You are on page 1of 977

Join Telegram--@mainchannelpdf

· A TEXTBOOK OF

I _ _ · · IC
C I
for JEE Main & Advanced
A TEXTBOOK OF:

for JEE Main & Advanced

Dr. RK Gupta

,::arihant
ARI HANT PRAKASHAN (Series), MEERUT
,:carihant
ARIHANT
PRAKASHAN (Series), MEERUT
All Rights Reserved

!ii ©AUTHOR
No part of this publication may be re-produced, stored in a retrieval system or by any means, lectronic
mechanical, photocopying, recording, scanning, web or otherwise without the written permission of the
publisher. Arihant has obtained all the information in this book from the sources believed to be reliable
and true. However, Arihant or its editors or authors or illustrators don't take any responsibility for the
absolute accuracy of any information published, and the damages or loss suffered thereupon.

All disputes subjtct to Meerut (UP) jurisdiction only.

!ii ADMINISTRATIVE & PRODUCTION OFFICES


Regel Office
'Ramchhaya' 4577/15, Agarwal Road, Darya Ganj, New Delhi -110002
Tele: 0ll- 47630600, 43518550; Fax; Oil- 23280316

Head Office
Kalindi, TP Nagar, Meerut (UP) - 250002
Tel: 0121-2401479, 2512970, 4004199; Fax: 0121-2401648

!ii SALES & SUPPORT OFFICES


Agra, Ahmedabad, Bengaluru, Bhubaneswar, Bareilly, Chennai, Delhi, Guwahati, Hyderabad, Jaipur,
Jhansi, Kolkata, Lucknow, Meerut, Nagpur & Pune

!ii ISBN 978-93-12147-02-3

!ii Price , 780.00

Printed & Bound by Arihant Publications (I) Ltd. (Press Unit)

For further information about the books from Arihant,


log on to www.arihantboob.com or email to info@arihantboob.com
PREFACE
In the recent years, the format and pattern of JEE Main & Advanced have been constantly changing. In pursuant to these latest
changes, I present a new text book the Inorganic Chemistry to help the students master the Inorganic Chemistry. The book covers
all the aspects of Inorganic at the level of Classes XI-XII with a view to master the syllabus of all Engineering Entrances.

Features of the Book


• Text has been devised in such a way as to be assimilable, without a prior knowledge of the Inorganic Chemistry.
• The basic thrust of the NCERT books has been followed and most of the problems from the NCERT books have been included
in each chapter.
• The whole text has been divided into twenty chapters, out of which seven devoted to General section, the backbone of Chemistry from
which almost 40% questions are being asked in JEE Main & Advanced every year.
• p-block elements have been sub-divided into six separate blocks to make it more relevant to NCERT pattern and examination oriented.
• Every chapter starts with Quick-Points that is the idea in nut-shell of the concepts/basics for application in the current unit and thus
helpful in creating confidence among aspirants.
• Target-Exercises at the end of each Section are meant to check your ability at the spot before you proceed to the next Section. This is
followed by Practice-Exercises on NCERT-based questions.
• There are lots of other exercises (with complete solutions) for testing analytical-skills. They include Comprehension-Based Questions,
True-False, Fill-in-blanks, Assertion-Reason, Matrix-Matching, Integer Answer Types, MCQs with One and More than One Correct
Options, Very Short and Analytical Questions compiled Sections-wise .
• Brain -Twisters are provide to sharp your knowledge specially of Olympiad Aspirants.
• Deep - Focus are scattered throughout the text. These are the warning concepts to catch your eyes for spot revision before examination.
• Essential Inorganic CHEMISTRY is'thought to be non-conceptual, nevertheless, I hope this text will be able to debunk this
misconception and will be immensely helpful.

Acknowledgement
Finally, there are many faculty and students, who have helped in the development of contents by their valuable suggestions and
unsolicited comments .They know who they are, and I hope that they will accept my sincere thanks for all they did.
It is also a pleasure to acknowledge the many people for their friendly assistance in completing this book, especially Shri Y C Jain,
Chairman, Arihant for encouraging me at each step, and for being an exemplary Publisher, and good friend too. My special thanks
are also to Daya Sindh Computers, Meerut for completing the book well in time. I am also indebted to MS Rawat, Keshav Agarwal,
Harsh Kumar and Narendra Singh who left no stone unturned for completing this project successfully.
I must thank Almighty God for His inspiration and guidance, and my family members for their unquestioning timely
encouragement without which this work would not be possible.
I would be glad to from the faculty and other of any errors which may have escaped. Any suggestion whereby the book can be
improved will be heartly welcomed.

Dr. RK Gupta
raghvesh@gmail.com
CONTENTS

1. Fundamental Concepts of Inorganic Chemistry 1-31


• Matter and Energy • Physical and Chemical Changes
• Laws of Chemical Combination • Dalton and Constitution of Matter
• Atomic Number, Mass Number and Isotopes • Equivalent Weight
• The Mole , • Per cent Composition and Empirical Formula

2. Chemical Bonding 33-96


• Ionic Bonding • Covalent Bonding
• Coordinate Bonding • Formal charge
• VSEPR Theory • Assessing the Molecular Shapes
• Sigma and Pi Bonds • Valence Bond Theory
• Hybridisation • Molecular Orbital Theory
• Bonding in Complexes • Dipole Moment
• Fajan's Rule • Hydrogen Bonding
• Resonance • Metallic Bonding

3. Periodicity of Elements 97-166


• The Origin of the Periodic Table • Classification of the Elements in Blocks
• Periodicity • Valency and Oxidation Number
• Shielding (Screening) Effect and Effective • Ionisation Energy
Nuclear Charge • Electronegativity
• Size of an Atom and Ion • Hydration and Hydration Energy
• Electron Affinity • Hydrides
• Metallic Nature • Density
• Acid-Base Character of Oxides • Atomic Volume
• Melting Points and Boiling Points

4. Oxidation-Reduction 167-213
• Oxidation Number and Oxidation State • Oxidation and Reduction
• How to Balance a Redox Reaction? • Balanced Redox Reactions
• Equivalent Weights

5. Radioactivity 215-255
• Radioactivity • Units of Radioactivity
• Theory of Radioactive Disintegration • Nuclear Stability
• Nuclear Reactions • Nuclear Fission
• Nuclear Fusion • Artificial Radioactivity
• Binding Energy and Packing Fraction • Applications of Radioactive Isotopes
• Radiations and Matter: Detection and
Biological Effects

6. Chemical Equations and Reactions 257-289


• Chemical Equations • Stoichiometry
• Limiting Reactants • Yields of Chemical Reactions
• Sequential Reactions

7. Volumetric 291-353
• Titrimetry • Methods of Expressing Concentration of a Solution
• Mole Fraction • Per cent Concentration
• Molarity (M) • Molality (m)
• Equivalent Mass and Normality • Volumetric Titrations
• Acid-Base Titrations • Redox Titrations
• lodometric/Iodimetric Titrations

8. s-Block Elements - The Alkali Metals 355-402


• Introduction • Periodicity at a Glance
• Study of Group 1 (IA)-The Alkali Metals • Compounds of Sodium
• Compounds of Potassium • Diagonal Relationship-Anomalous
• Additional Reactions and Facts of Alkali Metals Behaviour of 1A Lithium

9. s-Block Elements - The Alkaline Earth Metals 403-430


• Reactions of Alkaline Earth Metals • Periodicity at a Glance
• Compounds of Alkaline Earth Metals • Anomalous Behavior of Beryllium
• Additional Reactions and Facts • Alkaline Earth Metal Compounds

10. Hydrogen and Its Compounds 431-465


• Hydrogen • Hydrogen Peroxide

11. p-Block Elements-Boron Family (Group 13) 467-500


• Group 13(IIIA) (The Boron Family) • Compounds of Boron
• Compounds of Aluminium . ‘ ,

12. Group 14 (IVA-The Carbon Family) 501-540


• Group 14 (IVA-The Carbon Family) Study of Properties
• Compounds of Carbon • Compounds of Graphite
• The Carbon Cycle and Green-House Effect • Silicon
• Compounds of Silicon • Compounds of Tin
— Compounds of Lead

13. Group 15 (VA-The Nitrogen Family) 541-588


• Group 15 — Periodicity • Study of Nitrogen and Its Compounds
• Study of Phosphorus and Its Compounds
14. Group 16 (VIA-The Chalcogens) 589-617
• Periodicity in Group 16 (VIA-The Chalcogens) • Ozone (03)
• Allotropy and Polymorphism of Sulphur • Study of Compounds of Group 16 (VIA)

15. Group 17 (VIIA-The Halogen Family) 619-650


• Gradation of Physical Properties • Study of Halogens (X2)
• Interhalogen Compounds • Pseudohalogens and Pseudohalides
• Anomalous Behaviour of Fluorine • Halogen Acids
• Oxoacids of Halogen • Oxides of Halogen

.16. The Transition and Inner Transition Elements 651-736


• Transition Elements (d-Block) • Physical Properties
• Compounds • Inner-Transition Elements ( f-Block)

17. Metallurgical Extraction 737-785


• The Occurrence and Isolation of the Elements • Extraction of Copper
• Extraction of Magnesium • Extraction of Lead
• Extraction of Tin • Extraction of Silver
• Extraction of Aluminium • Extraction of Iron
• Extraction of Zinc

18. Coordination Compounds 787-835


• Double Salts and Coordination Compounds • Ligands
• Coordination Number • Nomenclature
• Effective Atomic Number • Isomerism in Complexes
• Werner Theory of Coordination Compounds • Bonding in Complexes
• Stability Constants and Stability of • Applications of Coordination Chemistry
Coordination Compounds

19. Salt Analysis 837-910


• Physical Examination • Analysis of Acid (Anion) Radicals
• Analysis of Basic (Cation) Radicals • DryTests
• One Reagent for Testing Different Ions

20. Group 18 (VIIIA-The Noble Gases) 911-931


• Discovery • Isolation of the Elements·
• Position of Noble Gases in the Periodic Table • Periodic Properties
• Structure of Xenon Compounds • Study of Compounds of Group 18

Appendices
Previous Years' Questions JEE Main & Advanced (2017-2010)
9 -944
"----------------------------- -3-
3
2
1-2 l
Chemistry touches almost every aspect of our lives, our culture,
and our environment. Its scope encompasses the air we breathe,
the food we eat, the fluids we drink, our clothings, dwellings,
transportation and fuel supplies, and our fellow creatures.

Fundamental Concepts
of
Inorganic Chemistry

Chemistry is the science that describes matter-its chemical


and physical properties, the chemical and physical changes it
undergoes, and the energy changes that accompany those processes.
Matter includes everything that is tangible, from our bodies and
the stuff of our everyday lives to the grandest objects in the wiiverse. • Matter and Energy
Some call chemistry as the central science. It rests on the
• Physical and Chemical Changes
foundations of mathematics and physics and in turn underlies the
life sciences biology and medicine. • Laws of Chemical Combination
■ Dalton and Constitution of Matter
All living matter contains carbon combined with hydrogen. The
chemistry of compounds of carbon and hydrogen is called Organic ■ Atomic Number, Mass Number and

Chemistry. The study of substances that do not contain carbon Isotopes


combined with hydrogen is called the Inorganic Chemistry. The ■ Equivalent Weight
branch of chemistry that is concerned with the detection or
■ The Mole
identification of substances present in a sample (qualitative
analysis) or with the amount of each that is present (quantitative • Per cent Composition and Empirical
analysis) is called Analytical Chemistry. Physical Chemistry Formula
applies the mathematical theories and methods of physics to the
properties of matter and to the study of chemical processes and the
accompanying energy changes. As its name suggests,
Biochemistry is the study of the chemistry of processes in living
organis ms.

Matter and Energy


Matter is anything that has mass and occupies space. Mass is a
measure of the qu�tity of matter in a sample of any material. The
mor� massive an
object is, the more force is required to put it in the
motion. All bo
dies consist of matter. Our senses of sight and touch
usually tell us
that an object occupies space. In the case of colourless,
odourles s, a
nd tasteless gases (such as air), our senses may fail us.
2 Essential Inorganic Chemistry

Energy is defined as the capacity to do work or to Solids


transfer heat. Energy can be in the form of In the solid state, the intermolecular force is very
(i) mechanical energy strong. The molecules in a solid species, therefore, cling
(ii) light energy
together, with a great force. Thus, solids have definite
(iii} electrical energy shape and volume.
(iv) heat energy
Energy is classified into two principal types: Liquid
In the liquid state, the intermolecular forces are very
Kinetic Energy feeble. They are still perceptible. Thus, liquid has no
A body in motion, such as rolling boulder, possesses definite shape but takes the shape of that part of the vessel
energy because of its motion. Such energy is called in which it is stored.
kinetic energy. It represents the capacity for doing work
directly. It is easily transferred between objects. Gases
In the gaseous state, the intermolecular force is
Potential Energy negligible and molecules lie far apart from one another.
It is the energy an object possesses because of its They move in different directions with different velocities
position, condition or composition. Coal, for example, colliding among themselves and against sides of the
possesses chemical energy, a form of potential energy, container.
because of its composition. Thermal plants burn coal,
Ice Steam
producing heat which is converted to electrical energy. A Properties
(Solid �O)
Liquid �O
(Gaseous �O)
boulder located at top of a mountain possesses potential
energy because of its height. It can roll down the mountain Rigidity Rigid (definite Flows and Fills any container
side, and potential energy is converted into kinetic energy. shape and assumes shape completely
volume) of container (no (no definite
definite shape but volume and
Different States of Matter definite volume) shape)
Expansion on Slight Slight Expands infinitely
Matter heating
Compressibility Slight Slight Easily
i
Solid
i
Fluid
Melting point solid> liquid> Compressed gas
We know that when water is boiled, it disappears;
l
Liquid Gas
when a candle burns, both the wick and the wax entirely
disappear, practically all its weight is lost. Again a piece of
magnesium wire, on being strongly heated in air, burns
A molecule is the smallest particle of any kind of with a brilliant light and leaves a white residue, which
matter, element or compound, which can exist in the free when weighed, will be found to be heavier than the piece of
state and can still preserve the character of that kind of magnesium taken.
matter. The properties of a substance, are the When we try to follow these, we are confronted with
properties of its molecules. some questions which naturally come to our mind? Is
water actually destroyed? Is there is destruction of the
substance of which the candle is decomposed? Whence
does the additional weight of magnesium come? Can
cu
II)
C
matter be created? We can certainly answer these
Q)
"'C
C
questions one by one.
• When water is boiled, it only changes its state and is
0

8
0

not destroyed.
Melting
�O(l) ----+ lliO(g)
Freezing
The candle loses weight due to combustion and th e
products are colourless gases that are disappearing into
Fig. 1.1 the atmosphere.
Chapter 1 : Fundamental Concepts of Inorganic Chemistry 3

Matter Chemical Change


(solid, liquid or gas)
Anything that occupies If there is change in the specific properties of the
space and has mass matter due to change in the composition or constitution of
Physically separable into its molecules, this is termed as chemical change. There is
formation of new molecules as a result of a chemical
change.
Homogeneous matter Heterogeneous matter
Uniform composition Non-uniform composition Illustration :
throughout
• Acidified water is converted into hydrogen and oxygen due to
Physically separable into electric decomposition
2Hp -. 2H2 (g)+ 0 2 (g)
Solutions Pure substances This is a chemical change, since Hp is converted into new
compounds H2 and 02 .
Homogeneous mixtures, Fixed combustion
uniformly composition cannot be further purified • When iron piece is exposed to air for a long time, resulting
that may vary widely
• I i takes place. Rust appears in the form of brown-coloured layer
and is formed when iron reacts with moist air .
Compounds Chemically Elements 0 0
_ separable into Fe -4 FeO �2 Fe 0 · xH 0
2 3 2
Elements combined in '
� Cannot be subdivided rust
fixed ratios Combine by chemical or
chemically to form physical processes
Think Yourself !
Fig. I. 2 Classification of matter When a candle (of wax) burns, wax melts.
After sometime wax is completely converted into
Magnesium gains weight due to its reaction with the CO2 and H 20. What is the type of this change?
oxygen of the atmosphere and gives white residue We know that wax is made of carbon, hydrogen
(of magnesium oxide). But in no chemical action any kind and oxygen when wax melts, it is a physical
change. But when it is converted into CO 2 and
of matter is destroyed or created.
H20 due to burning, it is a chemical change.

Physical and Chemical Changes Table 1.1 Distinction between Physical and Chemical
Change
Matter is capable of undergoing variety of changes as
Physical change Chemical change
Example
Physical Change CaC0 3 (s) �

Ifthere is change in some of the specific properties like CaO(s) + CO 2 (g)


the state (solid to liquid or vice-versa), texture, electrical or Composition no change, water in Changes due to
solid and liquid state formation of new
magnetic without bringing about any change in the have identical compounds. CaC0 3 (s),
composition or constitution of its molecules, then this type · composition H and 0 CaO(s) and CO 2 (g) have
being in 2 : 1 ratio completely different
of change is termed as physical change. composition.
Original state can be restored by change in Properties Physical properties Physical properties and
experimental condition. differ but chemical chemical properties of the
properties are identical. reactants and products
are quite different.
Illustration : Stability A temporary change, A permanent change,
above equilibrium is reaction is spontaneous
Hp (liquid) is converted into Hp vapour (steam) on heating,
reversible. Original state going to completion.
and steam into Hp (liquid) on condensation. Hp (liquid) is Original state is not
is restored by change in
converted into ice at freezing point, and ice melts to Hp(Q at melting exp- erimental condition. restored.
point.
Thermal effect Physical change can Chemical change initially
Hp (s) � Hp (Q � H20 (g). take place in the endothermic (takes
absence or presence of place with absorption of
Thus, it is completely a physical change. heat. heat). Above change
Above change absorbs requires heat.
• When an electric current is passed into copper wire wrapped
heat in forward side and
over iron, it is magnetised. When flow of electric current stops,
loses heat in backward
magnetic behaviour is also lost. Thus, it is a physical change. · side.
4 Essential Inorganic Chemistry

Laws of Chemical Combination • CaCO3 irrespective of its source has Ca, C and 0 as
40%, 12%, 48% by mass.
Law of Conservation of Mass
(Lavoisier-1789) Numerical Problem Solving 2
“As a result of chemical 12 g magnesium combines with 8 g oxygen to form magnesium
change, the total mass of the oxide. If 32 g oxygen combines, what is the mass of magnesium
combining with oxygen? Do they differ in composition.
reactants is equal to the total mass
2Mg+ O2 ---- > 2Mg (Mg = 24,0 = 16)
of the products.”
......g Mg
Thus, of masses m and n of
reactants A and B react to form masses
x and y of the products C and D Law of Multiple Proportions
A + B ---- > C + D (Dalton-1803)
m n. x y
Then, m +n=x+y “When one element combines with a second
2H2(g) + O2(g) ---- > 2H2O(g) element to form two or more different compounds,
4g 32g 36g the mass of one of the elements which combines
with the constant mass of the other, bears a simple
CaCO3(s) ---- > CaO(s) + CO2(g)
ratio to one another.”
10°g 56g 44g
CH3C00H + CjHgOH ---- > CHjCOOCjHg + HjO Simple ratio means whole number multiples
30g 23g 44g 9g A combines with B to form AB and ABj. Then ratio of B
(by mass in AB and AB2) that combines with a definite
mass of A is in a simple multiple ratio.
Numerical Problem Solving 1
H(2g)
AgNO3+ KC1 ---- > AgCl +KN03
17g 7.45 g 14.35 g ? +16g 02 \ 32g O2

What is mass of KNO3?


ab2 h2o h2o2
• 2 g H2 combines with 16 g O2 to form 18 g HgO
Law of Definite or Constant Composition 2g H-j combines with 32 g O2 to form 34 g
(Proust-1799) Thus, ratio of oxygen (by mass) that combines with
lgHL, is8:16 = l:2
“A chemical substance always consists of the
same elements combined together in the same
proportions by mass.” Numerical Problem Solving 3
If the elements A and B combine together to form the (i) Carbon and oxygen form two oxides carbon monoxide and
compound AB, then in whatever manner AB is formed, carbon dioxide. What is the ratio of oxygen that combines
then it is always composed of these two elements and mass with 1 g carbon in each?
2C + O2 —> 2CO
percentage of A and B remains constant.
C + O2 --- ) co2 Ratio =
AB can be formed by two methods. (ii) Consider the following reactions.
A + B ---- » AB (a) 2C + H2 ---- > C^ (b) 2C + 2H, ---- >
(c) 2C + 3H2 ---- > C2H6 (d) v,^
C + 2H2 ---- > CH4
First method ag bg ~ thus, ratio ofA and
Is law of multiple proportions followed.
Yes! No
B in AB by mass
x
Second method x g y g Law of Reciprocal Proportions
y
(Richter-1792)
a X
thus,
b y “The masses of two or more different elements
which separately combine with a definite mass of
• irrespective of its source has H and 0 in 1:8 mass another element are either the same as or are
ratio in pure state. simple multiples of the masses of these different
elements when they combine amongst themselves.”
Chapter 1: Fundamental Concepts of Inorganic Chemistry 5

A combines with C to form AC Equivalent mass of calcium = 20


B combines with C to form BC Equivalent mass of chlorine = 35.5
A combines with B to form AB
H2 + Cl2 ---- > 3HC1
Then ratio ofA and B by mass in AB is same as ratio of Ratio by mass 2g 2 x 35.5 g 2 x 36.5 g
A and B, and AC and BC.
Hg and Cl2 combine in the mass ratio = 2:2x35.5
(a) C + 2Hg ---- > CH4
= 1:35.5
(b) O2+2Hg ---- > 2HgO(O2H4)
Thus, ratio is also the ratio of their equivalent masses.
(c) C + O2 —> C02
H 2H, + O, ---- > 2H0O
4g 32g 36g
CHy \h2o H-2 and O2combine in the mass ratio = 4:32 = 1:8
This is also the ratio of their equivalent masses.
C< \o2
CO2 Law of Reciprocal Proportion is a special case of
Law of Equivalent Proportion
4 g Hg combines with 12 g carbon in (a)
4 g Hg combines with 32 g oxygen in (b)
Thus, ratio of carbon and oxygen in (a) and (b) by mass Numerical Problem Solving 5
that combines with a definite mass of Hg(in this case 4 g) Consider the following reactions (Mg = 24, Cl = 35.5, Ca = 40)
= 12:32 = 3 :8 in (c) 12 g carbon combines with 32 g oxygen (a) Mg+H2 —> MgH2
thus ratio of carbon and oxygen (by mass) in CO2 is = 3 : 8 (b) Mg + Cl2 ---- > MgCl2
(c) Ca + Cl2 ---- > CaCl2
Compound Elements Mass ratio Equivalent mass of hydrogen is 1, derive equivalent mass of
CH4 C and H magnesium, chlorine and calcium.
Ans. Mg = Cl = Ca =
h2o 0 and H
co2 C and 0

Law of Gaseous Volume or


Numerical Problem Solving 4 Gay-Lussac’s Law (Gay-Lussac-1808)
Phosphorus forms PH3 andPCl3 withH2 andCl2 respectively. When gases combine together they do so in simple
P +|h2—> PH, ratio by volume and the volume of the product formed
(if gaseous), also bear a simple relation to the volume of
31g 3g
the reacting gases, measured under the same conditions of
P + -Cl2 —>PC13
temperature and pressure.
2 2
31g 3x 35.5 g (1 mole of every gas at NTP = 22.4 L and equal moles
In which ratio by mass H2 and Cl2 combine to form HC1? occupy equal volumes)
Ans.
Gaseous Reactions RofV P
Reactants
Law of Equivalent Proportions (a) H, + + Cl2 2HC1 1:1 : 2
1 mol 1 mol 2 mol
(Richter-1792) 1 volume 1 volume 2 volumes
“Elements combine in the ratio of their (b) N2 ++ 3Hj 2NH3 1:3 : 2
equivalent mass or in simple multiple of them. 1 mol 3 mol 2 mol
1 volume 3 volumes 2 volumes
(Equivalent mass - Refer in this unit)”
(c) 2Hg + 02 2H2O 2:1 : 2
Equivalent mass of hydrogen = 1 2 mol 1 mol 2 mol
Equivalent mass of oxygen = 8 2 volumes 1 volume 2 volumes
Equivalent mass of magnesium = 12 *R of V = Ratio of volume, **P = Product
6 Essential Inorganic Chemistry

Numerical Problem Solving 6 or more atoms with same atomic number but different
atomic mass are called isotopes).
Nitrogen combines with oxygen in the following reaction
• 62C, g3C, g4C, have atoms of different masses. }H, 2H,
N2(g) + O2(g) ---- > 2NO(g)
What is ratio of the reactants and products? 3H have atoms of different masses.
(a) By mass (b) By equivalent
(c) By moles (d) By volume Avogadro’s Hypothesis
Ans. n2 O2 NO Berzelius assumed “equal volumes of different
(a) gases under identical conditions of temperature
and pressure contain the same number of atoms
(b)
(The word atom was at that time used for both
(0
elements and compounds as molecule was not
(d) thought of). We consider the formation of HjOtg) from
H2(g) andO2(g)
Dalton and Constitution of Matter 2H2 (g) + O2(g) ---- > 2H2O(g)
2 volume 1 volume 2 volume
Dalton Atomic Theory (1808) Berzelius 2 atoms 1 atom 2 atoms
• All elements are composed of minute particles called 1 atom 1 .
- atom 1 atom
atoms, which do not undergo subdivision during 2
chemical reactions and cannot be created or destroyed
1 atom ofH, combines with 1/2 atom ofO2.This allows
by any chemical process. (Taylor)
• Atoms of the same element are similar to one another atom to be divisible. This is against Dalton’s atomic
in all respects and are equal in weights. theory. Hence, Berzelius hypothesis was discarded.
• Atoms of different elements have different masses Failure led to assume that gases exist in polyatomic state
(called atomic masses) and different properties. (diatomic, triatomic etc.) and ultimate particles which can
• Chemical combination between two or more elements exsit in the free state were defined as molecule.
takes place by interaction of their atoms in simple Avogadro’s hypothesis is based on molecules :
numerical proportions (as 1:1; 1:2; 1:3; etc.) “Equal volume of all gases under the same
Ratio conditions of temperature and pressure contain the
H2 +C12 ---- > 2HC1 1:1 same number of molecule”.
N2+3H2 ---- > 2NH3 1:3 Avogadro’s hypothesis led to
C + O2 ---- > CO2 1:1 - the word molecule and made the distinction
• Combining masses of the elements represent the between atoms and molecules.
combining masses of the atoms. - conclude that molecules of hydrogen, chlorine,
Dalton atomic, theory can explain various laws of oxygen, nitrogen and fluorine are diatomic.
chemical combination.
- conclude that molecular weight = 2 x vapour density.
- Conclude that one gram-molecular mass
Limitations and Defects of Dalton’s Atomic (molecular weight) has volume 22.4 L
Theory (22.4 x 10-3 m3) at NTP (pressure = 1 atm and
T = 273.15 K)
• It can explain the laws of chemical combination by
weight, but it fails to explain Gay-Lussac’s law of Number of molecules present in amount equal to molecular
gaseous volume. weight (which we shall call one mole) are 6.02 x 1023 (called
Avogadro’s number)
• Distinction between the ultimate particles of an
element and the smallest particle of a compound, both H2 + Cl2 2HC1
of which are made of atoms, could not be made. 1 volume 1 volume 2 volumes
1 mole 1 mole 2 moles
• It assumes that atoms of same element are like and 6.02 x 1023 6.02 x 1023 2x 6.02 x 1023
molecules molecules molecules
have same mass. But this is not true for isotopes (two
Chapter 1: Fundamental Concepts of Inorganic Chemistry 7

Dalton’s Theory as Modified by Avogadro’s different mass numbers). There are three isotopes of
hydrogen
Hypothesis
hydrogen 1P 0 7V
1. Substances (elements and compounds) consist of
molecules which are composed of indivisible ?H deuterium 1P IN
particles called atoms. Molecules are capable of tritium 1P 2N
independent existence and are divisible into atoms. Thus, isotopes have different number of neutrons.
2. Molecules of a particular substance are similar to • The chemical properties of an element are determined
one another; with identical properties and mass. primarily by the protons and electrons in its atoms;
Molecules of one substance are different from those neutrons do not take part in chemical changes under
of other substances. normal conditions. Therefore, isotopes of the same
3. When there is chemical reaction between two or element have similar chemical reactions.
more substances, the molecules of each substance
are first disintegrated into atoms which combine Atomic Weight
together to produce new molecules of the new The atomic weight of an element is the smallest
substances. weight of it present in the molecular weights of its
N2(g) + 3H2(g) ---- > 2NH3 compounds.
It may also be defined thus —
2N + 6H ---- > 2NH3 The atomic weight of an element is a number which
expresses how many times the weight of one atom of the
element is greater than the weight of one atom of
Atomic Number, Mass Number and hydrogen.
Isotopes At. wt. of an element
_ wt. of 1 atom of the element
• All atoms can be identified by the number of protons
and neutrons they contain. The atomic number (Z) is wt. of 1 atom of hydrogen
the number ofprotons in the nucleus of each atom ofan Thus, the atomic weights are not the actual weights of
element. atoms but the relative weights of the atoms of various
• In a neutral atom the number of protons is equal to the elements expressed in terms of hydrogen as unit. Hence,
number of electrons present in the atom. when we say that the atomic weight of nitrogen is 14, we
• The chemical identity of an atom can be determined mean that an atom of nitrogen is 14 times heavier than an
solely from its atomic number. For example atomic atom of hydrogen; that is to say, if the weight of one atom
number of oxygen is 8. This means each neutral of hydrogen is taken as unit, the weight of 1 atom of
oxygen atom has 8 protons and 8 electrons. nitrogen will be 14 times the weight of 1 atom of hydrogen.
• The mass number (A) is the total number of neutrons Now a days the standard for atomic weight is C12. On
and protons present in the nucleus of an atom of an this basis the atomic weight may be defined as :
element. The atomic weight of an element is a number
• Except for hydrogen (which has only one proton), all which indicates as to how many times an element of
atomic nuclei contain both protons and neutrons. that element is heavier as compared with 1/12 of the
• The mass number is given by mass of an atom of C-12 (^C).
mass number (A) = P+ N = Z + N wt. of an atom
At. wt. of an element =
= atomic number + no. of neutrons — x wt. of an atom of l2C
12
Thus, N (number of neutrons) = A - Z
mass number \ The gram atomic weight (or the gram-atom) of an
element is its atomic weight expressed in grams. Thus,
AA
z X 1 g-atom of magnesium stands for 24 g of Mg.
atomic number /
The French chemists Dulong and Petit in 1819 found
• Atomic number, number of neutrons, and mass out that the product of the specific heat of an element in
number all must be positive integers (whole numbers). the solid state and its atomic weight is constant, and is
• Atoms of a given element do not all have the same equal to 6.4.
mass. Most elements have two or more isotopes
Specific heat x atomic weight = 6.4 (approx.)
(atoms that have the same atomic number but
8 Essential Inorganic Chemistry

Atomic Mass Unit Hence, (Z) atomic number of


M 2+ = 22+2 = 24
An early observation regarding atomic weight was
that carbon and hydrogen have relative atomic masses, or number of protons (P) in M 2+ =24
also traditionally called atomic weight, AW, of number of neutrons (N) in M 2+ = Z + 2 = 24+2 = 26
approximately 12 and 1, respectively. Thousands of ionic mass of M 2+ = P+ N =24+26 = 50
experiments on the composition of compounds have
resulted in the establishment of scale of relative weights Illustration 2 Ionic mass of x 2- is 16. It has 8
based on the atomic mass unit (amu), which is defined as. neutrons. Determine the number of its (a) protons (b)
"Exactly 1/12 of the mass of an atom of a number of electrons and (c) atomic number Z.
particular kind of carbon atom, called carbon-12. Its
Solution Ionic mass of x2 - =16, neutrons =8
value is 1.66054 x 10-24 g.
On this scale, the atomic weight of hydrogen is (a) protons =16 - 8 = 8
1.00794 amu :. electrons in neutral X =8
Sodium is 22.989768 amu - 23 times of H (b) X gains two electrons to form x2 -,
Magnesium is 24.3050 amu - 24 times of H hence, number of electrons is x2 - = 8 + 2 = 10
• amu is also called dalton (Da). (c) Z= atomic number = 8
• Mass of one 1lC atom = 12 amu (exactly)
Illustration 3 Write the appropriate symbol for each
mass of one �v2 C atom of the following isotopes.
1 amu=
12 (a) Z=74,A=186 (b) Z=80,A=201
= 1.66054 X 10-24 g Solution For any isotope, system is
1
• Thus, mass of one H-atom = -=1 amu mass number,
No 'i..Az X
16
mass of one O-atom = - =16 amu atomic number/
No (a) Z=74,A=186 6 x t:
Average Atomic Weight �i
(b) Z :::: 80, A=201 1 X

Average mass number of natural occurring element Illustration 4 1iB and 1 1B are two isotopes of boron.
containing two or more isotopes is given by
If average mass number is 10.2, what is the percentage of
,A= L AX = A1X1+ �X2 + ... each?
:EX X1+ X2 + ...
Solution Let the percentage of i0 B = x
where, A1, �, ... are mass numbers of the different
then percentage of � 1B = (100 - x)
isotopes with percentage or ratios as X1, X2, ...
�J
If �Cl and Cl are two isotopes of chlorine in the Average mass number A= i i
A X + �X2
X1+ X2
ratio of3 : 1, then average mass number of Cl is
lOx+11(100 - x)
A (Cl) = 35 x 3 + 37 x 1 = 35.5 10_2 =
x+ 100-x
3+1
lOx+1100 - 11.x
10_2 =
lliustration 1 A divalent cation (Atomic number Z) is 100
isoelectronic of CO2 and has (Z + 2) neutrons. What is ionic x=80
mass of divalent cation?
Thus, percentage of �0 B = 80%
Solution A divalent cation (M 2 + ) has as many as and percentage ofA1 B = 20%
electrons as CO2 =6 + 16 = 22
Chapter 1 : Fundamental Concepts of Inorganic Chemistry 9

Target Practice 1
23
Avogadro's nwnber = 6.02 x 10 atoms/mo! 8. 3 L of a gas weighs 2 g. What is the molecular mass? .........
1 mole of a gas at STP occupies 22.4 L 9. What volume does 22 g of CO 2 at STP occupy? ........ .
1. How many atoms of oxygen are there in 18 g of water?
IO. How many atoms of hydrogen are in 67.2 L ofH 2 at STP?

2. How many atoms of hydrogen are there in 18 g of water?


3. How many molecules of H2O are there in 18g of water? ...... 1 I. You are given 1.0 g samples of He, Fe and C. Which
contains the largest number of atoms? Which contains the
4. What is the mass of 1 mole of 0 2 ? ......... smallest? (He= 4, Fe= 56, C = 12)
5. What is the mass of 1 molecule of0 2 ? ........ . Note: Avogadro's number = 6.02x 10 2.1 atoms/mol
6. What is the mass of 2 moles ofH�SO.?
L 'I
.........
.
. 1 mole of a gas at STP occupies 22.4 L
7. What is the density (mass/voume) ofO2 at STP? .........

Answers
1. 6.02x 10 23 2. 1.204x 10 24 3. 6.02x 10 23 4.32g 11. He= No Fe= N� C= No
5. 5.32 X 10-23 g 6. 196 g 7. 1.43 g/L 8. 14.9 g/L 4' 56' 12
9. 11.2 L 10.3.612x 10 24 He (Largest) Fe (smallest)

Equivalent Weight and equivalent weight of ammonia= 5.66 g equivalent-I


It is also called the chemical equivalent (or simply => Mg + H:zSO4 � MgSO4 + �
equivalent) or the combining weight. 24 g 98 g 120 g 2g

The weight of an element which can combine with or 2 g � is displaced by 24 g Mg


displace from a compound 1 part by weight of Thus, 1 g H 2 is displaced by 12 g Mg
hydrogen, or 8 parts by weight of oxygen or 35.5
Thus, equivalent weight of magnesium
parts by weight of chlorine is called the equivalent
weight of the element. = 12 g equivalent-1

1 part by weight of hydrogen is actually the Also, 2 g � is displaced by 98 g H 2 SO4


reference weight. Thus, 1 g � is displaced by 49 g �SO4
• �+ Cl2 � 2HC1 Thus, equivalent weight off½SO4 = 49 g equivalent.
2g (35.5 X 2) g 2 X 36.5 g
=> 2 g hydrogen is equivalent (s) to 71 g chlorine (Cl2 ) DEEP Focus
thus, 1 g hydrogen is �quivalent to(=) 35.5 gCl2 Equivalent weight is not a fixed quantity. It varies
Thus, equivalent weight ofC12 = 35.5 g equivalent- 1 reaction to reaction.
and also equivalent weight of HCl = 36.5 g equivalent-1
N2 + 3H:z � 2NH 3
=> 2� + 02 � 2H 2 O
4g 32g 36 g Equivalent weight of nitrogen (N2 ) as equivalent to
1 g � = 4.66 g equivalent-1
4 g hydrogen = 32 g oxygen
N2 + 02 � 2NO
Thus, 1 g hydrogen = 8 g oxygen 28 g 32 g
Thus, equivalent weight of oxygen 32 g oxygen!! 28 g nitrogen
= 8 g equivalent-1 Thus, 8 g oxygen!!!! 7 g nitrogen
Thus equivalent weight of water = 9 g equivalent-1 Thus, equivalent weight of nitrogen
⇒ N2 +3� � 2NH 3 = 7 g equivalent-1
28 g 6 g 34 g
6 g hydrogen !!!: 28 g nitrogen and 34 g ammonia In acid-base reactions, one equivalent weight, or
equivalent (eq), of an acid is defined as the mass of acid
23
Thus, 1 g hydrogen (in grams) that will furnish 6.022 x 10 hydrogen ions
28 34 (1 mole) or that will react with 6.022 x1023 hydroxide ions
= (= 4.66 g) nitrogen and (5.66 g) ammonia. 3
(1 mole). One mole of an acid contains 6.022 x1()2 formula
6 6
Thus, equivalent weight of nitrogen = 4.66 g equivalent-1 units of the acid.
10 Essential Inorganic Chemistry

h2o The SI unit for amount is the mole, abbreviated mol.


HC1 H+(aq) + Cl (aq)
1 mol 1 mol 1 mol It is defined as the amount of substance that contains as
36.50 g 1-00 g 35.50 g many entities (atoms, molecules, or other particles) as
6.022 x 1023 FU 6.022 x 1023 FU 6.022 x 1023FU there are atoms in exactly 0.012 kg of pure carbon-12
atoms. Many experiments have refined the number, and
We observe that one mole of HC1 can produce
the currently accepted value is
6.022 x IO23 H+, hence one mole of HC1 is one equivalent.
The same is true for all monoprotic acids (with one 1 mole =6.0221367 x 1023 particles
ionisable H+) One mole is always associated with a chemical
formula and refers to Avogadro’s number of particles. It is
Sulphuric acid(H2SO4) is a diprotic acid. One molecule
designated by No
of H2SO4 can furnish 2H+ ions
No =6.022 xlO23 mol"1
h2o?
h2so4 2H+(aq) 16 g oxygen atom = 1 mole of oxygen atom
1 mol 2 mol
6.022 x 1023 FU 2x (6.022 x 1023)FU = 6.022 x 1023 atoms

FU = Formula Unit + SC^"(aq) 32 g oxygen gas = 1 mole of oxygen (O2) gas


1 mol = 6.022 x 1023 oxygen molecules
6.022 x 1023 FU
= 2 x 6.022 x 1023 oxygen atoms
Thus, equation shows that one mole HgSC^ can
produce 2(6.022 x 1023) H* ions therefore, one mole of
H2SO4 is two equivalent weights in all reactions in which
DEEP Focus
both acidic hydrogen ions react. When to use 1.008 g/mol for the molar mass of hydrogen and
when to use 2.016 g/mol?
One equivalent weight of a base is defined as the
The statement, “a mole of hydrogen” is ambiguous; you
mass of the base (expressed in grams) that will furnish should always say either a mole of hydrogen atoms or a mole of
6.022 x 1023 hydroxide ions or the mass of the base that hydrogen molecules. Better still write 1 mol H or 1 mol HLj. If
will react with 6.022 x IO23 hydrogen ions. you do this is, then you will see that the molar masses should be
Detailed study of Equivalent weights based on redox expressed as 1.008 g H/mol H and 2.016 g I^/molHj. This
reaction are taken in unit-3 distinction is very much like the distinction between one dozen
socks and one dozen pairs of socks (that is, the H atom is
analogous to a single sock and the H2 molecule to a pair
Numerical Problem Solving 7 of socks).
(i) Taking equivalent weight of hydrogen as (1) and that of
oxygen as (8) derive equivalent weights of the underlined. The mass of one mole of atoms of pure element.
(a) Mg + H2 ---- > MgH2 in grams is numerically equal to atomic weight of
(b) Mg + - O, —-> MgO that element in amu. This is also called molar mass of
2 1 -----
the element; its units are grams/mole
(c) C + O2 ---- > CO2
(d) 1n2 +O2--- > NO2........ Molar mass of a substance is the mass in gram of one
2
mole of that substance.
(ii) How many equivalents are in (based on ionization)? ,, . r , mass in grams
Moles of substance = ------------------- 5------------ —
(a) 3.65gHCl(^H++C1’) molar mass in grams/mole
(b) 9.8gH2SO4 2H+ +SO2’)
Given
(c) 9.8 g ofH3PO4 3H+ +POJ") .............. Grams of element A
(iii) Cobalt has three radioactive isotopes used in medical studies. Divide by
Atoms of these isotopes have 30, 31 and 33 neutrons, atomic mass
respectively. Give the symbol for each of these isotopes Moles of A

Multiply by Wq
The Mole Atoms of A
You are already familiar with the dozen (12 items)
and the gross (144 items). Fig. 1.3
Chapter 1: Fundamental Concepts of Inorganic Chemistry 11

------------------------- ..Given
~ ? “T1 z Given Grams of substance A _____
Atomic mass of A
t /[Divide by
Divide by No i- x. molar mass
Moles of A

Atomic mass unit of A - Mass of one atom of A Multiply by Nq

Fig. 1.4 Molecules of A

Fig. 1.5

Table 1.2 One Mole of Some Ionic Compounds


Compound Formula weight A sample with a mass of 1 mole Contains
Sodium chloride 58.44 58.44 g NaCI 6.022x 1023 Na* ions or 1 mole of Na* ions
6.022 x 1023 Cl" ions or 1 mole of Cl’ ions

Calcium chloride 111.0 111.0 g CaCI2 6.022x 10.23


23 Ca22+* ions or 1 mole of Ca2+ ions
2(6.022 x 1023) CC ions or 2 moles of Cl" ions

Aluminium 342.1 342.1 g AI2(SO4)3 2(6.022x 1023) Al3* ions or 2 moles of Al3* ions
sulphate
3(6.022x 1023) SO2- ions or 3 moles of SO2- ions

25.6
Illustration 4 Sulphur is a non-metallic element. Its =-----x 6.022 x 1023 x 4H atoms
presence in coal gives rise to the acid-rain phenomenon. 60
How many sulphur atoms are in 10 kg of coal sample = 1.0274 x 1024 H atoms
containing 1% sulphur. (S = 32 g mol~l)
Illustration 6 Mass of one atom of an element X is
Solution 10 kg coal= 10 x 103 g coal 2.32481 x 10-23 g. Express this in amu.
10 x 103 x 1 Solution Let atomic mass of the element be = A
g sulphur
100
Number of atoms in one mol of X - NQ
= 102/32mol sulphur A
Hence, mass of one atom of X = —
= (102/32) x 6.022 x 1023 atoms of sulphur No
A
= 1.882 x 1024 sulphur atoms — = 2.32481 xlO-23 g

Illustration 5 How many H atoms are present A = 2.32481 x 1(T23 xN0


in 25.6 g of urea [(NH2)2CO)], having molar mass of
60 g mor1? = 2.32481 xlO-23 x 6.022 x IO23
25 6 = 14
Solution 25.6 g urea = mol urea 14
Hence, mass of one atom of X = — = 14 amu
25.6 No
= -—x 6.022 x 1023 molecules
60

Target Practice 2
1. Which of the following has the greatest mass? (c) Ammonium nitrate NH^NO,'3
(a) 200 molecules of water (d) Guanidine HNC(NH,),
(b) 100 atoms of Fe
(c) 200 molecules ofO2 3. One atom of an element has a mass of 9.123x10,-23 g-
(d) 200 molecules of CH 4 There is only one isotope of the element. The element is
(a) chlorine (b) chromium
2. If the fertilizers listed below are priced according to their (c) magnesium (d) manganese
nitrogen content, which will be the least expensive per 50
kg bag? 4. One mole of (NH4 )2Cr2O7 contains
(a) Urea(NH2)2CO (a) one mole of nitrogen (b) four moles of hydrogen
(b) Ammonia NH,w (c) one mole of chromium (d) seven moles of oxygen
12 Essential Inorganic Chemistry

5. Which of the following statements are true? N


8. There are —2-atoms
atomsin givenquantity
ingiven quantityof carbon-12.Same
ofcarbon-12. Same
(a) 1 mole H atoms = 6.02 x 1023 H atoms
(b) 6.02 x 1023 H atoms have a mass of 1.008 g number of atoms are present in
(c) The formula mass ofO2 = 32.00 amu (a) 6 g Mg (b) 12 g Mg
(d) All of the above are true (c) 16gO2 (d)8gCH4
6. Which of the following has the largest mass of carbon per 9. Mass of O2, N2 and CH4 containing equal moles will be in
gram? the ratio of
(a) H2CO3 (b) CI’3002H (a) 16 :14 : 16 (b) 8 : 7 : 4
(c) CH3OH (d) CH3UH2OH (c) 1: 1: 1 (d) 4 : 7 : 4
7. 1 g urea (NH2CONH2), 1 g acetic acid (CH3COOH) and 1 g 10. Which has maximum volume at STP? (1 mole of a gas
formaldehyde (HCHO) will have H-atoms in the ratio = 22.4 L at STP)
(a) 2 : 2 : 1 (b) 1:1: 2 (a) 1.6gCH4 (b) 6.02 x 1022 N2 molecules
(c) 1:1:1 (d) 1: 2 :1 (c) 4.4gCO2 (d) Equad

Answers
200 3600 „TT Q 19 24
1. (a) 200 molecules H2O = —- mol H2O =---- - g H2O 6. (a) H2CO3, — g carbon (b) CH3CO2H, — g carbon
No No
19 24
100 (c) CH3OH, — g carbon (d) CH 3CH 2OH, — g carbon (max.)
(b) 100 atomsFe=mol Fe x 56 = g Fe
No No No
/ \ onn i in Thus, (d)
(c) 200 molecules O2 = 200
—- molin 6400 n
O2 =------ g O2
2 No N,o
N 7. 1 g urea = — mol = - — H-atom
60 60
(d) 200 molecules CH 4 1 g acetic acid = — mol = ^2. H-atom
= 200
—x in 10 g ru
CH4 3200 gCH4 60 60
No No 1 27V
1 g formaldehyde - — mol = —~ H-atom = —- H-atom
Thus, (c) 30 30 60
2. (c) Thus, 1:1:1
3. 9.123 x 10rax6.02x 1023 g mol"1 =55 Thus, (c)
Mn. Thus, (d)
8. (b) 9. (b) 10. (d)
4. (d) 5. (d)

Practice Exercise 1
1. Substances X and Y are colourless gases obtained by 6. Fill in the blanks in the following table :
combination of sulphur and oxygen. Substances X results Symbol |jFe
from combination of 6.00 g of sulphur with 6.00 g of oxygen 79 86
Protons 5
and substance Y results from combination of 8.60 g of Neutrons 6 16 117 136
sulphur with 12.9 g of oxygen. Show that the mass ratios in Electrons 5 18 79
the two substances are simple multiples of each other. Net charge -3 0
• 2. In addition to carbon monoxide (CO) and carbon dioxide
7. How many H atoms are in 6.0 g of isopropanol C3H8O?
(CO2), there is a third compound of carbon called carbon
suboxide. If a 2.500 g sample of carbon suboxide contains 8. How many moles of cobalt atoms are there in 6.022 x 109 Co»
1.32 g of C and 1.18 g of O, show that the law of multiple atoms?
proportions is followed. What is possible formula of carbon 9. The density of water is 1.00 g/mL at 4°C. How many water
suboxide? molecules are present in 2.56 mL water at this
temperature?
3. In which one of the following pairs do the two species
10. Aspirin has the formula C9H8O4. How many moles of
resemble each other most closely in chemical properties?
Explain. aspirin are there in a tablet weighing 500 mg? How many
molecules?
(a) jHandjH* (b) 14 N and 147 N3-
11. What is molecular weight of chloroform if 0.0275 mol
(c) g2Candg3C
weighs 3.28 g?
4. One isotope of a metallic element has mass number 65 and 12. An average cup of coffee contains about 125 mg caffeine,
35 neutrons in the nucleus. The cation derived from the C8H10N4O2. How many moles of caffeine are in a cup? How
isotope has 28 electrons. Write the symbol for this cation. many molecules?
5. Which of the following symbols provides more information 13. Select correct alternate. Only one alternate is correct.
about the atom : 23 Na or nNa? Explain. . (A) In which case purity of the substance is 100%?
(a) 1 mole ofCaCO3 gave 11.2 L CO2 (at STP)
Chapter 1: Fundamental Concepts of Inorganic Chemistry 13

(b) 1 mole of MgCO3 gave 40.0 g MgO (F) M2+ ion is isoelectronic ofSO2 and has (Z + 2) neutrons
(c) 1 mole of NaHCO3 gave 4 gH2O (Z is atomic number of M}. Thus, ionic mass of Af2+ is
(d) 1 mole ofCa(HCO3)2 gave 1 mole CO2 (a) 70 (b) 66
(B) Consider following laws of chemical combination with (c) 68 (d) 64
examples: (G) X+, y2+ and Z~ are isoelectronic of CO2. Increasing
I: Law of multiple proportion : N2O, NO, NO2 order of protons in X+, 72+and Z“ is
II: Law of reciprocal proportion : H2O, SO2, H2S (a) = y2+ = z- (b) x+ < y2+ < z~
Which is correct with examples? (c) z~ < x+ < y2+ (d) y2+ <x+ <z-
(a) I and II (b) I only
(H) X~, Y2~ and Z3- are isotonic and isoelectronic. Thus,
(c) II only (d) None of these
increasing order of atomic number of X, Y and Z is
(C) H2S contains 94.11% sulphur; SO2 contains 50% oxygen
(a) X<Y<Z (b)Z<y<X
and H2O contains 11.11% hydrogen. Thus, (c)x=y=z (d)z<x<y
(a) law of multiple proportion is followed (I) If each O-atom has two equivalents, volume of one
(b) law of reciprocal proportion is followed equivalent of O2 gas at STP is
(c) law of conservation of mass is followed (a) 22.4 L (b) 11.2 L
(d) all of the above are followed
(c) 5.6 L (d) 44.8 L
(D) Sodium combines with 33 Cl and 3JC1 to give two
(J) Each drop of H2O has 0.018 mL at room temperature.
samples of sodium chloride. Their formation follows the
law of Number ofH2O molecules in one drop is
(a) gaseous diffusion (b) conservation of mass (a)lxlO’3 (b) 6.02 x 1020
(c) reciprocal proportion (d) none of these (c) 22.4x 10'3 (d) 6.02x3x 102
(E) According to Dalton’s atomic theory, the smallest (K) 1 g CH4 and 4 g of compound X have equal number of
particle in which matter can exist is called moles. Thus, molar mass of X is
(a) an atom (b) an ion
(a) 16 g mol-1 (b) 32 g mol-1
(c) an electron (d) a molecule
(c) 4 g mol-1 (d) 64 g mol-1

Answers
Ratio 7. 4.8176 x 1023 H atoms
S 0 8. IO"14 mol cobalt atoms
1. X : SO2 1 : 2
y : SO3 1 : 3 9. 8.56 x 10 22 water molecules
Thus ,OatominSO2andSO3 are simple multiple of each other. 10. 2.78 x IO"3 mol, 1.67 x 1021 molecules
2. Carbon suboxide C3O2 11. 119.27 g mol’1
O=C=C==C=O 12. 6.44 x 10-4 mol, 3.88 x 1020 No molecules
3. (c) 4. 65 X/2 + 5. 23 Na 13. A. (b) B. (a)
30 M
C. (b) D. (d)
6. Symbol 11 M
26 Fe
re i^P3- ffAu E. (a) F. (a)
Protons 5 26 15 79 86 G. (c) H. (b)
Neutrons 6 28 16 117 136 I. (c) J. (b)
Electrons 5 26 18 79 86 K. (d)
Net charge zero zero -3 zero zero

Per cent Composition and Empirical because it tells the ratios of atoms in a compound. The

Formula simplest or empirical formula for a compound is


the smallest whole number ratio of atoms present.
• The per cent composition of a compound is expressed For molecular compounds the molecular
by identifying the elements present and giving the formula indicates the actual number of atoms
mass per cent of each. present in a molecule of the compound. It may be the
• By knowing per cent composition of compound makes same as the simplest formula or elase whole-number
it possible to calculate the chemical formula of the multiple of it. Simplest and molecular formulae of
compounds. The method involved is to find the water are both H>0. However for hydrogen peroxide
relative number of mole of each element in the they are HO and H2O2 respectively.
compound and then use & numbers to establish the
mole ratios of the elements. molecular weight
Multiple n =
• A formula derived up to above stage from data about empirical formula weight
per cent composition, is called an empirical formula
14 Essential Inorganic Chemistry

• Molecular formula = (empirical formula) n % of oxygen = x 100 = 53.33%


180
Convert mass Convert mass Find mole Ratio gives
per cent to mass to moles ratio formula Illustration 3 Analysis of a pure compound of an
oxide of sulphur gave 50.1% sulphur and 49.9% oxygen by
%A x mol A —I x mol A mass. What is the simplest formula of the compound?
%B gs y mol B — y mol B Solution
Relative Smallest
Illustration 1 What is simplest formula of a salt that Atomic Number of Divide by whole
Relative
Element weight atoms smallest number
contains 40% calcium, 12% carbon and 48% oxygen? mass
number ratio of
(At. wt.) (Divide by
Solution At. wt.) atom

Smallest
S 50.1 32.1 —— =
§2J = i
1.56 —=
.56 — = i(S)
1(S) 1
Atomic Relative Divide by whole 32.1 1.56
Relative
Element weight number smallest number 2
mass 0 49.9 16.0 = 3.12 — = 2(0)
(At. wt.) of atoms number ratio of 16.0 1.56
atoms
40 1 Thus, simplest formula = SO.'2
Ca 40.0 40.0 — = 1.0 - = 1 (Ca) 1
40 1
21 = 1(C) 1 Problem Solving TiPs
c 12.0 12.0 — = 1.0
12 J
(Known common fractions in decimal form)
O 48.0 16.0 — =3.0 1 = 3(0) 3
As Illustration (1) illustrates, sometimes we must convert a
16
fraction to a whole number by multiplication by the correct
Thus, simplest formula = CaCO.'3 integer. But we must first recognise which fraction is
represented by a non-zero part of a number. The decimal
Illustration 2 Glucose or blood sugar has the equivalents of the following fractions may be useful.
molecular formula C6H12O6. What is the empirical formula Decimal equivalent To convert to Integer
Fraction
and what is the per cent composition of glucose? (to 2 places) multiply by

Solution Molecular formula: CgH12O6 1/2 0.50 2


1/3 0.33 3
Mol ratio of C H O
6 12 6 2/3 0.67 3
simplest mol ratio 12 3 1/4 0.25 4
hence, empirical formula = CH2O 3/4 0.75 4
mass of 1 mole glucose = 12x6 + 1x12+16x6 1/5 0.20 5
2/5 0.40 5
= 180 g mol-1
12x6 3/5 0.60 5
% of carbon = -------- x 100 =40.0% 4/5 0.80 5
180
% of hydrogen = x 100 = 6.67%
180
Table 1.3 A Summary of Terms Used
Term Definition or usage
Atomic weight standard An atomic weight of 12.00000 is arbitrarily assigned to 162 C.
Isotopic mass (nuclidic mass) The mass, in atomic mass units, u, of a single atom, on a scale in which the mass of an atom of 162C is arbitrarily
defined as 12.00000 u (e.g., p Cl has an isotopic mass of 34.968852 u).
Atomic weight (relative A dimensionless (pure) number that expresses the mass of the naturally occurring mixture of isotopes of an
atomic weight) element, relative to an arbitrarily assigned atomic weight of 12.00000 for carbon-12. These are the values listed in a
table of atomic weights (e.g., atomic weight of Cl= 35.4527).
Formula weight (relative A dimensionless (pure) number that expresses the mass of a formula unit of a compound, relative to the atomic
formula weight) weight standard, carbon-12 (e.g., formula weight of NaCI = 58.44). Tabulated atomic weights are used in
computing formula weights.
Molecular weight (relative A dimensionless (pure) number that compares the mass of a molecule to the atomic weight standard, carbon-12
molecular weight) (e.g., molecular weight of CCI4 = 153.82). Tabulated atomic weights are used in computing molecular weights.
Mole An amount of substance containing the same number of elementary units (6.02214x 1023) as there are I2 C atoms in
12.00000 g ’2C.
Molar mass (molar weight, The mass of one mole of a substance, whether the substance is composed of individual atoms (e.g., 35.45 g
mole weight)__________ Cl/mol Cl), formula units (e.g., 58.44 g NaCI/mol NaCI), or molecules (e.g., 153.82 g CCI4/mol CCI4).’

•Where ever the term “weight” occurs, the term “mass” is equally (or even more) appropriate, e.g., atomic weight = atomic mass.
Chapter 1: Fundamental Concepts of Inorganic Chemistry 15

Practice Exercise 2
1. Write the atomic masses of the following in amu. 7. Convert the following per cent composition into molecular
(a) 23 Na (b) “C formulae:
(a) Diborane H: 21.86%, B : 78.14%,
(c) 1I4N (d) 27 Al
AO
Mol. wt.: 28 g mol'1
(e) l2Mg21- containing 10 electrons (b) Trioxan C : 40.00%, H : 6.71%,
(0 8O2-containing 10 electrons 0 : 53.28%, Mol wt.: 90 g mol"1
2. Which is heavier in each pair? 8. A12(SO4)3 xH2O has 8.20 per cent Al by mass. Calculate
(a) 1 amu of 12C or 1 amu of 13C value of x.
(b) 1 mol of23 Na or 1 mol of24 Na 9. Select correct alternate. Only one alternate is correct.
(c) 0.5 mol of 4He or 1 mol of 2H (A) If the equivalent weight of an element is 32, then the
3. The “lead” in lead pencils is actually almost pure carbon percentage of oxygen in its oxide is
and the mass of a period mark made by a lead pencil is (a) 16 (b) 40
about 0.0001 g. How many carbon atoms are in the period? (c) 32 (d) 20
4. A binary compound of zinc and sulphur contains 67.1% zinc (B) A hydrocarbon has 3 g carbon per gram of hydrogen,
by mass. What is the ratio of zinc and sulphur atoms in the hence simplest formula is
compound? (Zn = 65; S = 32)
(a) CH4 (b) CgH6
5. What is the empirical formula and what is the per cent
(0 C,H„ (d) CH,
composition of dimethylhydrazine, C^N^ a colourless JO Z

liquid used as a rocket fuel? (C) Molar ratio ofNa2S03 andH2Ois 1:7 in Na2SO3 xH2O.
6. What is the empirical formula of an ingredient in bufferin Hence, mass percentage is
tablets that has the per cent composition? (a) 12.5 : 87.5 (b) 87.5 :12.5
C : 14.25% O : 56.93%, Mg: 28.83% by mass (c) 50 : 50 (d) 75 : 25

Answers
1. (a) 23 amu (b) 14 amu (c) 14 amu (d) 27 amu 4. Zn : S :: 1:1
(e) 24 amu (f) 16 amu 5. Empirical formula : CH4N, C : 40%, H : 13.3%, N : 46.7%
2. (a) lamuof13C (b) 1 mol of24 Na (c) equal mass 6. CO,Mg
O'-*
7. (a) B,H
B2H6 (b) C,H6O,
Zu Q u
8. x = 18
3. 5.018 x 1018 atoms 9. A. (d) B. (a) C. (c)
- -

Total Practice Set


(Read, Plan & Solve\)

Problem 1. Analysis of chlorophyll shows that it Problem 3. Myoglobin stores oxygen for metabolic
contains 2.68 per cent magnesium. How many atoms of processes in muscle. Chemical analysis shows that it
magnesium does 100 g of chlorophyll contain? contains 0.34% Fe by mass. What is the molar mass of
myoglobin? (There is one Fe atom per molecule).
Plan First we determine moles of magnesium in 1 g of
chlorophyll and Plan Every myoglobin molecule has one Fe atom. Every
number of ~M.g atoms = mol x No 100 g of it has 0.34 g Fe hence, molar mass of myoglobin
containing 56 g Fe per mol can be determined.
Solution Mg in chlorophyll is 2.68%
2 AR Solution If 0.34 g Fe is present then myoglobin
Thus, 100 g chlorophyll = 100 x g Mg
= 100 g
2-68 ...
= — ~ mol Mg If 56 g of Fe is present then myoglobin = x 56
_2.687V0
atoms of Mg = 16470.6 g mol-1
24
2.68 x 6.022 x 1023
Problem 4. A compound X contains 63.3 per cent
24 manganese (Mn) and 36.7 per cent oxygen by mass. When
= 6.7246 x 1022 atoms X is heated, oxygen gas is evolved and a new compound Y
containing 72.0 per cent Mn and 28.0 per cent O is formed,
Problem 2. Carbohydrates are compounds contain­ (a) Determine the empirical formula of X and Y. (b) Write
ing carbon, hydrogen and oxygen in which the hydrogen to a balanced equation for the conversion of X to Y.
oxygen ratio is 2 : 1. A certain carbohydrate contain 40% (Mn = 55, O = 16)
carbon by mass. Derive molecular formula of the
compound if molar mass is 180 g mol-1. Plan From the percentage of Mn and O, empirical
formula ofX and Y are derived.
Plan Molar ratio of hydrogen and oxygen and mass per
cent of carbon have been given. Molecular weight and Solution For the compound X
molecular formula are thus derived.
mol. %
% Simplest ratio
Solution Let carbohydrate be C^HgO^ where x and y atomic mass

are whole numbers. Mn 63.3 1.15 1


12x + 18y = 180 0 36.7 2.29 2
Also 180 g of the carbohydrate has = 12 x carbon Empirical formula: MnO2
100 g of the carbohydrate has
12xxl00„ For the compound Y
=----------- % carbon Whole
180 mol.____*___
1200x % Simplest ratio number
atomic mass
= 40 ratio
180 3~
Mn 72.0 1.31 1
x=6 O 28.0 1.75 1.33 4
12x6 + 18y = 180
y=6 Empirical formula : Mn3O4
Carbohydrate is = Cgd^Og 3MnO2 ---- > Mn3O4 + O2
= C6H12O6
Chapter 1 : Fundamental Concepts of Inorganic Chemistry I 17

Problem 5. Ferrocene, a substance proposed for use P present in one mole= 31 g


as a gasoline additive, has the per cent composition 5.42% 31
% of P inH:f04 = x 100 = 31.63%
H, 64.56% C, and 30.02% Fe. What is the empirical 98
formula of ferrocene? 0 present in one mole= 64 g
Plan Per cent composition is to be converted to moles, 64
% of O in one mole= x 100=65.31%
then to molar ratio and then into integral mole ratio (if it is 98
fraction).
Problem 7. What is the ratio of the masses of oxygen
Solution that are combined with 1.08 g of nitrogen in the
Mol=
compounds N203 and NO?
Element % mole ratio
atomic mass
Plan First we calculate the mass of O that combines with
12c
64.56 5.38 10 one gram of N in each compound. Then we determine the
5.42 5.42 10 g(O)
IH ratio of the values of for two compounds.
56Fe 3002 0.54 g(N)

Empirical formula is C10H10Fe. Solution In N2O3 : ?g(O) - 48·0 g(O) = l.7l g(O)
g(N) 28.0 g(N) g(N)
Problem 5. Phosphoric acid (H:f04 } is a colourless, In NO
: ? g(O) 16. g(O) 1.14 g(O)
= =
syrupy liquid used in detergents, fertilizer, tooth pastes, gN 14.0 g(N) g(N)
and in carbonated beverages for a "tangy" flavour. O)
Calculate the per cent composition by mass of H, P and 0 The ratio is g( in (N203 ),
in this compound.(H= l,P= 31,0= 16)
g(N) t
1.71 g(O) / g(N) 1.5 -�
Plan In one mole(= 98 g) amounts of H, P an,d Oare =
(1.14 g(O) I g(N) 1.0 2
known, hence their mass per cent can be calculated. g(O)
in (NO}
Solution One mole ofH:f04 = 1 mole x 98 g moi-1 g(N)
=98g Thus, we observe that the ratio is 3 mass units g(O)
H present in one mole= 3 g in (N203) to 2 mass units of O (in NO or N202). Thia
., represents law of multiple proportions.
% of H in H:f04 = .::!_ x 100 = 3.06%
98
Master Exercises - - - - -

Exercise 1
(Stage 1 : Learning)
Short Answer Type Questions 12. What is the relation between specific heat and atomic
weight?
1. What is the name of the following process?
gas -----+ solid 13. The specific heat of a metal M is 0.25. Its equivalent
weight is 12.
2. What is the valency of carbonate if its equivalent
weight is 30 g equiv- 1 ? (a) What is its atomic weight?
3. 24 g of magnesium combines with oxygen forming (b) What is the formulas of its (i) chloride,
40 g of oxide. What is equivalent weight of (ii) sulphate and (iii) phosphate?
magnesium? 14. Calculate number of moles and atoms of each in 1.8 g
4. 27 g of aluminium combines with hydrogen forming
30 g of hydride. What is equivalent weight of CsH120e.
aluminium?
5. H:fO2 only forms Na�PO2 . What is its equivalent Analytical Questions
weight of acid? Law of Chemical Combination
6. H:fO3 forms Na�HPO3 and Na2HPO3. What is 1. Benzene, ethane and ethylene are just three of a
equivalent weight of acid at each stage of salt large number of hydrocarbons-compounds that
formation? contain only carbon and hydrogen. Show how the
7. H:fO4 formsNa�PO4 ,Na2HPO4 andNa;?O4 . What following data are consistent with the law of multiple
is equivalent weight of acid at each stage of salt proportions.
formation?
8. When a piece ofsulphur is burnt in air, it disappears. Mass of carbon In Mass of hydrogen In
Compound 5.00 g sample
5.00 g sample
How could you prove that it is not destroyed?
Benzene 4.61 g 0.39g
9. Taking VD of hydrogen as 1, determine
Ethane 4.00 g 1.00 g
(a) VD ofCH4 (b) VD ofO2
Ethylene 4.29g 0.71 g
(c) VD ofCO2
10. Calculate number of equivalents in 2. Ammonia,NH 3 and hydrazine,N2H4 , are both binary
(a) 1.6 g 02 gas (b) 0.49 g HiSO4 compounds of nitrogen and hydrogen. Based on the
(c) 1.80 g �O (d) 1.6 g CH4 law of multiple proportions, how many grams of
11. Calculate equivalent weights ofthe underlined based hydrogen would you expect 2.34 g of nitrogen to
on following reaction : (Take molecular weight = M) combine with to yield ammonia? To yield hydrazine.
(a) � + 3� -----+ �3 3. If 3.670 g of nitrogen combine with 0.5275 g of
hydrogen to yield compound X, how many grams of
(b) Ca(OH)2 +HJJO4 � CaHPO4
nitrogen would combine with 1.575 g of hydrogen to .
(c) Zn + �SO4 � ZnSO4 + � make the same compound? ls X ammonia or
(d) 2FeSO4 + �SO4 + 0 -----+ Fe2 (SO4 ) 3 + �O hydrazine?
Chapter 1: Fundamental Concepts of Inorganic Chemistry | 19

Mole 12. Disilane, Si2Hx, is analysed and found to contain


4. The table below gives number of electrons, protons 90.28% by weight silicon. What is the value of x ?
and neutrons in atoms or ions of a number of (Si= 28)
elements. Answer the following : 13. Cytochrome C is an iron-containing enzyme found in
Atom or Ion of element A B C D E F G the cells of all aerobic organisms. If cytochrome C is
Number of electrons 7 10 7s 28 36 7T 0.43% Fe by weight, what is its minimum molecular
Number of protons 5 7 19 30 35 5 9 weight?
Number of neutrons 5 7 20 36 46 6 10 14. What is empirical formula of stannous fluoride, a
(a) Which of the species are neutral? compound added to tooth paste to protect teeth
(b) Which are negatively charged? against decay? Its mass per cent composition is
(c) Which are positively charged? 24.25% F, 75.75% Sn.
(d) What are the conventional symbols for all the 15. Analysis of a metal chloride XC13 shows that it
species? contains 67.2% Cl by mass. Calculate the molar mass
5. The carat is the unit of mass used by jewellers. One of X and identify the element.
carat is exactly 200 mg. How many carbon atoms are 16. A sample of a compound of nitrogen (N) and oxygen
present in a 24 carat diamond? (O) contains 1.52 g of N and 3.47 g of O. The molar
6. The natural abundance of the two stable isotopes of mass of this compound is known to be between 90 g
hydrogen (hydrogen and deuterium) are |H: 99.985% and 95 g. Determine the molecular formula and
and 2H : 0.015%. Assume that water exists as either molar mass of this compound. (N = 14,0 = 16)
or D2O. Calculate the number of D2O molecules
in exactly 400 mL water. (Density = 1.00 g mL-1) NCERT Problems
7. A solution containing 10.0 millimol of CaCl2 is 1. Calculate the molar mass of
diluted to 1 L. Calculate the number of grams of (i) H>0 (ii) CO2
CaCl2 • 2H2O per mL of final solution.
(iii) CH4
Percentage Composition and Empirical Formula 2. Determine the empirical formula of an oxide of iron
8. All of the substance fisted below are fertilizers that which has 69.9% iron and 30.1% oxygen (Fe= 55.85
contribute nitrogen to the soil. Which of these is the amu;0= 16.00 amu).
richest source of nitrogen on a mass percentage 3. How much copper can be obtained from 100 g of
basis? anhydrous copper sulphate (CuSO4)? Cu= 63.5;
(a) Urea,(NH2)2CO S = 32,0 = 16 (all in amu).
(b) Ammonium nitrate, NH4NO3
4. Calculate the following in three moles of ethane gas
(c) Guanidine, HNC(NH2)2
(d) Ammonia, NH3
(W
(i) moles of carbon atoms
9. The aluminium sulphate hydrate [A12(SO4)3 xHjO] (ii) moles of hydrogen atoms
contains 8.20 per cent Al by mass. Calculate x, that is, (iii) molecules of ethane
the number of water molecules associated with each 5. Which one of the following will have largest number
A12(SO4)3 units. ofatoms?[Au= 197,Na= 23,Li= 7,C1= 35.5inamu]
10. Calculate the per cent composition by mass of all the (i) lgAu(s) (ii) IgNa(s)
elements in calcium phosphate Ca3(PO4)2. (iii) IgLi(s) (iv) lgCl2(g)
11. Platinum forms two different compounds with 6. What will be the mass of one 12 C atom in gram?
chlorine. One contains 26.7 per cent Cl by mass, and 7. Calculate the number of atoms in each of the
the other contains 42.1 per cent Cl by mass. following:
Determine the empirical formulae of the two (i) 52 moles He (ii) 52 u of He
compounds. (Pt = 195) (iii) 52 g He
20 | Essential Inorganic Chemistry

Exercise 2
(Stage 2: High Skill Problem Solving)
Only One Option Correct 11. Na2SO3 xH2O has 50% H2O by mass. Hence, x is
(a) 4 (b) 5
1. In which of the following pairs do the two species
(c) 6 (d) 7
resemble each other most closely in chemical
properties? 12. Mass of one atom of X is 6.66 x 10i23 g. Hence,
(a) |Hand2H (b) ^0 and ^O2' number of mole of atom X in 40 kg is
(a) 103 mol (b) 10“3 mol
(c) ^Mg and ?^Mg.2+
2+ (d) 74N and 74N3~
40 x 103 40 x 103
2. One isotope of a non-metallic element has mass (c) mol (d) mol
6.66 x 10’23 6.022 x 1023
number 127 and 74 neutrons in the nucleus. The
anion derived from the isotope has 54 electrons. 13. To make 0.01 mol, which of the following has
Hence, symbol for the anion is maximum mass?
(a) g7X" (b) 127 X' (a) NaHCO3 (b) Na2CO3
(0 gx- (d) 5’44X- (c) Na2SO4 (d) Na2C2O4
3. Which of the following is the richest source of 14. In a glass-tube, there is 18 g of glucose. 0.08 mol of
ammonia on a mass percentage basis? glucose is taken. Glucose left in the glass-tube is
(a) NH4NO3 (b) NH2CONH2 (a) 0.10 g (b) 17.92 g
(c) 3.60 mol (d) 3.60 g
(c) NH3 (d) HNC(NH2)2 Qi

15. Rest mass of an electron is 9.11 x 10 kg. Molar


4. Which of the following substances contains greatest mass of the electron is
mass of chlorine?
(a) 1.50 x 10"31 kg mol (b) 9.11 x 10~31 kg mol
(a) 5.0gCl2 (b)0.5molCl2
(c) 5.5 x 10-7 kg mor1 (d) 6.02 x 1023 kg mol"
(c) 0.10 mol KC1 (d) 30.0gMgCl2
16. A sample of ammonium phosphate (NH4)3PO4
5. When 0.273 g of Mg is heated strongly in a nitrogen
contains 3.18 moles of H atoms. The number of moles
(N2) atmosphere, 0.378 g of the compound is formed.
of O atoms in the sample is
Hence, compound formed is
(a) 0.265 (b) 0.795
(a) Mg3N2 (b) Mg3N (c) 1.06 (d) 3.18
(c) Mg2N3 (d) MgN
17. A sample ofCuSO4 contains 3.782 g of Cu. How
6. A certain metal sulphide, AfS2, is used extensively as many grams of oxygen are in this sample? (Cu = 63.5)
a high temperature lubricant. If MS2 is 40.06% by (a) 0.952 g (b) 3.80 g
mass sulphur, metal M has atomic mass (c) 4.761 g (d) 8.576 g
(a) 160 amu (b) 64 amu
(c) 40 amu (d) 96 amu 18. Cortisone is a molecular substance containing
21 atoms of carbon per molecule. The mass percentage
7. The molar mass of a compound if 0.372 mol of it has a of carbon in cortisone is 69.98%. Its molar mass is
mass of 186 g is (a) 176.5 (b) 252.2
(a) 200 g (b) 372 g
(c) 500 g (d) 186 g (c) 287.6 (d) 360.1
8. Which of the following has maximum number of C 19. In a hydrocarbon C is 3 g per g of hydrogen. Hence,
atoms? hydrocarbon is
(a) 4.4gCO2 (b) 3.0gC2H6 (a) CH4 (b) q>H6
(c) 4.4gC3H8 (d) 1.3gC6H6 (c) C3H8 (d) C4H10
9. Number of atoms present in 1.8 g HjO, 1.7 g NH3 and 20. Which has maximum percentage of Cl?
1.6 g CH4, in the increasing order is (a) C6H6C16 (b) CHC13
(a) HjO < NH3 < CH4 (b) CH4 < NH3 < HsO (c) CgHgCl (d) CH3C1
(d) CH4 =NH3=H2O (d) NH3 < CH3 < HjO
21. A spherical ball of radius 7 cm contains 56% iron. If
10. Number of atoms in 20 g Ca is equal to number of density is 1.4 g/cm3, number of moles of Fe present
atoms is approximately is
(a) 20 g Mg (b) 1.6gCH4 (a) 10 (b) 15
(c) l.SgHsO (d) 1.7gNH3 (c) 20 (d) 25
Chapter 1 Fundamental Concepts of Inorganic Chemistry | 21

22. In the following final result is 0.1 mol (b) 46 g CaCl2 would contain the same number of
CH4 + 3.01 x 1023 molecules CH4 - 9.6 g CH4 =x mol chloride ions as are contained in 48.6 g NaCl
H atoms (c) 17 gNH3 would contain the same volume ofHj at
(a) 0 mol H atom (b) 0.2 mol H atom NTP as are contained in 18 g H2O at NTP
(c) 0.3 mol H atom (d) 0.4 mol H atom (d) 1 mole CO would combine with same mole ofO2 to
23. If we assume that No = 1.2 x 1023 mol-1, then molar form CO2 as required by 1 mole of SO2 to form SO3
mass of 092 will be taken as QO 6. Equal mass of water is lost on heating of 1 mole of
(a) 32 g moK1 (b) — g mol"1 each of
6 (a) NaHCO3 (b) Ca(HCO3)2
lx 1023 (c) KHCO3 ’(d) CuSO4 -5H2O
(c) 32 x IO23 gmol (d) gmol
32
7. Empirical formula will be same in with following
24. CO, CO2, CgO3 follow compounds
(a) law of definite proportion (a) CH3CO2H (b) C6(H2O)6
(b) law of multiple proportion (c) HCHO (d) C12(H2O)n
(c) law of conservation of mass
(d) all of the above 8. Consider following species
I: CO; II: NO;
25. In a gas S and 0 are 50% by mass, hence, their mol III:N2; IV: HCHO
ratio is
(a) 1:1 (b) 1:2 Select species with equal number of neutrons,
(c) 2 : 1 (d) 3 :1 protons and electrons in each.
(a) CO,N2 (b) CO, HCHO
One or More Than One Options Correct (c) NO, HCHO (d) N2,NO,CO

1. Equal moles of the substance are present in 9. Which of the following may contain two neutrons and
two protons?
(a) 1.6gCH4 (b) 1.7 gNH3
(a) He (b) D2
(c) l.SgH^O (d) 1.2 g Mg
(c) He2* (d) H>
2. In which cases law of conservation of mass are
followed? io. |2C and g4C resemble in
(a) 10 g CaCO3 on heating gives 0.1 mole CaO and (a) number of protons (b) number of electrons
2.24 LCO2 at NTP (c) number of neutrons (d) chemical properties '
(b) 1 mole N2 and 1 mole Hj combine to form 2 moles
ofNH3 Brain Twisters
(c) 0.2 mole SO2 combines with 3.2 g O2 to form 1. What is equivalent weight of H3O+?
0.2 moleSO3
(d) 1 gHj combines with 0.5 gO2 to form 1.5 gHgO 2. Equivalent weight of
. . TT molecular weight
3. One formula unit of a substance (underlined) is (a) HgPO2 =--------- ------ —
present in each of ... rr nn molecular weight
(D) ------------------
(a) 200 g CaCO3 with 50% impurity A
(b) 38 g H2O and D2O with 1 mole D2O , x u nn molecular weight
(c) H3rU3 =--------- -
(c) 360 g C12H22O11 and C6H12O6 with 0.1 mole o
^6^12^6 Explain the difference.
(d) 46 gN2H4CO with 60% purity 3. Determine equivalent weight of carbon in the
following:
4. Which represents one mole? (a) C + O2 ---- > CO2
(a) 22.4 L He at STP (b) C + 2H2 ---- > CH4
(b) 16gO2
(c) 2C + Ha ---- > QHo
(c) 158gKMnO4
(d) 2C + 2H2 ---- > C^H4
(d) 392 gFeSO4 (NH4)2SO4 ei^O
I (e) 2C + 3H2 ---- > C^H6
5. Select correct altemate(s). (f) 3C + 4HJ ---- > CjHg
(a) 23.5 gSO2 would contain the same mass of oxygen
as is contained in 33.7 g of As2O5 (As= 75) What do you concludes about equivalent weight?
22 | Essential Inorganic Chemistry

4. Equivalent weight of two oxides A and B of sulphur Passage 2


are 16 and 13.33 g equivalent"1 respectively. What Study the following passage regarding fertilizer K^O
are these oxides? and answer the questions at the end of it.
5. Equivalent weights of two hydrides of nitrogen are Potash is any potassium mineral that is used for its
5.66 and 8. Derive formulae of hydride. potassium content. Most of the potash produced in the
6. A solution contains 200 g ppm CaCO3. Express this United States goes into fertilizer. The major sources of
concentration in potash are potassium chloride (KC1) and potassium
(a) millimoles per gram sulphate (K2SO4). Potash production is often reported as
(b) number of CaCO3 molecules per gram the potassium oxide (K2O) equivalent or the amount of
K2O that could be made from a given mineral. KC1 costs
7. How many carbon atoms are present in a diamond ? 50 per kg.
weighing 200 mg and of 20 carat purity?
8. 100 g of ethyl alcohol-water mixture contains 1 mole 1. What is the cost of K per mol of the KC1 sample?
of ethyl alcohol. What is molar ratio of ethyl alcohol (a) ? 13.42 mol’1 (b) ? 3.73 mol"1
and water in the mixture? (c) ? 1.00 mol"1 (d) ? 2.00 mol”1
9. Students A and B derived equivalent weights of 2. For what price must K2SO4 be sold in order to supply
KHC2O4 quite different. Explain. the same amount of potassium as in KC1?
10. 0.24 g of magnesium was burnt in the atmosphere of (a) ? 58.40 kg-1 (b) ? 50.00 kg-1
oxygen when residue formed weighed 0.40 g. Derive (c) ? 42.82 kg"1 (d) ? 25.00 kg’1
equivalent weight of magnesium.
3. What mass (in kg) of K2O contains the same number
11. Cost of a fertilizer is based on the percentage of
nitrogen in it. Which of the fertilizers should be of moles of K atoms as 1.00 kg KC1?
priced maximum? (a) 0.158 kg (b) 0.315 kg
(c) 1.262 kg (d) 0.631kg
(a) NH4NO3 (b) NT^CONH)
(c) KNO3 (d) NH4C1 Passage 3
12. One equivalent of calcium phosphate Ca3(PO4)2 is
Study the following observations and answer the
dissolved in HC1 questions at the end of it.
Ca3(PO4)2 +6HC1---- > 3CaCl2 + 2H3PO4 The mass spectrum (given below) of magnesium has
(a) How many moles of CaCl2 are formed? three peaks, which indicates that magnesium has three
(b) How many moles of H3PO4 are formed? isotopes. The questions given below are based on this mass
spectrum.
Passage Comprehension Questions -100

Passage 1 <D
O
C -80
Study the following observations and answer the CO

questions at the end of it. c


.D -60
The following is a crude but effective method for CO
0)
estimating the order of magnitude of Avogadro’s number >
using stearic acid (C^H^Og). When stearic acid is added Io -40
to water, its molecules collect at the surface and form a tr
monolayer; that is, the layer is only one molecule thick. -20
The cross-sectional area of each stearic acid molecule has
been measured to be 0.21 nm2. In one experiment it is , I
24 25 26
found that 1.4 x IO-4 g of stearic acid is needed to form a
Atomic mass (amu)
monolayer over water in a dish of diameter 20 cm. (the
area of a circle of radius r is nr2.) 1. Which isotope has maximum number of atoms per
1. Based on these measurements value of Avogadro’s gram of each?
number is (a) Mg-24 (b) Mg-25
(a) 3 x10s* (b)exlO23 (c) Mg-26 (d) Equal
(c) 4 x Iff* (d) 1 x 10s* 2. Number of atoms in one mole of each isotope is placed
2. What is equivalent of 1g H in amu for this value of in increasing order
Avogadro’s number? (a) 24Mg<25Mg<26Mg
(a) 1.66 x 10’24 g (b) 3.33 x 10’24 S
(d) lx IO"23 g (b) 26Mg<25Mg<24Mg
(c) 2.5 x 10'24 g
Chapter 1: Fundamental Concepts of Inorganic Chemistry | 23

(c) 26Mg=25Mg=24Mg Fill in the Blanks


(d) given data is insufficient
1. NO and NO2 follow the law of
3. Average atomic mass of Mg is approximately
(a) 25.0 (b) 24.3 2. SO3 contains sulphur and oxygen in the mass ratio of
(c) 25.2 (d) 25.8
3. Hydrocarbon containing 4 g carbon per gram
Assertion & Reason hydrogen is
4. If X2r is isotonic (same number of neutrons) of E^O
Codes:
(a) Both A and R are true and R is the correct and contains 10 electrons then ionic mass of X2+ is
explanation of A.
(b) Both A and R are true but R is not the correct 5. 1 mole of barium nitrate and 1 mole of ammonium
explanation of A, phosphate form mole of barium phosphate.
(c) A is true but R is false. 6. Percentage of sodium sulphite (Na2SO3) in hydrated
(d) A is false but R is true. salt is 50. Hence, molecular formula of hydrated salt
is
1. Assertion (A): When 10 g of CaCO3 is decomposed,
7. Concentration 1 ppm = ppb.
5.6 g of residue is left and 4.4 g of CO2 escapes. 8. Concentration 1 microgram per gram =
Reason (R): Law of mass conservation is followed. ppm.
2. Assertion (A) : SO2 and SO3 obey “law of 9. E^O, HjS and SO2 follow law of
multiple proportion”. 10. Atomic mass unit is also called
Reason (R): Every sample of SO2 contains 1 part
sulphur and 1 part oxygen by mass. Matrix-Match Type Questions
3. Assertion (A): SO2 from oxidation of sulphur or H2S I. Only One Correct
contains sulphur and oxygen in the mass ratio of 1: 2. 1. Match % of carbon (in List I) with the compound
Reason (R): Different samples of a pure chemical (in List II).
substance always contain the same proportions of
elements by mass. List! List II
A. 20% 1. ch4
4. Assertion (A): In Fe2+, there are 24 electrons and
30 neutrons and thus, ionic mass is 56. B. 52.2% 2. co2
Reason (R): Ionic mass= neutron + proton C. 75% 3. C2H6O
and proton = electrons in neutral species D. 27.3% 4. cn2oh4
E. 12% 5. CaCO3
True & False
1. If there are 10 electrons and 8 neutrons in X2~, then 2. Match Laws of chemical combinations (in List I) with
its ionic mass is 18. the examples followed (in List II).
2. SO2 contains sulphur and oxygen in 1:1 mass ratio. List! List II
3. 20 g CaCO3 (50% pure) on strongly heating gives 5.6 g A. Law of conservation of 1. CH.l4 has carbon and
residue. mass hydrogen in 3:1 mass
ratio.
4. Mass of 1 mole of electrons is 5.48 x 10-7 kg. B. Law of multiple proportion 2. 10 mL N2 combines with
5. Number of atoms in 1.6 g CH4, 1.7 g NH3 and 1.8 g 30 mL of H2 to form 20 mL
ofNH3.
Eire equal.
C. Law of definite proportion 3. S and O2 combine to form
6. If 1 mole ofHg is taken out from 2 gH2, 1 gH2 is left. SO2 and SO3.
7. 1 amu is equal to 1.66 x 10-24 g- D. Law of reciprocal 4. In H2S and SO2 mass ratio
proportion of H and O w.r.t. sulphur is
8. Out of CH3C1, C6H6C1 6, and CC13CHO, maximum 1:16, hence in H2O, mass
ratio of H and O is 1:8.
percentage of chlorine is in C6H6C16.
E. Gay Lussac's law 5. 4.2 g MgCO3 gives 2.0 g
9. 1 ppm is equal to 1 mg per kg. residue on heating.
10. Equivalent mass of NH3 is 17.
24 | Essential Inorganic Chemistry

II. One or More Correct 2. Na2SO3 • yH^O has 50% H2O by mass. Thus, y is
1. Match the item in Column I with the compounds in 3. If Avogadro’s number would have been lx IO10,
Column II. instead of 6.02 x IO23, then mass of one atom of H
Column I Column II would be amu.
A. Same % of C (40%) 1. ch3cooh 4. Sum of neutrons and electrons in H2+ is
B. Same % of H (6.67%) 2. nh2conh2
5. When neutron changes to proton, atomic number
C. Same % of 0 (53.3%) 3. HCHO
changes by unit.
D. Same empirical formula 4. CH3CH2CH2OH
5. hcooch3
6. When 1 mole of Na2CO3 is heated, CO2 lost is
mole.
2. Match the item in Column I with the given amount in 7. Empirical formula of substance with molar mass 180
Column II. is CH2O.Thus, ratio of molecular formula to empirical
Column I Column II formula is
A. 1 equivalent 1. 8 g oxygen (0) 8. When 2.495 g of CuSO4 • xH20 (molar mass 249.5) is
B. 0.5 mole 2. 1 g hydrogen (H)
heated 0.05 mole of H2O is lost. Thus, x is
C. N2-
— *
o atoms 3. 35.5 g chlorine (Cl2)
2 Set II
D. No molecules 4. 66.0gH3PO2
1. In Japan, radioactive water is leaking into sea from
5. 49 g H2SO4
nuclear reactors (April 04, 2011). 0.060 kg of
radioactive water has number of moles =
Integer Answer Types 2. Cl and 3? Cl are two isotopes of chlorine. If atomic
This section contains 8 questions. The answer to each mass of chlorine is 35.5, then mass ratio of these
of the questions is a single digit integer, ranging from 0 to isotopes is x : 1 where x =
9. The appropriate bubbles below the respectively
question numbers in the ORS have to be darkened. For 3. Mass of one atom of element is 6.66 x 10~23 g. Number
example, if the correct answers to question number X, Y, Z of gram atoms in 0.320 kg of element would be
and W (say) are 6,0,9 and 2, respectively, then the correct
darkening of bubbles will like the following.
4. X2~ is isoelectronic of 10Ne. 1.6 g of X2 has 0.1 mole.
X Y Z W
Thus, there are neutrons is X2~.
© © © ©
© © © 5. In the following reaction,
© ©. © © SO2(g) + 2H2S(g) » 3S(s) +21^0(1)
© © © © 2 moles each of SO2 and l^S forms sulphur =
© © © © moles
© © © ©
©. © © © 6. In the lightest isotope of hydrogen, number of
© © 0 0 neutrons present is
© © © © 7. Total number of ions in aluminium phosphate is
© © © ©

Set I 8. 11.2 g of Fe completely react with 3.36 L of O2 gas


under standard condition to formFe_Ov. Thus,(x + y)
1. 1 g of hydrogen has x g of carbon in methane. This x is is
Answers & Solutions
Numerical Problem Solving
1. Mass of reactants = mass of products 32g CO2 s 28 g N2
17 + 7.45 = 14.35 + KNO3(x) 8 g O2 — 7g N2
x = 10.10 g Thus, equivalent mass of N2 = 7 g equivalent’
2. Mg + 1 O2 —> MgO 28
Hence, equivalent of N2 used = — = 4

24 g
(?)• Equivalent mass ofO2 - 8 g equivalent
32
Hence, equivalent used of O2 = — = 4
12 g 16 g
8
? 32 g
32gO2 =2x30gNO
Mg= 24 g. MgO has constant composition.
8 g O2 = 15 g NO
3. (i) C + - O2 ---- > CO2 Thus, equivalent mass of NO= 15 g equivalent
2
Hence, equivalent of NO formed =4
12 g 16 g
16 N2 O2 NO
lg 12 S
(a) 7 8 15
C + 0.2 ' CO2
(b) 1 1 1
12 g 32 g
32 (c) 1 1 2
lg 12g (d) 1 1 2
Ratio of oxygen in CO and CO2 = = 1:2
7.(i) (a) 2gH2 = 26gMgH2
(ii)
IgH^lSgMglL,
Amount of hydrogen
Carbon Hydrogen combines with 12 g Ratio (b) 16 g O2 = 40 g MgO
carbon 8 g O2 =20 g MgO
24 g 2g Tg " 1
24 g 4g 2g 2 (c) 32gO2 = 44gCO2
24 g 6g 3g 3 8g02=llgC02
12g 4g 4g 4
(d) 32gO2 = 46gNO2
Ratio of hydrogen in C^, C^, C^Hg and CH4 that
8 g O2 =11.5 g NO2
combines with 12 g carbon is = 1:2:3 :4.
Thus, law of multiple proportion is followed. (ii) (a) Equivalent of weight HC1 = 36.5
3 65
4. 1:35.5 hence, equivalents of HC1 = —— = 0.1
36.5
24
5. (a) Mg= — = 12 g equivalent-■1
”2 (b) Equivalent weight of H2SO4 = 49
(b) Cl2 = 35.5 g equivalent-1 98
hence, equivalent ofH2SO4 =-^ = 0.2
40
(c) Ca=—= 20 g equivalent
2 98
(c) Equivalent weight ofHgPO4 =—
6. N2(g) + O2(g) —> 2NO(g) 3
98
mass 28 g 32g 2 x 30 g hence, equivalents ofH.jP04 =—:— = 0.3
98/3
mole 1 mol 1 mol 2 mol
equivalent 4 4 4 (iii) g'Co | 27 Co | ®?Co (Z(Co) = 27)
22.4 L 22.4 L 2 x 22.4 L at NTP
26 | Essential Inorganic Chemistry

Master Exercises
Exercise 1
1 RO
Short Answer Type Questions (c) —— = 0.2 (equivalent weight of H2O = 9)
9
1. Deposition
2. Since, carbonate is = 60 g mol-J (d) — = 0.4 (equivalent weight of CH4 = 4)
4
, , molar mass
u.v.u. 60 n
11. (a) N2 = 6H
hence, valency =-------- ------------ = — =2
equivalent mass 30
, „ N, 28 M
1H=—=—=—
3. Mass of magnesium = 24 g 6 6 6-
Mass of oxide = 40 g 2NH3 = 6H
Oxygen combined = 40- 24= 16 g Ijj _.2NH3 _ NH.i3 = 17 = Af
6 3 =3 3
16 g oxygen = 24 g magnesium
24 (b) H3PO4 = 2OH-
8 g oxygen = — x 8= 12 g
16
/. Equivalent weight of Mg = 12 2 2
(c) Zn=2H+
4. Mass of aluminium = 27 g
M
Equivalent weight of Zn = —
Mass of hydride = 30 g
Hydrogen combined = 3 g (d) 2FeSO4 = 1 (O) = 16 g= 2x 8 g
3 g hydrogen = 27 g aluminium .-. 8g(O)=lFeSO4
27 Equivalent weight of FeSO4=M
1 g hydrogen = — x 1 = 9 g
3 12. Specific heat x atomic weight = 6.4
Equivalent weight of Al = 9 (Dulong and Petit’s law)
5. HgPOs + NaOH---- > NaH^O.2 13. (a) Atomic weight x specific heat = 6.4
HgPO2=lNaOH 64
Atomic weight = 25.6
Thus, equivalent weight = molecular weight
atomic weight
= 66 g equivalent-1 ... Valency =—1AJ—-TT-1611-- =— = 2.13=2
equivalent weight 12.0
6. HgPO.l3 ---- > NaHrPOg ---- > Na2HPO3
(b) Thus, (i) MC12 (ii) MSO4 (iii) M2(PO4)2
HgPO3 = 1H+ (NaHjPOg) 1 8
14. 1.8 g C6H12O6 =—— mole CgH12O6
Equivalent weight = molecular weight loU

- 82 g equivalent-1 = 0.01 mole


HgPO;, = 2 H+ (NajHPOg) = 0.01 x 6 moles of carbon
. - . , . molecular weight = 0.01 x 6 x Nq atoms of carbon
Thus, equivalent weight =------------------ —
2 = 0.01 x 12 moles of hydrogen
= 41 g equivalent-1 = 0.01 x 12 x No atoms of hydrogen
Equivalent weight = 0.01 x 6 moles of oxygen
7. HgPO4 = NaH^O, = 1H+ = M =98 = 0.01 x 6x Nq atoms of oxygen
HgPO4 = Na2HPO4 = 2H+ = — =49
2 Analytical Questions
HgPO4 s NagPO4 = 3H+ = — =32.66 C H
3 1. Benzene 4.61 0.39
8. Sulphur changes to SO2. 12 1
9. (a) 8 (b) 16 (c) 22 Ethane 4 1
Ethylene 6 1
10. (a) 1.6 g O2 = — = 0.2 equivalent
8 Thus, mass ratio of C (w.r.t. H= 1 g) in benzene, ethane
0 49
(b) = 0.01 (equivalent weight of H^C^ = 49) and ethylene is 6:2:3
Chapter 1: Fundamental Concepts of Inorganic Chemistry | 27

2. For NH3, 0.5014 gH; for N^, 0.3342 g H. 11. I II


3. N = 3.6700 g % Mol Ratio % Mol Ratio
H = 0.5275 g Pt= 733 0.38 1 Pt= 57.9 0.30 1
Total mass of compound X =4.1975 g Cl = 26.7 0.75 2 Cl =42.1 1.18 4
Compound: PtCI2 PtCI4
N(%) = 87.43%
H(%) = 12.57% 12. x=6
% Ratio 13. Let molar mass be = M
N 87.43 6.24 1 If 0.43 g Fe then molar mass = 100 g
H 12.57 12.57 2
100x56
56 g Fe than molar mass =
Thus, X is N;jH4 0.43
N in y = 10.9578 g.Xis N^ = 13023.26 g mol-1
4. (a) Neutral species : A, F, G 14. SnF2
(h) Negatively charged : B, E 15. Cr: 52 g mol"1
(c) Positively charged : C, D
16. N2O4,92 g mo!
(d) Conventional symbols
10 *
5
14t»3-
7 D ,
39p+
19'-' >
66r\2+
30u >
Slp-
35^ » “F, i9
9k
r
NCERT Problems
5. One carat = 200 mg diamond
1. (i) 18 g mol (ii) 44 g mol (iii) 16 g mol"
24 carat = 200 x 24 x 10"3 g Integer ratio
2. % Ratio
200X24X10"3 , 1X ^-= 1.25
=------- —------- g atom (mol) Fe 69.9 1=2
55.85
= 0.4 g atom 0 30.1 = 1.88 1.5=3
16.00
= 0.4x6.02x 10i23
: atom
Thus, Fe2O3
= 2.408 x 1023 atoms
3. CuSO4 = 159.5 g mol = 63.5 g Cu
6. Water = 400 mL= 400 g
D2O = 400x —15 g Cu in 100 g CuSO4 = ^xl00
159.5
2 100
= 39.81 g
400x0.015 ,
=------------- moles 4. (i) 6 moles carbon
20 x 100
(ii) 18 moles hydrogen
= 1.8066 x 1021 molecules
(iii) 3N0 molecules ethane
7. 10 millimol CaCl2 • 2H2O in 1000 mL Atoms
Millimol per mL = -^-=0.01 5. moles in (i) —
1 A
1000 197 197
= 0.01x 10"3 mol 1 No
(ii)
= lx 10“5 mol 23 23
1
Mass of CaCl2 2H2O per mL = 147 x 10"5 g mL" (iii) z — (max)
7 7
= 1.47x 10"3 gmL
1 No
(iv) zy
8. (d) NH3 71 71
9. Molar of A12(SO4)3 xILjO = 342 + 18x 6. 6.02 x 1023 atoms = 12 g carbon
„ 54x100 12
% of Al =----------- = 8.2 1 atom = S
342 + 18x 6.02 x 1023

x = 18 7. (i) 52N0 (ii) ~x2V0=52


No
10. Ca: 38.71 0:41.29 P: 20.00
(iii) ~xJV0
4
28 | Essential Inorganic Chemistry

Exercise 2
Only One Option Correct Fe (56%) = - x — x (7)3 x 1.4 x — g = 1126.4
3 7 100
1. (a) 2. (b) 3. (c) 4. (b)
Moles = 20
5. Mg =0.273 g
Thus, (c)
Compound of Mg with N = 0.378 g
22. (a) 23. (a) 24. (b) 25. (b)
N-reacted =0.105 g
Gram Mole Ratio One or More Than One Options Correct
Mg 0.273 0.011375 1.51 3 1. (a, b, c) 2. (a, c) 3. (a, b, c)
N 0.105 0.0075 1 2 4. (a, c, d) 5. (a, b, d) 6. (a, c)
7. (a, b, c) 8. (d) 9. (a, b) 10. (a, b, d)
Thus, Mg3N2 (a)
6. MS2 = 2S Brain Twisters
M + 6Q 64
100 40.06 1. H3O+ H2O + H+
M + 64_ 64
1H+ = 1 H□3O+
100 ~ 40.06
Equivalent weight of H3O+ = molecular weight
M=96
= 19 g equivalent-1
Thus, (d) 2. Equivalent weight in acid is based on number of
7. (c) 8. (c) 9. (a) 10. (b) ionisable protons which is called Basicity. lonisable
11. Na2SO3-xH2O = 126 + 18x hydrogen is attached to electronegative atom as
18x O, andX(X=F, Cl, Br, I)
----------- = 0.5
126 + 18x (a) HgPOg has one ionisable hydrogen (monobasic acid)
H3PO2 H^O" + H+
x=7
hence, equivalent weight = molecular weight
Thus, (d)
•23 (b) HTO, HPO2" + 2H+
12. Mass of one atom = 6.66 x 10"~ g (dibasic)
Atomic mass = 6.66 x 10"23 x 6.02 x 1023 2H" = HgPOg
= 40 g mol-1 + H3PO3
1 H s—---- i
40000 g 2
Number of moles in 40 kg = = 1000 mol
40 g mol" TT . 1 x • ux molecular weight
Hence, equivalent weight =---------- -----------
Thus, (a)
(0 H3PO4 PO3- + 3H+
13. (c) 14. (d) 15. (c) (tribasic)
16. Number of moles of 0 atoms = - moles of H 3H+ = H TO.
3
3.18 mol 1H+b H3PO4
” 3
= 1.06 mol 3
, . . . . , . molecular weight
Thus, (c) hence, equivalent weight =-----------------2—
3
17. (d)
(In the Unit-Chemical Bonding, structures have been
18. Let molar mass be = M given; they can be used to predict ionisable hydrogens).
Mass of 21 carbon atoms = 252 0 O
252 x100 „
(h)-O—P—h
I @-O—P-O-®
% of carbon = = 69.98
M
M = 360.1 H H
Thus, (d) O
~ I
19. (a) 20. (b) ©-O-P-O-®
4 3 4 22
21. Volume of spherical ball = -nr3 = -x—x(7)'>3
3 3 7 ' ' 0
Mass = Vxd = ^ Xy x (7)3 x 1.4 g
(g)
Chapter 1: Fundamental Concepts of Inorganic Chemistry | 29

lonisable hydrogens (circled) are attached to oxygen x 1 number of nitrogen atom


atoms. y 2 number of hydrogen atom
3. (a) 32 g O2 s 12 g C Thus, hydride appears NH2 (unstable).
8 gO2s3gC Thus, actual hydride is (hydrazine).
(b) 4 g H2 = 12 g C 6. (a) 200 g parts per million (out of 106 g)
lgH=3gC 200 . _
(c) 2gH2s24gC = moles CaCO3 per million gram
lgH2sl2gC
= 2000 millimoles CaCO3 per million gram
(d) 4gH2H24gC
2000 _ in-3 ii- i
lgH2a6gC = —= 2 x 10 millimoles per gram
(e) 6gH2 = 24gC
lgH2=4gC (b) 2 moles CaCO3
(f) 8 g H2 = 36 g C = 2 x No molecules CaCO3 per million gram
1 gH2=4.5 g C 2xN0 . .
molecules per gram
Equivalent weight is not a definite property. It depends 10'
on the reaction and product formed. « x 20 ,An 250
7. Purity x 100 =
Punty = — xl00 =---- gg
4. Molecular weight
Let A be = SxOy 32x + 16y (24 carat is assumed to be 100%)
Pure diamond in 200 mg sample
andB be = ~ Jq
SpnO, 32p + 16g
200 250 1 1
InA 16 y g oxygen = (32 x + 16y) g SxOy =----- x----- x — = - g
1000 3 100 6
32x +16y
hence, 8 g oxygen = = 16 (given) Number of moles = - x — g carbon
2y 6 12
32x o Number of carbon atoms
---- + 8 —16
y 6.02 xlO23
= 8.33 x 102L atom
X 1 number of S atom
— —--------------------------- 72
y 2 number of 0 atom 8. 1 mole ethyl alcohol = 46 g ethyl alcohol (CH3CH2OH)
Thus, A is SO2 Water present = 100 - 46 = 54 g
In B 16 q g oxygen = (32p + 16g) g SpOQ 46
Moles ofCH3CH9OH = — = 1
32p +16g 3 2 46
8 goxygen = = 13.33 (given) 54
2g Moles of H2O=—= 3, Ratio =1:3
2 18
^=5.33
7 9. KHC2O4 K* + hc2o;
P 5-33 1 number of S-atom hc2o; h+ + c2o2-
~q “"IT” 3 number of O-atom One of the students (say A) would have derived
Thus, B is SO3 equivalent weight based on H+ which is = molecular
weight
5. Hydrides
Another student (say B) would have derived equivalent
(l) N^y weight based on C2O2- (oxalate)
molar mass =(14x + y)
10. Magnesium = 0.24 g
y gH = (14x + y) g NxHy
14x + y „ Magnesium oxide (residue) = 0.40 g
1 gH =--------- = 5.66
y Oxygen combined = 0.16 g
14x 0.16 g oxygen s 0.24 g Mg
— =4.66
y Thus, 8 g oxygen = 12 g Mg
x 1 number of nitrogen atom
— — — =------------------------------------ 28 x 100
11. (a)NH4NO3 =
y 3 number of hydrogen atom 80
28 x 100
Hence, hydrogen is NH3 (b) NH2CONH2 = Max
14x + y „ , . 60
(II)---------=8 (given) 14
y (c) KNO,=—x 100 least
3 101
14x
---- = 7 14 x 100
y (d) NH4C1 =
53.5
30 | Essential Inorganic Chemistry

12. Ca3(PO4) 3Ca 2+ +2PO3"


3Ca Passage 3
Based on valency, equivalent weight of 1. (a) 2. (c)
n molecular weight 3. (b) For spectrum, approximate percentage of
Ca3(rO4)2 - -
24Mg= 80
mole
Hence, one equivalent Ca3(PO4)2 = “Mg = 8
"V”
26Mg= 12
1 mole Ca3(PO4)2 = 3 moles CaCl2 = 2 moles HgPO4 M.X. + M2* X452 + M3O X.«.
Average atomic mass = X X = 24.32
- moleCa3(PO4)3 =- mole CaCl2 = - mole HgPO4 100
6 2 3

Passage Comprehension Questions Assertion & Reason


1. (a) 2. (b) 3. (d) 4. (a)
Passage 1
1. (a) Dish of 20 cm diameter (10 cm = 0.1 m radius) True & False
has area = nr2 = rc x 0.01 m2
1. F(16) 2. T 3. T 4. T 5. F (moles one equal)
0.21 x 10"18 m2 is the area of = 1 molecule
n 2. xi z. rc x 0.01 6. F (no H2 is left) 7. T 8. T 9. T
.*. n x 0.01 m is the area of =------------ molecules 17
0.21 x 10"18 10. F(itis—-g)
i . 1.4x 10"4 o
1.4 x 10 g steanc acid =—— mol
1.4 x IO"4 Fill in the Blanks
No molecules
284 1. multiple proportion 2:3
2. &.o 3. ^2116
«. 4. 20
1.4x10-*^ ..r0_ nxO.Ol 5. 0.33 6. Na2S2O3-7H2O 7. 103 8.1
284 0.21x 10"18 9. reciprocal proportions 10. Dalton (Da)
No=3.O3xlO23
1 Matrix-Match Type Questions
2. (b) —= = 3.33x 10"24 g
3.03 xlO23
I. Only One Correct
Passage 2
1. (A)-(4); (B)-(3); (C)-(l);
1000
1. (b) Rs. 50 per kg = Rs. 50 per mol (D) —(2); (E) —(5)
74.5
2. (A)-(5); (B)-(3); (C)-(l);
Rs. 50
C0St“ 1000 per mol = Rs. 3.73 per mol (D)-(4); (E) —- (2)
II. One or More Correct
74.5
1000 1. (A) —(1,3,4,5); (B) — (1,2,3,5); (O-(1,3,5);
2. (c)lkgKCl = mol K+ costing Rs. 50/ -
74.5 (D) — (1,3,5)
1000
Re 1 gives = mol of K+ (in KC1) 2. (A) —(1,2,3,4,5); (B) — (1,3,5); (C)-(l);
74.5x50
1000 , (D)-(4)
lkgK2SO4 = ------mol
174
2000 Integer Answer Types
mol K+ costing Rs. x
174 Set I
1000x2
Re 1 gives = molofK* (in
174x Questions —► 1 2 3 4 5 £ 7_ 8
1000 _2000 Answers —► 3 7 1 0 1 0 6 5
74.5x50”174x © © © © ® ©I© ©
x = Rs. 42.82 kg"1 © (3) O ®
3. (d) Let mass ofK2O=x g © © © © © @_®_ ©
©. © © © © @ © ©
=—mol K2O = —mol K+
94 2 94
1000 . 1000 © ® ® ® ® <® _® ®
1000gKCl = ----- mol KC1 =
74.5------------- 74.5
mol K+ ® ® ® ® ® >® ®
2x 1000 © © © © ©
94 74.5
® ® ® ® ® J)_® ®
x= 631g = 0.631 kg
© © © © © © © ©
Chapter 1 Fundamental Concepts of Inorganic Chemistry | 31

1. CH4 = 16 g mol-1 AjXj + A2X2


2. Average atomic mass
Xi +x2
4g H has = 12 g carbon
12 35x + 37
Thus, 1 g H has = — = 3 g =----------=35.5
4 x+1
Thus, x = 3 Thus, x = 3
3. One gram atom has No atoms = 6.02 x 10123
1 atoms
SO3 • yH2O
2. Na22^3

126 g
nr
18 y
If atomic mass
then mass of one atom
=A
— = 6.66 x 10"23 g
18y _ 50 No
126 + 18y 100 — =6.66 x KT23
"o
y=7
A =6.66 x 10"23 x6.02 x 1023
3. 1 amu
= 40 g mol"1
4. H2+ H2 has = 2 electron, 0 neutron
Number of gram atoms (moles) in 0.320 kg
H2+ has = 0 electron, 0 neutron 0.320 x 1000
1 _ ____ . Itr , 0 =8
5. 0n * 1“ + -le " 40
due to emission of electron, atomic number increases by 4. X2~ is isoelectronic with Ne.
1 unit Thus, electrons in X2~ = 10
6. Na2CO3 is thermally stable, no effect of heating. Proton (= atomic number) in X2~ = 8
., , Mass
Also mole =---------------
7. Empirical formula weight = 30 atomic mass
Thus, — =6 0.1 = 1®
30 A
2.495 _ A1 , A = 16
' 8. ------ =0.01 mol
249.5 Also, neutrons + protons = Atomic mass = 16
= 0.01x H2O neutrons = 16 - 8 = 8
= 0.05 mol 5. SO2(g) + 2H2S(£) ---- > 3S(s) + 2^0(1)
x=5
As per equation 1 mol 2 mol 3 mol 2 mol
Set II taken: 2 mol 2 mol ?
Questions —► 1 2 3 4 5 £ 7 8 If one takes 2 moles SO2(g) then 4 moles of H^g) are
Answers —► 3 3 8 8 3 0 2 5 required. But 2 moles ofH^glhave been taken. Hence,
© © © © © ©I© © H^g) is limiting reagent that forms 3 moles of
© 0 0 0 © ®_© 0 sulphur.
© © © © © <© _@
© Thus, 3
© © © © ©. >_© © 6. neutrons = 0
©. © © © © ©_© ©
© © © ©, ©. ©© © 7. A1PO44 '
Al3+ + PO3 (two ions)
© © ® ® ® ® 8. xFe + yO2 ---- > FezO.y
© © © © © ©_© © x mol y mol
® ® ® ® © ® ® ® = (22.4 y L)
x _ 22.4y
© © © © © © © © 0.2 ” 3.36
119
1. Radioactive water is heavy water with heavy isotope 11.2 g = —— = 0.2 mol
56
(2H) of hydrogen. Thus, molar mass of heavy water x 22.4y
= 2 x2 +16 (12 3.36
= 20 g mol"1 x 2
0.060 kg = 0.060 x 1000 g = 60g = ^ = 3 moles
y 3
Thus, 3 Thus, Fe2O3 x + y = 5
Chemical Bonding
The theory of the chemical bond......... is still far from perfect. Most of the
principles that have been developed are crude, and only rarely can they be
used in making an accurate quantitative prediction. However, they are the
best that we have, as yet, and I agree with Poincare's that "it is far better to
forsee even without certainty than not to forsee at all".
Linus Pauling

Quick Points
• Formal charge is given by
s
F=v---u
2
where v = valence electron
s = shared electrons forming bonds Ionic Bonding
I

u = unshared electrons Covalent Bonding


• 1 pm = 1 x IO-12 m = 1 x IO"10 cm Coordinate Bonding
• Charge on electron = 4.8 x 1O“10 esu Formal charge
I
= 1.6 x IO-19 C VSEPR Theory
• In a complex, [MLn ], M is the metal and L the ligand and n Assessing Molecular Shape
the number of ligands.
Sigma and Pi Bonds
• If ligand is weak, (as H20) arrangement of electrons
in valence shell is not affected but with strong ligand, (as Valence Bond Theory
CN’) unpaired electrons get paired. Hybridisation
!< Molecular Orbital Theory
Fe2+: [Ar] 3d6 lPp|p|j1p| Change to A with CN“
Bonding in Complexes
L'
ligand and no change
3d6 with H2O ligand Dipole Moment
Fajan's Rule
Hydrogen Bonding
i-
* OH no change
Resonance
(A) Metallic Bonding

• If there are N unpaired electrons, magnetic moment is given


by
g = /MAT + 2) BM
34 | Essential Inorganic Chemistry

A molecule will only be formed, if it is more stable, and


has a lower energy than the individual atoms. Normally
Ionic Bonding
only electrons in the outermost shell of an atom are • An ionic bond is formed when a metal atom transfers
involved in forming bonds and in this process each atom one or more electrons to a non-metal atom.
attains a stable electronic configuration of inert gas. We Na---- > Na+ + e~
divide elements into three classes : ci+e“ —> cr
(A) Electropositive elements, whose atoms give up one As a result of this transfer the metal atom becomes
or more electrons easily. They have low ionisation
cation and the non-metal the anion. IA(1), IIA(2) and
potentials.
(B) Electronegative elements, which can gain VIA(16), VIIA(17) elements are involved (Table 2.1).
electrons. They have higher value of • We can understand this exchange of electrons by
electronegativity. showing electron configuration.
(C) Elements which have little tendency to lose or gain 3s 3p _3s_ 3p
electrons.
uNa[Ne)[]] Na+ [Ne]__

Types of Bonds
17Cl[Ne] || || || | J Cl-[Ne] || [| || [|
Atoms involved Bonding
(I) A+ B Ionic
(ii) 8+ B Covalent
(iii) A+ A Metallic
(iv) Electron deficient molecule or ion (Lewis acid) Coordinate
and electron-rich molecule or ion (Lewis base)
(v) H and electronegative element (N, O, F) Hydrogen

Table 2.1 Simple Binary Ionic Compounds


Group
General formula Ions present Example Melting point (°C)
Metal Non-metal

IA(1) + VIIA(17) MX (M+, X~) LiBr 547


+ VIIA(17) MX2 (/W2+, 2X") MgCI2 708
IIA(2)
+ VIIA(17) mx3 (M3+,3X‘) GaF3 800 (subl)
IIIA(13)

IA(1) + VIA(16) m2x (2M+, X2’) Li2O > 1700


IIA(2) + VIA(16) MX (M2+, X2’) CaO 2580
IIIA(13) + VIA(16) M2X3 (2M3+,3X2-) AI2O3 2045

IA(1) + VA(15) m3x (3M+, X3’) Li3N 840


IIA(2) + VA(15) m3x2 (3M2+,2X3‘) Ca3P2 -1600
IIIA(13) + VA(15) MX (M3+, x3-) AIP

• Cations and anions attain stable noble gas - are soluble in polar solvents and insoluble in
configuration with outer shell octet of electrons. In non-polar solvents,
some cases cations may not have octet but different - have high m.p. and b.p.,
outer-shell (see Tables 2.2 and 2.3) - have strong force of attraction between cation and
• In the solid state each cation surrounds itself with anion (Coulombic force)
anions, and each anion with cations. These very large p _= ^e2
F
number of ions are arranged in an orderly network Dr*
called ionic crystal. where Zj and are charges on the ions, e the
• Ionic solids electronic charge; D the dielectric constant of the
- are good conductor of electricity in fused state and medium and r the inter-ionic distance (i.e., the bond
in aqueous solution, length).
Chapter 2: Chemical Bonding I 35
Table 2.2 Electronic Configuration of Some Metal Ions Solubility of Ionic Solids
"Octet"• "18" "18 + 2" Variable By the Coulombic force of attraction (F) between
2 2• cation and anion.
Na•, Mg 2+
cu• zn • in• Cr [Ar] 3d
4

K• I Ca • 2
Ag• Cd 2+ TI• Cr • 3
[Ar] 3d3 F=�

Rb•, Sr 2 Au• Hg 2+ 2
Sn • M n2 + [Ar] 3d 5 Dr1-
cs•, Ba 2 + Ga 3• Pb2+ Mn3• [Ar] 3d• we can study the solubility of ionic solids in different
Fr ♦ • Ra 2+ Sb 3+
Sb3+ 2
Fe • [Ar] 3d6 solvents.
AI 3+
Tl 3+ Bi3+ Fe3+ [Ar] 3d 5
Sc3 + Co2

[Ar] 3d
7 DEEP Focus
• Greater the force of attraction between ions, smaller the
yJ + Co3• [Ar] 3d6
tendency of the ions to go into solution and thus, smaller the
La 3 • Ni2+ [Ar] 3d8 solubility.
Ni3+ [Ar] 3d7 - For a given solute, greater the value of D (Dielectric
constant of the solvent) then smaller the force of
Table 2.3 Lewis Dot Formulae for Representative Elements attraction between positive and negative ions, hence
greater the solubility.
Group
IA IIA IIIA IVA VA VIA VIIA 0
(1) (2) (13) (14) (15) (16) (17) D(Hp)>D(CH3CH 2OH)> D(CH 3OCH :; )

Number of hence solubility of ionic solid (say NaCl) in


2 3 4 5 6 7 8
electrons in (except H:P > CH 3CHpH > CH 30CH3
valence shell He) - Greater the tnterionic distance (r) between positive and

..
negative ions, smaller the force of attraction between

..F :
Period 1 H• He:

. •O. : •..
positive and negative ions hence greater the solubility
•• •• ••
.. in the given solvent

.. ..• • ..p. • . ••s. : •..Cl:.. ..


Period 2 L i• Be: 8 • C• •N • :Ne:
LiF LiCl LiBr Lil

=�!:
(max)
interionic distance increases

.. .. •As• •Se: ..
Period 3 Na• Mg: Al• Si•
.. •Br: ..
:..
. . . ..
LiF NaF KF RbF Cs�
(max)

..In• Sn•
... •Sb•. •Te:... • ....I :
Kr:
..
Period 4 K• Ca: Ga • Ge• interionic distance increases
(maximum towards the direction of arrow)

.. (max) -----------------

..Tl• Pb•
..• •Bi... • •P...o: •At:.... =�..� =
Period 5 Rb• Sr: :Xe:
force of attraction increases
(max)
solubility in water increases
Period 6 Cs• Ba:
However, if one of the ions is very big, the solubility
Period 7 Fr• Ra: increases with decreasing size of the other ion.
CsF > CsCl > CsBr > Csl (very big cation)
lliustration 1 Explain the formation o/Li20 based on Lil> Nal >KI> Csl (very big anion)
electrons configuration. LiNO3 > NaNO3 > KNO3 > CsNO3 (very big anion)
- Also, greater the charge (z1 and z2) then greater the force of
Solution The Lewis dot formula for the atoms and ions
attraction between positive and negative ions hence smaller
are the tendency of the ions to go into the solution hence smaller
ls 2s the solubility.

[TI] [D er 2+
Na• Ba Cr
aLi (NaCl) (BaC12)

[TI] OJ
charge increases max
Li
3 max solubility increases

sO [TI] [TI] I t1 I l I l I Na• c1-


x
2Li + • :o• -----+ NaCl
charge increases max

ls 2s 2p max solubility increases


Li+
[TI] le-lost

Energetics of Ionic Bond Formation


Li+
[TI] ..---.....-.
le- lost
The energy given off when gaseous positive and

[TIJ [TI] I 11111111 I 2e- gained


02- negative ions come together to form 1 mole of the solid
ionic compound is called lattice energy (U).
36 | Essential Inorganic Chemistry

Na+(g) + Cl-(g) - * NaCl(s), AH = -U • If the anion is large compared to the cation, the lattice energy
will remain almost constant within a particular group. Since
For the reverse process the hydration energies decrease down a group, solubility will
NaCl(s)-----> Na+(g) + CF(g), AH = + U decrease as found for alkaline earth metal carbonates and
sulphates.
Lattice energy can be calculated using Born-Haber
cycle
Covalent Bonding
DEEP Focus If duplet (2) or octet (8) is completed by sharing of
(I) M(s)+h,(g) ---- > M+X'(s), AH° = Q = AHj electrons between two electronegative elements, the bond
2 “ formed is called covalent bond.
M(s) ---- > MtgY, AH = S
M(g) ---- > M+(g)+e'; AH = I The Lewis Theory
1 X2(g) ---- > X(g); The Octet rule
2 2 2
X(g)+e~ ---- > X'(g); AH = -E For many light atoms a stable arrangement is
M+(g)+ X~(g) ---- > Af+X'(s); AH = -U attained when the atom is surrounded by eight electrons—
the octet rule. (In case of H2, duplet is completed)
(II) M(s)+-X2(g) ---- > M+X-(s);
2 2 • Number of bonds between two atoms
AH2 = S + I + --E-U _ electrons shared
2 2
2
where AH° is the standard heat of formation of MX(s),
Electrons
S = the heat of sublimation of M(s), Molecule Bonds
shared
I = the ionisation energy of M(g)
D = the dissociation energy of X2(g), h:h 2 1 (single) H —H
E = the electron-affinity of X(g) 4 2 (double) 0=0
:o: :o:
I and II represent the same chemical change hence,
:n::n: 6 3 (triple) N=N
AHy = AH2 0 —H
:o;h
•X
2 (between H 1 (two single I
Q=S+I+--E-U H and O) bonds) H
2
H—N—H
U=S+I+--E-Q h; •nX ;h 2 (between H 1 (three single I
2 H and N) bonds) H
H H
(a) M+X’(s) + ag * M\aq) + X~(aq) 2 (between H 1 (four single
h; XC; h and C) bonds)
H-C-H
enthalpy change = AHsoln •
H H
(b) M+(g) + X~(g) + aq' ---- > M+(aq') + X~(aq)
• All the valence (outer-shell) electrons of the atoms in a
enthalpy change = A#Hydration
Lewis structure must be shown in the structure.
(c) ATX’(s) * M+(g) + X~(g)
• Usually, all the electrons in a Lewis structure are
enthalpy change = U paired.
We find (a) = (b) + (c)
• :ci:ci: or Id—lone pairs
AH,, .
, = Hydration
AH soln + U V— bond pair.
Both lattice energy and hydration energies decrease
with increase in ionic size.
DEEP Focus
DEEP Focus • :o::b‘: or ;o=6;
• If the anion and the cation are of comparable size, the This is actually incorrect since it does not have unpaired
cationic radius will influence the lattice energy. Since electrons which account for the paramagnetic character of 02
lattice energy decreases much more than the hydration (by MO theory, 02 is said to have two unpaired electrons.)
energy with increasing ionic size, solubility will increase as A resonance hybrid of the following two contributing
we go down the group. structures has unpaired electrons and also conforms to the fact
(BeOH)2 < Mg(0H)2 < Ca(OH)2 < Sr(OH)2 < Ba(0H)2 that the experimentally measured bond length in 02 shows to
-> have some multiple bond character.
solubility increases max
:o• —6:
• <—> :b’=b‘:
Chapter 2: Chemical Bonding | 37

• If there are odd number of electrons (15 as in NO), InHaSC^.


then there is at least one unpaired electron
N = 8 x 5 (one sulphur and four oxygen) + 2 x 2(H)
somewhere in the structure.
= 44
• At least one atom lacks a completed octet of
electrons and species is paramagnetic. A = 2(for two H - atoms) + 6 (for sulphur atom)
complete octet of N and 0 + 24(for four oxygen atoms)
.. .. ft; i = 32
unpaired electron •n=o: •n::o: 8 = 44-32 = 12
Thus, HgSC^ has six o-bonds (each bond is formed by
electrons shared between N and 0=5
sharing of one pair of electrons)
number of bonds = 2.5 0
Octet of electrons around each oxygen atom (four in double
bonds and four in lone pairs) H-O-S-O-H

0
b=c)=o
Octet of electrons around carbon atom (four in each of two Exceptions to the Octet Rule
double bonds) In so many cases, the octet rule is violated :
• In BeF2, octet of Be is not complete
octet incomplete
t ..I I ..
Octet of electrons around each N atom (Six in triple bond :F’X Be*x F;
and two in lone pair) L2J
• A mathematical relationship can be used to determine octet complete
total number of electrons shared : • In BF3, octet of B is not complete
••
S-N-A :x»f:
where 8 is the total number of electrons shared in the
molecule or polyatomic ion; N is the number of valence octet incomplete [B; Fl] octet complete
shell electrons needed by all the atoms in the molecule or :f;
ion to achieve noble gas configurations (N = 8 x number of
• The octet rule is also violated where atoms have an
atoms (H excluded) + 2 x number of H atoms); A is the extra energy level which is close in energy to the p
number of electrons available in the valence shell of all of level, and may accept electrons and be used for
the representative atoms. This is equal to the sum of their bonding. PF3 obeys the octet rule, butPF5 does not. PF5
periodic group numbers. has ten outer electrons, and uses one 3 s, three 3 p and
In C02, A for each 0 atom is 6 and for C atom is 4. one 3d orbitals
Thus, A = 4 + 2x6 = 16 T
N = 8x3=24
T | TIT 3d excite T|T|T 3d
Thus, S = 24-16 = 8.
Thus, there are, eight electrons shared to form bonds
P T1
3s
3p 0
3s
3p

:6::c::6; (ground state) (excited state)


Any compound with more than four covalent bonds
Illustration 1 Calculate total number of shared must violate the octet rule, and these become very
electrons in H2SO4 and thus, assign structure. common in the elements after the first two periods of eight
Solution Let S be the total number of shared electrons elements in the Periodic Table.
then In SF6, there are six S—F bonds having in all 12
S = N-A electrons in sulphur.
F
where N is the number of valence shell electrons
needed by all atoms in a molecule or ion and A is the F-S-F
number of electrons available in the valence shell of |\f
all of the respresentative atoms F
N = 8 x number of atoms (excluding H) + 2 x InPC15, there are ten electrons in phosphorus. In such
number of H-atoms cases, Sugden introduced singlet linkage in which one
A = sum of the periodic group number of atoms. electron is donated (instead of one pair of electrons) to the
38 | Essential Inorganic Chemistry

electron deficient atom so that octet rule is not violated. Number of Bond
This is indicated by (—). Thus, PC15 and SFfi6 are electron-pairs In Shapes of molecules angles
represented as outer-shell >

:ci: :ci:
Cl/ /Cl
6 Octahedron 90°
:ci*xPx.ci:
•• X ••
or Cl—P—Cl

:ci:
••
Cl
••
:f:
• V
:¥:
7• F 72° and
F\ 7 Pentagonal
• • xx.. 90°
F^S- bi pyramid al
IF x S x F. or ■F
•• x x ••
F
f: Illustration 2 Draw the Lewis structure for
aluminium tri-iodide (A1I3).
Solution The outer-shell electron configuration of Al
Sidgwick-Powell Theory is 3s23//.The Al atom forms three covalent bonds with the
Depending on the number of electron pairs in the I atoms as follows :
outer-shell of the central atom, shape of the molecule can
:i:
be decided based on this theory. In this concept bond pairs
and lone pairs are taken as equivalent since all electron :i—ai
pairs take up some space, and also their tendency of
repulsion. This repulsion is minimised if the electron pairs •I
••
are oriented in space as far apart as possible. Molecular
shapes predicted by Sidgwick-Powell theory have been We find that the octet rule is satisfied for the I atom,
there are only six valence electrons around Al atom. The
summarised in Table 2.4.
molecule is an example of the incomplete octet. Thus,
Table 2.4 Molecular Shapes Predicted By Sidgwick-Powell A1I3 is a Lewis acid—an electron pair acceptor.
Theory Illustration 3 Draw the Lewis structure for PF5 in
Number of which all five F atoms are bonded directly to P atom.
electron-pairs in Bond
Shapes of molecules
angles Solution The outer-shell electron configuration for
outer-shell
2 Linear 180° P:3s23p3, F:2s22p5
Total number of valence electrons = 7x5 + 5 = 40
3 Plane triangle 120° P is a third-period element and it can have an.
expanded octet. The Lewis structure of PF5 is
••
:f: ..
.. I ,f:
4 Tetrahedron 109° 28' :f— p<<
f:

Illustration 4 In the vapour phase, beryllium chloride


consists of discrete BeCl2 molecules. Is the octet rule
satisfied for Be in the compound. If not, can you form an
5 Trigonal 120°and
bipyramid 90° octet around Be by drawing another resonance structure?
How plausible is this structure?
Solution In single unit, octet of Be is short of four
electrons. OO 0 0

:ci:Be:ci:
oo o o
Chapter 2 : Chemical Bonding | 39

Octet around Be is formed in the following resonance H F


structure but it is not plausible. • NH3 and BF3
I I
00 2 90
H-N: -> B—F
+Cl=Be=Cl +
00 00
H F
-12+
OH,
H,0 OH,
Coordinate Bonding • [Fe(H2O)8]2+
A covalent bond results from the sharing of pair of H,0
OH, OH,
electrons between two atoms where each atom contributes
one electron to the bond. It is also possible to have an Properties of the coordinate compounds are
electron pair bond where both electrons originate from one intermediates of ionic and covalent compounds. A
atom and none from the other. Such bonds are called comparison is summarised in Table 2.5.
coordinate bonds or dative bonds. Since in coordinate
bonds, two electrons are shared by two atoms, they differ Table 2.5 Comparison of Ionic, Covalent and Coordinate
from normal covalent bonds only in the way they are Compounds
formed, and once formed they are identical to normal Ionic Covalent Coordinate
covalent bonds. (1) Binding force between ions between in between
It is represented as : ---- > strong (coulombic) molecules
smaller (van
• Atom/ion/molecule donating electron pair is called der Waals')
DONOR or LEWIS BASE (2) m.p./b.p. high less than in between
• Atom/ion/molecule accepting electron pair is called ionic
ACCEPTOR or LEWIS ACID; (-*) points donor to (3) Conduction conductor of bad greater than
acceptor. electricity in fused conductor covalent
state and in
• NHj : NH3 has three (N—H) bonds and one lone pair aqueous solution.
on N-atom. In NH4 formation this lone pair is (4) Solubility in high less in between
polar solvent
donated to H+ (having no electron) (H2O)
NH3 + H+ ---- > NH4+ (5) Solubility in low high in between
Lewis base Lewis acid non polar
e
r h H solvents
I I (ether)
H-N:-> H+ or H—N->H (6) Physical state generally solid liquid and solid, liquid,
gas gas
H H
Illustration 1 Classify the following bonds as ionic,
• BF4" : In BF3, B has incomplete octet. F" can donate its
polar covalent or covalent, and give your reasons :
one of the lone pairs to B forming BF4 (a) SiSi bond in Cl3SiSiCl3
F F T (b) SiCl bond in ClgSiSiClg
: f: b—f or F->B—F (c) CaF bond in CaF2
I (d) NH bond in NH3
F F Solution (a) Covalent due to identical
0 electronegativity.
T
•SO3 • 0=S->0 (b) One electrons pair is shared between Si and Cl and
Cl thus, covalent bond is expected but
electronegativity of Cl is greater than that of Si and
Cl\ I some polarity develops giving polar-covalent
• pci; C12>p<-Cl
Cl^| nature.
Cl (c) Ionic since Ca completes its octet by transfer of two
Cl outer-shell electrons and two F atoms gain these
I two electrons thus, completing their octets
• SbF0" F-^>Sb«-F Ca:[Ar]4s2, F:[He]2s22p5
F^ |
Cl (d) Polar covalent, explanation as in (b).
40 I Essential Inorganic Chemistry

Illustration 2 In the gas phase, aluminium chloride structure with no more than one large formal charge
(-2, +2 and so on) is preferable to a structure with
exists as a dimer (a unit of two) with the formula Al2 Cl6 .Its several formal charges.
skeletal structure is given by
(c) Lewis structure should have adjacent formal charges of
Cl Cl Cl
>Al < >Al< zero or of opposite sign.
Cl Cl
,,
Cl (d) When we must choose among several Lewis structures
with similar distribution of formal charges on the more
Complete the Lewis structure and indicate the electronegative atom is preferable.
coordinate covalent bonds in the molecule.
•• •• ••
.. .. /:Al ..
Thiocyanate ion can be assigned three structures
:c1) /
.,c1:,� <c1:
Solution Al
:ci ':ci ci:
Formal charges Total
charge

.. ..
•• •• •• N C s

.. ..
I [:N=C=s:r -1 0 0 -1

Formal Charge
.. ..:r
II [:C=N=s:r +1 -2 0 -1
The formal charge of an atom in a mol ecule is the
Ill [: C=S=N -1 -2 +2 -1
hypothetical charge the atom woul d have if we coul d
redistribute the electrons in the bonds evenl y between the
In all these cases total formal charge is -1. But we
atoms in the bond. Formal charge (F) is determined using
prefer structure I because it has the lowest number of
s
F=v---u atoms with non-zero formal charge [guideline (b)].
• Generally the most stable atomic skeleton is also that
where, v = valence electrons for which the sum of the electronegativity differences
s = shared electrons (forming bonds) of adjacent atoms is maximum. Thus, for the
u = unshared electrons arrangements HOCl and HCIO, the
• In BrC1 3 Cl-Br-Cl electronegativities of the atoms yield :
I 2.2 3.44 3.16
Cl H- 0 - Cl; l: = (3.44 -2.20) + (3.44 - 3.16) = 1.52
2.2 3.16 3.44
V s u Formal charge
H- Cl - 0 ; l: = (3.16-2.20) + (�.44 -3.16) = 1.24
Br 7 6 4 0
Cl 7 2 6 0 Thus, H-0-Cl is the most stable structure. This is
also consistent with formal charge distribution and
• In IC14 Pauling's electroneutral ity principle: in a molecule,
electrons are so distributed that the residual charge on
V s u Formal charge
each atom becomes zero or nearly zero; hydrogen and the
I 7 8 4 (one extra) -1 most electropositive metals may acquire partial positive
Cl 7 2 6 0 charge and the most electronegative atom may acquire the
negative charge.
• InNH1
Illustration 1 Calculate the formal charge on each

.. .. ..
V s u Formal charge
atom in the following electron-dot structure for S0 2 :
N 5 8 0 +1
H 2 0 0 :0-8=0
•• ••
• In many cases, we can draw more than one structure
for a molecule, with different arrangement of atoms.
Solution Let v = valence electrons, F = formal charge,
We can use the following guidelines involving formal s = shared electrons,
charge to decide between two or more possible u = unshared electrons.
structures for a molecule or ion :
Atom V s u F=v-�-u
DEEP Focus Sulphur 6 6 2 +1
(a) A molecular structure that gives a Lewis structure in
which all formal charges are zero is preferable to one in Single-bonded oxygen 6 2 6 -1
which some formal charges are not zero. Double-bonded oxygen 6 4 4 0

(b) If the Lewis structure must have non-zero form.al


charges, the structure with the lowest number of 802 may be written as
..
0 :0-8=0
$
•• ••
non-zero form.al charge is preferable and a Lewis •• ••
Chapter 2: Chemical Bonding I 41
lliustration 2 We can write two electron-dot Which of the two electron-dot structures gives a more
structures forN2 0, one of which has a negative formal accurate representation of the molecule?
charge on the oxygen atom and the other of which has a
negative formal charge on the terminal nitrogen atom.
Solution Since oxygen is a more electronegative
•• •• EB ••
element than nitrogen, the structure that places a
N
.• -N-o•. 9
� 9N
-• •• =N=O
-•• negative formal charge on oxygen is probably lower in
•• energy than the structure that has a negative formal
(II)
charge on nitrogen. Thus, (I) is more favourable than (II).

Target Practice 1
.. .. ..
.. .. .. .. <h> [�=Z-R :r
1. Which one would better represent bonding in the molecule
HNO?
I: H-N=O: ......... and II: H-O=N ••
..
·o·
. . ••
••
2. In each of the following Lewis structure, Z represents ·a I
main-group element. Name the group to which Z belongs in ..
(c) H-O-Z-O: .•
each case and give an example of a such a compound or ion I
:o:
that actually exists. ••

(a)
.. . . ..
:o=Z=O:
Group of Element Z Example 3. Draw Lewis structure for
No; (nitronium ion).
(i) c10; (chlorate ion) and (ii)
•• (i) ............ (ii) .......... ..

Answers
1. I is better as octets ofN and Oare complete and duplet of His ·o.
also complete.
2. Group of Element Z Example
. . .... .. (c) VII (17) H-0
.. �!;-0:
I ..
(a) Zero (18) :O=Xe =O: .·o·...
(b) V (15)
rO=N-O:
l
••. . ·· .••. T
J
..
·o.
a. Ci> [:R��!�� :r cm [R =N =�I

Practice Exercise 1
1. Atomic number of the four elements A, B, C and D are 6. Propose Lewis structures for the following ionic species
respectively Z - 1, Z, Z + 1 and Z + 2. If Z = 9, what is type containing sulphur-to-sulphur bonds
of bonding between (a) s 22 - (b) s 32 -
(a) A and D (b) Band D 2
(c) A and A (d) B andB (c) s 42- (d) s 5-
2. Valence electrons in the elementsX and Y are 3 and 6. What 7. Explain the formation of following compounds
is the formula of the compound formed fromX and Y? (a) CaO (b) Mg 3N 2
3. Classify Lewis acid and Lewis base out of: 8. Which of the following atoms would be expected to form
(a) H20 (b) NH3 negative ions and which would be expected to form positive
ions in binary ionic compounds?
(c) tt+ (d) BF3
(a) P, I, Mg, Cl, In, Cs, 0, Pb, Co
(e) A1H3 (f) ROH
(b) Ca, Br, Na, N, F, Al, Sn, S, Cd
(g) CH 3 NH 2 (h) CN-
9. Calculate the lattice energy of a salt MX (s) from the data
(i) +cH3 given below:
4. Use your knowledge of electronegativities, to arrange the Heat of formation of MX (Ml)= - 550 kJ mo1- 1
following bonds in terms of increasing ionic character Heat of sublimation of M (S) = 80 kJ moi-1
C-H, F-H, Na-Cl, Br-H, K-F Heat of dissociation of X2 (D) = 155 kJ mo1-1
5. Represent each of the following ionic compounds by an Ionisation energy of M (I)= 374 kJ moi-1
appropriate Lewis structures Electron affinity ofX (E ) = - 343 kJ moi-1
(a) IQ03 10. Arrange the following in increasing lattice energy:
(b) Ca (OCl) 2 (a) BeCO3, MgCO3 , CaCOa, SrCO3 , BaCO3
(c) NH4Cl0 4 (b) Li1CO3 , Na�pO3 , K2CO3
42 | Essential Inorganic Chemistry

(c) LiCl, NaCl, KC1 (a) Identify the incorrect bonds and explain what is wrong
(d) LiF, LiBr, Lil with them
11. The lattice energy of CaO (s) is -3460 kJ mol-1; the lattice (b) Write the correct Lewis structure.
energy of K2O is - 2240 kJ mol-1. Account for this :o:
h
difference. I I
h=c-c-o.-h
12. Specify which compound in the following pairs of ionic I ••
H
compounds has the higher lattice energy?
(a) KC1 or MgO (b) LiF or LiBr 20. Give an example of an ion or molecule containing Al that
(c) Mg3N2 or NaCl (a) obeys the octet rule
13. Bond length can be calculated by merely adding covalent (b) has an expanded octet
bond radii which are H= 0.28 A, C = 0.77 A, N= 0.70A, (c) has an incomplete octet
0= 0.66A, Cl= 0.99A, (C=) = 0.67A, (C =0=0.61A, 21. Why does NH3 forms coordination complexes, whereas the
(N s=)= 0.55 A. Calculate bond lengths in:
(a) NH3 (b) CH2C12 isoelectronic species CH does not ?
22. PF3 can act as a donor molecule while NF3 shows little
(c) HOC1 (d) HCN
tendency to act as a donor. Account for this observation.
(e) CH2=CH2 (DCH=CH
23. Indicate the coordinate covalent bond (or bonds) in the
14. In which molecule is the van der Waals’ force likely to be the product:
most important in determining the melting point and (a) Al3+ + 6H"2oO —> Al (OH)’" + 6H+
boiling point? » ocbf3 . . -
(b) CO+ BF.3
IC1, Br2, HC1, H2S, CO
(c) Sn2+ + 3Cr---- > SnClI0
15. Write Lewis structure of: (d) H2O+ H+ ---- > H3O+
(a) Cj- (b) PF“
(e) SO,0 + O2- ---- > SO2* ‘
(c) ici; 24. Write structures of the following indicating coordinate
16. The chlorine-to-oxygen bond distance in C1O“ is 1.44 A. bonds wherever exist:
What do you conclude about the structure of this ion? (a) N2O (b) Cr2O2-
17. Show that by using an expanded octet for the sulphur atom (c) CrO5 (d)A12C16
in thionyl chloride (SOC12), you can write a Lewis structure (e) H3PO4 (f)H3PO3
with no formal charge. (g) H3PO2 (h) no2
18. Write Lewis structures for the following molecules : 25. For each of the following pairs of elements, state whether
(a) H,NOH (b) N/2 the binary compound they form is likely to be ionic or
(c) HONO (d) H2NNO2 covalent.
19. The skeletal structure of acetic acid shown below is correct, (a) B and F (b) K and Br
but some of the bonds are wrong.

Answers
1. (a) Ionic (b) Ionic (c) Covalent (d) Covalent O:ls22s22p4 ---- > O2’ls22s22p6
2.
• • 2-
Ca2+ KO3
3. Lewis acid : (c), (d), (e), (i); Lewis base : (a), (b), (f), (g), (h)
(b) 3Mg*+2*N------ - 3Mg2+, 2[ZNa3- or by electron
4. C—H < Br—H < F—H < Na—Cl < K—F
configuration.
5. (a) K+ :o—i—o: 8. Refer Table 2.1
9. -738.5 kJ mol-1
zoz 10. (a) BaCO,0 < SrCO,0 < CaCO,0 < MgCO„0 < BeCO,0
(b) K2CO3 < Na2CO3 < Li2CO3 (c) KC1 < NaCl < LiCl
(b) i:o—ci:r ca2+[;o—ciir (d) Lil < LiBr < LiF
11. K+ >Ca2+ (size), K+ <Ca2+ (charge)
O2- is identical, polarising power of K+ < Ca2+ thus, K2O is
-If
:o:
■ ? more ionic than CaO thus, lattice energy of CaO > K2O.
(C) H—N—H :o I —o:
• • —ci •• 12. (a) MgO (b) LiF (c) Mg3N2
H :o: 13. (a) N-H 0.98 A (b) C—Cl 1.76 A
C—H 1.05 A
(c) 0—H 0.94 A (d)H—C 1.05 A
6. (a) [Z S—S Z]2“ similarly others. O-Cl 1.65 A C=N 1.16 A
(e) C—H 1.05 A (f) C—H 1.05 A
7. (a) Ca:[Ar]4s2 ---- > Ca2+ [Ar]
C=C 1.34 A C=C 1.22 A
Chapter 2 : Chemical Bonding | 43
!

14. Br2, other molecules have a dipole in addition to van der


Waals’ forces.
:o
h
Cl 19.
T I ..
H—C—C— O-H
/F"
15. (a) "!c—C==C: (b) F-: ’—F (c) Cl—1 H
F Cl 20. (a) A1H" (b) A1F3- (c) A1CL
‘i O J

I 16. There must be considerable double bond character in the 21. Due to lone-pair on N
bonds. 22. In PFg lone-pair on P is easily donated compared to that
17. :o = s-ci: ofN.
23. Lewis bases (donor) are H2O (OH~), CO, Cl", H2O, O2'
:ci: 0 0 i2-
T ?
Formal charge = valence electrons 24. (a) N=N -> 0 (b) O<-C1—0— C1-»O
_
bonding electrons
- - lone pair electrons I I
0 0
4
Oxygen = 6------ 4 = 0 CL CL Cl
2
Sulphur = 6 - — - 2 = 0 A>Cr<A (d)
CL
Al:
CL
:AI<
Cl
o O-H H
Chlorine = 7- -- 6= 0
2
(e) H-O-P—O-H (f) H-O-P—O-H
18. (a) H; N.-O-xH
•X oo
H-N-O
I 00
—H I I
H 0 0
X X X X XX .. •• XX 0
(b) jfx »n: :n-*f
XX XX
X
X *F
XX
—N=N—XX
F* (g) H-P-O-H (h) •
4* 0
0
• • —N=o:
(C) H-O
25. (a) Covalent (b) Ionic
0 make dots yourself
(d) H-N-N-
0
H

Valence Shell Electron Pair Repulsion DEEP Focus


(VSEPR) Theory (Gillespie Theory) To find which geometry applies, we determine the Steric
number, SN, of the central atom, which is defined as
• The shape of the molecule is determined by repulsions
between all of the electron pairs present in the valence "Number of atom bonded' Number lone pair'
shell. SN = to central atom +
, on central atom ,
• There can be
The steric number is used to predict molecular geometries.
— (lone pair-lone pair) If there are no lone pairs on the central atom then
— (lone pair-bond pair)
— (bond pair-bond pair) repulsions. SN = number of atoms bonded to central atom = n
• Repulsion is in order There is no repulsion and geometry is retained when
lone-pairs are present, the situation changes slightly due to
(Ip - Ip) > (Ip - bp) > (bp - bp)
various types of repulsion as shown. Double-bonded or
• The magnitude of repulsion between pairs of electrons triple-bonded atoms count the same as single-bonded atoms in
depends on the electronegativity difference between determining the steric number.
the central atom and the other atom.
• Double bonds cause more repulsion than single bonds, InCO2, two double-bonded oxygen atoms are attached
and triple bonds cause more repulsion than a double to central carbon and there are no lone pairs on that atom,
bond. thusSN =2
• From the Table 2.6, it is quite evident that presence of o=c=o
lone pair will change geometry of the molecule (which ? ?
Consider this double-bonded atom as single atom.
is not shown in Table 2.4).
44 | Essential Inorganic Chemistry

Effect of Electronegativity lone pair in NF3 causes a greater distortion from


tetrahedral and gives a F—N—F bond angles of 102° 30'
NF3 and NH3 both have structures based on compared with 107° 48' inNH3.The same effect is found in
tetrahedron with one comer occupied by a lone pair. The H2O (bond angle 104° 27') and F2O (bond angle 102°) and
high electronegativity of F pulls the bonding electrons also in H2O (bond angle 104° 27') and H2S (bond angle
further away from N than in NH3. Thus, repulsion 90°).
between bond pairs is less in NF3 than NH3. Hence, the

Table 2.6 The Effect of Bonding and lond Fairs on Geometry and Bond Angles
Number of
Distorted
Orbitals on Theoretical
Example Bond angle geometry due
central atom shape VSEPR
Bond pairs Lone pairs to repulsion
notation

BeCI2 2 Linear 2 0 ax2 2 180° Linear


BF3 3 Plane triangle 3 0 ax3 3 120° Plane triangle
SO2 3 Plane triangle 2 1 ax2e 3 119° Angular
CH4.SiH4 4 Tetrahedral 4 0 ax4 4 109° 28' Tetrahedral
nh3 4 Tetrahedral 3 1 ax3e 4 107° 48' Trigonal
pyramidal
NF3 4 Tetrahedral 3 1 ax3e 4 102° 35' Pyramidal

h2o 4 Tetrahedral 2 2 ax2e2 4 104° 27' Angular

f2o 4 Tetrahedral 2 2 ax2e2 4 102° Angular


h2s 4 Tetrahedral 2 2 ax2e2 4 90° Angular

pci5 5 Trigonal 5 0 ax5 5 120° and 90° Trigonal


bipyramidal bipyramidal
XeOjFj. SF4, 5 Trigonal 4 1 ax4e 5 101° 36'and Irregular
if; bipyramidal 86° 33' tetrahedral
CIF3 5 Trigonal 3 2 ax3e2 5 87° 40' T-shaped
bipyramidal
I^.XeF? 5 Trigonal 2 3 ax2e3 5 180° Linear
bipyramidal (also see text)
sf6 6 Octahedral 6 0 ax6 6 90° Octahedral
XeOF4,BrF5 6 Octahedral 5 1 ax5e 6 84° 30' Square
pyramidal
ICi;,XeF4 6 Octahedral 4 2 AX4E2 6 90° Square planar

IFz 7 Pentagonal 7 0 ax7 7 72°,90° Pentagonal


bipyramidal bipyramidal

E stands for lone pair.

Isoelectronic Principle Applications


Isoelectronic species usually have the same structure.
This may be extended to species with the same number of
valence electrons.
Species Sterlc number Structures
(a) Ammonia (NH3) There
are (Ip - bp) and (bp - bp)
repulsions. The steric
number is 4, and the
H
t lone-pair
bond-pair

electron pairs arrange


ch4 . nh; . bf; 4 Tetrahedral themselves into an approximatly tetrahedral
CO^- , NO3 . so3 3 Planar triangle structure. The lone-pair is not identical to three
co2. N3 , no2 2 Linear bonding pairs, however, so there is no reason for the
pci5 5 Trigonal bipyramidal electron pair structure to be exactly tetrahedral. It
is found that lone pair tends to occupy more space
sf6 6 Octahedral
than bonding pairs hence bond angles are
reduced to 107° 48' and structure is pyramidal.
Chapter 2: Chemical Bonding | 45

DEEP Focus (g) Iodine heptafluoride (IF7) This is 7


The presence of the lone pair causes slight distortion from the only common example of a Fr-~
109°28'to non-transition element using seven F
F
orbitals for bonding giving a
(b) Water (H2O) : There are two bond H H pentagonal bipyramid. F-
pairs and two lone pairs. Presence of F
(h) CIO3 The central chlorine atom
two lone pairs distorts the bond angle
from 109° 28'to 104° 27'. has all of its valence electrons (Cl loses one electron
(c) Phosphorus pentachloride in the formation of CD involved in bonds to three
1 oxygen atoms and no lone pair exists. Steric
(PC15): There is no lone pair
number is three and structure is thus trigonal
so the structure is not Cl< Cl planar.
distorted. However, a trigonal
bipyramidal is not a completely cv (i) C1O2 Chlorine (being positively charged) has six
regular structure, since some valence electrons. Two pairs (in all four electrons)
bond angles are 90° and others are involved in bonding with two oxygen atoms and
120°. Cl thus one lone pair exists. Thus, steric number
Symmetrical structures are (2 + 1 = 3) is three. The predicted geometry is bent
usually more stable than asymmetrical ones. Thus, PC15 is with an angle some what less than 120°.
highly unstable, and in the solid state it splits into [PC14 ]*
Illustration 1 Use the VSEPR model to predict the
and [PCl6r ions. geometry of the following
2PCI5 —» [pci4r +[pci6r (a) AsH3 (b) OF2 (c) Aici; (d) CA (e) C1F2+
tetrahedral octahedral
unsymmetrical Solution
symmetrical
(d) Chlorine trifluoride (C1F3) Repulsion/
Species Structure Geometry
F angle
There are three bond pairs and
(a) AsH3 Ip-bp trigonal pyramidal
two lone pairs. As a general H— As—H
bp - bp
rule, if lone pairs occur in a Cl H bond angle
trigonal bipyramidal they will less than
be located in the equatorial ’ 109.5°
positions (round the middle) (b) OF2 Ip-Ip bent shape (like
rather than the apical positions •F-O-
• •• F
Ip-bp H2O),
(top and bottom), since this bp - bp since
arrangement minimises the repulsive forces. much less electronegativity
(e) Sulphur tetrafluoride than that of of F > 0, hence
(SF4) There are four bond AsH3 bond angle is less
than that of H2O
pairs and one lone pair. To
F
minimise the repulsive forces :ci:
bp - bp
the lone pair occupies an : ci—ai—ci:
(c) aici; (no lone pair) tetrahedral
equatorial position, and F 109.5°
atoms are located at the other • :ci:
four corners.
(d) C2H4 H H bp-bp trigonal planar
(f) Triiodide ion (Ip I3 ion is C=C
H H (no lone pair)
made up of an molecule with ..
120°
an I" bonded to it by means of __
(e) CIF2+ as in (b) angular
a coordinate bond in which — :ci— f :
is lone pair acceptor \ |\
(Lewis acid) and I" the lone xA :f:
pair donor (Lewis base). \
There are two bond pairs and Illustration 2 The triiodide ion (I3) in which the I
three lone pairs in the
outer-shell of central atom. To minimise the atoms are arranged in a straight line is stable, but the
repulsive forces the three lone pairs occupy the corresponding F3“ ion does not exist. Explain.
equatorial position. The ion is therefore linear in Solution F3 and I3 are of same group. I can expand its
shape with a bond angle of exactly 180°.
octet but F cannot and thus, in I3 octet rule is not violated,
I2 + r ---- > Il- but in F3" octet rule is violated.
Lewis acid Lewis base
46 | Essentia/Inorganic Chemistry

Assessing Molecular Shape Sigma and Pi Bonds


There can be following types of overlapping along the
DEEP Focus axes (end to end):
By use of VSEPR theory, we can decide the shape of the
molecule based on four-step procedure.
• Draw a plausible Lewis structure of the species (molecule or
polyatomic ion) Fig. 2.1 Sigma overlap (lobes point along nuclei)
• Determine the number of electron pairs around the central
atom and identify each pair as being either a bond-pair or a
lone pair.
• Establish the geometrical orientation of the electron pairs
around the central atom (linear, trigonal planar,
tetrahedral, trigonal bipyramidal, or octahedral). Fig. 2.2 Pi overlap (lobes are at right angles to the line
• Use the positions around the central atom that are occupied joining the nuclei)
by other atoms (not lone-pair electrons) to specify the
geometrical shape using Table 2.6. - (s-s) overlapping when s-orbital overlaps with
another s-orbital
cios- - (s - p) overlapping, (p can be px or py or p2)
• Number of valence electrons is “ (px - px) overlapping
from Cl from 0 due to charge - any of the hybrid orbitals overlaps with another
7 + 3x6 + 1 =26
hybrid orbital or s or p orbital, (to be taken later)
• Less electronegative atom is central atom in the Lewis
structure. Thus, plausible structure is Bond formed in this manner is called sigma (o) bond
in which electron density is concentrated in between the
:o—ci—0" two atoms, and on a line joining the two atoms.
:o: • Double or triple bonds occur by sideways overlap of
orbitals (like (py - py)and(pz - pz) orbitals) giving pi
• There are three bond pairs (—) and one lone pair (..)
on chlorine atom total being four. (k) bonds in which electron density also concentrates
between the atoms, but on either side of the line
• By Table 2.6, the orientation of four electron pairs is
joining the atoms.
tetrahedral.
• In VSEPR notation the CIO3 ion is AX3E (E stands for • The shape of the molecule is determined by the
lone pair of electrons — Table 2.6). The ion has a o-bonds (and lone pairs) but not by the k bonds. Pi
pyramidal shape. bonds merely shorten bond length. Thus,
ici; (C=C)<(C=C)< (C—C)
• Number of valence electrons bond length
*
from I from Cl due to charge bond energy
7 + 7x4 + 1 =36 • If only single bonds exist in a molecule, all will be of
• I is the central atom being less electronegative and a type. In a double bond, one will be o and the other
four Cl atoms around it. 7t type while in a triple bond, one will be o and other
• To complete octet, total electrons required = 32 two it.
a
that means four electrons are extra and lie on I atom H —H 0=0 N^N
as two lone pairs. Thus, plausible structure is a it 2n
•• ••
:ck /Ci: H

••
’’ four bond pairs (—)
C=N
I
:ci xci: two lone pairs (*)
•• •• H—C C—H
N= =N I I
• By Table 2.6, the orientation of six electron-pairs H—C. ,C—H
C=N
around the central atom is octahedral.
(8n, 8a)
• The ICi; anion is of the type AX’4£2 and thus, it is H
square planar. (3jt, 12a)
Chapter 2: Chemical Bonding | 47

• In a reaction, it is the it bond that actually takes part — axial (p-p) overlap (o) —
H H — lateral p-p overlap (n) —
H a H I a I |— lateral p-p overlap (n) —
c=c + H, ---- > H—C-C—H
H- K H || 2p T T T T 2p
H H
Px Py Px Pt Py Px
Thus, K-bond is more reactive than g bond. 2s U 2s

Valence Bond Theory Is n n Is

The covalent bond is a region of high electron Py. Py


charge-density (high electron-probability) that results Pz jr )/-\Pz
from the overlap of atomic orbitals between two atoms. In
general, the greater the amount of overlap between two Px Q Px
orbitals, the stronger the bond. For each bond there is a
condition of maximum atomic orbital overlap leading to
maximum bond strength at a particular internuclear Fig. 2.3 Structure of Nj by valence bond method
distance (bond length). This is called Valence Bond
• In NH3 : Three o bonds are by (s - p) overlap of H
Approach.
• This is a localised electron model of bonding. and N-atoms
7T
S\
• Most of the electrons retain the same orbital locations Hr—*■
,'T\ ls(H)
; i
i i
as in a separated atoms, and the bonding electrons are
localised (fixed) in the region of atomic orbital overlap. N-atom r2p Sl
• In Hg molecules, H—H o bond is by s-s overlapping Px Py pz s - p overlap

s T
2s n
I n I _ Is u
s
Is H2 molecule

• In F2 molecule F—F a bond is by axial overlapping Hybridisation


of two p-orbitals. We consider formation of CH4 molecule based on
(p-p) overlap-. valence bond approach. To account for tetravalent
nature of carbon, we take it in excited state in which there
are four unpaired electrons and four C—H g bonds are
2p H n ? ? n 2p
supposed to be by overlap of s - s, s - px , s - py and s- p2
Pt Py Px Px Py Pt types.
2s 1X U 2s

Is U Is

• In O2 molecule One o bond is by axial overlap of T T 2p3


2p2
p-orbitals and it bond is by lateral overlap of | TX | 2s2
p-orbitals.
Is
p-p lateral overlap (n)
p-p axial overlap (a) Ground state Excited state
What do you think about these (C—H) bonds?
2p | TX 1 T | T|
Pt Py Px
m
Px Py
nnPt
2p
- are they equivalent?
2s Tn I H 2s
- are they having different bond lengths and bond
energies (due to different types of overlap)?
Is u Is
our answer is:
- they are equivalent;
• In N2 molecule One a bond is by axial overlap of
p-orbitals and two n bonds are by lateral overlap of - they have equal bond energy and bond length.
p-orbitals.
48 | Essential Inorganic Chemistry

This is due to formation of new orbitals before How to Determine the Type of Hybridization
overlapping takes place, by a process called
Hybridization. For this, one is need to know first of all the numbers of
lone pairs (by use of VSEPR theory). We adopt two
Hybridization is defined as the concept of methods for study of type of Hybridization :
intermixing of orbitals of same energy or of slightly Method I : Count number of atoms directly
different energy to produce entirely new orbitals of attached to central atom + lone pairs + single electrons
equivalent energy, identical shapes and Method II: Counter bonds +---- > (coordinate) bonds
symmetrically disposed in plane. New orbitals formed + lone pairs + single electrons
are called hybrid orbitals. This number comes out to be
• Only the orbitals of an isolated single atom can 2 => sp hybridization
undergo Hybridization. 3 =» sp2 hybridization
• The hybrid orbitals generated are equal in number to 4 sp3 hybridization or
that of the pure atomic orbitals which mix up. dsp2 Hybridization
• A hybrid orbital, like the atomic orbitals, cannot have
more than two electrons of opposite spins. (decided by way of orbital
used)
Hybrid orbitals do not make tc bonds. If there are
n-bonds, equal number of atomic orbitals must be left 5 => sp3d hybridization
unhybridised for it bonding. 6 => sp3d2 hybridization or
eftsp3 hybridization
Number of orbitals and type of Hybridization have (based on outer d or inner d-orbitals are used).
been summarised in Table 2.7. We illustrate methods in Table 2.8.

Table 2.7 Hybrid Orbitals and their Geometric Orientation


Hybrid orbitals Predicted bond Actual shape
Atomic orbitals (number) Orientation Example (Refer table 2.6)
(number) angle

s + p (two) sp (two) linear BeCI2, CO2, C2H2 180° linear


s + p + p (three) sp2 (three) trigonal planar BF3> C2H4, SO2, SO3 120° trigonal planar
s + p + p + p (four) sp3 (four) tetrahedral ch4 109° 28' tetrahedral
nh3 107° 48' trigonal pyramidal (due to
h2o Ip - bp repulsion) angular
104° 27' (V-shaped) due to Ip - Ip
and Ip - bp repulsion
s+p+p+p+d (five) sp3d (five) trigonal PCI5 120°and 90° trigonal pyramidal
pyramidal 101° 36'and irregular tetrahedral
SF4
86° 33'
I3 180° linear
s+p+p+p+d + d(six) sp3 d2 (six) octahedral SF6 90° octahedral
[Fe(H2O)6]3+ 90° octahedral
90° square planar (with two lone
XeF4
pairs)
d+d+s + p+ p+ p (six) d2sp3 (six) octahedral [Fe(CN)6]4- 90° octahedral
[Co(NH3)6]3+ 90° octahedral

d + s + p + p (four) dsp2 (four) square planar [Ni(CN)4]2- 90° square planar


[Pt(NH3)4]2+ 90° square planar
Chapter 2: Chemical Bonding | 49

Table 2.8 Method of Determination of Type of Hybridization


Number of electrons attached to central atom Total hybrid orbitals
Central
Molecule or Ion atoms a bonds
atom Ip single -» Method I Method II
electron (d) bonds (e) (a + b + c) (b + c+ d + e) Type
(a) (*>) (c)

H—Be—H Be 2 2 2 2 Sp
H\ B 3 3 3 3 sp2
8—H
H
H C 4 4 4 4 sp3
I
■C
H‘
I ■H
H
0=C=0 C 2 2 2 2 sp
H-C=C-H C (any) 3 3 3 3 sp2

H H

H—C=C—H C (any) 2 2 2 2 sp
N 3 3 1 4 4 sp3
N-
H •H
H
0 2 2 2 4 4 sp3
.0-
••
H' XH
Cl •

P 5 5 5 5 sp3d
Cl Cl
Vl Cl
F
F -^S-F sp3d2
S 6 6 6 6
IV
F
/O N 2 1 1 1 3 3 sp2
•N
0
:n=n: N (any) 1 1 1 2 2 Sp
S 2 1 1 1 3 3 sp2
or
j o
0
I C 3 3 3 3 sp4
2
H3C-C-O-H
2
ch2=ch—ch=ch2 C (any) 3 3 3 3 Sp

0
T sp3
H-O-S-O-H S 4 2 2 4 4
i
0
0
I SP3
H-O-S-O-H S 4 4 4 4
I
0
N 2 2 2 2 sp
:na =n♦ =o:

0 1 1 2 3 3 sp2
a Na 1 1 1 2 2 Sp
50 | Essential Inorganic Chemistry

Applications (e) SF6


(a) BeH2 3s 3p 3d
Is 2s 2p S [Ne] n nTT ?
u,
excite
T
i.
T ground state
3d
3s 3p
Is 2s/^\ 2p / S [Ne] : T ? ? T T T
sp excited state
Is ’ S (hybridised) sp3d2 3d

EH M 'H(hybri
— dised)
[Ne] 1 1 1
A B C D
1 1
E F
A and B sp-hybrid orbitals from o bonds with H by sp3(P hybrid orbitals form o-bonds with F-atoms
(sp-s) overlap.
(b) BH3
Is 2s
DEEP Focus
There, are also two types of new hybrid orbitals
n T TTT (i) an sd hybrid orbital
and (ii) an a pd hybrid orbital.
(i) is formed by interaction of one s- and one d 2 - orbital
sp2 s + d 2 ---- > two sd hybrid orbital
bh
ground state Combination of d 2 with s increases the (+) lobes of d?2 and
Is
diminishes the negative one. Angular hybrid results from
T! T T T (hybridised)
combination of s with other d-orbitals.
ABC
A, B and C sp2-hybrid orbitals form a bonds with H by
(sp2 - s) overlap.
(c)CH4
Is 2s _2P

n T tItIt
Is 2s/ 2p x/1excite
sp3
c0 0 |T| t| I ground state s + dx2_y2
Is T
[tT] | T | T | T | T | (hybridised)
A B C D
A, B, C and D sp3-hybrid orbitals form a bonds with H
by (sp3 - s) overlap.
(d) PC15
3s 3p 3d
(ii) pz ± d 2 -> Each combination increases one lobe of
P [Ne] T1 T T T
and diminishes the other.
ground state
3s 3p 3d

P (NeJ T T T T T +
excited state
P (hybridised) vsp3d 3d
Pz^z2
[NeJ T | T | T | ? | T
A B C D E
• While determining type of hybridization on the atom,
sp3d hybrid orbitals form o-bonds with Cl atoms k bonds are never taken into account, but lone pairs
are always considered.
Chapter 2 ; Chemical Bonding I 51
H
la :N !!!!: N:
H�C�H H..5l..�C-H one a bond
la 27t and one lone pair on each N
( sp)
four a bonds three a bonds with each C two CJ with each C Bond energies and bond lengths of different types of
(sp3) (sp2) ( sp)

..
•• •• bonds have been summarised in Table 2.9.
N -N-H H-0-H

three a bonds and


twoa bonds
and two lone pairs on 0
one lone pair on N
(sp3)
(sp3)

Table 2.9 Some Representative Bond Energies and Bond Lengths


Bond
Bond energy Bond energy Bond length Bond energy
Bond Bond length pm Bond
kJ mor 1 Bond
kJ mor 1 length
kJ mor1 pm
pm
H-H 435 74 C=C 611 134 N-N 163 145
C-H 414 C!!!!C 837 120 N=-N 418 123
sp 3 -H 110 C-N 305 147 N:!!!!!:N 946 109
sp 2 -H 109 C=N 615 128 N--0 230 136
sp-H 108 C!!!!N 891 116 N=O 590 115
H-N 389 100 C-0 360 143 0--0 142 145
H--0 464 97 C=O 728 123 0=0 498 121
H--S 368 132 C-CI 326 177 F-F 159 128
H-F 569 101 Cl-Cl 243 199
H-CI 431 136 Br-Br 192 228
H-Br 368 151 1-1 151 266
H-1 297 170
C-C 347
sp 3 -sp 3 154
sp 3 -sp 2 150
2
sp -sp 2
147
sp3 -sp 146
sp 2 -sp 143
sp-sp 137

Target Practice 2
; 1. Sterle number of the central atom in IF;
is ........ . 4. Using the general principle of isoelectric groups, identify
2. Sterle number and predicted geometry of AX6 is ........ . the species of line (II) that are "isoelectronic" with those of
3. Which of the following in each pair will have the larger bond line (I). Each entry might have more than one
angle? "isoelectronic" partner in the other line.
I II
(a) CH,, NH3 (I) (H2 N)2 CO,HONO2 , OCO CO(CN)2 c10; Si 3 O:-
(b) OF2, OCl 2 (II) H2CCO, (HO) 2 CO, ONN,H2 CNN,H3CNO2 ,F2CO, BF,
(c) NH 3, NF3 Bp=, (CH:: )2 CO, CH2 CN-, SO2, Cyclic (SO3 )3
(d) PH 3 , NH3
52 | Essential Inorganic Chemistry

Answers
1. Two lone pairs + four bonding atoms = 6 4. For each horizontal line shown the molecules, ions, or the
/ groups underlined are “isoelectronic”.
F F (I) (II)
HONO2 CH3NO2
F F
/ (H2N)2CO (HO)2CO,F2CO, (CH3)2CO
2. Six, octahedral (CN)2 B2O2 (2x9 electron)
3. (a) CH4 (b) OC12 (c) NH3 (d) NH3 C1O2* SO2 (18 electrons)
Si3O«- (SO3 )3 (3 x 40 electrons)

Practice Exercise 2
1. The H—P—H bond angles in PH3 are smaller than the 13. Match each of the following species with one of these
H—N—H angles in NH3. Explain. Hybridization schemes
2. PC15 has the shape of a trigonal bipyramid whereas IF5 has (I) (ID
the shape of square pyramid. Explain. (A) sp (a) SF6
3. Which one of each of the following pairs is expected to have (B) sp2 (b) CS2
the larger bond angle? (C) sp3 (c) SnCl4
(a) H,0 and NH, (b) SF2 and BeF2 (D) sp3d (d) NO-
(c) BF3 and BF” (d) NH, and NF, (E) sp3d2 (e) AsF5
4. Which of the following molecules are linear?
14. The ion C1F“ is linear, but the ion C1F* is bent. Describe
IC1;, IF*. 0F2, Snl2, CdBr2
5. Which molecule out of AX., AX,, AX,0 and AX,D , is most Hybridization schemes for the central Cl atom to account
0 4
for this difference in structure.
likely to have a trigonal bipyramidal structure? If the
central atom, A, has no lone pairs, what type of 15. SbF3 reacts with XeF4 and XeF6 to form ionic compounds
Hybridization will it have? XeF*SbF"
O O
and XeF^SbF;.
0 0

6. Two of the following species have the same shape. Which Describe the geometries of these two compounds.
are these two ? What is their shape ? What are the shapes of 16. What are the Hybridization states of the C and N atoms in
the other two? NI3,1", S03”, NO”? the following molecule?
7. Use the VSEPR model to predict the shape of nh2
(a) OSF, (b) O2SF2
(c) XeF4" (d) CIO;
(e) I3” N c^H
8. Predict the geometrical shapes of the following molecules
,C
I I
c
and then predict which molecules you would expect to have H
resultant dipole moments : (also refer sec-dipole moment) 0^ N^
(a) S02 (b) NH3 I
H
(c) H,S (d) C^
(e) SF6 (f)CH2C12 17. What are the total number of c and n bonds in
9. Match the terms in A with those in B (a) methyl isocyanate CH3NCO ?
(b) tetracyanomethane C(CN)4 ?
(A) (B)
(c) naphthalene C10HB
(a) NH3 molecule (i) Heitler London
18. When ethene reacts with H22*, ethane is formed. Which bond
(b) SF6 (ii) sp3d2
(c) Metallic bond (in) Pyramidal shape is affected?
(d) VB approach (iv) Mobile electrons 19. Although both carbon and silicon are in group IVA (14),
very few Si = Si bonds are known. Explain.
(e) MO theory (v) Bond order
(vi) Bond angles
20. Aspirin has the following H o
(0 VSEPR theory connections among atoms. I I /H
10. Two different bond lengths are observed in PF5 molecule, Complete the electron dot c c CT
C
structure for aspirin. Also give the
but only one bond length is observed in SF6. Explain.
o and n bonds and Hybridization of C.
I
11. Draw an electron dot structure for Br3. Deduce an .C
approximate value for the bond angle and explain your
each atom. I
0
deduction.
12. Predict the shapes of
0
cI C<H
(a) TeCl4 (b) C1O3
(0 CIO* (d) CIO* H
Chapter? . Chemical Bonding | 53

Answers
1. See text 2. See text 16. Each C and N, both are sp2 hybridised.
3. (a) NH3 (b) BeF2 (c) BF3 (d) NH3 4. ICl2,CdBr2 H C=N
5. AX5,sp3d 17. (a) H—C—N=C=06o,2rt (b) N=C—C—C=N 80, 8n
6. NI3 and S03~ are trigonal pyramidal; I3 is linear; NO3 is
trigonal planar. H C=N
7. (a) trigonal pyramidal (b) tetrahedral (c) sqare planar
H
(d) tetrahedral (e) linear.
H
8. Resultant dipole moment in (a), (b), (c) and (f).
9. (a>—(iii) (b)—(ii) (c)—(iv) (d)—<i) (e)—(v) (0—<vi)
(0 19a,5n
H
10. PC16 has trigonal bipyramidal structure in which bond angles
of 90° and 120° exist, hence, two types of bond length, while in H
case of SF6 there is ony one type of angle (90°) in octahedral
structure. 18. n-bond of C=C is affected, thus n bond is more reactive than
G-bond.
11. The molecule should be linear, using Br
two axial dsp3 orbital, with three other 19. Larger size of silicon (Si) results in poor sideway overlap of
dsp3 orbitals occupied by lone pairs. p-orbitals to form n-bond.
Br
12. (a) seesaw shaped, (b) trigonal •. H :b
pyramid with bond angles less than
109.5°, (c) angular, with bond angles
T Br ^C
II
../H
less than 109.5° (d) trigonal. H—C '0
••
20. 5x, 21g, C and 0 with
13. A—b; B—d; C—c; D-e; E-a H-C. multiple bonds -sp2, C and 0
14. C1F2 — sp3d (three lone pairs on Cl at three comers and two F with single bonds -sp3
at vertices)
H

15.
CIF2+ — sp3 (two lone pairs on Cl)
XeF3+ T-shaped
.0A<H
%u. IXH
XeF5+ Square pyramidal H
SbF6- Octahedral

Following MOs are formed when atomic orbitals overlap:


Molecular Orbital Theory
Molecular orbitals
• There is Linear Combination of Atomic Orbitals
(LCAO) to form Molecular Orbitals (MOs) Atomic orbitals Bonding Anti-bonding
1s + Is G 1S g * 1s
• The number of MO produced is equal to the number of
2s + 2s G 2s a * 2s
atomic orbitals that are combined.
2Px + 2px c2px o’ 2px
• Two types of MOs are formed : 2py + 2py n 2px 7t* 2Py
* Bonding MO at a lower energy than the original 2pz + 2pz ic2pz- 7C* 2p^
atomic orbital
(0 before MO indicates that bond formed is 0 type and
* Anti-bonding MO at a higher energy it before it indicates bond formed is tc. indicates
anti-bonding
anti-bonding MO)
O* Is • Electrons are filled in the increasing energy of the MO
which is in order (Aufbau rule)
(a)0 1s,0*ls, o2s, 0*2s, ©2Px,
« %Py Z*2py
o*2px
rt2pz k*2Pz’
<u
c
W increasing energy (for electrons > 14)
I
\0 Is n2py
(b) 0 Is, a* Is, a 2s, a* 2s 0 2px,
LITf ’ ^Pz

bonding increasing energy (for electrons < 14)


54 I Essential Inorganic Chemistry

DEEP Focus Energy


Atomic
orbitals
Molecular
orbitals
Atomic
orbitals Energy
• 1t 2py , 1t 2p, and also 1t • 2pY , 1t * 2 p7 are degenerate
levels (with same energy). Electrons enter MO of
identical energies singly before they pair up (Hund's
rule).
• The maximum number of electrons that can be
assigned to given MO is two (Pauli's exclusion
principle).
• Number of bonds between two atoms is called
bond-order and is given by
Bond-order=( N,; NA) -·· Ji ....
········a· • 2s······•··••••"..
····•·•
where NB = number of electrons in bonding MO
t-t• 2s
a 1s
NA = number ofelectrons in anti-bondingMO
• For a stable molecule/ion, NB > NA
• Given molecule or ion is paramagnetic if it has
unpaired electrons in MO, otherwise diamagnetic.
• MOs formation for � and 02 are represented in
Figs. 2.4 and 2.5 respectively.
0 02 0
Atomic Molecular Atomic
Energy Energy Atom Molecule Atom
orbitals orbitals orbitals
Fig. 2.5 Electronic configuration, atomic and molecular
orbitals for oxygen

• Various other species have been summarised in


Tables 2.10 and 2.11.

H H2 H
Atom Molecule Atom
Fig. 2.4 Electronic configuration, atomic and molecular.
orbitals for hydrogen

Table 2.10 MO Electronic Configuration of Diatomic Species up to 14 Electrons


Electronic configuration Paramagnetic (P}
Molecule or Total no. of Na NA Bond
S.no. order diamagnetic (D)
Ion electrons
a1s (J ., s a 2s a• 2s 7t 2Py 7t 2p, (J 2p/C
1. H2 2 2 2 0 1.0 D
2. H2 1 1 0 0.5 p
3. He2 4 2 2 2 2 0.0 D
4. He; 3 2 1 2 1 0.5 p
5. Li 2 6 2 2 2 4 2 1.0 D
6. Be2 8 2 2 2 2 4 4 0.0 D
7. 82 10 2 2 2 2 1 1 6 4 1.0 p
8. C2 12 2 2 2 2 2 2 8 4 2.0 D
9. N2 14 2 2 2 2 2 2 2 10 4 3.0 D

10. a:· 14 (same asN 2 } 10 4 3.0 D

11. N2+ 13 2 2 2 2 2 2 9 4 2.5 p


12. cw 14 (same asN2 } 10 4 3.0 D
Chapter 2: Chemic;al Bonding I 55
Table 2.11 MO Electronic Configuration of Diatomic Species of More Than 14 Electrons
Electronlc configuration ;
s. Molecule
or
Total no. of
Ns NA
� Paramagnetic (P)
No. electrons a 1 s a� s a 2s a*2s a 2p 7t 2Py 7t. 2P,
'tJ
Diamagnetic (D)
Ion ., 7t 2pz n*2pz a*2p.,

1. 02 16 2 2 2 2 2 2 2 10 6 2.0 p
2. o•2 15 2 2 2 2 2 2 2 10 5 2.5 p
3. NO 15 2 2 2 2 2 2 2 1 10 5 2.5 p
4. o; 17 2 2 2 2 2 2 2 2 10 7 1.5 p
(superoxide
ion)
2
5. 02- 18 2 2 2 2 2 2 2 2 2 10 8 1.0 D
(peroxide
ion)
6. F2 18 2 2 2 2 2 2 2 2 2 10 8 1.0 D
7. Ne2 20 2 2 2 2 2 2 2 2 2 2 10 10 0.0 D

• MO electronic configuration is written similar to that Bonding in Complexes


of atomic orbitals. For example for He2 ( 4 electrons) we
write Valence Bond Theory
• The metal M loses requisite number of electrons to
cr 1s2 cr * 1s2 or KK * (K indicates first bonding form cation. Number of electrons lost corresponds to
MO and K* anti-bonding MO and are core the oxidation number of the metal ion.
electrons) • The metal ion makes available a number of empty
orbitals equal to its coordination number for the
and for � we write formation of coordinate bonds with the ligand orbitals.
KK * cr 2s2 cr * 2s2 cr 2p! 7t 2p; 7t 2p� 7t * 2p; 7t * 2p� • A weak ligand (like �O) will not affect the electronic
configuration of the metal/metal ion.
In Oi, 7t * 2pY and 7t * 2p 2 each has one electron being • A strong ligand (like CN-, OH- etc.) will make
degenerate levels (of equal energy) hence it is unpaired electrons paired. (Also refer chapter-18)
paramagnetic.
I IiI i I i I
,will remain as such

.l
• Greater the bond order, larger the bond length and Fe2• jJ, i I ·when H20 is ligand
greater the bond energy. Thus for 02 , o;, 02, �-
o; 02 02 �- will change to when CW is ligand
bond length
bond order
I u I rJ. I J,t I
bond energy • The metal ion orbitals hybridise to form a new set of
equivalent hybridised orbitals with definite
stereochemical directions and summarised in
Table 2.12.

· Table 2.12 Common Type of Hybridization and Geometry of Complexes


Coordination Examples
Hybridization Geometry
number (CN)
2 sp Linear [Ag(NH3 )2] +
2
3 sp Trigonal planar [Hgl3r, [Cu(PMe3hr
[V0 4 )3-, [Mno 4r, [NiCl4 ] 2-. Ni(CO),
J
4 sp Tetrahedral
4 dsp 2
Square planar [Cu(NH3 )4] •, [Ni (CN) 4 ] -. [Pt (NH3 )4 ]
2 2 2+

5 3
sp d,dsp 3
Trigonal bipyramidal
3
[CuC l 5 ] 3 -, [Ni (CN) 5 ] -, [Fe(C0)5 ]
6 d 2 sp 3 , sp3d 2 Octahedral [Fe(CN) 6]
4
-,
3
[Fe(H 20)6 ] +
56 | Essential Inorganic Chemistry

DEEP Focus • All other characteristics remain as in [Fe(CN)6 J1 .


• If there are unpaired electrons the complex ion is said to be (iii) [FefH.OIg]31
paramagnetic and magnetic moment is = ^N(N+ 2) BM oxidation number of Fe = +3
where N is the number of unpaired electrons. C.N. ofFe= 6
• The complex with so many unpaired electrons is high spin
complex and that with paired electrons and one or two
unpaired electrons is called low-spin complex.
Fe [Ar] Tl|T|T|T|t EHO
• If inner d-orbital is involved in hybridisation, it is called 3d6 4s2 4p
inner-d complex as in
[Fe(CN)6]4'; d2sp3 Fe2+ [Ar] | T| ? I T| ?|T
and if outer d-orbital is involved then it is called outer-d 3d5 4s 4p
complex as in
[Fe(H2O)6]3*; sp3d2 Fe3*in[Fe(H2O)6]3* |t|t|t|t|t| Q | | I I

[Ar]
(i) [FefCNJg]4-
oxidation state of Fe = +2 (H2O is a weak ligand hence no change in
electron-spin. Outer d-orbitals will be used for taking
C.N. ofFe= 6
electron pairs from H2O)
[in all six empty hybrid orbitals are required by six
CN- ligands (a strong ligand)] 4s 4p 4d
[Fe(H2O)6]3+ |t|T|t|t|t| E3 |xx|xx|xx| j i I
Fe [Ar] |T1|T | T| T |T n [Ar] 3d
sp3d2
3d6 4s2 4p
• sp3cfi hybridised Fe
Fe2+ [Ar] H J f T T
• Outer-d complex, H2O being a weak ligand does not
Fe2+in
[Fe(CN)6]',4-
n|n|n affect electron spin in Fe3+.
• A high spin complex with five unpaired electrons,,
3d6
paramagnetic; magnetic moment is V35 BM.
(CN" is a strong ligand making unpaired electrons
• Octahedral geometry, bond angle 90°.
paired)
--------- ----------~l (iv) [FeFe]3~
I

[FefCNld4- [Ar] HHU XX XX XX XX xx xx ; F“ is a weak ligand like H2O hence structure is very
similar to [Fe(H2O)6 ]3+
d2sp3
Electron pair of ligand (v) [Cr(NHs)6]s'
• Complex ion has (Psp3 hybridised Fe2+. oxidation number of Cr = +3
• Inner-d complex (inner d-orbital takes part). C.N. ofCr= 6
• Diamagnetic (since no electron unpaired). As discussed above
• Magnetic moment = 0 (zero spin complex or 4s
3d ------____ 4p
spin-paired complex). [Cr(NH3)6]3* [Ar] | f | T 11 |xx|xx| 0 I.xxjxxjxxj
• Octahedral geometry, bond angle 90° (Table 7.8).
(ii) [Fe(CN)6]3- d2sp3
oxidation number of Fe = +3 • (Psp3 hybridised Cr
If we proceed as in [Fe(CN)6 J1- we have • Inner -d complex
3d _ 4s 4p • Highly paramagnetic, magnetic moment V15 BM.
[Fe(CN)6]3- [Ar] |T1|TJ.| T xx XX XX XX XX XX • Octahedral geometry, bond angle 90°.
d2sp3 (vi) [NiCH/))^

• It has one unpaired electron hence it is paramagnetic Oxidation number of Ni = + 2


with magnetic moment V3 BM. CNofNi= 4
Chapter 2:Chemical Bonding | 57

Ni [Ar] n|u|n| t 111 0 Illi • sp3 hybridised Ni,


• Diamagnetic,
3d8 4s2 4p
• Tetrahedral geometry, bond angle 109° 28'.
Ni2+ [Ar] |n|n|Ti|T|t
3d8 4s 4p
Dipole Moment
In H2, there is no displacement of the electric charge
Ni2+in [Ni(H2O)4]2* |n|n|Tl| TIT I Fl Illi due to same electronegativity of both H-atoms and the
[Ar] 3d 4s 4p bond is non-polar. In HC1, the Cl atom has a more
electronegativity than does the H atom. Electronic charge
(no change, H2O being a weak ligand)
distribution is shifted towards the Cl atom. The H—Cl
3d _4?_jP bond is said to be polar.
[Ni(H2O)4]2+ [Ar]
|tj.|u|u| t | ? | R'hEF h: h h8+:ci:5-
""J" The magnitude of the charge displacement in a polar
• sp3 hybridised Ni covalent bond is measured through a quantity called
• Paramagnetic with two unpaired electrons, magnetic dipole moment p. It is the product of the magnitude of
moment is 2-V2 BM charges (8) and the distance separating them(d ).(Here the
• Tetrahedral geometry, bond angle 109° 28' symbol (8) suggests a small magnitude of charge, less than
(vu) [Ni (CN)412- the charge on an electron)

CN" is a strong ligand, hence unpaired electrons in p =8 x d


Ni2+ are paired making one inner d-orbital vacant for the • If 8 = 4.8x IO"10 esu and d = 1 A= lx 10”8 cm
ligand. thenp = 4.8 x 10F’ 10x lx 10-8

Ni2* [Ar] |n|n|n|?|T| | | | | = 4.8 x 10’18 esu cm


A molecule will have a dipole moment of 1 Debye (ID)
3d8 4s 4p if charges of 1.0 x 10’10 esu are separated by a
distance of 1A
Ni2*in[Ni(CN)4]2~ |TJ.|U|T!|U| ~| |~[ I I I I Thus, rio
4.8 x 10~10 esu cm = 4.8 D
[Ar] 3d8 4s 4p
• In S.I. unit, 1 D = 3.33 x IO"30 coulomb metre (when
[Ni(CN)4]2~ [Ar] nnun XX XX XX XX
charge 8 = 3.33 x 10‘20 C and d = 1 x 10“10 m).
• In diatomic molecule,
dsp2
p =8xd
• dsp2 hybridised Ni, inner d-orbital complex but in polyatomic molecule with angle 0, resultant
• Diamagnetic dipole moment is the vector summation of the vector
moments.
• Square planar geometry, bond angle 90°
/A
(viii) [Ni(CO)4] oxidation number ofNi= 0 /0/2 (&
BJ /____ resultant — AB COS I q
C.N. ofNi= 4 \ 0/2 \"4
XA
(CO (carbonyl) is a strong ligand hence electronic
configuration is affected) where p is the vector moment of the side AB.
fO'l
Ni [Ar] |tl|tl|Tl| 1111 |F Illi • Also M resultant X COS I q I
\Z/
3d8 4s2 4p 1
oc —
0
Ni in [Ni(CO)4] [Ar] |U|TJ-|U|TJ-|tI| [J | | | | Thus, greater the value of 0, smaller the value ofp.
3d10 4s 4p • Symmetrical molecules without lone pair of electrons
will have p = 0
Unpaired electrons are paired and outer
4 s electrons are accommodated in 3d. P2 H , Hi
------->-H ^=^1+^2
4s 4p H-^
= =0
[Ni(CO)4] [Ar] F XX XX XX
In this case resultant p2 of H—B—H on the left is
3d10 „_3
sp cancelled by pT of the right.
58 | Essential Inorganic Chemistry

• o=c=o 0 = 180°, g=0 As the difference in electronegativity increases, ionic


H character increases.
H- • Based on Pauling’s electronegativity scale
H
(EN)X - (EN)H = 0.208 7AH_Z
0 = 120°, planar, p = 0
H H where AH_X is the extra bond energy in kcal mol-1
(0.208 is a conversion factor when electronegativities
H (EN) are given in eV).
• p HqO(0 = 104.5°) > p NH3 (9 = 107°) > p SOo (0 = 120°) ah-x = (actual bond energy (BE) of H - X bond)
Cl Cl Cl -V(BE)h.h (BE)X.X
Cl • Geometry of the molecule can be decided based on the
value of the dipole moment as given in Table 2.13.

Cl
0 = 180°
Table 2.13 Relation Between Molecular Geometry and
0 = 60° 6 = 120° Dipole Moment
Cl
ortho meta para Formula Molecular geometry Dipole moment

Theoretical p = 6.30 D p = 3.80 D AX Linear can be non-zero


p = 0.00 D AX2 Linear zero
Experimental p = 6.00 D p = 3.79 D Bent can be non-zero
p = 0.00 D AX3 Trigonal planar zero
In this case p (o-isomer) > p (m-isomer) > p (p-isomer) Trigonal pyramidal can be non-zero
but experimental value of o-isomer < theoretical T-shaped can be non-zero
value. AXa Tetrahedral zero
This is due to dipole-dipole repulsion in o-isomer that Square planar zero
increases bond angle greater than 60° and p decreases. See saw can be non-zero
AX5 Trigonal bipyramidal zero
OH Cl ch3 Square pyramidal can be non-zero
ax6 Octahedral zero
H Cl ch3
0 All X atoms are assumed to be identical.
• Dipole moment of cis-geometrical isomer is greater
attraction due to repulsion no repulsion than frans-geometrical isomer
H-bonding increases 0, or attraction H3C—C—H H3C—C—H
0 decreases, p decreases no change in
p increases actual value. I I
HjC—C—H h-c-ch3
p = +ve p =0
% ionic character in a molecule
observed value of p Illustration 1 Arrange the following compounds in.
x 100
theoretical value of p order of increasing dipole moment:
Dipole moment Bond length Per cent ionic Cl Cl
Molecule
(Debye) (pm) character Cl
Cl
HF 191 101 39.4%
(a) (b)
HCI 103 136 16.0%
HBr 0.78 151 10.8%
HI 0.38 170 4.7%
Cl
Thus, HF is having only approximately 40% ionic
character, the order being
HF > HCI > HBr > HI
• Ionic character can also be decided based on (0 (d)
electronegativity difference. Molecule H—X is 50%
ionic if electronegativity difference ofX and H is 1.7 eV.
Chemists have found that a correlation exists between Cl Cl
the difference in electronegativity of bonded atoms and
Solution (b) and (d) are symmetrical and thus
degree of ionicity
resultant dipole moment is zero.
(expressed as “% ionic character”)
Chapter 2: Chemical Bonding I 59
In (a) bond moments are towards the same direction It is the trans-isomer. Dipole
but in (c) there is net dipole at the Oz position. Thus, dipole moments of C-F bonds cancel one
moment of (a) is maximum. each other and thus, µ = 0.
(b) = (d) < (c) < (a)
Cl Cl
Fajan's Rule
Cl�Dt Cl f Cl
ca) I
'?
(c)
,:f?' 1-7 This rule is used to decide relative ionic and covalent
character in a molecule. A molecule is predominant
� � covalent if
tCI - smaller the size of cation
- larger the size of anion
- greater the charge on cation and anion
Illustration 2 Predict whether each of the following - ion does not have inert gas configuration.
molecules has a dipole moment : We can also say that these factors favour polarisation.
(a) !Br (b) BF3 (c) CH2 Cl2
• Greater the covalent nature
Solution (a) Since IBr molecule is +-----+ - lower the m.p./b.p.
diatomic, it has a linear geometry. Bromine is I-Br - larger the lattice energy
more electronegative than iodine, hence IBr is polar with - smaller conducting nature
bromine at the negative end. Thus, the molecule does - lower solubility in polar solvent
have a dipole moment. - higher solubility in non-polar solvent.
(b) Since fluorine is more electronegative (a) LiCl, NaCl, KCI, RbCI, CsCI
than boron, each B----F bond in BF3 is polar and Size of cation is the deciding factor (charge of each ion
three bond moments are equal. However, the and size of Cl is same)
symmetry of a trigonal planar shape means that Li• < Na• < K• < Rb• < Cs + size of cation
the three bond moments exactly cancel one
another. ------------ polarisation
(c) CH2Cl2 is similar to CR& (tetrahedral), LiCI > NaCl> KCI > RbCl > CsCl covalent
but it has three different bond angles: HCH, ------------ lattice energy
HCCI, and C lCCl, which are close to but not
l\qual to 109.5°. Since Cl is more -----------➔ m.p./b.p.

t. �ctronegative than C and which is more ------------ conductivity


el '.!tronegative than H, the bond moments do not cancel -----------➔ solubility in H2 0
and the molecule possesses a dipole moment. CH2Cl2 is (polar solvents)
thus a polar molecule. -----------➔ solubility in non
Resultant dipole moment is indicated by+--+ polar solvent
lliustration 3 Predict whether or not the following (b) NaF, NaCl, NaBr, Nal
molecules will have non-zero dipole moments. Deciding factor is the size of the anion. Larger size of
F <H the anion, greater the polarisation, hence greater the
Oa, CS2, SCl2, ClfoO, )c=C covalent nature.
F F F- <CI-<Br<1-
---------t size of anion
�...)5� . .
F-<CI-<Br<I­
-------- polarisation
Solution In 08, all of the atoms are the
same, the central oxygen is hypervalent, :v-
o:
•• NaF<NaCl<NaBr<NaI
covalent nature
sharing three instead of the usual two +---------- lattice energy
electron pairs, and hence is electron-poor and must bear a
partial positive charge. VSEPR rules predict a bent (c) NaCl MgC� AICl3 SiCl& PCl5
molecule, making µ non-zero. Na + > Mg2 + > AI3+ > Si4+ > P5+
•• •• max size of cation
:S==C=S: C�, like COi has zero dipole moment.
•• •• •• max--➔
---------- charge on cation
:Cl- S- C
•• •• ••I: SC�, has two lone pair on S atom, and Greater the charge, smaller the size then greater the
shows a non-zeroµ due to noncancelling S-Cl dipoles. polarisation
O Due to bent structure, it has trigonal Na + <Mg2 + <CI3+ <Si4 + <P5+
------------➔ polarisation
II planar structure and thusµ * 0
NaCl<MgC� <AICla < SiClt <PCL.
➔ covalent nature
C
H/ '--H
60 I Essential Inorganic Chemistry
(d) NaF, Na20 Na�
DEEP Focus
3
F- < d- <N- • Of all the electronegative donor atoms, only F, N and
size of anion
0 atoms enter into stable hydrogen bond formation.
F- < 02- < N3-
charge on anion • The weak electrostatic interaction leading to the
F- < ()?-- < N3- hydrogen bond formation is shown by dotted (.... )
polarisation lines. Thus X-H....Y represents hydrogen bonding
NaF < Na2 0 < Na 3N between hydrogen and Y atom.
covalent nature
(e) CuCI and NaCl
[Cu�] [Ar]3d 10 Nature of Hydrogen-Bonding
[Na�] [Ne] Since the hydrogen bond is formed by the most
Cations with 18-electron shell have greater polarising electronegative elements only, it is considered that its
power than the 8-electron-shell ions with the same charge nature is dipole-ion, dipole-induced dipole or dipole-dipole
and size. This is due to the increased electronegativity of interaction of the type :
6- 6+ 6- 6+ 6- 6+
the IS-electron shell ions as the inner electrons have poor A -H... B- or A-H.. B-R
shielding effect on the nucleus. Thus, CuCl is covalent and
NaCl is ionic. (R is the part of the molecule B-R)
6+ 6- o+
DEEP Focus Bond energy of H-bonding in (H... F - H) is
The conditions predicting the nature of bonds are : 113.4 kJ mo1- 1 due to dipole-dipole interactions.
Ionic Covalent • Energies of H-bonds formed by neutral molecules
1. Large cation 1. Small cation have been summarised in Table 2.14.
2. Small anion 2. Large anion
Table 2.14 Energies of Hydrogen Bonds Formed by Neutral
3. Low charge on cation/or 3. High charge on cation
anion and/or anion
Molecules
4. Cation has noble gas 4. Cation does not have Bond Energy of the bond Is kJ moI-1
configuration noble gas configuration
F-H.... O 45
F-H ....F 23-25
illustration 1 Which of the compounds MgCO3 and
0-H ....O 20-30
ZnCO3 is thermally more stable?
Explain. 0-H .... S 15-20
0-H .... F 12-15
Solution Mg2 + : [Ne]
N-H .... N 12-25
Zn2 + : [Ar] 3d10 N-H ....O 25
Mg2 + has noble-gas electron configuration while Zn 2 +
has an 18-electron configuration of the valence-shell.
Thus, polarising power ofZn2+ on an anion is larger than a lntramolecular H-Bonding
cation of the same size and charge. Thus, lattice energy of This type of H-bonding occurs when polar H and
ZnCO3 is greater than that ofMgCO3 and thus, ZnCO 3 is electronegative atom are present in the same molecule.
thermally more stable. �cCH�Q (XO'H
(a) II 0--H
: (b) I :
Hydrogen Bonding � � N-::7'0
t
• The concept of the hydrogen bonding was introduced by o•hydroxy benzaldehyde
0
Latimer and Rodebush (1920). o-nitro phenol
• Hydrogen bonding is said to be formed when slightly
acidic hydrogen attached to a strongly electronegative O-----H-O
atom such as F, N and 0, is held with weak
H3C-C=N
I
N=C-CHa
t
electrostatic forces by the non-bonded pair of electrons
(c I I
)
of another atom. The coordination number of > Ni<
hydrogen in such cases is two. It acts as a bridge H3C-C=N N=C-CH3
between two atoms, to one of which it is covalently i I
bonded, and to the other attached through O-----H-O
electrostatic forces, also called Hydrogen Bond. nickel dimethyl glyoximate (a chelate)
Chapter 2: Chemical Bonding I 61
The necessary conditions for the formation of (b) In hydrofluoric acid (Ill'), there is again
intramolecular hydrogen-bonding are association by H-bonding.
(a) the ring formed as a result of hydrogen bonding
o+ o+ o+ o+
should be planar. H-,, _,,-H........__ _,,-H·, ·'_,H-......._,
(b) a 5- or 6- membered ring should be formed. F F F F
6-- 6-- 6-- 6--
(c) interacting atoms should be placed in such a way
that there is minimum strain during the ring However in the gaseous state, several polymeric forms
closure.
of the HF molecules exist in which the monomers are
In case of anti-form of pyridine-2-carboxaldoxime
there is intramolecular hydrogen bonding but it is not held together through H-bonding. A pentagonal
arrangement of H-F molecules is shown below

'
shown in syn-form
F H 101 pm
-
(lC_,,H
,,' ',
K!!J....1s� ,
'K.; 150 pm
H
0

',,H i�a F,'�


: II �--H/
H, .,.,N
O (c) There is also similar H-bonding in alcohol (R-OH)
anti syn
ammonia (NH 3) and phenol (C6 �OH) molecules.
(d) Carboxylic acid dimerises in gaseous state due to
Intermolecular H-Bonding -
H bonding
This type of H-bonding takes place between H and
electronegative element present in the different molecules 160 pm--
H 1 1 100 H
pm

of the same substance (as in between H2 O and H2 O) or , �•�


different substances (as in between H2 O and NH 3) I /O---H-o, I
H-c-c( /C-C-H
(a) In water molecules I O-H---O I
H H
Due to polar nature of H2 O, there is association of
water molecules giving a liquid state of abnormally high (e) Alcohol is said to be highly soluble in water due to
boiling point. intermolecular H-bonding (between H2 O and
crossed
o+ &- o+ &- o+ &- o+ o-­
H-O---H-O---H-O---H-O R-OH molecules).
I I I I R H
I
R
I I
H6+ H6+ H6+ H 6+
... O-H. .. O-H ... O-H...
If we actually observe, one water molecule is joined to �- o • 6- 0� 6- o �
four water molecules - two with H-atoms and other two However isomeric ether is less soluble in water due to
with O-atoms. Thus coordination nwnber of water
its (ether) non-polar nature.
molecule in water is four.
o- 6+
H CH 3 -CH2 -0-H
.._.,_...,, ·-.-·
polar
H-O
I
less polar
M
H-..'-
I
6+
6+ A&--6+ 6-/H DEEP Focus
: &-: H---0\ 6+
)O---H/V;:-..._
6 +H _,
ti_ H (f) Though the hydrogen atoms in a methyl groups are not

I polarised, if an electronegative group like chloro, carbonyl,

l
O nitro or cyano is attached to it, the C-H bond gets polarised
......._,H due to the inductive effect and the hydrogen atom becomes
slightly acidic resulting in the formation of weak hydrogen
DEEP Focus bonds.
• When ice is formed from liquid water, some air gap is H H
formed (in tetrahedral packing of water molecules). Due to H- -C.aN---H-?--C,.. N---H-?--C""'N
this volume of ice is greater than liquid water and
thus ice is lighter than water. We can say that density H H

�y
decreases when ice is formed. Reversely when ice melts,
density increases but only up to 4°C, after this CH 3
intermolecular H-bonding between water molecules breaks CH3 -C- -H---O=C<
hence volume increases and hence density decreases. Thus,
? CH
water has·maximum density at 4° C.
62 | Essential Inorganic Chemistry

Consequences of the Hydrogen-Bonding species, and in which the true structure cannot
(a) Due to H-bonding boiling points of water, be written at all.
ammonia, hydrofluoric acid are abnormally high (Fig. 2.6.) • 0= O bond length is 121 pm and O—O bond length is
(b) B.P.: H2O > HF > NH3 > CH, 145 pm but in ozone experimental measured O—O
M.P.: H2O > NH3 > HF > CH, bond length is 128 pm suggesting that resonance
hybrid is a true representation.
(c) Solubility of the organic compounds in water is due
to H-bond formation. • In benzene, D and E are extreme resonating structure,
(d) Due to hydrogen-bonding, viscosity (r|) of the liquid while Fisa resonance hybrid of these two structures,
increases as given below (in centipoise (CP).
CH2OH
ch2oh
ch3ch2oh CHOH
ch2oh D
I
ch2oh and again carbon—carbon bond length is 140 pm
n= 1.2 CP T]= 17 CP n= 1070 cp which is intermediate of C—C ( 154 pm) and C=C
400-
(134 pm).
H2O\
• Difference in the energies of the canonical forms and
HF
H2Te resonance hybrid form is called resonance
300- H2Se
SbH3 stabilisation energy (AE), and provides stability to
* h2s^:
nh3
§ 200-
^*SnH4 the molecule.
Q. PH3’ GeH4
O>
c
75 100- XsiH4
rn ch4

Fig. 2.6 E
E5
®
a
Resonance UJ

We can write two electronic structures of ozone, O3.

A, .. °
:o: 6:
vo: :b'• • o:
A B • In nitrate ion, nitrogen-oxygen bond lengths are
In A, the oxygen—oxygen bond on the left is a double equal due to resonance,
bond and the oxygen—oxygen bond on the right is a single
bond. In B, the situation is just the opposite. Experiment o: :o:
shows, however, that the two bonds are identical. .. I ..
Therefore, neither structure A nor B can be correct. :Q-n-q: :q—n=o:
• One of the bonding pairs in ozone is spread over the
region of all three atoms rather than associated with a
particular oxygen—oxygen bond. This is called :o:
delocalised-bonding— a type of bonding in which a
bonding pair of electrons is spread over a number of :o=n—q:
atoms rather than localised between two.
• 1, 3-butadiene can have following structures:
® e
ch2=ch—ch=ch2 <—* CH2=CH—CH—CH2
:o
••
z o: © e
<—> CH2—CH=CH—CH2 <—»...
c
• Structures A and B are called resonating or • Vinyl chloride also has following resonating
canonical structures and C is the resonance structures:
•• e ®
hybrid of these two structures. This phenomenon is ch2=ch-ci: <—> ch2—ch=ci
called resonance: a situation in which more than
one plausible structure can be written for a and C—Cl bond is stable.
Chapter 2: Chemical Bond

• C-Cl bond in chloroben zene is stable due to


+
H-N =N=-N f--➔ H- N -N=:
- + +

..
resonance.
ci:
(I) (II)

6· ·6
9 • • EB
:c1: :c1 f--➔ H-:

e6 .. • The larger the number of the contributing s


the greater the stability of the molecule.
► • Greater numbers of the covalent bonds a
..
e stability of the canonical forms.
In BF3 , the actual bond order of B-F hon
:c1: than 1 and is found to have 54% 7t character
.. ..
resonating structures can be written as:
:F: :F:
..
:F: :F:
'V
B .....--.. �
B ...._. etc .
Vinyl chloride and chlorobenzene do not give S N
reactions due to resonance stabilisation of C-Cl bond.
I II
:F
••: :F:
• Phenoxide (C6Ht,O-) ion is stable due to resonance. (I)
$
(II)

Thus, {II) is significant due to the additi


formed.
lliustration 1 Draw two resonance stru
diazomethane (CH2N 2 ). Show formal charges.
Solution The skeletal structure ofCH2N 2 is,
• Thus phenol (which is also an oxo-acid) is more acidic H
than an alcohol because of resonance charge >C-N -N
delocalisation of the phenoxide ion. H
• Carbon-oxygen bond lengths in carboxylate ion are Two resonance structures are:
equal due to resonance, and thus stability of the H H
I � .• I �
..
carboxylate ion is increased.
0 09 0 H-C=N=N-
•• f--+ H-C - NE::::a
R-C< � R-C < f--➔ R-c(' e
'09 �o 'o Illustration 2 Draw three resonance structu
molecule N 20 in which the atoms are arranged ir
NNO. Also indicate formal charges.
Conditions for Resonance
• The canonical forms should not differ in the atomic Solution Resonance structures are shown be

.. .. ..
•• + •• + •• ••
arrangements. Only positions of the electrons can -N
•· -N =0 :N :!!!!!!N-0: - 2
t-➔ f--➔ - :N-
change. Different canonical forms are separated by
f--➔ (double headed arrow)
• The number of unpaired electrons should be the same
in all the canonical forms. Thus, following structure of Metallic Bonding
benzene is incorrect:
• • Most metals crystallise in close-packed s
The ability of metals to conduct electricity
must result from strong electronic interactic
8 to 12 nearest neighbours (which is a
coordination number). Bonding in metals
• The positive charge should reside as far as possible, on metallic bonding. It results from the
the less electronegative element and vice-versa. Thus, attractions among positively charged metm
H-F can be represented as mobile, deloca!ised electrons belonging to the
+ - - +
H-F � HF � HF a whole
correct incorrect
• Two models are considered to explain
• Like charges should not reside on adjacent atoms. For bonding:
HN 3, the structure (II) becomes unstable because of
the positive charges on the adjacent nitrogen atoms. (a) Band model
(b) Electron-sea model
64 | Essential Inorganic Chemistry

Band Model metal decreases as temperature increases. The


increase in temperature causes thermal agitation of
• The interaction of two atomic orbitals, say the
the metal ions. This impedes the flow of electrons
3s-orbitals of two sodium atoms, produces two when an electric field is applied.
molecular orbitals, one bonding orbital and one
anti-bonding orbital. If N atomic orbitals interact, N • Crystalline non-metals, such as diamond and
molecular orbitals are formed. Atoms interact more phosphorus, are insulators—they do not conduct
strongly with nearby atoms than with those farther electricity. It is due to the fact that their
away. The energy that separates bonding and highest-energy electrons occupy filled bands of
anti-bonding molecular orbitals resulting from two molecular orbitals that are separated from the lowest
given atomic orbitals decreases as the interaction empty band (conduction band) by an energy difference
(overlap) between the atomic orbitals decreases. called the band gap. In an insulator, this band gap
When we consider all the possible interactions among is an energy difference that is too large for electrons to
one mole of Na atoms, there is formation of series of jump to get to the conduction band. (Fig. 2.9).
very closely spaced molecular orbitals (formally) (3os
Metals Insulators Semiconductors
and 3o * s).This consists of a nearly continuous band
of orbitals belonging to the crystal as a whole. One
mole of Na atoms contributes one mole (6.02 x 1023) of | j Empty band
valence electrons (Fig. 2.7) thus, 6.02 x 1023 orbitals in Empty band
the band are half-filled. Small energy gap:

Half-filled
band of
w
or
0H
_■
Forbidden zone
(large energy gap)

1 ’ Filled band
N molecular
O) orbitals Partially Overlapping
c filled band
LU band
Fig. 2.9 Distinction among metals, insulators and semiconductors.
Na Na2 Na3 Na8 NaN In each case an unshaded area represents a conduction band.
Fig. 2.7 The band of orbitals resulting from interaction of the
3s-orbitals in a crystal of sodium • Elements that are semiconductors have filled bands
that are only slightly below, but do not overlap with
• The empty 3p atomic orbitals of Na atoms also empty bands. They do not conduct electricity at low
interact to form a wide band of3 x 6.02 x 1023 orbitals. temperatures, but a small increase in temperature is
The 3s and 3 p atomic orbitals are quite close in sufficient to excite some of the highest-energy
energy, so that these bands of molecular orbitals electrons into the empty conduction band.
overlap (Fig. 2.8). The two overlapping bands contain
4x 6.02 x 1023 orbitals and only 6.023 x 1023 electrons.
Because each orbital can hold two electrons, the
Electron-Sea Model
resulting combination of bands is only one-eighth full. Metals have
Energy band of • ability to conduct electricity
3N molecular • ability to conduct heat
orbitals from 3p • ease of deformation [that is, the ability to be flattened
3p orbitals
into sheets (malleability) and to be drawn into wires
} Bands overlap (ductility)]
• lustrous appearance
Half-filled band of N One over simplified model that can account for some of
molecular orbitals these properties is the electron-sea model. The metal
from 3s orbitals
is pictured (Fig. 2.10) as a network of positive ions
Fig. 2.8 Overlapping of a half-filled "3s" band with an empty
“3p" band of Na/; crystal
immersed in a “sea of electrons”. In lithium, the ions
would be Li+ and one electron per atom would be
contributed to the sea. These free electrons account for
• According to band theory, the highest-energy the characteristic metallic properties. If the ends of a
electrons of metallic crystals occupy either a partially bar of metal are connected to a source of electric
filled band or a filled band that overlaps an empty current, electrons from the external source enter the
band. A band within which (or into which) electrons bar at one end. Free electrons pass through the metal
must move to allow electrical conduction is called a and leave the other end at the same rate.
conduction band. The electrical conductivity of a
Chapter 2 : Chemical Bonding I 65
In thermal conductivity no electrons leave or enter the perhaps by hammering, the internal structure
metal, but those in the region being heated gain remains unchanged as the sea of electrons rapidly
kinetic energy and transfer this to other electrons. adjusts to the new situation (Fig. 2.11)
E9 EB EB EB EB E9 GHB EBE9E9EBE9 E9E9E9E9E9E9E9E9
E9E9E9EBE9E9E9E9 E9E9EBEBE9E9E9E9 � r

EB EB EB EB EB E9E9EBE9E9E9EBEB­ EB EB EB E9EBE9 e EB
EB EB EB E9 EB
E9E9E9EBEBE9EBE9
EBE9E9E9E9E9E9E9 !For�
Initial
>
E9E9E9E9E9E9E9E9
EBEBEBEBEBEB e EB

Fig. 2.10 Metallic Bonding in electron-sea model No new repulsive forces occurs
and, sheets and wires remain Intact

According to the electron•sea model, the case of Fig. 2.11 Effect of distortion (by hammering) on metal sheet
(assuming electron-sea model)
deformation of metals can be thought of in this way: If
one layer of metal ions is forced across another,

Target Practice 3
I. The molecular ion S 3 N3 has the cyclic structure (a) B-F and B-Cl
(b) Si-O and P-P
3. What is per cent ionic character in the molecule (M-X) if
electronegativity difference is 1.7 eV? .......... ..
4. Arrange the following in increasing values of dipole
moment (i) HF, HCI, HBr, HI and H 2 ••••••••••••
All (�N) bonds are equivalent (ii) H2O, H:zS, SO2, CO2
Give at least two equivalent resonance structures 5. Select the species with zero dipole moment and maximum
dipole moment.
I II CH 4, CH 3Cl, CH 2Cl2, CHC1 3, CCI 4
2. For each of the following bond pairs, decide which is the
more polar and indicate the negative and positive poles

s Answers

[✓ i-
3. 50%

1· I
s, ,

Iis .._. 4. H 2 < HI< HBr < HCI < HF
Ci)
CO 2 < H 2S < SO 2 < H 2O
(ii)
N 5. Zero: CC1 4, CH 4
o+ o- Maximum : CH 3 Cl
2. (a) B-F > B-Cl B-F
l+ 5 Order being: CH4 = CC1 4 < CHCI 3 < CH 2Cl2 < CH 3CI
(b) Si-O > P -P (non-polar) Si-O-

Practice Exercise 3
1. Which of the following is paramagnetic? (c) indicate whether the species is paramagnetic or
(a) o;- Cb) 0 2 diamagnetic.
(c) BN 5. Arrange the following species in order of increasing
2. N 2 has a greater dissociation energy than N;, whereas 02 stability: Li2 , Li;, Li;
has a lower dissociation energy than o;. Explain. Justify your answer using M.O. theory.
3. What are the bond•order in each of: 6. Explain:
(a) CN + (b) CN (a) [Fe(H 20)s)2+ has magnetic moment(µ) of 4.90 BM, but
( c) CN-? [Fe(CN)sl 4 - hasµ = 0.
Which of these species should have the shortest bond length? (b) [Co(CN)6 ] 3- is stable but [Co(CN)/- is unstable.
4. For each of the following species : H2; c; and F; (c) [NiC1 4 ]2- is paramagnetic but [Ni(CN),a12- is
(a) write the molecular orbita1 diagram diamagnetic.
(b) determine the bond order (d) [Co(NH 3 )sl 3+ is diamagnetic but [CoFs13 - is
paramagnetic.
66 I Essential Inorganic Chemistry

7. Clnssify weak and strong ligands out of . 18. Which of the ion Ca 2+ or Cd2+ has a stronger polarising
(a) OW (b) F- ' action on anions?
(c) r (d) NH 3 19. In going from CsF to Csl, the melting point of the crystal
<cl CH:,coo­ (0 en decreases. Explain.
<g> cw <h> er 20. Which one in each of the following pairs is expected to
(i) Hp exhibit hydrogen• bonding ?
(a) CH 3CH 2OH and CH3OCH3
8. NH 3 (aq) reacts very easily with Cu 2•; if acid is added into
(b) CH 3 NH 2 and CH 3SH
aqueous solution of NH 3, there is no reaction. Explain.
(c) CH3OH and (CH3 )3 N
9. Mntch list I (Complex ions) with list II (number of unpaired
electrons) and select the correct answers: 21. Which is expected to have the highest melting point?
PH 3, NH3, (CH 3)3 N
List I List II 22. Explain:
A 4
[CrF6 ] - 1 one (a) Maleate acid ion is more stable than fumarate acid
2 ion.
B [MnF6] 4 - two
(b) Dipole moment of o-chlorophenol is less than
C 1Fe(CN) 6 ] 4 - 3 zero
o-fluorophenol.
D [Mn(CN}6 )4 - 4 four
23. In each of the following pairs, select the species having the

-w- -w-
5 five greater resonance stabilisation :
(a) HNO3 and NO3
A B C D
(a) 4 1 2 5 (b) H Hand H o-
(b) 2 5 3 1 0 0
(c) 4 5 3 1 (c) H 2CO3 and co:-
(d) 2 1 5
3 24. Draw all possible octet structural formulae for N 3.
10. Select the species which is best described by the statement
to the right : 25. The structure of 1,3.butadiene is often written as
(a) CO2, NH 3, CO has a zero dipole moment. CH2 =CH-CH =CH2 • The distance between the central
Cb) CH 4• NH 3• HF has highest boiling point. carbon atoms is 1.46 A. Comment on the adequacy of the
assigned structure.
(c) Cl 2 , Br2 , 1 2 has the lowest boiling point.
26. Carbon submode has the formula C:p2 • C-C distances are
(d) HOI, HOBr, HOCI is the weakest acid.
11. Which of these molecules would you expect to be polar and found to be 130 pm and C-O distances are 120 pm. Write
which are to be non-polar? various Lewis structures.
Cl 2 , ICJ, BF3 , NO, SO2 , XeF4 , H 2CC1 2 , OCS. 27, How many energy levels are present in the 3s conduction
band of a single crystal of sodium weighing 26.8 mg? How
12. Calculate the dipole moment of HCJ molecule if the bond
many electrons are present in this band?
length is 1.2476 A. 28. Which of the following factors are especially important in
13. If the observed dipole moment of LiF is 6.32 D, calculate determining whether an element has metallic properties?
percentage ionic character; given Li-F bond length is (a) Atomic number
0.156 nm. (b) Atomic weight
14. Does the molecule OCS have a higher or lower dipole (c) Number of valence electrons
moment than CS 2 •
(d) Number of vacant atomic orbitals
15. Which has larger lattice energy in :
(1) Al2O3 or Al2Se3 (II) ZnO or NaCl (e) Total number of electronic shells in the atom.
16. Aqueous solution ofa mixture contains LiCI, NaCl and KCL 29. What is the energy gap in band theory? Compare its size in
How is LiCJ separated ? conductors, semiconductors and insulators.
17. Arrange the folJowing in increasing solubility in H 2O : 30, Compare the temperature dependence of electrical
conductivity ofa metal with that ofa typical metalloid.
Ca) NaF, MgO, AIN
31. What single factor accounts for the ability of metals to
(b) MgF2 , CaF2 , AgF conduct both heat and electricity in the solid state? Why are
(c) Agel, AgBr, AgI, AgF ions solids poor conductors of heat and electricity even
though they are composed of charged particles?

Answers
1. (b) 02 with one unpaired electron as decided by M.O. theory. c; : B.O. = 1.5, paramagnetic
2. N 2 lcses bonding electron hence, destablished; 02 loses F2• : 8.0. = 1.6, paramagnetic
anti-bonding electron hence, stabilised. (Refer Table 2.10, 2.11)
3. (a) 2 (b) 2.5 (c) 3 5. Li2 = Li2 < Li 2
CN- has the shortest bond-length. 6. See text
4. Hi : B.O. = 0.5, paramagentic
f

Chapter 2: Chemical Bonding | 67

7. F ", I", Cl" and H 2O are weak ligands, other are strong ligands.
8. NH3 forms deep blue coloured complex with Cu2+ and colour
change appears rapidly. Acid neutralises NH40H (aq NH3) but H-C/C~°
reaction appears bit slowly. II no H-bonding possible
9. (c) /C-H
10. (a) CO2 (b) HF (c) Cl2 (d) HOI HO—C
11. Polar : IC1, NO, SO2, CH2C12, OCS (due to electronegativity
II
0 (fumarate acid ion)
difference between the bonded atoms)
Non-polar: Cl2 (due to identical atom) Cl F- —H
BF3, XeF4 (due to symmetry) oh
12. 5.99 Debye 13. 84.4% 14. Higher dipole moment of OCS (b) (no H-bonding),
15. (I) A12O3 (II) ZnO
16. Ether extracts LiCl (covalent) from mixture. H-bonding in o-fluorophenol decrease angle (< 60°) hence, p is
17. (a) AIN < MgO < NaF (b) CaF2 < MgF2 < AgF increased.
(c) Agl < AgBr < AgCl < AgF 23. (a) NO3 (with three equivalent resonating structure)
18. Cd2+ 19. Due to decrease in ionic character. (b) H—C—0“ (with two equivalent resonating structures)
20. (a) CH3CH2OH (b) CH3NH2 (c) CH3OH
21. NH3, 0
(c) CO|-
0
24. :n: :n :: n: :n -”n :n:2‘ *.n: n »n:
H-C 25. It is due to resonanting structures that all C—C bond lengths
22. (a) stable due to intramolecular H-bonding are equal.
1
.0 26. O=C=C=C=O, 0-CsC-C=0+
Cx
II (maleate acid ion)
27. 7.015 x IO20 energy levels in the 3s conduction band.
0 7.015 x 1020 electrons in the band.
28. (c) and (a) 29. See text 30. See text 31. See text
Total Practice Set
(Read, Plan & Solve'.)

109°28'
Problem 1. The platinum-chlorine distance has been = sin v (in tetrahedral structure
2
found to be 2.32 A in several crystalline compounds. If this
value applies to both of the compounds shown in Fig., what 6 = 109°28')
is the chlorine-chlorine distance in (a) structure I and (b) = sin (54° 44')
structure II? AP= AO sin (54° 44')
NH3 NH3 = 1.54x 0.82= 1.26A
Cl Pi Cl Cl Pl NH3 AB = 2AP = 2.52A

NH3 Cl Problem 3. The dipole moment of HBr is


(I) (ID 2.60 x IO30 C.m. and the interatomic spacing is 1.41 A.
What is the per cent ionic character of HBr ?
Plan I and II are geometrical isomers and based on
Plan Per cent ionic character = °bserved value x iqq
position of C\-atoms, distance between them can be
theoretical value
determined.
Solution (a) Given (Pt—Cl) distance = 2.32 A Solution Theoretical value of dipole moment of a 100%
ionic character = e x d
Since both Cl and Pt atoms are in straight line hence in (I)
= (1.60x 10‘19 C)(1.41x 10’10 m)
Cl—Cl distance 2.32 A
Cl Pt = 2.26 x 10‘29 Cm
= 2x 2.32 A
= 4.64 A 2.32 A observed value of dipole moment = 2.60 x IO-30 Cm
.... . observed value , AA
(b) In (II), Cl—Cl distance per cent ionic character=--------------------- x 100
theoretical value
= 7(2.32)2 + (2.32)2 Cl r30
= 2.60 x 2,60 X 10
= 2.3272 x 100=11.5%
2.26 x 10“29
= 2.32 x 1.414
= 3.28 A Problem 4, A diatomic molecule has a dipole moment
of 1.2 D. If its bond distance is 1.0 A, what fraction of an
Problem 2. The C—C single-bond distance is 1.54A. electronic charge exists on each atom?
What is the distance between the terminal carbons in Plan Based on dipole moment of 1.2 D, electronic charge
propane? Assume that the four bonds of any carbon atom is calculated. This value is compared to theoretical value of
are pointed towards the corners of a regular tetrahedron. charge (= 4.8 x 10-1° esu).
Plan Structure of propane is assumed to be tetrahedral Solution For a diatomic molecule dipole moment
with bond angle of 6 = 109°28'. Thus, from the value off-1 p =exd
1.2 x 10 18 esu cm
and sin values, distance between terminal C-atoms can be :. electronic charge = — =
d 1.0 x 10"8 cm
determined.
= 1.2 x IO"10 esu
Solution Two terminal A P
B actual value of electronic charge = 4.8 x 10-1° esu
carbons can be assumed to be
1 2x 10-1°
at A and B, while the central fraction of the electronic charge = —------- = 0.25
CM CXJ
4.8 x 10
carbon at O. Then, 1.54A
AB = 2AP Problem 5. At 300 K and 1.00 atm, the density of
But
AP
---- - sin
AO 2J
-1 O gaseous HF is 3.17 g/L. Explain the observation, and
support you explanation by calculations.
Chapter 2: Chemical Bonding | 69
Plan HF molecules are associated by H-bonding. Overall Solution (a) is symmetrical hence no dipole,
molecular weight of the associated molecule is determined, (b) and (c) have dipoles.
hence number of HF molecules associated can be predicted.
Problem 8. Azobenzene, (C6H5)2N2, is an important
Solution PV = nRT = — RT
m starting material in the production of dyes.
dRT (a) Describe the hybridisation scheme for the atoms
P^ — RT =
Vm m marked with (*).
dRT 3.17x 0.0821x 300 (b) Indicate the value of the bond angles labelled
m (molecular weight) -
P 1 a and p.
= 78.08 g mol-1
But normal molecular weight of HF = 19 g mol-1. N==N
v-/a
Thus, four molecules of HF are associated in
gaseous phase by hydrogen bonding, forming Plan From the type of hybridisation, bond angles can be
tetramer of molecular weight 78.08 g mol-1. predicted.
Problem 6. Dipole moment of H20 is 1.85 D. If bond Solution (a) Each sp2 (b) a = p = 120°
angle is 105° and 0—H bond length is 0.94 A, determine
magnitude of the charge on the oxygen atom in the water Problem 9. Write Lewis structure for the following
molecule. compounds :
H c (a) BaO (b)MgCl2 (c) K2S
% Plan Based on valence-shell electron-configuration,
Lewis structure can be written.
52.5°
0 52.5°
52.5’ A B Solution (a) Ba = [Xe]6s2 : two electrons excess to
inert gas configuration; can lose two electrons.
0= Is2 2s2 2p4- : two electrons less than octet; can gain two
H
electrons
.x f-
Plan From the bond angle and vector moment p = ex I Ba
X
+ 2 02
<( •
[Baf :o:
• X

Thus, e (charge) can be determined. Lewis structure


(b) Mg= [Ne]3s2 : two electrons excess to inert gas
Solution AB = BC cos 52.5°
configuration; can lose two electrons.
= 0.94x 0.609 = 0.572 A Cl= [Ne]3s23p5 : one electron less than octet; two
= 0.572 x 10"8 cm chlorine atoms gain two electrons lost by
Also |1 = e x AB magnesium
p 1.85 x 10-18 esu cm *••
e = -r- =------------------ a
AB 0.572 xlO"8 cm Mg Cl.x [Mg2"] :ci.x
X
••
= 3.23x 10-10 esu . ci: Lewis structure

Problem 7. Draw all the geometric isomers of PBr2Cl3 XYX X
molecule. State whether each isomer has a dipole moment. (c) + s: [K]+ ss:
••
[K]+
CIBr C1C1 BrCl Lewis structure

Cl Br Cl Br Problem 10. Applying the VSEPR theory, predict the


Cl Cl
shape of methyl isocyanate CH3NCO.
Br Br Cl Plan Using valence-shell electrons and Lewis structure,
(a) (b) (0 shape of CH3NCO is predicted.

Plan Symmetrical molecules will have net dipole zero Solution Number of valence electrons
while unsymmetrical molecules will have dipoles. C H3 N C 0
! I M I
4 + 3 + 5 + 4 + 6 = 22
70 | Essential Inorganic Chemistry

We assign structure I with six bond pairs (-) and five Again note here, we have not taken n bonds into
lone pairs (..) account to decide shape of the molecule.
H
Problem 11. By the following method you can predict
(I) H—C—N—c—o: the geometrical shape of species with only one central
8 • •

atom, without first drawing the Lewis structure :


H
1. Total number of electron pairs
But this structure lacks octet of N and C, hence by
shift of electrons of lone pairs to bonding pairs, we = (number of valence electrons
write structure II in which each of atoms has ± electrons (for ionic) charge) / 2
completed octet (H with duplet). 2. Number of bond electron pairs = number of atoms - 1
H 3. Number of electron pairs around central atom
= total number of electron pairs
(ID H— C n=c=•o• :
-3 [number terminal atoms (except H)]
H 4. Number lone pair
Two bonding pairs one lone pair (trigonal planar) = (number of central electron pairs - number bond
pair)
What is the shape of the following based on this method?
:n*t«- Two bonding pairs (linear) (a) PC15 (b) NH3 (c) H2O
*|>120 (d) ClOi (e) ICI4
,C\ 1
H' | XH Four bonding pairs (tetrahedral) Plan We follows the steps gives in the question to predict
H J the shape.
Solution Solution is in the Table 2.15.

Table 2.15 Solution of Example 11


Steps
Species VSEPR notation Structure (Table 2.6)
1 2(X) 3 4(E) i
(a) PCI5 5±Ex7) = 2Q 5 20-15=5 5-5 = 0 AXS trigonal bipyramidal
2
5+3 A
(b) NH3 3 4-0=4 4-3 = 1 AX3E trigonal pyramidal
2
2+6= 4 .
-------
(c) H2O 2 4-0=4 4-2 = 2 AX2E2 angular
2
7+18+1
(d) CIO3 = 13 3 13-9=4 4-3 = 1 AX3E pyramidal
2
7+28+1
(e) ICi; = 18 4 18-12 = 6 6-4 = 2 ax4e2 square planar
2

All the structures determined by this new method is similar to that used by VSEPR theory and is widely
acceptable.
Problem 12. In certain polar solvents PC15 undergoes What are the changes in the geometrical shapes that
an ionisation reaction in which Cl“ ion leaves one PC15 occur in this ionisation?
molecule and attaches itself to another. Plan We follow the steps as given by Example 11.
2PC15 PC14+ + PCle
Chapter 2: Chemical Bonding I 71
Solution
Steps PCl5 PC14 PCl6
5 5 5+28-1 5 + 42 + 1
(1) Total number of electron pairs +3 =20 = 16 = 24
2 2 2
(2) Number of bonded electron pairs 5 4 6
(3) Number of electron pairs around central 20-15=5 16-12=4 24 - 18 = 6
atom
(4) Number of lone pairs 0 0 0
(5) VSEPR notation AXs AX4 AXs
( 6) Structure trigonal bipyramidal tetrahedral octahedral

If you manage to determine type of hybridisation bond-length = 1.40 A radius of iodine atom = 1.33 A,
of P atom in each, then structure can be decided, radius of carbon atom = 0. 77 A)
but this will also require whether lone pair is
(a) ©( (b) [91I
present or not.
PCl r; s/d trigonal bipyramidal
PCI: tetrahedral
PC18 octahedral
Plan In o- m- and p-isomers, bond angles are respectively
60 °, 120° and 180° hence, distance between two-atoms can
Problem 13. The HF; ion exists in the solid state and be calculated.
also in liquid HF solution, but not in dilute aqueous
solution. Explain. Solution (a) I-atoms are at C and D positions. CD=?
Solution In the solid state and in liquid HF, the HF2- ,,C

0;:�:- i
," I

ion is held together by hydrogen bonding. In aqueous


solutions, there is hydrogen bonding, but each HF
molecule being stronger acid than H20, forms H30+ and F-. �B"...... :
H2F2 � HF2- + H
+
(in liquid HF) ",
'---..!
I

n
"', I

HF+ H2 0 � H30 + + F- (in solution)

Problem 14. Copper (II) iodide is unstable even at Also, /J. OCD is equilateal triangle.
ordinary temperature. Explain. CD=OC=OA+AC
Plan Explanation can be made in terms ofpolarisation of = 1.40 + 1.33 + 0.77
cation by anion and vice-versa as predicted by Fajan's rule. .. a.so A
Solution The Cu2 + ion with a 17-electron outer-shell (AC is the sum of radii of carbon and iodine and OA = AB)
and a comparatively small radius (0.08 nm) has a strong (b) CD=2DE

1
polarising power, while the iodide ion with a larger radius

c
(r = 0.22 nm) has a high polarisability. Hence, the _A
polarisation of the r by the Cu2 + leads to the complete
transition of an electron from the anion to the cation : the 0
Cu2 + ion is thus reduced to Cu+ ion and r is oxidised to free B 6oy'E
iodine. The compound C% therefore, does not exist.
2Cu� ---+ 2Cul + �
21- ---+ � + 2e- D
2
Cu + + e- ---+ Cu But DE . 600
-=sm
OD
Problem 15. Assuming the additivity of covalent radii DE=OD sin 60°
in the C-1 bond, what would be the iodine-iodine distance
in each of the diiodibenzene? Assume that the ring is
=(OB+BD) sin 60°
regular hexagon and that each C-I bond lies on a line = (1.40 + 0.77 + 1.33) sin 60°
through the centre of the hexagon. (Take C-C
72 | Essential Inorganic Chemistry

= 3.50 x —= 3.03 A C = 135 - 7104 x 38


2 = 135-62.86
CD = 6.06 A
= 72.14 kcal mol
(BD = AC as in (a))
(0 OC = OD = 3.50 A Problem 18. Calculate the steric numbers for iodine
CD = 2x3.50 = 7.0 A D in (IF4_) and for bromine in (BrO4).
Plan SN = (Bonding atom + Lone pairs)
Problem 16. Calculate the electronegativity of
chlorine from the bond energy of Cl—F bond (61 kcal mol-1),
F—F bond (38 kcal mol-1) and Cl—Cl bond (58 kcal mol-1), Solution The central I has eight valence electrons
and electronegativity of fluorine 4.0 eV. ’I* x (gained electrons)
Plan Based on Pauling’s scale :
(EN)f - (EN)C1 = ky[E = 0.208VA Each F atom has seven valence electrons of its own and
needs to one of the electrons from I" to attain noble-gas
where A is resonance energy and k is the conversion factor
configuration (s2p6). Thus, four, of the 1“ valence electrons
(which is 0.208 for converting kcal into eV).
take part in covalent bonds, leaving the remaining four to
Solution A = (BE)C1_ F — 7(BE)q]_-ci (BE)f_ F form two lone pairs. Thus,
= 61 - 758 x 38
= 61-46.95 = 14.05 kcal :f:
(EN)C1=(EN)F- 0.208VA :f—i-2-f:
= 4.0-0.208714.05
= 4.0 - 0.78 = 3.22 eV :f:
Problem 17. Bond energy of H—H, F—F and H—F
bonds are 104, 38 and 135 kcal mol-1 respectively. SN = 4 (bonded atoms) + 2 (lone pairs) = 6
Calculate resonance energy in the H—F molecule. The central Br~ has also eight electrons. In BrOi, each
oxygen atom needs to share two of electrons from Br" to
Plan As in Problem 16. attain noble-gas configurations thus all the electrons are
Solution Resonance energy used in bonding and no lone pair exists.
Ah_f = (BE)h_f - 7(BH)H2 (BE)F2 Thus, SN = 4 (bonded pair) + 0 (lone pair) = 4.
. X__

Master Exercises
Exercise 1
(Stage 1: Learning)
Short Answer Type Questions Analytical Questions
1. Predict whether the molecules NH3 and SF6 will have Ionic, Covalent and Coordinate Bonding
dipole moments. 1. Indicate what is wrong with each of the following
2. Acrolein is the starting material for certain plastics Lewis structures? Replace each with a more
H H H acceptable structure.

H-C=C-C=O: (a) [:s‘-c=N:r (b) [:cir [:o:f“ [ci:r


(a) Which bonds in the molecule are polar and which (C) :o=n=o: (d) :ci-n=ci:
are non-polar?
(b) Which is the most polar bond in the molecule? :ci:
Which is the most negative atom of this bond?
3. Consider the nitrogen—oxygen ond lengths isNQj, 2. Assign formal charges to the following species. If
NO2 and NO3. In which ion is the bond predicted to be there are no formal charges present for certains of
longest? In which is it predicted to the shortest? these species, so indicate
4. Explain why the species I in each set has larger bond :o:
angle than II?
I II (a) :o—ci-qi: (b) :o
/sv0:
(a) CH, NH3
(b) OC12 of2
:f:
(c) NH3 nf3
(d) NH3 ph3 (c) :f—b -g: (d) [:n=n=n:]“
5. Explain why the valence bond structure of carbon
monoxide is best described as
:f:
:c=o (e) [:n—n=n:]~ (0 [:o=n=o:]+
with a dative bond from oxygen atom to the less
electronegative carbon atom. 3. In which of the following molecules would you expect
6. The bond dissociation energy of C2 (599 kJ mol-1) the nitrogen-to-nitrogen bond to be the shortest?
decreases slightly on forming C£ (513 kJ mol-1) and (a) N2H4 (b) N2
increases greatly on forming C£ (818 kJ mol-1). Why (c) N2O4 (d) N2O
is the change much greater for the addition of an 4. The common forms of N and P are N2 and P4
electron? respectively.
74 | Essentia/ Inorganic Chemistry

model. Describe the bonding, using valence bond


theory.
:N; n: p:
16. I. Which one of the following molecules is planar?
(a) NF3 (b) NC13
(c) PH3 (d) BF3
Using the following data, explain why the stable II. What is the type and number of bonds between
forms areN2, notN4; and P4 notP2? the two carbon atoms in CaC^?
Mean bond enthalpy, (kJ mol-1). 17. Each of the following compounds has a
N=N 946 P=P 481 nitrogen-nitrogen bond
N—N 160 P—P 215
N2, N2H4, N2F2. Match each compound with one of the
5. A sample of lead (II) bromide, PbBr2, was placed in a following bond lengths
crucible. The ends of two carbon electrodes were put 1.10 A, 1.22 A, 1.45 A. Describe the geometry about
in the solid and connected to a lamp and power pack. one of the N atoms in each compound. Why hybrid
At first the lamp did not light, but when the bromide orbitals are needed to describe the bonding in valence
was heated and began to melt the lamp began to
bond theory?
glow. Explain these observations.
18. “El” is the general symbol for a representative
6. What are two types of bonds present in B2H6? element. In each case, name the probable element El.
7. The experimentally determined N—F bond length in (a) [0—El—0 r (b) H—0—EI=O
NF3 is greater than the sum of the single bond
covalent bond radii of N and 0. Explain.
I
0
8. The correct order of increasing C—0 bond length of r o i2-
CO.CO^CCh is
(a) C0|" < CQa < CO i(b) CO2<C0|'<CO (0 O—El—0 (d) H—0—El—H
(c) C0< COl" < co2 (d) CO<CO2<CO23- I
0 H
9. In the dichromate dianion
(a) 4 Cr—0 bonds are equivalent 19. A monomeric anhydrous complex of Co gives the
(b) 6 Cr—0 bonds are equivalent following results:
(c) all Cr—0 bonds are equivalent Co : 21.24% SO?" : 34.65%
(d) all Cr—0 bonds are non-equivalent NH3 : 24.77% H2O : 6.53%
cr : 12.81%
Hybridization/Shape of Molecules (Co : 58 , Cl= 35.5,8= 32, N= 14); Z (Co = 27)
10. Two of the following species have the same shape. The compound is diamagnetic and gives white ppt
Which are these two ? What is their shape ? What are with BaCl2. What is its empirical formula ? What is
the shapes of the other two ? its structure ?
NI3,15, SO2,-, N05 [Hint : Determine molar ratio. Since complex is
11. Write plausible Lewis structures for the following diamagnetic, hence, cobalt has no unpaired electron.
molecules SO?' is outside coordinate sphere since it gives white
(a) H2N0H (b) N2F2 ppt withBaCl2.]
(c) HONO (d) H2NNO2
20. N(CH3)3 is pyramidal but N(SiH3)3 is planar.
12. PF5 and BrF6 appear similar formulae, but
Explain.
structures are different, explain.
13. C1F2" ion is linear but C1F2 is bent. Describe 21. Fill in the blanks
(a) The shape of CH3 is...........
hybridisation schemes for the central Cl atom to
account for this structure difference. (b) The valence atomic orbitals on C in silver
acetylide is..........hybridised.
14. The hyponitrite ion, "0—N=N—O', exists in solid
22. Using the VSEPR theory, identify the type of
compounds as the trans isomer. Using VB theory,
explain why cis - trans isomers might be expected for hybridisation and draw the structure of 0F2. What
this ion? are the oxidation states of 0 and F?
15. A molecular compound is composed of 60.4% Xe, 23. Among the following species identify the
21.1% O and 17.5% F, by mass. If the molecular isostructural pairs
weight is 217.3 u, what is the molecular formula? nf3, no3, bf3, h3o+, nh3
Predict the molecular geometry using the VSEPR
Chapter 2 : Chemical Bonding | 75

24. I. Select correct alternate 37. Explain how the dipole moment could be used to
Among KO2 , AIO^, BaCXj andNO^, unpaired distinguish between the cis and trans isomers of
electron is present in : 1,2-dibromoethene?
(a) NO-2 and BaO2 (b) KQ, and AIO^ Brx /Br H Br
(c) KQj only (d) BaC^ only C=C C=C
II. Among N2O, SO2,13 and I3, the linear species are H H Br H
........and ......... cis trans
III. The angle P—P—P in P4 molecule is ........
38. The length of the dipole of HF molecule is 4 x 10"11 m.
25. What is the geometry and the type of hybrid orbital
present about the central atom in BF3? Calculate the dipole moment in coulomb-metres and
in debyes.
26. Interpret the non-linear shape of H2S molecule and
39. Two substances A and B having the same molecular
non-linear shape of PC13 using VSEPR theory. formula, C4H8O , were examined in the gaseous state
27. Discuss the hybridisation of carbon atoms in allene by electron diffraction. The carbon-oxygen distance
(CjHj) and show that n-orbital overlap. was found to be 1.43 A in the compound A and 1.24 A
Select the correct alternate (Q. 28 to 29) in compound B. What can you conclude about the
structure of these two compounds ?
28. The geometry of Ni(CO)4 andNi(PH3)2Cl2 are
40. TheN2F2 molecule can exist in either of the following
(a) both square planar
(b) tetrahedral and square planar two forms:
(c) both tetrahedral F-
N=N' N=N
(d) square planar and tetrahedral
29. In the compound CH2=CH-CH2-CH2-C=CH (I) (II)
the C2—C3 bond is of the type (a) What is the hybridisation of N in the molecule?
(b) Which structure has a dipole moment?
(a) sp-sp2 (b) sp3-sp'>3
(c) sp-sp3 (d) sp2-sp .3 41. The dipole moment of HCN molecule is 2.9 D.
Calculate the length of the dipole.
30. Deduce the structure of [NiCl4f“ and [Ni(CN)4f-
considering the hybridisation of the metal ion. 42. The dipole moment of the NH3 molecule is 1.48 D.
Calculate the magnetic moment (spin only) of the Calculate the length of the dipole. Can the molecule
species. be assumed to have the shape of a regular triangle.
43.1. Arrange (A) toluene, (B) m-dichlorobenzene,
MO Theory (C) o-dichlorobenzene and (D) p-dichlorobenzene
31. Suggest reasons why the following do not exist as in order of increasing dipole moment.
stable molecules? II. Select the correct alternate :
(a) H3 (b) HHe The geometry of H2S and its dipole moment are
(c) He2 (d) H3O (a) angular and non-zero (b) angular and zero
(c) linear and non-zero (d) linear and zero
32. Write MO diagram of (a) CN+ and (b) BN.
33. What is the molecular orbital configuration of HeH+? Fajan’s Rule/Lattice Energy
Do you expect the ion to be stable.
44. Compare the stability (in the solid state) of the
34. (a) When N2 goes to N2, the N—N bond distance ... following pairs of compounds:
and when O2 goes to O£, 0—0 bond distance ... (a) LiF and LiF2 (b) Cs2O and CsO
(b) Write MO electron distribution of O^ Specific its (c) CaBr2 andCaBr3
bond order and magnetic property. 45. Assume the following bonds in order of increasing
Dipole Moment ionic character:
C to H, F to H, Br to H, Na to I, K to F, Li to Cl
35. Estimate the per cent ionic character of the HBr
molecule, given that the dipole moment (p) is 0.79 D 46. Explain.
and HBr bond length 151 pm. (a) AuC13 is less stable than AuCl and PbCLj is less
stable than PbCl2.
36. (a) Draw Lewis structure for nitrosyl fluoride FNO. (b) K2CO3 melts at 890°C without decomposing while
(b) What is the geometrical shape ? Ag2CO3 decomposes at 220°C.
(c) Explain why the measured resultant dipole (c) BaCl2 ionises in aqueous solution while HgCl2
moment for FNO, 1.81 D, is so much larger than does not (or very smaller).
the value for nitryl fluoride, NO2F (0.47 D)?
76 | Essential Inorganic Chemistry

(d) The ionic radii of Na+ and Cu+ are the same 53. The central C—C bond in 1,3-butadiene is shorter
(0.098 nm) but melting point of NaCl (801°C) is than that of n-butane. Explain.
much higher than that of CuCl (430°C).
54. Select the correct alternate :
47. Anhydrous A1C13 is covalent. From the data given The most unlikely representation of resonance
below, predict whether it would remain covalent or structure ofp-nitrophenoxide is:
become ionic in aqueous solution. "0. ^0 ■0“
“0
Ionisation energy for Al = 5137 kJ mol-1
^hydration <A13+) =- 4665 kJ moP1
AHhydration (CP)= - 381 kJ moP1 (a) (b)

Hydrogen Bonding/Metallic Bonding/General


O' 0
48.1. There is no S—S bond in :
(a) S2OT (b) S20f- '0.
"N
(c) SjO2,' (d) S20?-
II. Out of Ni(CO)., Ni(CN)J" and NiCl2" select
paramagnetic and diamagnetic. (c) (d)
III. What is the type of hybridisation of Cj—C3 single
bond in HC=C—CH=CH2 ?
IV. OutofSnCl2, NCO", CS2, N0£ andSC^ select the O' 0
one with linear structure.
V. What is increasing strength of hydrogen bonding NCERT Problems
(X.... H—X) (X = 0, S, F, Cl, N)?
1. Two p-orbitals from one atom and two p orbitals from
49.1. What is type of hybrid orbitals used by the another atom are combined to form molecular
chloride in CIO2 ? orbitals. How many MOs will result from this
II. The cyanide ion, CN" and N2 are isoelectronic. combination? Explain.
But in contrast to CN",N2 is chemically inert
because of......... 2. Show the shapes of bonding and anti-bonding MOs
III. The maximum possible number of hydrogen formed by combination of (a) two s orbitals (b) two p
bonds a water molecule can form is.......... orbitals (side to side).
TV. Which of the following have identical bond order 3. How do the bonding and anti-bonding MOs formed
CN", OJ, N0+, CN+ ? from a given pair of AOs compare to each other with
V. Which has highest dipole moment out of respect to (a) energy (b) presence of nodes (c)
(a) 2, 2-dimethyl propane intemuclear electron density?
(b) irans-2-pentene
4. Arrange the following species in order of increasing
(c) cjs-3-hexene
stability : Li2, Li2, Li2. Justify your choice with a
(d) 2,2,3,3-tetramethyl butane?
molecular orbital energy level diagram.
50. (I) The structure of the enolic form of
CH3—CO—CH2 —CO—CH3 with 5. Use molecular orbital theory to explain why the Be2
intra-molecular H-bonding is......... molecule does not exist?
(II) Pick out the isoelectronic structure from the 6. Explain why the bond order of N2 is greater than N2,
following: but the bond order of 02 is less than that of 02 ?
CH3, h3o+, nh3, ch; 7. Compare the relative stability of the following species
51. Explain why o-hydroxybenzaldehyde is a liquid at and indicate their magnetic properties (diamagnetic
room temperature while p-hydroxybenzaldehyde is or paramagnetic); 02 , 02, 02 (superoxide), Oj'
high melting solid? (peroxide ion).
Resonance 8. Explain the significance of bond order. Can bond
52. NO2 gas is paramagnetic at room temperature. On order be used for quantitative comparisons of the
cooling below 0°C, its molecular weight increases and strength of chemical bonds.
it loses its paramagnetism. When it is reheated, the 9. What is the energy gap in band theory? Compare its
behaviour is reversed. Explain. How does this size in conductors, semiconductors and insulators.
phenomenon differ from resonance?
Chapter 2 : Chemical Bonding I 77
10. Which of the following substances will exhibit 11. How can one non-polar induce a dipole in a nearby
H-bonding? Draw the H bonds between two non-polar molecule?
molecules of the substance where appropriate. 12. What type(s) of intermolecular forces exist between
0 the following pairs? (a) HBr and H2S, (b) Cl2 and CBC&,
I (c) � and NO3, and (d) NH 3 and C6H 6 ?
(a) CH 3CH2OH (b) CH 3 -C-OH [Hint : To identify intermolecular forces, it is useful
0 0 to classify the species being considered as
II I ( 1) non-polar molecules, (2) polar molecules, and
(3) ions. Keep in mind that dispersion forces exist
(c) CH 3 -C-CHa (d) CH3 -C-NH2
between all species.]

Exercise 2
(Stage 2 : High Skill Problem Solving)
Only One Option Correct 10. Delocalised molecular orbitals are found in
1. In which of the following molecules would you expect (a) H2 (b) Hs-
the nitrogen-to-nitrogen bond to be the shortest? (c) Cl!i (d) c�-
(a) N2� (b) N2 11. A substance in which large energy gap separates the
(c) N 2O4 (d) N2 O valence and conduction band is a
(a) metal (b) metalloid
2. One of the following ions has a trigonal planar shape (c) semi conductor (d) insulator
(a) s�- Cb) Pot
12. Which of the following has the highest percentage of
(c) CN- (d) c�- ionic character in its bonding?
3. Which of the following molecules or ions is not linear? (a) Lil (b) MgC12
(a) BeC12 (b) ICI2 (c) CsF (d) Csl
(c) CS2 (d) ICl; 13. In which of the following sets do all the three
4. The formal charge of the O atoms in the ion compounds have bonds that are mainly ionic?
•• •• (a) NaCl, NC1 3 , CC14 (b) CsBr, BaBr2 , SrO
[: 0 N =O: ]'" is
(c) CsF, BF3, NH3 (d) Al2O3, CaO, S�
(a) -2 (b) -1
(c) 0 (d) +1 14. Which of the following salts is not soluble in water?
(a) K 2CO3 (b) Rh2SO4
5. All the following molecules are linear except one (c) BaCO3 (d) NalO3
(a) S� (b) C�
15. Which of the following is an electron-deficient
(c) HCN (d) NO
compound?
6. All the following molecules are polar except one (a) NaB�
(a) BC1 3 (b) CH2Cl2 (c) BiH 6
(c) NH3 (d) FNO
16. Which of the following is non-polar but contains polar
7. Correct Lewis structure is bonds?
•• ••
(a) [:O-C
•• N:r (b) [C==C:f- (a) HCI
•• •• •• •• (c) NH3
(c) (:CI-O:r
•• •• (d) :N=O: 17. Following consists of discrete polar molecules at room
8. In carbon-hydrogen-oxygen compounds temperatures
(a) all O-to-H bonds are 1t bonds (a) CS2
(b) and C-to-H bonds are CJ bonds (c) HCN
(c) all C-to-C bonds are CJ and 1t bonds 18. One of the following is least volatile
(d) all C-to-C bonds are 1t bonds (a) HCN (b) H-F
9. The hybridisation scheme for the central atom (c) H2O (d) CS 2
includes a d-orbital contribution in 19. Which of the following does not obey 18-electron rule?
(a) Ia (b) PCI3 (a) Cr(CO) 6 (b) Fe(CO)5
(c) NOa (d) H2 Se (c) V(CO) 6 (d) Mn 2 (CO)10
78 I Essential Inorganic Chemistry

20. Which of the following will have the molecular shape 82. The molecules that will have dipole moments are
of a trigonal bipyram.id? (a) 1,2-dichloro benzene (b) cis-3-hexene
(a) PF:Pl 2 (b) IF5 (c) trans-2-pentene (d) all of these
(c) BrFs (d) Sb!if- 33. Which of the following have identical bond order?
21. Which of the following diatomic molecules would be (I) CN- (II) Qi
stabilised by the removal of an electron? (III) No +
(N} cN
+

(a) Ci (b) CN (a) I, III (b) II, IV


(c) N2 (d) 0i (c) I, II, III (d) I, IV
22. The expected spin-only magnetic moments (BM) for 34. Ci -Ca sigma single bond in vinyl acetylene
[Fe(CN)s ]'- and [FeF6 ]3- respectively are CH2 =CH-C•CH is due to overlapping of
(a) 1. 73 and 1. 73 (b) 1. 73 and 5.92 (a) sp-sp (b) s/-sl
(c) 0.0 and 1.73 (d) 0.00 and 5.92 (c) sp-s,J (d) s/-sp
23. Which ofthe following species/molecules has a planar 35. C-H bond distance is the longest in
geometry? (a} CzH 2 (b) CilI..
(a) Ni(CO)4 (b) SF4 (c) CiH (d) CiH2Br2
. s
(c) Coe�- (d) XeF4
36. Among the following species, identify the
24. Oi molecule is paramagnetic due to presence of isostructural pairs
(a) two unpaired electrons in anti-bonding MO NFa, NO3, BF3, H 3O+ , HN 3
(b) one unpaired electron in anti-bonding MO
(c) two unpaired electrons in bonding MO (a) [NFa, NOi] and [BF3, H 3O+]
(d) one unpaired electron in bonding MO (b} [NF3, HNal and [NO3, BFsl
(c) CNFa, HaO+] and [NO3, BFal
25. Resonance is not observed in (d) [NFa , HaO+] and [HN3, BF3]
(a) CO.z (b) NH 3
(c) SOa (d) NOi 37. Mg2Ca reacts with water forming propyne. ds- has
26. Which of the following can provide an electron pair (a) two sigma and two pi bonds
for the formation of a coordinate covalent bond? (b) three sigma and one pi bonds
�)W �) H- (c) two sigma and one pi bonds
(c) H2 (d) He2 + (d} two sigma and three pi bonds
27. Allyl cyanide has 38. Which of the following statements is correct for
(a) 9 sigma bonds and 4 pi bonds CsBr3?
(b) 9 sigma bonds, 3 pi bonds and I lone pair (a) It is covalent compound
(c) 8 sigma bonds and 5 pi bonds �) It contains Cs 3+ and Br­
(d) 8 sigma bonds, 3 pi bonds (c) It contains Cs + and Br3
(d) It contains cs+ , Br- and lattice Bt:z molecule
28. (C-Cl) bond in CH2 =CH- Cl (vinyl chloride) is
39. Among KO2 , AIO,i, Ba� and NO;, unpaired electrom
stabilised in the same way as in
(a) benzyl chloride (b) benzoyl chloride is present in
(c) chlorobenzene (d) allyl chloride (a) KOi only ili) N� and BaO.z
(c) KO,.z and AIOa (d) BaO.z only
29. Arrange the following compounds in increasing
dipole moment : toluene (I), m-chlorobenzene (II), 40. In which case there is change in hybridisation whem
o-dichlorobenzene (Ill), p-dichlorobenzene (IV) (a) NH 3 is protonated
(a) l< IV< II< III (b) IV< I< II< III �) AIH 3 combines with H­
(c) IV< I< III< II (d) IV< II< I< III (c) in both cases
(d) in none of the above case
30. When N2 is formed from N2 bond-order .......... and
when o; is formed from Oi bond-order ........ : 41. Molecular shapes ofSF4, CF4 and XeF4 are
(a) increases (a) the same, with 2, 0 and 1 lone pair of electrons
(b) decreases respectively
(c) increases, decreases ili) the same, with 1, 1 and 1 lone pair of electrons
(d) decreases,increases respectively
31. Which of the following is most polar bond ? (c) different with 0, 1 and 2 lone pair of electron:s
respectively
(a) CI-Cl (b) N-F
(d) different with 1, 0 and 2 lone pair of electrons
(c) C-F (d) O-F
respectively
Chapter 2: Chemical Bonding I 79
42. The hybridisation of atomic orbitals of N inNQ!, N03 52. Which has maximum dipole moment?
and NH; are respectively Cl Cl
c1kc1
3 3
(a) sp, sp-, sp (b) sp, sp , sl

lQJ
Cl
(c) s#, sp, sp 3 (d) sp-, sp3, sp
(a) (bl
43. Amongst H20, H2S, H2Se and H2 Te, the one with the
highest boiling point is
(a) H2 0 because of hydrogen bonding Cl
(b) H2 Te because of higher molecular weight Cl
(c) H2S because of hydrogen bonding
(d) H2Se because of lower molecular weight CIWCI
(c) (d)
44. Which is paramagnetic and coloured?

(a) K2Cr2 01 (b) (Nff..MTiCl6 ]
(c) VOS04 (d) K3[Cu (CN)4 ] Cl Cl
45. MgS04 is soluble while BaS04 is insoluble in H2 0.
53. Bond order of 02 , 02, Qi and�- is in order
This is because
(a) lattice energy of BaS04 is greater thanMgS04 (a) Di' <Oi- <Oi <C>;
(b) BaS04 is more covalent thanMgS04 (b) Di- < Di' < � < o;
(c) o; < � < Oi < �
(c) hydration energy ofMg2 + is greater than Ba2 + +

(d) lattice energy ofMgS04 is greater than BaS04


(d) � < 02 < 02 < �-
46. The strength of bonds by overlapping of atomic
orbitals is in order 54. Central o- atom in 03 is ...... hybridised.
(b) s#

&
(a) s-s > s-p > p-p · (b) s-s < p-p < s-p (a) sp '
3
(c) s-p < s-s < p-p (d) p- p < s-s < s-p (c) sp (d) dsp2

47. Correct order of dipole moment is 55. Which of the following species contain at least one
OH CHa atom that violates the octet rule?
(a) 0-Cl-O (b) F-Xe-F

lQJ lQJ
�N02 Cl �CHa

O
(c) Both of these (d) None of these
(I) (Ill (III) 56. ZnC0 3 is thermally more stable than MgC03,
because
(a) I= II!! III (b) I< II< III (a) Mg(OH)i exhibits only basic properties while
(c) l>II> III (d) II< III< I Zn(OH)2 is amphoteric
(b) polarising action of Zn2 + with 18-electron
48. A : tetracyanomethane B : carbon dioxide configuration on the anion is larger than that of
C : benzene D : 1,3-buta-di-ene Mg 2 + with a noble-gas electron configuru t,ion and
Ratio of a and 1t bonds is in order of same size and charge.
(a) A= B < C < D (c) both (a) and (b) are correct
(b) A= B < D < C (d) none of the above is correct
(c) A= B = C = D 57. Polarising action of Cd2 + on anions is stronger than
(d) C < D <A< B that of Ca2 + because
49. Which of the following molecules has the shortest (a) the charges of the ions are same
nitrogen-to-carbon bond? (b) their radii are same (Ca2 + = 0.104 nm;
(a) CiH.i Cb) CiHs Cd2 + = 0.099 nm)
2
(c) �H2 (d) CiCle (c) the Ca + ion has a noble-gas electron
configuration, and the Cd 2 + ion, an 18-electron
50. Which of the following species are isoelectronic?
configuration of its outer shell
(a) NHt, C� (b) N2 , CO (d) all of the above are correct
(c) C6H 6 , B�aff6 (d) All of these
58. Which of the following is not stable?
51. Which of the following molecules does not have a (a) H2 (b) H2
dipole moment? (d) HHe
(c) He2
(a) IBr (b) CHC1 3
(c) CH2Cl2 (d) BF3 59. Which of the following has the smallest bond length?
+
(a) NO (b) NO
(c) No- (d) Equal
80 | Essential Inorganic Chemistry

60. Some of the following properties are important in 66. Metallic bonding is explained by
determining whether an element has metallic (a) band model (b) electron-sea model
properties (c) both (a) and (b) (d) None of these
I: atomic number 67. Energy gaps between the highest filled band and the
II: atomic weight lowest empty bands in elements A, B, C and D are 0.,
III: number of valence electrons 70, 530 and 110 kJ mol'1. Thus
IV : number of vacant atomic orbitals (a) element A is metal
V : total number of electronic shells in the atom (b) element C is insulator
Select correct properties from the codes given below (c) element B and D can be semiconductors
(a) I, II, III, IV (b) I, III, IV, V (d) all are correct conclusion about elements A, B, C
(c) III, IV (d) III, IV, V and D
61. Azide ion (N3) exhibits on N—N bond order of 2 and 68. Select correct statements.
may be represented by resonance structures I, II and (a) The delocalised electrons in the %-orbitals of
III given below graphite cause this allotropic form to conduct
electricity
:n=n=n: <—> 2-:n—n=n: <—> :n=n—n:2- (b) Pauling assigns “metallic valences” equal to their
i ii III group number to the elements of IA to VIA
Select correct statements. (c) Diatomic lithium molecules Li2 are stable
(a) Structures I and II make greater contributions towards dissociation
than III (d) All the above are correct statements
(b) Structures II and III make greater contributions
than I 69. Solutions of alkali metals in liquid ammonia at -33°C
(c) Structures I and III make greater contributions conduct electricity without chemical reaction at the
than II electrodes. Hence, conductivity should
(d) All the above three structures make equal (a) increase as temperature is lowered
contributions (b) decrease as temperature is lowered
(c) remain constant on changing the temperature
62. Which of the following molecules are expected to (d) there is always interaction of alkali metal with
exhibit intermolecular H-bonding? liquid ammonia
I: Acetic acid II: o-nitrophenol
III: m-nitrophenol IV: o-boric acid 70. Which represents metallic character in an element?
Select correct alternate.

r
(a) I, n, III (b) I, II, IV

i
(c) I, III, IV (d) II, III, IV (a) (b)
63. In terms of polar character, which one of the
following order is correct?
(a) NH3 < H2O < HF < H2S
(b) H2S < NH3 < H2O < HF
(c) H2O < NH3 < H2S < HF
(d) HF < H2O < NH3 < H2S (c) ■ (d) None of these
64. How many o and %-bonds are there in the molecule of
tetracyano-ethylene?
(a) 4a, 14% (b) 5a, 13% 71. Isoelectronic species are
(c) 8a, 10% (d) 9a, 9% (a) CO,CN',NOX,N2,BF
65. Match List I with List II and select the correct
answer
(b) -ch3,-nh2,-6h,-f:
List I (species) List II (O—N—O angle)
A. NOj 1. 180°
(c) >ch2,>nh,>o:
B. NO2 2. 132° (d) all of the above
C. NOJ 3. 120° 72. Select, isoelectronic species out of
D. NO3 4. 115°
5. 109°
co.^ch, -^n:=ch2,=o,^o®,=nh>nh
A B C D A B C D (a) -^CH -^NI ^>6® (b) =CH2,=NH,=O
(a) 5 4 3 2 (b) 5 2 4 3
(0 1 2 4 3 (d) 1 4 3 2 (c) both (a) and (b) (d) None of these
Chapter 2 : Chemical Bonding | 81

73. Some, group in species in Column I are isoelectronic 82. Which combination will give the strongest ionic
with that in II bond?
(a) K+andCl" (b) K/andO2"
Column I Column II
(c) Ca2+ and Cl" (d) Ca2+ and O’2"
hono21 oco, co (OH)2CO.F2CO,ONN. bf, 83. Ionisation energies (IE) and electron affinities (EA) of
(CH3)2CO F and Cl are given below:
Select isoelectronic groups in Column I to Column II Element IE EA
Column I Column II F 17.4eV 3.45 eV
(a) (H2N)2CO (HO)2CO,F2CO,(CH3)2CO Cl 13.0eV 3.61 eV

(b) HONO2 CHgNO2 Compound formed between F and Cl is


(c) CO BF predominantly
(d) all of the above (a) F+C1" (b) Cl+F"
(d) C1</FF
74. Decreasing bond-order are expected to be (c) F-Cl
(a) CIO; > IO4" (b) CIO3 > BrO3 > IO3
(c) both (a) & (b) (d) None of these 84. Elements A, B and C have respectively 1,3 and
5 electrons in valence shell. Ionic compound is not •
75. Out of NO, NO+, O2, NO", O2 select species with same formed from
bond-order and paramagnetic. (a) A and C (b) A and B
(c) B and B (d) B and C
Electron Bond-order
85. Mg^O2" is formed since
(a) NO 15 2.5 Paramagnetic
(a) IE of Mg is low (b) EA of 0 is low
O2+ 15 2.5 (c) IE of Mg is high (d) IE of 0 is low
(b) O2 16 2 Paramagnetic 86. Solubility of KC1 is maximum in
NO" 16 2 (a) CH3CH2OH (b) CH3OCH3
(c) H2O (d) CH3COOH
(c) Both (a) and (b)
87. Which pair is not correct order of lattice energy?
(d) None of the above
(a) KC1 > MgO (b) AIN > MgO
76. Formation of anion from a neutral atom Xis favoured (c) BeCO3 > MgCO3 (d) BeCO3 = MgCO3
by 88. Solubility of NaCl, Na2SO4 and Na3PO4 in water in
(a) high electron affinity increasing order is
(b) large size ofX (a) NaCl < Na2SO4 < Na3PO4
(c) low ionisation potential
(b) NagPO4 < Na2SO4 < NaCl
(d) high charge on anion X
(c) NaCl < NaaPO4 < Na2SO4
77. When a chemical bond is formed, there is decrease in (d) Na2SO4 < NaCl < NaaPO4
(a) kinetic energy (b) potential energy
(c) repulsive force (d) attractive force 89. Select correct statement.
(a) LiF and MgO are isostructural and also
78. Which of the following bonds is the weakest? isodimensional, but a crystal of MgO is much
(a) Coordinate bond (b) Hydrogen bond harder than one of LiF
(c) van der Waals’forces (d) Covalent bond (b) The thermal stability of the isomorphous
79. An ionic compound A+B~ is most likely to be formed sulphates of Ca2+, Sr2+ and Ba2+ w.r.t.
from A and B when decomposition into metal oxide and sulphur
(a) ionisation energy of A is low trioxide increases in the order CaSO4, SrSO4,
(b) electron affinity of B is low BaSO4
(c) electronegativity of B is low (c) Both (a) & (b)
(d) ionisation energy of B is low (d) None of the above
90. Select correct statement.
80. In I3, Lewis base is (a) Both lattice energy and hydration energies
(a) I, (b) I" decrease with ionic size
(c) I2+ (d) £ (b) Lattice energy can be calculated using Born-Haber
81. Which set contains no ionic species? cycle
(c) If the anion is large compared to the cation, the
(a) NH4C1,OF2,H2S (b) COa.CCU.Cla lattice energy will remain almost constant within
(c) BF3,A1F3,T1F3 (d) la.CaO.CHaCl a particular group
(d) All the above are correct statements
82 f Essential Inorganic Chemistry

91. Which of the following compounds has both ionic and 101. Geometrical configuration ofBF3 and NF3 molecules
conllent bonding? is
(a) NaBr (b) Ba(CN)2 (a) the same because of same covalency of the centraD.
(cl PC1s (d) CH 3CH2OH atom
(b) different because BF3 is polar and NF3 is
92. Compound Xis highly volatile and insoluble in water.
non-polar
Bonding in X is (c) different because BF3 is non-polar and NF3 is
(a) ionic (b) covalent polar
(c) polar covalent (d) coordinate (d) none of the above is correct
93. The maximum covalency is equal to the number of 102. Coordinate covalent bond is absent in
(a) paired p-electrons
(b) unpaired s-clectrons
(a) [Fe(CN) 6 r-
(b) adduct ofNH 3 and BH 3
(c) unpaired s- and p-electrons (c) H 3o+
(d) s- and p-electrons in the valence shell (d) PC1 3
94. Select incorrect statement. 103. The compound NH 3 -BF3 can be easily separated
(a) Double bond is shorter than a single bond into its compounds because
(b) cr-bond is weaker than a x-bond (a) BF3 is highly reactive
(c) Double bond is stronger than a single bond (b) NH 3 is highly reactive
(d) Covalent bond is stronger than a hydrogen bond (c) BF3 and NH 3 are unstable
95. The As-CI bond distance in AsCI 3 is 2.20 A. Ifradius (d) BF3 and NH 3 have their independent existence
of chlorine atom is 0.99 A then radius of As atom is 104. Consider following compounds
<a> o.24 A Cb) 3.19 A I: K4 [Fe(CN)s1, II: Nli&Cl, III: H2 SO4
(c) 1.21 A (d) 2.09 A Ionic, covalent and coordinate bonds are present in
(a) I, II and III (b) I and III
96. CovaJency of carbon in the CO molecule is three (c) II and III (d) I and II
because 105. Which statement is not correct about NO2 ?
(a) an unexcited carbon atom has two unpaired
electrons (a) It is paramagnetic
(b) the carbon atom can be an acceptor of an electron (b) It forms dimer and paramagnetism is lost
pair (c) It has one coordinate bonds
(c) the carbon atom has four valence electrons (d) It has nitrogen oxygen triple bond
(d) maximum covalency of carbon is three 106. In ammonia, H-N-H bond angle is
(a) 106. 7° (b) 104.5 °
97. The number of electron dots in Lewis structure (c) 109.5° (d) 120°
indicates
(a) number of valence electrons in an atom 107. The angle between the bonding orbitals of a moleculle
(b) number of protons in the nucleus of the atom AX3 with zero dipole moment is
(a) 120 ° (b) 109°
(c) total number of electrons in an atom °
(d) total number of electrons and protons in an (c) 104 (d) 180°
atom 108. Number of bonding pairs (X) and Ione pairs (Y)
98. If there are five electron pairs in outer shell, then around the central atom in the I3 ion are
structure and bond angle as predicted by X Y X Y
Sidgwick-Powell theory is (a) 2 2 (b) 2 3
(a) octahedron, 90° (c) 3 2 (d) 4 3
(b} trigonal bipyramidal, 120 ° and 90° 109. The nitrogen atoms in NH 3, NH2 and NH,t are all
(c) pentagonal bipyramidal, 72° and 90 ° surrounded by eight electrons. When these species
(d) tetrahedron, 109 ° 28' are arranged in increasing order of H-N-H bomd
99. Octet rule is not followed in angle, correct order is
(a) CC1 4, N 2O4 and N2 O5 (b) BF3, BeCl2 and NOi (a) NHa, NH2, NHt Cb) NHt, NH2, NHa
(c) NaCl,MgC1 2, MgO (d) PCI 3,NH 3,H2O (c) NHa, NHt, NH2 (d) NH2, NH 3, NH.t
100. Which has maximum number of lone-pairs of 110. Which of these molecules have non-bonding electron
electrons on the central atom? pairs on the central atom?
(a) XeF2 (b) H30+ I : SF4; II : ICl3; III : SOi
(c) XeF.. (d) XeF6 (a) II only (b) I and II only
(c) I and III only (d) I, II and III
Chapter 2 : Chemical Bonding | 83

111. Which species has the same shape as the NO3 ion? (c) hybrid formation
(a) SO3 (b) SOl- (d) none of the above
(c) C1F3 (d) C1O3 121. The hybrid orbital of the central atom in A1F4“ is
112. Which one of the following compounds has the (a) sp (b) sp2
electron-pair geometry as the trigonal bipyramidal (c) sp3 (d) dsp2
with three equatorial positions occupied by lone pairs 122. What hybrid orbitals are employed by carbon atoms
of electrons? 1, 2 and 3, respectively as labelled in the compound
(a) [A1C13] (b) XeF2 shown?
(c) [Pt(NH3)2Cl2] (d) CH3—Mg—Br
113. Electron pairs occupy localised orbitals. Their h3c—c—c=n:
orbitals are oriented in such a way that the repulsion 12 3

between electron clouds becomes (a) sp3,sp,sp (b) sp3, sp2,sp


(a) zero (b) minimum (c) sp3, sp, sp2 (d) sp3, sp2, sp2
(c) maximum (d) infinite 123. The geometry of the atoms in the species PC14 is best
114. When the number of electron pairs on the central described as
atom is six, then geometry of the molecule is (a) tetrahedral (b) see-saw
(a) octahedral (b) trigonal bipyramidal (c) square (d) trigonal bipyramidal
(c) equilateral triangle (d) linear
124. In vinyl acetylene CH=C—CH—CH2, type of
115. PC15 has a shape of trigonal bipyramid whereas IF5
has the shape of a square pyramid. It is due to overlapping in (CjQ C3) bond is
(a) presence of unshared electron pair on I which is (a) sp2-sp (b) sp-sp2
oriented so as to minimise repulsion while P in (c) sp3-sp3 (d) sp3-sp2
PCI5 has no unshared pair 125. In which case, hybridisation of the central atom is
(b) octet of P is complete while that of I is incomplete affected when
(c) P and I are of different groups (a) NH3 changes to NH4 (b) A1H3 changes to AlH^
(d) F and Cl have different extent of repulsion (c) in both cases (d) in none case
116. Select correct statement about valence-bond 126. Hybridisation of the nitrogen atom and electron
approach. geometry around nitrogen atom in pyridine is
(a) Each bond is formed by maximum overlap for its
maximum stability
(b) It represents localised electron model of bonding
(c) Most of the electrons retain the same orbital
locations as in a separated atoms
(d) All the above are correct statements (a) sp3, pyramidal (b) sp2, planar trigonal
(c) sp2, linear (d) sp3, tetrahedral
117. Which has a maximum repulsive interaction?
(a) bp-bp (b) Ip-lp 127. Which pair of substances will have the most similar
(c) lp-bp (d) Equal geometry?
118. A molecule of the type AX5 has square pyramidal (a) SOj.SO^ 0» SO3,SOT
geometry. Hence, number of lone pairs on A is (c) SOs.CO?- (d) SOf.COT
(a) 4 (b) 3 128. Hybridisation of the underlined atom is affected
(c) 2 (d) 1 when
119. The bond angle inH20 is 105° and inH2S it is 90°. It is (a) CH3COOH is decarboxylated
due to
(b) CH3C H2OH is dehydrated
(a) the larger size of S atom as compared to 0 atom
which minimises repulsion and allows the bonds (c) CH3C H3 is chlorinated
in H2S to be purely p-type (d) C6H6 is nitrated
(b) liquid state of H2O as compared to gaseous state 129. Hybridisation on carbon in short lived species CH|+ is
ofSC^ (a) sp-linear (b) sp2-trigonal
(c) both (a) & and (b) (c) sp3-tetrahedral (d) none is correct
(d) none of the above is correct e
130. Hybridisation on carbon in carbanion CH3 is
120. True structure is predicted by
(a) valence-bond approach (a) sp3 (b) sp3^2
(b) Sidgwick approach (c) sp2 (d) sp3d
84 | Essential Inorganic Chemistry

131. The cyanide ion CN and the nitrogen molecule N2 140. A set of inner d-complex is
are isoelectronic. However, in contrast to CN“,N2 is (a) [Fe(CN)6 ]•", [Ni(CN)4 [Pt(en)2 f*
chemically inert due to (b) [Fe(H2O)6 ]3’, [CoF6 J3', [Cr(CN)6 ]3’
(a) unsymmetrical electron distribution (c) both (a) and (b)
(b) low bond energy
(d) none of the above
(c) absence of bond polarity
(d) presence of greater number of electrons in
bonding One or More Than One Options Correct
©
132. Select correct statement about carbonium ion CH5 1. Which of the following have identical bond-order?
(a) CN" (b) Ga­
(a) This cation shares eight electrons among five
te) N0+ (d) CN+
bonds
(b) There is no empty orbital 2. Select correct statement(s) about C3" ion
(c) It is not electron deficient (a) It reacts with H20 forming C3H6
(d) All the above are correct statements (b) It reacts with H20 forming CgELj
©
133. Select correct statement about carbenium ion CH3 (c) It has two sigma and two pi bonds
(d) It has three sigma and one pi bonds
(a) It is planar with empty p-orbital 3. The planar shape of N(SiH3)3 is explained by the
(b) There are three sigma (C—H) bonds
(c) It is electron deficient with six electron in (a) type of hybrid orbitals of nitrogen
outer-shell (b) additional dit- pit overlap along the N—Si bond
(d) All the above are correct statements (c) higher electronegativity of nitrogen
(d) higher electronegativity of silicon
134. Which of the following has maximum bond energy?
(a) Oa’ (b) 02 4. Which are the species in which sulphur undergoes
sp3 hybridisation?
(c) 02 (d) ($-
(a) SF4 (b) SC12
135. In which pair or pairs is the stronger bond found in (c) SO?" (d) H2S
the first species?
I: Ol'A; II:N2,N2; III:N0+,N0“ 5. Which of the following are non-polar?
(a) I only (b) II only (a) SiF4 (b) XeF4
(c) I and III only (d) II and III only (c) SF4 (d) BF3
136. Which plot best represents the potential energy (E) of 6. In which of the following pairs, molecules/ions have
two hydrogen atoms as they approach one another to similar shape?
form a hydrogen molecule? (a) C02 and H20 (b) BF3 and (CH3)3C®
(c) CC14 and PtCl4 (d) NH3 and BF3
I I 7. In BrF3 molecule, the lone pairs occupy equatorial
(a) E E
(b)
position to minimize
(a) lone pair-bond pair repulsion only
distance —- distance —•
(b) bond pair-bond pair repulsion only
t I (c) lone pair-lone pair repulsion and lone pair-bond
E (d) E pair repulsion
(0
(d) lone pair-lone pair repulsion only
distance — distance —* 8. H20 is dipolar, whereas BeF2 is not. It is because
(a) the electronegativity of F is greater than that of 0
137. Number of unpaired electrons in tV(H2O)6 ]3+ is
(b) H20 involves hydrogen bonding whereas BeF2 is a
(a) 2 (b) 3 discrete molecule
(0 4 (d) 5
(c) H20 is linear and BeF2 is angular
138. dsp2 hybridisation is in (d) H20 is angular and BeF2 is linear
(a) [Ni(CN)4f- (b) [Ptfen), f*
(d) none of these 9. Among the following the pair in which the two specie!
(c) both (a) and (b)
are not isostructural is
139. Select outer d-complexes
I: [Fe(CN)6]3"; II:[Fe(H20)sr (a) SiF4 andSF4
(b) I03 and XeO3
III:[CoF6]3-
(c) BH; andNHJ
(a) II, III (b) I, II
(d) I, II, III (d) PF6" andSF6
. (0 I, III
Chapter 2: Chemical Bonding | 85

10. PbCl4 exists but PbB^ and Pbl* do not because of 19. The correct structural representation of diborane is
(a) Br- and I" ions are bigger in size H H
(b) chlorine is a gas
(c) Br2 and I2 are more electronegative
(a) [B^riB^r (b) HB—BH
(d) inability of bromine and iodine to oxidise Pb2+ to H H
Pb4+
H H H H
11. Which is/are correct statements? (c) (d) H2 B=B
(a) A solute will dissolve in water if hydration energy H H H H
is greater than lattice energy
(b) If the anion is large compared to the cation, the 20. Which of the following is/are electron deficient
lattice energy wil remain almost constant compounds?
(c) Solubility of IIA hydroxide is in order.
(a) NaBHi
Be(OH)2 < Mg(OH)2 < Ca(OH)2< Sr(OH)2 (b) B2H6
(d) None of the above is correct (0 A1C13
12. In which molecule is the van der Waals’ force likely to (d) C*3"
be most important in determining melting point and
boiling point?
(a) IC1 (b) Br2
Passage Comprehension Questions
(c) H2S (d) CO Passage 1
13. Which have linear structure? A paper published in the research journal Science in
(a) BeF2 (b) AgfCNK 2007 (S. Vallina and R. Simo, Science, 305, 506 Jan 26,
(c) CO2 (d) XeF2 2007) reported studies of dimethyl sulphide (DMS, an
important green house gas that is released by marine
14. Inl3 phytoplankton. This gas “represents the largest natural
(a) is Lewis acid and I" as Lewis base source of atmosphere sulphur and a major precusor of
(b) I2 is Lewis base and I" as Lewis acid hygroscopic (i.e., cloud forming) particles in clean air over
(c) I3 itself behaves as amphoteric anion the remote oceans, thereby acting to reduce the amount of
(d) I3 does not exist solar radiation that crosses the atmosphere and is
15. In the following case (s), hybridisation of the absorbed by ocean.”
underlined atom is affected Answer the following questions
(a) PC15 (solid) dissociates into PCI4 and PClg
1. Steric number of sulphur in DMS is
(b) LiH reacts with A1H3 forming LiAlH4
(a) 2 (b) 3
(c) NH3 is protonated
(c) 4 (d) 5
(d) H3PO2 is heated forming PH3 and H3PO3
16. The correct order of the 0—0 bond length in O2, H2O2 2. Select correct statement(s).
(a) Bond pair-bond pair repulsions of the two
and O3 is
(S—CH3) bonds in DMS would be greater than
(a) H2O2 > O3 > (^
the bond-pair bond-pair repulsions of the S—CH3
(b) O2 > O3 > H2O2
and S—H bonds in thiol.
(c) O2 > H2O2 > O3 (b) Bond angle ZC—S—C in DMS is greater than
(d) O3 > H2O2 > O2 bond angle ZC—S—H in thiol
17. Which combinations of the compounds and their (c) Both (a) & (b) are correct
geometry are correct? (d) None of the above is correct
(a) HgCl2—linear (b) C1F3—V-shaped
3. The mean sea water concentration of DMS in the
(c) CIF3-T-shaped (d) ICI4—square planar
ocean is 2.7 nm (nanomolar). How many molecules of
18. Which combination of the compounds with their DMS are present in 1.0 m3 of sea water?
magnetic moments are correct? (a) 16.254 x 1017 molecules
(a) K3Mn(C2O4)3-3H2O H = 4.90 BM (b) 2.7 x 1017 molecules
(b) K3Fe(C2O4)3 -3H2O (1=5.92 BM (c) 2.7 x IO-6 molecules
(c) Ni(CO)4 p=0.00 BM (d) 16.257 x 109 molecules
(d) [Ni(CN)4f- >1=0.00 BM
86 | Essential Inorganic Chemistry

Passage 2 Passage 4
Assume following structure of N20 and answer the Consider the following molecules and answer the
questions. questions given:
N=N=0 :n—N=0 :n=n—o: I:H2O; II:CO2; III:NH3
I ’* ” n
in IV:CC14; V: C1F
1. In which compound are the bonds most polar?
n— n=o :n=n=o :n=n=o: (a) H2O (b) COa
IV V VI (c) CC14 (d) GIF
1. Which is most favourable structure? 2. Which compounds are not polar?
(a) I (b) III (a) H2O,CO2 (b) CO2,NH3
(c) V (d) VI
(c) H2O,CC14 (d) CO2,CC14
2. Select correct statements).
3. Which atoms in H2O and C1F are negatively charged?
(a) Structure I is not acceptable as there are
10 electron on N atom and 18 electrons total (a) 0 in H2O and Cl in C1F
(b) Structure IV is not acceptable as there are only (b) H in H2O and F in C1F
three bonds (c) 0 in H2O and F in C1F
(c) Structure VI is not acceptable as there are
10 electrons on oxygen atom (d) None of the above is correct
(d) All the above are correct statements
Passage 5
3. Structure III is represented as
The platinum-chlorine distance has been found to be
(a) :nsN- 6:" (b) :n=n—6:+ 2.32 A in several crystalline compounds. This value
applies to both of the compounds shown in figure,
(c) :n=n—6: (d) :n=n —o:+
NHa NH3
Passage 3 ci ci Cl^Pfr»NH3
We consider dissociation energy of different species.
NH3 Cl
Dissociation energy (A) (B)
Species
(kJ mol'1)
O2+ 642.9 Based on the above structures, answer the following
O2 493.6 questions
o-2 395.0 1. Cl—Cl distance in structure (A) is
N2+ 840.7 (a) 2.32 A (b) 4.64 A
N2 941.7 (c) 1.16 A (d) 9.28 A
n-2 765.0
2. Cl—Cl distance in structure (B) is
NO’ 1046.9
(a) 2.32 A (b) 1.52 A
NO 026.9
(c) 2.15 A (d) 3.28 A
NO" 487.8
3. A is
Answer the following questions : (a) cis-isomer (b) frans-isomer
1. In which case bond-order increases? (c) chiral isomer (d) none of these
(a) O-j changes to (b) N2 changes to N2 4. B is
(c) NO changes to NO" (d) In all cases (a) cis-isomer (b) nuclear isomer
2. In which case electron from anti-bonding MO (c) chiral isomer (d) coordinate isomer
(HOMO) is removed? (HOMO : Highest Occupied
Molecular Orbital) Passage 6
(a) Oz to 0£ (b)N2toN2 By the following method you can predict the
(c) N2 to N2 (d) In all cases geometrical shape of species with only one central atom,
3. In which case electron from bonding MO is removed? without first drawing the Lewis structure:
(alOztoOz* (b)N2toN2
1. Total number of electron pairs
= (number of valence electrons ± electrons (foi
(c)NOtoNO+ (dlCKjtoO^ ionic chargeVS
Chapter2: Chemical Bonding | 87

2. Number of bond electron pairs = number of atoms 2. Number of lone pairs around phosphorus in PC15,
-1 PC14 and PClg,are respectively
3. Number of electron pairs around central atom = (a) 0,1,2 c :; . (b) 0,0,0
total number of el&tron pairs (0 1, 2, 3 ' (d) 0, 2,1
- 3 [number terminal atoms (except H)]
4. Number of lone pair = (number of central electron Passage 8
pairs - number bond pairs) Read the following short write-up and answer the
Read the above method and answer the following questions at the end of it.
questions: “Each of the following compounds has a
1. IfAisthe central element of the molecule containing nitrogen-nitrogen bond”
A and X element and E the number of electron pairs (A) N2, (B)N2H4
round it then VSEPR notation AX^E will be for the (C) n2f2
molecules
Molecules may be polar or non-polar, that is
(a) PC15,IC14 (b) NH3,H2O
dependent on the electronegativity difference between two
(c) NH3,C1O3 (d) IC14-,C1O3 atoms. Their bond-lengths (between N-atoms) have been
2. VSEPR notation ofPCl5,H2O andIC14 are 1.45 A, 1.10 A and 1.22 A but not in correct order.
PC16 H2O ici; 1. Compounds have been matched with their bond­
length (between N-atoms). Which is not the correct
(a) ax5 AX2E2 ax4e2 matching?
(b) ax5 ax2e ax4e (a) A-1.10 A (b) B- 1.45 A
(c) ax5 ax2 ax4e (c) C - 1.22 A » - (d) None of these

(d) AXg ax2e2 A^E 2. Select incorrect statemei^t(s).


(a) N2 is an inert gas
3. Square planar structure is predicted for (b) N2F2 has sp2-hybridised N-atom
(a) IC14,C1O3 (c) N2Hi is an inert solvent
(b) PCl4+,PClg * (d) NjHj is used as rocket fuel
(c) IC14-,PC14+
(d) ICl4-,[Ni(CN^-] Passage 9
4. Based on above method, structure of the some of the Read the following short write up and answer the
molecules have been matched. Which is the incorrect questions at the end of it.
matching? Two substances having the same molecular formula,
(a) PC15 — trigonal bipyramidal C4H8O, were examined in the gaseous state by electron
(b) C1O3 — square planar diffraction. The carbon-oxygen distance was found to be
143 pm in the compound A and 124 pm in compound B.
(c) ICI4 — square planar
C—0 bond length = 143 pm
(d) PC14 — tetrahedral
C=O bond length = 123 pm
Passage 7 1. Which of the above compounds is inert?
Read the following short write-up and answer the (a) A .(b) B
(c) Both (a) and (b) (d) None of these
questions at the end of it.
In certain polar solvents PC15 undergoes an ionisation 2. Which of the above can show functional isomerism?
reaction in which Cl“ ion leaves one PC15 molecule and (a) A (b) B
attaches itself to another. (c) Both (a) and (b) (d) None of these
2PC15 PC14+ + PClg 3. Keto-enol tautomerism occurs in
1. Select incorrect statement(s). (a) A (b) B
(a) Dissociation is a redox reaction (c) both (a) and (b) (d) none of these
(b) Hybridisation changes from sp3d to sp3(f (PClg) 4. Distinction between two functional isomers of the
andsp3(PCl4) same main functional group can be made by
(c) Structure changes from trigonal bipyramidal to (a) Iodoform test (b) Tollen’s test
tetrahedral (PC14) and octahedral (PClg) (c) Br2 water (d) Phthalein test
(d) None of the above is incorrect
88 I Essential Inorganic Chemistry

5. Probable structures of A and B can be


Assertion & Reason
Q
A B Codes:
(a) (a) Both A and R are true and R is the correct
0 explanation of A.
(b) Both A and R are true but R is not the correct

Q
Cb)� explanation of A.
0 (c) A is true but R is false.
(d) A is false but R is true.
0
( )
c 0
0
�H
1. Assertion (A): On cooling, the brown colour of
nitrogen dioxide disappears.
Reason (R): On cooling N02 undergoes

'-A
OH OH
dimerisation resulting in the pairing of odd electrons
(d) ofN02 .
�H
2. Assertion (A): Tin (Sn) exhibits +2 and +4
oxidation states only and not +3 oxidation state.
Passage 10
Reason (R): Inert pair effect of electron pair ill
Read the following experimental fact and answer the
s-orbital is responsible.
question at the end of it.
A sample of lead (II) bromide, PbBr2, was placed in a 3. Assertion (A): PbC12 is more stable than PbC14 •
crucible. The ends of two carbon electrodes were put in the Reason (R): PbC14 is powerful oxidising agent.
solid and connected to a lamp and power pack. At first the
lamp did not light, but when the bromide was heated and 4. Assertion (A): BF3 is a weaker Lewis acid than
began to melt the lamp began to glow. BC1 3 •

1. Select correct statement(s). Reason (R): The planar BF3 molecule is stabilised
(a) PbBr2 is non-conducting in solid state to a greater extent than BC1 3 by B-X 1t-bonding.
(b) PbBr2 is conducting in aqueous solution 5. Assertion (A): 02 is paramagnetic in nature.
(c) PbBr2 is conducting in fused state
Reason (R): Bond-order of oxygen is two.
(d) PbBr2 is non-conducting in moJten state
6. Assertion (A): K� is paramagnetic.
Passage 11 Reason (R): K+ ion has unpaired electron.
Read the following short-write up and answer the 7. Assertion {A) : Dipole moment of o-dichlol'lo
questions at the end of it. benzene is larger than the theoretical value.
"Methyl isocyanate is used to make certain pesticides.
Reason (R): Due to dipole-dipole repulsion of two
In December 1984, water leaked into a tank containing Cl-atoms in ortho position bond angle is greater tham
this substance at a chemical plant, producing a toxic cloud 60° .
that killed thousands of people in Bhopal.
8. Assertion (A): N-atom in NH3 i� sp3 hybridised
1. Which is not the resonance structure of methyl and bond angle is 107 ° .
isocyanate? Reason (R): lp-bp repulsion (VSEPR) decreases
•• ••
(a) CH3-N=C=O:
9 EB
(b) CH3 -N-C!!!!!O:
•• bond angle,to 107° .
•• e EB •• 9 9. Assertion .(A) : P4 exists but N4 does not exist.
(c) CH3 -N!!!!!!!C-O: (d) CH3-N !!!!C-0:
••
e ••
Reaso� (R): In P4 form P-atoms are joined by
2. Which of the atoms in CHaNCO has maximum single bonds while in N4, two N-atoms are joined by
s-character? double and other two by single bonds.
(a) N (b) 0 10. Assertion (A): PC15 exists butNC15 does not exist.
(c) C(in CH3 ) (d) C (bonded to O and N)
Reason {R): d-orbital in P-atom takes part in
3. Total number oflone pairs are chemical bonding; this lacks in N-atom.
(a) 2 (b) 3 11. Asserton (A): The C-C bond distance in methyl
(c) 4 (d) 5 acetylene is unusally short (145 pm) for a single bon.d.
Chapter 2: Chemical Bonding I 89
Reason (R) : Hyperconjugation involving "no 18. Structures of [Ni(H20)4 f• and [Ni(CN)4 f- are
bond" structure, different.
H 19. Dipole moment of[Ni(CN)4 f- is zero.
H-C=C=C<
20. Dipole moment of o-dichloro benzene is smaller than
H
the theoretical value, while in case of o-nitrophenol,
increases the bond order of the C - CH3 bond. value is larger than the theoretical value.
12. Assertion (A) : Bond angle in NH3 is greater than Fill in the Blanks
that in PH 3 • 1. The complex ion [Pt(NH 3 )4 f + has a ............
Reason (R): P-H bonds are longer and the lower structure.
electronegativity of P permits electron density to be
2. AlC1 3 is covalent while AIF3 is ionic. The above fact
displaced towards H to a greater extent than in the
case ofNH 3 • can be justified on the basis of ............
13. Assertion (A) : Following structure is acceptable 3. Water has maximum density at ............ °C.
•• 4. �is ............ in nature because it has two unpaired
:N�N-O:
•• electrons.
Reason (R) : Octet rule is followed for each atom 5. The abnormally large specific heat of water molecule
central N has positive charge and oxygen has can be explained on the basis of ............
negative charge. 6. The bond formed between two atoms one of low
ionisation potential and the other of high electron
True & False affinity is ............
1. Solid sodium chloride does not conduct electricity. 7. By the axial overlapping of two p-orbitals of same
2. Dipole moment of HF is less than that ofHCl because energy, a strong ............ bond is formed.
of shorter bond length of HF. 8. When NH3 and BF3 are bonded, NH3 is a ............ and
3. Bond energy ofall the P-Cl bonds in PCI 3 and PC1 5 is BF3 is a ............
same. 9. Types ofbonding in�Cl are ............ and ............
4. The covalent molecule BeCl2 has permanent dipole
10. N�is ............ but N204 is ............
moment.
5. The H-N-H bond angle in NH 3 is greater than the 11. Bond orders of oxygen, superoxide ion and peroxide
ion are ............ and ............ and of these ............
H-As-H bond angle in AsH 3•
and ............ are paramagnetic.
6. All the molecules with polar bonds have dipole
12. Hybridisation of N atom in N�, N03 and NH 3 are
moment.
respectively ............ and ............
7. Sugden introduced singlet linkage in which one
electron is donated instead of one pair of electrons. 13. Ni atom in [Ni(CN)4 f- is ............ hybridised and in
8. PC1 5 in solid state dissociates into PC1% and PC1 6. Ni(C04 ) is ............ hybridised.
9. 02 , 02 and�- are paramagnetic species. 14. Magnetic moment ofiron in [Fe(H20) 6 f + is ............

10. Dipole moment ofBH 3 is zero. 15. A molecule like H-X is 50% ionic ifelectronegativity
difference ofH andXis ............ eV.
11. Dipole moment ofCHC1 3 is larger than that ofHCl.
16. Out ofNH 3 , PH 3 and AsH3 boiling point of ............ is
12. All the B-H bonds in BiH6 are equivalent. maximum and that of ............ is minimum.
13. Hybridisation of sulphur in S�is sp. 17. Coordination number of H20 in liquid state is
14. Bond length of (C=O) bond is larger than (C-0)
bond in C�-- 18. When N2 changes to N2, the N-N bond distance
15. The highest occupied energy band is called the ............ and bond order............
valence band and the lowest unoccupied energy band 19. When � changes to Oi the 0-0 bond distance
is called the conduction band.
............ and bond order ............
16. NaCl conducts electricity only in aqueous solution.
20. CN- and N2 are isoelectronic. But in contrast to cN-,
17. Ci and C3 atoms in CH==C-CH=CH2 are
N2 is chemically inert because of ............
respectively sp and sp2 hybridised.
90 | Essential Inorganic Chemistry

21. Unsymmetrical PCl5(s) splits into and 5. Match type of bond (given in List I) with method of
formation (given in List II).
22. In I3,12 is a and I” is a List I List II
23. Metallic bonding is explained by and A. o-bond 1. Lateral overlapping
B. Covalent bond 2. Sharing of electrons
24. In the case of insulators, the energy gap is C. Ionic bond 3. Transfer of electrons
while in case of semiconductors energy gap is D. n-bond 4. Donating an electron
5. Accepting an electron
25. The highest occupied energy band is called the 6. Axial overlapping
and lowest unoccupied energy band is called
the 6. Match the hybridization (in List I) with the
respective geometrical shape of the hybrid orbitals
Matrix-Match Type Questions (in List II).

I. Only One Correct List! List II

1. Match the compounds (in List I) with the structure A. sp 1. Tetrahedral

(in List II) B. sp2 2. Linear


C. sp3 3. Trigonal
List I List II D. dsp2 4. Square planar
A. XeF4 1. Distorted octahedral E. d2sp3 5. Pyramidal
B. XeF6 2. Tetrahedral 6. Octahedral
C. XeO3 3. Square planar
D. XeO4 4. Pyramidal 7. Match the list of type of bonds (List I) with list of
compounds (List II).
2. Match the type of species (in List I) with the List I List II
compound (in List II).
A. Ionic 1. H2O
List I List II B. Single covalent 2. nh3
A. Ionic species 1. HCI C. Coordinate covalent 3. CaCI2
B. Non-polar covalent 2. LiCH3 D. Hydrogen bond 4. c2h2
C. Polar covalent species having 3. KCH3 E. Multiple covalent 5. ch4
sigma bond only 6. nh4+
4. NO
8. Match the species (in List I) with the bond order
3. Match the compounds (in List I) with the type of (in List I).
hybridisation on the central atom/ion (in List II).
List I List II
List I List II
A. n2 1. 1.0
A. BCI3 1. sp3
B. o2 2. 2.0
B. NH3 2. sp3d2
C. f2 3. 2.5
C. [Cu(NH3)4]2+ 3. sp2 D. o2 4. 3.0
D. [Fe(H2O)6]2+ 4. dsp2
9. Match the items under List (I) with items under
4. Match the compounds (in List I) with the structures List (II).
(in List II).
List (I) Molecule List (II) Shape
List I List II
A. PCI5 1. V-shaped
A. (CH3)3B 1. Square planar B. F2O 2. Triangular planar
B. NH4CI 2. Trigonal bipyramid C. bci3 3. Trigonal bipyramidal
C. [IChf 3. Tetrahedral D. nh3 4. Trigonal pyramidal
D. PCI5 4. Trigonal 5. Tetrahedral
Chapter 2: Chemical Bonding |

10. Match List I (Hybrid bond orbitals) with List II in Integer Answer Types
(species) and select the correct answer.
This section contains 8 questions. X Y Z
List I List II The answer to each of the questions is © © ©
sp3
a single digit integer, ranging from 0 to (j) © O
A. 1. ICI4
9. The appropriate bubbles below the
B. dsp2 2. TeCI4 © © ©
respectively question numbers in the
© © ©
C. sp3d 3. MnO4 ORS have to be darkened. For
D. sp3d2 4. Ni<CN)i example, if the correct answers to
question number X, Y, Z and W (say) © © ©
11. Match the species (in List I) with bond-order (in are 6, 0, 9 and 2, respectively ® ® ®
List II) and number of unpaired electrons (in List III). 0 © 0
© © ©
List I List II List III © © ©
(Species) (Bond-order)
Setl
(Unpaired electron)
1. Steric number ofCHt is
Be2 2 2
O2 2.5 0
2. Steric number of H2O is
O2+ 0.5 1 3. Lone pairs in I3 are
Of- 1.0 1 4. Bond-order in NO+ is
5. Number of hybrid orbitals in XeF4 is
12. Match the species (in List I) with Bond-order
(in List II) and isoelectronic species (in List III). 6. Number of unpaired electron(s) in O2 is/are
7. Formal charge on sulphur in SO2 is
List III
List I List II
(Isoelectronlc 8. In a compound of Xe, F and O
(Species) (Bond-order)
species)
Xe = 60.4% (131.3)
NO + 2.5 O2 F = 17.5% (19.0)
NO 3.0 O2
Thus, number of oxygen atom
NO" 2.0 N2
Set II
II. One or More Correct This section contains 9 questions.
1. Match the species in Column I with corresponding 1. A slightly polar molecule AB has dipole moment
characters) in Column II. 0.24 D. If bond-length is 1 A, ionic character is
Column I Column II 2. In a very ideal imaginary condition, a sample of
was taken, and its osmotic pressure was found to
A. ch3 1. Bond order 3 0.274 atm at 300 K. (density = 1 g/mL). Thus, numl
® of water units attached to one water unit is
B. ch5 2. Paramagnetic
3. In O3, central oxygen atom is hypervalent. It sha]
C. NO® 3. sp2 electron pairs.
D. O2 4. Carbocation 4. Total number of orbitals involved in the formation
E. N2 5. Carbonium ion hybrid orbitals in C1F2“ is
6. Octet rule violated 5. Total number of orbitals involved in the formation
hybrid orbitals in C1F2+ is
2. Match the species in Column I with the 6. AY4 forms square planar type structure. Distai
corresponding character(s) in Column II. between A and X atoms is 4 units then distai
Column I Column II
between two X atoms in trans position is
7. Out of CH2C12, CH4, CC14, H2O, CHC
A. co2 1. Zero dipole moment
d-dichlorobenzene, o-cresol, p-xylene, SC12, BF3,1
B. ch4 2. Expansion of octet and CH2O, non-zero value of dipole moment is
C. cif2® 3. sp3 hybridisation
8. Superoxide ion has electrons in anti-bondi
D. PCIs 4. sp3d hybridisation
molecular orbitals.
E. CIF® 5. Linear
9. Difference of a and n bonds in C(CN)4 is
Answers & Solutions
Master Exercises
Exercise 1
Short Answer Type Questions 5. C: ls 22s22p 2•
1. The NH3 molecule has a dipole moment because the We consider formation of two sp-hybrid orbitals, one
three N-H bond dipoles add to give a net dipole pointing directed toward oxygen and directed 180° opposite. This
upward to the N atom from the base of the pyramid that contains carbon lone pair, leaving empty sp hybrid
defines the NH3 structure. orbita]s with a filled oxygen a-orbital.
The SF6 molecule has no dipole moment because each The two unhybridised singly occupied carbon 2p-orbitals
S-F bond dipole is balanced by one of equal magnitude form n: bonds by overlap with singly occupied oxygen 2p
pointing in opposite direction of the other side of the orbitals.
molecule. 6. C 2 (12 e) has molecular orbital electronic configuration
2. (a) CH and CO bonds are polar. as:
(b) The CO bond is most po]ar, and O-atom is the most (crls)2(cr * ls )2 (a2s)2 (cr *2s)2(n2p; )(1t2p; )cr2p�
negative atom. Bond-order
Electrons in (bonding MO - anti-bonding MO)
CH2 � CH f'o f-+ CH2 -CH=CH-08
2
3. Bond-order NO is = 8-4 =2
2 in No2• 2
1.5 in NOi c;(t3e) : (crls)2(cr * 1s)2 (a2s) 2 (cr *2s)2(1t2p!1t2p;) cr 2p;
1.33 in NO3 9-4
Bond-order = -- = 2.5
Thus, longest NO bond in NO3 and shortest in No;
Fact: Larger the bond-order, shorter the bond-length Electron is added to higher energy orbital (a2p:r ) and
bond-order increases.
No·2 No-2 No-3
Bond order +--
_ax
m
______ _ c;cne): (a1s>2 (cr *1s)2(o2s)2(o *2s)2(1t2p�1t2p;)
m ax 7-4
Bond length _______ _ _ Bond-order = -- = 1.5

4. (a) The bond angle is smal1er in NH3 than in CH 4 Electron is removed from lower-energy orbital and
because of the repulsion between the lone pair on N bond-order decreases. This bond-dissociation energy of
and the bonding pairs. c; is much lower than ofC2 in which bond-order is higher
(b) The bond angle is greater in OC12 than in OF2 and also electron is added to higher energy orbital.
because there is some 1t interactions in OC12
involving donation from the filled p-orbitals on Analytical Questions
oxygen into the empty d-orbitals on chlorine No
1t-bonding is possible for OF2 because all orbitals are
filled on both atoms.
..
incomplete octet, [:S=C=N:t,
..
1. (a) Two valence electrons m1ssmg, S and C have

(c) The bond angle in NH3 is larger than that in NF3


because the N-F bonds are larger than N-H bonds (b) Correct,
and the electron density is displaced towards the (c) One valence electron missing; odd electron species
•• • ••
more electronegative F, both effects diminish the with structure :o=N=O:
bonding pair-bonding pair repulsion.
(d) The bond angle in NH3 is larger than in PH3 because (d) N cannot have an expanded octet, correct structure
the P-H bonds are longer and the lower is:
•• •• ••
electronegativity of P permits electron-density to be
displaced towards hydrogen to a great.er extent than
.• I ••
:cI- N- Cl:

in the case ofNH3 .Both of these effects diminish the :c1:


••
repulsion due to bond-pairs.
Chapter 2 . Chemical Bonding | 93

H
V s u F=v~--u
2
ii. (a) h-n-o-h, (b) :f-n=n-f:,
(a) O 6 2 6
2. Cl 7 6 2 +2
S 6 8 2 0
H :o;
(b)
O 6 4 4 0
I ..
(c) H-O-N=o; (d) H-N-N=o:
(c) B 3 8 0
F 7 2 6 0 12. PCl5:sp3d trigonal bipyramidal, BrF5 sp,33dj2 square
(d) N 5 8 0 1 based pyramid.
(centre)
N 5 4 4 13. (cf—ci— f:f sP3d] [cf—ci-f:? sp3i
(terminal)
(e) N 5 8 0 +1 2-
(centre)
-12- .. ..:o:
N
14.
5 2 6 -2
(left) :o:
•• :o:
••
:o:
N cis- trans-
5 6 2 0
(right)
(f) N 5 8 0 +1 Since the rc-bond between the nitrogen atoms must be
O 6 4 4 0 broken to interconvert these two forms, hence, this ion
will show cis-trans isomerisation.
3. (b) Structure with the highest bond order and the
shortest bond length (N=N) 15. Atom % Atomic mass Mol Molar ratio

4. BE of fission of 6 (P—P) bond in P4 is greater than 6 Xe 60.4 131.3 0.46 1


(N—N) bond in N4; and BE of(N=N)is greater than that O 22.1 16.0 1.38 3
of(P=P). F 17.5 19.0 0.92 2

5. The glowing of lamp tells us that the solid conducts when Thus, empirical formula = XeOgF2
in molten state in which there are ions which are free to
Empirical formula weight =217.3
move. This also suggest that PbBr2 is ionic.
Molar mass =217.3
6. (a) Which are formed by two electrons.
Molecular formula = XeO3F2
(b) Which are formed by one electron.
16. I. (a) II. one o, two n
Hx? 6/Hx& yH
a>B<* &>B<e 17. N2:1.10 A linear, sp hybrid
NgE): 1.22 A trigonal planar, sp2 hybrid
7. N and F both have high electron density due to smaller NgHjj: 1.45 A tetrahedral, sp3 hybrid
atomic radii. Thus, lp-bp repulsion increases
18. (a) Cl (b) N (c) S (d) N
bond-length.
8. Bond-length is inversely proportional to bond-order Atomic mass/ Molar
19. Atom % Mol
e ® Molar masses ratio
ro<-C->or c=o o=c=o Co 21.24 58 0.367 1

0 NH3 24.77 17 1.46 4


SO^ 34.65 96 0.36 1
Bond-order : between 2 and 3 3 2 h2o 6.53 18 0.36 1
Thus, (d) Cl 12.81 35.5 0.36 1
2-
f 0 O T
Empirical formula Co(NH3)4SO4 H2OC1
9. O----- Cr----- O----- Cr----- O The compound gives white ppt with BaCl2
1.62 A | ’ 1.77 A |
Hence, SO4 is outside coordination sphere
0 O
[Co(NH3)4(Cl)(H2O)]SO4
Thus, (b) 20. In N(CH3)3, lone pair on N remains on it causing lp-bp
10. NI3 and SO3" are trigonal pyramidal; I3 is linear and NO3 repulsion while in N(SiH3)3, lone pair is transferred to
is trigonal planar. vacant d-orbital of Si.
94 | Essential Inorganic Chemistry


® 38. I - length =4 x 10“n m =4x10F9 cm
21. (a) CH3 has three orbitals
e = charge =4.8 x 10-1° esu.
H p = e x I = 4.8 x 10'10 x 4 x 10“9 esu-cm
H
Thus, C-atom is sp2 hybrid with triangular planar = 1.92 x 10“18 esu-cm = 1.92 D
structure Since, 1D = 3.3 x 10-3° C-m
(b) Silver acetylide is Ag2C2 (AgC=CAg) |i = 1.92 x 3.33 x 10~3° C-m
= 6.4 x 10-3° C-m
Thus, C-atom is sp-hybridised.
39. C—O single bond length = 0.77 + 0.66 = 1.43 A
22. sp3, + 2, C=O double bond length = 1.22 A (given). It is very near
F' F to predicted for a double bond. Hence, structure are :
23. (NF3, H3O) and (NO3, BF3) pairs isostructural. O
24. KO2 has superoxide ion O2 with 17 electrons. Thus, one
and
electron remains unpaired making it paramagnetic.
25. Trigonal planar, sp2 26. Refer VSEPR theory.
40. (a) sp,2 (b) II
27. Allene is CH2=C=CH2 28. (b) 29. (d)
v2 »p *p2 41. p = exZ
30. Refer text 31. Refer text p=2.9D=2.9xlO-10 esu-cm
32. (a) and (b) both have 12 electrons. e =4.8 xlO-10 esu.
p 2.9 x 10“18 esu-cm
a 2s o* 2s 2Py
e 4.8xlO"10esu
KK- U T1 n n = 6.04 x 10“9 cm = 0.0604 nm
33. Total electrons = 2 IT X | ols 2 42. 0.0308 nm, No since p * 0
bond order = 1, stable. 43. I. D<A<B<C H. (a)
14. (a) Refer Table 2.10 : 44. LiF > LiF2, CsO > Cs2O, CaBr2 > CaBr3
Bond Comparison Comparison of 45. C—H<Br—H<Na—I<F-H<Li-Cl<K—F
Species
order of bond order bond distance
46. Refer text
N2 3 N2 > N+2 N2 <N£
47. Hydration energy of
n2 2.5 increases
O2 < O2 AlCl3=AHHyd (Al3+) + 3AHHyd (CD
O2 2 O2 > O2
2.5 decreases = -4665-3 x381
q;
= -5808 kJ mol"1
35. Calculated value ofp = e x I IE of A1CL = 5137 kJ mol-1
= 4.8 xlO-10 esu x 1.51 x 10"10 cm Since, hydration energy > IE, hence, A1C13 ionises (by
= 7.248 x 10"18 esu cm hydration energy) thus it is ionic in aqueous solution.
(1 x 10”18 esu-cm = 1 Debye) = 7.248Debye 48. I. (d)
Observed value of p = 0.79 Debye II. Ni(CO)4 and Ni(CN)4“ are diamagnetic, NiCl3- is
„ , observed value x 100 paramagnetic.
% ionic character =----- -—----- -— ------
calculated value III. sp3-sp IV. NCO",CS2,NO; V. S<C1<N<O<F
= ±Zlxioo 49. I. sp3 II. absence of bond polarity IH. four
7.248
IV. CN~, NO+ (each has 14 electrons)
= 10.9%
/H-.
36. (a) :F-N=O; (b) angular (bent) 0 'O
50. (c)(1)
I (II)H3O+,NH3, CH3’
(c) larger p is a result of the molecule being asymmetric
(bent) and having rather larger (EN) difference. NO^F
CHAh- XC-CH3

molecule has a symmetric shape. 51. There is intramolecular H-bonding in


o-hydroxybenzaldehyde (weak) while intermolecular
37. > P-tra/u 0) H-bonding in p-isomer (strong).
Chapter 2: Chemical Bonding I 95

52. NO2 has one unpaired electron, hence paramagnetic. On 53. Refer sec. 7.15. Due to resonance in conjugate diene
cooling, thee is formation of new N-N bond (in N 2O4 ) 54. (c)
using this unpaired electron; hence, paramagnetism is
lost and molecular weight increases. NCERT Problems
2NO2 � N 2O4 Refer Text
Resonance forms are only multiple representation of the
same structure.

Exercise 2
Only One Option Correct
1. (b) 2. (d) 3. (d) 4. (c) 5. (a) 6. (a) 7. (c) 8. (b) 9. (a) 10. (d)
11. (d) 12. (c) 13. (b) 14. (c) 15. (c) 16. (a) 17. (b) 18. (c) 19. (c) 20. (a)
21. (b) 22. (d) 23. (a) 24. (a) 25. (b) 26. (b) 27. (b) 28. (c) 29. (b) 30. (d)
31. (c) 32. (d) 33. (a) 34. (d) 35. (c) 36. (c) 37. (a) 38. (c) 39. (a) 40. ( b)
41. (d) 42. (a) 43. (a) 44. (c) 45. (c) 46. (a} 47.. (d) 48. (a) 49. (c) 50. (d)
51. (d) 52. (a) 53. (b) 54. (b) 55. (c) 56. (b) 57. (c) 58. (c) 59. (a) 60. (c)
61. (b) 62. (c) 63. (b) 64. (d) 65. (c) 66. (c) 67. (d) 68. (d) 69. (a) 70. (a)
71. (d) 72. (c) 73. (d) 74. (c) 75. (c) 76. (a) 77. (b) 78. (c) 79. (a) 80. (b)
81. (b) 82. (d) 83. (b) 84. (c) 85. (a) 86. (c) 87. (a) 88. (b) 89. (c) 90. (d)
91. (b) 92. (b) 93. (d) 94. (b) 95. (c) 96. (b) 97. (a) 98. (b} 99. (b) 100. (a)
101. {c) 102. (d) 103. (d) 104. (a) 105. (d) 106. (a) 107. (a) 108. (b) 109. (d) 110. (d)
111. (a) 112. (b) 113. (b) 114. (a) 115. (a) 116. (d) 117. (b) 118. (d) 119. (a) 120. (c)
121. (c) 122. ( b) 123. (a) 124. (a) 125. (b) 126. (b) 127. (c) 128. (b) 129. (a) 130. (a)
131. (c) 132. (d) 133. (d) 134. (b) 135. (d) 136. (a) 137. (a) 138. (c) 139. (a} 140. (a)

One or More Than One Options Correct Passage6


1. (a, c) 2. (b, c) 3. (a, b) 4. (b, c, d) 1. (c) 2. (a) 3. (d) 4. (b)
5. (a, b, d ) 6. (b) 7. (c) 8. (d) 9. (a) Passage7
10. (a) 11. (a, b, c) 12. (b) 13. (a, b, c) 1. (a) 2. (b)
14. (a) 15. (a), (b) 16. (a) 17. (a, c, d)
Passage 8
18. (a, b, c, d) 19. (c) 20. (b, c)
1. (d) 2 . (c)

Passage Comprehension Questions Passage 9


Passage1 1. (a) 2. (c) 3. (b) 4. (a), (b) 5. (a), (c)
1. (c) 2. (c) 3. (a) Passage 10
Passage2 1. (c)
1. (b) 2. (d) 3. (a) Passage 11
Passage3
1. (c) 2. (d) 3. (b)
1. (a) 2. (a) 3. (b)
Passage4 Assertion & Reason
1. (a) 2 . (d) 3. (c) 1. (a) 2. (a) 3. (b) 4. (a) 5. (b) 6. (c)
7. (d) 8. (a) 9. (c) 10. (a) 11. (a) 12. (b)
Passage5
13. (a)
1. (b) 2. (d) 3. (b) 4. (a)
96 I Essential Inorganic Chemistry

True & False 12. List I List 11 List Ill


1. T 2. F 3. F 4. F 5. T 6. F 7. T 3.0
8. T 9. F 10. T 11. T 12. F 13. F 14. F NO 2.5
15. T 16. F 17. F 18. T 19. T 20. T
NO- 2.0
Fill in the Blanks II. One or More Correct
°
1. square planar 2. Fajan's rule 3. 4 1. (AH3,4); (BH5,6); (CHI); (DH2,3);
4. paramagnetic 5. H-bonding 6. ionic 7. sigma {EHl)
8. Lewis base, Lewis acid 2. (AHI,5); (BHl,3); (CH3); (D)-(2,4);
9. ionic (between Ntt: and ci-), covalent (between N and H), (EH3,5)
coordinate (between H+ and NH3 ).
10. paramagnetic, diamagnetic Integer Answer Types
11. 2, 1.5, 1 oxygen, superoxide ion 12. sp 2 , sp2 , sp3
Set I
13. dsp 2 , sp 3 14. M BM 15. 1.7 16. NH3 , PH3
Questions-+ 1 2 3 4 5 6 7 8
17. 4 18. increases, decreases 19. increases, decreases Answers -+ 4 4 3 3 6 1 0 3
20. large N!!!! N bond energy 21. PCI:, PC1 6
@ @ @ @ @ @ @ @
22. Lewis acid, Lewis base (D (D (D (D (D (D CD CD
23. electron sea model, band model ® ® ® ® ® @® ®
24. very large, very short @ @ @ @ @ @ @ @
25. valence band, conduction band @ @ @ @ @ @ @ @
@ @ @ @ @ @ @ @
Matrix-Match Type Questions @ @ @ @ @ @ @ @
I. Only One Correct (J) (J) (J) (J) (J) (J) (J) (J)
1. (AH3); CBHl); (CH4); (DH2) @ @ @ @ @ @ @ @
2. (AH3 ); (BH2); (CHl) ® ® ® ® ® ®® ®
3. (AH3); (BHl); (CH4); (DH2)
Set II
4. (AH4); (BH3); (CHU; (DH2)
5. (AH6); (BH2); (CH3); (D}-(1) Questions-+ 1 2 3 4 5 6 7 8 9
6. (AH2); CBH3); (CHI); CDH4); Answers -+ 5 4 3 5 4 8 7 7 0
(EH6) @ @ @ @ @ @ @ @ @
7. (A )-{3); (BH5); (CH6); (DHl); CD CD G) G) CD G) G) CD CD
(E)-{4); ® @ ® @ @ ® ® ® ®
8. (A)-{4); (BH2); (CHI); (DH3) @ @ @ @ @ @ @ @ @
9. (AH3); (BHl); (CH2); (DH4)
@ @ @ @ @ @ @ @ @
10. (AH3); (CH2); CDHl)
@ ® @
(BH4);
@ @ @ @ @ @
11. Ust I List II List Ill @ @ @ @ @ @ @ @ @
Be; 0.5 1 (() (J) (() (J) (J) (J) (J) Cl) (()
02 2.0 2 @ @ @ @ @ @ @ @ @
o; 2.5 1
® ® ® ® ® ® ® ® ·®
-
02 1.0 0
The members of the same group of elements (arrranged as every
eighth in a group) stand to each other in the same relation as the
extremities of one or more octaves in music. John Newlands

Periodicity of Elements

Quick Points
Electrons in various subshells of a shell (orbit) are filled based on
—Aufbau rule
—Hund’s rule
—Pauli’s exclusion rule The Origin of The Periodic Table
• a-particle is positively charged helium nucleus Classification of the Elements in Blocks
Its emission causes decrease in atomic number by two units and Periodicity
atomic mass by four units Valency and Oxidation Number
“jRa —> ^Rn + $He Shielding (Screening) Effect and
Emission of [J-particle increases atomic number by one unit Effective Nuclear Charge
resulting in the formation of isobar. Size of an Atom and Ion
79°Au —» 28°o°Hg+ _°e Ionisation Energy
This is due to conversion of neutron to proton Electron Affinity
Electronegativity
In --- > JH+Jje
Metallic Nature
• ns2 pair in p-block elements is inert if it can’t be unpaired
Hydration and Hydration Energy
(inert-pair effect). If a p-block element Mx* does not have noble
gas configuration (n2np6 or Is2 (He)) then it is expected to show Acid-Base Character of Oxides
inert-pair effect where x is the number of valence electrons. Hydrides
Melting Points and Boiling Points
Inert pair
Atom Z E.C. x Mx* E.C.
effect Density
B 5 1s22s22p1 3 b3* 1s2 No Atomic Volume
Al 13 1s22s22p63s23p1 3 Al3* 1sz2s22p6 No
Ga 31 [Ar] 3d’°4s24p1 3 Ga3* [Ar] 3d10 Yes
P 15 1s22s22p83s23p3 5 p5 + 1s22s22p6 No
As 33 [Ar] 3d104s24p3 5 As5* [Ar] 3d10 Yes
.-------
98 I Essential Inorganic Chemistry

Ga [Ar} 3d 10 valency + 1 if ns2 Non Decomposable Substances


(ground state) pair is inert.
Simple
Slmple Simple
Metallic
combustible Alkalies
substances substances earths

LIi 1' I 1' I


ns2 substances
excited sUlte) 2 N As Fe Si0 2 NaOH
:;�,� j valency +3 if ns
pair is excited, and C Mo Sn Al 20 3 KOH
thus three unpaired 0 s W Pb BaO NH 40H
electrons take part H p Mn Cu CaO
MgO
l
ns
. in chemical bonding. Cl Ni Hg
B Co Ag
(For detail refer Inert-Pair Effect in Chemical­ F Bi Pt
Bonding Chapter-2) etc. Sb Au
The concept of chemical periodicity is central to the Zn
study of Inorganic Chemistry. Chemical periodicity and
1817-29 J.W. Dobereiner discovered many triads o[
the Periodic Table now find their natural interpretation in
elements and compounds, the combining weight of
the detailed electronic structure of atom; indeed they
the central component being the average of its
played a major role at the tum of the century in
partners (e.g. CaO, SrO, BaO, and NiO, CuO.
elucidating the mysterious phenomenon of radioactivity
ZnO).
and the quantum effects which led ultimately to Bohr's
theory of the H-atom. 1850 Johann Dobereiner's Triads (1780-1849)
Triads are found with sequence of three similar
elements, where the middle element has a mass equal tm
The Origin of The Periodic Table the average of the least and most massive. The diagram
below, updated from here, uses mid-nineteenth century
• A brief historical survey of various development in the
study of Periodic Table is given in Table 3.1. There is atomic mass information rather than modem data. If
no single best form of the Periodic Table since the atomic number(Z) are used (a property unknown in 1850)�
the triads are exact :
choice depends on the purpose for which the table is
used. The most convenient form for various studies is Atomic mass (1850) Atomic number
"long form" with separate panels for the actinide and
71 Li 31
7 +3 3 +19=11
lanthanide elements. Li
23 9= 2 3 11
1---+
Na Na
• There has always been confusion over the numbering 1---+ 2 2
39J

!] _,
K K 19J
system to be used for the individual groups in the
table. We adopt the 1-18 numbering scheme 401 0+137 20 +56 38
Ca Ca
recommended by IUPAC in 1988. The 'long form' of 87 ---+ 4 = 88.5 =
1
Sr
2
Sr
2
13 7 J 56
the Periodic Table is given in Table 3.2. For
Ba Ba
comparison, numbering in Roman digits I, II, .... is
also given. p afl 1s·1
1 +122
p
15 +51
As 75 ---+ 3
• One advantage of the IUPAC system is that the
groups headed by Fe, Co, and Ni have separate
1
Sb 122j
2
=76.5 As
Sb
33
51J
I
---+ 2
= 33

designations (8, 9 and 10, respectively) instead of all


s 32 1 s 16"1
2 + 128 16 +52
78 ---+ 3
I
being placed together in VIII B. Also, for the transition
Se = 80 Se 34 ---+ = 34
1 2 2
elements
group number= number of electrons in (n - l)d and ns Te 128j Te 52J
orbitals Cl as.s·1
35.5+ 121
Cl 17]
Br 35 ---+ 17 + 53 _ 35
and for the p-block elements
group number = 10 + outer-shell electrons
Br
ao
I 127J
---+ I 2
I 53
2

Table 3.1 Mendeleef's Periodic Table = 81.25

G
Precursors and Successors
1772 L.B.G. de Morveau made the first table of "chemically
simple" substances. A L. Lavoisier used this in his Li Be B C N 0 F Ne
traite Elementaire de Chimie published in 1789. Na Mg Al Si p s Cl Ar
de Morveau's Table of Chemically Simple K Ca Ga Ge As Se Br Kr
Substances Rb Sr In Sn Sb Te I Xe
de Morveau's table of chemically simple substances Cs Ba Tl Pb Bi Po At Rn
(updated with modem representations by Mazurs):
Chapter 3: Periodicity of Elements | 99

1843 L. Gmelin included a V-shaped arrangement of 1864 L. Meyer published a table of valencies for 49
16 triads in the 4th edition of his Handbuch der elements.
Chemie. 1864 W. Odling drew up an almost correct table with
O N H 17 vertical columns and including 57 elements.
F Cl Br I Li Na K 1865 J.A.R. Newlands propounded his law of octaves
S Se Te Mg Ca Sr Ba after several partial classifications during the
P As Sb Be Ce La preceding 2y; he also correctly predicted the
C B Bi Zr Th Al atomic weight of the undiscovered element
Ti Ta W Sn Cd Zn germanium.
Mo V Cr U Mn Ni Fe 1868-9 L. Meyer drew up an atomic volume curve and a
Periodic Table, but this latter was not published
Bi Pb Ag Hg Cu
until 1895.
Os Ir Rh Pt Pd Au
1869 D.I. Mendeleef enunciated his periodic law that
1857 J. B. Dumas published a rudimentary table of “the properties of the elements are a periodic
32 elements in 8 columns indicating their function of their atomic weights". He published
relationships. several forms of Periodic Table, one containing
1862 A.E.B. de Chancourtois first arranged the 63 elements.
elements in order of increasing atomic weight;
he located similar elements in this way and 1871 D. I. Mendeleef modified and improved his tables
published a helical form in 1863. and predicted the discovery of 10 elements (now
known as Sc, Ga, Ge, Tc, Re, Po, Fr, Ra, Ac and
0i 1 1 1 1 1 1 1 1 r 1 1 1 1 1 1 1
Pa). He fully described with amazing prescience
H
the properties of 4 of these (Sc, Ga, Ge, Po).
4 Note, however, that it was not possible to
predict the existence of the noble gases or the
number of lanthanide elements.
8 Li
1894-8 Lord Rayleigh, W. Ramsay and M. W. Travers
Be detected and then isolated the noble gases (He,
B
Ne, Ar, Kr, Xe).
12 C 1913 N. Bohr explained the form of the Periodic Table
N
on the basis of his theory of atomic structure
and showed that there could be only 14
16o 0 lanthanide elements.
1913 H.G.J. Moseley observed regularities in the
F characteristic X-ray spectra of the elements; he
20
thereby discovered atomic numbers Z and
provided justification for the ordinal sequence
Na
24
Mg
of the elements.
1940 E. McMillan and P. Abelson synthesized the
Al first transuranium element 93NP. Others were
28
Si synthesized by G.T. Seaborg and his colleagues
during the next 15 years.
P 1944 G. T. Seaborg proposed the actinide hypothesis
32s S
and predicted 14 elements (up to Z = 103) in this
group.
36 The Mayan Periodic Table
Cl
The Mayan Periodic Table of Elements, named for its
K similarity to the ancient Mesoamerican calendar, is based
40
Ca on electron shells.
100 | Essential Inorganic Chemistry

Main Features of Mendeleef’s Periodic placed in the same vertical column. The horizontal series
are generally written as periods, and the nine vertical
Classification columns-headed by the Roman numerals from I to VIII
Study and explanation of the table and zero (0) are called groups. The periods are short or
In the table, the elements are arranged in order of long.
their increasing atomic weights in horizontal series, in
such a way that elements having similar properties are

Periodic Table
Table 3.1 With Atomic numbers written above and Atomic weights written by the side of the Symbols
Periods Group I Group II Group III Group IV Group V Group VI Group VII Group VIII Group 0

r?o R2O2 R2O3 R2O4 R2O5 RaOe R2O7 R2O8 «- formula of or


RH, RH3 rh4 RH3 rh2 RH oxides formula of Gr. VIIIB
RH
hydrides.
A B A ' B
~
A ~' B A 'B A B A B A B A

1 1 2
H 1.008 He 4
2 3 4 5 6 7 8 9 10
Li 6.94 Be 9.02 B 10.82 C 12 N 14.01 0 16 F 19 Ne 20.2
3 11 12 13 14 15 16 17 18
Na 23 Mg 24 3 Al 26.97 Si 28 06 P 31.04 S 32 Cl 35.46 Ar 39.94
4 19 20 21 22 23 24 25 26 27 28
K 39.1 Ca 40 Sc 45.1 Ti 48.1 V 51 Cr 52 Mn 54.9 Fe 55.8 Co 58.9 Ni 58.6
29 30 31 32 33 34 35 36
Cu 63.5 Zn 65.3 Ga 70 Ge 72.5 As 74.9 Sc 79.2 Br 79.9 Kr 83.7
5 37 38 39 40 41 42 43 44 45 46
Rb 85.4 Sr 87.63 Y 88.9 Zr 91.2 Nb 92.9 Mo 96 Tc 97.8 Ru 101.7 Rh 103 Pd 106
47 48 49 50 51 52 53 54
Ag 107.8 Cd 112 In 114.8 Sn 119 Sb 121.76 Te 127.6 I 126.9 Xe 131
6 55 56 57 72 73 74 75 76 77 78
Cs 132.9 Ba 137.37 La 139 Hf 178.6 Ta 180.8 W 184 Re 186.3 Os 191 Ir 198 Pt 195
79 80 58-71 82 83 84 85 86
Au 197.2 Hg 200.6 Rare Earths Pb 207.2 Bi 209 Po 210 Rn 222
81
Tl 204
7 87 88 89 90 91 92
Ra 226 Ac 227 Th 232.12 Pa 231 U 238.14

Short and Long Periods


4 The fourth period begins with K which strongly resembles
Period Elements Na; but in this case we have to pass over 18 elements
(instead of 8) before another element Rb (rubidium)
1 H He (Short period)
strongly resembling K is reached. Such a period of 118
2 Li Be B C N O F Ne elecments is called Long period. K19 ----------- Kr (36)
Electropositive Electronegative Inert 5 Rb ----------- Xeof 18 elements is another long period. The
abrupt increase from 8 to 18 elements in passing from the
There is a regular change from the strongly electropositive short to the long period is due to the interpolation of so
element Li to the strongly electronegative element F, which called transitional elements.
is followed by the inert element Ne. 6 Cs ----------- Rn consists of 32 elements and is thereto re
called a very long period, or a monster period. This increase
3 Na Mg Al Si P S Cl A from 18 to 32 elements is due to the introduction of 14
There is same periodicity as in second period. Na rare-earth elements.
resembles Li; Mg resembles Be; Al resembles B and so on. 7 It is fragmentary. It starts like a long period, but soon comes
to an end. Uranium (at fifth place) is the element of highest
Such periods of eight elements are called short periods.
atomic weight known at that time.
Chapter 3: Periodicity of Elements I 101
Intervals of Perodicity non-metals (less electropositive) followed by
electronegative elements as halogens
Thus, we observe that intervals of periodicity are 2n2
(n = period), i.e., 2, 8, 18, 18, 32 Li Be B C N O F
Na Mg Al Si P S Cl
Even series The first seven elements of a long period
forri.... the even series. 3. Other Properties The same periodicity in the
Odd series The last seven elements of a long period
properties can be observed with regard to the
form the odd series. (a) melting point
(b) boiling point
Group Zero The inert gases like helium, argon, etc. (c) specific gravity
present in the air were discovered afterwards. They were (d) atomic volume
placed in zero group. They being inert and havig no In atomic heat periodicity is not observed.
valency, stands between the highly electropositive alkali

w
metals and the strongly electornegative halogens.
Applications of the Periodic Law

IT]t []t
1. In classification of the elements
(a) We can form an idea of the properties of an
element and its compounds by knowing the group it
Most electronegative Inert
t
Most electropositive
belongs to
(b) We can compare the properties of different
Partington considered group zero as sub group B of elements and their compounds belonging to different
group VIII. groups.
Subgroups Each group is divided into subgroup A 2. In the prediction of new elements When
and B. The elements belonging to the even series of their Mendeleef first prepared the Periodic Table, many blank
respective long periods are placed to the left (sub group A), spaces were automatically left in it. Space of the elements
while those belonging to the odd series are arranged on the (not known at that time) Sc, Ga, Ge were left blank.
right-hand side (subgroup B) of each vertical column. The However he predicted their properties by naming
members belonging to a subgroup are more closely related eka-boron (below boron)
than the other members of the same group. eka-aluminium (below aluminium}
The Rare Earths Lanthanum in Group III and in eka-silicon (below silicon)
the 6th period forms a cluster with 14 other elements B Al Si
(atomic number 58-71) which have got atomic weights
Named by Mendeleef1 Eka-B, Eka-Al Eka-Si
differing by one or two or four; they are analogous in
chemical properties, and it is very difficult to separate Named afterwards] Ga Ge
them in analysis. They are known as the elements of the The following table summarises the predicted
rare earths. (15 years before discovery by Mendeleef in 1871) and
The Radioactive Elements There is also a class of observed (discovered by Winkler in 1886)
elements, placed in last period of the Periodic Table, which Germanium
differ from the rest in as much as they are radioactive, i.e., Property
Ekosilicon
(discovered In
they constantly liberate energy, whch appears to be (1871)
1886 by Winkler)
derived from the disintegration of atoms. At. wt. 72 72.3
Periodicity in the properties of the elements, when Sp. wt. 5.5 5.469
arranged in the Periodic Table is shown in terms of. At. vol. 13 13.2
1. Valency The valencies towards hydrogen Oxide EsO 2 GeO2
regularly increase up to the middle of a short period and Sp. wt. oxide 4.7 4.703
then gradually fall and come back to unity. The valencies
Chloride EsCI 4 GeCI 4
towards oxygen, on the other hand increase regularly from
Boiling point chloride < 100 ° 86°
1 to 7, and then begin with 1. Elements of group zero have
zero valency. Density chloride 1.9 1.887
LiH BeH2 BH3 C� NHa OH2 FH Fluoride EsF4 GeF4 • 3H2O
1 2 3 4 3 2 1 Not gaseous white solid mass
N a20 MgO Al20s Si02 N205 SOa Cl201 Ethyl compound EsEt 4 Ge(C2 HsO)4
1 2 Boiling point ethyl compound 160 °
160°
3 4 5 6 7
Sp. wt. ethyl compound 0.96 a little <1
2. Electrochemical Character In group I,
elements are highly electropositive. In between there are *Eka a prefix being the sanskrit numeral one
102 | Essential Inorganic Chemistry

3. In correcting doubtful atomic weight An in their properties. The arrangement of elements in


element has got only one proper place in the Periodic this table is also in keeping with their electronic
Table. Position is fixing by comparing its properties with structures.
those of neighbouring elements.
• This table also explains the cause of periodicity of
properties (discussed in following sections) and the
Elements 113 and 115 Discovered reason why similar properties recur at certain regular
Two super heavy elements, element 113 and element intervals.
115, have been discovered by scientists. The team
observed atomic decay patterns or chains, that confirm the • The horizontal rows constitute periods while the
existence of these two elements. vertical rows constitute groups or families.
In these decay chains, element 113 is produced via the • 4th, 5th, 6th and 7th periods also include transition
alpha decay of element 115. The observations were made elements.
in experiments conducted at the JINR (Joint Institute for
• 6th and 7th periods also include inner transition
Nuclear Research in Russia) U400 cyclotron with the
Dubna gas-filled separator between July 14 and elements (rare-earth) and grouped at the bottom of the
August 10, 2003. Scientists from the Glenn T. Seaborg main Periodic Table.
Institute and the Chemical Biology and Nuclear Science • Elements (58-71) are called lanthanides while
Division at Lawrence Livermore National Lab, worked in elements (90-103) are called actinides. Each is a
collaboration with researchers from the JINR. The results group of 14 elements and grouped in IIIB. (3rd period)
have been published in Physical Review C.

Explanation for Aufbau Principle


Modern Periodic Law
The properties of elements are periodic The arrangement of the long Periodic Table may be
functions of their atomic numbers. If element are explained from the distribution of electoms in various
arranged in increasing order of their atomic numbers, quantum levels. Each period starts with an element
there is repetition of properties after 2, 8, 8,18,18 and 32 having one valence electron in the s-orbital as given
elements. The numbers 2,8,18 and 32 are also referred as below:
magic numbers.
Element Valence electron
Repetition of properties after these numbers is due to
repetition of electronic configurations of the outermost H 1s1
orbit. Li 2s1
Hi Is1 Na 3s1
Li3 Is2,2s1 K 4s1
Nan Is2,2s2 2p6,3s1 H to Fr have been placed Rb 5s1
together [in group 1 (IA)]
K19 ls2,2s22p6,3s23p6,4s1 due to the fact that Cs 6s1
Rb37 (Kr) 5s1 outermost orbit has ns1 Fr 7s1
configuration.
Cs55 [Xe]6sl Electrons in various subshells have been filled based
P^87 [Rn] 7s1 on Aufbau principle i.e.,
ls<2s<2p<3s<3p<4s<3d<4p<5s<4d<5p<6i
Long form (Modern form) of Periodic Table < 4f < 6d < 6p < 7s < 5f..
It is annexed in Table 3.2. The elements have been Thus, 3d orbital which is filled after 4s is a part of
arranged in the increasing order of atomic number. fourth period. Thus one can determine number of
elements in a given period assuming two elements in s, sis
• It can explain why certain elements resemble one
in -p, ten in -d and fourteen in /'-suborbits.
another and also why they differ from other elements
Chapter 3 : Periodicity of Elements I 103
Total The sixth period is further complicated by the filling of
Period Suborblt Elements Comment
elements 4/-orbitals after 4d-orbitals. AB 4d-suborbit may
1s 2 accommodate maximum of 14 electrons, we have the first
series of 14 inner-transition elements, each related to the
2 2s 2
entry of one electron in the 4/-orbital. If these are placed
2p 6
in the Periodic Table, it would require 14 extra group; this
3 3s 2 would make the table unnecessarily long. Hence, these
3p 6 elements have been placed separately at the bottom of the
4 4s 2 3d is in 4th
Periodic Table. Similarly elements with 5/-orbitals form a
3d 10 period separate block.
4p 6 L ___,1
5 5s 2 4d is in 5th
4d 10 period d
s
p
Sp 6

6 6s 2 4f and 5d I
4f 14 are in 6th
Sd 10 period f
6p 6

• The eight families belonging to the s- andp-blocks are often called the representative elements; those to the d-block
are called the transition elements and members of the {-block are termed the inner-transition elements.

, ,

f"i!Eli_i9pjj
lto7

t-1
1. H, He (2) � 1 2 3 4 6 6 7 8
1
9 10 11 12 13 14 15 16 17 18
2. Li-Ne (8) IAIIAIIIBIVB VB VIBVIIB VIIIB IB IIBIIIAIVA VA VIAVIIA 'vn;A(O)
t0''---"""'.______________�------------
1
3. Na-Ar (8) :ui s-block d-block p-block
4.K-Kr
5. Rb-Xe (18)
(18)
�1 f(inIIIB of 6th and 7th periods)

6. Cs-Rn (32)
7. Fr-(incomplete)
104 I Essential Inorganic Chemistry

Group IA IIA IIIB IVB VB VIB VIIB VIII IB 11B III A IVA VA VI A VILA 0

(1) (2) (3) (4) (5) (6) (7) (8) (9) (10) (11) (12) (13) (14) (15) (16) (17) (18)

1 1 2
Period H He
1 ls ls
3 4 5 6 7 8 9 10
Li Be B C N 0 F Ne
2 2s 2p
11 12 13 14 15 16 17 18
Na Mg Al Si p s Cl Ar
3 8s 8p
19 20 21 22 I 23 24 25 26 27 28 29 30 31 32 33 34 35 36
Ca Sc Ti V Cr Mn Fe Co Ni Cu Zn Ga Ge As Se Br Kr
4 4s 3d 4p
37 38 39 40 41 42 43 44 45 46 47 48 49 50 51 52 53 54
Rb Sr y Zr Nb Mo Tc Ru Rh Pd Ag Cd In Sn Sb Te I Xe
5 5s 4d 5p
55 56 57 72 73 74 75 76 77 78 79 80 81 82 83 84 85 86
Cs Ba La Hf Ta w Re Os Os Pt Au Hg Tl Pb Bi Po At Rn
6 6s 5d 6p

87 88 89 104 105 106 107 108 109 110 111 112


Fr Ra Ac Rf Db Sg Bh Hs Mt Ds
7 7s 6d

58 59 60 61 62 63 64 65 66 67 68 69 70 71
Ce Pr Nd Pm Sm Eu Gd Tb Dy Ho Er Tm Yb Lu
6 4f
90 91 92 93 94 95 96 97 98 99 100 101 102 103
Th Pa u Np Pu Am Cm Bk Cf Es Fm Md No Lr
7 5f

Fig. 3.1 A Periodic Table to show the kinds of atomic orbitals (sublevels) being filled, below the symbols of blocks of elements. The
electronic structures of the A group and 0 (VIIIA) group elements are perfectly regular and can be predicted
from their positions in the Periodic Table, but there are many exceptions in the d and f blocks.
Chapter 3: Periodicity of Elements

Table 3.2 Long Form of Periodic Table


Main-Group Elements Main-Group Elements

1 1 Atomic number
H -<■ Symbol
IA Atomic weight
1.00794
1
1 H 2 13 14 15 16 18
1.00794
II A IIIA IVA VA VIA VIIA
3 4 Transition Metals 5 6 7 8 9
2 Li Be B C N O F
6.941 9 012182 10.811 12.011 14.00674 15 9994 18,9984032
P
3 11 12 13 14 15 16 17
E Na Mg 3 4 5 6 7 8 9 10 11 12
Al Si P s Cl
22.989768 24.3050 IIIB IVB VB VIB VIIB VIIIB IB IIB 26.981539 28.0855 30.973762 32.066 35.4527
R
19 20 21 22 23 24 25 26 27 28 29 30 31 32 33 34 35
I 4 K Ca Sc Ti V Mn Fe Co Zn As
Cr Ni Cu Ga Gc Se Br
39 0983 40.078 44.955910 47.88 50.9415 51.9961 54.93805 55 847 58.93320 58.6934 63.546 65.39 69.723 72.61 74 92159 78.96 79.904
0
43 44 45 46
D 5
37
Rb
38
Sr
39
Y
40
Zr
41
Nb
42
Mo Tc Ru Rh Pd
47 48
Cd
49
In
50
Sn
51
Sb
52
Te
53
I
Ag
s 85.4678 87.62 88.90585 91.224 92.90638 95.94 (98) 101.07 102.9055 106.42 107.8682 112.411 114.82 118.710 121.757 127.60 126.90447

55 56 57 72 73 74 75 76 77 78 79 80 81 82 83 84 85
Cs Ba La’ Hf Ta W Re Os Ir Pt Au Hg TI Pb Bi Po At
6
132.90543 137.327 138.9055 178.49 180.9479 183 85 186.207 190.2 192.22 195 08 196.96654 200.59 204.3833 207.2 207.98037 (209) (210)

87 88 89 104 105 106 107 108 109 no 114


7 Fr Ra Ac” Rf Db Sg Bh Hs Mt Ds Uuq
(223) (226) (227) (261.11) (262.114) (263.118) (262.12) (265) (266) (269)
Inner-Transition Metals

58 59 60 61 62 63 64 65 66 67 68 69 70
’Lanthanides Ce Pr Nd Pm Sm Eu Gd Tb Dy Ho Er Tm Yb
140.115 140.90765 144.24 (145) 150.36 151.965 157.25 158.92534 162.50 161.93032 167.26 168.93421 173.04

90 91 92 93 94 95 96 97 98 99 100 101 102


’’Actinides Th Pa U Np Pu Am Cm Bk Cf Es Fm Md No
232.0381 (231) 238.0289 (237) (244) (243) (247) (247) (251) (252) (257) (258) (259)

Classification of The Elements in • General electronic configuration is


Blocks [inert gas] nsx (x = l,2)
In Periodic Table, the elements have been arranged in • If x = 1, elements constitute a group of alkali n
different periods and groups based on electronic placed in first group (IA).
configuration. Classification of the elements into following
• If x = 2, elements constitute a group of alk
four blocks is decided by the differentiating electron (last
filling electron). Block is that in which orbital earth metals placed in second group (IIA).
differentiating electron goes (Aufbau rule) • Alkali metals form monovalent cations
(a) s-block element (b) p-block element alkaline earth metals form divalent cations,
(c) d-block element (d) /-block element with nearest stable configuration of inert gas.
Atomic
E.C.
Last electron
Block
• Period of the element is the value of x (num
Element
no. goes into shells).
Na 11 [Ne] 3s1 s s-block
Electronic configuration of Groi
[Ne] 3s2Sp1 Group IA
Al 13 P p-block (Sec
(First)
[Ar] 4s2 3b1 d-block alke
Sc 21 d alkali
Metal A A* or S Metal B ea
metals (A)
Ce 58 [Xe] 6s2 472 f /-block meta

Li3 [He] 2s1 [He] [He] 2s2 Be


Blocks have been represented in Fig. 3.1 and Table 3.3.
Na„ [Ne] 3s1 [Ne] [Ne] 3s2
s-Block Elements K-I9 [Ar] 4s1 [Ar] [Ar] 4s2 Ce
If shells up to (n - 1) are completely filled and the last Rb37 [Kr] 5s1 [Kr] [Kr] 5s2 Sr
electron enters the s-orbital of the outermost (nth) shell, Cs55 [Xe] 6s1 [Xe] [Xe] 6s2 Bs
the elements of this class are called s-block elements. [Rn] 7 s' [Rn] [Rn] 7s2 Re
• Thus differentiating electron enters the ns-orbital.
106 I Essential Inorganic Chemistry

Table 3.3 The s, p, d and f-Block of the Periodic Table


Groups
IA IIA 111B IVB VB VIB VIIB VIII 1B 11B IIIA IVA VA VIA VIIA 0
(1) (2) (3) (4) (5) (6) (7) (8) (9) (10) (11) (12 ) (13) (14) (15) (16) (17) (18)
s-orbital block s2
,.------,
-t p-orbital block
t
51 : 2 :

n=1 ·------·
: He:
I

s2 52p1 82p2 s2p3 s2p4 82ps s2p6

n=2 3 4 5 6 7 8 9 10
Li Be B C N 0 F Ne
d-orbital block
n=3 11
Na
12
Mg d1s2 d2s2 d3s2 cf>sl d5s2 cf>s2
t d1s2 ds52 d1081 d1082
13
Al
14
Si
15
p
16
s
17
Cl
18
Ar

,�,
n=4 19 20 21 22 23 24 25 26 27 28 29 30 31 32 33 34 35 36
Sc Zn Ga Ge


Ca (-orbital Ti V Cr Mn Fe Co Ni Cu As Se Br Kr
block
37 38 39 40 41 42 43 44 45 46 47 48 49 50 51 52 53 54
n=S Rb Sr y Zr Mo Tc Ru Rh Pd Ag Cd In Sn Sb Te I Xe
d1s1 d8s1
55 56 57 58 71 72 73 74 75 76 77 78 79 80 81 82 83 84 85 86
n=6 Cs Ba la ce_..,.Lu Hf Ta w Re Os Ir Pt Au Hg Tl Pb Bi Po At Rn
d4s2 d9s1

n = 7 87 88 89 90 103 104 105 106 107 108 109 110 111 112
Fr Ra Ac Th_..,. Lr R.f Db Sg Bh Hs Mt Ds

Lanthanide 58 59 60 61 62 63 64 65 66 67 68 69 70 71 4fsubshell
n=6
Series Ce Pr Nd Pm Sm Eu Gd Tb Dy Ho Er Tm Yb Lu being filled
...
Actinide 90 91 92 93 94 95 96 97 98 99 100 101 102 103 Sf subshell
n=7 Series Th Pa u Np Pu Am Cm Bk Cf Es Fm Md No Lr being filled

•n is the principal quantum number. The d1 s2 ,d 2s2 , ••• designations represent known configurations. They refer to(n - l)d and ,as
orbitals.

p-Block Elements Illustration 1 Group the following electro•n


If shells up to (n - 1) are completely filled and configurations in pairs that would represent simillJr
differentiating electron enters the p-orbital of the n th chemical properties of their atoms :
orbit, elements of this class are called p-block elements. (a) 1s 2s2 2p6 3s2
2

• General E.C. is (b) ls2 2s2 2p3


ns2 npx (x= 1 to 6) (c) ls2 2s2 2p6 3s2 3p6 4s2 3d10 4p6
• Group is (2 + x) A based on old system IHA, IVA, ...... (d) ls 2s22

and (12 + x) based on new IUPAC system. (e) ls2 2s2 2p6
• The s-block and p-block elements together are called ({} ls2 2s2 2p6 3s2 3p3
representative elements or main group
elements. Solution If there is repetition of electron configuratic»n
(E.C.) of the valence shell, then chemical properties are
Period Elements also repeated. Thus pairs with similar chemical properties
2 B C N 0 F Ne are:
3 Al Si p s Cl Ar
Valence shell EC
4 Ga Ge As Se Br Kr I : (a) and (d) [inert gas] ns2
5 In Sn Sb Te I Xe II : (b) and (0 ns2 np3 -nitrogen family ofp-bloclk:
6 Tl Pb Bi Po At Rn III : (c) and (e) ns2 np6 -inert gas
4 2
E.C. ns np 1 ns2np 2
2
ns 2np 3 ns 2np ns np 5
ns 2np 6
X 1 2 3 4 5 6 d-Block Elements
old IIIA IVA VA VIA VIIA VIIIA (0) If outermost (nth) and penultimate [(n - l)th] &:rE
new 13 14 15 16 17 18 incompletely filled and differentiating electron enters thE
Chapter 3: Periodicity of Elements | 107

(n -l)d orbitals (i.e., d-orbital of penultimate shell) then Illustration 3 A M2* ion. derived from a metal in the
elements of this class are called d-block elements. first transition metal series has four electrons in 3d
• d-block elements are placed between s-block and subshell. What element might M be?
p-block elements and thus also called transition
elements. Solution EC of M2+ : [Ar]18 4s° 3d4
• EC is (n -l)dz nsy EC of M : [Ar] 4s1 3d5 (and not 4s2 3df)
where x = number of electrons in (n - l)d orbital Total electrons = 24
x = 1 to 10 (except 4 and 9) Hence, M is Cr.
y = number of electrons in ns-orbital.
y = 2 in all cases except in groups 6 (VIB) and /-Block Elements
ll(IB)y = l If n, (n -1) and (n -2) are incompletely filled and last
• IUPAC (new pattern) group = (x + y) but old group is electron enters into /’-orbital of ante penultimate i.e.,
decided by (n - 2) th shell, elements of this class are called /"-block
— group is (x + y)B if x < 6 elements.
— group is VIIIB if x = 6,7,8 • Thus differentiating electron enters (n -2) /‘orbital.
— group is yB if x = 10 • General E.C. is (n -2) f2~14 (n - 1) dQ’1 ns2
Z (atomic Group • If n =6, elements belong to lanthanides with E.C.
Element EC x y
number) New Old 4/’2-145d0’16s2 of the elements with atomic number
III B
Ce (58) to Lu (71)
Sc 21 [Ar] 3d' 4s2 1 2 3
CeM : [Xeb 4/25d°6s.2 Gdg4 : [Xek 4/‘75d16s2
Ti 22 [Ar] 3d2 4s2 2 2 4 IV B
Lu71 : [Xe^ 4/'u5d16s2
V 23 [Ar] 3d3 4s2 3 2 5 VB
Cr 24 [Ar] 3d5 4s1 5 1 6 VI B
• If n =7, elements belong to actinides with E.C.
5f2~146dQ,17si of the elements with atomic number
Mn 25 [Ar] 3d5 4s2 5 2 7 VII B
Th (90) to Lr (103).
Fe 26 [Ar] 3d6 4s2 6 2 8 VIII B Th 90 :[Rn]86 5/26d°7s2
Co 27 [Ar] 3d7 4s2 7 2 9 VIII B Cm 96 :[Rn]86 bf^ls2
Ni 28 [Ar] 3d8 4s2 8 2 10 VIII B 1^103 :[Rn]86 5fi46dlls2

Cu 29 [Ar] 3d’° 4s1 10 1 11 IB • All the elements of/‘-block (4/series—Lanthanides and


Zn 30 [Ar] 3d10 4s2 10 2 12 II B
5f series—Actinides) are placed in 3rd group (IIIB) and
form a separate block at the bottom of the Periodic
• d-block elements are further classified into following Table.
three transition series on the basis of which (n - l)d is
• They are also called inner-transition elements.
being filled:
Series Elements (n-1)d being filled DEEP Focus
3d SC21—Zriao 3d • They were also called rare-earth elements since their
Ad Y39—Cd^ Ad
oxides (earth) were rare in earlier days.
5d Las?. Hf72—Hggo 5d
Nomenclature of the Elements
Illustration 2 Specify the group of the Periodic Table Elements with atomic numbers greater than 92
in which each of the following elements is found? (atomic number of uranium) are called the transuranium
(a) [Ne] 3s1 (b) [Ne] 3s2 3 p3 (c) [Ne] 3s2 3 pe elements. All these elements are man-made through
(d) [Al] 4s2 3d8 artificial nuclear reactions.
---- > 2992U + y
Solution
239u _> 23993Np+_?e
EC Group Element
ilNp --- > 2gPu
(a) [Ne]3s1 IA(1) Na
• At present, transuranium elements with atomic
(b) [Ne]3s2 3p,3 VA(15) P numbers up to 110 have been reported.
(c) [Ne]3s23p6 VIIIA (18) Ar • Very recently, on August 16,2003, IUPAC approved
(d) [Ne] 4s2 3d8 VIIIB (10) Ni the name for the element of atomic number 110, as
darmstadtium, with symbol Ds.
108 I Essential Inorganic Chemistry

• CNIC (Commission on Nomenclature of Table 3.4 IUPAC Nomenclature of Elements with Atomic
Inorganic Chemistry) appointed by IUPAC in 1994, Number above 100
approved official names for elements with atomic
Atomic IUPAC IUPAC
number 104 to 109. With Z > 109, IUPAC number
Name Symbol official name symbol
recommended a nomenclature to be followed for
naming these elements until their names are officially 101 Un-nil-unnium Unu Mendelevium Md
recognised. This nomenclature is based on the Latin 102 Un-nil-bium Unb Nobelium No
words for the atomic numbers of Hua elements. 103 Un-nil-trium Unt Lawrencium Lr
• The names are derived by using root.c: for the t.l::·ne 104 Un-nil-quadium Unq Rutherfordium Rf
digits in the atomic number of the element and adding 105 Un-nil-pentium Unp Dubnium Db
the ending -ium. The roots for the numbers are 106 Un-nil-hexium Unh Seaborgium Sg
Digit Name Abbreviation 107 Un-nil-septium Uns Bohrium Bh
108 Un-nil-octium Uno Hassnium Hs
0 nil n
109 Un-nil-ennium Une Meitnerium Mt
1 un u
110 Un-un-nillium Uun Darmstadtium 0

2 bi b 111 Un-un-unnium Uuu ••


3 lri 112 Un-un-bium Uub ••
4 quad q 113 Un-un-lrium Uut a
114 Un-un-quadium Uuq ••
5 pent p
115 Un-un-pentium Uup a
6 hex h
116 Un-un-hexium Uuh •
7 sept s
117 Un-un-septium Uus •
8 oct 0 118 Un-un-octium Uuo •
9 enn e
* Elements yet to be discovered,
Thus element with atomic number 109 will be named * * IUPAC names yet to be announced.
as une (u for l, n for 0 and e for 9). * *
* IUPAC approved the name on August 16, 2003.
Table 3.4 summarises the names of the elements with a Discovered in 2003.
atomic number above 100.

Target Practice 1
I. What are the blocks of the following? 4. What are atomic numbers of the following elements?
(a) Sc3 • (18e) .•.••.••••.. (a) Uut ......... (b) Uuo ....... ..
+
(b) Na (lOe) ............ (c) Unt .........
+
(c) Al 3 (10e) ............ 5. What are symbols for the following having atomic
2. Lanthanides (58- 71) have ......... elements. n umbers?
3. In the absence of Aufbau rule, what are the blocks of (a) 117 ......... (b) 115 ....... ..
(a) K( 19) ......... (b) Sc (21) ......... (c) 104 •.•..•.••
(c) Ca(20) .........

Answers
1. Blocks are based on atomic number and not on the number of (b) Sc (21) l s 2 2s 22p 63s 2 3p 6 3d3 d-block
electrons. Thus, (c) Ca(20) ls 2 2s 22p 6 3s 23p 6 3d2 d-block
(a) Sc3 .. (18e, z = 2l)[Ar] 4s 2 3d1 d-block 4. (a) Uut 113
(b) Na + (lOe, z = ll)[Ne] 3s 1 s -block iii
3+ 2 p1
(c) Al (10e, z = 13)[Ne]3s 3 p-block 113
.2. 14 (b) Uuo 118
3. If Aufbau rule is not followed, electrons are filled in the (c) Unt 103
available orbitals. 5. (a) 117 uus (b) 115 uup (c) 104 uuq
(a) K.(19) 1 s 2s 2 p 3s 3p 3d
6 1
2 2 6 2
d-block
Chapter 3: Periodicity of Elements | 109

Practice Exercise 1
1. If each orbital can take maximum of three electrons 14. With reference to the Periodic Table, name
(elements), what is the number of elements in (a) a halogen element in the fourth period
(a) 2nd period, (b) 4th period?
(b) an element similar to phosphorus in chemical
2. By use of electron-configuration, decide the group and properties
period of the following elements with atomic number (c) the most reactive metal in the fifth period
13, 35, 56, 78, 65
(d) an element that has an atomic number smaller than 20
3. (a) If Aufbau rule is not taken into account, determine and is similar to strontium.
the number of elements in each period.
(b) Assume total number of 112 elements, then how many 15. Elements A, B, C, D and E have the following electron
periods would be required in the Periodic Table configurations:
without separate position of lanthanides and A:ls2,2s22p1 B:ls2,2s22p6,3s23p1
actinides? C : Is2, 2s2 2p6,3s2 3p3 D : Is2,2s2 2p6,3s2 3p5
E : Is2, 2s2 2p6, 3s2 3p6, 4s 2
4. What are the groups and periods of the elements with
atomic numbers 37 and 56? Which among these will belong to the same group in the
5. Without writing EC can you decide whether the element is Periodic Table?
alkali metal or alkaline earth metals of the given two types 16. By definition, d-block elements are those in which n and
of elements. (n -1) shells are incomplete and last electron is being filled
6. Without using Aufbau rule which of the following elements into (n- l)d-orbitals. For
would be placed in s-block? Zn(30) [Ar]i83d10 4s2
Z = 19,38,29 Cd(48) [Kr]36 4</10 5s2
7. If it can happen that each orbital can accommodate Hg(80) [Xe].M 4f14 5d10 6s2
3 electrons (for example EC of Na is ls32s32p5), what is You find, (n -IJd is completely filled. Are their positions in
block of the elements with Z =12,19,37 when Aufbau rule d-block justified.
(i) is also valid, (ii) is not valid? 17. What are the blocks, groups and period of the following
8. What is the most important relationship among elements in iso-electronic ions/species ?
the same group in the Periodic Table? (a) Na+, Mg2+, Al3+ (b) O2-, Na+, Ne
9. What would be the atomic number of the (c) Fe2*,Mn+,Cr
as-yet-undiscovered alkaline earth element of period 8? 18. If Aufbau rule is not followed, what are the blocks of the
10. What are the blocks if EC in the excited states are : following elements ?
(A): Is2, 2s22p6,3s13p33d2 (B): Is2,2s12p1 (a)K(19) (b) Fe (26)
(C): Is2, 2s12p3 (c) Ga (31) (d) Sn (50)
11. The p-block elements show two types of oxidation state due to 19. A metal ion with +3 charge has five electrons in the
inert pair effect 3d-subshell. Identify the metal.
n (valence electrons) 20. EC of Gd (64) is written as:
(n-2) [XeJ» 4/‘75dflt6s2
13 14 15
Also stability of nth state and not as [Xeta 4f8 6s2. Explain.
decreases and that of (n-2)th Ga Ge As
In Sn Sb 21. Last electron in Lu(71) goes into 5d, but it is studied in
increases as one goes down the
group. What are different Tl Pb Bi /■-block. Explain.
oxidation states of groups 13,14, 22. Match the block of the elements with their atomic number:
15 starting from 4th period? z Block
Also write order of their stabilities.
55 P
12. Following are the p-block elements:
45 f
Ge -As Se
81 s
32 33 34
64 d
If each orbital can take maximum of three electrons and in
the absence of Aufbau rule, specify the 23. Write EC of Cm (96). What is its
(i) block (a) group (b) period
(ii) group (c) block and (d) magnetic moment?
(iii) period of the above elements. 24. Identify the group, family, and/or other Periodic Table
13. A neutral atom of a certain element has 15 electrons. location of each element with the outer electron
Without consulting a Periodic Table, answer the following configuration:
questions: (a) ns2 np3
(a) What is ground state electron-configuration? (b) ns1
(b) How should the element be classified? (c) ns2 (n-l)d0-2 (n-2)/*1’14
(c) Are the atoms of the element diamagnetic or (d) ns2 (n -1) d1'10
paramagnetic.
110 | Essential Inorganic Chemistry

Answers
i. (a) Total orbitals available in 2nd period : 10. (A) p-block (B) s-block (C) p-block
one in 2s 11. Ga, In, T1 +3, +1 Ga* < In* < Tl*
= 4 hence, number of elements = 12
three in 2p Ge, Sn, Pb +4, +2 Ge2* < Sn2* < Pb2*
As, Sb, Bi +5, +3 As3* < Sb3* < Bi3*,Pb 2* > Pb4*,
(b) Total orbitals available in 4th period:
one in 4s Sn2*<Sn4*
five in 3d = 9 hence, number of elements = 27 12. Element (EC) Block Group Period
three in 4p
Ge^ 1s3 2s3 2 p9 3s3 3 p9 3d5 d 17 3
2. Z Group Period Asjb 1s32s32p93s33p93d6 d 18 3
13 13 (IHA) 3 1s32s32p93s33p93d7 d 19 3
35 17 (VII A) 4
13. (a) Is2 2s2 2p6 3s2 3p3 (b) p-block, group VA (15), period 3
56 2 (HA) 6 (c) paramagnetic since there are (three) unpaired electrons
78 10 (VIIIB) 6 14. (a) Br (b) N (C)Rb (d)Mg 15. A and B
65 3 (IIIB) 6 16. Zn, Cd, Hg, with (n - 1) d10 ns2 electronic configuration can’t
be considered in d-block due to completely filled (n - l)d
3. (a) Each group would be having number of elements = 2n2, orbital. Although these elements form +2 cations, they do not
where n stands for number of period. have much in common with Be, Mg and Ca to Ra group except
for some resemblance between Zn, Mg and Be.
n Elements 2(IIA) group elements form +2 cation with inert gais
1 2 configuration while 12(IIB) group elements form +2 cation with
(n - l>d10 configuration which makes study of these elements
2 8
in d-block.
3 18 17. It is atomic number and not the number of electrons that decide
4 32 position in the Periodic Table.
5 50
(a) lons/species Group Period Block
6 72
7 98 Na* KIA) 3 s
Mg2* 2(HA) 3 s
(b) Total number of six periods, sixth period having only 2 Al3* 13(IHA) 3 P
elements [based on part (a)].
O2‘ 16 (VIA) 2
(b) P
4. Z Period Group
Na* 1 (HA) 3 s
37 4th IA(1)
Ne 18 (VIIIA) 2 P
56 5th HA (2)

5. Yes : Atomic number of alkali metals is not divisible by 2 while (c) Fe2* 8 (VIIIB) 4 d
that of alkaline earth metals, it is divisible by 2. Mn* 7 (VIIB) 4 d
e.g., Na — 11 • Or 6 (VIB) 4 d
Mg — 12
18. In the absence of Aufbau rule, (EC) and last filling electron
6. Z ___________(EC) Block decides the block
(i) K19 ls22s22p63s23p63d1
19 1s22s22p63s23p63d1 cf-block
(ii) Fe26 ls22s22p63s23p63d8
38 1s22s22p63s23p63d10 d-block
(iii) Ga31 ls22s22p63s23p83d104s24p1
4s24p64d2
(iv) Sn60 ls22s22p63s23p63d104s24p64d104/,4
29 1s22s22p63s23p63d104s1 s-block
Changed block Original block
7. Atomic (EC)
no. (i) d s
Aufbau rule is
valid
Block Aufbau rule Is not Block (ii) d d
valid
(ill) P P
12 1s32s32p6 P 1s32s32p6 P
(iv) f P
19 1s32s32p93s33p1 P 1s32s32p93s33p1 P
37 1s32s32p93s33p9 d 1s32s32p93s3 d Thus, block would change in case of K and Sn if EC is writter
4s33d7 3p93d10 without Aufbau rule.
19. Fe(Z = 26) : [Ar]18 4s2,3d6
8. They have similar valence shell E.C. Fe3* : [Ar]18 4s°,3d6
9. Atomic number = 120
Chapter 3: Periodicity of Elements | 111

20. In this EC all the electrons in 4/7 are unpaired and thus is
stable. IRnlss |T|T|T|T|T|T|f] |T| | | I I 0
5f7 6c/1 7s2

4/7 (a) Group = 3 (I1IB)


(b) Period = 7th
21. Lu71 is studied in /-block based on its properties. (c) Block =f
22. (d) Magnetic moment = jMN + 2) BM = v80 BM
Z Block
N = number of unpaired electrons
55 s 24. Group Family
45 d (a) VA(15) nitrogen
81 P (b) IA (1) alkali
64 f (c) IIIB (3) inner-transition metals
(d) - transition metals
23. Cm(96)
EC[Rn]86 5/7 6c/1 7s2

Periodicity • Within a periodic group the physical properties vary


The recurrence of similar properties of elements after more predictably, especially if the elements are in
certain regular intervals when they are arranged in the same physical state. For example, the m.p. of Ar and
order of increasing atomic number is called periodicity. Xe are -189.2°C and -111.9°C, respectively. We can
• Periodicity is due to recurrence of similar outer estimate the m.p. of the intermediate element (which
electronic configurations of their atoms at certain is Kr) by taking the average of these two values as
regular intervals. (Table 3.5) follows :
• Recurrence of properties is after 2, 8, 8,18,18 and 32 (-189.2)+ (-111.9)
m.p. of Kr =----------- ------------
elements (these number are called magic numbers).
=-150.55° C
Table 3.5 E.C. of Elements of Group 1,17 and 18
This value is quite close to the actual m.p. of-156.6°C.
1 (Alkali metals) 17 (Halogen family) 18 (Noble gases)
Li [He] 2s1 F :1s22s22p5 He: 1s2
Na [Ne] 3s1 Cl:... 3s2 3p5 Ne:2s22p6 Valency and Oxidation tabs?
K [Ar] 4s1 Br:... 4s24p5 Ar:... 3s2 3p6
Valency and oxidation number of the elements are
Rb [Kr]5s1 I :... 5s25p5 Kr:... 4s24p6
determined along a group and period.
Cs [Xe] 6s' At;... 6s26p5 Xe:... 5s25p6
Fr [Rn]7s1 Rn:... 6s2 6p6
Variation in a Group
From left to right across a period there is a transition • For a group, valency is constant.
from metals to metalloids to non-metals. Consider the
• Valency = N if N < 4
third-period elements from Na to Ar.
= (N-S) if N>4
where N is the number of valence electrons.
DEEP Focus
• Sodium, the first element in the third period, is a very 1 2 13 14 15 16 17 18
reactive metal, whereas chlorine, the second-to-last (IA) (HA) (IDA) (IVA) (VA) (VIA) (VISA) (VIIIA)
element of that period, is a very reactive non-metal. In
Li Be B C N 0 F Ne
between, the elements show a gradual transition from
metallic properties to non-metallic properties. Z 3 4 5 6 7 8 9 10
• Na, Mg and Al all have extensive three-dimensional atomic
networks, which are held together by forces characteristic N 1 2 3 4 5 6 7 8 '
of the metallic state. valency 1 2 3 4 -3 -2 0
• Si is a metalloid; it has a giant three-dimensional structure -4
in which the Si atoms are held together very strongly.
Starting with phosphorus, the elements exist in simple,
discrete molecular units (P4, S8, Cl2 and Ar) which have low InC02, C is +4 and in CHj, C is -4.
melting points (m.p.) and boiling points (b.p.).
112 | Essential Inorganic Chemistry

Due to Inert-pair effect (refer p-block elements), Variation in Period


p-block elements in 4th and higher periods have two types • Valency, w.r.t. O(= 2), increases 1 to 7 in a period left
of oxidation states/valencies, N and (N -2). Stability of to right
higher Nth state decreases and that of lower (N - 2)th
state increases going down the group. Na Mg Al Si P ’ s ci
Oxidation Compounds Na2O MgO AI2O3 SiO2 p2o5 so3 Cl2o7
Group Element N Stability order with oxygen
state
Valency 1 2 3 4 5 6 7
13 (IIIA) Ga. In. Tl 3 +1, +3 3* > Tl,3+
Ga3* > In>3*
Ga* < In* < Tl*
Ga3* > Ga*
• Valency, w.r.t. H (= 1), increases 1 to 4 and then falls
Tl3* < Tl*
to 1.
14 (IVA) Ge. Sn, Pb 4 +2. +4 Ge4* > Sn4* > Pb4* Li Be B C N O F
Ge2* < Sn2* < Pb2* Compounds with LiH BeH2 BH3 ch4 nh3 h2o HF
Sn4* > Sn2* hydrogen
Pb4* < Pb2* Valency (numerical 1 2 3 4 3 2 1
15 (VA) As. Sb. Bi 5 +3. +5 As5* > Sb5* > Bi5* value)
As3* < Sb3* < Bi3*
As5* > As3* • Inert gases [group 18 (VIIIA)], have zero valency.
Bi5*< Bi3* Oxidation states/valencies of the elements have beer
given in Fig. 3.2.

(1) (17) (18)


VIIA 0
IA
1 1 2
H H He
+1 +1
(2) (13) (14) (15) (16) -1
IIA IIIA IVA VA VIA

3 4 5 6 7 8 9 10
Li Ba B C N O F Ne
+1 +2 +3 +4 +5 -1 -1
+2 +4 -2
+3
+2
+1
-3

11 12 13 14 15 16 17 18
Na Mg Al Si P S Cl Ar
+1 +2 +3 +4 +5 +6 +7
+3 +4 +5
-3 -2 +3
+1
(7) (8) (9) (10) -1
(3) (4) (5) (6) (11) (12)
IIIB IVB VB VIB VIIB Groups VIIIB IB IIB
19 20 21 22 23 24 25 26 27 28 29 30 31 32 33 34 35 36
K Ca Sc Ti V Cr Mn Fe Co Ni Cu Zn Ga Ge As Se Br Kr
+1 +2 +3 +4 +5 +6 +7 +3 +3 +2 +2 +2 +3 +4 +5 +6 +7 +4
+3 +4 +3 +6 +2 +2 +1 +3 +4 +5 +2
+2 +3 +2 +4 -3 -2 +3
+2 +3 +1
+2 -1

Fig. 3.2 Some non-zero oxidation states (number)


Chapter 3: Periodicity of Elements | 1

DEEP Focus -For H-atom (Z = 1), there is no screening, bei


one-electron system. Thus
Based on stability
o=0 and Z* = Z = 1
Sn2+ ---- > Sn4+ +2e“ AG°<0
Pb2+ ---- > Pb4++2e" AG°>0 - For He-atom (Z = 2), Is2, there is screening by o
electron on the other electron.
Tl3++2e’ ---- > Tl+ AG°<0
Bis+ + 2e’ ---- > Bi3+ AG°<0 o = 0B0 (of Is electron)
(Note : If AG° is negative, reaction is spontaneous) Z* = 2-0.30= 1.70
— For Li-atom, there is screening of outermost electr
by 2 electrons in 1st orbit, (n - 1).
Shielding (Screening) Effect and Thus o for two electrons in (n - 1) = 2 x 0.85 = 1.70
Effective Nuclear Charge Z * = 3-1.70= 1.3
A valence-electron in a multi-electron atom is
attracted by the nucleus and repelled by the electrons of
inner-shells. The combined effect of this attractive and DEEP Focus
repulsive force acting on the valence-electron is that the • In all cases, screening by (Z -1) electrons is to
valence-electron experiences less attraction from the considered.
nucleus. This is called shielding or screening effect. • One electron system will have <5=0 and thus Z * = Z.
Thus in case of H, He+, Li2+, Be3+... with one electron Z*
DEEP Focus atomic number.
• According to J.C. Slater, effective nuclear charge Z* due to
screening is not the actual charge Z of the nucleus ofthe atom, • In a Period
but depends on the type of orbital in which the electron is
housed, and on the ability of the other electrons in more Value of a for (Z -1)
electrons in
penetrating orbitals to screen (or shield) the electron in Element Z Z=Z-c
question from the nucleus.
1st orbit 2nd orbit Total
• The relative extent to which the various orbitals penetrate
the electron clouds of other orbitals is s > p >d > f. Thus, for Li 3 2 x 0.85 1.70 1.30
any given principal quantum number n, an electron will
Be 4 2 x 0.85 1x0.35 2.05 1.95
experience the greatest effective nuclear charge when in an
s-orbital, than a p-orbital and so on. B 5 2 x 0.85 2 x 0.35 2.40 2.60
• The electron is more stable in the more penetrating orbital. C 6 2 x 0.85 3 x 0.35 2.75 3.25
N 7 2 x 0.85 4 x 0.35 3.10 3.90
Effective nuclear charge Z* due to screening is given
O 8 2 x 0.85 5 x 0.35 3.45 4.55
by
F 9 2x0.85 6 x 0.35 3.80 5.20
Z* = Z-O
Ne 10 2 x 0.85 7 x 0.35 4.15 5.85
where Z is the atomic number and c Slater's screening
constant. Values of a are given below for various types of Thus, in a period (left to right), value of Z * increas
orbitals. In all cases 0 < ct < Z. increment being of 0.65 per element.

o per electron of orbit


Electrons in orbitals 1 DEEP Focus
(of shell) -> (n- 2), (n- 3)...
n (n-1) For second period with n valence electron
s-or p-orbital 0.35 0.85 1.00 Z* = 0.6511+ n]
b-or Aorbital 0.35 1.00‘ 1.00 For third period with n valences electron
*for all groups on left, including s or p-electrons of the same n. Z* = 1.55 + 0.65n
• For Is electron, o =0.30 (in case of two-electron • For higher periods with d- and /‘-orbitals, variation m
differ.
system)
114 I Essential Inorganic Chemistry

• n
I a Group
--- -- - __ ___
__.

Value of a for (Z- !)electrons In orbit


Element z n Total z·= Z-cs
n (n-1) (n-2) (n- 3) (n- 4) (n-5) (n- 6)
u 3 2 0 1.70 1.70 1.30
Na 11 3 0 6.80 2.00 8.8 2.20
K 19 4 0 6 80 8.00 2.00 16.8 2.20
Rb 37 5 0 6.80 8.00 18.00 2.00 34.8 2.20
Cs 55 6 0 6.80 18.00 18.00 8 00 2.00 52.8 2.20
Fr 87 7 0 6.80 32.00 18.00 18.00 8.00 2.00 84.8 2.20

• Electron in nth orbit is screened by electrons in For Kr, Z = 36, [Ar] 4s2 3d 10 4 p6 , electron of highest
inner-shells. hence o = 0 for electrons in nth orbit in energy is in 4p, n = 4
the above case. ,--
z = 11 (1-35-0-.7-) (-16-) =4.06
elf V
1312
DEEP Focus
• \-.·e fi11d omazinf! c/i'"cctil:e nuclear charge ,n a group. E.,·cept
for Li (211d period) all other elements hatie constant ualue of Size of an Atom and Ion
Z"' f2.2 in ,:ase ol alkali metals). Thus while studying Atomic radius is taken as the distance from the centre
variation of any propert�· in a group, not Z but principal of the nucleus to the outermost shell of the electrons.
quantum number, n will be the deciding factor. As we shall • Based on probability concept, an atom does not have
see in the coming sections, variation of radius in a group well defined boundary. Probability of finding the
electron is never zero even at large distance from the
(a = Bohr's radius)
0 nucleus. Hence, exact value of the atomic radius can't
\ill be decid�dby n and not by Z (that wil1 be actually Z* and be evaluated. Atomic radius is taken as the effective
size which is the distance of the closest approach of
vh1ch we ha\'e seen is constant).
one atom to another atom in a given bonding
Thus while studying periodicity in a group, do situation.
not consider effect of Z. • Atomic radius can be
• One way to illustrate electron shielding is to consider - Covalent radius : It is one-half of the distance
the amount of energy required to remove the two between the centres of two nuclei (of like atoms)
electrons from the helium atom. bonded by a single covalent bond.
For atoms known to form single bonds in their
He (g) �He (g) + e- E =2373 kJ moi-1
+
elemental state, the experimental bond length may be
He + (g) � He2 + (g) + e- E = 5231 kJ mo1- 1 halved to obtain the respective covalent radius. For
The reason it takes so much more energy to remove example
the second electron is that with only one electron present, Element Bond length (pm) Atomic radius (pm)
there is no shielding, and the electron feels the full effect of
74 37*
the +2 nuclear charge. H2
F2 142 71...
Illustration 1 One method for calculating Zerr is to · Cl 2 199 99
use the equation : C (diamond) 154 77
Z =" (E)(n2 ) "Accepted values are H: 29 pm: F: 64 pm.
elf
V 1312 kJ / mol Atomic radii of elements which do not form single
where E is the energy necessary to remove an electron bonds (in their elemental form) may be calculated from
from an atom and n is the principal quantum number of observed single bond lengths in compounds with other
the electron. Use this equation to calculate Zerr values for elements of known atomic radii.
The set of covalent radii obtained in the manner fit
the highest-energy electron in potassium
experimental bond lengths quite satisfactorily, as may be
(E = 418.BkJ mol- 1) and krypton (E = 1350.7 kJ moZ-1). observed from following table for the halides of carbon,
silicon and germanium. The experimental bond lengths
Solution For K, Z = 19 ls2 , 2s2 2p 6 , 3s2 3 p6 , 4s1 (in pm) given in parenthesis show fair agreement (without
4s 1 is the electron of highest energy, n == 4 ten per cent) except some fluorides. The atomic radius or
fluorine has been taken as 64 pm because this value fits
.8) (4 ) =2.26
zea = VJ(4181a12
2
more closely with experimental data on a large number of
fluorine compounds. The correction is empirical in nature.
Chapter 3: Periodicity of Elements | 115

Similarly, the radius of the hydrogen atom is taken to be to the atoms. Thus the carbon-carbon bond length of
29 pm. These adjusted values for some elements are 135 pm in ethylene leads to a double bond radius of carbon
shown below: equal to 67 pm. Some typical values are given below.
Molecule M=C M=SI M=Ge
B C N 0
MF4 141 (148) 181 (154) 186(167) Double bond radius 71 67 62 62
MCI4 176(176) 216(201) 221 (208) Triple bond radius 64 60 55
MBr4 ' 191(191) 231 (215) 236 (231)
Ml4 210(211) 250 (243) 255 (250) The periodic variation in covalent radii of atoms
follows the same trend as the atomic volume. The radii
decrease from left to right along a period with increase in
Covalent atomic radii (single bond), pm nuclear charge. The size should also increase downward
H 29 Be 89 B 80 C 77 N 70 OO 6666 FF 64 along any group with increasing principal quantum
Al 126 Si 117 P 110 104 ClCl 99
SS 104 number of the outermost shell.
Zn 131 Ga 126 Ge 122 As 121 Se 117 Br 114 - van der Waais’ radius : It is one-half of the
intemucleus distance between two adjacent atoms in
Cd 148 In 144 Sn 140 Sb 141 Te 137 1 113
two nearest neighbouring molecules of the substance
Hg 148 Tl 147 Pb 146 Bi 151 in solid state.
This is obtained from the shortest distance to which
In a heteronuclear, diatomic molecule of AB type, if
two non-bonded atoms can approach before repulsion
the bonding is purely covalent, then the bond length rA _ B
between their electron clouds come into play.
is equal to the sum of the covalent radii of A and B, i.e.,
X X X
rA-B=rA+rB
Thus, covalent radii are additive. Thus known the
value of rA_B and rA (or rB), value of rB (or rA) can be A B C
calculated. If there is difference in the electronegativities
of the bonded atoms, it causes shortening of the bonds.
Schomaker and Stevenson gave following relation to
calculate intemuclear distance of the molecule AB with
different electronegativity XA and XB of atom A and B 2 AB = /"covalent 2 CD - rcrystal
respectively. (of elementX) (of element X)
rA - B = rA + rB - °-07(XA - XB^
H H X

DEEP Focus E : F
Gr
Multiplicity of the bond also causes a shortening of the
bond. Usually a double bond is about 0.86 times and a triple
• bond about 0.78 times the single bond length for the second
period elements.
1
One important contribution is likely to come from the 29 EF = rvan dec Waais’ of hydrogen in HZ molecule
attraction between the opposite partial charge built up on
the atoms due to their electronegativity difference. The | GH = rvan der Waais’ of * m HX molecule.
gradual decrease in C—F bond length with increasing
substitution by fluorine in the following compounds ^covalent < ^crystal < ^van der Waais’
supports this view. - Crystal radius: It is one-half of the distance between
Compound C—F distance (pm) the nuclei of two adjacent metal atoms in the metallic
closed packed crystal lattice.
ch3f 139.1
ch2f2 135.8
• Atomic radius/ionic radius in the nth orbit is given by
2
71 Gq
chf3 133.2 rn =
cf4 132
y*-
where n is principal quantum number (i.e., number of
When two atoms are joined by multiple bonds, the shell), a0 the Bohr's radius of H-atom (= 0.529A) and Z*,
bond length is shorter than the sum of their single bond the effective nuclear charge.
covalent radii. From the observed bond-lengths, it is Radius is also dependent on the extent of force of
possible to assign “double bond radii” or “triple bond radii” attraction which pulls outer shell inward.
116 I Essential Inorganic Chemistry

Variation in a Period Variation in a Group


LI Be B C N O F Ne We consider 1st group (alkali metals) elements:
3 4 5 6 7 8 9 10 Element Z r n Radius (pm)
1.30 1.95 2.60 3.25 3.90 4.55 5.20 5.85 Li 3 1.3 2 123
n 2 2 2 2 2 2 2 2 Na 11 2.2 3 157
rn (pm) 123 90 80 77 75 74 72 160 � (van der K 19 2.2 4 203
(covalent) Waals') Rb 37 2.2 5 216
Cs 55 2.2 6 235
In a period left to right :
Fr 87 2.2 7
- Z (atomic number) increases (by one unit) increases
increases constant increases
- Z* (effective nuclear charge due to screening) also
increases (but by 0.65 unit)
- n (number of shelJ�) remains constant DEEP Focus
- Electrons are pulled close to the nucleus by the In a group, top to bottom,
increased Z*.
- Z increases
Thus, r oc- - but Z* (effective nuclear charge due to screening) almost
n Z* remains constant
DEEP Focus - n increases
Sincer oc �. hence rn cc n2 (Z* remaining constant) thus
Thus atomic radii decrease with increase in atomic number n z•
in a period left to right. In case of Noble gases (as in Ne) there is r11 increases moving along a group top to bottom.
no covalent bond formation, hence, only van der Waals' radius is
considered. Thus there is high jump in the value of radius from F Fig. 3.4 gives the variation of covalent radii of
(72 pm) to Ne (160 pm). the elements (H to K). In the case of the noble gases,
On going across the third period from Na to Ar, for van der Waals, radii have been plotted.
example, each additional electron adds to the same shell
(from3s1 for Na to3s2 3 p6 for Ar). Because electrons in the
same shell are at approximately the same distance from 200
t
the nucleus, they are relatively ineffective at shielding one
another. At the same time, though, the nuclear charge Z '?150 20
increases from +11 for Na to +18 for Ar. Thus, the effective
nuclear charge for the valence-shell electrons increases

I8
.'.: 100
across the period from 2.20 for Na to 6.75 for Ar, drawing
all the valence-shell electrons closer to the nucleus and
shrinking the atomic radii (Fig. 3.3). 50
200 �--..----r--�---,�---.......--...--.------.. 8

180 l---+�-+---+--+----+---+--.......,,,..._-1----1 7 Atomic Number (Z) --+-

e.e 1 so 1-----t-------+--+----+--+---,,,......-+----1 6 Fig. 3.4 Covalent radii of first 20 elements .

� 140 5 t
� J Ionic Radius
i 120 l-----+--+----l-.�-+---+--1--1----1 4 The effective distance from the nucleus of the ion up to
� which it has an influence in the ionic bond is called ionic
radius.
80 '----.X-___.____._____.____.____.__......____, 2 • Metal ions {positive ions) are smaller than the
Na Mg Al Si P S Cl Ar atoms from which they were formed due to two
11 12 13 14 15 16 17 18 factors:
Atomic number _.,. (i) The whole of the outer shell of electrons is usually
Fig. 3.3 A plot of both atomic radius and 4ft for the removed.
valence-shell electrons versus atomic number. As Zett (ii) In an atom, the number of positive charges on the
increases, the valence-shell electrons are attracted more nucleus is exactly the same as the number of orbital
strongly to the nucleus, and the atomic radius therefore electrons.
decreases.
Chapter 3: Periodicity of Elements I 117
• In the main groups, radii increase on descending the
Na)))
group because extra shell is added.
28 1
Li +
281 +
Na K+
When a positive ion is formed, the number of positive 76pm 102pm 138pm
charges on the nucleus exceeds the number of orbital • The ionic radii decrease moving from left to right
electrons, and the effective nuclear charge is increased. across any period in the Periodic Table.
This results in the remaining electrons being more
Mg2 + Ala+
+
Na
strongly attracted by the nucleus. These electrons are
102pm 72pm 53.5 pm
pulled in further reducing the size. A positive ion is thus
always smaller than the corresponding atom and
the more electrons which are removed (that is
DEEP Focus
The d and {-orbital do not shield the nuclear charge very
greater the charge on the ion), the smaller the ion
effectively. Thus there is significant reduction in the size of the
becomes.
ions just after d or {-orbitals have been filled completely. This is
Thus, Mg>Mg>Mg2 • called lanthanide contraction (refer d-block elements). Zr
Fe> Fe2 + >Fe3 + (160 pm) and Hf (159 pm) thus have equal size due to
lanthanide contraction.
• The negative ion is always larger than that of
the corresponding atom. This is due to
(i) negative ion is formed by gain of one or more
electrons in the neutral atom and thus number of Ionisation Energy
electrons increases but magnitude of nuclear • Ionisation energy (IE), sometimes also called
charge remains the same. ionisation potential (IP), of an element is defined
(ii) nuclear charge per electron is thus reduced and as the amount of energy required to remove an
the electron cloud is held less tightly by the electron from an isolated gaseous atom of that element
nucleus resulting in the expansion of the resulting in the formation of positive ion.
outer-shell. Thus size ofnegative ion is increased. M(g) (IE) > M (g) + e-
er- >o- >0,
+

F->F, 1->I> 1 +
• (IE) is thus a direct measure of the ease with which an
• The expansion, that occurs when a group 17 (VIIA, atom can change into cation. The smaller the
halogen) atom gains an electron to form an anion, can ionisation energy, the easier it is for the neutral atom
be explained by the change in the quantum number of to change into a positive ion.
the valence shell since the added electron simply • (IE) 1, (IE)2 , (IE) 3... are respectively first, second, third,
completes an already occupiedp-subshell [Ne]: 3s2 3 p5 ... ionisation energies required to remove first, second,
for a Cl atom to [Ne] 3s2 3p6 for Cl- anion. Thus, the third, ... electron from the isolated gaseous atoms.
expansion is due entirely to a decrease in effective (
M(g) [Eh> M + (g)+e­
nuclear charge and to an increase in electron-electron
repulsion that occurs when an extra electron is added. (I
E)
M + (g) -----4 M2 + (g)+e-
• !so-electronic species (containing equal number of
electrons) will have different size due to different M2+(g) (JE)3 > Ma+ (g) + e-
values of Z. Thus, N 3-, er-,
F-, Na + , Mg2+, Al3+, Si4 +
Thus, second ionisation energy, (IE°).i, causes
(each having lOe) are iso-electronic but their radii
ionisation of M + (g) to form M2 + (g) and third
differ and oc _!_ ionisation energy, (IE)a causes ionisation of M 2 •(g) to
form M 3+ (g) and so on ...
z Electron eJZ • (IE)3 is also called first ionisation energy of M 2+ (g)
N3
-
7 10 1.43 and second ionisation energy of M + (g). (IE� is also
called first ionisation energy of M +(g).
02- 8 10 1.25
1.11
• Han electron has been removed from an atom, it
F- 9 10
becomes increasingly difficult to remove the
11 10 0.91
second and subsequent electrons from the
Na•
Mg2+ 12 10 0.83 resulting positively charged ions on account of
Al 3+
13 10 0.77 electrostatic attraction. This is due to the fact
Si 4+
14 10 0.71 that after the removal of an electron, the
number of electrons decreases while the
Thus, greater the value of (e/Z), greater the size of nuclear charge remains the same.
iso-electronic species. Consequently, the remaining electrons are held
118 | Essentia/ Inorganic Chemistry

These electrons shield the outer


more tightly by the nucleus and it becomes electrons from the nucleus
difficult to remove second electron. Similarly it is •
further difficult to remove electron from Af2+(g).Thus
(IE\< (IE)2< (IE)3< ...
Note : 1 eV= 1.602 x 10’19 J, 1 J= 6.242x 1018 eV
Successive ionisation energies (kJ mol-1) of some
elements are given below :
This electron does not feel the full effect
Electronic («03 of the positive charge of the nucleus
Element (/Eh (IE>2
configuration

Li 1s22s’ 520.1 7297 11810 Shielding is most effective whenever there is a full shell
1s2 2s2
(or shells) of electrons between the outermost electron and
Be 899.3 1758 14810
the nucleus, as in case of noble gases (group VIII A). Hence,
B 1s22s22p1 800.6 2428 25026 there is a sharp decrease in (IE) going from noble gas to
C 1s22s22p2 1086.2 2353 4618 alkali metals as given below:
N 1s22s22p3 1402.1 2855 4577 He----> Li Ne-----> Na Ar---- > K
O 1s22s22p4 1313.7 3388 5297 IE 2372 520 2081 513 1521 419
F 1s22s22p5 1680.8 3375 6045 (kJ mor1)
Ne 1s22s22p5 2080 4 3963 6130 The shielding is not so important across a period as it
is down a group. One reason is that an electron in, say, a px
orbital has little shielding effect on an electron in a py or
Factors Influencing Ionisation Energy pt orbital (px, py and pz are mutually at right angles to
one another hence less shielding), s-orbitals are
(IE) variation in a period and group may or may not be
spherically symmetrical, hence filling of Is or 2s will
regular and can be influenced by :
experience shielding effect. This comparison is shown in
(1) Size of the Atom Fig. 3.5.
In a small atom, the electrons are tightly held, while (4) Penetration Effect
in a larger atom, the electrons are less strongly held, the The ionisation energy also depends on the type of
Coulombic force of attraction F being electron which is removed, s, p, d, and /"-electrons have
kZe2 orbitals with different shape. An s-electron penetrates
F=
nearer to the nucleus, and is therefore more tightly held
than a d-electron, and a d electron is more tightly held
where Ze is charge on nucleus, e the electronic charge, r than an /"-electron. Other factors being equal, ionisation
the atomic size and k the constant. energies are in the order s>p>d>f. Thus, the increase in
Thus, the ionisation energy decreases as the size of (IE) is not quite smooth on moving from left to right in the
the atom increases. Periodic Table. (IE\ of group 13 (III A) elements (where a
p-electron is being removed) is actually less than adjacen t
(2) Nuclear Charge group 2 (HA) (where an s-electron is being removed). Also
As given above, the force of attraction between the refer Fig. 3.5.
nucleus and the outermost electron increases with Increasing nuclear charge
increase in nuclear charge. Greater the nuclear charge, Increase
in shielding wins over shielding
greater the energy required to pull the electron from the wins over Min Max
-----------►
atom. Here (IE) increases with increase in nuclear charge. increasing
nuclear Max IE increases
charge across a period
(3) Shielding Effect
IE decreases
The electrons in the inner-shells act as a screen or down a group
shield between the nucleus and the electron in the
outermost-shell. This is called shielding or screening
effect. The larger the number of electrons in the Min
inner-shells, greater is the screening effect and smaller
Fig. 3.5 Comparison of shielding effect across a period and
the force of attraction and thus (IE) decreases.
a group
Chapter 3 : Periodicity of Elements I 119

(5) Electro�lic Configuration (/£) I


In kJ mo1-1 (/£) I

l
Group 1 Group 13
If an atom has fully-filled or half-filled orbitals, its
(IE) is higher than expected normally from its position in Li B 801
496
520
577 :..:_ decrease
the Periodic Table.
Na Al
He2 [!£J · highly stable due to
1
complete valence shell K 419 Ga 579 t constant
403 558
1� .----r- �--. · hence maximum (IE)
[!£] [!£] I i! I i!- Ii! I 1 in . their respective !
2
Rb In decrease
Ne10
376 589
t
. periods. Cs Tl increase
182 282 2PG
decrease
I I I} stabl: half-filled
} i i i
N
p-orb1tals
p n p3 As expected the first ionisation energies decrease
down the groups in case of the main group elements in the
Periodic Table. But in case of transition elements opposite
Variation of (/E) in a Group trends are observed. Thus, (IE) 1 of the corresponding
On moving down a group
elements of3d and 4d-series are almost similar but these
(i) nuclear charge increases
(ii) Z* (effective nuclear charge) due to screening is an smaller than the ([E) 1 of the 5d- series elements.
almost constant. Certainly, the higher values of ionisation energies of the
(iii) number of shells increases, hence atomic size 5d-transition elements are consistent with the relatively
increases. small size of the atoms.
(iv) there is increase in the number of inner electrons
which shield the valence electrons from the
nucleus.
Thus, force of attraction between electrons and
Variation of (IE) in a Period
nucleus decreases and tendency to remove the valence On moving across a period, the atomic size decreases
electron increases. Hence (IE) decreases on moving and nuclear charge increases and therefore the force of
down the group. Variations of (IE) 1 of alkali metals attraction exerted by the nucleus on the electron in outer
(group I) and boron family (group 13) are given in
Fig. 3.6. most shell increases. Hence (IE) increases along a

B
period from left to right. Fig. 3. 7 explains general
t variation of (IE) in a group and period. Fig. 3.8 shows the
(/E)1 pattern of the ionisation energies of the elements H to Ca.

3 4 5 6 7
Tl

a 10
z ------------------•
1.3 1.95 2.6 3.25 3.9 4.55 5.2 5.85
Al g

-r-----------------
n-----------------

F---------------
(/E) --------------►
Fig. 3.6 Variation of (/E) i of groups 1 (•) and 13 3 1.3 Li Be B C N 0 F Ne
(o), (not according to scale) 11 2.2 Na z•- atomic number,
z•- effective nuclear charge due to screening
19 2.2 K n- orbits , rn - radius
DEEP Focus
general the first ionisation energy decreases in a
37 2.2 Rb F- force of attraction between electron and nucleus
In
regular way on descending the main groups. A departure from 55 2.2 Cs (IE) - ionisation energy
this trend occurs in group 13 (boron family), where the expected
decrease occurs between B and Al, but the values for the 87 2.2 Fr --+ increases in the given direction
remaining elements Ga, In and Tl do not continue the trend and +----+ constant
are irregular. Ga with completely filled 3d-orbitals (no 3d in Al)
decreases screening thus Ga is smaller in size than it would Fig. 3.7 Variation of(/£) in a group and period
otherwise be.
120 I Essential Inorganic Chemistry
2500r------.-----------------,
- sodium {[Ne] 3s1 ) loses one electron easily
t 2000 t--++--+-----.tJ....· N_e___+-----1 - magnesium ([Ne] 3s2 ) loses only two electrons easily.
I A
- aluminium ([Ne] 3s2 3 p1 loses only three electrons
easily, and so on.
� 1500 l----i -+---1--==:::::::::::::=-1--1-l------+---.q.._r---1

.....------....
,
ei 8 electron in valence shells
! 1000
H
Na(1s2 2s2 2p6 3s1) =Na + (1s2 2s2 2p6) + e-
j soo-----:..4..,......-_,_-_J-------------.,-
� Ca
Illustration 1 Two atoms have the electronitr:
2p1 K configurations (EC) 1s2 2s2 2p6 and Is2, 2s2 2p6 3s1• (IE)1 of
one is 2080 kJ moZ- 1 and that of the other is 496 kJ moi- 1•
0 5 10 15 20
Atomic number -+
Match each (JE)1, with one of the given (EC). Justify your
choice.
Fig. 3.8 Variation of IE with atomic number showing apparent
anomalies of 8 and 0, and Al and S Solution
Element (EC)
DEEP Focus A 1s 2s2 2p 6
2 � [!I] 1u1r J.lrJ.I
2
ls 2s2
2p 6
• There are tu•o bumps, or dips, around beryllium /boron and
stable inert gas EC and removal c.C
nitrogen I oxygen. The dip from beryllium to boron occurs electron requires higher energy
because the extra electron of boron enters a 2p-orbital, and
this electron will feel the effects of the shielding by the pair of B ls2 2s2 2p 6 3s1 !TI]2 !TI]2 liJ.lt !liJ.! [TI
electrons in 2s-orbital. The shielding has the effect of 1s 2s 6
2p 'J/-
lowering of(IE). The increase from boron to carbon reflects has unpaired electron, removal o:f
the increased nuclear charge of carbon. Also, because the two electron requires lower energy
2p-electrlms of carbon are in different orbita�. they have
little shielding effect on one another. A similar state of affairs Thus (IE) 1 of A is 2080 kJ moi-1 and that of B i5
accounts for the rl.se from carbon to nitrogen. 496 kJ mo1-1.
• The dip from nitrogen to oxygen is a different matter.
Compared to nitrogen. the oxygen atom's extra electron must IDustration 2 Arrange the elements Se, Cl and Sin
go into a 2p-orbital that already has one electron in it. increasing order of ionisation energy.
Putting two electrons into the same orbital brings about
repulsion between them. For this reason it is easier to remove Solution Ionisation energy generally increases from
an electron from the 2p-orbital in oxygen containing two left to right across a row of the Periodic Table and
electrons than it is from a similar orbital in nitrogen decreases from top to bottom down a group.
containing only one electron. This effect overweighs the effect
of nuclear charge from nitrogen to oxygen. (IE) increases
• From ox,•gen to neon, the increase in ionisation energy (IE) S16 CJ l'i"
reflects the increase in nuclear charge. In a general trend,
there will be peaks for noble gases for the values of (IE). decreases t Se 34
Thus, (IE) of Cl > S I
• As Fig. 3.8 shows, metals have relatively low IE and (IE) of Se < S
compared to non-metals. Thus, the order is : Se < S < Cl.
• The IE, of the metalloids generally fall between those Illustration 3 Which has the larger fifth ionisation
of metals and non-metals. The difference in IE energy, Ge or As?
suggests why metals always form cations and
non-metals form anions. Solution Group
• It is relatively easy to remove an electron from a Ge (32) : [Ar] 4s2 3d10 4p2 IVA(14)
partially filled valence shell, where Zeff is lower but it
is relatively difficult to remove an electron from an As (33) : [Ar] 4s2 3d10 4p3 VA{15)
atom or ion that has a filled valence shell, when Zerr is As their positions in the Periodic Table indicate, the
higher. In other words, there is a large amount of group IVA (14) element Ge has four valence-shell
stability associated with filled s and p-subshells electrons and thus four relatively low ionisation energies,
(a noble gas electron configuration), which whereas the group VA (15) element. As has fi\'e
corresponds to having eight electrons (an octet) in the valence-shell electrons and five low ionisation energies. Ge
valence shell of an atom or ion. Thus, thus has larger fifth ionisation energy than As.
Chapter 3: Periodicity of Elements

Target Practice 2
1. Arrange the following atoms in order of increasing (e) Which of the fol1owing has the largest radius? ....
ionisation energy : 02-, N 3- orF-
Si, K, P, Ca 3. Select the atom or ion in each pair that has the
2. Answer each of the following questions. radius
(a) Of the elements S, Se and Cl which has the largest (a) Cl or Cl­
atomic radius? .......... .. (b) Al or 0
(b) Which has the layer radius, Br or Br- ? ........... . (c) I ori
+

(c) Which should have the largest difference between the


4. If screening effect is not considered, and thus ef
first and second ionisation energy: Sl, Na, P or Mg?
nuclear charge is the atomic number, then
(a) larger radius out of Li, Be is ........... .
(d) Which has the largest ionisation energy? (b) larger ionisation potential out of Li, Na is ........
N,P,As (c) larger radius out of Li, Na is ........... .

Answers
1. K < Ca < Si < P z2
(IE)z = (IE)H 2
(b)
2. (a) Se (b) Br - n
(c) Na (since Na + attains stable inert gas confi guration) (IE)u = (IE)" (�) = (2.25) (IE)H
(d) N (e) N 3-
3. (a) c1- Cb) Al (c) I 121
(IE)Na = CIE)H ( ) > (IE)u
4. (a) n 2a
rn =--o
9
2 Thus,Na
rn (Li)= 490 (c)
3
904
rn (Be)= - (as in Cl (a))
4
Thus, Li
Thus, rr. <Be) < rn (Li)
Thus, Li

Practice Exercise 2
1. There is recurrence of properties after 2, 8, 8, 18, 18 and 32 7. BiCl& does not exist. Explain.
elements in the Periodic Table. Explain. 8. Which of the following reactions are not possible/po:
2. What would be the number of elements of recurrence of (i) Sn 2 + + 2Hg 2+ � Hg;+ + Sn 4.;.
properties if each orbital can take three electrons instead of
(ii) Ge + + Pb
4+
two?
2
� Pb 2+ + Ge·'+
3. Group the following electronic configurations in pairs that (iii) Tl + + Ga 3 + � Ga + + TI 3 +
would represent similar chemical properties of their atoms: 9. Determine value of effective nuclear charge of
(a) 1s 2 ' 2s 2 2p 5 (i) Mg (Z =12) (ii) Sc (Z =21)
(b) 1s 2, 2s 1 (iii) Cu (Z =29)
(c) 1s 2 , 2s 2 2p 6 10. He and He+ have different size though each has sam
(d) 1s 2, 2s 2 2p 6, 3s 2 3p 5 of Zand same number of orbit (n = 1). Explain. Youn
(e) 1s 2, 2s 2 2p 6, 3s 2 3p 6, 4s 1 n 2a
equation rn = __ o
(f) 1s 2 , 2s 2 2p 6 , 3s 2 3p 6 , 4s 2 3d10 4p 6 z
4. The boiling points of neon and krypton are -245.9 °C and 11. Ionisation potential (IP) of the element/ion with
-152.9 °C respectively. Using these data estimate the number Z is given by
boiling point of argon. r z2
([P)z = 13.6 2 eV.
)
5. Which of the following properties show a clear periodic
'-;;-
variation along a group?
(a) First ionisation energy Compare values of IP of He and He+ .
(b) Molar mass of the elements 12. Ionisation potential ofBe·x-.. is found to be 217.6 eV v
(c) Number of isotopes of an element the first orbit. What is the value of x?
(d) Atomic radius.
13. Compare atomic radii of third period elements
6. Pb 4 + salts are better oxidising agents while Ge2 .. salts are moving left to right.
better reducing agents. Explain.
122 I Essential Inorganic Chemistry

14. As we shall be taking in further chapters, covalent radii of 28. Which element in each of the following pairs has the larger
Cr(24) to Cu(29) do not vary appreciably. Explain. ionisation energy (IE}?
(a} Kor Br (b) S or Te
Cr2, Mn25 Fe26 Co21 Ni 28 Cu29 (c) Ga or Se (d) Ne or Sr
radius (pm) 117 117 117 116 115 117 29. For one-electron species, (IE) is given by

15. Referring to the Periodic Table, arrange the following


(IE)= is.a [ :: )•v
atoms in order of increasing radius
(a) P, Si, N (b) Na ♦ • Ci-, 0 2-, Ne u
Calculate (IE) ofHe+ , 2 + and Be3+ .
18. On the basis of their positions in the Periodic Table, select SO. (JE)1 < (IE)2 for any case. Can you compare (IE) of He and
the atom with the larger atomic radius in each of the He+ using equation in Q. 29 above ?
following pairs: 31. (/E)1 of Cs is 376kJmoi-1• What is the maximum number of
(a) Na, Cs (b) Be, Ba cs+ ions that can be produced per joule of energy absorbed
by a sample of gaseous Cs atoms ?
(c) N, Sb (d)F, Br
82. Which atom should have a smaller vaJue of (IE)1 ?
(e) Ne, Xe
(a) Oxygen or sulphur (b) Nitrogen or phosphorus
17. Calculate atomic radius of Li and Na atoms
(i) without consideration of screening 33. Which of the following atoms should have a smaJler vaJue of
(ii) with consideration of screening. second (IE): Na or Mg?
Which gives better result? 84. (a) Which has the larger third ionisation energy, Be or N?
(b) Which has the larger fourth ionisation energy, Ga or
18. What is effective nuclear charge on Li if its radius is 1.23 A?
Ge?
19. Compare the ionic size of: 85. Three atoms have the following electron confi.g,µ-ations:
+ 2
(a)Fe 2+ andFe3• (b) Cu and Cu + (i) 1s 2 2 s2 2p 6 3s 2 3 p 1 (ii) 1s 2 2s 2p 3s 3 p
2 6 2 5
+
(c)H , Wand H (iii) 1s 2s 2p 3s 3 p
2 2 6 2 6 48 1

20. Arrange the following in increasing atomic radius : (a) Which of the three has the largest (/E)1 ?
(i) Cr2,, M112s, Feit (ii) Ni28, Cu29, Zll30 (b) Which has the largest (IE)4 ?
(iii) Gen, As33, Ses. (iv) Tl22, Zr"°, Hfa 36. Which has larger (IE) in each pair?
21. Arrange the following in increasing radius: (a) N or o+ (b) Be+ or B+

(i) Li\ Na\ K + (ii) K , Ca 2+ , Ar (c) Be or B (d) Na+ or Ne
+
(iii) Mg, Mg , Mg 2+
(iv) 02 -, N 3-, F- 37. (1E)1 of Be is greater than that of (B) but (IE)2 of (B) ia
(v) 0, o-, 02- greater than that of Be. Explain.
22. Calculate ionic radius of Li2+ , Be 3+ and He+ each 38. Match the electronic configuration with the vaJues of (IE) or
containing one electron in the ground state. the elements in the nth period (n -:t, 1) :
23. For each of the following pairs, indicate which one of the two EC (IE) kJ mor1
species is larger?
(a) N 3- or F- 2+
(b) Mg or Ca 2+ 1. ns 2 I 4000
(c)Fe2+ or Fe3+ (d) Na+ or Ne 2. ns 2np 1 II 1500
+
(e) Au or Au 3+
(0 p3- or N 3- 3. ns 2np 3 Ill 1200
24. The atomic radius of F is 72 pm and that of F- is 136 pm.
4. 2
ns np 6
IV 1400
CaJculate the per cent increase in volume that occurs when 89. In generaJ, ionisation energy increases from left to right
F(g) is converted toF-(&'). across a given period. Aluminium, however, has a lower
25. IE (in kJ moi-1) of Li and Be are given below: ionisation energy than magnesium. Explain.
40. (IE� and (IE)2 in kJ mo1-1 of Kand Ca have been given u
z (IE), (IE)2 (IE),
Li 3 520.1 7297 11810
Be 4 899.3 1758 14810 K 419 3052
Ca 590 1145
(IE)2 of Li is much higher than that of its (IE� but in case of Compare their values and comment on the differences.
Be (IE)3 is much higher than that of its (IE�. Explain. 41. Arrange the following isoelectronic species in order of
26. Can you identify the element of third period whose (a) increasing ionic radius and
successive (]Es) are given below: (b) increasing (IE)
02 -, F-, Na + , Mg 2+
Element (IE)i (/E):a 1
"2. The first four ionisation energies of an element (in kJ mor )
X 513 4562 6920 are approximately 738, 1450, 7.7 x 10 and 1.1 x 10 •
3 4

y 738 1451 7733 To which periodic group does this element belong? Why?
z 1521 2666 3931 43. Which element in each of the following sets has the smallert
first ionisation energy, and which has the largest?
1 1 (a) Li, Ba, K (b) B, Be, Cl
27. (IE) of Na is 513 kJ moi- • What is vaJue in eV molecule- ?
(c) Ca, C, Cl
Chapters: Periodicity of Elements | 123

44. The (IE\ and the (IE\ in kJ mol-1 of a few elements Which of the above elements is likely to be a
designated by Roman numerals are shown below : (a) reactive metal
Element__________ (/E),___________ (/E)2 (b) reactive non-metal
I 2372 5251 (c) noble gas
II 520 7300 (d) metal that forms a stable binary halide of the formula
III 900 1760 ax2?
IV 1680 3380

Answers
1. Electrons in various suborbits are filled in the increasing order Z * = 21 - 18.15=2.85
of their energies as decided by Aufbau rule. 3d-sub-orbit is filled (iii)Cu (Z =29)
after 4s; we can say that 3d is part of 4th period. Similarly 4d is EC Is2 2s22p6 3s23p63d10 4s1
a part of 5th period and so on. (n-3) (n-2) (n-1) n
Period Orbitals Elements
ct 2 + 8 + 6.8 + 10 = 248
(for s,p) (for d)
1 V 1 2 Z* = 29- 24.8= 4.2
2 2s22p6 4 8 10. For the (Is2) there is screening by one electron, hence
3 3s23p6 4 8 Z* = Z - ct = 1.70
4 4s23d104p6 9 18
rn = = 0.5882 a0
5 5s24d105p6 9 18
6 6s24f146d,06p6 16 32 In He* (Is1) there is no screening, hence Z * = Z - 0 = 2
Thus, there are respectively 2, 8, 8, 18,18 and 32 elements in r/ = - = 0.2500 a0
%>
first, second, third, fourth, fifth and sixth period. Hence,
recurrence of properties takes place after these number of Thus, radius ofHe> He*
elements. 11. For He, Z* = 1.70
2. If each orbital can take three electrons, number of elements of (IP)ne = 13.6x (1.70)2 = 39.30 eV] (for comparison only)
recurrence of properties would be 3, 12, 12, 27, 27 and 48
elements [see practice exercise 2 (above)] ForHe*,Z* = Z = 2
3. (a) and (d), (b) and (e), (c) and (f) (IP)UHe t = 13.6 x(2)2 = 54.40 eV
4. - 199.4°C 5. (a) and (d) Thus, (7P)ne> (^)He
6. By inert pair effect, stabilities of 12. x = 3 (since Z = Z* = 4, i.e., no screening when it is one-electron
Pb4 + <Pb2+ and Ge4 > Ge2+ system).
hence Pb 4++ 2e~ ---- > Pb2 +
Pb 13. Atomic radius decreases Na > Mg > Al > Si > P > S > Cl « Ar
and Ge2+ ---- > Ge4* + 2e” but there is sharp jump in the value of Ar (in which van der
are spontaneous towards more stable ions (AG° < 0).Pb4+ can Waals’ radius is taken).
be easily reduced to Pb2+ and thus is an oxidising agent; Ge2 + 14. It is due to the fact that attractive force just balances the
can be easily oxidised and thus, is a reducing agent. repulsive force due to screening.
7. Due to inert pair effect, stability of 15. (a)N<P<Si; (b) Na* < Ne<O2~ <C1"
Bi3+> Sb3 + > As3* and Bi6*< Sb5*< As5*
16. (a) Cs (b) Ba (c)Sb (d)Br (e)Xe
hence Bi5* salts are least stable; hence BiCl5 does not exist.
8. (i) possible, stability of Sn4 * > Sn2 * 17. ________
(ii) possible, stability of Ge4 * > Ge2 * Z Z' n = n2ap/Z =n2a0IZ'
(iii) not possible, stability ofGa3 * > Ga *, Tl3 * < Tl* Li 3 1.3 2 0.71 A 1.63A
9. (i)Mg(Z = 12) Na 11 2.2 3 0.43 A 2.16 A
EC Is 2 2s2 2p6 3s2
(n-2) (n-1) n Since ru < rNa (experimentally) hence results are better when
CT Of 2e in (n - 2)th 8ein(n-l)th le in nth orbit screening is considered.
2.0 + 6.8 + 0.35 =9.15 18. r =
n Z*
Z* = 12-9.15=2.85
(2)2 x 0,529
Remember Z* increases by 0.65 unit per element in a 1.23 =
Z*
period. Z*(Na)= 2.20
(ii)Sc(Z = 21) Z* = 1.72
EC Is2 2s22p63s23p6 3d1 4s2 19. (a)Fe3+<Fe> ; (b)Cu*>Cu2* (c)H+<H<H-
(n-3) (n-2) n 20. (i) Cr = Mn = Fe (repulsive force just balances attractive force)
ct = 2 + 8 + 8 x 0.85 + 1 + 0.35 = 18.15 (ii) Ni < Cu < Zn (repulsive force overcomes attractive force)
for (s, p) (iii) Se < As < Ge (normal gradation)
for (d)
(iv) Ti < Zr = Hf (due to lanthanide contraction)
124 I Essential Inorganic Chemistry

21. (i)Li.< Nn•<K• (ii)Ca 2 •<K.<Ar 6.02 10 23


1 J ionises = atoms
X
(iii) Mg 2 • <Mg•< Mg (iv)F- < 0 2 -< N 3- 376 X 10 3
(v)O < o- < 0 2 -
= 1.6 x 10
18
atoms
22. Radius c,f Li 2 • = 0.1763 A
32. (a) S (b) P 33. Mg
Al
Be3 • = 0.1323 n 2a 34. (a) Be (b) Ga
use equation r" =�
He·= 0.2645 A J Z 35. (a) largest (/E) 1 = (ii)
(b) largest (/E) 4 = (i)
In this case, Z = Z* since each contains one electron and no

ud TI
36. (a)O• (b)B• (c)Be (d)Na•
screening takes place, i.e., CJ = 0.
23. (a)N 3- Cb) Ca 2 • (c) Fe 2.- (d)Ne (e)Au· (f) pJ- 37.
24. 5.74 X 10 % 2 Be[ITJ: B i
25. (IE)2 ofLi involves removal of electron from Li• (1s ) which has
2 2s 2 2s 2
inert gas (stable) helium configuration, hence (/E)2 of Li »> Unpaired eledron in B is easier to remove than that paired
(IE)1 • (/E)3 of Be involves removal of electron frora Be2 • (1s 2 ) electron m Be. Hence (/E) i ofBe > (/E) i of B, and by (JE)1 Be•
which is again stable hence (IE )3 of Be >» (IE )1 and B • are formed.
26. X: Na its (/E)'l is very high Be• O]: B• [IT]
Y: Mg its (IE)3 is very high 2s 1 2s 2
Z: Ar its (JE)1 is very high
Now reverse is the case; removal of unpaired electron from Be·
513000 J is easier than that of paired electron from B •. Hence (JE) of
27. 513 kJ moi-1 • molecule-• 2
6.02 X 10 23 B > (IE)2 of Be.

= 8.522 x 10-19 J molecule-1 38. EC (/E)


8.522 x 10-19 1 III
• eV = 5.326 eV
1.6 X 10-I9 2 IV
3 II
28. (a) Br (b) S (c) Se (d)Ne 4 I
Species (IE)
29. 39. · Refer Text
He• 54.4 eV Being single electron 40. (/E) 2 ofK »> (IE) 1 ofK sinceK• (after one electron is removed)
Li2• 122.4 eV sp ecies, Z* = Z (there has stable inert gas configuration as compared to
+
Be3 217.6 eV is no screening effect) corresponding values for Ca in which Ca + can lose second
electron easily.
30. In case of He, there is screening by one electron hence z• = 1.7 41. (a) Mg 2 • <Na <F- <0 2 -
+
(b) 02- < F- <Na < Mg •
+ 2

(IE) H,= 13.6x (1.7) = 39.4 eV,


2
42. Alkaline eart · ·lements.
(IE) ,• = 54.4 eV (in Q. 29) 43.
Min
H
31. 376 kJ mor1 Max
(a) K Li
376 10 3 J mor 1
Cl
X
(b) B
376 x 10 3 J ionise= 1 mol (c) Ca Cl
= 6.02 x 10 23 atoms

--------------�------------�-��--------------
44. (a) II (b) IV (c) I (d) III

Electron Affinity (EA) DEEP Focus


Electron affinity (EA) is the :iount of energy Two features of electron affinity based on above line to
associated with the gain of electrons by the gaseous atom. remember are :

F(g)+e- ----+ F-(g), EA=-328 kJ mo1- 1 (1) The electron affinity of an element is equal to the
enthalpy change that accompanies the ionisation process
Electron affinity is also called electron gain of its ion.
enthalpy. The sign of the enthalpy change indicates that (2) A large positive electron affinity means that the negative
this is an exothermic process; however the electron ion is very stable (that is, the atom has greater tendency
affinity of fluorine is assigned a value of+ 328 kJ mo1- 1. t.o accept an electron).
Thus, we can think of electron affinity as the energy that • The greater the energy released in the process of taking up
must be supplied to remove an electron from a negative the extra electron, the greater is the electron affinity.
ion. • The electron affinity of an atom measures the tightness
Ml=+ 328 kJ mo1- 1• with which it binds an additional electron to itself.
Chapters: Periodicity of Elements | 12

• In terms of orbital diagram: Variation of (EA) in a Period


f|u~| |u] |TX [TX | T [ + e" -> F: |u] [tx] |TX |TX [rx I EA = - 328 • On moving across a period, the atomic size decrease
inn t kJ mol-1 and hence, the force of attraction exerted by th
Is 2s 2p Is 2s 2p
nucleus on the electrons increases. Consequently, th
In becoming F", fluorine acquires the electron atom has a greater tendency to attract addition;
configuration of noble gas (Ne). electron, i.e., its electron affinity increases.
• By the line of reasoning outlined above, we might • Also (EA) values of metals are low while those <
expect that even metal atoms can form negative ions non-metals are high.
in the gaseous state, and this happens too. • Halogens have high electron affinities. This is due 1
their strong tendency to gain an additional electron 1
Li [tT| pi~| + e" -4 Li" : TX TX EA = - 59.6 change into the stable ns2np6 configuration.
Is 2s Is 2s kJ mol-1
Variation of (EA) in a Group
• There are also cases when addition of electron
experience repulsion; in such cases (EA) is a positive On moving down a group, the atomic size increas;
quantity. This is the case for the noble gases [group 18 and, therefore, the effective nuclear attraction decreas;
(VIII A)], where the added electron goes into (n + l)s and thus, electron affinity decreases.
orbital of higher energy.
Ne TX TX tx TX TX
DEEP Focus
2p • The fact that fluorine has a less negative electron affinit
Is 2s
than chlorine seems to be due to the relatively greate
Ne" : TX TX | TX [ TX | TX T EA = +29 effectiveness of 2p-electrons in the small F atom to rept
Is 2s 2p 3s kJ mol-1 the additional electron entering the atom than d
3p-electrons in the larger Cl atom.
~ The group 2 (HA) elements with filled s-subshell (ns2),
also have positive (EA).
• The results have been represented in Fig. 3.9.
- Even better examples of positive electron affinities
The positive value of (EA) of Be, Mg and N are al:
are those associated with gaining a second electron.
interesting. The stability of the filled s-shell (in Be ar
Here the electron to be added is approaching not a
Mg) and half-filled p-shell (in N) and the resultir
neutral atom but a negative ion. A strong repulsion is
felt and the energy of the system increases. reluctance of these atoms to add any extra electron
indicated.
0" +e" ---- > O2’ EA = 702 kJ mol"1
S’ + e"----> S2" EA = 332 kJ mol-1 300 --------------------------------------------------------------
This process [O“ (g) + e“-----> O2"^)] is unfavourable
in the gas phase because the resulting increase in 200
electron-electron repulsion overweighs the stability
gained by achieving the noble gas configuration. However, t 100
note thatO2- is common in ionic compounds; in solids, O2" o
E
is stabilised by neighbouring cations.
0
• (EA) values (kJ mol-1) of some elements are given below: J
H He ro -100
1 c
-72 (21) o
Li Be B C N O F Ne a? -200
2 -60 (241) -23 -122 o -142 -328 (29) UJ

Na Mg Cu Al Si p S Cl Ar
3
K
-53 (231)
Ca u -123
Ag Ga Ge
-44
As
-119
Se Br Kr
-74 -200 -348 (35)
-300

4
Rb
-48 (156)
Sr u -125 (-36) -116
Au In Sn Sb
-77 -194 -323
Te I
(39)
Xe
^100
0 5 10 15 20 25

u
5 Atomic number -
-47 (119) -222 (-34) -120 -101 -190 -295 (40)
Cs Ba Tl Pb Bi Po At Rn Fig. 3.9 A plot of electron affinity versus atomic number for the
6 first 20 elements. The general horizontal trend is that electror
-101 (-173) | (-270)

7 Fr
-45 (52)
XX (-48) -101 (40)
affinities become more negative (more energy is released as ar
(-44)
M
*Estimated value are in parentheses
extra electron is added) from group IA (1) through group Vll^
(17) for a given period. Exceptions occur at the HA (2) and W
(15) elements.
126 | Essential Inorganic Chemistry

Illustration 1 Why does nitrogen have a less Variation of (EN) in a Period


favourable (more positive) electron-affinity than its
• While moving across a period left to right
neighbours on either side, C and O?
- Z, Z* increases,
Solution Electron configurations of these elements are: - n (number of shells) remains constant,
C6 : Is2 2s2 2p\ 2p\ 2p° - r decreases,
N7 : Is2 2s2 2p] 2p\ 2p\ Ze2}
- F =k <> increases
O8 : Isi2s22p22p\2p\ I J
Carbon has only two electrons in its 2p-subshell and hence, (EN) increases along a period.
can readily accept another in its vacant 2pz-orbital.
Nitrogen, however, has a half-filled 2p-subshell, and the Li Be B C N 0 F Ne
additional electron must pair up in a 2p-orbital, when it
Z,Z*_ max
feels a repulsion from the electron already present.
constant
Thus the electron affinity of nitrogen is less
favourable than that of carbon. Oxygen also must add an n ----- ►
electron to an orbital that already has one electron, but the max (except for Ne)
additional stabilising effect of increased effective nuclear *-
charge (ZefT), across the Periodic Table counteracts the max
effect of electron repulsion, resulting in a more favourable F _ —►
electron-affinity for O than for N. max
EN. —►

Electronegativity (EN) max


• Fluorine is the most electronegative element in
Electronegativity is a measure of the tendency
the Periodic Table.
of an element to attract electrons to itself.
Fig. 3.10 represents variation of electronegativity of
X(g) + e~ —> X~(g)
elements with atomic number. In this :
In a molecule, tendency of the atom to attract bonding
pair towards itself is its electronegativity. B is said to be - the strongest non-metals, the halogens, appear
more electronegative than A if it pulls bonding pair at the peaks;
towards itself. the alkali metals mark the troughs. This is largely a
8+ 6- result of new shell of electrons starting with noble
(A)’X(B) (A)—-(B) gases which have zero values of
electronegativities.
The absolute magnitude of the electronegativity value
(EN) of an element is however not fixed. Different values (ENjf
4.0 - £
may be assigned to the same element from different
theoretical approaches. Accordingly, different (EN) scales 3.5 - O<
Cl
have been suggested.
3.0 - N
r
Variation of (EN) in a Group 2.5 - C
/s
/p Se/
• On moving down the group H
/si
As/
2.0 B
- Z increases but Z* almost remains constant, /ai Ga/Ge
Co^Cu
- number of shells (n) increases, 1.5 - Be; X"Ni^Zn
Mg V »Mn/Fe
- rn (atomic radius) increases, Sc Cr
1.0 - Li Na n
p- kze‘)
- force of attraction
f- —
k
between added
fn 7 0.5 -
K 'Ca
Rb

electron and nucleus decreases, 'He Ne Ar Ar /


I I 1 ii j___ L i 1 1 1 1 1 , 1 1 1 1 4 I ,,
0
2 4 6 8 10 12 14 16 18 20 22 24 26 28 30 32 34 36 38
Therefore, (EN) decreases moving down the
group. Fig. 3.10 Electronegativities of the elements H(l) to
Rb (37). The (EN) of the noble gases have been set at zero.
Element Z Z* n rn F (EN)
A • the group 1 (IA) metals have their nuclei quite
F 9 max max
Cl 17 strongly shielded. Hence, they show little tendency to
Br 35 gain new electron.
I 53 the electronegativities of transition metals like their
constant max max (IE), do not change greatly from one another.
Chapters: Periodicity of Elements | 127

Diagonal Relationship and Electronegativity CH.t C2H., c2h2


hybridization of C-atom sp3 sp2 sp
• Moving one place to the right across period, the
increased nuclear charge holds the electrons more s-character 25% 33.3% 50%
------- ►
tightly to the atom. Moving down one place in a group, (EN)
the extra shell of electrons decreases the attraction of acidic nature
the nucleus for the outer electrons. A diagonal move
means that these effects tend to compensate for one The estimated values (in Pauling equivalents) for C
and N in different hybridization states are :
another.
Atom sp3 sp2 sp
• The electronegativity increases as we go from Li to Be
but it decreases as we move from Be to Mg. As we C 2.5 2.75 3.3
move diagonally, these two effects partly cancel each N 3.7 3.9 4.7
other and there is no marked change in
electronegativity and thus Li and Mg have close Thus, carbon in is more electronegative than
values of electronegativities. Because of this, Li and chlorine; sp-hybridised nitrogen is more electronegative
Mg show similarity in properties— a diagonal than fluorine.
relationship (between elements lying diagonally in
Periodic Table). Pauling’s Electronegativity
• Elements’ pairs showing this relationship are : Let us consider two covalent molecules H—H and
Li—Mg, Be—Al, B—Si X—X
(EN) increases H—H + X—X ---- > 2H—X
-------
<0
Along a diagonal there is both The above reaction is generally exothermic, the bond
0)
<n increase and decrease in (EN) H—Xis thus more stable and hence stronger than either of
co values, each tends to cancel the
£
o effects of other. the bonds H—H or X—X. If the bond H—X were purely
0)
*O covalent (like the bonds H—H of X—X, i.e., the electron
pair forming the bond would have been equally shared by
H and X, the bond H—X would not be so. Pauling
suggested that extra stability was derived from unequal
sharing of the bonding electrons between A and B, giving
DEEP Focus rise to some ionic character. The extent of this ionic
Electronegativity of an element depends on its valence character, as measured by the extra stability of the bond,
state and hence it will change from one compound or one is dependent on unequal sharing of the bonding electrons,
bonding situation to another. An atom will have a greater which depends on the (EN) difference of H and X. Extra
attraction for electrons when its oxidation state is high, rather stability of the bond H—X has been correlated to the
than when it is in a low oxidation state — increase of positive difference in (EN).
oxidation state or formal charge of an atom increases its (EN). This scale determines electronegativity of the element
Thus, (EN) value of sulphur in
X w.r.t. hydrogen H
SF6 > SC12 Xx ~ Xh = 0-208 7aH-x
Fe (II) 1.83 where x (chi) represents electronegativity, AH_X is the
Fe (III) 1.96 extra bond energy in kcal mol-1
T1(I) 1.62
T1 (III) 2.04 and ah-x=£'h-x-
H-X -H E
^X - X
Sn (II) 1.80 £h-x»Eh-h “d Ex-x represent bond energy of
Sn (IV) 1.96
H - X, H - H and X-X bonds respectively.
Pb (II) 1.87
Converting the equation to SI units
Pb (IV) 2.33
Xx ~ Xh =0.1017 -J^h-x
DEEP Focus where AH _ x is measured in kJ mol-1.
Hybrid orbitals with greater s-character should be more • Xh = 2-05 (and not zero) to avoid any negative value of
electronegative than hybrid orbitals with less s-character. As
the element.
s-character increases, (EN) value increases hence acidic
character increases. Some other elements have following values of
electronegativity based on Pauling’s scale :
128 I Essential Inorganic Chemistry

-
(1)
IA
,-
Metas
I
(18)
VIIIA Mulliken-Jaffe Electronegativity
1
Electronegativity (EN) can be regarded as the average
-
2
H (2)
2.1 IIA
Non-metals I I (13) (14) {15) (16) (17) He
IIIA IVA VA VIAVIIA
of the ionisation energy (IE) and the electron affinity (EA)
3
,._
4 Metalloids I ,l 5 6 7 8 9 10 of an atom.

--
2 Li Be B C N 0 F Ne IE+ EA
1.0 1.5 2.0 2.5 3.0 3.5 4.0 (EN)=
2
11 12 (9) 13 14 15 16 17 18
3 Na Mg (3) (4) (5) (6) (7) (8)VIIIB(10) (11)(12) Al SI p s Cl Ar If both (EA) and (IE) are determined in eV units then
,.o 1.2 111B IVBVBVIBVIIB� 18 118 1.5 1.8 2.1 2.5 3.0 Pauling's electronegativity (EN)p is related to Mulliken's
19 20 2 1 22 23 24 25 26 27 28 29 30 31 32 33 34 35 36 electronegativity (EN)M by the equation:
4 K Ca Sc 1i V Cr Mn Fe Co Ni Cu Zn Ga Ge As Se Br Kr
0.9 1.0 1.3 1.4 1.5 1.6 1.6 1.7 1.7 1.8 1.8 1.6 1.7 1.9 2.1 2.4 2.8 (EN)p= 0.336 [(EN)M - 0.615]
37 38 39 40 41 42 43 44 45 46 47 48 49 50 51 52 53 54
5 Rb Sr y Zr Nb Mo Tc Ru Rh Pd Ag Cd In Sn Sb Te I Xe • If(EA) and (IE) are in eV, then (EN) in eV is given by
0.9 1.0 1.2 1.3 1.5 1.6 1.7 1.8 1.8 1.8 1.6 1.6 1.6 1.8 1.9 2.1 2.5
0.187 (IE+ EA)+ 0.17
55 56 57 72 73 74 75 76 17 78 79 80 81 82 83 84 85 86
6 Cs Ba La Hf Ta w Re Os Ir Pt Au Hg Tl Pb Bi Po At Rn • Mulliken's values were about 2.8 times larger
0.8 1.0 1.1 1.3 1.4 1.5 1.7 1.9 1.9 1.8 1.9 1.7 1.6 1.7 1.8 1.9 2.1 than the Pauling's values.
87 88 89 t
7 Fr Ra Ac
0.8 1.0 1.1 Allred-Rochow's Electronegativity
58 59 60 61 62 63 64 65 66 67 68 69 70 71 Allred and Rochow defined electronegativity as the
Ce Pr Nd Pm Sm Eu Gd Tb Dy Ho Er Tm Yb Lu
1.1 1.1 u 1.1 1.1 1.1 1.1 1.1 1.1 1.1 1 1 1.1 1.0 1.2
force exerted by the nucleus of an atom on its valence
electrons:
t 90 91 92 93 94 95 96 97 98 99 noo
101 102 103 (EN) -R = 0.359 z:ffective + 0.744
Th Pa u Np Pu Am Cm Bk er Es Fm Md No Lr A
r
1.2 1.3 1.5 1.3 1.3 1.3 1.3 1.3 1.3 1.3 1.3 1.3 1.3 1.5
where Zeffective is the effective nuclear charge and r the
Electronegativity values are given at the bottoms of the boxes. covalent radius (in A).
The noble gases are not included in this discussion. The relative (EN) values of different elements in the
• If two atoms have similar electronegativity, the bond same scale is extremely helpful in studying and predicting
between H and X will be predominantly covalent. the nature of bonds formed between them.

• A large difference in electronegativity leads to a bond


with a high degree of polar character, and bond will be
predominantly ionic.
Metallic Nature
Metals have the tendency to form cations by loss of
• Ionic character of a bond varies with the difference in electrons and this property makes the elements as
electronegativity as given in Fig. 8.11. This is based on electropositive elements or metals.
the ionic character; HI: 4%, HBr: 11%, HCl: 19% and
M(g) --+ M + (g) + e
HF : 45% which are known from dipole moment
measurements. • The tendency of an element to lose electron is closely
connected to the (IE) of the element. The smaller the
100 (IE) of an element, the greater will be its
tendency to lose electrons and thus greater will
be its metallic character.
• Tendency to oxidise itself provides reducing property
to the elements thus, smaller the (IE), greater thie
metallic character, hence greater the reducing natur,e
50 ···-·-····· 1
Qi.,::;_......--.;_• ___,,.---• : (IE) increases moving along a period left to righ.t
1.7 2 and decreases down the group, hence metallic and
Fig. 3 .11 Electronegativity difference reducing nature decrease along the period and
increase down the group.
• 50% ionic character is shown in the cases when Tendency
e1ectronegativity difference is about 1.7.
Nuclear
Shleldlng to make Reducing Metallic
charge +ve lons power nature
Thus,
(Xx-X 8 )= 1.7 50% ionic But the "'
"'
1/) Ul ti) (I)
Q) Q) Q) Q) Ill
(h shielding II) ti)

... ...
(/)
co
predominantly ionic ca (13 Ill di
(Xx-XH )> 1.7 Q)
....
Q)
effect wins Q) Q)
0
.s
0 (.) 0
so�
<xx-X H )< 1-7 predominantly covalent
.5 _i;; .£ .!:
Chapter 3: Periodicity of Elements > 129

Nuclear charge increases


>
Hydration and Hydration Energy
Shielding-------------------increases
_____-------- >
Hydration energy is the enthalpy change that
But the nuclear charge wins so => accompanies the dissolving of one mole of gaseous ions in
decreases L to R water.
Tendency to form +ve ions
decreases L to R Li+(g)+H2O---- > [LittiaO)]*,
Reducing power _________
AW = - 806 kJ mol-1
decreases L to R
Metallic nature ----------------
• Size of the ion and its charge determines extent of
Notice that the most reactive metals (the top of the hydration.
activity series) are on the left of the Periodic Table,
whereas the least reactive metals (the bottom of the • Greater the charge, smaller the size of the ion, greater
activity series) are in the transition metal groups closer to the attraction for the lone pair of O of HgO, hence
the right side of the table as shown in Fig. 3.12. greater the extent of hydration and hence greater the
hydration energy
- size of the hydrated ions increases,
i
- ionic mobility decreases [heavier (hydrated) ions
—I L__, moves slower).
i
More active Less active1--” (A) (B) (Q (D) (E) Na* Mg2* Al3+ Si •i+
metals metals Li*-------------- *► Charge on the ion
i
(A)
I Na*-* Size of the ion (B)
Fig. 3.12 K* — * Hydration and
hydration energy
Redox reactions are common for almost every element (Q
in the Periodic Table except for the noble gas elements of Rb* -► Size of the hydrated
group VIIIA (18). In general, metals act as reducing ion (D)
agents, and reactive non-metals such as O2 and the Cs*^ — Ionic mobility (E)
halogens act as oxidizing agents as given in Fig. 3.13.
(Min.---- ► Max.) (arrow points min. to max.)

Oxidizing i Table 3.6 compares extent of hydration and other


agents; i related properties of 1(IA) group cations.
i
tend to i
gain e- i
i
i
i
Reducing agents; i
tend to lose e~ I

Fig. 3.13

Table 3.6 Hydration and Ionic Mobilities


Ionic Ionic Radius of Hydration energy**
Ion Oxidation energy** AH°0X redn. (V)
radius (A) mobilities* hydrated ion (A) ^°hyd.

Li* 0.76 33.5 3.40 -544 169 - 3.045


Na* 1.02 43.5 2.76 -435 208 -2.714
K* 1.38 64.5 2.32 -352 191 - 2.925
Fib* 1.52 67.5 2.28 -326 200 - 2.925
Cs* 1.67 68.0 2.28 -293 196 - 2.923

* Values in 10"
10 8 m2 V"1 s
** Values in kJ mol-1
130 | Essentia/ Inorganic Chemistry

• Oxidation of an alkali metal M to ions M*(aq) involves Increasing acidic character


4)

the hypothetical steps : o IA HA IIIA IVA VA VIA VIIA


d
G5 (1) (2) (13) (14) (15) (16) (17)
1. Sublimation of the solid metal to the gaseous state, o
0) Li2O BeO B2O3 C02 N2Os F20
M(g), cn
cC
JO SO3 CI2O7
2. Ionisation of the gaseous metal to the gaseous tUD
Na2O MgO AI2O3 SiO2 P4O10
metal ions, M 7 g), .su> K2O Ga2O3 GeO2 As2(>5 Br2O7
as CaO SeO3
3. Hydration of the gaseous metal ions to produce
C Rb2O SrO In2O3 SnO2 Sb2Os TeO3 I2O7
Af *(aq).
Cs2O BaO T12O3 PbO2 Bi2O5 POO3 At2O?
DEEP Focus
From Table 3.6 we see that they are positive quantities,
DEEP Focus
but the value is smallest for Li. This is primarily because of the
• Oxides of the following elements are amphoteric
less negative AH for the hydration of Li*. Thus, it appears that
Li(s) should be easiest to oxidise of the alkali metal and is thus H, Be, Al, Ga, In, TI, Sn, Pb, Sb, Bi, Po
best reducing agent of the alkali metals but in aqueous solution H2O is amphoteric (also called amphiprotic)
which is also confirmed by its most negative value of H2O + H2O —> H3O* + OH-
(standard reduction potential) of since it is H* acceptor (base) as well as H* donor (acid).
Af*(aq) +e" ---- > M(s) BeO, Al2O3,SnO2,PbO2,... are amphoteric since they form
salts with acid as well as with base
M*(g) + e-
ALO, + 6HC1 —> 2A1CL + 3H2O
base acid
(3)
AWionls AFf’hydra
Al2o3 +2NaOH + 3H2O---- > 2Na[Al(OH)4]
acid base
(2)
ox = AH’ sub + AH’ lonij + AH” hydra “ oxide is acidic if it reacts with a base.
M(g) ^f(aq) + e~ ~ oxide is basic if it reacts with an acid.

IA(1)
(1) AH sub ox
M(s) . Rb Cs
Li Na K
normal oxides Li2O Na2O K2O Rb2O Cs2O
peroxides Li2O Na2O2 k2o2 Rb2O2 Cs2O2
Acid-Base Character of Oxides superoxides NaO2 ko2 RbO2 CsO2
On moving across a period, the basic character
of the oxides gradually changes first into IIA (2)
amphoteric and finally into acidic character.
Be Mg Ca Sr Ba
• On moving down the group, reverse behaviour is BeO MgO CaO SrO BaO
observed, i.e., from more acidic to more basic.
CaO2 SrO2 Ba02
Oxides of the element M in produce AfOH
1 18
- If electronegativities difference of M and 0 is greater H 2 n 14 15 16 17
than that of H and 0 in H^O then MOH is acidic due to
Li Be B C N 0 F
formation of H3O+ Transition metals
Na Mg 3 4 5 6 7 8 9 10 11 12 Al Si P B Cl
M —O—H + HgO----- > H3O+ +MO~
K Ca Sc Ti V Cr Mn Fe Co Ni Cu Zn Ga Ge As Se Br
- If electronegativities difference of M and 0 is less than
Rb Sr Y Zr Nb Mo Tc Ru Rh Pd Ag Cd In Sn Sb Te l
that of H and 0 in HgO then AfOH is basic due to
formation of OH" Cs Ba La Hf Ta W Re Os Ir Pt Au Hg TI Pb Bi Po At

M—O—H + H—O—H----- > [MOHJ* + OH' Fr Ra Ac

• Stability of oxides decreases across a period. Predominantly base oxides Acidic, covalent oxides
• Behaviour of the elements is shown in Fig. 3.14.
Oxides with amphoteric nature generally ionic

Fig. 3.14 The nature of oxides of the elements


Chapter 3: Periodicity of Elements |

Illustration 1 Arrange the following oxides in order


of increasing molecular (acidic) character :
Melting Points and Boiling Points
SO3 , C12O7 , CaO and PbO2 A solid melts when the forces holding its consti
units in position is overcome by thermal energy
Solution Acidic character of oxides increases as consequence of increase in temperature. The m
non-metallic character of the element that is combined temperature is determined by a number of difl
with oxygen increases
factors :
increasing non-metallic character
(1) Nature of packing of the atoms, ion
Ca < Pb < S < Cl
molecules,
Periodic group : HA IVA VIA VHA
(2) The lattice energy,
(2) (12) (16) (17)
(3) Association through hydrogen bond, etc.
increasing (molecular) acidic character
CaO < PbOo < SOo < CloO Boiling involves complete detachment of the pai
'2 3 2^7
of a liquid. Both the melting and boiling points
element thus indicate the nature and extent of bi
Hydrides between the atoms of that element. In spite c
contribution of a large number of variable factt
• Hydrogen combines with a number of other elements controlling the melting and boiling points, a sti
including metals and non-metals to form compounds
periodicity is observed in their values. Fig. 3.16
called hydrides (withH-).
• Covalent nature of hydrides increases across a period
and decreases down the group. 3500- CI
Ji
• Ionic hydride are better reducing agents than covalent ~ 3000-
<u Mo
hydride and reducing nature of hydride decreases I 2500-
across a period and increases down the group.

uM
• Covalent and ionic hydrides are classified in Fig. 3.15. £ 2000-
| 1500-
(A) (B) (C) LiH NaH MgH2 A1H, SiH4 sg. 1000-
covalent
NaH ------------------------------------------------- ►(A) E
KH
i°nic (B) £ 500-

reducing nature 0- / i
RbH (Q 4 4 12 16V 20 24 28 32 $ 40 44
-500- He Ne Ar Kr
CsH (Min. Max.) Atomic number

IA HA IIIB IVB VB VIB VIIB VIII IB IIB IIIA IVA VA VIAVIIAVIIIA Fig. 3.16 Periodic variation of melting point of elemei
1 18
H 13 14 15 16 17 represents a plot of melting points of some elei
2
against their respective atomic numbers. The shape
Li Be B C N O F
Transition metals curve indicates periodic variation. It may be fi
Na Mg 3 4 £ 6 7 8 9 10 11 12 Al Si P S Cl
observed that:
K Ca Sc Ti V Cr Mn Fe Co Ni Cu Zn Ga Ge As Se Br
(i) The noble gases have very low melting points
Rb Sr Y Zr Nb Mo Tc Ru Rh Pd Ag Cd In Sn Sb Te I
(ii) The melting point of the elements in any ]
Cs Ba La Hf Ta W Re Os Ir Pt Au Hg TI Pb Bi Po At
increases gradually to group 14 where it be<
Fr Ra Ac maximum; iot then decreases to the next noble
(iii) The melting points of elements occurring in a
Mainly ionic with often of uncertain formula :
usually decrease downwards, i.e., as the elei
Covalent, molecular structures fixed formula bear higher atomic numbers.
absorbed into metal lattice

Fig. 3.15 The nature of hydrides of the elements


132 I Essential Inorganic Chemistry

DEEP Focus atom. This is why the density of the central elements im
any period is highest.
(i) The nob]e gas atoms contain a comp1eted e1ectron shell.
So there exists little tendency for these atoms to combine 12
between themselves. This is why they occupy the lowest
range of melting points. 10
(ii) The alknli metn]s have only one e]ectron in their
outermost shells. Their contribution to metallic bonding 8
is, therefore, weak, resulting in low melting points. -�

(iii) As the number of electrons participating in metallic Q) 6
0
bonding increases. the melting point also increases: Be> 4
Li; Mg> Na.
(iv) Boron, carbon and silicon form giant three-dimensional 2
structures; breaking of these arrangements require 1arge
amount of energy. Hence. they have very high melting
points. 4 8 12 16 20 24 28 32 36 40 44 48 50
(v) Nitrogen. oxygen, fluorine-an form stable diatomic Atomic number
molecules containing triple, double and single bonds Fig. 3.17 Periodic variation of density with atomic number
(between their atoms) respectively. They do not form
giant three-dimensional lattices-may be due to greater The increase of density down any group in the
repulsion between the non-bonding electrons and the Periodic Table is similarly explained from the increasing
greater stabilities of these multiple bonds. At the same
time, very poor binding exists among the molecules number of neutrons and protons in the nuclei.
themselves. Consequently they have very low melting
points.
(\i) As we proceed downwards along a group, the elements
involve increasingly highly orbital for binding their
Atomic Volume
atoms. Poorer bonding, therefore, results in lowering of Lothar Meyer, in 1870, independently introduced the
melting point (Li-Na-K-Rb-Cs). Exceptions to this idea of periodicity in properties of the elements from study
take place in group 15 to 17 (e.g., P > N; S > O; I >Br> Cl of their atomic volumes. The atomic volume is the
> F). Phosphorus and sulphur have a diminished volume of one mole of atoms of an element; it was
tendency to bond multiplicity (larger size poor lateral
overlap). They form single bonded polyatomic structures obtained as atomic volume (cc mor 1) = atomioc
having higher melting points. Among the halogens, weight/density. This is only a rough indication of atomi,c
polruisabihty of the halogen atoms increases with their size since the density of an element is determined by
size; stronger van der Waals' interaction is established, several factors like physical state, allotropic form, numbe:r
resulting in gradually higher melting point. Thus of atoms per molecule, nature of packing and so on. Yet a
bromine is a liquid and iodine is a solid at room periodic variation is obtained when the atomic volumes etf
temperature.
elements are plotted against their atomic numbers.
The variation of melting point along a transition series is
irregular. The d-orbitals and the electrons there in contribute
(Fig. 3.18)
to metallic bonding to a varying degree. A sharp drop in melting
point is observed just after the completion of a transition series, 70 Cs
e.g., Cu: 1356 K vs. Zn: 692.5 K; Ag: 1233.8 K vs. Cd: 593.9 K
It appears that the completed d-orbitals do no longer 60- Rb
effectively participate in metallic bonding.
c-
0
50-
K

Density '; 40-


The densities of the elements in the solid state also
exhibit periodic variation when plotted against their
atomjc numbers. It may be observed from Fig. 3.17 that
i 30-
Na
-�
the densities of the alkali metals occupy the minima in the
curve; the density first rises and then decreases along a <( 20-
period, the elements in the middle having the highest
density values. 10-
Here again the underlying reason is largely Ni Os
B
complicated by the nature of packing of the atoms. A 0 I
simple interpretation may be provided by remembering 10 20 30 40 50 60 70 80
that stronger and compact binding leads to high Atomic number
density. The binding force is again dependent on
Fig. 3.18 Periodic variation of atomic volume
the number of valence electrons provided by each
Chapter 3: Periodicity of Elements | 133

Target Practice 3
1. Any salt AfCl2 is soluble in water. Relate lattice energy and 5. What is enthalpy change in the formation of MgO?
hydration energy for MC1O to be water soluble.......... (i) Mg(s) ---- > Mg(g) ; &H = S
2. Arrange hydroxide of alkaline earth metals in increasing (ii) Mg(g) ---- > Mg+(g)+e”
solubility in water............ (iii) Mg+(g) ---- > Mg2+(g) + e” ; AH=Z2
3. Arrange solubility of sulphates of (iv) O2(g) ---- > 20(g) ; AH = D
(a) alkali metals
(b) alkaline earth metals (v) 0(g) + e ---- > O”(g) ; AH=-EX
in increasing order..................... (vi) O”(g)+e” ---- > O2”(g) ; AH=E2
4. Which is more stable (thermally) in each pair (vii) Mg2r(g) + O2”(g) ---- > MgO(s) ; &H = -U
(a) BaCO3,SrCO3 (b) BeCO3,CaCO3
(c) MgCO„BeCO„ (d) Na9CO3,Li,CO.
(e) NaHCO3, Na2CO3

Answers
1. For a salt to be water soluble, hydration energy should exceed 4. (a) BaCO3 > SrCO3
the lattice energy (b) CaCO3 > BeCO3
hydration energy > lattice energy (c) MgCO3 > BeCO3
2. Be(OH)2 < Mg(OH)2 < Ca(OH)2 < Sr(OH)2 < Ba(OH)2 (d) Na2CO3 > Li2CO3
For salts of small anions (as fluorides, hydroxides), the lattice (e) Na2CO3 > NaHCO3
energy decreases from Be to Ba due to increase in size. 5. Add all steps (i) to (vii) (with (iv)/2)
Hydration energy also decrease but this decrease is less than Mg(s)+102(g) ---- > MgO(s)
decrease in lattice energy. As a result hydration energy 2
increases. Hence, solubility of hydroxides increases. Net enthalpy change is
3. (a) Li2SO4 < Na2SO4 < K2SO4 < Rb2SO4
AH = S + Z1 + Z2 + --Ei + E2-[/
(b) BaSO4 < CrSO4 < CaSO4 < MgS04 < BeSO4

Practice Exercise 3
1. Which has greater (EA) value in each pair? 8. Calculate effective nuclear charge that added electron
(a) For Ci­ (b) 0 or 0“ would experience in F-atom in the formation of negative
te) Ne+ orF (d)ForF- ion.
2. EA of Cu is 123 kJ mol but that of Zn is -87 kJ mol-1. 9. (a) Explain Na2CO3 is thermally stable, but Li2CO3 and
Explain. MgCO3 both decompose on heating.
3. We consider formation of NaCl(s) using Born-Haber cycle (b) BeCl2 and A1C13 are Lewis acids.
which involves following steps : (c) B and Si resemble each other in their properties.
(I) Na($) + lci2 ---- > NaCl(s) 10. Which is more electronegative in each pair?
(a)F”orNa+ (b)NeorF"
(ID Na(s) ---- > Na(g) &H = S (c) F or Cl (d) H” or He
(III) Na(g) —> Na+(g) + e‘ &H = I 11. Electronegativity of F on Pauling’s scale is 4.0. What is the
(IV) |ci2(g) —> Cl (g) - value on Mulliken's scale ?
2 12. A+B~ and A~B+ can be formed from elements (A) and (B).
(V) Cl(g) + e’—> Cl” (g) AH = ? Explain their formation based on relative values of (EN),
(VI) Na+(g) + Cl”(g)---- > NaCl(s)' AH = -U (EA) and (IE).
Can you derive value of (EA) of Cl(g) atom. 13. Covalent radius of F is 0.72 A. Calculate its Allred-Rochow’s
4. Why are the electron affinities of the alkaline earth metals electronegativity.
positive? 14. Covalent radius of oxygen is 0.74 A and its A.R.
5. Is it likely that Ar will form the anion Ar“. electronegativity is 3.5. What is effective nuclear charge
6. Considering their electrons affinities, do you think it is experienced by oxygen ?
possible for alkali metals to form an anion like M” where M 15. Make a Born-Haber cycle for the preparation of MgBr2(s).
represents an alkali metal. 16. Calculate the electronegativity of carbon from the following
7. What is relationship between the electron affinity of a data:
univalent cation such as Na+ and the ionisation energy of Eh-h = 104.2 kcal mol”1; Ec-c = 83.1 kcal mol”1
the neutral atom? Ec-h = 98.8 kcal mol-1
134 | Essential Inorganic Chemistry

17. (IE) and (EA) of fluorine are respectively 17.418 eV and 23. In the preparation of hydrocarbon by Kolbe’s electrolysis,
-9.278 eV. Calculate (EN) of fluorine atom. generally RCOONa or RCOOK are taken but not RCOOLi.
Explain.
18. Four atoms are arbitrarily labelled D, E, F and G. Their
electronegativities are as follows D = 3.8, E= 3.3, F = 2.8 24. LiCl is hydrated but NaCl is always anhydrous. Explain.
and G = 1.3. If the atoms of these elements form the 25. Arrange NH3,PH3 and AsH3 in increasing reducing nature.
molecules DE, DG, EG, and DF, how would you arrange 26. Fill in the blanks :
these molecules in order of increasing covalent bond (i) NaH is........... hydride.
character.
(ii) BgHg is called
19. Tendency to lose electrons shows reducing property of the
element. Arrange the following in order of reducing (iii) SiH< is called .
property: (iv) Most widely used mixed hydride in organic synthesis is
(a) Na, K, Rb (b) Na, Mg, Al
(c) F‘, CP, Br’, I" (d) Mg, Ca, Sr 27. Write the formulae and names of the hydride of the
20. Li is better reducing agent than Na. Explain. following second period elements :
21. Which of the following reactions are possible ? Li, C, N, 0, F.
(a) 3Mg+ 2A13+ ---- > 2A1+ 3Mg2* 28. SnO2 forms two types of colloidal sols, positively charged im
(b) ci2+2r -- > i2+2cr an acidic medium and negatively charged in a basiic
(c) K" + Na —> K+ Nax medium. Explain.
29. Arrange following oxides in increasing acidic nature
22. Arrange the following ions in increasing
Na+,Mg2+,Al3* Li,O,Z BeO, B,O,
Z v

(i) extent of hydration 30. Explain:


(ii) hydration energy (i) Aqueous CO2 can cause rusting.
(iii) size of the hydrated ions (ii) A12O3 is amphoteric while B2O3 is acidic.
(iv) ionic mobility (iii) BeO and A12O3 both are amphoteric.
(v) standard reduction potential (^°rodn), 31. Which oxide is more basic, MgO or BaO? Why?
Mn'f(aq) + ne~ -----> M(s) 32. Write the formulae and names of the oxides of
(vi) enthalpy of oxidation (AH°0X) of M(s). second-period elements (Li to N). Identify the oxides as
acidic, basic or amphoteric.

Answers
1. (i)F (ii) O’(more positive) (iii)Ne+ (iv)F 6. M(g)+e~ —> M~(g) AH =-ve
2. Cu(Ar)3d10 4s1 + e’---- > Cu’ (Ar) 3d10 4s2 ns ns
In case of Cu, added electron goes into empty 4s-orbital where it T U
can get repulsion to a certain extent giving a positive value of Yes, since M~(g) has paired ns-electrons
(EA). In case of Zn, added electron can be accommodated into
7. Na + (g)+e’ ---- > Na(g) AH = - (EA)
4p-orbital giving a negative value of (EA).
Na(g) ---- > Na+(g)+e" AH = + (IE)
3. (v) involves addition of electron to Cl(g)
(IE)=-(EA)
Cl(g)+ e- ---- » Cl'(g) EA-x
8. Z* = Z - S = 5.85
Energy released is (EA)
9. (i) Due to diagonal relationship, Li and Mg resemble.
Adding (ii) to (vi) gives (ii) Due to diagonal relationship.
Na(s)+ 1 Cl2(g)---- > NaCl(s) (iii) Due to diagonal relationship.
2
10. (a) both zero (b) both zero (c) F (d) both zero
which is also the step (i), hence
11. 11.2 (4.8 times larger)
S + 1 + — + x(EA)-U = Q 12. A---- > A+ + e~ if (IE)A < (IE)B
2
hence, x is calculated. B + e ---- > B’if(EA)A<(EA)B
Thus, A+B’ is formed if (IE)a + (EA)a <(IE)b + (EA)b
4. M(g)+e“---- >M’(g)
(IE)A + (EA)a (IE)b + (EA)B
The extra electron must enter np-subshell, which is effectively or <
shielding the two ns-electrons and the inner electrons. 2 2
Consequently, the alkaline earth metals have little tendency to or (EN)a < (EN)b
pick up an extra electron. Thus, A*B" exists if electronegativity of A < B. Similarly,
5. Added electron enters 4s orbital of higher energy where it gets A'B+ is formed if electronegativity of A > B.
repulsion. Thus, (EA) value is positive. Thus, Ar’ is not formed.
Chapter 3 : Periodicity of Elements I 135
13. 4.7952 14. 4.2039 21. (a) and (b)
22. (i) Na• <Mg 2♦ <Al3+ , (ii) Na• <Mg 2• <Al3+

l Ml•DJ
- Mg(s) + Br2(g) llHi • Q · MgBr2(s) I (iii) Na• <Mg <Al 3+ , (iv) Al 3+ <Mg 2+ <Na•
2+

(v) Al 3+ <Mg 2+ <Na• (E� • -ve), (vi) Al<Mg<Na


Ml•S
23. Ionic mobility of Li+ « Na• or K + (Refer Table 3.6)

1-2'- -2E1
Thus, Na + or K + reaches cathode faster than Li + and thus is
Mg(s) Br(g) n discharged at a faster rate. (This is due to greater hydration of
-U
Ml • I Ml • +2'- Li + than other ions)
24. Li + (smaller size) is more hydrated than Na + 0arger size).
2
Mg +(s) + 2Br_-_______, 25. NH 3 <PH 3 <AsH 3
26. (i) an ionic (ii) diborane (iii) silane (iv) LiAlH 4
fl H11 = S + I + D - 2E - U = fl H1 = Q
27. Lili, CH 4, NH 3 , H 2O, HF
S = beat of sublimation ofMg(s) 28. SnO2 forms Sn"• in acidic medium andSno�-in basic medium.
I = ionisation energy of Mg(g) These ions are preferentially adsorbed on SnO2 forming
D = dissociation energy of Br2 (g) colloidal sol.
-E = electron affinity of Br(g) SnO2 + 4W � Sn .c+ + 2H 2O
-U = lattice energy ofMgBr2 (s) SnO2 + Sn .c+ �
[SnO2 ]Sn .c+
Q = heat of formation ofMgBr2 (s) positively charged
18. Using Pauling's scale : SnO2 + 2OH- � Sno;- + H 2O
Xe = XH + 0.182.Jll.e-H SnO2 + Sno:- � [SnO2 ] Sno;­
negatively charged
XH = 2.05
Ae-H = Ee-H - '1EH-H · Ee-e 29. Li 20 < BeO < B 20 3
basic ampbot.eric acidic
= 98.8-.J104.2x 83.1 = 5.75 30. (i) CO2 is an acidic oxide
Xe = 2.49 CO2 + H 2O -.+ H 2CO3 � ff+ + Hco;
17. 4.07 eV H + can react with Fe formingFe 2 + and thus cause rusting.
18, Smaller the (EN) difference, larger the covalent nature. Fe+ 2H + � Fe 2+ + H 2
DG<DF<EG<DE (ii) While going down the group, basic nature increases and
increasing covalent bond character acidic nature decreases, hence
B 20 3 is an acidic oxide.
19. (a) Na<K<Rb (b) Na> Mg> Al (c)F- <c1-<Br-<i­
Al2O3 is an amphoteric oxide.
(d)Mg < Ca < Sr
(iii) Due to diagonal relationship.
20. The large hydration energy is responsible for the case with
which lithium ion will be made in solution. The electrode 31. BaO
potential of Li reflects this tendency for Li to convert into 32. Li 2O basic
hydrated ion, rather than its inherent ability as a reducing BeO amphoteric
agent. If lithium takes part in reactions that do not involve B2O3 acidic
water, then it does show less reducing power than the other CO2 acidic
members of the group.
N2 sO acidic
Total Practice Set
(Read, Plan & Solve'.)
SSEOEIJ:- j*I :~7< LjSegET-.CTEHBB

Problem 1. Draw a graph that shows how the second Solution In the Periodic Table (IE), (EA) and
(IE) (given in kJ mol"1) of the elements helium to sodium non-metallic behaviour decrease down the group and
change. Explain the shape of the graph. increase across a period while radius increases down the
He Li Be B C N O F Ne Na group and decreases along a period.
(IE)., 525 729 175 242 235 285 333 347 395 456 (a) Sr has a lower (IE) than Be.
' 0 8 7 7 3 6 8 1 2 2 (b) P has more non-metallic behaviour than Ga.
(c) Sr has larger atomic radius than Mg.
Solution Once Li has lost an electron, it has noble gas (d) Br has more (EA) than I.
electron structure, i.e., a filled set of electron shells. Like
the noble gas it has high (IE)2. When Be has lost one Problem 3. In the modern Periodic Table, the
electron, it is isoelectronic of Li. Its single outer electron is elements in groups IB(ll) and IIB(12) are shown in
shielded from the nucleus by a filled shell of electron. This d-block. This block is the place where the transition
gives a low (IE)2. In similar fashion, there is a trend elements are found. Why do you think some chemists
upwards across the period, with dips at places when an argue that the Group IB(ll) and IIB(12) elements should
electron is outside a filled s-orbital, or where two electrons not be considered as d-block elements?
begin to fill a single p-orbital. Thus, behaviour of Solution In the d-block, the 3d, 4d, or 5d orbitals are
nonovalent cation (formed after one electron is lost) with being filled. The properties of the transition elements are
itomic number Z, is just parallel to the atom with atomic a result of their having unfilled d-orbitals. However IB
.lumber (Z -1). and IIB elements have full d-orbitals.
T_ b IB (11) —(n - l)d10 ns1 e.g. Cu^ : [Ar] 3d10 4s1
£ 7000 -
-> IIB (12) — (n - l)d10 ns2 Zn30 : [Ar] 3d10 4s2
6000 -

vz
o> Indeed, these elements have s-orbitals filling. This is one
® 5000 - Na
reason why some chemists think them misplaced tacked
§ 4000 - O Ne on the end of the trar sition elements. However, the:
F chemical properties of the IB metals make to include them,
S 3000 - B
c N among the transition metals, viz., Cu2+ is coloured, forms,
o
"O 2000 - C complexes etc. The IIB metals are best excluded.
Be
8 10001- Problem 4. There are certain exceptions to the
W 0L >
123456789 1011 properties of the elements.
Atomic number (i) Which metal is liquid at room temperature ?
A graph of second ionisation energy against atomic (ii) Which non-metal is a solid with an extremely high
number for the elements helium to sodium melting point?
(iii) Which non-metal is a liquid at room temperature?
For (/E)2 He4 Li4 Be4 B4 C4 N4 04 F4 Ne4 Na4 (iv) Which two elements would you expect to combine ia
behaviour H He Li Be B C N O F Ne the most violent fashion?
is similar to
Solution (i)Hg
(ii) Carbon (diamond or graphite)
Problem 2. Of the following pairs, which would you (iii) Br
expect to have (iv) Cs and F
(a) the lower first (IE), Sr or Be?
(b) the more non-metallic behaviour, Ga or P? Problem 5. The heats of formation (AH°f) of the
(c) the larger atomic radius, Mg or Sr? oxides of the third period, sodium to chlorine, are in
(d) the more negative (EA), Br or I? kJ mol-1
Chapter 3: Periodicity of Eh

Na20 MgO A12O3 SiO2 P,O10 SO3 C12O7 Solution


-416 -602 -1676 -911 -2984 -395 +250
A £
Divide these values by the number of oxygen atoms in
&) (vi
the formula of the oxide. The resulting figure is a
(b) ('
measure of the strength of the oxide. What is the link
between the figures and the structures of the oxides?
(c) 0
(d) I

Solution (e) (v
(f) (i
Number of AH?/x
Oxide AH? (kJ mor1) O-atoms = x (g) (
(h) (i
Na2O -416 1 -416
MgO -602 • 1 -602 Problem 7. What is effective nuclee
Al 2O3 -1676 3 -558.7 periphery of nitrogen atom when an e
SiO2 -911 2 - 455.5 added in the formation of an anion. Cor
when the atom is ionised.
P4O10 -2984 10 -298.4
so3 -395 3 -131.7 Solution N(7) = ls2 2s22p3
Cl2O7 +250 7 35.7 Added electron will be screened by five (2
in 2nd orbit and two (Is2) electrons in Is
More negative the value of(^H°f /x), greater the stability,
Added electron i
Thus, MgO is most stable and C12O7 is least stable,
stability being :
C12O7 < SO3 < P4O10 < Na2O < SiO2 < A12O3 < MgO (n -1) n
In general, stability decreases across the period from left The screening constant, o = 5x 0.35 in
to right (except Na2O). This is shown in given fig. = 2 x 0.85 in
100-
Total or = 3.45
0-
'*CI2O7 Effective nuclear charge = 7 - 3.4f
-100- SO3
In the formation of positive ion, electron
—200- AH°f orbit is screened by four electrons in ntl
P4O10
electrons in (n — l)th orbit
—300-
—400- +Na2O
This electron is no
—500- SiO2
i (n -1) n
-600-
MgO AI2O3
—700- Screening constant, 0 = 4x 0.35 i
= 2 x 0.85 i
Total value of <5 = 3.1
Problem 6. Match the columns in (A) with those given
in (B). /. Effective nuclear charge = 7- 3.1

________ (A)_____________ _________ (B)_________ Problem 8. The sums of first and s


(a) Inner transition (i) Diagonal relationship energies and those of third and fourth ioi
elements (in MJ mol-1) of nickel and platinum are
(b) d-block elements (ii) p-block elements
(IE). + (IE)2 <
(c) Elements with unpaired (iii) Typical elements
electrons Ni 2.49
(d) Be—Al (iv) Block deciding rule Pt 2.66
(e) s and p-block elements (v) Transition elements
(f) Elements of 3rd period (vi) Paramagnetic Based on this information, write
(i) the most common oxidation states
(g) ns2np' tons2np5 (vii) Normal elements
(ii) name of the metal (Ni or Pt) which
(h) Aufbau principle (viii) f-block form compounds in its + 4 oxidatio
138 | Essential Inorganic Chemistry

Solution (i)Ni = +2,Pt=+4 since (IE)1 + (IE)2 ofNi is Solution Stepwise ionisation of Li(g) is written as:
less than its (IE)3 + (IE)4 and reverse is the case in Pt. Li(g) Li+(g) + e~ (IE\ = 520 kJ moL
(ii) Platinum forms more stable complexes in +4 state
due to its higher stability than +2 state. Li+(g) Li2+(g) + e" (ZE)2 = ?=
Li24(g) Li3*(g) + e~ (ZE)3 = ?=
Problem 9. Which atom should have a higher second
IE : Li or Be? Li(g) Li3*(g) + 3e~
Solution Ionisation of Li and Be can be represented as (ZE)Tota] = 1.96 x 104 kJ mol-1
Li2+(g) is isoelectronic of H(g), hence
Li (g)---- >Li+ (g)---- >Li2+(g)
Is2 2s1 Is2 Is1 (IE)3 = (2.18x 10’18)(Z2)
Be(g)--- >Be*(g)---- >Be2+(g)
is2 2s2 Is2 2s1 Is2 = 2.18 x 10 18 x 9 J atom 1
Since, ls-electrons shield 2s-electrons much more = 2.18x 10-18 x 9x 6.02x 1023 J mol'1
effectively than they shield each other, we predict that it y =11.81 x 103 kJ mol-1
should be much easier to remove a 2s electron from Be+
than to remove a Is-electron from Li+ which has inert gas (IE)^=(IE)l+(IE)2+(IE)3
[He] configuration. 1.96x 104 = 520 +x + 11.81X 103
Problem 10. A technique called photoelectron x = (IE)2 = 7.269 x 103 kJ mol-1
spectroscopy is used to measure the IE of atoms. A sample
is irradiated with UV light, and electrons are ejected from Problem 12. Which has the larger lattice energ;
the valence shell. The kinetic energies of the ejected NaCl or CsCl?
electrons are measured. Since the energy of the UV photon
Solution Lattice energy is defined as the energy whe
and the kinetic energy of the ejected electrons are known,
gaseous ions combine to form ionic solid.
we can write
hv = (IE) + - mu2 Na+(g) + Cl~(g)---- > NaCl(s), lattice energy = -<
2 The magnitude of a substance's lattice energy is affecte
where v is the frequency of the UV light, and m and u are . both by the charges on its constituent ions and by the sizf
the mass and velocity of the electron, respectively. In one of those ions. The higher the charges on the ions an
experiment the kinetic energy of the ejected electron from smaller the sizes of the cations and larger the size of th
potassium is found to be 5.34 x 10"19 J using UV source of anion, the larger the lattice energy.
wavelength 162 nm. Calculate IE of potassium Charges on each ion are identical.
(h = 6.62 x 10-34 J s , c = 3 x 108 m s-1). Na+ is smaller than Cs+.
Cr is identical
Solution hv = (IE) + - mu2 Thus, lattice energy of NaCl > CsCl.
2
IE = Av - - mu',2 Problem 13. — atoms of X(g) are converted ini
2 2
he 1 X*(g) by energy atoms of X(g) are converted ini
=------- mu' ,2 2
X 2
(6.62 x IQ"34 x 3x 108 X+(g) and — atoms if X(g) are converted into X~(g) t
-(5.34x 10-19) 2
< 162x 10 energy AH2. Calculate (IE) and (EA) of X(g).
=6.91 x 10"19 J atom-
Solution Let (ZE) of X(g) = Zper atom
=416.54 x 103 J moK1
and (EA) of X(g) = - E per atom
= 416.54 kJ mol’1
(i) X(g) ---- > X+(g) + e-
Problem 11. The energy needed for the following
process is 1.96x 104 kJ mol-1 Energy required to ionise
N
atom of (X)g =w
2
Li (g)---- >Li3+ (g) + Se­
lf the first ionisation energy of lithium is 520 kJ mol-1, 2 1
calculate the second ionisation energy of lithium, that is 2AHj
the energy required for the process 1=
Li*(g)---- >Li2‘(g) + e-
Chapter 3: Periodicity of Elemer

(ii) X(g) ---- > X+(g) + e~, Solution By Pauling’s electonegativity scale
N0I for atoms Xf “ Xh = o.is27ah_f
Energy =
2 2
(Values of BE are in 1
f X(g) + e~ - -> X'(g), A H— F = ^HF “ —H ^F—
~N0E' f No .
Energy = - for —- atoms = 134.6 - ^104.2 x 36.i
2 2
N0I N0E = 72.84 kcal
= AH2
2 2 XF - xH = 0.182V72.84 = 1.553
2AH\ N0E Xf ~ Xh + 1-5534
M) 2N0 = ah2
2 = 3.6034
-N0E Problem 15. (IE)1 and(ZE)2 ofMg(g) are 741
= AH2 - AHj
2 kJ mol-1. Calculate percentage of Mg+(g) and M
r _ 2fAH2 - AHJ
—= —■—-—--- -------------------------------------------------------------- g of Mg(^) absorbs 50.0 kJ of energy.
No
Solution Number of moles of Mg(g) = — = I
24
Ionisation Energy (IE) = atom ,
Energy absorbed in the ionisation of Mg(g)
Electron Affinity (EA) = 0.0417 x 740
2(AH2 - AHJ = 30.83 kJ
atom-1
M) Energy unused = 50- 30.83= 19.17 kJ
19.17 kJ will be used in the ionisation ofMg+(#)
i Problem 14. Calculate the electronegativity of
Hence, number of moles ofMg+(g) converted to
fluorine from the following data :
19 17
H = 104.2 kcal mol-1 = ±±±±=0.0132
1450
Ef_f = 36.6 kcal mol-1 Mg+(g) left as Mg+(^) = 0.0417 - 0.0132 = 0
Eh_f = 134.-6 kcal mol-1 is 4-
Percentage ofmMg +r(g)\ = 0-0285 x 100 = 6$
Electronegativity of H is 2.05.
and percentage of Mg2+(g) = 31.65%
Master Exercises
Exercise 1
(Stage 1: Learning)
Short Answer Type Questions (c) Electronegativities of atoms change wher
1. Comment on the following melting point (given in °C) crossing the d-block of elements than wher
sequences. crossing the p-block.
(d) Theoretical radii of atoms < rmax > from lefl
Cu 1083 Zn 419.5
to right across the /-block in the sixth period, hut
Ag 961 Cd 320.8
real radii across the /-block thar
Au 1064 Hg -38.9 they do across the p-block.
2. For the effective nuclear charge for the last valence (e) Radii of atoms from top to bottom going
electron (n) of the second period down group 2; radii from top to bottom
Z * = 0.65 + 0.65n = 0.65[l + n] going down Group 13(111).
Calculate Z * for Li, Be, B and C.
Analytical Questions
3. Compute the oxidation numbers of each element in
the ions 1. Arrange the following species in order of increasing
(a) HjIO?- (b) S30^“ size:
(c) Na2O2 Ar, K+, CL, S2-, Ca2+
4. Rank the 5s, 5p, 5d, 5f and 5g orbitals in order of 2. Refer to the Periodic Table and identify :
increasing (a) an element that is both in group 15 (IIIA) and ir
(a) penetrating ability the fifth period
(b) shielding ability and (b) an element similar to sulphur
(c) degree to which they are shielded by inner (c) a highly reactive metal in the sixth period
electrons (d) the halogen element in the fifth period
(e) an element with atomic number greater than 5(
5. The single-bonded metallic radius of Na is 157 pm. that is similar to the element with atomi<
Give a reasonable estimate of the following radii, number 18.
assuming that the increment between radii of
3. Match each of the lettered items in the column on t.h<
different magnitudes is 60 pm
left with an appropriate numbered items in th<
(a) the covalent radius of Na column on the right. Some of the numbered items ma.’
(b) the cationic radius of Na be used more than once, and some not at all.
(c) the anionic radius of Na (a) Tl 1. an alkaline-earth metal
(d) the van der Waals’ radius of Na (b) Z =70 2. element in the fifth period and
6. Without the use of tables, arrange each of the group 15 (VA)
following sets in order of increasing size (c) Ni 3. largest atomic radius of all the
(a) Li, C, F, Ne (b) Be, Ca, Ba, Ra elements
(c) B, Al, Ga, In, T1 (d) V, Nb, Ta, Db (d)[Ar]4s2 4. an element in the fourth period
7. Fill in each blank with one of the following; increase; and group 16(VIA)
decrease; stay the same; more rapidly; less rapidly. (e) a metalloid 5.3d8
(a) Electronegativities of atoms from left to (0 a non-metal 6. one p-electron in the shell of
right across a period. highest n
(b) Electronegativities of atoms from top to 7. d-block element
bottom down a group. 8. /'-block element
Chapters: Periodicity of Elements | 141

4. For each of the following pairs, indicate the atom that probable values for each of the following elements,
has the larger size : expressed as greater than, about equal to, or less
(a) Br or As (b) Sr or Mg than the value for Ge.
(c) Ca or Cs (d) Ne or Xe Covalent radius (IE)!
(e) C or 0 (0 Hg or Cl
(a) Ge 122 pm 762 kJ mol
5. Arrange the following in the increasing value of(IE\ : (b) Al
Sr, Cs, S, F, As (c) In
6. Listed below are the electronic configuration of five (d) Se
elements. Arrange the elements in the increasing 14. For the following groups of elements, select the one
order of metallic character :
that has the property noted
(a) [Ar]3d104s•:2 (b) [Ar] 4s2 (a) The largest atom : H, Ar, Ag, Ba, Te, Au
(c) [Ar] 3d10 4s2 4p6 5s2 (d) [Ar] 3d10 4s2 4 p5
(b) The lowest (IE)l : B, Sc, Al, Br, Mg, Pb
(e)[Ar]3d10 4s2 4p6 5s1
(c) The smallest (most negative) (EA)
7. With reference only to the Periodic Table, indicate : Na, I, Ba, Se, Cl, P
which of the atoms Bi, S, Ba, As and Ca : (d) The largest number of unpaired electrons
(a) is most metallic; : F,N,S2’,Mg2+,Sc3+,Ti 3+
(b) is most non-metallic;
15. An element is oxidised by fluorine and not by chlorine.
(c) has the intermediate value when the five are
Could the element be sodium? Aluminum? Sulphur?
arranged in order of increasing first ionisation
energy. Oxygen? Explain your answer.

8. Arrange the following elements in order of decreasing 16. Explain why the most negative oxidation state of an
metallic character : Sc, Fe, Rb, Br, 0, Ca, F, Te. atom of group 15 (VA) element is equal to the number
of electrons required to fill the valence shell of the
9. Which of the following species would you expect to be neutral atom?
diamagnetic and which paramagnetic : K+, Cr3+,
17. Predict which member of each of the following pairs
Zn2+,Cd,Co3+, Sn2+, Br.
will
10. Assuming that the seventh period is 32 members (a) conduct electricity : Ca or S?
long, what should be the atomic number of the noble
(b) form a negative ion : Co or Cl?
gas following radon (Rn)? Of the alkali metal
following francium (Fr)? What would you expect their (c) form a solution of an acid in water N2O3 or CaO?
approximate atomic weights to be? (d) form an ionic compound with : Al or P?
I
11. Use the following system of naming elements in fluorine
which first alphabets of the digits are written (e) form a covalent compound with 0 : K or N ?
I
collectively, (f) have a higher electronegativity : Ba or Br
I
0123456789 3 (g) form a solution of base in water : CO2 or BaO
nil uni bi tri quad pent hex sept oct enn
(h) a base : CsOH or
to write three-letter symbol for the elements with BrOH
atomic number 101 to 109.
(i) an acid : In(OH)3 or
[Example : 101 is Unu ...] B(OH)3
12. Match the following lettered items in the column on : S or Na
(j) an oxidising agent
the left with an appropriate numbered items in the
column on the right. All the numbered items should (k) an ionic compound : RbClorBrCl
be used at least once, and some must be used more Q) more easily reduced : Ca or 0
than once: (m) react as a base : P(OH)3 or
(a) Z = 37 1. two unpaired p-electrons A1(OH)3
(b) Z=9 2. diamagnetic
(n) contain ions : KNO2or
(c) Z = 16 3. more negative electron affinity
CINO,
than elements on either side
(d) Z = 30 of it in the same period. (o) easily hydrated : Li+ or Na+
(e) Z =82 4. lower (ZE)1 than Ca but greater (p) greater (IE)j : Be or B
(f) Z = 12 than Cs.
(q) greater (EA) : ForCr
13. Listed below are two atomic properties of the element (r) positive (EA) : F or O~
germanium. Refer to the Periodic Table and indicate (s) greater (IE), : N or 0
142 | Essential Inorganic Chemistry

(t) max. (IE)! : Na*,Ne+,F\ H2 gas. Element Y is a light-yellow solid and does n o'
O+ conduct electricity. Element Z has a metallic lustun
and conducts electricity. When exposed to air, i'
(u) zero (EN) : Ne or Na slowly forms a white powder aqueous solution o
(v) liquid element : Hg or Zn which is basic. What can you conclude about tin
(w) amphoteric oxide * B0Q3 • AI2Q} elements from these observations?
(x) acidic oxide : B2O3, AI2Q3 21. Listed below are the locations of certain elements ir
(y) basic oxide • N2O5, In2O^ groups and periods of the Periodic Table. Arrange
these elements in the expected order of increasing
(z) harder element : carbon first ionisation energy.
(diamond) or
lead (a) element in the fourth period and group IVA
(b) element in the third period and group VIA
18. Match each of the following elements on the right (c) element in the sixth period and group IIIA
with its description on the left :
(d) element in the second period and group VIIIA
(a) A dark-red liquid I : Ca
(b) A colourless gas that bums in oxygen gas II : Au (e) element in the fourth period and group VIA
(c) A reactive metal that attacks water III : H2 22. The (IE\ and the (IE)2 (in kJ mol-1), of three element
(d) A totally inert gas IV : Ar I, II and III are given below :
(e) A shiny metal that is used in jewellery V : Bi^ I II III
19. You are given four substances: a fuming red liquid, a (IE\ 403 549 1142
dark metallic-looking solid, a pale yellow gas, and a (IE)2 2640 1060 2080
yellow-green gas that attacks glass. You are told that Identify the element which is likely to be :
these substances are the first four members of group (a) a non-metal
VIIA. Name each. (b) an alkali metal
20. A student is given samples of three elements, X, Y, (c) an alkaline-earth metal
and Z, which could be an alkali metal, a member of 23. The element 119 has not been discovered. Wha
group IVA, and a member of group VA. She makes would be the IUPAC name and the symbol for thu
the following observations : element? Write the formula of the most stabli
Element X has a metallic lusture and conducts chloride and oxide.
electricity. It reacts slowly with aq HC1 to produce

Exercise 2
(Stage 2: High Skill Problem Solving)
Only One Option Correct (q) 2E\ 2(2?, - 2^) \ 2Ey 2En
b)—x,—
1. If Aufbau rule is not followed, K-19 will be placed in No ATo
....block. (#! - E2) 2E2
(0 (d) none is correct
(a) s (b) p No ’ Nq
(c)d (d)f
5. Which is maximum hydrated?
2. First, second and third IP values are 100 eV, 150 eV (a)NaCl (b)MgCl2
and 1500 eV. Element can be
(c)A1C13 (d)SiCl4
(a) Be (b) B
(c) F (d) Na 6. Which has maximum polarising power in cation ?
3. Which has maximum ionisation potential? (a) Li* (b)Mg2*
(a) N (b) O (c) Al3* (d)O2-
(c)O+ (d)Na 7. Electron affinity is positive when
4. N0/2 atoms of X(g) are converted into X+(g) by (a) O" is formed from O
energy Ev N0/2 atoms of X(g) are converted into (b) O2" is formed from O-
X~(g) by energy Hence, ionisation potential and (c) O+ is formed from 0
electron affinity ofX (g) are (d) electron affinity is always a negative value
Chapter 3: Periodicity of Elements I 143
8. Following triads have approximately equal size (c) large charges on cation or anion
(a)Na+ , Mg2+ , Al 3+ (iso-electronic) (d) in all the above cases
(b)F-, Ne, er- (iso-electronic) 21. Inert pair effect is shown by
(c) Fe, Co, Ni (a) s-block (b) p-block
2+
(d) Mn + , Fe , Cr (iso-electronic) (c) d-block (d) {-block
9. Ionic radii of 22. Stability of ions of Ge, Sn and Pb will be in the order
(a) Ti4+ < Mn 7 + (b) 35 Cl- < 37 c1- (a) Ge2 + <Sn 2+ < Pb2 + (b) Ge4 + > Sn 4 + > Pb4 +
+
(c) K > c1- (d) p a+ > p5+ (c) Sn 4 + > Sn2 + (d) all are correct
IO. Which of the following ions has the smallest radius? 23. Which is incorrect statement?
(a) Ti2+ (b) pt;2+ (a) Tia � salts are better oxidising agents
2+
(c) Ni (d) zr2
+
(b) Ga + salts are better reducing agents
11. When the following five anions are arranged in order {c) Pb·h salts are better oxidising agents
of decreasing ionic radius, the correct sequence is (d) As5 � salts are better oxidising agents
(a)Se2-,I-,Br-,er-,F- (b)r-, Se2-, er-,
Br-, F- 24. The first e]ement of a group in many ways differs
(c) Se -, 1-, Br-, F-,
2
er- (d) r, Se -, Br-, (f--, F-
2 from the other heavier members of the group. This is
due to
12. For which of the following crystals would you expect (a) the small size
the assumption of anion-anion contact to be valid ? (b) the high electronegativity and high ionisation
(a) CsBr (b) NaF potential
(c) KCI (d) NaI (c) the unavailability of d-orbitals
13. Which of the following ions has the largest heat of (d) all of the above
hydration? 25. Catenation properties of C, Si, Ge, Sn, Pb are in order
(a)Na+ (b) Al a+ (a) C » Si > Ge• Sn >> Pb
(c)F- (d) sr2
+
(b)C<Si< Ge<Sn< Pb
14. Which of the following anions is most easily (c) C > Si > Sn > Ge > Pb
polarized? (d) None of the above is correct
2
(a) c1- (b) Se - 26. Melting points of NaCl, NaBr, Nal and NaF will be in
(c) Br- (d) Te2 - order
15. Of the four t:J1. values needed to calculate a lattice (a)Nal <NaBr<NaCl<NaF
energy using the Born-Haber cycle, the one that is (b)NaF<NaCl< NaBr<Nal
most difficult to measure is (c)NaBr <NaF<NaCl< NaI
(a)the heat of sublimation of the metal (d) NaCl<Nal <NaF<NaBr
(b) the heat of formation of gaseous atoms of the 27 . Melting point is maximum for
non-metal (a) LiCl (b) NaCl
(c) the ionization energy of the metal (c) KCl (d) RbCl
(d) the electron affinity of the non-metal 28. Which is/are amphoteric oxides?
16. A molecule H-X will be 50% ionic if (a) BeO (b) SnO
electronegativity difference of H and Xis (c) ZnO (d) All of these
(a) 1.2 eV (b) 1.4 eV 29. The correct order of increasing ionic character is
(c) 1.5 eV (d) 1.7 eV (a)BeC1 2 < MgC12 <CaC12 <BaC12
17. Which pair is different from the others? (b)BeC1 2 < CaCl2 < MgC12 <BaCI2
(a)Li-Mg (b)Na-K (c) BaC12 < CaC1 2 < MgCI2 <BeC12
(c)Ca-Mg (d) MgCI 2 < CaC12 <BeC12 <BaCI2
(d) B-Al
30. Atomic number 64 will have electronic configuration
18. Which pair is different from the others?
(a) [Xe�4 6s2 4( 8 (b) [Xe14 6s2 4{ 7 5d 1
(a) Li-Mg (b)B -Si
(c) [Xe� 4 4{10 (d) [Xet4 6s2 4/ 7 5p1
(c)Be-Al (d) Li-Na
31. ��8 U(IIIB) changes to �4 Th by emission of a.-particle.
19. Representative elements belong to
Daughter element will be in
(a) s-andp-block (b) d-block
(a) III B (b) I B
(c) d- and {-block (d) {-block
(c) VB (d) II A
20. Covalency is favoured in the following cases 32. Which is correct order of size oro-, er-, F- and F?
(a) a smaller cation
er- > o- > F- > F (b) o- > (j-- > F- > F
er-
(a)
(b) a larger anion
er-
(c) > F- > F > o- (d) > F- > o- > F
144 | Essential Inorganic Chemistry

33. The electronegativities of N, C, Si and P are such that 42. Following are the values of the electron affinities c
(a)P<Si<C<N (b)Si<P<N<C (in kJ mol-1) the formation of 0“ and O2- from 0
(c)Si<P<C<N (d)P<Si<N <C (a)-142,-702 (b)-142, 702
34. The electron affinities of N, 0, S and Cl are such that (c) 142, 702 (d) -142, -142
(a)N<O<S<Cl (b)O<N<Cl<S 43. The ionic radii of O2-, F“, Na* and Mg2* ar
(c) 0 « Cl < N = S (d) 0 < S < Cl < N 1.35,1.34, 0.95 and 0.66 A respectively. The radius c
35. Fluorine has the highest electronegativity among the the Ne atom is
ns2 np5 group on the Pauling scale, but the electron (a) 1.39 A (b) 1.12 A
affinity of fluorine is less than that of chlorine (c) 0.85 A (d) 0.50 A
because
44. The element which does not exist in liquid state a
(a) the atomic number of fluorine is less than that of
chlorine room temperature are
(b) fluorine being the first member of the family (a) Na (b) Br
behaves in an unusual manner (0 Hg (d) Ga
(c) chlorine can accommodate an electron better than 45. The statement that is not correct for the period!
fluorine by utilising its vacant 3d-orbital classification of elements is
(d) small size, high electron density and an increased (a) the properties of elements are the periodi
electron repulsion makes addition of an electron to functions of their atomic numbers
fluorine less favourable than that in the case of (b) non-metallic elements are lesser in number tha
chlorine. metallic elements
36. (A), (B), (C) are elements in the third short period. (c) the first ionisation energies of elements along
Oxide of (A) is ionic, that of (B) is amphoteric and of period do not vary in a regular manner wit
(C) a giant molecule. (A), (B) and (C) will have atomic increase in atomic number
number in the order (d) for transition elements the d-subshells are fillo
(a)(A)<(B)<(C) (b) (C) <(B) <(A) with electrons monotonically with increase i
(c) (A) <(C) <(B) (d) (B)<(A) <(C) atomic number
37. Going down in a group from F to I, which of the 46. Which one of the following has the maximum numbe
following properties decreases? of unpaired electrons?
(a) Ionic radius (b) Ionisation energy (a) Mg2* (b)Ti3*
(c) Oxidising power (d) Electronegativity (c)V3* (d)Fe2*
38. Which of the following will have maximum electron 47. The following acids have been arranged in the orde
affinity? of decreasing acid strength. Identify the correct orde
(a) Is2 2s2 2 p5 (b) ls22s22p6 C10H (I) BrOH(II) IOH(HB
(c) Is2 2s22p6 3s23 p5 (d) Is2 2s2 2 p6 3 s2 3 p6 (a) I > II > III (b) II > I > III
39. Size of cation is smaller than that of the atom because (c)III > II > I (d) I > III > II
of
(a) the whole of the outer shell of electrons is removed 48. The statement is not true for the long form of tb
Periodic Table
(b) effective nuclear charge increases (a) It reflects the sequence of filling the electrons i
(c) due to gain of electrons the order of the sub-energy shells s, p, d and/
(d) statement, that cation is smaller than atom, is (b) It helps to predict the stable valency states of th
wrong elements
40. The sizes of the second and third row transition (c) It reflects trends in physical and chenrid
elements being almost the same. This is due to properties of the elements
(a) d-and /-orbitals do not shield the nuclear charge (d) It helps to predict the relative ionicity of the bor
very effectively between any two elements
(b) lanthanide contraction 49. The electronic configuration
(c) both (a) and (b) are true Is2,2s2 2p6,3s2 3 p6 3c?10,4s2 4 p6 4d10,5s2 is for
(d) None of the above is true (a) /-block element (b) d-block element
41. The factors that influence the ionisation energies are (c) p-block element (d) s-block element
(a) the size of the atom 50. Which of the following statement is false?
(b) the charge on the nucleus
(a) Elements of IB and IIB groups are transitu
(c) how effectively the inner electron shell screen the
elements
nuclear charge
(d) all of the above (b) Elements of VB group do not contain metalloid)
Chapter 3: Periodicity of Elements I 145
(c) Elements of IA and IIA groups are normal 58. Stability order ofgroup IIIA (boron family) element is
elements (a) Ga + < In + < Ti + (b)Ga 3+ > Ga +
(d) Elements of IVE group are neither strongly (c) both (a) & (b) (d) None of these
electronegative nor strongly electropositive.
59. The metal which gives no amphoteric oxide is
51. Transition metals are characterised by the properties (a) Zn (b) Cu
(a) variable valency and tendency to form complex (c) Sn (d) Al
(b)coloured compounds
(c) high melting and boiling points 60. The first ionisation potential (ineV)ofN, 0 atoms are
(d) all of the above (a) 14.6, 13.6 (b) 13.6, 14.6
(c) 13.6, 13.6 (d) 14.6, 14.6
52. Following graph shows variation of ionisation
potential (IP) with atomic number in second period 61. Atomic radii of F and Ne in A are given by
(Li-Ne). Value ofionisation potential (IP) of Na (11) (a) 0.72, 1.6 (b) 1.6, 1.6
will be . (c) 0.72, 0.72 (d) none of these
Ne 62. Match list-I (atomic number of elements) with list-II
(position of elements in Periodic Table) and select the
correct answer using the codes given below the lists

i List I
A. 19
List II
1. p-block
B. 22 2. {-block
C. 32 3. d-block
D. 64 4. s-block
Codes
3 4 5 6 7 8 9 10 11
Z---+ A B C D A B C D
(a) 1 2 3 4 (b) 4 3 1 2
(a) above Ne (b) below Ne but above 0 (c) 4 1 3 2 (d) 2 1 3 4
(c) below Li (d) between N and 0
63. Fluorine does not form any polyhalide as other
53. Based on the fig. in Q. 52, which is incorrect halogens because
statement (a) it has maximum ionic character
(a)11 ofBe >11 ofB but l2 ofBe <12 ofB (b) it has low F-F bond energy (38.5 kcal mor 1 )
(b)l1 ofBe <11 ofB but l2 ofBe <12 ofB (c) of the absence of d-orbitals in the valence shell of
(c) 12 ofBe+ <11 ofB+ fluorine
(d)11 ofBe2+ is abnormally high (d) it brings about maximum coordination number in
54. Which is/are correct order ofionic mobility? other elements
(a)Li + < Na+ < K+ (b) Al 3+ < Mg2+ < Na + 64. Consider the following statements
+
(c) Both (a) and (b) (d) None of these 1. Cs is more highly hydrated than the other alkali
55. M(g) -----+ M + (g) +e-, Ml =100 eV metal ions.
2. Among the alkali metals Li, Na, Kand Rb, lithium
M(g) -----+ M2+ (g)+ 2e-, Ml =250 eV has the highest melting point.
3. Among the alkali metals only lithium forms a
which is incorrect statement?
+ stable nitride by direct combination.
(a)l1 ofM(g)is 100 eV (b)J1 of M (g)is 150 eV
(c)12 ofM(g) is 250 eV (d) 12 ofM(g) is 150 eV Ofthese statements
(a) 1, 2 and 3 are correct (b) 1 and 2 are correct
56. AB is predominantly ionic as A + B- if (c) 1 and 3 are correct (d) 2 and 3 are correct
(a)(lP)A <(IP)8 (b)(EA)A <(EA)8
65. Match list I (atomic number of the elements) with
(c)(EN) A <(EN)8 (d)(lP)8 <(IP)A list II (position in the Periodic Table) and select the
(IP stands for ionisation potential, EA for electron correct answer using the codes given below the lists:
affinity and EN for electronegativity). List I List II
57. EN of the element (A) is E1 and IP is E2. Hence, EA A. 52 1. s-block
will be B. 56 2. p-block
(a) 2E1 - E2 (b)E 1 -� C. 57 3. d-block
(c)E1 -2E2 (d) (E1 + E2 )/2 D. 60 4. {-block
146 j Essential Inorganic Chemistry

Codes 75. The first ionisation potential of Al is smaller thar


A B C D A B C D that of Mg because
(a) 2 1 3 4 (b) 2 1 4 3 (a) the atomic size of Al > Mg
(c) 1 2 3 4 (d) 1 2 4 3 (b) the atomic size of Al < Mg
66. If Aufbau and Hund's rule are not used, then (c) Al has one unpaired electron in p-orbital
incorrect statement is (d) the atomic number of Al > Mg
(a) K't would be coloured ion
76. The non-metallic cation is in
(b) Na will be in same s-block (if these rules are true)
(c) Cu would be s-block element (a)CrO2Cl2 (b)VOCl
(d) magnetic moment of Cr(24) would be zero (c)NH4Cl (d)PCl3
77. Which of the following properties shows a cleai
67. Which set has all the coloured ions?
periodic variation?
(a)Cu\ Cu2+, Ni2+ (b)Cu2*, Fe2+, Co2+
(a) First ionisation energy
(c)Cu2+, Co2+, Sc3+ (d)Na+, Mg2+, Al3+
(b) Molar mass of the element
68. For the element (X), student (A) measured its radius (c) Number of isotopes of the atom
as 102 nm, student (B) as 103 nm and (C) as 100 nm
(d) All of the above
using same apparatus. Their teacher explained that
measurements were correct by saying that recorded 78. Ionisation energy of the element X(g) is I and the
values by (A), (B) and (C) are electron affinity of X+(g) is E then
(a) crystal, van der Waals’ and covalent radii (a)Z = E (b)Z = -E
(b) covalent, crystal and van der Waals’ radii , .T E .,.TE
(c) van der Waals’, ionic and covalent (c) I = — (d)ZI = - —
(d)
2 2
(d) none of the above is correct 79. For the process
69. Which set shows inert pair effect? X(g) + e“---- »X-(g), AZZ = x
(a) Fe, Co, Ni (b) B, Al, Ga
(c) Ge, Sn, Pb (d) He, Ne, Ar and X~(g)---- > X(g)+e~, &H=y
Select correct alternate
70. Gd (64) has unpaired electrons with sum of spin
(a) Ionisation energy of X~(g) is y
(a) 7, 3.5 (b) 8, 3 (b) Electron affinity of X(g) is x
(0 6, 3 (d) 8, 4 (c) Electron affinity ofX(g) is -y
(d) All the above are correct statements
71. Element Unq has atomic number of
(a) 102 (b) 103 80. The most metallic of the following elements is
(c) 104 (d) 112 (a) Mg (b) Li
72. The relative thermal stabilities of alkali metal (c) K (d) Ca
halides are such that 81. The most negative electron affinity of the following
(a) CsCl > RbCl > KC1 > NaCl > LiCl elements is that of
(b) LiCl > NaCl > KOI > RbCl > CsCl (a) Br (b) Sn
(c) CsCl > RbCl < KC1 > NaCl < LiCl
(c) Ba (d) Li
(d) CsCl < RbCl > KC1 < NaCl > LiCl
82. The highest first ionisation energy of the following
73. Consider the isoelectronic ions
elements is that of
K+,S2",Cr andCa2+
(a) Cs (b) Cl
The radii of these ionic species follow the order (01 (d) Li
(a)Ca2+ > K+ > Cl" > S2’ (b) Cl“ > S2’ > K+ > Ca,2; +
(c) S2’ > Cl" > K+ > Ca2+ (d) K+ > Ca2+ > S2‘ > Cl“ : , Sr21 and Ba2+ in the
83. The heat of hydration of Cai2+
decreasing order is
74. Astatine is the last element in halogen group (VII). It
is expected to be (a) Ca2+ > S?+ > Ba2+ (b) Ca2+
(b)Ca 2+ > Ba2+ > S?
(a) more electronegative than iodine (c) S?+ > Ba2+ > Ca2+ (d) Ba2+
(d)Ba 2+ > S?+ > Ca2+
(b) composed of diatomic molecules 84. The compound of vanadium has magnetic moment, ol
(c) limited to an oxidation number of -1 in its 1.73 BM. The vanadium chloride has the formula
compounds (a)VCl2 (b)VCl3
(d) a solid at room temperature (c)VCl4 (d)VCI6
Chapter 3: Periodicity of Elements | 147

85. Which of the following does not represent the correct (a) 1986 (b) 1906
order of the properties indicated? (c) 1908 (d) 1988
(a) Ni2+ > Cr24 > Fe2+ > Mn2+ (size) 96. Element with valence shell-electronic configuration
(b) Sc > Ti > Cr > Mn (size) as (n - Dc^ns1 is placed in
(c) Ni2+ < Co2+ < Fe2+ < Mn2+ (unpaired electron) (a) IA, s-block (b) VIA, s-block
(d) Fe2+ > Co2+ > Ni2+ > Cu2+ (unpaired electron) (c) VIB, s-block (d) VIB, d-block
86. Which of the following order is wrong? 97. Which set does not show correct matching?
(a) NH3 < PH3 < AsH3 — acidic
(a) Sc3+ [Ne]3s23p6 zero group
(b) Li<Be<B<C — (IE\
(c) A12O3 < MgO < Na2O < KjO — basic (b) Fe2+ [Ar] 3d6 VIII group
(d) Li2+ < Na+ < K+ < Cs+ — ionic radius (c) Cr[Ar] 3 cP 4 s1 VIB group
(d) All of the above
87. The ions O2-, F", Na+, Mg24 and Al3+ are 98. Two new elements (discovered in Aug 2003) with
isoelectronic. Their ionic radii show atomic number 113 and 115 are to be placed in
(a) an increase fromO2’ toF- and then decrease from (a) s-block (b) p-block
Na+ to Al3+ (c) d-block (d) /"-block
(b) a decrease from O2- to F" and then increase from 99. Most stable cation of element 113 will be
Na+toAl3+ (a) M3+ (b) M2+
(c) a significant increase from O2" to Al3+ (c) M+ (d) M2+
(d) a significant decrease from O2" to Al3+ 100. Element with atomic number 115 has configuration
88. Ionic radii are as and with most stable cation as......
(a) inversely proportional to effective nuclear charge (a) [Rn]7s25d104/147p3 M3+
(b) inversely proportional to square of effective (b) [Rn]7s25d104/147p3 m5+
nuclear charge (c) [Rn]7s25d104/147p3 M+
(c) directly proportinal to effective nuclear charge
(d) directly proportional to square of effective nuclear (d) [Rn]5d104/147p5 M5+
charge 101. Main group elements constitute:
(a) s-and p-block (b) p-and d-block
89. Sodium forms Na+ ion but it does not form Na2+ (c) s-and d-block (d) d-and/-block
because 102. Elements X and Y have valence shell electron
(a) very low value of (IE\ and (IE)2 configuration as
(b) very high value of (IE)1 and (IE)2 X: ns2np5; Y: ns1
(c) low value of(IE\ and low value of (ZE)2 Which compound is likely formed from X and Y ?
(d) low value of(ZE)1 and high value of (IE)2 (a) X3r5 (b)F3X5
90. The correct decreasing order of ionic radius is (c) XY (d) YX
(a)C4- >N3" >O2“ >F“ (b) F" > 02" > N3" > C4" 103. Recently discovered elements (August 2003) with
(c)(/’>F_ >C1->N3- (d) N3-> 02" > F" > C4" atomic number 113 and 115 have valence electrons in
(a) s-orbital (b) p-orbital
91. First long period contains ....... elements. (c) d-orbital (d) /-orbital
(a) 8 (b) 18
(c) 32 (d) 2 104. Chalcogens are elements of
(a) group 16 (b) p-block
92. Recently (in 2003) element with atomic number 110 (c) ns2np4 configuration (d) all are correct
has been named by IUPAC as
(a) Hs (b) Mt 105. M3+ has electronic configuration as [Ar] 3d10, hence it
(c) Ds (d) Sg lies in
(a) s-block (b) p-block
93. Recently (in Aug 2003) two new elements have been (c) d-block (d) /-block
discovered with atomic numbers
(a) 113,114 (b) 114,115 106. Transition elements have vacant
(c) 115,116 (d) 113,115 (a) s-orbitals (b) p-orbitals
94. Element 113 is produced via (c) d-orbitals (d) /-orbitals
(a) a-decay of element 115 107. Of the following pairs, the one containing examples of
(b) P-decay of element 114 metalloid elements in the Periodic Table is
(c) a-decay of element 111 (a) Na and K (b) F and Cl
(d) P-decay of element 112 (c) Cu and Ag (d) B and Si
95. Numbering of groups as 1, 2, 18 was adopted by 108. Pick out the property which is not shown by
IUPAC in transition elements.
148 | Essential Inorganic Chemistry

(a) Show variable oxidation state (a) Si (b) Ga


(b) Impart colour to flame (c) Bi (d) At
(c) Are paramagnetic in nature 121. Which has maximum stability?
(d) Act as catalytic agents (a) AsCl3 (b) SbCl3
109. Which of the following is a transition element? (c) BiCl3 (d) Equal
(a) Al (b) As 122. Which reaction is most spontaneous?
(0 Ni (d) Rb (a) Ga3* + 2e“ ---- > Ga+ (b) In3*+2e’ ---- > In+
110. Which of the following has the electronic (c) Tl3* + 2e’ ---- > Tl* (d) Cannot be predicted
configuration [Ar] 3d5? 123. With respect to oxygen maximum valency is shown
(a) Cr (b) Fe3* by
(c) Mn (d) V (a) halogen family (b) oxygen family
111. Which of the following forms a stable +4 oxidation (c) nitrogen family (d) boron family
state? 124. Which is best oxidising agent?
(a) Lanthanum (b) Cerium (a) Ge4* (b) Pb4*
(c) Europium (d) Gadolinium (c) Sn4* (d) Sn2*
112. The lanthanides have electron configuration with6s,2: 125. Valence electrons in the element A are 3 and that in
in common but with variable occupation of the element B are 6. Most probable compound formed
(a) 6p-level (b) 5/>level from A and B is
(c) 5d-level (d) 4/-level (a) A^B (b) AB,
113. Which group of elements is analogous to the (c) A6B3 (d) A^B^
lanthanides?
(a) Halides (b) Actinides 126. The relative extent to which the various orbitals
penetrate the electron clouds of other orbitals is
(c) Chalcogenides (b) Borides
(a) s> p> d> f (b) s< p<d<f
114. EC of Gd (64) is written as (c) s<d< p< f (d) d<s< p< f
(a) [Xel;, 4f15d'6s.22 (b) [Xe^ 4fa6s.2
127. Screening effect is not observed in
(c) [Xe^ 4/'96s1 (d) [Xe^ 4/10
(a) He* (b) Li2*
115. Recently discovered element with atomic number 115
(c) Be3+ (d) in all cases
is
(a) Uun (b) Uub 128. In which case effective nuclear charge is minimum?
(c) Uup (d) Uus (a) Be (b) Be2*
116. In Periodic Table, metallic elements appear (c) Be3* (d) Equal
(a) in the left-hand columns 129. ForBe,Zefr = 1.95and for Bex*,ZelT = 2.30.Hence, ion
(b) in the top rows
is
(c) in the right-hand columns
(d) in the bottom rows (a) Be* (b) Be2*
(c) Be3* (d) Be0
117. The atoms of the elements belonging to the same
group of the Periodic Table will have 130. For Cu, effective nuclear charge felt by a 4s-electron
(a) the same number of protons is
(b) the same number of electrons in the valence-shell (a) 2.2 (b) 2.95
(c) the same number of neutrons (c) 1.0 (d) 1.65
(d) the same number of electrons 131. State, which one of the following has the largesl
118. Which will have graded property similar to EC atomic radius?
Is2 2s2 2p6 3s2 3 p6 4s1? (a) Cs (b) Mg
(a) [Ar] 3d10 4s1 (b) [Kr]4d105sl (c) Ba (d) Cr
(c) [Kr]5s] (d) All of these 132. Which one of the following has the smallest atomi*
119. Recently discovered element with atomic number 115 radius?
has configuration (a) F (b) Cl
(a) ns2np3 (d) ns2np1 (c) Cs (d) Mg
(c) ns2np5 (d) ns2npi 133. Covalent radius of nitrogen is 70 pm. Hence, covalen
radius of boron is about
120. Element with atomic number 113 has been reported
(a) 60 pm (b) 110 pm
in August 2003. Its electronic configuration is similar
(c) 50 pm (d) 40 pm
to that of
Chapter 3: Periodicity of Elements I 149
134. In which case bond length is shortened? (c) electron is attracted more by the core electrons
(a) When multiplicity occurs between atoms (d) none of the above is the correct explanation
(b) When electronegativities are different 142. SI unit of IE is
(c) In both cases (a) kJmo1- 1 (b) eVatom- 1
(d) In none of the cases (c) Jmo1-
1
(d) kcal moi- 1
135. Select correct statement about radius of an atom 143. Which has maximum IE?
(a) Values of van der Waals' radii are larger than (a) Mg (b) Mg +
those of covalent radii because the van der Waals'
forces are much weaker than the forces operating (c) Mg2 + (d) Equal
between atoms in a covalently bonded molecule 144. Which pair represents incorrect first (IE)?
(b) The metallic radii are smaller than the van der (a) Be > B (b) N > 0
Waals' radii, since the bonding forces in the (c) Li> Na (d) He> He+
metallic crystal lattice are much stronger than
145. Which represents alkali metals based on (IE )i and
the van der Waals' forces
(c) Both (a) & (b) are correct (IE )i values?
(d) None of the above is correct (IE)i (IE)2
136. The separation of lanthanides in ion-exchange (a) X 100 110
method is based on
(a) basicity of the hydroxides (b) y 95 120
(b) size of the hydrated ions (c) z 195 500
(c) size of the unhydrated ion (d) M 200 250
(d) the solubility of their nitrates
137. Select correct statement(s). 146. Which element has the highest first (IE)?
{a) Across a transition series, there is only a small (a) N (b) Ne
decrease in atomic radius from one element to (c) He (d) H
another due to very small increase in effective 147. IE of an element does not depend on
nuclear charge (a) its nuclear charge (b) the shielding effect
(b) The rate of decrease in the size across the (c) electron neutrality (d) penetration effect
lanthanide series is less than the across the first
transition series 148. The dominant factor in determining the IE of the
elements on moving down the groups is its
(c) Both (a) & (b) are correct statements
(a) atomic radius
(d) None of the above statement is correct
(b) effective nuclear charge
138. Select incorrect order of size of ions/atoms (c) both (a) and (b)
(a) r+ < I < :r (b) Fe = Co = Ni (d) none of the above
(c) Ni < Cu < Zn (d) None is incorrect
149. Select correct statement.
139. Select correct alternate. (a) (IE) 1 of the corresponding elements of 3d and 4d
(a) Due to lanthanide contraction Zr and Hf have series are almost similar
almost equal size
(b) (IE)1 of 5d-series elements are smaller than that of
(b) Due to completion of 3d-subshell, the electronic
3d and 4d-series elements
charge denisty in this subshell becomes very high
which increases the inter-electronic repulsion (c) both (a) & (b) are correct
hence, size increases (d) both (a) & (b) are incorrect
(c) Both (a) & (b) are correct statements 150. Higher values of ionisation energies of the
(d) Both (a) & (b) are incorrect statements 5d-transition elements are consistent with the
140. The lanthanides contraction refers to (a) relatively smaller effective nuclear charge
(a) ionic radius of the series (b) relatively smaller size of their atoms
(b) valence electrons of the series
(c) relatively smaller penetration
(c) the density of the series
(d) electronegativity ( d) all the above are correct

141. The second ionisation energy is always higher than


the first ionisation energy because the One or More Than One Options Correct
(a) ion becomes more stable attaining an octet or 1. Stability of ions of Ge, Sn and Pb will be in order
duplet configurations (a) Ge2 + < Sn2 + < Pb2 + (b) Pb4 + < Sn4 + < Ge4 +
(b) electron is more tightly bound to the nucleus in an
(c) Sn2 + < Sn4 + (d) Pb4 + < Pb2 +
ion
150 | Essential Inorganic Chemistry

2. The first element of a group in many ways differs (c) the difference in electronegativity between Rb
from the other heavier members of the group. This is and Br is smaller than the difference between Na
due to and F.
(a) the smaller size (d) the intemuclear distance, rc + ra is greater for
(b) the high electronegativity and high ionisation RbBr than for NaF
potential 9. Select correct statement(s).
(c) the unavailability of d-orbitals (a) Cs+ is more highly hydrated than the other alkali
(d) the higher shielding effect metal ions
3. The factors that influence the ionisation energies are (b) Among the alkali metals Li, Na, K and Rb, lithium
(a) the size of the atom has the highest melting point
(b) the charge on the nucleus (c) Ionic mobility ofLi+ is maximum among Li+, Na ’
(c) how effectively the inner electrons screen the and K* ions
nuclear charge (d) Ionisation potential of Li is smaller than that of
(d) the atomic mass Na
4. Which is/are correct order of ionic mobility? 10. Select correct statements.
(a) Li* < Na* < K+ • (b) Na+ < Mg2* < AI3* (a) Ce, Pr and Nd are /-block elements
(c) Al3* < Mg2* < Na* (d) K* < Na* < Li* (b) Cu, Pd and Ni are d-block elements
5. Consider following ionisation steps: (c) Cu([Ar]3d104s1) and KffArHs1) have been placed
M(g) ---- > M*(g) + e'; AH = 100 eV in s-block
M(g) ---- > Af2*(g) + 2e~; AH = 250 eV (d) Si, Ge and As are metalloids
Select correct statement(s).
(a) (ZE)iofAf(^)is 100 eV Brain Twisters
(b) (ZE)jofM*(g)is 150 eV 1. Let us suppose that you have discovered a new
(c) (ZE)2 of M(g) is 250 eV element of atomic number 162, which you choose to
(d) (ZE)2 of Af (g) is 150 eV name khalidium (Kh). Assume that the normal order
of filling of orbitals still prevails in the eighth period.
6. An increase in both atomic and ionic radii with (a) Write the full electron configuration of this
atomic number occurs in any group of the Periodic element.
Table and in accordance with this the ionic radii of (b) Write the valence electron configuration of this
Ti(lV) and Zr(IV) ions are 0.68 A and 0.74 A element and of its +2 ion.
respectively; but for HfllV) ion the ionic radius is 0.75 (c) Note that a new type of orbital is used in Kh that
A which is almost the same as that for Zr(IV) ion.
is not used in any known atom. How many nodal
This is due to
planes will there be in a sketch of this type of
(a) greater degree of covalency in compound of Hf4* orbital? How many nodal spheres will there be?
(b) lanthanide contraction Will the electrons in this type of orbital be good tor
(c) difference in the coordination number of Zr4* and poor at shielding the valence electrons of Kh? Will
Hf4r in their compounds electrons in this type of orbital be good or poor ai
(d) actinide contraction penetrating the inner orbitals of Kh?
(d) Calculate Z * fo the outermost valence electron ol
7. When an isotope undergoes K-capture, its mass Kh.
number (e) Using an estimated covalent radius of 150 pm
(a) remains the same while the atomic number calculate the Allred-Rochow electronegativity o'
increases by one Kh. Do you think that the Pauling
(b) remains the same while the atomic number electronegativity of Kh will be higher, lower, oj
increases by two about the same as this value?
(c) remains the same while the atomic number
2. How many joules of energy must be absorbed b<
decreases by one
(d) as well as the atomic number decreases by one convert to Li* all the atoms present in 1.00 mg o
gaseous Li? (IE)1 of Li is 320.3 kJ mol-1. (Li = 7)
8. The melting point of RbBr is 682°C while that of NaF
is 988°C. The principal reason that the melting point 3. How much energy in kJ must be absorbed to ioni&
of NaF is much higher than that of RbBr is that completely all the third shell electrons in a mole ol
(a) the molar mass of NaF is smaller than that of gaseous phosphorus atoms?
(ZE)! = 1012 (ZE)2 = 1903 (ZE)3 = 2912
RbBr
(ZE)4 = 4957 (IE)t = 6274 (values in kJ mol'1)
(b) the bond in RbBr has more covalent character
Also express result in eV per atom.
than the bond in NaF
Chapter 3: Periodicity of Elements | 151

4. Use (IE) and (EA) listed below to determine whether Passage 2


the following process is endothermic or exothermic : Following rule in general, classifies p-block elements
Mg(g) + 2F(g) ---- > Mg24(g) + 2F-(g) into metals, non-metals and metalloids (where P is the
(ZE), ofMg(g) = 737.7 kJmol-1 period and TV the valence electrons)
(IE)2 of Mg(g) = 1451 kJ mol"1 (P +1) > N, the element would be metal
(P + 1) < N, the element would be non-metal
(EA) of F(g) = - 328 kJ mol"1
(P + 1) = TV, the element would be metalloid
5. What is the value in kJ mol-1 which is equal to Answer the following questions
1 eV atom-1?
1. Metalloid will be out of elements with atomic number
6. A mixture contains F and Cl atoms. The removal of
13,14,15,16
an electron from each atom of the sample requires
(a) 13 (b) 14
284 kJ while the addition of an electron to each atom
(c) 15 (d) 16
of the mixture releases 68.8 kJ. Determine the mole
% composition of the mixture. 2. Metals will be out of 13, 31, 35, 54
(a) 13, 31 (b) 31, 35
(IE) per atom (EA) per atom
(c) 35, 54 (d) 13, 54
F 27.91 x 1CT22 kJ -5.53 x 10'22 kJ
3. Non-metals will be out of 13, 31, 35, 54
Cl 20.77 x 10’22 kJ -5.78x 10’22 kJ
(a) 13,31 (b) 31,35
7. A hydrogen like ion is an ion containing only one (c) 35, 54 (d) 13, 54
electron. The energies of the electron in a hydrogen 4. Which will find use in semi-conductors?
like ion are given by (a) 5, 13 (b) 5, 14
En = -(2.18x10'l"18)Z2
(l)joule (c) 13,14 (d) 14, 16

Passage 3
where, n is the principal quantum number and Z is
the atomic number of the element. Calculate the In the following table Zp Z2 and Z3 of the main group
ionisation energy (in kJ mol"1) of the He* ion. elements in 2nd period have been given.
8. Plasma is a state of matter consisting of positive Li Be B C N o F Ne
gaseous ions and electrons. In the plasma state, a
mercury atom could be stripped of its electrons and 513 899 801 1086 1402 1314 1681 2080
therefore would exist as Hg80*. ^2 7298 1757 2427 2352 2856 3388 3374 3952
^3 11815 14848 3660 4620 4578 5300 6050 6122
Calculate energy required for the step
Hg,9*(g) —> Hg80+(g) + e~ Answer the following question based on the above
9. The atomic radius of K is 216 pm and that of K* is table.
133 pm. Calculate the per cent decrease in volume 1. There is increase in ionisation energy along a period
that occurs when K(g) is converted to K*(g). going Li to Ne, butZj of B is less than that of Be. This
is due to
Passage Comprehension Questions (a) valence electron in B is in 2p-orbital of higher
energy than that of Be in 2s-orbital of lower
Passage 1 energy
Following questions are based on Sc(Z = 21) (b) removal of paired electron in Be requires higher
energy than that of removal of unpaired electron
1. Out of Sc3+, Sc2* and Sc* paramagnetic as well in B.
coloured ions are (c) both (a) & (b) above
(a) Sc*, Sc2* (d) None of the above
(b) Sc*,Sc3*
(c) Sc2*,Sc3* 2. We
------------------------- that Zj! „of Be > Zj of B
observe--------------
(d) all being d-block element butZ2 ofBe< Z2 of B.
2. For any Is electron, Z * (effective nuclear charge is) Select correct explanation(s).
(a) 21.0 (b) 20.65 (a) Electron in Be in paired while in B, electron is
(c) 16.85 (d) 9.75 unpaired, hence of Be > Zj of B
3. For3d-electron, Z * is (b) Electron in Be* is unpaired while in B*, electron is
(a) 20.65 (b) 16.85 paired, hence Z2 of Be < Z2 of B.
(c) 9.75 (d) 3.00 (c) Both (a) & (b) above
(d) None of the above
152 | Essential Inorganic Chemistry

3. These occurs a fall while going from N to O. 3. Which represents a noble gas?
It is due to (a) A (b) B
(a) stable configuration of N | T | T | T |
(c) C (d) D
(b) higher atomic number of O 4. Which of the above elements forms a stable binary
halide of the formula MX2?
(c) stable configuration of O |
(d) lone pairs on O. (a) A (b) B
(c) C (d) D
Passage 4
The sums of first and second ionisation energies and Passage 6
those of third and fourth ionisation energies (in MJ mol-1) The heats of formation (A//r°) of the oxides of the third
of nickel and platinum are sndium to
nd sodium
period, chlnrinn are
tn chlorine, mnl-1
arp in kJ mol

(/gh + (/g)2 (/g)3 + (g)4 Na.,0


IN d 2^- MgO A12O3 SiO'22 P4OI0 so3 cia
Ni 2.49 8.80 -416 -602 - 1676 -911 -2984 -395 +250
Pt 2.66 6.70
Based on these data, answer the following questions
Based on this information, answer the following 1. Which oxide has maximum negative heat of
questions. Name of the metal (Ni or Pt) which can more formation per oxygen atom?
easily form compounds in its +4 oxidation state.
(a) P4O10 (b) A12O3
1. Most common oxidation states of Ni and Pt are (c) Na2O (d) MgO
respectively
(a) +2, +2 (b) +4, +2 2. Most stable and least stable oxides are
(c) +2, +4 (d) +4, +4 (a) P4O10,Cl2O7 (b) Na2O,Cl2O7
2. Which is most stable complex of Ni? (c) MgO,Cl2O7 (d) Cl2O7,MgO
(a) [Ni(CN)4f- (b) [Ni(CN),f- 3. Stability order of the above oxides is
(c) [Ni(CN)3r (d) [Ni(CO)6] (a) C12O7 < SO3 < P4O10 < SiO2 < A12O3 < MgO
3. Which is least stable complex of Pt? < Na2O
(a) KolPtClg] (b) [Pt(en)3]Cl4 (b) C12O7 < SO3 < P4O10 < Na2O < SiO2 < A12O3
(c) K^PtClJ (d) K2[Pt(C2O4)3l < MgO
(c) C12O7 < SO3 < Na2O < MgO < SiO2 < A12O3
Passage 5 < P4O10

The (IE\ and the (IE)2 in kJ mol of a few elements (d) P4O10 < A12O3 < SiO2 < MgO < Na2O < SO3 < Cl207
designated by Roman numerals are shown below:
Passage 7
Element
(1E1. (/H)2
Consider following table comparing ionic radius
A 2372 5251
lon-> N3’ O2‘ F'
F’ Na+ Mg2*
B 520 7300
Number of electron 10 10 10 10 10
C 900 1760
Number of nuclear protons 7 8 9 11 12
D 1680 3380
Ionic radius (pm) 146 140 133 98 79
Based on above information, answer the following Answer the following questions.
questions:
1. Which of the above elements is likely to be a reactive 1. Select correct alternate(s) in terms of size.
metal? (a) Na > Na+ (b) Mg > Mg+ > Mg2+
(c) F“>F (d) tf->O->O
(a) A (b) B
(c) C (d) D 2. All the above species are isoelectronic but they differ
in size. It is due to
2. Which of the above elements is likely to a reactive
(a) increase in number of protons
non-metal?
(b) removal of valence shell
(a) A (b) B (c) decrease in repulsive force
(c) C (d) D (d) addition of additional shell
Chapter 3: Periodicity of Elements | 153

Passage 8 6. Assertion (A): Sc(Z = 21) is placed in d-block


The singly-bonded metallic radius of Na is 157 pm. elements.
Assume that the increment between radii of different Reason (R): Last filling electron goes into
magnitudes is 60 pm. Answer the following questions. 3d-suborbit.
1. The covalent radius of Na is 7. Assertion (A): Ionisation energy decreases on
(a) 157 pm (b) 97 pm moving down the group.
(c) 217 pm (d) 267 pm
Reason (R): Force of attraction between electrons
2. Ionic radius of Na+ is
and nucleus decreases on moving down the group.
(a) 157 pm (b) 97 pm
(c) 217 pm (d) 267 pm 8. Assertion (A): Li and Mg shows diagonal
3. Ionic radius of Na" is relationship.
(a) 157 pm (b) 97 pm Reason (R): Li and Mg are diagonal to each other
(c) 217 pm (d) 267 pm in the Periodic Table.
4. van der Waals’ radius of Na is 9. Assertion (A): Fluorine has a less negative
(a) 157 pm (b) 97 pm electron affinity than chlorine.
(c) 217 pm (d) 267 pm
Reason (R): There is relatively greater
effectiveness of 2p-electrons in the small F atom to
Assertion & Reason repel the additional electron entering the atom than
Codes: to 3 p-electrons in the larger Cl atom.
(a) Both A and R are true and R is the correct 10. Assertion (A): SnCl2 is a reducing agent.
explanation of A.
(b) Both A and R are true but R is not the correct Reason (R): SnCl2 reduces FeCl3 to FeCl2 and
explanation of A. HgCl2 to Hg.
(c) A is true but R is false.
(d) A is false but R is true.
True & False
1. Assertion (A): Tin exhibits +2 and +4 oxidation
states only and not +3 oxidation state. 1. Electron affinity of Cl is greater than that of F.

Reason (R): Inert pair effect of electrons in the 2. A12O3 in aqueous NaOH forms Na[Al(OH)4].
s-orbital is responsible. 3. CO is an acidic oxide.
2. Assertion (A): Na+ and Al3+ are isoelectronic but 4. PbO2 is a reducing agent.
the magnitude of ionic radius of Al3+ is less than that
ofNa+. 5. Ionic mobility of Li+ is greater than that of Na+.
Reason (R): The magnitude of effective nuclear 6. BgOg is an amphoteric oxide.
charge of the outershell electrons in Al3+ is greater
7. ZnO is amphiprotic.
than that inNa+.
8. LiCl can be used to control humidity.
3. Assertion (A): The atomic radii of the elements of
the oxygen family are smaller than the atomic radii of 9. (IE)2 of H is less than that of He.
the corresponding elements of the nitrogen family. 10. First long period has 18 elements.
Reason (R): The members of the oxygen family 11. Na2CO3 is thermally stable butLi2CO3 is not.
are more electronegative and thus have lower values
of nuclear charge than those of the nitrogen family. 12. Atomic size of Fe > Co > Ni.

4. Assertion (A): PbCl2 is more stable than PbCl4. 13. Atomic size of Ne > F.
14. (IE)2 of Li is maximum in the Periodic Table.
Reason (R): PbCl4 is powerful oxidising agent.
15. The noble gas core together with (n - l)d10 electrons
5. Assertion (A): The mobility of sodium ion is lower
than that of potassium ion. is called pseudo-noble gas core.
Reason (R): The ionic mobilities depend upon the 16. Helium is the most unreactive element of the Periodic
effective radius of the ion. Table.
154 I Essential Inorganic Chemistry

Fill in the Blanks 2. Match the type of elements (in List I) with the
corresponding electronic configuration (in List II).
1 . ............ is the lightest of all metals.
List I List II
2............. is the only non-metallic element which is 1
A. Inert gas elements 1. ns to ns np 52
liquid at room temperature.
B. Main group elements 2. 1s 2 to ns 2 np 6
3.Two atoms of similar electronegativity would be C. Transition elements 3. (n _ 2),1-1•
expected to form a ............ compound. (n - 1)s 2 p6d1o ns 2
D. Inner transition elements 4. (n - 1)d 1-9ns2
4. Of the halide ions, ............ is the most powerful 5. {n - 1)d1o ns 2
reducing agent.
3. Match the classification (in List I) with the element
5. As the atomic radius increases, ionisation energy
(in List II).

List I -List I
6. The element having electronic configuration
p2
ls2 2s2 2 p6 3s2 3 has valency ............ A. Alkaline earth metal 1. Rb
B. Radioactive element 2. As
7.The substance which loses electrons is said to be
C. Alkali metal 3 Mg
....•....... and the substance which gains electrons is
D. Metalloid 4. Fr
said to be ............

8.Elements of group ............ have greater tendency to 4. Match the element (in List I) with its unique
form positive ions than elements of group IIA. properties (in List II).

9.An element which can exist as a positive ion in acidic List I List II
solution and also as a negative ion in basic solution is A. F 1. Maximum ionisation energy
said to be ............ B. Cl 2. Maximum electronegativity

10.On Mulliken scale, the average of ionisation energy C. Fe 3. Maximum electron-affinity


and electron affinity is called ........... . D. He 4. Recently named by IUPAC
E. Ds 5. Variable valency
11............. is the most electronegative element in the
Periodic Table. 5. Match the atomic number (in List I) with its IUPAC::
nomenclature(in List II).
12. Ionic mobility of Al 3+ is ............ than that of Na + .
List I List II
13.(IE) i of Be is ............ than that ofB but([E)2 ofBe is
A. 105 1. Uuh
............ than that of B.
B. 107 2. Uun
14. Electron affinity ofF ............ than that of Cl. C. 109 3. Uns
15.In aqueous solution ............. is the best reducing D. 110 4. Unp
agent among alkali metals. E. 116 5. Une

II. One or More Correct


Matrix-Match Type Questions
1.Match the element in Column I with its
I. Only One Correct corresponding property (ies) in Column II.
1. Match atomic number of elements (in List I) with Column I Column II
position ofelements in the Periodic Table(in List II).
A. F 1. Maximum electron-affinity
List I Llstll B. Cl 2. Maximum electronegativity
A. 19 1. p-block C. Br 3. Exists as X2 in liquid state
B. 22 2. f-block D. 4. Exists as X2 in solid state

C. 32 3. d-block E. At 5. X2 sublimates on heating


4. s-block 6. X2 is the best oxidising agent
D. 64
7. Radioactive element
Chapter 3: Periodicity of Elements | 155

2. Match the oxide in Column II with its corresponding 5. An ionic compound is formed of the type XY from X
property (ies) in Column II. with electronic configuration ns1 and Y with valence
Column I Column II shell electrons
A. Na2O 1. Soluble in HCI
6. Maximum oxidation state shown by chalcogens is
B. MgO 2. Soluble in NaOH
C. AI2O3 3. Turns blue litmus red
D. SiO2 4. Turns red litmus blue 7. Ionisation energy of M3+ is 217.6 eV compared to that
E. P2O5 5. Giant molecule of He+ (54.4 eV). Thus, atomic number of Af3+ is
F. SO3 6. Acid-rain

8. Semi-conductor with minimum value of atomic


Integer Answer Types number is
This section contains 8 questions. X y Z w
The answer to each of the questions is © © © ©] Test Yourself on Periodic Table
a single digit integer, ranging from 0 to © © O o 1. Test your knowledge of the Periodic Table by
9. The appropriate bubbles below the © © © ©
respectively question numbers in the © ©. © indicating where on the blank outline elements that
ORS have to be darkened. For
©
© © © © meet the following descriptions appear:
example, if the correct answers to
question number X, Y, Z and W (say)
© © © © (a) Elements with the valence-shell ground-state
are 6,0, 9 and 2, respectively © © © © electron configuration ns2 np5
© © © © (b) An elements whose fourth shell contains two
Set I © © ©. © p-electrons
© © © © (c) An element with the ground-state electron
1. Magnetic moment of an element
is 1.73. Thus, unpaired electron can be .... configuration [Ar] 4s2 3d10
2. Maximum oxidation state shown by Mn is
, ™ (ZP) + (EA)
o. Electronegativity =----- —----- .

This N =
4. Stability of T1 is maximum when its valency is
5. Catention is maximum for the element with atomic
number
6. Lanthanides have been placed in period 2. Circle the approximate part or parts of the Periodic
7. Actinides have been placed in period Table where the following elements appear :
8. Number of unpaired electron in Gd(64) is (a) Elements with the smallest values of (IE)l
(b) Elements with the largest atomic radii
Set II (c) Elements with the most negative values of EA
1. Effective nuclear charge of an element with three
valence electrons in 2.60. Thus, atomic number of the
element is
2. Number of amphoteric oxides out of Na2O, MgO,
Al2O3,SnO2,BeO2 an^ CaO is.............
3. Maximum electron affinity is shown by the species at
serial number.
(1) 0 (2) F (3) Cl" (4) He (5) N
3. Test your knowledge of the Periodic Table by
4. Diatomic molecule is formed by the element at serial
number (1) and at serial number indicating approximately where on the blank outline
(1) [Ne]3s23p1 (2) [NeJSs1 the following elements appear :
(3) [Ne]3s2 (4) [He]2s22p:.3
(5) [Ne]3s23p6 (6) [Ne]
156 | Essentia/ inorganic Chemistry

ncl
DI

(e) Which of these oxides has the highest melting


point?
Which has the lowest melting point?
(a) Main groups (b) Halogens
(c) Alkali metals (d) Noble gases 5. Look at the location of elements A, B, C, and D in the
(e) Alkaline earths metals (f) Group 3A elements following Periodic Table:
(g) Lanthanides (a) Write the formula of the simplest binary hydride
of each element.
4. Look at the location of elements A, B, C, and D in the (b) Classify of these hydride as ionic, covalent or
following Periodic Table:
interstitial.
(a) Write the formula of the oxide that has each of (c) What are the oxidation states of hydrogen and
these elements in its highest oxidation state.
other elements in the hydrides of A, C and D?
(b) Classify each oxide as basic, acidic, or amphoteric.
(c) Which oxide is the most ionic? Which is the most
covalent?
(d) Which of these oxides are molecular? Which are ©
solids with an infinitely extended S
three-dimensional crystal structure?
Answers & Solutions
Master Exercises
Exercise 1
Short Answer Type Questions 4. (a), (b)5g<5{<5d<5 p <5s
1. Melting point is a reflection of the attractive force which (c) 5s<5 p<5d<5{ <5g
holds the constituent particles of the solid element. In
5. (a) 157 pm (b) 97 pm (c) 217 pm (d) 217 pm
metals, this force depends primarily on the size and
nature of packing of the atoms in the crystal and on the 6. (a) F<C<Li< Ne (Ne has van der Waal's radius)
contribution of valence shell electrons in bonding. The (b) Be<Ca <Ba <Ra
lower melting point of Ag suggests that d-electrons in
(c) B < Al = Ga<In "'Tl
metallic bonding are involved to a smaller extent
compared to Cu and Au. This is confirmed by the fact (d) V< Nb = Ta = Db
that common oxidation of Ag is +1 while that of Cu and 7. (a) increase (b) decrease (c) less rapidly
Au are +2 and +3 respectively. (d) stay the same; decreases; less rapidly
The lower melting points of Zn-Cd-Hg in comparison (e) increase; increase, less rapidly
to Cu-Ag-Au also indicate that there is contraction of
d-orbital from increasing nuclear charge; in Hg this is Analytical Questions
reinforced by still poor shielding by the {-electrons.
1. Ca 2 + < K + <Ar< c1- < s 2-
2. This is general formula to calculate Z * directly by use of
valence electrons n. 2. (a) In (b) C (c) Cs (d) I (e) Xe
3. (a) 6 (b) 8 (c) 7 (d) 1 (e) 2 (f) 4
Element z EC
n (valence
electrons)
Z • = 0.65[1 + n] 4. (a) As (b) Sr (c) Cs (d) Xe (e) C (f) Hg
5. Cs < Sr < As < S< F
Li 3 1s 22s 1 1 1.30
6. (d) < (a) < (b) < (c) < (e)
Be 4 15 22s 2 2 1.9 5 7. (a) most metallic : Ba (b) most non-metallic : S
B 5 1s 22s 22p 1 3 2.60 (c) intermediate (IE� : Bi
8. Rb > Ca > Sc > Fe > Te > Br > 0 > F
C 6 1s 22s 22p 2 4 3.25
9. Diamagnetic (which does not have unpaired electrons):
K + , Zn2+ , Cd� Sn 2+
3. (a) H being least electronegative, its oxidation number
is taken as = + 1. Paramagnetic (which has unpaired electrons) :
and O being most electronegative, its oxidation number Cr3 •, Co3+
is taken as = -2. Take oxidation number of I =x 10. Noble gas has Z = 118 and alkali metal Z = 119; atomic
Thus, H2IO�- weight should be about 310.
j i 11. 101 102 103 104 105 106 107 108 109
+2 + X -12 =-3 Unu Unb Unt Unq Unp Unh Uns Uno Une
x=+ 7
12. (a) 4 (b) 3 (c) 1 (d) 2 (e) 1 (0 2
(b) Oxygen being more electronegative than sulphur is
assigned oxidation number = -2 13. Covalent radius (/E�
and oxidation number of sulphur = x (b) Al 125 pm> Ge 577< Ge
Thus, S3o:- (c) In 1 50 pm> Ge 558 < Ge
(d) Se 114 �m < Ge 941 > Ge
·3x -12 =-2
X =3.33 14. (a) Ba (b) Al (c) Cl (d) N
15. Oxygen is more electronegative than chlorine and
+2 + 2x =0 cannot be oxidised by it.
x=-1
158 | Essential Inorganic Chemistry

16. Outer shell electronic configuration : ns2np3 19. a fuming red liquid — Br2
(Three electrons less than octet) a dark metallic looking solid -I2
Number of electrons required to fill the valence shell = 3 a pale yellow gas — Cl2
Hence, X3~ has configuration = ns2npe yellow green gas -F2
X3~ has stable nearest inert gas configuration hence 20. X: Sn or Pb, Y: P, Z: alkali metal
most negative oxidation state = - 3
21. (a) Ge, (b) S, (c)Tl, (d)Ne,
17. (a)Ca (b) Cl (c)N2O3 (d)Al (e)N (OBr (g)BaO
(e) Se c <a <e <b <d
(h) CsOH (i) B(OH)3 (j) S (k) RbCl (1) 0 22. (a) a non-metal : III
(m)Al(0H)3 (n)KN02 (o)Li+ (p) Be (q) F (b) an alkali metal :I
(r)O" (s)O (t)Na* (u)Ne (v)Hg (w)A12O3 (c) an alkaline earth metal : II
(x)B2O3 (y)In2O3 (z) carbon (diamond)
23. IUPAC name : Uue, Chloride : MCI, Oxide : Af2O
18. (a)-V (b)-III (0 — 1 (d) —IV (e) —II

Exercise 2
Only One Option Correct 5. Smaller cation, layer charge, larger hydration
1. K-19 (in absence of Aufbau rule): Size of cation Na+ > Mg2+ > Al3+> Si4+ Minimum size
Charge----------------------------------► Maximum charge
ls22s22p63s23p6
Thus, SiCl4 is maximum hydrated.
last-filling electron goes into d-orbital. Thus, d-block Thus, (d)
element. 6. (c) 7. (b)
Thus, (c)
8. Repulsive force are balanced by attractive force, thus, (c)
2. I3 is very high indicating +2 oxidation state is most
9. (d) 10. (c)
stable. Thus, element is of group 2.
11. 0 F F<0 I > Br > F
Thus, Be.
Thus, (a) S Cl Thus, r < O2" F > Br" > F"
N(7) ' Se Br Se> 0
3. 1s22s2 2p*2p*2p’
and 0+(8)
Maximum multiplicity most stable I Se2" > O2"

0] ls22s22p22p^2pj I>Se
(IP) is dependent on the value of Z. r>Se2"
ZofO+(8)>ZofN(7) Thus, I" > Se2-> Br" > O2" > F"
Thus, (IP) of 0+ > N Thus, (d)
Na with one valence electron has least IP 12. (d)
Thus, (c) 13. Maximum hydrated ion is Al3+.
4. Let IP = I Thus, (b)
EA=E 14. Smaller cation, larger anion, maximum polarising
X(g) ---- > X+(g) + e~ power
^I=Ei Thus,Te2"
2 1 Thus, (d)
15. (d) 16. (d)
17. (a) Li—Mg diagonal relationship
X(g) + e~ —-> X"(g)
(b) Na-K
E2
2 (0 Ca—Mg same group
2E„ (d) B—Al
E = ^-
Nn
Thus, (a)
Thus, (b)
Chapter 3: Periodicity of Elements | 159

18. Except (d) all show diagonal relationship. 32. O>F


Li. Bev Bx C O" > F" Thus, O2" > 0" > F" > F
02">F"
Na Mg ‘Al ‘Si Thus, (a)
Thus, (d) 33. (c) 34. (a)
19. (a) 20. (d) 21. (b) 22. (d) 35. Due to empty d-orbital, screening effect decreases,
hence tendency to attract electron increases.
: + Ze-
23. (a) Tl3+ ->Tl+
stable Thus, (c)
Tl3+ is an oxidising agent thus correct. 36. Oxides of C is a giant molecule indicating covalent
(b) Ga+ ---- > Ga3+ + 2e~ nature
s table Oxide of B is amphoteric
Ga+ is a reducing agent thus correct. Oxide of A is ionic
(c) Pb4++2e" ---- > Pb2+ Thus, A is largest in size.
stable Thus, atomic number of A is least
Pb4+ is an oxidising agent thus correct. A <B <C
(d) As5* + 2e" ---- > As3+ Thus, (a)
stable
As5+ cannot be reduced 37. Z rn (IE) (EN) Oxidising power
Thus, As5+ salts are not oxidising agent. F F2
Thus, (d) Cl Cl2
24. (d) 25. (a) Br Br
26. Smaller cation, larger anion, larger charge (on cation or I
5
anion)
Thus, (a)
greater the polarising power,
38. (a) —F (b) —Ne zero CEA) (c) —Cl
larger the covalent nature
(d) —Ar zero (EA)
smaller the melting point
(c) Cl > (a) F due to empty d-orbital in Cl
Na+ (same), charge on cation, anion-same
Thus, (c)
Thus, size of anion will decide polarising power.
39. (b) 40. (c) 41. (d)
Size: F"<Cr<Br"<F"
42. 0(g)+ e~ ---- > O"(g) EA = -ve
Polarising power : F" < Cl" < Br" < I"
Covalent NaF < NaCl < NaBr < Nal O"(g)+e" ---- > O2“(g) EA = + ve
Melting point NaF > NaCl > NaBr > NaF Thus, (b)
Thus, (a) 43. Ionic radii of O2- > F" > Na+ > Mg2+
27. (b) We take van der Waals’ radius of Ne which should have
maximum size.
28. BeO, SnO, ZnO, all are soluble in alkalis as well as in
acids, thus are amphoteric oxides. Thus, (a)
MO + 2HC1 ---- > MC12+H2O 44. (a) 45. (d)
MO + 2NaOH ---- > Na ^2 + H2O 46. Mg2+ : ls22s22p6 unpaired electron zero
3d
Thus, (d)
Ti3* : ls22s22p63s23p63d1 ? 1
29. Polarising power ofBe2+ > Mg2+ > Ca 2+ >Ba2+
y3+ : ls22s22p63s23p63d2 ? ? 2
Thus, ionic nature of BeCl2 < MgCl2 < CaCl2 < BaCl2
Thus, (a)
Fe2+ : ls22s22p63s23p63d6 u? ? ? ? 4
30. Stable configuration with all unpaired electron in
/'-orbital is [Xe] 6s24/‘75d1 47. i>n>in
Thus, (b) Thus, (a)
31. Element (58-71) and (90-103) are placed in IIIB. 48. (d) 49. (b)
238U ---- > M4Th + a 50. (IV) B contains C, Si, Ge, Sn, Pb
niB niB Ge, Sn, Pb are strongly electropositive.
Thus, (a) Thus, (d)
160 | Essential Inorganic Chemistry

51. (d) (IP) + (EA)


57. (EN) =
2
52. I.P. ofLi>Na
(£A) = 2(EH)-(ZP)
Thus, Na falls below Li
= 2Ei -E,'2
Thus, (c)
Thus, (a)
Be B
58. (c) due to inert-pair effect.
53. /, of Be > /, of B 1s2 2s2 1s22s22p1 59. Copper forms only basic oxide
/2 of Be < /2 of B 1 /, l/i Thus, (b)
/3 of Be > /3 of B 1s22s1(Be+) ls22s2(B+)
1 /2 60. (ZP)ofN>O
1 /2
1s2(Be2+) 1s22s1(B2+) Thus, (a)
1 /3 1 /3 61. In case of Ne we take van der Waals’ radius
1s1(Be3+) 1s2(B3+) Thus, (a)

Thus, (b) is incorrect. 62. A-19 s-block


Thus, (b) B-22 d-block
C-32 p-block
54. smaller the size (P) of the ion
D-64 /■-block
larger the charge (Q)
Thus, (b)
then larger the hydration (R) and
thus, smaller the ionic mobility (S) 63. (c) 64. (d) 65. (a)
P Q R S 66. (a) K(19) ls22s22p63s23p63da:
Li+ n Am ax n
K+(19) ls22s22p63s23p6
Na+ same colourless being no unpaired electron in d-orbital
K* " Thus, (a) is incorrect.
max max
(b) Na(ll) ls22s22p63sx s-block — true
Thus, (a) is true (c) Cu(29) ls22s22p63s23p63dX04sx s-block—true
Na+ Mg2+
P <------
Al3+
(d) Cr(24) ls22s22p63s23p63d6 u|u|n| | ~
max Hund’s rule absent
Q -------- ► Thus, (d) is true
max
Only (a) is incorrect.
R -------- ►
max
-------- ►
67. Cu+ is colourless
S
max Sc3+ is colourless
Thus, (b) is also true Na+, Mg3+ 3+, Al3+ — colourless
Thus, (c) as they have no electron unpaired in d-orbitals.
AH Thus, (b)
68. Crystal > van der Waals’ > covalent
55. Af(s) M+(g) + e“ 100 eV
Thus, (a)
Z1 + Z2
M(g) •> Af2+(g) + 2e~ 250 eV 69. p-block element of higher periods
Ge, Sn, Pb
Thus, M+(g) M2\g)+e~ 150 eV
Thus, (c)
Thus, (a) is true, (b) is also true of (g)is I2 ofM(g)) 70. Gd(64) 4/‘75d1
Thus, (c) is incorrect
eight unpaired electron
56. A ---- > A+ + e" if (ZP)X <(ZP)B Sum of spin = 8 x | = 4
B + e~ B~ if (EA)b >(EA)a
(IP) + (EA) Thus, (d)
(EN) =
2 71. Unq
A*B~ is formed X1 X
if (EN)A <(EN)b 104
Thus, (c) Thus, (c)
Chapter 3: Periodicity of Elements | 161

72. Size of cation decides lattice energy and thermal V(23): ls22s22p63s23p63d34s 2
stability. V4+ :ls22s22p63s23p63dl
Smaller size, larger polarising power thus larger the Thus, VC14
lattice energy. Thus, (c)
Thus, LiCl> NaCl>KCl>RbCl>CsCl 85. Ni2+ is smallest thus, (a)
Thus, (b)
86. (IE\ of Be > B
73. z electron Thus, (b) is incorrect
K+ 19 18 87. Size « — for isoelectronic ion.
s2- 16 18 Z
Thus, O2'> F" > Na+ >A13+
cr 17 18
(Z) 8 9 11 13
Ca2+ 20 18
Thus, (d)
1 „2„
Above are isoelectronic species, hence size oc --- n aQ
z 81. rn =
Z*
Thus,S2’>Cr >K+ >Ca2+ 1
rn
Thus, (c) z*
74. (d) Thus, (a)
3s2 89. (d)
75. Mg Is2 2s2 2p6 3s2 n 90. Size « — for isoelectronic ion
Z
? Size: C4 >N3->O2’>F-
Al Is2 2s22p63s23p1 Z: 6 7 8 9
Thus, (a)
Thus, (c)
91. (b) 92. (c) 93. (d)
76. (c) NHJ is from non-metal N
94. 115* U3y+jHe
77. (a)
Thus, (a)
78. X(g) —> X+(g) + e“; AH = Z
95. (d)
X*(g)+e~ ---- > X(g); AH = E = -/ 96. (n - IJc^ns1 represents group VIB (transition metals) of
Thus, (b)
d-block
79. X(g) + e~ ---- > X~(g); = x = EA of X(g) Thus, (d)
X~(g) (X)g + e~; &H = y = (IE) of X“(g) 97. Sc2+(18e)[Ne]:3s23p6
= -(EA) of X(g) Sc [Ne]: 3s23 p64s23d1 Thus, d-block IIIB group
Thus, (a), (b), (c) are true Thus, given matching is incorrect.
I
I
Thus, (d)
80. (c) 81. (a) 82. (b)
Thus, (a)
98. 113 :[Rn] 7s25d104f147px
83. Smaller size, larger charge => larger hydration and 115 7s27p3 : [Rn] 7s25d104/'147p3
hydration energy
Thus, p-block element
Size Ca2+ <Si2+ <Ba2+
Thus, (b)
hydration Ca2+ >Sr2+ >Ba2+
99. Valence electrons in M(113) = 3
Thus, (a)
Thus, M3+ is the stable cation but due to inert pair effect
84. Unpaired electrons = N
M+ is most stable
Magnetic moment = jN(N + 2) BM Thus, (c)
+ 2) = 1.73 100. M(115): [Rn] 7s25dx04/-147p3
MAT + 2) = (1.73)2=3
Valence shell is 7s2 7p3 with five electrons.
N2 + 2N-3=0
(2V + 3)(N-1)=O Thus, +5, +3 are variable oxidation state. But due to
inert-pass effects Af5+ <Af3+ (stable).
Thus, N = 1 (one unpaired electron)
Thus, (a)
162 | Essential Inorganic Chemistry

101. (a) 121. Due to inert-pair effect


102. X : ns2np5 electronegative element Stability of Al3+ <Sb3+ <Bi3+
valency = -l Thus, (c)
Y: electropositive alkali metal 122. Stability of Ga+ > In* > Tl+
valency = +1 and Ga+ < In3+ < Tl3+
y+' and Tl+ > Tl3+
have inert gas configuration
Thus, Tl3+3+ salts are easily reduced to Tl+. Most
spontaneous reaction is
Compound is YX
Tl3+ + + 2e~ Tl+
Thus, (d)
Thus, (c)
103. (b)
123. Compound with
104. Oxygen family is chalcogens. Element Valency
oxygen
Thus, (d) Halogen CI2O7 +7
105. Af3+: [Ar]3d10 Oxygen o3 0
M -.[ArjSd^s^p1
Nitrogen N2O5 +5
Thus, p-block
Boron b203 +3
Thus, (b)
106. (c) 107. (d) Thus, (a)
108. Alkali (Li, Na, K) and alkaline earth metals (Ca, Sr, Ba) 124. Stability of Pb2+ >Sn2+ >Ge2+
impart colour in flame Pb4+<Sn4+<Ge4+
Thus, (b) Pb2+ >Pb4+
109. (c) Sn4+>Sn2+
110. Fe :[Ar]3d64s2 Ge4+>Ge2+
Fe2+ :[Ar]3d5 Thus, Pb2++2e“ —> Pb2+
Thus, (b) Thus, Pb4+ salts are better oxidising agent being easily
reduced to Pb2+.
111. (b)
Thus, (b)
112. Lanthanides are /‘-block elements.
Thus, (d) 125. Valence electrons in A = 3
Valence electrons in B = 6
113. Actinides and lanthanides constitute /"-blocks
Thus A is electropositive and B is electronegative; A can
Thus, (b) lose three electrons and B can gain two electrons to
114. Gd(64) [Xe] 6s22 |T|'r|T|T|?|?[? T| attain stable configuration.
Thus, A exists as A3+
4/"7 5J1
I| and B as B2~
all electrons, impaired, hence stable Thus, compound is A^B3
Thus, (a) Thus, (d)
115. 115 126. (a)
TTT 127. He+, Li2+
2+ and Be3+ each has one electron, hence no
Uup
screening.
Thus, (c)
Thus, (d)
116. (a) 117. (b)
128. Effective nuclear charge Zefr = Z - S
118. ls22s22p63s23p64sl: it is a case of alkali metals (K).
where Z = atomic number
Thus, graded properties will be shown by element with and S = screening constant
identical configuration of the valence shell. = 0.35 per electron for electron in nth orbit
Thus, (c) = 0.85 per electron for electrons in (n - lKh
119. (a) orbit
120. Element (113) lies below T1 (group 13, i.e., IIIA) = 1.00 per electron for electrons in
Thus, it resembles Ga (n - 2)th, (n - 3)th, (n -4)th orbit
Thus, (b) = 0.30 per electron in ls-orbital (when alone)
Chapter 3: Periodicity of Elements I 163
Be ls 2 2s 2 one valence-electron in 2s is screened by one 143. As electron is lost, effective nuclear charge increases,
electron in 2s-orbital (nth orbit) and two electrons in ls force of attraction increases. Hence,
orbital ((n -1) th orbit) (JE)1 < (JE)2 < (JE)a
S =0.35 + 2x 0.85 = 2.05 Thus, Mg has maximum (IE)
2+

Zeff = 4- 2.05 = 1.95 (given) Thus, (c)


Be + 1s 2 2s 1 2 s-electron is screened by two electrons in
ls-orbital ((n -1) th orbital) 144. He ..!.!-. He• � He 2+
S = 2 x0.85= 1.70 12 > 11, hence, (d) is incorrect.
Zerr ::;:; 4- 1.70= 2.30 Thus, (d)
2+ 2
Be ls one electron in ls-orbital (alone exists) is 145. For alkali metals
screened by another electron in same orbit. Thus,
S = 0.30 Thus, (c)
Zerr= 4- 0.30= 3.70
3+ 1
146. (c) 147. (c)
Be ls no-screening (single electron)
148. Along a group Z * (effective nuclear charge) is almost
Thus, Zeff = 4.
constant. Thus, (IE) depends mainly on rn .
Thus, zetrect1ve: Be< Be + < Be2+ < Be a+
Thus, (a)
Thus, (a)
149. (a) 150. (b)
129. Refer question 128.
Thus, (a) One or More Than One Options Correct
130. Cu(29) 1. (a), (b), (c), (d) 2. (a), (b), (c) 3. (a), (b), (c)
(n -3) (n -2) (n -1) n
r--71 I I 117 4. (a), (c) 5. (a), (b), (d) 6. (b) 7. (c) 8. (d)
1s 2 2s 2 2p6 3s 23p63 d10 4s 1 9. (b) 10. (a), (b), (d)
t
This electron is screened Brain Twisters
by 18 electrons in (n -1), 1. ls 2 2s 2 2 p63s 23 p64s 23 d104p6 5s 2 4d 105p6 6s 2 4f 145d10
(a)
8-electrons in (n - 2} and
2-electrons in (n -3) 6ps 7s 2 6f146d10 7ps 8s 2 5g1s 6fu 7d10
(n -· l)s 2p 6 (n - l)d10 (n - 2)d10 (n - 3)s 2
.-·-. ,........._ ,.....�
Thus, a= 8 x 0.85 + 1 x10 + 8 x1 + 2 x1
------ (b) 8s 7d , 7d
2 10 10

(c) Four nodal planes, zero nodal sphere


= 26.8 poor shielders, poor penetrators.
Thus, Z * = 29 - 26.8 = 2.2 (d) Z * =4.35
Thus, (a) (e) X A R = 0.744 + [3590 x(4.35-0.35)]/(150)2
-
= 0.744 + 0.64 = 1.384
131. Atomic number (Z) Period (n)
The value should be much higher.
(a) Cs 55 6 - 1 X 10-3
2. 1.00 mg = 1 x10 3 g = -- g mol Li
(b) Mg 12 3 7
1 mol of Li is converted by = 520.3 kJ
(c) Ba 56 6
1 X 10-3
(d) Cr 4 • --- mol is converted to 11+ by

!
24 7
-
52Oxl xlO 3
rn oc n2, but in case of Cs and Ba, rn oc = kJ
7
Thus, (a) = 0.0743 kJ

132. (a) 3. (IE>row = (IE)i + ... + (JE)s


133. rn oc .! along a period. kJ moi-1
= 1. 7058 x 10 4
z = 1.7058 x 107 J moi- 1
Thus, B(5) > C(6) > N(7)
Since, 1.6x 10-19 J = 1 eV
Thus, (b)
134. (c) 135. (c) 136. (b) 137. (c) 138. (d)
. 1.7058x 107 J
= l.7058 x 1 7
� eV moi-1
1.6 X 10-1
139. (c) 140. (a) 141. �b) 142. (a)
= 1.0661 x 1026 eV mo1-1
164 | Essential Inorganic Chemistry

1.0661 x 1026 2.18 xlO"18 x(79)2


J atom
6.02 x 1023 Hg 79+ ” 1

= 177.1 eV atom = 1.36 x 10"17 J atom-1


_ 2.18 x 10"18 x(79)2 x 6.02 x 1023
AZZ kJ mol
" 1000
4. Mg(g) ---- > Mg*(g) + e" 737.7 kJ
= 8.19 x 106 kJ mol"1
Mg*(g) ---- > Mg2+(£)+e" 1451 kJ 4,4
2F(g)+2e" ---- > 2F"(g) -328x2 kJ
9. Volume of K atom = - nr3 = - n(216)'>3
3 3
Mg(g) + 2F(g) ---- > Mg2+(g) + 2F"(g) AZZ = 1532.7 kJ 4
Volume ofK* ion = - n(133)3
Thus, endothermic
5. 1 eV atom-1 = 1.6 x IO-19 J atom Decrease in volume = - it [(216)” — (133 )3]
3
= 1.6 x 10"19 x 6.02 xlO23 J mol 4
- jt [(216)3-(133)3]
= 96.32xlO3 J mol"1 % decrease = - ----- ------------------ x 100
4
= 96.32 kJ mol"1 - n(216)3
3
6. ZE of F =27.91 x 10r22 kJ atom"1 = 76.66%
= 27.91 xl0’22 x 6.02 xlO23 kJ mol'
= 1680.2 kJ mol"1 Passage Comprehension Questions
IE of Cl = 20.77 x 10~22 x 6.02 x 1023 kJ mol"1 Passage 1
= 1250.35 kJ mol"1 1. (a) 2. (b) 3. (d)
EA of F = -5.53 x IO"22 x 6.02 x 1023 kJ mol
Passage 2
= -332.91 kJ mol'1
1. (b) 2. (a) 3. (c) 4. (b)
EA of Cl = -5.78 x 10"22 x 6.02 x 1023 kJ mol
= -347.96 kJ mol"1 Passage 3
LetF = 74 moles and Cl = n.;2 moles 1. (c) 2. (c) 3. (a)
For the formation of cation Passage 4
1680.2/^ + 1250.35n2 =284 1. (c) 2. (a) 3. (c)
or n, + 0.744n2 =0.1690 ...(i) Passage 5
and for the formation of anion
1. (b) 2. (d) 3. (a) 4. (c)
332.91^ + 347.96n2 =68.8
Passage 6
or, Tij + 1.045n2 =0.207 ...(h)
From (i) and (ii) 1. (d) 2. (c) 3. (b)
Tij =0.0785 mol Passage 7
ti2 =0.123 mol 1. (a), (b), (c), (d) 2. (a)
Total moles =0.123 + 0.0785 =0.2015 Passage 8
n 191
Mol % ofF = x 100 =61.04% 1. (a) 2. (b) 3. (c) 4. (c)
0.2015
Mol %F =38.96%
Assertion & Reason
7. (ZE),'h.-
1. (b) 2.
2. (a)
(a) 3. (c) 4. (b) 5. (b) 6. (a)
2.18x10r18 z2 7. (a) 8.
8. (b)
(b) 9. (a) 10. (b)
- J atom
7l2
2.18xl0“18 x(2)2 T . m
=---------- ----------- JJatom
atom for He (n = 1, Z = 2) True & False
= 2.18 x 10“18 x 4 x 6.02 x 1023 J mol"1 1. T Due to shielding effect
= 5.2494 x 106 J mol"1 2. T • 2NaOH + A12O3 + 3H2O 2Na[Al(OH)4]
= 5.2494 x 103 kJ mol-1 3. F Neutral oxide
8. Hg79*(g) ---- > Hg^gJ+e" 4. F PbO2 (oxidation number of Pb = +4) is a-'
oxidising agent since Pb2+ is more stable tha:
Hg79+(g) is isoelectronic of hydrogen (electron = 1)
Pb4+ due to inert pair effect.
(Z = 79, n = 1)
Pb4+ + 2e" ---- > Pb2*
From Q. 7.
Chapter 3: Periodicity of Elements | 165

5. F Smaller the size of the cation, larger the 4. (AM2); (B>—(3); (C>—(5); (DM1);
hydration and hence smaller the ionic mobility. (EM4)
Li+ is smaller than Na+ hence, it is more 5. (AM4); (B>—(3); (C>—(5); (D>—(2);
hydrated than Na+ hence, its ionic mobility is
less than that of Na+. (EMD
6. F . B2O3 is an acidic oxide. II. One or More Correct
B.2O3 +2NaOH 2NaBO2 + H20 1. (AH2,6); (B>—<1); (C>—(3); (DM4,5);
acid base (EM7)
B2O3 is not soluble in acid. 2. (AMI,4); (BM1,4); (CM(1,2); (DM1,2,5);
7. F . ZnO is amphoteric but not amphiprotic. (EM2,3); (FH2.3.6)
Amphoteric oxide is that which can be an acid as well
as a base. Amphiprotic oxide is that which is H+ ion Integer Answer Types
donor as well as its acceptor. Set I
H20 + H20---- > H30+ + OH’ Questions—► 1 2_ 3 4 5 £ 7_ 8
H20 is an amphiprotic. Answers —► 1 7 2 1 6 6 7 8
8. T' . LiCl can absorb moisture due to greater tendency © © © © © ©|© ©
of hydration of Li+ hence, LiCl is humidity © 0 © O © © ©. ©
controller. © © © © © ©
9. F' . Hydrogen has only one electron. Its (IE)2 is simply © © © © © ©© ©
imagination. © © © © © ©
10. T © © © © © ©__© ©
11. T : © © © © © © ® ©
Li2CO3 Li2O + CO2
© © © © © ©
12. F : They have approximately equal size. © ©
© © © ©
13. T : Atomic size °= — but in case of Ne, van der © © ©
© © © © ©
Waals’ radius is considered hence
Set II
atomic radius of Ne > F.
(^)i T-
Questions—► 1 2 3 4 5 £ 7_ 8
14. T : Li ----- Li + e Answers —► 5 3 2 4 7 6 4 5
(IE)2■> Li2+ + e~ © © © © © ©I© ©
Li+
0 © 0 © © ©_© ©.
Li+ is isoelectronic of He. Hence, after losing one © © © © © _©_© ©
electron, Li+ attains He electronic configuration. Hence, © © © © © <©_© ©
its (IE), (which is (IE)2 of Li), is maximum. © © © © © (£_© ©
15. T © © © © © ®_© ©
16. T: Its (IE\ is maximum in the Periodic Table. © © © © © >_® ©
© © © © © © © ©.
© © © © © ©
Fill in the Blanks © © ©
© © © © ©
1. Li 2. Br 3. covalent 4. T 5. decreases
6. four 7. reducing agent, oxidsing agent 8. IA
9. amphoteric 10. electronegativity 11. F 12. less Test Yourself On Periodic Table
13. greater, less 14. less 15. Li. „ (C) (b) (al

Matrix-Match Type Questions


I. Only One Correct 1.

1. (AH4); (BH3); (CHD; (DM2)


2. (AH2); (BHD; (CH4); (DM3)
3. (A)—(3); (BH4); (CHD; (DM2)
166 | Essential Inorganic Chemistry

(C) (e) (f)


□1 1 n

2.

t
(d)
(9)

(a)&(b)

4. (a) A : CaO, B : A12O3, C : SO3, D: SeO3


(a)
(b) basic : CaO (c) most ionic : CaO
acidic : SO3> SeO3 most covalent: SO3
amophoteric; A12O3
(d) CaO, A12O3 : three-dimensional structure
3.
SO3, SeO3: molecular
(e) CaO : highest melting point
SO3 : lowest melting point
5. (a) A: NaH (b) ionic (c) Na = +1, H = -1
B:PdHx interstitial
(b)
covalent S = -2,H = + 1
in CzHjS
D: HI covalent I = -l, H = + l

(c>
wt, , aa

Oxidation-Reduction
A fair business of gain of electrons (reduction) and loss of
electrons (oxidation) is a redox reaction. How much is lost in a
half-reaction, is also gained in equal amount (electrons) —
actually the best fair deal.

Quick Points
• Number of ionisable H atoms (attached to electronegative earj I
atom 0) determines basicity of an acid.
HNO3,H3PO2 (monobasic acid) — one H attached to 0-atom 1P■j

H2SO4, H3PO3 (dibasic acid) — two H attached to 0-atoms
HgPO^ (tribasic acid) — three H attached to O-atoms ■ Oxidation Number and Oxidation State
• Superoxide ion is Oj ■ Oxidation and Reduction
• Peroxide ion is O2“ ■ How to Balance a Redox Reaction
• Mole =—— ■ Balanced Redox Reactions
molar mass
• jVq (Avogadro’s number) = 6.02 x 1023 mol"1 ■ Equivalent Weights
• AG°=-nFE°
where, n = number of electrons exchanged
F = number of faradays
Ea = standard electrode potential
AG° = standard Gibb’s free energy change
• If different number of electrons are exchanged in a series of
reactions then
AG° = AG° + AG2°

or Eo _ + n2g°
3 n3

0.0591
logQ
E = Hi----------------
n
where, Q = reaction quotient.
2.3Q37?T
E° = lOg^
nF
at equilibrium when E = 0
and Q = equilibrium constant =
168 | Essential Inorganic Chemistry

Every time you oxidise the vanadium by removing


Oxidation Number and the Oxidation another electron from it, its oxidation state increases by 1.
State Fairly obviously, if you start adding electrons again
The oxidation number (O.N.) of an atom in a molecule the oxidation state will fall. You could eventually get back
or in a poly atom ion is a hypothetical charge the atom to the element vanadium which would have an oxidation
would have if the electrons in each bond were located on the state of zero.
more electronegative atom. What if you kept on adding electrons to the element?
The oxidation number (or oxidation state) of an You can’t actually do that with vanadium, but you can
element in a compound is determined as the number of with an element like sulphur.
electrons that have passed from one atom of a given Oxidation states are represented by integers, which
element to other atoms (positive oxidation) or to one atom can be positive, zero, or negative. In some cases average
of a given element from other atom (negative oxidation) oxdidation state of element is a fraction such as 8/3 for iron
Oxidation states simplify the whole process of working out in magnetite.
what is being oxidised and what is being reduced in redox
reactions.
• Oxidation state is the oxidation number per atom. Working Out the Oxidation States
• An unreacted element has oxidation number of zero. You don’t work out oxidation states by counting the
• A monoatomic ion has oxidation number equal to its numbers of electrons transferred. It would take far too
charge. long. Instead you learn some simple rules, and do some
very simple sums.
Oxidation Number of Elements in Covalent Compounds and • The oxidation state of an uncombined element is zero.
in Ions That’s obviously so, because it hasn’t been either
O.N. Element/lon O.N.
oxidised or reduced yet. This applies whatever the
Element/lon
structure of the element-whether it is, for example Xe
0 Mg2+ +2 or Cl2 or S8, or whether it has a giant structure like
O2 0 Fe3* +3 carbon or silicon.
n2 0 Mn7* +7 • The sum of the oxidation states of all the atoms or ions
Na 0 o-2 - 0.5 (per atom) in a neutral compound is zero.
Mg 0 O2’
-2 • The sum of the oxidation states of all the atoms in an
H* +1 o| —1 (per atom) ion is equal to the charge on the ion.
H" • The more electronegative element in a substance is
given a negative oxidation state. The less
Vanadium forms a number of different ions — for electronegative one is given a positive oxidation state.
example, V2t and V 3+.If you think about how these might
Remember that fluorine is the most
be produced from vanadium metal, the 2+ ion will be
formed by oxidising the metal by removing two electrons: electronegative element with oxygen second.
V ---- > V2++2e" • Some elements almost always have the same
oxidation states in their compounds
The vanadium is now said to be in an oxidation state - ■

of+2. Element
Usual oxidation
Exceptions
Removal of another electron gives the V3+ ion state
V2+ ---- > V3++e’ Group 1 metals always +1
Group 2 metals always +2
The vanadium now has an oxidation state of +3.
Oxygen usually -2 except in peroxides and
Removal of another electron gives a more unusual F2O
looking ion, V(y+. Hydrogen usually +1 except in metal hydrides
V^+HjO ---- > VO2++e’+2H+ where it is -1
The vanadium is now in an oxidation state of +4. Fluorine always -1
Notice that the oxidation state isn’t simply counting the Chlorine usually -1 except in compounds with
charge on the ion (that was true for the first two cases but 0 or F
not for this one).
The positive oxidation state is counting the total The reasons for the exceptions
number of electrons which have had to be (i) Hydrogen in the metal hydrides Metal hydrides
removed-starting from the element. include compounds like sodium hydride, NaH. In
It is also possible to remove a fifth electron to give this, the hydrogen is present as a hydride ion, H“.
another ion (easily confused with the one before). The The oxidation state of a simple ion like hydride is
oxidation state of the vanadium is now +5. equal to the charge on the ion-in this case, -1.
Chapter 4: Oxidation-Reduction | 169

Alternatively, you can think of it that the sum of the Elements with Multiple Oxidation States
oxidation states in a neutral compound is zero. Most elements have more than one possible oxidation
Since Group 1 metals always have an oxidation state. Carbon has nine integer oxidation states, and there
state of +1 in their compounds, it follows that the are also molecules in which the average degree of
hydrogen must have an oxidation state of -1 oxidation of several carbons is fractional.
(+1-1 = 0). Integer oxidation states of carbon with examples
NaH 1. -4:CH4
T
1+x=0 x = -1 2. -3:0^
(ii) Oxygen in peroxides Peroxides include hydrogen 3. -2:CH3C1
peroxide, H2O2. This is an electrically neutral 4. -1 : QA
compound and so the sum of the oxidation states of 5. OiCHjCL,
the hydrogen and oxygen must be zero. 6. +1 :CHC12— CHC12
Since, each hydrogen has an oxidation state of +1,
each oxygen must have an oxidation state of -1 to 7. +2:CHC13
balance it. 8. +3 : CgClg
HA 9. +4:CC14
T Examples of fractional oxidation states for
2 + 2x = 0 carbon
x = -1 1. - (6/5) AHg"
(iii) Oxygen in F2O The problem here is that oxygen 2. - (6/7): C7H;
isn’t the most electronegative element. The fluorine
is more electronegative and has an oxidation state 3. - (10/8): C8H28-
of—1. In this case, the oxygen has an oxidation state Oxygen has 8 different oxidation states
of+2. 1. -2 in oxides, e.g., ZnO, CO2, H2O
(iv) Chlorine in compounds with fluorine or oxygen 2. -1 in perioxides, e.g., HA
There are so many different oxidation states that 3. - (1/2) in superoxides, e.g., KO2
chlorine can have in these, that it is safer to simply
remember that the chlorine doesn’t have an 4. - (1/3) in inorganic ozonides, e.g., RbO3
oxidation state of -1 in them, and work out its 5. 0 in O2
actual oxidation state when you need it. You will 6. + (1/2) in dioxygenyl, e.g., dioxygenyl
find an example of this below. hexfluoroarsenate O2 [AsF6 ]“
• While naming the ion or compound, it is desirable to 7. +1 in O2F2
put oxidation number per atom (i.e., oxidation 8. +2 in OF2
state) in Roman figures within brackets (anion end Note that since fluorine is more electronegative than
with ate) (Table 4.1). oxygen, O2F2 and OF2 are considered as fluorides, rather
than as a peroxide and an oxide.
Table 4.1 Naming the Acid and its Ions
Oxoanions Formula Acids Formula Fractional Oxidation States
Bromate (I) BrO" Bromic (I) HBrO Fractional oxidation states are often used to represent
Bromate (III) BrO; HBrO2
the average oxidation states of several atoms in a
Bromic (III)
structure. For example, in KO2, the supoeroxide ion has a
Bromate (V) BrOj Bromic (V) HBrO3 charge of-1 divided among two equivalent atoms, so each
Bromate (VII) BrO; Bromic (VII) HBrO4 oxygen is assigned an oxidation state of -1/2. This ion is
Chromate (VI) CrO4” Chromic (VI) H2CrO4 decribed as a resonance hybrid of two Lewis structures,
and each oxygen has oxidation state 0 in one structure and
Nitrate (V) NO3 Nitric HNO3
-1 in the other.
Nitrate (III) no; Nitrous hno2 For the cyclopentadienyl ion C5H^, the oxidation state
Phosphate (V) po^- Phosphoric H3PO4 of C is (-1) + (-1/5) = - 6/5. The -1 occurs because each C
is bonded to one less electronegative H, and the —1/5
Sulphate (IV) SO|- Sulphurous H2SO3
because the total ionic charge is divided among five
Sulphate (VI) SO^- Sulphuric h2so4 equivalent C.
Peroxodisulphate (VI) S2O| Peroxy­ H2S2O8 If the average refers to atoms which are not
disulphuric equivalent, the average oxidation state may not be
170 | Essential Inorganic Chemistry

representative of each atom. This is true in magnetic — Greater the O.N. of the element forming
Fe3O4 whose formula leads to an average oxidation state of oxyacids, greater the acid strength except
+8/3. In fact two thirds of the iron ions are Fe3+ and for oxyacids of phosphorus : Thus, acid
one third Fe2+. strength increases.
Similarly the ozonide ion 0; has an average oxidation In (3) H2S2O3 < H2SO3 < H2SO4
state of -1/3. This ion is V-shaped with a central oxygen (4) HC1 < HC1O < HC1O2 < HC1O3 < HCIO4
which is not equivalent to the two others and cannot be
order of acid strength
assumed to have the same oxidation state.
You will have come across names like iron (II) If acid is strong its conjugate base (formed when H+ is
sulphate and iron (III) chloride. The (II) and (III) are the lost from an acid) is weak and vice-versa. Thus, basic
oxidation states of the iron in the two compounds: +2 and strength of conjugate bases of oxyacids of Cl will be in
order:
+3 respectively. That tells you that they contain Fe2+ and
Fe3+ions. cio; < cio; < cio; < cio-
This can also be extended to the negative ion. Iron (II) Hence, greater the O.N. of the element in
sulphate is FeSO4. There is also a compound FeSO3 with conjugate bases, smaller the basic strength.
the old name of iron (II) sulphite. The modern names Table 4.2 gives oxidation number of underlined atom.
reflect the oxidation states of the sulphur in the two
Table 4.2 Oxidation Number of the Atom in Different
compounds. Species (lon/Molecule)
The sulphate ion is SO^“. The oxidation state of the
[Nj(CN)4]2- +2 hno2 +3
sulphur is +6. The ion is more properly called the sulphate
(VI) ion. Ni(CO)4 0 hno3 +5
The sulphite ion is SO;-. The oxidation state of the (Fe(H2O)]2+ +2 h2n2o2 +1
sulphur is +4. This ion is more properly called the Mrp; +7 c6h12o6 0
sulphatedV) ion. The ate ending simply shows that Mrp2- +6 co2 +4
the sulphur is in a negative ion. MrO2 +4 nh3 *3
So, FeSO4 is properly called iron (H) no2 +4 n2h4 -2
sulphate(VI), and FeSO3 is iron(H) sulphate (IV). In nh2oh n2o +1
fact, because of the easy confusion between these NO +2
names, the old names sulphate and sulphite are n2o5 +5 (per atom)
normally still used in introductory chemistry n2o3 +3 (per atom)
courses.
kq2 -0.5 (per atom)
CrO5 +6 0
Oxidation numbers of underlined atoms have been O\ II /O
determined by known values of the O.N. of other atoms in it. l>c<l
(1) (a) CrO*“ x-8=-2 x=6 (Y XO
It has two peroxy linkages
(b) Cr2O^“ 2x -14 = - 2, 2x = 12 CrO(O2)2
or x = 6 (per atom) H2SO5 +6 It has one peroxy linkage
(2) (a) H3PO4 3 + x-8 = 0, x = + 5 T
(b) H3PO3 3 + x - 6 = 0, x = + 3 H—0—S—0—0—H
1
(c) H3PO2 3+x-4 = 0, x = + l 0
(3) (a) H2SO3 2+x-6=0 x = + 4 HjSjOg +6 (per atom) It has one peroxy
linkage
(b) HaSO, 2+x-8=0 x = + 6 [Fe(CN)6]<- +2
(c) HaS2O3 2 + 2x-6 = 0 x = + 2 [Fe(CN)6]3' +3
(4) (a) HC1O4 l+x-8=0 x = + 7 [Fe(CN)5(NO)*]: +2
(b) HC1O3 l+x-6=0 x = + 5 POCI3 +5 0 as well as Cl are
(c) HC1O2 1 + x-4 =0 x = + 3 POCI3 -2 attached to P directly
hence ON of 0 = - 2, and
(d) HC1O l+x-2=0 x = + l that of Cl = - 1
(e) HC1 l + x = 0 x = -l POCI3 -1 (per atom) 'CH2NH2'
en = is neutral
— In (2), (3) and (4) oxyacids have atoms with [Cr(en)3]3" +3
ch2nh2
different O.N. \
Chapter 4: Oxidation-Reduction | 171

4 ’
Table 4.3 gives common oxidation numbers for group Do you know
A elements. By convention, oxidation numbers are represented as +n
and -n, while ionic charges as n+ and n~; as +3, Fe3+, -3, X3~
Table 4.3 Common Oxidation Number (States) for Group A usually, the element with thze positive oxidation number is
Elements in Compounds and Ions
written first. (dldr^co")
Element (s) Common Other O.N. However, for historic reasons, in compounds containing
Examples
O.N. nitrogen and hydrogen, such as NH3, and many other
H -1 in metal
compounds containing carbon and hydrogen, such as CH4,
+1 H2O.CH4,NH4CI
hydrides, hydrogen is written last, although it has a positive oxidation
e.g., NaH, CaH2 state.
Group IA +1 KCI, NaH, RbNO3, None
K2SO4
Most transition metals and the metals of Group IIIA
(boron family), IVA (carbon family) and VA (nitrogen
Group IIA +2 CaCI2, MgH2, None
family), exhibit more than one oxidation number. These
Ba(NO3)2,SrSO4
metals may form two or more binary compounds with the
Group IIIA +3 AICI3,BF3>AI(NO3)3, None ini common
same non-metal. To distinguish among all possibilities,
Gal3 <compounds
the oxidation number of the metal is indicated by a Roman
Group IVA +2 CO, PbO, SnCI2, Many others are
also seen for C numeral in parentheses following its name. This method
+4 Pb(NO3)2, CCI4,
and Si can be applied to any binary compound of a metal and a
SiO2,SiOf-,SnCI4
non-metal, whether the compound is ionic or molecule. An
Group VA -3 in binary Mg3N2, Na3P, +3, e.g., NO2,
PCI3.
older method still in use but not recommended by the
compounds Ca3As
with metals +5, e.g., NO3, IUPAC uses “ous” and “ic” suffixes to indicate lower and
-3inNH4, PO4~, AsF5,P4O10 higher oxidation number respectively.
binary
compound NH3, ph3, AsH3>
s with H nh; Oxidation
Formula number of IUPAC name ; Common name
Oxygen -2 H2O, P4O10, Fe2O3, +2 in 0F2- "1 in metal
peroxides, e.g.,
J
CaO, CIO3 Cu2O +1 Copper (I) oxide Cuprous oxide
H2O2,Na2O2,-
CuF2 +2 Copper (II) fluoride Cupric fluoride
in superoxides,
e.g., KO2,RbO2 FeS +2 Iron (11) sulphide Ferrous sulphide
Group VIA -2 in binary H2S, CaS, Fe2S3, +4 with 0 and the Fe2O3 +3 Iron (III) oxide Ferric oxide
(other than compound Na2Se, (NH4)2S, lighter halogens,
SnCI2 +2 Tin (II) chloride Stannous chloride
oxygen) s with e.g., SO2, SeO2,
metals and (NH4)2Se Na2SO3, so*-. SnCI4 +4 Tin (IV) chloride Stannic chloride
H SF4 PbO +2 Lead (II) oxide Plumbous oxide
+6 with 0 and the
lighter halogens, PbO2 +4 Lead (IV) oxide Plumbic oxide
-2 in binary e.g., SO3, TeO3, Mercury (I) chloride Mercurous chloride
Hg2CI2 +1
compound h2so4,so4-,sf6
s with NH4+ HgCI2 +2 Mercury (II) Mercuric chloride
chloride
Group VIIA -1 in binary MgF2, KI, ZnCI2, Cl, Br, or I with 0
compound FeBr3,NH4CI,NH4Br or with a lighter
s with halogen Formula Oxidation number Name
metals and +1, e.g., BrF,
H, -1 in CIO’.BrO": H2SO3 Sulphurous acid
binary +3, e.g., ICI3; 2-units
compound H2SO4 +6 Sulphuric acid
CIO2,BrO2;
s with nh;
+5, e.g., BrF5; HNO2 +3' Nitrous acid
CIO3,BrO3; 2-units
hno3 +5 Nitric acid
+7, e.g., IF7,CIO;,
BrO; HBrO2 +3' Bromous acid
2-units
HBrO3 +5 Bromic acid
Table 4.4 gives some non-zero oxidation numbers/
states.
172 | Essential Inorganic Chemistry

DEEP Focus Do you know.....


Ternary acids that have one fewer 0 atoms than the “-ous" Oxidation state of the central atom is equal to its periods
acids (two fewer 0 atoms than the “-ic” acids) are named using group number, except for halogen
the prefix “hypo-” and the suffix “-ous”. IIIA VIIA
IVA VA VIA
h3bo3 H2CO3 hno3
Formula Oxidation number Name boric acid carbonic acid nitric acid
+6
HCIO +1 '.jpochloruus acid H4SiO4 H3PO4 H2SO4 HCIO3
silicic acid phosphoric sulphuric chloric acid
h3po2 +1 hypophosphorus °r '' acid acid
+5 +6 +5
HIO +1 hypoiodous acid H3AsO4 H2SeO4 HIO3
H2N2O2 +1 hyponitrous acid arsenic acid selenic acid iodic acid
I

DEEP Focus Illustration 1 What is the name of CIO- ion?


Acids containing one more oxygen r - central Solution We have
non-metal atom than the normal “-ic acid” are named “-per” Oxidation number of Cl = x
acids. Oxidation number of O = - 2
Total charge on ion = - 1
Formula Oxidation number Name Thus, O.N.(Cl) + O.N.(O) = total charge
HCIO4 +7 perchloric acid x - 2 = -1 x = +1
HBrO4 +7 perbromic acid Since, it is an oxoanion, hence it ends with ate. Thu
HIOj +7 periodic acid
it is chlorate (I) ion.
Note : Its common name is hypochlorite.
Ternary acids (oxoacids) are compounds of Illustration 2 What is the formula of
hydrogen, oxygen, and usually a non-metal. Non-metals (a) iron (II) chloride (b) iron (III) chloride
that exhibit more than one oxidation state form more than (c) copper (II) nitrate
one ternary acid. These ternary acids differ in the number
of oxygen atoms they contain. The suffixes “-ous” and “-ic” Solution (a) FeCl2 (b) FeCl3 (c) Cu(NO3)2
indicate lower and higher oxidation states, respectively.
Illustration 3 Thiosulphuric acid (a dibasic aei<
The common ternary “-ic” acids are shown below.
contains one sulphur and three oxygen atoms dire<cti
Acids containing one fewer oxygen atom per central joined to central sulphur atom. What is oxidation numb
atom are named in the same way except that the “-ic” of sulphur atom?
suffix is changed to “ous”. The oxidation number of the
central element is lower by 2 in the “-ous” acid than Solution Thiosulphuric acid is a dibasic ac
in the “-ic” acid. represented as
0
Thus, oxoacids of chlorine can be named as
I
Formula Oxidation number Name H—0—S—O—H
HCIO +1 hypochlorous acid
I
S
HCIO2 +3 chlorous acid
The difference in the environment of two S-atom.s
hcio3 +5 chloric acid
shown by radioactive tracer studies using
hcio4 +7 perchloric acid (a radioactive isotope of sulphur)
Problem-Solving Tip Naming ternary acids and SOj“ +S‘ s’so|- s* +so23-
their anions. Thiosulphuric acid H2S2O3H2S2O3 decomposes in
Ternary acid Anion 2 o sulphurous acid and sulphur and the enti
<D c E E E
CD n c radioactivity is found associated with sulphur atom.
•S PerXXXic acid PerXXXate
w E
B2ro
o £ co XXXic acid XXXate .£ ® 2 H^SA H2SO3+S
g.Pc XXXous acid XXXite Thus, oxidation number of terminal S atom
Qg 8 bo o
x hypoXXXous acid hypoXXXite H2S2O3 = — 2
o and oxidation number of central S-atom inH2S2C)3 =
The stem (XXX) represents the stem of the name e.g., net oxidation number of two S-atom =4
oxidation number per sulphur atom = + 2
“nitr”, “sulphur” or “chlor”
Chapter 4: Oxidation-Reduction | 173

Table 4.4 Some Non-zero Oxidation States* (Numbers)


•. IA VILA 0
1 1 2
H H He
4-1 4-1
-1

ILA IIIA IVA VA VIA


3 4 5 6 7 8 9 10
Li Be B C N 0 F Ne
+1 4-2 4-3 44 4-5 -1 -1
4-2 4-4 —2
4-3
-4
4-2
4-1
-3
11 12 13 14 15 16 17 18
Na Mg Al Si P S Cl Ar
+1 +2 4-3 4-4 4-5 +6 +7
4-3 44 4-5
-3 —2 4-3
4-1
Groups -1
IIIB IVB VB VLB VIIB (VI B) IB IIB
19 20 21 22 23 24 25 26 27 28 29 30 31 32 33 34 35 36
K Ca Sc Ti V Cr Mn Fe Co Ni Cu Zn Ga Ge As Se Br Kr
+1 +2 4-3 +4 4-5 +6 +7 4-3 +3 4-2 4-2 4-2 +3 +4 4-5 4-6 4-7 4-4
+3 4-4 4-3 +6 +2 +2 4-1 4-3 4-4 +5 4-2
+2 +3 4-2 44 -3 -2 4-3
+2 4-3 +1
+2 -1

O.N. shown in bold are common

Illustration 4 Determine change in oxidation number Solution (i) HaOj, —> O2


in of the underlined of the following conversions: T
-2 0] for every two oxygen atoms
(a) Cl" changes to CIO4 (&) MnO^ changes to MnO2
(c) CjC^- changes to CO2 (d) Cr90^~ to Cr3+ Total change in oxidation number = 2 units
Solution Change in oxidation number per atom= 1 unit
(ii) HA ---- > 211,0
Oxidation number of the underlined
coefficient 2 is used to balance O
In reactant In product change -2 -4] oxidation number of two O-atoms
(a) cr = -1 CIO; = +7 8 Total change in oxidation number = 2 units
(b) MnO; = 4-7 MnOz = +4 3 Change in oxidation number per atom = 1 unit
(c) C2O^ =4-3 CO2 = 4-4 1 (per atom)
Illustration 6 SO2 under atmospheric condition
(d) Cr20f’ = 4-6 Cr3+=+3 3 (per atom)
changes to SC?'. If oxidation number of S in SO^" is +6,
what is SO^" ?
Illustration 5 H2O2 changes to O2 or H^O depending on
Solution Oxidation number of S inSO^"= + 6
the medium. What is change in oxidation number of oxygen
per atom? Oxidation number of 0 = - 2
174 | Essential Inorganic Chemistry

Thus, 5 O2’ Illustration 10 In qualitative analysis, a


? T confirmatory test for chromate ion involves the formation
6 - 2x = - 2 of blue transient peroxo species, CrO5, which is better
2x = 8 represented as CrO(O2)2.fr is formed when H2O2 is added
x=4 to an acidic solution containing Cr2O^". What is oxidation
Thus, SO^’isSOj". state of Cr in CrO(O2)2? Write the equation offormation of
CrO5.
Illustration 7 Match the species with correct Solution
oxidation number of the underlined:
C^O2/ +2H+ +4H2O2 ---- > 2CrO(O2)2 +5^0
Species Oxidation number Oxidation state of oxygen in peroxide = -1
A: CrO2CI2 I: +7 Oxidation state of oxygen in oxide = - 2
B: MnO3CI II: +1 Hence in, CrO(O2)2
C: NH2OH
D: OF2
III:
IV:
+6
+2
/H
x-2-4=0
0

?>1<?
E: N2O V:
x=+6 ox x o
Solution Illustration 11 The complex [FeCHjO^NOf* is
Species O.N. formed in the 'ring-test' for nitrate ion when freshly
A +6 prepared FeSO4 solution is added to aqueous solution of
B +7 NO3 followed by addition of cone. H2SO4. If NO exists as
C nitrosyl NO+, what is oxidation number of iron1
? Explain.
D +2 Also calculate magnetic moment of Ye.
E +1
Solution Let oxidation state of Fe = x
Illustration 8 What is the oxidation state of hydrogen [FelH^NOf*
in LiAlH,? Write an equation for the reaction of hydrogen
in that oxidation state with water. x+0+1=2
Solution '

m
LiAlH4

+1 + 3 + 4x = 0
x = -1
x = +1
Thus, oxidation state of iron = +1
Iron is added as FeSO4 with oxidation state of iron=+2.
It is due to charge transfer from NO to Fe2+. N0 changes to
Thus, hydrogen is in the form of hydride (H") inLiAlH4. NO+andFe2+toFe+.
H" + HjO---- > Ha + OH" NO---- > NO++e"
Fe2+ + e~ —> Fe+
Illustration 9 What is the oxidation number of Cl in
(a) Ca(C102)2 (6) Ba(C103)2 (c) CaOC^? [Ar] f| f f f f
Solution (a) Ca(C102)2 has chlorite ion C1O2 Electronic configuration of Fe2+ = [Ar] 3d6

x-4=-l
x=+3
n Fe2+ gains one electron from NO hence, it has now
three unpaired electrons (N = 3)
[Ar] |tl|tl|t |t It
(b) Ba(C103)2 has chlorate ion C1O3 from NO
C1O3 Magnetic moment = ^N(N + 2) BM = ^3x5 BM
= J15 = 3.873 BM
x-6=-1
x=+5 Illustration 12 The tetrathionate anion, S4Og", is a
(c) CaOCl2 is bleaching powder having two types of sulphur oxoanion derived from the compound tetrathionic
chlorine, Cl’ and CIO" (hypochlorite ion) acid, H4S4O6. Anion can be viewed as the adduct formed
oxidation number of Cl in Cl" = - 1 from the reaction between Lewis base S2" and Lewis acid
and oxidation number of Cl in CIO’ = +1 so3.
q2-
Thus, in combined state oxidation number = 0 °2 + 2SO3 ---- > S4OJ-
Chapter 4 : Oxidation-Reduction I 175
Two of the sulphur atoms are present in the oxidation = 10 =
Thus, oxidation number per sulphur atom 2.5
state O and two are in oxidation state +5. 4
Alternately it can be explained as Note 2S2 �- --+ S 4 �- + 2e­
s�- + 2S03 --+ S4�­ i i
i i oxidation number per atom 2 2.5
Oxidation number of -2 +12
two sulphur atom in

Target Practice 1
1. Oxidation number ofCr in Cr02Cl2 is +6. What is oxidation (d) Hg 2Cl2
number (per atom) ofoxygen? ............ (e) Fe(NO 3 )3
2. H2S06 +H 2o ---+ H2s0 4 +Hp2 • 4. Chloride ofa copper metal is colourless. What is its IUPAC
name?
Derive oxidation number of sulphur in H 2S0 5. ............
5. Compare basic strength ofthe following •
3. Write the common and IUPAC names of the following
compounds. (a) H�O� l I H 2Po3
Common ruPAC (h) HPO4 - I
2
I HP032-
HI0 3
,_,
(a) (c) c10- L._ J c10;
(b) HCIO
(d) NH 3
(c) FeC1 2

Answers
1. Cr0 2CI2 ---·- -- ...

t\\ 3. Common IUPAC


6+2x-2=0 (a) lodiC acid hydroiOdiC lV) acid
Thus, x (oxidation number of oxygen) = - 2 (b) hypochlorous al :ct hydrochloric (I) acid
2. Formation ofH 20 2 indicates that there is one peroxy linkage in (c) ferrous chloride iron (II) chloride
H 2S0 5• (d ) mercurous chloride mercury (I) chloride
(e) ferric nitrate iron (Ill) nitrate
II �
HO-S �H
I 4. Since copper salt is colourless, it means there is no electron
unpaired ind-orbital.
Cu: [Ar] 3d 10 4s 1
H 2S0 3 (0 2)
Cu [Ar] 3d10
T :

i ii i
Thus, copper is +I
+2+x-6-2=0
Thus, it is Cu 2Cl 2 or CuCI: Copper (I) chloride
x = 6. Thus, oxidation number of sulphur = + 6
5. (a) H 2P04 < H 2P03 (b) HPO�- < HPo�-
(c) c10- > Cl02 (d) NH 3 < NH2

Oxidation and Reduction Because both reduction and oxidation are going on
side-by-side, this is known as a redox reaction.
Oxidation and Reduction in Terms of Oxygen
Transfer Oxidising and Reducing Agents
• Oxidation is gain of oxygen. An oxidising agent is substance which oxidises
• Reduction is loss of oxygen. something else. In the above example, the iron (III) oxide
For example, in the extraction of iron from its ore : is the oxidising agent.
reduction A reducing agent reduces something else. In the
equation, the carbon monoxide is the reducing agent.
Fe203 + 3CO --+ 2Fe + 3C02 • Oxidising agents give oxygen to another substance.
I t • Reducing agents remove oxygen from another
oxidation substance.

176 | Essential Inorganic Chemistry

Oxidation and Reduction in Terms of Hydrogen If you look at the equation above, the magnesium ia
reducing the copper(II) ions by giving them electrons t*
Transfer neutralise the charge. Magnesium is a reducing agent.
• Oxidation is loss of hydrogen. Looking at it the other way round, the copper (II) ions
• Reduction is gain of hydrogen. are removing electrons from the magnesium to create th«
Notice that these are exactly the opposite of the magnesium ions. The copper (II) ions are acting as ar
oxygen definitions. oxidising agent.
For example, ethanol can be oxidised to ethanal:
CH3CH2OH ---- > CH3CHO Writing Ionic Equations for Redox Reactions
I t
oxidation by loss of hydrogen Electron-Half-Equations
You would need to use an oxidising agent to remove What is an electron-half-equation?
the hydrogen from the ethanol. A commonly used When magnesium reduces hot copper (II) oxide tc
oxidising agent is potassium dichromate (VI) solution copper, the ionic equation for the reaction is :
acidified with dilute sulphuric acid. Cu2+ + Mg ---- > Cu + Mg2+
Note The equation for this is rather complicated for this You can split the ionic equation into two parts, and
introductory page. If you are interested, you will find a
similar example (ethanol to ethanoic acid) on the page look at it from the point of view of the magnesium and oi
dealing with writing equations for redox reactions. the copper (II) ions separately. This shows clearly that the
magnesium has lost two electrons, and the copper (II) ions
Ethanal can also be reduced back to ethanol again by
have gained them.
adding hydrogen to it. A possible reducing agent is sodium
tetrahydridoborate, NaBH4. Again the equation is too Mg ---- > Mg2+ + 2e’
complicated to be worth bothering about at this point. Cu2++2e" ---- > Cu
reduction by gain of hydrogen These two equations are described as
I
CH3CHO ---- > CHjCHjOH
I “electron-half-equations” or “half-equations” or
“ionic-half-equations” or “half-reactions”
Any redox reaction is made up of two half-reactions:
DEEP Focus in one of them electrons are being lost (an oxidation
• Oxidising agents give oxygen to another substance or process) and in the other one those electrons are being
remove hydrogen from it. gained (a reduction process).
• Reducing agents remove oxygen from another substance or
give hydrogen to it. DEEP Focus
• An oxidising agent oxidises something else.
Oxidation and Reduction in Terms of Electron • Oxidation is loss of electrons (OIL RIG), that means that am
Transfer oxidising agent takes electrons from that other substance.
• Oxidation is loss of electrons. • So an oxidising agent must gain electrons.
• Reduction is gain of electrons. Or you could think it out like this :
on Ring • An oxidising agent oxidises something else, that means that
oxidation is loss reduction is gain the oxidising agent must be being reduced.
A simple example The equation shows a simple • Reduction is gain of electrons (OIL RIG), so an oxidising
redox reaction which can obviously be described in terms agent must gain electrons.
of oxygen transfer.
A complete reaction showing oxidation and reduction
CuO+Mg ---- > Cu+MgO together is called a redox- reaction.
Copper (II) oxide and magnesium oxide are both ionic.
Fe+Cu2+ ---- > Cu + Fe2+
The metals obviously aren’t. If you rewrite this as an ionic
equation, it turns out that the oxide ions are spectator ions
i R t
________ o______
and you are left with: —-
Some examples of redox reactions are:
reduction by gain of electrons
I I (a) Sn2+ +2Hg2+ ---- > Hg|+ + Sn.4 +
Cu2++Mg ---- > Cu + Mg2+ I R f f
0
I________ J
oxidation by loss of electrons
Chapter 4: Oxidation-Reduction I 177

(b} Mn04 +5Fe2+ + 8H+ --+ 5Fe 3+ + Mn 2+ + 4li.i0 Oxidation number of P (in H 3P03) is +3 which is the

I I
2+
� t t intermediate of +5 (in H 3P04) and -3 (in PH3). Such
substances have oxidation- reduction duality.
The reaction between chlorine and cold dilute sodium
(c) C12�- +6Fe + 14H +
➔ 6Fe + 2Cr 3+ + 7li.i0
3+

I 7
I i t t
hydroxide solution is :
Cl2 + 20H- � c1- + cio- +f¼O

I
(d) 3Cu +2N03 + SH+ --+ 2NO + 3Cu2 + + 4li.i0
I � t j
Obviously the chlorine has changed oxidation state
because it has ended up in compounds starting from the
original element. Checking all the oxidation states shows :
DEEP Focus Cl2 + 20H- � er + c10- + f½O
i ii i ii i i
If one of the half-reactions does not take place, other half 0 -2 +l -1 +l -2 +l -2
will also not take place. We can say oxidation and reduction go
side by side. The chlorine is the only thing to have changed
oxidation state. One atom has been reduced because its
In the above examples, two substances react - one of oxidation state has fallen. The other has been oxidised.
them is an oxidising agent and the other is a reducing This is a good example of disproportionation reaction.
agent. Such reactions relate to intermolecular oxidation­ A disproportionation reaction is one in which a single
reduction ones. substance is both oxidised and reduced.
Consider the following example:
Illustration 1 Write unbalanced oxidation and
3Cl2 + 60H- --+ 5Cl- +Cl03 + 3l¼O reduction half-reactions for the following processes:

� t
-1

I'
O.N. =0 +5
(a) Te (s) + N03 (aq) � Te02 (s) + NO(g)
+ (b) �02 (aq) + Fe2+ (aq) --+ Fe 3+ (aq ) + f¼O (l)
(c) Mn(s) + N03 (aq) � Mn2 (aq) + N02 (g)
+
In this case we find that C� has been oxidised as well
as reduced. Such type of redox reaction is called (d) Mn 3+(aq) --+ Mn02 (s) + Mn2 +(aq)
disproportionation (autoxidation-autoreduction) Solution Te(s) + N03 (aq) � Te02 (s) + NO(
(a) g)
reaction. Examples are: 0 +5 +4 +2


-I----� ---♦ t
+ Disproportionates Oxidation : Te(s) --+ Te02 (s)
2Cu Cu + Cu2 +
Reduction: N03(aq) � NO(g)

• (b) H�P2(aq )tFe+22 + (aq ) � Fe+33+ (aq)+ �O(l)

1-I ---.,,,....� ____.+ t


2HCHO +OH- � CH 30H + Hcoo- -1 -2
2 3
Oxidation : Fe •(aq) � Fe +(aq)

• Reduction: �02(aq) � �O(l)


(c) Oxidation: Mn(s) � Mn2+ (aq)
Reduction: N03 (aq) � N02 (g)
• 3MnO;- + 2�0 --+ Mn02 + 2Mn04 + 40H- (d) Oxidation: Mn 3+ (aq) � Mn02 (s)

I • � t t
Reduction: Mn a+(aq) � Mn2 +(aq)
In this reaction, Mn a+ has been oxidised to Mn02 (s) as
CHO CR,OH
• I + oH- --+ I ..
well as reduced to Mn2 +(aq). Thus, it is a
disproportionation reaction.
CHO coo-
• 2�� --+ 2�0 + 02 Illustration 2 Identify the species oxidised and
reduced in the following:
In disproportionation reactions, compounds are
simultaneously formed that contain a given
element in a more oxidised and more reduced state (a) 2@-cHO + OW- @-CH20H
than the initial one. The initial substance displays
functions of both an oxidising and a reducing agent.
Such reactions are possible if the relevant element is an +@-coo-
intermediate oxidation in the initial compound. In the
following disproportionation reaction
4H3P03 --+ 3H:f04 + PH 3,
178 | Essentia/ Inorganic Chemistry

A A
(0 PC15 PC13 + C12 (b) NH4NO3 N20 + 2H2O
CHO COOH There are two types of nitrogen, one in NH4 ioi
(oxidation number = -3) and the other in NO
NO2 NH2 (oxidation number = + 5). On decomposition, N2<
(d) (oxidation number = +1) is formed. Thus,
NH4 is oxidised to N20 and N03 is reduced to N20.
But it is not the disproportionation reaction.
Solution (a) It is a case of Cannizzaro reaction. Both the elements are in the same initial substance
RCHO has been oxidised to 2?COO" as well as reduced to Reactions of this kind are known as intramoleculai
RCH2OH in a proton-hydride transfer reaction. Thus, it is oxidation reduction reactions.
a disproportionation reaction. PC15 -A> PC13+C12
(c)
CHO coo- tt tt t
oxidation
(O.N.) +5 -1 +3 -1 0
Phosphorus (V) of PC15 is thus reduced to P(III) 0
CHO PC13. Cl (-1) is oxidised to Cl2 (0).
CH2OH
reduction (d) —CHO group is oxidised to —COOH and —NQ
group is reduced to —NH2.
Thus, it is intramolecular redox-reaction.

Target Practice 2
1. I' changes to I" in a reaction. Is it oxidation or reduction? 4. NH4NO2 changes to N2 on heating.

NH4NO2 A N,+2H2O
3
2. On passing CO2 into aqueous yellow coloured CrO>2- 2"
solution, an orange coloured Cr2O2- solution is obtained. Indicate the species (i) oxidised (ii) reduced
This is oxidation or reduction?
CO„ , 5. Select oxidant and reductant in the following :
CrO‘“ (aq) ----- 2-> CrQO2- (a) Sn2+ + 2Hg2+ ---- > Hg2+ +Sn4+
yellow
3. Anhydride of an acid is formed by dehydration and (b) Zn + 20H" ---- > ZnO2-+H,2
oxidation number of central atom remains unchanged. (c) Mn02 ---- > Mn2+ + MnO;
Acid: H2N2O2,HNO2,HNO3
Anhydride : N,O5, N,O, NO, N.,O3 Oxidant Reductant
Match acid with anhydride.

Acid Anhydride
H2N2O2
hno2
hno3

Answers
1. 21' i--> I3
I2 JL 3. H2N„O2---- >N20
hno2 —» n2o3
I" is oxidised tol2 which is soluble in KI formingKI3 (Ij). Thus,
hno3--- » N2Av
O5
I2 is formed by oxidation. v

------ 4+4 ---- > N


4. NH ,920
v
2. CrO2' ---- > Cr2O2-
T
T T +1 Oxidation
+6 +6
In both anion, oxidation number of chromium is +6. Hence, no NOj > N 20
eh an gp Acidic medium due toC02 converts CrO2' intoCr2O2'. +5 +1 Reduction
CO2 + H2O H2CO3 2H++CO2"
5. Oxidant Reductant
2CrO2-+2H+ ---- » Cr2O2"+H2O
yellow orange (a) Hg2* Sn2 +
(b) OH~ Zn
(c) MnOg MnO2
Chapter 4 : Oxidation-Reduction I 179

Practice Exercise 1
I. What is oxidation number of carbon in CN-? (a)K 4f 20 7 (b) KAu CI
4
2. What is the change in oxidation number of the underlined (c)Rb 4NafHYi0028 ] (d) _!Cl
atom in following conversions? (e)Ba(Cl02 )2 CO QFv
(a) NaHg() 3 + HCI ---+ NaCl + H 20 + CO 2 (g)Ba2 Xe06
(b) 2NaOH +Br 2 ---+ NaBr + NaBrO + H 20 11. Identify oxidant and reductant in the following redox
.!
(cl K ---+ KI03 reactions :
(a) 2Mn0 2 + 4KOH + 02 ---+ 2K 2Mn04 + 2H 20
(d) H 2 §06 + 2K .! ---+ K 2S04 + H 0+ 1 2 2
(b) 2K 2Mn04 + Cl2 --+ 2KCI + 2KMn0 4
(e) 2H�O�. ---+ H.-0 + N -0...
1 2 p
ent
(f) Ca0Cl 2 + H 20 ---+ Ca(OH)2 + Cl 2 (c) 2MnO!- + 2H20 curr ) 2Mn04 + 20H- + H2

(g) MnO:. +30H- ---+ Mn02 + H:zO (d) P4 + 502 ---+ P4010
(dil.) 2
(e) Hg 2+ + N0 + H 20 ---+ Hg + N0 + 2H + 3
(h) As 2S 3 + OH- ---+ Ass:- 12. Identify disproportionation reactions in the following:
(a) 3Mno;- + 4W --+ Mn02 + 2Mn04 + 2H 20
3. In H2S 208 oxidation state of sulphur is six? Explain by
structure. Cb) 2KC103 --+ 2KC1 + 302
4. Write a formula for (c) 2Cs1f 5CHO + OH- ---+ Cs1f 5COo- + Caff 5CH 20H
(a) sulphate of iron with Fe in the oxidation state +3. Al(OEt)
(d) 2CH"CHO
., � CH �COOCH 2CH".,
(b) an oxoacid of nitrogen with N in the oxidation state
+3. (e) 4H 3P03 .A.+ 3Hf'O4 + PH3
(c) an oxide of chlorine with Cl in the oxidation state +7.
5. Which is the anhydride ofHN03 out of N 20, NO, N 205 ? (f) P4 + 3NaOH + 3H20 � 3NaH:zP02 + PH3
8. Which is stronger base in each pair? (g) NH4 N03 � N 20 + 2H 20
(a) HSO,I Hso; 2,
(b) N0 N0 3 13. Underline the species which are oxidised and reduced in the
(c) ci-, c10-
following (unbalanced) reactions:
7. vo:+ and V02+ are known as vanadyl ion. Determine (a) CN- + OCI- --+ N 2 + HC0 + c1- 3
oxidation number of vanadium in each. (b) Caff 50H + 0 3 --+ CO2 + H 20 + 0 2
8. Assign oxidation numbers to each atom in the following r
(c) + 03 --+ 02 + 12
substances :
(d) s20;- + Cl2 --+ HSO; + c1-
(a) ethane (Cfi 6 )
(e) C + ZnO ---+ Zn + CO
(b) borax (Na 2B407 )
14. Classify each of the following unbalanced half-reactions as
(c) silicate mineral of magnesium (Mg 2Si20 6 )
either an oxidation or a reduction:
9. (a) What is the type of link.age in (NH3 ) 3 CrO4 if oxidation (a) 02 (g)--+ OH- (aq)
number of Cr is +4?
(b) HP2 (aq) ---+ 02 (g)
(b) K 3Cr(02 ).x has peroxy linkage with oxidation number
of Cr as +5. What is value ofx? (c) MnO�(aq) --+ MnO!-(aq)
10. Determine the oxidation state of the underlined element in : (d) CH30H(aq) � CH20(aq)

Answers
1. +2 (Hint: In the cyanide ion, CN-, N is more electronegative
0 0
T t
than C and taking the common negative oxidation state of N as H-O- S-0-0- S-OH
-3, oxidation state ofC = + 2)
2. (a) No change
0! i
peroxy linkage
0
,1.
(b) Decreases to -1, w.r.t. NaBr and increases to +1 w.r.t.
with oxidation state of
NaBrO 0 = -1 (2 for two oxygen atoms)
(c) 6
Other six oxygen atoms have oxidation state of -2 each, hence,
(d) No change in S of H 2S0 5 (since its perox.y linkage is oxidation state of sulphur is six :
affected) and 1 in KI 2x + 2(for two H)- 12(for six 0)- 2 (perox.y) = 0
(e) No change
:. x = 6 per sulphur atom .
(0 There are two types ofCl inCaOCl2 .Cr (oxidation number
4. (a) Fe2 (SO4 )3 (b) HN0 2 Cc) Cl207
= - 1) and Cl in c10- {oxidation number = +1) c1- is
5. N 20 5 (oxidation number of the element (N) in oxyacid and its
oxidised to CI 2 and Clo- is reduced to Cl2 we find overall
anhydride remains same).
change is zero (since in Ca0Cl2 total oxidation number is 0)
(g) 3 (h) 2
6. (a) HS03 (b) N02 (c) er
Smaller the oxidation number, stronger the conjugate base.
3. H 2S 208 (called Marshall's acid) is perox.y disulphuric acid as
shown: 7. +6 in vo: + and +4in vo z..
180 | Essential Inorganic Chemistry

8. (a)C=-3, H = +l (b) Na = + 1,B = +3,O = -2 (b) Cl ofKClO3 (O.N. = + 5) is reduced to Cl" and 0 ofKC103
(c) Mg=+ 2,Si = +4,O = -2 (O.N. = -2) is oxidised to O2 hence, redox but not
9. (a) (NH3)3CrO4, take oxidation number ofO = x disproportionation.
?0 T T (c) C6H5CHO is oxidised to C6H5COO" as well as reduced to
+4 +4x o C6H5CH2OH (Cannizzaro reaction) hence
X = -1 disproportionation.
Thus, oxygen atoms are in the form of peroxy linkage. The (d) CH3COOC2H6 (ester) is formed from CH3CO0H
compound can be written as (NH3 )3Cr(O2 )2. (oxidation product of CH3CHO) and CH3CH2OH
(b) K3Cr(O2)J hasCr(O2)3’ ion (reduction product of CH3CHO), hence CH3CH0
Cr(O2)3- disproportionates in presence of aluminium ethoxide
T ? (Tischenko reaction).
+ 5 — 2x ~ — 3 (e) H3PO3 (O.N. ofP=+3) disproportionates to H3PO4 (O.N.
ofP=+5) and PH3 (O.N. ofP =-3).
-2x = - 8 (f) P4 (O.N. = 0) disproportionates to NaH2PO2 (O.N. of
x=4 P = +1) andPH3 (O.N. of P= -3).
It isK3Cr(O2)4 (g) NH*4 (O.N. of N = -3) oxidises to N2O (O.N. of N=+1).
10. (a) P = + 5 (b) Au = + 3 (c) V = + 5 (d) I = + 1 NO3 (O.N. of N = +5) redues to N2O; hence redox but not
(e)Cl = + 3 (f) O = + 2 (g) Xe = +8 disproportionation.
13. (a) Oxidation of C(O.N. = + 2) and N (O.N. = - 3) of CN" into
11. Oxidant Reductant
HCO3 (O.N. ofC= +4) and N2 (O.N. of N= 0) respectively
(a) 0, MnO2 and reduction of OCl' (O.N. of Cl = +1) into Cl" (O.N. of
(b) Cl2 K2MnO4 Cl = -1).
(c) H 2O MnO2- (b) Oxidation ofC6H5OH into CO2 by O3
(d) O2 P4 (c) Oxidation of I" into I2 by O3
(e) Hg2* NO2 (d) Oxidation ofS2O2’ into HSO4 by Cl2
(e) ZnO is reduced to Zn
12. (a) MnO'i2- is reduced to MnO2 as well as oxidised to MnO4
14. (a) Reduction (b) Oxidation (c) Reduction
hence, disproportionation.
(d) Oxidation
•_*a

Do you Know..... How to Balance a Redox Reaction?


A redox predominance diagram shows the predominant Working out electron-half-equations and using them
(thermodynamically most stable) oxidation state and chemical to build ionic equations.
form of an element at any given potential. In this type of
diagram, elements with no redox chemistry (such as Ar) will Illustration 1 The reaction between chlorine and iron
predominate at all potentials and will cover the entire diagram. (II) ions
Among those with more than one thermodynamically Chlorine gas oxidises iron (II) ions to iron (III) ions. In
stable oxidation state, more strongly oxidising stable
the process, the chlorine is reduced to chloride ions.
chemical forms are confined to higher regions of redox
predominance diagrams. Correspondingly strongly reducing
You would have to know this, or be told it by an
stable chemical forms will be low in the predominance examiner. In building equations, there is quite a lot that
diagrams Fig 4.1 represents redox predominance diagram of you can work out as you go along, but you have to have
iron. Of the chemical forms of iron shown, the most strongly somewhere to start from!
oxidising is the ferrate ion.FeO2-; the most strongly reducing is You start by writing down what you know for each of
iron metal, Fe. the half-reactions. In the chlorine case, you know that
chlorine (as molecules) turns into chloride ions :
Oxidation
state
Half-reaction E°/V ci2 —> cr
The first thing to do is to balance the atoms that you
+6 FeO2- + 8H‘ + 3e" Fe3* +2.20 V FeO2"
have got as far as you possibly can :
+ 4H2O 2.20
Cl2 ---- > 2C1’
+3 Fe3* Fe2* +0.77 V Fe3* Always check that you have the existing atoms
0.77 balanced before you do anything else. If you forget to do
+2 Fe2* + 2e" Fe(s) -0.44 V Fe2* this, everything else that you do after wards is a complete
-0.44 waste of time!
Fe Now you have to add things to the half-equation in
order to make it balance completely.
Fig. 4.1 Redox predominance diagram for the element iron,
All you are allowed to add are :
with the corresponding reduction half-reactions
• electrons
• water
Chapter 4: Oxidation-Reduction | 181

• hydrogen ions (unless the reaction is being done under you will mess up the oxygens again-that’s obviously
alkaline conditions in which case, you can add wrong!
hydroxide ions instead) Add two hydrogen ions to the right-hand side.
In the chlorine case, all that is wrong with the existing HA ---- > O2 +2H+
equation that we’ve produced so far is that the charges Now all you need to do is balance the charges. You
don’t balance. The left-hand side of the equation has no would have to add 2 electrons to the right-hand side to
charge, but the right-hand side carries 2 negative charges. make the overall charge on both sides zero.
That’s easily put right by adding two electrons to the HA ---- > O2 +2H+ +2e~
left-hand side. The final version of the half-reaction is : Now for the manganate(VII) half-equation:
Cl2+2e" ---- > 2C1’ You know (or are told) that the manganate(VII) ions
Now you repeat this for the iron (II) ions. You know (or turn into manganese(II) ions. Write that down.
are told) that they are oxidised to iron (III) ions. Write this MnO4’ ---- > Mn2+
down: The manganese balances, but you need four oxygens
Fe.22++ ---- > Fe3+ on the right-hand side. These can only come from
The atoms balance, but the charges don’t. There are 3 water-that’s the only oxygen-containing thing you are
positive charges on the right-hand side, but only 2 on the allowed to write into one of these equations in acid
left. conditions.
You need to reduce the number of positive charges on MnO^ ---- > Mn2+ + 41^0
the right-hand side. That’s easily done by adding an
By doing this, we’ve introduced some hydrogens. To
electron to that side :
balance these, you will need 8 hydrogen ions on the
Fe2+ ---- > Fe3++e" left-hand side.
It is obvious that the iron reaction will have to happen MnO; +8H+ ---- > Mn2+ + 4^0
twice for every chlorine molecule that reacts. Allow for
that, and then add the two half-equations together. Now that all the atoms are balanced, all you need to do
is balance the charges. At the moment there are a net 7+
2x[Fe2+ ---- > Fe3+ + e“ charges on the left-hand side (1- and 8+), but only 2+ on
Cl2 + 2e“ ---- > 2C1" the right. Add 5 electrons to the left-hand side to reduce
2Fe2*+Cl2 ---- > 2Fe3*+2Cr the 7+ to 2+.
MnO; + 8H+ + 5e" ---- > Mn2+ + 4^0
But don’t stop there!! Check that everything
This is the typical sort of half-equation which you will
balances-atoms and charges. It is very easy to make small
have to be able to work out. The sequence is usually :
mistakes, especially if you are trying to multiply and add
up more complicated equations. • Balance the atoms apart from oxygen and hydrogen.
• Balance the oxygens by adding water molecules.
Illustration 2 The reaction between hydrogen • Balance the hydrogens by adding hydrogen ions.
peroxide and manganate (VID ions • Balance the charges by adding electrons.
The first example was a simple bit of chemistry which Combining the half-reactions to make the ionic
you may well have come across. The technique works just equation for the reaction
as well for more complicated (and perhaps unfamiliar)
The two half-equations we’ve produced are :
chemistry.
This reaction needs 5 electrons
Manganate (VII) ions, MnO^, oxidise hydrogen /
peroxide, HA, to oxygen gas. The reaction is done with MnO; + 8H+ + 5e~ ---- > Mn2+ + 411,0
potassium manganate (VII) solution and hydrogen
peroxide solution acidified with dilute sulphuric acid. HA ---- > 02 +2H++2eL
During the reaction, the manganate (VII) ions are but this is only producing 2 electrons
reduced to manganese (II) ions.
You have to multiply the equations so that the same
Let’s start with the hydrogen peroxide half-equation. number of electrons are involved in both. In this case,
What we know is : everything would work out well if you transferred
H2O2 > O2 10 electrons.
The oxygen is already balanced. What about the 2 x (MnO4“ + 8H+ + 5e"---- > Mn2+ + 4^0)
hydrogen?
5x(HA ---- > O2+2H++2e~)
All you are allowed to add to this equation are water,
2MnO; + 16H+ + 5HA ---- > 2Mn2+ + 8H2O + 5O2
hydrogen ions and electrons. If you add water to supply
the extra hydrogen atoms needed on the right-hand side, + 10H+
182 | Essential Inorganic Chemistry

But this time, you haven’t quite finished. During the Illustration 3 Cr202' + Fe2+ ---- > Fe3+ + Cr3+
checking of the balancing, you should notice that there are In this Cr atoms on RHS are not balanced. Hence, fi rd
hydrogen ions on both sides of the equation : step is to balance Cr3+ and take O.N. of two Cr atoms
hydrogen ions on both sides of the equation < Cr.O2/ +Fe2+ ---- > Fe3+ +2Cr3+
+12 .+2 *3 +6
2MnO; +16H+ +5H2O2 ---- > 2Mn2++ SHjO + 5O2
+ 10H+
I t
By taking steps of illustration (1), balanced equation is
You can simplify this down by subtracting 10
hydrogen ions from both sides to leave the final version of C^O2/ + 6Fe2+ + 14H+ ---- > 6Fe3+ + 2Cr3+ + 7H2C
the ionic equation-but don’t forget to check the balancing
of the atoms and charges! Illustration 4 MnO; + C20j“-----> 2CO2 + Mn2*
2MnO4‘ + 6H+ + 5H2O2 ---- > 2Mn2++ 81^0 + 502 MnO; +C2(^- ---- > 2CO2 +Mn2+
+7 +6 +8 +2
You will often find that hydrogen ions or water I -2g~ I
molecules appear on both sides of the ionic equation in __________ +5e~_______
complicated cases built up in this way. Always check, and
then simplify where possible. Carbon atoms are taken balanced and taking usua
steps:

Working out oxidation number and using them 2MnO; + 5A0J- + 16H+ * 10CO2 + 2Mn2+ + 8HgO
to balance equation Illustration 5 H2O2 + MnO; —> Mn2+ + 02
In. the entrance examination, it is never mentioned -l2+ +02
H2O2 +Mn0; ---- > Mn
what method is to be used. We adopt here “quick” method -2 +7 +2 0
that will certainly be a time-saving method. I R f
0

Acidic Medium H2O2 has been oxidised to 02. Thus, oxygen atoms
undergoing oxidation are balanced. Taking O.N. of both
Illustration 1 We want to balance following
equation: oxygen and as by usual steps, balanced equation is
Cu + NO; ---- > NO + Cu2+ 5H2O2 + 2Mn0; + 6H+ ---- > 2Mn2+ + 5O2 + SHjO
Cu +N0; ---- > NO +Cu2+ Total H on LHS = (double ofE^O)- (H of HA)
0 +5 +2 +2
I +3e~ f Illustration 6 We may face some difficulties when
-(2e“)
two or more atoms are oxidised or reduced. In the following
ALERT example
• Affected atoms are balanced. As2S3+NO;+H+---- > AsO3-+ S + N02 +1^0
• Take total O.N. of the affected atom on each side. As3+ as well as S2- have been oxidised to AsO3' and! S
(a) Cross multiply by change in O.N. respectively. In such case take total change (increase or
3Cu + 2NO; ---- > 2NO + 3Cu2+ decrease) in O.N. for cross-multiplication.
2As 3+ +3S-«2’ +N0; ---- » N0 2 +3S+2A sO3“
(b) Balance oxygen by EL/) +« +53 +4 0 +10

3Cu + 2NO3 ---- > 2N0 + 3Cu2+ + 41^0 I ■—t t


(c) Balance hydrogen by H+ 0

3Cu + 2NO3 + ---- > 2N0 + 3Cu2+ + 4^0


Total change in O.N. of oxidised part = 10
Illustration 2 MnO; + Fe2+
1 -----> Fes+ +Mn2+ Hence, by usual steps, balance equation is
MnO; +Fe2+ ---- > Fe3++Mn2+ As2S3 + IONO3 + 4H+---- > 10NO2 + 2AsO43“ + 3S + 2H/0
-r“. ♦
. +5e fi
Basic Medium
step (a) MnO; + 5Fe~+ 5Fe3+ + Mn2+ We adopt here same method as in acidic medium,
difference is only in balancing of oxygen.
step (b) MnO; + 5Fe2+ 5Fe3+ + Mn2* + 4H/)
— Cross-multiply by change in O.N.
step (c) — Add H2O on excess oxygen side.
MnO; + 5Fe2+ + 8H+ 5Fe3* + Mn2+ + 4^0 — Add 0H“ twice of H2O on the opposite side.
Chapter 4 : Oxidation-Reduction | 183

Illustration 1 B12 disproportionates into Br" and Illustration 4 Write unbalanced half-reactions for the
BrO3 in alkaline medium. following net ionic equations'.

I b* 4
0 0
Br +BrO3
-1 +5
(a) Mn2+(ag) + C103(ag) -> MnO2(s) + C102(ag)
(b) CejO^"(aq) + Fe2+(ag)---- > Cr3+(ag) + Fe3+(ag)
I__ R f (c) Cl2(g) + OH"(ag)---- > Cl"(ag) + C103(ag)
o
Oxidation Reduction
Cross multiplication by change in O.N. gives (a) Mn2’(aq) —» MnO2(s) CIO; (aq) - -» CIO2 (aq)
3Br2 ---- > 5Br" + BrO3 (b) Fe2’(aq) —> Fe3* (aq) Cr2O2'(aq) —> Cr3+(aq)
(We have taken Bi^ for oxidation and reduction to (c) Cl2(g) —> CIO; (aq) Cl2(g) —< Cl" (aq)

balance affected atoms.) Illustration 5 Balance the following half-reactions in


Three 0-atoms are extra on RHS, hence, add three acidic medium. You may balance charges by putting electrons.
HgO molecules (excess O-side) on RHS, and add 6OH" on (а) V02+(ag)---- > V3+(ag)
LHS
(б) N03(ag)---- > N02(s)
3Br2 + 60H" ■> 5Br" + BrO3 + 3^0 (c) I03(ag)---- > I3(ag)
Basic MnO2 +Q2
Illustration 2 Mn04" +11,02 (a) ....
VC^^ag) y’^a?)
+7-2 +4 0 T T
+4 +3
Affected atoms are balanced; by cross-multiplication
VO2* is reduced to V3+ by gain of one electron.
2MnO4 + 3H2O2 ---- > 2MnO2 + 3O2
VO2+(ag) ---- > V3+(ag)
14(0) wW Balance 0 : V02+(ag) ■> V3+(ag) + HjCXZ)
add H20 on LHS (excess O-side) Balance H : VO^ag) + 2H+(ag) * V3+(ag) + 1^0(0
2MnO4 + 3H2O2 + 4^0---- > 2MnO2 + 3O2
Balance charge:
add OH"twice ofH2O on RHS
V02+(ag)+ 2H+(ag) + e" * V3’(a9) + H/X/)
2MnO4 + 3H2O2 + 4^0---- > 2MnO2 + 3O2 + 80H"
We have proceeded in a right way, but we have not (b) NO3(ag) * NO2(g)
+5 +4
considered H due to H2O2. Hence, H-atoms as well as
charge are not balanced. Hence, add 6H+ on RHS which NO3(ag) + 2H+(ag) + e~ ---- > NO2(£)+ H2O(Z)
will neutralise OH" forming 6^0 and thus, final result (c) IO3(ag)-----> I3(ag)
will be IO3(ag) is first oxidised to which dissolves in I"
2MnO4 + 3H2O2 —-> 2MnO2 + 3O2 + 20H" + 2H2O forming I3

Illustration 3 A purple solution ofaqueous potassium IO3(ag)


+5 2 o
permanganate (KMn04) reacts with aqueous sodium
sulphite (Na2SO3) in basic solution to yield the green iljfsl + SHaCKZ)
IO3(ag) + 6H+ + 5e
manganate ion (Mn0^“) and sulphate ion (SO^"). The 2
unbalanced net ionic equation is: Note decrease in oxidation number = electrons added on LHS
Mn04(ag) + SOg" (ag) MnOj"(ag) + SOj [aq) increase in oxidation number = electrons added on RHS
Balance the equation. Illustration 6 Balance the following half-reactions in
change in O.N.
a basic medium :
reduction4 Mn0^"(ag) (a) Cr(^"(ag)---- > Cr(OH);(ag)
MnO4 (ag) lunit
+7 +6
(b) Bi3+(ag)----> BiO3(ag)
SO^" (ag)---- > SO^" (ag/ x 2 units (c) C10"(ag)--- > Cl"(ag)
+4 +6
Cross-multiplication followed by addition gives
(a) Cr<^~(aq) --- > CrtOH);(a9)
2MnO4 (ag) + SO^" (ag) ---- > 2Mn0^"(ag) + SO^"(ag) T T
+6 +3
11 oxygen 12 oxygen
CrC^-(ag) + 4H2O(Z) + 3e" ■> Cr(OH)4(ag)
Add H^O on extra oxygen side and add twice OH" on
+ 4OH"(ag)
opposite side
(b) Bi3+(ag)---- > BiO3(ag)
2MnO4(ag) + SO^"(ag) + 20H"(ag)---- > +3 +5

2Mn0^"(ag) + SO^"(ag) + H20(ag) Bi3+(ag) + 60H" ---- > BiO3(ag) + 3^0(2) + 2e"
184 | Essential Inorganic Chemistry

(0 CIO- (ag) - 4 Cl"(ag) + 2H2O Illustration 8 Write balanced net ionic equations fr
+1
the following reactions in acidic solution:
C10"(aq) + H2O(Z) + 2e" -----> Cl“(ag) + 20H"(ag) Zn2+(ag) + V3+(ag)
(а) Zn(s) + VO2+(ag)
Illustration 7 Write balanced net ionic equations for (б) Ag(s) + N03(ag) Ag+(ag) + N02(£)
the following reactions in basic solution: (c) Mg(s) + VOj’(aq) Mg2+(ag) + V2+(ag)
(а) Mn04(ag) + IO3(ag)----- > Mn02(s) + lO^Cag)
(б) Cu(0H)2(s) + N2H4(ag) -----> Cu(s) + N2(g) (d) I~(aq) + IO3(ag) • E3(aq)
(c) Fe(OH)2"(s) + CrC^"(ag)-----> Fe(0H)3(s) Solution
+ Cr(OH)4 (aq) (a) Zn(s) + 2VO2+(ag) + 4H+(ag)-----> Zn2+(ag)
(d) HjOoCag) + C104"(ag) 4 C102(ag) + 02(g) + 2V3+(ag) + 2^0(1
Solution (b) Ag(s) + 2H+(aq) + NO3(ag)-----> Ag+(ag)
(a) 2MnO7(ag) + 3IO3(ag) + H2O(Z) ---- > + NO2(g) + H20Xi
2MnO2(s) + 3IO7(ag)+ 20H" (aq) (c) 3Mg(s) + 2VO3-(ag) + 16H+(ag)-----> 3Mg2+(ag)
(b) 2Cu(OH)2(s) + N2H4(ag)-----» 2Cu(s) + N2(g) + 2V2+(ag) + 8H20'(Z
+ 4H2O(Z)
(c) 3Fe(OH)2(s) + CrOj" (aq) + 4H2O(Z) (d) I03(ag) + 8I"(ag) + 6H+(ag)----- > 3I3(ag)
4
+ 3H2O(Z
3Fe(OH)3(s) + Cr(0H)4"(ag) + OH’(ag)
(d) C107(ag) + 2H2O2(ag) 4 C102(ag) + 2H2O(Z)
+ 2O2(g)

Balanced Redox Reactions


Some balanced redox reactions are in the medium specified: 20. 2Fe2S3 + 6H2O+ 3O2 ---- > 4Fe(OH)3+6S
1. 3S2O^- + 2CrOj- + 4H2O---- > eSO^ 21. 3IBr + 2BrO3 + 3H>0 — 3IO3 + 5Br" + 6H+
(dithionite ion) 22. As2S3 + 120H" + HHA —> 2AsOj" + 20H20
+ 2Cr(OH)3 + 2H+ + 3S0j
2. 3CN" + 2MnO4" + H20 4 2MnO2 + 3CN0" 23. 2CrI3 + 27H2O2 + 10OH" ---- > 2CrOj- + 6IO4-
+ 20H" + 32H2<
3. 4[Fe(CN)6]3- + N2H4 + 40H" 4[Fe(CN)6]4-
24. 4Ag + 8CN" +02 + 2^0 4 4[Ag(CN)2F
+ N2 + 41^0 + 40H
4. 4Fe(OH)2 + 02 + 2H>0 ---- > 4Fe(OH)3
25. B2C14 + 60H" 4 2BO2 + 4C1’ + 2^0 + H2
5. 3C2H5OH+ 4Mn0; — 4 3CH3COO- 3’ + 4MnO2 26. 2Ag2S + 8CN" + 02 + H/) 4 4[Ag(CN)2r
+ 41^0 + OH" + 2S + 40H
6. S2O23‘+5H2O + 4C1.2 ---- > 2SO5" + 8C1“ + 10H+ 27. H2O2 + 2H++ 2Fe2+ ---- > 2^0+2Fe3+
7. 2NH4 + Ciotff —4 Cr>03 + N2 + 4H2O 28. CrO + 2C10- + 20H“ ---- > CrO2- + 2C1' +
8. P4 + 8H+ + 20NO3 + 8H0O---- > 12H2PO4- + 20NO 29. 5As2S3 + 4MnO4 + 9^0 + 12H+ ---- > 10H3As04
9. 2MnO4 + 6H+ + 5NO2---- > 2Mn2+ + 5NO3 + 31^0 + 15S + 4Mn2
10. S8 + 120H" ---- > 4S2" + 2S2O^- + 6H>0
30. O3 + 2r+H2O---- > 02 + + 20H"
11. 5S2O|" + 8MnO4 + 14H+ ---- > lOSO^" +8Mn2+
+ 7^0 31. S20|- +4C12 +5H2O---- > 2HS0; + 8H+ +8C1-
12. 2HS" + 4HSO3 - 3S2O23" + SHzjO 32. 2CN’ +5C10" +1^0--- > 2HCO3 + 5C1' +N2
13. 4Fe3+ + 2NH2OH 4 4Fe2++ N2O + 4H+ 33. 6C1O2 + 60H" ---- » 5C1O3 + Cl" + 31^0
14. 4O2 + 21^0-----> 4OH" + 3O2 34. 4Zn + 10H+ + N03 ---- > 4Zn2+ + NH4+ + 3^0
(superoxide ion)
35. 4Zn + 10H+ + 2NO3 ---- > 4Zn2+ + N20 + 5^0
15. 2(Fe(CN)6J4" +H2O2+2H+---- > 2[Fe(CN)6]3'
(ferrocyanide ion) (ferricyanide ion) 36. 2Mn0 + 5PbO2 + 8H+---- > 2MnO4" + 5Pb2+ + 4H,(
+ 21^0 37. 20C1" + 20H" + Bi2O, ► 2BiO3 +1^0 + 201-
16. H2C2O4 + Mn02 + 2H+ - ------ > Mn 2+ + 2^0 + 2CO 2 38. 8A1 + 30H" + 3HSO4 - 4A12O3 + SHjO + 3S2-
17. IO3 + 3HSO3 ----- > 1r + 3H+ + 3S0J-
39. 4Ag + 8CN" + 02 + 2H>0---- > 4Ag(CN)2 + 40H-
18. 51" + IO3 + 6H+ ------4> 3H2O + 3I2
40. P4 + 40H" + 2^0---- > 2PH3 + 2HPO23"
19. 3MnOj" + 21^0----- > Mn02 + 2MnO4 + 40H"
Chapter 4 : Oxidation-Reduction I 185

Target Practice 3
Balance the following redox reactions 6. Br2(aq) + OW(aq) ----+ Br-(aq) + Br03(aq)
1. S02(g) + HN02(aq) � H 2S04 (aq) + NO(g) 7. Mn(s) + HN0 3(aq) ----+ Mn .. (aq) + NO 2(g)
2

2. Al(s)+ H 2S0 4 (aq) � Al 2 (S04)3(aq)+ H 2(g) 8. l2(s) + OCr(aq) ----+ I03(aq) + Ci-(aq)
3. Au (aq) + r(aq) � Au(s) + l 2(s)
3+
9. Cr:_p;-(aq) + HN02(aq) � Cr • (aq) + NO;(aq)
3

4.
2
s-(aq) + 12(s) ----+ SOt(aq) + r(aq) 10. CrOt(aq) + s 2 -(aq) ----+ Cr(OH) 3 (s) + S(s)
5. H20 2(aq) + CIO�(aq) ----+ 02(g) + CIO;(aq)

Answers
7. Mn(s) + 2NO3(aq) + 4H (aq) --+ Mn 2 •(aq)
+
1. SO 2 (g) + 2HNO 2 (aq) --+ H 2 SO 4(aq) + 2NO(g)
2. 2Al(s)+ 3H 2SO 4 (aq) --+ Al 2 (SO 4 )3(aq)+ 3H 2 (g) + 2NO 2(g) + 2H 2 O

3. 2Au 3 ..(aq) + 6r-{aq) --+ 2Au(s) + 3I 2 (s) 8. I2(s) + 5O Ci-(aq)+ H 2O(l) --+ IO;(aq) + 5Ci-{aq)
2
4. S -(aq) + 4l2(s) + 4H20([) --+ Sr(aq) + So�-(aq)
+ 2H� (aq)
9. Cr2 O�-(aq) + 6NO2(aq)+ BW(aq) ---4 2Cr 3 •(aq)
+ aH•(aq)
+ 6NOj(aq)+ 4H2O(l)
5. 2H 2 O 2(aq) + C10 4 (aq) ---4 2O 2(g) + ClO2(aq) + 2H 2 O(l)
10. 2cr0!-(aq) + 3S2-(aq)+ 8H2O(l) --+ 2CrtO H)a(s)
6. 3Br2(aq) + 6OW( aq) --+ 5Br-(aq) + BrOj(aq)+ 3H 2 O(l)
+ 3S(s) + l0OW(aq)

Equivalent Weights (ii) Conversion into oxide


In unit 1, detailed study of equivalent weight has Mass of non-metal
been given. To determine number of equivalents or Compound Oxygen combining with 8 g
Non-metal oxygen = equivalent
with oxygen combined
, equivalent weight we can compare the molecular weight to weight

= gH mol ofH
1 =1
+
Carbon-12 CO 2 32g 12
-X8=3g

= 17 OH- 1 mol of OH­


g =
Nitrogen-14 NO 16g
32
14
-X8=7g
!! 12 g C 16

= 35.5 g Cl Nitrogen-14 NO 2 32 g 14
-X 8= 3.5 g
32
=8 g 0 Nitrogen-14 N2O 16g 28 8 = 14g
- X
16

In the above example, it is clear that depending upon


Equivalent Weight of Non-metals the compound formed nitrogen has different equivalent
Equivalent weight of non-metals can be determined weight.
by
Equivalent Weight of Metals
(I) Direct combination with hydrogen
1. Replacement of hydrogen from acids
Equivalent We compare the mass of metal that displaces 1 g (H2 )
Mass= of
Compound Mass of Zn + 2HC1 --+ ZnC12 +H2
Non-metal non-metal 2g
with hydrogen hydrogen (M) g
combining
with 1 g H H2 !!Zn
2gH2 !!!!!M g metal
(a) Carbon-12 CH4 4g 3g
M
lgH2 !!!!!
(b) Boron-10.8 BH 3 3g 3.6g 2
Atomic weight
(c) Nitrogen-14 NH3 3g 4.66g . . Equivalent weight ofZn =
2
186 | Essential Inorganic Chemistry

(b) 2A1 +6HC1 AICI3 +3H2 3. Conversion of Metals into Chlorides


(2Af)g 6g
We compare the mass of metal that combines with
3H2 = 2 Al 35.5 g of chlorine.
6gH2 = 2Afg Let mass of metal = a g
M
1 g H2 = — - Mass of metal chloride = b g
b 3
Chlorine combined = (b - a) g
Equivalent weight of Al = atomic weight
3 (b- a) g chlorine = a g metal
35.5 g chlorine = —-— x 35.5 g
(b - a)
2. Conversion of Metals into Oxide
35.5a
We compare the mass of metal that combines with 8 g /. Equivalent weight of metal =
(b-"a)
oxygen.
2M + xO2 ---- > Af2Ox Alternatively
2M = 32r g (x = valency of metal M) M + — Cl2 ---- > MClx equivalent weight of M
2
32.r g O2 - 2 atomic weight _ atomic weight
2xatomicweight x
32r
atomic weight Li + -Cl2 LiCl x -1; equivalent weight of
2
2x Li = atomic weight
We can also calculate equivalent weight by taking Mg + Cl2 MgCl2 x = 2; equivalent weight of
weight of metal and metal oxide. , , atomic weight
Mg------------------
Let weight of metal = a. g
Weight of metal oxide = b g 3
Al + - Cl2 ---- > A1C13 x = 3; equivalent weight of
Oxygen combined = (b - a) g 2
.. _ atomic weight
(b - a) g oxygen combined with = a g metal
3
8 g oxygen combined with = —-— x 8 g
b-a Illustration 1 (a) 0.24 gof magnesium combined with
80 oxygen forming 0.40 g magnesium oxide. Derive equivalent
Equivalent weight of metal =--------
(b - a) weight’of magnesium.
(a) 2Na + -O2 ■> Na20 (b) 0.27 g aluminium combined with hydrogen
2 2 forming 0.30 g aluminium hydride. Derive equivalent
1 weight of aluminium.
x=-
2 (c) 1.95 g of platinum forms 3.37 g of chlorides on
Equivalent weight of sodium = atomic weight reaction with chlorine. Derive equivalent weight of
= 23 g equivalent-1 platinum.
(b) 2Mg + O2---- > 2MgO Solution (a) Mass of magnesium = 0.24 g
x=l Mass of magnesium oxide = 0.40 g
atomic weight
Equivalent weight of magnesium = Oxygen combined = 0.40 - 0.24
= 24 = 0.16 g
2 0.16 g oxygen combined with = 0.24 g magnesium
= 12 g equivalent-1 0 24
Q 8 g oxygen combined with = x 8 = 12 g
(0 2A1 + -0, —> A12O3
2
3 Equivalent weight of magnesium
x=—
2 = 12 g equivalent-1
_ atomic weight (b) Mass of aluminium = 0.27 g
Equivalent weight of aluminium
~ -
Mass of aluminium hydride = 0.30 g
= 27
3 Hydrogen combined = 0.03 g
= 9 g equivalent' 0.03 g hydrogen combines with = 0.27 g aluminium
Chapter 4: Oxidation-Reduction | 187

0 27 Illustration 1 Calculate moles o/“Ca(OH)2 required to


1 g hydrogen combines with = —— x 1 g = 9 g
0.03 carry out following conversion taking one mole in each
Equivalent weight of aluminium
= 9 g equivalent
—-
case'- COOH coo
(a) I into I Ca (6) H3PO4 into CaHPO4
(c) Mass of platinum = 1.95 g COOH CO*
Mass of platinum chloride = 3.37 g (c) NHjCl in to NH3 (d) NaHCO3 into CaCO3
Mass of chlorine combined = 3.37 - 1.95 COOH
Solution (a) I is a dibasic acid
= 1.42 g COOH
1.42 g chlorine combines with = 1.95 g platinum COOH COOH
• qe c ki • k- -^k 1-95 x 35.5 I +Ca(OH)2 Ca
.. 35.5 g chlorine combines with = —^42— COOH COOH
1 mol 1 mol
= 1?® = 48.75 g
4 Ca(OH)2 required = 1 mol
(b) H;jPO4 + Ca(OH)2 —> CaHPO4 + 2H2O
Equivalent weight of platinum
1 mol of HjjPQj =2H+ neutralised by 1 mol ofCa(OH)2
= 48.75 g equivalent
Ca(OH)2 required = 1 mol
(c) 2NH4C1 + Ca(OH)2 ---- > CaCl2 + 2NH3 + 2H2O
Equivalent Weight of Acids and Bases 2 mol NHjCl =1 mol Ca(OH)2
In a Neutralisation Reaction 1 mol NHjCl = 0.5 mol Ca(OH)2
We compare molecular weight w.r.t. ionisable H+ (in (d) 2NaHCO3 + Ca(OH)2---- ■> Na2CO3 + CaCO3
acids, called basicity) or OH- (in bases, called acidity). + 2H2O
M 2 mol NaHCO3 = 1 mol Ca(OH)2
HC1 H+ + cr E = M/1 36.5 1 mol NaHCO3 = 0.5 mol Ca(OH)2
HgPOj, H+ +H2PO2 E = M/1 66.0 Illustration 2 Calculate equivalent weight of the
E = M/1 61.0 underlined:
HCO3 H+ + col-
(a) Ca(OH)2 + 2H2SO4 ---- > Ca(HSO4)2 + 2H2O
H2SO4 2H+ + SOT E = M/2 98.0
(b) [A1(OH)3(H2O)3] + OH- —> [A1(OH)4(H2O)2]
HgPOa 2H+ + HPOl" E = M/2 82.0 + h2o
(c) [A1(OH)3(H2O)3] + H3O+ —4 [A1(OH)2(H2O)4 ]
H2C2O4 2H+ + QOt E = M/2 90.0
+ h2o
H^ 3H+ + PO4“ E = M/3 98.0
Solution (a) H2SO4 changes to HSO4 i.e., only one H+
NaOH Na+ + OH" E=M 40.0 reacts
Ca(OH)2 Ca2+ + 2OH" E = M/2 74.0 H2SO4 = H+
Equivalent weight of H2SO4 = molecular weight
A1(OH)3 Al3+ + 3OH" E = M/3 78.0
= 98
Facts: It must be noted that equivalent weight (b) [A1(OH)3(H2O)3] = 1 OH-
is not a definite property but varies reaction to equivalent weight = molecular weight
reaction.
(c) [A1(OH)3(H2O)3]=1H3O+
H3PO4 is a tribasic acid. But in the following reaction equivalent weight = molecular weight
only two H+ ions have been neutralised:
HaPO4 + Ca(OH)2 ---- > CaHPO4 +2H2O
In a Redox Reaction
Hence, equivalent weight of H3PO4 = M/2 = 49
Depending on the nature of products of a redox
and if reaction proceeds completely reaction, equivalent weight is determined.
H3PO4 + 3NaOH---- > NagPO4 + 3H2O • Compare molecular weight w.r.t. 8 g (O) if oxygen is
Equivalent weight = y- = = 32.66 gained or lost. For this convert every (O) atom gained
or lost into two parts :
188 | Essential Inorganic Chemistry

(a) Cl2 + H2O-----> 2HC1 + 0 (0 MnO2 + 4HC1---- > MnCl2 + Cl2 + 2H2O
Cl2 = 1 (O) taking weight as 16 Cl2 + 2KI---- > 2KC1 +12
= 2 (0) taking weight as 8 MnO2 = I2 = 21
Eq. wt. of MnO2 =
M
Equivalent weight of Cl2 = — (d) Cr2O27" + 6F + 14H+ ---- > 3I2 + 2Cr3+ + 7H20
2
(b) KMnO4 in acidic medium loses 2.5 oxygen atoms Cr2O^“ = 3I2 = 61

2KMnO4 + 3H2SO4-----> K2SO4 + 2MnSO4 + 3H2O Eq. wt. of CrjjO?- = y


+ 5(0)
Thus,KMnO4 = 2.5 (0) atoms taking weight as 16 DEEP Focus
= 5 (0) atoms taking weight as 8 • Easiest method of determination of eq. wt. in a redox
KMiA^ o) reaction is by
5 formula weight (stoichiometric)
Eq. wt. =
M change in O.N. or electron exchanged
Eq. wt. of KMn04 in acidic medium = —
5 We have used the term “formula weight” to simplify the
(c) KMn04 in basic and neutral medium loses 1.5 definition. While using this term, check
~ affected atoms are balanced
oxygen atoms
- take total O.N. of the affected atom of each side.
2KMnO4 + H20-----> 2MnO2 + 2K0H + 3(0)
Thus,KMnO4 = 1.5 (O) atoms taking weight as 16 Illustration 1
= 3 (O) atoms taking weight as 8
KMnO4 x (Q) (a) Mn04“ » Mn2+
+7 +2
3
M „ . mol. wt. of one unit of Mn04
.-. Eq. wt. of KMnO4 in basic medium = — Eq. wt. =----------------------------------- -

(d) KMnO4 in strongly alkaline medium loses 0.5 (b) H2O2 ---- > 02
-2 0 O.N. of two (0) on each side
oxygen atom
T7 X M
KMnO4 = 0.5 (O) atom taking weight as 16 Eq. wt. = —
= 1 (0) taking weight at 8.
M (c) CigO?' ---- > Cr3+ check here Cr atoms are no
Eq. wt. of KMnO4 in neutral medium = —
ofKMnO
balance
Thus, eq. wt. is not a definite property. It also depends Ci^O?" ---- » 2Cr■3+
: now balance
on medium. Eq. wt. of KMnO4 as we shall above is quite 12 6 O.N. of two Cr atoms
different in three media, acidic, basic and neutral, (mol. 17 x M
wt. = 158) Eq. wt.= —
Acidic = — = 31.6
5 We can also take it as follows :
M
Basic and neutral = — = 52.66 CrzO?- —> Cr3+
3
M - Cr2O^~ ---- > Cr3+ Cr atoms are balance
Strongly alkaline = — = 158.0 2 6 3 O.N. of one Cr atom on each side
f |) formula weight
• In certain cases I2 is formed or used. In such cases
M/2 M
compare molecular weight with one I atom. Eq. wt. = ------ =----
3 3 6
(a) 2CuSO4 + 4KI-----> I2 + Cu2I2 + 2K2SO4
2CuSO4 = I2 = 21 (half molecule is on LH
1= CuSO4 (d) N2 + 3H2 2NH3
0 -6 O.N. of two N-atoms
Eq. wt. of CuSO4 = M
Eq. wt. of NH3 =2 formula weight of NH3 = M = J
(b) 2Na2S2O3 + I2 ---- > 2NaI + Na2S4O§
6 3
2Na2S2O3 = I2 = 21 28 14
Eq. wt. of N2 = —■
Eq. wt. of Na2S2O3 = M 6 3
Chapter 4: Oxidation-Reduction | 189

(e) 2MnO^" ---- > MnO2 + MnO4" :. Equivalent weight of Br2 in oxidation process
MnO^“ disproportionates to MnO2 and MnO4. If ER is = 96-80 = 16
the equivalent weight in the reduction half-reaction and Let oxidised product may contain one bromine
Eo is the equivalent weight in the oxidation half-reaction - Br2 ---- > oxidised part
then net equivalent weight E 2 n
0
E = ER+E0 80
Equivalent weight = 16 ------------ ~v
MnO^- ---- > MnO2 change in O.N.
+6 +4 R 2 _ 80
M n
MnOT ---- > MnO4 Eo = —
+6 +7 11 80 ..
n= —=5
M M , _„ 16
E(net) = — + —-1.5
= 1.5 M
2 1 Thus, oxidation number of Br in oxidised part is +5. It
(f) ci2 + oh- —> cr + C1O2 is BrO3

- ci2 —> cr ER = —
R 2
Illustration 3 HCHO disproportionates to HCOO”
2 and CH3OH in presence of OH~ (Cannizzaro reaction).
0 -1
2HCHO + OH" ---- > HCOO’ + CH3OH
-Cl2 C1O2
2 Calculate equivalent weight of HCHO.
0 +3
(0.5 mol of Cl2 is used in each half-reaction)
Solution HCHO oxidation> HCOO"
HCHO
?
„ M M 2 KZ O.N. = 0 2
=—+—=-M
(net) 2 6 3 molecular weight
Equivalent weight of HCHO =
change in O.N.
Illustration 2 Equivalent weight ofHr2 is 96 in the
M
following disproportionation reaction:
2
Br2 + OH- ---- > Br" + H2O + ? (oxidised product) reduction,
HCHO
Identify the oxidised product. (Br= 80) ?
o T)
Solution Molar mass of Br2 = 160 g mol'1 2
M
1 Br2 reduction, Br- Equivalent weight of HCHO = —
2 0 -1
In a disproportionation reaction, net equivalent
Equivalent weight of Br2 = f°™ul*weight weight = sum of equivalent weights of oxidation and
= “ = 801 reduction reactions
M M w Oft
= — + — = M = 30
1 2 2
Net equivalent weight (given) = 96

Target Practice 4
1. In the following redox reactions, write values of a to f
I. aMnO; + bFe2+ + cH+ ---- > dMn2* + eFe3+ + fH,0 Equivalent weight of
Compound formed
KMnO4
II. aCr2O2- + bFe2+ + cH+ ---- > dCr3+ + eFe3+ + fH,0
158
III. aMnO; + bH2O2 + cH+ ---- > dMn2+ + eO2 + fH,0
52.66
a b c d e f 31.6
I
3. MnO4 is an oxidant for Fe2>, C2O4" etc. Mention moles of
II
MnO4 required by 1 mol each of
III
MnOj required
2. Molecular weight ofKMnO4 is 158. KMnO4 can be reduced
(a) Ferrous oxalate
to MnSO4, K2MnO4 and MnO2 and equivalent weight of
(b) Ferric sulphate
KMnO4 comes out to be 158, 52.66, 31.6. Match the
equivalent weights with compounds formed by reduction. (c) Ferrous chloride
190 | Essential Inorganic Chemistry

4. Write half reactions using electrons Oxidation..............


(a) Reduction.............
2MnO; +5C2O;‘ + 16H* 10CO2 + 2Mn2* + 8H2O
5. KHCO3 and KHC2O4 can be estimated by base or/ar
Oxidation ....
MnO;. Derive equivalent weight
Reduction ....
(b) Cr2O2- + 6Fe’; * + 14H* 2Cr3* + 6Fe3* + 7H2O Using base Using MnO;

Oxidation .... KHCO3


Reduction .... khc2o4
(c) 3C12 +6NaOH 5NaCl + NaC103 + 3H2O

Answers
(c) FeCl2 Fe2*
1. a b c d e f
5Fe2* = MnO;
I 1 5 8 1 5 4 1 Fe2+ s 0.2 mol MnO<
II 1 6 14 2 6 7
4. (a) Oxidation : Fe 24 ---- > Fe3* + e~
III 2 5 6 2 5 8 Reduction:
MnO; + 8H* + 5e‘ ---- > Mn2+ + 4H2O
2. Equivalent weight Compound formed
(b) Oxidation : Fez+ ---- > Fe3* + e"
158 K2MnO4 Reduction:
52.66 MnO2
Cr2O2-+ 14H* + 6e~ ---- > 2Cr3+ + 7H2O
31.6 MnSO4
(c) Oxidation : - Cl2 + 6OH~ ---- > CIO; + 3H2O + 5e"
3. (a) FeC2O4 Fe2* + C2O2- 2
5Fe2* + MnO; ---- > Mn2* + 5Fe3* Reduction: - Cl2 + e~ ---- > Cl~
2
5CzO$-+2MnO; ---- > 2Mn2* + 10CO2
5FeC2O4 = 3MnO; 5. Compound Using base Using MnO4
(1 Fe C2O4 = 0.6 MnO; KHCO3 E=M cannot be used
MnO 4 = 0.6 mol KHC2O4 E=M
(b) Ferric sulphate Fe2 (SO4 )3 is already has iron in +3 state. 2
Hence, no further oxidation by MnO;
MnO; = Zero

Practice Exercise 2
1. Balance the following redox reactions in acidic solution: 4. Balance the following equation taking place in ba:
(a) VOj- + Fe2* ---- > Fe3* + VO2+ solution: i
(b) BrO3 ---- > Br“ +N2 Fe(OH)2(s) + O2(g)---- > Fe(OH)3(s)
(c) UO2 + Cr2O2’ + H* ---- > UO2* + Cr3* + H2O 5. Write balanced redox reactions of the following:
2. Balance the following redox reaction in basic medium: (a) H2O2 (aq) reduces Cr2O2-(aq) to green colour*
(a) Mn2* +H2O2 ---- > MnO2 +H2O Cr3+ (aq) in acidic medium.
(b) H2O2 (aq) oxidises Mn2* (aq) to MnO2 in basic medium
(b) MnO; + NO2 ---- > MnO2 + NO3
(c) 4 moles of Sn2* (aq) reduces IO4 (aq) to I”(aq).
(c) Mn2* + CIO’ ---- > MnO2 + C1O2
(d) MnO; + NO2 ---- > MnO2 + NO3 6. A given amount of Fe2* is oxidised by x moles of MnO'
acidic medium. Calculate number of moles of CfjC
3. Balance the following redox reactions in acidic or basic required to oxidise same amount inFe2+ in acidic medium
aqueous solution as indicated:
7. C2O2~ can be oxidised to CO2 by MnO; in acidic as well ai
(a) HjS + NO3 ---- > NO2 + S8 (acidic)
(b) MnO; +SO2 ---- > SOj‘ +Mn2* (acidic) basic medium. What is ratio of moles of MnO; used per t
(c) Bi(OH)3 + Sn(OH)3 ---- » Sn(OH)2’ + Bi (basic) Ofc2o;-?
8. How many moles of KI are oxidised by number of mole;
(d) MnO; +1“ -----♦ MnO2 + IO3 (basic)
KIO3 formed when 1 mole ofl2 is boiled with excess of KQ
(e) H2O2 + MnO; -----> O2 + MnO2 (basic)
6KOH + 3Lit —> 5KI + KIO,o + 3HIt2O
(f) Al + NO3 ----- > Al(OH); + NH3 (basic)
(g) Zn + NO3 ----- > Zn2* +N2 (acidic) ki + kio3
•j
I2 +h2o
ft L
Chapter 4: Oxidation-Reduction I 191
9. Calculate number of moles of Mn04 required to oxidise 14. Calculate the equivalent weight ofthe following substances
1 mole ofFe(HC 2O4 )2 in acidic medium. How many moles of as acids or bases:
(a) KH(I03 )2 (b) KHC20.1 ·H 2C204
Ca(OH) 2 are required to react with 1 mole ofFe(HC 2O 4)2 ?
(c) Al(OH)4 (d) SO 2 (OH) 2
10. H2O2 (aq) reduces Cl04 (aq) in acidic medium. If 1 mole of (e) PO(OH) 3 (0 Zn(OH)!-
H 2O2 reduces 0.5 mole of Cl04 (aq), what is the reduced 15. Calculate equivalent weight of KHC 2O 4 (potassium acid
species? oxalate)
11. MnO4 (aq) can oxidise I03 (aq) to I04 (aq) and itself gets (a) when it is neutralised by KOH, and
reduced to MnO 2 (s). Calculate number of moles ofl03 (aq) (b) when it is oxidised by MnO� in basic solution.
that can reduce 0.1 mole of MnO4 (aq). 16. 0.24 g of a metal on heating in air forms 0.40 g of metal
12. Calculate equivalent weight of the underlined : oxide. What is equivalent weight of metal?
17. Equivalent weight of KMn0,1 in acidic medium is 31.6.
(a) Na 2CO3+ H 2SO4 ---> Na 2SO4 + H 2O+ CO 2
What is corresponding value in basic medium?
(b) NaHCO3 + HCI ---> NaCl+ H 2O+ CO2 18. Determine equivalent weight of underlined species taking
its molecular weight as M
(c) 2C6H:PH+ 2Na ---> 2C6H ,.ONa+ H 2 (a) 4H 3P03 ---=-+ 3H 3PO4 +PH 3
CH 2 NH 2 CH 2NH;c1- t!,.
(d) I + 2HCl ---> I (b) 2H
--2O 2 ---=-+ 2Hp+0 2 -
CH 2 NH 2 CH 2 NH;c1- (c) Mn0 2 OH-) Mn04+ Mn +
2

(c) CH-CH+ 2Na ---> NaC �cNa+ H 2 fl 2H +0


(d) 2H 0 ---=-+
2 2 2

13. Mercuric oxide, HgO, can be analysed by reaction with 19. Calculate the grams of one equivalent each of the following
iodide and then titration with an acid: for the indicated reaction:
(a) FeSO4 ( ---> Fe3•) (b) H 2S(---> S 0)
HgO+ 4r ---> Hgl�-+ 2OH­
(c) H 2O 2 (---> 0 2) (d) Hp 2 (---> Hp)
What is its equivalent weight?

Answers
1. (a) Vo!- + Fe 2
++ 6W ----+ Fe 3+
+ vo 2+ + 3H
.,O 7. 5C 2O!-+ 2MnO4 .!!.:+ 10CO 2 + 2Mn 2+
Cb) 2BrO3 + 3N 2H 4 ----+ 2Br-+ 3N 2 + 6H 2O
3C 2O�-+ 2MnO4 � 6CO 2 + 2MnO 2
(c) 3UO 2 + Cr2O�-+ 14W ----+ 3UO�• + 2Cr 3• + 7H 2O
2. (a) Mn 2+ + H 2O 2 + 2OH- ----+ MnO 2 + 2H 2O 1 C2O�- = � MnO4 (acidic)
5
(b) 2MnO4 + 3NO2 + H 2O ----+ MnO 2+ 3NO3 + 2OH-
1 C 2O!- = � MnO4 (basic)
(c) Mn 2+ + 2ClO3 ----+ MnO 2 + 2ClO 2 3
(d) MnO4 + 3NO 2 + 2OH- ----+ MnO 2 + 3NO 3+ H 2O ratio ofmole ofMnO4 used= 3: 5
3. (a) 8H 2S+ 16NO 3 + 16W ----+ 16NO 2 + S 8 + 16H 2 O 8. By given reaction
Cb) 2MnO4 + 5SO 2 + 2H 2O ----+ 2Mn 2• + 5So:- + 4W 31 2 !! K.10 3
(c) 2Bi(OH) 3 + 3Sn(OH)3+ 3OW ----+ 2Bi+ 3Sn(OH)�­ 11 2 =!_ K.10 3 - !_ KI
3 3
(d) 2MnO4 + r + H 2O----+ 2MnO 2 + l0 3 + 2OH- 1
Hence,- mol.
(e) 3H 2O 2+ 2MnO4 ----+ 30 2 + 2MnO 2 + 2OH-+ 2H 2O 3
(0 8Al+ 3NO3 + 5OH-+ 18H 2O ----+ 8Al(OH)4 + 3NH 3 9. Fe(HC 2O 4)2 � Fe 2•+ 2HC2 O4
(g) 5Zn+ 2NO3 + 12W ----+ 5Zn 2• + N 2 + 6H 2O MnO4 oxidises Fe 2+ to Fe J+ and HC 2 O4 to CO 2•
4. 4Fe(OH) 2 (s)+ 2H 2O(l)+ O 2 (g) ----+ 4Fe(OH) 3 (s) MnO4 + 5Fe 2+ ----+ 5Fe3+ + Mn :.i+
5. (a) Cr2O�-+ 3H 2O 2 + SW ----+ 2Cr 3+ + 30 2 + 7H 2O 2MnO4 + 5HC 204 ----+ 10 CO 2 + 2Mn 2•
Cb) Mn 2+ + H 2O 2+ 2OH- ----+ MnO 2+ 2H 2O 5Fe2+ = 1 MnO4
(c) 4Sn 2++ l04+ SH + ----+ 4Sn 4++ r + 4H 2O 1Fe 2+ !_ MnO4
!!
'5
6. 6Fe2++ Cr2O�- __!:!:.+ 2Cr 3+ + H 2O+ 6Fe3+

i
5HC 204 = 2MnO4
5Fe 2+ + MnO4 � Mn 2+ + H 2O+ 5Fe 3+ 2HC 2O4 = MnO4
mole of MnO 4 5 --- x 5
----� - -- - (ratio of Fe 2
mole of Cr2O�- - 6 ' 6
+
mole of Cr 2 O�- - Total MnO4 required by 1 mol ofFe(HC 2 O 4 )2
oxidised ) 1 -=
= -+ 4 1 mol
5 5
192 | Essential Inorganic Chemistry

Fe(HC2O4)2 has two acidic H* 14. If Mis the molecular weight and E the equivalent weight, then
Fe(HC2O4)2 Fe2* + 2HC2o; (a) E = M (b) E = — (0 E =
3 4
I (d) E = — (e) E = — (f) E = —
2 3 4
2H* + 2C2or
2H* + Ca(OH)2 ---- > Ca2* + 2H2O 15. (a) KHC2O4+KOH---- » K2C2O4+H2O
Thus, Ca(OH)2 required by 1 mol of Fe(HC2O4 )2 = 1 mol KHC2O4 s lH*s 1 OH" (H * ion is neutralised)
mol. wt.
10. H2O2+C1O; ---- » O2 + reduced species (Z) Equivalent weight = =128
-2 +7 0 O.N. =x 1
Balancing O.N. (b) KHC2O4 +MnO; MnO22+ 2CO 2
(7 - x) H2O2 + 2C1O; ---- > (7 —x) O2 + 2Z
T T ? T
6 +7 +4 8
(for two C) (for two C)
(7-x) molH2O2 reduces = 2 mol CIO;
2 (C2O$“ ion is oxidised)
1 mol H2O2 reduces =------ mol CIO;
7-x mol wt.
Equivalent weight of KHC2O4 = = 64
2 2
= 0.5 mol CIO; (given)
7-x 16. 12 g/equivalent.
4=7-x 17. 31.6 x - = 52.66 g / equivalent.
x=3 Eq. wt.
18. (a) H3PO3 —> H3PO4, H3PO3=H3PO4=^;
Oxidation number of Cl in Z is = + 3, Z is C1O2.
+3 +5
11. 2MnO; +310; ---- » 310; +2MnO2 M
+7 +5 +7 +4 h3po3 —> ph3, H3PO3=PH3=^
+3 -3 6
2 mol MnO; h 3 mol IO3
In total reaction, equivalent weight of the species undergoing
0.1 mol MnO; s 0.15 mol IO3 disproportionation
12. Equivalent weight of = sum of equivalent weight of half reactions
mol. wt M M
(a) Na2CO3 = = 53 = — + — = -M
2 2 6 3
mol. wt. _ 0
O2-_-M
(b)NaHCO3 = (b) 2H2O2 —> 2H2O + O2) H2O2 = —,
i 22 2

(c) C6H5OH =
mol. wt.
= 94 (c) MnO2 ---- > MnO; + Mn 2* MnO2 = —, MnO; = —,
2 3
1
ch2nh2
mol. wt.
(d) 2H2O---- > 2H2 + O2, O -M °2-T,
h2 = "
2
(d) I = 30
CH2NH2 2 ■ .> Fe
19. (a) Fe2* ---- _ _ 3*, 56 g
+2 +3
mol. wt
(e) CH=CH= = 13 (b) H2S —> S, 17 g
2 —2 0
13. HgO=20H“=2H*
(c) H2O2 ---- > O2, 17 g 1 O.N. of two O -atoms
... -2 Q have been considered
2
Equivalent weight of HgO = mo^ecu^ar we^t (d) H2O2 2H20,17 g O.N. of two O-atoms have
—2 been considered
2
Total Practice Set
(Read, Plan & Solve!)

Problem 1. If 10 g V2O5 is dissolved in acid and (b) Cl2 + 2OH" ---- > Cl" +C1O- + l^O
reduced to V2+ by treatment with zinc metal, how many T t ?
0 -1 +1
moles of I2 could be reduced by the resulting V2+ solution,
as it is oxidised to V4+? (V= 51) Cl2 is reduced to Cl“ as well as oxidised to CIO" hence, CLj
undergoes disproportionation,
V2O5 + 10H+ + 6e‘ ---- > 2V2+ + 51^0
(c) CaCO3 ---- > CaO +CO2
V2+ +12 +H2O---- > 21- + VO2+ +2H+ T T T T
Plan On comparison of stoichiometric coefficients, we +2+4 +2+4
find no change
1 mol V2O5 = 2 mol (d) 2HgO ---- > 2Hg +O2
Solution From the above equations,
T ? T ?
+2-2 0 0
1 mol of V2O5 is reduced to 2 mol of V2+
: There is oxidation of O and reduction of Hg. It is a redox
1 mol of V2+ reduces 1 mol of reaction but not disproportionation.
Thus, 1 mol of V2O5 reduces 2 mol oflj (e) CuaO + 2H+ ---- > Cu +Cu2+ +
hence, 10 g of V2O5 =f=-^ mol | reduces $ ? T
+1 0 +2
x. 182 J
There is oxidation of Cu+ to Cu2+ as well as reduction to
=2x mol 1% Cu, hence, Cu+ disproportionates.
182
= 0.11 mol (f) CuS +O2 ---- > Cu + SO2
T ? ? T ?T
+2 -2 0 0 +4 -2
Problem 2. Which of the following are examples of
disproportionation reactions? This is also a redox but not disproportionation reaction.
(a) Ag(NH3)2 + 2H+ ---- > Ag+ + 2NH4+ (g) 2HCuC12 ---- > Cu +Cu2+ +4C1- + 2H+
T T2 T T T
(b) Cl2 + 2OH" ---- > Cl' + CIO’ + HaO +1 -1 0 +2
(c) CaCO3 —> CaO+ CO2
Cu+ disproportionates to Cu2+ and Cu.
(d) 2HgO---- > 2Hg + O2
(e) 0^0 +2H+ ---- > Cu + Cui2+5 + 0,0 Problem 3. MnO4 can oxidise NO2 to NO3 in basic
(f) CuS + O2 —> Cu + SO2 medium. How many moles ofNO2 are oxidised by 1 mole of
MnO4"?
(g) 2HCuC12---- > Cu + Cu2+ + 4G!" + 2H+
Plan On comparison of change of oxidation number,
Plan A disproportionation reaction is that in which the
same species is oxidised as well as reduced. 2 mol ofMnO4 = 3 mol of NO2
Cu+ Oxidation^ Cu2+ Solution NO2 is oxidised to NO3 by MnO4 (in basic
Reduction } qu medium) which is reduced to MnO2
Cu+
MnO4 +NO£ ---- > NO3 +MnO2
+7+3 +5+4
Solution (a) Ag(NH3)2+ + 2H+ —> Ag+ +NHJ | Oxidation | a
T T t t Reduction
+1-3 +1-3
no change, neither oxidation nor reduction
194 | Essential Inorganic Chemistry

Thus, MnO4 ---- > MnO2 oxidation number decreases Thus, 6FeC2O4 b SC^O?'
by 3-units
1 mol FeC,O4 = 0.5 mol of Cr2O?"
NO2 - NO3 oxidation number increases by 2-units
Problem 6. The following is a completely balana
Thus, 2MnO; = 3NO2 equation:
Q
lMnO4“B-NO2 3Sn + 12HC1 + 4HNO3 ---- > 3SnCl4 + 4N0 + 8H2'O
2 In this reaction, what fraction of the formu
= 1.5 mol NO2 weights are the equivalent weights of Sn and HN C
respectively?
Problem 4. In certain laboratory procedures it is formula weight
Plan Equivalent weight =
necessary to remove NH4 ions from solution by heating change in oxidation number
with HNO3. Write balanced ionic reaction if NH4 ion is formula weight
oxidised to N2 by HNO3, which in turn is reduced to NO2. or
number of electrons exchange
Calculate moles ofHNO3 required by 1 mole of(NH4)2SO4.
6NO3 +2NH.; + 4H+ ---- > N2 +6NO2 +6H2O Solution The half-reactions are
oxidation : Sn + 4C1" ---- > SnCl4 + 4 e~
Plan From stoichiometric coefficients :
reduction : NO3 + 4H* + 3e" —> NO + 21^0
6 mol HNO3(NO3)b 2 mol NH4+
Equivalent weight of Sn =——
= 1 mol (NH4)2SO4
Thus, in the oxidation, there are 4 equivalents S
Solution In the product, we find there are two types of per mol Sn.
nitrogen (as N2 and as N02). To know which nitrogen is Equivalent weight of HN03 = ^ormu^
from which reactant, isotopes of nitrogen are used. If N of
NH4 is labelled by its certain isotope, we find that nitrogen Thus, in the reduction, there are 3 equivalentsHNI
certainly appears in N2 per mol HN03.
(NH4)2S04 b 2NH4 = 6NO3
Problem 7. Aqueous solution of sodium hypochlori
hence, one mole of (NH4)2SO4 is oxidised by six moles (NaOCl) is a household bleach and is a strong oxidi so
ofHNCk. 0
agent that reacts with chromite ion [Cr(OH)4 ] in ba«
solution to yield chromate (CrO^“) and chloride ic
• Determine
Problem 5. Calculate number of moles of (a) MnO4
(a) number of equivalents of each reactant
and (blCioO^- to oxidise 1 mole ofFeCjC^ (ferrous oxalate)
in acidic medium. (b) number of moles of NaOCl required by 1 mole of
[Cr(OH)4].
Plan Fe2+ and C2C^“ (in FeC2O4) both are oxidised by Plan (a) Number of equivalents = number <
MnO^/FT as well as by Cr2O}"/H+. Thus, on comparison of moles x change in oxidation number
stoichiometric coefficients, number of moles of C^O^- and (b) If change in oxidation number of each reactant
that of MnO4 required by 1 mole of FeC2O4 can be known, stoichiometric coefficients are also known. Hen:
calculated. number of moles of NaOCl can be determined.
Solution
Solution Fe2+ is oxidised to Fe3+ and C^O^ is oxidised
C10"(ag) +Cr(OH)4 (aq) CrO^ (aq) +Cr(aqj
to C02 both by Mn04 and Ci^O^" in acidic medium. ? ? T ?
+1 +3 +6 -1
(a) 5Fe2+ + Mn04" + 8H+ ---- > 5Fe3+ +Mn2+ +4H20 3 units f n
2 units
5C2O^" + 2Mn04' + 16H+---- > 10CO2 + 81^0 + Mn2+
Thus, 5FeC204 b 3Mn0; Thus,
1 mol FeC,04 b 0.6 mol MnO4 3C10“(047) + 2Cr(OH)4(aq)---- > 2CrO^(a(?) + 3CHc
- 6Fe3+ + 2Cr3+ + 71^0 (a) Equivalents of CIO" (aq) used = 3 x 2 = 6
(b) 6Fe2+ + C^O2/ + 14H+-
* 12CO2 + 4Cr3+ + 141^0 Equivalents of Cr(OH)4(aq) used = 2x3=6
6q,C^“ + 2Ci2O27' + 28H+
Chapter 4: Oxidation-Reduction | 195

(b) 2 mol [Cr(OH)7] = 3 mol C1O" millimol of alcohol hence, mass per cent of alcohol level in
Q
1 mol [Cr(OH)7 ] = - = 1.5 mol C1O" blood can be determined.
2
1 Note: Remember, in a redox reaction, number of equivalents of Solution 2 millimol of solution
each reactant is always same. = 1 millimol ofGjHgOH
'8.76x 0.05 millimol of Ci^O^- solution
Problem 8. Alcohol levels in the blood can be
determined by a redox titration with dichromate solution =- x 8.76 x 0.05 millimol of CjHgOH
in acidic medium according to the balanced equation. 2
G2H6OH(aq) + 2Ci2O27"(aq) + 16H+(a^) ---- > = 0.219 millimol
2CO2(£) + 4Cr3+(aq) + HI^CKZ) = 0.219 x IO"3 mol
= 0.219 x IO-3 x 46 g CgHgOH
What is the blood alcohol level in mass per cent if 8.76 mL
( of 0.05 M Ci^O2/ solution is required for titration of a 10.00 = 0.01gC2H5OH
g sample of blood? Thus, 10 g blood sample has= 0.01 g CjHgOH
Plan Based on stoichiometric coefficients, number of hence, alcohol level (mass per cent)= x 100
millimol (= molarity x mL) ofCx$i~ are to be related to
= 0.10%

I
Master Exercises
■Kjl> JiZrtfiDSJ? " *b» '£.:JTJ5. C _,. 2 . •L”Z^232~' "~Z.~3

Exercise 1
(Stage 1: Learning)
Short Answer Type Questions 2. Arrange the following sulphur containing anions u
1. When Cl- is converted into Cl2, what is change in order of increasing oxidation state of sulphur:
oxidation number? SO2,-; S2O5-; S2O2,-; HSO;; HS-; S.OJ'
2. What is oxidation number of oxygen in superoxide ion 3. Nitrogen forms five different compounds will
QJ? oxygen. Write appropriate formulae for thes
compounds if the oxidation state of N in them is +1
3. Equivalent weight of MnO4 is equal to molecular +2, +3, +4 and +5.
weight, when it changes into new species X. What is
X? It is by oxidation or reduction. 4. A compound is made of Ba (oxidation number = ♦ $
4. What is equivalent weight of NaHC2O4 when Xe(oxidation number = 8) and oxygen. Xe and 0 ar
present in the atomic ratio of 1:6, what is th
(a) it behaves as an acid?
compound?
(b) it behaves as a reducing agent?
5. Oxidation number of sulphur in H2SO5 is +6. Explain 5. KMnO4 oxidises Xn+ ion into XO3, itself changing t
by structure. Mn2+in acid solution. 2.68 x 10~3 mol of Xn+ require
1.61 x 10~3 mol of MnO4". What is atomic weight of
6. Oxidation number of sulphur in H2S2O8 is +6. if the weight of 1 g equivalent of XCln is 56?
Explain by structure.
7. Derive equivalent weight of Redox Reactions
G) HgPO, (ii) H3PO3
(iii) H3PO4 6. Balance the following redox reactions. Also underlin
disproportionation reactions:
at every stage of neutralisation by NaOH.
(a) MnO4 +1" + H+ ---- > Mn2+ + + H2O
8. When CO2 is passed into aqueous K2CrO4 solution
(yellow), it changes to orange coloured solution.
(b) ci2 + r + h2o —> 1O3 + cr + h+
Explain by reaction. (c) BrOg +N2H4 ---- > Br" +N2 + H2O
9. When SO2 gas is passed into aqueous K2Cr2O7 (d) VO43- + Fe2+ + H+ ---- > VO2* + Fe3+ + H20
solution (orange), it changes to green coloured (e) UO2* +Cr2O27- +H+ ---- > UO1* + Cr3+ +H2O
solution. Explain by reaction. (0 Pb(NO3)2 ---- > PbO + NO2 + O2
10. Fe2+ is oxidised to Fe3+ by MnO4 or Cr2OV in acidic (g) IO3 + HSO; + H* ---- > I2 + SO?" + H2O
medium. Explain by reaction with change in colour. (h) S2O?- + H2O---- > S2O23- + HSO3
(i) Cl2 + OH’ —> CIO" + Cl" + H2O
Analytical Questions
(j) CeHgCHO + OH"---- > CeHsCOO" + CgHgCHj-O;
Oxidation Number
(k) SzOi" + H+ ---- > H2O + SO2 + S
1. The ion An+ is oxidised to AO3 by MnO4, changing to
(l) Cl" + NO3 + H* ---- > NO2 + Cl2 + H2O
Mn2+ in acidic solution. Given that 2.68 x IO’3 mol of
An+ requires 1.61 x IO'3 mol of MnO4. What is the (m) CIO” + H2O---- > C1O2 + Cl2 + OH"
value of n? (n) Cl2 + S2" ---- > S + Cl"
Chapter 4: Oxidation-Reduction | 197

(o) 03 + Pb2+ + h2O---- > PbO2 + H+ + 02 Equivalent Weight


(p) H02 + Br" + H20---- > OH" + Br2 13. Determine the equivalent weight of the underlined by
(hydrogen taking its molecular weight as M:
peroxide ion)
(a) 2KMnO4 + 5H2SO4 + 10FeSO4 ■> 5Fe2(SO4)3
(q) Au + CN" + 02 + H20---- > [Au(CN)2]" + OH" + K2SO4 + 2MnSO4 + 5H2O
7. Identify the oxidising agent and the reducing agent in (b) K2Cr2O7 + 7H2SO4 + 6FeSO4 ---- » K2SO4
the following redox reactions: + Ci2(SO4 )3 + 3Fe2(SO4)3 + 7H2O
(a) 5C1O3 + 3As2S3 + 9H2O 501" + 6H+ (c) CH3CHO + 2[Ag(NH3)2 ]* + 30H" ---- >
+ 6H2AsO4 + 9S 1 ’ CH3COO" + 2Ag + 4NH3 + 2H2O
(b) 7CN" + 20H" + 2Cu(NH3)4+ ---- > 2Cu(CN)l" (d) 2KC1O3 ---- > 2KC1 + 3O2
+ 8NH3 + CNO" + H20 (e) 3C12 + 6NaOH---- > 5NaCl +NaC103 + 3H2O
8. H202 can be used as oxidising agent or reducing agent (f) NH4NO3 ---- > N20 + 3H2O
depending on the medium. Explain, why this (g) PC15 ---- > PC13 + Cl2
behaviour is possible? Give one example of each. (h) 2CuSO4 + 4KI---- > Cu2I2 +2K2SO4 + L>
9. Balance the following redox reactions: (i) S2O?3~ + 5H2O + 4C12 ---- > 2S0J" + 8C1" + 10H+
(a) Zn + N03 + OH" ---- > NH3 + ZnO^" (j) 2HS~ +4HSO3 ---- > SSsjOj" + 3H2O
(b) HgS + Cl" + N03 + H+ ---- > HgCl2" + NO + S (k) 4Fe2+ +2NH2OH---- > 4Fe3+ + N20 + 4H+
(l) IO3 + 3HSO3 ---- > I" + 3H+ + 3SO5"
(c) Mn04 + H+ + H2O2 ---- > Mn2+ + 02
(m) 3MnO2 + 6K0H + KC103---- » 3K2MnO4
(d) P2H4 ---- > PH3 + P4H2
+ KC1 + 3H2O
(e) C102 +01" ---- > C102 + 02
(n) H&04 + Mn02 + 2H+---- > Mn2+ + 2H2O + 2002
(f) Cl2 +103 + OH" ---- > I04" + Cl-
14. Photosynthesis of a carbohydrate in plants is given by
10. Calculate the number of moles of Mn04 that will be chemical equation
needed to react with one mole of SO5" (sulphite ion) in 6CO2 + 12H2O ■> C6H12O6 + 6H2O + 602
acidic solution. Calculate:
11. H2O2 acts both as an oxidising agent and as a (a) equivalents of 02 formed per mol of C0.2
reducing agent in alkaline solution towards first row consumed.
transition metal ions. Illustrate both these properties (b) equivalents of C6H12O6 per mol ofH2O used.
(c) equivalents ofCO2 used per mol ofH20 used.
of H2O2 using chemical equations.
(d) If you have net result as:
12. Give an example of oxidation of one halide by another 6CO2 + 6H2O---- > C6Hl2O6 + 6O2
halogen. Explain the feasibility of the reaction.
It is correct or incorrect? Explain.

Exercise 2
(Stage 2: High Skill Problem Solving)
Only One Option Correct (c) H3AsO3 ---- > HAsOj"
1. A sulphur-containing species that can’t be a reducing (d) A1(OH)3---- > Al(OH);
agent is 3. In balancing the half-reaction
(a) BQ, (b) SO,- CN" ---- > CNO" (skeletal)
(c) H2SO4 (d) S2-
the number of electrons that must be added is
2. Which of the following changes requires a reducing (a) 0
agent? (b) 1 on the right
(a) CrOt —> C^O?" (c) 1 on the left
(b) BrO3 ---- > BrO" (d) 2 on the right
198 | Essential Inorganic Chemistry

4. For the reaction between MnO; and in basic 14. In the reaction, 4 moles of electrons are transferred
solution, the unbalanced equation is 1 mole ofHNO3.The possible product obtained duett
MnO4- + C2O^~ ---- > MnQ2(s) + CO?- reduction
(a) 0.4 mol of N2
In a balanced equation, the number of OH" ions is
(a) 0 (b) 4 on the right (b) 0.5 mol of N2O
(c) 4 on the left (d) 2 on the left (c) 1 mol of NOu
5. Which of the following is disproportionation reaction? (d) 1 mol of NH3
(a) CaCO3 + 2H+ ---- > Ca2+ + H2O + CO2 15. 1 mole of N2Hj loses 10 mol of electrons to form a new
(b) 2CrOl- + 2H+ ---- 4> Cr2O27~ + H2O compound K Assuming that all the nitrogen appeal
(c) CraO?- + 2OH- ---- > 2CrOj- + H2O in the new compound, what is the oxidation state ol
nitrogen in Y? (no change in the oxidation state of Hl
(d) Cu20 + 2H+ ---- > Cu + Cu2+ + H2O (a) -1 (b) -3
6. When MnO4 and T react in a strongly basic solution, (c) +3 (d) +5
the products will most likely be 16. Oxidation number of P in Ba(H2PO2)2 is
(a) Mnandl2 (b) MnO?j-,IO3 (a) +3 (b) +2
(c) MnO.2,Q2,IO" (d) Mn2+,I2 (c) +1 (d) -1
7. Of the following acids, the one that is strongest is 17. Which combination appears odd w.r.t. oxidatior
(a) HBrO4 (b) HOC1 number per atom of the underlined?
(c) HNOa (d) H3PO3
(a) H2SO5!H2S2O8)K2Ci2O7
8. When PbS(s) is treated with warm dilute HNO3, the (b) CrO5,CrOi-,SOt
products most likely will be (c) Both (a) and (b)
(a) Pb2+, S.NOa (b) Pb2+,S,NO
(c) PbO, S, NO (d) PbOa,SO$-,N2 (d) None of the above

9. In an experiment 20 g of vanadium (V) oxide (molar 18. In[Cr(O2)(NH3)4(H2O)]Cl2 oxidation number.of Cr if


mass = 182) was reduced by excess of zinc dust in +3, then O2will be in the form
acidic solution to vanadium (II) ions. The required (a) dioxo (b) peroxo
number of moles of iodine to reoxidise vanadium (II) (c) superoxo (d) oxo
to VO2* is
19. Strongest conjugate base is
(a) 0.22 (b) 0.11
(c) 0.30 (d) 0.23 (a) CIO- (b) C1O2
(c) C1O3 (d) C1O4-
10. For a titration of 100 cm3 of 0.1 M Sn2+ 2+ to Sn4+,
50 cm3 of 0.40 M Ce4* solution was required. The 20. Which is not a disproportionation reaction?
oxidation state of cerium in the reduction product is Al(OEt)3
(a) +1 (b) +2 (a) 2 CHO
(c) +3 (d) 0
11. In the following reaction (unbalanced), equivalent
weight of As2S3 is related to molecular weight M by COOCH
As2S3 + H+ + NO3 --- > NO + H2O + AsOf- + SO$"
CHO CHoOH COO"
(a) — (b) —
2 4 (b) | + OH" > |I +COO-
l
COOH COO"
(c) — (d) —
28 24 (c) NaH + H2O---- NaOH + H2
12. 1 mole of FeC2O4 is oxidised by x mole of Cr2O^- in (d) All of the above
acidic medium, x is 21, Oxidation number of Cr in the following compound
(a) 3 (b) 1.5 are x, y, z and w.
(c) 0.5 (d) 1.0 K(CrO(O2)(OH)], K3[Cr(O2)4] (NH3)3[Ci<02)2],
X y
y Zz
13. 1 mole of MnO4 will oxidise x mole of ferric oxalate in CrOA
acidic medium, x is w
(
(a)6
5
(b) I5 These values are
(a) 6,13, 5, 6 (b) 4, 4, 4, 6
(c) 5 (d) 6 (c) 4, 5, 4, 6 (d) 4, 5, 6, 6
Chapter 4 Oxidation-Reduction | 199

22. If a small amount of KMnO4 is added to cone. H2SO4, a OH


green coloured solution with oxidation number of Mn
as +7 is formed. Probable compound is
(a) MnO3HSO4
(c) MnO2HSO4
(b) K2MnO4
(d) MnOa
23. Equivalent weight ofMn3+ in the following reaction is
(Mn=55)
Mn 3+ ---- > Mn2+ +MnO2
2AgBr(s) + 2OH (aq) +

0 OH 0

(a) 27.5
(c) 110
(b) 55
(d) 165
24. Which of the following is the example of a
2Ag(s) + 2H2O(Z) + 2Br"(aq) + |
0 0
disproportionation reaction?
Select correct statement.
(a) CaCO3 ---- > CaO + CO2 (a) Hydroquinone is the oxidant
(b) 2HCuC12 ---- > Cu + Cu2+ + 4Cr + 2H+ (b) Ag+ is the oxidant
(c) PC15 ---- > PC13+C12 (c) Br“ is the oxidant
(d) 2H2S + SO2 ---- > 3S + 2H2O (d) Ag+ is the reductant
25. Out of the following redox reactions 30. In the following redox reaction
I: NH4NO3 N2O + 2H2O Cu(OH)2(s)+ N2H4(aq)---- > Cu(s)+ N2(g)
number of moles of Cu(OH)2 reduced by one mole of
II: NH4NO2 N2 + 2H2O
N2H4 is
A
PC15 -A» PC13 + Cl2
III: (a) 1 (b) 2
(c) 3 (d) 4
disproportionation is not shown in
(a) I, II (b) H, III 31. In the following half-reaction in basic medium
(c) I, III (d) I, II and III Bi3+(aq) ---- > BiO3(aq)
26. Bleaching powder (CaOCl2) has two types of chlorine one mole of Bi3+ is equivalent to
atoms with oxidation number of +1 and -1. It can be (a) 1 (b) 2
represented as: (c) 3 (d) 4
(a) Ca2+(C102)Cr 32. The complex [Fe^O^NC)!2* is formed in the
(b) Ca2+(C103)Cr ring-test for nitrate when freshly prepared FeSO4
solution is added to aqueous solution ofNO3 followed
(c) Ca2+(C10+)Cr by addition of cone. H2SO4 .This complex is formed by
(d) Ca2+(C10-)Cr charge transfer in which
27. In the following redox reaction (a) Fe2* changes toFe3+ andNO* changes to NO
Zn(s) +NO3(aq)+ H+(aq) Zn2+(aq) + NHj(aq) (b) Fe2+ changes to Fe3+ and NO changes toNO+
(c) Fe2+ changes toFe+ and NO changes to NO"
Zn(s) and NO3(aq) respectively are: (d) no charge transfer takes place
(a) oxidant, reductant
(b) reductant, oxidant 33. When KCN is added to CuSO4 solution there is
formation of the stable water soluble complex. This
(c) both oxidant
complex is
(d) both reductant (a) Kt [Cu(CN)6 ] (b) K3[Cu(CN)4 1
28. Out of the following select species with different (c) K2[Cu(CN)4] (d) K[Cu(CN)3]
oxidation states of the underlined atoms 34. In cyanide method, silver metal is obtained as
I: CaOClj; II:: H2S
II 2O8;
S2O III. Na2S4O6
2K[Ag(CN)2] + Zn---- > K2[Zn(CN)4 ] + 2Ag
IV: CiC5; V: N2H4O3 In this
(a) All except I (a) Ag has been oxidised and Zn has been reduced
(b) All except II (b) Ag+ has been reduced and Zn has been oxidised
(c) All except III (c) both the metals have been oxidised
(d) All except IV (d) both the metals have been reduced.
29. Photographic paper is developed with alkaline 35. In the following unbalanced redox reaction
hydroquinone H2S(g) + SO^g)---- > S(s) + H2O
200 | Essential Inorganic Chemistry

total number of equivalent of SC^ is 45. In the following redox reactions, NH3 appears either
(a) 4 (b) 3 in reactant or product. In which case equivalent
(0 2 (d) 1 weight of NH3 is maximum?
36. In the above reaction of question (35), one equivalent (a) N2 + 3H2 ---- > 2NH3
of H2S(g) will reduce (b) 4NH3 + 5O2 ---- > 4NO + 6H2O
(a) 1 mol ofS02 (b) 0.5 mol ofSQj (c) 2NH3 + 2Na---- > 4NaNH2 + H2
(c) 0.25 mol ofSO2 (d) 2 mol ofSQj (d) Equal in the above all cases
37. Which has maximum number of equivalent per mol of 46. In the following unbalanced redox reaction,
the oxidant?
CuaP + CrgO?’ ---- > Cu2+ + H3PO4 + Cr3+
(a) Zn(s) + VO2+(aq)---- > Zn2+(ag) + V3+(ag)
(b) Ag(s) + NO3(aq)---- > Ag+(ag) + NO2(g) Equivalent weight of H3PO4 is
(c) Mg(s) + VO3"(ag)---- > Mg2+(aq) + V2+(aq) (a) — (b) —
3 6
(d) I”(aq) + ICaCaq)---- > 13(09) , x M ... M
(c) — (d) —
38. Which has maximum oxidation number of the 7 8
underlined atom in the following? 47. In the following reaction
(a) MnOt (b) CiO5
Zn + 2OH" + 2H2O---- > Zn(OH):•I" + H2
(c) CrO2Cl2 (d) Equal
Species which has been reduced is
39. Which has least number of equivalent per mol of the (a) Zn (b) OH"
reactant? (c) H2O (d) none of these
(a) MnO^ changes to MnO2
48. H2S2O8 and H2SO5 both have +6 oxidation state of
(b) MnO4 changes to Mn2+ sulphur. It is due to
(c) MnO^ changes to MnO^" (a) presence of peroxy group
(d) MnO4 changes to Mn2O3 (b) presence of superoxo group
40. Oxidation number of Cr is +5 in (c) presence of neutral O2
(d) presence of ozone
(a) K3CrO8 (b) (NH3)3CrO4
(c) K2CrO4 (d) [Cr(NH3)5(H2O)]Cl3 49. H2O2 oxidises MnO2 to MnO4 in basic medium. H2O2
41. Sulphur atoms are of two environments in 8406”, and MnO2 react in the molar ratio of
difference in oxidation states of two types of (a) 1:1 (b) 2:1
sulphur is (c) 2:3 (d)3:2
(a) 5 (b) 6 50. Equivalent weight of HgPO2 in a reaction is found to
(0 7 (d) 10 be half of its molecular weight. It can be due to its
42. When Br2 is passed into hot aqueous solution of (a) reaction of its two H+ ions
Na2CO3, (b) oxidation to H3PO3
(c) both (a) and (b)
(a) Br" is formed by reduction (d) None of the above
(b) BrO3 is formed by oxidation
51. Oxidation number of Cl atoms in CaOCl2 are
(c) COja is evolved
(d) all of the above are correct Cl
43. Consider following unbalanced reactions Ca
I: Zn + dil. H2SO4 ---- > ZnSO4 + H2 OC1
II: Zn + cone. H2SO4 ---- > ZnSO4 + SO2 + H2O (a) zero on each
III: Zn + HNO3 ---- > Zn(NO3)2 + NH4NO3 + H2O (b) -1 on Cl and +1 on Cl
Oxidation number of hydrogen changes in (c) +lonCl* and-Ion Cl**
(a) I, II, III (b) I, II (d) +1 on each
(c) II, III (d) I 52. Prussian blue has two types of iron with oxidation
44. NH3 is oxidised to NO by O2 (air) in basic medium. number as shown
in 11
Number of equivalent of NH3 oxidised by. 1 mole of Fe[Fe(CN)6]
(^ is What is the net charge on Prussian blue?
(a) 4 (b) 5 (a) -1 (b) +1
(c) 6 (d) 7 (c) 0 (d) -2
Chapter 4: Oxidation-Reduction I 201
63. Consid er following redox reactions CHO coo- CH2OH
I: (�)2 Cr2O7 --+ N2 + Cr2O3 + H2O (c) I +OH- � I +I
COOH coo- coo-
II : Nll.N03 � N20 + 2H2O
IlI: 2KC1O3 � 2KCI + 30i (Intramolecular Cannizzaro reaction)
OH-
Select intramolecular redox reactions (d) Mn()_z MnO4 + Mn2• (Redox reaction)
(a) I, II (b) II, III
(c) I, III 59. If CGi is passed into waste water containing Crai­
( d) I, II, III
54. 0.05 mole of Ca(OH)2 can neutralise H2SO4 . This (yellow) then
+
(a) solution turn green due to formation ofCr 3
H2S04 can be neutralised by
(b) solution turns blue due to formation ofCrO5
(a) 0.05 mol ofNaOH (c) solution turns orange due to formation ofCr2 �­
(b) 0.10 mol ofNaOH (d) no effect takes place
(c) 0.05 mol of POH(OH)2 60. In the following reaction
(d) None of the above is correct
v01- +MnO4 � Mn2+ + VOt°
66. NaHCi04 is neutralised by NaOH and can also be
1 mole of v�- is oxidised by MnOi
oxidise d by KMnO4 (in acidic medium). Equivalent (a) 0.2 mol (b) 0.4 mol
weight is related to molecular weight (M) of
(c) 0.8 mol (d) 1.0 mol
NaH2Ci04 in these reactions as
(a) M, M (b)
!,! One or More Than One Options Correct
( c) � , M (d) M, � I. One mole of ferric oxalate is titrated with acidified
potassium permanganate solution. In this
56. If OX 1+ae- � Redi , E1 ° (a) 3 mol of potassium permanganate are required
OX2+be- � Red2, �
0 (b) 1.2 mol ofpotassium permanganate are required.
Then potential at the equivalence point is (c) oxalate ion is oxidised to carbon dioxide.
= aE1 o + b& o aE1 o - b& o (d) ferric ion is oxidised to ferrate (VI) ion
(a) E (b) E =
(a+ b) (a+ b) 2. Consider following reaction :

(c) E =
bEi, o - aE1 o
(d) E
= aE1 o - b& o CHO coo-
(a+ b) (a - b) l +OH- --+ I
CHO CH2OH
5 7. In the following redox reaction,
(a) It is a disproportionation reaction
5Fe2 + + MnO4 + SH+ � Mn2 + +5Fe3+ +4H2 O
(b) It is intramolecular redox reaction
Given: Fe 3+ + e- --+ Fe2 +, E1 ° (c) OH- is a reducing as well as oxidising agent
MnO4 + SH + 5e- --+ Mn + + 4H2O,
+ 2
Ez 0 CHO
Potential at the equivalence point is (d) I is a reducing as well as oxidising agent
(a) E = (E1 °+ Ei, 0) - 0.08pH CHO
{b) E = E1 o + 5 Ez o 3. Out ofFeo�-, FeC1 3, FeCI2 and Fe,
- 0.08pH
6 (a) best reducing agent is Fe
(c) E = (E1 ° - Ez O) - 0.08pH
0 o
(b) best oxidising agent is Fe�­
(d) E = E1 _5Ez +0.08pH (c) best oxidising agent is FeCia
6
(d) best reducing agent is Fe�-
58. Which reaction is suitably matched?
CHO CH2OH 4. 0.1 mol ofNaH�O4 is
(a I
) +oH- --+ I (a) neutralised by O.l mol ofNaOH
CHO coo-
(b) neutralised by 0.05 mol ofCa(O!Di
(crossed Cannizzaro reaction) (c) oxidised by 0.04 mol ofKMnO4 is acidic medium
(b) 3HaP� � 2HaPO 3 + PHa (d) oxidised by 0.02 mol ofK2 MnO4 is basic medium
· (disproportionation reaction)
202 I Essentia/ Inorganic Chemistry

5. Select correct statements. Passage Comprehension Questions


Equiva]ent weight ofKMn04 (molar mass= ) is
M Passage 1
(a) M in acidic medium
5 Consider following Redox Predominance Diagram
for iron in different oxidation state
(b) M in neutral basic medium
3 Oxidation
Half-reaction E 0/V
state
(c) Min strongly alkaline medium
M +6 Feo�- + 8W + 3e- ¢ Fe3+ +2.20 V Feoi-
(d) in acidic, basic and neutral media 2.20
2 + 4H20
+3 Fe3 • + e- ---+ Fe2 • +0.77 V Fe3+
6. Intramolecular redox reactions are 0.77
(a) PC15 -4 PCI 3 + Cl2 +2 -0.44 V Fe2 •
Fe2• + 2e- ¢ Fe
(b) 2KCI0 3 --4 2KCI + 3� -0.44

coo- Fe
CHO
(c) I: + oH- .� I Answers the following questions.
CHO CH20H
1. Select best oxidising and reducing agent
(d) NH.iN� � N2 +2H2O (a) Fe, Fe
2+
(b) Fe�-, Fe
3+

7. Out ofH2 S04 , H28 20s, H2S05 , H2S203 and H2 S201 (c) Fe�-, Fe2+ (d) Fe�-, Fe
.. (I) (II) (III). (IV) . (V)
2. Most thermodynamically stable species is
(a) oxidation number of sulphur is +6 in all except IV 3+
(a) Fe�- (b) Fe
. (b). peroxy li�age is �n I, II _and III 2+
(c) Fe (d) Fe
(c) pe�oxy linkage is in II and III
(d) oxidation number of sulphur is -2 in I and V 3. Least thermodynamically species is
3+
(a) Fe�- (b) Fe
8. Select correct statement(s). 2
(c) Fe + (d) Fe
(a)..Equivalent
. ' weigh� ofCa(HCi04 )2 is M when it is a
. . 2
reducing agent Passage 2
. (b) Equiva]ent weight of Ca(H½04 )2 is M when it Every disproportionation reaction is a redox reaction
but every redox reaction may not be disproportionation.
behaves as an acid Disproportionation reaction may be intramolecular ifsame
+
. (c) Ca(HCi04 )2 can be estimated by Mn04 /H species is oxidised and reduced.
(d) Ca(HCi04 )2 can be estimated by an acid Based on the above statement answer the following
questions.
9. G_onsider following reaction
1. A : 2C6�CHO +KOH � C6 l¼COOK
Br2 �OH_. :· ��?i +Br- +H20 (unbalanced)
..

+C6HsCH20H
Select correct statement.
(a) Equivalent weight ofBr2 when it is reduced to Br­
B : 3Cl2 +6KOH � 5KCI +KC103 +3H20
is 80 Select correct statement.
(b) Equtva]ent weight of Br2 when it is. oxidised to
(a) Both are redox reactions
(b) Both are disproportionation reaction
Br03 is 96 (c) A is a disproportionation and B is a redox reaction
(c) Net equivalent weight ofBr2 is 96 (d) A is a redox reaction, B is a disproportionation
(d) It is a disproportionation reaction reaction
10. F�llowing is/are disproportionation reaction(s) 2. Following reaction is used to produce Br2
(a) 2CCI 3CHO +NaOH � CC1 3COONa 5Br- +Br03 +6H+ � 3Br2 +3H20
In this reaction.
(b) N�N03 -� N2 0 +2H20 (a) Br- disproportionates to Br2
(b) Br03 disproportionates to Br2
(c) 2KC103 � 2KCl +302 (c) Br- has been oxidised andBr03 has been reduced
(d) 5Cl2 + 60H- � ClOa +5Cl- +3H20 (d) Br- has been reduced andBr03 has been oxidised.
Chapter 4 : Oxidation-Reduction I 203
Passage 3 4. The alcohol content in a 10.0 g sample of blood from a
Following passage is for the drunken drivers on roads driver required 4.23 mL of 0.07654 M �C12O1 for
whfch are becoming fatal by their wrong driving. titration. Should the police prosecute the i�dividual
for drunken driving.
Every year in our country about 25,000 people are
(a) yes
killed and 500,000 more are injured as a result of drunk
(b) no
driVing. In spite of efforts to educate the public about the (c) nothing can be predicted
dangers of driving while intoxicated and stiffer penalties (d) None of the above
for drunk driving offenses, law enforcement agencies still
have to devote a great deal of work to remove drunk drivers
from roads.
Assertion & Reason
Codes:
The police often use a device called a breath analyser
(a) Both A and R are true and R is the correct
to test drivers suspected to being drunk. The chemical explanation of A
basis of this device is a redox reaction. A sample of the (b) Both A and R are ,rue but R is not the correct
driver's breath is drawn into the breath analyser, where it
explanation of A
is treated with an acidic solution of potassium dichromate.
(c) A is true but R is false
3CH3CH2OH+ 2K2C12O7 + 8H2 SO4 ----+ 3CHaCOOH (d) A is false but R is true
+ 2Cr2 (8O4 )s + 2K2SO, + llH2O 1. Assertion (A) : �NO3 u.n�ergoes a redox
The driver's blood alcohol level can be determined reaction to N2O and "10.
readily by measuring the degree of this colour change (read Reason (R) : It is a disproportio�ation reaction.
from a calibrated meter on the instrument). The current
legal limit of blood alcohol content is 0.1 per cent by mass. 2. Assertion (A) : Oxidation num�r of sulphur in
Anything higher constitutes intoxication.
Reason (R) : It has one peroxy linkage.
Read the passage and answer the questions given
below: 3. Assertion (A) : Oxidation number of chromium in
c� is+a.
1. Above test is based on
(a) smell of ethanol and ethanoic acid Reason (R) : Oxidation number of each oxygen
(b) change of colour yellow to blue atom is -1.5.
(c) change of colour orange to green 4. Assertion (A) : B� is oxidised · to BrO; in basic
(d) change of colour orange to colourless medium.
2. Equivalent mass of K2Cr2 O7 is Reason (R): When-B� is passed into hot NaOH, it
(a) molar mass molar mass
(b) disproportionates to Br- and BrOi.
6 3
(c) molar mass molar mass 5. Assertion (A) : 1 mole of FeG,iO4 is oxidised by
(d)
2 1 0.6 mole of MnO, in acidic medium.
3. Equivalent mass ofCH3CH2OH is
Reason (R) : MnO4 oxidises both Fe2+ as well as
(a) molar mass
ei�-.
molar mass
(b)
True & False
(c) molar mass
1. Lewis structures should have adja�t formal charges
of zero or of opposite sign. ,,
(d) molar mass
2. Oxidation number of sulphur in �S(\ is +6.
204 | Essentia/ Inorganic Chemistry

3. Oxidation number of sulphur in H2S2O8 is +7. Matrix-Match Type Questions


4. Following reaction is disproportionation reaction, I. Only One Correct
NIH +NO3 ---- > N2O + 2H2O 1. Match the different compounds of Mn (in List I) with
oxidation number (in List II).
5. SOI" is named is sulphate (IV).
List I List II
6. HCHO disproportionates in presence of OH“.
A. MnO3HSO4 1. +4
7. Oxidation number of iron in brown complex (obtained
B. K2MnO4 2. +7
in ring-test ofN03) is +1.
C. MnO2 3. +6
[Fe(H2O)eNOf*
D. Mn2O3 4. +2
8. Equivalent weight of MnO4 when it changes to MnO2 E. Mn(HCO3)2 5. +3
in concentrated basic medium is one-fifth of molar
mass. 2. Match the half-reactions (in List I) with change in
9. Oxidation number of chromium in CrO2+ is +6. oxidation number (in List II).
10. Oxidation number of each chlorine in CaOCl2 is -1. List I List II

A. cr —> cio; 1. 2
B. Cr3+ —> CrO5 2. 8
Fill in the Blanks C. H2O2 -----> O2 3. 0
1. Oxidation number of Mn in MnOjz, MnO^" and MnO4 D. CrO2+ —> CrO4‘ 4. 3
are ,.............and respectively.
2. Oxidation number of Mn in MnO3Cl is II. One or More Correct
3. Br2 disproportionates to and in 1. Match the compounds in Column I with the matter in
coldNa2CO3 solution. Column II.
4. Equivalent weight of Cl2 in the following reaction is Column I Column II

A. H2S2O8 1. Peroxy linkage


ci2 —> cio3 + cr B. Cr O5 2. Oxidation number of the
underlined atom 6
5. Oxidation number of Fe in [Fe(H2O)5NO] is +1 due to
charge transfer c. h2so5 3. H2O2 is the hydrolysis product
D. K2CrO4 4. Caro’s acid
6. CrO5 has oxidation number of Cr as +6 hence, it has
IV
E. (NH3)3CrO4 5. Marshall's acid

2. Match the compounds/Ions having underlined atoms


7. HO OOH of different oxidation states in Column I with values
in Column II.
lNO2 ‘NH2
Column I Column II
This reaction is redox-reaction.
A. CaOCI2 1. 6
8. 1 mole of C^O?" oxidises moles of FeC2O4
B. Na2S2O3 2. 3
in acidic medium.
c. N2H4O'2 3. 2
9. In copper extraction, final stage reaction is
D. 4.
(Prussian blue)
CHO COO' E. CrO5 5. 1
OH^
10. I I 6. -2
CHO ch2oh
7. -3
is a type of reaction.
Chapter 4 : Oxidation-Reduction I 205
rnteger Answer Types 2. In the following reaction
This section contains 8 questions. The answer to each 2NH 3 ---+ N2 +2NH3
of the questions is a single digit integer, ranging from Oto . molecular weight
equivalent weight ofNH 3 1s = ------
9. The appropriate bubbles below the respectively question
numbers in the ORS have to be darkened. For example, if What is value ofy?
the correct answers to question number X, Y, Zand W (say)
are 6, 0, 9 and 2, respectively, then 3. In a Cannizzaro reaction ofHCHO with NaOH
2HCHO + NaOH ---+ HCOONa + CH 3OH
X Y Z W
@ @ @ @ Molecular weight ofHCHO = z x equivalent weight.
CD G) CD CD What is value ofz?
® ® ® ® 4. Oxidation number of Cr in (NH 3)3Cr(�)2 is ......
@ @ @ @
@ @ @ @ 5. Number of peroxy linkage in K 3Cr(0i).x: having
@ @ @ @ oxidation number ofCr as +5 is .........
@ ® @ ® 6. Difference in oxidation states oftwo types ofsulphur
(J) (J) (J) (J) ins.�- is ........ .
@ @ @ @
® ® ® ® 7. Based on redox predominance diagram, out ofFe�-,
Fe3+, Fe2+ and Fe, best oxidising agent is the species
with oxidation number ofiron as .........
1. Cr2�- changes to Cr3+ in acidic medium. Thus,
8. Maximum oxidation state ofchlorine is ........ .
molecular weight ofCr2 �- = x x equivalent weight.
What is the value ofx?
Answers & Solutions
Master Exercises
Exercise 1
Short Answer Type Questions H3PO3 HPO:o + 2H+ E-™
E~~2
1. cr —> -ci2
2 2 (hi) HjjPO^+H* E=M
-1 0 tribasic acid
Change in oxidation number = 1 unit HPO2- + 2H+
2. 02, If oxidation number of oxygen = x 2
HsPO4 POj- + 3H+ e-m
then2x = -l x=-~
2
E~T
8. CO2 + H2O H2CO3 H+ +HC0I
3. Mn04 (oxidation number of Mn = + 7) ft £ ft O Q

CO, makes the solution acidic, which converts CrO?“ into


It is maximum value of oxidation number. In the product
formed, oxidation number should be less than +7.
Cry­
Since, equivalent weight = molecular weight, hence 2CrO42' +2H+ Cr2O?-+H2O
yellow orange
oxidation number of Mn = + 6 in new species formed by
reduction. 9. Cr,O?~ + SO, ----- > SO2" + Cr3+
orange green
This is Mn04‘
4. NaHC2O4 Na*+HC204 H+ +C2O2’ oxidation
ft ft *t ft *1

reduction
(a) It has one ionisable IF thus equivalent weight
= molecular weight 10. 5Fe2 + Mn04 + 8H+ ----- > Mn2+ + 5Fe3+ +4H2O
pinkish colourless
(b) C2O4~ is a reducing agent, itself oxidised to C02.
violet
c2o42- ----- > 2CO22 + 2e" 6Fe2+ + Cr2O2" + 14H+ ----- > 6Fe 3+ + 2Cr3+ + 7H2O
T ?
+g (for two C-atoms) orange green *
+6
Change in oxidation number = 2 units
_ . , . , molecular weight
Analytical Questions
Thus, equivalent weight =----------- - ----------- 1. An+ —> -AO;o change
O.N.-> +n +5 (5-n)
5. It has one peroxy linkage in which oxidation number of
each oxygen is -1 MnO; ----- > Mn2+
O.N. -> +7 +2 5
H^Og +H2O ----- > BLjSO,, +H2O2
O<--2 Balancing (O.N.)
I (5 - n)MnO4 + 5 A'n + ____ 5AO3 + (5 - n) Mn2+
HO—S—O—O—H moles of MnO4 1.61 x 10~3
I -2
Thus,
moles of An+ 2.68 x 10"3
0
? 5 - n 1.61 x IO"3
-2
~5~~ 2.68 x 10"3
6. H^Og + 2H2O —> 2H2SO4 +H20.2
(5-n) = 3
It has also one peroxy linkage n =2
00 2. HS-<S2O2-<SO2"<S4O2-<S2O2- =HSO7
I I
HO—S—0—0—S—OH Oxidation state is the oxidation number per atom
I I S2Og_ has one peroxy linkage.
0 0
3. N2O(+1), NO(+2), HNO2(+3), NO2(+4), HNO3(+5)
7. (i) H3PO2 H+ + H^Oj no further ionisation
monobasic 4. Ba Xe 0
acid n : 1 : 6
Thus, equivalent weight = M 2n + 8 -12 = 0
(ii) H3PO3 H^Og+H* E=M n=2
_ 3
dibasic BaZ2XeOD6
Chapter 4: Oxidation-Reduction | 207

HgOjj as reducing agent


5. Asin 11.
n =2 MnO2 + H2O2 ---- > Mn 2+ + O2 + 2OH"
X2+ XO3 change 12. 21’ + Cl2 ---- > 2CP + I2
+2 +5 3 oxidising power is in order:
M I2 < Br2 < Cl2 < F2 based on electrochemical series.
Equivalent mass of XC12 = — = 56 .. .. ... _ m
13. (a) E = — (b) E = — (c) E = M (d) E = —
M =56x3 =168 5 6 6
(e)E = ^ (f)E = ^ (g)E = ^ (h)E = M
XC12=M +35.5x2 = 168
5 4 2
M =97 M .. _ M
(1) E = — (j) E(HS ) = — (k) E = —
6. Refer Text (h), (1), (j), (k), (m) are disproportionation 4 2
reaction. (l) E(HSO;) = —, E(IO.) = —, E(HSOp = y
7. Oxidising agent (a) C1O3 (b) Cu(NH3 )2+ 3 2 6
(m) E = M M E = ^- M
Reducing agent (a) Ag2S’33 (b) CN“ 2
8. 2H2O2 ---- > 2H2O + O2 14. (a) 6CO2 = 6O2
1 mol CO2 = 1 mol 0, s 32 g O2 = 4 equivalents
Since, H2O2 disproportionates to H2O and O2.
(b) 12 mol H2O used = 1 mol C6H12O6
9. (a) 4 Zn + NO3 + 7OH" ---- > NH3 + 4ZnO2- + 2H2O
= 24 equivalents C6H12O6
(b) 3HgS + 12C1- + 2NO3 + 8H+ 3HgCl2’ + 2NO 1 mol H2O used = 2 equivalents C6H12O6
+ 3S + 4H2O 6CO2Z ---- > COfiH..,O
1Z o6

O.N = 24 =0
(c) 2MnO; + 6H+ + 5H2O2 ---- > 2Mn 2+ + 5O2 + 8H2O
change = 24 units
(d) 5?^4 ---- > 6PH3 + P4H2
5P2H „ . , . . ,. 6 mol mol. wt.
Equivalent weight of CO2 = - • =
(e) 2C1O2 + O2- ---- > 2C1O2 + O2
(0 Cl24 ++ IO
Cl, 3 ++ 2OH-
io; 2OH" ----- > IO; + 2Cr + H2O 1 mol CO2 = 4 equivalents CO2
Equivalent weight of C6H12O6 = m°^’ Wt‘
10. 0.4 mol (2MnO4 + 5SO3 ---- > 5S0j" + 2Mn2+)
24
11. 2H2O2 —> 2H2O + O2 .-. 1 mol C6H12O6 = 24 equivalents of CgH12O6
Since, H2O2 disproportionates to H2O (by reduction) and (c) 2 equivalents of CO.,
02 (by oxidation) hence acts both as an oxidising agent (d) No,O2 (product) is from H2O(reactant), hence 12H2O
and as a reducing agent. Examples are is taken as reactant. It is confirmed by taking
as oxidising agent isotopic O18 when all O18 appear in O2.
6CO,4 + 12H2O 18 ---- > CO6H.,O6 + 6H,0 + 6O14,8
2MnO2 + 3H2O2 + 2OH~ 2MnO4 + 4H2O 4 14 O 4

Exercise 2
3. CN“ -> CNO“
Only One Option Correct
1. If the species is a reducing agent, it means it can be O.N. of nitrogen 4 + x = -1 4 + x - 2 = -1
oxidised easily thus it should have an O.N.
<- — less than x = -5 x = -3
maximum values of O.N. CN“ is thus oxidised. Thus, two electrons are lost.
O.N. Thus, (d)
so2 4 4. (c) 5. (d) 6. (b)
SO2- 4 7. Strongest acid is that which has maximum value of O.N.
SO,,
L1
SO2o ’ and S2" can be reducing agent of the central atom
SO2“ 6
HBrO4 O.N. ofBr = +7
S2- -2
HOC1 O.N. ofCl = + l
SO2- can’t be.
but HNO2 O.N. ofN = + 3
H3PO3 O.N. ofP = + 3
Thus, (c)
Thus, (a)
2. (a,d) no change
8. (b)
(b) reduction, (c) oxidation
Thus, (b)
208 | Essential Inorganic Chemistry

5
9. V2OS ----- > V2* ----- > VO2+ 1 mol of Mn04 oxidises = - mol of C2O4
2
V2O5 ----- > 2V2+ 5
= - mol of Fe2(C2O4)3
20 g
0.11 mol 0.22 mol Thus, (a)
V2+ *2
VO2++ 2r 14. HNOa+4e" Product has O.N. = 1
+2 0 +4 -2
T
+5
Thus, V2O5 = 2V 2+ =2VO2+ = 2I2
0.22 mol In N20oxidation number ofN = + l.Thus, 0.5 mol ofN20
0.11 mol
are formed by reduction of 1 mol of HN03 by 4 mol of
Thus, (a) electrons thus, (b)
10. Sn2+ ----- > Sn4+ + 2e~ increase in O.N. = 2 15. N2H4 ----- > N2(—) + 10e~
Ce4+ + ne" Ce(4-n) decrease in O.N. = n T ?
To balance O.N. two N-atoms of = -4 O.N. of two N-atoms in oxidised species
nSn2+ + 2Ce4+ nSn4+ + 2Ce(4-n> = — 4 + 10 = + 6
Millimoles ofSn2+ = 100 x 0.1 = 10 Thus, oxidation state of N in new compound = +3.
Thus, (c)
Millimoles of Ce4+ = 50 x 0.4 = 20
16. (c)
Thus,
71 10
— =--- 0
2 20 ?
17. HgSOg (peroxy sulphuric acid) H— 0—0 —S—0—H
n=1 peroxy X
0
Thus, oxidation state of cerium in the reduced product linkage
= (Ce4-1 =Ce3+) = + 3 HjSjjOg (peroxy disulphuric acid)
Thus, (c) 0 v 0
T I . ?
11.
O.N.
As2S3 ----- * AsO4"+3SO:
2As = + 6 2x5 3x6
•t X X
H— 0—S—0—0—S—0—H

|3S = -6| = 18 | 0 0
I = 10
All of the above have O.N. = 6 in sulphur, peroxy linkage
Net = 0 Total = 28 Net change = 28 (—0—0—) has -1 O.N. for oxygen, CrO5 has two peroxy
_. _ M linkage, thus O.N. of Cr = + 6
Thus, equivalent mass of A 383 = —
- " 28 0
Thus, (c) 0
12. FeC2O4 Fe2+ + C2O2"
0
Fe2+ is oxidised to Fe3+ and C2O2- to CO2 by Cr2O2-
K2Cr2O7 and CrO4‘ both has +6 oxidation number for Cr.
Fe2+ ----- > Fe3++e“]x6
Thus, (d)
Cr2O2- + 14H+ + 6e“ ----- > 2Cr3+ + 7H'22O
18. [Cr(O2)(NH3)4(H2O)]Cl.2
Thus, 6Fe2+ = 1 Cr2O2- X‘ ‘ X ■
+3 +x +0
+0 -2=0
1 Fe2* a 1 Cr2O2“
x=-l
Thus, (02) exists as 02 (superoxide ion)
C2°4" ----- > 2CO2x2e-lx3
Thus, (c)
Cr2O?" + 14H+ + 6e“ ----- > 2Cr3+ + 7H2O
19. If acid is weak, its conjugate base is strong. Greater the
3C2O2" ----- > Cr2O2- O.N. of Cl, stronger the acid and thus weaker the
ic2oj- —> IcijO2- conjugate base
hcio hcio2 hcio3 HC1O4
Thus, total Cr2O|" required =1+1=1 =0.5 mol O.N. of Cl +1 +3 +5 +7
6 3 2 acid strength --------------------------------------------------------------- ►

Thus, (c) conjugate base CIO- C1O2 ClOj cio;


13. MnO4 will only oxidise oxalate (C2O4-), ferric is already base strength --------------
in oxidised state Thus, (a)
2MnO;+5C2O; ----- > 10CO2+2MnO4 20. NaOH + H2O ----- > NaOH + H2
Fe2(C2O4)a 2Fe3++3C2O4" T ? T T
-1 +1 +1 +0
2 mol of MnO4 oxidises = 5 mol of C2O4~ H“ changes to H2 — oxidation
Chapter 4 : Oxidation-Reduction I 209
Ir changes to H2 - reduction (II) NH: + NC>; --+ N 2
Thus, different H are involved. i t i
-3 +3 0
Thus, it is a redox but not disproportionation.
Same as (I)
Thus, (e)
21. K[CrO](O2 )(OH)l (III) PCl5 --+ PCI3 + Cl2
it it i
iii i i +5-1 +3 -1 0
+l + .:c - 2- 2- 1 = 0
p +5 is reduced to p 3+
X =4
c1- is oxidised to Cl2
K 3[Cr(O2)4]
Thus, different species are involved. Thus, it is also not a
i i i disproportionation reaction.
3 + y - 8 = 0, y =5
Thus, (d)
(NH3)3[Cr(02>2l
i 26. Ca 2+cc10-)(Ci-)
i i
0 +z -4 =0 thus, z =4 Thus, (d)
CrO2Cl2 27. Zn � Zn2+
Zn is oxidised hence, it is a reducing agent (reductant}

Thus, (e)
w -4 -2 = 0, w = 6 NO3
+5
--+ NH:
-3
22. (a) MnOa!ISO4 NCJ; is reduced hence, it is oxidising agent (oxidant}
i i Thus, (b)
x-6-1=0 ci-�-1
x=+7 • 28. I: Ca <2+

Thus, (a) oc1- �+1


23. Mn + Mn 3+ ___. Mn2+ + MnO2
3+
II :+6 both and S

1 I
+3 +3 +2 +4
1 -::___,
-- : I
III : 0 and 5 IV -2, 1
V: NH; NG;
Mn disproportionates to Mn2+ and Mn02
3+ i i
-3 +5
2 units of Mn3+ Thus, (b)
Thus, equivalent mass of Mn3+ = -----
2 29. Ag + is reduced hence, it is oxidant. Hydroquinone is
=-M =55 oxidised hence, it is reductant.
Thus, (b) Thus, (b)
-2 -2 30. 2Cu2 + + N2 H4 --+ Cu+ N2 +4H+
! -2 t i i
24. (a) Ca CO3 ---4 Ca O + CO2
+2 -4 0 0
ii i i
+2+4 +2 +4 :. Thus 1 mol NJ-14 !!! 2 mol Cu 2+
No change, Thus, (b)
(b) 2HCuCl2 ___. Cu+ Cu2+ + 4Cr + 2H+ 31. Bi3+ --+ BiO3
+3 +5
it t i i i i
+l+l -1 0 +2 -1 +1 Change in oxidation number = 2 units
Thus, Cu+ disproportionates to Cu and Cu 2+ . Thus, number of equivalents in one mole Bi3+ = 2
Thus, (b) Thus, (b)
25. m NH: + No; ---4 N20 + 2H2O 32. Fe 2+ + e- � Fe +
t i t . NO --+ NO• + e-
-3 +5 +1
! reduction f Based on magnetic properties of the complex, it is found
that iron has three unpaired electrons thus it is as Fe +
oxidation
formed by reduction ofFe 2+ by NO which is oxidised to
It is a redox but not disproportionation. Since, different NO•.
N-species are involved. Thus, (c)
88. (b) 34. (b}
210 | Essential Inorganic Chemistry

35. 2H2S(g) + SO2(g) ---- > 3S(s) + 2Ho0(7) 41. (a)


?
-2
T+4 J0 42. (d) Na2CO3+H2O ---- > 2NaOH + H2CO.3
Change in oxidation number = 4 units H42CO,0 ---- > H42O + CO42
Thus, number of equivalent ofSO2 =4 6NaOH + Br2 ---- > 5NaBr + NaBrO3 + 3H2O
by reduction by oxidation
Thus, (a)
36. H,S changes to S Thus, (d)

Thus, change in oxidation number = 2 units 43. I: Zn + 2H+ h2 changes


+1 0
Thus, one equivalents H2S = 2
2H._>S + SO2 3S +2H,0
3 mol
II: Zn + H+ + SO;
T
•r ---- > Zn2+ + SO2 + H2O no
?
2 mol 1 mol +1 +1
4 equivalent 1 mol change
1 equivalent ofH^ will reduce 0.25 mol of SO2 III: Zn + H+ + NO: ---- > Zn2+ + NH4 + H,0
-2^
Thus, (c) T ? ?
+1 +1 +1
Change in Number of no change
37. Reaction
O.N. equivalent
Thus, in (I) only
(a) VO2’(aq) V3 (aq)
T Thus, (d)
? ?
x-2 = +2 +3 1 unit 1 44. 4NH,■3 + 50,2 ---- > 4NO + 6H,0
■2
x = +4
T T
-3 +2
(b) NO3 (aq) NO2(g)
4
T T 1 unit 1
1 mol 092 oxidises = ’ mol of NH3
+5 +4 5
4
= - x 5 equivalent of NH3 = 4
(c) VO3’(aq) —» V2’(aq) 5
T T Thus, (a)
+5 +2 3 unit 3
45. (a) 3H24 =2NHO3
(d) IO3 (aq) <3 (aq)
NH,
T T 1H = _____ o
+5 1 16 16 3
3 3 „ M
3 Equivalent weight of NH3 = —
3
Thus, (d) (b) 4NH3 = 5O2
38. (a) MnO2’ x = 6 = 10(0) atoms taking weight as 16
= 20(0) atoms taking weight as 8
(b) CiOr
— x =6
Thus, 1(0)s-NH3
(c) CrOpClo x = 6 5
M
Thus, (d) Equivalent weight of NH3 = —
5
39. Change in O.N. Equivalent per mol
(c) 2NH3=H2
ofMnO4
IHhINH,
(a) M_nO4 MnO, 3 3
Equivalent weight of NH3 = M
+4 "
(b) Mn2+ 5 5 Thus, (c)
+2 46. Cu3P ---- > H3PO4
(c) MnO2“ 1 1 T T
+6 4 -3 +5
(d) - Mn2O3 4 4 Change in oxidation number = 8 units of phosphorus
2 +3 M
Equivalent weight of H3PO4 = —
Thus, (c)
40. K3CrO8 has [Cr(O2)4]3' with four peroxy linkage Thus, (d)
[Cr(O2)4l3- 47. OH’ ---- > H2
? ?
x-8 = -3 -2+l 0
x =+5 Thus, OH" is reduced to H2
Thus, (a) Thus, (b)
Chapter 4: Oxidation-Reduction | 211

O O H2SO4 + 2NaOH Na2SO4 +2H2O


T T —OH
43. H&Og HO—S-4Q>-S 0.05 00.10 mol
10 mo1
! r J,
O I O
peroxy linkage
t Thus, (b)
55. NaHC2O4 + NaOH ---- > Na2C2O4+H2O
O
?
I NaHC2O4 = HC2O; sH+ = OH~
ILSOjj HO—3-
S i=6\H Thus, E(NaHC2O4) = M
i
O HC2O; =C2O3~ =2CO2
Thus, (a) ? T
+6 +8 ] for two C-atoms
Thus, E(NaHC204) = ^
49. H2O2 + MnO2 ---- > MnO4+2H2O Z 4 2
+4
-2+4 +7
4-7 _4
Thus, (d)
3e~
56. At the equivalent point, all species are in unit
+2e~ concentration and
(ox. no. of two O atoms taken) AG°(net) = AG° + AG2°
Balancing of O.N.
Total electrons exchanged = (a + b)
2O2+2MnO2 ---- > MnO;+6H2O
3H■2^2 AG°=-nFE°
Thus, H2O2 : MnO2 = 3 :2 -(a + b)FE° = -aFE° - bFE°
Thus, (d) EO_aE^ + bE°2
50. (a) H^O^HPO" + 2H+ 3 a+b
ButHgPOg is a monobasic acid, hence, false Thus, (a)
(b) H3PO2 —> H3PO3 57. AG°(net) = AG1°+- &G°
O.N. +1 +3 -6FE°=-FE° -5E°
O 1 il

M
Equivalent weight = —
z 3 6
Cl Except H+, all other species =1M
51. Ca< 8
OC1 Cl is -1 ...
OC1 is -1 E° + 5E° 0.0591
“6 6
x81og(iF,
Thus, OC1
?T _ E° + 5E°
-2 +x = -1 -0.08 pH
~ 6
.’. x = + l
Thus, (b)
Thus, (b)
m n 58. (a) Intramolecular Cannizzaro
52. Fe[Fe(CN)6] (b) Disproportionation thus, true
i +3+2-6 = -! Thus, (b)
Thus, (a) O T— HjCOg
59. CO24-HL22W 2H+
53. Intramolecular redox reactions are in which different 2CrO2- ■+2H* H,0 + Cr2O?"
atoms are oxidised and reduced in same substance. yellow orange
Substance Oxidised Reduced Thus, (c)
I. (NH4)2Cr2O7 NH4 toN2 Cr2O?‘ toCr3+ 60. VOo2’ + MnO44 Mn2+ + vo;
T ? T ?
II. NH4NO3 nh; toN2o NO3 to N2O 44 +7 +2 +5
III. KCIO3 0" toO2 cioji+s) to cr 5 unit t
1 unit
Thus, (d)
Thus, 5VO2- + MnO4 ---- > Mn2+ + 5VO;
54. Ca(0H)2 + HjSO, ---- > CaSO44-2H2O
0.05 mol 5 mol VO^- s 1 mol MnO4
Thus, 1 mol Ca(OH)2 = 1 mol H2SO4 1 mol VO2“ 2 0.2 mol MnO4
hence, 0.05 mol Ca(OH)2 = 0.05 mol H2SO4 Thus, (a)
212 | Essentia/ Inorganic Chemistry

One or More Than One Options Correct 6. (a) PC15 PC13+C12


?? T T
1. Fe2(C2O4)3 2Fe3++3C2O2~ +5 -1 +3 0
1 mol 3 mol
Fe3+ is already in the oxidised state (b) 2KC1O3 —> KC1 + O2
C2O2" is oxidised to CO2 by MnO; in acidic medium. T T ?? T
+5 -2 -1 0
2MnO4+5C2O4“ ---- > 10CO2+2Mn2+
5 mol C2O4" = 2 mol MnO; CHO COO-
oxidation
3 mol C9O4" = - *— = 1.2 mol (0 I
2 4 5 CHO reduction CH2OH
Thus, (b) is correct (d) NH4NO2 N2
C2O4" is oxidised to CO2 ? T 0
Thus, (c) is correct -3 +4
Thus, (b) and (c) Thus, (a),(b),(c) and (d)
2. One CHO is oxidised to COO"
Oxidation Peroxy
and one CHO is reduced to CH2OH 7. Acid Structure number of linkage
Thus, it is not a disproportionation reaction. It is sulphur
CHO I. H2SO4 0 +6 No
intramolecular redox reaction. Thus, (b) is true and I sulphuric II
CHO acid HO—S—OH
is reducing as well as oxidising agent. II
S
Thus, (d) is also true.
II. H2S2O8 O 0 +6 Yes
Thus, (b) and (d) peroxydi­ II II
3. Refer redox predominance diagram (3.1). Species which sulphuric HO—S—O—O—S-OH
has least oxidation number is oxidised and thus is the acid II II
s s
best reducing agent. Thus, (a) is true
III. H2SO5 O +6 Yes
Species which has maximum oxidation number is
reduced and thus is the best oxidising agent. Thus, (b) is
peroxy­ II
sulphuric HO-S-O-O—H
also true. acid II
S
Thus, (a) and (b)
4. NaHC2O4 Na+ + H+ + c2ot IV. H2S2O3
thiosulph­ II
0 +2 No
0.1 0.1 o.i HO-S-OH 0
? T uric acid
neutralised oxidised by II
by OH" MnO4 s
1 V. H2S2O7 SO3 dissolved in H2SO4 +6 No
1H+ = 1 OH’ = NaOH = - Ca(OH)2
0.1 mol 0.1 mol 0.1 mol 2 0.05 mol (oleum)

Thus, (a) is true (a) is true


Thus, (b) is also true. (b) is not true
5C,O?-+2MnO: ---- > 2Mn2+ + 10CO2 (c) is true
2^44
Z 4
5C2O2- = 2MnO4 (d) is not true
0.1 mol C2O4” = 0.04 mol MnO4 Thus, (a) and (c)
Thus, (c) is also true 8. C2O2’ ---- > 2CO2
Thus, (a), (b) and (c) are true +6 +4
Equivalent weight Equivalent weight =
M
5. (a) MnO; Mn2+ in acidic medium = — Thus, (a) is true
+7 +2 5
Ca(HC2O4)2 2HC2o;
M
(b) MnO; MnO, in basic neutral medium = — hc2o; H+ +C20j-
+7 +4 2 3
Equivalent weight = M
M Thus, (b) is true
(c) MnO; MnO2" in strongly alkaline medium = —
+7 +6 1 C2O4" can be estimated by MnO4/H+
Thus, (a), (b) and (c) are true. Thus, (c) is also true
Chapter 4: Oxidation-Reduction | 213

Ca(HC2O4)2 cannot be estimated by acid 4. (a) 4.23 x 0.07654 millimol K2Cr2O7


Thus, (d) is not true = 4.23 x 0.07654 x 6 milliequivalent of K2Cr2O7
Thus, (a), (b) and (c) are true. = 4.23 x 0.07654 x 6 milliequivalent of CH3CH2OH
.4.23 x 0.07654 x 6 x 23
9. Br2 + OH’ ---- > BrO3 + Br’ + H2O 1000 3
(a) ^Br2 —» Br’ a 0.045 g in 10 g sample
2o -1 0.045
Thus, % of alcohol content = xlOO
M 10
Equivalent weight = — = 80 = 0.45 >0.10
Thus, (a) is true. Thus, (a)
(b) ifir2 —» BrO,
0 *5 Assertion & Reason
1. (c) 2. (a) 3. (c) 4. (d) 5. (a)
Equivalent weight = =16

Thus, (b) is not true True & False


(c) Net equivalent weight = Equivalent (in oxidation 2. T 3. F 4. F 5. T 6. T 7. T
1. T
part + in reduction part) 9. T 10. F
8. F
= 16+80=96
Thus, (c) is true Fill in the Blanks
(d) Br2 disproportionates to Br" (reduction) and BrO3 3. Br’.BrO" 4. 42.6
1. +4, +6, +7 2. +7
(oxidation)
5. from NO to Fe2* 6. two peroxy linkages
Thus, (d) is also true.
7. intramolecular 8. 2 9. disproportionation
Thus, (a), (c) and (d) true.
10. intramolecular disproportionation.
10. (a) CC13CHO ---- > CC13COO’ + CC13CH2OH
• by oxidation by reduction
Thus, (a) is true
Matrix-Match Type Questions
I. Only One Correct
(b) nh; + no;; —> N2O + 2H2O
-3 +5 +1 1. (A)-(2); (B) — (3); (C)-(l);
(E) —(4)
I Loxidation
reduction t (D)-(5);
2. (A)-(2); (B) —(4); (C)-(l); (D) — (3);

II. One or More Correct


Different ions are involved, hence (b) is not true (C) —(1,2,3);
1. (A)-(1,2,3,5); (B) —(1,2);
(c) C1O3 ---- > Cl’+O.2 (D) —(2); (E)—(1)
tt t ? 2. (A)-(4,5); (B) —(1,6); (O- (2,7);
+5 -2 -1 0
(D) — (2,3); (E)—(4,6)
(d) Different atoms are involved, hence (c) is not true (d)
is true (refer question 9(d)).
Thus, (a) and (d) are true
Integer Answer Types
Questions 1 _3_ _4 £ 7_
Passage Comprehension Questions Answers 6 3 I 1 4 5 6
Passage 1 © © © ® © ©I© ©
1. (d) 2. (a) 3. (d) O © ® © O O
Passage 2 © © ® ©) © © © ©
1. (b) 2. (c) ® © © © _©_© ©
Passage 3 ® ® ® © ®
@ (4)
® ® ® ® © ®
(5) (5)
1. (c) 2. (a) © ® ®
® ® ® ® ©
3. (b) CH,CH2OH —> CHoCOOH
0 4 O ©
-2 0 ®_®_ ®
® © ® ® ©
Change in O.N. = 2 units
® © ® ® © ®| (9) ®
M
Thus, £=—
2
"If the radiance of a thousand suns
were to be burst into the sky, that
would be like the splendour of the
mighty one - yet when the sinister
and gigantic clouds rose up ........ .
I am become death, the shatters of
world ...... "
A passge from Bhagvad Gita was

Radioactivity
quoted by Robert Jungk in his
book while describing the first
testing of Atom Bomb in Japan.

Quick Points
• Elements in the Periodic Table, have been placed in the
increasing order of atomic number.

• In the element ix, • Radioactivity


A= atomic mass = proton + neutron = p + n ■ Units of Radioactivity
P = proton = atomic number = electrons in neutral atom ■ Theory of Radioactive Disintegration
• E = mc2 by Einstein equation, where m is the mass and c the ■ Nuclear Stability
velocity of light. • Nuclear Reactions
• By first-order kinetics, • Nuclear Fission
dN
rate -- ='AN • Nuclear Fusion
' dt
• Artificial Radioactivity
where A.= disintegration constant
(a1so called rate constant) in �ime-1 unit
■ Binding Energy and Packing Fraction
A. _ 2.303 log �
■ Applications of Radioactive Isotopes
--- 10
t N • Radiations and Matter : Detection and
where N0 is the number of atoms disintegrating at start and Biological Effects
N is the number of atoms disintegrating at time t.
• When 50% disintegr ation has taken place

N = No , t = t50 called half-life period


2
2.303 log 2 0.693
tso = A.
=
A.
Thus, t50 is independent of initial number of atoms.
216 | Essential Inorganic Chemistry

1 y Table 5.1 Types of Radioactive Decay


• Also, N = No
2 c Nuclear change
total time o Usual
where y = Types of .2 Equivalent
nuclear
^50 decay process Atomic Mass
condition
number number

Alpha £He -2 -4 Z > 83


Radioactivity emission (a)
Beta 10n-> }p + + 1 0 "too
Radioactivity was discovered by Henri Becquerel in emission (p) z
1896, while investigating any possible relation between large
fluorescence and X-rays. A crystal of potassium uranyl Positron ?e 10n+ ?e -1 0 "too
emission®*) z
sulphate was used as a source of fluorescence material. On small
exposure to sunlight, it emitted radiations which could Electron X }p + °e-» 'on -1 0 "too
penetrate paper, glass and aluminium sheets and ionise capture* rays Z
(E.C.) small
gases. These types of radiations were also observed in
Gamma 0 0 Excited
darkness. Nature and origin of these radiations were emission (y) nucleus
investigated by Becquerel, Rutherford, Pierre Curie and
Marie Curie. Uranium changed to thorium during the (N = neutrons, Z = atomic number)
*Also called K capture-an electron from the K'-shell (n = 1) is
emission of these radiations. captured by the nucleus
Radioactivity is the phenomenon of spontaneous • P-rays are the same as cathode-rays but p-electrons
disintegration of certain nuclei to form new elements with travel with much higher speeds than the cathode-ray
electrons. Also the velocity of cathode-ray electrons is
the emission of “active radiation”, they are called constant while the P-electrons have a wide velocity
active as they affect photographic plates and interact range.
with electric and magnetic fields. Active radiations are • Emission of a- and p-radiations do not cause health
summarised in Table 5.1. hazard to a greater extent due to their low penetrating
power (see Table 5.2).
238it ____ .
a-particles: 92 u ------- * 29304Th +^He(a) • y-rays are extremely penetrating and represent a
234 mt ____ . greater health hazard to those who work with
P-particles: 90 lh------ > 2394iPa+_ie° (P) radioactive chemicals:
y-rays: 230 rpr * ____
9q in > 239°0Th + y Characteristics of a-, P- and y-radiations are
summarised in Table 5.2

Table 5.2 Properties of Alpha, Beta and Gamma Rays


Speed
Type Alternate name Charge Mass Penetrating power Stopped by

Alpha (a) Helium nucleus +2 4mH 2 x 107 Low Air or paper


(2 protons + 2 neutrons) (deflection towards -ve | — mm of Al |
plate in electric field) lioo J
Beta (P) Electron 2x10® Medium Thin aluminium
(deflection towards +ve | — mm of Al |
plate in electric field) Uo J
Gamma (y) High energy electro­ 0 0 3x10® High Thick lead
magnetic radiation (no effect in electric or 25 cm of steel/ 5 cm of
magnetic field) lead
Positron Positive electron +1 me Medium Thin aluminium

Illustration 1 Write balanced nuclear equations for Solution The key to writing nuclear equations is to
each of the following-. make sure that the number of nucleons is the same on both
sides of the equation and that the number of elementary
(a) a-emission from curium-242
and nuclear charges is the same.
(b) ^-emission from magnesium-28
(c) positron emission from xenon-118 (a) ’gCm —> jHe +2”Pu (b) ^Mg
(c) ‘“Xe—■> .Je+’s’l
Chapter 5: Radioactivity I 217
Table 5.3 Distinction between a Radioactive Change and a number increases by one unit. The nutrinos are energy
Chemical Change particles.
4 4
;� Pb ---. �; Bi + -�e
Radioactive change Chemical change
( 1) New elements are produced TVA VA
( 1) One compound can be
1.16 converted to other
68 Ra--+ �Rn+ ;He Illustration 1 Elements with atomic number 89 to
CaCO 3 � CaO + CO 2 103 all have been placed in group IIIB (actinides). What
(2) It is irreversible (2) It can be reversible or are the groups of the elements formed in the following
�Rn+;He � irreversible disintegration?
;�u --=-.!4 A -a, B � C � D � E
X
N2 + 3H2 � 2NH3 2
(3) Rale of a reaction is (3) Rate is affected by change in
independent of imposed
external conditions hke
temperature, pressure etc. Solution 2;i U belongs to actinide series hence, is of
temperature, pressure, etc. IIIB. Other elements are placed in the groups as shown:
(4) 11 is spontaneous and needs (4) It Is made spontaneous by
not be initiated by external external source as heat. Elements Atomic Number Group
means pressure 90
A 1118
Heat
2H20 � 2H2 + 02 B 88 IIA (alkaline-earth)
Ignite C 86 Zero (noble gas)
C + 02 CO2
(5) large amount of heat is (5) Change may be exothermic or D 87 IA (alkali)
1l berated about one million endothermic. Heat change is E 88 IIA (alkaline-earth)
_
t�mes greater than that generally very low.
hberated m a chemical
change.

Units of Radioactivity
Radio Elements and the Periodic Table Various types of units are-used for the radioactivity,
each one named after the scientist using it. These are
Elements formed due to emission of a- and �-- summarised in Table 5.4.
particles are placed in groups other than parent element.
This is by Soddy and Fajan's group displacement
Table 5.4 Units of Radioactivity
law (1913).
Quantity
• When an a-particle is emitted by a radioactive atom, Name Symbol Defined to be
measured
the product has an atomic number two units less than Curie Ci 3.7x 1010 Decay events
that the parent element and it occupies a position two disintegration per
places to the .'t.:ft of the parent element in the Periodic second (dps)
Table. Rutherford Ru 1 x 106 dps Decay events
�� Ra --+ �Rn + ;He Becquerel Bq (SI unit) 1 dps Decay events
6

II A Zero Gray Gy 1 kg tissue receiving Energy absorbed


1 J of energy

2
Rn ---. �8 Po + ;He Rad Rad 1 rad = 0.01 Gy Energy absorbed
Zero VIA Sievert Sv gray x quality Tissue damage
8
� Po ---. �4 Pb + ;He number of radiation
(e.g., 20 for alpha
VIA TVA and 1 for beta)
• When a �-particle is emitted by a radioactive atom, the Rem Rem 1 rem = 0.01 Sv Tissue damage
atomic number of the product becomes one unit greater
than that ofits parent; the product occupies a position 1 millicurie (mCi) = 3. 7 x 107 dps
one place to the right of the parent element in the
1 microcurie (µCi) = 3.7x 1()4 dps
Periodic Table.
9
1 Ci = 3.7x 1a1° Bq = 37 GBq (G == giga, 1 G = 10 )
P·emission involves the conversion of a neutron into
proton and electron Specific activity of a radionuclide is its activity
t i
n --+ H + -�e + nutrino per kilogram (or dm3 ) of the radioactive material.

The electron escapes the atom as �-particle, leaving (In some cases specific activity can also be taken as the
one proton extra in the nucleus, due to which atomic activity per gram)
218 | Essential Inorganic Chemistry

Theory of Radioactive Disintegration Rate of Radioactive Disintegration


According to this theory (Rutherford, 1903) the atoms Radioactive decay law states :uThe rate of
of a radioactive element are inherently unstable. They disintegration of a radioactive material (the decay rate) is
disintegrate spontaneously, emitting a- or p-particles, and directly proportional to the number of atoms present.”
dN xr
resulting in the formation of a new element which is
physically and chemically different from its parent dt
dN ,xr
element. The new element formed may, in turn, be again
radioactive and will disintegrate in a similar fashion. The dt
dN
process will continue until the atoms of stable element are where------ represents rate of decay in disintegration
dt
reached. Following radioactive disintegration series have of atoms per unit time (say second), N the number -of
been identified among the naturally occurring radioactive atoms disintegrating at that time and X the disintegration
elements and are summarised in Table 5.5. Series is constant (also called decay constant).
decided by dividing the atomic mass of the parent . _ rate of decay
element by 4. Series will be (4n + x), where x is the N
remainder. Radioactive decay is a first-order process*.
Neptunium series is artificial series while the other If is the number of atoms at the start of decay and
are natural series. The thorium, neptunium and actinium N the number of atoms at a given time then by first order
series are so named because an isotope of these elements kinetics,
rdN_
has the largest half-life in the respective series. The
J dt = X Jtv
actinium series was once thought to originate with
actinium-227—the isotope having the longest half-life . _ 2.303
amongst the series members. However, it has been
A —--------
t
10gl° )
investigated that the series actually initiates with a very N = Noe-u
long-lived uranium-235 isotope.
If T50 (or f1/2, both notations have been used in further
There is no cross-linking between the four series. parts) is the half-life which is the time for half of the
Thus, an isotope of one series will not decay to an isotope substance to disappear,
belonging to another series. thus, at
2.303
Table 5.5 Radioactive Disintegration Series then X = log 2
^1/2

Series Starts with Ends with _ 2.303 log 2 = 0.693


^1/2
232 X X
4n - Thorium series 90 ln ^Pb
(4n + 1) - Neptunium series 2£Pu ^Bi
Thus, half-life period is independent of number of
238 u
atoms present at a given time.
(4n + 2) - Uranium series 92 U ^Pb
235.J
Half-life provides an easy way to estimate the time
(4n + 3) - Actinium series 92 U ^Pb
required before a radioactive element is no longer a health
hazard. Strontium-90, for example, is a ^-emitter with a
Illustration 1 In Table 5.5, four disintegration series half-life of 29.1 years. Significant quantities of
have been given. Calculate number of a- and ^-particles strontium-90 were dispersed into the environment in
emitted in each series. atmospheric nuclear bomb tests in the 1960s and 1970s.
Solution and from the half-life, we know that a little less than half
is still around. Sr-90 is taken into the body and deposited
Number of in bone, taking the place of calcium. Radiation damage by
Net disintegration particles Sr-90 in bone has been directly linked to bone-related
Series reaction
a cancers.
P
4n - Thorium series ^Th—-» **Pb 6 4 If y = = number of half-lives in a time t
*1/2

(4n + 1) - Neptunium series £1Pu—> ^Bi 8 5 y

2381J >»
then
(4n + 2) - Uranium series 92 U ^Pb 8 6
235 U __
(4n + 3) - Actinium series 92 u * §7Pb 7 4 and in concentration terms C = Co
,2J
Chapters: Radioactivity | 219

The average-life period (iav) is related to iV2 and


' dV' ----- Illustration 2 The decay constant for sodium-24, a
disintegration constant by equation: radioisotope used medically in blood-study, is
j =1—= 1.443 tm 4.63 x 10~2 h'1. What is the half-life ofuNa?
" X 0.693 v
Solution Half-life (7^0) is related to the decay constant
X of the radioactive disintegration (which follows
20- first-order kinetics) by equation
_ 0.693
-10- so -
V) co
0.693
% 5— T50 = = 14.97 h
| 2.5 - 4.63 x IO-2 h“
The half-life of sodium-24 is 14.97 h
0 Illustration 3 The half-life ofradon-222, a radioactive
n 2 4 6 K
__J\.—half-life gas ofconcern as a health hazard is 3.823 days. What is the
decay constant when it undergoes first-order radioactive
Time (minute)
disintegration?
■ ^
iiiiu ;
iiiiiimig

Fig. 5.1 Decay of 20.0 mg of oxygen-15. After each half-life Solution As given in above example
period of 2.0 min, the mass of oxygen-15 decreases by one half m 0.693
^50-------

Illustration 1 Phosphorus-82, a radioactive isotope . 0.693 0.693


or A —---------- days
used in leukemia- therapy has a half-life of 14.28 days. ^50 3.823
What percent of a sample disintegrated after 35.0 days? = 0.181 day'
Solution Let sample initially present = No
and after 35.0 days sample = N Radioactive Equilibrium
decay constant X = =1^= 0.0485 day"1 According to the theory of radioactive disintegration,
T50 14.28 the product of a radioactive disintegration may also be
radioactive, and the rate of disintegration of the daughter
. 2.303!i element depends upon the amount of it present at
-log —-
N different time. When steady state is reached
0.0485=—:log —- pAT dN
35 N V dt parent dt daughter
i 35 x 0.0485
log—- = = 0.7375
5 N 2.303 or Xj-AZi = X27V2
N where Xi and X2 are the disintegration constants for the
—^■ = 5.464
N decay of parent and daughter radioactive species and
N and N2 are the number of atoms disintegrating at a
— = 0.183
given time.
ATr = X2 = (half-life)
% of sample left= 0.183x 100= 18.3 Thus,
7V2 Xx (half-life)
% of sample disintegrated = 100 - 18.3
If the parent (say A ) has smaller X than the daughter
= 81.7% (B), but none are very small, a steady state is reached at a
Alternate Method stage when
Let initial amount of sample = No = 100 Nb_
and sample left after time t = N Na ~ ^a
1 y This is called a transient equilibrium, which is in
then N = N0
2 fact a steady state.
total time (t) ^1=2.451 Illustration 1 Radioactive equilibrium is set up
where y =----------
F50 14.28 between uranium and radium atoms in a uranium
2.451 mineral in the ratio of 28 : 1 7^0 for radium is 1620 years.
AZ = 100 f— 5 = 18.3% Calculate the disintegration constant of uranium.
12.
.*. Sample disintegrated = jV0 - N = 100 - 18.3 Solution When radioactive equilibrium is set up
X1Ar1 = X22V2
= 81.7%
220 | Essential Inorganic Chemistry

*u.= /
1 0.693
Xg - — x
^Ra 28 k1620 years
*UU _ 1 = 1.528 x 10-5 yr
0.693 28
^5o)Ra

Target Practice 1
1. Complete the following nuclear equations. Write the mass 3. Which of the following nuclei decay by _°p decay?
number and atomic number for the remaining particle, as 3H 16O 20F 13 N
well as its symbol. 4. Which of the following nuclei decay by +°P decay?
(a) ^Fe+lHe
zo z
---- > 2*n
u
+? 238U 19F 22Na wNa ......
(b) JjAl + jHe --- » J5°P+...
10 days • •
(0 32S + in —> }H+........ 5. --------------- ►

(d) isF _> i*o +


(e) ? ---- » gMg
«Mg + > A A
2. What particle is emitted in the following nuclear reactions?
(a) Gold-198 decays to mercury-198 Xdays
(b) Radon-222 decays to polonium-218
(c) Cesium-137 decays to barium-137 A A
(d) Indium-110 decays to cadmium-110 Specify X.

Answers
1. (a) IfNi (b) on (0 ?52P (d) °e (e) pNa 3. Refer text
2. (a) ^Au ---- > ^Hg+.Je 4. Refer text
(b) |!2Rn ---- > 2]8Po + 4He
do
5. X = 10 (Half-life of first-order disintegration is independent of
13" .
initial radioactivity)
(0 55 US * ’do37Ba+—1 °.e
110T . UOpj , 0
(d) 49 *n * 4S "*■ 1 ®

Practice Exercise 1
1. Although a neutron within a nucleus is stable, a neutron in 5. Identify chemical change and radioactive change in the
free space decays into a proton and an electron (p-particle). following:
Write its equation. 0
(a) “C---- > 14N + e
2. What are approximate atomic weight of electron and (b) nAl + > —» ?5°P + \n
positron per mol?
27A1 + 3!H
(c) lu 1
A1H,o
3. 2^Th disintegrates to give 2^Pb as the final product. How
(d) 23Na + 2H —> JfNa + JH
many a- and p-particles are emitted during the process?
6. 2392 U (group III B) emits one a particle. What is the group of
4. Fill in the blanks :
(a) Alpha-rays are fast moving nuclei. new element formed?
7. Assign groups to the elements A, B, C, D and E starting from
(b) Emission of one a-and two p-particles will result in the element X (group II A)
formation of
X A B C D
(c) rays cause health hazards to maximum extent. (II A)
(d) Nuclear fusion is a reaction. What is relation between A and D?
(e) A radioactive change is always What is relation between A, B and C?
(0 The rate of radioactive change is independent of 8. If a radioactive element 79Au emits p-particle, what is the
pressure and new group? Is there any change in the number of moles of
(g) New elements are produced in daughter element? 79Au belongs to group I B (copper
family).
Chapters: Radioactivity I 221
9. What is relation between following units: 11. The decay constant for mercury-197, a radioisotope used
(a ) Bq and Ci (b) Bq and Ru medically in kidney-scans, is 1.08x 10-2 h-1• What is the
(c) Ru and Ci half-life of mercury-197?
IO. (- : ='AN) represents the rate of decay in disintegration 12. The half-life of carbon-14 is 5715 yr. What is decay
constant?
of atoms per unit time, where N is the number of atoms 13. Write balanced nuclear equations for each of the following:
disintegrating at that time and A the disintegration (a) (l-emission from ruthenium-106
constant. Calculate number of curies when number of (b) ex-emission from bismuth-189
atoms equal to one Avogadro's number disintegrates in the (c) electron capture by polonium-204.
case when)..= 0.6146 x 10- 10 s- 1•

Answers
I In o -
· o � 1P + -I� 8. 79 Au � 80 Hg +p . 80 Hg belongs to II B. (zinc family)
1

2. atomic weight of electron or positron There is no change in number of moles since atomic mass
=9.lx 10-28 x 6.00x 10 23 g moi-1 = 5.48x 10 --4 gmol- remains unchanged.
1

3. 7a and 6� 9. (a) 1 Ci= 3.7x 10 10 Bq (b) 1 Bq = 1x 10 6 Ru


4. (a) helium; (b) an isotope; (c) y (d) thermonuclear (c) 1 Ci= 3.7x 10 4 Ru
(e) spontaneous (f) temperature (g) radioactive change. 10. 1000 Ci 11. 64.17 h 12. 1.212x 10 --4 yr-1
5. (a), (b) and (d) radioactive change; (c) chemical change. 13. (a ) :Ru � -�e + ':sRh
1 1

6. III B (Z = 90 to 104 are in III B) (b) 1:Bi � ;He+ 1�1 Tl


1. A (Zero), B (IA), C(IIA), D (Zero)
A and D are isotopes; (c) �Po+ _�e --+ 2�i
A, B and C are isobars.

Nuclear Stability • Magic number for protons : 2, 8, 20, 28, 50, 82, 114
A number of basic questions have probably occurred to • Magic number for neutrons : 2, 8, 20, 28, 50, 82, 126
you as we have been describing nuclear decay processes. • �He (a-particle) is stable. It contains two protons and
Why do some radioactive nuclei decay by a-emission two neutrons hence, it contains magic number of
and some by p- -emission ? protons (2), neutrons (2).
23 2
Why do the lighter elements have so few naturally :2 U � 0:2 Pb (stable)
occurring radioactive nuclides, whereas those of the It <2':2 Pb) has 82 protons (magic number)
heavier elements all seem to be radioactive? Table 5.6 gives number of stable isotopes.
From the Fig. 5.2, it is clear that nuclides with even
number of protons and neutrons have higher binding Table 5.6 Number of Stable Isotopes with Even and Odd
energies than their neighbours and are stable. Numbers of Protons and Neutrons
Experimentally, it is found that nuclei with certain
number of protons or ,1eutrons appear to be very stable. Number of stable isotopes
These numbers called magic numbers and associated 157 52 50 5
With specially stable nuclei, were later explained by the
shell model. According to this theory, a magic number Number of protons Even Even Odd Odd
is the number of nuclear particles in a completed Number of neutrons Even Odd Even Odd
shell of protons or neutrons.

--
16Q • Finally when we plot each stable nuclide on a graph
9.00 between the number of protons (Z) and the number of
•• rr-· •
--
56
-r---
' � neutrons (N), these stable nuclides fall in a certain
/I �12c Fe •r--_.. region, or band, of graph. (Fig. 5.3). The band of
t
7.00
◄r' r-,4He stability is the region in which stable nuclides
Binding
lie in a plot of number of protons against
S.00
la e
energy number of neutrons. From this graphs it is clear
per that
nucleon 3 00
(MeV)
• Stable nuclei are those for which number of neutrons
and protons are equal and are generally upto 20.
1.00 (4 He , 1 2 C , 16 O 2s S1. -&o ca)
0 50 100 150 200 250 2 6 s • 14 ,20
Mass number--+- • For higher atomic numbers, because of increasing
Fig. 5.2 Binding energy per nucleon repulsive forces between protons, large number of
222 | Essential Inorganic Chemistry

106
neutrons must be present to stabilise a nucleus and 106 Pd
46ra .
the neutron/proton (n/ p) ratio increases. 37 Ar + _xe ___
is 37Vi
---- >* 17 C1
For bismuth the ratio is about 1.5 : 1
• All nuclides with atomic number greater than 83 are
• Isotopes of all the known elements with Z > 83 are beyond the band of stability and are radioactive. Many
radioactive. of these decay by a-emission.
120 ■
2|6 Ra---- > 2 8g Rn+o He
Neutron-rich 218°Po---- > 20862Pb +2He
nuclei 1.5
(/)c 100 Illustration 1 Consider the following radioactive
2 /• • •9 change
<D
nNa^?02Ne
C 80
'What is the type of particle captured or emitted1?
o
E
Solution nNa —> gNe
□ 60
z There is change in atomic number of one unit, hence,
it can
40
(i) capture an electron
22Na + _°e ---- > 1022
22Ne
20 (ii) emit a positron 22Na---- > gjNe + +°P
Illustration 2 Identify probable models) of decay for
80 each isotope and write an equation for the decay process.
Number of protons (a) oxygen-15, (b) uranium-234 |24U (c) fluorine-29,
|°F(c0 manganese-56,25Mn
Fig. 5.3 Plot of the number of neutrons against the
number of protons in stable nuclei (shown by dots) Solution (a) Oxygen-15 has 7 neutrons and 8 protons,
so the nip ratio is less than 1-too low for 15O to be stable.
Nuclei with too few neutrons are expected to decay by
Mechanism of Radioactive Decay either positron emission or electron capture. In this
The emission of an a-particle by a nucleus is not instance, the process is ., B emission, and the equation is
difficult to visualise. A bundle of four nucleons (two 85O —» Jjp + ^N.
protons, two neutrons) is ejected from a nucleus, and the
nucleus becomes energetically more stable. In the (b) Alpha emission is a common mode of decay for
emission of P-particle, neutron is converted into proton. isotopes of elements with atomic numbers higher
238 tt ____ . 234
ot-emission than 83. The decay of uranium-234 is one example.
92u ------ * 90 Th + 2He
---- » |p p“-emission 92 > 901 h + 2a
+ -ie
Jp --- > in + P+-emission (c) Fluorine-20 has 11 neutrons and 9 protons, a high
1 „ , „0,0 _____ x
nip ratio. The ratio is lowered by p emission :
iP + -iC > on electron capture
• In general, if a nuclide lies above the belt of stability
“F —„ .’p + JjNe
(Fig. 5.3), the — | ratio is too high. (d) The atomic weight of manganese is 54.85. The
Ip; higher mass number, 56, suggests that this
A neutron is converted to a proton and a P~ particle is radioactive isotope has an excess of neutrons,
emitted. in which case it would be expected to decay by
14 ____ . 14 xt , P-emission :
6C---- > 7N + _je 0
gMn —a °iP + ^Fe
— =1.33 - = 1 (stable)
P P
• If a nuclide lies below the belt of stability, the —
\P.
Nuclear Reactions
We may classify nuclear reactions according to the
ratio is too low.
projectiles as:
Either a proton is converted to a neutron, followed by
P+ emission or electron capture occurs. Charged Particle Reactions
gK—> gAr+JJe (or +?p) Such reactions may be carried out by accelerated
158O—> 157N + +°p protons, deutrons or alpha particles:
Chapters: Radioactivity | 223

Projectile Reaction type • When a neutron strikes the g|5U nucleus, the nucleus
Proton splits into roughly equal parts, giving off several
Deutron neutrons.
(d, p),(d,n),(d,a),(d, Y)
Alpha particle 1452 Xe + 38 Sr + 4 (J, n)
(a, n), (a, p)
235
• By writing (p, n), it means, proton is absorbed grt + 92 l3596Ba + ;“Kr + 3(Jn)
and neutron is emitted. 1 “5Cs + 3°Rb + 2 (in)
Examples: • If the neutrons from each nuclear fission are absorbed
(i) p,n 24Na + 4 12 Mg + on by other uranium -235 nuclei, these nuclei split and
release even more neutrons. Thus, a chain reaction
(ii) p,a iLi + }H ^He + ^He can occur. A nuclear chain reaction is a
(iii) p,Y ?37ai + ;h —» ?JSi + Y self-sustaining series of nuclear fissions caused by the
absorption of neutrons released from previous nuclear
(iv) d, p- ?37ai +?h—4 If Al +}H fissions. This is shown in Fig. 5.4.
(v) d,n gAl + ?H--- > 14 Si + in 1 n 235 tt
X on 92
(vi) d,a 3 Li + 2H---- > 4He + J He 139 Ba
56 Ca 235
235
(vii) a, n (X)+Jn 92 u
49Be +^He---- > 126C + in 92
235
(viii) a, p Y 92 u
147N +^He---- > 178o +JH
• Reaction (ii) is the first artificial nuclear reaction 235 tt 1 X + 3jn
carried out with artificially accelerated particles. in 92
+ on Y
• Reaction (vi) gives one of the highest non-fission 94 TT X + 3jn
energies (22.2 MeV) recorded for light atoms. 36Kr +
on Y
• Neutron was discovered through the reaction (vii). (Y)
• Reaction (viii) is the first artificial nuclear reaction Fig. 5.4 Representation of nuclear chain reaction of nuclear
studied. fissions
• There should be a critical size of the fissionable
Neutral Particle Reactions material to maintain a fission chain. This in turn,
Neutrons are generally used as projectiles in such requires a minimum critical mass of the fissionable
reactions. Neutrons of high energy are obtained from material. It is the smallest mass of the fissionable
different artificial nuclear reactions. These are slowed
material in which a chian reaction can be sustained. If
down through moderators like graphite, heavy water,
paraffin, etc. Much of the energies of the neutron is the mass is much larger than this (a supercritical) mass,
transferred to these substances through elastic scattering. the number of nuclei that split multiply rapidly. An
Example: atomic bomb is detonated with a small amount of
238 tt , 1 . 239 tt , ,, chemical explosive that pushes together two or more
(i) n,y 92D + on ---- * 92D + Y masses offissionable material to get a supercritical mass.
(ii) n, p +in---- > 146C +}H
Containment
(iii) n,a f Di + Jn--- > jHe + shell \ Steam
Steam
turbine
• Reaction (i) shows the formation of transuranium
elements. ~Y~—__Electrical
-J output
Reaction (iii) shows the production of tritium.
Condenser
Reactions initiated by photons may also be considered Control \ (steam from
as neutral particle reactions. rods „
turbine is
Fuel J condensed)
?H +Y 4 + xon elements > X
?Be + Y 4 ®Be + in
38’C
Large water
Nuclear Fission
Reactor
Water-
Pump ZJ
r \ source

It is a nuclear reaction in which a heavy nucleus splits


Steam
into lighter nuclei and energy is released. generator Pump
• Californium -252 decays both by alpha emission (97%) Fig. 5.5 A diagram of a nuclear power plant. Heat produced in
and by spontaneous fission (3%). During spontaneous the reactor core is transferred by coolant circulating in a closed
fission, the nucleus splits into two stabler, fighter loop to a steam generator, and the steam then drives a turbine to
nuclei plus several neutrons. generate electricity.
Ts Cf—> 14526Ba + ^2 Mo + 4g n
224 | Essential Inorganic Chemistry

A nuclear fission reactor (Fig. 5.5) is a device that ^As + 2H ---- > §Mn + 9jH + 129n
permits a controlled chain nuclear-fissions. Control rods 238U + 2He---- > ffW + 20}H + 35^
made of elements such as boron and cadmium, absorb
additional neutrons and can therefore, slow the chain
reactions.
A nuclear reactor consists of three components:
Nuclear Fusion
In this type of nuclear reaction, certain light nuclei
(a) A fissile material (uranium enriched in 292U, say
may fuse together with the liberation of tremendous
2.3%) amount of energy. To achieve this, the colliding nuclei
(b) A moderator (graphite or heavy water D2O) to slow
must possess enough kinetic energy to overcome the
the neutrons so that they are captured and become
initial repulsion between the positively charged cores.
effective to bring about fission reaction.
This energy may be made available by raising the
(c) Control rods made of boron steel or cadmium, which
temperature of the reacting system to several million
are capable of absorbing neutrons and are used to
degrees. Such reactions are therefore, also known as
ensure that neutron flux is under control and thus
thermonuclear reactions.
chain-reaction is controlled. The large amount of
energy released from the fission is used for steam + fH---- > 2He + Jn + 17.6 MeV
generation through heat exchanges to produce The energy of a fusion process is due to mass defect
electricity. At present our country is proud of (converted to binding energy). The high temperature
having more than 12 such nuclear power plants required to initiate such reactions may be attained
producing electricity. initially through a fission process.
• A breeder reactor uses uranium fuel, but unlike a Spectroscopic evidence indicates that the sun is a
conventional nuclear reactor, it produces more tremendous fusion reactor consisting of 73% H, 26% He
fissionable materials than it uses. When and 1% other elements. Following fusion reactions are
uranium-238 is bombarded with fast neutrons, the believed to take place:
following reactions take place:
+?H---- » 23He
“’U + 'n ---- >> 23992U
3He+3He--- > ^He + 2jH
239 t j Tjq = 23.4 min 4

92 u 29339Np + }H+{H---- > ?H++?e


239 50 = 2.35 oay
T.50 day 239 0 A major concern in choosing the proper nuclear fusion
process for energy production is the temperature
In this manner the non-fissionable uranium-238 is necessary to carry out the process. Some promising
transmuted into the fissionable isotope plutonium-239. reactions are:
In a typical breeder reactor, nuclear fuel containing
uranium-235 or plutonium-239 is mixed with Reactions Energy released
uranium-238 so that breeding takes place within the core. fH+2H---- > 3H+jH 6.3x W13 J
For every uranium-235 (or plutonium-239) nucleus ?H+?H jHe+oO 2.8 x 10"12 J
undergoing fission, more than one neutron is captured by
fLi+fH 2^He 3.6 x 10‘12 J
uranium-238 to generate plutonium-239. Thus, the
amount of fissionable material can be steadily increased
as the starting nuclear fuels are consumed.
232 Th is also used. When it captures slow neutrons, it Artificial Radioactivity
is transmuted to uranium-233 which is also a fissionable Bombardment of stable elements with high energy
isotope. a-particles, protons, neutrons, deutrons or y-rays produce
^Th + Jn --- > 293o3Th radioactive nuclides. These radio-nuclides do not occur
T50 = 22 min % 233 naturally and may be called man-made or artificial. The
233 rrn
90 -lb > 91 Pa + 4(3 radioactivity exhibited by these artificial radio-nuclides is
T50 = 27.4 day^ referred to as artificial radioactivity or as induced
2333Pa “aU + _°p radioactivity. (I. Curie and F. Joliot-1934)
The decay resulting from artificial radioactivity is
usually a one-step process, i.e., the radioisotope
Spallation disintegrates into a stable isotope which does not show
Seaborg and his coworkers (1947) observed any further tendency to decay.
that bombardment with high energy charged particles (a) The first artificial breakdown of the atomic nucleus
(e.g., 400 MeV protons or 200 MeV deutrons) can break was achieved by Rutherford in 1919. He bombarded
some target nuclei into several smaller nuclei with the nitrogen with a-particles and obtained oxygen and
emission of a large number of nucleons (10 to 20 or more). protons
These are called spallation reactions. 147N +2He 870+}H
Chapter 5: Radioactivity I 225
(b) The newly formed isotope undergoes positron • The packing fraction does not have any precise
emission. theoretical significance but it gives an indication
about the stability of the nucleus.
�� Al + :He � �gp + �n • A negative packing fraction indicates that the isotopic
aop T5 0 = 3.25 min 3 o mass is less than the nearest whole number. This
15 14081.+1 e
suggests that a small fraction of the mass has been
artificial radioactive stable converted into energy of formation of that nucleus.
Hence, the more negative the packing fraction of
�Mg + iH � i�Na + �He an element, the more stable should be the
221 N Tso = 2.6 yr 22Ne +1o e nucleus.
1 a 10
__,;;;.;;.....__-4
• On the other hand, a positive packing fraction indicates
2
1 4Mg + 24He �
278 . + 1
14 1 on that the concerned nucleus should be less stable.
2
• Generalisations are shown in the Fig. 5.6.
i!Si � iiAl + �e
i B+:He �
0 �3:N+ in
�ac+ fe
+8
�sN � 4He

l
+6
(c) The newly formed isotope undergoes electron
emission (�-decay) +4

I
23Na+1 T50 = 15 h 4 o C:
+2
on� uNa ---- 2 21 Mg+_1e
24
u 12c

iBe+ i,H � 0Be+ iH ! 4:: 0

!
O>
0 Be � i0B+ -�e C: -2
31p l 2 a.. -4
1s + on � 138 Al + 24He Maximum
2s 2s . 0
81 + o
13 Al � 14 -le
stability
1 1 -8
io+�n� ic+�He
(d} The newly formed isotope undergoes K-electron -10
capture (electron from K-shell or ls energy level). In -12
this process energy is emitted in the form of X-rays. 50 100 200 250
◄s 2 Mass number, A
22 Ti + 1 8 � 49V2a + 1on
:�v + -�e(K-shell) � ::Ti+ X-rays Fig. 5.6 Plot of packing fraction against mass number
(e) There can be formation of isomer in artificial This shows:
nuclear reactions.
- The packing fractions of the stable nuclides lie

=�Kr+
SOB
35 B r+ 1on� 35 r
79 more or less on a smooth curve.
ggBr � _�e - The curve passes through a minima in the mass
Thus 80 Br exists as two nuclear-isomers. number range 50-60, implying the maximum
stability.
- After the minima, packing fraction steadily
Binding Energy and Packing Fraction increases and ultimately becomes positive for
The nearest integer to the mass of a nuclide (in the elements of high mass number.
atomic mass scale) was called the mass number of that
This is consistent with the observed radioactivity, i.e.,
isotope. The difference between the actual isotopic mass
inherent instability, of these elements.
and mass number was termed as mass defect of the
nuclide. Binding energy (B) = t:,m x c2
Thus, Am (mass defect) = isotopic mass (m) Binding energy of the nucleus is equal to the energy
- mass number (A) required to split the nucleus into its component nucleons.

Aston introduced a term "packing fraction" for each Nuclear binding energy is generally expressed in the
nuclide to compare their mass excess, it was defined as: non-SI unit of MeV (mega electron volt).
27
Since, 1 amu = 1.66 x 10- kg
Packing fraction (f) = t:,m x lo'
A .. (B) for one amu = 1.66 x 10- x (3 x 108 ) J
27 2
226 | Essential Inorganic Chemistry

_1.66x IO'27 x (3 x 108)2 J Illustration 1 Helium-6 is a radioactive isotope with


1.602x 1(T19 J/eV T5Q = 0.81 s. Calculate the binding energy if mass defect is
= 932.6 x 106 eV = 932.6 MeV 0.03141 g mol'1. Also calculate binding energy ofhelium-b
(B) = Am (in amu) x 931 MeV whose mass defect is 0.03038 g mol"1. Is a 6He nucleus
_ B more stable or less stable than a 4 He nucleus'!
Binding energy per nucleon (B) = —
A Solution Binding energy of 6He=
He-*Am
w“xwc'’‘2
total binding energy
<0.03141
total number of nucleons “I 1000 kg mol (3x 108 ms'1)2
A plot of average binding energy per nucleon against
= 2.8269 x 1012 J mol
mass number gives a nearly smooth curve (Fig 5.2). The
curve is practically an inversion of the curve obtained by Binding energy of
plotting packing fractions (Fig 5.6). Maximum binding 0.03038
4He = kg mol (3x 108 ms’1)2
energy per nucleon is indicated in the mass number range 1000
55-60. The average binding energies for4 He,12C and 16 0 = 2.7342 x 1012 J mol
lie above the curve, showing that they are exceptionally 2.8269 x 1012
stable. Binding energy of 6 He per nucleon=
6
This may be attributed to several factors. Firstly these = 4.7115 x 1011 J per nucleon
atoms have even-even combination of protons and n- ! 2.7342X 1012
neutrons in their nuclei. Binding energy of He per nucleon=------------ —
4
Nuclei Neutron Proton
= 6.8355 x 1011 per nucleon
jHe 2 2
Thus, 6 He nucleus is less stable than a 4 He
12C
6 u 6 6 nucleus.
16O
8 u 8 8

Secondly, number of protons and neutrons are equal.


thirdly the electrostatic repulsion between the protons in Applications of Radioactive Isotopes
these three cases is less than that of their immediate Carbon-Dating
neighbours. Electrostatic repulsion is measured by
Z2 Radiocarbon-dating of archaeological artifacts
~ (A)1'3 depends on the slow and constant production of
radioactive carbon-14 in the upper atomosphere by
where Z is the number of protons and A the mass number. neutron bombardment of nitrogen atoms (The neutrons
We compare this type of force with the immediate come from the bombardment of other atoms by cosmic
neighbours. rays).
147N+in ---- > 146C+}H
Table 5.7 Electrostatic Force of Different Nuclei
64C + O2 ---- > “CO,
Electrostatic
Nuclei Z A Carbon-14 atoms produced in the upper atomosphere
force
^He 2 4 (2)2 combine with oxygen to yield 14CO2, which slowly diffuses
= 2.53 into the lower atmosphere, where it mixes with ordinary
(4)1'3
12CO2 and is taken up by plants during photosynthesis.
3L1 3 6 4.97
When these plants are eaten, carbon-14 enters the food
l2C 6 12 15.78 chain and is ultimately distributed evenly throughout all
1/N 7 14 20.33 living organisms.
l6O 8 16 25.39
As long as a plant or animal is alive, a dynamic
9 19 30.45 equilibrium exists in which an organism excretes or
In case of helium-4 and oxygen-16, there is an exhales the same amount of 14 C that it takes in and the
additional reason for exceptional stability. In each case ratio of14 C to12 C in the living organism is the same as that
number of neutrons (2 and 8) corresponds to magic in the upper atmosphere. When the plant or animal dies,
number. however, it no longer takes in more 14 C, and the ratio of
14Cto 12 C slowly decreases due to decay of14 C
Note: To convert amu into MeV conversion factor is generally
taken 931. 146C---- > 14N+_?e
Chapter 5: Radioactivity | 227

Thus, from the activity of 14 C in the dead organism Medical Uses of Radioactivity
and that of the fresh sample (in the atmosphere), age can • French physician Henri Danlos first used radium in
be decided. the treatment of tuberculous skin lesion. Now, uses of
X(decay constant) =-^ No (original activity) radioactivity have become a crucial part of modem
log medical care, both diagnostic and therapeutic
i(age) N (final activity)
• Various techniques used in medical treatment are
Also, . 0.693 grouped into three classes:
A —--------- (half-life)
^50 (i) in vivo procedures,
Thus, t (age) can be determined. (ii) therapeutic procedures and
(iii) imaging procedures.
or 2V = N0
2 • In vivo studies—those take place inside the body—are
carried out to assess the functioning of a particular
where total time (t)
organ or body system. A radio-pharmaceutical agent
y ~ half-life(T50)
is administered, and its path in the body—whether it
is absorbed, excreted, diluted or concentrated is
Rock (Geological) Dating determined by analysis of blood or urine samples. 51Cr
Uranium-238 has a half-life of 4.46 x 109 years and radioisotope is used in this type of study.
decays through a series of events to yield lead-206. • Therapeutic procedures —those in which radiation is
238 tt 206 Ph used as a weapon to kill diseased tissue—can involve
92U * 82rD
either external or internal source of radiation.
The age of uranium containing rock can, therefore, be
determined by measuring the 238u/206Pb ratio. Cobalt-60 used in external radiation therapy for the
It is based on the kinetics of radioactive decay. It is treatment of cancer. It is embedded in the cancerous
assumed that no lead was initially present in the sample tissue and emitts both p- and y-rays
and the whole of it came from the uranium. 2®Co ---- > 28^1 + + Y (Y rays destroys cancer cells)
Initial radioactivity = [U] + [Pb] = No in terms of Sodium-24 injected alongwith salt solution to trace
gram atoms
the flow of blood.
Final radioactivity = [U] = N
We have assumed that due to high value of T50 of Iodine-131 used in internal radiation therapy for the
uranium, its amount remains unchanged. treatment of thyroid disease.
No =[U] + [Pb] 1 | [Pb] Phosphorus-32 used for leukemia therapy which
N [U]
CUI [U] controls polycythemiavera-a. condition in which the
bone marrow produces excess red blood cells. The
Thus, . 2.303
A —--------- radioactive P-32 (in the form of phosphate) gets
^50
preferentially absorbed in the bone marrow and emits
2.303 y-rays, which destroy excessive red blood corpuscles.
1
ogL cm. • Imaging procedures gives diagnostic information
about the health of body organs by analysing the
or 1+™1 = (2P distribution pattern of radioisotopes introduced into
[U1J the body.
where y = U
~ — hence, ti is calculated.
*50
Iodine-131 employed to study the structure and
activity of thyroid glands.
Thus, knowing the ratio of uranium to lead or thorium
to lead in a particular minerals and knowing the half-lives Iodine-123 used in imaging the brain.
of the radioisotopes, the age of the natural substance may Technetium-99 M used in bone scans.
be estimated.
Similarly, potassium-40 has a half-life of • MRI (magnetic resonance imaging) uses radio-waves
1.26 x 109 years and decays through electron capture and to stimulate certain nuclei in the presence of an
through positron emission to yield argon-40. extremely powerful magnetic field.
SK+_?e J?Ar
gK—>4?Ar+°e Power Generation
The fission of uranium-235 nucleus by slow neutrons,
The age of a rock can be determined by crushing a
releases energy to the tune of 200 MeV
sample, measuring the amount of40 Ar gas that escapes,
and comparing the amount of argon-40 with the amount of 29325U + Jn ---- > J^Ba + gKr + 2-3 in + 200 MeV
K remaining in the sample
228 | Essential Inorganic Chemistry

which may be converted into electrical energy. Power The two labelled chlorine would rapidly exchange
produced by nuclear energy is unmatched as compared to with three non-labelled ones, if all five chlorines are in a
any other energy resource. Theoretically, one tonne of similar environment, and as a result all five chlorines
uranium can give as much electrical energy as 3 million would have radioactive character. If above synthesised
tonne of coal or 12 million barrels of oil. Further, not only PC15 is hydrolysed.
the reserves of coal and oil are exhausting fastly, but
burning of these fossil fuels pollutes the environment. PC15 +H20 ---- > POC13 + 2HC1
Nuclear energy is a clean and abundant resource for HC1 is found to be radioactive andPOCl3 is found to be
power generation. non-radioactive. Thus, two chlorines are different from the
other three.
Agricultural Applications • To decide structure ofS2O^“.
Isotopes can be used to measure the uptake of The formula of thiosulphate ion is S2O3~. Following
nutrients. Plants take up phosphorus containing reaction is found to be reversible depending on the
compounds from the soil through their roots. By adding a • experimental conditions :
small amount of radioactive 32 P, a ^-emitter (with a TJ+
half-life of 14.3 days), to a fertiliser and then measuring SO|-(aq) + S(s) S2O|-(ag)
the rate at which the radioactivity appears in the leaves, If one starts with elemental sulphur enriched with the
plant biologists can determine the rate at which radioactive sulphur-35, the isotope acts as a “label” for
phosphorus is taken up. The outcome can assist S atoms. All the labels are found in the sulphur
agriculture-scientists in identification of hybrid strain of precipitated when S2O^“(ag) is treated with dilute
plants that can absorb phosphorus quickly, resulting in acids. Thus, two sulphur atoms are not structurally
faster-maturing crops, better yields per acre, and more equivalent as would be case if the structure were:
food or fibre at lower cost. To measure pesticide level, a rn
pesticide can be tagged with a radioisotope and then
n • 12-
[’ o- sq — no— so — o : ]2'
applied to a test field. By counting the disintegrations of Otherwise, the radioisotope would be present in both
radioactive tracer, information can be obtained about how the sulphur atoms. Based on this thiosulphate has the
much pesticide is in soil; is taken up by the plant, and is structure:
carried off in runoff surface water. (Radioactive isotope 0
2-

used to carry out these investigation has short


half-life and decays to harmless levels in a few days) :o-s s
• • I

Isotopes Dilution Technique is used to determine,


the volume of blood in an animal subject. A small amount :o:
of radioactive isotope is injected into the bloodstream.
Oxidation number of central S atom is +6 and that of
After a period of time to allow the isotope to become
terminal is -2, thus net oxidation number is 4 (for two
distributed throughout the body, a blood sample is taken
S-atoms) and difference is 8.
and its radioactivity measured.
Study of photosynthesis also uses radioisotope 18 Oto
determine the source of 02
Chemical Investigation 6CO2 + 12H218O---- > C6H12O6 + 60j8 + 6^0

Structural Determination On starting with 18 0 enriched water (H2 18 0) we find


that only 02 has radioisotope indicating that 02 comes only
When more than one atom of a particular element are from H20 and not from C02.
present in a molecule, it needs to be decided whether all of
• To study the mechanism of the following
these are equivalent or not. Radioisotope labelling settles
reaction:
the issue. 0 0
• To decide nature of bonding in PC15.
CH3-C—0—H + C^Hg S H
II 18
CH3—C-OQHs
In order to determine whether all five chlorides in
phosphorus (V) chloride are in a chemically similar + H,0
environment, the compound may be synthesised using 0 of CjHgOH is enriched with radioisotope 18 0. After
labelled chlorine molecules. esterification,18 0 is found in the ester and not in RO
indicating —OH of CH3C00H is replaced by —OCjHj
PC13 +C12 * PC15 of CjHgOH which acts as an acid in this reaction.
Chapters. Radioactivity | 229

Illustration 1 What is the age of a bone fragment that Alpha and beta-particles can be detected by a Geiger
showsan average of 2.9 disintegrations per minute (dpm) counter. To detect neutrons, boron trifluoride is added to
per gram of carbon? The carbon in living organisms the gas in the tube. Neutrons react with boron-10 nuclei to
undergoes an average of 15.3 dpm I min per gram, and the produce alpha particles, which can then be detected.
half-life of14C is 5715 years. >n + '»B---- , ’Li+42He
Solution T50 of 14C= 5715 yr
A scintillation counter is a device that detects
0.693 0.693 nuclear radiations from flashes of light generated in a
decay constant X =
^50 5715 yr material by the radiation. A phosphor is a substance that
emits flashes of light when struck by radiation. The
= 1.2126 x 10"4 yr"1 gamma rays are detected by this counter- a sodium iodide
Also, . 2.303 . No crystal containing thallium (I) iodide is used as a phosphor
X =------- log —-
t (age) N for gamma radiations. Rutherford used zinc sulphide as a
■where No the initial activity = 15.3 dpm and N the phosphor to detect alpha particles.
activity after time t = 2.9 dpm
1.2126 x IO"4
. 15.3 Biological Effects and Radiation Dosage
log----- Quantity of radiation intermixing with biological
t 2.9
t = 13718 yr organisms might be small but the effects are quite adverse
because important chemical bonds may be broken; DNA is
Illustration 2 A 1.00 mL solution containing specially affected.
0.240 pCi of tritium is injected into a dog’s bloodstream. Effect of nuclear radiations on biological tissues is
After a period of time to allow the isotope to be dispersed, a
expressed in terms of RAD (radiation adsorbed dose)
1.00 mL sample of blood is drawn. The radioactivity ofthis
which is the dosage of radiation that deposits 1 x 10"2 J of
sample is found to be 4.3 x 10"4 pCi/mL. What is the total
volume of blood in the dog? energy per kilogram of tissue.

Solution Let total volume = X mL • Neutrons are more destructive than gamma rays of
the same radiation dosage measured in rads.
Activity = 0.240 p.Ci A REM (roentgen equivalent for man) is a unit of
Activity per mL = 0.240 pCi/X radiation dosage used to relate various kinds of radiation
in terms of biological destruction.
After dilution in the blood stream Rem = radx RBE
activity per mL = 4.3 x lO^pCi/mL where RBE stands for the relative biological
Thus, 0.240pCi/X = 4.3 x 10"4 pCi/mL effectiveness.

X = 558.14 mL • For beta and gamma-radiations, RBE = 1 radiation.


• For neutron, RBE = 5
• For alpha-radiation, RBE = 10.
Radiations and Matter The biological effects of different radiation-doses are
Detection and Biological Effects given in Table 5.8
Radiations from nuclear processes affect matter in Table 5.8 Biological Effects of Short-Term Radiation on
part by dissipating energy in it. In some cases, these Humans
radiations may also excite electrons in matter. When these
electrons undergo transitions back to their ground states, Dose (rem) Biological effect
light is emitted. The ions, free electrons and light 0-25 No detectable effects
produced in matter can be used to detect nuclear 25-100 Temporary decrease in white blood cell count
radiations. Because nuclear radiations can ionise
molecules and break chemical bonds, they adversely affect 100-200 Nausea, vomitting, longer-term decrease in white
biological organisms. blood cells
200-300 Vomitting, diarrhoea, loses of apetite, restlessness
300-600 Vomitting, diarrhoea, hemorrhaging, eventually death
Radiation Counters in some cases
A Geiger counter, a kind of ionisation counter used above 600 Eventually death in nearly all cases.
to count particles emitted by radioactive nuclei consists of
a metal tube fitted with gas such as argon.
230 | Essential Inorganic Chemistry

Target Practice 2
1. What is Einstein mass-energy relationship?............. 5. Which radioactive isotope is used to control
2. When an electron is emitted, mass number remains polycythemia vera?
unchanged but atomic number increases by one unit. Write 6. Cobalt-60 is used to irradiate tumors with y-rays.
nuclear reaction............... 60p ____ . 60vn, 0„, „
27uo >
8. Write full name of MRI.
Which is responsible for destroying cancer-cells?
4. What is application of radioactive sodium chloride
containing sodium-24? 7. A museum wishes to analyse a piece of ruby for chromium
content. What should be preferred method of analysis?

Answers
,2
1. E = me' 5. Phosphorus-32
2. > 6. y-rays,
3. Magnetic Resonance Imaging 7. Neutron activation analysis.
4. It is used to study obstructive vascular diseases in the
extremities of the body.

Practice Exercise 2
1. The half-life periods of two elements A and B are 5 days and 8. Complete the following nuclear reactions :
5 months, respectively. Which of the two is more stable ? (i) 2gCa(n,a) (ii) gMg^a)
2. Which of the following are not stable ?
(iii) JH (a, n) (iv) “jAu(n,y)
(i) 14u C (ii) 13O C (v) J*Al(y,n) (vi) ®Be(a,n)
(iii)14 N (iv)24Na
9. What are the types of reactions?
3. A proposed series of reactions (known as the
carbon-nitrogen cycle) that could be important in every (a) 147Ca + 4He---- > 178O + JH
hottest region of the interior of the sun is: (b) «Li + Jn---- > 4He + fH
I2C+1H---- > A+y (c) 2H + y ---- > + Ton
A B+> 10. How a radioactive nucleus can emit a 0“ particle (electron) if
there are no electrons present in the nucleus?
B + ’H ---- » C + y
11. Why both a and 0-particles are emitted in a radioactive
C + ’H ---- > D+y
decay series?
D —» E + >
12. Why is U not suitable for a chain reaction ?
E + 1H---- > 12C+F
Identify the series labelled A to F. 13. A typical neutron-initiated fission of 2325 U yields ®2 Mo, two
4. Give product of the following nuclear reactions: neutrons and an isotope of what element?
(a) 10497Ag + ’He —> ? 14. If an atom of ^25U, after the absorption of a slow neutron
(b) ”B+42He—> 7+^n undergoes fission to form two fragments 139 Xe and ^Sr.i
what other particles are produced and how many?
(0 ^Cm^C—> ? + 4in
15. The binding energy per nucleon for 2325U is 7.5 MeV and
(d) ^Cf+^B—4 ? + 4’0n
8.5 MeV for nuclei of half that mass. If 235 U undergoes
5. Write balanced nuclear equations for the following
nuclear fission in two such nuclei of equal size, how much
processes:
(a) Alpha emission of 162Re energy is released in the process?
(b) Electron capture of138 Sm 16. The overall reaction in the sun responsible for the energy ii
radiates is
(c) Beta emission of188 W
4(}H)---- > 4He + 2°e(0+)
(d) Positron emission of165 Ta
It has been estimated that the sun produces 3 x 1022 kJ s-1.
6. Write the abbreviated notation for the following
How many moles of H-atoms are converted to He in 1
bombardment reaction, in which neutron was first second, to produce this energy ? (Take binding energy
discovered: 27 MeV)
®Be+2He—— 17. Fill in the blanks :
7. Write the nuclear equation for the bombardment reaction (a) ............ is a thermonuclear reaction.
denoted as 2J Al (p.d)™ Al.
Chapter 5 Radioactivity | 231

(b) The energy production in the sun and stars could be 18. Make distinction between nuclear fusion and nuclear
explained on the basis of .... ... by means of which fission.
hydrogen is converted into helium. 19. An artificial element shows activity of 12.5 cps (counts per
(c) was awarded the 1967 Physics Nobel Prize for second) at its formation as recorded in a G.M. counter. After
his theory of energy production in the sun and the 10 min activity is 7.5 cps. In the absence of element, G.M.
stars. counter records activity of 2.5 cps. What is its half-life
(d) Hydrogen bomb is based on period and activity after 20 days?
(e) Instrument used to accelerate nuclear projectiles is 20. Give one example each of (i) a-particle-induced reaction
called (ii) proton-induced reaction and (iii) neutron-induced
(D reaction.
was awarded the 1939 Physics Nobel Prize for
invention and development of cyclotron. 21. Packing fraction of element A, B, C are -20, 27, +18,
(g) ®Be + 4He---- > 12C led to the discovery of respectively. Arrange them in the increasing order of
by
stability.
Chadwick.

Answers
1. Element 3 is more stable. (v) 237A1+y—> “ Al + 9n
2. (i) “C (ii) C and (iv) 24 Na are not stable. (vi) 9Be+^He---- » 12
6r C j.
+1onn
3. A: 73N B : 63C C: D : ^0 E: “N 10. See text
9. (a) a, p ; (b) n, a ; (c) y, n.
F: 2He 11. See text 12. Due to less stability' 13. ^Sn
4. (a) n439In (b) 13 N (c) “4No (d) 257Tr
103 14. Three neutrons 15. 235 MeV
(a) 162 16. 4.61 x 1013 mol H atoms s"1.
75',Re —> 1578 Ta+^He
17. (a) nuclear fusion (b) carbon-cycle (c) H.A. Bethe
(b) 72Sm + _;e _> i38pm
(d) nuclear fusion (e) cyclotron (f) E.O. Lawrence
(0 W—> ’“Re+je (g) neutron
18. See Text
(d) ^3 Ta —> 167®Hf + °e
19. half-life - 10 days, 5 cps (2.5 cps is zero error of the G.M.
6. 9Be(a,n) 126C 7. fjAl+J H--- > 10
26A1+,2H
1
counter.)
8. (i)$Ca + 9n — -» 3jAr+2He 20. 147N(a,n)197F —>
n5B(p, n)“ C —■>
(ii) gMg+2H---- > 22Na+2He
33Na (n, y) ^Na ——» !224Mg+ _?e
(iii) ’H+*He —> fLi+'n
21. C<A<B.
(iv) ’7997 Au +9n —> 198 Au + y
Total Practice Set
(Read, Plan & Solve'.)

Problem 1. Calculate the nuclear binding energy per 34.56 x IO30 J day"1 of energy is obtained from
nucleon of C from the following data: 4 34.56 x IO30
Given mass of proton (mp) = 1.00758 amu —________ x---------------- gH
6.02 X 1023 43.2 xlO"13
mass of neutron (mn) = 1.00893 amu
mass of electron (mc) = 0.00055 amu = 53.16x 1018 g
Isotopic mass of 12C= 12.00000 amu. Take
1 amu = 931 MeV Problem 3. A chemist prepares 1.00 g of pure
Plan From the mass defect Am (amu), This isotope has half-life of 21 min, decaying by equation:
binding energy = Am x 931 MeV n6C—> “B+1°e
Am x 931 (a) What is rate of disintegration per second (dps) at
binding energy per nucleon = start?
number of nucleons
(b) How much of this isotope (^C) is left after 24 h of its
Solution ’gC contains 6 protons, 6 neutrons and preparation?
6 electrons hence, calculated atomic mass of - dN
Plan = AJV0, where No is the number of atoms of
= 6 mp + 6 mn + 6 me dt
at start in 1 g.
= 6 x 1.00758+ 6x 1.00893+ 6x 0.00055
. _ 2.303
= 12.10236 Use log —-
t N
mass defect = 12.10236- 12.00000
or C = C0
r y to calculate amount left after 24 h
= 0.10236 amu .2
binding energy = 0.10236 x 931= 95.30 MeV dN ,Ar
Solution (a) ------ = UV0
95 30 dt 0
binding energy per nucleon = —:— = 7.94 MeV 0.693
12 where A, (disintegration constant) = and No the
^50
Problem 2. The sun radiates energy at the rate of number of atoms present initially
4 x 1026 J s-1. If the energy of fusion process weight
4jH---- > *He + 2^6 *o = x Avogadro’s number
atomic weight
is 27 MeV, calculate amount of hydrogen that would be 0.693
dN ----- 1
consumed per day for the given process. —__ x__ X 6.02 x 1023 dps
dt 21x60 11
Plan Energy radiated by the sun per second is to be = 3x 1019 dps
converted per day. Energy of the fusion process is known,
hence, amount of JH can be calculated. 1 y
(b) Using equation: c = c0 2
Solution 27 MeV = 27 x 106 x 1.6 x 10"19 J
_ total time _ 24 x 60 min
= 43.2x IO'13 J where, = 68.57
TJ0 21 min
and 4x 1026 Js = 4x 1026 x 24x 3600 J day-1
Co = initial amount = 1 g
= 34.56 x 103° J day-1 C = amount of radioactive substance left after jr
43.2 x IO"13 J of energy is obtained from half-lives
= 4 amu of H 1' 68.57
C= 1
4 2.
gofH
"6.02x 1023 Taking log and then antilog, C = 2.28 x IO 21 g (n6C).
Chapter 5 : Radioactivity I 233
Problem 4. Calculate the weight in grams of 1 Ci of [U] - 3x 10-7
= 12.6x 10- 10 mol
6 238
14 C ('fso = 5000 yr), (1 yr = 31.536 X 10 S).
[
dN w 2.79x 10-10 ]
Plan + 1 =(2),
12.6x 10-10
- = W = A A No
dt
where w is the ·weight of the element of mass number A;
1.2214 = (2)Y
N0 is the Avogadro's number.
taking log, we have y = 0.2886
Solution Let weight required = w g
but y= total number ofhalf-life= _t_
= !!!_ x 6.02x 1a23 atoms Tso
14
t=yx 7;0
0.693
Dism
.. tegrat'1On constant ('\ ---------,,- s = 0.2886x 4.5x 109
-1
I\, )
6
5000 X 31.536 X 10
= 1.30 X 109 yr
and 1 Ci= 3.7x 1a1° dps
-- dN =
W
Problem 7. 1 g of radium produces 0.043 cm 3 of
dt helium gas at STP in one year. The number ofex-particles
produced in the same period is 11.6x 1017• Calculate the
3_ 7 X lOlO = 0.693 X W X 6.02 X la2
3
value ofAvogadro's number.
5000x 31.536x 106 x 14
Pla,i One mole = 22400 cm3 at STP = N0
w = 0.196 g
Thus, from the volume of the gas and number ofparticles,
Problem 5. How many mL of radon (Rn) under NO can be determined.
standard condition of temperature and pressure are in Solution 88 Ra � 86 Rn + �He
equilibrium with 1 g ofradium (Ra). Given half-life period
of 226 Ra= 1590 yr and for 222Rn= 3.82 days. thus, 0.043 cm 3 of helium gas at STP
= ll.6x 1017 ex-particles
Platt At radioactive equilibrium:
ll.Gx la17
N 22400 cm 3 ofhelium at STP = x 22400
N1A 1 = N2 A 2 :. _l = A'\ 2 0.043
N2 11,1
NO = 6.043 x Io23 mo1-1
Solution N1(Rn) A. 2 (Ra) _ Tso (Rn)
=
N2 (Ra) l 1(Rn) Tso (Ra) Problem 8. The curie was originally intended to be
N1(Rn) 3.82 days the activity ofone gram ofradium -226, [½o = 1620 a, here
(1/226) mol 365 x 1590 days 'a' stands for annum (year) in SI unit]. Calculate actual
weight ofradium which is equivalent to one curie.
N\(Rn)= 2.91x 10-8 mol given 1 a= 3.1536x 107 s
mL ofRn at STP = 6.52x 10 -4 mL dN - l\T - w
Plan - - = - JJ.v = - A -A No
dt
Problem 6. Estimate the age ofa rock which is found Thus, w (mass of the radioactive substance) can be
to contain 5 x 10-5 mL ofhelium at STP and 3 x 10-7 g of calculated.
uranium. (T,;0 of 23:2 U is 4.51x 109 yr and each 23:2 U is Solution 11,'\ = --
0.693 0.693
converted into 20;2 Pb). = -------7 8-1
1620 1620 X 3.1536 X 10
Pla,i 2
iiu--+ 2�;Pb +Bi3 +Sex(helium) but by definition, 1 Ci (curie)= 3. 7 x 1a1° dps
We take the equation [[� +
]
1] = (2)>
dN
--=W
dt
0.693
Where [Pb] is. the mol arratio,
- . y=-t 3.7 X 1a10 - N
[U] Tso 162 0x 3.1536x 107
N = 2.72766 x 1021 atoms
Solution Number ofmoles ofHe at STP
2.72766x 1021
5x 10-5 moles ofRa-226
---mo1 6.02x 102a
22400
= 0.004531
1 5x 10-5
[Pb]=- [He]=--- mol amount in g = molx 226
8 Sx 22400
= 2.79 x 10- 10 mol = 1.024 g
234 | Essential Inorganic Chemistry

Problem 9. The half-life of 32 P is 14.3 days. Calculate Plan From the mass defect (Am)
the specific activity of a specimen of phosphorus energy (in MeV) = (Am in amu x 931)
containing 1.00 part per million 32P. (atomic wt. ofP= 31)
Solution Mass defect
Plan Determine rate of disintegration per second (dps)
for the given isotope. Activity of 1 curie = 3.7 x 1010 dps. Am = (m(Cl) + m(n)]- [m(S) +m(H)]
Thus, activity in the given isotope can be calculated. = [34.9688+ 1.0087]- [34.9690+ 1.0078]
Solution The specific activity of a radionuclide is its = 0.0007 amu
activity per kg of the radioactive material. energy released = 0.0007 x 931
1 kg = 1000 g31P =^^ No atoms of31P
= 0.6517 MeV
31
But 32P content is 1 ppm (1 atm 32P out of 106 atoms) Problem 12. Calculate the maximum energy of a
32p_100QNo x 1 _ 1000x 6.02 x 1023 P-particle emitted from a 14 C nucleus which does not emit
X any y-radiation. The masses of C-14 and N-14 isotopes
31 X 106 31x 106
are 14.003242 amu and 14.003074 amu respectively.
= 1.942 x 1019 atoms of32P 64C---- >174N+_1e°
also ^ = XN,
dt
p
0.693^
^50
7
Plan In solving the above problem, the mass of emitted
^-particle is not taken into account. This is because the
dN _ 0.693 atom N-14 produced contains one more extra nuclear
x 1.942 x 1019 dps electron than the C-14 atom. But when a positron is
' dt “14.3x 24 x 3600
emitted two electron masses have to be taken into account
= 1.089 x 1013 s'1. (i) one for the positron emitted and
hence, specific activity (number of curies in 1 kg of 32P) (ii) one for the extra electron that has to leave the extra
1.089 x 1013 nuclear shells to maintain electrical neutrality. Thus
= 294.32 Ci kg in the reaction :
” 3.7 x 1010
gMg —> nNa+je0
Problem 10. An archaeological specimen containing Am =[mass of (Mg - 23)]- [mass of (Na - 23)
14 C gives 40 counts in 5 min per gram of carbon. A
specimen of freshly cut wood gives 20.3 counts per gram of + 2 mass of electron]
carbon per minute. The counter used recorded a Solution Am =m(C-14) - m(N-14) = 0.000168 amu
background count of 5 counts per minute in absence of any
14 C-containing sample. What is the age of the specimen? energy released in the process = 0.000168 x 931
[T50 of14C = 5668a] = 0.1564 MeV
Plan Initial and final activities (in terms of counts per This is maximum energy of p-particle.
min) have been given. Net value are determined (since
instruments has zero-error). Thus, age can be determined. Problem 13. A sample of |gSr originally had an
Solution Counter has zero-error of 5 counts per min activity of 0.500 mCi (millicurie) (a) What is the specific
(cpm) activity of the sample? (b) What is the activity of the
sample after 30.0 years? T30 of Sr is 19.9 years.
Initial activity = 20.3 - 5 = 15.3 cpm
f40 A Plan Initial activity (No) is known, hence activity (N)
Activity after time t = I — - 5 | = 3.0 cpm after time t can be calculated using
2.303. No
2.303 15.3 t =------ log —-
X= log t N
t 3.0
0.693 2.303
2.303.
------ =------- log 15.3 Solution The specific activity of a radio-nuclide is its
log ------
5668 t 3.0 activity per kg of the radioactive material.
Hence, first we calculate the weight of H Sr which has an
t = 13328 yr
activity of 0.500 mCi
Problem 11. Calculate the energy released in the
reaction : dt A
®?C1 + in---- » ?65S+,H!i ImCi = 3.7 x 107 dps
[(Cl-35) = 34.9688 amu, (S-35) = 34.9690 amu, 0.500 mCi = 0.5 x 3.7 x 107 dps
(H-l) = 1.0078 amu, Jn = 1.0087 amu]. Assume
1 amu = 931 MeV
Chapter 5 : Radioactivity I 235
0.693 X W X 6.02 X 1023 formed in the earth's crust? (The half-life of 238
U is
0.5 X 3.7 X 107 = ----------
19.9 x365x24x60x60x90 4.5x 109 years).
[Pb
w =2.5 x10� g = 2.5x10-9 kg Plan Use 1+ ]= 21 where y = __ t .
7 [U] Tr;o
SpeCl.6C act·lVl·ty = 0.5 x 3. 7 x 10
2.5 X 10-9 Solution If 0.214 g of206 Pb is present, then 1 g of 238 U
is also present.

(�r
= 7.4 x 1015 dis/kg-s
0·214 1
[Pb] = = 0.00104 mol, [U] =- - = 0.0042 mol
(b) using N=N0 206 238
[Pb]
_t 30·0 .. • 0.247
where y= = =1.5075 [U]
7;0 1 9 .9
For 'rock �ating' analysis, we use
1
-)
,1 .5075
N =0.5 ( mCi [Pb1 where y = _t_
\2 [1 + ] =2Y
[U] Tr;o.
taking log log N = log 0.5 -1.5075 log 2
[1+ 0.247]= (2) Y
= - 0.3010-1.5075 X 0.3010
Taking log, we get y = 0.3188
=-0.7548
_t = 0.3188
N =0.176 mCi Tso
=0.176x3.7 x 10 dps 7
t = 0.3188x Tso
= 0.3188x 4.5x 109
6
= 6.51 x 10 dps

Problem 14. In order to determine the volume of = 1.4344x 109 years


blood in an animal without killing it, a 1.00 mL sample of
an aqueous solution containing tritium is injected into the Problem 16. Given 2.00":2U kg of
animal's blood-stream. The sample injected has an (7;0 = 4.50x 10 years). The ultimate decay product in
9

activity of 1.8 x 106 cps (counts per second). After this series is i�Pb. (a) What weight of 23 U is left after :i
sufficient time for the sample to be completely mixed with 3.00x 109 years? (b) What weight of 2°;2Pb is produced in
the animals blood due to normal blood circulation, 2.00 mL this time?
of blood are withdrawn from animal, and· the activity of ,,v
the blood sample withdrawn is found to be 1.2x Hf cps.
Calculate volume of animal's blood.
Plan Use C= C0
2
l J
where C0 is the initial amount,

C after time t and y = �


Plan Compare the activity with volume of the blood '-50
sample to calculate volume of animal blood.
:2 U --. ':2 Pb + �(a) +GCP�
23
Solution (a) 2
Solution Let volume ofthe blood in the animal = V mL
particles
Total activity before mixing= 1.8 x106 cps . l' 3.00 X 109
2)
1
. l.2x Hf taking C = C0 ( where y = 0.667
. - 4_5x 109
· VmL= ---x V
To tal act1vi·ty after mixing m
2 0.667
1
=0.6x 1(}4 V cps C=2 ( )
2
Thus, 0.6 x Hf V = 1.8 x106
taking log on both sides
(assuming activity remains constant)
or log C = log 2- 0.667 log 2
V = 1.8 x 10 = 300 mL
6
= 0.3010- 0.666x 0.3010= 0.1003
0.6x1()4
C=l.26kg
Problem 15. A sample of uraninite, a uranium (b) 92 U disintegrated = 2.00- 1.26
containing mineral, was found on analysis to contain = 0.74 kg= 740 g
0.214 g of lead-206 for every g uranium. If all the lead 740
came from the radioactive disintegration of the uranium = = 3.109 mol
and assume that all isotopes of uranium other than 238 U 238
can be neglected, estimate the date when the mineral was
236 | Essential Inorganic Chemistry

Pb formed = 3.109 mol Plan Energy = Am x c2, 1 MW = 106 W = 106 J s"1


= 3.109 x 206 = 640.5 g Thus, energy in MW can be calulated.
= 0.6405 kg Solution Am = mass decay =m(f Li) +m(2H)- 2zn(2He)
Problem 17. A sample of14 CO2 was to be mixed with = (6.01512+ 2.01410)- 2x 4.0026
ordinary CO2 for a biological tracer experiment. In order = 0.02402 u
that 10 cm3 of the diluted gas at STP should have thus, energy of one atomic event (one mass decay)
104 dis/min, how many jiCi of radioacth - carbou-14 are
needed to prepare 0.06 m3 of the diluted gas ? = 0.0240 x 931 x 106 eV
Plan Given volume is to be related to the dps (activity) = 0.0240 x 931 x 106 x 1.6 x 10r19 j
and thus, number of microcuries (pCi) can be calculated. = 3.59x 10"12 J
Solution 10 cm3 of the diluted gas has activity energy per mol of LiH = 3.59 x 10"12 x 6.02x 1023
io4 = 2.16 x 1012 J mol"1
= 104 dis/min =----- dps
60 2.16 x 1012
energy per g of LiH = Jg
0.06 m3 ( = 60,000 cm3) of the diluted gas has 8
u 104 60000 , 2.16x 1012
activity=— x----- - dps J g"1 s
60 10 energy per g of LiH per second =
60000 8x 24 x 3600
hence, number of |iCi needed = 104 x
600 x 3.7 x 104 = 3.125 x 106 Js^g"1
(IpCi = 3.7 x 104 dps) = 3.125 x 106 Wg 1 (Js-1 = lW)
= 27.03 pCi
= 3.125 MW g"1 (1MW = 1O6 W)
Problem 18. The half-life of 212Pb is 10.6 h, that of its
daughter 212 Bi is 60.5 min. How long will it take for a Problem 20. Tritium, or hydrogen-3, is formed in the
maximum daughter activity to grow in freshly separated upper atmosphere by cosmic rays, similar to the formation
212Pb ? of C-14. Tritium has been used to determine the age of
Plan We define time of maximum activity by equation wines. A certain wine that has been aged in a bottle has a
2.303 . X, tritium content only 78% of that in a similar wine of the
^max ----------- log — same mass that has just been bottled. How long has the
aged wine been in the bottle ? T50 of tritium is 12.2 years.
where Xd is the disintegration constant of daughter 2.303. No , . 0.693
element and Xp that of parent element. Plan Use X = ------ log —- and X =
t N T50
o j . 0.693 0.693 Thus, time t can be calculated.
d T50 (212Bi) 60.5 min
. _ 2.303 1
Solution log —-
= 0.01145 min"1 t N
0.693 _ 0.693 0.693 2.303 1
■log —
T50 (212Pb)~ 10.6x60 ^50 t N
= 0.00109 min 0.693 2.303 . 100
log----
2.303 Iq 0.01145 12.2 t 78
^max
"(0.01145-0.00109) °g .0.00109
t = 4.375 years
= 227 min
Problem 21. When a positron and an electron collide,
Problem 19. An isotopic species of lithium hydride they are annihilated and two gamma photons of equal
6Li2H is a potential nuclear fuel, on the basis of the energy are emitted. Calculate the wavelength
reaction corresponding to this gamma emission:
fLi + ---- > 24He
Plan Energy per photon = mec2
Calculate the expected power production in MW,
associated with 1.00 g of 6Li2H per day (process is 100% E = ftv=*£
efficient). X
[fLi = 6.01512 u,?H = 2.01410 u , 2He= 4.00260 u.], u is Thus, X can be calculated.
amu and 1 u = 931 MeV
Chapter 5 : Radioactivity I 237
0
Solution 1e + _1e 0 �2 gamma photons of equal Plan The problem asks us to calculate the mass defect L\m
when the energy change L\E is known. To do this, we have to
energy rearrange the Einstein equation to solve for mass,
The energy of the above process is remembering that l J=1 kg m 2 /s2 •
E =2 me c2 Solution L\E = L\mc2
31 8 1)2
= 2 (9.llx 10- kg) (3x 10 m s-
E - 436 kJ
= 1.6398 X
10-Ia J L\m = L\ i =
c (3x 10 8 )2 m 2 s-2
energy of one photon= 8.199x 10-14 J
- 436x 10 3 J
c
takingE= � 9x la1 6 m2 s-2
6.62x 10-34 x x 10 8 _ - 436x 103 kg m2 s-2
..A -
=
he ----
= -3�- m -
9x la16 m 2 s-2
E 8.199x 10- 14
= 2.42 x 10-12 m = 2.42 pm =- 4.84 X 10-12 kg
Problem 22. Natural nitrogen is 99.63% 14
N with Problem 24. Calculate the volume of:H,z gas at STP
mass 14.00307 amu and 0.37% 15 N, with mass number which would be produced per minute from radiolysis of
15.00011 amu. Which isotope is more stable, 14 N or 15 N?
water in a 1000 kW reactor, assuming that 100 eV of
PIau . . L\m (amu) x 931 energy produces 1 molecule of hydrogen
Binding energy per nucleon = -------
number of nucleons
Isotope with larger binding energy per nucleon is more
2 HiO � I¼ + 'H.i02 {I eV= 1.6x 10- 19 J)
1
stable. Plan Energy given ink Wis to be converted into J s- and
thus, volume of Hi gas produced in this reactor can be
Solution There are 7n, 7p and 7 e- in 14 N calculated.
mass of nucleons = 7x 1.00782505+ 7x l.008664904
Solution Given 1 eV= l.6x 10-19 J
= 14.11543 amu
100 eV = l00x 1.6x 10- 19 J
mass defect = 14.11543- 14.00307
= 1.6 X 10-17 J
= 0.11236 amu
x 931 Also 1000 kW= l000x 1000 W
b.md"mg energy per nucI eon 0.11236
----- MeV
14 = lOOOx 1000 J s-1
= 7.472 MeV = 10
6
J s- 1
There are Bn, 7p and 7 e- in 15 N = 106 x 60 J in one minute
mass of nucleons= 8x 1.008664904+ 7x 1.00782505 (as required)
= 15.12409 amu 1.6 x 10- 17 J produces = 1 molecule of Hi
mass defect= 15.12409- 15.00011 60x 10 6
60 x 10 6 J produces - molecules
= 0.12398 amu l.6x 10-17
b.md'mg energy per nucleon= 0.12398x
----- MeV
931
= 37.Sx 1()23 molecules
15
37.5x lo"
= 7.695 MeV = mol Hi
6.02x 1a23
As binding energy per nucleon is larger for 15N,
hence, 15 N is more stable. = 6.23 mol Hi
1 mol at STP has volume= 22.4 L
Problem 23. W hat is the change in mass ( in grams)
when 2 mol of hydrogen atoms combine to form 1 mol of 6.23 mol at STP has volume= 139.55 L
hydrogen molecule?
L\E =- 436 kJ
L... z:z3
Master Exercises
K3E —

Exercise 1
(Stage 1: Learning)
Analytical Questions vessel, how much He will accumulate in 20 days at
STP?
Theory of Radioactive Disintegration
12. The disintegration rate of a certain radioactive
1. Half-life of 221Fr is 4.8 min. If 1 mg of the isotope is sample at any instant is 4750 dpm (disintegration
isolated, how much will remain after (a) 19.2 min per minute). Five minutes later, the rate becomes
(b) 48 min? 2700 dpm. Calculate half-life of sample.
2. 0.1 mg of 239Pu emits 1.4x 107 particles per minute. 13. Chlorine-34 has a half-life of only 1.53 s. How long
What is its half-life? does it take for 99.99% of a 34Cl sample to decay?
3. A radioactive isotope has a half-life of 14.8 hr. Nuclear stability and Nuclear Reactions
How much time it will take for the activity to drop to
(a) 10 per cent, (b) 5 per cent of its original value? 14. Calculate the number of a and P-particles in the
change:
4. Radioactive equilibrium is established in a uranium
mineral between radium and uranium atoms in the 23|U —> 2087Pb
ratio 1:28. If half-life period of radium is 1620 years, 15. 1 g of 19gAu(7g0 = 65 h.) decays by p-emission to
calculate the disintegration constant of uranium. produce stable Hg.
5. 1 g of 226Ra emits 11.6 x 1017 a-particles in a year. If (a) Write nuclear reaction for process.
X = 4.353 x 10-4 year-1, calculate the value of (b) How much Hg will be present after 260 h?
Avogadro’s number. 16. 928 U is radioactive and it emits a and [3-particles to
6. Calculate the mass of 14 C (T50 = 5720 years) atom form 2°82pb. Calculate the number of a and p-particles
emitted in this conversion. An ore of 2392U is found to
which has radioactivity of 1 mCi.
contain g28U and |26Pb in the weight ratio of 1:0.1.
7. The half-life of 40K is 1.28 x 109 years. What mass of The half-life period of |28U is 4.5 x 109 years.
this nuclide has an activity of l.OOjiCi ? Calculate the age of the ore.
8. 2&4Po decays with a-particle to 288Pb with a half-life 17. (a) The super heavy element 287 Uuq was made by
of 138.4 days. If 1.0 g of 2^Po is placed in a sealed firing a beam of 40 Ca ions at 240 Pu. Three
tube how much of helium will accumulate in 69.2 neutrons were ejected in the reaction. Write a
days? Express the answer in cm3 at STP. balanced nuclear equation for the synthesis of
287Uuq.
9. 10 g atoms of an a-active radioactive isotope are
(b) Boron is an effective absorber of neutrons. When
disintegrating in a sealed container. In one hour, the
He gas collected at STP is 11.2 cm3. Calculate boron-10 adds a neutron, ana-particle is emitted.
half-life of radioactive isotope. Write an equation for this nuclear reaction.
10. 0.1 g atom of radioactive isotope £ X (half-life 5 days) Binding Energy
is taken. How many number of atoms will decay
18. Calculate the mass defect and binding energy in
during eleventh day.
(a) 105B (isotopic mass = 10.01019 amu) and
11. A certain radioisotope gXdso = 10 days) decays to
(b) 3IA (isotopic mass = 7.01436 amu).
give z^Y. If one g atom of is kept in a sealed
Chapter 5 : Radioactivity | 239

19. The alpha-particles emitted by radium have energies 2. The atomic mass of 16 O is 15.995 mu while the
of4.795 MeV and 4.611 MeV. What is the wavelength individual masses of proton and neutron are 1.0073
of the gamma ray associated with the decay? and 1.0087mu. The mass of electron = 0.000548mu.
20. What is the wavelength (in nm) of gamma-rays Calculate the binding energy of the oxygen nucleus in
whose energy is 1.50 eV? joules.
21. Calculate the mass defect (in g/mol) and the binding 3. The isotopic masses of 2H and 2 He are 2.0141 and
energy (in MeV/nucleon) for the following nuclides. 4.0026 mu respectively and the velocity of light in
Which of the two is more stable? vacuum is 2.998 x 108 ms"1.Calculate the quantity of
(a) 50Cr (atomic mass = 49.94605 amu) energy (in J) liberated when two moles of 2H
(b) 64 Zn (atomic mass= 63.92915 amu) undergoes fusion to form one mole of 2 He.
4. The radioactive isotope 89 Co which has now replaced
Applications
radius in the treatment of cancer can be made by a
22. A sample of uranium ore was found to contain 0.28 g
one_ - (n, p) or(n, y) reaction. For each reaction, indicate the
206 Pb and 1.67 g of 238 U. Assuming that all Pb came appropriate target nucleus. If the half-life of 27 Co is
from the decay of 238 U, what is the age of the ore ? 7 yr, evaluate the decay constant in s’1.
(Given T50 of238 U is 4.51 x 109 years).
5. A piece of wood from an archaeological source shows
23. A sample of 238 U (half-life = 4.5 x 109 years) ore is a 14 C activity which is 60% of the activity found in
found to contain 23.8 g of 238 U and 20.6 g of 206Pb. fresh wood today. Calculate the age of the
Calculate the age of the ore. archaeological sample. (^ = C = 5770 yr)
6. Calculate the mass of 140 La in a sample whose
NCERT Problems activity is 3.7 x IO10 Bq (1 becquerel, Bq = 1
1. What is meant by nuclear binding energy? Calculate disintegration per second) given that its ^/2 is 40 hr.
the binding energy per nucleon of Li isotope, which 7. Calculate the binding energy per nucleon for12C,14N,
has the isotopic mass of 7.016 mu. The individual 16O, and comment on their relative magnitudes.
masses of neutron and proton are 1.008665 mu and Masses of proton and neutron are 1.0078 and 1.0087
1.007277 mu, respectively and the mass of electron mu respectively. (1 mu = 931 MeV)
= 0.000548 mu.

Exercise 2
z (Stage 2: High Skill Problem Solving)
Only One Option Correct 3. 13 Al is a stable isotope. 13 Al is expected to
1. In the chain reaction disintegrate by
(a) a-emission (b) 0-emission
292 U---- > Ba + Kr + 3on + energyE
(c) positron emission (d) neutron emission
neutrons and energy produced at nth step will be: 238
4. The number of aand0 particles lost when 92 U
(a) 3n, nE (b) 3n , nE
(c) 3,l,3n“1£ (d) S'1-1, nE changes to 2s26Pb are
(a) 8a, 60 (b) 6a, 80
2. A radioactive element decays by the sequence, and (c) 6a,60 (d) 8a,80
with half-lives, given below :
5. Assuming that only particles emitted from atoms
X — * Y —Z during natural radioactive decay are a and
30 min 2 days
0-particles, which of the following atoms could not
Which of the following statements about this system possibly result from the natural decay of
are correct ? (a) 283<}Ac (b) 28297 Ac
(a) After two hours, less than 10% of the initial X is
left (c) 28295Ac (d)28°JPb
(b) Maximum amount of Y present at any time before 6. Of the following types of radiations, the only one to be
30 min is less than 50% of the initial amount ofX deflected in a magnetic field is
(c) Atomic number of X and Z are same (a) X-ray (b) y-ray
(d) All of the above (c) 0-ray (d) neutrons
240 | Essential Inorganic Chemistry

7. A process that produces a one-unit increase in atomic 18. Consider the following nuclear reactions
number is
(a) electron capture (b) p"-emission
1. 14 N +tHe * VO+iH
(c) a-emission (d) y-ray emission 2. 94Be +JH ----- > fLi+lHe
Be+jH
8. Of the following nuclides, the one most likely to be 3. +|He---- > gSi+Jn
radioactive is 4. ^B+jHe---- > ^N+Jn
(a) ?5‘P (b) f§Zn Examples of induced radioactivity would include
(c) 14,N (d) ‘64C reactions:
9. One of the following elements has eight naturally (a) 3 and 4 (b) 1 and 2
occurring stable isotopes (c) 1, 3 and 4 (d) 1, 2, 3 and 4
(a) Ra (b) Au 19. Match list I with list II and select the correct answer,
(c) Cd (d) Br
using the codes given :
10. Of the following nuclides, the one most likely to decay
by positron (p+) emission is List! List II
(a) 59Cu (b) 63Cu A. "V 1. Location of tumour in brain
(c) 67 Cu (d) 68 Cu B. 24 Na 2. Location of blood clots and
11. Among the following nuclides, the highest binding circulatory disorders
energy per nucleon is found for C. “Co 3. Radio-therapy
131|
(a) ,H (b) ‘86O D. 4. Agriculture research

(c) |Fe (d) 29325U Codes


12. The most radioactive of the isotopes of an element is A B C D
the one with the largest value of its (a) 4 1 2 3
(a) half-life (b) neutron number (b) 4 3 2 1
(c) atomic number (d) decay constant (0 4 2 3 1
13. The radiation that produces the greatest number of (d) 3 1 2 4
ions as it passes through matter is 20. Aluminium-25 decays by emitting a positron. The
(a) a (b) y
species immediately produced has
(0 p- (d) p+
(a) 12p, 13n, 13e" (b) 13p, 12n, 13 e’
14. The type of radiation that has the greatest (c) 12p, 13n, 12e~ (d) 14p, Un, 14e“
penetrating power through matter is
(a) a (b) y 21. The half-life of 99 Tc is 6.0 h. Hence, average life is
(c) p" (d) visible light (a) 4.17 h (b) 3.0 h
15. In uranium mineral, the atomic ratio Nu_238 1 NPb_206 (c) 8.66 h (d) 8.00 h
is nearly equal to one. The age (in years) of the 22. Tritium, 3H has a half-life of 12.26 years. A 5.00 mL
mineral is nearly (half-life period of U-238 is sample of tritiated water has an activity of
4.5 x 109 years) 2.40 x 109 cpm. How many years will it take for the
(a) 3.0 x 103 (b) 4.5 x 109 activity to fall to 3.00 x 108 cpm?
(c) 3.0 x 109 (d) 4.5 x 108
(a) 6.13 (b) 12.26
16. For an a-emitting isotope, the value of disintegration (c) 24.52 (d) 36.78
constant is 0.49 x 10-1° per year. The amount of the
isotope of a given sample will reduce to half its value 23. In a series of three steps in radioactive disintegration
after a period (in years) of nearly sequence starting with 2288 Ra, the particles emitted
(a) 0.45 x 1010 (b) 0.9 x 1010 are, successively, P~, p" and a-particles. The
(c) 1.41 x 1010 (d) 2.82 x 1010 resulting product is an isotope of
17. In nuclear reaction (a) 92U (b) 90 Th
3Li + }H---- > 21He (c) 88Ra (d) 86Rn
the mass loss is nearly 0.02 amu. Hence, the energy 24. 1 g of 27°9 Au (T50 = 13 days) emits a P-particle forming
released (in units of million kcal/mol) in the process is
approximately a stable Hg atom. Hg formed at the end of 52 days
(a) 430 (b) 220 (a) 0.0625 g (b) 0.9375 g
(c) 120 (d) 50 (c) 0.7500 g (d) 0.2500 g
Chapter 5: Radioactivity I 241
25. At radioactive equilibrium, the ratio betweeen the 32. In the esterification of an organic acid R-COOH by
atoms of two radioactive elements A and B was found alcohol R' OH, 0 of acid is isotopic.
to be3.1 x 10 9 : 1 respectively. If 7],0 of the element A 0 0
is 2 x 1010 yrs, then Tso of the element B is 1s II I
(a) 6.2x 109 yr
R-C-OH +R'-0-H � R-C-OR' + H2 0
(b) 6.45 yr
(c) 2 x 10 10 yr (d) 3 .1x 10 9 yr Which of the following statement is correct?
(a) 18 0 is in ester
26. Weight of 14 C to have radioactivity 1 curie (b)
18
0 is in water
[1.. (disintegration constant)= 4.4x 10- 12 s- 1 ]is (c)
18
0 is in both
(a) 2x 10- 4 kg (b) 0.9x 10-4 kg (d) 18 0 is in none
(c) l.7x 10 -4 kg (d) 3.7x 10- 10 kg 33. Thiosulphate ion, S2 01- on acidification changes to
27. If half-life period is 100 yr, average life is nearly S().z along with precipitation of sulphur
(a) 100 yr (b) 70 yr S S01- + 2H+ � H2O + S().z + S
35 32

(c) 144 yr (d) 90 yr Which is correct statement?


35
28. There are0.618µg or 206 Pb and 0.238µg of 238 Uin a (a) 35 S is in sulphur (b) S is in S().z
35
(c) S is in both (d) 35 S is in none
rock. If Tso of 238 Uis 1.5 x 10 9 yr, age of the rock is
34. Water used as moderator in nuclear reactor is called
(a) l.5x 10 9 yr (b) 3.0x 10 9 yr (a) heavy water (b) hard water
(c) 4.5x 10 9 yr (d) 0.75x 10 9 yr (c) nuclear water (d) critical water
2 9. A kA> product, B k
B· product 35. 1
IN+tn � 3C+fH is written as
1
TA TB
(a) 1jN (n ,e) }H (b) 1/N (p .n) 13C
A and B are two radioactive elements with half-life (n ,p) 1:c
(c) 1#N (d) 1:c (p ,n.) 1:N
periods TA and T8 (in years) and kA (yr-1) and
36. � U (III B) emits two a-particles; new element will be
k8 (atom- 1 year- 1). If half-life periods are equal,
disintegration rate at the start of disintegration placed in
would be (a) VII B (b) VII A
(a) kA TA (c) II A (d) II B
(b) 0.693
(c) both (a) and (b) (d) None of these 37. After the em1ss1on of a �-particle followed by
30. Whi ch of the following nuclear reactions is an a-particle from �4 Bi, the number of neutrons in the
example of nuclear fusion? atom is
130 (b) 129
(a) ! C+tH � 1/N+ y
2 (a)
(c) 128 (d) 127
(b) �N+tn � 1JC+tH 38. Which of the following are fissile isotopes?
( c) iH+fH � iHe+�n I. 2
tiu . 11. �iu
(d) 21:u +in� tJBa+�!Kr+ ain
1
III. 2 Pu t� IV. �� U
31. Match list I with list II and select the correct answer: Select the correct answer from the following
(a) I and II (b) II and III
List I list 11 (c) I and IV (d) All of these
(Isotope) (Characteristic) 23
39. Na is the more stable isotope of Na. t1Na can
A. :ca 1. unstable, a-emitter
undergo radioactive decay by
B. fl 2. unstable, �-emitter (a) � - -emission
c. �,, 3. unstable, positron emitter (b) a-emission
(c) � + -emission
D. �Th 4. stable
(d) K-electron capture

A B C D 40. The missing fission product in the reaction


(a) 1 2 3 4 �iu +tn � 1�La+ ...+ 3&n is
(b) 1 3 2 4 (a) f�Br (b) �Br
(c) 4 3 2 1
(d) 4 2 3 1
(c) �gBr (d) g�Ge
242 | Essential Inorganic Chemistry

41. Which is the correct combination? 48. Beta-emission takes place


•Change in (a) from the elements above the band of stability
(b) when neutron is converted to proton
Atomic Mass Neutron (c) with shifting of the new element one grou.
Emission number number number
~~2~
towards right
(a) a -2 ~~-2
(d) following all the facts given above
(b) P’ +1 0
(c) 0 0 49. Mass defect of 1 g gives energy equal to
Y
(d) r 0 0 (a) 9x 1013 J
(b) 5.625 x 1032 eV
42. Match the radioisotopes with their applications from (c) both (a) and (b)
the alternates given. (d) None of the above
Radioisotopes Applications 50. Select correct alternate.
I. Cobalt-60 1. Leukemia therapy (a) ---- > qti is positron emission
II. Potassium-40 2. Thyroid therapy (b) |p---- > is K-electron capture
III. lodine-123 3. Geological dating (c) both (a) and (b)
IV. Phosphorus-32 4. Cancer therapy (d) None of the above
V. Carbon-14 5. Archaeological dating
One or More Than One Options Correct
I II HI IV V
1. Carbon-14 dating method is based on the fact that
(a) 1 2 3 4 5
(b) 5 4 3 2 1 (a) carbon-14 fraction is same in all objects
(c) 4 3 2 1 5 (b) carbon-14 is highly insoluble
(d) 4 3 2 5 1 (c) ratio of carbon-14 and carbon-12 remain;
43. Constituents of wine are carbon, hydrogen and constant during disintegration
oxygen. Half-lives of 13C, loO and 3H are respectively (d) carbon-14 is highly soluble
5730 years, 124 s and 12.5 years. A bottle of wine was 2. Select correct statement(s).
sealed about 6 years ago. To confirm its age, which of (a) 131I is used for the treatment of thyroid cancer
the isotopes would you choose to determine its age?
(b) 59 Co cannot be used for treatment of cancer
(a) ‘63C (b) 158O
(c) 33P is used for treatment of leukemia
(c) iH (d) Any of these
(d) excessive use of radioactive elements caus
44. Tat man’ relates to cancer
(a) Pubomb
(b) U bomb 3. Radioactive disintegration rate is affected by
(c) Th bomb (a) temperature (b) pressure
(d) literary word from a book (c) electric field (d) None of these
45. Tattle boy5 relates to 4. U (III B) undergoes follows emissions
(a) Pu bomb (b) U bomb
(c) H bomb (d) He bomb 23982U A B C
46. Thorium is found in Which is/are correct statements?
(a) pitchblende (b) uranite (a) A will be of IB group
(c) monazite (d) malachite green (b) A will be of IIIB group
47. Consider following statements (c) B will be of HA (alkaline earth metal) group
I. ‘Carbon-dating’ is based on the measurement of (d) C will be of IIIA (boron family) group
activity of14 C 5. Beta-emission takes place
II. 40K is used to determine age of the objects upto (a) from the elements above the band of stability
1 million years old (b) when neutron is converted to proton
III. The uranium-lead method is based on the natural (c) such that new element remains in the same grou;
uranium-238 decay series which ends up with the (d) such that atomic number of the element forme;
production of stable lead-206. increases by one unit
Select correct statements. 6. Mass defect of 1 g gives energy equal to
(a) I, II (b) II, HI (a) 9x 1013 J (b) 5.625 x 1032 eV
(c) I, III (d) I, II, III (c) 2.15 x IO10 kcal (d) 9x 106 ergs
Chapters: Radioactivity | 243

7. Target nucleus A is converted to product nucleus B by 6. What mass of14 C with T5Q = 5730 years has activity
(p, n) as: A( p, n) B equal to one curie?
In this case
(a) A and B are isotopes 7. What is the wavelength (in nm) of gamma-rays
(b) A and B are isobars whose energy is 1.50 eV?
(c) A and B are isotones 8. A positron has the same mass as an electron
(d) B has higher atomic number than that of A (9.109 x 10-31 kg) but an opposite charge. When the
8. In a nuclear reactor, oxides of which of the following two particles encounter each other, annihilation
metals are used as fuel material? occurs and only gamma-rays are produced. How
(a) Uranium (b) Thorium much energy (in kJ mol-1) is produced?
(c) Actinium (d) Plutonium 9. Radiolysis of water yields hydrogen according to the
9. Two radioactive substances X and Y have reaction:
disintegration constant in the ratio 2:1. If half-life of 2H2O---- > H2 + HA
Y is 4 days then after 12 days starting with equal
Calculate the volume of hydrogen produced per hour
mole fraction of each in mixture ofX and Y in a 200 kW reactor. The yield of hydrogen is
(a) mole fraction of X and Y remains unchanged 1.7 molecules per 100 eV of energy absorbed.
(b) mole fraction of X is larger than that of Y
(c) mole fraction of X is smaller than that of Y 10. The nuclide 2?Sc, mass 49.9516 amu, is neutron rich.
(d) half-life of X is smaller than that of Y It decays to form 22 Tb mass 49.94479 amu. If the
10. Select correct statement(s). emitted 0“ particle has a kinetic energy of 0.80 MeV,
what is the kinetic energy of the neutrino emitted
(a) Neutron-proton ratio after a nuclide, 298U loses
ana-particle is 1.6 simultaneously ?
(b) 13 Al can be converted to 3£P by (a, n) reaction 11. How much of heat would be developed per hour from
(c) Nuclear fusion reactions are known as 1 Ci 14 C source if all the energy of the P” decay were
thermonuclear reactions imprisoned ?
(d) Larger the value of disintegration constant,
greater the stability of radioactive element 12. In a thorium mineral the ratio by weight of
208Pb i232Th is 0.2. The decay constant of 232Th is
4.98 x 10"11 year-1. Calculate the age of mineral.
Brain Twisters
13. The activity of hair of an Egyptian mummy is
1. An atomic battery operated with watch uses 10 dismin"1 g-1 of 14 C. What is the age of the
p-particles from 6i7Pm (T50 =2.62 yr) as the energy mummy? 7£0 of14 C is 5730 years and disintegration
source. How long would it take for the rate of rate of fresh sample of14 C is 15 dis min-1 g-1.
p-emission in the battery to be reduced to 5% of its
14. 1 g of 14 C from an archaeological sample gives
original values?
9.4 counts min-1. 1 g of 14C from a recently cut tree
2. If 1 g of 226Ra emits 11.6 x 1017 a-particles in a year. If gives 15.3 counts min-1 under the same conditions.
X = 4.353 x 10"4 yr-1, calculate value of Avogadro’s How old is the archaeological sample? (T50 of 14 C is
number. 5730 years).
3. One of the hazards of nuclear explosion is the 15. Potassium-40 decays with a half-life of 1.3 x 109 yr.
generation of 90 Sr and its subsequent incorporation What is the age of lunar rock sample that contains
in bones. This nuclide has a half-life of 28.1 years. these isotopes in the ratio 34.1 atoms 40K : 65.9
Suppose one microgram (10-6 g) was absorbed by a atoms 40 Ar? Assume that no argon was originally in
new-born child, how much 90 Sr will remain in his the sample and that none has been lost by
bones after 20 years? weathering.
4. The activity of 30 pg of 247 Cm is 2.8 nCi (10 9 curie). 16. 292U by successive radioactive decays changes to
Calculate the disintegration rate constant and the 2g2Pb. A sample of uranium ore was analysed and
found to contain 1.0 g 238U and 0.1 g206Pb. Assuming
half-life of247 Cm.
that 206 Pb has accumulated due to decay of238 U, find
5. Upon irradiating californium with neutrons, a out the age of the ore. T5o of 238 U is 4.5 x 109 years.
scientist discovered a new nuclide having mass
17. As a result of being exposed to the radiation released
number of 250 and a half-life of 0.50 h. Three hours
during the Japan nuclear accident in very recent
after the irradiation, the observed radioactivity due
Sunami, the dose of iodine-131 in a person’s body is
to nuclide was 10 dis/min. How many atoms of the 7.4 mCi (1 mCi = lx 10-3 Ci). Calculate the number
nuclide were prepared initially?
of atoms of iodine-131 (T^o - 8*1 days).
244 | Essential Inorganic Chemistry

Passage Comprehension Questions 2. Select correct statements.


(a) Sr-90 and Cs-137 are the other by-products of
Passage 1 nuclear fission released through bomb testing
Back in 1953 the U.S. Army started an experimental (b) Sr-90 and Cs-137 can produce cancer of the bone
programme of food irradiation so that deployed troops (c) Sr-90 like Ca is incorporated into bone and thus
could have fresh food without refrigeration. The procedure produces bone-cancer
is a simple one. Food is exposed to high levels of radiation (d) Cs-137 through radioactive, is rapidly eliminated
to kill insects and harmful bacteria. It is then packaged in from the tissue being quite soluble like K
airtight containers, in which it can be stored for months
without deterioration. The radiation sources for most food Passage 3
preservation are cobalt-60 and cesium-137, both of which Read the following passage and answer the questions
are y-emitters, although X-rays and electron beams can at the end.
also be used to irradiate food. A person received an anonymous gift of a decorative
Read the above passage and answer the following cube which he placed on his desk. A few months later he
questions. became ill and died shortly afterward. After investigation,
the cause of his death was linked to the box. The box was
1. The benefits of food irradiations are air-tight and had no toxic chemicals on it. What might
(a) it reduces energy demand by eliminating the have killed the man?
need for refrigerations
(b) it prolongs shelf life of various foods 1. (a) a-rays emitting material
(c) both (a) and (b) are correct (b) p-rays emitting material
(d) None of the above is correct (c) y-rays emitting material
(d) KCN
2. There have been certain objections against food 2. Am-241 is used in smoke detectors because it has a
preservation by irradiations. These are: long half-life (458 y). Given the schematic diagram of
(a) irradiated food may itself become radioactive a smoke detector below, explain how it works?
(b) irradiation can destroy the nutrients such as Current
vitamins and amino acids
(c) ionising radiation produces reactive species such
as the hydroxyl radical which then react with
organic molecules to produce potentially harmful
substances.
(d) irradiated food may be much costlier
3. Select correct statement(s).
(a) A rad is a measure of the amount of high energy /
radiation absorbed by matter
(b) 1 rad of a-rays has the same effect on tissue as
1 rad of y-rays
'Am —Uh—
(c) Iodine-131 can be used for scanning the thyroid Battery
for disorders
(d) Strontium-90 is useful isotope for food radiation (a) it emits a-particles which are energetic enough to
ionise air molecules
Passage 2 (b) it emits p-particles which can complete the circuit
(c) it emits y-particles which have high penetration
In a series of extraordinary events, a natural nuclear
(d) it emits a- and 0-particles
fission reactor operated at the time when the first life
forms appeared on Earth. As is often the case in scientific
endeavors, humans are not necessarily the innovators but Passage 4
merely the imitators of nature. Radiolysis of water yields H2 as given below
Read the above passage and answer the following 2H2O---- > H2O2 + H2
questions. In the analysis it is found that 1.7 molecules ofH2 are
1. Nuclear-fission reaction is yielded per 100 eV of energy absorbed. A nuclear power
(a) chain reaction reactor of 200 kW capacity has been installed based on this
(b) induced by neutrons reaction.
(c) related to Pu-bomb Read the above passage and answer the following
(d) related to U-bomb questions.
Chapter 5: Radioactivity | 245

1. Above reaction is an example of 4. Select correct statement(s).


(a) disproportionation (a) Photon or particle is ionised and collected by the
(b) redox positive electrode in a Geiger counter
(c) decomposition (b) A scintillation counter contains a phosphor, a
(d) displacement substance that emits a flash of light when struck
by a photon or particle
2. What mass of heavy water is required to yield 448 kL (c) Both of the above
of heavy hydrogen at STP? (d) None of the above
(a) 996 kg (b) 448 kg
(c) 800 kg (d) 720 kg
Passage 6
3. Volume of H2 produced per minute in the above .
One of the products of radiation fall out from
reactor is atmospheric bomb testing or nuclear accidents as at
(a) 140 L (b) 70 L Chernobyl is iodine-131. People living in a region where
(c) 35 L (d) 28 L iodine-131 was known to have been deposited were
encouraged to use table salt enriched with non-radioactive
Passage 5 isotope of iodine.
To detect bombs that may be smuggled into airplanes, Read the above short write-up and answer the
the Federal Aviation Administration (FAA) will soon following questions.
require all major airports in the United States to install 1. Non-radioactive isotope prescribed in the table-salt is
thermal neutron analysers. The thermal neutron analyser (a) 1-130 (b) 1-127
will bombard baggage with low-energy neutrons, (c) 1-132 (d) 1-129
converting some of the nitrogen-14 nuclei to nitrogen-15, 2. The rate of iodine uptake by the thyroid is
with simultaneous emission of y-rays. Because nitrogen (a) dependent upon the iodine concentration
content is usually high in explosives, detection of a high (b) dependent upon pressure of iodine
dosage ofy-rays will suggest that a bomb may be present. (c) follows second-order kinetics
Read the above passage and answer the following (d) independent of iodine concentration
questions. 3. By use of non-radioactive iodine isotope in common
1. Nuclear process involves reaction salt
(a) rate of iodine-131 uptake is sharply reduced
(al^N+Jn---- > 75N compared to uptake of iodine-127
(b) “N+Jn ---- > 64C + ;H
■>
(b) a new non-radioactive isotope is formed
(c) taste of table salt becomes better
(c) 7^N —» JfC+fe (d) iodine-131 being water soluble is eliminated from
(d) “N---- > ^N+’Je tissue
2. Consider the following nuclear reactions
Passage 7
1. ^N+jHe---- > 87O+iH
Read the following figure in paragraph and answer
2. 4 Be +}H---- > J Li +lHe
the questions at the end of it.
3. laMg+jHe 4 ?JSi+o« A number of basic questions have probably occurred to
4. - 73N +ort you as we have been describing nuclear decay processes.
Examples of induced radioactivity would include Why do some radioactive nuclei decay by a-emission
reactions and some by -emission?
(a) 3 and 4 (b) 1 and 2
Why do the lighter elements have so few naturally
(c) 1, 3 and 4 (d) 1, 2, 3 and 4
occurring radioactive nuclides, whereas those of the
3. The sensitivity of the neutral activation analysis heavier elements all seem to be radioactive?
depends on the
Experimentally, it is found that nuclei with certain
(a) neutron flux available for irradiation
number of protons or neutrons appear to be very stable.
(b) availability of nucleus to absorb a neutron
These numbers called magic numbers and associated
(c) energy of decay process ■ with specially stable nuclei, were later explained by the
(d) All of the above are correct
shell model.
246 ] Essential Inorganic Chemistry
16Q

9.00 Assertion & Reason


Codes:
(a) Both A and R are true and R is the correct
5 7.00 explanation of A.
s (b) Both A and R are true but R is not the correct
explanation of A.
S (c) A is true but R is false.
? 5.00
c (d) A is false but R is true.
®
Q.
>. 1. Assertion (A) : When there is emission of
S’ p-particle atomic number increases by one unit.
® 3.00
® 14 p ____ . 14 KT . 0 „
gU ------ > 7 IN +_1C
o>
£
■o Reason (R): Neutron is converted to proton
=
m 1.00 L i „ ____. i„ , o„
on ---- > iP + _xe
0 50 100 150 200 250
Mass number 2. Assertion (A) : When there is an emission of
Binding energy per nucleon a-particle, new element is displaced two groups of the
parent element.
1. Which have comparatively higher energy?
(a) Even number of protons and even number of Reason (R): Atomic mass decreases by 2 units due
neutrons to a-particle emission.
(b) Even number of protons but odd number of 3. Assertion (A) : Half-life period of the radioactive
neutrons disintegration is independent of concentration.
(c) Even number of neutrons but odd number of
protons Reason (R) : 75% disintegration takes place in a
(d) Odd number of protons as well as odd number of time twice the 50% disintegration.
neutrons 4. Assertion (A) : y-rays produce a greater health
2. Magic number for protons are hazard to those who work with radioactive element.
(a) 2, 8, 20, 28, 50,126 Reason (R) : y-rays are extremely penetrating
■ (b) 2, 8, 20, 28, 50, 82, 114
high energy electromagnetic radiations.
(c) 2, 8, 20, 50, 82,126
(d) 1, 3, 5, 7, 9,11 5. Assertion (A): Element formed by the successive
3. Magic number for neutrons are emission of la-particle and 2p-particles is placed in
(a) 1, 3, 9,11,13 the same group as the parent element.
(b) 11,19, 37, 55, 87 Reason (R): a-particle is [^Hef+ while (J-particle is
(c) 2, 8, 20, 28, 50, 82,126
(d) 2, 8,18, 36, 54, 86
I
6. Assertion (A): Nuclear-fission is a chain reaction.
Passage 8 Reason (R) : Extra electrons formed in the
intermediate steps further attack the nuclei.
Don’t Press the Panic Button.
Despite the crisis in Japan’s Plants, India must not
take nuclear power off the table. True & False
Answer the following questions 1. A radioactive change is always irreversible and
1. In view of recent nuclear blasts due to Sunami in independent of temperature and pressure.
Japan countries are 2. Thorium (90) formed from emission ofa-particle from
(a) going to stop nuclear-energy plan uranium (92, group IIIB) has been placed in group
(b) devising methods to avoid this type of disaster IIIB.
(c) stressing thermal energy plan 3. Increase in atomic number by one unit due to
(d) stressing hydro-electric power emission of 0-particle is as a result of conversion of
2. Nuclear plan can’t be taken off the table because of neutron into proton.
(a) unusual toxicity of the radioactive substance 4. Successive emission of la and 2p-particles results in
(b) technology outweighs such risks the formation of isotopes.
(c) nuclear power remains the only economically 5. y-rays are not affected in magnetic or electric field.
viable renewable source of energy
(d) alternates (b) and (c) 6. y-rays have least penetrating power.
Chapter 5. Radioactivity | 247

7. Nuclear-fusion chain initiation is endothermic and 12. Nuclear reaction 14N +2He -—> +|H is written
further reaction is also endothermic. as
8. D2O or graphite is a moderator in nuclear reactor. 13. There should be a of the fissionable material
9. Elements with atomic number greater than 92 are to maintain a fission chain.
called actinides. 14. Graphite is used in the nuclear reactor as a
10. IUPAC representation of atomic number 108 is Uno. 15. IUPAC representation of the element of atomic
11. Ds is the latest IUPAC approved name of the element number 115 is
of atomic number 110. 16. The overall reaction in the sun responsible for the
12. The more positive the packing fraction of an element, energy it radiates is
the more stable should be nucleus. 17. Elements with atomic numbers greater than 92 are
13. Energy equivalent of 1 amu is 931 MeV. called the elements.
14. Age of earth is determined by Geological dating. 18. The more the packing fraction of an
15. MRI uses radio-waves. element, the more stable should be the nucleus.
16. Iodine-131 is used in imaging the brain. 19. Energy equivalent of 1 amu is MeV.
17. 2iNa changes to 12 Mg by bombardment of q/i and 20. Age of dead bodies is determined by
successive emission of 0-particle. 21. Cobalt-60 used in the treatment of
18. ‘Fat man’ relates to literary word from a book. 22 is a device that detects nuclear radiations
19. 40K is used to determine age of objects upto 1 million from flashes of light generated in a matter by the
years old. radiation.
20. ‘Little bo/ relates to U-bomb. 23. A REM is a unit of used to relate various
kinds of radiation in terms of biological destruction.
Fill in the Blanks 24. Unit of radioactive disintegration constant is
1. SI unit of radioactivity is ... which is equal to
dps. 25. Element formed from 29® U (group IIIB) by emission
2. Disintegration of 3.7 x 10jo 1 atoms per second of 1 a-particle is placed in group
(3.7 x IO10 dps) is called
3. Successive emission of 1 a- and 2 p-particles results in Matrix-Match Type Questions
the formation of
I. Only One Correct
4. There is an increase in the atomic number by 1 unit
due to 1. Match the atomic number of the transuranium
element (in List I) with their IUPAC representation
5. 904Th changes to 282Pb by emission of
2
in (List II).
a-particles and ...... P-particles.
List I List II
6. Series Starts with Ends with
A. 100 1. Unp
(a) 4n B. 101 2. Unh
(b) 4n + 1 C. 102 3. Unq
D. 103 4. Unt
(c) 4n + 2
E. 106 5. Unu
(d) 4n + 3 6. Unb
7. Radioactive disintegration follows order 7. Unn
kinetic.
8. Radioactive change is and independent of 2. Match the name of scientist (in List I) with the
and discovery or law (in List II).
9. k~\k is radioactive disintegration constant) is called List I List II
A. Henry-Becquerel 1. Artificial radioactivity
10. Concentration of the radioactive element after one B. Soddy-Fajan 2. Cyclotron
average life is of the original concentration. C. Rutherford 3. Group displacement law
(A*) 24-*r D. I. Curie, F. Jolit 4. Radioactive disintegration
11. ftNa ------ * nMg reaction is written as E. E.O. Lawrence 5. Radioactivity
248 | Essential Inorganic Chemistry

3. Match the series (in List I) with the end product Set I
(in List II).
1. In the following disintegration
List I List II
™U ---- > s26Pb+...
A. An 1. 82 Pb number of 0-particles emitted is
B (An + 1) 2. 2832Bi
2. In question (1) above, number of a-particles emitted
C. (An + 2) 3. ^Bi is
D. (An + 3) 4. ^Pb 3. 16 g X changes to 1 g (half life 1 day) after ....
5. 823pb days
6 «Bi 4. Atomic number of positron is taken as
5. When J Be captures one electron, mass number of the
4. Match the series (in List I) with starting element (in element formed is
List II) and end element (in List III).
6. Charge on a-particles is X+; X is
List I List II List III
7. Charge or y-particles is
A. An 1. 2w1Pu I. ^Pb
8. 10 g of a radioactive element loses 8.75 g in 9 days.
209D;
B. (An + 1) 2 iTh II. 83 bl Thus, half-life is days.
235i i
C. (An + 2) 3 92 u III. ^Pb
D (An + 3) 4. 238.)
IV. ^Pb
Set II
92 u
1. 1.00 mL solution containing 0.24 jiCi of tritium is
II. One or More Correct injected into a dog’s blood stream. After a period of
1. Match the Column I with the properties (one or more) time to allow the isotope to be dispersed, a 1.00 ml
in Column. sample of blood is drawn. The radioactivity of this
sample is found to be 0.04 pCi/mL. What is the total
Column I Column II volume of blood in the dog?
A. Determination of age of 1.
/V = A/o
7? 2. How many neutrons are contained in A formed by the
rock 2J reaction?
B. Carbon-dating 2. 0.693 4 Be (a, n) ---- > A
X
3. How many neutrons are emitted in the reaction?
C. Half-life 3. 2i®u —> ^Pb
236U ---- > 141Ba + 92Kr
D. a-particle 4. ii r __ ,
6v ----- > 7n 4. In the following decay process, half-life is days
E. p-particle 5
10g
2. Match the Column I giving particles with the
reaction involving these particles (on or more) in
Column II. 5
Column I Column II 2.5
1.25
A. jHe 1. »N----- > 178O
B. ]H 2. nAI —> gP
3. 30p ___ . 30 q;
C. on 15' ------ * 14 Ol time (days)
D. _°e 4. f?Na--- > fl Na 5. What is the number of particles left in the following
E. ?e 5. i2?Na----- > fjMg decay process?
• • • • 6 days 6 days
Integer Answer Types X Y Z IV
--------------- ► ?
This section contains 8 questions. © © © ©
The answer to each of the questions is © 0 6. In the following reaction,
a single digit integer, ranging from 0 to O O
© © © © II I8 I
9. The appropriate bubbles below the © © ® © CH3COH + CH3CH2OH —> CHsCOCjHj+H.0
respectively question numbers in the ©. ©
ORS have to be darkened. For molar mass difference between water formed in the
example, if the correct answers to
© © © reaction and ordinary water is unit.
question number X, Y, Z and W (say) ® ® ® ® 7. How many curies are contained in the radioactive
are 6, 0, 9 and 2, respectively, then the 0 0 0 0 change with activity 18.5 x 1010 dps?
correct darkening of bubbles will like ® ® ® ® 8. Carbon-dating uses carbon isotope with number of
the following: © © ® © neutrons
Answers & Solutions
Master Exercises
Exercise 1
N1 A1 = N2 'A. 2
Analytical Questions 4.
1. Half-life= 4.8 min radium uranium
(a) time = 19.2 min A 1 N2 28
= =
19·2 A. 2 Nl 1
y = number of half-life= =4
4.8 0.693
r )
I.. t50 (t50 >2 28
1 = --
1
C=C0 (U =-
0.693'
-,------, ...=..

(tso>i 1
r
=lmg (½)' -�J2
(t50 )2 = 1620 x 28 years= 1.43047 x 10 12 s
1
=-mg - 0.693 - 4 844 10-13 S -1
16
'l
11. ---- • X
2

= 0.0625 mg
t50

(b) time=48min 5. 1 g radium = N o atoms


48 226
y=-=10
4.8 dN =-
-- >No
226
time of99.9% reaction=10 times of half-life dt
4.353 x10-4 N0
hence, 99.9% reacted 11_6 x101, =
O.l 226
unreacted =0.1% = 1 x
100
No = 6.02 X 10 moi-1
23

=1 xl0 mg 6. 1 mCi =0.001 Ci= 3.7 x10 7 dps


-8

dN
2. --='J..N
dt t50 = 5720 years
0.1 X 10
0.1 mg 239Pu =0.1 x10 -3 g = --- mol = 5720 X3.1536 X 10 S
-3 7

239 (1 yr=3.1536 X 10 7 s)
0.1 X10 -3 X6.02 X 10 23
• 0.693 0.693
N =-------atoms
239 A= -- = ----------7 S-1
3 x6.02 x 10 23 t50 5720 x 3.1336 x10
x0.l x
. xl 0 7 _ l 10-
14 ---------
239 dN = 1.. (!!!...)
14
x6.02 x10 23 atom s-1
A =5.56 x10-11 min-1
dt
0.693 W X 6.02 X 10
- 0.693 3_7 X 107 =
23

-7•481 X 1011
t50-- X
--
')..
S
3.1536 X10 7 'X5720 14
= 2.372 years W =2.24 X 10
-4
g
3. l _ 2.303 I C0 1.00 µCi=1.0 x10 -6 Ci
-- t-
ogc 7.
2.303 =3.7 x 101 x 1.0 x 10 dps
log C0
° -6
t=
-dN =3.7 x 10 4 dps
"' C
2.303 x t50 C
lo ( o)
=
dt
0.693 g C 1 year= 3.1536 x 10 7 s
2.303 x 14.8 100 .. 0.693 = 0.693
(a) t=---- I og- =
0.693 10 A. � 1.28 X 10 X3.1536 X 10 7
9

= 1.72 X10-17 S-l


=49.184 h
2.303 x 14.8 100
(b) ts:.---- Iog-= 64 h - dN A.WNO
0.693 5 ="J.N=
dt 40
250 | Essential Inorganic Chemistry

4 _ 1.72 x IO-17 x w x 6.02 x 1°23 11.


O.« X JLU —----------------------------------------------------------
40
*50 = 10 days
w = 0.143 g t
8. 2!°Po ---- > 4ne + 2^Pb y=r 2
?°=2
l50
1 g Po = -i- mol Po X left after 20 days
210 1 / 1
Po left in 69.2 days = C = Co
ny c=c0 2
| = - g atom
2)
total time .*. Helium formed = amount of X decomposed
69.2 fl
where, y = ~-------
t50 138.4 ”
1/2
u = 1 - - = - mol
4 4
1 Q

C =1 .-. Volume at STP = - x 22.4 L


2 4
= 0.7071 g = 16.8L
Po changed to helium = 1 - 0.7071 g 12. - o at start
dt 0
= 0.2929 g dN
= 1.3947 x 10"3 mol ------ = XN after time t
dt
Helium formed = 31.24 cc at STP 4750 = XV0
9. Initial amount of an a-active isotope = No = 10 g atom 2700 = XV
Helium formed = 11.2 cc at STP No _4750
11 2 N ~ 2700
= —:— g atom at STP
22400 2.303 1
Also X= log —-
= 5 x 10"4 g atom t 6 N
2,303 4750 A11O . .
Element left =(10-5 x 10"4) log ------ =0.113 min
5 200
= 9.9995 g atom 0.693 0.693
, 2.303 . No *50 - ------ =------- = 6.1328 mm
10g^ X 0.113
13. Let initial amount = 10 = No
= 2.303 log = 5 x IO"5 h-1
9.9995 Amount decomposed =99.99%
0.693 0.693 , N = amount left = 100 - 99.99
*50 ~
X 5 x 10’3
= 0.01
= 13857 h = 1.582 years , _ 2.303
A. —--------
10. Amount left after 10 days = amount disintegrating on t log aF
0.693 2.303 , 100
11th day. log-----
1.53 t 0.01
1 y
C -Co t= 20.34 s
2 total time 235t t ____ .
=2 14. 92 u * 237Pb;7-a,4-p
= 0.1
n2 S8 Au —8Q8Hg + >(P)
2) 15.
Au and Hg are isobars.
= 0.1x- Half-life = 65 days, t = 260 days
4
total time
= 0.025 g atm after 10 days y =------------- =4
hal-life
Amount left on 11th day (y = - half-life |
y
\ 5 J 1 1 4 1
Au left C -Co =1
when Co =0.025 g atom 2, 2 = 16S
1/5 Hg formed =CO-C=1-^ 15
1 = 0.9375 g
C =0.025 g atom
2
'o.n
= 0.0218 g atom
Amount disintegrating on 11th day = Co - C
16. 1+™L1 + 206 = 1.1155 =(2)-v
. [U] J 1
= 0.025-0.0218 <238,
= 3.2 x 10"3 g atom t
= 1.926 x 1021 atom y=r l50
Chapters: Radioactivity I 251
y log 2 = log 1.1155 2. Mass defect=15.995 -[8(1.0073 + 1.0087 + 0.000548)]
y=0.1578
= 15.995-16.132384=0.137384 unit
t = 4.5 X 109 X 0.1578
= 7.1 x108 years BE=0.137384 x931 MeV
17. (a) �.2Pu + ;gca --+ �JUuq + �n =127.9045 MeV
= 127.9045x106 eV
Uuq has atomic number 114
19 J
ttt = 127.9045 X 108 X 1.6 X 10-
11 4
= 2.046 x 10-11 J per molecule
1
{b ) :B + �n � �i + :H e :::2.046 X 10-ll X 6.02 X 10
23
(a)
18. Ca) 69.9 MeV (b) 42.6 MeV = 1.232 X 10
13
J mor1
19. LlE=4.795-4.611=0.184 MeV 3.2;.fl��e

= 0.184x106 eV Mass defect = m<;He)-2m(fH)


= 0.184 X 10 6 X 1.6 X 10-19 J = 4.0026-2 X 2.0141
:::2.944 X 10-H J = - 0.0256 units
M = he = - 0.0256 x 10-3 kg unit
A, BE=Amc 2
he 6.62x10-34 x 3 x 108 =0.0256 X 10-3 X (2.998 X 10 8 )2
A= =
!J.E 2.944 X l0-l4
=2.3 x1012 J
=6.746 X 10-1 2 m
4• 60 N · In
28 1 + 0
(n, p)) soc°+ l1 H
21
=6.746 pm
20. 827.5 nm Target nucleus : Ni

21, Mass defect BE


;;co + �n � :,:co + y
. g mor1 MeV/neutron
Target nucleus = �o
(a) 0.467 8.72
Half-life of �Co = 7 years
(b) 0.60015 8.76
t50 = 7 X 3.1536 X 107 S
22. Refer Q. 22.
l. 0.693 0.693
= s_1
23. (1+ 1�1 )=2'
= 7
l50 7 X 3.1536 X 10

2 2
(l + 0.6/ 06) = 2" = 3.14 X 10 -8 8-l
, 23.8/238 A._ 2.303 I C0
5. - ogc
2=21 - t-

y:::l 0.693 2.303


= l �
t
=1 t50 t og C
t50
0.693 2.303 100
t = 4.5 x 10 9 years = Iog
5770 - t - 60

NCERT Problems t= 4254 years


1. Mass defect 6. - dN =AN
dt
=7.016-(4 X 1.008665 + 3 X 1.007277 + 3 X 0.000548)
=7.016-7.058135 0.693 X W X 6.02 X 1023
3_7 X loJ.O
=-0.042135 units 40x60x60 x 140
:. BE=0.042135x931 MeV=39.228 MeV
w =1.8x10-6 g
E
:. BE per nucleon= (!3 ) = 5.604 MeV/nucleon 7. Refer text.
7 J
I Essential Inorganic Chemistry

Exercise 2
Only One Option Correct 22. C=C0
1V
1. In the chain reaction, 2j
y
Energy Neutrons (3.00 x 108) = (2.40 x 109) f-
12.
1st step
2nd step
3rd step
3E
9E
E 3
9
27
(ir-s-er
J=3
nth step 3n'yE 3n or
150
Thus, (d)
t = 3 x T50 = 36.78 years
2. (d) 3. (d) 4. (a) 5. (c) 6. (c) 7. (b)
Thus, (d)
8. (d) 9. (c) 10. (a) 11. (c) 12. (d) 13. (a) 23. (c)
14. (b) 24. p-particle emission gives isobar.
15. To determine age by rock-dating
?9°Au ---- * + -le
l+™=2> y
[U] 1
C’=C0
(l+l) = (2)y 2
52
y=l y = — =4
13
—=y=1
T
2 50
C=1
fl> 4
t = T50 = 4.5 x 109 year I2J 16
Thus, (b) Au left = — g
16 5
16. Disintegration constant k = 0.49 x 10r10 year 1 15
0.693 0.693 Au changed toHg=l- — = — = 0.9375 g
r50 = k ~ 0.49 x IO'10 year 16
Thus, (b)
= 1.41xlO10 year 25. (r60)A
*B ^50 ^B
Thus, (c)
17. Mass defect = 0.02 amu 2 x IO10
3.1xl09 =
002 t _j
= —— gatom
(T60)b =6.45 year
002 x. Thus, (b) I
= —xN0gmol i
■^o
26. - — = kN = k — N0
= 0.02 g mol-1 dt A 0
E = me2 = 0.02 x 9 x IO20 erg mol-1 3.7 x IO10 = 4.4 x 10~12 x — x 6.02 x 1023
14
_O.O2x9xlO20 x0.002
million kcal mol w =2x10-4 kg (lCi = 3.7xlOl°dps)
8.314 x 107 xlO6
Thus, (a)
1 million = 106
= 430 27. (c)
k8.314 x 107 erg = 0.002 kcal. 0.618 x IO-6
28. [Pb] = mol = 3 x 10“9 mol
Thus, (a) 206
0.238 x 10r6
18. (d) 19. (c) [U] = — mol = 1 x IO-9 mol
238
20. Positron is emitted with proton changes to neutron
|H —> ‘n + > For rock-dating, fl + = (2)>
I [U1J
Thus, “Al—•» gX+‘e
y=2=-L
Number of electrons remained = 13 (as in Al) r50
Thus, (a) f25X has 12p, 13n, 13e. t = 3.0 x 109 years
21. (c) Thus, (b)
Chapter 5 : Radioactivity I 253

29. TA=
0·: 93 0.693 2.303 l 1 x10-6
by first order rate law (based on unit of kA) = og
A 28.1 20 C
1
T8 =-k by second order rate law (b ased on unit of k 8) C =0.6107 x10 g
--6

Ba =0.6107µg
( given) 4. 2.8 nCi =2.8 x 10-9 Ci
--=-
kA k 8a
(a =initial activity) =2.8 x 10-9 x3.7 x 1010 dps
=103.6dps
30 x10-6 x6.02 x1023
kA =0.693
a _ dN
kB 103.6 = =).. x
dt 247
(Rate)A =kia) 15 1
11. = 1.417 x10- s ··
(Rate)8 =k 8( a)
2
0 693
(Rate)A k t50 = · =4.891 X 10 14 S
= A =0.693 =kATA A
(Rate)8 k 8a
=1.551 x 107 years
Thus, (c) 0.693 0.693 _1
5. A= = h
30 . (c) 31. (d) 32. (b) 33. (a) 34. (a) t50 0.5
35. (c) dN
2 38u � 230x
--='AN
36. 92 88 dt
Element of atomic number 88 belongs to IIA - dN =10 dis min-1 =10 x 60dis h- 1
Thus, (c) dt
0·693
37. (c) 38. (c) 39. (a) 40. (b) 41. (b) 10 x60;;;; N
0.5
42. (c) 43. (c) 44. (a) 45. (b) 46. (c)
N =207.9 atoms
47. (d) 48. (d) 49. (c) 50. (c)
6. 1 Ci =3.7 x 10 dps
10

One or More Than One Options Correct t50 =5730 years =5730 x3.1536 x 107 s
1. (c) 2. (a),(b),{d) 3. (a) 4. (b),(c),(d) 8_1
0.693 0.693
A= =
5. (a),(b),(d) 6. (a),(b),(c) '1. (b),(d) 8. ( a),(b),(d) . t50 5730 x3.1536 x10 7

9, (c),(d) 10. (a),(b),(c) _ dN AW x6.02 x1023


=').N=
dt 14
Brain Twisters 0.693 X W X 6.02 X 1023
3. 7 X 10 = ----------
lO

1. A =2.303 log 100 5730 X3.1536 X 107 X 14


t 5
0.693 2.303 w =0.2244 g
= log 20
t50 t 7. 8.27 x 10-4 nm
0.693 2.303 8. Refer solved example 5.21
= log 20
2.62 t 9. Energy =200 kWh=200 x 103 watt
t =11.33 years
=200 X 103 J S-l
2. 1 g radium = No atoms =200 x103 x3600 Jh-1
226
200 x103 x3600
dN'ANo
--=- = eV
dt 226 1.6 X 10-lS
4.353 x10 N0 =4.5 x10 eV
4 27
17
11_6 x10 =
226 4.5 X 1027 X 1.7
Molecules of H2 produced =
N0 =6.02 x1023 moi-1 100
3. t50 =28.1 years = 7.65 x1025 molecules
7.65 X 1025
-----,-mo I H2
). --- l og S,
-2.303
C 6.02 X 1023
t

0.693 2.303 l =127.076 mol H2


= og �
t50 t C =2846.5 L at STP
254 | Essential Inorganic Chemistry

10. Mass defect =49.9516-49.94479 17. 7.4 mCi = 7.4 x 10’3 Ci = 7.4 x 10’3 x 3.7 x IO10 dps
= 6.81 x 10"3 amu = 2.738 xlO8 dps
Binding energy =6.81 x IO-3 x931 MeV
= 6.34011 MeV dt
Kinetic energy of the emitted 0” particle =0.80 V t60 = 8.1 days
Kinetic energy of neutrino emitted = 5.54 MeV = 8.1 x 24 x 3600 s
. _ 0.693
11. i4C —> “N+JJe A —---------
*50
Mass defect = 14C - 14N = 14.00324 -14.00307 0.693 AT
2.738 x 108 =
= 1.7 x IO-4 amu 8.1x24x3600
Binding energy = 1.7 x 10-4 x 931 MeV
N = 2.765 xlO14 atoms
= 0.1583 MeV per nucleon
1 Ci = 3.7 x IO10 dps Passage Comprehension Questions
= 3.7 xl010x 3600 dph
Total heat =3.7 x IO10 x 3600 x 0.1583 MeV Passage 1
= 2.1086xlO13 MeV 1. (c) 2. (a),(b),(c) 3. (a)
= 2.1086 xlO19 eV Passage 2
= 2.1086 xl019x 1.6 xlO’19 J 1. (a),(b),(c),(d) 2. (a),(c),(d)
[Pb] Zn 9 >
12. = ——x 232 =0.2231 Passage 3
[Th] k208 J
1. (c) 2. (a)
(2)y=l + — = 1.2231
[Th] Passage 4
y log 2 = 0.08745 1. (b) 2. (c) 3. (d)
y = 0.2905
Passage 5
— = 0.2905
*50 1. (a) 2. (d) 3. (d) 4. (c)
t= 0.2905 xT60 Passage 6
= 0.2905 x—0,693 - 1. (b) 2. (a) 3. (a)
4.98 x 10’11
Passage 7
= 4.04 x 109 years
1. (a) 2. (b) 3. (c)
. 2.303 . Co
13. X = —log^
t Passage 8
0.693 2.303 , 15 1. (b) 2. (d)
=------- log----
5730 t 10
t = 3353 years Assertion & Reason
_ 2.303 1. (a) 2. (c) 3. (b) 4. (a) 5. (b) 6. (a)
14. A —---------- logw
t c*
0.693 2.303 . 15.3 True & False
------- =-------- log
log ------
5730 t 9.4 7. F
1. T 2. T 3. T 4. T 5. T 6. F
t =4028.62 year
8. T 9. F 10. T 11. T 12. F 13. T 14. T
15. (Ar) = 65.9 atoms = No
15. T 16. F 17. T 18. F 19. T 20. T
(K) = 39.1 atoms = N
2.303 , ^0
Fill in the Blanks
X=
t logv 1. Bq (Becquerel), 1 2. one curie 3. isotope
0.693 2.303 log^
6 N 4. conversion of neutron into proton 5. 7,6
*50 t 238 tt
232rm,
*h; 208Pb (b) ^Pu, 28°39Bi (c) 92 ; Mpb
6. (a) 299032Th,
0.693 2.303 , 65.9
1.3 x 109 t 10g m (d) 285U 28°27Pb
t = 1.236 x 109 years 7. first 8. irreversible, temperature, pressure
Chapter 5: Radioactivity j 255

9. average life 10. -


Integer Answer Types
e
Set I
11. "Na+}H---- > ^Mg+’n 7_
id
Questions 1 2 | _4_ 6
12. “Nta.p)1^ 14. moderator
13. critical size Answers 6 8 1 2 0

15. Uup 16. 4{H - —> 2^6+2 °e(p+) 17. transuranium © ®_ © © ®l®
©
©. ©. © © © ©
18. negative 19. 931 20. carbon-dating 21. cancer
23. radiation dosage © ©_ © © © © © ©
22. scintillation counter
© ®_ © © © © © ©
24. time’1 25. IIIB © ©. © © ©
© _© © © © ©_©_ ©
Matrix-Match Type Questions © ©. © © © © © ©
I. Only One Correct © ©. © © © ©
1. (AM7); (B)-(5); (O—(6); (D)M4); © ®, © © © _©_© ©
(EM2); I® ©| © © © ®|® i©
2. (AM5); (B)—(3); (CM4); (D)—(1);
(EM2); Set II
(DM1);
H30
3. (AM5); (BH3); (CM); Questions —► 1 _3 4_ 6 7_ 8
(BM1MH); Answers —► 3 4 0 5 8
4. (AM2MHI); 6
(CM4MIV); (DM3MD; ©_ © ©| © ©|© ©
II. One or More Correct ©. © ©. ©
1. (AMI,2,3); (BM1,2,4); (CM1,2); (DM3); ©_ © ©_ ©J © _©_© ©_
(EM3.4) ©_ © ©_ ©^ © © © _©
2. (AMI,2); (BHD; (CM2,4); (DM5); ©. © @_ © © © @_
(EM3) ©. © © © ©I® _®
_® © _© © © © © ®_
©. © 0. © © ©_©_ ©
®_ © _© © © © © ®_
® I© © © © ©I© ®

a
Chemical equations represent a very precise, yet a very versatile,
language that describes chemical changes.

Chemical Equations and


Reactions

Chemical Equations . .. . . . .
Chemical reactions always involve changing one or more
substanc es into one or more different substances. That is, they
involve regrouping atoms or ions to form other substances.
A chemical equation represents the final change in • Chemical Equations
composition when two or more reactants react together to form
new substances. • Stoichiometry
Thus, chemical equations are used to
• Limiting Reactants
- describe chemical reactions, • Yields of Chemical Reactions
- and they show • Sequential Reactions
(1) the substances that react, called reactants
(2) the substances formed, called products and
(3) the relative amounts of the substances involved.
Take following chemical equation which represents a
reaction of stream of chlorine gas, Cl2, with solid phosphorus, P4
giving a liquid phosphorus trichloride (which appears as a
flame).
�4 (s)+ 6Cl2 (g) � 4PC1 3 (l�
_
Reactants Products
In a chemical equation, the formulas for
- the reactants are written to the left of the arrow.
- and that of the products are written to the right of the
arrow.
- the physical states of the reactants are also indicated by
(s) for solid
(l) for liquid
(,g) for gas
258 £ Essential Inorganic Chemistry

A substance dissolved in water, that is, an aqueous stoichiometry and the coefficients in a balanced
solution of a substance, is indicated by (aq) equation are stoichiometric coefficients
Based on the law of conservation of matter, which 2Fe(s) + 3Cl2(g)---- > 2FeCl3(Z)
states that matter can neither be created nor
destroyed, if the total mass of the reactants is 10 g, and if
t t t
stoichiometric coefficients
the reaction completely converts the reactants to products,
This equation is read as :
you must end up with 10 g of the products. This also means
that if 100 atoms (say) of a particular element are 2 atoms of iron (Fe) combine with 3 molecules of
contained in the reactants, then those of 100 atoms must chlorine (Cl2) to form 2 molecules of ferric chloride (FeCl3)
also appear in the products in same fashion. This equation also represents that 2 moles of iron
When we apply law of conservation of matter to the combine with 3 moles of chlorine (Cl2) to form 2 moles of
following reaction FeCl3.
P4(s)+ 6Cl2(g)---- > 4PC13(Z)
it tells that 1 molecule of phosphorus, P4 (with Balancing the Chemical Equations
4 phosphorus atoms) and 6 molecules of diatomic chlorine Balancing a chemical equation ensures that the same
(Cl2, with 12 chlorine atoms) are required to produce four number of atoms of each element appears on both sides of
molecules of PC13. Because each PC13 molecules contains the equation. Many chemical eqautions can be balanced by
1 P atom and 3 Cl atom, hence in all four PC13 molecules trial and error, although some will involve more trial than
are needed. others when balancing chemical equations,
(Cl-atoms) 6x2 = 12 4x3 = 12 — Formulae for reactants and products must be
correct
Ion the reactant side on the product side.
— Subscripts in the formulae of reactants and
P4(s) + 6Cl2(g) ---- > 4PC13(Z) products cannot be changed to balance equations.
(P-atom) j Thus, CO2 cannot be changed to CO
4 on the reactant side 4 on the product side — Look for elements that appear in only one place on
The numbers in front of formulae in balanced each side of the equation and balance these
chemical equations are required by the law of elements first.
conservation of matter. The relations"hip between the — If free, uncombined elements appear on either
quantities of chemical reactants and products is called side, balance them last.

Illustration 1 Al reacts with HCI to form A1C13 and H2. Unbalanced equation is
Al + HCI---- > A1C13 + H2
Solution In this form, the equation does not satisfy the Law of conservation of matter as except Al, other
atoms are not balanced (Cl one on the left and three on the right, H one on the left and two on the right)

Atoms of
Al Cl H Conclusion
Steps Reaction stage Al Cl H
Products
Reactants

I Balance Cl by placing 3 before HCI Al + HCI----- > AICI3 + H2 1 1 1 1 3 2 Unbalanced chlorine


Al + 3HCI----- > AICI3 + H2 1 3 3 1 3 2 balanced but reaction
unbalanced
II 3 H appears in LHS and 2 H on Al + 6HCI —> AICI3 + 3H2 1 6 6 1 3 6 H balanced but reaction is
RHS. Least common multiple is 6. unbalanced
Multiply HCI on LHS by 2 and H2 on
RHS by 3
III Balance Cl by multiplying AICI3 on Al + 6HCI ----- > 2AICI3 + 3H2 1 6 6 2 6 6 All balanced but Al is not
RHS by 2 balanced
IV Balance Al on LHS by multiplying 2AI + 6HCI ----- > 2AICI3 + 3H2 2 6 6 2 6 6 all balanced
Al on LHS by 2
V Specify physical state 2AI(s) + 6HCI(aq) —> 2AICI3(aq) + 3H2(g)
Chapter 6: Chemical Equations and Reactions | 259

Illustration 2 We now balance the following reaction Table 6.1 Can be used to represent a chemical
with same strategy : equation in aqueous solution many of the reactions are
C3H8(g) + O2(g)-----> CO(g)+H2O(Z) exchange reactions in which the ions of the reactants
change partners^-----
Reactants Products
Solution Step I Balance the C-atoms Multiply A+B~ +C*D~ ---- > A+D~+C+B~
CO2 on RHS by 3. Take the reaction of aqueous silver nitrate with
aqueous potassium chloride.
C3H8(g)+ (^(g) •> 3CO2(g)+ H2O(Z)
unbalanced in terms of H and 0 AgNO3(ag)+ KCKag)---- > AgCl(s)+ KNO3(ag)
Step II Balance the H-atoms Multiply H2O on RHS by 4 From the table 6.1, we observe AgNO3,KNO3 and KC1
both are water soluble and thus can give ions in aqueous
C3H8(g)+ 02(g)---- > 3CO2(g)+ 4H2O(Z)
solution. AgCl is insoluble and is thus precipitated.
imbalanced in terms of 0
Ag+(ag) + NO3(ag) + K+(uq) + Cl"(ag)
Step III There are in all 10 oxygen atoms in RHS Balance
0 on LHS by multiplying by 5 ---- > AgCl(s)+K+(aq) + NO3(ag)
Net reaction taking place is thus between Ag+ and Cl"
C3H8(^)4- 5O2(£) 3CO2(g)+ 4H2O(Z)
ion in aqueous solution
balanced
Ag+(ag) + C\~(aq) ---- > AgCl(s)

Numerical Problem Solving 1


Balance following reactions
Acid-Base Reactions
• Acid is a substance that gives H+ ion in aqueous
(a) KC1O3(s)---- > KCl(s)+ O2(s)
solution
(b) P4(s)+ O2(g)---- > P4O10(s)
hci h+ + cr
(c) Zn(s)+ HNO3(ag)---- » Zn(NO3)2(a^)+ NO2(g)
• Base is a substance that gives OH" ion in aqueous
+ H2O(Z) solution
(d) Al(s) + NaOH(ag) NaA102(ag)+ H2(g)+ H2O(Z) NaOH Na+ + OH"
Acid-base reactions are among the most important
Different Types of Chemical Reactions kinds of chemical reactions.
Reactions in Aqueous Solutions Many acid-base reactions occur in nature in both
A solution is a homogeneous mixture of two or more plants and animals.
substances. One substance is generally considered the The reaction of an acid with a metal hydroxide (base)
solvent, the medium in which another substance-the produces a salt and water. Such reactions are called
solute- is dissolved.
neutralisation reactions due to the fact that acidic and
There are many important chemical reactions basic properties are lost.
occuring in aqueous solutions where water is the solvent.
• In formula unit equations, we show complete
formulae for all compounds.
DEEP Focus
2AgNO3(ag)+ Cu(s)---- > 2Ag(s) + Cu(NO3)2(ag) Acid is H+ ion donor base is OH" ion donor.
In nearly all neutralisation reactions, the driving force is
• In total ionic equations, formulae are written to
the combination ofH+ (aq) from an acid and OH" (aq) from a base
show the (predominant) form in which each substance to formH2O.
exists when it is in contact with aqueous solution. Ions
that have common source or that remain in solution HCl(ag) + NaOH(ag) * H2O(Z)+NaCl(ag)
after the reaction in complete are shown in brackets in strong strong weak strong
acid base electrolyte
total ionic equations
2[Ag+(ag)]+ NO3(aq)+ Cu(s)---- > 2Ag(s) I I
+ [Cu2+(aq)+ 2NO3(ag)] By [H+(ag)+ Cl"(ag)] + [Na+(aq)+ OH~(ag)]
We observe that NO3 ions do not participate in the ionisation -----> H2O(Z)+ [Na+(ag)+ CHa?)]
reaction. Because they do not change, they are thus simply Cancelling common ion,
the “spectator” ions.
H+(ag)+ OH'(aq)-----> H2O(Z)
• The net ionic equation shows only the species that
react. It does not have the spectator ions. This is the net ionic equation for all reactions of strong
acids with strong soluble bases that form soluble salts and
2Ag+(aq)+ Cu(s)---- > 2Ag(s)+ Cu2+(ag) water.
260 | Essential Inorganic Chemistry

DEEP Focus Generally soluble Exceptions


A weak acid is that one that is not ionised completely in Carbonates (CO3') Soluble : those of NH},Na+,K*
aqueous solution. Neutralisation of weak monobasic acid by a Phosphate (PO4‘)
strong base results in the formation of soluble salt and H2O Oxalate (C2O2-)
CH3COOH (acetic acid) is such type one of the weak acids : Chromate (CrO4“)
CH3COOH(aq) + KOH(aq) ---- > CH3COOK(aq) +H2O(Z) Arsenate (AsO4")
weak acid strong base soluble salt weak
Thus, we write
Precipitation Reactions
CH3COOH(aq) + [K+(aq)+ OH'(aq)] ---- >
In precipitation reactions an insoluble solid, a
[CH3COO'(aq) + K+ (aq)] + H2O(Z) precipitate forms and then settles out of solution.
Net reaction is thus :
CH3COOH(aq) + OH'(aq)---- » CH3COO'(aq) + H2O(Z) DEEP Focus
The driving force for these reactions is the strong attraction
between cations and anions. This results in the removal of ions
Numerical Problem Solving 2 from solution by the formation of precipitate.
Write net reaction when a weak acid (as CH3COOH) is Both silver nitrate and potassium chloride are
neutralised by a weak base (as NH4OH) water-soluble ionic compounds. When combined in water,
they undergo an exchange reaction to produce insoluble
Table 6.1 Solubility of Common Ionic Compounds in Water silver chloride and soluble potassium nitrate
AgNO3(aq)+ KCl(aq)-----> AgCl(s)+ KNO3(aq)
Generally soluble Exceptions
Reactants Products
Common inorganic acids (HCI, No common exceptions Ag+(aq) + NOi(aq) Insoluble AgCl(s)
H2SO4,HNO3,H3PO4)
K+(aq) + Cl'(aq) K+(aq) + NO3(aq)
Low molecular weight organic No common exceptions
acid [CH3COOH, HCOOH,
(COOH)2] Predicting the Outcome of a Precipitation
Na+,K*,NH4 compounds No common exceptions Reaction
Chlorides (Cl“) Insoluble, AgCI, Hg2CI2 soluble in hot Whether the species in the product is precipitated or
water: PbCI2 not, can be predicted by solubility guidelines as given in
Bromide (Br “) Insoluble: AgBr, Hg2Br2,PbBr2 Table 6.1
Moderately soluble: HgBr2 => Lead nitrate (soluble) is mixed with potassium
chromate (soluble) to form potassium nitrate and lead
Iodides (I") Insoluble: Many heavy metal iodides
(Agl,Pbl2,Cdl2) chromate. In Table 6.1, potassium nitrate is soluble and
lead chromate is insoluble and thus is precipitated.
Sulphates (SO*') Insoluble: BaSO4,PbSO4,HgSO4
Moderately soluble : CaSO4, SrSO4, Pb(NO3)2(aq)+ K2CrO4(aq)-----> PbCrO4(s)
Ag2SO4 + 2KNO3(aq)
Nitrates (NO3), Nitrites (NO2) Moderately soluble: AgNO2 Reactants Products
Chlorates (CIOJ), Perchlorates Moderately soluble : CH3COOAg Pb2+(aq) + 2NO3(aq) ___ * PbCrO4(s)-insoluble
(CIO4), Permanganate (MnO4),
Acetates (CH3COO_) 2K+(aq)+ CrOl'taq) 2K+(aq)+ 2NO3(a?)

Sulphides (S2-) Soluble those of NH4,Na+,K+,Mg2+, K+(aq) and NO3(aq) appear on both sides and thus
Ca2+ final outcome is the reaction between Pb2+(aq) and
Oxides (O2‘), Soluble : Li2O, Na2O, K2O, BaO CrO^'(aq) ions.
Hydroxides (OH-) (dissolve in H2O with release of heat Pb2+(aq)+ CrO^ "(aq) * PbCrO4(s)
i.e., exothermic reactions forming
soluble hydroxide LiOH, NaOH, KOH,
Ba(OH)2) Numerical Problem Solving 3
Moderately soluble:
CaO, SrO (dissolve in H2O with Write the net ionic reactions of
release of heat, i.e., exothermic (a) K2CrO4 (aq) + AgNO3 (aq) ---- >
reactions forming Ca(OH)2, Sr(OH)2)
(b) BaCl2 (aq) + K2SO4 (aq) ---- >
(c) CaCl2 (aq) + Na2CO3 (aq) ---- >
Chapter 6: Chemical Equations and Reactions | 261

Mass relations in chemical reactions can be


Do You Know — illustrated by:
Our teeth and bones were formed by very slow
precipitation reactions in which mostly calcium phosphate, Illustration 1 Glucose, C6H12O6(s) on reaction with
Ca3(PO4)2 was deposited in the correct geometric
arrangements. (g) gives CO2(g) and H2O(Z)
Sea shells, which are formed in a slow precipitation C6H12O6(s) + 602(g) 6CO2(g)+ 6H2O(Z)
reactions, are mostly calcium carbonate, (CaCO3), a white
compound. Traces of transition metal ions give them colour. Suppose we have 18 g of glucose. Then often quantity ofC^,
CO2, and H2O can be determined based on stoichiometric
Strategy is that using a balanced equation, we can
factor. s+vntAorv
far'fnr
Stoichiometry perform the stoichiometric calculations.
A basic question raised in the chemical laboratory is, mass of H2O |
Mass O2 required mass of CO2
“How much product will be formed from specific amounts
of reactants?” Or in some cases, question may be How Multiply by Multiply by Multiply by
molar mass molar mass molar mass
much starting material (reactant) must be used to obtain a
specific amount of product. Stoichiometry (Greek words .
Moles of O2 required Moles of CO2 + Moles of H20
stoicheion - meaning “first principle or element , and
metron - meaning “measure”) describes the quantitative
relationships among element in compounds
(composition stoichiometry) and among substances as Mass of glucose
they undergo chemical changes (reaction stoichiometry).
Divide by
• Two moles of potassium chlorate (KCIO3) on heating molar mass
I(called thermal decomposition) give two moles of
potassium chloride (KC1) and three moles of oxygen Moles of glucose—I
(O2)gas
2KCl(s)+ 3O2(g) Step I: Balanced equation is
2KC1O3(s)
• Two moles silver nitrate (AgNO3) and1 one mole C6H12O6(s)+ 6O2(g)-----> 6CCW+ 6H2O(Z)
potassium chromate (K2CrO4) form one mole silver Step II: Convert amount of glucose into moles :
chromate (Ag2CrO4) and two moles potassium nitrate 18 g
18 g glucose =--------- =0.1 mole
(KNO3) is aqueous solution 180 g mol'
2AgNO3(aq) + K2CrO4(aq) Ag2CrO4($)
+ 2KNO3(ag) Step III: Stoichiometrically
moles of C6Hi2O6 _ 1
moles of O-2 6
Problem Solving Tip
.-. moles ofO2 required = 6x moles of C6H12O6
For stoichiometry
Write correct formulae for all reactants and products, and = 0.6 mole
balance the resulting equation. A balanced chemical = 0.6 x 32 = 19.2 g
equation must always include the same number of each moles of CO2 formed _ 6
kind of atom on both sides of the equation. Step IV: Also
moles of C6H12O6 reacts 1
— Convert the quantities of same or all given or known
substances (usually reactants) into moles. Moles of CO2 formed = 0.6 mole= 0.6 x 44= 26.4 g
— Use the coefficients in the balanced equation to calculate „ .. moles of H2O formed 6
the number of moles of the unknown quantities (usually Step V: Also, - --------------- ---------------- = -
moles of C6H12O6 reacted 1
products)
— Using the calculated number of moles and molar masses, .-. Moles of H2O formed= 0.6 mole = 10.8 g
convert the unknown quantities to the desired units. Results can be summarised in the tabular form :
[Moles of reactant Moles of product Equation : C6H12O6(s)+ 6Oo(g)-----> 6CO2<g)+ 6H2O(Z)
Initial amount
(in moles) 0.1 0.6 0 0
Mass of Mass of 0.6
change -0.1 -0.6 +0.6
reactant *■ product
After reaction 0 0 0.6 0.6
moles 0.6x 44 0.6x18
DEEP Focus 26.4 g 10.8 g
Take quantity of each reactant in terms of moles, for Illustration 2 The reaction between nitric oxide (NO)
stoichiometric calculations as coefficients in a balanced
chemical reaction represent moles. and oxygen to form nitrogen dioxide (NO<>) is a key step in
A2 + 3J% ---- > 2ABa photochemical smog formation’.
Correct 1 mol 3 mol 2 mol 2NO(g) +02(g) ---- > 2NO2(g)
Incorrect lg 3g 2g
262 | Essential Inorganic Chemistry

(а) How many moles of NO2 are formed by the 1 mol NO gives = 1 mol NO2
complete reaction of 0.25 mole ofO^l 1 20
(б) How many gram ofNO2 are formed by the complete hence, 1.20 g NO = mol gives
reaction of 1.20 g o/NO?
= 1x mol NO2
Solution (a) 2NO + O2 ----- > 2NO2 30
1 mol 2 mol
1.20
x 46 g NO2
1 mol O2 gives = 2 mol NO2 ” 30

hence, 0.25 mol O2 gives = 0.50 mol NO2 = 1.84 g NO2

(b) 2NO + 02 ----- > 2NO2


2 mol 2 mol

Target Practice 1
1. Balance the following equation : 4. Net reaction of
NH3(g)+ O2(g)---- > NO(g)+ H2O(Z) KOH(ag)+ HCl(ag)---- > KCl(ag)+ H2O(Z)is
OH“(aq)+ H+(ag)---- > H2O(Z)
2. What is net ionic reaction of but net reaction of
(a) FeCl3(ag)+ 3NaOH(ag)---- > Fe(OH)3(s)+ SNaCKmj) NH4OH(aq)+ HCl(ag)---- > NH4Cl(aq) + H2O(Z)
(b) Pb(NO3)2(aq)+ (NH4)2S(aq)---- > PbS(s) isNH4OH(a^)+ H+(ag)---- > NH^(ag) + H2O(Z)
+ 2NH.NO, (aq) Explain
3. What is the difference in the net reactions of the following: 5. Which compound or compounds in each of the following
HCOOH(ag)+ KOH(ag)---- > HCOOK(aq)+ H2O(Z) groups is (are) expected to be soluble in water?
andHNO3(aq)+ KOH(ag)---- > KNO3(aq)+ H2O(Z) (a) CuO,CuCl2,FeCO3 ........
(b) AgI,Ag3PO4,AgNO3
(c) K2CO,,KI,KMnO4

Answers
1. 4NH3(g) + 5O2(g) ---- > 4NO(g) + 6H2O(Z) H + (ag) + OH”(ag) > H2O(Z)
2. (a) Fe3+(a^)+3OH-(ag) ---- > Fe(OH)s(s) 4. Since NH 4OH is a weak base
(b) Pb2 + (ag) + S2"(a^) ---- > PbS(s) 5. (a) CuCl2 (b) AgNO3 (c) K2CO3, KJ,KMnO4
3. HCOOH(ag) + OH"(ag) HCOO“(ag) + H2O(Z)
weak acid

Practice Exercise 1
1. Consider the following reaction taking place completely Suppose that 1.64x 103 kg of Fe are obtained fron
2Na + 2H2O-----> 2NaOH + H2 2.62 x 103 kg sample Fe2O3. Assuming that the reactiM
goes to completion, what is the per cent purity of Fe2Os in
(a) How many moles of H2 will be formed from 0.46 g of the original sample?
Na?
4. Select correct alternate. Only one alternate is correct.
(b) How many gram of NaOH will be formed by 0.01 mole (A) 10 L of hard water with temporary hardness of
of Na? (Na = 23, H= 1,0= 16)
[Ca(HCO3 )2] required 0.56 g of lime :
2. Consider the following reaction: Ca(HCO3 )2 + CaO ---- > 2CaC03+H20
3Mg + N2 -----> Mg3N2 Temporary hardness in term of ppm ofCaCO3 is
(a) Calculate amount of Mg required to prepare 10 g of (a) 56 (b) 1/2
Mg3N2 in 100% reaction.
(c) 100 (d) 200
(b) How many moles of Mg3N2 are formed from 5.6 g of N2
(B) 5.0 g ofKC1O3 gave 0.03 mole ofO2. Hence, percentage
with excess of Mg in 100% reaction? (Mg = 14, N = 14)
purity of KC1O3 is :
3. One of the reactions that occurs in a blast furnace, where 2KC1O, 2KC1 + 3O2
iron ore is converted to cast iron, is
(a) 49% (b) 50%
Fe2O3 + 3CO-----> 2Fe + 3CO2
(c) 95% (d) 98%
Chapter 6 : Chemical Equations and Reactions I 263
{C) CaCO 3 is decomposed by HCI (density 1.825 g/c.c.) (a) 1.825 mL (b) 3.65 mL
CaC03 + 2HC1 � CaC1 2 + H20 + CO 2 (c) 0.9125 mL (d) 2 mL
Volume of HCI required to decompose 10 g of 50% pure
CaCO 3 is

Answers
l. (a) 0.01 mol (b) 0.40 g 2. (a) 7.2 g (b) 0.2 mol 4. (A) (d) (B) (a) (C) (d)
3. 89.42%

Limiting Reactants Also, 3 moles B react with = 1 mole A


Reactants are often carried out with an excess of one 2 moles B react with =� mole A = 0.66 mole A
reactant over that required by stoichiometry. This is 3
usually done to ensure that one of the reactants in the Thus A is in excess and B reacts completely.
reaction is consumed completely, even though some of B is thus limiting reactant.
another reactant remains unused. The extent to which a
reaction takes place depends on the amount of the
reactant s. Problem Solving Tips
• If you cannot decide how to solve a limiting reactant
- If the reactants are present exactly in the same problem, do the entire calculation based on each reactant.
stoichiometric ratio as required, then no reactant The smallest answer is always right
is left unused.
• Moles of product formed are always determined by the
- If one of the reactants is present in smaller starting moles of limiting reactant.
quantity than as required stoichiometricallly,
then other reactant is left unreacted and amount lliustration 2 We perform the following reaction with
of product formed is determined by the reactant four moles of CO and 3 moles Oi.
which is present in smaller quantity and is called 2CO(g)+ 0i(g) ---) 2C02(g)
Limiting Reactant. The other reactant is said to
Stoichiometry 2 mol 2 mol
be Excess Reactant. 1 mol
We take 4 3 ?
Illustration 1 In the following general example We determine the amount of COi (product) based on
A +3B---) 2C each reactant. The reactant giving the smaller amount of
1 mole A reacts with 3 moles B to give 2 moles C and 1 mole
COi will be the limiting reactant.
D. Tims, stoichiometric requirement for the reaction to go 2 moles CO give=2 moles COi
completely, is that molar ratio of A and B should be 3 : 1
4 moles CO give= 4 moles COi
nA =3
1 mole Oi gives = 2 moles COi
ns
3 moles Oi gives = 6 moles COi
Solution
Thus, smaller quantity of the product (C0i) is from
Case I: If
CO, which is thus limiting reactant and Oi is excess
n8 =2 reactant.
· nA ___1 How much excess reactant (�) is left, is decided
l.e.,
n8 2 by stoichiometric ratio of the reactants or from the
then Bis in smaller quantity, and amount of the products limiting reactant
is determined by B. Thus, B is limiting reactant. We 2 moles CO consume = 1 mole 0i
present the calculation as
Thus, 4 moles CO consume = 2 moles Oi
A + 3B---) 2C
Thus, �=�=== 4
Reaction says 1 mol 3 mol 2 mol n 02 1 2
We take 1 mol 2 mol ?
1 mole A forms = 2 moles C Thus, Oi left unreacted = 3 - 2= 1 mole
3 moles B form = 2 moles C After reaction is completed, reaction mixture contains
4 molesCOi (product) and 1 mole Ci (unreacted reactant)
Thus 2 moles B form = i = 1.33 moles C < 2 moles C
3
264 | Essential Inorganic Chemistry

We can represent the above change as Illustration 4 Urea (NH2)2CO, is prepared by­
2C0(g)+02(g) * 2002(g) reacting ammonia with carbon dioxide.
Stoichiometry 2 1 2 2NH3(g) + CO2(g) ---- > (NH2)2CO(a<7) + H2O(0
Initial moles 4 3 0 In one process, 637.2 g o/'NH3 are allowed to react with
Change -2 +4 1142 g o/*CO2 (a) Which of the two reactants is the limiting
After complete reaction 0 1 4 reagent? (6) Calculate the mass of (NH2)2CO formed
(c) How much of the excess reagent (in gram) is left at the
Illustration 3 We want to calculate the amount ofCO2
end of the reaction?
formed by the reaction of 16 g CH4 with 48 g Qz.
Solution Number of moles of
CH4 + 20a ----- > CQz + 2H2O NTTJ . 637.2 g
NH3 taken =---------- “ = 37.482 mol
Stoichiometry (moles) 1 2 1 2 17 g mol-1
We take 16 g 48 g ? ? 1142 g
Number of moles ofCO2 taken =
= 1 mole 1.5 mole ? ? 44 g mol-1
We do the calculation based on mole ratio :
, ,. 1 mole CHj 0.5 mole CHLj = 25.955 mol
Required ratio--------—— =-------- — 2NH3 + CO2 -----> (NH2)2CO+ h2o
2 moles O2 1 mole O2
(a) Limiting reagent is that one which is in smaller
Each mole of Qz would require exactly 0.50 mole of
quantity stoichiometrically.
CHj to be completely used up
2 mol NH3 react with = 1 mol CO2
Available ratio
1 mole CHj _ 0.67 mole CHj 37.482 mol NH3 react with
3 2
1.5 moles O2 1 mole O2
= 18.741 mol CO2
We have 0.67 mole of CFL for each mole of Qz. Thus,
Reversely,
there is Qz in small quantity and CQz formed is decided by
1 mol CO2 reacts with = 2 mol NH3
moles of O2
25.955 mol CO2 reacts with = 51.910 mol NH3
2 moles of give= 1 mole CO2
1.5 moles O2 gives = 0.75 mole CO2 Thus, NH3 is present in limiting quantity and CO? is
We do the calculation based on each reactant, and present in excess.
limiting reactant would be that give smaller amount of (b) Quantity of the product is always determined
CO2. from the limiting reactant.
1 mole CHj gives = 1 mole CQz 2 mol NH3 gives = 1 mol (NH2)2CO
on ^OO 1 XTTT • 37.482
2 moles O2 gives = 1 mole CO2 37.482 molNH3 gives =---------
2
1.5 moles Qz gives = 0.75 mole CQz (smaller)
- 18.741 mol (NH2)2CO
Thus, Qz is the limiting reactant.
(c) CO2 required by 37.482 mol ofNH3 = 18.741 mol
How much methane (CH4) is used :
Thus, CO2 left = 25.955 - 18.741
2 moles Qz consumes = 1 mole CHj
= 7.214 mol=7.21x44g
1.5 moles O2 consumes= 0.75 mole CH4 = 317.416 g
Thus, CHi left unreacted =1- 0.75
= 0.25 mole CH4 Illustration 5 Consider following reaction :
BaCl2(aq)+H2SO4(aq) -----> BaSO4(s) + 2HC«
= 4 g CH4
50 mL of 20.8% (w/V) BaCl2(aq) and 100 mL of 9.8^
(iy/V)H2SO4(aq) solutions are mixed. What is the amount
Numerical Problem Solving 4 o/BaS04(s) formed1 ? Which reactant is left over after the
In the following reaction, calculate the amount formed in reaction? (Ba = 137, Cl = 35.5, S = 32, H = 1,0 = 16) .
each case
N2(g) + 3H2(g) 2NH,□ Solution Number of grams of BaCl2 present in 50 mL
1 3 ? 20 8
Case I aqueous solution which is 20.8% (w/V) = x 50
Case II 2 3 ?
2 4 ? = 10.4 g
Case HI
1?^=0.05 mol
Case IV 1 1 ? moles of BaCl2 =
208
Chapter 6: Chemical Equations and Reactions I 265
Number ofgrams ofH2SO4 present in 100 mL Take 2 moles of KClO3(s) and assume product is
aqueous solution which is 9.8% w/V =9.8 g 2 moles 0i(g) due to side reaction, then per cent yield of
0i(g) is
moles ofH2SO4 = 9·8 = 0.1 mol
. 98 = actual yield x 100
BaCI2 (aq) + H2 SO4 (aq) --+ BaSO4 (s) + 2HCI(aq) theoretical yield
2
1 mol 1 mol 1 mol
?
=-xlO0
0.05 mol 0.10 mol
1 molBaC12 gives = 1 mol BaSO4 = 66.67%
0.05 molBaC12 gives= 0.05 mol BaSO4 Illustration 2 Acetaldehyde (CH3CHO) can be
= 0.05x 233 g obtained by the oxidation or ethyl alcohol (CH3CH2 OH).
=11.65 g BaS04 There is no side reaction and all CH3CH2OH is
converted to CH3CHO
1 mol BaCI2 reacts with =1 mol H2SO4
CH3CH2OH --? CH3CHO + !:Ii
0.05 molBaC12 reacts with = 0.05 mol H2SO4
Thus, yield is 100%
But, H.iSO4 taken = 0.10 mol H2SO4 There can be side reaction giving acetic acid
Thus, H2SO4 left unreacted = 0.10 - 0.05 = 0.05 mol (CH3COOH)
= 0.05x 98 g = 4.9 g CH3CH2OH --+ CH3CHO --+ CH3COOH
Volume of I:IiS04 (aq) solution left= 50 mL which is due to no 1 mol 1 mol
side reaction
9.8%(w/V).
If there is also formation of 0.2 mol CH3COOH, then
yield of CH3CHO would be 0.8 mol
actual yield
Yields of Chemical Reactions Per cent yield ofCHa<JHO =
theoretical yield
x 100
The maximum mass of product that can be obtained
from a chemical reaction is the theoretical yield. = Q.S X 100
Generally, loss of product occurs during the isolation and =80%
purification steps. In addition, some reactions do not go
completely to products, and reactions are sometimes
complicated by giving more than one set of products. For Numerical Problem Solving 5
all these reasons, the actual yield of the product - the Acetyle chloride (CH 3COC1) can be reduced to acetaldehyde
mass of material that is actually obtained in the (CH3CHO) in presence of BaSO 4 'Pd - reaction is called
laboratory or a chemical plant - is always less than the Rosenmund reaction. Some ethyl alcohol (CH3CH 2OH) is also
theoretical yield. In all such cases, it is desirable to report obtained due to further reduction ofCH 3CHO (intoCH 3CH2OH) if
a per cent yield, which specifies how much of the experimental conditions are not efficient. In an experiment one
mole of CH 3COC1 gave 2.3 g CH 3CH 2OH along with CH 3CHO
theoretical yield was obtained, is defined as (main product). What is per cent yield ofCH3CHO?
. actual yield
Per cent yield=------x 100%
theoretical yield
Illustration 3 Diethyl ether(C4H10O), the "ether" used
medically as an anaesthetic, is prepared from ethyl alcohol
Illustration 1 KC1O 3(s) decomposes into KCl(s) and (Ci�OH) in presence of an acid
Oz(g) 2Ci�OH(l) 1:��c, Ci�OCz�(l) + H2 O(l)
There is complete conversion of KCIO3 into the
products Calculate the per cent yield if 46.0 g of ethyl alcohol
gives 30.0 g of ether.
KC1O3(s)--? 2KCl(s)+ 3�(g)
Initial 2 mol O 0 Solution 2CiH2OH(l) --? Ci�OCi�(l) +H2O(l)
Change -2 +2 +3 2 mol 1 mol 1 mol
92 g 74 g 18 g
After O 2 3
Thus, yield is 100% Theoretical yield
There can be side reactions 92 g ofCi�OH gives = 74 g ether with 100% yield
74
2KC1Os(s) re�:n> 2KCI(s)+ 3�(g) :. 46g ofCi�OH gives = x 46 = 37 g ether with
92
3KC1O4 (s)+ KCl 100% yield
4KC1Oa(s) re=��on > But, actual yield is= 30.0 g
266 | Essential Inorganic Chemistry

actual yield many industrial process. These are called sequential


per cent yield = x 100
theoretical yield reactions. The amount of desired product from each
QA
= — x 100 = 81.08% reaction is taken as the starting material for the neit
37 reaction. We consider following illustrations.
Illustration 4 A 15.6 g sample of CgH6 (.benzene) is Illustration 1 1: 2KC1O3---- > 2KC1 + 3O2
mixed with excess HN03 in presence o/H2SO4. We isolate II: 4A1 + 3O2 ---- > 2A12O3
18.0gofC^{^02 (nitrobenzene). What is the per cent yield
of CeHgNOa in this reaction? KCIO3 decomposes in step I to give O2, which in turn, is
used by Al to form A12O3 in step II. First we determine Q
Solution
NO2 formed in step I and then Al used by this O2 in step II. Both
reactions can be added to determine amount of KC1O3 that
h2so4 can give required amount of O2 needed for Al.
+ HN03 + H2O
Net: 2KC1O3+4A1---- > 2KC1 + 2A12O3
H2SO4 Thus, KC1O3=2A1.
C6H6 +hno3 ■>
C6H5NO2 + H2 O
1 mol 1 mol
78 g 123 g Illustration 2 I: CaO + 3C CaCj + CO
15.6 g ? (theoretical yield)
II: CaCj + 2H2O---- > Ca(OH)2 + CA
Theoretical yield
78 g ofC6H6 gives = 123 g CeHgNOz CaCj (calcium carbide) is prepared in step I. It is used to
123 prepare acetylene (C^H2) in step II. Suppose we want to
15.6 g ofC6H6 gives = -—x 15.6 C^NC^ determine amount of CaO that can give enough CaC2 to be
78
converted into required amount of . Amount of CaO is
= 24.6 g
determined in step I and then amount of C^ in step □.
Actual yield = 18.0 g We can relate CaO and Q2H2 stoichiometrically by writing
per cent yield=—actual-^e^— x 100 net reaction which is
theoretical yield
13 0 CaO + 3C + 2H2O---- > Ca(OH)2 + CA + CO
= —x 100=73.17%
24.6 Thus, CaO = C2H2
Illustration 5 The per cent yield for the following
Illustration 3 Purification of Ni in Mond’s process is
reaction carried out in carbon tetrachloride (CC14) solution
is 67%. based on the conversion of Ni (impure) into pure Ni(C0)4
using CO (step II) which in turn is obtained in step I
Br2 + Cl2 ---- > 2BrCl
(a) What amount of BrCl would be formed from the Step I: c+
C +H
h22O
o red hot > CO + H2
reaction 0/0.025 mole Br2 and 0.025 mole Cl2?
(b) What amount of Br2 is left unchanged? Step II: Ni + 4CO ---- > Ni(CO)4

Solution Br2 + Cl2 ---- > 2BrCl If we combine both steps then
1 mol 1 mol 2 mol 4C + 4H2O+Ni ---- > Ni(CO)4+4H2
0.025 0.025 0.050 mol
(a) Theoretical yield of BrCl = 0.050 mol
Numerical Problem Solving 6
actual yield of BrCl = 0.050 x
100 Take illustration (3) given above. What mass of Ni(C0>4
could be obtained from the CO produced by the reaction of 72 g
= 0.0335 mol carbon? Assume 100% reaction and 100% recovering in both
R7 cases. (Ni= 58.7)
(b) Br2 reacted= 0.025 x---- = 0.01675 mol
100
Br2 left unreacted = 0.025 - 0.01675 = 0.00825 mol Illustration 4 Consider the formation of cyanogen.
C2N2, and its subsequent decomposition in water given bj
Sequential Reactions the equations :
2Cu2+ + 6CN" ---- > 2[Cu(CN)2r + <%
Often more than one reaction is required to change
starting materials into the desired product. This is true for QjNa + H2O---- > HCN + HOCN
many reactions that we carry out in the laboratory and for
Chapter 6: Chemical Equations and Reactions | 267

How much hydrocyanic acid, HCN, can be produced 2KC1O3 ---- > 2KC1 + 30a
from 20.0 g of KCN, assuming 100% yield? CH* + 2Oz ---- > COz + 2H2O
(H=1,C=12,N= 14,K= 39) (K= 39,0= 16, C= 12, H= 1,C1 = 35.5)
Solution
Solution Step 1: 2KC1O3 —> 2KC1 + 3O2
Step 1: 2Cu2+ + 6CN" -----> 2[Cu(CN)2 F + Q>N2 Step II: CH* +202 -----> CCKj + 2H2O.
Step II: Q>N2+H2O 4 HCN + HOCN KCIO3 gives O2, and 0% in turn is used to combust CH*.
Net: 2Cu2++ 6CN~ + H2O ■> 2[Cu(CN)2T +HCN Thus, 62 is the intermediate reactant. To cancel out Og
multiply step II reaction by 1.5, thus,
+ HOCN
Thus, 6 mol KCN gives = 1 mol HCN Net reaction is
20 90 2KCIO3 + 1.5 CH* ---- > 2KC1 + 1.5 CQ, + 3H2O
~ mol KCN gives=-=^- mol HCN
65 6 65x6 1.5 mol of CH* is combusted by O2 supplied by
= 0.05128 mol HCN = 2 mol KC1O3
= 0.05128 x 27 g HCN 33 2
hence, —— mol of CH4 is combusted by Q2 supplied by
16
= 1.38462 g HCN
2 33.2 .
= — x----- mol KCIO3
Illustration 5 What mass of potassium chlorate 1.5 16
(KCIO3) would be required to supply the proper amount of = 2.767 mol KC1O3
oxygen needed to burn 33.2 g of methane (CH*)?
= 338.92 g
r-

Target Practice 2
1. Which is the limiting reactant in the following? 4. 50 g of calcium carbonate sample, CaCO3, (lime stone) gave
1.12 L carbon dioxide gas measured at STP. (1 mole of
N2 (g) + 3H2(g) 2NH3(g)
every ideal gas at STP has 22.4 L volume)
14 g 3g
What is per cent purity ofCaCO3?
2. What is the amount of the product, NH3 in the question (1)
5. Take following sequential reactions :
react? 2H2O(Z)----- > 2H,(g) + O2(g)
i

i
3. Carbon monoxide (CO) and hydrogen (H2) react to form N2(g)+ 3H2(g)----- > 2NH3(g)
methanol (CH3OH) CaCO3(s) ---- > CaO(s) + CO2(g)
CO(g) + 2H2(g)---- > CH3OH(Z) NH3(g) + CO2(g) + H2O(0 ---- > NH4HCO3(aq)
We want to produce 0.10 mol NH4HCO3.
How much CH3OH(Z) obtained from 0.01 mole CO(g) and H2O(Z) required is............. j
I 0.08gH2(g)?
N2(g) required is.............

Answers
1. N2(g) + 3H2(g) ---- » 2NH 3 But actual yield = 1.12 L CO2 from 5.0 g pureCaCO3
1 mo? 3 mol 2 mol 5 J
Per cent purity = —- x 100 = 10% pure
28 g 6g 34 g
14 g 3g 17 g
! 5. (a) 2H2O(Z) ---- > 2H,(g) + O2(g)]x0.75
Both N2(g) and H2 (g) are in exact quantity.
2. 17 g (as in question (1)) (b) N2(g)+3H2(g) ---- > 2NH3(g)]x 0.50
3. CO(g) + 2H2 (g) ---- > CH3OH(Z) (c) CaCO3(s) ---- > CaO(s) + CO2(g)j
1 mol 2 mol 1 mol NH3(g)+CO2(g) + H2O(D NH*HCO3(ag)
0.01 mol 0.02 mol 0.01 mol
0.02 mol 0.04 mol 0.02 mol 0.75(a) + 0.50(b) gives
C0(g) = 0.01 mol 0.5 N2(g) + 1.5H2O(Z) ---- > NH3(g)+0.75O2(g) ■ >

H2 (g) = 0.08 g = 0.04 mol


Thus, N2(g) required = 0.05 mol for 0.1 mol NH4HCO3
Thus, CO(g) is limiting reagent
H2O(Z) required = 0.15 mol in (a) and (b) + 0.10 mol in final
CH3OH(Z) formed = 0.01 mol = 0.32 g
reaction = 0.25 mol
CaCO3 - • > CaO + CO2
1 mol (100 g) 1 mol = 22.4 L at STP 3
50 g 11.2 L CO2 ■_ -ai
268 | Essential Inorganic Chemistry

Practice Exercise 2
1. The per cent yield for the reaction In one experiment 1.0 mol of NO is mixed with 1.0 mol ofO2
PCI.o + Cl,Z ---- > PCL0 (a) Calculate which of the two reactants is the limiting
reagent.
is 83.2%. What mass of PC15 would be expected from the (b) Calculate also the number of mol of NO2 produced.
reaction of 56.7 g of PC13 with excess Cl2?
9. Select correct alternate. Only one alternate is correct.
(P= 31, Cl = 35.5)
(A) In the following reaction :
2. Calcium carbide is made in an electric-furnace by *'-j MnO2 + 4HC1---- > MnCl2 + 2H2O + Cl2
reaction:
2 moles MnO2 reacts with 4 moles of HC1 to form
CaO + 3C ■ ■-■) CaC2 + CO 11.2 L Cl2 at STP. Thus, per cent yield ofCl2 is
The crude product is usually 85% CaC2 and 15% unreacted (a) 25% (b) 50%
CaO. (a) How much CaO should we start with to produce (c) 100% (d) 75%
450 kg of crude product? (b) How much CaC, would this
crude product contain? (Ca = 40, C= 12,0= 16) (B) 2 moles of 50% pure Ca(HCO3)2 on heating forms 1
mole of CO2. Thus, per cent yield of CO2 is
3. If 1.87 g of acetic acid (CH3COOH) reacts with 2.31 g of
isopentyl alcohol (C5H]2O) to give 2.96 g isopentyl acetate Ca(HCO,)2 CaO + HZ2O + 2CO2Z
v Z

(C7H14O2), what is the per cent yield of the reaction? (a) 50% (b) 75%
CH.COOH
J
+ C,H
0
12O---- > C7i H14
IX
uO,Z + H,0
Z (c) 80% (d) 100%
4. Hydrogen, obtained by the electrical decomposition of (C) Mixture of 1 mole of Na2CO3 and 2 moles of NaHCOj
water, was combined with chlorine to produce 62.5 g of forms 1 mole ofCO2 on heating. Thus, per cent yield of
hydrogen chloride. Calculate the mass of . water CO2 is
decomposed. (a) 25% (b) 50%
2H2O--- > 2H2 + O2 (c) 75% (d) 100%
H2 + Cl2 > 2HC1 (D) Per cent yield of NH3 in the following reaction is 80%-
5. Urea, NH2CONH2, is formed when CO2 reacts with NH3 NH2CONH
Z Z
2 + 2NaOH NaZ2COO3 + 2NH3w
CO2 +2NH3 ---- > NH2CONH2 +h2o 6 g NH2CONH2 reacts with 8 g NaOH to form NH3
NH3 and CO2 are obtained in the following steps (a) 3.4 g (b) 2.72 g
I: N2z + 3H2z 2NH,o (c) 4.25 g (d) 11.2 g

II: CaCO3 50% CaO + CO2 (E) 100 mL solution of NaOH (containing 4 g NaOH per
litre) and 50 mL of HC1 (containing 7.3 g HC1 per litre)
Calculate amount of N2 and CaCO3 to produce 60 kg of react as:
urea. NaOH(aq)+HCl(aq) ---- > NaCl(aq) + H2O(Z)
6. The reaction between aluminium and iron (HI) oxide can 0.5 g of NaCl is formed. Thus, unreacted NaOH is
generate temperature approaching 3000°C and is used in
(a) 0.058 g (b) 3.66 g
welding metals:
(c) 10.8 g- (d) 0.63 g
2A1 + Fe2O3 ---- > A12O3 + 2Fe
(F) Al and KC1O3 react together to form A12O3 according to
In one process 124 g Al are reacted with 601 g of Fe2O.3’
(a) Calculate the number of moles of A12O3 formed. 2KC1O, ---- > 2KC1 + 30.2
(b) How much of the excess reagent is left at the end of the 4A1 + 3O2 ---- > 2A12O3
reaction?(Al= 27,Fe= 56,0= 16) 4 moles ofKC1O3 (50% pure) on reaction with excess of
7. H2O can be obtained from by the reaction Al forms A12O3
(a) 2 mol (b) 4 mol
2H2 + O2 ---- > 2H22O (c) 6 mol (d) 8 mol
If we take 1 mol O2 and 1 gH2, (G) NaOH is formed according to reaction
(a) Which is the limiting reactant? 2Na +- O2 ---- > Na2O
(b) How much of the excess reactant is left unreacted? 2
(c) How much of H2O is formed? Na2O + H2O---- > 2NaOH
8. Nitric oxide (NO) reacts instantly with oxygen gas to form To make 4 g NaOH, Na required is
nitrogen dioxide (NO2), a dark-brown gas (a) 4.6 g (b) 4.0 g
2NO(#)+ O2(£)---- » 2NO2(g) (c) 2.3 g (d) 0.23 g
Chapter 6: Chemical Equations and Reactions | 269

(H) NH3 is formed in the following steps (a) 0.67 mol (b) 1.12 mol
I: Ca + 2C---- > CaC2 50% yield (c) 1.33 mol (d) 2.66 mol
(J) NaOH can be prepared by two methods each with two
II: CaC2 + N2 ---- > CaCN2 + C 100% yield
steps having 100% extent.
III: CaCN, + 3H,0---- > 2NH, + CaCO, 50% yield
I: 2Na + 2H2O-- > 2NaOH + H2
To obtain 2 moles NH3, calcium required is 2H2 + O2 -- > 2H2O
(a) 1 mol (b) 2 mol
II: 2Na+-O, -- > Na,0
(c) 3 mol (d) 4 mol 2 2 2
(I) MnO, + 2KC1CL ---- > K2MnO4 + Cl2 + 2O2 Na2O + H2O 2NaOH
Cl2 + K2MnO4 ---- > 2KC1 + MnO2 + O2 Which gives better yield?
Each reaction takes place to the extent of 50%. 11.2 L (a) I (b) II
(c) Both equal (d) None is suitable
ofO2 at STP is obtained fromKClO3 using

Answers
1. 71.53 g 2. (a) 393.75 kg (b) 382.50 kg 7. (a) H2 (b) O2 left = 24 g = 0.75 mol (c) 9 g H20
3. 86.74% 4. 15.411g 8. (a) NO is the limiting reactant (b) 1 mol N02
9. (A) (b) (B) (a) (C) (d)
(B) (a) (D) (b)
(D) (b) (E) (a)
5. 31.111 kg N2, 200 kgCaCO3
(F) (a) (G)(G) (c)
(c) (H) (d) (I) (a)
(I) (a) (J) (c)
6. (a) 2.296 mol (b) 1.46 mol
COMX1
r— •- A-*/ S* I ----- .

Total Practice Set


(Read, Plan & Solve'.)
s

Problem 1. Assume that you have 1.39 moles ofN2 and K2PtCl6 + 2NH3 Pt(NH3)2Cl2 +2KC
potassium cisplatin
3.44 moles of Hg. (a) How many grams of ammonia (NH3) tetrachloroplatinate
can you make? (b) How many grams of which reactant will
be left over? (c) How many grams of the limiting reactant Assume that 10.0 g of KgPtCl4 and 10.0 g of NH3 an
will be required to react with the reactant present in larger allowed to react.
amount? (a) Which reactant is limiting and which is in excess?
N2 + 3Hg 2NH3 (b) How many grams of the excess reactant an
consumed, and how many gram remain?
N2 + 3H2 4 2NH3
1 mol 3 mol 2 mol (c) How many grams of cisplatin are formed?
(Pt= 195,K= 39, Cl = 35.5,N= 14,H= 1)
Plan N2 and Hg react in 1:3 mol ratio. Reactant present
in smaller quantity (in terms of mol) stoichiometrically is Plan Given amounts are converted into moles. Limiting
the limiting reactant and the other present in larger reactant is that which is present stoichiometrically m
quantity is left over. Thus, amount ofNH^ reactant left smaller amount. Quantity of the product is alwayi
over and the reactant required for complete reaction can be determined from the limiting reactant.
calculated.
Solution Number of moles of
Solution Number of moles of N2 = 1.39 mol
KgPtCl4 = 10 g = 0.0241 mol
Number of moles of Hg taken = 3.44 mol 415 g mol
Number of moles of Hg required by 1.39 moles of
Number of moles of NH3 =— = 0.5882 mol
N2 =1.39x3 = 4.17 mol 17
Thus, Hj is present in smaller quantity and is thus Moles ofNH3 required by 0.0241 moles of KgPtCl4
the limiting reactant.
= 0.0482 mol
(a) 3 mol of H2 gives = 2 mol NH3
2 but, NH3 taken = 0.5882 mol » 0.0482 mol
3.44 mol gives =—x 3.44
(a) Thus, KgPtCl^ is the limiting reactant and NHj
= 2.293 mol NH3 is in excess.
(b) 3 mol ofHg react with= 1 mol N2 (b) NH3 consumed = 0.0482 mol = 0.0482x 17 g
3.44 mol Hg react with=x 3.44 = 1.147 mol N2 = 0.8194 g
NH3 left = 10 - 0.8194 = 9.1806 g
Thus, number of N2 left over = 1.390 - 1.147
(c) 1 mole of KgPtCl4 gives = 1 mol of Pt(NH3)2Cl2
= 0.243 mol cisplatin
(c) Number of moles of .’. 0.0241 mole KgPtCl4 gives = 0.0241mol Pt(NH3)2ClJ;
Hg required by 1.39 mol N2 = 4.17 mol = 0.0241 x 300 g
Number of moles ofHg actually present = 3.44 mol = 7.23 g
Additional Hg required = 4.17 - 3.44
= 0.73 mol Problem 3. At high temperatures carbon reacts with
water to produce a mixture of carbon monoxide, CO ad
Problem 2. Cisplatin, an anticancer agent used for the hydrogen, Hg.
treatment of solid tumours, is prepared by the reaction of C + HgO CO + H,
ammonia with potassium tetra chloroplatinate:
Chapter 6: Chemical Equations and Reactions ? 271
QQ K

CO is separated from H^, and then used to separate Actual ROI0 formed = 2.1935 x —--
‘ 10 100
nickel from cobalt by forming a volatile compound, nickel
= 1.9632 mol
tetracarbonyl, Ni(CO)4.
Ni + 4CO---- > Ni(CO)4 In Step II: Theoretical H3PO4 formed
What mass of Ni(CO)4 could be obtained from the CO = 4 x naoles of P4
produced by the reaction of 75.0 g of carbon? Assume 100% = 4x 1.9632 mol H3PO'44
reaction and 100% recovery in both steps. (Ni= 59) 96 8
Actual H;jPO4 formed = 4 x 1.9632 x y—
Plan We determine the number of moles of CO formed in
the first reaction. = 7.6016 mol
From the number of moles of CO produced in the first = 7.6016 mol x 98 g mol
reaction, we calculate the number ofgrams o/Ni(CO)4 that = 744.96 g
would be formed in the second reaction. Or, if we combine
both the reactions
Problem 5. Lime stone (CaCO3) reacts with
4C + 41^0 + Ni---- > Ni(CO)4 + 4^ hydrochloric acid according to the equation:
we find that 4 moles carbon gives = 1 mole of Ni(CO)4. CaCO3 + 2HC1---- > CaCl2 + H2O + CO2
gC—> molC---- > mol of CO---- > molofNi(CO)4
If 1.00 mole of CO2 has a volume of 22.4 L under the
Solution Number of mol of carbon reaction conditions, how many litres of gas can be formed
=__ 75 g — = 6.25 mol by the reaction of 2.35 g of CaCO3 with 2.35 g of HC1?
12 g mol 1 Which reactant is limiting?
Plan Quantity of the product is always determined from
4 mol of carbon give= 1 mol of Ni(CO)4 the reactant present stoichiometrically in smaller amount
6 25
hence, 6.25 mol of carbon give=—— mol of Ni(CO)4 (expressed in terms of moles)
4
Reactant present in smaller amount is the limiting
171g reactant. (Ca = 40, C= 12,0= 16, Cl = 35.5,H= 1)
4
Solution CaCO3 +2HC1 CaCl2 + H2O + CO2
= 267.2 g 1 mol
1 mol 2 mo1
22.4 L
Problem 4. Phosphoric acid, (H^C^) is widely used to 9
CaCO, =— = 0.0235 mol
make fertilizers, and can be prepared in a two-step 3 100
process. 9 35
HC1=—=0.0644 mol
Step I: P4+5O2---- > P4O10 36.5

Step II: P4O10 + 6H2O---- > 4H3PO4 1 mol CaCO3 requires = 2 mol HC1
We allow 272 g of phosphorus to react with excess 0.0235 mol CaCO3 requires = 0.0470 mol HC1
oxygen, which forms tetraphosphorus decoxide (P4O10) in HC1 taken = 0.0644 mol
89.5% yield. In the step II reaction, 96.8% yield ofH^Qi is Thus, HC1 is present in excess and amount of the
obtained. What mass of H3PO4 is obtained? product is determined from CaCO3, which is the limiting
(P= 31,H= 1,0= 16) reactant.
Plan We convert amount of P4 given into number of 1 mol CaCO3 gives = 1 mol C02 = 22.4 L C02
moles. Also 1 mole P4 s 1 mole P4O10 thus, number of moles 0.0235 mol CaCO3 gives = 0.5264 L C02
(with 89.5% yield) o/'P4O10 are known.
Also 1 mole o/‘P4O10 gives 4 moles of H3PO4 in step II. Problem 6. What is the per cent free SO3 in an oleum
Thus, amount o/‘H3PO4 is also known (with 96.8% yield). (H2S2O7) that is labelled “109% H2SO4” ?
Plan Such a label refers to the total mass of pure
Solution In Step I: Number of mol of P4
H2SO4, 109 g, that would be present after dilution of 100 g
=--- ?72g— = 2 1935
of the oleum, when all free S03 would combine with water
4x 31 g mol
to form H2SO4.
TheoreticalP4O10 formed= 2.1935 mol
272 | Essential Inorganic Chemistry

Solution 9 g of JLjO is mixed with 100 g oleum 3x 233 x 0.7160


= 0.7002 g (given)
(2M+612)
HgO + SO3 ---- > H2SO4
(2M + 612) = 714.77
Since, 18 g combines with 80 g SO3
2M = 102.77
9 g H2O combines with 40 g SO3
M = 51.38
thus, 100 g of oleum has 40 g SO3
40% free SO3 in oleum Problem 8. A 0.4852-g sample of iron-ore is dissolve
in acid, and iron is oxidised to the +3 state and the
Problem 7. A 0.7160 g of a hydrated metallic sulphate precipitated as the hydrous oxide, Fe2O3 • xE^O. Th
of formula Af/SQ,)^ 18H2O precipitated 0.7002 g of precipitate is filtered, washed and ignited to Fe-jOg, whic.
BaSO4. When mixed with KgSQj and crystallised, the is found to weigh 0.2481 g. Calculate the percentage c
metallic sulphate yielded an alum isomorphous with iron (Fe) in the sample. (Fe = 56,0 = 16)
potash alum. Find the atomic weight of the metal. Plan Only Fe(II) in the pure state after oxidation 4
Plan Mx(SO4)v18H2O is isomorphous with alum with Fe(III) is precipitated as Fe2O3 xl^O which gives Fe^O;
formula K2SO4 A12(SO4)3-241^0 thus, x = 2, and y = 3. From the weight ofFe^O3,percentage of iron in iron-ore cat
From the stoichiometric amount of BaSO4, and given be determined.
amount o/'BaSO4, atomic weight ofM is determined.
Solution Let iron in iron-ore be = w g
Solution Crystallised substance formed with K^SC^ is
isomorphous with alum, 2Fe(II)---- > 2Fe(III) Fe2O3 xH2O---- > FegOg
2x56 g 160 g
K2SO4-Al2(SO4)3-24 HjO wg 0.2481 g

Thus, M is trivalent, M3+ and hydrated metallic w 0.2481


Thus,
sulphate is M2(SO4)3.18H2O 2x56 160
M2(SO4)3-18H2O + 3BaCl2 ---- > 3BaSO4 +.... 0.2481
w =-------- x 2 x 56 g pure Fe(II)
160
1 mol of hydrated salt gives = 3 mol of BaSO4
= 0.1737 g in 0.4852 g sample
or (2Af + 612) g of hydrated salt gives = 3x 233 g BaSO4
0.1737 x 100
0.7160 g of hydrated salt gives Thus, percentage of Fe(II) =
0.4852
3x233
x 0.7160 gBaSO4 = 35.80
"(2M +612)
Master Exercises
I

Exercise 1
(Stage 1: Learning)
Short Answer Type Questions 11. What mass of potassium can be produced by the
1. Classify following as soluble or insoluble salts in reaction of 150.0 g of Na with 150.0 g of KC1?
water. FeS, PbS, CaSO4, NaOH, CaO, CaCO3,CaC2O4 Na+KCl NaCl+K
2. What is driving force in a precipitation reaction?
12. How much 74.1% pure Na2SO4 could be produced
3. What is driving force in a neutralisation reaction? from 245 g of 93.9% pure NaCl?
4. One mole of HCl(aq) is neutralised by one mole of 2NaCl+H2SO4 ---- > Na^ +2HC1
NaOH(ag). If we take Ca(OH)2(ag), then how many
moles ofCa(OH)2 are required?
Analytical Questions
5. Calculate moles of H2SO4 required to complete
reaction of
Stoichiometry
(i) lmoleCa(OH)2 1. Dichloromethane (CHjC^), used as a solvent, is
(ii) 1 mole KOH prepared by reaction of methane (CH4) with chlorine
6. Consider the reaction (Cl2). How many grams ofCH2Cl2 result from reaction
of 1.85 kg of methane if the yield is 43.1%?
2NO + Bi2 ---- > 2NOBr
2. 4 g of a mixture of dry sodium carbonate and sodium
For every 5.00 moles of bromine that reacts, how
many moles of bicarbonate containing 75% of the latter was heated
until there was no loss in the weight. Calculate
(a) NO react and
volume of C02 evolved at STP.
(b) NOBr are produced?
3. Find the volume of hydrogen which would be evolved
7. What mass of C1O2 is required to produce 8.36 kg of
at STP by reacting zinc with 50 mL of 40% pure
HC1O3 by the following reaction? dil.HjaSQj (density = 1.3 g/ mL).
2C1O2 + 0,0---- > HCIO3 + HC1O2
8. We allow 32.0 g of methane, CH4, to react as Limiting Reactants
completely as possible with excess of O2, to form CO2 4. Lithium oxide, (Li2O) is used aboard the space shuttle
and H2O. to remove water from the air supply according to the
(a) Write the balanced equation equation
(b) What mass of oxygen will react?
Li2O(s) + H^g)---- > 2LiOH(s)
9. What mass of pentane, C^H^, produce 3.01 x IO22 CO2 If there are 80.0 kg of water to be removed and 65 kg
molecules when burnt is excess oxygen O2. of Li2O available, which reactant is limiting? How
10. Lime, CaO, dissolved in muriatic acid, HC1, to form many kilograms of the excess reactant remain?
calcium chloride, CaCl2, and water, H^O (Li= 7,0= 16,H= 1)
(a) How many moles of HC1 would be required to 5. Consider the reaction
dissolve 8.8 moles of CaO
MnOa +4HC1 MnCl2 + Cl2 + 21^0
(b) How many moles of H^O would be formed?
274 [ Essential Inorganic Chemistry

If 0.86 mole ofMnO2 and 48.2 g of HC1 react, which Sequential Reactions
reagent will be used up first? How many gram ofCl2
8. The chief ore of zinc is the sulphide, ZnS. The ore is
will be produced? (Mn =55,0= 16, Cl = 35.5)
concentrated by floatation process and then heated in
air, which converts the ZnS to ZnO.
Yields of Chemical Reactions
I. 2ZnS + 3O2 2ZnO + 2SO2
6. Ethyl alcohol ((XH^OH) is prepared industrially by
the reaction of ethylene (C,H4) with water. What is The ZnO is then treated with dilute H2SO4
the per cent yield of the reaction if 9.6 g of ethylene II. 2ZnO + 2H2SO4 - ' 2ZnSO4 + 21^0
gives 4.7 g of ethyl alcohol?
to produce ZnSO4(aq) which produces Zn metal on
C>H4(g) + H2O(Z)---- > QHOHfZ) electrolysis.
7. Industrially, vanadium metal can be obtained by III. 2ZnSO4 + 2H2O 98.2%, 2Zn + 2H2SO4 + O2
reacting vanadium (V) oxide with calcium at high
What mass of Zn will be obtained from an ore
temperatures containing 225 kg of ZnS? Efficiencies of the processes
5Ca + V2O5 ---- > 5CaO + 2V have been indicated above the arrow mark. (Zn= 65,
S=32,O= 16, H= 1)
In one process, 1.54xl03gof V2O5 react with
1.96 x 103 g of Ca, (a) Calculate the theoretical yield 9. How many grams of calcium carbonate are required to
of V. (b) Calculate the per cent yield if 803 g of V are produce enough carbon dioxide which can convert
obtained. (Ca= 40, V= 51,0= 16) 0.5 mole of sodium carbonate into sodium
bicarbonate?

Exercise 2
(Stage 2: High Skill Problem Solving)
Only One Option Correct 5. Reactant present in excess (with extra number of
1. What mass of C02 could be formed by the reaction of moles of that reactant) in question (4) above is
16 g CH4 with 48 g of 02? (a) BaCl2, 0.3 mol (b) (NH4)3PO4, 0.2 mol
CH4 + 2O2 ---- > C02 + 21^0 (c) (NH4)3PO4, 0.1 mol (d) BaCl2, 0.2 mol
(a) 44 g (b) 33 g 6. In the following reaction,
(c) 16 g ' (d) 24 g 2H2S + SO2 ---- > 2H0O + 3S
2. For the formation of 5.00 mol of water, which reaction 1 mol each of SO2 and H^S will give sulphur
uses the most nitric acid? (a) 1 mol (b) 3 mol
(a) 3Cu + 8HNO3 - * 3Cu(NO3)2 + 2NO + 4^0 (c) 1.5 mol (d) 2 mol
(b) AI2O3 + 6HNO3 2A1(NO3)3 + 3H2O 7. One mole BaF2 is treated with two moles of H>S04 .To
(c) 4Zn + IOHNO3 - -» 4Zn(NO3)2 +NH4NO3 make the resulting mixture neutral, NaOH is added.
+ 3H2O NaOH required in this process is
(d) Cu + 4HNO3 ---- > Cu(NO3)2 +2NO2 + 21^0 (a) 4 mol NaOH (b) 2 mol NaOH
(c) 3 mol NaOH (d) 1 mol NaOH
3. One mole of SO2C12 is dissolved in water and Ca(OH)2
is added to neutralise acidic solution. Moles of 8. Temporary hardness is due to HCO3 ofMg2+ andCa24.
Ca(OH)2 required are It is removed by addition of CaO.
(a) 2 (b) 3 Ca(HCO3)2 + CaO---- » 2CaCO3 +H^O
(c) 4 (d) 5 Mass of CaO required to precipitate 2 g CaCO3 is
4. 0.5 mole ofBaCl2 is mixed with 0.2 mole of(NH4 )3PO4. (a) 2.00 g (b) 0.56 g
Maximum number of moles of barium phosphate (c) 0.28 g (d) 1.12 g
formed in this reaction is 9. Hardness of water is measured in terms of ppm (parts
(a) 0.1 (b) 0.2 per million) ofCaCO3. It is the amount (in g) ofCaCOj
(c) 0.3 (d) 0.5 present in 106 g HjO. In a sample of water, 10 1
Chapter 6: Chemical Equations and Reactions | 275

required 0.56 g of CaO the remove temporary 18. The oxide of an element contains 67.67% of oxygen.
hardness of HCO3 Equivalent weight of the element is
Ca(HCO3)2 + CaO ■> 2CaCO3 + HgO (a) 2.46 (b) 3.82
Temporary hardness is (c) 4.36 (d) 4.96
(a) 200 ppm CaCO3 (b) 100 ppm CaCO3 19. Which pair has same percentage of carbon?
(c) 50 ppm CaCO3 (d) 25 ppm CaCO3 (a) CH3COOH and C6H12O6
10. Permanent hardness is due to SO^ SO^~- and Cl“ of Ca‘2+ (b) CH3COOH and C12H22O11
and Mg2+ and is removed by the addition ofNa2CO3 (c) CH3COOH and C2H5OH
CaSO4 + Na2CO3 ---- > CaCO3 + Na2SO4 (d) C6H12O6 and C^H^Ou
CaCl2 + Na2CO3 ---- > CaCO3 + 2NaCl 20. The mass of 70% HaSO* required for neutralization of
If hardness is 100 ppm CaCO3, amount of Na2CO3 1 mole of NaOH is
required to soften 10 L of hard water is (a) 70 g (b)35g
(a) 2.12 g (b) 0.106 g (c)30g (d) 95 g
(c) 10.6 g (d) 1.06 g
21. 1000 g CaCO3 solution contain 10 g carbonate. The
11. All the oxygen in a 0.5434-g sample of a pure oxide of
concentration of solution as carbonate is
iron is removed by reduction in a stream of The
loss in weight is 0.1210 g. Hence, formula of the iron (a) 10 ppm (b) 100 pm
oxide is : (Fe = 56) (c) 1000 ppm (d) 10000 ppm
(a) FeO (b) Fe2O3 22. 800 g of a 40% solution by weight was cooled. 100 g of
(0 Fe3O4 (d) FeO2 solute precipitated. The percentage composition of
12. A certain sample of phosphate rock contains 26.26% remaining solution is
P2O5. A 0.5428 g sample is analysed by precipitating (a) 31.4% (b) 20.0%
MgNH4PO4 • 6H2O and igniting the precipitate to (c) 23.0% (d) 24%
Mg2P2O7.Thus, Mg2P2O7 obtained is 23. The hydrogen phosphate of certain metal has AfHPO'44
(a) 0.8486 g (b) 0.1424 g
formula. The formula of metal chloride is
(c) 0.3648 g (d) 0.2228 g
(a) MCI (b) M2C1
13. The calcium in a 0.8432-g sample is precipitated as
CaC2O4. The precipitate is washed, ignited to CaCO3 (c) MC13 (d) MC12
and found to weigh 0.3462 g. Percentage of CaO in the 24. A bivalent metal has 37.2 equivalent weight. The
sample is molecular weight of its chloride is
(a) 11.5% (b) 23% (a) 216.6 (b) 148.8
(c) 18% (d) 46% (c) 145.4 (d) 172.8
14. Number of millilitres of a 1.6% BaCl2 (w /V) solution 25. A body drinks 500 mL of 9% glucose solution. The
which required to precipitate the sulphur asBaSO4 in number of glucose molecules he has consumed are
a 0.60-g sample that contains 12% S is [mol. wt. of glucose = 180]
(a) 58.50 mL (b) 14.63 mL (a) 0.5 xlO23 (b) 1.0 xlO23
(c) 29.25 mL (d) 21.00 mL (c) 1.5xlO?3 (d) 2.0xlO?3
15. An iron sample contains 18% Fe3O4. What is the
amount so that it is precipitated as Fe^ which One or More Than One Options Correct
weighs 0.40 g? 1. 1 mole aqueous solution of SO2C12 is neutralised by
(a) 2.15 g (b) 1.075 g
(a) 2 mol of Ca(OH)'22 (b) 1 mol of Ca(OH)2
(c) 4.30 g (d) 2.01 g
(c) 2 mol KOH (d) 4 mol KOH
16. An aqueous solution of glucose is 10%. The volume in
2. The molar mass of haemoglobin is about 65000 gmol-1.
which 1 g mol of it is dissolved will be
Every haemoglobin contains 4 iron atoms. Thus
(a) 0.9 L (b) 9 L ’
(a) iron content in haemoglobin is 0.35% by mass .
(c) 1.8 L (d) 18 L
(b) 1 mole of haemoglobin contains 56 g iron
17. In diammonium phosphate (NH4)2HPO4, the (c) 1 mole of haemoglobin contains 224 g iron
percentage of P2O5 is (d) if iron content is increased to 0.56%, molar mass of
(a) 35.87 (b) 46.44 haemoglobin would be higher than 65000 g mol-1.
(c) 51.99 (d) 53.78
4
4,0 I essential Inorganic Chemistry

3. In one process giving below AgNO3 solution gave 14.35 g of white precipitate
2NH3(g) + CO2(g) ---- > NH2CONH2(aq) + H2O(Z) Hence,
(a) ionic mass of M+ is 18 g mol-1
637.2 g ofNH3 are allowed to react with 1142 g ofCO2.
In this (b) mixture has equal mole fraction of MCI and KC1
(a) NH3 is the limiting reactant and CO2 is in excess (c) KC1 and MCI when separately treated with
AgNO3 solution would give same number of moles
(b) NH3 is in excess and CO2 is the limiting reactant
of while precipitates
(c) 1124 g of urea is formed
(d) AgCl is water soluble
(d) 319 g of CO2 is left unreacted
4. If 0.80 mole of MnO2 and 146 g of HC1 react
Brain Twisters
MnO2 +4HC1 —> MnCl2 +C12 +2H2O
CaCO3(s) + 2HCl(aq) ---- > CaCl2(ag)
then,
(a) 0.80 mole ofCl2 is formed CaCO3(s) + 2H+(aq) + 2Cl-(aq) ---- >
(b) 0.80 mole of HC1 remains unreacted [Ca2+(aq) + 2Cr(aq)] +
(c) MnO2 is completely reacts Thus, net reaction is
(d) MnO2 is the limiting reactant CaCO3(s) + 2H+(aq) ---- > Ca2+(aq) + H^CO^ng)
5. When 0.273 g of Mg is heated strongly in a nitrogen This is an exothermic reaction (in which heat is
(N2) atmosphere, a chemical reaction occurs. The evolved) and thus there is thermal decomposition of
product of the reaction weighs 0.378 g. Empirical carbonic acid into gaseous carbon dioxide and water.
formula of the compound Explain
(a) MgN (b) Mg3N2 H^OgCaq) ---- > HjOUJ + CO^)
(c) MgN2 (d) Mg2N 2. For each of the following write balanced
6. We have 1.6 g CH4, 1.7 g NH3 and 1.8 g H^O select (i) formulae unit (ii) total ionic, and
correct alternate(s) (iii) net ionic equations
(a) There are equal number of moles of each reactant (Refer Table 6.1)
(b) Total number of atoms in CH4 > NH3 > H^O (a) Black-and-white photographic film contains some
(c) Total number of H-atoms are in the ratio of 4 :3 :2 silver bromide, which can be formed by the
reaction of sodium bromide with silver nitrate.
(d) Total number of C-atoms in CH4 < that of N-atoms
in NH3 < that of O-atoms in HjO (b) Barium sulphate is used when X-rays of
gastrointestinal tract are made. It can be
7. Density of is 1 g mL-1. If we have 1 mLH^O then prepared by reacting barium chloride with dilute
(a) it is 55.55 mol L-1 suphuric acid.
(b) it has 3.33 x 1022 HjO molecules (c) In water purification, small solid particles are
(c) it has 3.33 x 1622 H-atoms often “trapped” as aluminium hydroxide
(d) it has 6.66 x 1022 O-atoms precipitates and falls to the bottom of the
sedimentation pool. Aluminium sulphate reacts
8. In diammonium hydrogen phosphate, (NH4)2HPO,’4> with calcium hydroxide (from lime) to form
percentage as aluminium hydroxide and calcium sulphate.
(a) P2O5 is 53.78% (b) NH3 is 25.76% (Do not use NaOH, why?)
(c) P is maximum (d) N is maximum (d) Our bones are mostly calcium phosphate. Calcium
chloride reacts with potassium phosphate to form
9. Mole fraction of ethanol (C^HgOH) is ethanol-water
calcium phosphate and calcium sulphate.
system is 0.25. Thus it has
(a) 25% ethanol by weight of solution 3. Aluminium building materials have a hard
transparent protective coating of aluminium oxide,
(b) 75% water by weight of solution
formed by reaction with oxygen in air. The sulphuric
(c) 46% ethanol by weight of solution acid, HaSO^ in acid rain dissolves this protective
(d) 54% water by weight of solution coating and forms aluminium sulphate and water.
10. 12.8 g of mixture of MCI (volatile) and KC1 (a) How many moles of H^SC^ are required to react
(non-volatile) on reaction with excess of aqueous AgNO3 with 4.0 moles of A12O3?
solution gave 28.7 g white precipitate. 12.8 g of same
mixture on a heating gave a gas that on passing into (b) How many moles of A12(SO4)3 are formed in (a)
Chapters: Chemical Equations and Reactions | 277

4. Calcium carbide is made in an electric furnace by the Passage Comprehension Questions


reaction.
CaO + 3C ---- > CaG, + CO Passage 1
When sulphuric acid dissolves in water, the following
The crude product is usually 85% CaCj and 15%
reactions take place
unreacted CaO.
H2SO4 ■> H*+HSO4 (100% ionisation)
(a) How much CaO should we start with to produce
450 kg of crude product HjSO; ■> H++SO^“ (10% ionisation)
(b) How much CaCj would this crude product We start with 0.2 M aqueous solution of H^SO^
Answer the following questions.
contain?
1. [SOt ] in the above case is
5. A mixture of CuSO4 and CuSO4 • 5H2O has a mass of
I. 245 g. After heating ot drive off all the water, the (a) 0.1 M (b) 0.01 M
mass is only 0.832 g. What is the mass per cent of (c) 0.2 M (d) 0.02 M
CuSO4 • 5H2O in the mixture? 2. [HSO4 ] in the above case is
6. Nickel forms a compound with carbon monoxide, (a) 0.1 M (b) 0.09 M
Nix(CO)r To determine its formula, you carefully (c) 0.18 M (d) 0.08 M
heat a 0.0973 g sample in air to convert the nickel to
3. [H3O+] in aqueous solution is
0.0426 g of NiO and the CO to 0.100 gCO2. What is the
empirical formulae of Nix(CO)y [Ni = 58.7] (a) 0.28 M (b) 0.22 M
(c) 0.02 M (d) 0.10M
7. Sodium azide, the explosive chemical used in
automobile airbags, is made by the following reaction. 4. Maximum concentration is species
NaNO3 + 3NaNHa -- > NaN3 + 3NaOH + NH3 (a) H2SO4 (b) H3O+
(c) SOt (d) hs°4-
If you combine 15.0 g NaNO3 with 15.0 g NaNHj,
what mass of NaN3 is produced?
Passage 2
8. Silicon and hydrogen form a series of compounds with
About half of the world’s production of pigments for
general formula SiJIy. A 6.22 g sample of the
paints involves the formation of white TiO2. In the united
compound is burnt in oxygen. All the Si is converted to
states, it is made on a large scale by the chloride process,
II.64 g ofSiO2, and all of the H is converted to 6.980 g
starting with ores containing only small amounts of rutile,
of I^O. What is the empirical formula of the silicon
TiO2. The ore is treated with chlorine and carbon (coke).
compound? (Si = 28)
This produces TiCl4 and gaseous products.
9. Copper metal can be prepared from copper ore is 2TiO2 +3C + 4C12 7°°% » 2TiCl4 +CO2 +2CO
following steps:
I: CujSte) + O2(g) ----- > 2Cu(s) + SO2(g) The TiCl4 is then converted into TiO2 of high purity.
II: CuS(s) + O2(g) ------ > Cu(s) + SO2(g) TiCl4 +O2 93 0% > TiO2 +2C12

Suppose an ore sample contains 11.0% impurity in Efficiencies have been indicated above the arrow
addition to a mixture of CuS and Ct^S. Heating mark. (Ti = 48, Cl = 35.5,0 = 16, C = 12)
100.0 g of the mixture produces 75.4 g of copper metal Answer the following questions
with a purity of 89.5% what is the weight per cent of 1. If we start with 100 kg of ore (containing 7.5% rutile),
CuS and Ci^S in the ore? pure TiO2 obtained is
10. In some countries, a person will receive a “driving (a) 4.8825 kg (b) 7.3238 kg
while intoxicated” (DWI) ticket of the blood alcohol (c) 9.7650 kg (d) 2.4413 kg
level (BAL) is 100 mg per decilitre (dL) of blood or 2. Cl2 required in this reaction is
higher. Suppose a person is found to have a BAL of (a) 0.1221 kmol (b) 8.666 kg
0.033 mole of ethanol (C^HgOH) per litre of blood. Will (c) 4.333 kg (d) 0.1221 kg
a person receive a DWI ticket?
278 | Essential Inorganic Chemistry

Passage 3 Answer the following questions


A fluorine disposal plant was constructed to carryout 1. Which reactant is limiting and which is in excess?
the reactions Limiting Excess
F2 +2NaOH | O2 + 2NaF + H^O (a) K2PtCl4 NH3
(b) NH3 KjPtCl*
2NaF + CaO + H2O ---- > CaF2+2NaOH
(c) Exact molar ratio
As the plant operated, excess lime was added to bring (d) Would be decided by the quantity of the product
about complete precipitation of the fluoride as CaF2. Over a formed
period of operation, 2000 kg of fluorine were fed into a
plant and 10,000 kg of lime were required. (F = 19, Ca = 40,
2. Number of moles of KjPtClg consumed is
O = 16,H = 1, Na = 23) (a) 0.048 (b) 0.024
Answer the following questions (c) 0.012 (d) 0.096
1. What was the percentage utilisation of lime (CaO)? 3. Number of moles of NH3 consumed is
(a) 29.5% (b) 14.75% (a) 0.048 (b) 0.024
(c) 59.0% (d) 60.0% (c) 0.096 (d) 0.192
2. CaO used by given quantity of F2 is 4. Number of moles of cis-platin formed is
(a) 1475 kg (b) 2947.4 kg (a) 0.012 (b) 0.024
(c) 5900 kg (d) 6000 kg (c) 0.048 (d) 0.192
3. CaF2 precipitated is 5. Number of moles of excess reactant which remains
unreacted is
(a) 37.79 kmol (b) 263.16 kmol
(a) 0.024 (b) 0.34
(c) 35.71 kmol (d) 26.32 kmol
(c) 0.54 (d) 0.56
Passage 4
Passage 6
11.2 g of mixture of MCI (volatile) and NaCl gave 28.7 g
Phosphoric acid, HgPO4, is widely used to make
of white precipitate with excess of AgNO3 solution. 11.2 g fertilisers, and can be prepared in a two-step process.
of same mixture on heating gave a gas that on passing into
AgNO3 solution gave 14.35 g of white precipitate Step I: P4 + 5O2 ---- > P4O10
Answer the following questions Step U: P4O10+6H2O---- > 4H3PO4
1. Ionic mass of M+ is We allow 310 g of phosphorus to react with excess
(a) 18 (b) 9 oxygen, which forms tetraphosphorus decoxide, P4O10, in
(c) 36 (d) 23 50% yield. In the step II reaction, 25% yield of HgPO4 is
obtained. (P = 31, H= 1,0= 16)
2. MCI and NaCl are the molar ratio
Answer the following questions
(a) 1:1 (b) 1:2
1. Assume 100% yield then number of moles of H3P04
(c) 2:1 (d) 1:3
would be obtained
3. In terms of moles, mixture has
(a) 1.25 (b) 10.0
(a) 0.2 mol (b) 0.1 mol
(c) 5.0 (d) 2.5
(c) 0.15 mol (d) 0.01 mol
2. Actual number of moles of H3PO4 obtained based on
Passage 5 experimental conditions is
Cisplatin, an anticancer agent used for the treatment (a) 5.0 (b) 2.5
of solid tumors, is prepared by the reaction of ammonia (c) 1.25 (d) 0.3125
with potassium tetra chloroplatinate 3. H3PO4 thus formed is converted into calcium
ICjPtC^ + 2NH3 ---- > [Pt(NH3)2]Cl2 +2KC1 phosphate by treating with 1 mole of calcium
potassium cis -platin hydroxide. Calcium phosphate obtained is
tetrachloroplatinate (a) 2.5 (b) 0.33
Assume that 10.0 g of I^PtC^ and 10.0 of NH3 are (c) 0.1563 (d) 1.25
allowed to react. (K= 39, Pt= 195, Cl = 35.5)
Chapter 6: Chemical Equations and Reactions | 279

Passage 7 Passage 9
Let us explore a reaction with a limiting reactant.
Chile salt peter is a natural source of NaNO3 which Here zinc metal is added to a flask containing aqueous HCI
also contains NalOo. The NaIO3 can be used as a source of and Hj gas is a product
iodine produced in the following reactions Zn(s) + 2HCl(aq) ---- > ZnCl2(aq) + H^g)
IO3 + 3HSO3 ---- > r + 3H+ + 3S0J' ...(i)
Each of three flasks contain Balloon being
51- + IO3 + 6H+ ---- > 312(s) + 31^0 ...(h) 0.10 mol HCI. Zinc is added to each inflated
1.00 L of the starting solution which contains 5.80 g flask in the following quantities :
NaIO3/L, is treated with stoichiometric quantity of Flask 1: 7.00 g Zn
NaHSO3.Then a further quantity of the starting solution is Flask 2 : 3.27 g Zn Zn + HCI
added to the reaction mixture to bring about the second
Flask 3 :1.31 g Zn
reaction. [I = 127, Na = 23, S= 321 When the reactants are combined, the IL, inflates the
Answer the following questions balloon attached to the flask.
1. How many grams ofNaHSO3 are required in step (i)? Based on the above experiment, answer the following
(a) 5.80 g (b) 17.4 g questions
(c) 9.14 g (d) 3.0 g 1. Balloon is inflated completely in
2. What additional volume of the starting solution must (a) flask 1 (b) flask 2
be added in step (ii)? (c) Both (a) and (b) (d) None of these
(a) 0.2L (b) 0.4L 2. Zinc reacts completely in
(c) 0.8 L (d) 1.0 L (a) flask 1 (b) flask 2
3. How many equivalents of IO3 are present in one mole (c) flask 3 (d) flask 2 and 3
of NaIO3 in step (i)? 3. Consider following statements
(a) 2 (b) 4 I: In flask 1, balloon inflates completely, but some
(0 6 (d) 8 Zn remains when inflation ceases
4. How many equivalents of IO3 are used in step (ii)? II: In flask 2, balloon inflates completely, but no Zn
remains
(a) 6 (b) 5 III: In flask 3, balloon does not inflate completely
(c) 0.06 (d) 0.035 and no zinc remains
Select correct statements.
Passage 8 (a) I, II, III (b) I, II
The depletion of ozone (O3) in the stratosphere has
(c) I, III (d) II, III
been a matter of great concern among scientists in recent
years. It believed that ozone can react with nitric oxide
(NO) that is discharged from the high altitude jet plane. Assertion & Reason
The reaction is Codes:
O3 + NO---- > O2 + NO2 (a) Both A and R are true and R is the correct
Answer the following questions explanation of A.
(b) Both A and R are true but R is not the correct
1. If 0.740 g of O3 reacts with 0.670 g of NO, how many
explanation of A.
gram of NO2 will be produced?
(c) A is true but R is false.
(a) 0.71 g (b) 0.74 g
(d) A is false but R is true.
(c) 0.68 g (d) 0.81 g
1. Assertion (A): In the following reaction
2. Which compound is the limiting reactant?
SO2 + 21^8---- > 3S + 21^0
(a) NO
(b) O3 1 mole SCkj and 1 mole form 3 moles sulphur.
(c) Both are in equimolar ratio Reason (R) : 1 mole S02 and 2 moles HjS form
(d) Both are in stoichiometric ratio 3 moles sulphur and 2 moles H^O.
3. Number of moles of the excess reactant remaining at 2. Assertion (A): 20 g of 50% pure CaCO3 on heating
the end of the reaction is gave 2.24 L CO2 at STP and 15.6 g non-volatile
(a) 0.007 mol O3 (b) 0.014 mol O3 residue.
(c) 0.007 mol NO (d) 0.014 mol NO CaCOou CaO + CO2
280 | Essential Inorganic Chemistry

Reason (R) : Pure CaCO3 is 10 g which gave 4.4 g Fill in the Blanks
CO2 (= 0.1 mol = 2.24 L at STP) and 15.6 g residue is
left. 1. 16.8 g of mixture of NaHCO3 and Na2CO3 (50% each)
on strongly heating gives L CO2 at STP.
3. Assertion (A): 2H2 + O2 ---- > 2H2O
2. In the following reaction,
2 g H2 react with 1 g O2 to form 2 g HjO.
MnO2 +4HC1---- > MnCl2 + Cl2
Reason (R) : 2 moles H2 react with 1 mole O2 to
1 mole each ofMnO2 and HC1 forms .. gCl2.
form 2 moles H2O.
4. Assertion (A): 2 moles N2 react with 3 moles E^ to 3. C(s) + H2O---- > CO + H2
form 2 moles NH3. Ni + 4CO---- > Ni(CO)4
N2 + 3H2 ---- > 2NH3 To prepare 1 mole of Ni(CO)4, C(s) required is
Reason (R) : N2 is excess and H2 is the limiting mole.
reactant. 4. In the reaction between 1 mole each of E^S and AsC13.
5. Assertion (A) : 4 moles of KC1O3 (50% pure) gave
2AsC13 + 3E^S — -> As2S3 + 6HC1
3 moles of O2 on heating strongly and thus yield is
100% limiting reagent is
2KC1O3 ---- > 2KC1 + 3O2 5. In the following reaction,
Reason (R) : 3 moles of O2 are obtained from AgNO3 +HC1 — » AgCl + HNO3
2 moles of KC1O3. lg lg
unreacted reagent is
True & False
1. 1 mole of Ag2CO3 on strongly heating gave 1 mole of Matrix-Match Type Questions
residue.
I. Only One Correct
2. 1 mole ofNa2C03 and 2 moles of NaHCO3 on heating
1. Match the number of moles (in List I) with the
gave 2 moles residue, 1 mole CO2 and 1 mole E^O.
compound (in List II).
3. 1 mole SO2 and 1 mole H2S form 3 moles S.
List I List II
SO2 + 2H2S---- > 3S + 21^0
4. In the reaction between 2 g E^ and 3 g O2, E^ is the A. 0.5 mol 1. 92 g CH3CH2OH
limiting reactant and O2 is in excess. B. 1.0 mol 2. 66 g CO2
2H2 + O2 ---- > 21^0 C. 1.5 mol 3. 54 g H2O
5. Percentage yield in the following reaction is 50%. D. 2.0 mol 4. 15gHCH0
NH4NO3 -A» N2O + 2H2O 44 g CH3CHO
E. 3.0 mol 5.
1 mole of 50% pure NH4NO3 gave 11.2 L N2O at STP.
2. Match the reaction (in List I) with the maximum yield
6. 100 mL of 20.8%BaCl2 (molar mass = 208) and 50 mL
of the product (in List II).
of 17.4% (molar mass = 174) form 0.5 moles of
BaSO4 (molar mass = 233) List I List II
7. 1 g AgNO3 reacts with 1 g E1C1 to form 1 g AgCl. A. 2H2 +O2 ---- > 2H2O 1. 1.028 g
19 ig
8. 2 moles of PCL are 50% dissociated at 400 K. After
reaction, there are total of 3 moles. B. 3H2 +N2 ---- > 2NH3 2. 1.333 g
1g 1g
9. In the following reactions,
C. H2 +CI2 ---- > 2HCI 3. 1.125 g
2KC1O3 4 2KC1 + 3O2 19 ig
Q
2A1 +- O2 -> A12O3
2 2 D. 2H2 + C CH4 4. 1.214g
ig 1g
1 mole of KC1O3 is required to form 1 mole A12O3.
10. Gravimetry is studied under stoichiometry.
I Chapter 6: Chemical Equations and Reactions | 281

II. One or More Correct 7. How many moles of KC1O3 would be required to
1. Match Column I (reactant) with the amount in supply the proper amount of oxygen needed to burn
Column II. 24 g of CH4 completely?
Column I Column II 2KC1O3 ---- > 2KC1+3O2
A. CH, COOH + NaOH ---- > 1. 1 mole CH4 +2O2 ---- > CO2 +2^0
1 mol
CH3COONa+ H2O 8. In the following sequence of reactions
NaOH used to react completely with CaOCl2 + 1^0 ---- » Ca(OH)2 + C1.2
CH3COOH
2. 1 equivalent 2KI + C12 ---- > 2KC1+12
B. MgCO3 MgO + CO2
84g I2 + 2Na2S2O3 ---- > Na2S4O6 + 2NaI
Residue formed
C. 2SO2 + O2 —> 2SO3 3. 40 g
Calculate moles of Na2S2O3 required for complete
2 mol 64 g reduction of obtained from 127 g CaOCl2 sample
Reactant in excess under standard (50% purity).
conditions
D. 2H2 + O2 —> 2H2O 4. 22.4 L at STP
Set II
O2 used to form 36 g H2O
E. N2H4 + O2 —» N2 + 2H2O 5. 4 equivalents 1. How many species form 0.05 mole from 3 g of
N2 formed from 32 g each reactant CH3COOH, NHgCONH,, HNO3 ?
2. How many substances underlined have equivalent
Integer Answer Types weight equal to molecular weight based on the
This section contains 8 questions. X y z IV reactions?
The answer to each of the questions is a © © © © H3PO4 +Ca(OH)2 -> CaHPO4 +2H2O
single digit integer, ranging from 0 to 9. O ©
The appropriate bubbles below the
0 H3PO2 +NaOH 4 NaHjPOa + ^0
respectively question numbers in the © © © ©.
ORS have to be darkened. For example, © © © © NaHGA +NaOH ■> Na2C2O4 +H2O
if the correct answers to question © © © © 3. 1 mole of C^H* is oxidised to CO2 and H^O by 3 moles
number X, Y, Z and W (say) are 6, 0, 9 © © © ©
© © © O2.Thus, 140 gCjH* has....... moles.
and 2, respectively, then the correct ©
darkening of bubbles will like the © © © © 4. Out of the following, how many substances have
following © © © © underlined atoms of different nature?
© © © ©
Set I Na2 S40g, N a2 S2O3, CaOCl2, N2H4O3,Fe3O4, KI^, CrO5,
1. What mass (in grams) of CH4 produces 3.01 x 1023 HjO K^Ci^O?
molecules when burnt to excess O2 ? 5. 6 g urea sample on heating with NaOH give 0.224 L
2. How many moles of H2O are formed by 10 moles of NH3 at STP.
HN03 and 10 moles Cu in the following reaction? NHjCONHj + 2NaOH ---- > 2NH3 +Na2CO3
3Cu + 8HNO3 ---- > 3Cu(NO3)2 + 2NO + 41^0 Thus, percentage yield of NH3 is.............
3. 20.8 g of AfClx (M = 137) or reaction with excess of 6. Sample of CaCO3 of 50% purity forms 0.56 g residue.
K2SO4 yields 23.3 g of sulphate of metal M. Calculate Thus, sample weighs......... g.
value ofx.
7. Pure nitrogen is obtained by decomposition of nitride
4. Free SO3 in one mole of pure oleum is mole.
Mx(N3)y where x + y = ?
5. The molar mass of haemoglobin is about 65000 g mol"1.
8. In the following reaction
Haemoglobin contains 0.35% Fe by mass. Thus, iron
atoms present in haemoglobin molecules are...... 2K2MnO4 +C12 ---- > 2KCl+2KMnO<'4

6. How many moles of oxygen are required to react 2 moles Cl2 and 2 moles K2MnO4 form moles
completely with 24 g of CH4 ? KMnO4.
Answers & Solutions
Numerical Problems Solving
1. (a) 2KC103(s) ---- > 2KCl(s) +3O2(g) +H2
5. CH3COC1 + H.•2 CH,CHO
o
-» CH3CH2OH
(b) P4(s) + 5O2(g) ---- > P4O10(s) 1 mol 1 mol 1 mol
44 g 46 g
(c) Zn(s) + 4HNO3(aq)---- > Zn(NO3)2(ag) + 2NO2(£)
+ 2H2O(Z) If no side reaction, CH3CHO = 44 g
(d) 2Al(s) + 2NaOH(ag) + 2H2O(Z)---- > 2NaA102(ag) but same CH3CH2OH is also formed.
+ 3H2(g) CH3CH2OH = 2.3 g for 4.4 g CH3CHO
2. CH3COOH(ag) + NH4OH(a<?) ---- > CH3COCT(ag) Thus, actual yield of CH3CHO = 44.0 - 4.4
+ NH3(ag) + H2O(Z) = 39.6 g
39 A
3. (a) CrO4"(a<?) +2Ag+(ag) ---- > Ag2CrO4(s) % yield = — x 100 = 90.0%
44.0
(b) Ba2+(a<?) +SO4~(aq) ---- > BaSO4(s) 6. C(s) + H2O ---- > CO(g) + H2(g)
(c) Ca2+(a<7) + CO2‘(ag) ---- > CaCO3(s)
Ni(s) + 4CO(g) ---- > Ni(CO)4
4. 1: 2 mol NH3
After balancing
II: 2 mol, N2 is in excess 4C(s) Ni(CO)4
III: 2.66 mol, N2 is in excess 4 x 12 g 170.7 g
IV : 0.66 mol, H2 is limiting reagent. 170.7
72g ---- > x 72 g
48
= 256.05 g

Master Exercises
Exercise 1
Short Answer Type Questions 6. NO = 10 moles
1. Soluble : NaOH, CaO NOBr = 10 moles
Partial soluble: CaSO4 7. 2C1O2 ---- > HC1O3+HC1O2
Insoluble : FeS, PbS, CaCO3, CaC2O4 2 mol 1 mol
135 g 84.5 g
2. Interaction between cation and anion that leads to
formation of insoluble salts 8.36 kg HC1O3 is from = x 8.36 CO2
3. H+(ag) + OH"(ag) ---- > H2O(a<?) = 13.356 kg C1O2
4. HCl(ag) + NaOH(ag) NaCKagJ + HjO 8. CH,
--- 4 + 20, CO2 + 2H2O
1 mol 1 mol 1 mol 2 mol
16 g 64 g
2HCl(a<7) + Ca(OH)2(ag) ---- > CaCl2(ag) + 21^0 32 g 128 g
2 mol 1 mol
1 mol 0.5 mol 9. C5H12(g) + 8O2(g) ---- > 5CO2(g) + 6H2O(Z)
1 mol 5 mol
5. Ca(OH)2 + CaSO4+2H2O 1 mole = 72 g 5 No molecules
1 mol 1 mol
5 x 6.02 x 1023 molecules of CO2 are from = 72 g C5H12
2KOH + H2SO4 K,>SO4 +2H2O Thus, 3.01 x 1022 molecules of CO2 are from
2 mol 1 mol 72x3.01xl022 _ ^TT
1 mol 0.5 mol =---------------- 57- = 0.72 g C6H12
5x6.02xl023
Chapter 6: Chemical Equations and Reactions | 283
75
io. CaO + 2HC1 ----- > CaCl2 + H2O NaHCOn in mixture = 4 x-----= 3 g = 0.0357 mol
1 mol 2 mol 1 mol 1 mol 3 100
8.8 mol 8.8 x 2 mol 8.8 mol 0.0357
CO2 formed = mol
2
11. Na + KC1 ----- > NaCl + K 0.0357
1 mol 1 mol 1 mol x 22.4 L CO2 at STP
23.0 g 74.5 g 39 g 2
I 150.0 g 150.0 g 78.5 g 254.35 g = 0.4L = 400mL
iI I t f 3. 5.943 L 4. 41 kg H2O, Li2O is limiting reactant
KC1 is a limiting quantity and thus, K formed = 78.5 g
5. MnO2 + 4HC1 MnCl2 + 2H2O + Cl2
12. 2NaCl +H2SO4 ----- > Na2SO4+2HCl 1 mol 4 mol 1 mol
2 mo1 1 mol 0.86 mol 48.2 g = 1.32 mol
2 x 58.5 g 142 g
HC1 required by 0.86 mol MnO2 = 3.44 mol
117 g NaCl yield = 142 g pure Na^C^
/ qq q\ 142x245x93.9 but HC1 taken = 1.32 mol
|^245 x^jg NaCl yield = ----------------------g
117x100 Thus, HC1 is the limiting reactant.
Thus, Cl2 formed = | x 1.32
pure NagSO,’4
= 279.21 g pure Na^C^ = 0.33 mol Cl2
= 279.21 x of 74.1% purity = 23.43 g
6. 29.80% 7. (a) 863.08 g (b) 93.04%
= 376.80 g
8. ZnS ----- > ZnO ----- > ZnSO4 ----- > Zn
97 g 65 g
Analytical Questions 225x90.6x100x98.2
kg gives Zn = 134.14 kg
1. 4236 g 100x100x100
A
2. 2NaHCO3 Na2CO3 + H■22O
’ + CO2 9. CaCO3 ----- > CaO + CO2
2 mol 1 mol = 22.4 L at STP
Na2CO, +H2O + CO2 > 2NaHCO3
Na2CO3 -±>X
Z 0
50 g

Exercise 2
Only One Option Correct 2HCl + Ca(OH)2 ----- > CaCl2+2H2O
i. ch4 + 2O2 ------ CO2+2H2O Thus, total Ca(OH)2 required = 2 mol
1 mol 2 mol 44 g Thus, (a)
16 g 64 g
4. 3BaCl2+2(NH4)3PO4 ----- > Ba3(PO4)2+6NH4C1
given 16 g 48 g = 1.5 mol 3 mol 2 mol 1 mol
Thus, O2 is the limiting reactant given 0.5 mol 0.2 mol 0.1 mol
2 mol O2 give = 44 g CO2 Thus, (NH4)3PO4 is the limiting reactant giving 0.1 mol
44 Ba3(PO4)2
1.5 mol O2 give = — x 1.5 = 33 g CO2
2 Thus, (a)
Thus, (b) 5. BaCl2 required by 0.2 mol of(NH4)3PO4 = 0.3 mol
2. Ratio of HNO3 per mol of H2O Thus, BaCl2 in excess = 0.5 - 0.3
(b) |=2 = 0.2 mol
(a) 7=2
4
Thus, (d)
(d) i=2
(c) — =3.33
3 6. 2H2S + SO2 2H9O+ 3S
2 mol 1 mol 3 mol
Thus, (c)
3. given 1 mol 1 mol ?
SO2C12 +2H2O H2SO4 + 2HC1
1 mol 1 mol 2 mol Thus, is the limiting reactant forming 1.5 mol S

+ Ca(OH)2 -----> CaSO4+2H2O Thus, (c)


284 | Essential Inorganic Chemistry

7. BaF2Z + H2ZSO44 ----- > BaSO4 + H2F2 = 0.1938 g


1 mol 1 mol D 1Q3R
1 mol 1 mol % of pure CaO = x 100
given 1 mol 2 mol 0.8432
Thus, unreacted H2SO4 = 1 mol = 23%
HgFg formed = 1 mol Thus, (b)
H2SO4+2NaOH ----- > Na2SO4+2H2O 14. S ----- > BaSO4 ----- > BaCl2
1 mol 2 mol 32 208 g
0.60 x 0.12= 0.072 g ?
H„F2 +2NaOH Na^ +2H2O BaCl2 required by 0.072 g sulphur to be precipitated as
1 mol 2 mol 9OR
BaSO4 =-—x 0.072
Thus, total NaOH required by 1 mol H^ (formed) and 4 32
1 mol HrjSCli (unreacted) = 4 mol = 0.468 g
Thus, (a) 0.468 x 100
Thus, volume of BaCl2 solution =
1.6
8. (b)
= 29.25 mL
9. Ca(HCO3)2 + CaO ----- > 2CaCO3+H2O
56 g 200 g Thus, (c)
0.56 g 2.00 g in 10 L water 15. Fe3O4 1.5 Fe2O3
10 L H2O (= 104 mL) has CaCO3 = 2 g 1 mol 1.5 mol
232 g 240 g
106 mL (ppm) has CaCO3 = 200 g ? 0.40 g
Thus, 200 ppm CaCO3 - 232.
Fe3O4 required by 0.40 g Fe2O3 = —- x 0.4
Thus, (a) - ’ 240
10. Na2CO3 sCaCOo = 0.3867 g (pure)
106 g 100 g = 0.3867 x^g
18
Hardness in 106 g H2O = 100 g CaCO3
= 2.15 g impure Fe3O4
Hardness in 10 L (104 mL = 104 g) H2O = 1 g CaCO3
Thus, (a)
100 g CaCO3 required = 106 g Na2CO3
1 gCaCO3 required = 1.06 g Na2CO3 16. 10 g glucose is in = 100 mL
100 x180
Thus, (d) 180 g glucose is in = ---------------- 1 mL
10
11. Iron oxide = 0.5434 g
= 1800 mL = 1.8 L
Oxygen lost as H2O = 0.1210 g
Thus, (c)
Iron = 0.5434 - 0.1210 = 0.4224 g
17. 2(NH4)2HPO4 =P2O5
2 x 132 g = 142 g
Element Amount % mole Ratio 142
Thus, % of P2O5 = — x 100 = 53.78
2 5 264
Iron 0.4224 g 77.73 1.39 1
Thus, (d)
Oxygen 0.1210 g 22.26 1.39 1
18. Oxygen = 67.6%
Thus, FeO (a)
Metal =32.33%
12. P2O5 * Mg 2^2^?
1 mol 1 mol Thus, 67.67 g oxygen combines with =32.33 g metal
142 g 222 g 32.33 x 8
hence, 8 g oxygen combines with =
67.67
P9OS in the mineral = 0.5428 x
2 5 100 = 3.82
= 0.1425 g = 1 x 10-3 mol Thus, (b)
Thus, MgjjPgO, obtained = 1 x 10"3 mol = 0.2228 g 19. (a)
Thus, (d) 20. 2NaOH+ H9S0. Na^ + 2H2O
13. CaO ----- > CaC2O4 CaCO3 2 mol 1 mol
56 g 100 g 1 mol 0.5 mol = 49 g H2SO4 (pure)
? 0.3462 g
Thus, 70% required = 49 x = 70 g
56
Pure CaO in the mineral = x 0.3462
Thus, (a)
Chapter 6: Chemical Equations and Reactions | 285

21. (d) Thus, molar mass is decreased


40 Thus, (d) is also false
22. Solute present in 800 g solution = 800 x----
100 Thus, (a), (c)
= 320g 3. (a), (c), (d)
Solute precipitated = 100 g . ... 146 ,
4. 146 g HC1 =-—— = 4 mol
36.5
Solute left = 220 g
Total weight of solution = 800 -100 = 700 g MnO2 + 4HC1 ---- > MnCl2 + Cl2 + 2H2O
1 mol 4 mol 1 mol
700 g solution has solute = 220 Initial 0.8 mol 3.2 mol 0 mol
220 after 0 mol 0.8 mol 0.8 mol
100 g solution has solute = --- x 100
Thus, MnO2 (less) is limiting reagent thus, (d) is true
= 31.43% HC1 (more) is in excess.
Thus, (a) Amount of product is decided by MnO2
23. MHPO, ,= ™
M2+ +HPOJ- Cl2 formed =0.8 mol thus (a) is true
Thus, metal is divalent forming MC12 HC1 unreacted = 0.8 mol thus, (b) is true
Thus, (d) Thus, (a), (b), (c) and (d) are correct alternates

24. Equivalent weight of M 2+ =37.2 5. Mg =0.273 g


Product formed with nitrogen =0.378 g
Thus, molar mass of M2+ = 37.2 x 2 = 74.4
Nitrogen reacted =0.105 g
Thus, molar mass of MC12 = 74.4 + 71 = 145.5
Thus, (c) Whole
Element Amount Mole Rate
number
9x500
25. Glucose in 500 mL solution =
100 Mg 0.273 g 0.011 1.5 3

= 45 g N 0.105 0.0075 1.0 2


45
= — =0.25 mol Thus, Mg3N2
80
Thus, (b)
= 0.25 x 6.02 x 1023 molecules
Total
= 1.5xl023 6. Species Amount Mole
atoms
H-atoms

Thus, (c)
ch4 1.6 g 0.1 0.5 No 0.4 No

One or More Than One Options Correct nh3 1.7 g 0.1 0.4 No 0.3 No

1. SO2C12 + 2H2O ---- > HaSO, + 2HC1 h2o 1.8g 0.1 0.3 Nq 0.2 No
1 mol 1 mol 2 mol
They have equal number of moles thus, (a) is true
1 mol H2SO4 = 1 mol Ca(OH)2 = 2 mol KOH Number of atoms is CH4 > NH3 > H2O thus, (b) is also
1 mol HC1 = - mol Ca(OH), = 1 mol KOH true
2 2 Ratio of H-atom 4:3:2 thus, (c) is also true.
2 mol HC1 = 1 mol Ca(OH)2 = 2 mol KOH Thus, (a), (b) and (c)
Thus, mixture of 1 mol H2SO4 and 2 mol HC1 requires 7. H2O = 1 g mL"1 = 1000 gL’1
2 mol Ca(OH)2 thus, (a) is correct 1000 1T-1
=------ = 55.55 mol L
4 mol KOH thus, (d) is correct 18
2. Iron in haemoglobin = —x 100 = 0.35 Thus, (a) is true
65000 1 i 6.02 x 10I23
1 g = — mol = — H2O molecules
Thus, (a) is true s 18 18
Iron in mole haemoglobin = 224 g = 3.33 x 1022 H2O molecules
Thus, (b) is false and (c) is true Thus, (b) is true
Molar mass of haemoglobin if iron content is 0.56% It has 6.06 x 1022 H-atoms thus (c) is false
100x4x56 It has 3.33 x 1022 O-atoms thus (d) is also false
= 40000 <65000
ft56 Thus, (a) and (b)
286 | Essential Inorganic Chemistry

8. 1 mole of (NH3)2HPO4 has 0.5 mole P2O5 and 2 mole of 143.5x5.35


Thus, ---------------= 14.35
NH3. (M + 35.5)
Thus, 1 mole (=132g) (NH4)2HPO4 has =0.5 mole M = 18
(=71g)P2O5 Thus, ionic mass of M+ = 18, thus (a) is true and mola
% of P2O5 = — x 100 = 53.78% mass of MCI = 53.5
2 5 132 5 35
Thus, (a) is correct. Thus, moles of MCI = — = 0.1
53.5
Also, 132 g (NH4 )2HPO4 has = 34 g NH3 12.80-5.35
Moles of KC1 = = 0.1
Thus, % as NH. = — x 100 = 25.76% 7L5 ”
3 132
Thus, (b) is correct. Thus, (b) is also true
Also, 132 g has = 28 g N, 9g H and 64 g 0 Thus, (c) is also true
Thus, % of 0 is maximum AgCl is water insoluble
Thus, (c) and (d) are incorrect. Thus, (d) is false
Thus, (a) and (b) Thus, (a), (b) and (c)
9. Mole fraction of ethanol (C^HgOH) = 0.25 =
Til + n2 Brain Twisters
Mole fraction of water (H2O) = 0.75 = n2 1. Heat evolved in the reaction (exothermic) cause
?ii + n2 decomposition of H2CO3(aq) into CO2(g) and HjOU)
Tij _0.25 1 H2CO3 ---- > H2O(Z) + CO2(g)
t^”O?75 3 Thus, H2CO3 is unstable.
CjjHgOH = Tij = 1 mole = 46 g 2. (a)
H2O = n2 = 3 moles = 54 g (i) AgNO3(aq) + NaBr(ag) ---- > AgBr(s) + NaN03(a^
Total mass = 46 + 54 = 100 g (ii) Ag+(aq) + NO3(aq) + Na+(aq) + Br"(ag)
Thus, it is 46% by weight of solution and 54% by weight ---- > AgBr(s) + Na+(aq) + NOjtety
of solution. (iii) Ag+(aq) + Br"(aq) ---- > AgBr(s)
Thus, (c) and (d) are correct. (b) (i)
10. MCI + AgNO3 ---- > AgCU BaCl2(aq) + H2SO4(aq)---- > BaSO4(s) + 2HCl(ag)
(Af + 35.5) 143.5 (ii) Ba2+(ag) + 2Cr(aq) + 2H+(ag) + SO4“(ag) —>
KC1 + AgNO.'3 AgCl J- BaSO4(s) + 2H+(a^) + 2Cn«?:
74.5 143.5
(iii) Ba2+(aq) + SO4“(aq) ---- > BaSO4(s)
Let MCl=Xg (c) (i) Al2(SO4)3(ag) + 3Ca(OH)2(ag) 2Al(OH)3(4f)
than KC1 = (12.8-x) g + 3CaSO4(a?l
143.5x (ii) 2Al3+(ag) + 3S0j’(aq) + 3Ca2+(aq) + 6OH"(a<7)
AgCl due to MCI =
(M + 35.5) ---- > 2A1(OH)3(s) + 3Ca2+(a<7) + 3SO;'(a41
143.5 (iii) 2A13+ + 6OH"(aq) ---- > 2A1(OH)3(s)
AgCl due to KC1 = (12.8-x)
74.5 NaOH dissolves Al(OH)3 hence can’t be used
143.5x 143.5(12.8 — x)
(M + 35.5) + = 28.7 (d) (i) CaCl2(a<7)+2K3pO4(aq) ---- > Ca3(PO4)2(s)
74.5 + 6KCKO4)
Since, MCI is volatile, in second case AgCl due to MCI (ii) 3Ca2+(aq) + 6Cr(aq) + 6K+(aq) + 2PO3’(ag)
only. ---- > Ca3(PO4)2(s) + 6K+(ag) + 6Cl'(o?)
143.5x (iii) 3Ca3+ +2PO3" ---- > Ca3(PO4)2(s)
Thus, = 14.35
M + 35.5
3. A12O3+3H2SO4 ---- > A12(SO4)3 +3H2O
143.5(12.8-x) 1 mol 3 3mol
mol 1 mol
14.35 + = 28.7 4 mol 12mol
12 mol 4 mol
74.5
10(12.8-x) H2SO4 required = 12 mol
1+ =2
7L5 A12(SO4 )3 formed = 4 mol
(12.8-x) = 7.45 4. CaO + 3C ---- > CaC2+CO
56 g 64 g
x= 5.35 g
Crude produce is = 85% CaC2 + 15% CaO (unreacted)
Chapter 6: Chemical Equations and Reactions | 287

15 9. Cu2S ---- > 2Cu


450 kg of crude product has 15% CaO = 450 x —
159 g 127 g
= 67.50 kg CaO CuS ---- > Cu
95.5 g 63.5 g
and 85% CaC, = 450 x —
2 100
There is 11.0% impurity. Thus, mixture has 89.0 g of
= 382.5 kg CaC,2 Cu-jS and CuS.
64 kg CaC2 is from = 56 g CaO
LetCugS = x g
Thus, 382.5 kgCaC2 is from = — x 382.5
64 then CuS = (89.0 - x) g
= 334.69 kg CaO 127x^1
Thus, Cu obtained from x g Cu^ =
Thus, total CaO required = 67.50 + 334.69 159 J g
= 402.19 kg 63.5(89.0-x)
and from (89.0 - x) g CuS = ----------------- g
(b) 382.5 kgCaC,'2 95.5
5. CuSO4 = x g Total Cu obtained
CuSO4-5H2O = (1.245-x) 127x 63.5(89.0-x) 75.4x89.5
159”+ 95^5 (given)
CuSO4 -5H2O ---- > CuSO4 + 5H2O(g)T 100
249.5 159.5 g . = 67.483 g
CuSO4 in (1.245 -x)g This gives x = Cu-jS = 62.45
159.5 n a x % of CugS = 62.45%
CuSO4-5H2O = ------ (1.245 — x)g
249.5
159.5 CuS =27.55%
Thus, (1.245-x) +x =0.832
249.5 10. 0.033 mol CgHgOH per litre = 0.033 x 46 x 1000 mg/L
x =0.1 g
= 1518 mg/L
CuSO4-5H2O = 1.145 g
= 151.8 mg/dL
6. Nix(CO)y ---- > xNiO + yCO.2
C58.7x + 30y) 74.7x 44 y This excess prescribed limit, hence yes DWI ticket
0.0973 g 0.0426 0.10
74.7x _ 0.0426 Passage Comprehension Questions
44 y 0.10
Passage 1
-=0.25 = -
y 4 Step I HgSO. ---- > H++HSO;100%
Thus, empirical formula is = Ni(CO),'4 step n hso; H+ + SO2" 10%
7. 3 mol NaNH2 form = 1 mol NaN3
H+ +H2O ---- > H3O+
0.0769 mol NaNH2 forms =0.0256 NaN3
SO4~ is from step II
Thus, NaNH2 is limiting reactant and NaN3 obtained
= 0.0256 mol Step I is 100% thus [H+ ] = 0.2 M
7. NaNO,+3NaNH.
'3 ■2
NaN3 and [HSO;] = 0.2M
1 mol 3 mol 1 mol
85 g 65 g
8.33 g (based as NaNH2) Step II is 10% thus [H+ ] = 0.2 x 0.1 = 0.02 M
(3 x 39) g
15 g 15g 7 11.47 g (based on NaNO3) and [SO^-] = 0.02M
I total [H3O+ ] = 0.2 + 0.02 = 0.22 M
Limiting reagent is NaNII2
[SO2-] = 0.02 M
and thus NaN3 obtained is 8.33 g
and [HSO4] = 0.20-0.02 =0.18 M
8. SiJL +O, —> xSiO, + -H2O
C8x + y) ' . 1. (d) 2. (c) 3. (b) 4. (b)
7’ 60x 9y H2O
11.64 g 6.980 g Passage 2
60x _ 11.64
1. (a) 2. (a,b)
9y 6.980
Passage 3
— =0.25 = -
y 4 1. (a) 2. (b) 3. (a)
Thus, SiJIy is SiH4
288 | Essential Inorganic Chemistry

Passage 4 = 2.5 x 0.5 x 0.25 x 4 (25% yield!


1. Let MCI = xg, NaCl =(11.2-x)g = 1.25 mol HgPO*

MNO3 + AgClX 3. (b) 3Ca(OH)2 +2HgPO4 ---- > Ca3(PO4)2


MCI + AgNO3
white ppt 3 mol 2 mol 1 mol

(M + 35.5) g MCI gives = 143.5 g AgCl 1 mol Ca(OH)2 =— mol Ca3(PO4)2


3
143.5X A
x g MCI gives = 777—g AgCl 2 mol H;jPO4 = 1 mol Ca3(PO4).2
(Al + Ou.O)
1 25
1.25 mol H3PO4 =— mol.
NaCl + AgNO» NaNO3 + AgCl J- 2
(58.5 g) (143.5 g)
Ca(OH)2 is limiting reagent.
143.5(11.2-x)
(11.2 -x) g NaCl gives =------ ——------- g AgCl ...(ii) Passage 7
DO. O
143.5x 143.5(11.2-x) . 1. (0 5.8 g NaIO3 =-^ mol NaIO3
+------------------ = 28.7 (given) ...(iii) 198
M + 35.5 58.5
5R
On heating MCI vaporises (being volatile) NaHSO, = 3 x —— mol
3 198
Thus, AgCl is due to MCI only _3x5.8
rm 143.5X , , „— x 104 g NaHSO3
Thus, ------------ = 14.35 ...(iv) 198
’ M + 35.5
= 9.14 g
Thus, from (III) 5.8 .
143.5(11.2-x) 2. (a) I formed in step I =—— ~:1 mol
= 14.35 198
58.5
5.9 .
Thus, gives x=5.35 I03 required in step II = ---------mol
198x5
From (IV) M =18 5.8 x 198
MCI in mixture = 5.35 g gNaIO3
f 198x5
NaCl in mixture = 5.85 g
= 1.16gNaI0;'3
Thus, ionic mass of M+ is 18.
Also, molar mass of MCI = 53.5 g mol Volume=— = 0.2 L
5.8
moles ofMCl = —= 0.1 3. (c) IO3 —+ r
53.5
e; qe +5 -1
moles of NaCl = — = 0.1
58.5 Change in oxidation number = 6 units
Total moles = 0.2 Thus, one mole NaIO3 = 6 equivalents
Mole fraction of MCI = 0.5 116
4. (d) 1.16 g NaIO3 =—mol
Mole fraction of MCI = 0.5 198
Thus, 1. (a) 2. (b) 3. (a) 6.96 . ,
=----- equivalent (since lOj =6D
198 H
Passage 5
= 0.035 equivalent
1. (a) 10 g KjjPtClg mol = 0.024 mol (limiting)
Passage 8
10 g NH3 = 0.588 mol (excess) 0 74
1. (a) 0.74 g O3 =— = 0.0154 mol O3
48
KgPtClg consumed = 0.024 mol = cis-platin formed
0.67 g NO = 0.0223 mol NO
NH3 consumed = 0.048 mol
NH,o unreacted = 0.588 - 0.048
03 is the limiting reagent and NO is in excess
= 0.54 mol
= 0.0223 - 0.0154 = 0.007 mol
2. (b) 3. (a) 4. (b) 5. (c)
Thus, O3 taken = NO2 formed
Passage 6
310 = 0.0154 mol NO2
1. (b) 310 g P4 mol P4 = 10 mol H.jPO4
124 = 0.0154 x 46 gNO2
2. (c) 2.5 mol P4O10 (100% yield) = 0.71g
= 2.5x^ (50% yield) 2. (b) 3. (c)
100
Chapter 6: Chemical Equations and Reactions | 289

Passage 9 Integer Answer Types


1. (0 2. (d) 3. (a) Set I
Questions 1 2 3 _4_ 5 £ 7_ 8
Assertion & Reason Answers 4 5 2 1 4 6 2 1
1. (d) 2. (a) 3. (d) 4. (a) 5. (b) © © © © © ©I© ©
© © CD © © GW ©
True & False © © © ©1 © ©_ ©_ ©
1. F 2. T 3. F 4. F 5. F 6. T 7. F © © © © © © © ©
© © © © _©_© ©
8. T 9. T 10. T © © © © © © © ©
© © © © © © © ©
Fill in the Blanks © © © © © _©_© ©
1. 1.12 2. 17.75 3. 4 4. 5. HC1 © © © © © © © ©
© © © © © ©I© |©
Matrix-Match Type Questions Set II
I. Only One Correct Questions—> 6_ 1_ 8
1. (A) —(4); (B) — (5); (C)-(2); Answers —►
1
3 2
_3_ 4
4 Tj jH 2 1 2
(D)-(l); (E)-(3) © © © w© ©I©
©
2. (A)-(3); (B) —(4); (C)-(D; (D) — (2); © © © © © ©
©
II. One or More Correct © © © © © © ©
©
(C) —(1,4,5); © © © © © _©_©
©
1. (A)-(1,2,3); (B) — (1,3);
© © © © © _©_©
©
(D) —(1,4,5); (E)-(1,4,5) © © © _©_©
©
© ©
© © © © © ©
© ©
© © © © ©
©©
© © © J) © ©
© © © © © ©I© ©

I
Phenolphthalein is colourless in acidic solution and pink in
alkaline solution. Methyl-red is red in acidic solution and yellow
in alkaline solution. What is an idea? A drop of phenolphthalein or
methyl red can indicate completition of acid-alkali reaction by
colour change in volumetric titration.

Volumetric

Quick Points
• Mole= mass
molar mass
• N0 (Avogadro's number) = 6.02 x 10 23 moi-1
1 equivalent of an acid :! 1 equivalent of a base in an • Titrimetry
acid-base reaction
• Methods of Expressing Concentration of
1 equivalent of a reducing agent = 1 equivalent of an
oxidising agent in a redox reaction a Solution
Equivalent weight in a redox reaction ■ Mole Fraction
=------------
molar mass (one unit) • Per cent Concentration
change in O.N. (equal on both sides) ■ Molarity (M)
MnO� � Mn 2+
E(MnO�) = M ■ Molality (m)
+7 +2 5
■ Equivalent Mass and Normality
Cr2O�- � 2Cr 3+ E(Cr2o;-) = M
i i 6 • VolumetricTitrations
12 6

• Acid-Base Titrations
• Redox Titrations
For redox reactions such that AGi° + flG; = .b.G; ■ lodometric/lodimetric Titrations

Eo _ 71iE; + n2E:
3 -
na
where n1 is the number of electrons e·xchanged in a reaction
with standard electrode potential E,°, F is number of
coulombs (1 F = 96500 coulombs) ·
Electrode potential is given by Nemst equation
2.303RT
E = E0 _
log Q
nF
292 | Essential Inorganic Chemistry

where, Q is reaction quotient. In most of the cases, above requirements are not met
At equilibrium, E = 0 and Q = and their standard solutions cannot be prepared by
„o 2.303BT. v dissolving a weighed amount in a given volume of solution.
E=—^~loeK^ They are termed as secondary standard.
0.0591
E° = log Standard Solutions
n
2.303AT The ideal standard solution (of known concentration)
(at 298 K when = 0.0591V) must
F
- be sufficiently stable so that it is necessary to
determine its concentration only once.
Titrimetry - react rapidly with the analyte.
Titrimetric methods include a large and powerful - give sharp end-point with the analyte.
group of quantitative methods that are based on
• If concentration can be known simply by dissolving
measuring the amount of a reagent of known
the known amount in a given volume of solutions, they
concentration that is consumed in an analysis. Titrimetric
can be are termed as primary standard solutions. In most
cases, concentration can be determined by
Volumetric Gravimetric Coulometric standarisation against primary standard solution.
involves measuring involves involves Such solutions are termed as secondary standard
the volume of a measurement of measurement of solution.
solution of known mass involved electric current
concentration that is instead of its volume required to bring
needed to react with about a particular Equivalence Points and End Points
the species under chemical change. The equivalent point in a titration is a theoretical
investigation point reached when the amount of added titrant is
Present unit describes stoichiometric based on chemically equivalent to the amount of analyte in the
sample. Equivalent point cannot be determined
Volumetric Titrimetry.
experimentally. We can only estimate its position by
observing same physical change associated with the
Terms to be used in Volumetric Titrimetry condition of equivalence. This is called end-point of
A standard solution (or a standard titrant) is a titration.
reagent of known concentration that is used to carry out a Equivalent point is reached when 1 mole of HC1 is
titrimetric analysis. neutralised by 1 mole of NaOH. But we cannot detect this
A titration is performed by slowly adding a standard equivalent point. If one drop of suitable indicator (in this
solution from a burette to a solution under examination case phenolphthalein) is added in the HC1 (analyte) then
(analyte) until the reaction between the two is judged additional drop of NaOH (from burette) will give light pink
complete. The volume or mass of reagent needed to colour. This is the end-point. Thus volume of titrant at the
complete the titration is determined from the difference end-point is larger than that at the equivalent point.
between the initial and final readings.
Sometimes even rough estimate of the concentration
It is sometimes necessary to add an excess of the is not known. In such cases excess of reagent is added so
standard titrant and then determine the excess amount by
that complete reaction of the analyte takes place. Suppose
back-titration with a second standard titrant.
we have been given NH3 solution. We allow its reaction
with excess of HC1 or H2SO4 of known concentration. Acid
Primary and Secondary Standards reacts equivalent toNH3 and unreacted acid is determined
A primary standard is a highly purified compound by titration with NaOH. Estimation of unreacted acid by
that serves as a reference material in volumetric and mass NaOH is called back titration.
titrimetric methods. The accuracy of a method is mainly
dependent on the properties of this compound. A primary
standard must be Explanation of Terms
1. of high purity We can determine the concentration of oxalic acid
2. of atmospheric stability, (no oxidation, reduction or (H2C2O4 -2^0), potassium dichromate (K^Ci^CM, copper
decomposition sulphate (CuSO4 ■ SHgO) by dissolving the known amount
3. anhydrous so that composition of the solid does not in a given volume of solution. These are stable to
change with variation in humidity atmospheric condition and their concentration does not
4. of reasonable solubility in the titration medium change. They are thus, primary standard. Ferrous salts
(Fe2+ as FeSO4), potassium permanganate (KMnO4),
5. of reasonably large molar mass so that the relative
error associated with weighing the standard is mineral acids (HC1, HNO3, HgSQj), sodium hydroxide
minimised. (NaOH) etc, are not stable to atmospheric condition. Their
Chapter 7: Volumetric | 293

solutions are prepared by dissolving a known amount or + 722) = moles of solution


volume but they have to be standarised using primary d = density in g cm-3
standard. Thus, all these are secondary standard. V = volume of solution in cm3
= equivalent weight of solute
Standarised against primary
Secondary standard
standard
NaOH H2C2O4 (oxalic acid) Expressing Concentration in Groups A and B
KMnO4 H2C2O4
Following are the terms given in Tables 7.1 and 7.2
FeS04-7H20
widely used to express concentration of solution.
K2Cr2O7
Na2S2O35H2O CuSO4-5H2O Table 7.1 Expressing Concentration in “Group A”
(sodium thiosulphate)
HCI,HNO3,H2SO4 NaOH (standarised with H2C2O4) o
XI Usual SI
'2 Na2S2O3 -5H2O (standarised with Name E Definition Formula
unit
CuSO4 -5H2O)
w

Volume per C amount of solute C _ 100W, kg m'3


Mineral acids are standarised using sodium hydroxide cent present in 100 mL of V
which is turn is standarised first using oxalic acid. solution
Volume per C amount of solute WOOw, gL-’
Iodine is standarised using sodium thiosulphate which c=
litre present in 1000 mLof V
in turn is standarised first using copper sulphate solution. solution
Molarity M number of moles of M _ 1000w, mol dm'3
Methods of Expressing Concentration solute present in 1 L
(1 dm3, 1000 cm3) of
my

of a Solution Normality
solution
N number of N _ 1000Wi g equiv
The concentration of a chemical solution refers to the g-equivalent of solute Ey dm'3
amount of solute that is dissolved in a solvent. We present in 1 L(1 dm3,
normally think of a solute as a solid that is added to a 1000 cm3) of solution
solvent (for example NaCl in H2O) but the solute could
exist in another phase. For example, if we add a small Table 7.2 Expressing Concentrations in “Group B”
amount of ethanol (C^HgOH) to water, then the ethanol is
solute and the water is the solvent. If we add a smaller o
•D
amount of water to a larger amount of ethanol, then water Name E Definition Formula Usual SI unit
could be solute. w

The amount of solute in a solution can be expressed in WOw,


% concentration % amount of %= dimensionless
terms of two system of units:
by mass of solute present W, + w2
(i) ‘Group A’ units specify the amount of solute in a solution in 100 g of
given volume of solution, such as density, molarity, solution.
etc. % concentration % amount of % = 100w, dimensionless
by mass of solute present w2
(ii) ‘Group B’ units specify the amount of solute for a solvent in 100 g of
given mass of solvent or solution, such as mass%, solvent
mole fraction, molality, etc. Molality m number of m = lOOOw, mol kg
moles of solute myW2
The advantage of ‘group A’ units is the ease of present in 1 kg
solution preparation and that of ‘group B’ units is solvent
temperature independence. Mole fraction X fraction of a X, = —ft ■ ■ dimensionless
In a given solution, let solute be represented by (1) and component n, + n2
out of total
the solvent by (2) moles present X2 = —
w-l = mass of solute + n2
in solution
w2 = mass of solvent X, + X2 = 1
= molar mass of solute
= molar mass of solvent Standard notation for molality is m and should not be
= moles of solute = — confused with 7nx (molar mass of solute) or (molar mass
of solvent)
= moles of solvent = — Closely related to weight per cent and particularly
Wl2
useful for dilute solutions are the concentration units
wx + w2 = mass of solution parts per million (ppm) and parts per billion (ppb)
294 | Essential Inorganic Chemistry

mass of solute component (a) mass per cent


Parts per million (ppm) = x 106 ( w A mass of solute ,
total mass of solution — =-------------------- x 100%
mass of solute component, VW J mass of solution
Parts per billion (ppb) = x 109 / \
total mass of solution x 100
wx +
k^i w2
(b) volume per cent
Mole Fraction W = .volume of solute x 100%
Fraction of the substance in the mixture expressed in k V J volume of solution
terms of mole is called its mole fraction. = — x 100
nA = moles of A in mixture of A and B V
nB = moles of B in same mixture (c) mass/volume per cent
/w mass of solute, g
nA x 100%
mole fraction of A in the mixture = XA = —■ k V J volume of solution, mL
nA. + nB

nB
= - x 100
mole fraction of B in the mixture = XB = — V
nA + nB • Note that the denominator in each of these
This is thus, the number of moles of a compound expressions refers to the solution rather than to the
solvent.
divided by the total number of moles of all chemical
• (a) and (b) do not depend on the unit employed while
species in the solution. Keep in mind, the sum of all mole
(c) has unit of mass/volume (say g/mL).
fractions in a solution always equals 1.
• Of the three expressions, only mass per cent is
Thus, XA+XB=1 temperature independent.
• To avoid confusion, we generally use terms like % by
mass of solution which means amount of solute
(wx) g present in 100 g solution
Illustration 1 What are the mole fractions of the
= - W1 x 100
components of the solution formed when 92 g glycerol wx + w2
(92 g mol-1) is mixed with 90 g water (18 g mol'1)?
and % by mass of solvent which means amount of
Solution solute (wY) g present in 100 g solvent
Components Step 1 Step 2 Step 3 = x 100
1^2
Glycerol convert amount Take total Mole fraction = • For every dilute solutions, parts per million, 106
into moles moles =1+5 mole of the
amount =6 species (ppm) is a convenient way to express concentration.
molar mass total moles
~ . .. z . mass of solute in6
Concentration (ppm)=-------- ----------- x 10
= — = 1 mol X - 1 mass of solution
92 A glycerol “ g a
• For even more dilute solutions, parts per billion, 10
Water . 90
X (ppb) rather than 106 (ppm) is used.
mole = — — —
''water - g
18 /-I x. / i \ mass of solute
= 5 mol
Concentration (ppb) =-------------------- x 109
mass of solution
Check: Sum of mole fraction is always equal to one • Concentration in parts per thousand (ppt) is also
commonly used.
Xglycerol + XWater = -g + -g = A1 mass of solute
Concentration (ppt)= x 103
mass of solution
Illustration 1 Aqueous urea solution is 20% by mass
Per cent Concentration of solution. Calculate percentage by mass of solvent.
In a solution, let there be W& solute (molar mass = mf) Solution If solute = 20 g
w2g solvent (molar mass = mf)
then solution wx + w2 = 100 g
V mL of solution of density d g/mL
solvent = 100 - 20 = 80 g
There are following types of expressions used to 20
denote % concentration: Hence, percentage by mass of solvent = — x 100=25%
Chapter?: Volumetric | 295

Alternate method Given, ^—x 100 = 20 Volume = 100 mL


wx + w2 = 100 L
wi 1 " 1000
Wi + w2 "5 = 0.1L
Thus, ,, . .. 0.1 mol
w1: (wr + w2) = 1:5 Molanty =---------
0.1 L
or wr: w2 = 1:4
= 1 mol L~
x 100 = 25%
w2
Equilibrium Molarity
Illustration 2 Equal moles of water and urea are The equilibrium molarity expresses the molar
taken in a flask. What is mass percentage of urea in the concentration of a particular species in a solution at
solution?
equilibrium. In order to specify the equilibrium molarity
Solution If H2O = x mol = 18x g of a particular species, it is necessary to know how the
solute behaves when it is dissolved in a solvent.
then urea = x mol = 60x g
• If analytical molarity of NaOH is 0.1 M, then
total mass of the solution = 18x + 60x = 78x g equilibrium molarity of NaOH, [NaOH] = 0.0 M
18x
mass percentage of urea =---- x 100 =23.077% (assuming 100% ionisation).
78x that of OH", [OH-] = 0.1M
and that ofNa+, [Na+] = 0.1M
- NaOH Na+ + OH"
Molarity (M) • If H2SO4 ionises as H2SO4 H+ + HSO4
Number of moles of solute present per litre of the
solution is its molarity. then 1.0 M H2SO4 (analytical) has equilibrium
moles of solute concentration as (assume 100% ionisation)
Molarity (M) = [H2SO4] = 0.0M
litres of solution
[HSO;] = 1.0M
If 0.1 mol of a solute is present in 100 mL (0.1 L)
solution then molarity H+as[H3O+] = 1.0 M
M = — = 1M • However, there is no difference between analytical
0.1 and equilibrium molarity when solute dissolved is
non-electrolyte or no ionisation takes place in
DEEP FOCUS solution. 1M glucose (non-electrolyte), 1 M urea
i (non-electrolyte), etc., are also 1 M as equilibrium
moles of solute = molarity x litres of solution molarity.
millimoles of solute = molarity x mL of solution
Illustration 2 Concentration of glucose (C6H12O6) in
normal blood is approximately 90 mg per 100 mL. What is
Analytical Molarity the molarity of the glucose solution in blood?
The analytical molarity of a solution gives the total 90
number of moles of a solute in one litre of the solution (or Solu tion Amount of glucose = 90 mg = - ~ = 0.09 g
the total number of millimoles in one millilitre). Thus,
analytical molarity specifies the amount by which the =----- = 0.0005 mol
180
solution can be prepared.
• Sulphuric acid solution that has analytical Volume of solution = 100 mL = 0.10 L
concentration of 1 mol per litre can be prepared by moles of solute 0.0005
Molanty =-------------------- =--------
dissolving 1.0 mol or 98 g of in one litre of litres of solution 0.10
solution. = 0.005 M
• Sodium hydroxide solution that has analytical
molarity of 0.1 M has 0.1 mol or 4 g of it in one litre of Illustration 3 Hydrochloric acid is sold commercially
solution. as a 12.0 M solution. How many moles and how many
grams o/HCl are in 300.0 mL of 12.0 M solution?
Illustration. 1 What is the molarity of solution made
moles of solute
when 6 g urea (molar mass 60 g mor1) is dissolved in water Solution Molanty =--------------------
to make 100 mL of solution? litres of solution
6 moles of solute (HC1)
Solution Solute = 6 g = — = 0.1 mol
60 ’ " 0.30 L
296 | Essential Inorganic Chemistry

moles ofHCl = 12 x 0.30 Illustration 8 What is molarity of K+ in aqueous


= 3.6 mol HC1 solution that contains 17.4 ppm K^SQj (174 g moL1)
= 3.6 x 36.5 gHCl Solution KjSC^ is 17.4 ppm i.e.,
= 131.4 g HC1
106 g (= mL) solution has = 17.4 g K2SO4
Illustration 4 How many moles of solute are present IO3 mL solution has =------------ g KjSC^
in each of the following solutions?
(a) 35.0 mL of 0.2 AfHNO3 = 0.0174 gL"1K2SO4
(b) 175 mL of 0.5 M glucose (C6H12O6) [K2SO4] = 5^- = lxl0^M
Solution Molarity x volume in mL = millimoles of solute 4 174
(a) 35.0 mLx 0.2 M = 7.0 millimol = 0.007 mol K2SO4 2K++SOt
(b) 175 mLx 0.5 M = 87.5 millimol = 0.0875 mol Molarity of K+ = 2 x IO-4 M

Illustration 5 How many millilitres of 0.45 M BaCl2 Illustration 9 How many millimoles of solute are
contained in
solution contain 15.0 g ofBaCl2 (208 g mol'1) 2 (a) 2.00L of 2.76x IO"3 MKMnO4?
Solution Let volume of the solution containing 15.0 g (b) 250 mL of a solution that contains 4.20 ppm CuS04?
BaCl2 be V mL Solution (a) Since,
V x 0.45 = 0.45 V millimol = 0.45 V x 10-3 mol volume in mLx molarity = millimol
.-. 0.45Vx10-3=— 2.00 L (= 2000 mL) of 2.76 x 10'3 M KMnO4
208
T7 15 = 2.76 x IO"3 x 2000
V =---- x — = 160.3 mL
208 0.45 = 5.52 millimol
Illustration 6 Calculate the molar concentration of a (b) 4.20 ppm means 106 mL solution has 4.20 g
solution that is 50% in NaOH (w/W) and has a specific CuSO4
gravity of 1.52. 4.20
=------ mol
mass . _o 159.5
Solution Specific gravity= ----------- = 1.52 4 20
volume hence, 250 mL solution has= —-— x mol CuS04
(w.r.t. HgO = 1) 159.5 106
50% NaOH (w/W) means = 6.58x 101-6
’ mol
100 g NaOH solution has = 50 g NaOH = 6.58 x IO-3 millimol
mL NaOH solution has= 50 g NaOH
1.52
1 50
.-. ----- L NaOH solution has = — = 1.25 mol NaOH Molality (m)
15.2 40 • The molality of a solution is defined as the number
Molarity of NaOH= 1.25x 15.2 M of moles of solute per kilogram of solvent (mol kg'1)
= 19 M ,, . ... . . moles of solute
Molality (m) —-------------------- -—-
mass of solvent (kg)
Illustration 7 HC1 is 80% ionised in 0.01 M aqueous
solution. What is its equilibrium molarity? What are other • The main advantage of using molality is that it is
species present? What are their molar concentrations? temperature independent because masses do not
Solution Initial [HC1] = 0.01 M change when substances are heated or cooled.
HC1 H+ + cr • The disadvantages of using molality are that amount
of solutions are measured by mass rather than by
H+ + H^O H3O+ volume and that the density of the solution must be
or HC1 + BjO h3o+ + cr known to convert molality into molarity.
HC1 ionised = 80% of 0.01 M
R0
= 0.01x—= 0.008 M DEEP Focus
100
• Molality (zn) is related to molarity (M) by relation
Equilibrium molarity lOOOAf
[HC1] = 0.010 - 0.008= 0.002 M m =---------------
lOOOd-Mr^
[H3O*1= 0.008 M
where d is the density (g mlT1) and is the molar mass of the
[cr]= 0.008 M solute
Chapter 7: Volumetric | 297

Because the density of water at 25°C is about


* kilogram per litre, molality is approximately equal to
Equivalent Mass and Normality
molarity for dilute aqueous solutions at this temperature. Equivalent mass
Remember I This is only for water (solvent) and if
• Equivalent mass of a substance is the mass which can
solution is not dilute and at other temperature, this
combine or displace. (For more details refer unit 2 and
approximation is not true.
Thus, 18 M HjSC^ (d = 1.8 mL"1, mx = 98 g mol"1) is unit 3)
500 molal. 1 g hydrogens 8 g oxygens 35.5 g chlorines 80 g
1000x18 bromines 127 g iodines 12 g carbon = 17 g OH"
m =--------------------------- = 500
(1000 x 1.8 -18 x 98) N2 +3H2 ---- > 2NH3
1 mol 3 mol 2 mol
28 g 6g 34 g
DEEP Focus
Molality (zn) expressed in terms of mole fraction of solute 6 g Hj s 28 g N2
op
(Aj)is given by relation: lgH^gN,
lOOOXj o
(1 - X, )m2 14 i
equivalent mass of N2 =— g equiv"
3
where m2 is the molar mass of the solvent.
Similarly, equivalent mass of
Illustration 1. 100 g solution of urea in water has 40 g 34 17 . _i
Nn3 = —=—g equiv
urea (molar mass= 60 g moZ"1). What is molality of urea b o
solution? What is mole fraction of urea in solution?
2H> + O2 -----> 2H,0
Solution Mass of urea and water = 100 g 2 mol 1 mol 2 mol
4g 32 g 36 g
mass of urea= 40 g
, r 40 2 32 g O2 s 36 g
moles of urea = — =
60 3 8g O2 = 9g H>O
= 0.667 mol
Thus, equivalent mass of 1^0 = 9 g
mass of = 60 g
(determined w.r.t. oxygen)
moles ofHjO =|j = 3.333 mol
Also, 4g s 36 g HgO
kg of solvent (HjO) = 0.06 kg IgHjeggHjO
Thus, w.r.t. hydrogen equivalent mass of HLjO is also
moles of urea 0.667
molality (m) = 9 g equiv"1
kg ofHgO 0.06 mass
• Number of equivalent =
= 11.12 molal equivalent mass
Total moles of urea and water = 3.333 + 0.667 = 4.00 Normality GV) Concentration of the solution
, P r 0.667 expressed in number of equivalent dissolved per litre of
•*. mole fraction of urea=—— = 0.167 solution is called normality.
4.00
Alternatively An equivalent is a measure of the reactive capacity of
If molality of urea solution is 11.12 mol kg 1 then a given molecule. Normality is the only concentration
unit that is reaction dependent. 1 M sulphuric acid is
using relation:
2 N for acid-base reactions because each mole of sulphuric
1000^!
molality (zn) = acid produce 2 moles of H+ ions. On the other hand, 1 M
(l-X^m? sulphuric acid is 1 N for sulphate ion, since 1 mole
n.i2=_W_ sulphuric acid provides one mole of sulphate ion.
(l-x^is H2SO4 2H+ + SC^"
1-*1 = 1000
-------------- — HgPO4 (ortho phosphoric acid) can be neutralised
*1 11.12x18 stepwise in three steps and depending upon the number of
—=1+5=6 H+ ions neutralised, normality would be different (say we
*1 take 1 MHgPOJ

X, = - = 0.167 HgPO4 + OH" ---- > HjPO; +H2O


1 6 H+ neutralised = 1
298 | Essential Inorganic Chemistry

molar mass Illustration 2 Determine equivalent mass of tl


Equivalent mass =
1 underlined:
Normality = IN (a) H3PO4 + Ca(OH)2 ---- > CaHPO4 + 2^0
H3PO4 +2OH- ---- > HPO2- + 21^0 (b) H3PO4 + 3NaOH---- > Na^ + 31^0
H+ neutralised = 2
t-, . . , molar mass (c) H3PO4 + NaOH---- > Na^PO* +
Equivalent mass =------ ------- (d) 2H, +02 ---- > 2^0
Normality = 2N (e) 2CuSO4 + 4KI---- > Cu^ +K2S04 + I2
H3PO4 +3OH- ---- > PO43- + 3H2O
Solution (a) H;jPO4 is a tribasic acid but in this cai
H+ neutralised = 3
only two equivalent H+ are neutralised by Ca(OH)2.
„ . . , molar mass
Equivalent mass =------ ------- H3PO4 = Ca(OH)2 = 20H"
1 oh- =
Normality = 3 N 2
Normality (N) = .number of equivalent molar mass 98
Thus, equivalent mass of H3PO4 =
litre of solution 2 2
If 0.40 g NaOH is dissolved in 100 mL (= 0.1 L) = 49 g equiv'1
solution, (b) Similarly, H3PO4 = 3NaOH= 30H-
then, number of equivalent of NaOH= °^°= 0.01
40 = 10H“
3
normality of NaOH solution = ™1 = o.in 98 1
0.10 equivalent mass of = — g equiv
3
Also, NV = milliequivalent (x) = equivalent.
(c) HgPO4 = NaOHs 1 0H“
NE = concentration in g L-1. equivalent mass of H3PO4 = 98 g equiv'1
(d) 2Hr> = 02
Illustration 1 Calculate equivalents of the underlined
4H=O2
based on redox, reactions I acid-base reactions.
1H=^_
(а) N2H4 +212 ---- > N2(g) + 4F + 4H+
4
(б) 5H2O2 + 2MnO4“ + 6H+ ---- > 2Mn2+ + 5O2 molar mass = 8 g equiv'
equivalent mass of 02 =
+ 8H2O 4~
(c) B4O27~ + 2H+ + 5^0 — + 4HgBO3 (e) 2CuSO4 = I2 - 21
(d) S(s) + CN- ---- > SCN" CuSO4 = I
equivalent mass of CuSO4 = molar mass
2SCN- + 3IO3 + 2H+ + 6C1’ ---- > 2S0j’ + 2CN*
+ 3IC12+H2O = 159.5 g equiv-1

Solution (a) Illustration 3 At a given condition, 1 mole of Q


N2H4 > N2
occupies volume 30 L. What is volume occupied b
1 equivalent of O2?
O.N. -4 0 (for two N-atoms)
Solution Molar mass of O2 (= 1 mol) = 32 g
change in O.N. = 4
1 mol N2H4 = 4 equivalents N2H4 equivalent mass of 02 = 8 g
hence, 1 mol 02 = 4 equivalent 0,
(b) H2O2 ----- * ^2
-2 0 (for two 0-atoms) If 1 mol 02 occupies = 30 L
30
change in O.N.= 2 then 1 equivalent O2 occupies =— = 7.5 L
4
1 mol H^ = 2 equivalents H^
Illustration 4 0.024 g of Mg on burning in oxygc
(c) B40^- = 2H+ gave 0.40 g o/MgO. What is equivalent mass ofMtf
1 mol B40^- = 2 equivalents Solution Mass of Mg = 0.24 g, mass of MgO = 0.40f
(d) S ---- > SCN" ---- > SO^- oxygen combined = 0.16 g
0 +6 0.16 g oxygen = 0.24 g Mg
1 mol S = 6 equivalents of S 8 g oxygen = 12 g Mg
Thus, equivalent mass of Mg = 12 g equiv'1
Chapter?: Volumetric | 299

HaCA = 2H
Illustration 5 When 6 g of a monobasic acid is
dissolved in 100 mL solution, it is 1N. What is equivalent /. Equivalent mass of
mass of the acid? ~ molar mass
- -—— = 45 g equiv
equivalent of substance
Solution Normality = n=L5M5 = 0133N
litre of solution
I 1N = —/o.l 0.250
D E'
Illustration 7 10 g of MnO2 on reaction with
1 = 0.1
E cone. HCI liberated 0.1 equivalent ofCl2. What is per cent
E = 60 g equiv-1 purity ofMnO2? (Mn =55,0= 16,C1= 35.5)
MnO2 + 4HC1----- > MnCl2 + Cl2 + 21^0
Illustration 6 Oxalic acid, I^CA, forms two series of
salt HCjO; and QA~. 1.5 g of I^CA is dissolved in Solution MnO2 = Cl2
250 mL solution. Determine its normality at each stage of 87 g 71g
salt formation. MnO2 = Cl2 = 2C1
i Solution I^CA H+ +HCA MnO2 87
1C1 = —— = 43.5 g
f^CA s 1 H
1 equivalent ofCl2 = 43.5 g MnO2
Hence, equivalent mass of
HgCA = molar mass = 90 g equiv-1 0.1 equivalent ofCl2 = 4.35g MnO2 (pure)

N = equivalent =W90 = 067 N But, MnO2 taken = 10 g


4 35
volume (L) 0.250 percentage of pureMnO2 =——x 100= 43.5%
HaCaO4 2H+ + CA“

■ Target Practice 1
1. How many grams of Na* are contained is 25 mL of 1 M 9. Calculate the volume of 4.00 M NaOH solution required to
prepare 200 mL of a 0.750 M solution of NaOH
i Na2SO4.....
■ 2. How many moles and millimoles of benzoic acid are 10. Complete the table for the following aqueous solution
contained in 10 mL of 0.5 M benzoic acid Mass solute Volume
3. Calculate molar concentration of ethanol (C^H^OH) in an Solute Molarity
(9) (litres)
aqueous solution that contains 2.3 g of CjHgOH in 3.5 L 0.01 (I)...
(a) HCI 0.365
solution
(b) H2SO4 0.98 (II)... 1.0
■ 4. Describe preparation of 2.00 L of 0.108 M BaCl2 from
(c) h3po4 (III) ... 1.00 0.1
BaCl22H2O(Ba = 137)
5. Describe the preparation of 500 mL of 0.01 M Cl solution (d) NaOH (IV) ... 2.00 0.25
from solid BaCL 2H9O (Ba = 137) Je) Ca(OH)2 7.40 1.00 (V)...
6. Calculate the resulting molarity when 145 mL of 6.00 M
H^C^ is mixed with 245 mL of 3.00 M HjSC^ 11. What is normality if molarity of the underlined solute is 1M?
7. A solution contains 0.10 mole per litre of each of the (a) H/A +Ca(OH)2 * CaHPO+2HO4 Z

following acids : HCI, H2SO4, H3PO4. (b) H3PO3 + NaOH - NaH^Og+HjjO..........


(a) Is the molarity the same for each acid? » Ca(OH)H2PO2 +H2O
(c) H3PO2+Ca(OH)2
(b) Is the number of molecules per litre the same for each
acid?
(c) Is the normality the same for each acid? (d) (NH4)2SO4 + 2NaOH Na2SO4 +2NH3 +2H2O
8. Calculate the volume of 0.0600 M Ba(OH)2 solution that
contains the same number of moles ofBa(OH)2 as 180 mL of (e) Ba(OH)2 +2HC1 BaCl,+2H2O
0.09 M Ba(OH)2 solution?

r Answers
= 25 millimol of Na2SO4
1. Na2SO4 2Na*+SO4“
i = 50 millimol of Na+
One mole furnishes two moles of Na* ions *
Moles of Na* =----- = 0.050
millimoles = volume (mL) x molarity 1000
= 25x1 Mole x molar mass = 0.050 x 23 = 1.15 g Na*
'4
300 | Essential Inorganic Chemistry

2. Benzoic acid = 10 x 0.5 millimol Mol


7. Molarity =
L
= 5 millimol = 0.005 mol
moles HC1 h2so4 h3po4
3. Molarity =
volume in litre (a) 0.1 M 0.1 M 0.1 M Molarity same-Yes
2.3 gC2H5OH = || = 0-05 mol (b) 0.1xN0 O.lx No O.lx No Molecules same-Yes
(c) 0.1 N 0.2 N 0.3 N Normality same-No
Molarity = = 0.0143 mol L-1 (2V0 = Avogadro’s number)
3.5
8. It is a case of dilution
4. BaCl2-2H2O = 2x0.108
= M2V2
= 0.216 mol 0.06 x Vi = 0.09 x 180
= 0.216 x 244 g V! = 270 mL
= 52.704 g 9. It is a case of dilution
We dissolve 52.704 gBaCl2 -2H2O in 2.00 L solution MMx=M2V2
4 x VT = 0.75 x 200
5. 0.01 MCI' = — MBaCl2-2H2O
2 Vj = 37.5 mL
= 0.005 MBaCl2 -2H2O 10. I—IM
= 0.005 mol L"1 BaCl, -2H2O II — 0.01 L
III — 9.8 g
= 0.0025 mol in 500 mL
IV — 20.0 g
Dissolve 0.0025 mol (= 0.61 g)BaCl2 ■ 2H2O in 500 mL solution. V —0.1M
_ D ... , .. 1MV Normal) tj
6. Resulting molarity =
11. (a) Two H + ions are neutralised 2N
„ -b M2V2
(b) OneH+ ion is neutralised IN
Vi + v2
(c) One H * ion is neutralised IN
_ 6 x 145 + 3 x 245
(d) One mole (NH4 )2SO4 is neutralised
145 + 245 by two (OH“) ions 2N
= 4.0125 M (e) One mole Ba(OH)2 is neutralised
by two HC1 2N

Practice Exercise 1
1. How many equivalents of hydrogen are in 1 equivalent of (b) 1.575 g of a diabasic acid H2C2O4 xH2O in 250 ml
H„PO ? solution is 0.1 N. What is the value ofx?
J 4
2. 0.400 g of iron (III) sulphate are present in 100 mL of 9. Urea solution is 20% by weight of solution. What i
solution. Express concentration of iron (III) and sulphate percentage concentration by weight of solvent?
ions in terms of 10. Calculate normality of:
(i) ppm (ii) molarity (a) 0.01 M Ba(OH)2 (b) 0.01 M H3PO3
3. H2SO4 (d = 1.80 g mL''1) is 98% by weight of solution. (c) 0.01 M H3PO2 (d) 0.01 M H„PO solutions.
U 4

Express concentration in terms of molarity of 11. What is weight of sucrose in 1 kg solution of 1 mil­
concentration?
(i) hydrogen ion, (ii) sulphate ion
12. One litre of 1 M solution of following are to be prepare!
4. AfClx is 100% ionised in 0.01 M aqueous solution. If
Which has maximum weight of solute?
[Crj= 0.03 M, what is the formula? Also calculate [Afx+] (Mn = 55, K = 39, Cr = 52, Fe = 56)
and [AfClx]. (a) KMnO4 (b) K2Cr2O,7
5. A solution of Na2SO4 is 50% ionised. What is analytical (c) FeSO4 • (NH4 )2SO4 • 6H2O
molarity of Na2SO4 if[Na+]= 1.0 M? 13. 2.78 g FeSO4 xH2O in 100 mL solution gives 0.1M solution
6. 6 g of an urea sample (molar mass = 60 g mol-1) was What is the value ofx?
dissolved in 100 mL aqueous solution and was 0.8 M. 14. CH3CH2OH solution in water is 46% by weight of solatia
Comment.
What is mole fraction of each component? What is molalir
7. The same amount of a metal combines with 0.4 g of oxygen as water solvent? What is molality as ethanol solvent?
and 4 gof a halogen. Determine equivalent mass of halogen.
15. Calculate volume of water that must be added to 250 ml o
8. (a) The equivalent mass of a metal is double that of 1.25 N solution to make it 0.500 N.
oxygen. How many times is the mass of its oxide
16. What volumes of 12.0 N and 3.00 N HC1 must be mixed t
greater than the mass of the metal?
give 1.00 Lof 6.00 NHC1?
Chapter 7: Volumetric | 301

Answers
1. 1 equivalent of H3PO4 has 1 equivalent of H* (as well as Molarity = m0^e = — = 1M
1 equivalents ofPO4") litre 0.1
2. Fe2(SO4)3 ---- > 2Fe3* + 3SO2" But it is 0.8 M (< 1 M)
iron (III) sulphate Thus, urea is 80% pure.
Molar mass ofFe2(S04)3 = 400 g mol-1 7. Equivalent of oxygen = Equivalent of halogen
0.4 — —
--- 4
Moles of Fe2(SO4)3 =9^22 =0.001 mol
8 E
„ 4x8
Volume of solution = 100 mL = 0.1 L = 80 g equivalent'
0.4
.’. Molarity ofFe2(SO4 )3 = = 0.01 M
8. (a) Equivalent mass of oxygen = 8
(i) Fe3* = 0.001 x 2 = 0.002 mol = 0.002 x 56 g Equivalent mass of metal = 16
= 0.112 g in 100 mL Equivalent mass of metal oxide = 24
0.112 xlO6 t equivalent mass of metal oxide 24 . -
=-------------- g in 106 mL Thus, --- = 1.0
100 equivalent mass of metal 16
= 1120 ppm (b) Molar mass ofH2C2O4 xH2O = (90 + 18x)
SO2" = 0.001 x 3 mol Moles ofH2C2O4 xH2O = 1575
= 0.001 x 3 x 96 g in 100 mL 224 (90 +18x)
0.001 x 3 x 96 x 106 . _
=------------------------- g in 10 mL Volume of solution = 250 mL = 0.250 L
100
1.575
= 2880 ppm (90 +18x)
(ii) Molarity :[Fe3*] = 0.01 x 2 = 0.02 M Molarity = m°— =
litre 0.250 ”
[SO^"] = 0.01 x 3 = 0.03 M 6.3
3. H2SO4 = 98% by weight of solution (90 + 18x)
= 98 g in 100 g solution Being a dibasic acid
mass
= 98 g in mL solution Volume = normality = 2 x molarity
1.8 density 2 x 6.3
98 = 1 mol
Moles ofH,SO, = — ” (90 + 18x)
2 4 n983
= 0.1 N (given)
100 2x6.3
Volume = — L=—L = 0.1
1.8 x 1000 18 (90 + 18x)
Molarity of H 2SO4 — = 18M x=2
litre 1/18
9. wr = 20 g solute
H2SO'4 2H*+SO4 w2 - mass of solvent
I
[H*] = 36 M u?! + w2 = 100 g
[SO2"] = 18 M w2 = 80 g
4. MC1X Mz+ + xcr 20
0.01 M 0.01 M O.Olx M % by weight of solvent = — x 100 = 25%
There is 100% ionisation 10. (a) 0.02 N [Ba(OH)2 Ba2* + 2OH"]
[Cl"] in MClx=0.03M (b) 0.02 N (H3PO3 has only two ionisable H* ions)
O.Olx = 0.03 (c) 0.01 N (H3PO2 has only one ionisable H * ion)
x=3 (d) 0.03 N (H3PO4 has three ionisable H* ions)
Thus, solute is AfCl3 Focus Hydrogen ion attached to oxygen atom in oxyacids is
[Af3*] = 0.01M ionisable. This decides basicity of an acid.
[MC13] = 0.00 M (none is left) O O O
5. Na2SO4 2Na* + SO2
I I I
H—P—H H—P-O-H H—O—P-O-H
[Na2SO4] =
[Na*] if 100% ionised I I I /
2 O-H O-H O—H/
= [Na *] if 50% ionised I
this H attached two H-attached three H-attached to
=1M to O-atom to O-atoms O-atoms (tribasic)
monobasic H3PO2 dibasic H3PO3 h3po4
6. Moles of urea = — = 0.1 mol
60 11. 1 molal sucrose means 1 mol sucrose (= 342 g) in 1 kg water.
Volume = 100 mL = 0.1 L Total weight of solution = 1000 + 342
= 1342 g
302 | Essential Inorganic Chemistry

1342 g solution has sucrose = 342 g mole of solute (CH3CH2OH)


Molality as water solvent =
Thus, 1000 g solution has sucrose =----- x 1000 = 254.84 g kg of solvent (H2O)
1
12. Moles in one 0.054
Molecular
Solute litre of 1 M = 18.52 molal
weight
solution Molality as ethanol (CH,CH„OH) solvent
KMnO4 158 g mol 1 mol _ mole of solute (H2O)
K2Cr2O7 294 g mol 1 mol kg of solvent (CH3CH2OH)
FeSO4 • (NH4)2SO4 • 6H2O 392 g mol 1 mol 3
0.046
Thus, maximum weight is of FeSO4 • (NH4 )2SO4 • 611,0 = 65.22 molal
13. Molar mass of FeSO, • xH„O = 152 + 18x 15. It is a case of dilution :
42 9 7R
Moles ofFeSO, xH,0 = ——— •NV=NV
£vivi ^*2’2
4 2 (152 + 184) 0.5 x =1.25x250
Volume = 100 mL = 0.1 L Vr =625mL
„ , 2.78/(152+ 18x) Thus, additional water to be added = 625 - 250
Molanty =-------------------
0.1 = 375mL
2.78 '0.1 =0.1 (given) 16. If volume of 12.00 N HC1 = x L
Thus,
152+ 18x then volume of 3.00 N = (1 - x) L
x=7 Final normality = 6.00 N
14. CH3CH2OH = 46% by weight of solution NV +NV
N (final) = 1 1 2-2-
Thus, CH3CH2OH = 46 g = 1 mol = 0.046 kg V1 + V2
H2O = 54 g = 3 mol = 0.054 kg 12x + 3(1 - x)
6=
Total moles = 4
Mole fraction of CH ,CH„OH = PH =- x = 0.333 L
Thus, volume of 12 N HC1 = 0.333 L
Mole fraction of H„O
2
= X,HjOn = -4 and volume of 3 N HC1 = 0.667 L

Volumetric Titrations 2NaOH + H2SO4 -----> Na2SO4 + 2^0


While diluting a solution from one concentration to Here, ATONaOH) = N2V2(H2SO4)
another, we can use
but M^NaOH) = 2M2V2(H2SO4)
Ig + 2Na2S2O3 -----> Na2S4O6 + 2NaI
■*

and W = N2V2
MV1d2) = N2V2(Na2S2O3)
(concentrated solution) (diluted solution)
but 2M1V1(I2) = M2V2(Na2S2O3)
• For a redox reaction or neutralisation reaction MnO4 + 5Fe2+ + 8H+ -----> Mn2+ + 4H2O + 5Fe.3*
:
jVjVj = N2V2 is always true
Here, N^fFe2*) = N2 V2(MnO4“)
but Mi Vi = M2 V2 is not always true
but M^Fe2*) = 5M2V2(MnO4)
• Normality = change in O.N. x molarity in a redox
reaction
DEEP Focus
(O.N. = oxidation number) Thus, it is always better to convert molarity factor inti
• Normality = molarity x basicity (of an acid) normality factor by multiplying it with change in O.N. oracidin
or basicity.
- molarity x acidity (of a base)
(MnO; ---- > Mn2+), 1 M MnO4 = 5 N Mn04
NaOH + HC1-----> NaCl +
(Fe2+ ---- > Fe3+), 1 M Fe2+ = 1N Fe2*
Here, W = N2V2 IMC# =6NCr)03;*
(C^O2,- ---- > 2Cr3+),
and also (HjSQj ---- > 2H+ + SOj"), 1 M H2SO4 = 2 N W,
Chapter 7: Volumetric | 303

• Normality of a mixture of two or more acids or bases • In all cases : equivalent of an acid = equivalent of a
(strong) base
NiV1 + N2V2+... I NV • When neutralisation point is reached NjVj = N2 V2
=
v1 + v2+... EV End point is the point of completion of the reaction
indicated by suitable indicator. Hence, it has additional
where NltN2,... are normalities and Vn V2... are volumes drop of titrating reagent but, we use
of acids or bases.
W = x2v2
• Normality of mixture of acid and bases (strong)
NaOH + HCI---- > NaCI + H2O
let N1 Vj = milliequivalent of acid
1 equivalent of NaOH a 1 equivalent of HCI
N2V2 = milliequivalent of base
- If NjVj = N2V2 mixture is neutral = 1 mol of HCI
~ If NlV1> N2V2, then mixture is acidic and = 1 equivalent of H2SO'44
resultant normality of acidic solution is: = 0.5 mol of H2SO4
Na2CO3 + 2HC1---- > 2NaCl + HjO + CO2
TV(resultant) = MVi - n2v2
Vi + v2 Na2CO3 + H>SO4 ---- > Na2SO4 + E^O + CO2
~ If N1V1< N2V2, then mixture is basic and 1 equivalent of Na2CO3 a 0.5 mol of Na2CO3
resultant normality of basic solution is: a 1 equivalent of HCI
//(resultant) = N2V2-MV1 = 1 mol of HCI
Vi + V2
= 1 equivalent of HgSO^j
= 0.5 mol of H2SO4
Acid-Base Titrations
Acid or acid mixture can be titrated against a suitable
base or vice-versa using a suitable indicator. These are Titration of Mixture of Bases with Two
summarised in Table 7.3 Indicators
Every indicator has a working range
Table 7.3 Acid-Base Titrations
Colour change Indicator pH range Behaving as
Combination Suitable indicator
I »
in acid In alkali Phenolphthalein 8-10 weak organic acid
1. Strong acid/ Methyl orange Red Yellow Methyl orange 3-4.4 weak organic base
strong base Bromothymol blue Yellow Blue
2. Strong acid/ Methyl red Red Yellow Thus, methyl orange with lower pH range can indicate
weak base complete neutralisation of all types of bases.
3. Weak acid/ Phenolphthalein Colourless Purple
strong base
4. Weak acid/ Not suitable for a
weak base titration

Extent of reaction of different bases with acid (HC1) using these two indicators is summarised below :
Mixture Phenolphthalein Methyl orange
NaOH 100% reaction is indicated 100% reaction is indicated,
NaOH + HCI —> NaCI + H2O NaOH + HCI NaCI + H2O
Na2CO3 50% reaction up to NaHCO3 stage is indicated 100% reaction is indicated,
Na2CO3 + HCI —> NaHCO3 + NaCI Na2CO3 + 2HCI---- > 2NaCI + H2O + CO2
NaHCO3 No reaction is indicated 100% reaction is indicated
NaHCO3 + HCI---- » NaCI + H2O + CO2

Suppose volume of given standard acid solution (say HC1) required


for complete reaction of Na2CO3 = x mL
for complete reaction of NaHCO3 = y mL
for complete reaction of NaOH = z mL
304 | Essential Inorganic Chemistry

There may be different combination of mixture of bases. We may adopt two methods:
Method I: We carry two titrations separately with two different indicators.
Method II: We carry single titration but adding second indicator after first end-point is reached.
These have been summarised in Table 7.4
Table 7.4 Results with Two Indicators
Method I Method II

Volume of HCI used with indicator Volume of HCI used with Indicator
Mixture
Methyl orange added after first
Phenolphthalein Methyl orange Phenolphthalein end-point is reached
1. NaOH + Na2CO3 (x + z) x
Iz+ -j z+*
I ?) 100% each I 2/ 2
T T ? ? (remaining 50% Na2CO3 is
100% 50% 100% 50% indicated)
2. NaOH + NaHCO3 z + 0 (y+z) (z+0) /(remaining 100% NaHCO3 is
100% no change 100% each indicated)
x
3. Na2CO3 + NaHCO3
2
T
+ 0
T
50% no change
(x+ y)
100% each
-+ 0
2 (H
(remaining 50% of Na2CO3 and
100% of NaHCO3 are indicated.)

• Bromocresol-green also indicates complete reaction of :. 0.6010 g AgCl is obtained from=^^~^- mol HCI
Na2CO3, NaHCO3 and NaOH, thus its behaviour is
similar to methyl orange. = 4.188x 10"3 mol HCI
• Bromocresol can also be used as an indicator for the This amount of HCI is contained in 50 mL solution
determination of phosphates when reaction of acid (= 0.050 L)
(say HCI) up to H^PC^ is indicated. 4.188X IO-3
PO^ + 2H+ ---- > B^PO; molar concentration of HC1=
0.050
• Thymolphthalein indicates reaction upto the
= 0.08376 M
formation ofHPO^".
PO3" + H+ ---- > HPO2’
Illustration 2 A 1.00 g sample of a metal X (that is
Illustration 1 Calculate the molar concentration of a known to form X2+ ions) was added to 0.10 L of 0.50 M
dilute HCI solution if HgSO^ After all the metal had reacted, the remaining acid
(a) titration of a 50 mL of 0.04 M Ba(OH)2 required required 0.0334 L of 0.50 M NaOH solution fv
20 mL of the acid. neutralisation. Calculate molar mass of the metal and
(b) 50 mL of HCI yielded 0.6010 g AgCl identify the element.
Solution (a) Ba(OH)2 +2HC1 ---- > BaCl2 +2^0 Solution X + H2SO4 —> XSO4 + Hj
0.04 M Ba(OH)2 = 0.08N Ba(OH)2 or X + 2H+ ---- > X2+ + H2
N1V1 = N2V2 2H+ + 2OH- ---- > 2HgO
HCI Ba(OH)2 1 mol HgSC^ = 1 mol of X = molar mass of X
NjX 20 = 0.08x50 Let unused acid = V L
N1(HCl)=0.20N then VLof 0.5 MHjSO^ 0.0334 Lof 0.5 MNaOH
molar concentration = 0.20 M or V L of 1 N H2SO4 = 0.0334 L of 0.5 N NaOH
(b) Ag+ + cr ---- > AgCl V = 0.0167 L of
(from HCI)
H2SO4 used = 0.10 - 0.0167 = 0.0833 L
1 mol 1 mol = 143.5 g
Number of moles of = 0.0833 x 0.5 mol i
? 0.6010 g
= 1 g metal X
143.5 g AgCl is obtained from= 1 mol HCI
Chapter 7 . Volumetric | 305

1 WT r. r 1.4 NV 1.4x0.1181x24.61
1 mol ofH2SO4 = g metal X Note: % of nitrogen =--------
0.0833x 0.5 1 w 1.047
where, N = normality of acid
= 24.01 g mol
V = volume of acid required to neutralise NH3
Metal is Mg. w = weight of sample
Note: This is a case of back-titration.
Illustration 4 A 0.9092 g sample ofa wheat flour was
analysed by the Kjeldahl procedure. The ammonia formed
Kjeldahl Method was distilled into 50 mL ofQ.Qb M HC1. A 7.50 mL of 0.05
Kjeldahl method is used to determine amount of M NaOH was required in back titration. Calculate
N-containing compounds asNH^N^CONH^NI^Cl, etc. percentage of nitrogen in the flour.
Compound is decomposed by alkali (NaOH, Ca(OH)2, etc.)
and NH3 formed is absorbed by excess of standard HC1 or Solution Nitrogen content (protein) of the flour was
H2SO4 solution. Unreacted acid is determined by converted into NH3 which was absorbed into HC1.
back-titration. N---- > NH3 NHL44 Cl
If V is the volume of acid or normality (N) required to
absorb NH3 from w g of the compound then, Let unreacted HC1 be = V mL
NH3bH+ V mL of 0.05 HC1 = 7.50 mL of 0.05 M NaOH
NV V = 7.50 mL
Equivalent of acid a equivalent of NH3 =
used HC1= 50.00 - 7.50= 42.50 mL
17 NV moles of HC1 = moles of NH3 = moles of N = 42-50x °-05
NH3 present= ---------- g
1000 3 1000
17 NV = 2.125x IO-3 mol
w g of the compound has = g NH3
nitrogen in the flour sample = 2.125 x 10-3 x 14 g
17 NV x 100
100 g of the compound has = NH3 = 0.02975 g
1000 oF
1.7 NV M c .x 0.02975
% of mtrogen =---- x 100= 3.272%
%ofNH3 = 0.9092
w
1.4 NV
% in terms of N= since N = NH3 = H+
w
Redox Titrations
% in terms of NH4C1 = 5-35NV since NH4C1 = NH3 = H+
w Reducing agents (R.A.) like Fe, Fe2+, H2O2, CaOCl2,
6 6 NV MnO2, etc. can be determined by suitable oxidising agents
% in terms of (NH4)2SO4 = —-----
w (O.A.) like MnO4, C^O^-, etc. in acidic or basic or neutral
since (NH4)2SO4 = 2NH3 = 2H+ medium. These are called redox titrations. For example,
3 0 NV Fe2+ is oxidised to Fe3+ by MnO4 which in turn is reduced
% in terms of NHaCONHj = —-----
w to Mn2+ in acidic medium.
since NH^ONF^ = 2NH3 = 2H+ 5Fe2+ + MnO4 + 8H+ ---- > Mn2+ + 5Fe3+ + 4^0
Illustration 3 A 1.047 g sample of canned tuna was Oxidising agents (O.A.) and reducing agents (R.A.) are
analysed by the Kjeldahl method', 24.61 mL of 0.1181 M summarised in Table 7.5.
HC1 were required to titrate the liberated ammonia. This will certainly help to choose a particular O.A. or
Calculate the percentage of nitrogen in the sample.
R.A. in a particular redox titration.
Solution N(in the sample) —> NH3 NH4C1 In all cases, again, we remind
moles ofHCls moles of NH3 = moles of N ^M = n2v2
24.61X 0,1180
but MxVx*M2V2
• ■

1000 mol. wt.


= 2.9064 x 10"3 and Eq. wt. =
total change in O.N.
N in the given sample = 2.9064 x 10-3 mol
mol. wt.
= 2.9064 x lO^x 14 g
number of electrons gained or lost
= 0.0407 g
Thus, in acidic medium MnO4 is changed to Mn2+ by
% of nitrogen = x 100= 3.89% gain of 5 electrons or there is change of 5 units in O.N.
hence,
306 |. Essential Inorganic Chemistry

Eq. wt. ofMnO^" (acidic medium) = M/5 Again note this point that it will cause error in resuli
Following example will certainly help, why to take if not considered.
total change in O.N. while determining equivalent weight Again take the following examples :
Cr2O^“ is changed to Cr3+ (a) CzO^ ---- > 2CO2 + 2e~
O.N. =6 4 for two carbon atoms
Cr-jO2," ---- > 2Cr3+
change in O.N. = 2
O.N. of two Cr atoms = 12 on LHS (6 per atom)
O.N. of two Cr atoms = 6 on RHS (3 per atom) eq. wt. of CzO^- = M/2
(b) HjOz (reducing agent)---- > O2 + 2H+ + 2e"
Hence, O.N. = - 2 0 for two oxygen
atoms
We can take it like this also change in O.N. = 2
-(Cr2O^ ) -> Cr3+ eq. wt. of H2O2 = ~
O.N. 6 3 (c) HzO2 (oxidising agent) > 2e-+2H++2H20
change in O.N. = 3 O.N. =-2 -4
eq wt - ^ormu^a as given in equation) for two oxygen atoms
change in O.N.
change in O.N. = 2
M/2 M M
eq. wt of H2O2 = —
3 6
Table 7.5 Oxidising Agents (O.A.)/Reducing Agents (R.A.) with Equivalent Weight
Electrons
Species Changed to Reaction exchanged or Eq. wt
change in O.N.
1. MnOHO.A.) Mn2+ in acidic medium MnO; + 8H+ + 5e~ Mn2+ + 4H2O 5 E-“
5
2. MnO;(O.A.) MnO2 in dilute basic* MnO; + 3© + 2H2O---- > MnO2 + 4OH 3 E-“
medium 3
3. MnO; (O.A.) MnO2" in neutral medium MnO; + ©“ ---- > MnO2- 1 E^
1
4. Cr2O2- (O.A.) Cr3+ in acidic medium Cr2O2- + 14H* +6e’---- » 2Cr3+ + 7H2O 6 E-“
6
5. MnO2 (O.A.) Mn2+ in acidic medium MnO2 + 4H+ +2e" —> Mn2+ + 2H2O 2 -
e--m
2
6. Cl2 (O.A.) cr Cl2 + 2e" —> 2CI" 2 E^
(in bleaching powder) 2
7. CuSO4(O.A.) Cu+ Cu2+ +e" —> Cu+ 1
(in iodometric titration)’ 1
8. S2O2' (R.A.) s4o2 2S2O2 * S4O6 + 2e“ 2 (for two ions) E= —= M
2
9. H2O2(O.A.) H2O H2O2 + 2H+ + 2e~ —> 2H2O 2
2
10. H2O2(R.A.) O2 H2O2 —> O2 + 2H+ + 2e" 2 E= —
(O.N. of oxygen is H2O2 is (-1) per atom) 2
11. Fe2+ (R.A.) Fe3+ Fe2+ —> Fe3+ + e" 1
1

♦In strong basic medium (which is not practically taken) 4MnO; + 4OH"---- > 4MnO2" + O2 + 2H2O, E = ^-

**In S2O|", there are two types of sulphur, terminal one with oxidation number -2 and central one with oxidation
number +6, thus net value of oxidation number of +4. In S4O|", oxidation number of two types of sulphur are 0 (zero) and
five and thus net value of+5. When S2O3- is oxidised to S4O^-, oxidation number increase by two units for two S-atoms.
Chapter 7: Volumetric | 307
■!

DEEP Focus x mL O, at STP are from = ——— mol H2O2


22400
“Volume’ concentration ofHQO2 can be expressed in terms -------mol H„O„ are in 1 mL H„O„ solution.
of normality or molarity. 22400------- 22 22
2H2O2 ---- > 2H2O + O2 Hence, molarity of “x volume” H2O2
2 mol 22400 mL at STP
= -A_ x 1000 = — M
IfH2O2 is ux volume” it means 1 mL ofH2O2 at STP gives 22400 11.2
xmL02. xN
Normality of “x volume” H2O,, =
From the above equation,
22400 mL of O2 at STP are from = 2 mol H2O2

Redox titrations (excluding iodometric and iodimetric) are summarised in Table 7.6
Table 7,6 Redox Titrations (Excluding lodometric/lodimetric)
Estimation of By titrating with Reactions Relation* between O.A. and R.A.
1. Fe2+ MnO; Fe2+ —> Fe3* + e' 5Fe2+ = MnO;
MnO; + 8H* + 5e" ---- > Mn2* + 4H2O Eq. wt.Fe2 + = M/1
Eq. wt. MnO; =M/5
2. Fe2+ Cr2O| Fe2+ —> Fe3+ + 6Fe2+ = Cr2O2-
Cr2O|~ + 14H* + 6e“ ---- > 2Cr3+ + 7H2O Eq. wt. Cr2O2- = M/6
3. C2O*- MnO4 C2O?" —♦ 2CO2 + 2e" 5C2O2‘ = 2MnO4
MnO; + 8H+ + 5e" —> Mn2+ + 4H2O Eq. wt. C2O^ =MI2
Eq. wt. MnO; = M/5
4. H2O2 MnO4 H2O2 —♦ 2H* + O2 + 2e~ 5H2O2 = 2MnO^
Eq. wt. H2O2 = M/2
Eq. wt. MnO; =M/5
5. As2O3 MnO; 2AsO:3' + 1OH* + 4e~
As2O3 + 5H2O —> 2AsO Eq. wt. As2O3 =M/4
6. AsO3' BrO3 AsO3" + H2O---- > AsO?" + 2H* + 2e" Eq. wt. of AsO3' =M/2
BrO3 + 6H+ + 6e" ---- > Br" + 3H2O Eq. wt. ofBrO3 =M/6

*Af is molar mass of O.A. or R.A.

Illustration 1 A solution o/KHCA AOA -21^0 is Illustration. 2 The concentration of a hydrogen


Q.2Nas an acid. What is its normality as reducing agent? peroxide solution can be conveniently determined by
titration against a standardised potassium permanganate
Solution KHQ>04 AQA K+ + 3H+ + 2CA"H+ solution in an acidic medium according to the following
ion is neutralised by OH" equation:
H+ + OH" ---- > HjO 2MnO4 + 5HA + 6H+ -----> 5O2 + 2Mn2+ + 8^0

GjO?" ion is oxidised hence, it is a reducing agent If36.44 mL of0.01652 MKMnO4 solution are required
to completely oxidise 25 mL of a H2O2 solution, calculate
GA- ---- > 2CO2 + 2e" the molarity of the HA solution.
Since, there are three ionisable H+ ions hence,
Solution Method I
Nj (acid), normality= M x3 reduction ? Mr2+ Change in O.N. Normality
and since there are two CA“ ions exchanging four MnO;
+7 +2
electrons, hence, 5 units =5xM
N2 (reducing agent), normality = molarity (Af) x 4
As an acid 3M = 0.2 N (given) (for two 0 atoms) HA O2 2 units = 2xM
-2 0
iz 0-2 M
M =—=— Thus, 25.00 mL of
3 15
1 4 H2O2 = 36.44 mL of 0.01652 M MnO;
N2 (reducing agent) = 4 x — = — = 0.267 M
Using, = N2V2
308 ] Essential Inorganic Chemistry

25.00 x 2MX H2O2 = 36.44 x 5x 0.01652 M MnO; Equivalent weight of = mo^ecu^ar


w 36.44x 5x 0.01652 2
1 25x 2 Molarity = 2 x normality
Equivalent weight of MnO4 = m0^ec.u)ar we*£—
• = 0.0602 M
I 5
Method II Number of moles of Molarity = 5 x normality
U j 36.44x 0.01652
MnO4 used=------------------- Let molarity of MnO4 =Mt
1000
normality = 5A4\
= 0.602x 10‘3 mol 28.85x 5M2
Number of moles ofH2O2 = 2.5 x mol of MnO4 Number of equivalents of MnO4 =
1000
= 1.505 x IO-3 mol 0.1467
Let molarity of H2O2 = Number of equivalents of Na2C2O4 =
67
25 x M i 28.85 x 0,1467
then number of moles of H.2O2 =
1000 1000 ” 67
25 x = 0.01518 M
= 1.505 x 10'3
1000 Method II (Based on stoichiometric molar coefficients)
1.505x 10-3x 1000 = 0()602M i rur 28.85MX
Moles of Mn04 =--------- -
25 4 1000
0.1467
Illustration 3 KBrO3 can be used to determine the Moles of Na2C2O4 =
134
concentration of As(III)
5 mol C^C^- = 2 mol MnO4
3H3AsO3 + BrO3 ---- > Br" + 3H3AsO4 0.1467 . ~ 2 0.1467 .,, n-
What is the molar concentration of As(III) in a solution -------- mol CioOi = - x mol MnO4
134-------- 2 4 5 134
if 22.35 mL of 0.10 M KBrO3 is needed to titrate 50.00 mL
of As(III) solution? 2 0.1467 28.85^
-x
5 134 1000
Solution Let molar concentration of a solution
= 0.01518 M
• containing As(III) = M1
Change in O.N. Normality Illustration 5 A 100 mL of sample of water was
treated to convert any iron present to Fe2+. Addition of
H3AsO3 oxidatio^ H3AsO4 2 units 2 x molarity 25 mL of 0.002 M K^Ci^O? resulted in the reaction :
+3 +5
6Fe2+ +Ci2O27- + 14H+ ---- > 6Fe3+ + 2Cr3+ + 7Hj,0
BrC£ reduction * Br“ 6 units 6 x molarity The excess was back-titrated with 7.50 mLof
+5 -1
0.01 M Fe2+ solution. Calculate the parts per million of
NiVi = N2V2 iron in the sample
As(III) BrO^ Solution In the given reaction equivalents of C12O7’;
2xM1x50= 6x O.lOx 22.35 = equivalents of Fe2+
6x0.10x22.35 Unreacted Ci^O?- has been determined by titration
M-, =------------------
1 100 with standard 0.01 M Fe2+ solution.
= 0.1341 M Method 1 Let unreacted Ci^O?" = V mL
Illustration 4 Titration of 0.1467 g of primary 0.01 MFe2+= 0.01 NFe2+
standard Na2C2O4 required 28.85 mL of KMnO4 solution. 0.002 M CrjO2/ = 0.012N C^O2/
Calculate the molar concentration of KMnO4 solution. MVi = n2v2
Solution +2MnO4 + 16H+ ---- > 2Mn2+ CijjO2/ Fe2+
+6 +7 +2
0.012x7 = 0.01x7.50
+ 10CO2 + 8H2O
8 V (unreacted C^O?") = °‘Qlx7’5° = 6.25mL
Method I (Based on number of equivalent) 7 0.012
In a reaction (redox/neutralisation) number of :. Volume of CrjOV used by Fe2+ (in water sample)
equivalents of different reactants are same. = 25.00-6.25
Equivalent ofCjO^- = equivalents of MnO4 = 18.75 mL
Chapter 7: Volumetric | 309

18.75x0.012
/. Equivalents of Ci^O?" used = Illustration 7 The SO2 present in air is mainly
1000 responsible for the acid-rain phenomenon. Its
= 2.25xl0“4 concentration can be determined by titration against a
•*. Equivalents of Fe2+ in water = 2.25 x 10-4 standard permanganate solution as follows :
Thus, Fe present in 100 mL water = 2.25 x 10-4 x 56 g 5SO2 + 2MnO4" + 21^0 5S0g“ + 2Mn2+ + 4H+
= 0.0126 g Calculate the number of grams of SO2 in a sample of
air if 7.37 mL ofQ.WS M KMnO4 solution are required for
Fe parts per million = 0 0126 x 106 the titration.
100
= 126 ppm Solution Number of moles of MnO4 required
Method II Volume of CigO^- used = 18.75 mL 7.37x 0.008 _QQC in_5 .
=-------------- = 5.896 x 10 mol
... j 18.75x0.002 1000
Moles of C19O7 used - ---------------- Also, number of moles of
7 1000
SO2 = 2.5 x number of moles of MnO4
= 3.75xl0"5 mol
= 1.474 x 10-4 mol
Moles of Fe2+ (based on reaction) = 6 x C^Oy” SO2 in air sample = 1.474 x 10^ x 64 g
= 6 x 3.75 x 10’5 mol = 9.434 x 10"3 g
Fe present in 100 mL = 2.25 x lO^1 mol
= 2.25 x 10-4 x 56 g
lodometric/lodimetric Titrations
= 0.0126 g
Compounds containing iodine are widely used in
Fe parts per million = 126 ppm titrations.
(i) Iodide ions can be oxidised to 1g by suitable
Illustration 6 Treatment of hydroxylamine (NB^OH)
oxidising agents
with an excess o/'Fe(III) results in the formation ofTLfoand 2I-(ag) ---- > ^(s) + 2e~
an equivalent amount of Fe(II)
(ii) Iodate (V) ions, IO3, will oxidise T to lg
2NH2OH + 4Fe3+ ---- > N2O(g) + 4Fe2+ + 4H+ + HgO
lO^aq) + 5T(aq) + 6H+(ag) ---- > 31g(s) + SJijCXZ)
Calculate the molar concentration of an NHgOH
solution if Fe(II) produced from 50 mL solution required (iii) Thiosulphate ions, (S2O^-) can reduce iodine to
23.61 mL ofO.0217 M K2Cr2O7. iodide ions.

Solution Fe2+ obtained by reduction ofFe3+ by NI^OH 2S2O23"(aq) +Vs) ---- > S4O26’ +21"
colourless black colourless
is estimated by C^O?“. Although solid iodine is black (deep violet) )and
2NHgOH + 4Fe3+ N2O(g) + 4Fe2++ 4H++H2O insoluble in water, it is converted into soluble triiodide
6Fe2++Ci2O27- +14H+---- > 6Fe
6Fe 3+ +2Cr3+ +7H2O ions (I3)
Based on stoichiometric coefficients : W +r Vag)
black dark brown
lCr^f =6Fe2+=3NH2OH
Small amount of iodine molecules (lg), gives blue
or 1 mol Cr2O^- = 3 mol NHjOH colour with starch, hence, completion of the reaction can
23.61x0.0217 be detected when blue colour disappears at the end-point.
mol C12O7
1000 It should be noted that starch, being colloidal, can adsorb
3x23.61x0.0217 I2 hence, it should be added when there is very small
=--------------------- mol NHoOH
1000 amount ofl^ left (indicated by appearance of pale yellow
= 1.537 x 10-3 mol NH2OH in 50 mL solution colour) otherwise exact point is not detectable.
1.537 x 10-3 In all the titrations, KI is added to oxidising agent and
Molar concentration of NH2OH = - —
formed is estimated by sodium thiosulphate solution.
= 0.0307 M These are summarised in Table 7.7.
310 | Essential Inorganic Chemistry

Table 7.7 Iodometric Titration (Titration Solution is Na2S203 5H20)


S. No. Estimation of. Reaction Relation between O.A. and R.A.
1. h l2 + 2Na2S2O3 ----- > 2Nal + Na2S4O;6 l2 = 2I = 2Na2S2O3
or l2 + 2S2O|- --- > 21" + S4Oi- Eq. wt. of Na2S2O3 = M/1
2. CuSO4 2CuSO4 + 4KI ----- > Cu2l2 + 2K2SO4 + '2 2C1JSO4 = l2 = 2I = 2Na2S2O3
Eq. wt. of C11SO4 = M/1
or Cu2+ + 41" Cu2l2 +l2
white ppt-
3. CaOCI2 CaOCI2 + H2O----- > Ca(OH)2 + Cl2 CaOCI2 = Cl2 s l2 = 2ls 2Na2SA
Cl2 + 2KI ----- > 2KCI + l2 Eq. wt. ofCaOCI2 =M/2
Cl2 + 21" ----- > 2CI" + l2
4 MnO2 MnO2 + 4HCI(conc.) —MnCI2 + Cl2 + 2H2O MnO2 b Cl2 b l2 = 2I = 2Na2S2O3
Cl2 + 2KI ----- > 2KCI + l2 Eq. wt. of MnO2 =MI2
or MnO2 + 4H* + 2CI" ----- > Mn2+ + 2H2O + Cl2
Cl2 + 21" ----- > l2 + 2CI"
5. IO3 IO3 + 5I" + 6H+ 3I2 + 3H2O IO3 = 3I2 s 6I b 6Na2S2O3
Eq. wt. of IO3 = M/6
6. H2O2 H2O2 + 2I- + 2H+ ----- > l2 + 2H2O H2O2 = l2 = 2I = 2Na2S2O3
Eq. wt. of H2O2 = M/2
7. Cl2 Cl2 + 2I- 2CI- + l2 Cl2 = l2 = 2I = 2Na2S2O3
Eq. wt. of Cl2 = M/2
8. O3 O3 + 6f + 6H+ ----- > 3I2 + 3H2O O3 = 3I2 = 6Ib 6Na2S2O3
Eq. wt. ofO3 = M/6
9. cicr CIO" + 21" + 2H+ ----- > H2O + Cl" + l2 CIO’s l2 = 2ls 2Na2S2O3
Eq. wt. of CIO- =M/2
10 Cr2O2 Cr2O?" +14H+ + 6I" 3I2 + 2Cr3+ + 7H2O Cr2O2" = 3I2 b 6I
Eq. wt. of Cr2O2- = M/6
11. MnO; 2MnO; + 101" + 16H* ----- > 2MnO; + 5l2 + 8H2O 2MnO4 = 5I2 b 101
Eq. wt. of MnO^ =MI5
12. BrO3 BrO3 + 6I_ + 6H+ ----- > Br" + 3I2 + 3H2O BrO3 b 3I2 = 61
Eq. wt. of BrO3 =M/6
13. As(V) H3AsO4 + 21" + 2H+ ----- > H3AsO3 + H2O + l2 H3AsO4 = l2 s 21
Eq. wt. of H3AsO4 = M/2
14. HNO2 2HN02 + 2I" I2 + 2NO + H2O 2HNO2 b l2 b 21
Eq. wt. of HNO2 =M/1
15. HCIO HCIO + 2I" + H* ----- > Cl" + l2 + H2O Eq. wt. of HCIO = M/2

In iodimetric (different from iodometric) titration, Weakly acidic medium is preferred since
solution is taken in burette and reducing agents like • Starch used for the end-point detection tends t
sodium thiosulphate (As2O3) etc. are taken in flask. hydrolyse or decompose in strong acid.
I2 + 2Na2S2O3 ---- > 2NaI + Na2S4O6 • Reducing power of several reducing agents t
increased in neutral solution.
212 + As2O3 + SHjO---- > 2AsOf" + 4F + 10H+
• I" produced in the reaction tends to be oxidised b
These titrations are usually performed in neutral or dissolved O2 in acid solution:
mildly alkaline (pH 8) to weakly acid solutions. If the pH is 41’ + O2 + 4H+ ---- > 2Lj + 21^0
too alkaline, will disproportionate to hypoiodate and The pH for the titration of arsenic (III) with L can N
iodide ions. maintained neutral by adding NaHC03. lodimein
L> + 2OH~ ■> IO" + I" + HjO titrations have been summarised in Table 7.8.
Chapter 7: Volumetric | 311

Table 7.8 lodimetric Titration (Titrating Solution is l2 In KI)


Reaction Relation between O.A. and R.A.
S. No. Estimation
H2S + l2, -----> S + 2I" + 2H* H2Ss I2s2I
1. H2S
(in acidic medium) Eq. wt. ofH2S = M/2

SO3- + l2 + h2o —> so4 + 21“ + 2H* SO|- e l2 = 2I


2. so|-
(in acidic medium) Eq. wt. of SO3- = Ml 2

Sn2+ + l2 —» Sn4+ + 21- Sn2+ = l2 s 2I


3. Sn2+
Eq. wt. of Sn2+ = M/2
(in acidic medium)
H2AsO3 + l2 + H2O-----> HAsO4“ + 2r + 3H* H2AsO3 = l2 = 2I
4. As (III) (at pH 8)
Eq. wt. of H2AsO3 =M/2

N2H4 + 2I2 N2 + 4H+ + 41" N2H4 = 2I2 = 41


5. N2H4
Eq. wt. of N2H4 = M/4

Illustration 2 A solution o/Na2S2O3 is standardised


Illustration 1 A 0.200 g sample containing copper is
analysed iodometrically. Copper (IF) is reduced to copper iodometrically against a 0.1262 g ofpure KBrO3, requiring
45.0 mL Na2S2O3 solution. What is the molarity o/Na2S2O3
(/) by iodide.
solution? (KBrO3 = 167 g mol-1)
2Cu2+ + 4F ---- > 2CuI +1,
W/iai is the per cent copper in the sample if 20.0 mL of Solution The reactions are
0.10MNa2S203 is required for titration of the liberated Lj? BrO3 + 61" + 6H+ -----> Br" + 3L> + SHjO
(Cu = 63.5 g mol"1) 3L> + 6S2O5“ -----> 61" + 3S4O26"
Solution 2Cu2+ + 41“ ■> 2CuI +12 molarity of Na2S2O3 solution = Mx
white ppt. Let,
Ja + 2S2O23" ---- > S40J' + 21" M1 X 45 = 0.045 M,
moles of Na2S2O3 used = 1000 1
By stoichiometric coefficients:
moles ofKBrO3 taken= 2^? = 7.56x 10-4
2Cu2+ = 1% = 2S2O^“ 167
Cu2+ = S2O|"
Based on stoichiometric coefficients:
20 mL of 0.10 M S2O|" = 20 x 0.1 millimol 6 mol ofS2O^“ = 1 mol of BrO3
= 2x 10"3 mol S2O^“ 0.045M
0.045 Mi mol of S2O23" = °-045Mi mol of BrO3
PureCu2+ = 2x 10"3 mol 6
= 2x 10“3x 63.5 g °-045M1 = 7.56 x 10^
„ ' , 2 x 10-3 x 63.5 x 100 6
% of pure Cu in 0.20 g sample- ------------—-----------
Mx = 0.10 M
= 63.5% Molarity ofNa2S203 solution = 0.10 M

Target Practice 2
1. 100 mL of NaHC2O4(A) is neutralised by 100 mL of 1 M 3. We have 100 mL of 1 MKMnO4. In acidic medium it is to be
used. Calculate volume of water to be added so that it is
NaOH. Thus 100 mL ofA will be oxidised by mL of
0.5 N
1M KMnO4. in acidic medium. 4. Calculate normality of the underlined if each reactant is
2. KMnO4 is reduced to MnCl2 in HC1 medium, to MnO2 in IM.
(a) 2CuSO4 + 4KI ---- * Cu^+2K2SO4+I2
KOH medium and to K2MnO4 in H2O medium.
Thus, (a) in 1 M KMnO4 = N in HC1 medium (b) 2Na2S2O3+I2 ---- > 2NaI + Na,S.Ofi.............
L 4 O

(b) 1M KMnO.4 = N in KOH medium (c) H2O2+2KI —-> 2KOH + I2


(c) 1M KMnO.4 = N in H£•90 medium (d) Fe2+ ---- > Fei3+
; + e-..............
312 | Essential Inorganic Chemistry

5. 10 mL of 1M Fe2* is oxidised by each of 1M MnO“ and 1M Calculate volume of NaOH in both cases.
p_
Cr2n2-
O2- in separate *•__ ;______________
*:*__titration in acidicJ;____
medium. (a)
Thus, volume of MnO" required = mL (b)
Volume of Cr,O2" required = mL 7. A100 mL sample of brackish water was made ammoniaca
6. 0.1 M HC1 solution is being titrated by two students using and sulphide it contained was titrated with 16.0 mL &
0.1 M NaOH. 0.025 M AgNO3 solution.
Student A: 20 mL of 0.1 M HC1 + 40 mL H2O 2Ag* + S2- ---- > Ag2S(s)
Student B : 20 mL of 0.1 M HC1 + 80 mL of H2O. Calculate concentration of H2S in water in ppm

Answers
1. 100 mL of NaHC2O4 is neutralised by 100 mL of 1 M NaOH. 6. In both case same volume of NaOH is used.
This is an acid-base titration. NaHC2O4 has one ionisable H * 20 x 0.1 N
ion, hence, (A) 2VHC1 (resultant) =
60 30
100 x NJ = 100 x 1 M NaOH 20 x 0.1 N_
(B) Nhc1 (resuitant) =
Mi = Nj (NaHC2O4) = 1 100 50
NaHC2O4 is oxidised to CO2 by KMnO4 in acidic medium. Thus, resultant volume in A = 60 mL
C2O2- ---- > 2CO2+2e” N
60 mL of — b V mL of 0.1 N NaOH
Thus, 1 M NaHC2O4 = 1 N as acid 30
= 2 N as reducing agent V = 20 mL
1M KMnO4 = 5 N as oxidising agent
Resultant volume of B = 100 mL
100 x 2N (A) = V x 5N KMnO4
100 mL of— = V mL of 0.1 N NaOH
V = 40 mL 50
2. (a) 5 N (b) 3 N (c) 1N V = 20 mL
3. 1M KMnO4 = 5 N in acidic medium
7. 16.0 mL of 0.025 M AgNO3 = 100 mL ofH20 containingHjS
100 x 5 N = V x 0.5 N
16 x 0.025
V = 1000 mL Moles of AgNO3 =
1000
Thus, water to be added = 900 mL
= 4x 10 mol
4. (a) 2 CuSO4 = I2 = 21,
CuSO4 = I 1M=1N Moles ofH2S = - mol of AgNOs
2
(b) 2Na2S2O3 = I2 =21
= 2 x 10~* mol H2S
Na2S2O3 si 1 M = 1N
= 2 x 10"4 x 34 g in 100 mL Ha0
(c) H2O2 = I2 = 21
H2O2 68 x 10-4 xlOx 10 6 . in6 T / iun
1= 1M = 2N --------------------g in l0b mL (ppmlHjjO
2 100
(d) Fe2* s l e~ = 68 ppm
1M=1N Alternatively:
5. Fe■2* ---- > Fe3* + e~
1 H2S b 2Ag*
MnO4 + 8H* + 5e” ---- > Mn2* + 4H2O
Thus, Normality of H2S = 2 x Molarity
10 mL of I MFe2* = 10 mL of l N Fe2*
V mL of I M MnO4 s V mL of 5 N MnO4 in acidic medium 100 x 2 x M (H2S) = 16.0 x 0.025
V x 5 N = 10 x I N M(H2S) = 2 x IO'3 mol L’1
V = 2mL = 68 x 10"3 gL’1
Cr2O2-+ 14H* + 6e” ---- > 2Cr3* + 7H2O
= 68 x 10-6 g mL-1
V mL of l M Cr2O2- s V mL of 6 N Cr2O2~
V x 6 N = 10 x I N = 68 x 10"6 x 106 g ppm

V = — = 1.67 mL = 68 ppm
6
Chapter 7 . Volumetric 5 313

Practice Exercise 2
1. Household ammonia, used as a window cleaner and for 9. What is molarity of a MnO~ solution if 32.00 mL of the
other cleaning purposes, is NH3 (aq). 31.08 mL of 0.9928 M solution is required to titrate 40.0G mL of 0.400 N Fe2*?
HC1 (aq) is required to neutralise the NH3 present in a MnO“+5Fe2+ +8H* ---- > Mn2* +5Fe3+ +4H2O
5.00 mL sample,
10. In a reaction, Cr2O2- is reduced to Cr3+. What is
NH„o + H+ ---- > NHt4
concentration of 0.1 MK2Cr2O7 in equivalent per litre?
What is the molarity of NH3 in the sample?
Cr2O2’ + 6H+ + 6e~ ---- > 2Cr3+ + 7H2O
2. What volume of a 0.232 N HC1 solution contains
(a) 3.17 meq. of solute (Here we have given half reaction; if not given be careful in
(b) 6.5 eq. of solute? balancing affected atoms and in taking oxidation number of
two Cr atoms)
3. Calculate the normality of each of the following solution:
(a) 7.88 g ofHNO3 per litre solution 11. What mass of phosphoric acid (H3PO4) is required to make
(b) 26.5 g of Na2CO3 per litre solution (if acidified to form 550 mL of 0.400 N solution?
CO2). (a) assuming complete neutralisation of the acid
4. How many (a) meq. (b) equivalent, (c) mole and H3PO4 + 3OH“ ---- > 3H2O + PO3’
(d) gram are present in 60 mL of 4.0 N H2SO4? (b) assuming reduction toHPO2-.
5. What is the purity of cone. H2SO4 (d = 1.8 g / mL) if 5.00 mL HjjPO, +2e' ---- » HPO2’ +H2O
is neutralised by 84.6 mL of 2.00 N NaOH? 12. The concentration ofFe2+ ion in aqueous solution can be
6. Urea sample is 60% pure. Calculate volume of 1N HC1 determined by redox titration with bromate ion, BrO',
required to absorb NH3 from 1.0 g of urea sample on according to net ionic equation:
6Fe2+ + BrO3 + 6H+ ---- > 6Fe3+ + Br’ + 3H2O
decomposition with NaOH.
7. Mn2t (aq) can be determined by titration with KMnO4 (aq) What is the molar concentration of Fe2+ if 31.50 mL of
0.10 M KBrO3 is required for complete reaction with
3Mn2+ + 2MnO“ + 2H2O---- > 5MnO2 + 4H+
10.0 mL of the Fe2+ solution?
A 25.00 mL sample of Mn2+ (aq) requires 35 mL of 0.06 M 13. A 1.100 g sample of copper ore is dissolved and the Cu2+ (aq)
KMnO4(aq) for its titration. What is the molarity of the
is treated with excess KI. The liberated 1, requires 12.12 mL
Mn2+(aq)?
of 0.10 M Na9S903 solution for titration. What is % copper
8. A particular acid-rain water has SO?-. If a 25.00 mL sample by mass in the ore?
of this water requires 34.08 mL of 0.01964 M KMnO4 for its 14. What mass of Na,,S903 • 5H2O is needed to make 506 mL of
titration, what is the molarity ofSO3 in acid-rain? 0.200 N solution for the reaction?
2MnO“ + 5SO," + 6H+ ---- > 5SO2’ + 2Mn2+ + 3H„O 2S„O 2- +1,4 ---- > S.O2‘ + 21-
4 0 4 0

Answers
1. M(NH3) x 5 = 0.9928 x 31.08 HC1 4. (a) meq = normality x volume (mL)
M(NH3) = 6.17 M = 4.0 x 60
2. (a) N x V (mL) = milliequiv. = 240
0.232 x V = 3.17 (b) eq = 0.240
y- 3-17 = 13.66 mL , . , 0.240 .
(c) mol =------- = 0.120 mol
0.232 2
N x V (mL) (d) gram = 0.120 x 98 = 11.76 g
(b) —---------- = equiv.
1000 5. 5.00 mL ofH2SO4 = 84.6 mL of 2.00 N NaOH
0.232 x V
” 1000
= 6.5 N (H2SO4) = 33.84 N
V = 2.8017 x IO4 mL = 28.017 L M (H2SO4) = 16.92 M
3. (a) 7.88 gL'1 HNO3 = — mol L-1 1000 mLH2SO4 has = 16.92 mol H2SO4
" 63
= 0.125 M since, mass/volume = density

= 0.125 N (being monobasic acid) 1000 x 1.8 gH,SO4 has = 16.92 x 98 g H2SO4
(b) Na2CO3+2HC1---- » 2NaCl + CO2 +H,0 16.92 x 98 x 100
100 gH2SO4 has =
Na2CO3 = 2H+ 1000 x 1.8
Normality of Na2CO3 = 2 x molarity = 92.12%
26.5 Na2CO3 per litre = mol L-1 = 0.25 M 6. NH2CONH2 + 2NaOH ---- > 2NH3+CO2+H2O
2NH3+2HC1 ---- > 2NH4C1
Normality = 0.50 N 1 mol NH2CONH2 s 2 mol HC1
314 ] Essential Inorganic Chemistry

1.0 g impure NH2CONH2 =0.6 g pure NH2CONH2 Thus, moles ofH3PO4 in 550 mL
= 0.01 mol _ 0.400 x 550
mol
HC1 required by 0.01 mol urea = 0.02 mol 2 x 1000
= 20 millimol = 0.11 mol
1 x V = 20 = 10.78 g
V = 20 mL 12. Fe2* ---- > Fe3* + e" Normality - molarity
7. Mn2* ---- > MnO2 Change in O.N. Normality +2 +3
+2 +4 2-unit 2 x molarity BrO3 Br" Normality =6 x molarity
+5 -1
MnO4 ---- > MnO2
+7 +4 3-unit 3 x molarity Fe2* BrO3
25 mL of2Mj Mn2* = 35 x 0.06 x 3 N MnO; 10.00 x Nj s 31.50 x 0.10 x 6 N BrO3
Afj = 0.126 M Mn2* N1(Fe2 + ) = 1.89 N
SO2" sol Change in O.N. Normality = 1.89 M Fe 2+
8.
13. 2Cu 2++ 41" -- > Cu2I2+L 2
+4 +6 2-unit 2 x molarity
2S2O|” + I2 ---- > S4O|" + 21"
MnO4 ---- > Mn 2 +
2Cu2+sI2 =21
+7 +2 5-unit 5 x molarity
1 Cu2+si = S2O|"
25 x 2Mj (SO1") in H,0 = 34.08 x 0.01964 x 5 MnO4 1 molCu2+ = 1 equivalent Cu2+
Afj = 0.0669 M SO3"
Also, 2S2O3 = I2 = 21
9. MnO4 Fe2*
1S2O2" =1
32 x Nj = 40 x 0.4 N
1 mol S2O|" = 1 equivalent S2O2" = Cu2*
Nj (MnO4) = 0.5N
n Moles ofCu2+ s moles of S2O|"
M, (MnO4) = — = 0.1 M
5 _ 12.12x0.1
10. 0.1 MK2Cr2O7 s 0.6 N K2Cr2O7 1000
11. (a) All the H * ions are neutralised. = 1.212 x 10"3 mol
Thus, 0.400 N = ^°° M H3PO4 = 1.212 xlO"3 x 63.5 g
3 3 4
0.400 x 550 = 0.076962 g
Thus, moles ofH3PO4 in 550 mL = It is present in 1.1 g impure Cu ore
3 x 1000
% ofCu2* in sample =
in sample = 0 076962 xlOO
MassofH3PO4 =0.073 mol 1.100
= 7.1867 g = 6.996%
(b) It is reduced to HPO3 14. As in Q. 13
1 H3PO4 = 2e" 500 mL of 0.200 N Na2S2O3 -5H2O
, - _ H3PO 4
2 s 500 x 0.200 millimol
0.400 = 0.1 mol
0.400 N = MH3PO4
2 = 24.8 g Na2S2O3 • 5H2O
•.-z»*x._:.x__r
r
Total Practice Set
(Read, Plan & Solve'.)

_135x 0.224
Problem 1. What are the molar concentrations of the mol MgSO4
1000
solutes listed below when dissolved in water?
(a) 3.28 mol C^OK in 7.16 L of solution = 0.0302 mol MgSO4
(b) 3.67 x 10-3 mol CH3OH in 50.00 mL of solution Same amount is present in 105 mL solution after
(c) 22.3 g (CH3)2 CO in 125 mL solution 3 evaporation of water.
(d) 14.5 mL of pure glycerol; C3H8O3 (d = 1.26 g/ cm ) Thus, 105 mL solution has= 0.0302 mol MgSO4
0 0302
in 425 mL solution 1000 mL solution has = —------x 1000 = 0.288 M
105
Plan Molarity = mol L"1 = __ g_ L' Alternate method MXVX = M2 V2
g mol
g x 1000 initial final
g mol'1 x mL 0.224x 135 =M2 x 105
M2 = 0.228 M
mass = volume x density
Solution (a) 3.28 mol C^HgOH is 7.16 L of solution Problem 3. Find the volume of a 0.2 M solution of
MnO4 which will react with 50.0 mL of 0.1 M solution of
MCCjHgOH) = y|| mol L"1 = 0.458 M in acidic medium.

(b) 3.67 x 10"3 mol CH3OH in 50.00 mL (= 0.05 L) Plan Normality of MnO4 = 5 x molarity
solution and normality ofC^f = 2 x molarity
3.67 x IO"3 use
a M(CH30H) = mol L-1 = 0.0734 M
0.05 Milliequivalents of reacting species are equal.
(c) 22.3 g(CH3)2CO in 125 mL solution Solution
= mol (CH3)2CO in 0.125 L solution 2MnO4 +5C2C^~ + 16H+ — -> 10CO2 + 2Mn2+ + 8H2O
+7 +6 r. , +8 +2
22.3 1 Oxidation
Af[(CH3)2CO] =-^_ mol L"1 = 3.076 M
32 0.125
Reductionzf I
Note: We have taken oxidation number of two C atoms i n Co02"
(d) 14.5 mL of pure glycerol = 14.5 x 1.26 g (true definition of O.N.) hence, we also take O.N. of two C atoms
= 14.5x 1.26 mol Q 425 L solution inCO2.
92 Thus, MMnO4) = change in O.N. x M
M (glycerol) = 0.467 M N = 5xO.2=lN
MCrjC^-) =2 x M = 2 x 0.1= 0.2 N
Problem 2. Water is evaporated from 135 mL of using ^(MnOp = N2V2(C^-)
1224 M MgSO4 solution until the solution volume 1.0 x Vi = 0.2 x 50
Jecomes 105 mL. What is the molarity of MgSO4 in the Vi = 10 mL 0.2 M MnO4
solution that results?
Note: We can’t use MXVX=M2V2 when change in O.N. or
Plan When solution is evaporated, volatile solvent (like acidity and basicity are different.
-IjO) evaporates, but non-volatile solute remains
-inaffected. Due to change in volume of solution, molarity Alternate method Let volume of MnO4 = V mL
also changes. V mL of 0.2 M MnO^ = 0.2 V millimol of MnO4
Solution 135 mL of 0.224 M MgSO4 solution 50 mL of 0.1 M CjO^- = 50 x 0.1 = 5 millimol of CjOJ-
Using reaction: 5 mol ofG>O^- are oxidised by
= 135 x 0.224 millimol ofMgS04
= 2 mol of MnO7 '4
316 | Essential Inorganic Chemistry

5 millimol ofC2O^ are oxidised by = 2 millimol ofMnO^ 100-x 100-x


g-equivalent of BaCO3 =
0.2 V = 2 197/2 98.5
V = 10 mL x 100-x
— +-------- = 2
42 98.5
Problem 4. Two acids, A and B are titrated This gives x = 72.11
separately. 25 mL of 1 N Na2CO3 solution requires 10 mL
MgCO3 =72.11%
of A and 40 mL of B for complete neutralisation. What
volume ofA and B would you mix to produce 1L of 1N acid BaCO3= 27.89%
solution?
Problem 6. Determine the equivalent weight in each
Plan Normality of each acid is known by titration. Thus, of the following
from total normality and the relation (a) the reduction half-reaction for the
NTotal
disproportionation of B^ in base,
y y (b) the oxidation half-reaction which yields bromate
ion,
volume of each acid is determined. (c) the overall reaction.
Solution 10 mL of NA acid A = 25 mL of 1 N Na2CO3 Plan Equivalent weight =------- formula weight—
change in oxidation number
ATa=2.5
40 mL of Nb acid B = 25 mL of 1 N Na2CO3 and in a disproportionation reaction =E0 + ER
AL, =0.625 where, Eo is the equivalent weight in oxidation
D
half-reaction and ER is the equivalent weight in reduction
Let volume of acid A = x L half-reaction.
and that of acid B = (1 - x) L
Resultant normality of mixture = 1N Solution (a) 2e~ + Br2 > 2Br"
. . S NV NaVa + NBBVrBB „ , mol. wt. of Bio 160
using relation N= _ =— „ Eq. wt. =---------------- - =----
yAA+yB 2 2
1_2.5x +0.625(1-x)
= 80 g equiv
1
(b) Br2 + 12OH" ---- > 2BrO3 + 6^0 + 10e"
x = 0.2L
Volume of acid A = 0.2 L mol. wt. of Bi, 160 * . -1
Eq. wt. =------- —------- “ *6 S equiv
Volume of acid B = 0.8 L
(c) 6OH" + 3Br2 ---- > 5Br" + BrO3 + 3^0
Problem 5. A sample supposed to be pure CaCO3 is Since, number of electrons involved in each
used to standardise a solution of HC1. The substance half-reaction should be identical, hence 10 electrons are
really was a mixture of MgCO3 and BaCO3, but the involved in oxidation then 10 electrons should also be
standardisation of HC1 was accurate. Find the percentage involved in reduction thus,
of BaCO3 and MgCO3 in the mixture. (Ca = 40, Mg = 24, 5 x (a) + (5)
Ba = 137)
12OH" + 6Brj ---- > lOBr" + 2BrO3 + 61^0
Plan CaCO3 and mixture of MgCO3 and BaCO3 give Hence, Eq. wt. (Bi^) =6x ^X 160 = 96.0 g equiv-1.
same result in the standardisation o/HCl hence, 10
Equivalent ofC&CO3 = equivalent ofMg££)z and BaCO3 or net eq. wt. = eq. wt. of step (a) + eq. wt. of step (b)= 96
Thus, by assuming a quantity ofCaCO3 {which is also
equal to mixture o/MgCO3 and BaCO3), % of each can be Problem 7. 100.0 mL sample of hard water is passed
determined. through a column of the ion exchange resin The water
coming off the column requires 15.17 mL of 0.0265 M
Solution Equivalent ofCaCO3 = equivalent of mixture NaOH for its titration. What is the hardness of the water
CaCO3 = (MgCO3 + BaCO3) expressed as ppm ofCa2+?
Let CaCO3 be 100 g , g-equivalent = — = 2 Plan Ca2+ in hard water replaces H+ of ion-exchangt
column
Let MgCO3 = x g Ca2+ = 2H+ = 2OH"
BaCO3 =(100-x) g mol of C*2+ =~xmol of OH"
x
g-equivalent of MgCO3 = —-
Thus, ppm concentration ofCa2+ can be determined.
Chapter 7: Volumetric | 317

Solution Ca2+ + ---- > 2H+ + CaZ? x i-^ = 0.02136


---- 4-
49 40
Every Ca2+ exchanged gives twoH+ ion.
2H+ + 2OH" ---- > 2H2O x = 0.7927 g H2SO4 in 1 g oleum.
Thus, 2OH" = 2H+ = 1 Ca2+ percentage of H2SO4 = 79.27%
moles of Ca2+ =- mol of OH" and SO3 = 20.73%
2
lx 0.0265x 15.17 Problem 9. A sample of water has a hardness
mol
2x 1000 expressed as 77.5 ppm Ca2-r. This sample is passed
= 2.01x 10"4 mol through an ion exchange column and the Ca2+ is replaced
100 mL of hard water has by H+. What is the pH of the water after it has been so
treated?
Ca2+ = 2.01 x 10"4 mol Ca2+
= 2.01x 10'4x40gCa2+ Plan Ca2+ in hard water replaces H+ of cationic ion
106 mL hard water has Ca2+ exchange column.
2.01 x 10“4x40x 106 Ca2+ = 2H+
100 g From ppm concentration ofCe?*, molar concentration of
H+ can be determined. Hence, pH=-logH+ can be
« 80.4 g
determined.
Hardness of water = 80.4 ppm Ca2+
Alternate method Solution Ca2+ + AH, -> /?Ca + 2H+
(ion exchange
Equivalent ofCa2+ = equivalent of H+ or OH" column)
= 15.17x 0.0265 = 4 02x 1q-4 every Ca2+ replaced gives twoH+ ions.
= 4.02x 10'
1000 106 mL of E^O has = 77.5 g Ca2+ = ~ mol Ca2+
Ca2+in 100 mL water = 4.02 x lO^4 x 20 g
. 4.02x 10-4 x 20x 106 q (77.5^ . tj+
La in 10 mL water =------------ —----------- = 2 ----- mol H
100 40 )
= 80.4 ppm 2x 77.5
103 mL of H2O has H+ = x 10"3 mol H
40
Problem 8. 1 g of fuming H2SO4 (oleum : it is a
= 3.875 x 10"3 M
mixture of cone. H2SO4 saturated with SO3 and having
formula HjSgO?) diluted with E^O. This solution is pH = - log [H3O+]
completely neutralised by 26.7 mL of 0.8 N NaOH. Find = 2.41
the percentage of free SO3 in the oleum.
Plan SO3 and H2SO4 in oleum both act as a dibasic acid Problem 10. A sample of water has its hardness due
■Mid are neutralised by NaOH. only to CaSO4. When this water is passed through an ion
A/so, equivalents of SO3 and E^SO,* = equivalents of exchange resin, SO^" ions are replaced by OH". A
NaOH 25.00 mL sample of water so treated requires 21.58 mL of
Thus, percentage of SO3 in the oleum can be determined. 1.00 x 10"3 M H2SO4 for its titration. What is the hardness
of the water; expressed in ppm of CaSO4? Assume the
Solution H^Ot+HjO —> 2H2SO4 density of the water is 1.00 gmL"1.
oleum
S03 part of H>S2O7 is converted into H2SO4, hence, SO3 Plan SO^" replaces OH" ions (in ion-exchange resin) and
acts also as a dibasic acid. OH' in turn is determined by titration with H9SO4.
M SO^" = 2OH" = H2SO4
Equivalent weight (SO3) = —- = 40
z Thus, hardness in ppm ofCaS>OA can be determined.
LetH2SO4 in the fuming EL2SO4 = x g
Solution Let an ion exchange resin be 7?(OH)2
and SO3=(l-x)g
X
Equivalent of HjSC^ = — 7?(OH)2 + SOj" ---- > 7?SO4 + 2(OH")
every SO^" exchanged gives two (OH“)
and that of so33 = —
40
OH" is neutralised by H9S04
26.7 x 0.8 2OH" + H2SO4 ---- > 2H>O + SO1"
Equivalent of NaOH used = = 0.02136
1000 Thus, 1 CaSO4 (in hard water) = 2OH' = 1 H2SO4
318 | Essential Inorganic Chemistry

21.58x IO"3 x volume in mL = sub$tanM


Moles of CaSO4 = moles of HgSO^ = Plan
1000 1000
= 2.158x 10'5 mol Based on reaction 2 mol AgNO3 = 1 mol Ag2CrO4
This amount is in 25 mL hard water hence, number of moles of Ag2CrO4 obtained from given
mol of AgNO3 can be calculated.
25 mL of hard water has hardness
= 2.158x 10"5 mol CaSO4 Solution From the reaction:
= 2.158x 10-5 x 136gCaSO4 2 mol of AgNO3 gives = 1 mol Ag2CrO4
106 mL (per million) of hard water has hardness hence, —— mol of AgNO3 gives
2.158 x 10-5 x 136 x 106 _25x 0.1122
gCaSO4
25 mol Ag2CrO4
1000 x 2
= 117.4 ppm CaSO4 = 0.0014 mol Ag2CrO4
Problem 11. What mass of Na2SO3 must have been = 0.0014 x 332 g Ag2CrO4
present in a sample that required 26.50 mL of 0.0510 M = 0.4656 g Ag2CrO4
KMnO4 for its oxidation to Na2SO4 in acidic solution?
Problem 13. A sample of pure KHCgC^ ■ • 2H2C
Plan Taking the fact that equivalent of requires 46.2 mL of 0.1 M NaOH for titration. How many
SO3" = equivalent of MnO4 amount of Na2SO3 can be millilitres of 0.1 M KMnO4 will the sample react?
calculated, or on comparison of moles o/MnO^ and SO^-,
amount o/’Na2SO3 can be calculated. Plan NaOH neutralises three ionisable H+ and each mole
of the sample has two moles of ions oxidised b)
Solution Redox reaction is MnO^.Also 2
SSOV +2MnO; + 6H+ - ■> 2Mn 2+ + 5S0J- + 31^0 moles o/'C2O^“ = — mol o/H+
3
| +5e~ __ t
-2e“ From the number of moles ofOPP, number ofmoles o/H
and hence that of CgO^" are known. Thus, volume o/'MnQj
M required to oxidise C^- can be calculated.
Equivalent weight of MnO4 = —
5
Solution H+ + OH" ---- > H2O
Equivalent weight of SO^~ = ~
502^" + 2MnO4 + 16H+ ---- > 2Mn2+ + 10CO2 +8H2.0
26.50 mL of 0.0510 M MnO; Number of moles ofOH“
= 26.50 mL of 0.2550 N MnO; 46.2x 0.1
= number of moles of H+ ion =
= 26.50 mL of 0.2550 N SO^ 1000
„ • 1 4. rv or-A 0.2550x 26.50
Equivalent of Na9S0q =------------------ = 4.62 x IO-3 mol
2 3 1000 Number of moles of C^C^" ion
= 6.7575 x 10"3 =-x 4.62 x 10"3 = 3.08 x 10"k-33 mol
3
.-. Weight of Na2SO3 = 6.75 x IO-3 x 63 g
Since,, 5 mol of C^C^- = 2 mol of MnO4
= 0.426 g 3.08x 10’3 mol of QjOj" =|x 3.08x 10"3
Alternate method
26.5x 0.051 = 1.232 x 10-3 molMnOj
Moles of MnO4 =
1000 If V mL of 0.1 M MnO^ are required, then
= 1.35 x 10“3 mol --X 0-1 = 1.232 x 10~3 .
Moles ofNa2SO3 =-x 1.35x IO-3 mol 1000
2
V = 12.32 mL
= 0.426 g
Alternate method
Sample has three H+ ions
Problem 12. 25 mL of 0.1122 M AgNO3 is added to an
.-. 46.2 mL of 0.1 M NaOH= 3H+ ions
excess of KjCrO^m?). What mass of Ag2CrO4 will
precipitate from the solution? (Ag= 108,Cr= 52,N= 14, Equivalent of salt as acid =—= 1.54 x KT3
0=16) 3x1000
2AgNO3 + K2CrO4 ---- > Ag2CrO4 + 2KNO3 Equivalent of C^C^" in two oxalates = 6.16x 10-3
Chapter 7: Volumetric | 319

Vx 0.1x5 Problem 15. 25 mL of a solution containing 6.1 g L 1


Equivalent of 0.1 M MnO4 =
1000
of an oxalate of formula KxHy(C2O4)2 ■ nH^O required
Vx O.lx 5 18 mL of 0.1 N NaOH and 24 mL of 0.1 N KMnO4 in two
= 6.16x 10"3
1000 separate titrations. What is the compound?
V = 12.32 mL Plan KxH/C204),nH20 has y ionisable H+ ions
Problem 14. A 0.8040 g sample of an iron ore is determined by NaOH and z oxidisable C.2O^~ ions
dissolved in acid. The iron is then reduced to Fe2+ and determined by MnO4, K+ and H+ both are monovalent
titrated with 47.22 mL of 0.02242 M KMnO4 solution. cations and CjO^" is divalent anion hence, x + y = 2 z
Calculate the results of this analysis in terms of Thus, from three linear equations, value of x, y and z
(a) % Fe (56 g mol"1) (b) % Fe3O4 (232 g mol"1) can be determined. Value of n is determined when
molecular weight of the compound is also known.
Plan (a) Fe2+ is oxidised to Fe3' by MnO4 in acidic
medium. Solution Concentration = 6.1 g/L
MnO4 + 5Fe2++ 8H+ ---- > Mn2++5Fe3++ 4^0 K+ and H+ are cations and oxalate C^O^" is an anion
From the stoichiometric ratio, on comparison, moles of hence, x + y = 2z ...(i)
Fe hence, its amont (in gram) is calculated. Thus % of Ye
is also determined. (a) NaOH will neutralize H+ part
(b) . 5Fe2+ = lMnO4 25 mL of (acid oxalate) = 18 mL of 0.1 N NaOH
18 x 0 1
5Fe3O4 s 15Fe2+=3MnO4 Ni (acid oxalate) = ----- = 0.072 N
Knowing number of moles ofFeJX which is equivalent
to 3 mol MnO4, its % can be determined. Eq. wt. of an acid salt = = 84.72 g equiv"1
0.072
Solution Moles of MnOJ = molarity x volume (in mL) (b) KMnO4 will oxidise C^C^"
4 1000
25mLN1'(C20^-)H24mLof0.01NKMn04
------ '4=
1 mol MnOj 5 mol of Fe2+
-----
N1,(C2(^") = 0.096 N
47.22x0,02242
mol MnO4 6.1 _.
1000 Eq. wt. of oxalate (reducing agent) = ------ = 63.54
0.096
5 x 47.22 x 0.02242 . _ 2+ mol. wt.
=----------------------- mol Fe Thus, = eq wt. (84.72) when it is an acid salt
1000 y
= 0.00529 mol Fe2+ mol. wt.
= eq. wt. (63.54)
2z
= 0.00529x56
when it is a reducing agent
= 0.2964 gFe2+ 2z 84,72
Thus, % of pure Fe(Fe2+) in 0.8040 g sample y 63.54
= ^x!00 y = 1.5z ...(ii)
0.8040 Using Eqs. (i) and (ii), x + 1.5z = 2z
= 36.87% x = 0.5z
(b) 3 mol MnO4 = 5 mol Fe3O4 x, y and z are in the ratio of 1:3 :2 (to balance charge)
47.22 x 0.02242 or x = 1, y = 3, z = 2
mol MnO4
1000 Mol. wt. = 84.72 x 3 = 254.16
5 47.22x0.02242 39x + y + 88z + 18n = 254.16
=-x mol
3 1000 n=2
= 0.001764 mol Fe3O4 hence, salt is KH3(C2O4)2 • 2H2O.
= 0.001764 x 232 gFe3O4 Problem 16. 30 mL of an acidified solution of 1.5 N
= 0.4094 g MnO4 ions, 15 mL of 0.5 N oxalic acid and 15 mL of 0.4 N
ferrous salt solution are added together. Find the
a v n 0.4094 1AA
% as Fe3O4 =-------- x 100 molarities of MnO4 and Fe3+ ions in the final solution.
3 4 0.8040
Plan MnO4 is reduced to Mn2+, Fe2+ is oxidised to Fe3+
= 50.91% and CjQt” is oxidised to CO2
320 | Essential Inorganic Chemistry

, .. normality hence, 10 g = mol of V2O5 reduces


molarity --------------------- —-----------
change in oxidation number 182
n 10 1 pt
= 2 x---- mol of L
M(MnO;) = — (MnO;) 182
5 = 0.11 mol of 12
M(Fe2*) = 2V(Fe2*)
Problem 18. 1.44 g pure FeCaQ, was dissolved
dil. HC1 and solution diluted to 100 mL. Calculate voluc
On comparison of milliequivalents of each reactant, of 0.01 M KMnO4 required to oxidise FeGjO^ soluti<
final concentration o/Fe3* and MnO; can be calculated. completely.
Solution Acidified MnO; oxidises Fe2+ into Fe3+ ;and Plan MnO; oxidises Fe2+ as well as C^O^-. Thus, from, ti
number of moles ofF^ and C^O^- number of moles hem
C2O^“ intoCO2.
volume o/MnO; can be determined.
MnO; +8H+ +5Fe2+ ---- > Mn2 + 5Fe3++ 41^0
2MnO; +502^“ +16H+ ---- > 2Mn2+ +10CO2 +8H2O Solution Fe2+ is oxidised to Fe3+ andQjOj' is oxidi*
to CO2 by MnO; in acidic medium.
Milliequivalents of MnO; = 30 x 1.5 = 45
Milliequivalents of Fe2+ = 15 x 0.4 = 6.0 5Fe2+ + MnO; + 8H+ ----
- > 5Fe3+ + Mn2+ + 4^0
Milliequivalents of oxalic acid = 15 x 0.5 = 7.5 5G2O^ + 2MnO; + 16H+ - -> 10CO2 + 2Mn2+ + 8B,(
Milliequivalents of MnO; used by 6.0 milliequivalents SFeQA + 3MnO; + 24H+ —» 5Fe3+ + 10C02
of + 3Mn2+ + I2H2O
Fe2+=6.0
Thus, 5 mol of FeG^ are oxidised by= 3 mol of MnO
milliequivalents of MnO; used by 6.5 milliequivalents of
oxalic acid = 7.5 1.44 g (= 0.01 mol) FeC^ is oxidised by
thus, total milliequivalents of MnO; used = 13.5 Q
= -x 0.01= 0.006 mol MnO;
milliequivalents of MnO; left = 45 - 13.5 5
= 31.5 in 60 mL If V mL of 0.01 MKMnO4 are required then
™ n-T 31.5 x IO"3 x 1000 N 31.5 XT
thus, [MnO4 ] =------------------------ =----- N = 0.0Q6 ,
4 60 60 1000
= — x^M= 0.105 M V = 600mL
60 5
Milliequivalents ofFe2+ consumed , (moles = mV- = QQ1X.K.=0.006)
1000 1000
= milliequivalents of Fe.3+
3+ formed
= 6.0 in 60 mL solution Problem 19. For titrating a certain volume of
reducing substance by 1 M KMnO4, it is found that 20 u.
thus, [Fe3+ ] = — x 10"3 x 1000 N = 0.1 N was used in acidic medium, 33.4 mL was used in alb
60
medium and 100 mL was used in neutral medium.
= 0.1 M MnO; is reduced to Mn2+ in acidic medium, to what stat
.. MnO; is reduced to in alkaline and neutral medium?
Problem 17. If 10 g of V2O5 is dissolved in acid and is •
reduced to V2+ by zinc metal, how many moles of could Plan From the volume ofT^nO^ used in acidic medium
be reduced by the resulting solution if it is further oxidised normality o/MnO; in basic medium as well as in neutr.
to VO2+ ions? (V = 51,0 = 16,1 = 127) medium can be determined. Since
normality = molarity x change in O. N.
Plan From the stoichiometric coefficients, number of
moles of L reduced by given amount of V2O5 can be hence, O.N. ofMn in final state is known.
determined. Solution In acidic medium MnO; is reduced to Mn‘
Solution (by gain of5e")
V2O5 +10H+ +3Zn - 3Zn2++2V2++5H2O Mn O? + 8H+ + 5e“ * Mn2++4H20
+7 +2
H^O + +V2* - * 2r+VO2*+2H+]x2
thus, 1M MnO; = 5 N MnO;
V2O5 +6H+ +3Zn + 2^ —> 3Zn2+ + 41“ + 31^0 (normality = molarity x change in oxidation number)
+ 2VO2* Let normality in alkaline medium be N' and in neutr;
Thus, 1 mol of V2O5 reduces = 2 mol of Lj medium N'’
Chapter 7: Volumetric | 321

20 mL of 5 N MnO4- = 33.4 mL N' MnO4" 0.004 0.175x0.446 IK<


--------- Mn = --------------- mol MnO4
5 2 1000
= 100 mL of N"MnO44
0.175x0.446x5
N' (alkaline) = 3 N M2 =
~iooo7aooZ-
N' ’ (neutral) = 1N
Af2(KHCA HC2O4) = 0,0976 M
Thus, change in oxidation number in alkaline medium = 3
Hence, MnO4 is reduced to MnO2 (+4) Problem 21. A mixture containingNa2CO3, NaOH and
Thus, MnO4 + 2H2O + 3e~ ---- > MnO2+4OH" inert matter weighs 0.7500 g. When the aqueous solution is
+7 +4 titrated with 0.5000 N HC1, the colour of the
MnO4 is reduced to MnO^ (+6) in neutral medium phenolphthalein disappears when 21.00 mL of the acid has
when change in oxidation number is one unit: been added. Methyl orange is then added and 7.00 mL more
Thus, MnO4 + e“ ---- > MnO?~ of the acid is requried to give a red colour to the solution.
+7 +6 Find the percentage composition of the sample.
Problem 20. If 1.00 mL KMnO4 solution will react Plan Phenolphthalein indicates complete reaction of
NaOH and 50% reaction of Na2CO3. Remaining 50%
with 0.125 g Fe2+ and if 1.00 mL of KHC^O4 H^C^
solution will react with 0.175 mL ofKMnO4 solution, how reaction o/‘Na2CO3 is indicated when methyl orange is
many millilitres of 0.20 M NaOH will react with 1.00 mL of added at this stage. Thus, from the volume of HC1,% of
KH(^O4 H2C2O4 solution in which all the protons (H+) are each is determined.
ionisable? (Fe = 56 g mol-1) I: NaOH + HCl ---- > NaCl+HjO
II: Na2CO3+HCl ---- > NaHCO3+NaCl
Plan We are to relate oxidisable C^O^- ions with ionisable
H+ ions in KHC^Qj • H2Cr>04. Molar concentration o/MnO4 III: NaHCO3+HCl ---- > NaCl+H2O + CO2
is determined by Fe2+. In turn C20j- is determined by Solution Let volume of 0.5000 N HC1 for complete
MnO4. Thus, number of ionisable H+ hence, volume of neutralisation of NaOH = x mL
NaOH can be determined.
and for that of Na2CO3 = y mL
Solution Phenolphthalein indicates 100% reaction of NaOH but
5Fe2+ +MnO4 +8H+ ---- > 5Fe3+ +Mn2+ + 41^0 50% reaction ofNa2CO3 uptoNaHCO3 stage.
5C$" +2MnO4 +16H+ ---- > 10CO2 +2Mn2+ +8H2O NaOH + HCl ---- > NaCl+HjO
H++OH" ---- > H2O Na2CO3+HCl ---- > NaHCO3+NaCl
Let molarity of MnO4 solution = Mj On the other hand, methyl orange indicates 100%
reaction each of NaOH and Na2CO3 both.
and molarity of KHC^ • H^C^ solution = M2 Thus, first end-point appears when complete NaOH and
Number of moles of Fe2+ = moi 50% Na2CO3 have reacted (stage I and II)
56
thus, X+2 = 21-00 mL
Number of moles of MnO4 = 1^1 = 0.001 Mx
1000 Second end-point appears when remaining Na2CO3
Also, 5 mol ofFe2+ = lmol MnOl reacts.
0.125 . 2+ 1 0.125 thus, — = 7 mL, y = 14 mL
------ mol of Fe = - x------- mol MnO4 2
56 5 56
0.125 n _A1 hence, x = 14 mL
-------= 0.001 Mi
5x56 thus, 14 mL of 0.5000 N HC1 b NaOH
M1(MnOp = 0.446 M 14 x 0.5 _ , __3 . .
thus, NaOH in mixture = --------- = 7 x 10 d equivalent
Also, 1.00 mL of MjKHCA 1000
= 7 x 10-3 x 40 = 0.28 g NaOH
= 0.175 mL of 0.446 MMnO4
1 x M2 millimol KHCjC^ ■ H^C^ Similarly, 14 mL of 0.5000 N HC1
= Na2CO3 = 7 x 10"3 equivalent
= 0.001 Af2 mole of KHC^ • H^C^
= 7 x 10-3 x 53 g = 0.371 g Na2CO3
= 0.002 ions
0 2ft
(since, KHCjO4 HjC^C^ has two oxidisable C^O^- ions) % of NaOH in mixture = —— x 100 = 37.33%
0.75
5 mol of C^- = 2 mol of MnO4 0 371
o % of Na2CO3 in mixture = —---- x 100 = 49.47%
0.002 M2 CjOJ- = - x 0.002 M2 MnO; 2 3 0.750
5
Master Exercises
Exercise 1
(Stage 1: Learning)
Short Answer Type Questions 9. In which of the following cases, equilibrium molarit
1. What volume of 0.125 M NaOH, in mL contains 4.0 g and analytical molarity are identical?
ofNaOH? (a) 1 M glucose solution
(b) 1 MH3PO4 solution
2. If 1.06 g of Na2CO3 is dissolved in enough water to (c) saturated Agl solution
make 250 mL of solution, what is molar concentration (d) solution of LiCl in ether.
ofNa2CO3? What are molar concentrations of Na+ and 10. Acetic acid forms 100% dimer in benzene by hydrogel
CO^“ ions?
bonding as shown below by — lines.
3. For each solution, identify the ions that exist in ,0—H—OK
aqueous solution, and specify the concentration of CHg—C
each ion
(a) 0.25 M(NH4)2SO4 (b) 0.123 MNa2CO3
(c) 0.056 MHNO3 If analytical molarity is 1 M, what is its equilibriun
4. If you dilute 25.0 mL of 1.50 M HC1 to 500 mL, what is molarity?
molar concentration of the dilute acid? 11. Calculate the percentage of MnO2 in a minera
5. Which of the following methods would you use to specimen if the iodine liberated by a 0.1344 g sampl;
prepare 1.00 L of 0.125 MH^SC^ ? in the net reaction
(a) Dilute 20.8 mL of 6.00 M H«SO4 to a volume of
MnO2(s)+ 4H+ + 2F ---- > Mn2+ + + 21^0
1.00 L.
(b) Add 950 mL of water to 50.0 mL of 3.00 MB^SO^ required 32.30 mL of 0.07220 M Na2S203.
6. What volume of 0.750 M Pb(NO3)2 is mL, is required (MnO2 = 87 g mol-1)
to react completely with 1.00 L of 0.225 M NaCl
solution? Analytical Questions
Pb(NO3)2(ag) + 2NaCl(oq) * PbCl2(s) 1. A5.00x 102 mL sample of 2.00 MHC1 is treated vritl
+ 2NaN03(o^)
4.47 g of magnesium. Calculate the concentration fl
7. If 38.55 mL of HC1 is required to titrate 2.150 g of the acid solution after all the metal has reacted
Na2CO3 as given, what is the concentration (mol L-1) Assume that the volume remains unchanged.
of the HC1 solution?
2. A useful application of oxalic acid is the removal fl
Na2CO3(a^) + 2HCl(a^) 2NaCl(aq) + CO2(g)
rust (Fe2O3) from a cloth:
>11,0(0
Fe2O3(s)+ eHjCA---- > 2Fe(C2O4)t + 3^0 + 6H’
8. You have 0.490 g of unknown acid, Hj>A, which reacts
Calculate the number of grams of rust that can b<
with KOH, according to
removed by 5.00 x 102 mL of 1 M oxalic.acid.
H>a9) + 2KOH(ag) ---- > l^A(aq) + 2^0(1)
3. 30 mL of x M HC1 neutralise completely 10 mL fl
If 100 mL of 0.1 M KOH is required to titrate the acid
to the second equivalence point, what is the molar 0.2 M NaHCO3 and further 5 mL of 0.2 M Na2C03t«
mass of acid? phenolphthalein end-point. What is value of x?
Chapter 7: Volumetric | 323

4. 300 mL of H2SO4 solution of a certain molarity was 13. 2.4 g of a mixture of (KC1 + NH4C1) is heated till
mixed with 200 mL of a Ba(OH)2 solution of same constant weight is obtained. The solid residue is
molarity. The pH of the mixture solution is 1. dissolved in water and the solution is made up to
Calculate normality of HgSC^ and Ba(OH)2. 250 mL. 25 mL of this solution required 15 mL of
5. What volume of 6M NaOH would be required to react 0.1 N AgNO3 for precipitation of chloride. Calculate
with the SO2 produced from 103 kg of coal containing percentage of chloride by weight in the mixture.
0.05% by mass of FeS2 (iron pyrites). 14. The concentration of Cu2+ ions in the water
2FeS2 + 5.5 O2 ---- > Fe^ + 4SO2 discharged from a certain industrial plant is
SO2 + 2NaOH---- > Na2SO3 + H2O determined by adding excess sodium sulp.iide
solution to 0.80 L of water.
6. 25 mL solution containing a mixture of Na2CO3 and
S2" + Cu2+ ---- > CuS(s)
NaOH required 19.5 mL of 0.995 N HC1 for the
phenolpthalein end-point and in a separate titration What is molar concentration ofCu2+ in water sample
25 mL of the same mixture required 25.9 mL of same if 0.0177 g of solid CuS is formed?
HC1 for methyl orange end-point. Calculate 15. Mn2+(aq) can be determined by titration with
concentration of each base in mixture. MnO4(aq).
7. A drop (0.05 mL) of 12.0 M HC1 is spread over a sheet Mn2+ + MnO4 + OH"---- > MnO2 + HgO (unbalanced)
of thin aluminium foil. Assuming that all the acid A 25.00 mL sample ofMn2r(aq) requires 34.77 mL of
dissolves through the foil, what will be the area, in
cm2, of the hole produced? (density of
0.05876 M KMnO4(aq) for its titration. What is the
Al = 2.70 gem-3; thickness of the foil = 0.10 mm) molarity of the Mn2+(aq) ?
8. Titration of a 0.7439 g sample of impure Na2B4O7 16. Oxalic acid (H2C2O4) is present in many plants and
(borax) required 31.64 mL of 0.108 M HC1 for vegetables. If 24.0 mL of 0.01 M KMnO4 solution is
reaction. Express the result of this analysis in terms needed to titrate 1.00 g of HjC^ to the
of per cent (B = 11) equivalence-point, what is the per cent by mass of
(a) Na2B4O7 (b) Na2B4O7 • 10H>O H^C^ in the sample?
(0 B2O3 (d) B 17. The calcium oxalate isolated from blood is dissolved in
B4O27" +2H3O+ +3H2O 4H3BO3 acid and titrated against a standardised KMnO4
9. Average human blood serum contains 18 mg of K+ solution. In one test, it is found that the calcium
and 365 mg of Cl- per 100 mL. Calculate oxalate isolated from a 10.0 mL sample of blood
(a) the molar concentration for each of these species; requires 24.2 mL of 9.56 x lO^4 M KMnO4 for
use 1.00 g mL-1, for the density of the serum titration. Calculate the number of milligrams of
(b) p and p for human serum. calcium per mL of blood.
10. A mixture of HC1 and H3PO4 is titrated with 0.1 M 18. A solution contains 2.52 g of a reductant per litre.
NaOH. The first end-point (methyl red) occurs at 25 mL of this solution of the reductant required 20 mL
35.00 mL and the second end-point (bromothymol of 0.01 M KMnO4 in acidic medium for oxidation. Find
blue) occurs at a total of 50.00 mL (15.00 mL after the the molecular weight of the reductant given that each
first end-point). Calculate the millimoles of HC1 and of the two atoms which undergo oxidation per
present in the solution. molecule of the reductant, suffers an increase in the
11. A sample of P2O5 contains some H3PO4 impurity. A oxidation state by one unit.
0.405 g sample is reacted with water 19. In an experiment 10 L of air (containing O3) at 1 atm
(P2O5+3H2O ---- > 2HgPO4), and the resulting and 27°C were passed through an alkaline KI
solution is titrated with 0.25 M NaOH requiring solution. After neutralisation, free iodine required
42.5 mL (upto Na2HPO4). What is the per centH3PO4 1.5 mL of 0.01 N Na2S2O3 solution. Calculate mol% of
impurity? O3 in air,
12. 1.2 g sample of Na2CO3 and KgCOs was dissolved in 20. A polyvalent metal weighing 0.1g and having atomic
water to form 100 mL of solution. 20 mL of this weight 51 reacted with dil. to give 43.9 mL of
solution required 40 mL of 0.1 N HC1 for complete Kj at STP. This solution containing the metal in lower
neutralisation. Calculate the weight of Na2CO3 in oxidation state is found to require 58.8 mL of 0.1 N
mixture. If another 20 mL of this solution is treated KMnO4 for complete oxidation. What are the
with excess of BaCl2, what will be the weight of the oxidation states of the metal in two reactions?
precipitate?
324 | Essential Inorganic Chemistry

21. 0.015 mol of K2Cr2O7 oxidises 2.18 g of a mixture of (b) What is the weight of one-gram equivalent
XO and X2O3 into XO; in acidic medium. If 0.0187 AC1„ for the above reaction if the atomic mass c
mol of XO; is formed, calculate atomic weight ofX. is 97.0. ?
6XO + SC^O2/ + 34H+ ---- > 6XO; + 10Cr3+ 28. 0.518 g sample of limestone is dissolved and Cala­
is precipitated as calcium oxalate CaC^O,. Af
+ 17H2O filtration and washing the precipitate, it requii
3X2O3 + 4Cr2O7“ + 26H+---- > 6XO; + 8Cr3+ + 13^0] 40 mL of 0.05 M KMnO4 solution acidified w
22. What mass of N2H.} can be oxidised to N2 by 24.0 g of sulphuric acid to titrate it. What is the percentage
K2CrO4 which is reduced to [Cr(OH)4 ]“ ? CaO in the limestone?
N2H4 ---- > N2 + 4H+ + 4e~
N,H 29. 20 mL of a solution containing 0.2 g of impure sam
CrOj- +4H+ +3e” * [Cr(OH)4r of H2O2 reacts with 0.316 g of KMnO4 in presence
H2SO4.
3N2H4 + 4CrO^~ + 4H*~ > 3N2 +4[Cr(OH)4]~ (a) What is % purity of H2O2?
23. The alcohol content in a 10.0 g sample of blood from a (b) Calculate the volume of dry O2 formed at 27°C a
driver required 4.23 mL of 0.07654 M K2Cr2O7 for 750 mmHg pressure.
titration. Should the police prosecute the individual 30. Na2G2O4 and KHCgO,, HgC^ are mixed in slid
for drunken-driving when the current legal limit is proportion by weight that the normality of i
0.1 per cent by mass? resulting solution as a reducing agent is 3.62 tin
3CH3CH2OH + 2Cr2O27~ + 16H+ ---- > 3CH3COOH the normality as an acid. What is the proportion?
+ 4Cr3+ + HHjO 31. A standard solution of potassium dichromi
24. A 1 g sample of H2O2 solution containing x per cent contains 5.442 g L"1. What is the amount in terms
HoO, by weight required x mL of KMnO4 solution for Fe3O4 per mL K^Ci^O-;?
complete oxidation in acidic medium. Calculate 32. A 0.5 g sample containing MnO2 is treated with H1
normality of KMnO4 solution. liberating Cl2. The Cl2 is passed into a solution of
25. In basic solution CrO^~ oxidises S2O^“ to form and 30.0 mL of 0.1 M Na2S2O3 are required to titn
the liberated iodine. Calculate the percentage
Cr(OH); and SO^*. How many mL of 0.154 M CrO^-
MnO2 in the sample. (Mn = 55)
are required to react with 40.0 mL of0.246 M S2O^“ ?
33. Calculate the mass of oxalic acid, H2C2O4, which c
[Hint: 0.154 M = 0.154 x 3N CrO*'
4
be oxidised to CO2 by 100 mL of an MnO; solntic
0.246 M= 0.246 x8NS2O2“] 10 mL of which is capable of oxidising 50.0 ml
26. 25 mL of a solution containing ferrous sulphate and 1.00 NT to
ferric sulphate acidified with H2SO4 is reduced by MnO; + 8H++ 5e" ---- > Mn2++411,0
metallic zinc. The solution required 34.25 mL of 0.1 N
HaQA ---- > 2CO2+2H++2e“
solution of KjCi^Oy for oxidation. However, before
reduction with zinc, 25 mL of same solution required 21“ ----- > + 2e“
22.45 mL of same KjCi^O.;. Calculate the weight of 34. 5.0 g of bleaching powder was suspended in waters:
FeSO4 and Fe2(SO4)3 per litre of the solution. volume made up to half a litre. 20 mL of il
27. (a) The ion An~ is oxidised to AO3 by MnO; changing suspension when acidified with acetic acid a:
to Mn2+ in acid solution. Given that treated with excess of potassium iodide solufi
2.68 x 10"3 mol of An~ requires 1.61 x IO-3 mol of liberated iodine which required 20 mL of a decino nr
MnO4, what is the value of n ? hypo solution for titration. Calculate the percents
of available chlorine in bleaching powder.

Exercise 2
(Stage 2: High Skill Problem Solving)
Only One Option Correct 2. 10.78 g of H3PO4 in 550 mL solution is 0.40 N. Tb
1. To prepare a solution that is 0.50 M KC1 starting with this acid
100 mL of0.40MKCl (a) has been neutralised to HPC^“
(a) add 0.75 gKCl (b) add 20 mL of water (b) has been neutralised to PO3-
(c) add 0.10 mol KC1 (d) evaporate 10 mL water (c) has been reduced to H2PO;
(d) has been neutralised to H2PO4
Chapter 7: Volumetric | 325

3. How many equivalents are there per mol of HjS in its 11. 2H>O2(Z) ---- > 2H2O(Z) + O2(g)
oxidation to SO2? 100 mL ofX molar H>02 gives 3 L of O2 gas under the
(a) 2 (b) 4 condition when 1 mol occupies 24 L, value of X is
(c) 6 (d) 8 (a) 2.5 (b) 1.0
4. In hot alkaline solution, Br2 disproportionates to Br" (c) 0.5 (d) 0.25
and BrO3 12. If equal volumes of 1 M KMnO4 and 1 M K^Ci^
3Br2 +6OH" ---- > 5Br" +BrO3 +3^0 solutions are allowed to oxidise Fe (II) to Fe (HI), then
hence, equivalent weight of Bi^ is (molecular weight = M) Fe (II) oxidised will be
(a) more by KMnO4 (b) more by KjCi^Oy
<46 (b)".5 (c) equal in both cases (d) data is incomplete
(0^ (d)™ 13. 100 mL of 1 M KMnO4 oxidised 100 mL of HjOg in
5 3 acidic medium (when MnO4 is reduced to Mn2+);
[Hint: Each half-reaction, oxidation and reduction should volume of same KMnO4 required to oxidise 100 mL of
involve equal number of electrons, balance it. 3Br2 = 5e ] H2O2 in basic medium (when MnO4 is reduced to
5. M (molarity) and M' (molality) are related to each MnO2) will be
other by equation: /(a)x ----
100mLT ... 500 T
(b) ---- mL
(m = molecular weight of solute, d = density g/mL] 3 3
m + 1 1000 M . . 300 T
(a)M = d (b)M' = (c) ---- mL (d) 100 mL
1000 AT 1000 d - Mm 5
(c) Both (a) and (b) (d) None of these 14. 10 mL of H2 O2 solution (volume strength = x)
N
6. Maximum number of moles of PbSO4 that can be required 10 mL of ----- MnO4 solution in acidic
0.56
precipitated by mixing 20.00 mL of 0.1 M Pb (NO3)2 medium. Hence, x is
and 30.00 mL of 0.1 M Na2SO4 will be (a) 0.56 (b) 5.6
(a) 0.002 (b) 0.003 (c) 0.1 (d) 10.0
(c) 0.005 (d) 0.001 15. Aqueous solution of chloride of an element A
7. [Na+] in a solution prepared by mixing 30.00 mL of containing 1 x 10"3 mol after complete hydrolysis
0.12 M NaCl with 70 mL of 0.15 M Na2SO4 is required 30 mL of 0.1 M AgNO3 solution to form AgCl.
(a) 0.135 M (b) 0.141 M Element A is
(c) 0.210 M (d) 0.246 M (a) Al (b) P
(c) Si (d) S
8. When 80 mL of 0.20 M HC1 is mixed with 120 mL of
16. Molality of 18 M H2SO4 (tZ = 1.8 g mL"1) is
0.15 M KOH, the resultant solution is the same as a
solution of (a) 36 mol kg"1 (b) 200 mol kg"1
(a) 0.16 M KC1 and 0.02 M HC1 (c) 500 mol kg"1 (d) 18 mol kg"1
i
(b) 0.08 M KC1 17. 10 L of hard water required 0.56 g of lime (CaO) for
(c) 0.08 M KC1 and 0.01 M KOH removing hardness. Hence, temporary hardness in
(d) 0.08 M KC1 and 0.01 M HC1 ppm of CaCO3 (parts per million ie 106) is
(a) 100 (b) 200
9. Equal volumes of 0.200 M HC1 and 0.400 M KOH are (c) 10 (d) 20
mixed. The concentration of the principal ions in the
resulting solution are: 18. 1 g equivalent of a substance is the weight of that
(a) [K+ ] = 0.400 M, [Cl"] = 0.200 M, [H+] = 0.200 M amount of a substance which is equivalent to
(a) 0.25 mol of O2 (b) 0.50 mol of O2
(b) [K+] = 0.200 M, [Cl"] = 0.200 M, [OH" ] = 0.100 M
(c) 1 mol of O2 (d) 8 mol of O2
(c) [K+] = 0.200 M, [Cr] = 0.100 M,[0H“] = 0.100 M
(d) [K+] = 0.200 M, [Cl"] = 0.100M, [OH"] = 0.200 M 19. In an experiment, 50 mL of 0 • 1 M solution of a
metallic salt reacted exactly with 25 mL of 0.1 M
10. Mass of KHC2O4 (potassium acid oxalate) required to solution of sodium sulphite. In the reaction SO^~ is
reduce 100 mL of 0.02 MKMnO4 in acidic medium (to oxidised to SO^“. If the original oxidation number of
Mn2+) is x g, and to neutralise 100 mL of 0.05 M the metal in the salt was 3, what would be the new
Ca(OH)2 is y g then, oxidation number of the metal?
(a) x = y (b) 2x = y (a) 0 (b) 1
(c) x = 2y (d) None of these (c) 2 (d) 4
326 | Essential Inorganic Chemistry

20. 20 mL of x M HC1 neutralises completely 10 mL of 29. 1 mol of ferric oxalate is oxidised by x mol ofMn04 ar
0.1 M NaHCO3 and a further 5 mL of 0.2 M Na2CO3 also 1 mol of ferrous oxalate is oxidised by y mol
solution to methyl orange end-point. The value ofx is MnO4 in acidic medium. The ratio — is
X
(a) 0.167 M (b) 0.133 M y
(c) 0.150 M • (d) 0.200 M (a) 2 : 1 (b) 1 : 2
21. The molality of a H2SO4 solution is 9. The weight of (c) 3 :1 (d) 1: 3
the solute in 1 kg H2SO4 solution is 30. 40 mL of 0.05 M solution of sesquicarbona
(a) 900.0 g (b) 468.65 g (Na2CO3 NaHCO3 -21^0) is titrated against 0- 05-
(c) 882.0 g (d) 9.0 g HC1. x mL of HC1 is used when phenolphthalein is tl
22. The density of 1 M solution of NaCl is 1.0585 g/mL. indicator andy mL of HC1 is used when methyl oranj
The molality of the solution is is the indicator in two separate titrations, hem
(a) 1.0585 (b) 1.00 (y - x) is
(c) 0.10 (d) 0.0585 (a) 80 mL (b) 30 mL
(c) 120 mL (d) None of these
23. Which is not true about H3PO2?
31. 3 moles of a mixture ofFeSO4 and Fe2(SO4)3 requirt
(a) It is tribasic acid
(b) One mol is neutralised by 0.5 mol Ca(OH)2 100 mL of 2 M KMnO4 solution in acidic medinr
(c) NaH2PO2 is normal salt Hence, mole fraction of FeSO4 in the mixture is
(d) It disproportionates to HgPO3 and PH3 on heating (a) 1 (b) |
3 3
24. RHo (i°n exchange resin) can replace Ca2+ in hard . . 2 ... 33
water (c) - (d) -
5 5
+ Ca2+ ---- > PCa + 2H+ 32. In the following reaction
1 L of hard water after passing through -RJL has 28NO3 + 3As2S3 + 4HjO 6AsO4~ + 28N0
pH 2. Hence, hardness in ppm of Ca2+ is + 9S0j"+8H
(a) 200 (b) 100
(c) 50 (d) 125 equivalent weight of As2S3 (with molecular weight^
is
25. 1 mol BaF2 + 2 mol '4 » resulting mixture
M
will be neutralised by (a) (b)
(a) 1 mol KOH
(a,T 4
(c)" M
(b) 2molCa(OH)2 (c) 777 <d)
(c) 3 mol NaOH 24 28
(d) 2 mol KOH 33. In the following reaction :
26. 100 mL of a mixture of NaOH and Na92S0, SO4 is Cr(OH)3 + OH“ + IO3 - •> CrO2" -t-HaO + r
neutralised by 10 mL of 0.5 M HaSQp Hence, amount (a) IO3 is oxidising agent
of NaOH in 100 mL mixture is (b) Cr(OH)3 is oxidised
(a) 0.2 g (b) 0.4 g (c) 6e" are being taken per I atom
(c) 0.6 g (d) 1.0 g (d) All of the above are correct
27. 10 mL ofNaHC^ solution is neutralised by 10 mL of 34. 5.3 g of M2CO3 is dissolved in 150 mL of 1 N HC
0.1 M NaOH solution. 10 mL of same NaHC^ Unused acid required 100 mL of 0.5 N NaOH. Hena
solution is oxidised by 10 mL of KMnO4 solution in equivalent weight of M is
acidic medium. Hence, molarity of KMnO4 is (a) 23 (b) 12
(a) 0.1 M (b) 0.2 M (c) 24 (d) 13
(c) 0.04 M (d) 0.02 M 35. Equivalent weight ofH3PO2 when it disproportionate
28. For a given mixture of NaHCO3 and Na2CO3, volume into PH3 and H3PO3 (mol. wt. = M) is
of a given HC1 required is x mL with phenolphthalein M
(a) M (b) —
indicator and further y mL required with methyl 2
orange indicator. Hence, volume of HC1 for complete
reaction of NaHCO3 is (0 T
4 (d) T"
4
(a) 2x (b)5 36. Lj obtained from 0.1 mol of CuSO4 required 100 ml<
(c) y (d) (y-x) 1 M hypo solution, hence, mol percentage of put
CuSO4 is
Chapter 7: Volumetric | 327

(a) 100 (b) 50 47. 0.116 g of C4H4O4(A) is neutralised by 0.074 g of


(c) 25 (d) 40 Ca(OH)2. Hence, protonic hydrogen (H+) in (A) will be
37. 1.2 gofMg is treated with 100 mL of lMHjSQp Molar (a) 1 (b) 2
concentration of the H2SO4 solution after complete (c) 3 (d) 4
reaction is 48. 10 mL of NaHCjC^ is oxidised by 10 mL of 0.02 M
• (a) 0.5 M (b) 0.005 M MnO^. Hence, 10 mL ofNaHCjC^ is neutralised by
(c)0.10M (d)0.20M (a) 10 mL of 0.1 M NaOH
38. Hardness in water is expressed in terms of ppm of (b) 10 mL of 0.02 M NaOH
CaCO3. If hardness is 200 ppm CaCO3, then in terms
of molarity it is (c) 10 mL of 0.1 M Ca(OH)2
(a) 0.004 M (b) 0.002 M (d) 10 mL of 0.05 NBa(OH)2
(c) 0.001 M (d) 0.0005 M 49. Oxalic acid (HjCjO^j) forms two series of salt HCjO^
39. 10 mL of a blood sample (containing calcium oxalate) and CjC^”. If 0.9 g of oxalic acid is in 100 mL solution,
is dissolved in acid. It required 20 mL of 0.001 M HGjO^- and C^C^- have normality respectively
KMnO4 (which oxidises oxalate to carbon dioxide)
hence, Ca2+ ion in 10 mL blood is (a) 0.1 N, 0.1 N (b) 0.1 N, 0.2 N
(a) 0.200 g (b) 0.02 g (c) 0.2 N, 0.2 N (d) 0.2 N, 0.1 N
(c) 2.00 g (d) 0.002 g 50. At a given condition, 1 mol O2 occupies 30 L. Volume
40. Volume of 18.0 M H2SO4 required to prepare 1.0 L of a occupied by 1 equivalent of O2 is
0.9 M solution of H2SO4 is (a) 7.5 L (b) 15 L
(a) 50.0 mL (b) 10.0 mL (c) 30.0 L (d) 22.4 L
(c) 500.0 mL (d) 5.0 mL 51. 10 g ofMnO2 on reaction with cone. HC1 liberated 0.1
41. Volume of 0.50 M NaOH solution required to react equivalent ofCl2 (Mn= 55). Hence, per cent purity of
with 40.0 mL of 0.05 M H2SO4 solution is MnO2 is
(a) 40.0 mL (b) 80.0 mL (a) 87.0 (b) 21.75
(c) 20.0 mL (d) 8.0 mL (c) 50.0 (d) 43.5
42. Stock phosphoric acid solution is 85% H3pO4 and has 52. The equivalent mass of a metal is double that of
a specific gravity of 1.70. Hence, molarity of HgPQj oxygen. How many times is the equivalent mass of its
solution is oxide greater than the mass of the metal?
(a) 8.51 M (b) 1.70 M (a) 2 times (b) 1.5 times
(c) 14.74 M (d) 7.37 M (c) 3 times (d) 0.5 time
43. 150 mL of 6.00 M H^SC^ solution is mixed with 53. 0.106 g of Na^Og completely neutralises 40.0 mL of
250 mL of 3.00 M H2SO4. Resulting molarity is HjSQp Hence, normality ofH2SO4 solution is
(a) 4.125 M (b) 8.250 M (a) 0.05 N (b) 0.025 N
(c) 4.500 M (d) 1.650 M (c) 0.10 N (d) 0.20 N
44. If 100 mL of 1.00 M is mixed with 200 mL of 54. Volume of 0.02 M MnO4 solution required to oxidise
I
l 1.00 M KOH solution, molarity of the salt formed is 40.0 mL of 0.1 M Fe2+ solution is
(a) 0.100 M (b) 0.200 M
(a) 200 mL (b) 100 mL
(c) 0.333 M (d) 0.666 M
(c) 40 mL (d) 20 mL
45. Commercial « 11.2 volume” H2O2 solution has a
55. A 20.0 mL solution of Na2SO3 required 30 mL of
molarity of
(a) 1 (b) 0.5 0.01 M K^O? solution for the oxidation to Na2SO4.
(c) 11.2 (d) 1.12 Hence, molarity of Na2SO3 solution is
(a) 0.015 M (b) 0.045 M
46. Equivalent weight of Ci^O^- (mol. wt. = M) in the
(c) 0.030 M (d) 0.0225 M
following reaction is
56. Normality of a solution that contains 12.64 g of
C^O2’ +61" + 14H+ ---- > 2Cr3+ + 312 + 7H2O
KMnO4 is 500 mL of solution to be used in the reaction
(a) - (b) — that produces K^MnC^ ion as the reducing product is:
6 3 (a) 0.16 N (b) 0.32 N
,. M (c) 0.80 N (d) 0.08 N
(d) M
T
328 ; Essential Inorganic Chemistry

57. Iodate ion, IO3, oxidises SO3' to SO^“ in acidic is then titrated with standard acid using
medium. If 100 inL sample of solution containing phenolphthalein indicator, what normality of the acid
2.14 g of KIO3 reacts with 60 mL of 0.5 M Na2SO3 will be found?
solution, then final oxidation state of iodine is (a) 0.190 N (b) 0.380 N
(a) +5 (b) +3 (c) 0.095 N (d) 0.0475 N
(c) +1 (d) -1
58. A 0.60 g nitrogen containing compound was boiled One or More Than One Options Correct
with NaOH and NH3, thus formed required 100 mL of 1. 1 g equivalent of a substance is the weight of that
0.2 N H2SO4 for neutralisation; percentage of nitrogen amount of a substance which is equivalent to
in the compound is (a) 0.25 mol O2 (b) 8 g O2
(a) 46.67% (b) 23.34% (c) 16 g O2 (d) 0.50 mol O2
(c) 60.00% (d) 20.00%
2. 2.84 g of P4O10 is dissolved in 1 L aqueous solution.
59. 1.00 L of 0.15 NaOH absorbed 11.2 millimol of CO2 Thus,
from air. Hence, new molarity of NaOH is (a) resulting solution is 0.01 M
(a) 0.1276 M (b) 0.1500 M (b) it is neutralised to Na3PO4 by 400 mL of 0.3 M
(c) 0.0224 M (d) 0.0112 M NaOH solution
60. Molarity of Na2CO3 solution formed in the above (c) it is neutralised to CaHPO4 by 400 mL of 0.1 M
Ca(0H)2 solution
question will be
(d) it cannot be neutralised by basic solution
(a) 0.0244 M (b) 0.0112 M
(c) 0.1500 M (d) 0.1276 M 3. A solution of KHG2O4 H2C2O4 -2H2O is 0.2 N as an
61. A sample of iron ore weighing 0.740 g containing 24% acid. Then it is
Fe2O3 is dissolved and the iron reduced to Fe2+ by the (a) 0.267 N as reducing agent
addition of 25.0 mL of 0.05 M SnCl2. The excess Sn2+ (b) 0.6 M as an acid
is oxidised to Sn4+ using 0.05 M HgCl2. Volume of (c) 0.067 M as an acid
HgCl2 used is (d) 0.067 M as reducing agent
(a) 5.2 mL (b) 6.2 mL 4. A sample of water has a hardness expressed as
(c) 5.6 mL (d) 2.6 mL 77.5 ppm Ca2+. This sample is passed through an ion
62. A sample containing only Na2C2O4 and KHC2O4 exchange column and the Ca2+ is replaced by H’.
Select correct statement(s).
required three times the volume of 0.1 N KMnO4 for
(a) pH of the water after it has been so treated is 2.4
titration as of 0.1 N base (same size sample in each
(b) Every Ca2+ ion is replaced by one H+ ion
case). Percentage of KHC2O4 in the mixture is
(c) Every Ca2+ ion is replaced by two H+ ions
(a) 65.64% (b) 34.36%
(d) pH of the solution remains unchanged
(c) 30.28% (d) 69.72%
5. 10.78 g of H3PO4 in 550 mL solution is 0.40 N. Thus,
63. A 0.6234 g sample that might contain NaOH, Na2CO3,
this acid
NaHCO3 or a mixture of NaOH+Na2CO3 or
(a) has been neutralised to HPO^“
Na2CO3 +NaHCO3 is titrated with 0.1062 M HC1 by
(b) has been neutralised to PO^“
two-indicator method. It is found that 40.38 mL of the
(c) has been reduced to HPO^-
acid is required to reach the phenolphthalein
(d) has been neutralised to H2PO4
end-point. Methyl orange is then added to the
solution, and the titration continued using an 6. Mass of KHC2O4 (potassium acid oxalate) required
additional 12.83 mL of the acid. Thus, sample to reduce 100 mL of 0.02 M KMnO4 in acidic medium
contained (to Mn2+) is x g and to neutralise 100 mL of 0.05 M
(a) NaOH+Na2CO3+NaHCO3 Ca(OH)2 isy g, then
(b) NaOH+Na2CO3 (a) x-y (b) 2x = y
(c) Na2CO3 +NaHCO3 (c) x = 2y (d) None of these
(d) can't be predicted
7. A mixture of Na2C2O4 (A) and KHC^. H2G2O4 W
64. Percentage of NaOH in the mixture of the above
example is required equal volumes of 0.1 M KMnO4 and 0.1 M
(a) 18.77% (b) 23.17% NaOH separately. Molar ratio of A and B in the
(c) 37.54% (d) 46.34% mixture is
(a) 1:1 (b) 1: 5.5
65. A bottle which contains 200 mL of 0.100 M NaOH (c) 5.5 : 1 (d) 3.1:1
absorbs 1.00 mmol of CO2 from the air. If the solution
Chapter 7: Volumetric | 329

8. and NaHC^Qj behave as acids as well as 4. We define p-value of a term (say x) as


reducing agents. Which are correct statements? Px = “ log x
(a) Equivalent weight of and NaHC^ are Calculate p-value for each ion in a solution that is
equal to their molecular weights when behaving 2.00 x 10'3 M in NaCl and 5.4 x 10’4 M HC1.
as reducing agents
(b) 100 mL of 1 M solution of each is neutralised by 5. Calculate molar concentration of Ag+ in a solution
equal volumes of 1 M Ca(OH)2 that has pAg = 6.6990
(c) 100 mL of 1 N solution of each is neutralised by 6. What mass of Ag2CO3 is formed when 25.0 mL of
equal volumes of 1 N Ca(OH)2
(d) 100 mL of 1M solution of each is oxidised by equal 0.2 M AgNO3 is mixed with 50.0 mL of 0.08 M
volumes of 1 M KMnO4 Na2CO3?
9. 100 mL of 1.44% solution of pure FeC2O4 in dil. HC1 is 7. What is the molar concentration of Na2CO3 in the
oxidised by 0.01 M KMnO4. Then volume of KMnO4 solution when 25.0 mL of 0.2 M AgNO3 are mixed
required is with 50.0 mL of 0.08 M Na2CO3 solution?
(a) 120 mL (b) 600 mL 8. Calculate the p-value for each of the indicated ions in
(c) 200 mL (d) 60 mL the followng
10. 1 mole of MnO^“ in neutral aqueous solution (a) Cu2+, Zn2+ ;and' NO3 in a solution that is
in Cu(NO
4.78 x 10"2 M __ ___ ’3)23). and 0.104 M in
disproportionates to Zn(NO3)2.
2 1
(a) - mol of MnO4 and - mol of MnO, (b) H+, Ba2+ and C1O4 in a solution that is
3 3
3.35 xlO-4 M in Ba(C104)4 and 6.75 xlO"4 M in
1 2
(b) - mol of Mn07 and - mol of MnO2 HC1O4.
3 4 3
9. Sea water contains an average of 1.08 x 103 ppm of
(c) mol of Mn2O7 and - mol of MnO2 Na+ and 270 ppm of SO^". Calculate
3 3
2 1 (a) the molar concentration of SO;’ and Na+ given
(d) - mol of Mn2O7 and - mol of MnO2 that average density of sea water is 1.02 g/mL
3 3
(b) the pN and pcr.2_ for sea water
10. A solution was prepared by dissolving 5.76 g of
Brain Twisters KC1 • MgCl2 • 6H2O in sufficient water to give 2.00 L.
1. You place 2.56 g CaCO3 (calcium carbonate) in a Calculate.
beaker containing 250 mL of 0.125 M HC1. When the (a) the molar analytical concentration ofKCl MgCl2
reaction has ceased, does any CaCO3 remain? What (b) the molar concentration of Mg2’ andCl-
mass of CaCl2 can be produced? (c) the number of millimoles of Cl“ in 25.0 mL of this
CaCO3(s) + 2HCl(aq) ---- > CaCl2(aq)+ CO2(g) solution
(d) ppmK+
+1^0(0
11. What volume of 0.01 M AgNO3 would be required to
2. Gold can be dissolved from gold-bearing rock by
treating the rock with sodium cyanide (NaCN) in the precipitate all the Tin a 200.0 mL of a solution that
presence of oxygen. contained 24.32 ppt of KI?
4Au(s) + 8NaCN(a^) + O2(g) + 2H2O(Z) ---- > 12. Exactly 750.0 mL of a solution that contained
480.4 ppm of Ba(NO3)2 were mixed 200.0 mL of a
4NaAu(CN)2(ag) + 4NaOH(aq)
solution that 0.03090 M A12(SO4)3
(a) Name the oxidising and reducing agents in this (a) What mass of solid BaSO4 was formed?
reaction. What has been oxidised and what has (b) What was the molarity of the unreacted reagent
been reduced? [A12(SO4)3 orBa(NO3)2l?
(b) If you have exactly one metric ton (1 metric ton =
13. The phosphorus in a 4.258 g sample of a plant food
1000 kg) of gold-bearing rock, what volume of
was converted to PO4" and precipitated as AggPO4
0.075 M NaCN, in litres, do you need to extract the through the addition of 50.0 mL of 0.082 M AgNO3.
gold if the rock is 0.0197% gold? (Au = 197) The excess of AgNO3 was back-titrated with 4.86 mL
3. 500 mL of 0.25 M Na2SO4 solution is added to an of 0.0625 M KSCN. Express the results of this
aqueous solution of 15.00 g ofBaCl2 solution resulting analysis in terms ofP2O5.
in the formation of white precipitate of BaSO4. Which P,O5 +9H2O » 2PO3-+6H3O+
is the limiting reactant? Calculate moles of BaSO4 2P0f~ +6Ag+ •» 2Ag3PO4(s)
formed. (Ba = 137, S = 32,0 = 16, Na = 23) ■> AgSCN(s)
Ag+ +SCN"
330 | Essential Inorganic Chemistry

14. How many mL of 0.30 M CaCl2 must be added to Passage 2


400 mL of 0.20 M KC1 to produce a solution of 0.25 M Copper containing alloy weighing 0.3175 g is dissolve^
er? in an acid, and an excess of KI is added. Estimation c
15. How many kg of wet NaOH containing 12% water is copper in alloy is based on following reaction
required to prepare 60 L of 0.50 M NaOH solution? 2Cu2+(aq) + 5I~(ag) ---- > Cu^s) + I3(aq)
16. Exactly 50.0 mL of Na2CO3 solution is equivalent to I3(aq) + 2S2O^"(aq) ---- > S4O^"(a(7) + 3r(aqF)
56.3 mL of 0.102 N HCI in acid-base neutralisation. Answer the following questions based on the abov-
How many gram CaCO3 would be precipitated if an reaction.
excess ofCaCl2 solution were added to 100 mL of this 1. Oxidising agent (O) and reducing agent (R) in th«
Na2CO3 solution? above reaction:
17. What mass of phosphoric acid, H3PO4 is required to O R
prepare 550 mL of 0.40 N solution? (a) Cu2+, S2O23“ i3,r
(a) assuming complete neutralisation of acid (b) Cu2+,S2O5“
(b) assuming reduction to HPO^~ (0 Cu2+,I3 r,s2o|-
HgPO4+3OH" ---- > 3H2O + PO43" (d) Cu2+,T I3 > S2O3
H3PO4+2e" ---- > HPO|"+H2O 2. Estimation is based on
18. The tetraethyl lead, TEL, [PbtC^Hglj ] in a 25.00 mL (a) iodometry (b) iodimetry
(c) gravimetry (d) colorimetry
sample of aviation gasoline was shaken with
15.00 mL of 0.02095 MI2 3. If 25 mL of 0.10 M Na2S2O3 is required, Cu present in
Pb(C2H5)4+I2 ---- > Pb^H^I + CAI alloy is
(a) 20% (b) 40%
After the reaction, unused 1^ required 6.09 mL of
(c) 45% (d) 50%
0.03465 M Na2S2O3 solution. Calculate molar
concentration of TEL. 4. formed is converted to KI3 in the given reaction.
This shows
Passage Comprehension Questions (a) is soluble in H2O (b) is a Lewis acid
(c) I2 is ionic compound (d) I" is a Lewis base
Passage 1
Chromium (III) ion forms many compounds with Passage 3
ammonia. To find the formula of one of these compounds, Consider the titration of solution X of 1 M HC1 using
you titrate theNH3 in the compound with standarised acid 0.1 M NaOH solution with phenolphthalein indicator, by
Cr(NH3)x Cl3(ag) + xHCl ---- > xNH4+(aq) + Cr3+(aq) two students A and B.
(A) Student Process Titre volume of NaOH
+ (x + 3)Cl-(ag)
A 20 mL HCI + 20 mL H?O + VA
Assume that 40 mL of 1.5 M HC1 is used to titrate 1 drop phenolphthalein
2.635 g of (A).
B 20 mL HCI+ 60 mLHjO +
Based on the above experiment, answer the following 1 drop phenolphthalein
questions.
1. Value of x is Answer the following questions
(a) 3 (b) 4 1. What is the relation between VA and VB ?
(c) 5 (d) 6 MVa = VB (b)2VA = VB
2. If 1.5 M H2SO4 is used instead of 1.5 M HC1, titre (c)Va=2Vb (d)3VA = VB
value is 2. Before titration, resultant molarity of HCI of the
(a) 40 mL (b) 20 mL student A after addition ofl^O is
(c) 10 mL (d) 15 mL (a) same as that of B (b) double as that of B
3. Probable complex (assume that coordination number (c) half as that of B (d) one third as that ofB
of chromium is six) can be 3. End-point is
(a) [Cr(NH3)6]Cl3 (a) appearance of blue colour
(b) [Cr(NH3)5Cl]Cl2 (b) disappearance of blue colour
(c) [Cr(NH3)4Cl2]Cl (c) appearance of pink colour
(d) [CrCl3]-6NH3 (d) disappearance of blue colour
Chapter 7: Volumetric | 331

Passage 4 Passage 6
For titrating a certain volume of a reducing substance Following experiment is designed to determine
by 1 M KMnO4, it is found that 20 mL was used in acidic calcium content in the blood. Read the expeiiment and
medium, 33.4 mL was used is alkaline medium and answer the questions at the end of it.
100 mL was used in neutral medium. MnO4 is reduced to Calcium oxalate (CaCjC^) is insoluble in water. For this
Mn2r in acidic medium. reason it can be used to determine the amount ofCa2+ ions
in fluids such as blood. The calcium oxalate isolated from
E
Based on this study, answer the following questions. blood is dissolved in acid and titrated against a
1. To what state MnO4 is reduced to in alkaline standardized KMnO4 solution.
i Answer the following questions
medium?
(a) +4 (b) +1 1. In one test it is found that the calcium oxalate isolated
(c) +5 (d) +6 from a 10.0 mL sample of blood requires 25 mL of
1 x 10”3 M KMnO4 for titration. Calculate the
2. To what state MnO4 is reduced to in neutral medium? number of milligrams of calcium per mL of blood
(a) +4 (b) +5 (a) 0.50 (b) 10.0
(c) +6 (d) +7 (c) 0.25 (d) 5.0
3. 1M KMnO4 expressed in terms of normality is in 2. Milliequivalents of KMnO4 used were
Acidic medium Alkaline medium Neutral medium (a) 0.125 (b) 0.025
(c) 0.250 (d) 0.050
(a) 1 N 1 N 1 N
3. Calcium content expressed in ppm is
(b) 5N 3N 1 N
(a) 250 (b) 200
(c) 1 N 3N 5N
(c) 150 (d) 100
(d) 0.2 N 0.33 N 1 N
4. Above titration takes place in acidic medium. Acid
chosen is
Passage 5 (a) H2SO4 (b) HC1
Both Ci^C^“(aq) and MnO^fug) can be used to titrate (c) CH3COOH (d) HCOOH
Fe2+(aq). You have been provided with 0.1 M solution of 5. Indicator in the above titration is
each and volumetric estimation is carried in acidic (a) phenolphthalein
medium. Suppose for the given volume of Fe2+ solution : (b) methylene blue
Vi = volume of MnO4 (c) methyl orange
(d) KMnO4 (self-indicator)
V2 - volume ofC^O^-
required in the oxidation to Fe3+.
Based on this experimental study, answer the Passage 7
following questions : Following titration method is given to determine total
content of the species with variable oxidation states.
1. Select correct alternate. Answer the questions given at the end of it.
lFe2+ is equivalent to A quantity of 25.0 mL of solution containing both Fe2+
(a) - mol of MnO4 and Fe3+ ions is titrated with 25.0 mL of 0.0200 M KMnO4
5 (in dilute H2SO4). As a result, all of the Fe2+ ions are
(b) | mol ofCrjO2" oxidised toFe3+ ions. Next 25 mL of the original solution is
treated with Zn metal. Finally, the solution requires
(c) 1 equivalent each of MnO4 and C^O^- 40.0 mL of the same KMnO4 solution for oxidation to Fe3+.
(d) All of the above MnO; + 5Fe2+ + 8H+ ---- > Mn2+ + 5Fe3+ + 41^0
2. (a) Vx = V2 Answer the following questions
(b) V1>V2 1. Molar concentration of Fe2-" in the original solution is
(0 ^<72 (a) 0.01 M (b) 0.02 M
(d) nothing can be predicted (c)0.10M (d)0.20M
3. 24.50 mL of 0.1 M Ci^O^- is required in the oxidation 2. Molar concentration ofFe3+ in the original solution is
(a) 0.06 M (b) 0.16M
of given volume of 0.1 M Fe2+. 0.1 M MnO4 required
(c) 0.032 M (d) 0.012 M
will be
(a) 29.40 mL (b) 24.50 mL 3. Zinc added in the second titration will
(c) 20.42 mL (d) 20.10 mL toFe
(a) oxidize Fe2+to Fe3+ (b) reduce Fe3+ toFe2+
(c) reduce Fe3+ to Fe (d) reduce Fe2+ to Fe
332 | Essential Inorganic Chemistry

4. If0.02MK2Cr207 is used instead of0.02MKMnO4, its The driver’s blood alcohol level can be determine
volume required in these titrations are respectively readily by measuring the degree of this colour change (res
(a) 25 mL, 40 mL (b) 25 mL, 15 mL from a calibrated meter on the instrument). The currea
(c) 20.8 mL, 33.3 mL (d) 10.4 mL, 16.7 mL legal limit of blood alcohol content is 0.1 per cent by mas
Anything higher constitutes intoxication.
Read the passage and answer the questions give
Passage 8 below
Following short write-up describe synthesis of L from
1. Above test is based on
natural source chile saltpetre. Answer uie questions at the
(a) smell of ethanol and ethanoic acid
end of it.
Chile saltpetre is a natural source of NaNO3 which (b) change of colour yellow to blue
also contains NaIO3. The NaIO3 can be used as a source of (c) change of colour orange to green
iodine produced in the following reactions: (d) change of colour orange to colourless
IO3 + 3HSO3 ---- > I" + 3H+ + 3S0j" ...(i) 2. Equivalent mass of is
, . molar mass ... molar mass
51" + IO3 + 6H+ ---- > Sljts) + 3F ~ ...(ii) (a)—6— (b)—-—
1.00 L of the starting solution which contains 5.80 g molar mass molar__
mass
NaIO31L, is treated with stoichiometric quantity of (c) (d)
2
NaHSO3.Then a further quantity of the starting solution is
added to the reaction mixture to bring about the second 3. Equivalent mass of CHgCH^OH is
reaction. [1= 127, Na = 23, S= 321 . . molar mass . molar mass
(a) ------ ------- (b)------ -------
Answer the following questions
molar mass ... molar mass
1. How many grams ofNaHSO3 are required in step (i)? (d)(d) -----------
(c) ----------------------------
3 4
(a) 5.80 g (b) 17.4 g
(c) 9.14 g (d) 3.0 g 4. The alcohol content in a 10.0 g sample of blood from i
driver required 4.23 mL of 0.07654 M KgCijO? fa
2. What additional volume of the starting solution must titration. Should the police prosecute the individua
be added in step (ii)? for drunken driving?
(a) 0.2 L (b) 0.4 L (a) Yes (b) No
(c) 0.8 L (d) 1.0 L (c) Nothing can be predicted
3. How many equivalents of IO3 are present in one mole
of NaIO3 in step (i)? Assertion & Reason
(a) 2 (b) 4
(c) 6 (d) 8 Codes:
(a) Both A and R are true and R is the correct
4. How many equivalents of IO3 are used in step (ii)? explanation of A.
(a) 6 (b) 5 (b) Both A and R are true but R is not the corred
(c) 0.06 (d) 0.035 explanation of A.
(c) A is true but R is false.
Passage 9 (d) A is false but R is true.
Following passage is for the drunken drivers on roads 1. Assertion (A) : If a solution is 20% by weight d
which are becoming fatal by their wrong driving. solution, then it is 25% by weight of solvent.
Every year in our country about 25,000 people are Reason (R): Solution also includes weight of solute
killed and 500,000 more are injured as a result of drunk and thus, pure solvent is 80 g which contains 20 j
driving. In spite of efforts to educate the public about the solute.
dangers of driving while intoxicated and stiffer penalties 2. Assertion (A) : Molality is independent a
for drunk driving offenses, law enforcement agencies still temperature.
have to devote a great deal of work to remove drunk drivers
from roads. Reason (R) : Volume of solution does not change
The police often use a device called a breath analyser with temperature.
to test drivers suspected to being drunk. The chemical 3. Assertion (A): In the following half reaction:
basis of this device is a redox reaction. A sample of the Ci^O2/ ---- > Cr3+
driver’s breath is drawn into the breath analyser, where it Equivalent weight of Ci2O27" -Molar mass
is treated with an acidic solution of potassium dichromate.
SCHgCHjOH + 2K2Ci2O7 + SH^ ---- > 3CH3COOH Reason (R) : Change in oxidation number a
+ 2Ci2(SO4)3 + 2K2SO4 + IIH2O chromium is 3 per atom.
Chapter 7: Volumetric | 333
I
4. Assertion (A) : 1.575 g HjQjQj ^HgO in 250 mL 3. Pyrolusite can be estimated by
solution makes it 0.1 N. 4. Concentration of Na2CO3 solution can be determined
Reason (R) : HjC^ • 21^0 is a dihydrate organic by titration with acid solution using as the
acid. indicator.
5. Assertion (A) : Following reaction is a 5. End point in the titration of CuSO4 iodometrically
disproportionation reaction: (with starch as the indicator) is the
Bi2 + 2OH- ---- > BrCT + Br" + HjO 6. Concentration of 1 mg per litre is equal to
Reason (R) : Br2 is oxidised to BrO" as well as ppm.
reduced to Br". 7. If 900 mL H2O is added to 100 mL of 1 M I^SC^, then
it is N H2SO4.
True & False 8. MnO4 oxidises Mn2+ to MnO2 and itself reduces to
in the titration method.
1. A solution 20% by mass of solvent is 25% by mass of
9. Normality of the mixture of 100 mL 1 N HC1 and
solution. 50 mL2NH2SO4 is.............
2. Equivalent mass of in the following reaction is 10. If 100 mL of SO2 solution required 50 mL of 0.01 M
M/6. Ci^O2/ in acidic medium then SO2 solution is
Ci^O2/ ---- > Cr3+ 11. Normality of the mixture of 100 mL 1 N HC1 and
3. 100 mL of 1M SO2C12 aqueous solution is neutralised 50 mL 1 N NaOH is and the mixture is
by 100 mL of 4 N NaOH.
4. HgOg can be estimated by MnO4 in acidic as well as in 12. 100 mL of a mixture of FeSO4 and Fe^SQJg required
basic medium. 50 mL of 0.1 N KMnO4 solution, then concentration of
5. lodimetric estimation includes CuSO4, Ci^O^", MnO4, Fe2+ is g L"1.
CaOCl2, MnO2/HCl which on treatment with KJ 13. Ozone in the atmosphere is determined by
liberate^. 14. B1O3 oxidises I" to and thus equivalent
6. In iodometric titration of Ci^O^", there is bluish green weight of BrOj is
colour at the end point. 15. Ig oxidises N2H4 to
7. Concentration of 1 ppm means that 1 L solution
contains 1 mg solute. Matrix-Match Type Questions
8. “10 volume HjQj” means 1 mL of HjOg concentrated
I. Only One Correct
solution has been diluted to 10 mL.
1. Match the reaction (in List I) with titration method
9. “1 volume HjO-j ” solution means that 1 mL of at
(in List II).
NTP gives 1 L ofO2.
10. Ferric sulphate is oxidised by MnO^ IH+. List I List II

11. In the titration of Fe2+ with MnO4, MnO4 is A. N2H4 + 2I2---- » N2 + 4H+ + 4F 1. Iodometric
self-indicator. B. 2CuSO4 + 4KI ---- > Cu2l2 2. Redox
12. Concentration of H^C^ can be determined by + 2K2SO4 + l2
MnO4/H+ as well as by NaOH. C. MnO4 + Fe2* Mn2+ + Fe3+ 3. Kjeldahl
13. Only 50% of Na2CO3 solution is indicated if it is
D. NH4CI + NaOH -A> NH3 4. lodimetric
neutralised by HC1 using phenolphthalein as the
indicator. 2NH3 + H2SO4-> (NH4)2SO4
14. Estimation of nitrogen is done by Kjeldahl method.
15. SO2 turns acidified I^Ci^O? solution to colourless 2. Match the half-reaction (in List I) with equivalent
mass (molar mass = M) (in List II).
solution.
List I List II
Fill in the Blanks A. Cr2O|" —> Cr3+ 1. M
1. If molecular weight of KMnO4 is 158, then its B. c2o42- —> CO2 2. MI2
equivalent weight is in acidic medium. C. MnO4 —> MnO2 3. M/6
2. Equivalent of KMnO4 is 158 when it changes to D. HC2O4 --- > c2ot 4. M/3
334 | Essential Inorganic Chemistry

3. Match the reactants (in List I) with the indicator (in Integer Answer Types
List II).
This section contains 8 questions. The answer to eacl
List I List II of the questions is a single digit integer, ranging from 01*
9. The appropriate bubbles below the respectively question
A. Fe2*/Cr2O2‘ 1. Ferric alum numbers in the ORS have to be darkened. For example, L
B AgNO3/CNS' 2. MnO4 (self) the correct answers to question number X, Y, Z and IV (say
are 6,0,9 and 2, respectively, then the correct darkening o-
C. CuSO4/KI/S2O:'3 3. K3[Fe(CN)6]
bubbles will like the following :
D. Fe2+/MnO4 4. Starch
X z w
Y

4. Match the reaction (in List I) with the colour at the


© © ® ®]
final stage (in List II).
0. O
© ©
List I List II ® ® ® ®
A. MnO4/H*/Fe2* 1. Deep reddish brown
®
_® © ®
B. MnO4 /cone. OH“ /Mn2+ 2. Colourless
_® ® ® ©
C. MnO4 ZH2O/dil.OH- 3. Blackish brown 0. 0 0
D. CuSO4/KI 4. Green _® ® ® ®
© ® © ©
II. One or More Correct
Set I
1. Match the reactant (in Column I) with the type of
titration (in Column II) used for estimation. 1. 1 M MnO4 solution is x equiv. L-1 when reduced to
Mn2+. Thus, x is
Column I Column II
2. 1 M CigO^- solution is y equiv. L-1. Thus, y is
A. KHC2O4 1. Acid-Base titration
3. 1 M C^O^- solution is z equiv. L-1. Thus, z is
B. FeSO4(NH4)2SO4-6H2O 2. Redox titration
C. CuSO4 3. Iodometric
4. 6.3 g of an acid (molar mass = 126 g mol-1) reacts
D. I2 4. lodimetric
completely with 100 mL of 0.5 Ca(OH)2 solution. Thus
basicity of acid is
E. K2Cr2O7
5. We want to prepare 2 L of 0.025 M CH3COOH
2. Match the half-reaction (oxidation or reduction) solution. Amount of CH3COOH dissolved is
(in Column I) with number of electrons exchanged 6. 100 mL of 0.01 M XO4 is reduced to xn+ by 100 mL of
(in Column II).
0.05 MFe2+ in acidic medium. Thus, oxidation state of
Column I Column II XinXn+is
A. 31’ ----- > Ij 1. 5 7. Cr(NH3)xCl3(aq) solution is titrated with HCl(o^)
B. Cr2O2- ----- > 2Cr 3+ 2. 6 solution.
C. Of----- > CIO3 3. 2 Cr(NH3)xCl3(ag) + xHCl(aq) ---- > xNH4+(a<?)
+ Cr3+(a^) + (x + 3)Cl'(aq)
D. N2 —♦ N2H4 4. 4
Assume that 24.26 mL of 1.5 M HC1 is used to titrate
E. MnO4----- > Mn2+
1.580 g of Cr(NH3)xCl3. Thus, x is
F. C2O^ ----- > 2CO2
8. 250 mL of 0.1 M NaOH contains x g of NaOH. Thus.x
is
Chapter 7 . Volumetric | 335

Set II Set III


1. Basicity of H3PO4 is . 1. Complex Cr(NH3)xCl3 (A) is titrated with 1.5 M HC1.
40 mL of HC1 are required to titrate 2.635 g of A.
2. Basicity ofH3P03 is . Thus, x is
3. Basicity ofH3P02 is 2. Mixture containing 1 mole BaF2 and 2 moles H2SO4 is
4. 2 moles of MnO4 oxidise moles ofNaHC^Qp to be made neutral by adding Ca(OH)2. Moles of
Ca(OH)2 required
5. lonisable H atoms in acetic acid is 3. If[BaCl2 ] = 0.05 M then p
ci- ...............
6. 20 mL of 1M MnO4 are required in acidic medium and 4. 2.78 g FeSO4 xHjO in 100 mL solution gives 0.1 M
100 mL of 1M MnO4 are required in neutral medium solution. Thus, x =
in a redox titration. If MnO4 is reduced to Mn2+ in
5. 25 mL of a solution containing 6.1 gL-1 of an oxalate of
acidic medium, oxidation number of Mn in reduced
formula KxHy(C2O4)x • nl^O required 18 mL of 0.1 N
species in neutral medium is NaOH and 24 mL of 0.1 N KMnO4 in two separate
7. 22.4 L is the volume of equivalent(s) of O2 gas titrations.
atNTP. Thus, x =
6. Thus, y =
8. 25 mL of 0.1 M solution of metallic salt (A) oxidises
25 mL of 0.1 M sodium sulphite to sodium sulphate. If 7. Thus, z =
oxidation number of the metal in the salt( A) is 3, then 8. Thus, n =
new oxidation number of the metal is (Refer Problem 15 (P-323) for solution of Question 5-8)
Answers & Solutions
Master Exercises
Exercise 1
Short Answer Type Questions 6. Moles of NaCl =1.00 x 0.225
1. NaOH = 0.125 M = 0.225 mol
Volume = V (in litres) = ? Moles of PbCl2 = V (required) x 0.750
Amount = 4.0 g 2 mol NaCl react with = 1 mol of Pb(NO3)2
MV = moles = moles x 40 g Thus, 0.225 mol reacts with = 0.1125 mol Pb(NO3)2
0.125 x V = moles V x 0.750 = 0.1125
0.125 x V x 40 = 4 g (given) V = 0.15L
V=0.8L = 800 mL = 150 mL
MmV 7. Method I: In terms of moles
or w =-------
1000 Volume of HC1 =38.55 mL
molarity x molar mass x volume (in mL) = 0.03855 L
amount in grams =---------------------- 1000----------------------
Moles of HC1 = 0.03855 x molarity (required)
.. lOOOio 1000x4 n
v =-------- ------------ = 800 mL Moles of Na,CO3 = = 0.0203
Mm 0.125x40 2 3 106
2. Na2£CO3o 2Na+ +CO2o ’ Based on reaction
1 mol Na2CO3 reacts with = 2 mol HC1
Moles of Na2CO3 = = 0.01 mol
2 3 106 0.0203 mol Na2CO3 reacts with = 0.0406 mol HC1
Volume = 250 mL = 0.250 L 0.03855 xM =0.0406
mole 0.010 A A . _. M = 1.05M
Molarity = ------ =--------= 0.04 M
litre 0.250 Method II: In terms of equivalent
[Na+] = 0.08M In any reaction.
[CO2’] = 0.04 M Equivalent of reactant I = Equivalent of reactant II
3. (a) [NH* ] = 0.50 M, [SO2-1 = 0.25 M Equivalent of Na2CO3 =-------2.150--------
(b) [Na+ ] =0.246 M, [CO2’] = 0.123 M 53 (equivalent mass)
(c) [H+] = [NO3] = 0.056 M NV AT x 38.55
Equivalent of HC1 =
4. For dilution 1000 “ 1000 ~
= M2V2 Nx 38.55 2.150
before after 1000 53
1.50 x 25.0 =M2 x500 MHCl) = 1.05N
1.50 x 25.0 M(HC1) = N(HC1) = 1.O5N
M2 =
500 8. As above
= 0.75 M Equivalent of H2A = Equivalent of KOH
5. Mi Vi =M2V2 0.490
Equivalent of H2A =
6.00 x 20.8 = M2 x 1000 E
100x0.1
M2 = 0.125 M of volume 1L Equivalent of KOH =
1000
(b) 50.0 mL of 3.00 M is diluted to 1000 mL
= M2V2 = 0.01
3.00 x 50.0 = M2 x 1000 — = 0.01
E
M2=0.15M
E = 49 g equivalent
Thus, method (a) is employed.
H2A is a dibasic acid, hence,
Chapter 7: Volumetric | 337

molar mass of H2A = 2 x equivalent mass Mg = 4.47 g = 0.18625 mol


= 98 g mol"1 HC1 required =0.3725 mol
9. (a) Glucose is non-electrolyte HC1 unreacted = (1 - 0.3725) mol in 500 mL solution
(b) H3PO4 H+ + HjjPO; Molarity of HC1 = = 1.255 M
(c) Agl is insoluble in H2O
(d) LiCl behaves like non-electrolyte in ether. 2. Fe2O, + 6H2C9O4 ---- > 2Fe(C2O4): +3H2O + 6H*
1 mol 6 mol
Hence, in (a), (c) and (d), analytical and equilibrium
concentrations (molarity) are identical. Oxalic acid taken = 500 x 1 =500 millimol = 0.5 mol
0.5 1
10. 2CH3COOH (CH3COOH)2 .-. Fe20q required = — mol = — mol
monomer dimer 2 3 6 12
In benzene, there is 100% dimer formation of acetic acid. = ^ x 160 = 13.33 g
Thus, molecular weight of acetic acid in benzene
= 120 g mol"1 3. NaHCO3 is completely neutralised and Na2CO3 is
[CH3COOH]eq = 0.0 M neutralised to phenolphthalein end point indicating
[(CHoCOOHkU = 0.5M neutralisation upto NaHCO3
Note: IfCH3COOH (without dimer) = 1 M it means 60 g L”1 NaHCO3+HCl ---- > NaCl + H2O + CO2
of solution. But when dimer is formed Na2CO3 +HC1 ---- > NaHCO3 + NaCl
60 gL"1 Thus, 0.2 M NaHCOo =0.2 N NaHCO,
equilibrium molarity = 0.2 M Na2CO, = 0.2 N Na2CO3
120 g mol-1 and X M X M

= 0.5 molL'1
11. MnO2(s) + 4H++ 21" ---- > Mn2++I2 + 2H2O
acid base (Na2CO3 and NaHCO3)
30x = 10x0.2+ 5x0.2
L+2S.O2- ---- > S4O2"+2I" x = 0.1 M
Method I: (Based on number of equilivalents) 4. Solution is acidic (pH = 1)
Equivalent of MnO2 = equivalents ofS203"
[H+] = 10"pH M
Equivalent mass of MnO2 =43.5
[H+] = 10-1M
0.07220 M S2O2" = 0.07220 N S2O2"
Let molarity of H2SO4 or Ba(OH)2 = 1M
Equivalents of S2O2" = °-07220x32gg = 2.332 x 10’3
Normality = 2 M
2 3 1000
(acid) = 300x2 = 600 M
Equivalents of MnO2 =2.332 x 10“3
N2V2 (base) = 200 x 2 = 400 M
Pure MnO2 in the sample = 2.332 x 10"3 x 43.5 g
= 0.1014 g
Vl +V2
% of pure MnO2 = x 100
200 M
0.1 =-------
= 75.48% 500
Method II: (Based on stoichiometric coefficients) 0.1x500
M= = 0.25
xt x r , „ ^2- 32.30x0.0722 200
Numbr of moles S20!" =-----------------
2 3 1000 Normality = 2 M = 0.5 N
= 2.332 xlO"3 mol 5. 2FeS2+5.5O2 ---- > Fe2O3+4SO2
mol of S2O3"
Number of moles of MnO2 = 4SO2 + 8NaOH ---- > 4Na2SO3+4H2O
2
Thus, 2FeS2 s4SO2 = 8NaOH
= 1.166 xlO"3 mol
lFeS2=4NaOH
Pure MnO2 in the sample = 1.166 x 10"3 x 87 g c c 0 05
FeS2 in 1000 kg (106 g coal) = 106 x = 500 g
% MnO2 = 75.48%
= — =4.167 mol
Analytical Questions 120
1. Mg +2HC1 ---- > MgCl2+H2 NaOH required = 16.67 mol
1 mol 2 mol 6 M NaOH contains 6 mol NaOH in = 1000 mL
24 g 73 g 1000x16.67 T
16.67 mol are in = ---------------- mL
HC1 = 5 x 102 x 2 millimol 6
= 1000 millimol = 1 mol = 2778mL
338 | Essential Inorganic Chemistry

or MV 31.64x0.108
---- = mol Moles ofHCl =
1000 1000
6x V = 3.417 xlO’3 mol
------ = 16.67
1000 Moles of Na2B4O7 = 1.7085 x 10-3 mol
V = 2778 mol Na2B4O7 = 1.7085 x 10’3 x 202 g
6. Phenolphthalein indicates 50% reaction of Na2CO3 = 0.3451 g in 0.7439 g borax
(upto NaHCO3) and 100% reaction of NaOH while (a) Per cent Na 2B4O7 =46.39%
methyl orange indicates 100% reaction of both.
(b) Na,>B4O7 • 10H2O = 1.7085 x 10"3 x 382 g
Let volume of 0.995 N HC1 required by Na2CO3 = x mL
Per cent Na^O, • 10H2O = 87.73%
and by NaOH = y mL
(c) B2O3 = 2 x moles of Na^O, i
with phenolphthalein
= 2 x 1.7085 x 10"3 x 70 g
— + y = 19.5 mL
2 = 0.2392 g
with methyl-orange Per centB2O3 = 32.15%
x + y = 25.9 mL (d) Na2B4O7 = 4B =4x 1.7085 xlO"3 mol
x = 12.8 mL = 4 x 1.7085 x 10"3 x 11g
y = 13.1 mL B = 0.0752 g
N^=N2V2 Per cent = 10.11%
Na2CO3 HC1 9. (a) 100 mL of serum has K+ = 18 mg
Nj x 25 =0.995x12.8 = 18 x IO-3 g
N1(Na2CO3 ) = 0.50944 N 18 x 10"3
mol
= 0.50944x53 39
18 x 10'3
= 27.0 gL-1 [K+] = '0.1
39
=N2V2
= 4.615 x IO"3 M
NaOH HC1
365 xlO-3 1
Ni x25 = 0.995x13.1 Similarly, [cr] = 'o.i
35.5
(NaOH) = 0.5214 N
= 0.103 M
= 0.5214x40
(b) pie=-log [K+] = 2.336
= 20.86 gL-1
Q Pr1- = - log (Crj = 0.988
7. A1 + 3HC1 A1C13+-H2
2 10. HC1 H+ + Cl- : end-point indicated by methyl-red
3 mol HC1 = 1 mol Al H3PO4 H+ + H^O^ : end-point indicated by
0.05x12 1TT„, 0.05x12 ,
----------- mol HC1 = —---------mol Al methyl-red
1000-------------- 1000x3
— H+ + HPO2- : end-point indicated by
= 2 x IO-4 mol Al bromothymol blue
= 2x10^ x27g Al Let HC1 = x millimol
2X10-4 x27 3 A1 H3PO4 = y millimol
=--------------- cm3 Al
2.7 then x + y (first ionisation step) = 35 x 0.1 = 3.5
= 2xl0-3 cm3 Al millimol
X
, mass y (second ionisation step) = 15 x 0.1 = 1.5 millimol
volume =------- -
density, HC1 = 3.5 -1.5 = 2.00 millimol
thickness x area = volume and HgPO4 = 1.5 millimol
— x area = 2 x 10’3 11. LetP2O5=xg= —mol= —molHgPO4
10 2 5 6 142 142 4
area =0.2 cm 2 (since one mole ofP9(X in aqueous solution forms 2 moles
8. Na;>B4O7 borax behaves like a weak base h3po4)
0.405-x
0.7439 g sample of borax =31.64 mL of 0.108 M HC1 HgPO4 = (0.405 - x) g = mol
98
Based on reaction
moles of Na2B4O7 _ 1 HgPO4 +2NaOH ---- > Na^PC^ +2H2O
41* ^Y
moles of HC1 2 NaOH required by P2O5 = mol = —
Chapter 7: Volumetric | 339

and NaOH required by HgPO4 NH4CI = 2.4 -1.1175


0.405 -xW0.405
1 -x = 1.2825 g
=2
98 49 = 0.024 mol
42.5x0.25 Total Cl = 0.039 mol
Actual NaOH required =
1000 = 1.3845 g
= 0.010625 mol % of Cl = x 100 = 57.69%
2x 0.405-x 2.4
--- + = 0.010625
71 49 14. S2- + Cu2+ CuS X
1 mol 1 mol
x =0.3040 g 63.5 g 95.5 g
H3PO4 = (0.4050 - 0.3040) = 0.1010 g 95.5 g of CuS is from = 63.5 g of Cu 2+
% HgPO4 =24.93% 63.5 x 0.0177
0.0177 g of CuS is from = g Cu2+
12. Na2CO3 + 2HC1 ---- > 2NaCl + H2O + CO2 95.5
K2CO3 + 2HC1 ---- > 2KC1 + H2O + CO2 = 0.01177 g
Cone, of mixture = 1.2 g/100 mL = 12 g/L = 1.8534x10^ mol Cu2+ in
Na2CO3=xgL-‘=^N 0.80 LH2O
[Cu2+]= 2.317x10“* M
12-x
K2ZCOO3 = N 15. Mn2+ + MnO4 + OH“ ---- > MnO2 + H2O
69 +2 +7 +4
^=1V2V2
(mix) x 20 =40x0.1
J 1
Normality of Mn2+ = molarity x 2
AT^O^N
x 12-x _ Normality of MnO4 = molarity x 3
--- + -- =0.2 Let molarity of Mn2+ =x
53 69
x =5.9625 g Molarity of MnO4 =0.05876 M
Na2CO3 = 5.9625 gL’1 = 0.59625 g/100 mL W=n2v2
K2CO3 = 6.0375 gL-1 = 0.60375 g/100 mL Mn2+ MnO~
Na2CO3+BaCl2 ---- > BaCO3 2x x 25 =34.77 x 0.05876 x 3
K2ZCO<52 + BaCLZ ---- > BaCO3 x = 0.1226 M
16. 24 mL of 0.01 M KMnO4 = 0.24 x 10"3 mol KMnO4
Na2CO3 in 20 mL solution = —------ x20g
3 1000 Redox reaction is:
5.9625x20 ,
=-------------- mol 2MnO74 + 5CZ2O42" + 16H+ ---- > 10COZ2+2Mn2+ + 8H2O
1000 x106
.-. C2O2- (pure) reacted = - x 0.24 x 10”3 mol
= 1.125 xlO-3 mol 2
BaCO3 formed = 1.125 xlO’3 mol = 0.60 x IO'3 mol
K2CO3 in 20 mL solution = - x20g Thus, oxalic acid (pure) = 0.60 x 10-3 x 90 g
3 1000
= 0.054 g in 1.0 g impure oxalic acid and
_ 6.0375x20
% purity = 5.4%
“ 1000 x138
17. Oxalate part of calcium oxalate is oxidised by KMnO4
= 8.75 x 10“* mol
„ , „ 24.2 x9.56 x 10"4
BaCOo formed = 8.75 x 10-4 mol Moles of MnO. used =---------------------
4 1000
Total BaCCK formed = 2 x 10"3 mol
V
= 2.31352 x 10‘5
= 2 x 10"3 x 197 = 0.394 g 5
.*. Moles of calcium oxalate = - x moles of KMnO.
13. ‘ KC1 + NH4C1 KC1X+ NH4C1? 2 4
sublimates
(Refer Q. 16 above) =5.7838 x IO-5 mol
Only residue left is KC1 (dissolved in 250 mL solution)
[Ca2+1 = 5.7838 x IO’5 mol
25 mL of KC1 s 15 mL of 0.1 N AgNO3
15x0.1 = 5.7838 x IO"5 x 40 g
2V(KC1) = = 0.06 N
25 = 5.7838 x 10-5 x40 x 1000 mg in
KC1 in mixture = 0.06 x 74.5 = 4.47 gL-1 10 mL blood
= 1.1175 gin 250 mL = 2.314 mg in 10 mL blood
= 0.015 mol = 0.2314 mg per mL blood
340 | Essential Inorganic Chemistry

18. Let molar mass of the reductant -mg mol 1 21. Let X0=wg= —- ---- mol
(X + 16)
Molarity = mol L"1 2.18 — w .
m and X2O3 = (2.18-w)g = ------------mol
0.01 M MnO~4 =0.05 N MnO74 (2X 4- 48)
25 mL of A\ (reductant) = 20 x 0.05 N MnO; 5w
Number of moles of Cr2O2 required by XO =
.... t . 20x0.05 6(X + 16)
•• N, (reductant) =-----------
25 Number of moles of Cr2O2 required by X2O3
= 0.04 N _ 4(2.18 — w)
Change in oxidation state of reductant = 2 units ~ 3(2X + 48)
Molarity (reductant) = 0.02 M w
Number of moles of XO4 from XO =
0.02 = 1“ (X + 16)
m
2(2.18-w)
m = 126 g mol"1 Number of moles of XO4 from X2O3 =
(2X + 48)
19. O3 4- 61" 4- 3H2O ---- > 3I2 4- 6OH“
5w 4(2.18 -w) ....
4-------------- =0.015
Eq. wt. of O3 = — = — = 8 6(X 4-16) 3(2X 4- 48)
3 6 6
w + 2<2-18_-^=q.O187
I2 + 2S,O2" ---- > S4O2" 4- 21" and
(X 4-16) (2X 4- 48)
Equivalent of S2O3" = Equivalent of I2 = Equivalent of
2.18 — w
°3 ---------- =0.0018
_ 1.5x0.01 2X4-48
= 1.5 x 10“5
1000 On solving w = 1.7372 g and X = 99
1.5 x IO’6
—------- 24
mol ***
22. 24 g K2CrO4 = ^ mol1 = r\0.1237
ir\nri__
mol
6 194
1X10 Q
Air taken = = ----------------mol N^ used = 0.1237 x
RT 0.0821x300
= 0.406 mol = 0.0928 mol
1.5 x 10’5 x 100 = 0.0928 x 32 g
Mol % of O, in air =
6x0.406 = 2.969 g
4.23 x 0.07654
= 6.16x10"*% 23. Moles ofK2Cr2O7 = = 3.24x10"* mol
1000
20. 2M 4- nHgSC^---- > Af2(SO4)n 4- nH2 3 i
2 x 51 g n. x 22400 mL at STP /. CH3CH2OH oxidised = 3.24 x 10"* x— mol
0.1g 2
43.9 mL at STP
= 4.86x10"* mol
2x51 n x 22400
= 4.86 x 10"* x 46 g
0.1 43.9
= 0.0223 g in 10 g sample of blood
n=2
= 0.223% above 0.1% prescribed limit
M is oxidised to M2+ by
Hence, police will prosecute the person.
M2+ 4-MnO44-H+ ---- > Afx+ + Mn2+ 24. 2MnO4 4- 5H2O2 4- 6H+ ---- > 5O2 4- 2Mn2+ 4- 8H2O
+2 4-7 (x > 2) 4-2
+x x
Pure H922
O9 = 1Q0 g 6
= equivalent
Also 0.1 N MnO4 = 0.02 M MnO4 1700
Nx
MnO4 used = equivalent
Change in oxidation number of M 2+ = (x-2) 1000
Change in oxidation number of MnO4 = 5 Nx _ x
1000 1700
.-. 5M2+ 4- (x - 2) MnO4 ---- > 5MX+ + (x - 2) Mn2+
N(MnO;) = ^
moles of Af2* 5 0-1/51
moles of MnO4 x-2 58.8 x 0.02/1000 25. 8CrO2"4-3S2O2 ---- > 6SO2" + 8Cr(OH);
+6 44 12 +3
x=3
[58.8 mL of 0.1 N MnO; =58.8 mL of 0.02 M MnO; 1
oxidation |
reduction
58.8x0.02
mol]
1000 0.154 M CrO2" = 0.154 x 3 N CrOj"
Chapter 7 : Volumetric | 341

0.246 M S2O2- = 0.246 x 8 N S2O2" H2O2 = - x 2 x 10 3 mol pure


2
= N2V2 = 5 x 10-3 mol pure
7x0.154x3 =0.246x8x40 = 0.17 g pure H2O2 in 0.2 g sample
V = 170.4 mL
% of pure H2O2 = 85%
Note: Oxidation number of two S-atoms in S2O3" = 4 and
(b) HA ---- > O2
that of in SO4" = 12
O2 formed from 0.17 g pure HA
26. K,Cr2O7 is required by oxidation of FeSO4 only 32
= ^x0.17g=0.16g
Fe2(SO4)3 is already in oxidised state. However, metallic
zinc reduces Fe2(SO4)3 to FeSO4 and thus, 0.16 .
=----- mol
22.45x0.1 32
N(FeSO4 before reduction) =
25 = 5.0xl0-3 mol
= 0.0898 N pV = nRT
nznnA n , X 34.25x0.1 _ nRT _ 5.0 x 10-3 x 0.0821 x 300
N(FeSO4 after reduction) - --------------
Zu "~~p~ " (750/760)
= 0.137 N
= 0.1248 L = 124.8 mL
:.N [total FeSO4 and Fe2(SO4)3] = 0.137 N 30. Let Na2C2O4 = x g = -|- mol = -|- mol C2O2"
FeSO4 in mixture = 0.0898 x 152
= 13.6496 gL-1 KHC2O4 HAO. = y g = J- mol = mol C2O’‘
Zlo Zlo
AT(Fe2(SO4)3 = 0.137 - 0.0898
= 0.0472 N and mol H+
218
Fe2(SO4 )3 in mixture = 9.44 gL-1 x t 2y'
Total C2O^" = mol
(Eq. wt. ofFeSO4 = 152 g equiv-1, .134 218.
Fe2(SO4)3 = 200 g equiv-1) Total H+ =-^ mol
27. An+ + MnO: ---- > Mn2+ + AO3 218
O.N. = n 4.7 +2 +5 If volume of the solution is V litres, then
+5e~ | x | 2y
M(C2<^-) = V
.134 218/
(5 - n) e~____
5An+ + (5-n)MnO4 ---- > (5 - n) Mn2+ + 5AO-
f—+ —
MC2O2") = 2 1134 218 V
5 mol of An+ 2.68 x 10~3
N(H+) = ^/v
(5 - n) mol of MnO4’ " 1.61 x IO"3 218/
5 - n =3 C2O^" is a reducing agent and H+ is an acid.
n=2 Given, MC2Oj") = 3.62 N (H+)
Chloride is AC12
molar mass —+ —
2fU34 ]
2187 _ 3.62 x3y /
.'. Equivalent wt. of AC12 =
change in O.N. V 218 7
97+71 x , 2y = 3.62 x 1.5y
= 56
3 134 218 218
28. 40 mL of 0.05 M MnO; =40mLof0.05 x5 NMnO4 — =2.1:1
y
= 10 milliequivalent
31. 6Fe2+ + Cr2O2" + 14H+ ---- > 6Fe3+ + 2Cr3+ + 7H2O
= 0.01 equivalent
5.442 .
C2O2" = 0.01 equivalent 1000 mL contains = 5.442 g K2Cr2O7 = -------mol
294
CaO = 0.01 equivalent
= 0.01851 molK2Cr2O7
= 0.005 mol = 0.01851 x 6 mol Fe2+
= 0.005 x 56 g= 0.28 g
_ 0.01851x6
in 0.518 g sample of limestone mol Fe3O4
" 3
% of CaO = 54.05% = 0.03702 mol Fe3O4
29. (a) 2MnO4 + 5H2O2 ---- > 5O2 + 2Mn2+
: = 0.03702 x 232 gFe3O4
Moles of KMnO, = = 2 x IO"3 mol Fe3O4 per mL = 8.59 x 10"3 g
4 158
342 | Essential Inorganic Chemistry

32. MnO2 + 4HC1 ---- > MnCl2 + 2H2O + Cl2 34. CaOCl2+H2O ---- > Ca(OH)2+Cl,2
ci2 + 2KI ---- > 2KC1 +12 Cl2 + 2KI --- > 2KC1 +12
2Na2S2O3 +12 ---- > 2NaI + Na2S4O6 I2 + 2Na2S2O3 = Na2S4O6 + 2NaI
2Na2S2O3 = I2 = Cl2 = MnO2 CaOCl2 =C12 sl2 =2Na2S2O3
30 xO 1 71
Moles of Na.>S2Oo used =------- — = 3 x 10"4 mol .'. Equivalent mass of Cl2 = — = 35.5
3 1000 2
30 x 10^
Moles of pure MnO2 = mol = n2v2
!2 Cl 2 in bleaching hypo
_ 30 x 10"4 powder
x 87 g pure MnO2
2 x20 = 0.1x20
= 0.1305 g in 0.5 g impure MnO2 AT^O.IN
% purity = x 100 = 26.1% CaOCl2=0.1N
0.5
Cl2 =0.1 x 35.5 gL-1
33. 10 mL of Ni MnO; = 50 mL of 1.0 N I"
= 3.55 gL"1 (pure)
10x2^=50x1
N,=5N Impure sample = —-^ = 10gL"1
-- 0.5 L
5 x 100 „
Equivalent of KMnO4 = ---------=0.5
1000 %ofpureCl2=^y xl00
Equivalents of oxalic acid = 0.5
Mass of oxalic acid = 0.5 x 45 g = 22.5 g = 35.5%

Exercise 2
Only One Option Correct 4. In the disproportionation reaction
1. 100 mL of 0.40 M KC1 = 0.04 mol KC1 net equivalent weight = (Eq. wt. )0 + (Eq. wt.
(Eq.wt.)0=^
100 mL of 0.50 M KOI = 0.05 mol KOI -Br2 ---- > BrO3
2 0 +5
Thus, KOI required = 0.05 - 0.04
= 0.01 mol -Br2 ---- > Br" (Eq. wt.)R=y
= 0.01 x 74.5 g 2 o -1
= 0.745 g KOI Net = —+ —= 0.6M
10 2
Hence, (a)
Hence, (c)
2. HjPO4 = 10.75 g in 550 mL
5. (c)
10.75 .. _____ 6. 20.00 mL of 0.1 MPb.2+ 1 = 2xl0"3 molPb2+
------ mol m 0.550 L
98 30.00 mL of 0.1 M SO2" = 3 x IO"3 mol SO2"
[H3PO4] = ----------- mol L 1 = 0.20 M Pb2+ + SO2" ---- > PbSO4 •
0.550 Pb2+ is in the limiting a quantity and every 1 mol ofPb2
But it is 0.40 N gives equal mole of PbSO4 hence, PbSO4 formed
Hence, change in oxidation number = 2 units = 0.002 mol.
or H+ neutralised = 2 Hence, (a)
In (a) HgPO4 changes to HPO4" 7. NaCl b Na+ + Ci­
Hence, (a) Na2SO4 st 2Na+ +SO2"
3. HjjS ---- > SO2 + M2V2
(Na+]raix =
T T
-2 +4 0.12x30 + 0.30x70
Change in oxidation number = 6 unit 30 + 70
1 mol HjjS = 6 equivalents BLjS = 0.246
Hence, (c) Hence, (d)
Chapter 7: Volumetric | 343

8. MjV/HCl, acid) = 16 H2O2 * ^2


13. (I) > O.N. change = 5
M2V2(KOH, base) = 18 > V^acid) MnO; > Mn2*
Mixture is basic. H2O2 > O2
(II) }O.N. change =3
[KOH]lmix MnO4 > MnO2
Vx + V2
Ini: (acidic) Nft = N2V2
18-16 _ „
=-------- =0.01M H2O2 MnO4
200
2M1 x 100 =0.5x100
KOH + HC1 ---- » KC1 + H2O
Mx (H2O2) = 2.5M
KC1 is from HC1 and 80 mL of 0.2 KC1 is diluted to 200 mL
In II: (Basic) = N2' V2'
80 x 0.2 = M(KC1) x 200
M(KCl) = 0.08M 2M\ xl00 = 3 xM2xV2'
Thus, resultant mixture has KOH (0.01 M) and KC1 2x2.5x100 = 3 xlxV2'
(0.08 M) Tr , 500 -
V9 =-----mL
Hence, (c) 2 3
9. KT and CP remain unchanged but due to dilution (in Hence, (b)
equal volumes), molar concentration of each is halved. ... _ . ___ xY
14. Normality of ux” volume H2O2 = —
[Cl-] =0.1 M " “ 5.6
[K+] = 0.2M 10 x (H2O2) = 10 x — (MnO;)
0.56
Also, [OH"1 > [H+1, mixture is basic
rrtTT_, 0.4-0.2 AT = — = —
u OH'U Jmix =---------
2 = 0-1 M 1 0.56 5.6
x=10
Hence, (c)
Hence, (d)
10. 5HCZ2O74 + 2MnOT4 ---- > 2Mn2+ + 10COz2
15. MCI„ + 3H2O M(OH)_ + nHCl
100 mL of 0.02 M MnO; = 0.002 mol MnO4 1 x 10“3 n x IO-3
= 0.005 mol of HC2O4 = x 30 mLofO.lM Ag+ =3 xlO'3 molAg*
100 mL of 0.05 M Ca(OH)2 = n x 10-3
_ 100x0.05 .-. n = 3 thus, trivalent chloride.
molCa(OH)2
1000 Hence, (a)
= 0.005 mol Ca(OH)<'2 1000 M
16. Use relation, M' (molality) =
= 0.01 mol'H+ (1000 d-Mm)
= 0.01 mol HC2O4 = y Hence, (c)
x 0.005 _ 1 17. Temporary hardness is due to HCO3 ofCa2+ and Mg 2+
y 0.01 “2
Ca(HCO3)2 + CaO ---- > 2CaCO3 + H2O
y = 2x 56 g 200 g
Hence, (b) 0.56 g 2g
11. 100 mL of XM H,O2 = 0.1 A mol HL2O2
Z Z 10 L (= 104 g) hard water contains = 2 g CaCO3
2H2O2 106 g (ppm) hard water contains = 200 g CaCO3
2H2O + O2
2 mol 1 mol = 24 L Hence, (b)
1
24 L O2 = 1 mol O2 18. 1 equivalent of O2 = 8 g =-mol
4
3 L O2 = i mol O2 = | mol H2O2
Hence, (a)
19. SO3" is oxidised to SO2- (change in O.N. = 2)
0.1X = -
4 25 mL of 0.1 M SO3' = 2.5 millimoles
X=2.5 = 5.0 milliequivalents of SO2-
Hence, (a) = 5.0 milliequivalents of M3+
12. MnO’ + 5Fe2+ ---- > 5Fe3+ + MnO; 50 mL of 0.1 M M3+ =5 millimol (given)
Thus, decrease in O.N. of M3+ should be 1
Cr2O2' + 6Fe2+ ---- > 6Fe3+ + 2Cr3+ So that 5 millimoles = 5 milliequivalents
Cr2O2" oxidises moreFe2+ Thus, new O.N. of M+ =2
Hence, (b) Hence, (c)
344 | Essential Inorganic Chemistry

20. Methyl orange indicates 100% reaction of Na2CO3 o.im = o.2NHC2o;


Na2CO33 +2HCl ---- > 2NaCl + H2O + CO.2
10 x0.2 = 10 X N'(MnOp
20x = lOx 0.1+5x 0.4 = 3 2V' = 0.2N
x=0.15M MnO4 is reduced to Mn2+
Thus, (c)
Hence, M(MnO4)x5 = N'
21. Molality =9
M(MnO4) = 0.04 M
9 mol HL>SO4 (= 9 x 98 g H.>SO4) is in 1000 g H,0 Hence, (c)
Total mass of 9 molal H2SO4 solution = 1000 + 98 x 9 28. With phenolphthalein, 50% reaction of Na2CO3 is on?
= 1882 g indicated. With methyl orange, next 50% reaction c-
1882 g solution has H2SO4 = 882 g CO, and 100% reaction of NaHCOu3 is indicated.
Na9Zu
1000g(= 1 kg) solution has H2SO4 = 469 g = volume of HC1 for 100% reaction of Na2CO3
Hence, (b) V2 - volume of HC1 for 100% reaction of NaHC03
22. As in (16)
1000x1 2
M =-------------------------------- = 1
[1000xl.0585-58.5xl] V
v2 + -^=y
Hence, (b) 2 2
23. HjjPOg is monobasic acid V2=(y-x)
H3PO2 H+ + HgPOj Hence, (d)
Hence, (a) 29. Fe2(C2O4)3 2Fe3+ + 3C2O4"
1 mol 3 mol
24. pH = 2 .-. [H+] = 10"2M
10“2 Only C2O4- is oxidised.
[Ca2+ ] = — M = — mol/L 2MnO4+5C2O2- ---- > 10CO2+2Mn2+
2 2
IO'2 5 mol C2O2- = 2 mol MnO4
= ^y-X40g/L
3 mol C2O4“ = 1.2 mol MnO4
IO"2 40 x 10G FeC2O4 = Fe2+ + C2O2-
= X------ q--- PPm 5Fe2+ + MnO4 ---- > 5Fe3+ + Mn 2+
2 103
= 200 ppm Ca2+ 5C2O4 + 2MnO7 ---- > 10CO2 + 2Mn2+
Hence, (a) 5 mol FeC2O4 = 3 mol MnO4
3
25. BaF, + H2SO4 BaSO4+H2F2+ H2SO4 1 mol FeC2O4 = - mol MnO4
1 mol 2 mol
unreacted
1 molFe9(C9O4L = - mol MnO4
= 2OH“ = 1 mol Ca(OH)2 = 2 mol KOH
H2SO4 = 2OH~ = 1 mol Ca(OH)2 = 2 mol KOH Thus, ratio of MnO4 required by 1 mol ofFe2(C204)3ani
2molCa(OH)2 =4 mol KOH FeC2O4 =2:1
Hence, (b) Hence, (a)
30. Naz2COu2 and NaHCO, are in 1:1 ratio with
26. NagSCXj remains unreacted O

phenolphthalein 50% reaction of Na2CO3 up to NaHCO,


^1=^2
is indicated and with methyl orange complete reactioncf
Nx x 100 = (2x0.5) x 10 Na2CO3 and NaHCO3 is indicated.
(NaOH) = 0.1 40 mL of 0.05 M Na2CO3 • NaHCO3 -2H2O
100 mL of 0.1 N NaOH = 0.01 mol NaOH = 2 millimoles
= 0.01 x 40 g = 0.4 g Na2CO3 = 1 millimol
Hence, (b) NaHCCL□ = 1 millimol
27. 10 mL of x M NaHC2O4 = 10 mL of 0.1 M NaOH x x 0.05 millimol HC1 = 0.5 millimol of Na2CO3 when 50^
H+ ion is neutralised of Na2CO3 (1 millimol) is neutralise!
x M = xN y x 0.05 millimol HC1 = 2
x = 10
10 xx = 10x0.1
y = 40
x=0.1M
(y-x) = 30mL
with KMnO4, C2O2- is oxidised Thus, (b)
Chapter 7:Volumetric | 345

31. MnO4 oxidises only FeSO4 = 5xl0'5 x40gCa2+


= 0.002 g Ca2+
MnO; + 5Fe2+
2+ ---- > 5Fe3+ + Mn2+
1 mol MnO4 oxidises = 5 molFe2+ (FeSO4) Hence, (d)
100x2 40. Af,v; = m2v2
mol MnO4 oxidises = 1 mol FeSO4 in 3 mol mixture
1000 1000x0.9
^i =
18~
mole fraction of FeSO4 = —
4 3
= 50 mL
Hence, (a)
Hence, (a)
32. (b) 33. (d)
41. (d)
34. V mL of 1 N unused HC1 = 100 mL of 0.5 N NaOH
42. 100 g HgPO4 solution has = 85 g HjPO,4
V=50 mL -yy mL H3PO4 solution has = 85 g H3PO4 = || mol
Used lNHCl = 100mL
100 mL of 1 N HC1 =0.1 equivalent HC1 85/98 mol
Molarity = y ^- = 14.74 M
= 0.1 equivalent of Af2CO3 100
L
2M + 60 ^1.7x1000 /
Equivalent mass of Af2CO3 = = (M + 30)
2
' -^=0.1 Hence, (c)
+ M2V2
M + 30 43. M (mixture) =
V1 + V2
M =23
= 4.125 M
Hence, (a)
Hence, (a)
35. Proceed as in Q. No. 4. (d)
36. 44. H^ + 2KOH + 2H,0
2CuSO4+4KI ---- » Cu^+I2+2K2SO4 1 mol 2 mol 1 mol
I2 + 2Na2S2O3 ---- > 2NaI + Na2S4O6 0.1 mol 0.2 mol 0.1 mol in 300 mL
2CuSO4 =2Na2S2O3 [K^OJ = = 0.333 M
CuSO4 = Na2S2O3
100 mL of 1 M hypo =0.1 mol hypo Hence, (c)
“x” volume
= 0.1 mol pure CuSO4 45. Molarity =
11.2
Hence, 100% pure 11.2
Hence, (a) 11.2
37. Mg + H2SO4 — MgSO4+H2 Hence, (a)
24 g (98 g = 1 mol)
46. (a)
1.2 g Mg reacts with = 0.05 mol H2SO4
47. 0.116 gC4H4O4(A) = On-=0.001 mol
H2SO4 taken = 100 mL of 1MH2SO,4 116
= 0.1 mol = 0.001 x equivalent
H2SO4 left = 0.1-0.05 0.074 g Ca(OH)2 = = 0.001 mol
= 0.05 mol in 100 mL solution
[H2SO4] = 0.05x10 = 0.5 M = 0.002 equivalent
Hence, (a) 0.001X =0.002
38. 106 mL of water has = 200 g CaCO3 x=2
Hence, (b)
103 mL of water has = 0.200 g CaCO3
48. Let [NaHC2O4 ] = x M
= 0.002 molCaCO3
[CaCO3] = 0.002 M = x N as an acid when H+ is neutralised
Hence, (b) = 2x N as a reducing agent when C2O4“
is oxidised
39. 5C2O4"+2MnO4 ---- > 10CO2+2Mn2+
10 mL of 2x NaHC2O4 s 10 mL of 0.02 M MnO4
Also, lCa2+=lC2O2- = 10 mL of 0.1 N MnO4
20 mL of 0.001 M MnO; = 2 x 10“5 mol MnO; x=0.05M
= 5xl0'5 molC2O4~ = 0.05 N
= 5xl0~5 molCa2+
346 | Essential Inorganic Chemistry

Hence, 10 mL of 0.05 N NaHC2O4 is neutralised by 56. MnO; MnO2-


10 mL of 0.05 N of any strong base. +7 +6
Hence, (d) Change in O.N. = 1 unit
49. H2ZC2Z0.4 H+ + HC2O; : H+ +C2O*- Normality = Molarity
I II
12.64 g KMnO4 = 0.08 mol in 0.5 L
Up to stage I — monobasic acid
= 0.16 M
II — dibasic acid
= 0.16 N
Hence, (b)
Hence, (a)
50. 1 mol O2 =4 equivalents O2 = 30 L
57. Oxidation state of I in KIO3 = + 5
1 equivalent O2 = 7.5 L
Sin SO:i2"=+4
Hence, (a)
Sin SO;f = + 6
51. MnO2+4HCl MnCl2 + Cl2 + 2H2O
io; +so2-
0
---- > SOj’+ F
o
0.1 equivalent Cl2 = mol Cl2 +5 +4 +6 x<5
2 60 x 0.5 x 2
= 0.05 mol MnO2 Equivalent of SO2- = = 0.06
1000
= 4.35 g MnO2 in 10 g sample 2 14
Equivalent ofIO3 = (5 - x) x
Thus, 43.5% pure.
Hence, (d) = (5-x)0.01
52. Equivalent mass of O = 8 (5-x) 0.01 =0.06
Equivalent mass of metal = 16 5 -x =6
Equivalent mass of metal oxide = 24 x= —1
= 1.5 times of metal Hence, (d)
1.4^ _ 1.4 x 0.2 x 100
Hence, (b) 58. N% =
w 0~6
0.106 .
53. 0.106 g Na2CO3 = ------ mol = 46.67%
106
Hence, (a)
= 2 x 10“3 equivalent
40 N 59. 2NaOH+CO2 ---- > Na2CO3+H2O
H2o04 = Yqq" equivalents 2 mol i mol 1 mol
40 N - 11.2 millimol = 11.2 x IO-3 mol CO.'2
------=2xl0'3
1000 1 mol CO2 reacts with = 2 mol NaOH
N = 0.05 N H2SO4 .•. 11.2 x 10~3 mol CO2 reacts with = 0.0224 mol NaOH
Hence, (a) NaOH at start =0.15 mol
54. MnO; + 5Fe2+ ---- > 5Fe3+ + Mn2+ NaOH unreacted =0.150 -0.224
A/jV/MnO^) = SMjVjCMnO^) milliequivalent = 0.1276 in IL
M2V2(Fe2+) = Af2V2(Fe2+) milliequivalent [NaOH] = 0.1276 M
ATiyi(MnO;) = N2V2(Fe2+) Hence, (a)
60. [Na2CO3] = [CO2] = 0.0112 mol per L
5x0.02x7! =40x0.1
^ = 40 mL = 0.0112 M
Hence, (c) Hence, (b)
55. SO2-+Cr2O2" 2Cr3+ + so2- 61. Mass of iron-ore= 0.740 g

O.N. = 4 12 6 6 .-. Fe2O3 = 0.740 x = 0.1776 g = 1.11 x IO”3 mol


+ 6e"
Fe3+ = 2.22 x 10"3 mol
-2e"
2Fe3+ +Sn2+ ---- > 2Fe2+ +Sn4+
2V1V1(SO3") = N2V2(Cr2O2-) Sn2+ +2Hg2+ ---- > Sn4++Hg2+
2MX x20 =6x0.01x30 Fe_3+
Sn2+ required by Fe3+ = mol
= 0.045 M 2
Hence, (b) 2.22 x 10l“3
— = 1.11 x 10"3 mol
2
Chapter 7:Volumetric | 347

q 2+ . , 25 x 0.05 2NaOH + CO2 ---- > Na2CO3+H2O


Sn taken =----------- = 1.25 x 10“3 mol
1000 Thus, NaOH reacted = 0.001 x 2 = 0.002 mol
Sn2* unreacted =(1.25 x 10"3 -1.11 x 10"3) mol Na9C0q formed =0.001 mol
= 0.14 x 10‘3 mol NaOH unreacted = 0.02 - 0.002 = 0.018 mol
Hg2+ required for oxidising Sn2+ Thus, HC1 required by NaOH for phenolphthalein end
= 2 x [Sn2+ ] mol point
= 0.28 xlO-3 mol = 0.018 mol
Let volume of HgCl2 = V mL Equivalent of Na2CO3 = 0.002 equivalents
Vx0.05 .-.Moles of HC1 required by 0.002 equivalents for
= 0.28 x 10"3
1000 phenolphthalein end point = =0.001 (50% reaction
= 5.6 mL 2
is indicated upto NaHCO3)
Hence, (c)
.-. Total HC1 = (0.018 + 0.001) mol in 200 mL
62. Na2C2O4 ---- > 2Na+ + C2O2- • (0.018+0.001) x
N (HC1) =------------------ mol L
KHC2O4 ---- > K+ +H+ +C2O2" 0.2
Let Na2C2O4 = x mol = 0.095 M
KHC2O4 = y mol = 0.095 N
Then C2O2- = (x + y) mol = 2 (x + y) equivalents Hence, (c)
H+ = y mol = y equivalents
Let volume of 0.1 N base required = V mL One or More Than One Options Correct
Then 0,lxV=y 1. O2 + 4e~ ---- > 2O2-
1000 4e~ = 1 mol O2 = 0.25 mol = 8 g O2
V = 10000? Thus, (a) and (b) correct
o ©4
Volume of 0.1 N KMnO4 = 3 V 2. 2.84 gP4Ow =“ = 0 01 mol L"1 P„O10
0.1x3 V Zo4
= 2(x + y) Also, P4O10 + 6H2O---- > 4H3PO4
1000
0.1 x 3 xlOOOOy Thus, 0.01 M P4O10 = 0.01 x 4 M H.}PO4
= 2(x + y)
1000 = 0.04 M H.,PO4
3y =2x + 2y 1 L of 0.04 M H.jPO4 e- 1 L of 0.12 N H3PO4
y = 2x = 1 Lof 0.12 N NaOH
Na2C2O4 = 134 x g = 400 mL of 0.3 N NaOH (to NagPO4 stage)
KHC2O4 = y = 2x mol = 2x x 128 g = 256x g = 400 mL of 0.1 M (= 0.2 N) Ca(OH)2 (to CaHPO4)
~ 256X X 100 Thus, (b) and (c) are true
% of KHC,O
'2^44 =---------------
2 4 256x + 134x 3. KHC2O4 -H2C2O4 -2H20has
= 65.64% (i) three ionisable H+. If it is 0.2 N then it is
09
Hence, (a) =— M = 0.067 M
3
63. Since, 40.38 mL > 12.83 mL, the sample must contain
(ii) two C2O4" oxidised to CO2
NaOH and Na,CO,.
4 O

Hence, (b) 2C,O2- ---- > 4CO2 +4e‘


64. The volume of titrant used by Na 2CO3 in the second step Thus, 0.067 M = 0.067 x 4 = 0.267 N
= 12.83 mL. Thus, (a) and (c) are true.
Thus, volume of HC1 used by NaOH in first step 4. Ca2+ + jRH,4 ---- > CaR + 2H+
= 40.38-12.83 77.5 ppm= 77.5 g Ca2+ per 106 mL
= 27.55 mL = 77.5x 10-3 gL-1 Ca2+
27.55x0.1062x40x100 77.5 x 10"3 1t4„
% of NaOH = =------------- mol L Ca 2+ ion
1000x0.6234 40
77.5 x 10'3 x 2
= 18.77 mol L-1 H+ ion
Hence, (a) 40
200x0.100 [H*] = 3.875 x 10-3
65. Moles of NaOH initially = = 0.02 mol
1000 pH = - log (H+) = 2.4
C094 absorbed = 1 millimol = 0.001 mol Thus, (a) and (c) are true.
348 | Essential Inorganic Chemistry

10.78/98 = VX~1- mol KMnO4


5. 10.78 gH3PO4 is 0.550 L = = 0.2 M
0.550 5 1000
But it is 0.4 N hence 5V
(x + 2y) =
H3PO4 has been changed to HPO4“ (by neutralisation of 20000
2H+). Thus, (a) is true. x + 2y 5x3
= 7.5
When H3PO4 (O.N. of P= + 5) is reduced to HPO2"
(O.N. ofP = + 3)
- + 2 = 7.5
Change in O.N. = 2 units y
Thus, 0.2M = 0.4N — =5.5
Then, (c) is also true. Thus, (a) and (c) y
6. KHC2O4 K++H++C2O2“ Thus, x :y = 5.5 :1
Thus, (c)
H+ s 1 OH’s - Ca(OH)2
2 z 8. (a) C2O2’ 2CO2 + 2e
5C2O4" + 2MnO; ---- > 2Mn 2+ + 10CO2 Molecular weight
Equivalent weight =---------- ----------
5C2O4-=2MnO; Ci

o
lC2O2“^MnO; Thus, (a) is not true.
(b) H2C2O4 has2H+ ions while NaHC2O4 has 1H+ ion.
100x0.02
100 mL of 0.02 M MnO4 = mol MnO; Thus, (b) is not true
1000
(c) Equivalent of H2C2O4 s NaHC2O4
2 mol MnO; = 5 mol KHC2O4
Thus, (c) is true
0.002 mol MnO; =0.005 molKHC2O4
(d) 5C2O4“ = 2MnO;
= 128 x 0.005 gKHC2O4
Thus, (d) is also true.
=x
100x0.05 Thus, (c) and (d)
100 mL of 0.05 M Ca(OH)2 = molCa(OH)2
1000 9. 100 mL has = 1.44 g FeC2O4
= 0.005x2 mol (OH-) 1 44
= — molFeC204
= 0.01 molH+ 144 2 4
= 0.01 mol KHC2O4 = 0.01 mol FeC2O4 0.01 molFe2+
= 0.01 x 128 gKHC2O4 = y + 0.01 mol C20;
x 128 x 0.005 _1 5Fe2+ + MnO; ---- > 5Fe3+ + Mn2+
y 128x0.01 “2 5C2O2- + 2MnO; ---- > 10CQ2+2Mn2*
2.x =y 5FeC2O4 s 3MnO;
q
Thus, (b) 0.01 mol FeC,O4 = - x 0.01
2 4 5
7. Let (A) Na2C2O4 = x mol = x mol C2O4~
(B)KHC2O4 -H2C2O4 = y mol = 0.006 mol MnO;
= 3y = mol H+ and 2y mol C2O4“ Let volume of KMnO4 = V mL
VxO.Ol
Total oxalate = (x + 2y) = 0.006
1000
Let volume of KMnO4 or NaOH = V mL 0.006 x 1000
V= = 600 mL
H+ + OH’ ---- > H2O 0.01
5C2O2-+2MnO; ---- > 2Mn2+ + 10CO2 Thus, (b)
1H* = 1 OH"
10. MnO; ---- > MnO2 + MnO;
3 y mol H+ = 3 y mol OH" +6 +4 +7
V xO.l ._TT MnO*- + 2e" ---- > MnO2
3y = --------- mol NaOH
1000 MnO' ---- > MnO; + e"]x2
V
y= 3MnO2’ ---- > 2MnO; + MnO2
30000
2 1
5C2O4"=2MnO; MnO2" ---- > - Mn07 + - MnO2
3 4 3 2
2(x + 2y)
(x + 2y) mol C20j" = mol MnO; Thus, (a)
5
Chapter 7: Volumetric | 349

Brain Twisters 6. 2AgNO3 + Na2CO.3 ‘ Ag2CO3 +2NaNO3


2 mol 1 mol 1 mol
1. CaCOoo + 2HC1 ----- > CaCl2Z +CO,l. +H2O
1 mol 2 mol 25.0 mol of 50.0 mol of
1 mol
2.56 , 250 x 0.125 mol 0.2 M 0.08 M
loo mo1 1000 0.005 mol 0.004 mol
0.0256 mol 0.03125 mol will react with : will react
CaCO3 HC1 0.0025 mol of : with 0.008 mol
reacts with reacts with Na2CO3 : ofAgNO3
0.0512 mol 0.015625 mol Thus, AgNO3 is limiting reactant
ofHCI ofCaCO3
Thus, Ag2CO3 formed = - mol of AgNO3
Thus, HC1 is limiting reactant and CaCO3 is in excess. 2
CaCO3 left unreacted =0.0256-0.0156 = 0.0025 mol
= 0.01 mol = 0.0025x276= 0.69 g
= 1.0g 7. In the above case Na2CO3 left unreacted
CaCl2 formed = moles of CaCO3 reacted = 0.004-0.0025
= 0.015625 mol = 1.7344 g = 0.0015 mol in total 75 mL
2. (a) Oxidising agent: O2, oxidises Au = (0.075 L)
Reducing agent: Au, reduces O2 0.0015 mol
Thus, molar concentration of Na2CO3 =
(b) Gold = 1000 kg impure 0.075 L
0 0197
= 1000 x 1000 x —------ g pure = 0.02 M
100 [Cu2+] =4.78 x 10’2
8. (a)
= 197g
= 1 mol Pcu 2+=-log [Cu2+]
= - log 4.78 x IO’2
Moles of NaCN = V (litres required) x M
= 2 - log 4.78
= 2-0.6794
4 mol Au require = 8 mol NaCN
= 1.3206
3. 1 mol Au requires = 2 mol NaCN
[Zn2+]= 0.104 = 1.04 x 10’1
0.075 V =2 PZn2+ =-log[Zn2*]
V= 26.67 L = - log 1.04 x 10’1
500 mL of 0.25 mol Na2SO4 = 0.125 mol
= 1 - log 1.04
BaCI2= 15 g =0.0721 mol
= 1-0.0170
BaCl2 + Na2SO4 ----- > BaSO44 + 2NaCl
= 0.9830
Thus, BaCl2 is limiting reactant [NO’ ] = 2 x 4.78 x 10’2 [from Cu(NO3 )2]
BaCl2 = BaSO4 = 0.0721 mol
+ 2 x 0.104 [from Zn(NO3)2]
4. NaCl and HC1 are strong electrolytes thus, ionise = 2.1756 xlO’1
completely in aqueous solution.
k”3 pN05 =-iog[NO;]
PNn+ = " log 2 x 10
= 3 - log 2 = — log (2.1756 xlO’1)
= 3 -0.3010 = 2.6990 = 1-log 2.1756
= - log 5.4 x 10"4 = 1-0.3376
= 4 - log 5.4 = 0.6624
= 4-0.7324 (b) [H+] = 6.75 xlO-4
= 3.2676
= -log [H+]
Total [CP] = 2 x 10’3 + 5.4 x IO-4
= 2.54xl0"3 M =-log 6.75 x IO"4
= 3.1707
= 2.595
[Ba2+] = 3.35 x IO-4
4 ++ =6.6990
5. pAg
2+ = - log 3.35 x 10
ba
[Ag+] = IO-66990 = 10 IO0'3010
= 2x10r7 = 3.4750
350 | Essential Inorganic Chemistry

[CIO; ] = 2 x 3.35 x 10'4 from Ba(C104 )2 12. A12(SO4)3+3Ba(NO3)2 ---- > 3BaSO4 + 2A1(NO3)3
+ 6.75 x IO-4 fromHClO4 1 mol 3 mol 3 mol
= 13.45 x 10"4 M Ba(NO3)2 = 480.4 ppm (parts per million)
^cio; = — log 13.45 x 10"4 = 480.4 g in 106 mL
480.4 x 750
= 2.8713 g in 750 mL
106
9. (a) Na+ = 1.08 x 10"r®” ppm = 0.3603 g
Thus, 106 g of sea water has = 1.08 x 103 g Na+ 0.3603
1q6 261
----- mL of sea water has = 1.08 x 103 g Na+
1.02 = 1.3804 x 10'3 mol
103 mL of sea water has A12(SO4)3 = 200 x 0.03090 millimol
_ 1.08 x 10>33 x 1.02 xlO3 = 6.18 millimol
g Na+
106 = 6.18 x 10’3 mol
= 1.1016 gL’1 A12(SO4)3
= 4.8 xlO’2 M Ba(NO3)2 = — based on stoichiometry
c- -i i rcn2-i 270 x 1.02 xlOi3: 3
Similarly [SO4 ] =-------------------- = 0.2754 gL 6.18 xlO-3
= 4.5
= 2.87x IO-3 M “ 1.3804 x 10’3

(b) = 1.32 Thus, Ba(NO3)2 is limiting reactant and A12(SO4)3 is in


^Na+
excess.
pso2- -2‘54 (a) Thus BaSO4 formed = 1.3804 x 10"3 mol
= 1.3804 x 10"3 x 233 g
10. KC1- MgCl2 -6H2O = 5.76 g in 2 L solution = 0.3216 g
5.76 .. T . . 1,3804 x IO-3
=------ mol in 2 L solution (b) Unreacted A12(SO3)3 = 6.18 x 10”3
277.5 3
= 0.01 M = 5.72xl0’3 mol in 950 mL
(a) [KCl-MgCl2] = 0.01 M = 6.02 x IO"3 M
(b) [Mg2+] = 0.01M 13. Let unreacted AgNO3 be = V mL
[Cr] = 0.03 M
V mL of 0.082 M AgNO3 = 4.86 mL of 0.0625 M KSCN
(c) Millimoles of Cl" in 25.0 mL solution v_ 4.86x0.0625
= 0.03 x 25 =0.75 0.082
(d) [K+]=0.01M = 3.70 mL
= 0.39 gL’1 used AgNO3 = (50.0 - 37.0) mL
0.39 1 „6 .n. = 46.30 mL
=---- - x 10 g per million
1000 46.30x0.082
Moles of AgNO3 used =
= 390 ppm 1000
11. KI = 24.32 ppt = 3.8 x 10“3 mol
P2O5 ->► 2PO3’
= 24.32 g per thousand mL solution
24 32 2PO3’+6Ag+ ---- >‘ 2Ag3PO4
= —:— x 200 g in 200 mL solution 3 ft x 1 (TI"3 .
1000 Thus, P2O5 content = - — mol
6
= 4.864 g
= 6.33 x 10i*4 mol
= 0.038 mol _ 3.8 x 10r3
Volume of AgNO3 required = V mL — x 142 g
6
Molarity =0.01 M = 0.0898 g
0.01 M x V = 0.01 V millimol Thus, % of P2O5 in plant food = x
= 1 x IO-5 V mol
1 x IO’5 V =0.038 = 2.11%
0.038 14. 0.30 MCaCl2 =0.60 MCI’ (inCaCl2)
= 3800 mL
” 1 x 10"5 Volume of CaCl2 = V; mL
KC1 = 0.20 M = 0.20 MCI’
Chapter 7: Volumetric | 351

Volume of KC1 solution =400 mL NEV


w (amount) =
1000
Resultant molarity of Cl“ = 0.25 M
0.40 x98 x550
M,V. + M,V2
Resultant molarity = X 1__________ f>
1000x3
v1 + v2
0.60^ +0.20x400 = 7.187 g
0.25 = (b) HgPO, + 2e" ---- > HPO3" + H2O
Vx + 400
98
Vx = 57.14 mL Equivalent weight of H.jPC)4 = — = 49
15. 60 L of 0.50 NaOH = 60 x 0.5 mol NaOH if pure NEV
w (amount) =
= 30 mol NaOH if pure 1000
= 30 x 40 g NaOH if pure 0.40 x 49 x 550
30x40x100 1000
g NaOH if wet
88 = 10.78 g
= 1363.64 g
16. 50 mL of Na2CO3 = 56.3 mL of 0.102 N HC1 Passage Comprehension Questions
50 x Nx =56.3 x 0.102 Passage 1
N. =0.115 N Na ,CO,
1 f» M
Molar mass of Cr(NH3 )XC13 = 52 + 17x + 35.5 x 3
= 0.0575 MNa2CO3 = (158.5+ 17x)
a 100 mL of 0.0575 M Na9CO„ = 5.75 x 10^ mol
ft o NH3 in the complex is neutralised by HC1.
CaCl, + Na9C0, ---- > CaCO, +2NaCl 40 x 1.5 n
1 mol 1 mol Moles of HC1 required = =0.06
1000
5.75 x 10’3 mol 5.75 x IO-3 mol ... _ . 2.635
CaCO, formed =5.75 x 10"3 mol Moles of A =----------------
(158.5+17x)
= 5.75x10^ x 100 g
0.06 2.635
= 0.575 g Thus,
x (1585 + 17x)
17. (a) IfH3PO4 is completely neutralised then it behaves as a
0.40 M x=5.89 = 6
tribasic acid thus, 0.40 NH^ = H3PO4
3 1. Thus, x = 6, Thus (d)
, ... 0.40 .
Moles in one litre =----- mol 2. 1.5 M being dibasic, volume of H2SO4 required
3
will be = 20 mL
M1 • «« t 0.40x550
Moles in 550 mL =------------ Thus, (b)
3 x 1000
3. Thus, complex is (Cr(NH3 )6 )C13
0.40 x 550 x 98
Amount of = ■---------------------------- g Thus, (a)
3 x 1000
Passage 2
= 7.187 g
1. (c) 2. (a) 3. (d) 4. (b), (d)
(b) H3PO4 + 2e~ ---- > HPO3~ + H2O Passage 3
Due to reduction as a result of gain of two electrons. 20x1
A. Resultant molarity of HC1 =
0.40 M 40
0.40NH3P04 =
2 due to dilution from 20 mL to 40 mL = 0.5 M
Moles of HgPO4 in one litre = 0.20 mol Thus, 40 mL of 0.5 M HC1 = VA mL of 0.1 M NaOH
t 0.20x550 VA=200mL
Moles in 550 mL =-
1000
0.20 x 550 x 98 B. Resultant molarity of HC1 due to dilution from 20 mLto
Amount of HgPO4 = -------------------- g _ T 20 x 1
1000 80 ml i =--------
80
= 10.78 g
= 0.25M
Alternate (short method)
Thus, 80 mL of 0.25 M HC1 s VB mL of 0.1 M
(a) HgPC^+SOH" ---- > 3H2O + PO4~
QQ
Vfl=200mL
Equivalent weight of HgPO4 = — 1. Thus, VA = VB, Thus, (a)
3
2. (b)
352 | Essential Inorganic Chemistry

3. At the end-point additional drop of NaOH, imparts pink Thus, 24.5 x0.6 = V x0.5
colour. V = 29.4 mL
Thus, (c) Thus, (a)
Passage 4 Passage 6
MnO~ -^+ Mn2+ Change in O.N. Volume ofKMnO.4 1. (c) 25 mL of lx 10r3 M MnO;
+7 +2 5 unit 20 mL = 25 mL of 5 x 10"3 N MnO4
| OH~ = 0.125 milliequivalent of MnO4
4 MnI+ (7 - x) unit V„ 33.4 mL
neutral „ v+ h0.125 milliequivalent of CaC2O4
-------- > Mny (7 - y)unit Vr 100 mL
s 0.125 milliequivalent of Ca2+
1 M MnO4 = 5 N in acidic medium = 0.125 x 20 mg of Ca2+ in 10 mL
= (7 — x) Nin basic medium = 0.250 mg/mL Ca2+ solution
= (7 - y)N in neutral medium 2. (a) Explained above
20 mL of 5 N MnO; = 33.4 mL of(7 - x) NMnO; ' 3. (a) 0.250 mg/mL = 0.250 g/L
100 n
----- = 7-x = 250 g/106 mL = 250 ppm
33.4
4. (c) In H2SO4, HC1, it is decomposed.
x =4
HCOOH is oxidised by KMnO4, hence cannot be used.
Also, 20 mL of 5 N MnO4 = 100 mL of (7 - y) N MnO4
5. (d)
(7-y) = l
y=6 Passage 7
2+
1. (a) 2. (c) 3. (b) 1. (c) Only Fe2+ is oxidised and MnO4 is reduced to Mn
Passage 5 25 x M (Fe2+) = 25 x 0.02x 5 (MnO;)
6Fe2+ ---- > 6Fe3+ + 6e’ M = 0.10M
Cr2O2’ + 14H+ + 6e“ ---- > 2Cr3+ + 7H2O. 2+
2. (a) Total Fe in second part (including that of from
N(Cr202-) = 6xM(Cr202~) Fe3+)’
= 0.6 N 25 xy = 40 x0.02 x5
5Fe2+ --- > 5Fe3+ + 5e~
y = 0.16 M = 0.16 N
MnO4 + 8H+ + 5e“ ---- > Mn2+ + 4H2O
Fe3+ = 0.16 - 0.10 (Fe2+)= 0.06 M
MMnO4) = 5 xM(MnO4)
3. (b)
= 0.5 N
Volume of Fe2+ = V mL (say) 4. (c) 25 mL of 0.10 M Fe2+ = mL of 0.02 M K2Cr2O7
Molarity of Fe2+ = 1 M (say) = 1 N = Vxx 0.02x6 NK2Cr2O7
6Fe2+ = Cr2O2- Vx = 20.83 mL
,2+ _ Cr2O2 (Cr2O^“ is reduced to Cr3+)
IFe
6 Similarly for 0.16 (Fe2+ to Fe3+)
5Fe2+ =MnO; V2 =33.33 mL
MnO4 Passage 8
1 Fe2+ =
5 5.8 g NaIO3 =^| mol NaI03
In all cases 1. (0
1 equivalent of X = 1 eqivalent of Y NaHSO3 =^A® mol
Thus, (d) 3 198
2. V x 1 = 0.5 x Vx for MnOl4 _3x 5.8x 104
g = 9.14 g
“ 198
V1=2V
58
V x 1 = 0.6 x V2 for Cr2O^“ 2. (a) Informed in step I =—— mol
198
V2=1.33 V 5.8
IO3 required in step II = mol = 1.16 g (NalOj)
Thus, (b) 198x5
3. 24.5 of mL 0.1 M Cr2O?“ = 24.5 mL of 0.6 N Cr2O?“ volume=U^ - 02 L
5.8
V mL of 0.1 M MnO4 = V mL of 0.5 N MnO4
Chapter 7: Volumetric | 353

3. (c) IO;o II. One or More Correct


+5 1. (AMI,2); (BM1.2); (CM2,3); (DM4)
change in ON = 6 units (EM2.3)
2. (AM3); (BM2); (CM2); (DM4)
Thus, 1 mol NaIO3 = 6 equivalents
1 Ifi (EMD; (FM3)
4. (d) 1.16 g NaIO3 =— mol
198
Integer Answer Types
- 0.035 equivalent
198 Set I
Passage 9 Questions 1 _3_ _4_ 6 7 8
Answers 5 2 2 2 6 1
1. (c) Cry­ Cr3+
orange green © © ©_ © ©I© ©.
2. (a) © © © © © © © ©_
3. (d) CH3CH2OH---- > ch3cooh © © © © © © © ©_
© © © © © © @
change in oxidation number = 4 units
© © © © ©
4. (a) Number of moles of K2Cr2O7 required © © © © © © ©
©
=£2.3 x 0.07654 =3 24xlQ- © © © © © _©_©
1000 © © © © © _©._© _©
© © © © © © © I®
Number of moles of CH3CH2OH
= 3.24x 10-4 x 1.5 © © © © © ©I© ar
= 4.86 x IO-4 mol
Set II
= 0.0224 g CH3CH2OH in 10 g sample
Questions —► 1 2 | 4 ! 5 £ 7 8
= 0.224% which is greater than legal Answers —► 11 5 1 6 4 1
limit. 3 2
© © ©. © © ©I© ©
© © ©. © © ©I© ©
Assertion & Reason © © ©. © © © ©
1. (a) 2. (c) 3. (d) 4. (b) 5. (a)
© © ©. © ©T© ©
© © ©. ® © _©_© ©
True & False © © ©. © © ©I© ©
1. F 2. T 3. T 4. T 5. F 6. T 7. T © © © © _® ® ©
8. F 9. T 10. F 11. T 12. T 13. T 14. T © © _© © © ©I© ©
15. F © © ®j © © ©I® ©
I© © ©I © © ©I© ©
Fill in the Blanks
1. 31.6 2. K2MnO4 3. iodometric method Set HI
Questions —► 1 2 5 6 7_ 8
4. methyl red 5. disappearance of blue colour 2
Answers —► 6 2 1 3 2
6. 1 7. 0.2 8. MnO'22 9. 1.33 N 10. 0.015 M
11. 0.333 N, acidic 12. 2.8 13. iodometric method
© © $
(F © ©I© ©
© © © _© ©_ ©
14. I2, M/6 15. N2
© © © ©^ © ©I© ©
© © © © © © © ©
Matrix-Match Type Questions © © ® © © ©._© ©
I. Only One Correct © © © © _©_© ©
(CM2); (DM3); © © © © © © © ©
1. (AM(4); (BHD;
(DMD; © © © © © _©_© ©
2. (AM(3); (BM2); (CM4);
© © © © © © © ©
(CM4); (DM2);
3. (AM(3); (BM1);
(DMD I© © © © © ©I© ©
4. (AM(2); (BHD; (CM3);
There is nothing in the universe but alkali and acid, from
which nature composes all things. 0. Tachenius

s-Block Elements
·-· The Alkali Metals

Quick Points
• Effective nuclear charge Z * (due to shielding effect)
Z*=Z-S
where Z is atomic number and
S is the shielding constant • Introduction Alkali
Value of Z * is derived by using Slater's rule;
S = 0.25 for ls-electron when it is alone (only for He) • Periodicity at a Glance -
metals 1 Alkaline
earths
1H metals
= 0.35 per electron in n th orbit • Study of Group 1 (IA) 2

= 0.85 per electron in (n - l) orbit - The Alkali Metals 3LI 4 Be


th

= 1.00 per electron in (n -2), (n -3) ... orbit • Compounds of Sodium


11Na 12Mg
Thus, for Li (Z = 3, ls 2 1
2s ) • Compounds of Potassium
19K 20Ca
Z * = 3 - shielding constant of two electrons in ls • Diagonal Relationship
-suborbit (n - l)
th
orbit 37Rb 38Sr
• -Anomalous Behaviour
= 3 - 2 X 0.85 = 1.30
of Lithium 55CS 568a
l
jf.2s
• Additional Reactions and a1Fr aaRa

1
e );\
Facts of Alkali Metals 1 A
, This electron is shielded by
two electrons in ls orbital
For second period, Z * = 0.65 (n + 1)
where n is the number of valence electrons.
For Na (Z = 11, 1/-2s22p6 a/)
Electron in 3s-suborbit is shielded by 8 electrons in 2nd, i.e.,
(n -l)th orbit and 2 electrons in 1st, i.e., (n - 2) Lh orbit

...
Thus, S =8 x 0.25 +2 xl =8.8
Z * = 11 - 8.8 = 2.2
• Species with N-unpaired electrons is paramagnetic and
' magnetic moment = ✓N(N + 2) BM
356 I Essential Inorganic Chemistry

• K02 has 02 ion (superoxide ion) and based on sub-orbit--elements under this class are called s-block
molecular orbital theory, bond-order is 1.5 elements.
• Amphoteric species are acidic {soluble in base) as well General electronic configuration of alkali group 1 (IA)
as basic (soluble in acid) is [inert gas] ns 1 where n signifies group.
• Amphiprotic species is H+ ion donor as well as H+ ion Alkali metals form compounds with +1 oxidation state
acceptor in which stable nearest inert gas configuration is attained.
HC0 3 � H+ + c�­ Alkali metals EC of M + (of IA) M 2•
acid
HC03 + H+ � H2C0 3 1(1A) z EC Ion (of IIA)
base Li 3 [He] 2s 1 +
[He]
u
• Born-Haber cycle is based on Hess's law of constant Na 11 [Ne] 3s 1 Na + [Ne]
heat summation and is used to derive Lattice energy K 19 [Ar] 4s 1 K+ [Ar)
of the compound and also other unknown parameter.
Rb 37 [Kr] 5s 1 Rb + [Kr]
For the salt MX(s)
Cs 55 [Xe] 6s 1 cs + [Xe]
Fr 87 [Rn]7s 1 Fr + [Rnl
M(s) ----t M(g)
• Group IA elements are collectively known as alkali
M(g) -➔ M+(g) + e-
metals after the Arabic word al-quis meaning plant
� X2(g) ----➔ X(g) ashes which are rich in the carbonates of sodium and
potassium.
X(g) + e- ---- ➔ x-(g) q4
M +(g) + x·(g) ----➔ MX(s) -U (lattice energy)
M(s) + � X2(g) ----➔ MX(s) t:Jl; = Q Periodicity at a Glance
Detailed periodicity has been given in chapter 3.
�� �+�+�+�-U=Q Table 8.1 describes periodicity of s-block elements at a glance.
Thus, U is calculated. • There is uniform gradation in properties except in
case of Li and Be which being smallest in their groups
Introduction show abnormal behaviour. Li resembles Mg and Be
resembles Al that we call diagonal relationship.
Shells up to (n - 1) are completely filled and
differentiating (last) electron enters into ns

Table 8.1 Periodicity ins-Block Elements


A Atomic number 2
B Effective nuclear charge (Z *=Z -cr) Li Be A
where cr is screening constant B
C Number of shells Na Mg C
D Atomic radius D
E Ionisation energy K Ca E
F Hydration and hydration energy
F
G Size of hydrated ion
Rb Sr G
H Polarising power (based on Fajan's rule)

I Irr
H
I Lattice energy of chloride
J Reactivity Cs Ba
K Ionic conductance

• From the Table 8.1, it is concluded that due to higher Role of s-Block Ions in Biological Systems
(IE) of alkaline earth metals, they are less reactive • The sodium and potassium ions, taken together,
than alkali metals. constitute sodium pump which is also known as
'Na + - K+ pump'. Operation of this process is
biological in each and every cell of all animals in which
Na + ions from intracellular fluids are transferred to
Chapter 8: s-Block Elements — The Alkali Metals | 357

the extracellular fluids with the help of DEEP Focus


carrier-proteins'. Transfer of K+ ions is however, in • Ionic size increases in the order :
a reverse way and thus an electrical potential Li* < Na* < K* < Rb* < Cs*
gradient is set-up across the cell-membrane which is and size of hydrated ion increases in the order (smaller the
responsible for the nerve signals in animals. size, larger the charge, greater the hydration):
• Some NaCl is essential in the diet of humans and Cs* < Rb+ < K* < Na+ < Li*
other animals because many biological functions are hence ionic conductance is in order :
controlled by the concentration of Na+ and Cl" ions. Cs* > Rb* > K* > Na* > Li*
The fact that salt has long been recognised as
important is evident in surprising ways. For example, Table 8.2 Electronic Structure
we are paid a “salary” for workdone. This word is Element Symbol Z Electronic structure
derived from the Latin salarium which meant [He] 3s'
Lithium Li 3 1s2 2s1
salt-money” because Roman soliders were paid in salt.
Sodium Na 11 1s22s22p63s' [Ne] 3s'
K 19 1s22s22p63s23p64s’ [Ar] 4s'
Study of Group 1 (IA)—The Alkali Potassium
Rubidium Rb 37 1s22s22p63s23p64s2 [Kr]5s1
Metals 4p65s’
[Xe] 6s1
Caesium Cs 55 1s22s22p63s23p63d’°
The elements of group 1 (IA) illustrate, more clearly 4s24p6 4d'°5s25p6 6s1
than any other group of elements, the effects of increasing Francium____ Fr 87 .............
87 ..... ........6s 6p67s'______ [Rn] 7s'
6sz26p°7s'
the size of the atoms or ions on the physical and chemical
properties which are closely related to their electronic • All the alkali metals give characteristic colours in
structures (Table 8.2) and sizes. bunsen flame. On heating in bunsen flame, outermost
• The elements are all metals, excellent conductors of electrons get excited to higher energy levels (due to
very low (IE)V When these electrons return to the
electricity, and are typically soft and highly reactive. ground state absorbed energy is released in the form
• They have one loosely held valence electron (ns1) in of light in the visible region of the spectrum.
their outer shell, and thus form univalent, ionic and
Element Colour A/nm
colourless compounds. M+ has stable nearest noble
Li crimson red 670.8
gas configuration.
Na golden yellow 589.2
• The hydroxides and oxides are very strong bases and
K violet 766.8
the oxo salts are very stable. 780.0
Rb red violet
• Lithium, the first element in the group, shows 455.5
_______ Cs blue______
considerable difference from the rest of the group.
This is due to high charge density of the Li+ ion Colour emission by these elements is the basis of
compound compared to other alkali metal ions, analytical methods (like flame photometry, atomic
(charge density = ionic charge/ionic radius) absorption spectroscopy) for the precise estimation of
these metals.

Occurrence and Uses of Alkali Metals


Main mineral Uses
Element Abundance
First isolated in 1817 from petalite (LiAISi4O10) All-purpose automative grease,
Lithium 0.002%
named after the Greek word lithos meaning stone.
Main source is spodumene (LiAISi2O6)
2.36% sixth most abundant element in First prepared in 1807 from caustic soda, NaOH. Glass, rubber, pharmaceutical agents
Sodium
the earth's crust and the most abundant NaCl (halite or rock salt), use sodium or its salts in their
metal, after Al, Fe, Ca and Mg Na2CO3 • NaHCO3 • 2H2O. (trona) NaNO3
(trona) production.
(saltpeter) are main sources
2.09% eighth most abundant element in First prepared in 1807, name derived from potash As a plant fertilizer.
Potassium
the earth's crust and next most (K2CO3)
abundant metal after Na. KCI (sylvite), KCI ■ MgCI2 • 6H2O (carnallite) are
main sources.

Although Na and K are almost equally abundant in abundant as K in the oceans. This is partly because K salts
with the larger anions tends to be less soluble than the Na
the crusted rocks of the earth, Na is some 30 times as
358 | Essential Inorganic Chemistry

salts and like wise, K is more strongly bound to the absorption bands — one for the solvated electrons and the
complex silicates and aluminosilicates in soils (ion other for the M~ species called alkalide ion. This if
exchange in clays) supported by the following facts :
Also K leached from rocks is preferentially absorbed (i) The solutions are diamagnetic, as expected for
and used by plants whereas Na can proceed to sea. spin-paired M~ ions in the ground state.
(ii) The molar conductivity of solutions containing
The Alkalide Ions sodium decreases slowly as the concentration is
raised. This is due to the formation of ion pairs
The reaction 2Na(g) Na+(g) + Na"(g)
Na+ Na".
is endothermic by 438 kJ mol"1 which indicates -1 (iii) Absorption band also confirms the existence ofNa'
oxidation state nonexisting but under unusual conditions
this -1 state exists as in crown ethers (polycyclic ethers).
Solutions of sodium in ethylenediamine or Periodicity of Alkali Metals
methylamine show electrical conductance of normal 1 : 1 Table 8.3 summarises physical properties of the
electrolytes— possibly due to Na+ and Na". Alkali metals elements of group 1 (IA). Form this we conclude
also dissolve in certain ethers, tetrahydro furan and other - they have low (IE
solvents forming relatively unstable solutions. The optical - they have large negative electrode potentials
spectra of such solutions clearly indicate two distinct - they are a group of active metals.

Table 8.3 Properties of the Alkali Metals


Properties LI Na K Rb Cs
Effective nuclear charge 1.3 2.2 2.2 2.2 2.2
Metallic radius (pm) 123 157 203 216 235
Ionic radius (pm) 76 102 138 152 167
Ionic charge density 1.32 40.98 0.72 0.66 0.60
(ionic charge/ionic radius (A)
Radius of hydrated ions (pm) 340 276 232 228 220
(EN), electronegativity 1.0 0.9 0.8 0.8 0.7
Melting point (°C) 181 98 63 39 29
Boiling point (°C) 1331 890 766 701 688
Density (103 x kg m-3) 0.53 0.97 0.86 1.53 1.87
(IE)vM(g)-^ M + (g) + e~
or (kJ mol’1) 520 496 419 403 376
(IE)2, M*(g) -» Mz+(g)+ e' kJ mol'1 7297 4561 3069 2650 2420
Sublimation energy
M(s)-> (kJ mol’1) 155 109 90 86 79
Hydration energy
M +(g)----- » M +(aq), (kJ mol’1) -506 -397 -318 -289 -259
AH°b,+ AH^.+ AHbyd, (kJmol’1) 169 208 191 200 196
^(V),M7aq)+ e- ----- > M(s) -3.045 -2.714 -2.925 -2.925 -2.923
Abundance on Earth% 0.002 2.36 2.09 0.009 0.0001
Ionic mobility 33.5 43.5 64.5 67.5 68.0
Lattice energy (of MC\), SHy kJ mol’1 -845 -770 -703 -674 -644
AS°hyd,. of/Vf*. JK"1 mol’1 -140 -110 -70 -70 -60
AG°hydr ofM’.kJ mol’1 -485 -379 -308 -285 -362

1. Size group from one element to the next. The addition of the
Alkali metals are the largest in their corresponding extra shell of electrons overweighs the effect of increased
periods in the Periodic Table. The size of the atom or its ion nuclear charge and thus there is an increase in size from
increases on descending the group. This is due to the Li to Cs. The size is based on
addition of an extra shell of electrons as we move down the
Chapter 8: s-Block Elements — The Alkali Metals | 359

n2a0
rn = 4. Thermal and Electrical Conductivity
Z*
In alkali metals, electrons of the noble gas core
where n = number of shells efficiently shield the lone valence shell electron from the
a0 = Bohr’s radius nuclear charge. Thus, effective nuclear charge felt by the
Z * = Effective nuclear charge due to screening electron in the valence shell of an atom of an alkali metal
(Z * = 1.3 for Li and 2.2 for Na—Fr) is the least and the sole valence shell electron is very
Thus, atomic ionic size is mainly dependent on n. loosely held by the nucleus. It can move freely from one
metal to the other in the lattice, thereby the alkali metals
Cs are good conductors of heat and electricity. The loosely
150-- K bound electron is also responsible for the silvery
a lusture of the alkali metals when freshly cut.
Na
g ioo--
*6
s 5. Ionisation Energy, Electrode
,J
o
c
Potential and Hydration Energy
Low values of (IE), reflect the tendency of the element
0 —I------- 1------- 1------- 1- -I to oxidise — the electropositive character. Rb and Cs are
10 20 30 40 50 60 so highly electropositive that they emit electrons even
Atomic number when exposed to light, that is they exhibit photoelectric
Fig. 8.1 Trend in ionic radii of group one elements. effect.
• What is the significance of the large negative electrode
2. Density potentials of the alkali metals ? They mean that the
In a group, density increases on going down the group. metal M+ can be reduced to the free metal M only with
Since, the elemental structure are often the same within great difficulty. On the other hand, the reverse
any group, the factors which determine the density are process— oxidation of the metals occurs with great
atomic mass and volume. In this group, increase in atomic ease.
weight from one element to the next in passing down the 550 T
group overweighs the effect of increase in the size of the
atom. Density of potassium is however less than that of _ 500"
sodium. o
E 450"
2
3. Melting Point and Boiling Point O)
400"
These metals are soft and can be cut with a knife. As a a>
g 350-
result of increase in size and repulsion of the non-bonded
electrons, their cohesive energy and tendency for metallic | 300-
bonding decrease down the group and thus softness 'c
o 0- -I-------- 1-------- 1-------- I- -I
increases from Li to Cs. These metals have low melting
10 20 30 40 50 60
point and boiling point which also reflect the low values of
Atomic number
cohesive energy between the atoms.
Fig. 8.3 Trend in first ionisation of alkali metals.
500 t

450"

£400"
DEEP Focus
• If we use the magnitude of the first ionisation energy (IE\
S.350-- as an indicator of ease of oxidation, Li is the most difficult of
Q)
C the alkali metals to oxidise. On the other hand, if we use
? 300-- E°, (standard reduction electrode potential) as our
S
criterion, Li is the easiest to oxidise. How can we explain
250 ■■
this discrepancy? Actually these two criteria (IE). and E° ,
0 -I------- 1------- 1------- 1- -4 measure the same thing. (IE)Y measures the tendency for a
10 20 30 40 50 60 gaseous atom to ionise, measures the tendency for a
Atomic number metal atom to go from solid state to the ion in aqueous
Fig. 8.2 Trend in the melting point of group one elements. solution which involves:
360 | Essential Inorganic Chemistry

(1) subliming the solid metal to the gaseous state Gradation in the Properties of Alkali Metals
(2) ionising the gaseous metal atoms to gaseous metal
mai
ions
(3) hydrating the ions by dissolving M+(g) in water to Atomic radii m.p. andb.p.
Li
produce M+(aq) Atomic volume Hardness
Thus, AH°X = AH^ + AH2° + AH3° Density Na Ionisation energy
’O Reactivity K Conductivity
= AH°ublj + AH°nis + AH,hydr.
Reducing power Electronegativity
From Table 8.3, it is clear that they (AH°xdn) are Electropositivity Rb Solubility of salts
positive values, but that the value is smallest for Anion stabilisation Cs having large anions
I
Li (169 kJ). This is primarily because of the large negative Solubility of salts
AH for the hydration of Li+. Thus it appears that Li(s) having small anions i u
max
should be easiest to oxidise of the alkali metals. If
lithium takes part in reactions that do not involve
water, then it does show less reducing power than DEEP Focus
the other members of the group. ,„) are related to
• Standard electrode potentials (E°In + IM
In terms of lattice energy AH^, heat of solution is energy changes of the process of atomisation
(sublimation), ionisation and hydration; among the
= AH°hydra. - AHU°
AH° solution alkali metals these changes almost cancel one
another and E° for the different M+ IM systems is
M /M
In general, ifAHhydr0 > AH„,
I/’ or AHsolution
0, h > 0, the salt nearly constant. The lower reactivity of Li towards water
dissolves in water. is therefore kinetic rather than thermodynamic in origin:
as a harder and less easily melted metal, Li is less easily
ForNaCl AH...
hydra = -772 kJ mol dispersed and hence, reacts more slowly than the other
metals.
= -776 kJ mol-1 • The second ionisation energies, (IE)2, of all alkali metals are
so high that the formation of M2+ ions under chemically
Action = - 772 - (-776) = 4 kJ mol realisable conditions is not possible.
hence NaCl dissolves in water.
• All of the halides (MX, X = F, Cl, Br, I) have strongly
M*(g) + e~ negative standard enthalpy of formation, , but the
actual values show two different trends as is evident
(2) j>
from Table 8.4.
AW°hydr.
AW°ionis.
(3) M(aq) + e (a) XH° for fluorides becomes less negative as we
M(g) go down the group, while reverse is true for
AH°0X. chlorides, bromides and iodides.
sub.
(b) For a given metal, always becomes less

• With increase in atomic number, the atoms become negative as we go from the fluoride to iodide.
larger and the strength of the metallic bonding
Table 8.4 Standard Enthalpies of Formation of Alkali Metal
decreases. Halides
• Enthalpies of hydration (AH°ydr) and entropies of
&H°f (MX)/kJmol
hydration (ASbydr) both become less negative as ionic M
MF MCI MBr Ml
size increases, but the variation in free energy of -270
Li -617 -409 -351
hydration (AGbydr) follows fairly closely that in
Na -575 -411 -361 -288
enthalpy of hydration. K -568 -328
-436 -394
• Due to the fact that the due to the largest size of the Rb -557 -443 -393 -333
hydrated Li+ ion, it is least conducting in solution Cs -555 -348
-435 -405
order being Li+ < Na+ < K+ < Rb+ < Cs+.
Chapter 8: s-Block Elements — The Alkali Metals | 361

5, Lattice Energy and Solubility Difference in lattice energy between NaCl and KC1 is
As shown in Table 8.5, the variation in lattice energy 67 kJ mol-1, and yet the difference in AG(hydrallon) forNa+
(which is defined as the energy released when cations and andK+ is 71 kJ mol"1. Thus, KC1 is less soluble than NaCl.
anions in gaseous state combine to form one mole of ionic
crystals) for fluorides is greater than the variation in Table 8.5 Lattice Energies of Alkali Metal Halides
lattice energy for chlorides, bromides and iodides. This is
U (MX)/kJ moi-1
( 1 A M
because lattice energy is proportional to and to MF MCI MBr Ml
the charge on the ions hence, the variation in lattice Li -1035 -845 -800 -740

energy is the greatest when r_ is smallest and least when r_ Na -908 -770 -736 -690
is largest. Lattice energy is the driving force for the K -803 -703 -674 -636
formation of an ionic compound and its stability. Rb -770 -674 -653 -615
Lattice energy is measured by Born-Haber cycle Cs -720 -644 -623 -590

ah; We observe that the lattice energies decrease


Aflsl + h*) —U MX(s) regularly from Li to Cs and from F" to I". (Table 8.5).
• The higher the charge and the smaller the size of the
D
S ion, the more is the hydration energy. In a group,
2- lattice energy and hydration energy decrease as we go
M(g) M(g) down. While the decrease in lattice energy favours the
I -e" -#| +e" -U solubility, the decrease in hydration energy makes the
compound insoluble.
M+(g) + X"(g)----------
8 = heat of sublimation of M(s) DEEP Focus
1 = ionisation energy of M(g) • Another important factor contributing to the solubility of
D = dissociation energy of X2(g) the compound is the comparison of the size of cation and
E = electron-affinity of X(g)
£ anion. Whenever cation is small and anion is large or
U = lattice energy of MX(s)
vice-versa, the solubility will be larger. Thus, solubility of
CsF > CsCl > CsBr > CsI and solubility of
= heat of formation of MX(s) LiF < LiCl < LiBr < Lil.
• LiCl, LiBr, Lil, and Nal are readily soluble in some
• Based on above cycle oxygen-containing organic solvents such as ether. Nal and
Mi°=S + I + --E-U Lil both are very soluble and form complexes in NH3. The
f 2 unstable complex[Na(NH3)( ] I has been isolated and shown
U = -Mi°f+S + I + ^-E to contain a tetrahedrally coordinated sodium ion.
• In the vapour phase, alkali metal halides are present
as ion-pairs, but measurement of bond length and
The solubility of most of the salts of group 1 (alkali dipole moment suggests that under these conditions
metals) elements M water decreases on descending electron-sharing is also involved to a considerable
the group. For a substance to dissolve, the energy evolved extent, especially for the lithium halides.
when the ions are hydrated (hydration energy) must be
larger than the energy required to break the crystal lattice • For the salts of small anions (like F", OH", O2", CO^"
(crystal energy). We can say that if the solid is insoluble. etc.), lattice energy which is inversely proportional to
(r+ + r ) is very sensitive to the change in the size of the
Hydration Energy < Lattice energy cation, and anion being very small in size has little
In case of alkali metals, lattice energy only changes contribution in the total and decreases sharply along
slightly, but the free energy of hydration changes rather the group, and decrease in lattice energy is greater
more. than decrease in hydration energy and thus solubility
of these salts increases as we go down the group.
Free energy of hydration Lattice energy Solubility of the alkali fluorides is thus in order:
A^hydratlon MCI(U)
Li*
LiF < NaF < KF < RbF < CsF
-485 -845 difference
Na’ -379 -770 67 Li2CO3 < Na2CO3 < K,CO3 < Rb2CO3 < Cs2CO3
K’ -308 -703
Rb’ -285 -674 Thermal Stability of Salts
Cs’ ________-362________ -644 The stability of the compounds can also be explained
__ _______ all values in kJ mol by using the concept of polarising power. The simple idea
362 | Essential Inorganic Chemistry

is that, as the charge density of the metal ions 2-


0—H—Ox
increases, the thermal stability of the salts of large O—C< >C—O
polarisable anions, relaive to some decomposition '0—H—0Z
product decreases. In general, the least polarising H-bonding in KHCO3 (a-dimer)
metal ions are those of the most electropositive metal ions
and these from the most stable salts with large anions. In o
other words, small cations form stable salts with /O ,0 (k
small anions and large cations form stable salts
■w C °"'H c c
with large 0 0
H-bonding in NaHCOa (a-chain)
DEEP Focus
The thermal stability of Group I hydroxides also
Small cations and small anions
follows a similar trend as that of carbonates. Thus, except
or Stable salts LiOH, which on heating decomposes to Li2O, all other
Large cations and large anions z Group I hydroxides are stable.

anions. Let us take the example of carbonates of Group I 2LiOH Li2O +


metals. The carbonates of sodium, potassium rubidium
and caesium are resistant to the heat of a bunsen burner 2NaOH
flame, however, lithium carbonate decomposes to its oxide KOH ■>
no effect, thermally stable.
and carbon dioxide under the same conditions :
Li9CO3 -----> Li2O + C02 The ease of thermal decomposition of a salt is related
A to the enthalpy of formation of the salt. The enthalpy
Na2CO3 -> No effect
of formation, AHp of a salt MA (M is the metal, A is
The tendency of Li2CO3 to undergo thermal
anion) is given by
decomposition may be explained in terms of the
AH° =(AHatnm + metal + (^atom A anion
gain in electrostatic attraction that occurs when
extremely small LiT ion combines with the smaller oxide where AHatom is the enthalpy of atomisation, I is the
ion rather than the much larger carbonate ion. The
other carbonates of Group I (Na-Cs) are more stable ionisation energy and EA is the electron affinity. Since for
because the cations have a lower charge density and any salt in a particular group the terms involving the
considerably larger in size and thus decomposition in less anion alone remain constant, the value of for such
favourable energetically. compounds is dependent upon the sum of the enthalpy
terms of the particular metal, (AHatora + /)metal and the
DEEP Focus lattice energy, AHlatt. The larger the lattice energy, the
AU the metals except lithium form stable bicarbonates more negative the enthalpy of formation and so, more
(lithium bicarbonate is formed only aqueous solutions and has stable is the compound. All these terms become smaller on
not been isolated. It is due to strong polarising power of Li+ on descending the series from lithium to caesium. The
the HCOj ion of larger size.) When we heat the alkali metal
relative stabilities of the salts are, therefore, decided by
bicarbonates, they are decomposed to carbonates and
simultaneously, carbon dioxide and water are liberated. the parameter which decreases more rapidly—the lattice
energy or the sum of the metal enthalpies.
2NaHCO,3 Na2CO3 + CO2 + H2O
In the salts having small anions of high charge
This reaction forms the basis for the use ofNaHCO3 in density, e.g., F", N3", OH", O2" etc., the change in
baking powder.
lattice energy is much dependent on the size of the
As the electropositive character increases down the cation and decreases rapidly on descending the
group the stability of carbonates and bicarbonates group. Thus, as the size of the cation increases, lattice
increases. (Na < K < Rb < Cs) energy decreases more than the change in the sum of the
The crystal structures of NaHCO3 and KHCO3 both metal enthalpies. Therefore, as we go down the group, the
show hydrogen bonding, but are different. In NaHCO3 the stability of these salts having small anions decreases.
HCO3 ions are linked into infinite chain, while in Thus, in alkali metal fluorides, the stability decreases in
KHCO3 a dimeric anion is formed. the order LiF > NaF > KF > RbF > CsF.
Chapter 8: s-Block Elements — The Alkali Metals | 363

K— Rb — Cs— Fr
DEEP Focus 19 37 55 87
IA(1)
Similarly, lithium nitrate also decomposes on heating to Li —A Na
five Lj2O, NO2 and O2 but all other alkali metal nitrates Cu----- Ag—[—Au
oecompose on strong heating to nitrites liberating oxygen: 3 /k*.'11 J 29 47 79
IB (11)
Stability being t bridge-element
LiNO < NaNO, < KNO, < RbNO < CsNO3
U MM’***

4LiNO3 ---- > 2Li2O + 4NO2+O2 (I) IA — Li, Na, K, Rb, Cs, Fr
(II) IB — Cu, Ag, Au.
2NaNO,. ---- > 2NaNO9+O2
Sodium (11) acts as a bridge for sub-groups A and B.
• The increasing stability of the peroxides (M2O2) or Distinction between alkali and coinage metals has been
superoxides (MO2), as the size of the metal ion
increases is due to the stabilisation of large anions by summarised below:
larger cations through lattice energy effects. Properties IA(1) IB(11)
• The m.p. and b.p. of alkali halides always follow trend [Inert gas] ns1 [Inert gas]
1. Electronic
MF > MCI > MBr > MI. configuration (n-1)c/'° ns1

2. Block s-block d-block


Hydride 3. (/£) low, form stable M * ion higher than IA, form
The alkali metals react withH^ at high temperature to M* ion, Cu also
forms Cu2* while Au
form the hydrides. form Au3* salts
1073 K
2Li +H2 -------- > 2LiH 4. Reactivity highly reactive even non-reactive with
with water water but tarnished
673 K by air
2K + H2 2KH
5. Nature of salts halides, sulphates, CuCI, Cui, CuBr,
• Metallic hydrides are ionic solids with high melting carbonates, AgCI, AgBr, Agl,
points. They form complex hydrides when find nitrates,
bicarbonates water CuS, Ag2S etc.,
applications in synthesis of organic compounds. soluble (Li* is insoluble
LiH + AlH3 ---- > LiAlH4 exception, its salts are
predominantly covalent
NaH+BH3 ---- > NaBH4 due to smaller size)
They also react with proton donors such as alcohols, larger than coinage Smaller than alkali
6. Size of ions
gaseous ammonia and alkynes. metal metal
Na* >Cu*,K*> Ag*
,____ above H in placed below H in
placed
Complex Halide and Polyhalides 7. Electropositive
electrochemical series, electrochemical
nature
Complex halide formation such as represented by the more reactive than series hence, less
equations: coinage metals reactive than alkali
Li* (aq) + e‘ — Li(s) metals, do not
MF + BF3---- >M[BF4] displace H2 with
E° = - 3.045 V
and MI + IC1---- >M[IC12] acids
Cu2*(aq) + 2e"
is favoured by increase in the size of the cation.
—> Cu(s)
• The alkali metal halides react ith the halogen and
E°= + 0.34V
interhalogen compounds forming ionic polyhalide
compounds
KI + L, ---- > K[I3] Reactions of Group I Metals
KBr + ICl ---- > KtBrICI] Alkali metals, due to low (ZE), are reactive and are
KF + BrF2 ---- > K[BrF4] always found in combined state. Table 8.6 summarises
reactions of alkali metals. The reactivity of the alkali
Alkali and Coinage Metals metals increases from Li to Cs except towards N2.The slow
Elements of group I except hydrogen are all metals reactivity of Li is due to kinetic factor.
arranged in two sub-groups:
364 | Essential Inorganic Chemistry

Table 8.6 Reactions of Group I Metals


Reaction Comment
2M + 2H2O 2M0H + H2 MOH are strongest bases. All alkali metals react with H2O. Lithium undergoes the reaction with
vigorous bubbling asH2 is released, sodium reacts rapidly with evolution of heat and potassium
reacts so violently that H2 produced bursts instantly into flame.
2Li+-O2 ---- > Li2O Monoxide is formed by Li and to a small extent by Na (oxidation number (O.N.) of 0 = - 2) Normal
2 vAiOes M2O, <M - Li, Na. K, Rb) have antifluorite structure . This structure is related to the CaF2
2Na + O2 NazO2 structure but the sites ?ccupied by the cations and anions interchanged so that M replaces F and
0 replaces Ca in the structure.
There is a trend to increasing coloration with increasing atomic number,
Li2O, Na2O — pure white
K2O — yellowish white
Rb2O — bright yellow
Cs2O — orange
Perox^ 'em- d by Na and to a small extent by Li (O.N. of 0 = - 1)
K + O2 ---- > KO2 Superoxide (MO2) formed by K, Rb and Cs. Superoxides are coloured and paramagnetic (ON of 0
= -1/2)
Under appropriate conditions pure compounds M2O, M2O2 and MO2 can be prepared for all five
metals.
2M+H2 2MH Ionic salt-like hydride, LiH is the most stable.

MH+ H2O > MOH + H2 Hydrides are decomposed by H2O forming H2 and thus are reducing agents
3Li+—N2 ---- » Li3N Only Li forms nitride, slow at room temperature rapid at elevated temperature.
2 2
2Na + C2H2 ---- > Na2C2 + H.l2 Carbide contains carbide ion (C=C)2~
2M+F2 ---- > 2MF All metals form fluorides, chlorides, bromides and iodides. The reactivity of the alkali metals
2M + Cl2 ---- > 2MCI increases as their (/£) decreases giving reactivity order:
2M + Br2 ---- > 2MBr Cs > Rb > K > Na > Li
2M + l2 ---- > 2A4l The higher members can form polyhalides, Kl3 (K+ and l3) and CsBr3 (Cs*,Br3)
Very vigorous reaction. Polysulphides are also formed. They are zig-zag chain structure of sulphur
2M + S ---- » M2S atoms.

2M + 2NH3 2MNH2 + H2 All the metals form amides. Dilute solutions of alkali metals in liquid NH3 are dark blue in colour and
the main species present are solvated electrons and the solvated metal ions. If the blue solution is
Na(/)---- > Na*(/) + e’ allowed to stand, the colour slowly fades until it disappears owing to the formation of metal amide.
e~ + NH3 ---- > NH2 +1 H2

Na* (am) + e~ (am)+ NH3(/)


---- > NaNH2(s)+-l H2

where ‘am’ denotes solution in ammonia.

Sesquoxide “Af2O3” have been prepared as The ozonides, on standing, slowly decompose to
dark-coloured paramagnetic powders by careful thermal oxygen and the superoxide MO2, but on hydrolysis forms
decomposition of MO2 (M = K, Rb, Cs) They can also be MOH
prepared by oxidation of liquid ammonia solutions of the MO3 ■> MO2 — O2
metals or by controlled oxidation of the peroxides, and are 2
considered to be peroxide disuperoxide [(Af+)4(O2“)(O2)2] 4AfO3 + 2H2O ---- > 4MOH + 5O2
• Ozonides MO3 have been prepared for Na, K, Rb and Rb and Cs also form suboxides in when the formal
Cs by the reaction of O3 on powdered anhydrous MOH oxidation state of the metal is considerably lower. Partial
at low temperature and extraction of the red MO3 by oxidation of Rb at low temperature gives Rb6O which
liquid NH3 decompose above -7.3°C to give copper-coloured metallic
3M0H(s) + 2O3(£) ---- > 2MO3(s) + AfOHHjOCs) crystals of Rb9O2 :
2Rb6O Rb9O2 +3Rb
Chapter 8: s-Block Elements — The Alkali Metals | 365

Alkali metals react with HjO, liberating H2 and associate to form electron-pairs and their
bluing hydroxides. The reaction becomes increasingly paramagnetic character decreases.
violent on descending the group. The Li reacts gently, The solutions of alkali metals in liquid NH3 are
sodium melts on the surface of water and molten metal extremely good conductors of electricity: they have higher
even catches fire. Similarly for K..... Thus, Na, K are conductivity than that of completely ionised salt in any
placed in kerosene oil. known solvent. But the equivalence conductance
decreases as the solution become more concentrated and
• Alkali metals are reactive and tarnish rapidly in dry
reaches a minimum around 0.05 M. Beyond this
air. Na, K, Rb and Cs form oxides of various types, but
Li forms mixture of the oxide and the nitride, Li2O and concentration the conductivity increases dramatically
UjN. Li is the only element of the alkali metals sharply and the saturated solutions practically conduct
electricity like liquid metals :
that forms nitride.
Conductivity of Hg = 104 ohm of Na in liquid
~ 0.5 x 104 ohm-1 NH3
Solutions in Liquid Ammonia
The alkali metals dissolve in pure liquid ammonia MNH3),r +[e(NH3),r---> [e(NH3),t
without evolution of hydrogen; the metals can be The solutions are themselves unstable with respect to
recovered from such solution by evaporation. The amide formation.
saturation solubilities are quite high, e.g., at -33 C. Na+(NH3) + e’(NH3) ---- > NaN^+Hj
Solubility in However, under anhydrous conditions and in the
Element kg/kg NH3 absence of catalytic inpurities such as transition metal
Li 108 ions, solutions can be stored for several days.
Na 250 3. Reducing Property Ammoniated solutions of
K 463
alkali metals are reducing agents due to the presence of
1. Colour Dilute solution N free or solvated electrons
)f alkali metals in liquid NH3 e"(NH3ly
H' hh O2 ■> O2
ire blue due to solvated
electrons and the solvated N<H
netal ions. But in the H
0 1. e-(NH3) v
e-(NH,)
K2[Ni“(CN)J + 2K-------- K4[Ni(CN)J
0

• The alkali metals dissolve readily in mercury forming


concentration of NH3 above H more reactive species M(Hg) called amalgams
Ilf, metal ion cluster are N M(s) + Hg(Z)---- >M(Hg), AH = -ve
ormed, the solutions; are (Solvated electron)
Jopper-coloured with Estimated diameter of Formation of an amalgam is an exothermic process.
netallic lustre. the cavity = 3 - 3.4A
2. Conductance and Solvation
nagnetic properties The solutions are good
■onductors of electricity and are paramagnetic in nature, When a metal ion is surrounded by the solvent
molecules, the phenomenon is called solvation of the
■shi ch decreases as concentration increases. metal ion. When the solvent is water it is called
Explanation hydration. The alkali metal ions are highly hydrated.
When alkali metal is dissolved in liquidNH3, there is The smaller the size of the ion, the greater is its degree of
Step I. Ionisation of alkali metal hydration. This is because the smaller the size, the more
Af(s)---- > M+(g) + e will be its charge density and the more will be its
Step II. Cation M+ and e~ both are solvated by NH3 attraction for the polar solvent molecules. Thus, Li+ ion,
which is the smallest gets more hydrated than theNa+ ion
□nning [M(NH3)J and [e(NH3),]'.
and so on. The degree of hydration decreases on moving
M++(x + y)NH3 ---- > [M(NH3)x]+ +[e(NH3)y] down the group. As a result of differences in their degree of
It is the ammoniated electron which is hydration, the hydrated ionic radii of the alkali metal ions
—sponsible for the blue colour of the solution, and decrease as we go down the group from lithium to cesium.
ne electrical conductivity is due to the Li+ ion has the largest hydrated radius while Cs+ has the
■nmoniated cation, [M(NH3)J+ as well as the smallest hydrated radius in the first group. One would
=nmoniated electron, [e(NH3)y)", values of x and y agree that the smaller the size of the ion and the lighter it
—pend on the extent of solvation (by NH3). Dilute is, the more will be its mobility and thus conductance. In
this regard we should expect the highest conductance for
■lutions are paramagnetic due to free electrons. In
Li+ which is the smallest of the alkali metals but this is not
■ncentrated solutions ammoniated electrons
366 | Essential Inorganic Chemistry

so. The hydrated radius of Li* is the largest of all the conductance in solution actually decreases in the
alkali metal ions, in solution. Therefore, Li* ion is order Cs* > Rb+ > K+ > Na+ > Li*.
the least conducting in solution. The ionic

Target Practice 1
1. Arrange Li, Na, K, Rb, Cs in increasing. (c) Li* and Mg2*
(a) atomic radii (d) Be2* and Al3*
(b) ionisation energy 4. The single-bonded metallic radius of sodium is 157 pm.
(c) electronegativity
Assume that the increment between radii of different
(d) electron-affinity
magnitudes is 60 pm. Calculate the
2. Arrange alkali metal carbonates in increasing
(a) covalent radius of Na
(a) thermal stability
(b) covalent nature (b) ionic radius Na*
(c) melting point (c) ionic radius Na"
(d) lattice energy (d) van der Waals’ radius Na
3. Compare hydration energy of 5. Write compounds of low solubility of Fr
(a) Na* and Be2* 6. How could you remove unreacted Na (metal) in liquid
(b) Li* and Be2* ammonia safely?

Answers
1. (a) Li < Na < K < Rb < Cs (b) Cs < Rb < K < Na < Li 3. (a) Be2* > Na* (b) Be2* > Li* (c) Mg2* > Li+
(c) Cs < Rb = K < Na < Li (d) Cs < Rb < K < Na < Li (d) Al3* > Be2*
2. (a) Li2CO3 < Na2CO3 < K2CO3 < Rb2CO3 < Cs2CO3 4. (a) 157 pm (b) 97 pm (c) 217 pm (d) 217 pm
(b) Cs2CO3 <Rb2CO3 <K2CO3 < Na2CO3 <Li2CO'3 5. FrC104, Fr2PtCl6 (due to larger alkali metal ion)
(c) Li2CO3 < Na2CO3 < K2CO3 < Rb2CO3 < Cs2CO3 6. Add NH4C1, (a strong acid in NH3)
(d) Cs2CO3 <Rb2CO3 <K2CO3 < Na2CO3 <Li2CO3 2Na + 2NH4Cl ---- > 2NaCl + NH3+H2

Practice Exercise 1
1. Differentiating electron in K(19) goes into ns and this is 8. Alkali metals are good reducing agents. Explain.
s-block element. What happens if Aufbau rule is not 9. (ZElj of Li is highest in first group but it is best reducing
followed ? agent in aqueous solution. Explain.
2. What would be electronic configuration of the following in 10. Arrange He, Li, Be, B, C, N, 0, F, Ne in increasing second
the absence of Aufbau rule? ionisation energy.
(a) Ca2* (b) Sr2* 11. In Table 9.3, size of the hydrated ion is in order
(c) Rb* Li* > Na* > K* > Rb* £ Cs* which is just reverse of ionic
3. Na and Mg are also called bridge element. Why? size. Explain.
4. Which is more covalent in each pair? 12. Calculate lattice energy of NaCl (s) from the following data:
(a) LiCl or NaCl (b) LiCl or BeCl2 heat of sublimation of Na(s) = S
5. (ZE)j of the two elements of s-block of 2nd period are 520 (ZE)i of Na (g) = Z
bond dissociation energy ofCl2(g) = D
and 899 kJ mol"1. Identify these elements. electron affinity of Cl(g) = - E
6. Compare Li and Be in the following properties: heat of formation of NaCl(s) = - Q
(a) (IE)2 13. Arrange the following in increasing lattice energy:
(b) size of hydrated ion (i) NaF, NaCl, NaBr, Nal
(c) polarisation (of cations) (ii) LiF, LiCl, LiBr, Lil
(d) thermal stability of carbonates (iii) LiCl, NaCl, KC1, RbCl, CsCl
(e) nature of oxide 14. A mixture contains 2 moles of Na„COq and 1 mole d
(f) nature of hydride
Li,CO,. Calculate volume of CO, formed on heating this
(g) oxidation state Xu l»

7. Compare Li and Mg in the following properties: mixture and the data converted at STP.
(a) effect of heat on their carbonates 15. Identify (A), (B), (C), (D) and (E) in the following:
(b) hydration of chloride A . H2O
Alkali metal (A) + N2 ——(B) —-—> (C) + (D)
(c) nature of oxides
(d) tendency to form nitrides (C) and (D) both react with HC1, but (D) gives white fumes
(e) electronegativity (E)withHCl.
Chapter 8: s-Block Elements — The Alkali Metals | 367

What are (A), (B), (C) and (D)? What is main application of
16. A binary salt (X) of alkali metal on heating with sulphur
give (Y); (Y) with BaCl2 solution gave white ppt. (Z) (D)?
insoluble in cone. HC1. 14.2 g of (X) gave 23.3 g of (Z). 18. Match the following column (A) with column (B):
Identify (X), (Y) and (Z) and explain reactions.
17. (a) Which element of the alkali earth metals (group II) will (A) (B)____________
form nitride? 1. KO2 (a) Diamagnetic
(b) 1 mole of a mixture of Li2CO3 and Na2CO3 forms (b) Conductor of electricity
0.75 mole of CO2 at NTP. What is percentage of each in 2. Na2O2
3. NaANH3 (c) Paramagnetic__________
the mixture?
H O
(c) Li + H2 ---- > (A) —2—> (B)+(C) 19. 2Na(s) + 2H2O(Z)---- > 2Na*(ag) + 20H_(a<?)+ H2(g)
I A1H3 Write the ON of all the atoms in various species.
(D)

Answers
1. K(19) in the absence of Aufbau rule will have EC as: 15. (A): Li (only Li forms nitride with N2) (B): Li3N
ls2,2s22p6,3s23p63d1 and thus differentiating electron (C): LiOH (D):NH3 (E): NH4C1
enters into 3d— a d-block element. 16. Let alkali metal be M, since Xis binary and combines with S to
give Y. Y gives white precipitate with BaCl2 insoluble in cone.
2. (a)Ca2* = Is2, 2s22p6, 3s23p6 HC1. It means Y has SO 2' and thus X is an oxide and thus, X is
(b) Sr2* = Is2, 2s22p6, 3s23p63d10, 4s24p6
an oxide. Since X is binary hence, X is
(c) Rb* = Is2, 2s22p6, 3s23p6 3d10, 4s24p6 BaCl,
3. Na is called bridge element since it links Group IA and Group 2MO2 +S ---- > Af22‘SO4 ------ =i BaSO4
2(M + 32) 233
IB 14.2 23.3
^Cu----- Ag----- Au
29 47 79 2(M + 32) _ 233
¥ Na
n _ ___ Rb---- C s----- Fr
19 37 55 87
Thus, 141 231
Thus, M = 39. Thus M is potassium.
Similarly, Mg links HA and IIB
^Zn---- Cd----- Hg (X) : KO, (potassium superoxide, K*O2 (binary)
30 48 g0
Be-fMg (Y) :K2SO4 (Z): BaSO4
4 M2> Ca-----Sr----- Ba----- Ra BaCl2
20 38 56 88
2KO2 + S---- > K,SO4 ------- > BaSO4
4. (a) LiCl (b)BeCl2 (X) (Y) (Z)
Smaller cationic size, larger, charge, larger polarising power 2 x 71 g 233 g
14.2 g 23.3 g
5. Li and Be
17. (a) Mg (due to diagonal relationship with Li)
6. (a) Li > Be (b) Li* < Be2* (c) Li* < Be2*
(d)Li2CO3 > BeCO3 (e) Li2O-basic (f) LiH—ionic (b) Li2CO3 - Li2O + CO2
(g)Li = +1 BeO—amphoteric BeH2—covalent Be = + 2 Na2CO3 A+ X (no effect)
7. (a) Both decompose to form CO2 and oxide
(b) Both hydrated (c) Basic oxides CO2 is due toLi2CO3 only
(d) Both form nitrides (Li3N, Mg3N2) hence, 0.75 mol CO2 = 0.75 mol ofLi2CO3
(e) Xu=10 XM “1.2 75 mol %Li2CO3
8. In case of alkali metals, outer ns-electron is .very well shielded 25 mol % Na2CO3
by the inner electrons. Thus effective nuclear charge (= Z - o) (c) (A): LiH (B): LiOH (C): H, (D): LiAlH4
is very small. As we go down the group, shielding wins over the (D) is a reducing agent in the preparation of alcohols from
effect of the increasing number of protons in the nucleus. Thus
(IE)} decreases, hence tendency to lose electrons, which is also carbonyl compounds, acids, esters, etc.
a measure of the reducing nature, increases down the group. ^>C=O LiA1H^ CHOH
9. See text
10. Be < C < B < N < 0 < F < Ne < He < Li LiAlH.
BCOOH -------- BCH,OH
11. Greater the charge or charge density, smaller the ionic size, greater LiAlH.
the hydration hence, greater the size of the hydrated ion. BCOOB' ------- BCHoOH + B'OH
12. Lattice energy -1/ = S + Z + - E-Q 18. (A) (B)
1. c
13. (i) Nal < NaBr < NaCl < NaF (ii) Lil < LiBr < LiCl < LiF a
2.
(iii) CsCl < RbCl < KC1 < NaCl < LiCl 3. b
14. 22.4 L (only Li2CO3 decomposes) 19. 2Na(s) + 2H.,O(Z) ■» 2NaT(aq) + 2OH’(ag) + H2(g)
Li2CO3 ---- » Li’22 O + CO2
at STP 1 mo1 1 mo1 t t
+1 -2 +1
tt 0
t
0
368 | Essential Inorganic Chemistry

Compounds of Sodium Nelson Cell (Diaphragm cell)


• The metals of this group are too reactive to be found in If H2 and Cl2 gases mix, they react with explosion. A
the free state. Their compounds are amongst the most porous diaphragm of asbestos is used to keep the H2 and
stable to heat, so thermal decomposition is Cl2 (and thus cathode and anode) gases separated from
impractical. one another. Product in this cell is a mixture of 11% NaOH
and 16% NaCl. Concentration in a steam evaporator gives
• The metals are the strongest chemical reducing
solution containing 50% NaOH and 1% NaCl. (Fig. 8.4)
agents and oxides can’t be reduced to metals.
Carbon anodes (+ve)
• The metals are generally obtained by electrolysis of a ■ . — TOT
Cl2 A
fused salt, usually the fused halide, often with
impurity added to lower the melting point.
<♦ Brine
• Sodium is obtained by electrolysis of molten mixture
of:
(a) 40% NaCl r j r n r n
(b) 60% CaCL in a Downs cell. "TOD-i y •& Asbestos
diaphragm
CaCl2 lowers the m.p. of NaCl (803° C) to 600° C. Some Wire gauze
cathode (-ve)
Ca is formed but it remains insoluble in sodium, which H2*
collects at a copper or iron cathode that surrounds a
circular graphite anode. Chlorine, a valuable by-product,
collects at anode.
Caustic soda out
Fig. 8.4 A Diaphragm cell
Sodium Hydroxide
Until World War II, the Soda-Lime process the
principal method of producing sodium hydroxide, NaOH, Mercury Cathode Cell
involved conversion of Ca(OH)2 to CaCO3 using Na2CO3 (Castner-Kellner Cell)
Ca(OH)2 +Na2CO3 ---- > CaCO3 + 2NaOH During the electrolysis of brine, Na+ is discharged at
cathode
The reaction goes to completion as is evident by Na+ +e~ ---- » Na
measuring pH of the solution after reaction.
If the cathode is made of mercury, the Na atoms
The insoluble CaCO3 by-product is filtered off, then produced are dissolved in mercury and form an amalgam.
heated (calcined) to convert it to lime (CaO), which is The amalgam is pumped to a different compartment and
recycled into the reaction system. water reacts to form NaOH.
Na (amalgam) + H2O * NaOH + ^Hj+Hg
CaCO3(s) ---- > CaO(s) + CO2(g)
CaOteJ + HjO ---- > Ca(OH)2 Originally the anodes were made of graphite, but
• Electrolysis of aqueous NaCl gives NaOH at cathode because traces of oxygen are produced in a side reaction,
they become pitted with corrosion. The anodes are now
NaCl(aq) Na+ + Cl~
made of steel coated with titanium (Fig. 8.5).
Anode: 2Cr ---- > Cl2 + 2e~ Positive carbon anodes
Cathode: Na+ +e" ---- > Na A A
-'TOT
Cl2 A
l|l
2Na + 21^0---- » 2Na0H + H,
■»

Side reactions may also occur if the products formed Spent Strong
mix with one each other. brine brine in

2NaOH + Cl2 ---- > NaCl + NaClO + HjO


out
qu on ^Hg in

or 20H“ +C12 ---- > 20C1- +H2


and following reaction may also occur at the anode to a Na/Hg
smaller extent: out
W
40H- 02 + 2H2O + 4e Moving mercury cathode (-ve)
Fig. 8.5 The Castner-Kellner cells
Chapter B : s-Block Elements - The Alkali Metals I 369
Properties With Oxide
White crystalline deliquescent solid, highly soluble in Amphoteric oxides (as well as hydroxides) dissolve in
w-ater but sparingly in alcohol corrosive to skin. It breaks NaOH.
down the proteins of skin to a pasty mass hence called
taustic soda. Al203 + 2NaOH--+ 2NaAl02 + �
• A strong base ionised almost completely Al{OH) 3 +NaOH--+ NaAl02 + 2�0
NaOH ¢ Na+ +oH- ZnO + 2NaOH --4 N¾ZnOi + �O
NaOH+H + ¢ Na + +�0 Zn(OH)2 + 2NaOH --+ Na2 Zn02 + �O
Equivalent weight ofNaOH = molecular weight (similarly with SnO, PbO)
= 40 g equiv-1
With Salts
With Acidic Oxide
Salts like ZnC12,SnC12, AlCI 3 etc. give white ppt with
Itcan absorbCO2 , S02 N02 etc. (acidic oxide) to form salt NaOH; however precipitate dissolves in excess ofit.
2NaOH + CO2 --+ Na2C0 3 + 8i0 NaOH
ZnC12 + 2NaOH --+ Zn(0 H)2 -- Na2 Zn02
2NaOH +S02 --+ Na2S0 3 + liiO white ppt. soluble
2NaOH + 2N02 --+ NaN02 + NaN0 3 + �O NaOH
AlC1 3 + 3NaOH--+ Al(OH) 3 -- NaAl02
white ppt. soluble
With P4 NaOH
SnC12 + 2NaOH--+ Sn(OH)2 • Na2Sn02
P4 + 3NaOH + 3�0 � 3NalizP02 + PH3 white ppt.
yellow sodium hypo phosphine
phosphite • NH; salts are decomposed by NaOH forming NH3

Withs NH; + oH- � NH3i + �o


It is a disproportionation reaction ofP4 to �P02 (by • Salts of transitional metals give insoluble hydroxide
oxidation) and PH 3 (by reduction) with NaOH.
' 4S +6NaOH � Na2 S2 0 3 · + 2Na2 S+ 38i0 FeC1 3 NaOI-\ Fe(OH) 3 -1.
hypo brown ppt.
(sodium
CrC1 NaOI-\ Cr(OH) -1.
thiosulphate) 3
green ppt.
CuS04 NaOI-\ Cu(OH)2 !
1 With Halogens blue ppt.
1
X2(Cl2, B�. 1J disproportionates to
cold and dilute alkaline solution (NaOH)
x- and xo-in
However, AgOH and Hg(OH)2 decompose to oxide on
heating.
• X2 +2NaOH --+ NaX + NaXO +�O
cold sodium 2Ag0H � Ag2 0 + �O
hypohalite
• X2(Cl2, B�, lz) also disproportionates to x-
and X03 Hg(OH� � HgO+ �O
in hot and concentrated alkaline solution (NaOH)
3X2 +6NaOH --+ 5NaX + NaX0 3 + 38i0 With CO
hot sodium haloate HCOONa is formed when CO is passed into NaOH at
high pressure and temperature
With Metals
5-IO atm
Elements like Zn, Al, Sn, B etc. liberate� with NaOH CO+ NaOH HCOONa
100-200° c
Zn+2Na0H--+ Na2 Zn� + 8i
sodium zincate Application
Sn+ 2NaOH+ �O--+ Na2Sn0 3 + 2� NaOH finds wide application in
sodium stannate (1) paper industry
2Al + 2NaOH+ 2�0--+ 2NaA102 + 3lii (2) as a laboratory reagent.
sodium (3) alumina production
meta aluminate (4) soap industry
2B+2Na0H+2�0--+ 2NaB02 + 3� (5) rayon industry
sodium meta borate
370 I Essential Inorganic Chemistry

Aq. NaCl
DEEP Focus
• Na2O2 is diamagnetic (�11 electrons paired) and is regarded
as a salt of dibasic acid H 20 2• o;- is isoelectronic ofF2
Na 202 + 2H20 ---+ 2Na0H + Hp2
• Na:P :1 is colourless in pure form but is pale yellow in colour
due to presence of sodium superoxide Na02 and is an
oxidising agent.

- Al is oxidised to A1 203•
3Na202 + 2Al ---+ Al203 + 3Na2 0
- Cr salt is oxidised to Cr�- (yellow).
3+

Na202 + 2ft.?O ---+ 2NaOH + �02


2Cr + 3ft.?0 + lOOH- ---+ 2Cr�- + 8�0
3+
2
yellow

Microcosmic Salt
• Colourless crystals of microcosmic salt are obtained DEEP Focus
when NH.1 Cl and Na2 HP04 in 1 : 1 molar ratio are Because it reacts with CO2 in the air, it has been
dissolved in hot water and cooled. used to purify air in submarines and confined spaces, as
it both removes CO2 and produces 02 •
NH4 Cl + N32HP04 ---+ Na(NH4 )HP04 + NaCl
• On heating, NaP03 is formed, which forms coloured Na 20 2 + CO ---+ Na 2C03
beads of orthophosphates with oxides of transition 2Na 20 2 + 2C02 ---+ 2Na 2C03 + 03
metal and cloudy bead with Si02 •
Na(NH4 )HP04 ....!.. NaP0 + ft.?O t + NH t
3 3
> 400 c
• 2Na2 0---
°
Na202 + 2Na (vapour)

NaP03 + CuO ....!.. CuNaP0 (blue bead)


4
In the absence of oxygen on oxidisable material, tne
+ CoO ....!.. CoNaP0 (blue bead)
peroxides (except Li202) are stable towards, thermal
NaP03 4
decomposition up to quite high temperature
NaP03 + MnO � NaMnP04 (violet bead) (Na202 == 675° C).
� HCI � Na2HPQ4 .....----- • Na202 gives Hi02 with dil acids
N2 + H2--+ �__..,.. � Na(NH4)HPO4
Na202 +ft.?S04 ---+ H202 + Na2S04
dil
CuO,O __.__MnO
CuNaPO4 NaMnPO4
blue .1 6 violet bead • 02 is formed when Na202 reacts with hot acids .
N3202 + 4HC1 (hot)--+ 4NaCl + 2�0 + 0i
CoNaPO4
blue bead • Na202 ox.ides Mn(OH)2 to Mn� - (green)
Mn(OH)2 + 2Na 202 ---+ Na2Mn04 + 2NaOH
Sodium Oxide and Peroxide
• Sodium peroxide along with sodium oxide is formed When mixture ofCa(OH)2 and Na202 is compressed
when sodium bums in air or with NaN02• Ca02 is formed.
2Na + 02 ---+ Na202 Na202 + Ca(OH)2 ---+ Ca02 + 2NaOH
1
2Na + 02 ---+ Na20
2 NaOH
Na202 + 2Na ---+ 2Na20
2NaN02 + 6Na ---+ 4Na20 + N2
Oxone is a commercial name for the preparation of
I NaAlO2 I
Na202 that is used in breathing apparatus.
• Na20 is colourless ionic solid and aqueous solution is a
strong base.
Chapter 8: s-Block Elements — The Alkali Metals | 371

Reactions involved in the preparation of Na2CO3 by


h2o2 | the Solvay process are :
NH3 + H2O + CQ, ---- > NHtHCOg
02
dil. H2SO4 NaCI + NH4HCO3 ---- > NaHCO3 + NH4CI
MnO42-
2NaHCO3 -^3 Na2CO3 + H2O + CQj

Na2CO3 |
co rX fo
<h2o NaOH + H2O21 CQj is obtained by the decomposition of CaCO3
CO2/ Al CaCO3 A’ —°-°C> CaO + CO2
AI2O3 + Na2O^
Na2CO3 + O2 CaO (aq) can decompose NH^Cl to formNH3 which can
$ Cr(OH);
be reused
CaO2 | Ca(OH)2 + 2NH4C1---- > 2NH3 + 2H2O + CaCl2
CrO4~ |
Thus, NH3 is required in first instance only; it is
0?" has following MO electronic configuration recycled in further cycles. It is shown in the
(ols)2(o * 1s)2(o2s)2(o * 2s)2(o2px)2 (Fig. 8.6 - 8.8).
(n2py)2(7r2pz)2(K * 2py)2(n * 2pz)2 nh3 + h2o + CO2J NH4HCO3 } > + NaCI |
with bond-order one and diamagnetic (all electrons Filter
paired)
A NaHCOs}—►[
A CO2 Na2CO3 NH4CI

Sodium Sulphate CaCO3 |

Anhydrous sodium sulphate (Na2SO4), called salt


I
cake, is the by-product of HC1 industry Ca(OH)2|-
H2O
CaO
2NaCl + H2SO4 ---- > Na2SO4 + 2HC1
I

• If aqueous solution of Na2SO4 is cooled below 32 C,


Fig. 8.6 Solvay process for the manufacture of
Glauber’s salt Na2SO4- 10H2O separates. However, if
Na2CO3 (...) represents recycled process.
cooling is done below 12° C,Na2SO4 • 7H2O crystals are
formed. Brine
Excess NH3
•In dry air Na2SO410H2O effloresces forming
anhydrous Na2SO4
Na2SO4 • 10H2O---- » Na2SO4 + 10H2O
• Na2SO4 finds use in paper industry, detergent and
glass manufacture.

Sodium Carbonate
Since pre-historic times a natural deposit called
Trona, Na2CO3NaHCO3-2H2O (sesqui-carbonate) has
been obtained from dried-up lake beds in Egypt. Trona is
sometimes called sodium sesqui-carbonate (sesqui means
one and a half) and on heating it, Na2CO3 is obtained. NH3 gas

2(Na2CO3 NaHCO3 -2H2O) 3Na2CO3 +CO2


+ 5H2O
Ammoniated
The Solvay (ammonia-soda) process brine

• The Solvay (ammonia-soda) process is widely used to


produce Na2CO3 synthetically after Leblanc process
Fig. 8.7 The ammoniation of brine solution (Solvay process)
proved non-economical.
372 I Essential Inorganic Chemistry

Sodium Bicarbonate (Baking Soda)


Sodium bicarbonate is the intermediate compound 1
the manufacture of sodium carbonate by the Selva
Tower filled with --+• process, principal chemical reaction being:
ammoniated NaCl+ NH3 + CO2 + H2 ---+ NaHCO3+ N�O
• KHCO3 can't be prepared by Solvay process, KHC()
brine

being highly soluble in water.


• 2NaHCO3 Na2CO3 + H2O + C{}z
50-100 °C
----

Due to this property of giving C� on gentle heatini


Perforated noads
to distribute C0---1--­
through the brine
NaHCO3 finds use in
- Baking powder (which contains NaHC03,
Ca(H2 PO4 )2 and starch). Improved baking powde
contains 40% starch, 30%NaHCO3, 20%NaAl(SO4)
and 10% Ca(H2PO4 h.
- Pharmaceutical products (including antad
preparation for indigestion)
- Fire extinguishers.
Paste of
- NaHCO3 is also added in small amount to table sa!
(NaCl). NaCl is often contaminated with smal
NaHC03-+-i

amount of MgC1 2 which being hygroscopic maki


out
Fig. 8.8 Carbonating tower (Solvay process) NaCl moist. Adding NaHCO3 converts MgCl2 �
MgCO 3 which is non-hygroscopic salt.
• Na2CO3-10H2O-washing soda widely used in soap
and detergent MgC1 2 (s)+ 2NaHCO 3(s) ---+ MgCO3 (s) + 2NaC1h
+ H20(l) + C0i(g
• Na2CO3 • 7H2O-crystal carbonate I heavy ash
Na2CO 3-H2O widely used in glass sheets and • It is amphiprotic: HCOi + H + ¢ H2 COa
glass-fibres manufacturing HCOi ¢ H+ + cda­
• Na2CO3 - soda ash or light ash earlier used Amphiprotic species is H+ ion donor (acid) and H1" tor
to prepare NaOH, and in petroleum acceptor (base).
refinery and textile industry
• Na2CO3 • lOH2O effioresces to anhydrous Na2CO 3•
Compounds of Potassium
Potassium compounds occur in the earth's Crill'
Na2COa·lOH2O ¢ Na2CO3 + 10H2O especially as silicates. All land plants contain considerabli
• Na2CO 3 is thermally stable and does not give CO2• amounts of potassium salts due to absorption from th,
soil. Deficiency of the potassium salts in soil is made up b,
Na2 COa + H2O + CO2 ---+ 2NaHCO3
adding potassium fertilizers.
Na2CO3 + Ca(OH)2 ---+ 2NaOH+ CaCO3
Potassium can also be made by electrolysis of molte1
• Aqueous solution is alkaline due to hydrolysis. KCI. But molten potassium is soluble in molten KCI
Na2 CO 3 + 2H2O ¢ 2NaOH+ H2CO3 thus separation of metal is difficult. The preferred met.b
for preparation of potassium uses the reaction of sodiun
NaOH + CaC03 vapour with molten KCl, with potassium bein
continually removed from the equilibrium mixture
Ca(OH)2 Na(g)+ KCl(l) ¢ K(l) + NaCl(l)

Potassium Superoxide
• It is formed as a chrome-yellow (orange) powder
burning potassium in excess of moisture-free oxy�
or air.
K+ 02 ---+ KO2
• KO2 is better (than K2 O2 ) oxidising agent since I
produces G.? and H2O2 both in aqueous or acidic soluti0t
Chapter 8: s-Block Elements — The Alkali Metals | 373

2KO2 + 2H2O---- > 2K0H + HA + Q, • Colourless deliquescent powder soluble in water


2KQ, + H2SO4 ---- > K2SO4 + HA + O2 (more than Na2CO3 and Li2CO3).
2KO2 + CO---- > K2CO3 + Oa • Aqueous solution is alkaline due to hydrolysis.
4KO2 + 2CO2 ---- > 2K2CO3 + 302
K2CO3 + 2H2O---- > 2K0H + H2CO3
4KO2 + 4CO2 + 2H2O---- > 4KHCO3 + 302
K02 thus finds applications to be used in space K2CO3 + H20 (steam) 2K0H + COj
capsules, submarines and breathing masks, K2CO3 finds wide use in the manufacture of soft soap,
because it both produces 02 and removes C02 and and is washing wool.
CO. Both functions are important in life support
system.
Potassium Sulphate
• The superoxide contains 02 ion, which has an
unpaired electron and hence, it is paramagnetic. Bond • A naturally occurring mineral, Schonite —
order is 1.5 K2SO4 MgSO4-6H2O is treated with KC1 in aqueous
solution and colourless anhydrous crystals of K2SO4
k + o2
separate.
co K2SO4MgSO4 • 6H2O + 2KC1 -> 2K2SO4
+ MgCl2 + 6H2O
K2SO4 + H2O2 + O2 IS—
H2SO4 ./,•
K2SO4
C. A
k2s • It is highly soluble in H20 and forms double salts like
potash alum
Aq. CO2 h2o
co co2 K2SO4 + A12(SO4)3 + (aq)---- >
khco3+o2 H2O2 + O2 + KOH K2SO4A12(SO4)3-24H2O
k2co3 + 02
potash alum
I. (potassium aluminium sulphate)
K2SO4 +4C K2S + 4CO?
Potassium Carbonate (Potash or Pearl Ash) BaSO4 |
• Solvay process is not adopted to prepare KHC03
(orK2CO3) being highly water soluble. BaCI2

• Leblanc process involves conversion of KC1 (from


carnallite) first into K2SO4 and then heating it k + o2 KO2 ^4
re
^0
k aqAI2(SO4)3
Alum |

(K2SO4) with carbon and limestone (CaCO3).


KC1 + H2SO4 ---- » KHSO4 + HC1 C. A
KHSO4 + KC1 - -> K2SO4 + HC1
K2SO4 + 2C + CaCO3
A
K2CO3 + CaS + 2CO2 ? k2s I
• Magnesia process (also called Precht’s process)
involves passing of C02 into aqueous solution of KC1 Potassium Nitrate
and MgC03-3H20 at 17°C, when a double salt NaNO3 (chile saltpeter) is converted to KNO3 by
KHCO3-MgCO3- 4H2O is precipitated.
following exchange reaction:
KC1 + 3(MgCO3-3H2O) + C02 ---- >
NaNO3(ag) + KCl(ag) KNO3(a<7) + NaCl(s)
2(KHCO3-MgCO3- 4H2O) + MgCl2
white ppt- (A) Above reaction is spontaneous in forward side as,
NaCl being less soluble than KNO3, is removed by
• Precipitate (A) on heating under pressure gives a
mixture ofK2C03 (soluble) andMgCO3 (insoluble) and crystallisation, and KNO3 remains in solution, and can be
thus separated. MgCO3 is reused in further obtained by evaporation of water. KNO3 is used in gun
preparation. powder (a mixture of sulphur, carbon and KNO3)
A 2KNO3(s) + 4C(s) > K2CO3(s) + 3CO(g) + N2(g)
2(KHCO3-MgCO3 • 4H2O) --- ;------------>; K2CO3
under pressure
2KNO3(s) + 2S(s) > K2SO4(s) + SO2(g) + N2(g)
+ 2MgCO3+CO2 +9H2O
374 I Essential Inorganic Chemistry

Diagonal Relationship • Lithium forms more stable covalent bonds than other
alkali metals and, therefore, forms more stable
-Anomalous Behaviour of Lithium complex compounds. For example, lithium cannot be
recovered unchanged from its liquid ammonia
solution, owing to the formation of[Li(NH3 )4 t.
The first element of a group in many ways differs from
the other heavier members of the group. These differences
arise on account of three factors The first element of group often shows resemblance to
(a) the small size the second element of the neighbouring group on the righL
(b) the high electronegativity, and This type of behaviour is known as diagonal
(c) the unavailability of d-orbitals among the first row relationship.
elements (Li to F). Li Be B C
� �

Its polarising power (i.e., charge/radius ratio) is the
Na �Mg �Al "'Si
greatest of all the alkali metal ions. This results in
increased covalent character of lithium compounds which This similarity between Li and Mg is particularly
is responsible for their solubility in organic solvents. striking and arises because of their similar ionic sizes
(Li + = 76 pm; Mg2 + = 72 pm). The ionic potentials
Due to the very small size of lithium, the metallic (charge/size ratio) are very similar for the pair Li and Mg.
bonding between the atoms in the metallic lattice is very
strong giving rise to strong cohesive forces. This is shown Li and Mg show close resemblance in the following:
in its relatively higher melting and boiling points, Nitrides Li and Mg both form nitrides. Other alkali metals oo
hardness and homonuclear bond energy. not.
The relatively higher attraction of lithium for its outer 6Li + N2 _...!.+ 2Li 3N
3Mg + N2 � Mg 3N 2
I!
electron results in its relatively higher electronegativity,
ionisation energy, hydration energy, electron affinity and Carbonates Like MgC0 3 , Li 2 C0 3 is decomposed by heat (ll"ie
and other alkali carbonates are thermally stable). Boln
bicarbonates carbonates are insoluble while Na 2 C0 3, K 2C03. -
of course smaller atomic radii relative to the other
are soluble.
homologues.

MgC0 3 � MgO + CO 2
Similar anomalies are also found in the chemical
L1 2 C0 3 � Li20 + CO2
properties but the differences appear greater.
Solid bicarbonates of lithium and magnesium do oo
exist
• Lithium salts of large polarisable anions are

Nitrates LiN0 3 decomposes to give Li 20 like Mg(N03)z, bu:


thermally less stable than those of other alkali metals,
other alkali metal nitrates give nitrite.
e.g., lithium carbonate decomposes at 950 � whereas

Mg(N0 3 h � MgO + 2N0 2 +..!. 02


no decomposition of sodium carbonate take place
2
above 1050 K
2LiN0 3 � Li2 0 + 2N0 2 + ..!_ 02
• Lithium forms no solid bicarbonate, triiodide and
superoxide as these are unstable at room 2
2NaN0 3 ---+ 2NaN0 2 + 02
I
temperature, whereas those of other alkali metals
Oxides Both give their normal oxides, Li 20, MgO, when !hi'/
burn in oxygen; peroxides and superoxides are rd
require a higher temperature to affect their
formed. Na forms peroxide, Na 20 2 while K forms
decomposition.
• Lithium salts of anions of high charge density are less superoxide, K0 2 •
Chlorides Both chlorides are hydrated, LiCl-2H 20,MgCl2•6H2').
both are deliquescent and anhydrous salts are
soluble than those of other alkali metals. The halides
soluble in ether.
of lithium are more covalent than the other halides
Phosphates Solubility of both Li and Mg phosphate is low.
and are more soluble in organic solvents.
• Lithium forms stable salts with anions of high charge Fluorides LiF and MgF2 are sparingly soluble in water.
density owing to their high lattice energy. For Hydration Both u + and Mg 2 + are heavily hydrated.
example, in air, lithium forms the normal oxide, Hydroxide LiOH, Mg(OHh both are weak bases and Ver)
slightly soluble in water, and their hydroxides
decompose on heating.
whereas the others form higher oxides. Lithium reacts
Perchlorates Lithium perchlorate (UCl0 4 ) and magnesi.r.i
with nitrogen to form nitride, LiaN, the others do not
perchlorate [Mg(Cl0 4 h] both are extremely soluble
react. Lithium hydride is more stable than the other
hydrides and lithium carbide is formed more easily in alcohol and used as water absorber.
Alkyl and aryl Li alkyls and aryls are important in organic syntheses
with acetylene.
• Lithium reacts slowly with water. and Grignard reagent (Rrv'lgX)
Chapter 8: s-Block Elements — The Alkali Metals | 375

Uses of Alkali Salts fluorescent lamps; it is also used in china wares,


textile dyes and pigments.
• Lithium stearate is used as a thickener and gelling
• KNO3 is a powerful oxidising agent and mainly used
agent to transform oils into lubricating greases.
in gunpowders and pyrotechnics, and in fertilizers.
• Lithium hydride (LiH) is used to generate hydrogen in
• KMnO4 is used as an oxidiser, decolourisers, bleacher
military, meterological, and other applications.
and purification agent; its major application is in the
• LiAlK, is a reducing agent in organic synthesis. manufacture of saccharin.
22COQR' • KO2 is used in breathing apparatus as an auxilliary
BCHO supply ofO2 in mines, submarines and space vehicles.
72COOH LiA]H4> #CH2OH 4KO2 + 2CO2 -----> 2K2CO3 + 3O2
BCOC1 4KO2 +4CQ2 +2H2O -----> 4KHCO3 +3O2
(7?CO)2O • KCIO3 is used in matches and explosive.
• KBr in photography and as a source of Br2 in organic
• Li2CO3 when taken orally provides an effective synthesis.
treatment for manic-depressive psychoses. • NajjQj finds widespread use as a bleaching agent for
• Li/FeSx battery systems are emerging as a potentially fabrics, paper pulp wood etc.
viable energy storage system and as a non-polluting • Li is used in batteries and alloys.
silent source of power for electric cars. • K is used for analytical reagents because K+ salts not
• NaOH, Na2CO3 find applications in soap, paper and so highly solvated as those ofNaT, making it easier to
glass industry and as reagent in the laboratory. obtain anhydrous reagents.
• K2CO3 is used chiefly in high-quality decorative • Cs is used in photocells because of its low ionisation
glassware, in optical lenses, colour TV tubes and energy.

Target Practice 2
1. Mixture containing one 1mole Na9CO»3 and one mole 4. In United States, sodium carbonate is obtained from
NaHCO3 is heated. What is amount (in moles) of the naturally occurring deposits of the mineral trona. Write
residue? fobmula of trona
2. Table salt is always contaminated with NaHCO3. It is to 5. How can you stabilise solutions containing Na’ ?
6. Name oxides of alkali metals which are
3. NaOH is prepared industrially by electrolysis of brine (i) paramagneticin nature
(NaCl) (ii) binary salt
2NaCl(ag) + 2H2O(Z) ---- > Cl2(g) + 2NaOH(a<7) + H2(g) (iii) used in life savingdevices
What is the name of this process? (iv) diamagnetic

Answers
heat,
1. Na2CO3 ----- MgCl2 + 2NaHCO3 ---- > MgCO3 + NaCl + H2O + CO2
> no effect. Thus, residue is 1 mol Na2CO3
1 mol 3. Soda-lime process
heat 4. Na2CO3 NaHCO3-2H2O
2NaHCO3 ----- > Na2CO3 +H2OT + CO2T
2 mol 1 mol 5. Larger anion as O2 (superoxides) can stabilise Na ’.
1 mol 0.5 mol
NaO2 is stable.
Residue is 0.5 mol Na2CO3. Thus total residue is 1.5 mol 6. (a) KO2 (due toO2)
Na2CO3. (b) KO2
2. Table salt NaCl has also MgCl2 as impurity (from sea). The (c) Na2O2
magnesium salt is hygroscopic; that is it picks water up from
the air and thus NaCl becomes wet. If NaHCO3 is added to (d) Na2O2
table salt, it converts MgCl2 to MgCO3 which is
non-hygroscopic salt.
376 | Essential Inorganic Chemistry

Practice Exercise 2
1. What is the nature of CO when it reacts with NaOH? 2. Bond orders of O2", O2,02 are respectively
,and
2. Gas(A)+ NaOH -----> (B) —>(C) (D) 3. Na2O2 is absorber and producer
and is thus used in
irCaCl2 18. What happens when equimolar quantity of Na2SO4 and
white ppt (E) cone. H2SO4 is cooled?
(C) and (D) both decolourise acidified KMnO4. Na2[Fe(CN)5NO] ,
Identify (A), (B), (C) and (D) and explain reactions. 19. Na2SO4 + carbon (A) —---------- 2----- > (B) (purple)
3. (A) gives white ppt with AgNOa soluble in aq NH3. (A) gives
CdCO3
green ppt (B) with NaOH but ppt dissolves to yellow
solution (C) when heated with H2O2. (C) gives yellow ppt (D) yellow ppt (C)
with BaCl-a. What are (A), (B), (C) and (D)? Identify (A), (B) and (C).
4. (A) gives white ppt (B) with NaOH, but (B) dissolves in 20. A colourless salt (X) has 50% Na 2SO3 and 50% H2O. What is
excess of NaOH forming (C). (C) gives white ppt (D) when the formula of (X)? How much of SO2 at NTP is obtained
H.,S gas is passed into it. (A) also gives white ppt (E) with when 2.52 g of (X) reacts with excess of dil. H2SO4?
BaCl0. (E) is insoluble in cone. HNO3. Identify (A), (B), (C), 21. Na2CO3 is widely used in softening of hard water. If 1L of
(D) and (E).
hard water required 0.0106 g of Na„C03, calculate
5. PbO2 is soluble in NaOH and also in HC1. What does it hardness in ppm (parts per million i.e., 10^mL) ofCaC03.
reflect about the nature of PbO, ? 22. Silver can be extracted from silver coin and Na2CO3 is used
6. A mixture contains Cr3* and Zn2+ salts. How will you in one of the steps. Outline the extraction process.
separate and analyse them? 23. 3 moles of mixture of NaHCO,U and Na,CO 3 is strongly
Zu
7. Colourless salt (A) -^£9^—> (B) (gas)
heated at 200°C, when 1 mole ofCO2 is obtained. What is
the weight of residue? What is molar percentage of
CaCl2 solution
NaHCOo and Na9CO3 in the mixture?
white ppt (C) 24. Baking powder is said to contain NaHCO3, CaCHjPOJ,,
(B) gives white fumes with HC1. starch and NaAl(SO4)2. What is the function of each
(C) decolorises acidified KMnO4. content?
What is (A)? Explain reactions. 25. Fire extinguisher contains NaHCO3 and the compound (X).
8. If NaOH solution is placed in an open vessel, will there be What is the compound (X)? Explain.
any change in its weight? Explain. 26. Arrange the following in increasing solubility in water.
9. What is difference in the chemistry of microcosmic bead test (i) LiOH, NaOH, KOH
and borax bead test? (ii) Li2CO3,Na2CO3,K2CO3
NH 4.Cl (iii) LiHCO3, NaHCO3, KHCO3
10. H3PO4+2NaOH ---- > (A) ---- —> (B) ---- > (C)
(iv) LiCl, NaCl, KC1
A> Violet bead 27. What is the formula of the three oxides of potassium with
percentage of oxygen being
(E) is obtained along with gas (F) when (D) is heated. Gas (A): 17.02% (B): 29.09%
(F) turns lime water milky andK2Cr2O7/H+ solution green. (C): 45.07%
What are (A), (B), (C), (D), (E) and (F) and explain reactions. Which of them will show paramagnetism?
11. Sodium oxide solution can't be stored in Zn or Al vessel. 28. Which is most thermally stable out of Li2CO3, MgC03 and
Explain.
K2CO,?
12. There is white deposit in a glass vessel in which NaOH is
29. 1 mole of mixture of Li2CO3 and K2CO3 on strongly heating
stored. Explain.
gave 0.25 mole ofCO2. What is mole % of each in the mixture1
13. What do you consider about Na2O2, a salt of monobasic acid
or a salt of dibasic acid? 30. Which indicator, phenolphthalein or methyl orange will you
choose in the titration of K2CO3 solution with HCl?
14. How will you make distinction between Na2O2 and Na2O?
31. Identify (A) to (C) in the following:
15. Sodium forms two oxides (A) and (B) having 25.81% and
a BaCl,
41.03% oxygen respectively. What is reaction of (A) and (B) KO2+S (A) ------ 4 (B)
with
(a) H2O (b) H2SO4 (c) Cr(OH)3 (d) MnO2 A12(SO4)3 solution
16. Sodium superoxide NaO2 can’t be prepared directly from (C)
sodium, but from Na2O2. Write balanced chemical equation. 32. 10 mL of a solution of K2SO4 on reaction with BaCl, gave
How much of Na2O2 is required to prepare 11 g of NaO2?
0.233 g of white ppt. What is concentration of KJSO(
17. Fill in the blanks : solution?
1. Na2Ohas structure. (Ba = 137, S = 32, 0 = 16, K = 39)
Chapter 8: s-Block Elements — The Alkali Metals I 377

M. A gaseous mixture contains N2, O2 and CO2. It is passed (b) complex formation
(c) solubility ofC03', POjJ" andF" inH2O
over heated Mg, then over heated Cu and finally into
aqueous KOH. What happens? (d) nature of Cl"
(e) nature of bicarbonate
34. Li + N2 (A) - H2°> (B) + (C) gas
36. Sodium does not form carbide Na2C2 with carbon but Na2C2
,, A can be formed by other source. What can be it?
(D) + (E) gas 37. Which is more stable out of
What are (A), (B), (C), (D) and (E)? (i) Li2O and Li2CO3
35. Compare Li+ and Na+ salts in terms of (ii) Na2O or Na2CO3
(a) formation of ionic carbide

Answers
ii. Zn or Al reacts with Na2O (aq), (actually with NaOH)
1. CO +NaOH ---- > HCOONa Zn + Na2O + H2O ---- > Na2ZnO2 + H2
and base
CO is acidic oxide in this reaction. 2A1 + Na2O + 3H2O---- > 2NaA102 + 3H2
2. (A):CO (B): HCOONa (sodium formate) 12. Glass has SiO2. SiO2 reacts with NaOH.
COONa SiO2 + 2NaOH---- > Na2SiO3 + H2O
(0:1 (sodium oxalate)
Na2SiO3 appears as white deposit.
COONa
CD): (oxalic acid) (E): CaC2O4 (calcium oxalate) 13. Na2O2 is a saltof dibasic acidH2O2 which is formed on reaction
H* withH2O.
CO + NaOH---- >HCOONa (COONa)2 (COOH)2 Na2O2 + 2H2O---- > 2NaOH + H2O2
(A) (B) -H2 (C) (D)
14. Na2O does not show oxidising properties while Na2O2 gives
2MnO4 + 5C2O4" + 16H * ---- » 10CO2+2Mn 2* + 8H2O
Na2O + H2O---- > 2NaOH
3. (A):CrCl3 (B):Cr(OH)3 (C): Na2CrO4 (D): BaCrO4 Na2O2 + 2H2O---- > 2NaOH + H,0 2
4 (A):ZnSO4 (B): Zn(OH)2 (C): Na2ZnO2 (D): ZnS Add Cr3* salt into aqueous solution of each
(E):BaSO4
—Na20 gives green ppt ofCr(OH)3
6. PbO2 is an amphoteric oxide
Cr3* + Na2O + H2O---- > Cr(OH)3 i
PbO2 + 2NaOH---- > Na2PbO3 + h2o green
sodium plumbate (IV) —Na2O2 oxidisesCr3* intoCrO2" (yellow)
PbO2 + 4HC1---- > 2H2O + PbCl4 Na2O2 + H2O + Cr3* ---- > CrOj"
2+ (i) NaOH, excess4 green ppt. Cr(OH)3l yellow
6. Cr^+Zn
(ii) filter | A B
—i
Na2ZnO2 (soluble) NaOH, H2O2 0 Na 0 Na
A ,, A 25.81 74.19 41.03 58.97
15.
Mol ratio 1.61 3.22 2.56 2.56
Na2CrO4
soluble (yellow) 1:2 1:1

ZnS (white ppt) (CH3COO)2Pb _____________ B__________


A
CHoCOOH
Na2O Na2O2
PbCrO4 (yellow ppt) 2NaOH (a) Na2O2 + 2H2O —>
(a) Na2O + H2O-
7. (A): (NH4 )2C2O4 (B):NH3 (C) :CaC2O4 2NaOH+ H2O2
8. NaOH absorbs CO2 from atmosphere hence weight of the (b) Na2O + H2SO4---- > (b) Na2O 2 + H2 SO4 ---- >
solution would increase. 2Na2SO4 + H2O Na2SO4 + H2O2
A (c) Na2O + Cr(OH)3---- > (c) Cr(OH) 3 + Na2O2 + H2O
9. Na2B4O7 2NaBO, + B2O3 —> Na2CrO4
no effect (yellow)
B2O3 + CuO---- > Cu(BO2)2
(d) Na2O + MnO2 (d) Na2O2 + MnO2 + H2O
In the case of borax-bead test we take borax when coloured bead
no effect ---- > Na2MnO4
of the type Cu(BO2 )2 is formed. (green)
In case of microcosmic bead test, we take microcosmic salt. Na2O2 is oxidant in (c) and
(d) due toH2Q2.
10. (A) : Na2HPO4 (B): Na(NH4)HPO4 (C): NaPO3
(D):MnSO3 (E): MnO (F):SO 2 450°^ 2NaO2
16. Na2O2 + O2
78 g 300 atm 2 x 55 g
Ca(OH)2 + SO2 ——> CaSO3i 7.8 g NaO2 11 g
milky
17. (1) antifluorite (2) 1,1.5 and 2 (3)CO2,02, submarines
Cr2O,‘/H* + SO,
378 | Essential Inorganic Chemistry

18. Prism type crystals of NaHSO4 are formed. broken and X and NaHCO3 react instantly to give CO2 which
Na2SO4 +H2SO4 ---- > 2NaHSO4 can put off fire.
19. (A):Na2S (B): Na2[Fe(CN)5NOS] (C): CdS 26. In all cases compound ofLi+, Na + and K + have solubility in the
order:
20. (X): Na2SO3 • 7H2O
Li+ < Na+ < K+
NaoSO, -7HoO+ H2SO4 ---- > Na2SO4+SO,T +8H2O
" 252 g " 22.4 L 27. (A):K2O (B):K2O2 (C):KO2
atNTP KO2 is paramagnetic (with one unpaired electron in O2) while
SO2 : 0.224 L at NTP from 2.52 g Na2SO3 • 7H2O. K2O andK2O2 are diamagnetic.
21. CaCl2 + Na2CO3 ---- > CaCO3X+2NaCl 28. K2CO3
106 g" 10*0 g
29. Li2CO3 : 25% K2CO3 : 75% (CO2 is due toLi2CO3 only
0.0106 g 0.010 g in 1 L (1000 mL) water
0.016 x 10 6 6 Li2CO3 Li2O + CO2
— g in 106 mL H2O (ppm)
io3 K2CO3 —no effect)
10 ppm CaCO3 30. Methyl orange; phenolphthalein ceases to indicate colour
boil with change after 50% ofK2CO3 has reacted with acid.
22. Silver coin 4 AgNO3 + Cu(NO3) etc.
HNO3 K2CO3 + HC1---- > KHCO3 + HC1
(other remain soluble) 31. (A):K2SO4 (B): BaSO4 (O: K2SO4 • A12(SO4)3 -24H2O
NfloCOg
■> Ag2CO3 J- Ag 32. 17.4 gL"1
A 33. (a) Mg reacts with N2 as well as O2.
23. 2NaHCO3 Na2CO3 + H2O? + CO2T 3Mg + N2 -A> Mg3N2
2 mol 1 mol 1 mol 1 mol
Na2CO3 —no effect (b) O2 if left unreacted in (a) reacts with Cu
Cu + O22 —2CuO
CO2 is due to NaHCO3 only
(c) KOH absorbs CO2
1 mol CO2 = 2 mol NaHCO3
2KOH + CO2 ---- > K2CO3 + H2O
NaHCO3 = 66.67% Na2CO3 = 33.33%
34. (A): Li3N (B): LiOH (C): NH3 CD): Li (E): N2
residue = 1 mol Na2CO3 (in mixture)
35. (a) Li forms Li2C2) Na does not form carbide with carbon
+ 1 mol Na2CO3 (from 2 mol NaHCO3) (b) [Li(NH3 )4]I exists, Na does not form
= 2 mol Na2CO3 (H2O and CO2 escape at 200°C) (c) Li2CO3, Li3PO4, LiF are insoluble while sodium salts are
24. NaHCO3 ‘ Ca(H2PO4 )2 is acidic in aqueous solution. soluble
Ca(H2PO4 )2 acidic solution of Ca(H2PO4 )2 liberates CO2 (d) LiCl is covalent while NaCl is ionic
. on reaction with NaHCO3 (e) NaHCO3 is stable in solid and aqueous solution while
starch ] as a filler LiHCO3 exists in aqueous solution but solid LiHCO3 does
NaAl(SO4 )2] retarder so that CO2 is given off more slowly not exist.
while preparing cake or bread. 36. Sodium acetylide (Na2C2) is formed when sodium reacts with
25. Fire extinguisher contains NaHCO3 and compound X can be an acetylene
acid (like CH3COOH) that can decompose NaHCO3 to give CO2. CH=CH + 2Na---- > Na—C=C—Na
NaHCO3 +H+ ---- > Na++H2O + CO2 (sodium acetylide)
(X) 37. (i)Li2O (ii)Na2CO3
NaHCO3 and acid (X) are in a sealed flask (but placed inside
separately), in case of emergency (when there is fire), seal is

Additional Reactions and Facts of 7. KF + BrF3 K[BrF4]


Alkali Metals IA 8. 2[Co(NH3)6]Cl3 + 6NaOH---- > 12NH3 + CO2O3
Reactions: + 3NaCl + 3H2O
500°C 9. 2Na2O2 + 2CO2 ---- > 2Na2CO3 + O2
1. 2NaNO3 \ 2NaNO2 + O2
800°C __ -__ 10. 4KO2 + 4CO2 + 2H2O---- > 4KHCO3 + 30a
2. 4NaNO3 —> 2Na2O + 5O2 + 2N2
A ~ 11. Na2CO3 + NO + NO2 ---- > 2NaNO2 + CO2
3. KNO3 + Zn ---- > KNO2 + ZnO
12. 2NaOH + Cl2 ---- 4> NaCl + NaClO +H2O
4. 2K0H + 4NO » 2KNO2 + N2O + H2O cold sodium
hypochlorite
4K0H + 6NO - ■> 4KNO2 + N2 + 2H2O
13. 6NaOH + Cl2 ---- > 5NaCl + NaC103 + 3H2O
5. KI + I; KI•3 hot sodium chlorate
6. KBr + ICl KIBrICI]
Chapter 8: s-Block Elements — The Alkali Metals | 379

14. 2CuSO4 + 4KI Cu2l2 + K2SO4 + I2 • Li forms LijjN (nitrides).


white ppt • Li* being of smallest size has the highest hydration
enthalpy which accounts for its high negative E° value
L2 +KI KI3 and its reducing power.
(reddish brown)
• Alkali metals dissolve in liquid NH3; solutions are
More reactions of KI are given in halogen family of
coloured due to solvated electrons. In dilute solution,
p-block elements.
ammoniated solution is paramagnetic but in
15. Mg2C3 + 4H2O * 2Mg(OH)2 +CH3—C=CH concentrated solution, unpaired electrons get paired
propyne and it becomes diamagnetic.
16. NaF + Lil—3 Nal + LiF • NaOH • nH2O (n = 1, 2, 3, 4, 5 and 7) has been
17. ^C-C1 + KF----- >C—F + KC1 prepared.
----- >>
• LiXhave partial covalent character due to smaller size
200 °C
18. 2BF3 +6NaH -^3 + 6NaF of cation.
Et2°’125^NaBH4+3NaF • Carbonates and in most cases bicarbonates also
19. BF3 + 4NaH - (except that of Li) are highly stable to heat. Along a
20. AlBr3 + 4NaH NaAlHj +3NaBr group, stability increases. LiHCO3 does not exist in
solid state.
21. 2SO2 + 2NaOH ---- > 2Na2S2O4 + H2
• LiCl is deliquescent and crystallises as a hydrate
sodium
dithionite LiCl- 2H2O.
22. COa +NaH -> HCOsNa • KQj is CC^ and CO absorber, and (X emitter. This
sodium formate finds application in submarines, space capsules and
23. TiCl4 +NaH ^3 Ti + 4NaCl+H2 breathing masks.
• NaOH is prepared from electrolysis of aqueous NaCl
solution using
Important Facts of Alkali Metals — Castner Kellner cell
• Francium is radioactive; its longest-lived isotope 223rtxr_ — Diaphragm cell
has a half-life of only 21 min. — Nafion membrane cell
• Li and Mg, show diagonal relationship. • Na2CO3 is prepared by Solvay process.
• Among the alkali metal K, Rb and Cs form • Alkali metals impart colour in flame.
superoxides MO2 ; O2 is paramagnetic. • KNO3 has been used for centuries as the oxidising
• Increasing stability of the peroxide or superoxide, as agent in gun powder. A mixture ofKNO3, charcoal and
the size of the metal ion increases is due to the sulphur will spontaneously react when ignited.
stabilisation of large anions by larger cation through • Li2CO3 has been used for than 40 yr as a treatment for
lattice energy effects. bipolar disorder.
Total Practice Set
(Read, Plan & Solvel)
~~~....... -xj'

MnOo
Problem 1. On the basis of the following values of (B)g ----- (C) gas
hydration energies and lattice energies, calculate heat of
solution of LiCl, NaCl and KC1. (A) + NaCl (D) + (B) gas
(Z?) + carbon + CaCO3 ---- > (E) + (F)
Hydration energy Lattice energy
AHhydr (kJ mol-1) AHy (kJ mol-’) CaCO3 (G) + (ZZ) gas
Li* -499 LiCl -840 (O + HjO---- > (Z)
Na* -390 NaCl -776 (Z) + (E)---- > NaOH + CaCO3
K* -305 KCI -703
cr -382 Solution NaCl + cone. ---- > NaHSO4 +HC1
(A) (B)
Solution AH . . =AZZk □ -AH,, MnO2
HC1 > Cl2
A solute M+X~ dissolves in water if AZZsolutlon (heat of (B)
solution) is positive. A
MnO2 +4HC1 MnCl2 +C12 +2^0
^hydr. : LiCl = -499 -382 = -881 kJ mol"1 (B) (C)
NaCl = -390 -382 = -772 kJ mol"1 A
NaHSO4 + NaCl Na2SO4 + HC1
KCI = -305 -382 = -687 kJ mol"1 (A) (D)
AHsol., : LiCl = -881 -(- 840) = -41 kJ mol"1 Na2SO4 + carbon + CaCO3 Na2CO3 +CaSO4
NaCl = -772 -(- 776) =+4 kJ mol"1 (D) (E) (F)
_____ A
KCI = -687 -(- 703) = +16 kJ mol"1 CaCO3 —CaO +CO2
(G) (ZZ)
Problem 2. Which of the following methods would you CaO + H2O -> Ca(OH)2
use to extinguish a fire of lithium, sodium or potassium (Z)
metals ? Explain why some of these are unsuitable, and
give the reaction involved? Ca(OH)2 +Na2CO3 * 2NaOH + CaCO3
(a) Water (b) Nitrogen (c) Carbon dioxide (Z) (E)
(d) Asbestos blanket
Problem 4. Propose a method of synthesising Na2CO3
Solution (a) Water can't be used since metal catches from NaCl.
fire forming hydroxide and hydrogen.
Solution
2M + 2H2O---- » 2AZOH + H,
(b) Nitrogen forms nitride with Li. NaCl(s) + H2S04(conc.) NaHSO4(s)+ HCl(g)
6Li + N2 ---- > 2Li;jN NaCl(s) + NaHSO4(s)---- » Na2SO4(s) + (HCl)(g)
(c) If metal has converted to oxide or peroxide, CO2 may Na2SO4(s) + 4C(s) Na2S(s) + 4CO(g) T
react producing O2 that would further worsen the
A
situation. Na2S(s) + CaCO3(s) —> CaS(s) + Na2CO3(s)
2Na2O2 + 2CO2 ---- > 2Na2CO3 + O2
(d) Asbestos blanket is best suited. Problem 5. In some foam-type fire extinguishers the
reactants are A12(SO4)3 (aq) and NaHCO3(aq). When the
Problem 3. NaCl in earlier days used to manufacture extinguisher is activated, these reactants are allowed to
NaOH and Cl2 involving following steps. Identify (A) to (Z) mix producing Al(OH)3(s) and CO2(g). The A1(OH)3 - C02
in the following: foam extinguishes the fire. Write net ionic equation to
A
NaCl + cone. * (A) + (B) (gas) represent this reaction.
Chapter 8; s-Block Elements—The Alkali Metals | 381

— smaller the cation


Solution A12(SO4 )3 solution is acidic due to hydrolysis. — larger the anion
A12(SO4)3 + 6H>0 2A1(OH)3 + 3H2SO4 — greater the charge
HjSO, 2H+ + SOj" — thus greater the polarising power
NaHCO3 + H+ ---- > Na+ +1^0 + C02 hence, greater the covalent nature.
(d) Smaller the size of cation, larger the charge (hence,
Al3+ + 3HCO3---- > A1(OH)3 + 3CO2 greater the charge density) then greater the
or [A1(H2O)6]3++ 3HCO3 ---- > Al(OH)3(s)l hydration. The dense charge of Li+ attracts several
+ 3CO2(g) + 61^0 layers of water molecules around it. They increase
the effective size of the ion, thus slowing it down.
Problem 6. Mention various factors involved in the (e) The small size of both the Li+ and F" ions leads to a
control of the pH of blood. very large lattice energy hence solubility is least.
Solution Several factors are involved in the control of Problem 8. Give the distinct features of structures of
the pH of blood. A particularly important one is the ratio of sodium and potassium bicarbonates.
dissolved HCO3 (from NaHC03) to H,C03 (aq. C02,
C02 + HjO). C02 is soluble in water and in aqueous Solution The structures of sodium and potassium
solution it reacts to a limited extent to produce F^CC^. bicarbonates provide interesting examples of hydrogen
I^O + CCXj HgCO^aq) bonding. The potassium salt contains a dimeric anion as
shown in the following structure.
H2CO3 + HoO H30+ + HCO3 0.
118° —-
K1=4.2x IO"7, 6.38
HCO3 + H20 H30+ + COV K2 =5.6 x IO'11, 0 C< 118° C—0
1.28A H
pK2 = 10.25 123° o---------- -^cr
Since K2<<<KX, hence is of importance and 1.27A
H---- 2.54A---- H
involved in the buffer. Then
„ „ , [HCO,] In the sodium bicarbonate, the HC03 anions form an
pH = pK'+10fW0j infinite chain
0
Generally JHCQ31 = 10 :1 so that pH (blood) = 7.4 |1.25A
[H2CO3]
If lactic acid is produced due to exercise, HC03 can be
H' <1165oV
c
112° C<"

converted to I^COg, but due to large ratio of


0
[HCO3]/ [HgCO-j], pH remains almost constant.
To control alkalinity, extra H2CO3 is required. C02 (in covalent
Problem 9. Lithium compounds have
the lungs) can be absorbed to keep H2CO3 in the blood
character. Explain.
constant and thus its pH.
Solution High polarising power of Li+ ion due to its
Problem 7. Explain the following: smaller size makes lithium compounds (particularly
(a) Why should an alkali never be put into a burette?
(b) Large quantities of rock salt are used on roads in Li+
winter (in cold countries). /
(c) Why is it that lithium salts have a greater degree of /
I

0
i
covalent character than other halides of the group?
(d) Why is it that although Li+ is far smaller than the © i
i

\ I
i
i
i

other metal ions, it moves through a solution less \


rapidly than the others?
(e) LiF has the lowest solubility of the group 1 metal
Polarisation of electron charge cloud of iodide by lithium ion
halides.
lithium halides) predominantly covalent. If LiT" is
Solution (a) It gives a deposit of a carbonate with C02 considered fully ionic compound, its dipole moment in the
in the air, which can jam the tap. gaseous state would be 11.5 debye (D) but its experimental
(b) They lower the freezing point of water, thus value comes out to be 3.25 D. This is due to distortion of
preventing ice formation. electron charge cloud of the iodide ion by the lithium ion,
(c) The small size of Li+ gives it a large polarising which results in decrease in ionic character (hence the
power hence covalent nature of its compounds decrease in dipole moment) and increase in covalent
increases. It can be viewed in terms of Fajan’s rule: character of Lil.
382 | Essential Inorganic Chemistry

- Lil > Nal > KI > Rbl > CsI (solubility)


Explanation
(a) LiF is much less soluble than LiCl, but AgF is Lattice energy —-—
much more soluble than AgCl.
- LiF has such a high lattice energy (U) that the high Since, r_ > r+, the sum will not change too much asr4
solvation energy of F" is not sufficient to make it increases. As a result the lattice energy will not
more soluble than LiCl. The difference in U is not change significantly as r+ increases. Thus decrease
so great for AgF and AgCl and the high solvation in lattice energy is not as fast as decrease in
energy of F“ determines relative solubilities. hydration energy. Thus, more decrease in
(b) Lithium salts are not involved in alums. hydration energy results in decrease in solubility.
I II Thus, solubility decreases as
- In alums, M2SO4 • M2(SO4)3 • 24^0 Lil > Nal > KI > Rbl > CsI
(M1 = alkali metal, M111 = a trivalent metal), each With a small anion as (F“), the lattice energy
alkali metal is coordinated to six separate water decreases more rapidly along a series of salts with
molecules: increasing cation size and thus solubility increases.
LiF < NaF < KF < RbF < CsF
[M(H2O)6]*[Al(H2O)6]3’'[S0j~J!
(e) Ionic mobility of alkali metals ion is in order
Lithium does not form alums, probably because the
Cs* > Rb* > K+ > Na* > Li*
small [Li(H2O)6 ]* is too small to form stable lattice
required for alum formation. - Size of alkai metal ions is in order
(c) In lithium salts, lithium is surrounded Li* > Na* < K* < Rb* < Cs*
octahedrally by six water molecules. Smaller the size, larger the hydration. Thus,Li+15
- Due to smallest size, Li* ion is hydrated to most hydrated and Cs* is least hydrated.
maximum and it is surrounded octahedrally by six Hydration results in decrease in ionic mobility.
water. Thus, Li* has least ionic mobility and Cs*
Molecules — the octahedral share faces, forming maximum, order being:
chains. Hydration : Li* > Na* > K* > Rb* > Cs*
+ oh + + Ionic mobility : Li* < Na* < K* < Rb* < Cs*
—> L14-OH, Li4-OH, -A Li— (f) On being heated in excess supply of air, K, Rb
Z ^OH/ ^OH/ and Cs form superoxide rather than oxides
and peroxides.
(d) Solubility of iodides of alkali metals in water
- As a general rule, to the large cations stabilise
is in codes larger anions. K*, Rb* and Cs* are larger cations
Lil > Nal > KI > Rbl > CsI and O2 (superoxide) is also larger anion. Thus,
But that of fluorides is in reverse oxides when AfO2 is formed, it is stabilised, hence its
LiF < NaF < KF < RbF < CsF formation is preferred to oxides and peroxides
(which are relatively smaller ions).
Master Exercises
M&lll'NUWlil

Exercise 1
(Stage 1 : Learning)
Short Answer Type Questions 14. Li2C0 3 decomposes into CO2 and Li20 but other alkali
1. Name the bicarbonate of alkali metal which does not carbonates are thermally stable. Explain.
exist in solid state? 15. What is oxone?
2. Name the binary salt of potassium which shows 16. Why sodium is less reactive than potassium?
paramagnetic character and which is used is
life-saving devices? 17. Cs metal is used in photoelectric cells, Li is not used.
Explain.
3. Compare the following :
+ 18. Lithium is the only metal in alkali metals that from
(a) Li and Be2+ in ionic conductance
+ + nitride. Explain.
(b) Li and K in hydrated ionic size
(c) LiCl and NaCl in lattice energy 19. (a) What is formula of potassium superoxide?
(d) Na2C03 and Li2C0 3 in thermal stability (b) What is oxidation state of potassium?
(e) Li + and Na + in polarising power (c) What is bond-order of superoxide ion?
(f) Lil and CsI in solubility in water. (d) What is magnetic moment of superoxide ion?
4. Arrange Na + , Mg 2 + , a2-, F-, Al 3+, N� (each having 10 20. Aqueous Na2C03 is alkaline in nature. Explain
electrons) in increasing ionic radii. 21. In the laboratory, you would have noticed that NaOH
5. Arrange LiCl, NaCl, KCl, RbCl and CsCl in solution has some white deposit inside reagent bottle.
increasing lattice energy. Explain.
6. Arrange Li2C03 , Na2C0 3, !½C0 3 and Rb2C0 3 in 22. Pallets of KOH become wet when exposed to air.
Explain.
increasing melting point.
23. Name the metal which floats on water without any
7. Identify A to in the following sequence of reaction : apparent reaction with water.
(B) H + fK2 Cr20 1
Na +02 --+ (A)� (C} 24. Li resembles to element X which is not the member of
ether blue in alkali metals.
ethereal layer
(a) NameX
8. Arrange LiC104 , NaC104 , KC104 and RbCl04 in (b) What is group of XJ
increasing solubility. (c) Mention two properties which resembles to that
of Li
9. Arrange fluorides of alkali metals in increasing
solubility. 25. Lithium on heating in air forms mainly �20 (oxide)
and not peroxide (Li202). Explain.
10. Potassium cobaltinitrite is used in qualitative
analysis. Explain. 26. Give an example to verify that larger anion stabilise
larger cation.
11. Na+ (cation) is smaller than Na (atom)
27. Name the salt of alkali metal that does not form
12. You have a mixture of LiCl and NaCl. Suggest a double salts such as alums.
suitable solvent to separate the mixture.
28. Baking soda or baking powder, they are different or
13. LiCl has lower melting point than NaCL Explain
same.
384 | Essential Inorganic Chemistry

29. KHCO3 or K2CO3 cannot be obtained Solvay process. 8. Identify A to C in the following
Explain. (CHjjCOOJgPb
CrCl3 A N°2°2 > B ----- —>
30. Give two oxidising properties given by Na2O2. yellow
C
yellow precipitate
Analytical Questions
1. The ionisation energies of group 1 elements (alkali 9. (a) A white solid is either Na2O or Na2O2. A piece of
metals) suggest that caesium should be most reactive red litmus paper turns white when it is dipped
but standard electrode potentials suggest that into a freshly made aqueous solution of the white
lithium is the most reactive. Reconcile these two solid. Identify the substance and explain with
observations. balanced equation.
(b) Explain what would happen to red litmus if the
2. In the following sequence of reactions (unbalanced)
white solid were the other compound?
Li +N2 ---- > LigN
10. (A) is binary salt of alkali metal. 0.71 g ofA on heating
LigN+H2O ---- > NH3 with excess of sulphur forms (B) and which in
LigN+HoO + CCt, ---- > NH4HCO3 aqueous solution gives 2.33 g white precipitate
NH4HCO3+NaCl ---- > NaHCO3 (insoluble in cone. HC1) with aqueous BaCl2 solution.
A crystalline double salt (C) formed from A12(SO4)3
NaHCO3 ---- > Na2CO3
and (B) (Ba = 137, Al = 27, S = 32)
Calculate volume of N2 required at NTP to form (a) What is A? (b) What is B?
10.6 kg of Na2CO3. (c) What is C? (d) What is amount of C?
3. Comment on each of the following observations: 11. Calculate the energy change for the alternating
(a) The mobilities of the alkali metal ions in aqueous reaction that yields chlorine sodium (Cl+Na-) and
solutions are: then explain that why sodium chloride is prepared
Li+ < Na+ < K+ < Rb+ , Cs+ 2Na(s) + Cl2(g) ---- > 2C1+Na"(s)
(b) Lithium is the only alkali metal to form a stable Lattice energy ofCl+Na“ = - 787 kJ mol-1
nitride. Electron affinity of Na = - 52.9 kJ mol-1
(c) The solubility of sodium sulphate in water Ionisation energy of Cl = + 1251 kJ mol-1
increases up to 32°C and thereafter decreases. BE of Cl2 = 244 kJ mol-1
(d) E° for M+(aq) + e~ M(s) is nearly constant
Heat of sublimation of Na(s) = 107.3 kJ mol-1
for the alkali metals.
(e) For the replacement of chlorine in organic AH^NaCl) = - 411 kJ mol-1
compounds by fluorine by the autoclave reaction Also make Born-Haber cycle for Cl+Na"(s)
^C—Cl + MF---- > ^C—F + MCI
NCERT Problems
KF is a better reagent than NaF.
1. The alkali metals follow the noble gases in their
4. One mole each of CJ^OH and CH=CH is treated atomic structure. What properties of these metals can
with excess of Na metal. be predicted from this information?
(a) Write reactions 2. What happens when
(b) In which case, yield of H2 is better (in terms of (i) sodium metal is dropped in water?
volume of H2 at STP) (ii) sodium metal is heated in free supply of air?
(c) Compare % yield (H^ in terms of mass of (iii) sodium peroxide dissolves in water?
reactant.
3. Name the alkali metals which form superoxides when
5. Explain polarisation by Li+ ion. heated in excess of air.
6. Na and K are strong reducing agents. Give examples. 4. Name the metal which floats on water without any
7. Identify A to C in the following : apparent reaction with it.
Na2O2 Cl2 Na2O2/H+ 5. List three properties of lithium in which it differs
Mn(0H)2 ---------- > (A)A' * (B) ------------- >
green violet from the rest of the alkali metals.
(C) 6. Name the chief factor responsible for the anomalous
colourless behaviour of lithium.
Chapters: s-Block Elements—The Alkali Metals | 385

7. When an alkali metal dissolves in liquid ammonia the 17. How do the following properties change on moving
solution acquires different colours. Explain the from group 1 to group 2 in the Periodic Table?
reasons for this type of colour change. (i) Atomic size (ii) Ionisation enthalpy
8. What is meant by ‘diagonal relationship’ in the (iii) Density (iv) Melting points
Periodic Table? What is it due to? 18. Like lithium in group 1, beryllium shows anomalous
9. Describe in detail the manufacture of sodium behaviour in group 2. Write three such properties of
carbonate by the Solvay process. State the principles beryllium which make it anomalous in the group.
involved in this process. 19. Comment on each of the following observations :
10. Describe three industrial uses of caustic soda. (a) The mobilities of the alkali metal ions in aqueous
Describe one method of manufacture of sodium solution areLi+ < Na' < K’ < Rb* < Cs+
hydroxide. What happens when sodium hydroxide (b) Lithium is the only alkali metal to form a nitride
reacts with (i) aluminium metal (ii) CO2 (iii) SiO2 ? directly
(c) E° for M2+(aq) + 2e~ ---- > M(s) (where M = Ca,
11. Why is that the s-block elements never occur free in Sr or Ba) is nearly constant
nature? What are their usual modes of occurrence and (d) LiF is least soluble among the fluorides of alkali
how are they generally prepared? metals
12. Compare the solubility and thermal stability of the 20. Why does the following reaction
following compounds of the alkali metals with those of
the alkaline earth metals ^C—Cl + MF ---- » C—F + MCI
(a) nitrates (b) carbonates
(c) sulphates proceed better with KF with than NaF?
13. Explain what happens when 21. Compare and contrast the chemistry of group 1
(i) sodium hydrogen carbonate is heated? metals with that of group 2 metals with respect to
(ii) sodium amalgam reacts with water? (i) nature of oxides
(iii) fused sodium metal reacts with ammonia? (ii) solubility and thermal stability of carbonates
(iii) polarizing power of cations
14. Mention the general trends in Group 1 and in Group 2
(iv) reativity and reducing power
with increasing atomic number with respect to
(a) density (b) melting point 22. Describe two important uses of each of the following.
(c) atomic size (d) ionization enthalpy (i) caustic soda (ii) sodium carbonate
15. Starting with sodium chloride how would you proceed (iii) quicklime
to prepare (state the steps only)? 23. State as to why
(i) Sodium metal (ii) Sodium hydroxide (a) A solution of Na2CO3 is alkaline?
(iii) Sodium peroxide (iv) Sodium carbonate (b) Alkali metals are prepared by electrolysis of their
16. State as to why fused chlorides?
(i) lithium on being heated in air mainly forms the (c) Sodium is found more useful than potassium?
monoxide and not the peroxide? 24. Write balanced equations for reactions between
(ii) an aqueous solution of sodium carbonate gives (a) Na2O2 and water
alkaline tests? (b) KO2 and water
(iii) sodium is prepared by electrolytic method and not (c) Na2OandCO2
by chemical method?

Exercise 2
(Stage 2: High Skill Problem Solving)
Only One Option Correct 2. Select correct statement(s).
1. Which is/are not correct configuration of s-block (a) Li2CO3 decomposes into oxides while other alkali
elements? carbonates are thermally stable
(b) LiCl is predominantly covalent
(a) (Ar) 3d10 4s2 (b) (Ar) 3d10 4s1
(c) LigN is stable
(c) Both (a) and (b) (d) None of these
(d) all of the above
386 | Essential Inorganic Chemistry

3. Alkali metals resemble IB (copper family) in following 14. Some of alkali metal salts are coloured. Na2CrO4—
respects yellow, KMnO4—pink, K2MnO4 —green. It is due to
(a) +1 valency (a) cations are coloured ions
(b) sulphates are water soluble (b) anions are coloured ions
(c) oxides are strong base (c) both (a) & (b) are correct
(d) oxides are strong acids (d) None of the above is correct
4. Select incorrect statement. 15. Na and Li are placed in dry air. We get
(a) Na2CO3 is thermally stable while Ag2CO3 (a) NaOH, Na2O, Li2O
decomposes into Ag, CO2 and O2 (b) Na2O, Li2O
(b) Ag’ forms complexes, Na+ does not (c) Na2O, Li2O, Li3N, NH3
(c) NaCl is water soluble, AgCl is insoluble (d) Na2O, LigN, Li2O
(d) NaCl and AgCl both give colour in flame when 16. All alkali metals form following compound except
ignited (a) amide, MNH2
5. A colourless solid (X) on heating evolved CO2 and also (b) superoxide like KO2
gave a white residue, soluble in water. Residue also (c) ionic ‘salt-like’ hydride AZH
gave CO2 when treated with dilute acid. (X) is (d) basic oxides
(a) Na2CO3 (b) CaCO3 17. Match the compounds (given in X) with their colours
(given in Y)
(c) Ca(HCO3)2 (d) NaHCO3
6. Which of the following is preferentially formed on X ______ Y
reaction of oxygen and potassium? A. Li3N I Pale yellow
B. K20 II Orange
(a) KA (b)KO2
C. Cs2O, ko2 III Bright yellow
(c)K2O (dJKjOj D. Rb2Q IV Ruby red
7. CO2 gas along with solid (Y) is obtained when sodium
A B C D A B C D
salt (X) is heated. (X) is again obtained when CO2 gas (a) IV I II III (b) II III I IV
is passed into aqueous solution (Y). (X) and (Y) are (0 I II III IV (d) None of these
(a) Na2CO3, Na2O (b) Na2CO3, NaOH 18. Which of the following is bestCO2 absorber as well as
(c) NaHCO3, Na2CO3 (d) Na22CO
(d) Na NaHCO;3
CO33, , NaHCO source of Qj in submarines?
8. Which salt can be used to identify coloured cation? (a) K02 (b) NaOH
(a) Borax (b) Microcosmic salt (c) KOH (d) LiOH
(c) Both (a) and (b) (d) None of these 19. Following are the ionisation potential values of
9. Intermediate formed by heating microcosmic salt and (ZJ 899 kJ mol-1, (Z2) 1757 kJ mol-1,
which forms coloured bead with coloured cation is (Z3) 15000 kJ mol-1
(a) NH3 (b) H3PO3 (a) Na (b) K
(c) NaPO3 (d) H>0 (c) Be (d) Ne
10. Molten sodium chloride conducts electricity due to the 20. A compound which can be used in space vehicles both
presence of to absorb CO2 and liberate O2 is
(a) free electrons (b) ions (a) NaOH * (b) Na2O
(c) Na atom (d) Cl atom (c) Na2O2 (d) CaO + NaOH
11. Which is the major constituent of gun powder? 21. Which of the following changes is not realised in the
(a) Nitre (b) Sulphur laboratory ?
(c) Charcoal (d) Chile salt petre (a) Absorption of NO by alkaline sodium sulphite to
form a compound
12. When SO2 gas is passed into aqueous Na2CO3, (b) Combustion of metallic Mg in CO2
product formed is (c) Heating hydrated magnesium chloride to get the
(a) NaHSO4 (b) Na2SO4 anhydrous salt
(c) NaHSO3 (d) Na2SO3 (d) ‘Displacement' of chlorine from KC1O3 by iodine to
form KIO3
13. CO2 cannot be obtained by heating
22. Which is not s-block element?
(a) Na2CO3 (b) CaCO3 (a) [Ar] 4s2 3d10 4 p6 5s1 (b) Is2 2s2 2 p1
(c) Li2CO3 (d) Ca(HCO3)2 (c) ls22s12p1 (d) [Ar]4s14p1
Chapters: s-Block Elements—The Alkali Metals | 387

(c) Hydration of alkali metal ion is less than that of


23. Which will give N2O on heating?
II A.
(a) NH4NO2 (b) NH4NO3 (d) Alkaline earth metal ions, because of their much
(c) NaNO,o (d) LiNO3 larger charge to size ratio, exert a much stronger
--- ---------- ■ ’ r a.

24. Nitrate can be converted into metal oxide on heating electrostatic attraction on the oxygen of water
in case of molecule surrounding them.
(a) Li (b) Na 35. NaOH is not used in
(c) both (a) and (b) (d) None of these (a) paper industry (b) soap industry
25. Automobile grease is obtained from (c) rayon industry (d) plastic industry
(a) CH3COONa (b) C17H35COONa
(c) CH^COOLi (d) C17H35COOLi 36. Match compounds given in (X) with their uses in (Y)
X
26. Li2CO3 is I glass
A. Na2CO3
(a) added to bauxite in the electrolytic production of
B. Na2SO3 II bleach
aluminium
(b) used to toughen glass C. NaOH III SO2 absorber
(c) used as medicine as it affects the balance between D. NaOCI IV detergent
Na+ andK+,andMg2+ andCa2+ hence, correct order is
(d) all of the above are correct A B C D A B C D
27. Which cannot be used to generate (a) IV I III II (b) I III IV H
(a) Al + NaOH (b) Zn + NaOH (c) II IV I III (d) III II IV I
(c) Mg + NaOH (d) LiH + ^0 37. Match the compounds/metal in (X) with their uses in
28. There is loss in weight when mixture of Li2CO3 and (Y).
Na2C03.10H20is heated strongly. This loss is due to x Y
(a) Li2CO3 (b) Na^Oa-lO^O I. Breathing apparatus
A. Liquid sodium metal
(c) both (a) and (b) (d) None of these submarine
29. Which disproportionates into Af2O2 (peroxide) and M B. Potassium stearate II. Explosive
(metal) on heating? C. Potassium nitrate III. Coolant in nuclear reaction
(a) Li2O (b) Na2O D. Potassium superoxide IV. Soft soap
(c) Both (a) and (b) (d) None of these
hence correct order is
30. Reagent used as fixer in photography A B c D A B C D
(a) AgBr (b) AgNO3 (a) I III II IV (b) III IV II I
(c) Na2S2O3-5H2O (d) both (a) and (c) (c) II I III IV (d) IV II III I
31. Which can dissolve ^7 38. Lattice energy (numerical value) of chloride of alkali
(a) KI (b) Nal metals is in order
(c) Both (a) and (b) (d) None of these (a) LiCl > NaCl > KC1 > RbCl > CsCl
32. The deep colour produced when iodine is dissolved in (b) LiCl < NaCl < KC1 < RbCl < CsCl
a solution of potassium iodide is caused by the (c) NaCl < KC1 < LiCl < RbCl < CsCl
presence of (d) NaCl < KC1 < RbCl < CsCl < LiCl
(a) 12 (b) F 39. Which is most stable halide of alkali metal?
(c) I3 (d) £ (a) Sodium fluoride (b) Sodium bromide
33. Alkali metals dissolve in liquid NH3, then which of the (c) Sodium chloride (d) Sodium iodide
following observation are true? 40. Which is a pair of paramagnetic species?
(a) Hg gas is liberated (a) KO2,NO2 (b) K,O2,KO2
(b) Solution is blue due to the presence of solvated (c) K2O,NO2 (d) NO2,N2O2
electrons 41. Baking powder contains
(c) Solution is conducting (a) NaHCO3, Ca(H2PO2)2 and starch
(d) All of the above are correct (b) NaHCO3, Ca(H2PO2)2
34. Which is incorrect statement? (c) NaHCO3, starch
(a) The heats of hydration of the dipositive alkaline (d) NaHCO3
earth metal ions decrease with an increase in 42. Which does not exist in solid state?
their ionic size. (a) NaHCO3 (b) NaHSO3
(b) NaNO3 forms Na2O on heating (c) LiHCO3 (d) CaCO3
388 | Essential Inorganic Chemistry

43. Following compounds are used in fire-works 53. The common name, “cream of tartar" refers to
(a) LiNO3 (b) BaCl2 (a) KHC4H4O6
(c) (NH4)2Cr2O7 (d) all of these (b) K(SbO)C4H4O6
44. When NO is passed into KOH solution, products are (c) KNaC4H4O6
(a) N, (b) N,0 (d) Mg2(OH)2(C4H4O6)H2O
(c) both (a) and (b) (d) None of these 54. Which of the following mixtures cannot be prepared?
45. Which has least molar solubility in H2O? (a) NaHCO3 + Na2CO3
(a) LiCl (b) NaCl (b) Na2CO3 + NaOH
(c) KC1 (d) CsCl (c) NaHCO3 +NaOH
(d) H2CO3 + NaHCO3
Q. 46 to 49 are based on following reactions:
55. Noble gases (like He, Ne, Ar, Kr etc.) are isolated from
(A) ——> (B) (oxide) + CO2 air. One of the steps is/are
(B) + H2O ---- > (C) (a) heating air with Li or Mg
(C) + CO2 ---- > (A) (milky) (b) bubbling air into NaOH solution
(C) + NH4C1 —> (D)gas (c) both (a) and (b) are correct
(d) None of the above is correct
(D) + H2O + CO2 —■> (E)
(E) + NaCl---- > (F) 56. Which is used to treat acid indigestion?
(a) Be(OH)2 (b) KOH
(F) Na2CO3 + CO2 + H2O (c) Mg(0H)2 (d) Ca(OH)2
46. Name of the process is 57. Which of the following is not used as food
(a) Solvay (b) ammonia-soda preservatives?
(c) both (a) & (b) correct (d) None is correct (a) NaCl (b) CH3COOH
47. (A) is (c) CgHgCOONa (d) HCHO
(a) Ca(HCO3)2 (b) CaCO3 58. Which are involved in various physiological functions
(c) CaO (d) Na2CO3 in animals?
48. (B) and (C) are (a) Na+ (b) K+
(a) CaO, Ca(OH)2 (b) Ca(OH)2, CaCO3 (c) Both (a) and (b) (d) None of these
(c) CaCO3, Ca(OH)2 (d) Ca(OH)2, CaO ‘ 59. Which is used in the treatment of manic-depressive
49. (D), (E) and (F) are disorders?
(a) Na2CO3 (b) Li2CO3
(a) NH3, NH4C1, NH4HCO3 (c) ICjCOg (d) MgCO3
(b) NH3, NH4HCO3, NaHCO3
60. Select correct statement(s).
(c) NH4HCO3, Na2CO3, NaHCO3 (a) Li2CO3 is only sparingly soluble in water and no
(d) None of the above LiHCO3 has been isolated.
50. Out of AgF, CaF2, BeF2, MgF2 (b) K^COg cannot be made by a method similar to the
(a) AgF and BeF2 are soluble, and CaF2 and MgF2 are ammonia-soda process.
insoluble in water (c) Li2CO3 and MgCO3 both are thermally stable.
(d) Na2CO3 NaHCO3 -2^0 is a mineral called trona.
(b) AgF is soluble, others insoluble
(c) AgF is insoluble, others are soluble 61. Select correct statement(s).
(d) all are insoluble (a) CaCO3 is more soluble in a solution of C02 than in
H2O
51. To an acidified dichromate solution, a pinch of Na2O2
(b) Na2CO3 is converted to Na20 and CO2 on heating
and ether is added and shaken. What is observed? (c) Li2CO3 is thermally stable
(a) Deep violet colour (d) Presence of CaCl2 or CaSO4 in water causes
(b) Red colour changing to green temporary hardness
(c) Copious evolution of oxygen 62. The alkalide ion is
(d) Bluish-green precipitate (a) diamagnetic (b) represented as CH3
52. Nitrolim is a +
(c) paramagnetic (d) represented as CH3
(a) mixture of calcium carbide and nitrogen
(b) mixture of calcium cyanamide and carbon 63. Select correct statement(s).
(c) mixture of calcium cyanide and carbon (a) Mg2+ ions are necessary for the activation of
(d) mixture of NH4CN and CaCN phosphate transfer enzymes.
Chapter 8: s-Block Elements—The Alkali Metals | 389

(b) Mg is present in chlorophyll used in 72. MO3 an hydrolysis forms


photosynthesis in green plants. (a) MOH and H2 (b) AfOHandO2
(c) Operation of Na+ - K+ pumps is biological. (c) AfOH and W2O2 (d) MOHandH2O2
(d) All the above are correct statements
73. Which is used as a treatment for bipolar disorder (an
64. The alkali metals dissolve in liquid NH3, it is found illness that involves alternating periods of
that depression)?
(a) the dilute solutions are blue but the colour (a) Li2CO3 (b) K2CO3
changes to bronze with increasing concentration (d) LiCH3
(c) LiCl
(b) the blue colour is due to the presence of solvated
electrons 74. NaNO3 is converted to KNO3 by following reaction :
(c) the blue solutions are paramagnetic but the NaNO3(aq) + KCl(aq) KNO3(aq)+NaCl(s)
bronze-coloured solutions are diamagnetic
(d) all the facts given above are found In the above equilibrium, which is least soluble in hot
65. Select correct statement(s). water?
(a) Stability of peroxides and superoxides of alkali (a) NaNO3 (b) KC1
metals increases with increase in size of the metal (c) KNO3 (d) NaCl
ion 75. Gun powder spontaneously react when ignited.
(b) NaOH does not form hydrated salt Reaction can be
(c) Increase in stability in (a) is due to stabilisation of K2CO3(s) + 3CCXg)
(a) 2KNO3(s) + 4C(s)
large anions by larger cations through lattice
energy effects + N2(g)
(d) The low solubility of LiF is due to its high lattice (b) 2KNO3(s) + 2S(s) K2SO4(s) + SO2(g)
energy whereas low solubility of CsI is due to + N2(g)
smaller hydration energy
66. Which one of the following is correct sequence (c) both (a) & (b)
followed by molar ionic conductance of the ions? (d) None of the above
(a) Li+<Na+<K+<Rb+
(b) Rb+< K+< Na+< Li+ One or More Than One Options Correct
(c) Sr2+<Ca2+<Mg2+<Be.2: + 1. Select correct statement(s).
(d) Na+<K+<Li+<Rb+ (a) MO2 contains the paramagnetic ion O2
67. Which one among the following is the most soluble in (b) O2 is stable only in the presence of large cations as
water? K, Rb and Cs ‘
(a) Lil (b) LiBr (c) KO2 is paramagnetic and has orange colour
(c) LiCl (d) LiF (d) K2O has antifluorite structure
68. The stability of K^O, K2O2 and KO2 is in order 2. Recently sodium naphthenide (C10H8Na) has been
K^O < Kz.02 < KO2 .This increasing stability at the size used as reductant in complex formation. C10H8Na is
of metal ion increases is due to stabilisation of (a) paramagnetic (b) deep-green in colour
(a) larger cation by smaller anions through lattice (c) diamagnetic (d) colourless
energy effects 3. Sesquoxides (Af2O3) of alkali metals
(b) larger cation by larger anions through lattice
energy effects (a) have been prepared by careful thermal
(c) smaller cations by smaller anions through m.p. decomposition of MO2
(d) smaller cations by larger anions through m.p. (b) are dark-coloured paramagnetic substances
(c) are colourless diamagnetic substances
69. Af2O3 (sesquoxides) is paramagnetic in nature. It is (d) are coloured diamagnetic substance
due to presence of 4. Select correct statement(s).
(a) peroxide ion (b) superoxide ion
(a) Alkali metals ozonides are coloured and
(c) oxide ion (d) all of these paramagnetic
70. Which is/are coloured and paramagnetic? (b) Alkali metal-ozonides are colourless and
(a) KO2 (b) K2O3 diamagnetic
(c) Both (a) and (b) (d) None of these (c) Sesquoxides of alkali metals are peroxides
disuperoxides (M+)4(C^")(O2 )2
71. Suboxide of Rb has the formula
(d) Sesquoxides are paramagnetic due to presence of
(a) RbO2 (b) RbO3
superoxide ion O2.
(c) Rb2O2 (d) Rb6O
390 | Essential Inorganic Chemistry

5. Select correct alternate(s). 13. Which is/are true statement(s)?


(a) In NaHCO3, the HCO3 ions are linked into infinite (a) The heats of hydration of the dipositive alkaline
chain earth metal ions decrease with an increase in
(b) In KHCO3, a dimer is formed by H-bonding. their ionic size
(c) In NaHCO3, a dimer is formed by H-bonding (b) Hydration of alkali metal ion is less than that of
(d) In KHCO3, the HCO3 ions are linked into infinite HA.
chain (c) Alkaline earth metal ions, because of their much
6. Select correct alternate(s). larger charge to size ratio exert a much stronger
electrostatic attraction on the oxygen of water
(a) All alkali metals form solid bicarbonates
molecule surrounding them
(b) Except LiHCO3, all alkali metals bicarbonates are
(d) None of the above statements is correct
solid
(c) Li2CO3 decomposes into CO2 and Li2O 14. Which are true statements about s-block elements?
(d) Na2CO3 decomposes into CO2 and Na2O (a) Metals are obtained by the electrolysis of fused
chlorides
7. Select corrrect alternate(s).
(b) Oxides are basic except BeO
(a) Solid LiNO3 and NaNO3 and deliquescent
(c) +1 valency by alkali metals and +2 valency by
(b) KNO3 is used in preference to NaNO3 in
alkaline earth metals is shown
gun powder
(d) Carbonates are thermally stable
(c) NaNO3 is used in preference to NaNO3 in
gun powder 15. Li2CO3 is
(d) NH4HCO3 exists in solid state (a) added to bauxite in the electrolytic production of
8. LiAlH4 can reduce aluminium
(b) used to toughen glass
(a) carbonyl compounds into alcohols
(c) used as medicine as it affects the balance between
(b) alkenes into alkanes
Na+ and K+, and Mg2* and Ca2+
(c) —CHO group into —CH 3
(d) thermally stable
(d) either into alcohols
9. Select correct statement(s).
(a) Radius of hydrated Li+ ion is smaller than that of
Passage Comprehension Questions
hydrated Cs+ ion Passage 1
(b) Ionic mobility of hydrated Li+ ion is smaller than Read the following passage and answer the questions
that of hydrated Cs+ ion at the end of it.
(c) Hydrated Cs+ is more conducting that hydrated Dilute solutions of alkali metals in liquidNH3 are blue.
Na+ ion
It is the ammoniated electron which is responsible for the
(d) Ionic mobility of hydrated
blue colour of the solution, and the electrical conductivity
Cs+> Rb+> K+> Na+> Li+ is due to the ammoniated cation, [M(NH3)X]+ as well as the
10. Li does not resemble other alkali metals in following ammoniated electron, [e(NH3)y]", values ofx andy depend
properties on the extent of solvation (by NH3). Dilute solutions are
(a) Li2CO3 decomposes into oxides while other alkali paramagnetic due to free electrons.
carbonates are thermally stable
(b) LiCl is predominantly covalent 1. What happens if alkali metal is allowed to react with
(c) LigN is stable concentrated ammonia solution?
(d) LiCl is non-conducting in molten state (a) Paramagnetic character of solvated electrons is
retained
11. Alkali metals resemble IB (copper family) in (b) Solvated electrons associate to form
following respects electron-pairs and paramagnetic character
(a) +1 valency decreases
(b) sulphates are water soluble (c) Reducing character is increased
(c) oxides are strong bases (d) Reducing character is not affected
(d) oxides are strong acids
2. Ammoniated solutions of alkali metals are reducing
12. Na+ and Ag+ differ in agents due to the presence of free or solvated
(a) NajjCOg is thermally stable while Ag2CO3 electrons and can reduce
decomposes into Ag, CO2 and O2 (a) O2 to O$~
(b) Ag+ forms complexes, Na+ does not (b) K2(Ni(CN)JtoK4[Ni(CN)4]
(c) NaCl is water soluble, AgCl is insoluble (c) both (a) & (b)
(d) NaCl and AgCl both give colour in flame when (d) none of the above
ignited
Chapter 8: s-Block Elements—The Alkali Metals | 391

Passage 2 4. Following are some statements about species C.


Select correct statement(s).
Questions given below are based on the following
values of hydration energy and lattice energy. (a) CisNHjCl
(b) CisNaHCO3
Hydration energy Lattice energy (c) C is recycled into Ca(OH)2 chamber to form NH3
AWhydr. (kJ mol-1) AHy (kJ mol'1) (d) CisNa2CO3
Li* -499 LiCl -840
Na* -776
Passage 4
-390 NaCl
K*
Read the following passage and answer the questions
-305 KCI -703
given at the end.
cr -382 One of the cheif uses of sodium carbonate has been in
the conversion of Ca(OH)2(s) to concentrated aqueous
1. Which salt has maximum heat of hydration?
solution of NaOH which in turn was used in the
(a) LiCl (b) NaCl
manufacture of soap
(c) KCI (d) LiCl and KCI equally
Ca(OH)2(s) + 2Na+(ag) + CO^“(ag) CaCO3(s)
2. Heat of hydration (numerical value) of LiCl, NaCl and
KCI in the increasing order is + 2Na+(aq) + 2OH"(ag)
(a) LiCl < KCI < NaCl (b) LiCl < NaCl < KC1 In 1775, in order to gain independence from the
(c) LiCl = KCI < NaCl (d) KC1 < NaCl < LiCl importation of natural Na2CO3, the French government
offered a prize to any one who could devise a process to
3. Maximum heat is absorbed in the dissolution of one
prepare Na2CO3 from NaCl.
mole of
(a) LiCl (b) NaCl 1. Who first devised a process of preparing Na2CO3 from
(c) KCI (d) equally NaCl?
4. Which salt can be used to control humidity? (a) Solvay (b) Nicolas Leblanc
(a) LiCl (b) NaCl (c) Peterson (d) Hofmann
(c) KCI (d) None of these
2. A more economical process but based on same
principle was given by
Passage 3 (a) Solvay (b) Leblanc
Answer the questions based on following flow-sheet. (c) Williamson (d) Haber
NH3 + H2O + CO2 NH4HCO3 + NaCl 3. What is natural source of Na2CO3?
I Filter (a) Nitre
I
I D (b) Chile salt peter
I D A C
I T (c) Dolomite
I I
I
I CaCO3 I (d) Trona
I
I I
I 1 4. Reaction given in the passage is spontaneous because
I I
_____ I
H2O I (a) NaOH formed is ionic
Ca(OH)2 I
CaO I (b) CaCO3 is removed from the reaction mixture
I
1
1
I
I
(c) reaction is exothermic
(d) reaction is endothermic
1. This flow-sheet represents 5. NaOH can also be prepared by electrolysis of aqueous
(a) Solvay process of NaOH NaCl. Amount of NaOH formed when 0.445 L of NaCl
(b) Solvay process of Na2CO3 (at?) is electrolysed for 137 s with a current of 1.08 A,
(c) Dow process of Na2CO3
is
(d) none of the above is correct
(a) 0.09 g
2. Filtrate A contains (b) 0.12 g
(a) NH4C1 (b) Na2CO3 (0 0.06 g
(c) NaHCO3 (d) CaCO3 (d) amount of NaCl has not been given
3. Species B formed is
(a) NH3 6. pH of the above solution (in Q. 5) is
(b) CO2
(c) Na2CO3 (d) NaHCO3 (a) 11.70 (b) 11.53
(c) 11.83 (d) data is incomplete
392 | Essential Inorganic Chemistry

Passage 5 Passage 7
NaHCO3 can be used on its own to make cakes or bread Sesquoxides “M2O3” have been prepared for K, Rb and
“rise”. Baking powder is more commonly used, and Cs. Ozonides AfO3 have been prepared for Na, K, Rb and Cs.
contains NaHC03, Ca(H2PO4)2 and starch. An improved Answer the following questions.
combination baking powder contains about 40% starch, 1. Which are paramagnetic in nature?
30% NaHCO3, 20% NaAl(SO4)2 and 10% Catf^POJ,,. (a) M2O3,MO3,MO2 (b) M2O2,M2O
Answer the following questions based on above study. (c) Both (a) & (b) (d) None of these cases
1. Rise in bread and cakes is due to .......... when it 2. Sesquides is said to be double oxide of
decomposes between 50°C and 100°C (a) oxide, peroxide (b) oxide, superoxide
(a) CO2 (b) HoO (c) peroxide, disuperoxide (d) oxide, disuperoxide
(c) both (a) and (b) (d) None of these 3. A typical sesquoxide of Rb/Rb4O6 can be represented
2. Ca(H2PO4)2 in baking powder as
(a) behaves as an acid in moist condition and (a) [Rb4(O|')(O2)2] (b) Rb^O2"),;
decomposes NaHCO3 to give CO2 (c) RbO2,(Rb2O2)2 (d) Rb20(Rb202)2
(b) behaves as a base in moist condition and gives
HaPO4 Passage 8
(c) behaves as a binder Reduction potentials of group 1 (alkali metals) are
(d) behaves as a filler given below:
3. Filler in baking powder is Element E°(U)
(a) NaHCO3 (b) Ca(H2PO4)2
Li*+ e" —> Li -3.045
(c) starch (d) NaAl(SO4)2
Na * + e~ —> Na -2.714
4. In improved baking powder, NaAl(SO4)2 K+ + e_ —> K -2.925
(a) makes the decomposition of NaHCO3 faster Rb* + e~ ---- > Rb -2.925
(b) slows the decomposition of NaHCO3 Cs* + e~ —> Cs -2.923
(c) acts as filler
(d) makes the bread rise Answer the following questions.
1. Which is the best reducing agent?
Passage 6 (a) Li (b) Na
The crystal structures of NaHC03 and KHC03 both (c) Rb (d) Cs •
show hydrogen bonding, but are different. Answer the 2. Which is the poorest reducing agent?
following questions based on this (a) Li (b) Na
0—H---0
i2- (c) Rb (d) Cs
0 —c C-0 3. E° can be related to
(a) AG° (b) AH0
0 (c) both (a) and (b) (d) None of these
dimer (I)
4. Li(s) Li+(aq) + e" involved
0
(a) sublimation (b) ionisation
(\ 0
c 'H CX
(c) hydration (d) all of these
•C
0
0 o Assertion & Reason
chain (II)
Codes:
1. Which structure is adopted by NaHCO3 ? (a) Both A and R are true and R is the correct
explanation of A,
(a) I (b) II (b) Both A and R are true but R is not the correct
(c) Both of these (d) None of these explanation of A.
2. Which structure is adopted by KHCO3 ? (c) A is true but R is false.
(a) I (b) II (d) A is false but R is true.
(c) Both of these (d) None of these 1. Assertion (A): Li+ forms series of trihydrates
3. Which is used in baking powder? LiX • 3HaO [X = Cl, Br, I, C1O2, C1O4, MnO4, NO3, BF4)
(a) LiHCO3 (b) NaHCO; 3 Reason (R): This is due to its smaller size (Li+) and
(c) KHCO3 (d) Na2CO3 highest ion dipole interaction.
Chapter 8: s-Block Elements—The Alkali Metals | 393

2. Assertion (A): Lithium does not form alums. Reason (R): Smaller the radius of the hydrated
Reason (R): Small [Litt^C) 6 ]* is too small to form cation, lower is the electrical conductance in aqueous
solutions.
a stable lattice required for alum formation.
15. Assertion (A): Ether can extract LiCl from the
3. Assertion (A) : Alkali metals form solid aqueous solution of a mixture of LiCl, NaCl and KC1.
bicarbonates.
Reason (R): Hydration of Li+ ion is greater than
Reason (R): LiHCO3 exists in aqueous solution. that of Na+ and K+ ions.
4. Assertion (A): NH4HCO3 exists in solid state. 16. Assertion (A): NaOH cannot be stored in a vessel
made of Al or Zn.
Reason (R): Alkali metals form solid bicarbonates. Reason (R): A protective layer of oxide is formed
5. Assertion (A): NaHCO3 and KHCO3 exist in solid on the surface of the metal.
state.
Reason (R): Alkali metal ions are basic in nature. True & False
6. Assertion (A) : K^COg cannot be prepared by 1. KgOj is paramagnetic but KO2 is diamagnetic.
solvay process. 2. When aqueous KO2 solution reacts with CO2, O2 is
Reason (R) : KHCO3 being fairly soluble in water formed thus KO2 is used in submarines.
does not precipitate in carbonation tower. 3. Borax or microcosmic salt can be used to identify
coloured cations.
7. Assertion (A) : CsF2 could not be formed from its
4. Li2CO3 and Na2CO3 are thermally stable.
elements.
Reason (R): Enthalpy of formation of CsF is much 5. When 1 mole of Na2CO3 is heated strongly, 1 mole of
larger which makes CsF2 unstable. CO2 is formed.
8. Assertion (A) : The superoxide MO2 contains the 6. Solution of Na in liquid NH3 is of blue colour due to
paramagnetic ion O2. solvated electron.
7. Lattice energy of alkali metal chlorides is in order:
Reason (R) : O2 is stable in presence of large
LiCl > NaCl > KC1 > RbCl > CsCl
cations as K, Rb, Cs. 8. LiHCO3 does not exist in solid state.
9. Assertion (A) : FrCl has CsCl structure with 9. Polarising power of Li+ ion is nearly the same as that
coordination number 8. of Mg2* ion.
Reason (R) : CN 8 is favoured by the large cation 10. Alkali metals form covalent hydrides.
size. 11. The enthalpy of hydration decreases in going fromLi*
10. Assertion (A): Fr burns in air forming FrO2. to Cs+.
12. Potassium burns in air forming potassium
Reason (R): The larger alkali metal ions stabilise superoxide.
02 (superoxide).
13. Aqueous solution of brine on electrolysis forms
11. Assertion (A): FrC104and Fi^PtCl6 are the salts of sodium.
low solubility. ‘ 14. Carnallite is KC1 • MgCl2 • GH^O.
Reason (R) : All alkali metal ions form insoluble 15. Li2SO4 forms double salts with A12(SO4)3.
salts.
16. K3[Co(NO2)6 ] is insoluble in water.
12. Assertion (A): Li is used in photocells.
17. KHCO3 is less soluble than NaHCO3 and is thus
Reason (R) : Cs has lower ionisation energy than prepared easily by following reaction.
Li.
NH4HCO3+KC1 ---- > NH4C1+KHCO3
13. Assertion (A): The mobility of sodium ion is lower 18. Alkali metal hydrides are ionic in nature.
than that of potassium ion.
19. The solubility of carbonates and bicarbonates of alkali
Reason (R): The ionic mobilities depend on the metals increases down the group.
effective radius of the ion.
20. Zinc displaces Hj on reaction with aqueous NaOH as
14. Assertion (A): Among the alkali metals, lithium well as with dil. HC1.
salts exhibit the least electrical condutance in
aqueous solutions.
394 | Essential Inorganic Chemistry

Fill in the Blanks 24. Sodium silicate dissolved in water, and its
concentrated aqueous solution is called
1 of IA and of IIA show diagonal
relationship. 25. The Castner’s process uses molten for the
2 is binary oxide of alkali metal. extraction of sodium.
3. NaOH can’t be stored in the vessel made of
4 can react with NaOH as well as with HC1.
Matrix-Match Type Questions
This salt gives CO2 on heating. 1. Only One Correct
5. The fact that Li has maximum reducing nature is due 1. Match the commercial names (in List I) with the
to its maximum negative value which is chemical formulae (in List II).
actually in aqueous solution M*(aq) + e~ ---- > M(s). List II
List I
6. The first element of a group in many ways differs from
A. Caustic potash 1. Na2CO3
the other heavier members of the group. These
differences arise on account of three factors : B. Common salt 2. Na2SO4-WH2O
C Soda ash 3. NaCI
(i) (ii) (in)
D. Lithia 4. KOH
7. Alkali metals dissolve in liquid ammonia to give
coloured solutions which upon standing E. Caustic soda 5. NaHCO;'3
slowly liberate H2. The colour of F. Baking soda 6. KNO3
metal-ammonia solutions is due to the presence of G. Glaubers’ salt 7. Li2O
electrons.
H. Salt petre 8. NaOH
8. Of the alkali metals only forms nitrides.
2. Select the metals (in List I) with the specific
9. Of the alkali metal carbonates, only
decomposes to give CO2 and metal oxide. properties of the compounds (in List II).
10. Alkali metal and alkaline earth metal chlorides can List! List II
be detected by test. A. Li 1. Superoxide
11. KO2 is absorber and producer and is B. Na 2. Lewis acid—a chloride
thus used in C. K 3. Most negative value of E°edn
12. A solute dissolves in water if AHhydration is D. Be 4. Thermally stable carbonate

than AHy. E. Mg 5. No flame colouration of the chloride

13. Of the O2, O|~ and O2, has odd order bond and, 3. Match List I (Compounds) with List II (Associated
and are paramagnetic. Uses).
14. is used to control humidity. List! List II (Associated Uses)
15. LigN is decomposed by water forming gas. (Compounds)
A. KO2 1. Baking
16. Lithium ion has the hydration enthalpy of
B. NaHCCT'3 2. Photovoltaic cell
all the alkali metals.
C. LiCI 3. Submarine
17. Alkali metals are good agents.
D. Cs 4. Humidity control
18. The phenomenon of crumbling to powder of sodium
carbonate crytals on exposure to air is called II. One or More Correct
19. Potassium salts impart colour to the flame.
1. Match the species in Column I with given property
20. Aqueous solution of Na2CO3 is due to (ies) in Column II.
of Column I Column II
21. i?COOH decomposes NaHCO3 to liberate CO2 which A. Alkalide of halonium 1. Paramagnetic
comes from B. Superoxide of alkali metal 2. Diamagnetic
22. Sodium metals when dissolved in ammonia gives a C. Peroxide of alkali metal 3. Life saving devices
solution. D. Oxide of alkali metal 4. Oxidant
23. Sodium metal when dissolved in ammonia conducts 5. Bleaching agent
electricity because of 6. Dissolves Zn, Al
Chapter B : s-Block Elements-The Alkali Metals I 395
2. Match the species in Column I with given property 6. Na2CO3 removes permanent hardness :
{ies) in Column II. CaSO4 + N32CO3 --> Na28O4 + CaCO3
Column I Column II 10 L of hard water required 0.0848 g N f¼CO3 • Thus,
A. Li 3N 1. Paramagnetic hardness in ppm ofCaCO3 is ............
B. LiCI 2. Ether soluble
C. K02 3. Humidity control
7. Potassium superoxide has ............ unpaired
D. Rb 20 3 4. Automobiles air bags
electron(s).
5. Coloured compounds 8. Bond order of oxide in sodium peroxide is ............

Integer Answer Types Test Yourself on Periodic Table


This section contains 8 questions. x y z w 1. Mark the element which is invariably bivalent and
Th: answer to each of the questions is a @ @ @ @ whose oxide is soluble in excess of NaOH and its
I sin;ledigit integer, ranging from Oto 9. dipositive ion has a noble gas core
-
----
G) G) G) G)
Tht appropriate bubbles below the ,_
l?Sfectively question numbers in the � ® (2) (2)

ORS have to be darkened. For example, @ @ @ @


if the correct answers to question © © © ©
number X, Y, Zand W (say) are 6, 0, 9 ® ® ® ®
and 2, respectively, then the correct @ @ @ @
darkening of bubbles will like the (!) (!) (!) (!)
f1>llowing : @ @ @ @
® ® ® ® 111111111111111
Seti
2. Identify the elements in the Periodic Table:
1. Unpaired electron(s) in superoxide ion ......... A : which shows diagonal relationship to Be
2. Potassium sesquoxide has actually ......... 0 atoms B : which shows diagonal relationship to Li
3. When one mole ofNa 2CO3 is heated, CO2 obtained is C : whose alkali metal carbonate is decomposed on
......... mole. heating
D : coinage metals
4. Number ofHCO3 ionsjoined by H-bonding inKHCO3
----
'
is .........
5. WhenLiNO3 is heated, change in oxidation number of
--
>--
,_

Nis .........
6. When NaNO3 is heated, change in oxidation number
ofNis .........
7. Number ofs-electrons in Na- is .........
8. Number of elements in second period showing
diagonal relationship is ......... 11111111111111 I
3. Out ofthe elements marked A, B, C, D, E, F, G, and H
Set II (a) which form superoxide?
1. Number ofradioactive elements ins-block is ............ (b) which form thermally stable carbonate?
2. Hydrated sodium sulphite has 50% �O. Number of (c) which forms strongest base?
molecules of hydration is ............ (d) which show diagonal relationship?
3. One mole oflithium nitride is decomposed by �O and
resultant solution is neutralised by HCI. Number of
-
(e) which forms amphoteric oxide?
,_

A
moles of HCl required is ............
4. Cl2 disproportionates to NaCl andNaC1O3 on passing E F
o
into hot NaOH solution. In this process each mole of GH
Cl2 consumes ............ mole(s) of NaOH.
5. K02 is CO2 absorber and 02 producer as shown'
4KO2 + CO2 --> K2CO3 + xO2
Value ofx is ............ 11 I 11111111111 I
Answers & Solutions
Master Exercises
Exercise 1
Short Answer Type Questions 11. Na ---- > Na+ + e~
1. LiHCO3 V When cation is formed, outermost shell is removed (in
2. KO2 : It absorbs CO2 and releases O2 thus is used in case of alkali metal) and also due to increase in force of
attraction, outermost shell is pulled inward. Hence, ionic
life-saving devices. six decreases.'
4KO2 + 2CO2 ---- > 2K2CO3 + om2
Z Z if O it
12. In aqueous solution of LiCl and NaCl, add ether. LiCl
It is paramagnetic due to unpaired electrons in O2 being covalent is extracted into ether. Evaporate ether
(superoxide ion). when LiCl is left as residue.
3. (a) Smaller the size, larger the charge hence greater the 13. Li+< Na+(size)
hydration and smaller ionic conductance.
Thus, polarisation by Li+ is larger than that of NaT.
Li+ Be2* Thus, LiCl is more covalent than NaCl. Hence, m.p. of
> Size LiCl (being covalent) is lower than that of NaCl.
< Charge 14. Refer text
< Hydration 15. Oxone is the commercial name for the preparation of
> Ionic conductance Na2O2 (sodium peroxide)
(b) Li+ >K+ 2Na+O2 ---- > Na2O2
(c) LiCl > NaCl (smaller cation Li+ polarises to a greater Na2O2 is used in life-saving devices in submarines.
extent than Na+) Na2O2 absorbs CO2 and releases O2
(d) Na2CO3 >Li2CO3 2Na2O,+2CO2 ---- > 2Na2CO3+O2
(e) Li+ > Na+ 16. Along a group, ionisation energy decreases and thus
(f) LiI>CsI tendency to oxides increases.

4. rn « (for isoelectronic species)


Na ---- > Na+ + e~ A
K ---- > K+ + e~ A'<A
Al3+ < Mg2+ <Na+ <F-<O2-<N3’ Thus, Na is less reactive than K.
5. CsCl < RbCl < KC1 < NaCl < LiCl 17. Lithium has highest ionisation energy while caesium
has the lowest among the alkali metals. Photoelectric
6. Li2CO3 < Na2CO3 < K2CO3 < Rb2CO3 cells are based on emission of electrons. Hence, Cs with
least ionisation energy has better tendency (as compared
7. A: Na2O2 to Li) to emit electrons hence thus finds use in
B: HA photoelectric cells.
C : CrO3 (chromic anhydride blue in ethereal layer) 18. N3" is very small in ionic size and thus can be set into
crystalline lattice with smaller Li+ ion. Thus, Li3N is
8. RbC104 <KC1O4 < NaC104 < LiC104 formed
(Due to sharp decrease in hydration energy compared to 6Li + N2 ---- > 2Li,N
3
lattice energy). 19. (a) KO2 (with superoxide as -1)
9. LiF<NaF<KF<RbF<CsF (b) K+O2 thus K = +1
With smaller ion (as F") decrease in lattice energy more (c) O2(17e~)has M.O. electronic configuration as
than decrease in hydration energy. als2 o*ls2 o2s2 a*2s2 a2p2 n2p2 7t2p2
10. Low solubility of K3[Co(NO2)6] is used to detect n*2p2 K*2p\
potassium ion in solution in qualitative analysis. Nb = Electrons in bonding MO = 10
Na = Electrons in anti-bonding MO = 7
Chapter B: s-Block Elements-The Alkali Metals I 397
10-7
Number of bonds= N8 -NA = - - = 1.5 30. (a) 2Na 202 + M n(OH)2 ----+ Na 2 Mn04 + 2NaOH
2 2 green
Number of unpaired electron= N= 1 (b) 3Na 202 + 3H2 0 ---+ 6NaOH + 3 [OJ
Th us, magnetic moment= ✓N<N + 2) BM 2Cr(OH)3 + 4NaOH +3 [OJ � 2Na 2Cr04 + 5H20
= ✓3BM 2Cr(OH)3 + 3Na 202 ----+ 2Na 2Cr04 + 2Na0H
Green Yellow
It is paramagnetic due to unpaired electron. p pt
ZO. Na 2C03 is salt of weak acid (H2C03 ) and strong base
(NaOH). co:- is hydrolysed forming free OH- thus it is
alkaline in nature Analytical Questions
Na 2C03 + 2H20 � 2NaOH + H2C03
f �t weak acid
1. The oxidation of a metal (M) to form M + (aq) takes place
in following stages :
2Na + +Cot +2H20 � 2Na + +20H- +H2 C03 Stage I : Sublimation

Thus, I coi- + 2H20 � 20H- +H2 C03 I M(s) ---+ M(g) tJl;ub Endothermic
Stage II : Ionisation
21. NaOH absorbs atmospheric CO2 and thus while deposit M(g) ----+ M + (g) + e-
of Na 2C03 is formed. Also Si02 of glass reacts with tJ/1:ru Endothermic
NaOH forming insoluble Na�i03 (white deposit) Stage m : Hydration
2NaOH + CO2 ----+ Na 2C03 +H20 M + (g) + H20(excess) ---+ M + (aq)
2NaOH +Si02 ----+ Na�i03 +H20 .6.H;ydrabon Exothermic
Overall: M(s) + H20(excess) ----+ M (aq) + e-
+
22. KOH is hygroscopic and thus absorbs H20. Thus, KOH
pallets become wet. It is feasible only if more energy is released in stage III
23. Lithium (density= 0.53 g mL -1) is lighter than water than that absorbed in stages I and II. Li+ being smallest
hence it floats on water. Being less reactive (due to high in size, its hydration is maximum hence energy released
IE) it does not react with H20 to a greater extent. in stage III {exothermic) is much larger than that
absorbed overall in stages I and II together. Thus, Li acts
24. (a) Li resembles Mg as strongest reducing agent and is the best reducing
(b) Mg belongs to group II (alkaline earth metals) agent.
(c) (i) MgC03 � MgO + CO2 2. Li+ N2 ---+ 2Li3 N] X 6
Li 2C03 � Li 20 + CO2 Li3 N +3H20 ----+ 3LiOH + NH3 J x 2
(ii) 3Mg + N 2 ----+ Mg 3N2 NH3 + H20 + CO2 --4 NH4HC03 ] x 2
NH4HC03 + NaCl ----+ NaHC03 + NH4Cl] x 2
6Li + N2 ----+ 2Li3 N
2NaHC03 ----+ Na 2C03 +�O + CO2
�. Li+ being smaller in size cannot stabilise larger anion as
Overall
�- (peroxide). Thus, Lip2 is not formed.
6Li + N 2 + 7�0 + CO2 + 2NaCl----+ 6LiOH + Na 2C03
26. Lil + KF � LiF + KI
+2NH4Cl
This reaction is spontaneous in forward side indicating Thus, every one mole of N2 gives enough NH3 that can be
that Li+ (smaller cation) is attached to F- (smaller anion) used to absorb C O2 and thus gives one mole NazC03
and K + (larger cation) is attached tor Oarger anion)..
N2 !!!Na 2C03
27. Li�04 is the only alkali metal salt that does not form 1 mole !!! 1 mole
double salt (as alum) as it cannot set cavity of alum. =
22.4 L at NTP 106 g
28. Baking soda and baking powder, they are different. 22.4 kL at NTP !!: 10.6 kg
Baking soda is NaHC03 and baking powder is a mixture Thus, 22.4 x 10 3 L N2 is required.
ofNaHC03, Ca(HP04 )2 starch and potassium hydrogen 3. (a) Greater the charge, smaller the radius, larger the
tartarate. hydration and thus heavier (hydrated) ion more
slower.
29. In Solvay process, intermediate reactions are
NH4HC03 +KCl ---+ KHC03 + NH4Cl ionic size --------------max
KHC03 is highly soluble in water and cannot be hydration ,_m_ax
_________ ___
separated by crystallisation. Ionic mobility------------� max
398 | Essential Inorganic Chemistry

(b) Smaller cation (Li*) can accommodate smaller anion 8. CrCl3 CrfOH)3 +3NaCl
o +3NaOH
(N3) hence, Li+ forms nitride LiN3. (A)
(c) When Na2SO4 dissolves in water, there is hydration
Cr(OH)3 + Na2O2 +H20 ---- > Na2CrO4
also (B) yellow
Na2SO4+xH2O NagSC^-xHgO, AH = -ve
Na2CrO4 +(CH3COO)2Pb PbCrO4 X
This hydration process is exothermic, hence increase
(C) yellow ppt.
in temperature shifts above equilibrium in the
direction in which heat is absorbed (i.e., backward 9. (a) Na2O2+2H2O---- > H2O2+2NaOH
side) hence, solubility is decreased. This is in C it Lt Lb

accordance with Le-Chatelier’s principle. H2O2 bleaches red litmus


(d) Refer text Thus, white powder is Na2O2.
(e) ^C+ + Cl" + M+ + F" ---- > ^>C—F + MCI (b) If it would be Na2O, then red litmus turned blue by
alkaline (NaOH) solution
only variable is U(MC\) - U(MF) Na2O + H2O ---- > 2NaOH
This opposes reactiqn but less when M+ is larger.
10. A contain oxygen since B gives white precipitate with
4. (a) CgHgOH + Na---- > CJ^ONa + AHh2.
j
BaCl2 indicating that B has SO4“.
2 ‘
1“°I 0.5 mol = 11.2 L at A is a binary salt with M+ cation and oxygen containing
46 g NTP 1 g H2 anion.
CH=CH + 2Na----> NaC^CNa
NaC^CNa ++ H2 Thus, A is MO2
1 mol 1 mole = 22.4 L at NTP
26 g 2gH2
MO2 + S ---- > A(m2so4)
z
(M + 32) g 2M + SO4'
(b) CH=CH gives better yield in terms of volume of H2 = (M + 48)
(c) % yield from CoH5OH in terms of mass . 2 ,

= — x 100 =2.174% + BaCl2 ---- > BaSO4 +2MC1


46 (233 g)
2x100 MO2 0.71
% yield from in terms of mass = = 7.692%
26 BaSO. '2.33
5. The distortion of the electron cloud of a neighbouring M + 32 0.71 71
anion by a cation is called polarisation. Polarisation 233 "2.33 233
leads to lowering of the charges on the both the ions due Af =39
to partial neutralisation of charges and thus compounds
(a) Thus, M is K (potassium)
assume covalent nature factors that favour polarisation.
(i) Smaller cation (b) BisK2SO4
(ii) Larger anion (c) C is K2SO4 • A12(SO4)3 • 24H2O
(iii) Larger charge on ion (cation/anion) (d) KgSC^ obtained from 0.71 gKO2 =0.87 g and double
(iv) Presence of d- and /--electrons salt (alum) obtained = 4.74 g
Li+ being smallest in ionic size has maximum polarising 11. Cl(g) ---- > Cl-(g)+e- f Na(g)+e" ---- > Na"(g)
tendency, and it can easily polarise larger anion as I", AH = 1251 kJ mol-1 (IE) j-AH = -52.9 kJ mol"1 (EA)
thus Lil shows covalent nature compound to Nal, KI etc.
Na"(g) + Cl+(g) —»
6. A12O,+6Na ---- > 3Na2O + 2Al Cl+Na"(s)
BeCl2+2Na ---- > Be+2NaCl i AH =-787 kJ mol"1 (fJ)
3CO,+4Na ---- > C + 2Na2CO3
|ci2(g) Cl(g) Na(s) + Aci2(g) —»
SiO2+4Na ---- > Si + 4Na2O z
Similarly with K. AH = 122 kJ mol"1 Cl+Na"(s)
7. 2Na2O2 + Mn(OH)2 Na2MnO4 +2NaOH Na(s)---- > Na(g) AH = 640.4 kJ mol"1 by
(A) green AH = 107.3 kJ mol addition
2Na2MnO4 + Cl2 2NaCl + 2NaMnO4
(B) violet AHf(Na+Cl") = -411 kJ mol"1
AHf(Cl+Na") = + 640 kJ mol"1
Na2O2+2HCl ---- > 2NaCl+H2O2
Thus, formation ofCl"Na“ is not favoured.
MnO4 is decolourised by H2O2 in acidic medium.
H2°2 ---- > 2H+ + °2 + 2e"
NCERT Problems
MnO4 + 8H++5e“ ---- > Mn2+ + 4H2O
colourless Refer Text
Chapter B : s-Block Elements-The Alkali Metals I 399

Exercise 2
Only One Option Correct 7. NaHC03 --+ Na 2C03 + H20 + CO2
10 2 (X)
I. (a) [Ar] 3 d 4s - Last filling electron is in 3 d thus it is (Y)

d-block element
I CO 2
> NaHC03
10 1
(b) [Ar] 3d 4s - same as (a) Thus, (c)
Thus, both are d-block element 8. (c) 9. (c)
Thus, (c) 10. In molten state NaCl(l) � Na + + ci-
2. (a) Li:PO:J � Li 2O + CO2 Na ,JJCJa � X Thus, (b)
Thus, (a) is true 11. (d)
(b) LiCl soluble in ether thus NaCl insoluble in ether 12. Na 2C03 + S02 ---+ Na:t'303 + CO2
covalent but soluble in H20 thus Thus, (d)
Thus, (b) is true ionic
13. Na 2C03 is thermally stable
(c) Li forms nitride Na does not form nitride Thus, (a)
Thus, (c) is true 14. Alkali salts are colourless
Thus, (d) NaCl colourless Na ,J:,rO., - yellow since Cro�- is
yellow
3. IA (Alkali) 18 (Coinage)
KCl colourless K,E:,r04 - yellow
(a) NaCl CuCI + 1 valency KMn04 - pink since MnO� is pink
thus true
CuS0 4 ,Ag2 S04
Kµn0.1 - green since Mno!- is
less soluble green
Thus, (b)
15. Na + 02 ---+ Na 20
Thus false
(d) also false Li+02 � Li20
Li + Ni --+ Li3 N
Thus, (a)
Thus, (d)
4. NaCl - golden yellow 16. Li and Na do not form superoxide M02
AgCl - no flame
Thus, (b)
Thus, (d) is incorrect
17. (a)
5. (a) Na 2C03 � no C02 18. 4K02 + 2�0 + 4C02 ---+ 4KHC03 + 302

I'
(b) CaC03 � CaO+CO2 Thus, (a)
• Ca(OH)2 19. Third IE (J3 )is very high, thus it represents alkali earth
1: metal (Be)
no gas
Thus, (c)
(c) Ca(HC03 )2 --+ CaC03 + H20 + CO2 20. 2Na 202 + 2C02 ---+ 2Na 2C03 + 02
insoluble Thus, (c)
in H20
21. (c)
(d) 2NaHC03 --+ Na 2C03 + H20 + CO2
I
HCl ) co t
2
22. (a) (s-block)
(b) (p-block)
2 2 2 1 1
Na 2C03 is water soluble (c) 1s 2s in ground state - (s-block) and ls 2s 2p in
Thus, (d) excited state.
(d) [Ar] 4s 2 - ground state
6. K02 is stable due to larger anion o; which polarises K +
[Ar] 4s 1 4p1 - excited state
Thus, (b) Thus, s-block.
Thus, (b)
400 | Essential Inorganic Chemistry

23. NH44NO,o ---- > NX2O + 2Hit2O Solution 46-50


Thus, (b) CaCO. ---- > CaO + CO,
24. 2LiNO, Li2O + 2NO2 +|02 (A) (B)
CaO + H2O ---- > Ca(OH)2
Mg(NO3)2 ---- > MgO + 2NO2 + - O2 (C)
2
Ca(OH)2 + CO2 CaCO3
Thus, (c) (A)
25. (d) 26. (d) Ca(OH)2 + NH4C1 NH,
27. Mg does not react with NaOH. (D)
Thus, (c) + co2 + h2o---- > NH4HCO3
nh3
28. Li2CO3 ---- > Li2O + CO2?
Li,CO. CE)
Naz2CO,10H2O ---- > Nait2COo3 + 10Hi2OT NH4HCO3 + NaCl -> NaHCO, + NH4C1
o it
(F)
Thus, (c)
46. (a) 48. (a)
47. (b) 49. (b) 50. (a)
29. 2Na2O ---- > Na2O2+2Na
51. Na2O2+2H+Cl_ —> H2O2 +2NaCl
Thus, (b)
30. Na.>S2O3 (as fixer) dissolves unreacted AgBr as H2O2 + Cr2O7~ + 2H+ CrO5
deep violet soluble
2Na2S2O3 + AgBr ---- > Na3[Ag(S2O3)2] + NaBr in ether
Thus, (c) Thus, (a)
31. KI + I2 KI,o 52. (CaCN2+C)
Thus, (a)
Thus, (b)
32. Refer Q. 31
53. Potassium antimonyl tartrate
Thus, (c)
Thus, (b)
33. (d)
34. 2NaNO3 ---- > 2NaNO2 + O2 54. NaHCO3 is acidic and decomposed by NaOH forming
Na2CO3
Thus, (b) is incorrect. it O

35. (d) 36. (a) 37. (b) NaHCO, + NaOH ---- > Na2CO3 + H2O
acid base
38. Smaller cation polarises anion to a greater extent hence
Thus, (c)
larger covalent nature and thus lattice energy.
Thus, (a) 55. (a) Li + N2X ---- > Li,N
O

39. (a) Mg + O2 ---- > MgO


40. KO2 (O2 is paramagnetic due to one unpaired electron) O2 and N2 are removed
NO2 (is paramagnetic due to one unpaired electron on N) (b) NaOH + CO2 ---- > Na2CO3
Thus, (a) CO2 is removed
41. (a) 42. (c) Thus, (c)
43. LiNO3 on ignition imparts crimson red, 56. (c)
BaCl2 ---- > green 57. HCHO is used as preservatives of dead biological
(NH4)2Cr2O7 ---- > green specimen.
Thus, (d) Thus, (d)
58. (c) 59. (b)
44. (c)
45. Molar solubility is in mol L (Reference concise 60. Li2CO3 and MgCO3 are decomposed by heating
inorganic chemistry J.D. Lee) Li2CO3 ---- > Li2O + CO2
LiCl = 19.6 MgCO3 —> MgO + CO2
NaCl =6.2 Thus, (c)
KC1=4.8 61. CaCO3 + CO2 + H2O Ca(HCO3)2
RbCl = 7.5
CsCl = 11.0 Na2CO3 X
Thus, (c) A
Li2CO, Li2O + CO2
Chapter B: s-Block Elements-The Alkali Metals I 401
CaCl2 /CaS04 make permanent hardness. Passage 3
Thus, (a) 1. (b) 2. (c) 3. (c) 4. (a), (c)

6!. Na - is aJkalide ion Passage 4


Na(ls 22s 22p 6 4s 1 ) +-Paramagnetic due to unpaired 1. (b) 2. (a) 3. (d) 4. (b)
electrons . 40 X 1.08 X 137
5. (c) w=zit------= 0.06 g
Na-(ls 2s 2p 3s ) +- diamagnetic due to paired
2 2 6 2
96500 X 1
electrons
6. (b) 0.06 g NaOH per 0.0445 L,
Thus, (a) O.OG
Cone. in mol L-1 = = 3.37 x 10-a M
63. (d ) 64. (d) 65. (b) 40 x0.0445
66. Smaller cation, larger hydration thus smaller molar
[OH-] =3.37 x 10
-3
M
ionic conductance
Ionic size Li+ < Na + < K + < Rb+ PoH =2 .47
p8 =11 .53
Hydration Li+ >Na + >K + >Rb +
Size ofhydrationion Li+ >Na + >K + >Rb+ Passage 5
1. (a) 2. (a) 3. (c) 4. (b)
Molar ionic conductance Li+ <Na < K < Rb
+ + +

Passage 6
Thus, (a)
57. (a) 68. Cb) 1. (b) 2. (a) 3. (b)
69. M203 actually exists as M406 [Mi0:-)(02)2 ] Passage 7
02 is paramagnetic. 1. (a) 2. (c) 3. {a)
Thus, (b) Passage 8
iD. K02 is p aramagnetic due to 02 1. (a) 2. Cb) 3. (c) 4. (d)
K203 is actually K 4O6
K 4 (0�-)(02>2 Assertion & Reason
i i 1. (a) 2. (a) 3. (b) 4. (b) 5. (a) 6. (a)
peroxide superoxide 12. (d)
7. (a) 8. {b) 9. (a) 10. (a) 11. (c)
Thus, it also paramagnetic. 13. (a) 14. (c) 15. (b) 16. (c)
Thus, (c)
il. (d) 75. (c)
72. (b) 73. (a) 74. (d) True & False·
1. F, (K02 is paramagnetic) 2. T 3. T
One or More Than One Options Correct
· 1. (a), (b), (c), (d) 2. (a), (b) 3. (a), (b) 4. F: Li2C03 is decomposes into CO2 andLi20, Na 2C03 does

f. (a), (c), (d) 5. (a), (b) 6. (b), (c) 7. (a), (b), (d) not
8. (a), (d) 9. (b), (c), (d) 10. (a), (b), (c) 11. (a) 5. F, Na 2C03 is not decomposed on heating.
12. (a), (b), (c) 13. (a), (b), (c) 14. (a), (b), (c) 6. T 7. T 8. T 9. T
15. (a), (b), (c)
10. F : Ionic hydride 11. T 12. T

Passage Comprehension Questions 13. F: Na formed reacts with Hp giving NaOH and H2•

Passage 1 14. T
I. (b) 2. (c) 15. F : Smaller size of Li cannot set into double salt.
+

Passage 2
16. T 17. F: KHC03 is more soluble.
I. (a) AH(Hydration) = AHhydration (cation)+ Al:lhydration (anion)
18. T 19. T 20. T
2. (d)

3. (c) fJIaotution =tJ{hydration -Ali,U Fill in the Blanks


AH10lutioo (LiCl) =-499 -382 + 840 1. Li, Mg 2. K02 3. Zn/ Al/Be 4. NaHC03
=-41 kJ moi-1 5. standard reduction electrode potential
NaCl= -4kJ moi-1 6. smaller size, greater charge density, lack of d-orbital
KCI =+ 16 kJ moi-1 7. blue, blue, solvated 8. Li 9. Li2C03
4. (a)
402 | Essential Inorganic Chemistry

10. flame 11. co.•2, O2, submarines 12. greater Set II


13. O2;O2,O2 14. LiCl 15. NILo 16. highest Questions —> 1 2 3 4 5 6 7_ 8
Answers —► 2 7 4 2 3 8 1 1
17. reducing 18. efflorescence 19. violet
20. alkaline, hydrolysis, CO: 21. NaHCOoo © © © © © ©I© ©
© © © ©
22. conducting 23. solvated electrons © © © © © ©
24. water glass 25. NaCl © © © © j©_©
© ©
© © ® ®© (4)
Matrix-Match Type Questions © © © © ©©__© ©
I. Only One Correct © © © © ©©._© ©
1. (A)—(4); (B)—(3); (C)—(1); (D)—(7); © ©. © © ©0 ©
(E)-(8); (F)—(5); (G)—(2); (H)-(6); © © © © © ©
2. (A)-(3); (B)—(4); (C)—(1); (D)—(2) © © © © © © © ©
(E)-(5)
3. (A)—(3); (B>—(1); (C)—(4); (D)—(2) Test Yourself on Periodic Table
II. One or More Correct 1.
1. (A)—(2); (B)—(1,3,4); (C)—(2,4,5); (D)—(2,6) Zl"-Be □_
2. (A)—(4,5); (B)—(2,3); (C)—(1,5); (D)—(1,5)

Integer Answer Types


Set I
Questions—► 1 2 3 4 5 £ 7_ 8
Answers —► 1 6 0 2 1 2 6 3
© © © © © ©I© © 2.
© © © © O C
© © © © © _© _© © B D A
© © © © © ©© © I

© © 131
® © © I

© © © © © (©_© ©
© © © © © ©.© ©
© © © 0 ©
© ® © © © ©__© ©
© © © © © © © © 3. (a) EandG (b) C,E,andG (c) G (d) AandD
(e) B
s-Block Elements
-The Alkaline Earth Metals
Group 2 family members (alkaline-earth elements) are not free from tension. The
youngest of the family (Be) and the younger one (Mg) does not have patch with other
members. Be has tied up with Al (Group 13) and Mg with Li (Group 1) - a diagonal
relationship. Ghar ka Bhedi Lanka Dhaye. n
66

• General electronic confi guration is


(Inert gas] ns2 - Alkaline earth metals; Group 2(IIA) n
signifies period and number of valence electrons in ns, its group.
Alkallne Earth Metals
• 1H
Iron EC z 2(11A) Periodicity at a Glance 1

Be2 • [He] 2s 2
4 Be • Reactions of Alkaline
3Li Be
Mg 2
+ 2
12 Mg
■ Earth Metals 4
(Ne] 3s
Ca2

[Ar] 4s 2 20 Ca Compounds of Alkaline 11 Na 12Mg

5r 2+
[Kr] 5s 2 38 Sr ■ Earth Metals a
19K 20C

Ba2 [Xe] 6s 2
56 Ba • Anomalous Behaviour
Ra 2

Ra of Beryllium 37Rb 38Sr
[Ra] 7s 2
88
• Additional Reactions ssCs 388a
Alkali earth metals form compounds with +2 oxidation state and Facts
ions, having nearest stable noble gas configuration. a1Fr 88 Ra
• Alkaline Earth Metal
• Magnesium is an important constituent of chorophyll which
Compounds
initiates the process of photosynthesis in green plants.
• Magnesium ion are concentrated more in intracellular than in
extracellular fluids in animal bodies. Their presence is also
necessary for the activation of phosphate-transfer enzymes.
These enzymes take part in the biochemical process
(exothermic) occurring in animal body. Mg2 + ions are also
involved in carbohydrate metabolism.
• Ca2 • ions are essential for the formation of bones and teeth. The
enamel on teeth is a double salt3Ca 3(PO4 )2 -CaF2 •
• M g2• and Ca2 + ions are also involved in the transmission of
electrical impulses along the nerve fibre and for the contraction
of muscles.
404 | Essential Inorganic Chemistry

Periodicity at a Glance
Detailed periodicity has been given in chapter 3. Periodicity of s-block elements is summarised below.
There is uniform gradation in properties except in case of Li and Be which being smallest in their groups shov
abnormal behaviour. Li resembles Mg and Be resembles Al that we call diagonal relationship.

Periodicity in s-Block Elements


A :Atomic number 1 2
B :Effective nuclear charge (Z * = Z - o) Li Be A
where o is screening constant B——————*
c : Number of shells Na Mg C +---------------------------------- *
D : Atomic radius D-----
E : Ionisation energy K Ca E-------------------------------►
F : Hydration and hydration energy
F--------------------- —
G : Size of hydrated ion
Rb Sr G---------------- —*
H : Polarising power (based on Fajan’s rule)
H---------------►
I : Lattice energy of chloride
J : Reactivity Cs Ba /--------------- ►

K : Ionic conductance J*-----


• From the above table, it is concluded that due to higher (IE) of K*-
alkaline earth metals, they are less reactive than alkali metals. lll II
Table 9.1 Summarises some of the properties of alkaline earth metals [Group 2(IIA)]
Table 9.1 Properties of the Alkaline Earth Metals
Properties Be Mg Ca Sr Ba Ra
Atomic number 4 12 20 38 56 88
Atomic mass 9.012 24.3050 40.078 87.62 137.327 226.0254
Effective nuclear charge 1.95 2.85 2.85 2.85 2.85 2.85
Ground state electron configuration [He] 2s2 [Ne]3s2 [Ar] 4s2 [Kr]5s2 [Xe] 6s2 [Rn] 7s2
Metallic radius, (pm) 112 160 197 215 222
Ionic radius, (pm) 31 72 100 118 135
Ionic charge density 6.45 2.78 2.00 1.69 1.48
(ionic charge/radius (A))
(/E)1tkJ mol*1 899 737 590 549 503 508
(/E)2, kJ mol*1 1757 1450 1145 1064 965 975
[(/E), + (/E)2]kJ mol*1 2656 2187 1735 1613 1468 1483
(/E)3 ,(kj mol*1) 14850 7730 4940 4150 3440
Pauling's electronegativity 1.5 1.2 1.0 1.0 0.9
^^°hydr', Hydration energy, kJ mol*1'
-2494 -1921 -1577 -1443 -1305
M2t(g)+aq —> M* (aq).

AS hydra of M2+/JK*1 mol*1 -300 -320 -230 -220 -200


AG “hydra, of M2+/kJ mol*1 -2410 -1845 -1525 -1400 -1265
AHalomjS,/kJ mol*1 326 149 177 164 178 130
Density at 25°C (g cm-3) I. 848 1.738 1.55 2.54 3.51 5
Colour Grey Silver Silver Silver Silver-yellow Silver
Enthalpy of fusion AHf“S10n/kJ mol II. 6 8,95 895 9.62 7.66 7.25
Bond enthalpy of M2 (kJ mol-1) 9.46
Electrode potential, E°, (V)]
M2*(aq) + 2e~ ----- » M (s)
-1.85 -2.37 -2.87 -2.89 -2.90 -2.92

Melting point/’C 1287 649 839 768 727 700


Boiling point/°C 2500 1105 1494 1381 1850 1140
Abundance on earth % 2.8 x 10*3 2.33 4.15 0.038 0.042 trace
Chapter 9: s-Block Elements — The Alkaline Earth Metals | 405

• Like the alkali metals they also form predominantly The atomic and ionic radii increase from Be to Ra due
ionic compounds but tendency towards covalency is to the effect of extra shells of electrons being added. This
greater, particularly with Be and Mg because of their outweighs the effect of increased nuclear charge.
smaller atomic and ionic radii. These elements are denser than group 1 metals
• Be forms compounds which are essentially covalent. because they have two valence electrons per atom for
• Alkaline earth metal ions, because of their larger bonding the atoms into a metallic lattice and as a result
charge to size ratio, exert a much stronger more mass can be packed into a smaller volume. The
electrostatic attraction on the oxygen of water density decreases slightly on moving down the group from
molecules surrounding them. Be to Ca but increases considerably there after up to Ra.
• Since the alkaline earth metals (except Be) tend to
lose their valence electrons readily, they act as strong
reducing agents as indicated by E°ed. values.
Melting Point
(Table 9.1). The particularly less negative value for Be Group 2 metals have higher melting points when
arises from the large hydration energy associated with compared to the group 1 metals. The reason being +2
the small size of Be2+ and the relatively large value of charge on the cations in the metallic lattice, causing them
heat of sublimation. to be more strongly attracted to the ‘sea of electrons’ and
• Basic nature of oxides increases down the group but making it difficult to pull them apart.
solubilities of sulphates and carbonates decrease as
ionic size increases.
Ionisation Energy
Thermal Stability of Oxy Salts The first ionisation energy (Table 9.1) of alkaline
earth metals is more than that of corresponding alkali
All the group 2 elements form oxysalts. The thermal
metals. This is because the alkaline earth metals have
stability of the oxysalts increases with the increase in
electropositivity of the metals. Thus it increases down higher effective nuclear charge and are smaller in size,
the group. The thermal stability of the salts of group 2 thereby the electrons are more tightly held to the nucleus.
is less than those of the group 1 metals. The second ionisation energy of these elements is almost
twice first ionisation energy. This is because once one
• The sulphates are stable to heat whereas the
electron has been removed, the effective nuclear charge
carbonates decompose to give MO and CO2, the
temperature of decomposition increasing from Mg to felt by orbital electrons is increased, so that the remaining
Ba. BeCO3 is kept in the atmosphere of CO.>2 to electrons are more tightly held and hence, much more
energy is needed to remove the second electron. However,
prevent its decomposition.
their second ionisation energy is less than that of the
MCO3 —> CO2 + MO corresponding alkali metals because of stability of a closed
mso4 —> so3 + mo shell configuration of the univalent cations that are
BeCO3 MgCO3 CaCO3 SrCO3 BaCO3 formed in the latter cases. The ionisation energy of
alkaline earth metals also decreases on moving down the
<100°C 540°C 900°C 1290°C 1360°C
group.
BeSO4 MgSO4 CaSO4 SrSO4
575°C 875°C 11OO°C 1375°C Lower second ionisation energies of the alkaline earth
metals as compared to those of alkali metals can be
Alkali metal nitrates decomposes into nitrites on
Aixali examplified below:
heating whereas alkaline earth metal nitrates decompose
Mg Mg+ + e~ (Electronic
on heating to metal oxide, nitrogen dioxide and oxygen. (2, 8,2) (2, 8,1)
configruration of
2Ca(NO3)2 * 2CaO + 4NO2 +O2
Mg+ Mg2* ■ e~
+ the metal and the
(2, 8,1) (2,8) ion is given in
DEEP Focus stable closed brackets)
shell
Atomic and Ionic Radii Na+ ----- > Na+ +e“
The atoms of the alkaline earth metals are smaller than (2, 8, 1) (2, 8)
stable
those of the corresponding group 1 elements. This is because of
the increase in effective nuclear charge with increase in atomic Na+ -----> Na2++e
number. Due to increase in effective nuclear charge, valence (2, 8) (2, 7)
shell electrons are pulled in more firmly by the nucleus, thereby stable closed
reducing the size of the atoms. Similarly, their ionic radii are shell
also smaller than those of group 1 elements, because the Electropositive character and the reducing property
removal of two orbital electrons increase the effective nuclear (tendency to lose electrons) increase on moving down the
charge even further.
group.
406 | Essential Inorganic Chemistry

As the alkaline earth metals may lose electrons quite Table 9.2 Enthalpies of Hydration, AHhyd, of Alkaline Eartl
easily, they form divalent cations which have a noble gas Metal Ions, M2* and Lattice Energies, AH,att, of their
structure with no unpaired electrons. Therefore, their
Oxides, Carbonates Fluorides and Iodides in kJ mol
compounds are diamagnetic and colourless, unless the
anion iis coloured. Ca, Sr and Ba compounds give A^hyd
characteristic flame colourations— which
....—i are used to Element
identify them — M2* MO MCO3 mf2 MI2
Be -2494
Ion Colour
Mg -1921 -3923 -3178 -2906 -2292
Ca2* Brick-red
Ca -1577 -3517 -2986 -2610 -2058
Sr2* Crimson
Sr -1443 -3312 -2718 -2459
Ba2* Apple-green
Ra2* Carmine-red Ba -1305 -3120 -2614 -2367

Table 9.3 Occurrence and Uses of Alkaline Earth Metals


Solubility, Lattice Energy and Hydration
Element Abundance Uses
Energy Main mineral
Beryllium 2.8 x 10^% First detected in Used in
The metal ions are easily hydrated, e.g., MgCl2 ■ GH^O,
1798 in the corrosion
CaCl2 • 6H2O, BaCl2 • 2H.,O. The hydration energies of gemstone beryl resistant alloys
these ions are much greater than those of alkali metal ions and emerald
(Table 9.2), because of their smaller size and increased (Be3AI2Si6O18).
cationic charge. The lattice energies (Table 9.2) of alkaline Magnesium 2.33%, seventh Pure Mg first When alloyeo
earth metal salts are also much higher than those of alkali most abundant prepared in with Al, Mg is
element in the 1808, named widely used as
metal salts. earth's crust after the structural
Magnesia district material
in Thessaly, because of ft
DEEP Focus Greece where high strength,
Hydration and lattice energies decrease with increase in large deposits of low density and
the mineral are ease n
size of metal ions. Decreasing lattice energy favours increased found. machining
solubility, whilst decreasing hydration energy favours
decreased solubility. If on moving down a group the hydration Calcium 4.15%, fifth most CaCO.'3. As an alloy ng
energy decreases more rapidly than the lattice energy, the abundant CaSO4 ■ 2H2O agent to harcFer.
compound becomes less soluble. element in the (gypsum), aluminium.
earth's crust obtained in pure calcium is ths
form in 1808, primary
This occurs with most of the compounds except for calcium is constituent d
fluorides and hydroxides for example, solubility of derived from the teeth and bones
sulphates decreases from BeSO4 to BaSO4. Due to their Latin word calx,
meaning "lime".
small ionic radii, Be2+ and Mg2* have high hydration
Strontium 0.038% Discovered in SrCO'3: is uses
energies. Thereby BeSO4 and MgSO4 are soluble in water. ths
1787 and named for
CaSO4 is only slightly soluble in water, whereas SrSO4 after the small manufacture di
and BaSO4 are almost insoluble in water. This solubility of town of Strontian glass for coor
sulphates is in order : (Scotland) TV picture tubes
Barium 0.042% Found in the BaSO4 is used h
BeSO4 > MgSO4 > CaSO4 > SrSO4 > BaSO4 minerals medicine as a
witherite contrast medtr:
(BaCO3) and for stomach and!
DEEP Focus barite (BaSO4) intestinal X-raj-s I
In case of florides and hydroxides, the lattice energy after which it is
decreases more rapidly than their hydrration energy. This named.
causes a reverse trend, i.e., the fluorides and hydroxides Radium traces Isolated as Used in cancer
increases in solubility on moving down the group. Thus, order chloride in 1898 radiotherapy
of solubility is from the mineral
pitchblende
Ba(OH)2 > Sr(OH)2 > Ca(OH)2 > Mg(OH)2 > Be(OH)2
Chapter 9: s-Block Elements — The Alkaline Earth Metals | 407

Group IIA (Alkaline Earth Metals) and Group Table 9.5 Reactions of Group 2 (HA)
HB (Zn, Cd, Hg) Reaction Comment

Mg acts as a bridge element between IIA (2) and M + 2H2O ----- > M(0H)2 + H2 Be probably reacts with steam,
Mg with hot water, and Ca, Sr
HB(12): ' and Ba react rapidly with cold
Ca—Sr—Ba—Ra IIA (2) water.
20 38 56 88
M+2HCI----- > MCI 2 + H2 All the metals react with acids
Be—<' Mg liberating H2. With HNO3, Be
4 \ 12 J Zn—Cd—Hg
becomes passive due to
IIB (12) formation of oxide layer.
30 48 80
Bridge element 3M + N2 —> M^2 All form nitrides at high
temperatures. Stability
M3N2 + 6H2O---- > 2NH3
Be > Mg > Ca (hydrolysis to
+ 3M(0H).'2 NH3). '
Table 9.4 Distinction Between IIA(2) and IIB(12) Elements
M + X2 —» MX2 All the metals form MX2. No
IIA IIB
Properties polyhalides are formed, (except
(Be, Mg, Ca, Sr, Ba, Ra) (Zn, Cd, Hg) (X2 = F2, Cr2, Br2, l2)
BeX2)
1 Electronic [Inert gas] ns2 [Inert gas] M + 2NH3 —> M(NH2)2 + H2 All the metals dissolve in liquid
. configuration (n - 1)cf10 ns2 NH3 to give deep blue black
solutions from which
2 Block s-block d-block ammoniates [M(NH3)6]2+ can
be recovered and form amides
3 Oxidation state +2 +2. mercury also at high temperatures.
forms dimeric
2M + O2 ----- > 2MO All the metals form normal oxide.
Hgf+ Ba also forms peroxide (BaO2).
Ba + O2 —> BaO2
4 Nature of oxide BeO is amphoteric, other ZnO is
oxides are basic amphoteric, CdO M + H2 MH2 Ionic salt-like hydrides formed at
MgO < CaO < SrO < BaO < and HgO are high temperatures by Ca, Sr and
RaO basic. Ba. Be forms electron-deficient
hydrides.
5 Nature of Electron-deficient BeX2 ZnCI2, CdCI2 are
. halides covalent, others (MX2) are ionic but less than Not with other alkaline earth
Be + 2H2O + 2OH' ----- >
ionic; HgCI
IIA, IIA,HgCI2 2is metals.
[Be(OH)4]2-+ H2(g)
MgCI2 < CaCI2 < SrCI2 covalent.
< BaCI2 M(s) + 2C(s) ----- > MC2(s) At high temperature Be forms
Be2C, ionic compounds.
6 Nature of Less soluble in water and more soluble than
. sulphates solubility decreases down IIA. M+S MS(s) The sulphides are soluble, but
the group hydrolyse if heated in water.
BeSO4 > MgSO4 >
CaSO4 > SrSO4 > BaSO4
7 Nature of Solubility of hydroxides Solubility of With Water
• hydroxides increases as we move hydroxides
down the group decreases as we • Ca, Sr and Ba have reduction potentials (E°) similar
move down the to those of the corresponding alkali metal and are
group. quite high in the electrochemical series. They react
8 Nature of Soluble ZnS, CdS, HgS
with cold water quite readily.
• sulphides insoluble and Ca + 21^0---- > Ca(OH)2 + H,
precipitate in salt
analysis. • Mg has an intermediate value and it does not react
with cold water but it decomposes hot water.
9 Reactivity Increases as we move Decreases as we
down the group move down the Mg + 211,0---- > Mg(0H)2 + Ha
Be < Mg < Ca < Sr < Ba group or Mg + H2O---- > MgO +
Zn > Cd> Hg
Mg forms a protective layer of oxide, so despite its
favourable reduction potential, it does not react readily
unless the oxide layer is removed by amalgamating with
Reactions of Alkaline Earth Metals mercury. In the formation of the oxide film, it resembles
Table 9.5 summarises reactions of group 2 (IIA). From aluminium.
this, it is evident that as we go down the group, reactivity • Be is amphoteric, as it reacts with acid as well as with
increases. base.
408 | Essential Inorganic Chemistry

Be + 2NaOH + 2^0 Na2[Be(OH)4] + 1^ its electrons with several neighbours and receives a share
sodium beryllate in their electrons to acquire a stable configuration.
Mg, Ca, Sr and Ba do not react with NaOH and thus
are purely basic. Formation of Carbides
All the metals in the Mg-Ba series of their oxides react
With Ammonia directly with carbon to give the carbides (acetylides), MCj.
• All the metals dissolve in liquid ammonia as do the These carbides are ionic in nature and have a NaCl type of
alkali metals. Dilute solutions are bright blue in structure with M2+ replacing Na+ and C=C?_ replacing
colour due to the spectrum from the solvated Cl“. Beryllium forms methanide, Be2C, with carbon and
electrons. These solutions decompose very slowly, acetylide, BeCj, with acetylene. Magnesium on heating
forming amide and H2, but the reaction is accelerated with carbon forms Mg2C3 also, which is an allylide since
by many transition metals and their compounds. with water it liberates allylene (methylacetylene).
2NH3 + 2e~---- > 2NH2 + H2 The carbides are named depending upon the
Evaporation of the ammonia from solutions of alkali hydrocarbon they liberate on reaction with water. If it
metals yields the metal, but in case of alkaline earth liberates acetylene, it will be named acetylide, and if
metals, evaporation of ammonia gives hexammoniates of methane is liberated, it will be called as ‘methanide’, etc.
the metals, which slowly decompose to give amides:
CaQz + 211,0 -----> Ca(OH)2 + GJl,
Af(NH3)6 -----> M(NH2)2 +4NH3 +H2 carbide acetylene
Concentrated solutions of the metals in * 2Be(OH)2 + CH4
Be2C + 41^0
ammonia are bronze coloured, due to the formation methanide methane
of metal clusters. Mg2C3 + 41^0 2Mg(OH)2 +CH3-CsCH
allylide allylene
DEEP Focus
Formation of Hydrides Formation of Nitrides
All the group 2 element except beryllium form hydrides, The alkaline earth metals burn in nitrogen and
AfH2, by direct combination with hydrogen. Beryllium hydride form nitrides, AfgN2. It requires a lot of energy to
can be formed by reducing beryllium chloride with lithium convert the stable N2 molecule into nitride ion, N3', and
aluminium hydride, LiAlH4. All these hydrides are reducing
agents which react with water and liberate hydrogen. Calcium, this is recovered from the very high lattice energies of the
stronium and barium hydrides are ionic and contain the alkaline earth metal nitrides. The beryllium compound is
hydride ion, H\ Beryllium and magnesium hydrides are rather volatile while others are not. They are all colourless
covalent and polymeric. (BeH2)n has an interesting structure. crystalline solids which decompose on heating and react
The polymeric solid contains hydrogen bridges between with water to liberate ammonia and form either the metal
beryllium atoms. oxide or hydroxide, e.g.,
H 3Mg(s) + N2(g) > Mg’-Njhs)
Be Be Be Mg^Nf-GO+ei^cxz) ■» 3Mg2+(OH")2 + 2NH3(g)
H
Beryllium hydride polymer
Each beryllium atom is bonded to four hydrogen atoms and
Complexation Behaviour
each hydrogen atoms forms two bonds as it is bridging two Be Complex formation is favoured by small, highly
atoms. Since Be has two valence electrons and H only one, it is charged cations with suitable empty orbitals of
apparent that there are not enough electrons to form the usual approximately the right energy with which the ligand
electron pair bonds in which two electrons are shared between orbitals can combine.
two atoms. Instead of this, three-centre bonds are formed
Alkaline earth metals form more complexes as
in which a ‘banana-shaped’ molecular orbital covers
compared to alkali metals. The tendency to form
three atoms Be...BL..Be, and contains two electrons. The
complexes (mostly with 0 and N donors) decreases with
monomeric molecule BeH2, if formed with normal bonds, would
have only four electrons in the outer shell of the beryllium atom increasing atomic number. Thus, of the heavier ions only
and would be electron deficient. Ca2+forms a complex wth ethanol. Beryllium having the
smallest ion in the group tends to form complexes most
This would make the molecule very unstable; readily. It mostly form complexes with tetrahedral
that is why BeH^ exists as a polymer. This is an arrangement because of the available orbitals as shown
example of a cluster compound in which each atom shares below:
Chapter 9: s-Block Elements — The Alkaline Earth Metals | 409

Is 2s 2p Magnesium is known to form a very important


4 Be (ground state) | | | | No unpaired electrons complex occurring in nature, viz; chlorophyll; a green
hence no covalent bond pigment of the plants which produces sugar for the plants
formation in presence of sunlight, CO2 and H2O in a process called
photosynthesis.
Be (excited state) HT] [71 111 I I Two unpaired electron
— — ——— can form £W0 COvalent Chlorophyll „
6CO2 +6H2O a ]. .. > C6H12O6 +6O2
sunlight
bonds in BeF2.
Magnesium, in chlorophyll, is coordinated to four
nitrogen atom in the heterocyclic prophyrin ring system as
Bein(BeF4)2- jf| g] Electron donated by two shown.
fluoride ions forming ^8
coordinate bonds.
However, once these are formed, all the Be—F bonds 7?7i |/?2
tend to become similar.
Mg N,
F
*6I J*3
l

7?5
F4 ^F
Skeleton of chlorophyll molecule
I I
I I
The rest of the element from calcium to barium form
complexes only with strong complexation agents such as
F acetylacetone, ethylenediamineteraacetic acid (EDTA),
In the hydrated salts, e.g., BeCO3 • 41^0 and etc. In fact, titrations are performed using EDTA in buffer
BeSO4 • 4^0, beryllium ions exist in the form solutions to estimate the amounts of Ca2* and Mg2*
present in water, to determine the hardness of water.
[BetHjO^f*, where they show a coordination number of
four.

Target Practice 1
1. Out of Be2*, Mg* and Ca 2. BeCO3 is decomposes easily. Hence, it is kept in the
(a) Which has maximum IE? atmosphere of to prevent its decomposition.
(b) Which is maximum hydrated? 3. In Group 2 (HA)
(c) Which has least size? (a) Maximum basic oxide is
(d) Which has maximum size? .... (b) Maximum soluble sulphate is
(e) Which is most stable? (c) Maximum soluble carbonate is
(f) Which is least stable? (d) Maximum soluble fluoride is
(e) Maximum hydrated ion is

Answers
1. (a) Be2* (b) Be2* (c) Be2* (d) Ca (e) Ca (f) Mg* 3. (a) RaO (b) BeSO4 (c) BeCO3 (d) RaF2 (e) Be2*
2. C02

Practice Exercise 1
1. Arrange the following properties, as indicated, in the (d) Stability ofBeSO4, MgSO4, CaSO4
increasing order (e) Size of the hydrated ions of Be2*, Mg2*, Ca2*
(a) Lattice energy of BeCl2, MgCl2, CaCl2, SrCl2 (f) Solubility ofBe(OH)2, Mg(OH)2, Ca(OH)2
(b) Solubility of BeSO4, MgSO4, CaSO4, SrSO4, BaSO,4 (g) Covalent nature of BeCl2, MgCl2, CaCI2
(c) Stability ofBeC03, MgCO3, CaCO3 (h) Hydrolysis of BeCl2, MgCl2, CaCl2
410 | Essential Inorganic Chemistry

2. Lattice energies of BeF2, MgF2 and BaF2 are given in 4. What are the possible products when Mg is burnt in air and
Table 9.2. they are decomposed by H2O?
Hydration energies of Be2*, Mg2*, Ba2* and F* are - 2494, 5. Identify (A) to (E) in the following:
-1921, -1305 and - 457 kJ mol'1. Which of the BeF2, MgF2 Mg + air —(A) + (E)
and BaF, is soluble in water? CO2
3. Arrange the following in increasing order: (A) + H2O---- > (B) •» (O
(a) Basic nature of Be, Mg, Ca, Sr (E) + H2O-----> (D) + (B)
(b) Reactivity of BeO, MgO, CaO with H,0
(c) Refractory properties of BeO, MgO, CaO white fumes
(d) Reaction of Li, Mg and Ca with N2
1 mole of (B) neutralises 2 moles of HC1.
(e) Reaction of Be, Mg, Ca with H2O

Answers
1. (a) SrCl2 < CaCl2 < MgCl2 < BeCl2 ^Solution = MHydration “ Lattice energy
(b) BaSO4 < SrSO4 < CaSO4 < MgSO4 < BeSO4 BeF2 = -3408 - (-2906)= -502 kJ mol"1
(c) BeCO3 < MgCO3 < CaCO3 MgF2 = -2835 - (-2610) = -225 kJ mol"1
(d) BeSO4 < MgSO4 < CaSO4 BaF2 = -2219 - (-2367) = 148 kJ mol"1
(e) Ca2* < Mg2* < Be2*
In BaF2, AHsoln is +ve hence, BaF2 is soluble in water.
(0 Be(OH)2 < Mg(OH)2 < Ca(OH)2
3. (a) Be < Mg < Ca < Sr (b) BeO < MgO < CaO
(g) CaCL < MgCl2 < BeCl2 (c) CaO < MgO < BeO (d) Ca < Li < Mg (e) Be < Mg < Ca
(h) CaCl2 < MgCl2 <BeCl2 4. MgO, Mg3N2 changing to Mg(OH)2 and NH3. (also refer Q.5
2. Hydration energy of below)
BeF2 = -2494+ 2x (-457) = -3408 kJ mol 5. (A): MgO (B): Mg(OH)2 (C):MgCO3 (D):NH3
MgF2 =-1921+ 2x (-457) = -2835 kJ mol" (E):Mg3N2
BaF, = -1305 + 2x (-457) = -2219 kJ mol

Compounds of Alkaline Earth Metals BeO + 2NaOH---- > Na2BeO2 + HjO


Oxides and Hydroxides sodium beryllate
• Oxides are formed either by heating the metals in • CaO combines with solid acidic oxides at high
oxygen: temperatures
2Be + O2 ---- > 2BeO CaO + SiO'22---- > CaSiO3
2Ca + O2 ---- > 2CaO 6CaO + P4O10--- > 2Ca3(PO4)2
or by heating their carbonates at high temperatures: • MgO is not very reactive, especially if it has been
ignited at high temperature and for this reason it is
CaCO3 —CaO + CO2 used as a refractory. BeO is also used as a refractory.
A
MgCO3 MgO + CO2 • Solubility of hydroxides increases while going down
• The oxides are highly stable due to large ionic crystal the group
lattice energies. The values are so high in the case of Be(OH)2 < Mg(OH)2 < Ca(OH)2 < Sr(OH)2 < Ba(OH)j
oxides of Be and Mg that these compounds are almost
insoluble in water. BeO is amphoteric and other • Ca(OH)2 is called lime water or slaked lime and
oxides are basic and yield hydroxides in aqueous Ba(OH)2 is called baryta water; lime water or baryte
solution. turns milky by CO2
MgO + HjO---- > Mg(OH)2 Ca(OH)2 + CO2 ---- > CaCO3 +
CaO + HgO---- > Ca(OH)2 milky
• Basic strength of the hydroxides increases going down Ba(OH)2 +CO2 ---- > BaCO3 +H2O
the group milky
Be(OH)2 < Mg(OH)2 < Ca(OH)2 < Sr(OH)2 < Ba(OH)2 Ba(OH)2 gives turbidity (milkyness) even on exhaling
BeO is insoluble inH^Obut dissolves in acid and alkali breath on it.
and is thus amphoteric
Milkyness disappears when excess of CO2 is passed,
BeO + 2HC1 BeCl2 +H2O
due to formation of soluble bicarbonates.
Chapter 9: s-Block Elements — The Alkaline Earth Metals | 411

CaCO3 + HgO + CO2 ---- > Ca(HCO3)2 Stalactite The downward growth of CaCO3 formed
excess soluble on the roof of a cave by the trickling of water containing
Temporary hardness in water is due to dissolved calcium compounds.
Ca(HCO3)2 and Mg(HCO3)2
Stalagmite The upward growth from the floor of a
• Ca(OH)2 + Na2CO3 ---- > CaCO3 X + 2NaOH
cave; of the same nature and origin as a stalactite.
This reaction was used in earlier days to manufacture
NaOH fromCa(OH)2.
• 2Ca(OH), + 2CL---- > CaCL + Ca(C10)2 + 2H2O Sulphates
calcium • The solubility of the sulphates in water decreases
hypochlorite
down the group.
• Solutions of beryllium salts are acidic and dissolve
appreciable quantities of Be(OH)2. In alkali solution BeSO4 > MgSO4 > CaSO4 > SrSO4 > BaSO4
[Be(OH)4 f" is formed soluble sparingly insoluble
soluble
[Be(H2O)4f+ [Be(H2O)3(OH)l++ H+
[Be(H2O)3(OH)]+ [Be(H2O)2(OH)2 ] + H+ High solubilities of BeSO4 and MgSO4 are due to the
high enthalpy of hydration of the smaller Be2+ and
[Be(H2O)2(OH)2] [Be(H2O)(OH)3r + H+ Mg2* ions, which overcomes the lattice energy factor.
[Be(H2O)(OH)3r [Be(OH)4f“ + H+
• The sulphates decompose into oxides and SO3.
MSO4 MO + SO3
Bicarbonates and Carbonates
• The carbonates are all ionic, but due to excessive Thermal stability of the sulphates increases going
hydration of Be2*, BeCO3 is unusual because it down the group as shown by decomposition
contains the hydrated ion [BeCK^O^f+. temperatures :
BeSO4 MgSO4 CaSO4 SrSO4
• BeCO3 decomposes at low temperature and is placed 500°C 845°C 1149°C 1374°C
in the atmosphere of CO2
• MgSC^ THjO is called Epsom salt and is used as mild
BeCO3 ---- > BeO + CO2 laxative. Correct formulation is [MgCHgOg ] S04 H^O
Thermal stability increases with increase in atomic and is isomorphous with zinc sulphate ZnSO4 ■ 7^0.
weight down the group or with increasing cationic MgSO4 is deliquescent and readily soluble in water. It
size. forms double salts with alkali metal sulphates.
K^SO, MgSO4 6H2O is sold as a fertilizer in the name
BeCO3 < MgCO3 < CaCO3 < SrCO3 < BaCO3
of ‘potash magnesis’.
• Ca(HCO3)2 and Mg(HCO3)2 exist in water causing • CaSO4 -2^0 is called gypsum. Heating converts it into
temporary hardness which can be removed as variety of compounds depending on the temperature.
insoluble carbonates by boiling:
CaSO^HjO -^3 CaSO^^G-^^ CaSO4
Ca(HCO3)2 CaCO3 X + H^O + CO2
gypsum Plaster of Paris anhydrite
A>
Mg(HCO3)2 MgCO3 1 + HaO + CO2 1100°C> CaO + S03
• Ca(HCO3)2 and Mg(HCO3)2 are amphoteric thus when powdered Plaster of Paris, CaSO4H^O
neutralised by acid as well as base:
Ca(HCO3)2 + H2SO4 ---- > CaSO4 + 2H>0 + 2CO2 (2CaSO4 H2O or (CaSO4)2 HgO) is mixed with
stoichiometric amount of water, it sets into a solid
Ca(HCO3)2 + Ca(OH)2 ---- > 2CaCO31 + 2^0 mass CaSO4-2H2O — gypsum. Plaster of Paris finds
• All of these carbonates are much soluble in a solution use
ofC02 than in water due to formation of bi carbonates - for plastering walls
CaCO3 + H^O + C02---- > Ca(HCO3)2 - for making plaster casts (moulds)
- as industrial material
• SrCO3 is used in the manufacture of glass for colour - as sculptural material and
TV picture tube. - for encasing limbs so that broken bones are set
straight.
Calcium and barium bicarbonates are stable only in
solution and decompose on removal of water to give • CaSQfHgO is called Alabaster — a shiny-like marble
carbonates. This property of bicarbonates is the reason for and is used to make ornaments.
stalactite and stalagmite formation. • MS04 +4C MS + 4C0?
412 | Essential Inorganic Chemistry

• BaSO4 is used in medicine as a contrast medium • BeCl2 forms complexes Af2(BeCl4) with alkali metal
for stomach and intestinal X-rays. chlorides but are decomposed by water.
• BeF2 is very soluble in water owing to the high
Halides solvation energy ofBe2+ in forming [Be(H20)4f+. The
The tendency for the smaller cations to get hydrated is other fluorides AfF2(Af2+ - Mg2+, Ca2+, Sr+, Ba2+)
so strong that it is not possible to prepare anhydrous salts are all almost insoluble.
by simply heating the hydrated ones. Thus, anhydrous • The chlorides, bromides and iodides of Mg, Ca, Sr and
BaCl2 may be prepared by heating the hydrated salts Ba are ionic and are readily soluble in water.
beyond 127°C.
• The halides form hydrates and they are hygroscopic.
BaCl2 -2H2O heat BaCl2 +2H2O
• CaCl2 is widely used for treating ice on roads,
However analogous reaction cannot be carried out for particularly in very cold countries when it prevents ice
magnesium chloride. formation even up to -55° C.
heat
MgCl2 -6H2O - ---- > MgCl(OH) +HC1 +5Hj>0 • Mixed chloride hydrides of formula AfCIH which have
layer lattices (the different anions occupying different
MgCl(OH) ---- > MgO + HCl
layers) are formed when the hydrides and chlorides of
If anyhydrous MgCl2 is required, it may be obtained alkaline earth metals are heated together:
by mixing the hydrated salt with thionyl chloride SOC12. CaH2 + CaCl2---- > 2CaHCl
MgCl2 • 6H2O + 6SOC12 ---- > MgCl2+6SO2+12HC1
• Halides MX2 of alkaline earth metals can be prepared
by heating the metals with the halogen acid on either Anomalous Behaviour of Beryllium
metal or its carbonate :
• Because of high (IE) and small atomic size it forms
M + 2HC1 ■* mci2 + h2
compounds which are largely covalent and its salts are
MCOo t
jkvuj + 2HC1 ---- > MC12 + H^O + C02 easily hydrolysed.
• The beryllium halides are covalent, hygroscopic
• Be (Is2 2s2 2p°) can use only one 2s and three 2p
and fumes in air due to hydrolysis:
orbital in coordination thus maximum coordination
BeCl2 + 2H2O---- > Be(OH)2 + 2HC1 (fumes) number (CN) of Be is 4 while other elements can show
• BeX2 sublimes on heating and are bad conductor of CN of 6 in their compounds by use of d-orbitals in
electricity. In aqueous solution it is difficult to addition to s- and p-orbitals.
crystallise anhydrous BeX2 since dehydration results
in hydrolysis: • BeO and Be(OH)2 are amphoteric while other oxides
A are basic.
[Be(H2O)4]Cl2 Be(OH)2 + 2HC1 + 2H2O
• The cohesive properties of beryllium are much greater
• Anhydrous BeX2 can be obtained by : than those of magnesium and other elements in the
BeO + C + Cl2 BeCl2 + CO group. As a result, it is much harder and has higher
melting and boiling points.
2BeO + CC14 2BeCl2 + C02
• Its higher polarising power leads to all its compounds
NH4F A being largely covalent, with lower melting and boiling
BeO A >3 (NH4)2BeF4 ■> BeF2 + 2NH4F
points and enthalpies of formation. (MP of BeF2 is
Anhydrous BeCl2 is polymeric. Vapours exist as 800°C that of other fluorides about 1300°C). All the
monomer BeCl2 and dimer (BeCl2)2. Bond length data compounds of Be are more soluble in organic solvents
show that the polymer has three centre two-electron than the corresponding magnesium compounds. They
bonds. hydrolyse in water, and in this respect they rather
CR resemble aluminium (diagonal relationship)
Cl—Be *Be—Cl
Diagonal Relationship
dimer • Beryllium shows a diagonal relationship to
Cl CL aluminium :
Be Be Be Be - The chlorides of both Be and Al have bridged
Cl ^cr structures in vapour phase
polymer (BeCl2)n
Chapter 9: s-Block Elements — The Alkaline Earth Metals | 413

.Ck /Ck /Cl-.. Al forms octahedral complexes like A1F63 and


>Be< >Be< [akcam3-.
•••cr xcr xciz
Additional Reactions and Facts of
Alkaline Earth Metal Compounds (HA)
~ Two metals have similar standard electrode Reactions
potential: A
1. 2Ba(NO3), 2BaO + 4NO,(g) + O2(g)
Be2+(aq) +2e~ ■ ->Be(s), E°=-1.85V
2. CaH2 + 2H2O---- > Ca(OH)2 + 2H2
Al3+(ag) +3e“ -»Al(s), E° = -1.66 V
“ Both are rendered passive by nitric oxide due to 3. CaJ3N„L + 6H..0
L
---- > 3Ca(OH)„Zu
+ 2NH
formation of oxide layer
4. Be2C + 4H2O 2Be(OH)2 + CH4
2HNO3 -----> HaO + 2NO2 + 0 methane
HNO3
Be + 0 BeO ■------ no reaction 5. CaC2 + 2H2O---- > Ca(OH)2 + C2H2
passive ethyne
HN03
Al + O ■> ai203 ------- > no reaction 6. CaC2 + N2 CaCN2 +C
passive nitrolim
Thus, HNO3 can be stored in a vessel made of Al 7. CaCN_ +5H,0---- > CaCO, +2NH.0H
or Be.
• Both form electron-deficient hydrides and volatile —BaC,
8. 1 2 ---- > Ba(CN)2
TN,
------ '2 +
chlorides. Bef^, A1H3, etc, are some electron-deficient 9. Ca(OH)2+Cl2 CaCl OCT + H.,0
hydrides. bleaching powder
• Both form carbides which on hydrolysis give methane. (Bleaching powder contains Cl” and CIO" ions)
A14C3 + 121^0-----> 3CH4 + 4A1(OH)3 10. Mg„C3+4H2O--- > 2Mg(OH)2+CH3—C=CH
2 ° ~ Ar
propyne
rxnwno
Be2C + 2H2O-----> 2BeO + CH4
BeO + H2O-----> Be(OH)2
• Both metals dissolve in aqueous alkali evolving Important Facts of Alkaline Earth Metals
Be + 2NaOH ■> Na2BeO2 + H2 • Be and Mg are kinetically inert to oxygen and water
2A1 + 2NaOH + 2H2O 2NaA102 + 3H2 because of the formation of a surface film of oxide.
• Oxides and hydroxides are amphoteric • Be and Mg bum in air forming oxides and nitrides
(BeO, MgO/Be3N2, MggN?). Ca, Sr and Ba are readily
In dil acid (showing basic In alkali solution (showing
nature of acidic attacked by air forming oxides and nitrides.
oxide or hydroxide) nature of oxide or hydroxide)
• Alkaline earth metals dissolve in liquid NH3 to give
BeO + 2HCI—» BeCI2 + H2O BeO + 2NaOH—> Na2BeO2 deep blue black solutions from which ammoniates
Be{OH)2 + 2HCI—> BeCI2 + H2O [M(NH3)6f+ can be recovered.
+ 2H2O Be(OH)2 + 2NaOH —
• Metals are electropositive and strong reducing agents.
Na2BeO2 + 2H2O
sodium berrylate The particularly less negative value for Be arises from
the large hydration energy associated with the small
A!jO3 + 6HCI —> 2AICI3 + 3H2O AI2O3 + 2NaOH---- >
size of Be2+ and the relatively large value of heat of
A-'(OH)3 + 3HCI —> AICI3 2NaAIO? + h2o
+ 3Hi?2O
< sodium meta-aluminate sublimation.
• The solubility, thermal stability and the basic
• The two elements have same electronegativity character of hydroxides increase with atomic number
(Be = 1.5 and Al = 1.5) and their charge/radius ratios
from Mg(0H)2 to Ba(OH)2 • Be(OH)2 I BeO is
(0.064 and 0.060 respectively) are very similar
indicating similar field strengths. amphoteric.
• Except BeX2 (X = F, Cl, Br, I), all other hallides MX2
• They have also strong tendency to form complexes:
are ionic. BeX2 exists in gas phase only but in
Be forms tetrahedral complexes as BeF42- and condensed phase it exists as polymer. Solubility in
WW2f~ water increases with increase in size of cation.
414 | Essential Inorganic Chemistry

• Carbonates and bicarbonates decompose on heating. • Oxalates of Ca, Sr, and Ba are sparingly soluble and
BeCO3 is least stable and is kept in the atmosphere of solubility increases from CaG2O4 to BaG2O4.
CO2. • Nitrates decompose to form oxide, O2 and NO2
• Sulphates are stable to heat. BeSO4 and MgSO4 are
soluble in water but solubility decreases from CaSO4 2M(NO3)2 2MO + 4NO2 + O2
to BaSO4. The greater hydration energies of Be2+ and • Be shows diagonal relationship to Al.
Mg2+ ions overcome the lattice energy factor, and
hence sulphates are soluble.

Target Practice 2
1. Mixture containing one mole each of CaCO3 and Ca(HCO3 )2 BeO + 2NaOH ---- > Na,BeOL
2+Hh2O
L
is strongly heated. What is the amount (in moles) of the BeO + 2HCl ---- > BeCl,+H2O
residue left?
5. CaO is based on following reaction
2. For several thousand years, lime has been used in mortar (a
lime, sand and water paste) in building houses, etc. Explain 6CaO + P.O,
4
n -----> 2CaJPO.)„
LU u • Z

its reaction 6. Give the reaction used to prepare NaOH from Ca(OH)j is
3. (a) What is hybridisation of Be in BeF,? earlier days
Q &

(b) When BeF4 is formed hybridisation of Be changes 7. MgSO4 • 7H2O is correctly represented as
from to 8. Name the salt which is used in medicine as a contrast
4. BeO is based on following reactions medium for stomach and intestinal X-rays

Answers
1. CaCO3 CaO + CO,T 3. (a) sp (b) sp, sp 3
1 mol 1 mol 4. amphoteric
Ca(HCO3)2 CaO + H,Ot +2CO2T 5. basic
1 mol 1 mol 6. Na2CO3+Ca(OH)2 - 2NaOH + CaCO3i
Residue is 2 moles CaO 7. [Mg(H2O)6]SO4-H2O
2. CaO + H2O ---- » Ca(OH)2 8. BaSO4

Practice Exercise 2
1. Gas (X) turns lime water milky. Gas (X) also turns acidified 5. Ca(HCO,O ),Z and Ca(OH),Z can't be stored in the same vessel
K2Cr2O7 green. What is gas (X)?
Explain.
2. Beryllium turns passive in presence of HNO3. Explain. 6. Ca(HCO3 )2 is determined volumetrically by titration with
3. Identify (A) to (C) and the metal ion Af 2+. acid. Which of the following indicators is better?
MCO (A) + (B) (a) Phenolphthalein
(X) (solid) (gas) (b) Methyl red
(A) + H2O ---- > (C) 7. Ca2+ can be determined by precipitating it as CaCjO4
(C) + (B)-----> (X) gravimetrically. Calculate percentage of Ca2+ in a sample,if
0.5 g ofCa2+ salt gave 1.28 g of CaC2O4.
Paste of the salt (X) in cone. HC1 given green flame on
(Ca= 40, Ca= 12,0 = 16)
ignition.
4. Following reaction is used to remove temporary hardness 8. Identify (A) to CD) in the following:
which is due to dissolved bicarbonates ofCa2+ and Mg2+ A * ‘ K' BaC12> (/J) (white ppt) ■Carbon.
(A)
KO2 +S —— 7

Ca(HCOJ„ + Ca(OH)z2 -----> 2CaCO,0 I + 2H,0 A


o z z
Actually lime (CaO) is added which in H2O forms Ca(OH)2 (C) ® D(gas)
and precipitates CaCO3 (jD) gas turns lead acetate paper black.
CaO + H„O
z -----> Ca(OH),z 9. Presence of CaSO4 causes ‘permanent hardness' in water. It
If 10 L of hard water requires 0.56 g of CaO, what is can be removed by precipitating it as CaCCh by addin?
temporary hardness in ppm of CaCO3 (parts per Na2CO3. Calculate amount of Na2CO3 to soften 100 L of
million—ppm.)? hard water which is 200 ppm CaCO3.
Chapter 9 : s-Block Elements - The Alkaline Earth Metals I 415
10. While moving down the group (in alkaline earth metals) 13. Arrange BeC1 2, MgC1 2, CaC1 2, SrC1 2 and BaCI 2 in
following property decreases increasing
(a) radius (i) tendency of hydrolysis
(b) solubility of hydroxides (ii) lattice energy
(c) solubility of sulphates (iii) solubility in H 2O
(d) thermal stability of carbonates (iv) covalent nature.
Choose the correct alternate.
14. Match the following chlorides with the colour they impart
11. CA) is binary oxide of alkaline earth metal. On reaction with on ignition in flame:
HCl, (A) forms solid (B) and liquid (C). (B) gives apple green
flame on ignition and liquid (C) decolourises KMnO, in Salt Colour in flame
acidic medium. What are (A), (B) and (C)? 1. CaCl 2 (A) Apple green
12. BeC1 2 in aqueous solution exists as [Be(H2O)4] 2+ rather 2. SrCl 2 (B) Brick red
3. BaCl 2 (C) Crimson red
than Be2 •. This is acidic in nature. Explain.

Answers
I. Gas (X) is SO2 9. Na 2CO 3 + CaSO, � CaCO 3 J. + Na 2SO 4
106 g 100 g
Ca(OH)2 + SO 2 --t CaSO3 +H 2O
milky (1 million)= 10 6 mL of hard water has= 200 g CaCO
3
100 L = 10 11 mL of hard water has= 20 g CaCO 3
SO 2 + Cr20;- + ff+ --+ Cr a+ +so:-+ H 20
green ff hardness is 100 g, Na 2CO 3 required= 106 g
2. HN0 3 is an oxidising agent. It oxidises Be to BeO. This, oxide ff hardness is 20 g, Na 2CO3 required= 21.2 g
layer prevents attack ofHNO3 on it. Hence, Be turns passive. 10. (c)
3. (X):BaCO 3 ,M 2+ :Ba 2• (A):BaO (B):CO 2 11. (A) : BaO 2 � Ba 2+ + o�- (B) : BaCl 2 (C) : H20 2
(C) : Ba(OH)2
12. [Be(H 2O),J 2• is acidic because of hydrolysis.
Ca<HCOa >�
4. CaO --+ Ca(OH)2 2CaCO3 [Be(H 2O)4 ] 2+ +H 2O � [Be(H 2O)3 (0H>r + H 3O+
56 g 200 g
0.56 g This happens because the Be-O bond is very strong, and this
2.00 gin 10 4 mL H20
weakens the 0-H bond and thus, proton is lost.
10� mLH 2O hardness= 2 g
H-O-H � W+OH
10 6 mL water has hardness = 200 ppm
5. Ca(HCO 3 )2 behaves as an acid with Ca(OH)2 [Be<H 2O),]2+ + OH- ¢ [Be<H:,i0>3 COH>r + H 2O

Ca(HCO 3 ) 2 + Ca(OH)2 --+ 2CaCO 3 J, +H 2O H 20 + H• ¢ H 3Q•


Hence, neutralisation takes place. 13. (i) BaC1 2 < SrC1 2 < CaC1 2 < MgC1 2 < BeC1 2
8. Methyl red is only used. Phenolphthalein does not indicate any (ii) BaCl 2 < SrCl 2 < CaCl 2 < MgCl 2 < BeC1 2
colour change. (iii) BeCl2 < MgC12 < CaC1 2 < SrC1 2 < BaCl2
7. 80% pure. (iv) BaCl2 < SrCl 2 < CaC1 2 < MgC1 2 < BeC1 2

-=------------,-----------------------·---- ---=-----
8. CA): KaSO 4 (B) : BaSO 4 (C) : BaS (D) : H 2S 14. 1---(B) 2--(C) 3-(A)
Total Practice Set
(Read, Plan & Solve'.)

Problem 1. What is biological role of Ca2+ and Mg2+ Solution A given reaction is spontaneous, if AG0^ is
ions? negative.
Solution Mg2+ ions are concentrated in animal cells, 3CaO + 2A1---- > A12O3 + 3Ca
and Ca2+ are concentrated in the body fluids outside the AG°Z =AG°f(Al2O3)-3AG°f (CaO)
cell. Mg2+ is important in chlorophyll, in the green parts of
= -1582.4 - 3(-604.2)
plants.
Ca2+ is important in bones and teeth as apatite = -1582.4 + 1812.6
Ca3(PO4)2 and the enamel on teeth as fluoroapatite = +230.2 kJ mol"1
[3(Ca3(PO4)2)CaF2l.
The positive sign tells us that the reaction cannot occur
Ca2+ ions are important in blood clotting, and are under standard conditions.
required to trigger the contraction of muscles and to
maintain the regular beating of heart. Note: Under non-standard conditions e.g., under high
temperatures, AG is negative and reaction takes place.
Problem 2. Beryllium gives a compound X with the AG° (element) = 0.
following percentage composition : Be-6.1%; N -37.8% Cl
-48%, H —8.1%. Molecular weight of (X) is 148 g mol"1 and Problem 4. 0.2 g of magnesium ribbon was placed in a
that of Be is 9 gmol"1. crucible and heated with the lid on until the magnesium
(i) What is the molecular formula of the compound? began to bum rapidly. At the end of the experiment there
(ii) In aqueous solution, 1 mole of the compound reacts was 0.3 g of a white powder left. Show that this result does
with 2 moles of AgNO3 giving white ppt. What is not agree with the equation:
structural formula of (X)?
2Mg(s) + O2(g)---- > 2MgO(s)
Solution
What might have gone wrong?
Atomic Simplest
Atom % %/A Solution 2Mg(s) + O2(g) —> 2MgO(s)
weight (A) ratio
2 x 24 g 2 x 40 g
Be 9 6.1 0.68 1
N 14 37.8 2.70 4 48 g of Mg gives = 80 g of MgO
Cl 35.5 48.0 1.35 2 80
H 1 8.1 8.10 12 0.2 g of Mg gives =— x 0.2

(i) Molecular formula is BeN4Cl2H12. = 0.33 g of MgO


(ii) Since, 1 mole of X reacts with 2 moles of AgNO3,
hence two Cl are outside the complex. Since, there But residue left = 0.30 g (given)
are four N and twelve H, this indicates four NH3 Hence, the results are not in accordance with this
molecules attached to Be. This gives structural equation.
formula as: It can be due to
[Be(NH3)4] CI2
(1) some MgO may have escaped as vapours
Problem 3. A student suggested that calcium should (2) some Mg may not have reacted
be made if calcium oxide is reacted with aluminium (3) some Mg3N2 can be formed.
powder. Was the student correct ?
AG° (CaO) = -604.2 kJ mol"1, Problem 5. Give the trends in solubilities of chlorides,
AG°Z (A12O3) = -1582.4 kJ mol"1 carbonates, sulphates and hydroxides of alkaline earth
metals.
Chapter 9 .� s-Block Elements-The Alkaline Earth Metals I 417
Solution Problem 8. Give a comparison of the elements Be and

r
Al.
Chlorides Carbonates Sulphates Hydroxides
Be

!
Solution Be [2nd period, group 2 (IIA)] and Al [3rd

J l
least soluble most soluble most soluble least soluble
Mg period, group 13 (IHA)] show diagonal relationship. Be
Ca differs in most of the properties of its group but resembles
Sr Al (diagonally placed to Be) in following respects:
Ba most soluble least soluble least soluble most soluble
Properties Explanations
Problem 6. What is the importance of calcium and WithHN0 3 Both metals are rendered passive and do not
magnesium carbonates and hydrogen carbonates in dissolve due to formation of oxide layer.
making water hard or soft? With alkali Both dissolve in alkali giving off H 2 •
Solution When slightly acidic rain water percolates 2AI+ 2Na0H + 2Hp--. 2NaAl0 2 + 3H 2
through carbonate rocks, some of the rocks dissolve. Ca 2
+ Be+ 2Na0H ---+ Na 2 Be0 2 + H 2
and Mg ions are dissolved in the water. It is these ions
2+
Amphoteric nature Both metals react with acid and base and thus
which give scum with soap or make scale (white deposit) are amphoteric.
when hard water is boiled. Oxides Both BeO and Al 20 3 are amphoteric.
Chlorides I BeCl2 and AICl3 both are electron deficient
Problem 7. Explain anomalous behaviour of I and thus are Lewis acids.
beryllium.
Solution
Problem 9. The standard enthalpy of formation
• It is extremely small in size and as determined by AHf' of hypothetical CaCl(s) theoretically found to be
Fajan's rule, smaller cations tend to polarise larger
anion and thus, Be2 + forms covalent compounds. -188 kJ mo1-1 and that of CaC12 (s) -795 kJ mo1- 1•
• Be has high electronegativity and compounds like BeF2 Calculate the value of standard heat of reaction for the
and BeO are covalent. following disproportionation reaction:
• BeX2 exists as polymeric (BeX2 )4 in solid state and 2CaCl(s) � CaCl2 (s)+ Ca{s)
coordination number = 4 is shown in which each
halogen forms Solution A disproportionation reaction is that in which
the same species (in this case Ca + ) is oxidised as well as
Ch,.. reduced.
Cl-Be.,/ )B e-Cl 2 CaCl(s)� CaC12 (s) + Ca(s)
"'cv
one normal covalent bond, and uses a lone pair to form +l
I +2
t t
0
a coordinate bond. oxidation
• In water, beryllium salts are extensively hydrolysed to reduction
give a series of hydroxo complexes of the type :
[HO, / OH, / OHr- To determine standard heat of reaction, we use
Be Be
HO) <_OH) �OH t:J{ 0 = t:J:lf (products) - t:J:lf (reactants)
Ml; of the element = 0
tilir
and
If alkali is added to these solutions, the polymers
break down to form [Be(OH)4 f- - a tetrahedral Thus, Ml° = (CaCl 2 ) + AH 1 1
(Ca)- 2Ml (CaCD
structure. = (-795 + 0) -(-2 X 188)
• Be2 + salts are extensively hydrated due to larger
(char ge/radius) ratio (called charge density). = -419 kJ mo1-1
[Be(II.iO)4 ] S04 , [Be(�O)4 ](NO 3 )2 and [Be(H2O)4 ] Cl2 This shows that CaCI(s) does not exist and any
are known. These salts are acidic due to following
equilibrium : synthetic route to get MX in case of the alkaline
earth metals will result in the formation of MX.2•
[Be(�O)4 f + + �O � [Be(�O)3OHj + Hp
+ +
r-
it

Master Exercises
Exercise 1
(Stage 1: Learning)
Short Answer Type Questions earth metals) and state how these properties
1. Arrange Be, Ca, Ba, Ra in increasing order of differ between the two groups ?
(a) size (b) ionisation energy (i) (ZE^ and (ZE)2
(c) electronegativity (d) electron-affinity (ii) cationic radii
2. Arrange alkaline-earth metal chlorides in increasing (iii) rates of reaction with cold water
(a) covalent nature (b) melting point (b) For the same s-block elements, discuss :
(c) lattice energy (d) thermal stability (i) action of heat on their nitrates
3. Compare thermal stability of (ii) the crystal structures of their chlorides, XY type
only.
(a) NaHC03 andCa(HCO3)2
(b) CaCO3 andNa2CO3 6. Compare the chemistries of Mg and Ca with reference
to the following:
(c) BeCO3 and Li2CO3
(i) solubilities of their sulphates in water
(d) NaHC03 andNa2CO3
(ii) the thermal stabilities of their carbonates
4. When magnesium burns in air, it forms both an oxide (iii) the reactions of their oxides with
and nitride. Write balanced equations.
7. (A) is binary electrolyte of a divalent cation. (A) on
5. In the preparation of barium oxide by thermal reaction with H2SO4 gave a white ppt. (B) and a
decomposition of barium carbonate, why is carbon colourless liquid (C) that can decolourise KMnO4 in
added to the reaction mixture? acidic medium. 1.69 g of (A) gave 2.33 g of (B). What
6. What is sorel cement? are (A), (B) and (C) and explain reactions?
8. “The chemistry of beryllium is not essentially ionic.’
Analytical Questions Justify the statement by making reference to the
1. Do you know of a biologically important molecule in nature of oxide, chloride and fluoride of beryllium.
which magnesium atoms are held in a position by 9. Comment on each of the following observations
organic groups? (a) E° for the reaction, M2+(aq) + 2e~---- > M(s) is
2. Beryllium chloride exists as BeCl2 • 41^0 due to nearly constant {M =Ca, Sr, Ba).
(b) LiF is least soluble among the fluorides of alkali
solvation (hydration). What is the hydrated cation ?
metals.
Does it behave as an acid or base, explain?
10. How would you explain?
3. Ca(0H)2 +Na2CO3 ---- > CaCO3>l + 2Na0H
(a) BeO is insoluble but BeSO4 is soluble in water.
This reaction made the basis of manufacturing of (b) BaO is soluble but BeSO4 is insoluble in water.
NaOH in earlier days and pH of the solution was (c) Lil is more soluble than KI in ethanol.
found to be greater than 14. What do you think about (d) NaHCO3 is isolated in solid state but Ca(HCO3)2
the completeness of reaction ? Explain. is not isolated in solid state.
4. When BeCl2 is hydrolysed, there is formation of 11. Make distinction between in terms of action of heat
fumes, explain. (a) Na2CO3 and CaCO3
(b) MgCl2 -6^0 and CaCl2 -6H2O
5. (a) Describe how the following properties vary on
descending group IA and IIA (alkali and alkaline (c) Ca(NO3)2 andNaNO3
Chapter 9 : s-Block Elements—The Alkaline Earth Metals | 419

N CERT Problems 10. Compare and contrast the chemistry of group 1


metals with that of group 2 metals with respect to
1. Compare the solublity and thermal stability of the (i) nature of oxides
following compounds of the alkali metals with those (ii) solubility and thermal stability of carbonates
of the alkaline earth metals. (iii) polarizing power of cations
(a) nitrates (b) carbonates (iv) reactivity and reducing power
(c) sulphates
11. Describe the importance of the following
2. Mention the general trends in group 1 and in group 2 (i) limestone (ii) cement
with increasing atomic number with respect to (iii) Plaster of Paris
(a) density (b) melting point
(c) atomic size (d) ionisation enthalpy 12. Mention the main constitutes of Portland cement.
3. How do the following properties change on moving 13. What happens when
from group I to group 2 in the Periodic Table? (i) magnesium is burnt in air?
(i) Atomic size (ii) Ionisation enthalpy (ii) quicklime is heated with silica?
(iii) Density (iv) Melting point (iii) chlorine reacts with slaked lime?
(iv) calcium nitrate is heated?
4. Like lithium in group I, beryllium shows anomalous
behaviour in group 2. Write three such properties of 14. Draw the structure of
beryllium which make it anomalous in the group. (i) BeCl2 (vapour) (ii) BeCl2(s)
5. Beryllium exhibits some similarities with 15. List the raw materials required in the manufacture
aluminium. Point out three such properties. of Portland cement. What is the role of gypsum in it?
6. The chemistry of beryllium is not essentially ionic. 16. Differentiate between (a) quick-lime (b) lime-water
Justify the statement by making a reference to the (c) slaked-lime.
nature of oxide, chloride and fluoride of beryllium. 17. How is plaster of Paris prepared? Describe its chief
7. Name the chief forms of occurrence of magnesium in property due to which it is widely used.
nature. How is magnesium extracted from one of its 18. How would you explain?
ores? (i) BeO is insoluble but BeSO4 is soluble in water
8. Commercial aluminium always contains some (ii) BaO in soluble butBaSO4 is insoluble in water
magnesium. Name two such alloys of aluminium. (iii) Lil is more soluble than KI in ethanol
What properties are imparted by the addition of (iv) NaHCO3 is known in solid state butCa(HCO3)2 is
magnesium to these alloys? not isolated in solid state
19. Contrast the action of heat on the following and
9. The enthalpy of formation of hypothetical CaCl(s)
explain your answer
theoretically found to be -118 kJ mol-1 and the AfHe (i) Na2CO3 andCaCO3
forCaCl2($) is -795 kJ mol-1. Calculate the A^H3 for (ii) MgCl2 • GHjO and CaCl2 • 61^0
the disproportionation reaction. (iii) Ca(NO3)2 andNaN03
2CaCl(s) ---- > CaCl2(s) + Ca(s) 20. Complete the following equations for the reaction
[Hint. A,H0(CaCl,)-2A,H°(CaCl) between
(i) Ca + HjO (ii) Ca(OH)2 +C12
= -795+2x188
(iii) BeO+NaOH (iv) BaO2 +H2SO4
= - 419 kJ mol-1]

Exercise 2
(Stage 2: High Skill Problem Solving)
Only One Option Correct 2. When Cl2 is passed into moist slaked lime, compound
1. Mg and Zn does not resemble in following properties formed is
(a) oxides are amphoteric (a) CaO2Cl2 (b) CaO,Cl
(b) carbonates on heating form metal oxides (c) CaOCl2 (d) CaCl2O4
(c) widely used as electrodes 3. A colourless solid (X) on heating evolved CO2 and also
(d) used to prevent corrosion gave a white residue, soluble in water. Residue also
gave CQj when treated with dilute acid. (X) is
420 ] Essential Inorganic Chemistry

(a) Na.,CO3 (b) CaCO3 (b) Be is amphoteric


(c) Ca(HCO3)2 (d) NaHCO3 (c) MH2 is ionic ‘salt-like’ hydride
4. II A (alkaline earth metals) and II B (zinc family) (d) All the above are correct statements
resemble 13. Choose the incorrect statement.
(a) MgSO4 • 7H2O is isomorphous with ZnSO4 • 7H2O (a) BeCO3 is kept in the atmosphere ofCO2 since it is
(b) IIA and II B cations are not precipitated by H2S least thermally stable
in acidic medium (b) Be dissolves in alkali forming [Be(OH)4f“
(c) both (a) and (b) (c) BeF2 forms complex ion with NaF in when Be
(d) None of the above goes with cation
(d) BeF2 forms complex ion with NaF in which Be
5. Be and Al show diagonal relationship hence, both goes with anion
have 14. Which are true statements about s-block elements?
(a) same degree of electronegativity (a) Metals are obtained by the electrolysis of fused
(b) amphoteric nature of oxides chlorides
(c) approximately same charge/radius ratio (b) Only one type of valency, +1 for IA and +2 for II A,
(d) all the properties above is shown
6. Be and Al do not resemble in (c) Oxides are basic except BeO
(a) both become passive on reaction with HNO3 due (d) All the above are correct statements
to formation of oxide layer 15. Which cannot be used to generate Hj?
(b) their chlorides are Lewis bases
(c) chlorides exist in polymeric form (a) Al + NaOH (b) Zn + NaOH
(d) hydroxides are soluble in alkali as well as in acid (c) Mg + NaOH (d) LiH + H2O
7. Going down to II A group, following properties 16. Portland cement does not contain
increase except (a) CaSiO4 (b) CaSiO3
(a) solubility of hydroxides in H2O (c) Ca3Al2O6 (d) Ca3(PO4)2
(b) hydration energy 17. Setting of plaster of Paris is
(c) thermal stability of carbonates (a) oxidation with atmospheric oxygen
(d) ionic radius (b) combination with atmospheric CO2
8. Identify the correct statement. (c) dehydration
(a) Gypsum is obtained by heating plaster of Paris. (d) hydration to yield another hydrate
(b) Plaster of Paris can be obtained by hydration of 18. Mixture of MgCl2 and MgO is called
gypsum. (b) Sorel's cement
(a) Portland cement
(c) Plaster of Paris contains higher percentage of (d) None of these
(c) double salt
calcium than does gypsum.
(d) Plaster of Paris is obtained from gypsum by 19. Setting of cement is
oxidation. (a) exothermic reaction
9. Following are the ionisation potential values of (b) endothermic reaction
(A) 899 kJ mol"1, (/2) 1757 kJ mol"1, (c) neither endothermic nor exothermic
(/3) 15000 kJ mol"1 (d) None of the above
(a) Na (b) K 20. Which is main constituent of egg-shell?
(c) Be (d) Ne (a) CaCO3 (b) CaSiO3
10. Which is most stable out of the following? (c) CaSO4 i H2O (d) CaSC^O
(a) [BetHjO),/* (b)
(c) [CafHjO),]2" (d) [SrtHjO.f* 21. The deliquescent among the following is
(a) CaCl2 (b) FeS04-7H20
11. Lattice energy of II A group compounds (oxides, (c) CuSO4-5H2O (d) BaCl2-2H2O
carbonates, flourides)
(a) decreases as size of the ion increases 22. Which is incorrect statement?
(b) increases as size of the ion increases (a) The heats of hydration of the dipositive alkaline
(c) constant for a given type of anion earth metal ions decrease with an increase in
(d) all the above are incorrect their ionic size.
12. Which of the following statements are true about IIA (b) NaNO3 forms Na2O on heating
group elements? (c) Hydration of alkali metal ion is less than that of
(a) All form nitrides in air II A.
Chapter g: s-Block Elements-The Alkaline Earth Metals l 421
(d) Alkaline earth metal ions, because of their much 33. Select correct statement(s).
larger charge to size ratio, exert a much stronger (a) Beryllium and magnesium hydride are covalent
electrostatic attraction on the oxygen of water and polymeric
molecule surrounding them. (b) Caliz, SrH2 and Baf½ are ionic
23. Which is used for fixing atmospheric nitrogen? (c) Belii contains three-centre two-electron bond
(a) CaCN2 (nitrolim) (b) LiaN (d) All the above are correct statements
(c) MgaN2 (d) All of these 34. Select correct statement(s).
fl
24. Ba(; + N2 ---+ (A) (a) CaC03 is more soluble in a solution ofC02 than in
fi:iO
fl (b) NaC0 3 is converted to Na20 and CO2 on heating
Ca(; + N2 ---+ (B)
CA) and (B) are (c) Li2C03 is thermally stable
(d) Presence of CaCI2 or CaS04 in water causes
(a) BaCN2 , CaCN2 (b) Ba(CN)2 , Ca(CN)2
(c) Ba(CN)2 , CaCN2 temporary hardness
(d) None is correct
35. Select correct statement(s).
25. Estimation of calcium and magnesium is done by
(a) Mg2 + ions are necessary for the activation of
(a) EDTA (b) oxalate phosphate transfer enzymes.
(c) phosphate (d) None of these (b) Mg is present in chlorophyll used in
26. Which of the following is/are correct statement(s)? photosynthesis in green plants.
(a) Ca 3(P04)2 in part of bones (c) Operation ofNa + - K + pumps is biological.
(b) 3Ca 3(P04) · CaF2 is part of enamel on teeth (d) All the above are correct statements
(c) Ca2 + ions are important in blood clotting 36. A metal M readily forms water soluble MS04 • It also
(d} All of the above are correct forms oxide MO which becomes inert on heating.
27. 1 mole of a substance (X) was treated with an excess Hydroxide M(OH)2 is insoluble in water but soluble
of water. 2 moles of readily combustible gas were in NaOH solution. What is M?
produced along with solution which when reacted (a) Mg (b) Ba
With CO2 gas produced a white turbidity. The (c) Ca (d) Be
substance (X) could be 37, Which one of the following has magnesium?
(a) Ca (b) Caliz (a) Vitamin B12 (b) Chlorophyll
(c) Ca(OH)2 (d) Ca(N0 3)2 (c) Haemocyanin (d) Carbonic anhydrase
28. A b asic refractory material among the following is
(a) Al 2 03
38. In which of the following the hydration energy is
(b) Si02
(c) Fe203 (d) CaO higher than the lattice energy?
(a) BaS04 (b) MgSO.,
29. In water
(c) RaS04 (d) SrS04
(a) temporary hardness is due to the bicarbonates of
Ca2 + and Mg2 + 39. Epsom salt is
(b) permanent hardness is due to chlorides and {a) BaS04 • 2Rz0 Cb) CaS04 • l½O
sulphates ofCa2 + and Mg2 + (c) MgS04 • 2�0 (d) MgS04 • 7�0
(c) hardness can be removed by adding phosphates 40. Slaked lime and chlorine reacts to produce
(d) all of the above properties are true (a) quick lime
30. Na2 [Be(OH)4 ] is formed when (b) CaC12
(a) BeO reacts with NaOH solution {c) Ca0Cl2
(b) Be reacts with NaOH solution (d) mixture ofCaC12 and Ca(OC12 )
(c) both (a) and (b) correct 41. Mg and Li and similar in their properties due to
(d) None of the above is correct (a) same e Im ratio (b) same electron affinity
31, Which is used to treat acid indigestion? (c) same group (d) same radius
(a) Be(OH)2 (b) KOH 42. Which one of the following hydroxide is insoluble in
( c) Mg(OH)2 (d) Ca(OH)2
water?
32. Which is not obtained when metal carbides react with (a) Ca(OH)2 (b) Ba(OH)2
RiO? (c) Be(OHJ2 (d) Mg{OH)2
(a) Al4C3 + I½O � CH!!!!!CH 43. The pair of amphoteric hydroxide is
(b) Ca� + I½O ---+ CH!ll!CH
(a) Al(OH) 3 , LiOH (b) Be(OH)2 ,Mg{OH)2
Cc) M g4C3 + l½O ---+ CH 3C!!!!!!CH
(c) B(OH) 3 ,Be(OH)2 (d) Be(OH)2, Zn(OH)2
(d) B�C + l½O ---+ CH4
422 | Essential Inorganic Chemistry I

44. The pair whose both species are used in antacid 56. Be in BeCl2 undergoes
medicinal preparation is (a) linear hybridisation
(a) NaHCO3 and Mg(OH)2 (b) trigonal hybridisation
(b) Na2CO3 and Ca(HCO3)2 (c) tetrahedral hybridisation
(c) Ca(HCO3)2 andMg(OH)2 (d) no hybridisation
(d) Ca(OH)2 andNaHCO3 57. CaCl2 is used as
45. Lithophone is mixture of (a) disinfectant (b) desiccant agent
(a) ZnCO3,BaCO3 (b) ZnS,Na2SO4 (c) medicine (d) None of these
(c) ZnSO4,BaSO4 (d) ZnS,BaSO4 58. The product obtained on fusion ofBaSO4 andNa2COj
46. Gypsum is is
(a) CaSO4 -2^0 (b) CaCO3-2H2O (a) BaCO3 (b) BaO
(c) Ba(OH)2 (d) BaHSO4
(c) CaSO^j -HgO (d) SiO2
2 59. Which of the following is known as dead burnt?
47. Alkaline earth metals are (a) Gypsum (b) Plaster of Paris
(a) reducing agent (b) dehydrating agent (c) Anhydrite (d) None of these
(c) amphoteric (d) oxidising agent 60. The decomposition temperature is maximum for
48. A covalent chloride is (a) MgCO3 (b) CaCO3
(a) BeCl2 (b) NaCl (c) BaCO3 (d) SrCO3
(0 MgCl2 (d) CaCl2 61. Bleaching powder loses its power on keeping for a
49. Plaster of Paris is hardened by long time because
(a) giving out water (a) it changes into calcium hypochlorate
(b) uniting with water (b) it changes into calcium chloride and calcium
(c) changing into CaCO3 hydroxide
(d) liberating C02 (c) it absorbs moisture
(d) it changes into calcium chloride and calcium
50. Which of the following halide of calcium is insoluble chlorate
in water?
(a) CaCl2 (b) Ca^ 62. Among Na+, Na, Mg and Mg2* the largest particle is
(c) CaF2 (d) CaB^ (a) Mg2* (b) Mg
51. The substance used as pigment in paint is (c) Na (d) Na+
(a) borax (b) alumina 63. The electron affinity of Be is similar to
(c) lithophone (d) None of these (a) He (b) B
52. Lightest alkaline earth metal is (c) Li (d) Na
(a) Be (b) Mg 64. Bleaching action of CaOCl2 is due to
(c) Ca (d) Sr (a) nascent oxygen (b) chlorine
53. Which of the following is not an ore of magnesium? (c) HC1O (d) HC1
(a) Carnallite (b) Magnesite 65. In alkaline earth metal sulphates, the value of
(c) Dolomite (d) Gypsum hydration energy decreases down the group because
54. The activity of alkaline earth metals as reducing of the
agent (a) decrease in size
(a) decreases from Be to Ba (b) increase in size
(b) increases from Be to Ba (c) greater lattice energy
(c) increase form Be to Ca and decreases from Ca to (d) None of the above
Ba 66. The correct order of increasing ionic character is
(d) decreases from Be to Ca and increases from Ca to (a) BeCl2 <CaCl2 <MgCl2 <BaCl2
Ba (b) BeCl2 < MgCl2 < BaCl2 < CaCl2
55. Magnalium contains (c) BeCl2 < BaCl2 < MgCl2 < CaCl2
(a) aluminium + magnesium (d) BaCl2 <CaCl2 <MgCl2 <BeCl2
(b) magnesium + copper 67. Ripening of fruits can be caried out in presence of
(c) magnesium + iron (a) Na2SO4 (b) NaCl
(d) magnesium + silver (c) CaCl2 (d) CaGj
Chapter 9: s-Block Elements—The Alkaline Earth Metals | 423

68. Toe right order of the solubility of sulphates of 78. Select correct statement(s).
alkaline earth metals in water (a) Presence of MgCl2 in table salt causes it to clump
(a) Be > Ca > Mg > Ba > Sr (b) Addition of NaHCO3 to table salt converts MgCl2
(b) Mg > Be > Ba > Ca > Sr to non-hygroscopic salt
(c) Be > Mg > Ca > Sr > Ba (c) Both (a) and (b)
(d) Mg > Ca > Ba > Be > Sr (d) None of the above
69. Burning of Mg is extinguished by 79. In polymeric (BeCl2)n, there are
(a) throwing N2 liquid (b) throwing sand (a) three centre two-electron bonds
(c) throwing ice (d) throwing water (b) three centre three-electron bonds
70. Flame test is not given by (c) two centre three-electrons bonds
(a) Ca (b) Ba (d) two centre two-electron bonds
(c) Mg (d) Li 80. Be and Al resemble in the following but not in
71. Lithium shows similarities with magnesium in its (a) both form electron deficient hydrides
chemical behaviour because (b) both are rendered passive by HN03
(a) similar size, greater electronegativity and lower (c) both form amphoteric oxides
polarising power (d) both have sp-hybridisation in their compounds
(b) similar size, same electronegativity and lower 81. Select incorrect statement.
polarising power (a) NaOH can be stored in a vessel made of Al
(c) similar size, same electronegativity and similar (b) HNO3 can be stored in a vessel made of Be/Al alloy
high polarising power (c) HF can be stored in a vessel made of wax
(d) None of the above (d) HF attacks glass
72. The element whose electronic configuration is 82. Metal carbides on reaction with HjO from CH4.
ls22s22p63s2 is
i Carbide can be
(a) non-metal (b) noble gas (a) CaQ, (b) Mg^
(c) metalloid (d) metal (c) Be2C (d) SiC
73. On strong heating MgCl2 • 61^0 the product obtained 83. Select correct statement.
is (a) Be and Al show diagonal relationship
(a) MgCl2 (b) MgO (b) Be form tetrahedral complexes [BetC^O.^ f"
(c) MgCl2 • 21^0 (d) MgCl2 • 41^0 (c) Al forms A1F63~, an octahedral complex
74. Which of the following carbonate decompose most (d) All the above are correct statements
easily on heating?
(a) Rb2CO3 (b) KsCOg Brain Twisters
(c) Na2CO3 (d) MgCO3
1. Metals reacts with hydrogen halides (such as HC1) to
75. The solubilities of carbonates decrease down the give metal halide and hydrogen :
magnesium group due to a decrease in M(s) + nHX(g) ---- > MXn(s) + lnH2(g)
(a) lattice energies of solids Li
(b) hydration energies of cations
(c) interionic attractions AG° = AG°(AfXn) - nAG°(HX(g))
(d) entropy of solution formation AG°(HCl(g)) = - 95.1 kJ mol’1.
76. The charge/size ratio of a cation determines its What is relative value of AG° for MXn for the reaction
polarising power. Which one of the following to be product-favoured?
sequences represents the increasing order of the
polarising power of cationic species; K+, Ca2*, Mg2*, 2. A mineral MgxBay(CO3)2 was analysed as given
Be2*? below:
(a) Mg2* <Be2* <K* < Ca2* A sample of the mineral was dissolved in excess HC1
(b) Be2* <K* <Ca2* <Mg2* and the solution made up to 100 mL with water.
(c) K* <Ca2* <Mg2* <Be2* During the process 48 mL of CO2 measured a 25°C
(d) Ca2*<Mg2* <Be2* <K* and 1 atmosphere pressure were evolved.
77. What is impurity (as a salt) associated with table salt A 25.0 mL portion of the resulting solution required
obtained from sea-water? 25.0 mL of EDTA solution of 0.02 M to reach end
(a) NaHCO3 (b) MgC03 point. A further 25.0 mL portion gave a precipitate of
(c) MgCl2 (d) Nal BaSO4 of mass 0.058 g on treatment with excess of
424 | Essential Inorganic Chemistry

dil.H2SO4. You may assume that group 2 (alkaline 2. In the mineral barium ion can be identified by
earth metals) metal ions form 1:1 complexes with (a) green colour in flame
EDTA. (b) yellow precipitate with CrO^- solution
(Given molar volume of any gas at 25°C and (c) white precipitate with SO^"
1 atmosphere pressure = 24 L) (d) all of the above
3. Identical mineral of calcium and magnesium ions is
3. Use the following data to calculate second electron
affinity of oxygen, i.e., for the process (a) dolomite (b) magnesite
(c) epsomite (d) talc
O"(g) + e-(g) ---- > O2"^)
Is the O2- ion stable in the gas phase? Why is it stable Passage 2
in solid MgO?
Questions given below are based on the following
Heat of sublimation of Mg(s) = + 147.7 kJ mol-1 enthalpy values
Ionisation energy of Mg(g) to form
The standard enthalpy of formation AHp of
Mg2+(g) = + 2189.0 kJ mol-1
hypothetical CaCl(s) theoretically found to be
Bond dissociation energy for O2 = + 498.4 kJ mol-1 -188 kJ mol-1 and that of CaCl2(s) -795 kJ mol-1.
First electron affinity of 0(g) = - 141.0 kJ mol-1
1. Which of the following compounds is more stable?
Heat formation of MgO(s) = - 601.7 kJ mol-1 (a) CaCl(s)
Lattice energy of MgO = - 3791.0 kJ mol-1 (b) CaCl2(s)
4. A 1.005 g sample of an unknown alkaline earth metal (c) Both (a) & (b) are equally stable '
was allowed to react with a volume of chlorine gas (d) Nothing can be said
that contains 1.91 x 1022 Cl2 molecules. The resulting
metal chloride was analysed for chlorine by 2. Select most spontaneous reaction
dissolving 0.436 g sample in water and adding an (a) 2CaCl(s)---- > CaCl2(s)+ Ca(s)
excess of AgNO3(aq).The analysis yielding 1.9126 g of (b) CaCl2(s)+ Ca(s)---- > 2CaCl(s)
AgCl.
(c) CaCl(s)---- > Ca(s)+ Cl(g)
(a) What is the identity of the alkaline earth metal?
(b) What is the % of Cl in the alkaline earth chloride. (d) CaCl2(s)---- > Ca(s)+ Cl2(g)

Passage Comprehension Questions Passage 3


AG° values (kJ mol-1) for the following cases can given
Passage 1 MC03 ■» MO + CO2
Based on the following analytical data, answer the -394.4
questions. Mg -1028.2 -568.9 64.9
A mineral, which can be represented by the formula Ca -1129.2 603.4 165.9
MgrBa>.(CO3)z, was analysed as described below: Ba -1134.4 -520.4 219.6
A sample of the mineral was dissolved in excess Answer the following questions
hydrochloric acid and the solution made up to 100 cm3 1. Maximum value of AG° is for the reaction when Mis
with water. During the process 48 cm3 of carbon dioxide, (a) Mg (b) Ca
measured at 25°C and 1 atmosphere pressure, were (c) Ba (d) equal for all cases
evolved.
2. Thermal stability of the carbonates is in order
A 25.0 cm 3 portion of the resulting solution required
(a) MgCO3 < CaCO3 < BaCO3
25.0 cm 3 of EDTA solution of concentration 0.02 mole dm 3
to reach an end-point. A further 25.0 cm portion gave a (b) BaCO3 < CaCO3 < MgCO3
precipitate of barium sulphate of mass 0.058 g on (c) MgCO3 < BaCO3 < CaCO3
treatment with excess dilute sulphuric acid. You may (d) CaCO3 < BaCO3 < MgCO3
assume that group-2 metal ions form 1:1 complexes with 3. Which is decomposed most easily?
EDTA. (a) CaCO3 (b) MgCO3
Molar volume of any gas at 25°C and 1 atmosphere (c) BaCO3 (d) Equally
pressure = 24 dm3). 4. BeCO3 decomposes at low temperature. To avoid its
1. Formula of the mineral is decomposition, it is placed in the atmosphere of
(a) MgBa(CO3)4 (b) MgBa(CO3)2 (a) N2 (b) He
(c) MgBa(CO3)3 (d) Mg2Ba(CO3)4
(c) Ne (d) CO2
Chapter 9: s-Block Elements-The Alkaline Earth Metals I 425

Passage 4 True & False


Ca'cium oxide, CaO, is used to remove S02 from 1. When CO2 is passed into lime water, white turbidity
power plmt exhaust."
Baseci on the above statement, answer the following appears which dissolves in excess ofCO2 •
questions 2. S02 turns lime water milky.
I. In what form CaO removes S02 ? 3. [Be(�0) 4 f + is an acid.
(a) CaS0 3 (b) CaS04
{c) Ca(HS03)2 (d) Ca(HS04 )2 4. Solubility of sulphates of alkaline earth metals
increases going down the group.
2. What mass ofS02 can be removed using 1.4 x 103 kg
5. Mg 2C3 on reaction with water forms propane.
CaO?
(a) 1.4 x 10 3 kg (b) 1.6 X 10 3 kg 6. CaC03 is the main constituent of egg shell.
(c) 2.2 x 10 kg
3
(d) 2.4 x 103 kg
7. EDTA is used to estimate Ca and Mg.
3. Presence of S02 from power plant exhaust can be
detec ted by 8. Permanent hardness is due to dissolved bicarbonates
{ a) C��- which changes to green ofCa2 + and Mg2+.
(b) Mn04 which changes to colourless 9. BeC12 is a nucleophile.
(c) both (a) & (b) above
(d) None of the above 10. BeO and Al 203 are amphoteric.
11. Be and Al are made passive by HN03 •
Assertion & Reason 12. Chlorophyll contains Mg and chlorine.
Codes: 13. Chlorophyll produces sugars for the plants in
(a) Both A and R are true and R is the correct presence ofsunlight, CO2 and �O by photosynthesis.
explanation of A
14. Chlorophyll is an organometallic compound.
(b) Both A and R are true but R is not the correct
explanation of A 15. The lattice energies of alkaline earth metal salts are
(c) A is true but R is false. much higher than these of alkali metal salts.
(d) A is false but R is true. 16. Decreasing lattice energy favours increased
I. Assertion (A) : BaS04 is likely insoluble in water solubility, while decreasing hydration energy favours
but readily dissolves in solution of sodium salt of decreased solubility.
ethylene diamine tetra acetic acid (EDTA) . 17. BeO has hexagonal ZnS structure.
Reason (R) : Ba2 + forms a very stable complex
18. Beryllium forms hydrate peroxide Be02 • �O.
with the anion of the sodium salt EDTA which is
water soluble. 19. Calcium and barium bicarbonates are stable only in
2. Assertion (A) : Temporary hardness can be solution and decompose on removal of water to give
removed by addition of lime. carbonates.
Reason (R): Ca(HC03) 2 in hard water is converted 20. Formation of stalactite and stalagmite is the
to insoluble CaC03 on moderate heating. property of calcium and barium bicarbonates which
3. Assertion (A) : Group 2 metals have higher change to carbonates.
melting points when compared to the group 1 metals.
Reason (R) : +2 charge on the cations in the Fill in the Blanks
metallic lattice causes them to be more strongly 1. . .. .. ... .... of IA and .. .... . .. ... of IIA show diagonal
attracted to the 'sea of electrons' and making it relationship.
diffic�lt to pull them apart.
2. (JE) 1 ofBe is ........ .... than that ofB and (IE) 2 ofBe is
4. Assertion (A) : The metals of group 2 (except Be)
from ionic compounds. ............ than that of B.
Reason (R) : Electropositive character and the 3. Of the alkaline earth metals only ............ carbide
reducing property increase on moving down the group. gives CH4 on reaction with �O.
5. Assertion (A) : Divalent compounds of alkaline 4. Alkali metal and alkaline earth metal chlorides can
earth metals are diamagnetic. be detected by ............ test.
2+
Reason (R) : M ions of group 2 have noble gas 5. BeC12 is an .............. and has .............. structure in
configuration. solid state.
426 | Essential Inorganic Chemistry

6. Be(OH)2 is 2. Match the mineral names (in List I) with the


chemical formulae (in List II).
7. Solubility of sulphates and carbonates of alkaline
earth metals as the atomic number of the List I List II
metal increases (down the group). 1. KCI-MgCI2-6H2O
A. Magnesite
8. BeCO3 is placed in an atmosphere of since, it B. Dolomite 2. MgSO4 H2O
is
C. Kieserite 3. MgCO3
9. Be and show diagonal relationship.
D. Carnallite 4. MgCO3-CaCO3
10. Be is not readily attacked by acids due to formation of
layer. 3. Match compounds (in List I) with the specific
11. Be dissolves in alkali to give properties of the compounds (in List II).
12. HCO3 is List I List II

13. Be forms complexes with coordination number A. BeCO3 1. Temporary hardness

B. BaCI2 2. Permanent hardness


14. The thermal stability of group 2 oxysalts is C. MgSO4 3. Decomposes readily
than those of the group 1 oxy salts.
D. Ca(HCO3)2 4. Gives green edged flame
15. Ca(NO3)2 decomposes into gases and

II. One or More Correct


16 is the primary constituent of teeth and
bones 1. Match the species in Column I with their
corresponding property (ies) in Column II.
17. Amphoteric oxide of group 2 is
Column I Column II
18. MgO is used as
19. Ca(OH)2 is called and Ba(OH)2 is called A. BeO 1. Amphoteric

B. AI2O3 Diagonal relationship

20. Most soluble sulphate of group 2 is C. MgO 3. Lime water

21. Potash magnesis is ... D. CaO 4. Bartya water

22. CaSO4 ■ HjO is called and is used to make E. BaO 5. Water insoluble
Maximum soluble in water

23. BeCl2 fumes in air due to formation of 7. Estimation by EDTA

24. BeCl2 exists as monomer in state. 2. Match the species in Column I with their
25. Polymeric BeCl2 has centre corresponding property (ies) in Column II.
electron bonds. Column I Column II
A. Li 1. Diagonal relationship
Matrix-Match Type Questions B. Be 2. Maximum standard reduction
I. Only One Correct electrode potential (E”)

1. Match the commercial names (in List I) with the C. Mg 3. Cation is maximum hydrated
chemical formulae (in List II). D. Al 4. Cation has maximum ionic
mobility in aqueous solution
List! List II
E. Ca 5. Fluoride and carbonate least
A. Magnesia 1. BeO soluble in water
B. Epsom salt 2. Ca(OH)2
6. Coordination number six in the
C. Anhydrone 3. MgSO4-7H2O complexes
D. Hydrolith 4. CaH2
7. Ions are involved in the
E. Beryllia 5. MgO transmission of electrical
F. Slaked lime 6. Mg(CIO4)2 impulses in human body.
Chapter g: s-Block Elements-The Alkaline Earth Metals I 427
lntege1 Answer Types 5. 96 g of Mg is burnt in air in a closed chamber. On
analysis 25% of Mg is converted into oxide and
This Eection contains 8 questions. The answer to each remaining Mg into other possible product. Residue is
or the questions is a single digit integer, ranging from Oto dissolving in H:zO and neutralised by �SO4 • Number
9. The awropriate bubbles below the respectively of moles of �SO4 required is ............
question numbers in the ORS have to be darkened. For
6. E� 2• 1 M values of some metals are
uample, if the correct answers to question number X, Y, Z
and lV (say) are 6, 0, 9 and 2, respectively, then the correct Be - 1.85 V
darkening of bubbles will like the following: Mg -2.37V
Ca -2.87V
X Y Z W
Sr -2.89V
@ @ @ @ Cu +0.34V
CV G) G) G) Hg 0.80 V
® @ @ ® Number of metals which can displace� gas from
@ @ @ @
acid is ............
@ @ @ @
@ @ @ @ 7. Mg�a y CCO3) z has 8.54% Mg, 48.75% Ba and
® ® ® ® remaining carbonate. 0.281 kg is decomposed by
(J) (J) (J) (J) �SO4• CO2 formed is ............ mole(s).
@ @ @ @ 8. In question (7), x+ y + z = ........... .
® ® ® ®

Set I
Test Yourself on the Periodic Table
1. Identify the elements in the Periodic Table
1. Coordination number of complexes of beryllium is A : which shows diagonal relationship to Be
········· B: which shows diagonal relationship to Li
2. Number of water molecules in gypsum is ......... C : whose alkali metal carbonate is decomposed on
heating
3. Epsom salt contains ......... water molecules (of
D : coinage metals
hydration)
4. Magnesium allyide is Mg 2Cx where xis .........
5. Magnesium nitride is Mg� Y where (x + y) is .........
6. Magnesium is coordinated to ......... N atoms in
chlorophyll.

111111111111111
7. Bleaching powder has two types of chlorine one with
oxidation number -1 and other with oxidation
number .........
8. One mole of Ca(HCO3 )2 is neutralised by ......... 2. Out of the elements marked A, B, C, D, E, F, G, and H
(a) which form superoxide?
mole(s) of KOH.
(b) which form thermally stable carbonate?
Set II (c) which forms strongest base?
(d) which show diagonal relationship?
1. Number of electrons in magnesium ion formed by (e) which forms amphoteric oxide?
three steps ionisation of magnesium is ............ - ,-
· 9 Be 3 + 1s -
2• Number of neutrons m •

o
4 ............
3. Magnesium is coordinated to N-atoms in chlorophyll. E F
Number of rings in chlorophyll is ............ GH
4. When Cl 2 is passed into hot alkali solution, products

1111 I .111111111 I
formed have two types of chlorine. Difference in
oxidation number of two chlorine atoms is ............
Answers & Solutions
Master Exercises
Exercise 1
Short Answer Type Questions When mixed with saw-dust or cork waste, sorel cement
1. (a) Be <Ca <Ba <Ra gives a weather proof composition.

(b) Ba < Ra < Ca < Be


Analytical Questions
(c) Ba <Ca <Be
1. Chlorophyll
(d) Ba <Ca <Be(=0)
2. Hydrated cation is [Be(H2O)4 ]2+.
2. (a) RaCl2 < BaCl2 < SrCl2 < CaCl2 < MgCl2 < BeCl2
[Be(H2O)4]2++H2O ---- > [Be(H2O)3(OH)]++ H3O+
(b) BeCl2 < MgCl2 < CaCl2 < SrCl2 < BaCl2 < RaCl2
(c) RaCl2 < BaCl2 < SrCl2 < CaCl2 < MgCl2 < BeCl2 hydrated cation is H+ ion donor, hence, it behaves as an
acid (Lowry-Bronsted protonic concept).
(numerical value)
3. This indicates that concentration of NaOH is very high
(d) RaCl2 < BaCl2 < SrCl2 < CaCl2 < MgCl2 < BeCl2
hence, pH is greater than 14.
3. (a) Ca(HCO3)2 < NaHCO.3 4. BeCl2 + 2H2O ---- > Be(OH)2 + 2HC1
(b) CaCO3 < Na2CO3 HC1 causes fumes formation.
(c) BeCO3 < Li2CO3 5. (a) See text (b) See text 6. See text
(d) NaHCO, < Na 2CO3
O it Q 7. (A):BaO2 (B):BaSO4 (C):H2O2 8. Refer text
4. Mg + - O2 ---- > MgO 9. (a) Refer text (b) Refer text 10. (a) to (d) Refer text
2
3Mg + N2 ---- > Mg3N2 11. (a) Na2CO3 —no effect
5. Barium carbonate due to very high lattice energy CaCO3 CaO + CO2
decomposes at about 1400°C
BaCO3 ---- > BaO + CO2 (b) MgCl2 -6H2O Mg(OH)Cl-5H2O + HCl
At such high temperature, the reaction tends to become CaCl2 -6H2O CaCl2+6H2O
reversible. Carbon added combines withCO2 forming CO
and thus reaction is made irreversible. (0 2Ca(NO3)2 -A* 2CaO + 2NO2 + 3O2
CO2 + C ---- > 2CO A
2NaNO3 2NaNO2 + O2
6. If strongly ignited magnesium oxide is added to a
concentrated solution magnesium chloride, the slurry so
obtained solidifies in the course of a few hours, to a solid
NCERT Problems
mass. This is called sorel cement or magnesia cement. Refer Text

Exercise 2
Only One Option Correct
1. (a) 2. (c) 3. (d) 4. (c) 5. (d) 6. (b) 7. (b) 8. (c) 9. (c) 10. (a)
11. (a) 12. (d) 13. (c) 14. (d) 15. (c) 16. (d) 17. (d) 18. (b) 19. (a) 20. (a)
21. (a) 22. (b) 23. (a) 24. (c) 25. (a) 26. (d) 27. (b) 28. (d) 29. (d) 30. (c)
31. (c) 32. (a) 33. (d) 34. (a) 35. (d) 36. (d) 37. (b) 38. (b) 39. (d) 40. (d)
41. (d) 42. (c) 43. (d) 44. (a) 45. (d) 46. (a) 47. (a) 48. (a) 49. (b) 50. (c)
51. (c) 52. (c) 53. (d) 54. (b) 55. (a) 56. (a) 57. (b) 58. (b) 59. (c) 60. (c)
61. (d) 62. (c) 63. (a) 64. (c) 65. (b) 66. (a) 67. (d) 68. (a) 69. (c) 70. (c)
71. (c) 72. (d) 73. (b) 74. (d) 75. (a) 76. (c) 77. (c) 78. (c) 79. (a) 80. (d)
81. (a) 82. (c) 83. (d)
Chapter 9: s-Block Elements—The Alkaline Earth Metals | 429

Brain Twisters MC12 + 2AgNO3 -> 2AgCl + M(NO3)2


(A+ 71) 245.3 x 2
1. If the reaction is to be product-favoured, i.e., it is 0.436 1.126
spontaneous in forward side then AG° should be A+ 71 243.5x2
negative and AG° for MXn should be greater than and
0.436 1.9126
more negative than (95.1) x n kJ.
A = 40 g mol"1
2. 24 L of CO2 at 298 K and 1 atm = 1 mol CO2
(a) Thus element is Ca
48 mL (= 0.048 L) = — x 0.048 (b) Molar mass ofCaCl2 = 111 g mol'
24
= 0.002 mol CO2 in 100 mL solution 71
%ofCl = —xlOO
= 0.02 M 111
= 63.96%
EDTA complexes with Mg2+ and Ba2+
M1V1 =M2V2
Mj(Mg2+ and Ba2+) x 25 = 25 x 0.02
Passage Comprehension Questions
= 0.02 M = 0.02 molL"1 Passage 1
Af1(Mg2+)+M1(Ba2+) = 0.02M 1. (b) 2. (d) 3. (a)
Ba2+ + SO2" ---- > BaSO4 Passage 2
137 g = 1 mol 233 g = 1 mol 1. (b) 2. (a)
233 g BaSO4 is from = 137 g Ba2+ =1 mol Ba2+
Passage 3
2+
0.058 gBaSO4 is from = = 2.5 x 10“* mol Ba 1. (c) 2. (a) 3. (b) 4. (d)
4 233
in 25 mL Passage 4
[Ba2+] =0.01 mol L"1 1. (a) 2. (b) 3. (c)
[Mg2+] = 0.01 mol L"1
Thus, Mg2+,Ba 2+ and CO," are in molar ratio of 1:1:2 Assertion & Reason
a Salt is MgBa(CO3 )2
1. (a) 2. (b) 3. (a) 4. (a) 5. (b)
3. Stepwise formation of MgO involves
Step Reaction AW kJ mol True & False
1. T 2. T 3. T 4. F(decreases)
I Mg(s) ---- > Mg(g) 147.7
5. F (propyne) 6. T 7. T
II Mg(g) —> Mg2 + (g)+2e" 2189.0
8. F (due to Cl" and SO2")
102(g) —> 0(g) 498.4
III
2 9. F (BeCl2 is an electrophile)
IV O(g)+e~ —> O"(g) -141.0 10. T 11. T 12. F 13. T
V O"(g)+e" —> O2"(g) q 14. F (Mg is coordinated to N)
VI Mg2 + (g)+O2-(g) ---- > MgO(s) -3791.0 15. T 16. T 17. T 18. F 19. T 20. T

Mg(s) +1 o2(g) — MgO(s) Fill in the Blanks


1. Li, Mg 2. higher, smaller 3. beryllium
AH = -1346.1 + 9 4. flame 5. electrophile, polymeric 6. amphoteric
Mg(s)+|o2(g) ---- > MgO(s)
7. decreases 8. CO2, thermally unstable 9. Al
AH2=-601.7 10. oxide 11. Na,BeO2andH2 12. amphoteric
By Born-Haber Cycle (based on Hess law) 13. four 14. less 15. NO2,O2 16. Ca3(PO4)2
AH1 - AH2 17. BeO 18. refractory 19. lime water, baryta
-1346.1 +q = -601.7 water
9 = 744.4 kJ mol"1 20. BeSO4 21. KoSO, MgSO4 -6H2O
4. Let molar mass of the element (M) = A 22. Alabaster, ornaments 23. HC1 24. vapour
Moles of element = - 25. three, two
A
430 | Essential Inorganic Chemistry

Matrix-Match Type Questions Set II


I. Only One Correct Questions —► 1 2 3 4 5 £ 7_ 8
Answers —► 9 5 8 8 5 4 2 4
1. (A>-(5); (B)—(3); (C)—(6); (D)—(4);
(E)—<1); (F>—<2); © © © © © ©I© ©
2. (A)-<3); (B)—(4); (C)—(2); (D)—(1) © © © O
3. (A)-(3); (B)—(4); (C>—(2); (D>—(1) © © © © © ©
© © © © © ©@ ©
II. One or More Correct © ©
1. (A)—(1,2,5); (B)-(l,2,5); (C)—(2,7); © © © © © ©__©
(D>—(3,7); (E)-(4,6) © © © © © ©._© ©
2. (A)—(1,2,); (B)-(l,3); (C)—(1,6,7); © © © 0 ©
(D>—(1,6); (E)—(4,5,6,7) © © © © © ©
© © © © © © © ©
Integer Answer Types
Set I Test Yourself on the Periodic Table
Questions—► 1 2 3 4 5 7 8 i. n
Answers —► 4 2 7 3 5 4 1 2 c~ nn-
© © © © © ©I© © B D A
© ©. O © © ©
© © © © © ©|© © 1
© © © © © © © ©
© © © @|©
© © © © © ©
© © © © © ®|© ©
© © © © © 2. (a) E and G (b) C, E, and G (c) G
© © © © © ©I® © (d) A and D (e) B
© © © © © © © ©
The science fiction writer Jules
Verne had an uncanny knack of
predicting the future. In 1874, he
wrote ''I believe that water will
one day be employed as fuel, that
hydrogen and oxygen which
constitute it, used singly or

Hydrogen and
together, will furnish an
inexhaustible source of heat and
light" Many now believe that this

Its Compounds
prophecy has come true in the
twenty.first century when a major
research work is being carried out
with hydrogen and it may become
a universal fuel.

Quick Points
• Ionisation Energy is the energy required to convert gaseous
atom to gaseous ion, i.e.,
MI =IE • Hydrogen
• Electronegativity is measured in Pauling scale (w.r.t. H) ■ Hydrogen Peroxide
• Entropy is the measure of the disorder of the system

� = !!._

• Ionic product of H2 0, K,. = [H 30+ ][0H-] = 1 x 10- 14 at


298K
Ionisation constant of a weak acid HA (with ionisation step
HA+ H20 --+ H 30 + K) is
+

[H o+ uA-1
� = __,3'-----
--a
[HA]
• In an electrochemical cell, oxidation takes place at anode
and reduction takes place at cathode.
• Mno; is used to determine concentration of H202
volumetrically
2
Mno; + 8W + 5e- � Mn • + 4H 20
HP2 � 2H +�-
+
+ 02
• 1 mol of every gas at STP occupies volume 22.4 L or
3
22400 mL or 0.0224 m
1
• Molarity is the concentration in mol L-
• Nonnality is the concentration in equivalent L-1
432 | Essential Inorganic Chemistry

In many ways, hydrogen is unique and although it is - by forming an electron pair (covalent) bond with
hardly found on earth as the free element, life here would another atom as in , H2O, NH3 HC1, CH4
be impossible without it (Fig. 10.1). It accounts for - by losing an electron to form H+; a proton is ol
virtually all of our energy, either directly in the form of approximately 1.5 x 10"5 A radius compared tc
sunlight, or indirectly as fossil fuels. Without hydrogen that of H which is 0.7414 A. Due to its very small
there would be no water to drink and DNA molecules size, H+ has a very high polarising power, anil
would not form the double-helix structure that allows our therefore distorts the electron cloud on other
genetic code to be copied and passed on to future atoms. H+ (protons) is always associated with
generations. other atoms or molecules, as H3O+, H90^ or
HG^O)* ions.
Other 1^0 +H+ ---- > H3O+
(H2O)n + H+ ---- > [H(H2O)J+
Methanol Free protons do not exist under normal
manufacture ; conditions though they are found in low
' j
Fuel-p pressure gaseous beams (as in spectrometer)
- by gaining an electron to form H“; crystalline solid
Food
such as LiH contains the H” ion and are formed by
processing highly electropositive metals (as groups I, II).
Petroleum -
treatment ■ !
Position in the Periodic Table
• Hydrogen is the first element in the Periodic Table.
Ammonia - Hydrogen is placed in no specific group due to its
manufacture
property of gaining electron (when H' is formed) and
/ also losing electron (when H+ is formed).
0 5 10 15 20 25 30 35 40 • Hydrogen is placed in Group I (alkali metals) as:
Fig. 10.1 Some of the most important industrial uses of - it has one electron in its (outer) shell—Is1 like
hydrogen other alkali metals which have [inert gas] ni
configuration
- it forms monovalent H+ ion like Li+, Na+,....
DEEP Focus - its valency is also 1
Hydrogen is the most abundant lightest element in the - its oxide (HgO) is stable as Li2O, Na20
universe and oxygen is the most abundant element on the
earth’s surface. In the earth's crust and oceans, hydrogen is - it is a good reducing agent (in atomic as well as
the ninth most abundant element on a mass basis (0.9%) molecular state) like Na, Li,....
and the third most abundant on an atom basis (15.4 atom • Hydrogen also resembles halogens (Group 17) as:
%). Hydrogen combines with every element in the Periodic - it is also diatomic (H^) like F2, Cl2,....
Table except the noble gases and forms more compounds than
any other element. - it also forms anion H” like F", Cl” by gain of
one electron
Henry Cavendish, an English chemist and physicist, - H” has stable inert gas [He] configuration as F”,
was the first person to isolate hydrogen in pure form (but Cl”, Br”, and I" have
did not name it) by action of dilute acids such as HC1, - it forms covalent compounds with carbon as CH4,
C^Hg like halogens CC14, CF2C12 etc.
on metals such as Zn, Sn, etc.
~ H is one electron short of duplet (stable
Zn(s)+ 2H3O+(aq)---- > Zn2+(aq)+ H^gH 2^0^) configuration) like F, Cl, ... which are also one
The French chemist Lavoisier called the gas electron deficient than octet
“hydrogen” meaning “water former” because it F:2s22p5 Cl:3s23p5
combines with oxygen to produce water. - (IE) of H(1312 kJ mol”1) is of the same order as
that of halogens, as :
Hydrogen Atom (/E), kJ mol
Electronic Structure (1s1) H 1312
Hydrogen has the simplest atomic structure of all the F 1681
elements, and consists of a nucleus containing one proton Cl 1256
with a charge +1 and one orbital electron (Is1). Atoms of Br 1143
hydrogen may attain stability in three different ways : I 1009
Chapter 10: Hydrogen and Its Compounds | 433

• (IE) of H is very high in comparison with alkali This mixture of CO and H2 is also known as water
metals. Also size of H+ is very small compared to that gas and the reaction is referred to as water gas reaction.
of alkali metal ions. H forms stable hydride only with As this mixture (CO + H2) is used for the synthesis of
strongly electropositive metals due to smaller value of methanol an a number of hydrocarbons., this is also called
its electron affinity (72.8 kJ mol"1). synthesis gas.
• In view of the anomalous behaviour of hydrogen, it is (2) Hydrogen is now produced by the reaction of
difficult to assign any definite position to it in the natural gas (chiefly methane) with steam in presence of a
Periodic Table. Hence, it is customary to place it in nickel catalyst.
Group I (along with alkali metals) as well as in Group Ni catalyst
CH4 +H2O 920-1170 K> CO + 3H2
(17) VIIA (along with halogens).

1(IA) 17(VIIA) Similar reaction can occur with other hydrocarbons


H H also. In both the above processes CO is converted to CO2
Li and more hydrogen generated by passing the gases
F
(00+-^) and steam over iron oxide or cobalt oxide
Na Cl
catalyst.
K Br
catalyst
Rb I (iron oxide)
Cs At
CO+HjO + Hg 450-550 K > CO2 + 2Hp
Fr
This reaction is called water gas shift reaction
Table 10.1 A Comparison of Some Properties of Hydrogen,
Lithium, Carbon and Fluorine DEEP Focus
Hydrogen and carbon dioxide can be easily separated from
Property Lithium Carbon Fluorine each other by bubbling the gas mixture through water in
Hydrogen
(Group 1) (Group 14) (Group 17) which CO2 is fairly soluble and H2 virtually insoluble.
Electron 1s1 [He] 2s1 [He] 2s22p2 [He] 2s22p H9Z0 + C0Z9 ---- > H,CO,
Zu
configuration
First ionizalion +1312 +519 + 1090 + 1681 (3) At higher temperature, in the presence of
energy/kj catalysts (silica, alumina) hydrocarbons decompose and
mor'
rearrange in what are called cracking reactions. These
Electron -72 -52 -120 -348 reactions, which are used in refining of petroleum,
cfnity/kJmor1 produce hydrogen as a by-product.
Peclronegati-
v?/ (Pauling)
Structure
2.1

diatomic
molecules
1.0

metallic
lattice
2.5

covalent
macromolec
4.0

diatomic
molecule
2CH3—CH2— CH3
propane s CH<+CI^=CH»
+ CH3—CH=CH2 +H2?
ule
Atomization +218 + 161 +715 +79 (4) Electrolysis of acidified water using platinum
energy electrodes. On a large scale, very pure hydrogen
Wmol'1
(> 99.95%) can be obtained from the electrolysis of
aqueous solutions of barium hydroxide between nickel
Manufacture of Hydrogen electrodes.

• Water is the natural abundant source for the


21^0 current^ 2H,(g) + O2(g)
at cathode at anode
manufacture of hydrogen.
Hydrogen is economically obtained as a by-product in
Reduction of water of hydrogen can be carried
the electrolysis of brine solution in the manufacture of
out.
sodium hydroxide.
(1) Chemically (2) Electrolytically common reducing
At anode:
agents are (1) Coke (2) Carbon monoxide (3) Hydrocarbons
During electrolysis there is a competition at the
(1) Hydrogen is manufactured by allowing steam to anode between the oxidation of chloride ion and the
react with red hot coke at about 1250 K. oxidation of water.
2Cl"(aq) ■> Cl2(g) + 2e~
C + HgO
2^0(1) + O2(g) + 4H+(aq) + 4e~
434 | Essential Inorganic Chemistry

When a concentrated salt solution (brine) is used, the Table 10.2 Physical Properties of Hydrogen
first reaction is the one that is observed.
At cathode : Relative atomic mass A.008
The reaction is the reduction of water because it is Electron structure IS
more easily reduced thanNa-’ (/E),H(g) —» H+(g)+e- ^12^°'
Electronegativity (Pauling scale) 2-A
2H2O(/) + 2e“ 4 H2(g) + 2OH"(ag)
Electron affinity _72V.Jr0°'
Overall reaction is
Molecular formula Hz
2Cr+2H2O -----> Cl2(g) + H2(g) + 2OH~ Melting point -259°c
Water is very stable to thermal decomposition, it does Boiling point ,253°C
not decompose to hydrogen and oxygen to any appreciable Density at STP 0.09
extent at practically viable temperatures. Great efforts BE. (H—H) 436^^°'
are being made to make electrolysis of water more Bond length (H—H) 74p^
energy-efficient so that the use of hydrogen as a fuel can Covalent radius 37
become economically viable. van der Waals’ radius A20Prn
(5) Non-electrolytic methods of preparation of Ionic radius (H-) 208P^A m
hydrogen, e.g., thermochemical and photochemical. In Standard entropy 731 ,
thermochemical hydrogen generation, water is heated Heat capacity (25°C) 297 K m°'
with certain chemicals to liberate hydrogen at much lower
temperatures. Two potentially useful cycles are given • At ordinary temperatures and pressures, hydrogen is
below : a colourless, odourless and tasteless gas composed of
diatomic Hj molecules. Because Hg is a non-polar
775 K
(i) 3FeCl2 + 4^0 ------ > Fe3O4 + 6HC1 + H2 molecule that contains only two electrons,
intermolecular forces are very weak hence, it (in
Q
375 K
(ii) Fe3O4 +-C12 +6HC1 * 3FeCl3 + 3H2O the solid form it has hexagonal close-packed
Cycle 1 2
structure) has very low melting point (-259°C) and
+ -O2 low boiling point (-253°C). The bonding forces within
2 2 the H2 molecule are exceptionally strong and H,
|(iii)3FeCl3 3FeCl2 + |ci2 molecule is thus, thermally stable. Even at 2000 K.
z only 1 of every 2500 H^ molecules is dissociated into
H-atoms.
(i)CaBr2 + H2O -°25K) CaO + 2HBr
Atomic Hydrogen
375 K T Hydrogen is the lightest element known. It is a
(ii)Hg+2HBr ------- > HgBr2 + H^
Cycle 2 colourless, odourless and tasteless gas. Hydrogen
280 K molecule is thermally stable and has little tendency to
(iii)HgB^ +CaO --------> HgO+CaBr,
dissociate at normal temperature, since the reaction.
775K TT - 1 H;(£) -----> 2H(g), AH = 436kJmor’
(iv)HgO ------- > Hg + sO2
2 is highly endothermic. However, at high temperatures in
Thus, by repeating reactions (i)—(iii) in cycle 1 or an electric arc or under ultraviolet light, it does dissociate.
reactions (i)—(iv) in cycle 2, water can be splitting to H2 The atomic hydrogen produced, exists for less than half
andO2. a second. after which it recombines to give
molecular hydrogen and liberates a large amount of
Net reaction is H2O > H2 + - O2 energy (436 kJ mol-1) in the form of heat. Most of the
transition metals catalyse the recombination reaction of
Decomposition of water by solar energy in presence of atomic hydrogen.
catalyst is known as photochemial decomposition of
water. Using special catalysts, scientists in France have Atomic hydrogen is a powerful reducing agent and
been able to achieve the efficient decomposition of water reduces copper, silver and mercury salts to the metallic
under visible and ultraviolet light. state. It combines with alkali metals to form hydrides
reduces sulphur to hydrogen sulphide and carbon
monoxide to formaldehyde. It also reacts with oxygen at
Properties room temperature to form hydrogen peroxide. The
reactions can be written as :
• Some of the physical properties of hydrogen atoms are
summarised in Table 10.2. HgO+2H ----- > Hg+HX)
CuO + 2H ----- > Cu + ELjO
Chapter 10: Hydrogen and Its Compounds I 435

AgN03 +H --+ Ag+HN03 Reducing Agent


Na+H--+ NaH Hydrogen is easily oxidised to water and; therefore, it
CO+2H --+ HCHO acts as a very good reducing agent in a variety of
02 +2H--+ �02 situations. Hydrogen is used in metallurgy to reduce
S+2H--+ �S metal oxides to metals in cases where carbon cannot be
used because the metal, e.g., Mo and W can form a
Atomic hydrogen is generally produced by passing
carbide:
ordinary hydrogen through an electric arc maintained
between two tungsten electordes. The atoms set free are W03 +3[½ --+ W +3[½0
tarried away by a stream of incoming hydrogen gas. These (NH4 }2 Mo04 +3[½ --+ Mo +2NH3 + 4[½0
free atoms recombine at once on coming in contact with a
metallic surface liberating a large amount of heat and • Hydrogen adds on the multiple bonds in organic
thus raising the temperature of the metal very high, say to compounds. In the presence of catalysts such as finely
4000-5000 K. This principle is utilised in making of divided nickel, palladium or mixtures of metal oxides,
the 'atomic hydrogen welding torch' (Fig. 10.2). It unsaturated organic compounds are thus reduced to
provides an opportunity of welding at a very high saturated compounds.
temperature but in a reducing atmosphere. 278 K
CH 3CH=CHCH3 + � CH 3C�C�CH 3
Despite fairly high bond dissociation energy of 2-butene Pd, 1 atm n-butane
hydrogen molecule, it is moderately reactive 673 K
because hydrogen forms strong bonds with many CH 3C!!!!!!:N +2� CHaC�NRz
ethyl cyanide ZnO/Cr2O3 ethyl amine
other elements. It reacts with almost all elements except
the noble gases. • Catalytic hydrogenation of unsaturated liquid
vegetable oils to solid edible fats.
CHa(C�) 7 CH=CH(C�hCOOR + �
(oil) oleate
---¼ CH 3 (C�) 16 COOR
(fat) stearate

• Hydrogen reacts with carbon monoxide in the


presence of catalysts to form methanol:
CO +2[½ cobal\ CH30H
This reaction is known as hydroformylation
reaction and is used in industrial preparation of methanol.

Fig. 10.2 Atomic hydrogen welding torch

Isotopes of Hydrogen
Hydrogen has three isotopes:

Name Atomic Nuclear Nuclear spin


Symbol p N Percentage
stability quantum number
Relative atomic mass
mass

Protium 1 H1 1 1 99.986 Stable -2 1.007825

Deuterium 2 2 1 1 0.014 Stable 1 2.014102


1 H (D)
Tritium 3
1H (T) 3 1 7 X 10-is Radioactive - 3.106049
2

• Physical properties of �, D2 and T2 are summarised in Table 10.3. These isotopes have same electronic
configuration and therefore their chemical properties are almost the same. The only difference is in the rate of
reaction. Hydrogen has lower energy ofactivation than deuterium in its reaction with halogen hence, reacts faster.
436 ] Essential Inorganic Chemistry

Table 10.3 Physical Properties of Isotopes of Hydrogen


Physical contents P2 t2
Freezing point -259°C - 254.3°C - 252.4°C
Boiling point - 252.6° C - 249.3°C - 248.0°C
Tc (critical temperature) -239.8°C - 234.6°C - 232.4°C
Bond length (H—H) 74 pm 74 pm 74 pm
BE (H-H) 436.0 kJ moF’ 443.3 kJ mol" 446.9 kJ mol'1
Latent heat of vaporisation 0.904 kJ mol-1 1.226 kJ mol“ 1.393 kJ mor’
Latent heat of fusion 0.117 kJ mol-1 0.197 kJ mol" 0.250 kJ mor1
Water HjO d2o t2o
Melting point
0.00°C 3.81°C 4.48“C
Boiling point
100.0°C 101.42’C 101.51°C
Density at 25°C
0.997 g/cm3 I. 104 g/cm3 1.214 g/cm3
Dissociation constant (Kw) 1.01 x W14 0.195 x IO'14 0.06 x KT14
Temperature of maximum density 4°C II. 6°C

DEEP Focus It is now prepared on a large scale by attack of slow


Although, DoO chemically resembles H20 in many respects, neutrons on isotopes of lithium-6
it is toxic substance. The reason is that deuterium is heavier fLi+in-----> fT+2He
than hydrogen; thus, its compounds often react more slowly
than those of the lighter isotope. Regular drinking of D20 Following reaction also occurs in nature
instead of H20 could prove fatal because of the slower rate of “N + in -----> ,3T+’2C
transfer of D+ compared with that of H' in the acid-base
Tritium disintegrates emitting p-particle, half-life
reactions involved in enzyme catalysis. This kinetic isotopic
effect is also observed in the ionisation of an acid. Ionisation period being 12.3 years.
constant of acetic acid. ?T-----> 23He + fe (P-particle)

CH3C00H(a^)^ CH3COO“(ag)+H+(aq),
DEEP Focus
Ka = 1.8x10
Tritium can be easily incorporated into biological molecules
is about three times as large as that of deuterated acetic because it behaves chemically, just like ordinary hydrogen.
acid: The radiation that tritium gives off within an organism, as a
CH3C00D(ag) CH3COO’(ag)+D+(ag), result of its decay, can cause many diseases as cancer.
Tritium is non-toxic, except when inhaled in large
Ka = 6x10>-6 quantities.
Deuterium undergoes exchange reactions, replacing
protium.
Ortho and Para Hydrogen
R, +D2 -----> 2HD
• In the H2 molecule, the two nuclei may be spinning in
CH4 + 2D2 —> CD4 +2H, either the same direction or in opposite direction. This
2NH3+3D2 -----> 2ND3+3H2 gives rise to spin isomerism, that is, two different
To prepare deuterium rich compounds, heavy water forms of hydrogen may exist:
(D2O) is used - if two nuclei have same spin then it is called ortho
MggN2+3D2O -----> 2ND3+3MgO H2
- if two nuclei have different spins then it is called
SO3 + D2O -----> D2SO4
paraBLj.
P4O10 +6D2O -----> 4D3PO4

00
(If T2O is used, tritium rich compounds are formed)
Mg3N2 + 3T2O -> 3NT3 + 3MgO Total spin = 1 = 1 (ortho Hj)

Tritium differs from the other two isotopes of


hydrogen in being radioactive. It was first obtained

00
synthetically by the bombardment of deuterium
compounds such as(ND4)2SO4 with fast deuterons
Total spin = |-| = O(paraH2)
2D+2D-----> 13t+ih
Chapter 10: Hydrogen and Its Compounds | 437

• There are considerable differences in physical With alkaline electrolyte, the half-cell reactions are:
properties of the ortho and para H2 because of at anode: H^g) + 2OH“ ---- > 2H2O(Z) + 2e~,
differences in their internal energy. Para H^ has a
lower internal energy than the ortho H?. E°=0.83 V
Hydrogen gas is an equilibrium mixture of ortho and at cathode HjOU) + - O2(g) + 2e~ 2OH",
fora hydrogen. The ratio of ortho to para hydrogen varies 2
»ith temperature as shown in Fig. 10.3 E° = 0.40 V
100 HzCXZ) E° = 1.23 V
SO
80
70 75.0% Thus, E° is 1.23 V irrespective of the electrolyte
60
66.67% because we are using the same reactants (H2 and O2) at the
o electrodes. Fuel-cells have greater efficiency (about 75%)
150
^40
than other rechargeable secondary cells (about 40%)
J* Combustion of hydrogen is a highly exothermic
30
reaction and produces no pollutants :
20
10 2H>(g) + O2(g) ---- > 2^0(0, AH = - 572 kJ
- Deuterium D2 H Tritium T2
0 ------ 1------- 1------ L- -I------ +----- Liquid hydrogen is, therefore, used as a rocket fuel.
100 200 300 0 100 200 300 0 100 200 300 It has been recognised during the past two decades
Temperature (K) that the world reserves of fossil fuels like coal, oil and gas
Fig. 10.3 Ortho-para equilibria for Hg, Dg and T2. are finite, so they cannot last for ever. Similarly, the
nuclear and hydroelectric power also cannot meet all our
Evidently, this ratio increases with the rise in energy requirements. Moreover, these sources pose a
temperature upto a temperature of about 300 K(27°C) and danger of environmental degradation. This has prompted
remains constant thereafter. The percentage of ortho an active search for alternative sources of energy. In
hydrogen at 300 K and above is 75%. This means that it is
addition to solar power, hydrogen is being considered a
not possible to get 100% ortho hydrogen at any
temperature. The equilibrium mixture of para and ortho potential fuel for the future. Hydrogen-run vehicles are
hydrogen changes to almost 100% para hydrogen when being tried. Hydrogen as a fuel has many advantages over
cooled nearly to absolute zero. Para hydrogen is stable for the conventional fossil fuels and electric power.
*eeks in the absence of catalysts like activated charcoal,
Fe, Ni, Pt, O2,NO2, etc., which catalyse the conversion of Hydrides
para to ortho hydrogen. Similarly, deuterium and
tritium also exhibit spin isomerism and exist in Binary compounds of the elements (metal or
ortho and para forms. However, the ratio of ortho non-metal) with hydrogen are called hydrides. The type of
and para forms in deuterium and tritium is hydride which an element forms depends on its
different from that in hydrogen. The variation of electronegativity and hence the type of bond formed.
ortho Ipara ratio at different temperatures is also While there is no sharp division between ionic, covalent
different. From Fig. 10.3 it is evident that tritium and metallic bonding, they are classified as:
resembles hydrogen more closely than deuterium in this (a) ionic or salt like hydride
respect. (b) covalent or molecular hydride
(c) metallic or interstitial hydride
DEEP Focus They have been summarised in Fig. 10.4.
• In spacecrafts, H2 gas is used in fuel cells for generating i 18
electrical energy and for providing clean drinking water to H 2 13 14 15 16 17
the astronauts. In fuel cell, electrical energy is generated _U_ * — Transition metals----------------- > B C N O F
by the reaction ofH2 and O2 without evolution of any heat: Na
Mg |
3 4 5 6 7 8 9 10 11 12 aT Si P S cT
called “cold combustion”, A H2—O2 fuel cell may have
an alkaline or acidic electrolyte.
K
Rb t 'J Ga Ge As Se Br
In Sn Sb Te I
With acidic electrolyte, the half-cell reactions are: Cs E3 La Ti Pb Bi Po At
Fr E3 Ac
it anode: H^g) ---- > 2H+ + 2e“, E° = 0V
at cathode: 2H+ 4 O2(g) ---- > 1^0(0, F | Mainly ionic with fixed formula Often of uncertain formula;
= 1.23 V
2 absorbed into metal lattice
; ~| Covalent, molecular structures (Intermediate)

•> HjCXZ), E° = 1.23 V Fig. 10.4 Hydrides of the Elements


JU
438 | Essential Inorganic Chemistry

Ionic Hydrides Mg3N2 +6H2O


600-1000 atm >
------------------- Mg(OH)2 +2NH.
• They are formed when molecular hydrogen combines
directly with any alkali metal or with the alkaline CaC2 +2H2O -----> Ca(OH)2 +QH*
earth metals Ca, Sr or Ba (electropositive metal). Zn+HCI
AsC13+6H ------------ > 4AsH3 +3HC1
2Li(s)+ H2(g) 2LiH(s) ether
4PC13 + 3LiAlH4 -------> 4PH,o +3L1C1 +3A1C1:
973 K
Ca(s)+ H^g) ------- > CaH^s)
DEEP Focus
All ionic hydrides are solids that have the high These hydrides have a molecular lattice made up cf
melting point characteristic of ionic compounds and individual saturated covalent molecules, with only weak van
conduct electricity in fused state liberating H2 at the der Waals’ forces and in some cases hydrogen bonds holding the
molecules together. This accounts for their softness, lo*
anode. Anion of these hydrides is H“ which is a very strong
melting and boiling points, their volatility and lack cf
Bronsted base (H+ ion acceptor) and accepts proton from conductivity. Some covalent hydrides are unstable in the
a proton donor such as water: presence of air, e.g., stannane, SnH4.
H~(aq) +H2O(Z) -----> OH’faq) + H2(g)
Base! Acid2 Base2 Acid! Some covalent hydrides of Groups 2 and 13 are
electron deficient. These have structures between ionic
Due to their high reactivity with water, ionic and covalent hydrides. These are either dimeric, e.g,
hydrides are used to remove traces of water from boron hydride (r^Hg). or polymeric, eg., beryllium hydride
organic solvents. (BeHg)^, magnesium hydride (MgHg)^ and aluminium
They are powerful reducing agent specially at higher hydride (AlH3)n.
temperature, e.g.,
2C0+NaH -----4> HCOONa+C Metallic Hydrides
SiCl4 + 4NaH ------»> SiH4 + 4NaCl On heating, hydrogen reacts with many transition
metals, lanthanides and actinides to form metallic
PbSO4 + 2CaH2 -----4> PbS + 2Ca(OH)2
PbS+2Ca(OH).
hydrides. Most of these have metallic appearance and are
Li+H“ and Na+H“ are used in making valuable less dense than the parent metal. They all conduct heat
reducing agents like lithium aluminium hydride (LiAlH4) and electricity though not as well as the parent metal (of
and sodium borohydride(NaBH4). These complex hydrides ionic hydrides). Unlike the saline hydrides, they are
are frequently used in the reduction of aldehydes, ketones, almost always non-stoichiometric, being deficient in
acids and their derivatives of give alcohols.
hydrogen. For example, TiH15_18, VH056, CH17,
LiAlH4/NaBH4
R—COOH ----------- > R— CH>OH NiHo.6-o.7» Pd^o.6-o.8» TaH2 76, LaH2 87, YbHgss, etc,
organic acid primary alcohol approach the stoichiometric formulae given in following
LiAlH4/NaBH4 Table. Most of these hydrides are stable to water up to 375
/C=O ------------------- > /CH—OH K but are quantitatively decomposed by acids and show
ketone/aldehyde secondary alcohol reducing properties.

Covalent Hydride Stoichiometric Formulae of Some Metallic Hydrides


Covalent hydrides are formed by elements of MH MH(M = V, Cr, Ni, Cu, Ta, Pd)
comparatively higher electronegativity, generally p-block (M = Ti, Zr, Hf, V,Nb,Cr, Zn, Y; La-Lu except
elements and Be and Mg. The bonds in this class of Pm; Th, Np, Pu, Am, Cm)
hydrides are mostly covalent in character but in some MH3 (M = Y, La, LA-Lu except Pm and Eu; Pa, U, Np,
cases, for example, in HF, the bond may be partially ionic. Pu, Am)
The covalent hydrides can be prepared either by direct
reaction of non-metals with hydrogen under suitable Formerly, these hydrides were termed as interstitial
conditions or by the reaction of HgO or acids on nitrides, compounds in which hydrogen was thought to be
carbides, borides, silicides, stannides of alkali and accommodated in the interstices in the metal lattice
alkaline earth metals or by the reduction of halides. These producing distortion but no change in its type. But recent
are illustrated by the following reactions : studies have shown that except for the hydrides of nickel,
palladium, cerium and actinium, other hydrides of this
catalyst, 750 K „VTTT
N3+3H2 ------ ------------ > 2NH3 class have lattice of a type different from that of the parent
metal.
Chapter 10: Hydrogen and Its Compounds | 439

DEEP Focus period as nuclear charge increases and atomic radius falls,
For example, the hexagonal close packed lattice of but decreases down a group as both atomic size and the
some lanthanides is transformed to a face-centred cubic number of inner screening electrons increase.
lattice in their dihydrides. These interstitial hydrides
are poorer conductors of electricity, exhibit less Although all of the hydrides shown in Table 10.4 are
paramagnetism and are more brittle than the parent described as covalent, the bonds are polar to some extent.
metal. These characteristics suggest that hydrogen is
The degree of this polarity depends upon the
present in the metal lattice as hydrogen atoms rather
than as hydrogen molecules. The single electron of electronegativity difference between the two
hydrogen is paired with an electron of the metal, elements concerned. In the case of the Group 14
thereby reducing the extent of metallic bonding. hydrides, however, this bond polarity does not result in a
Breaking of the H—H bond is in agreement with the fact
that these metals catalyse reactions of hydrogen. polar molecule. In the tetrahedral XH4 molecule, there is
no overall dipole moment as the centre of the 8+ charges
The physical and chemical properties of these coincides with the centre of the 8" charge, for example in
I compounds change from ionic to covalent across a
methane.
given period (moving left to right).
As they have no overall dipole, the only forces of
LiH BeHj ^6 ch4 attraction between XH4 molecules are relatively
F ~T
ionic high m.p. covalent polymeric
?
non polar weak van der Waals’ forces due to temporary
(950 K) in solid state dipoles. Since, these forces result from mutual repulsion
of the electron clouds of neighbouring molecules, they
NH,u HjO HF Xacidic in nature increase as the size of the molecule increases, thus,
accounting for the steady increase in the boiling point of
polar with H atom as positive end
of the polar bond the hydrides on passing down group 14.

• As we move down any group, the compounds change In the case of the hydrides of Group 15, 16 and 17,
from covalent to ionic. bond polarity does result in a permanent molecular dipole.
BeHa
MgHa <D Q

CaHa co 8+h
>
o h —4 Ox
SrHa U H 8+ H 8+
BaHa 8+ 8+

The AHJ values (Table 10.4) show that the energetic


stability of the covalent hydrides tends to increase on
passing from left to right across a period, but to decrease f XH
on passing down any group. The main reason for this is the 8+ H 6+
8+
change in the strength of the X—H bond as shown by the
Permanent dipoles in period 2 hydrides
bond enthalpy figures in Table 10.4. The attraction of an
atom for a shared pair of electrons increases across a
440 | Essential Inorganic Chemistry

Table 10.4 Some Common Covalent Hydrides of Periods 2-5. Electronegativity Differences in Brackets Indicate that
Hydrogen is more Electronegative Than the p-Block Element, that is, the Polarity of the Bond is X —H
Group 14 Group 15 Group 16 Group 17
Period 2 HF-
CH4 NH, HjO
AH /kJ mor1 -74 -46 -286 -271
Bond enthalpy/kJ mol-1 +436 +431 +494 +574
Boiling point/K 112 238 373 293
Electronegativity difference (Pauling) 0.35 0.84 1.24 1.78
Period 3 SiH4 PHa HjS HCI
AH/kJ mol-1 34 -10 +20 -92
Bond enthalpy/kJ mol-1 +323 +323 +364 +432
Boiling point/K 161 185 213 188
Electronegativity difference (Pauling) (0.3) (0.01) 0.38 0.96
Period 4 GeH4 ASH3 HjSe HBr
AH/kJ mol'1 +91 +66 +73 -36
Bond enthalpy/kJ mol’1 289 +363
Boiling point/K 185 211 232 206
Electronegativity difference (Pauling) (0.19) (0.02) 0.35 0.76
Period 5 SnH4 Sbhtj HJe HI
AH/kJ mol-1 + 163 + 145 + 100 +26
Bond enthalpy/kJ mol-1 253 +295
Boiling point/K 221 256 269 238
Electronegativity difference (Pauling) (0.24) (0.2) (0.1) 0.46

In these cases, as well as attraction due to temporary 400+


dipoles, there is an extra attraction due to these
permanent molecular dipoles. The positively charged 350+
hydrogen atom will attract a lone pair of electrons on the
p-block atom of a neighbouring molecule. 300t
—■— Period 2
DEEP Focus 250“ —A— Period 3
As a result of this extra permanent dipole attraction, K Period 4
in any period the boiling points of the Group 15,16 and 17
200--
hydrides are all higher than that of the non-polar Group -0- Periods
14 hydride. This effect is much greater in period 2 than in the
lower periods (Fig. 10.5). Since, the attraction resulting from the 150+
presence of N—H, O—H or F—H bonds in much stronger than
other permanent dipoles, in these cases only the effect is referred
to as hydrogen bonding. Typically such ‘hydrogen bonds’ 5-10% 100+
of the strength of a covalent and in the case of water this is
enough to make it liquid at room temperature. 50+

0 4--------- 1--------- 1-------- 1


Group 14 Group 15 Group 16 Group 17
Fig. 10.5 Boiling point of covalent hydrides plotted by period
H
8+
At first glance it might seem surprising that hydrogen
r^F5" fluoride has a lower boiling point than water, even though
& it contains the more polar H—F bond. Whilst the
8+
individual hydrogen bonds in HF are indeed stronger than

kH 8+ those in water, the later can form more hydrogen bonds.


Each water molecule contains two H5+ atoms and
each O6" has two lone pairs and so may form up to

Hydrogen bonding in hydrogen fluoride


I four hydrogen bonds with its neighbours.
Chapter 10;Hydrogen and Its Compounds | 441

Atomic Hydrogen
Hydrogen gas dissociates into atoms when it (H2) is
subjected to an electric discharge under low pressure,
electric discharge
H2 2H
Hg+
• Atomic hydrogen is very reactive and can be used as a
reducing agent as well as oxidising agent
• As a reducing agent
CuO + 2H -----> Cu + H^O
y/VW'r“'V 05"
A“
/ • \ 0+ Is)
NaN03+H -----> Na+HN03
AgCl+H -----> Ag+HCl
8+
(
AuC13+3H -----> Au + 3HC1
^08- • As an oxidising agent (when hydrides are formed)
Hz \H 6+ Li+H -----> LiH
8+ n8+
Ca+2H -----» CaHa
Water can hydrogen bond to several other water molecules. Mg+2H -----> MgHa
On average both HF and NH3 can form two A1+3H -----> A1H3
hydrogen bonds with neighbouring molecules and, • Reaction with non-metals
as expected, hydrogen bonding between the less S+2H -----> HjS
polar NH3 molecules results in a lower boiling point P + 3H -----> PH3
for ammonia than hydrogen fluoride.

Target Practice 1
(iv) Para hydrogen is more stable at lower temperatures.
!• Why does hydrogen resemble the alkali metals?
(v) At 100 K, percentage of ortho hydrogen in the mixture
is 70%.
4. Complete the following chemical reactions which take
2. Chemical properties of isotopes of hydrogen are similar.
place during the formation of hydrogen
Explain.
(i) C + H2O ---- > +
(ii) CH4+H„0 ---- > +
3. Select True/False (iii) CO + H2O ---- > +
(i) Ortho and para hydrogen forms are different due to (iv) CaH2+H2O ---- > +
difference in their nuclear spins. Hydroformylation reaction is the industrial preparation of
(ii) Ortho and para hydrogen forms are different due to 5.
methanol. Write this reaction > CH3OH.
difference in their electron spins.
(iii) Physical properties of ortho and para hydrogen are
similar.

Answers
1. Hydrogen resembles alkali metals because like alkali metals it 3. (i) T (ii) F (iii) F (iv) T (v) F
also has one electron in s-orbital 4. (i) > CO + H2
H;ls2 (ii) > CO + 3H2
Li:ls22sl (iii) > CO2+H2
Na: [Nel 3s1 (iv) > Ca(OH)2 + H,
2. Most of the chemical properties are governed by the electronic
5. CO + 2H2 CH30H
configuration of the element. As the isotopes of hydrogen have
the same electronic configuration, their chemical properties
are closely similar.
442 | Essential Inorganic Chemistry

Practice Exercise 1
1. What is the equation for the nuclear reaction that takes if H2 gas is passed into FeCl3 solution, there is no change.
place when tritium undergoes beta decay? What is the Explain.
product in the reaction ? 4. For H2O, Xw = lxl0-14; D2O, Kw = 3.0x 1O'1S
2. Before natural gas can be used to make hydrogen, it has to Compare [H+] and [D+] in H,,0 and D2O. If both are
be purified. Especially sulphur compounds must not be
electrolysed, which is liberated faster, H2 or D2?
allowed through; why ?
5. Ionic hydrides are the ready-made source of hydrogen.
3. If Zn pieces are added into aqueous FeCl3 solution, deep Explain.
yellow colour ofFeCl3 fades and changes to light green. But 6. What is the nature of hydride in the following reaction,
LiH + H„O
z
-----> LiOH + H„z

Answers
1. i*H--- > ’He+.fe 4. H2O H+ + OH-, Kw = 1x10
2. Sulphur is very effective poison of catalysts, so it has to be
[H+] = Vl x 10"14 = 1 x 10-’ M
removed before it reaches the nickel.
3. FeCl3 solution is acidic in nature due to hydrolysis : D2O \ k D+ + OD , Kw = 3.0x10
Fe3+ + 3H2O Fe(OH)3 + 3H* [D*] = V3.0x 10-15 M
On adding Zn, nascent hydrogen is formed in situ, that being Thus, [H+] > [D*]
very reactive causes reduction of Fe3+ (deep yellow) to Fe2 +
H2 gas is liberated much faster than D2 at cathode.
(light green). Molecular H2 is not effective in this case :
5. CaH2 + 2H2O---- > Ca(OH)2 + 2H2
Zn + 2H+ Zn2*+2H
6. LiH (H-) is H + acceptor from H2O and changes to H2. Hence,
FeCl3 + H---- > FeCl2 + HC1 LiH is a Lowry-Bronsted base

Hydrogen Peroxide This solution is concentrated to 98 per cent by low


• It is the hydride of oxygen. pressure fractionation.
• BaO2 is decomposed byH2SO4;Na2O2 andKO2 by H>0 • Industrially it is obtained by a Cyclic Process.
forming HA: 2-ethyl anthraquionol (A) is oxidised by air to 2-ethyl
anthraquinone (B) and HA: (B) is reduced back to (A)
BaO2 + H2SO4 ■» BaSO4 + HA with H2 at a moderate temperature using Pt, Pd or
barium peroxide
Raney Ni as a catalyst. The cycle is repeated. The
Na2O2 + 21^0---- > 2NaOH + HA reaction is carried out in a mixture of organic solvents
sodium peroxide which must:
- dissolve both (A) and (B)
2KO2 + 21^0---- > 2K0H + HA + O2 OH 0
potassium superoxide C2H5 p2 r C2Hs
• Electrolysis of HjSO,, or NH4HSO4 at high current H2/Pd
density using platinum electrodes forms
peroxosulphuric acid which on subsequent hydrolysis OH 0
gives HA : (A) (B)
current + H2O2
H2SO4 H+ + HSO;
- resist oxidation
At anode: 2HSO4~ ---- > HjSA + 2e~ - be immiscible with water.
H^A + H>0 H2SO5 +H2SO4 Solvent is generally a mixture of ester and
hydrocarbon or octanol and methyl naphthalene.
HjSOg + HjO H2SO4 + HA
HA is extracted with water as a 1% solution. This is
At cathode : 2H+ + 2e~ ---- > Hj concentrated under reduced pressure. HA solutions are
stored in plastic or wax coated glass vessels, often with
2NH4HSO4 ---- >.H2 +(NH4)2S2O8 negative catalysts such as urea or sodium stannate
(NH4)2S2O8 + 2H>O---- > 2NH4HSO4 +HA added as stabilisers.
Chapter 10: Hydrogen and Its Compounds | 443

• An alternative process is based on the partial • H2O2 as reducing agent: (when itself oxidised to O2)
oxidation of 2-propanol; HA — 2H* + O2 + 2e"
OH ♦ 2MnO4‘ + 5HA + 6H+ -» 2Mn2+ + 5O2 + 8H2O
MejCHOH + O2 ■> Me2C
* IO4 + P^C^ - -> IO3 + O2 + H2O
OOH
* 2Ce4+ +HA - -> 2Ce3+ + O2 + 2H+
/OH
Me2C\ Mf^CO + H^ * Ag2O + H2O2 - -> 2Ag + HjO + O2
^OOH
* PbO2 + HA - -> PbO + H2O + O2
• Pure HjOa is> a colourless liquid which resembles * O3 + pp2^2 ■ —> 2O2 + H2O
water quite closely. It is more hydrogen bonded than is * Ai^O + HA -----> 2Au + H2O + O2
water so has a higher boiling point and is more dense
than H,O. * MnO2 + H2O2 + 2H+ ---- > Mni2+ + O2 + 2H2O

Property H2O2 H2O * [Fem(CN)6]3- +HA +2OH" -—> [FeI,(CN)6J‘-


152°C (decomposes) 100°C + 2H2O + O2
boiling point
melting point -0.4°C o°c * 2MnO4 + 3H2O2 $ 2MnO2 X + 3O2 + 2H2O
density 1.4 g cm-3 1.0 g cm-3 + 2OH-

• At 68 mmHg pressure, it boils at 84°C.


DEEP Focus
• H2O2 is unstable and the rate at which it Traces studies using 18 O show that in reactions all the
disproportionates, depends on the temperature and oxygen that is formed comes from the Ho02, none from the
concentration. Many impurities catalyse the solvent H2O, so the 0—0 bond is not broken.
decomposition, which may become very violent,
• H2O2 as an oxidising agent: (when itself reduced to
especially with concentrated solutions.
HA
2HA -----> 2H2O + O2 H2O + O
reduction a a HA
HA + 2H+ + 2e~ 2H2O
oxidation
and at[H+]= 1 M, E°=+1.77V
Catalysts include metal ions Fe2+, Fe3+, Cu2+, Ni2+,
metal surfaces such as Pt or Ag, MnO2, charcoal or hence, it is a strong oxidising agent.
alkali. They are best stored in polythene bottles in a • Its bleaching action is also due to its oxidising
refrigerator. Traces of complexing agents (e.g., property.
8-hydroquinoline)or adsorbing material (e.g., sodium
Oil paintings contain Pb2+ salts which are blackened
stannate) are often added as stabilisers.
by atmospheric H^S (due to PbS). PbS (black), on
• Mixtures of hydrogen peroxides and organic or other
reaction with H2O2, changes to PbSO4 (white), hence
easily oxidised materials are dangerously explosive.
HA is used to clean oil paintings :
Mixture of hydrogen peroxide and hydrazine
PbS + 4H2O2 PbSO4 + 4H2O
has been used for rocket propulsion.
• The ionisation constant of hydrogen peroxide in
* ‘ + 2H+ + HA -----> 2Fe 3+ + 21^0
2Fe>2+
aqueous solution at 20°C for the acid equilibrium is ♦ so32- + ha--- > SO/'+^O
SO42"
HA H+ + HO2 * no2 + ha NO3 + P^O
[H-XHO£j= 2x 10.12 * Ij +21^0
Ka = 2F + HA + 2H+
[H&]
This reaction can be used to estimate HA
In alkaline medium, therefore, it is present as the HO2 iodometrically. formed is titrated against Na-jSA
ion.
solution:
+ 2S2O2"-----> 2F + S4O62-
DEEP Focus
H2O2 — = 2S2O3
• Many reactions ofH2O2 are radical reactions. The oxidation
of Fe(II) at low hydrogen ion concentrations proceeds as: Io — 21= H2O2 = 2S2O3
Fe2+ + H2O2 ---- > FeOH2+ + OH mol. wt.'l
Eq. wt. (H2O2) =
Fe2+ + OH---- > FeOH2+ . 2 J
and a mixture of H2O2 and Fe(II) is thus a source of Eq. wt. (S2O2“) = mol. wt.
hydroxyl radicals for organic reactions.
444 | Essential Inorganic Chemistry

* AsO3 “ + H2O2 -----> AsO43- + H2O This amount is in 1 ml H2O2 (by definition)
* 2[Fen(CN)6]*- + H2O2 + 2H+ — -» 2[Feni(CN)6]3-
Thus, 68 x
1 mL H2O2 = g
+2H2O 22400
* Mn2+ (colourless when dilute) changes to MnO2 density = x g/mL
(black) and Cr■’3+ (green) changes to CrO42- (yellow) in 22400
alkaline H2O2. cone, in g/L = x. x 1000
Mn2+ +H2O2 + 2OH“ * MnO2 J, + 2H2O2 22400
black 680 x
g/L cone. = g/L
2Cr3+ + 3H2O2 + 10OH" -> 2CrO2" + 8H,0 224
green yellow
m 68 X
% cone. =---
224
DEEP Focus 680 x 1
• HCHO (formaldehyde) is oxidised to HCOOH (formic acid) Normality (N)=conc *n - ----- x—
in presence of pyrogallol and is itself reduced to hydrogen. equiv. wt. 224 17
2HCHO + H,O2 ---- > 2HCOOH + H2 ,, X
N=—
The reaction in the dark is accompanied by emission of light 5.6
(chemiluminescence). cone, in g/L _ 680 x 1
Molarity (M) = x—
• As an acid : mol. wt. 224 34

H2O2 decomposes CO2 from carbonates and thus


behaves as an acid. It turns blue litmus red but 11.2
aqueous solutions has no effect. CH, CH2OH
Na2CO3 + H2O2 -----> Na2O2 + H2O + CO2 I + ^2^2
* CI^OH
CH,
A 30% solution of H2O2 is sold in the market under the glycol
trade name perhydrol. This is also known as “100
volume” peroxide. This reaction confirms the presence of two —OH
groups in H,O,
• Concentration Determination:
Concentration of H2O2 is expressed in terms of
“volume of O2” H-^-o ......
,.45*
“10 volume” H2O2 means that 1 mL of H2O2 at NTP
o <*■■■■
gives 10 mL O2 gas. These values can be changed
H
into other concentration terms like density, gL-1,
normality and molarity.
111*30'
2H>O-----4> 2H,0 + O2
68 g • 1 mol = 22400 mL at NTP
? x mL • HgO, finds use
22400 mL O2 is from= 68 g H,O2 at NTP ~ as a bleaching agent
- as an antiseptic in surgery.
x mL O2 gas is from = ■ g H2O2 NTP
~ as a fuel for rockets (85-90% solution).
Chapter 10 : Hydrogen and Its Compounds I 445

OH

OH

at anode
j"·····-·····-··............
i
<it

intermediate l
. .................. -·····••-'.
l is HS04- :
\.

2H2Q .....:--=..::__.=_.::a_.--____,.__.....____
PbS04

Fe3•

soi-

Ag
Asa
[Fe(CN)6

C 4 �
� CH20H
u I
,:t CH20H

Target Practice 2
l. Write ionic reactions showing H2O2 as 5. H2O2 behaves as an acid. Write reaction
(a) Reducing agent : ................................................
{b) 0xi"di smg
· agent : ................................................
2. H,0 2 is used to clean oil-paintings. Write reactions making 6. Structure ofH:P2 is represented as
paintings black and then cleaning with H202• Angle La= ...........................
················································································ Angle LP= ...........................
.......................................................................................... Bond-angle length x = .......... ..
3. Specify the meaning of "10 volume" Hp2 •

..................................... ..................... ..................................................


....... ............... .. ... ii •••••••••• •••••••••••••••••••••••••••••••••••••• •••••• ••

"· Molarity of H 2O2 solution is 0.01 mol L-1• Thus, its


-Volume" concentration is .......................... .
446 | Essential Inorganic Chemistry

Answers
1. (a) Reducing agents : 2H2O2 ---- > 2H2O + O2
H2O2 ---- > 2H++O2+2e“ 1 mL 10 mL at NTP

(b) Oxidising agents: x volume


4. Molarity =
H2O2+2H+ + 2e“ ---- > 2H2O 11.2
2. Oil paints contain lead acetate (white) which is blackened by Volume concentration = 11.2 x 0.01
atmospheric H2S due to formation of PbS (black). = “0.112” volume
(CH3COO)2Pb + H2S ---- > PbS+2CH3COOH 5. Na2CO3+H2O2 Na2O2 + H2O + CO2
3 2 2 black 3 base acid
H2O2 oxidises PbS (black) toPbSO4 (white) 6. Za=94°48'
Thus, oil-paintings are cleaned by H2O2 Z0 = lll°30'
x = 1.48 A
PbS + 4H2O2 ---- > PbSO4 + 4H2O
3. It means that 1 mL H2O2 on decomposition gives 10 mL of O2
gas at NTP

Practice Exercise 2
1. H2O2 is used to counteract Cl2. How does it act in this 4. Represent the following by chemical equations :
process? (a) Alkaline Cr3+ salts are oxidised by H2O2.
2. Complete: (b) H2O2 is added to a mixture of cone. H2SO4 andK2Cr2Or
A A 5. What do you conclude about H2O2 from the reaction
(a) Na2'0
a9 9 + CO
“'2 _ 2 (b) Na2O2+Fe(OH)2
current,
(c) nh4hso4 ------ —— (d)KO2+CO2 ---- > H,O, +H,0 H„O+ +HO2-?

(e) Ba(OH)2 + H2O2 ---- > 6. When a few drops of blood are added to aqueous H20,, it
(H2O2) decomposes rapidly. Explain.
3. When H2O2 reacts with O3, is there any change in magnetic
behaviour ofO,?

Answers
1. H2O2 acts as a reducting agent to counteract Cl.2 4. (a) 2CX3* + 3H2O2 + 10 OH” ---- > 2CrO3’ + 8H2O
H2O2 + Cl2 --- > 2HC1 + O2 (b) Cr2O?- + 2H* +4H2O 2CrO5 + 5H2O
2. (a) 2Na2O2+2CO2 ---- > Na2CO3+O2
(b) Na,FeO,
Na2O2 + Fe(OH)2 ---- > an d 2 r CW4 +H
t n2 0 0
2NH4HSO4 current-■> (NH4)2S2O8 + h2 I I
(0 0 0
anode cathode
(NH4)2S2O8 +2H2O---- » 2NH4HSO4 +H2O2 (CrO5 gives blue colour in ethereal layer; it is diperoxide)
(d) 4KO2 + 2CO2 ---- > 2K2CO3 + 3O2 - 5. This indicates that H2O2 behaves as acid (though weak)
(e) Ba(OH)2 + H2O2 ---- > BaO2 + 2H2O H2O2 +H2O H3O+ +ho2
3. Peroxide (O2“) is diamagnetic and O3 is also diamagnetic. acid base acid base
When both react O2 and H2O are formed: 6. The decomposition of H2O2 is catalysed by catalase, an
O3 + H2O2 ---- > 2O2 + H2O enzyme present in blood. There is formation of bubbles due to
O2 is paramagnetic. Thus, O3 (diamagnetic) is oxidised to O2 O2.
(paramagnetic) but H2O2 (diamagnetic) has no effect in its
property as H2O is also diamagnetic.
*
J

Total Practice Set


(Read, Plan & Solvel)
kJ

Normality of is related to “x volume” strength by


Problem 1. Compare bond lengths, bond order and relation
magnetic behaviour of
N=—
^2 »O2, O2 5.6
x = N1 x 5.6
Solution Based on molecular orbital theory :
= 0.24 x 5.6
bond order of 0^ “ = 1
= “1.344 volume1
O2 =1.5
O2 =2.0 Problem 4. (a) Give structures of O2 , and O2.
Greater the bond order, smaller the bond length, thus (b) What is their behaviour towards H2O ?
bond length of O2 < O2 < O|“ (c) Give applications of and KO2.
r .. 12- |2-
O2 has two impaired electrons hence, paramagnetic, Solution (a) o o;o o;o;
0^ has one unpaired electron hence, paramagnetic,
0^ ~ is without unpaired electron hence diamagnetic. (b) O2- + 11,0 2OH"
oxide

Problem 2. When blood is added to a solution of , 20?' + 211,0 O2 +4OH"


peroxide
the solution bubbles furiously. This reaction is also
observed when H2O2 is poured on a skin wound to cleanse 4O2 + 2H2O > 3O2 +4OH"
it. What is the overall reaction that occurs? superoxide
Solution An enzyme or biological catalyst (called All the three ions are unstable towards HoO and give
catalase), present in tissue and in blood, decomposes the alkaline solution. The decomposition of O^“ and O2
H.2O2 and produces bubbles of O2. The natural function of involve oxidation and reduction. Both of these ions,
this enzyme is to prevent the accumulation of in the especially O2, are good oxidising agents.
body, since such a build-up of H2O2 could damage tissue. (c) HgChj is used as :
- bleaching agent
Problem 3. To 25 mL H2O2, excess of acidified solution
- a substitute for Cl2 in water and sewage treatment
of KI was added. The iodine liberated required 20 mL of - an antiseptic in curing wounds.
0.3NNa2S2O3 solution. Calculate volume strength ofH2O2
solution. KO2 is used in life supports in spacecrafts, submarines
and emergency breathing apparatus since it absorbs
Solution ll,O2 + 21’ + 2H ■> 2H2O +12 CO2 and releases O2
I2 + 2S2O^ S4O26"+2r 4KO2 + 2CO2 ■> 2K.,CO3 + 3O2
^=^2
Problem 5. Starting with deuterium oxide (D2O),
H2O2 Na2S2O3
describe how you would prepare?
Nx x 25 = 0.3x 20 (a) NaOD, (b) DC1, (c)ND3, (d)C^D2,
Ni = 0.24 N (e) CD4, (f) D^O,
448 | Essential Inorganic Chemistry

Solution (a) 2Na + 2D2O 2NaOD + D2 Solution


(b) D20 + Cl2 ---- > DC1 + D0C1 H3C h3c OH
Step I !H0H + 02 ---- >
lElcctr°lysis> D, 2DC1 h3c h33c
’ xo-oh
d2 2-propanol intermediate
at cathode H3c
H3c oh
- n2 Step II C=O + H2O2
(0 d2o —>> dD<2----- > nd3 h3c 0—OH kH3C
(d) CaC2 + 2D2O---- > Ca(0D)2 h3C\ H3C'
Net CHOH + 02 C—0+ HjOj
(e) A14C3 + 12D2O -> 4A1(OD)3 + 3CD4 h3c h3c
(f) S03 + D20 ■> D2SO4
Problem 8. Oxygen forms three types of oxides: O2',
Problem 6. A thermochemical cycle for hydrogen 0|‘ and 02. Classify them as base/acid when reacted with
production involves at least one element (say Cu) that can 1^0. Also classify the reaction
exist in two different oxidation states; in a typical cycle
A (ox) represents oxidised form and A (red) represents Solution :o:2~ :o:b:2" :b:o:
reduced form in a photolytic production of H2 as given oxide peroxide superoxide
below: The ions are all strong Bronsted-Lowry bases and
react with water as follows :
H20 + hv + 2A (ox) * 2A(red) + 02 + 2H+ +2e~ Oxide ion C?~(aq) + H20 (Z)---- > 20H’(ag)
T T T ? T
-2 +1, J2 -2,+l
2A(red)+ 2H+ +2e~ ---- > 2A(ox) + H2
Starting with FeCl2, H^O and Cl2, explain formation of There is no change in oxidation number of 0 or H,
H2. You may consider oxidised species as FeCl3. hence, it is simply hydrolysis of O2-.
Solution 6FeCl2 + 8H2O ^4 2Fe3O4 + 12HC1 + 2^ Peroxide ion 0^" (oq)+ 2H2O(Z) ■> O2(g) + 40H"(a9)
T T TT
2 Fe3O4 + 3C12 + 12HC1 6FeCl3 + GHjO + 02 -1 0 "2+1
6FeCl3 6FeCl2 + 3CLl2 There is change in oxidation number of 0, thus it is a
redox reaction (disproportionation of 0^')
Net 21^0---- > 2Hg + 02
Various devices may be employed to get round the Superoxide ion
difficulty created by the simultaneous production of the
two gases (Kj and 02) 4O2(a<7)+ 2^0(1) 3O2 (g) + 4OH"(a<7)
T T
-0.5 0 -2 +1
Problem 7. Hydrogen peroxide can also be obtained
by partial oxidation of 2-propanol. Outline the scheme of This is also a redox reaction (disproportionation of Op
this oxidation.
'W

Master Exercises
Exercise 1
(Stage 1: Learning)
Short Answer Type Questions Analytical Questions
1. Which of the following compounds would be expected 1. A 1 g sample of H2O2 solution containing x per cent
to form intermolecular hydrogen bonds in the liquid AO2 by weight required x mL of KMnO4 for complete
state? oxidation in acidic medium. Calculate normality of
(a) CH3OCH3 (b) CH4 KMnO4 solution.
(c) HF (d) CH3CO2H 2. A 5.0 mL of H2O2 solution liberates 0.508 g of from
(e) Br (f) CH3OH an acidified KI solution. Calculate the strength of
H2O2 solution in terms of volume strength at STP.
2. What is meant by metallisation of hydrogen?
3. You have been given two liquids (A) and (B) made of
3. Name the binary compounds of hydrogen. same atoms. Both are diamagnetic. Liquid (A)
4. Hydrides are reducing agent. Explain
+ KI/H+ + starch---- > blue colour, liquid (B) is
neutral to litmus and also does not give above test
5. (a) Hydrides of boron are called............. with starch. Identify (A) and (B).
(b) Hydrides of silicon are called............. 4. Na[Cr(OH)4 ] green solution changes to yellow colour
(c) Hydrides of carbon are called............. solution when boiled with H2O2. Yellow solution
(d) Hydrides of that can reduce aldehydes to alcohol changes to orange on acidification. Write reactions of
these chemical changes.
6.2C0+NaH A —B 5. HjQ, ionises as:
HC1 HA H+ + HO2
If pH of H2O2 at 20°C is found to be 5.91, calculate
C
ionic product (auto-protolysis constant) of HA-
B and D decolourise KMnO4 /acidic
6. To prepare A from water, would you allow water to
C reduces Tollen’s reagent
What are A to D? react with an oxidising agent or a reducing agent?
Which of the following metals could be used in the
7. Complete the following reactions : reaction?
(a) SiCl4 +Li[AlH4] ---- > (a) Ag (b) Al
(b) AgNO3+SiH4 ---- > (c) Au (d) Ca
(c) CuSO4 +PH3 ---- > 7. Which of the following molecules have similar
(d) SiH4 +C12 ---- > properties and which have quite different properties?
16 17
(e) A14C3 + AO ---- > (a) AO (b) D2O
16 17
8. Name the hydrides used in nuclear reaction (c) HA (d) H^O
technology. 17
(e) D2O2
450 | Essential Inorganic Chemistry

8. Ionic metal hydrides react with water to give 11. Identify A to E in the following
hydrogen gas and an aqueous solution of the metal
hydroxide. H2Q(g)
Ni, 800°C O2
(a) On reaction of equal masses of LiH and Cal^ CH4(g) B HjO
with water, which compound gives more H2?
(b) How many kilograms of CaH2 is needed to fill a B C
Fe/Cu
A
100 L tank with compressed H2 gas at 150 atm H2O(g), 400°C
pressure and at 25°C? (Li= 9,H= l,Ca= 40)
LiH + H2O---- > LiOH+-H2 C
Ca(OH)2 Milky Soluble
2
CaH2 + 2H2O---- > Ca(OH)2 + 2^ D E

9. Which of the following are oxidised by water? Which 12. What conclusion can you draw about the structures of
are reduced by water? Which undergo a A, and B based on following reactions?
disproportionation reaction when treated with water?
(a) Cl2 (b) F2 H2SO5 + H2O---- > H2SO4 + HA
(c) K (d) (A)
10. How many kilograms of water must be processed to H2S2O8 + 2H2O---- > 2H2SO4 + HA
obtain 2.0 L of D2 at 25°C and 0.90 atm? Assume that (B)
deuterium abundance is 0.015 per cent and that
recovery is 80 per cent.

Exercise 2
(Stage 2: High Skill Problem Solving)
Only One Option Correct (b) KjO > H2Te > H2Se > H2S
1. HLj gas is liberated at cathode and anode both by (c) HF > HI > HBr > HC1
electrolysis of the following aqueous solution except (d) All of the above are correct
in 6. Which statements is/are correct?
(a) NaH (b) HCOONa (a) Boiling point of H2O, NH3, HF are maximum in
(c) NaCl (d) LiH their respective group due to intermolecular
2. Statement S: Trimethyl ammonium hydroxide (A) H-bonding
is a weaker base than tetramethyl ammonium (b) Boiling point of CH4 out of CH4, SiH4, GeH4 and
hydroxide (B). SnH4 is least due to lack of H-bonding
Reason R: O of OH group in (A), is hydrogen bonded (c) Formic acid forms dimer by H-bonding
to the MejjNH—group while in (B), hydrogen bonding (d) All the above are correct statements
can't occur.
7. HA can be obtained when following reacts with
(a) S and R both are correct
(b) S is correct but R is incorrect H2SO4 except with
(c) S is incorrect but R is correct (a) PbO2 (b) BaO2
(d) S and R both are incorrect (c) Na2O2 (d) SrO2
3. In the absence of hydrogen bonding, boiling point of 8. The set of elements which could form stable, covalent
water would have been hydrogen bonded hydrides are
(a) 100°C (b) 0°C (a) nitrogen, oxygen and fluorine
(c) -100°C (d) 373°C (b) lithium, sodium and potassium
4. Which is most volatile out of O-, m- and (c) sulphur, selenium and tellurium
p-nitrophenol? (d) chlorine, bromine and iodine
(a) o- (b) m-
(c) p- (d) All are equally volatile 9. The fuel gas obtained by blowing steam over
incandescent coal is known as
5. Which is correct order of boiling point of VA, VIA and (a) coal gas (b) water gas
VILA hydride? (c) producer gas (d) natural gas
(a) NH3 > SbH3 > AsH3 > PH3
Chapter 10: Hydrogen and Its Compounds | 451

10. Steam is passed over red hot carbon and the gaseous 20. Temporary hardness and permanent hardness in
products cooled and passed first through a solution of water can be removed respectively by addition of
alkali, then through ammoniacal cuprous chloride (a) CaO, CaCO3 (b) CaO, Na2CO3
and then through water. What is finally collected? (c) Na2CO3, CaO (d) NaHCO3, CaCl2
(a) CO2
21. Decomposition of HaO2 can be prevented by addition
(b) A mixture of hydrocarbons
(0 CO + Ha of
(d)Ha (a) ferrous sulphate (b) oxalic acid
11. Which among the following is a hydride? (c) ozone (d) glycerol
(a) Rongalite (b) Nitrolim 22. Which is true statement?
(c) Hydrolith (d) Minium (a) The layer of ice on the surface of river in the
winter acts as a thermal insulator between the
12. Which of the following easily reacts with water water below and the air above
producing hydrogen? (b) The fish and other marine organisms are enabled
(a) PH3 (b) B2H6 to survive long periods of freezing weather due to
(c) CH4 (d) HaS the fact that ice is lighter than water
13. How does HaO2 differ from O3 in its chemical action? (c) Both (a) and (b) are correct
(a) In oxidising PbS to PbSO4 (d) None of the above is correct
(b) In liberating L> from KI 23. Hydrides of metals are named like
(c) In decolorising acidified KMnO4 (a) alkane (b) alkene
(d) In oxidising K4 [Fe(CN)6 ] to K3[Fe(CN)6 ] (c) alkyne (d) none is correct
14. Which of the following statements about HaO2 is not 24. In water, or in aqueous solutions of HC1 or HaSO4,
true? proton exists as
(a) HaO2 is used to clean oil paintings (a) H3O+ (b) HtfW
(b) HaO2 acts as oxidising as well as reducing agent (c) H(HaO): (d) all are correct
(c) Two hydroxyl groups in H2O2 lie in the same 25. Hydrogen
plane (a) is placed in IA since it forms monovalent cation
(d) It retains same structure in liquid and solid form H+
15. HaO2 is used as but not as (b) is placed in VII A (halogen family) since it forms
monovalent anion H"
(a) oxidant, reductant (b) bleaching agent
(c) is placed in IV A (carbon family) since both have a
(c) antiseptic (d) catalyst
half filled shell of electrons
16. Which is accepted structure of HaO2 in gas phase?
(d) follows all of the above facts
% 26. Which is true statement about HaO?
H
(a) O->O (a) Hardness can be removed by passing through ion
H
(b) I exchange resin
XH (b) Its presence can be detected by anhydrous CuSO4
(c) Both (a) and (b) (d) None of these (c) It is amphiprotic
17. “10 volume” HaO2 means (d) All the above are correct statement
27. HaO2 is “5.6 volume”, then
(a) 1 mL HaO., gives 10 mL O2 at NTP
(a) it is 1.7% weight by volume
(b) 1 gHjOa gives 10 mL O2 at NTP (b) it is 1 N
(c) 1 mol HaO2 gives 10 mL O2 at NTP (c) both (a) and (b) are true
(d) none of the above is correct (d) none of the above is true
18. Industrially HaO2 is obtained from 28. Bond angles H—0—H and H—O—0— in water and
(a) 2-ethyl anthraquinol by oxidation and then hydrogen peroxide respectively are
reduction in a cyclic process (a) 104.5° in both (b) 94.8° in both
(b) HaSO5 (c) 104.5°, 94.8° (d) 94.8° and 104.5°
(c) HaS2O8 29. HaO2 ---- > HaO + O2
(d) BaO2 This represents
(a) oxidation of HaO2
19. Density of HaO is maximum at (b) reduction of HaO2
(a) 0°C (b) 100°C (c) disproportionation of HaO2
(c) -273°C (d) 4°C (d) acidic nature of HaO2
452 | Essential Inorganic Chemistry

30. Which is a source of nascent hydrogen? 39. H2 can be obtained from


I: Zn + dil. HC1, (a) water gas (CO + Hg) by liquefaction of CO at low
II: CH3OH + Na temperature under pressure
III: Electrolysis of H2O (b) water gas by oxidation of CO into CO2 (by steam)
which can be easily removed by dissolving in H20
IV : Silent electric discharge
(c) electrolysis of water
(a) I, II (b) II, III (d) all the above methods
(c) I, II, III, IV (d) IV
40. Out of the following metals which will give Hg oa
31. Select correct statement(s) abut lime light.
reaction with NaOH
(a) Oxy-hydrogen flame
(b) Used in welding I: Zn, II: Mg, III: Al, IV : Be
(c) Temperature of lime light is sufficient to melt (a) I, II, III, IV (b) I, III, IV
even platinum (c) II, IV (d) I, III
(d) All the above are correct 41. Choose correct statement(s).
32. H2 is adsorbed on palladium surface. It is a case of (a) Hg is more rapidly adsorbed on to surfaces than
D2
(a) occlusion (b) diffusion
(b) Hg reacts over 13 times faster with Cl2 than D,
(c) effusion (d) electro-osmosis because Hg has a lower energy of activation
33. D2O (heavy water) and H2O differ in following except (c) Both (a) and (b) are true
in (d) None of the above is true
(a) freezing point 42. Abundance of Hg in the earth’s atmosphere is very
(b) density
small. This is because
(c) ionic product of water
(a) the earth's gravitation field is too small to hold so
(d) its reaction with sodium
light an element
34. (A)FeCl3 solution + Zn---- > products (b) Hg exists in ortho and para form
(B) FeCl3 solution + H2 gas---- > product V (c) Hg is diatomic gas
FeCl3 solution gives blue colour with K4[Fe(CN)6] (d) Hg is not the metal
hence 43. Hg reacts much faster with Cl2 than D2, because
(a) X also gives blue colour with K4[Fe(CN)6 ] (a) rate of diffusion ofH2 is greater thanD2
(b) Y also gives blue colour with K4 [Fe(CN)6 ] (b) Hg has lower energy of activation than D2
(c) both X and Y give blue colour with K4[Fe(CN)6 ] (c) both (a) and (b) are correct
(d) none gives colour with K4 [Fe(CN)6 ] (d) none of the statements is correct
35. H+, D+ and T+ differ in all except in 44. Which is true statement about D2O and HgO?
(a) number of electrons (b) number of neutrons (a) D2O has lower dielectric constant than HgO
(c) ionic mass (d) all of these (b) NaCl is more soluble in D2O than in HgO
36. Which is radioactive isotope of hydrogen? (c) Both (a) and (b) are correct
(a) }H (b) *H (d) None of the above is correct
(c) ?H (d) All of these 45. Out of LiH, MgHj and CuH
37. Which is true about different forms of hydrogen? (a) all are ionic hydrides
(a) Ortho hydrogen has same spins of two nuclei (b) LiH, MgHg are ionic and CuH covalent hydride
clockwise or anticlockwise (c) all are covalent hydrides
(b) Para hydrogen has different spins of two nuclei (d) LiH is ionic, MgH2 and CuH are intermediate
(c) At absolute zero, there is 100% para form and at hydrides
high temperature, there is 75% ortho form 46. There are three isotopes of hydrogen and three
(d) All the above are correct naturally occurring isotopes of oxygen (160,170and
38. Which is not correct statement? 18 0). How many kinds of water are possible?
(a) s-block elements, except Be and Mg, form ionic (a) 18 (b) 16
hydride (c) 8 (d) 9
(b) BeHg, MgHg, CuHg, ZnHg, CaHg and HgH2 are 47. In the following compounds H is covalent bonded in
intermediate hydride case of
(c) p-block elements form covalent hydride
(a) BaHg (b) CaH2
(d) d- and /‘-block elements form ionic hydride
(c) SiH4 (d) NaH
Chapter 10: Hydrogen and Its Compounds | 453

48. Which of the following disproportionates when 57. H~ is a


treated with water? (a) Lewis base (b) Lowry-Bronsted base
(a) S03 (b) F2 (c) both (a) and (b) (d) None of these
(c) N2 (d) CL l2 58. Consider following reactions
49. Select the correct statement. I: A1H3 + H" -----> aih;
(a) Ammonia is more soluble in aqueous ammonium II: HjO + H" ------> H2 + OH-
chloride than in pure water. Select correct statements based on these reactions.
(b) Solid ammonium fluoride and ice are miscible in (a) H- is a Lewis acid in I and Lewis base in II
all proportions. (b) H- is a Lewis base in I and Bronsted base in II
(c) Both (a) and (b) are correct (c) H~ is a Lewis acid in I and Bronsted acid in II
(d) None of the above statements is correct (d) H- is a Lewis base in I and II
50. H, D and T (isotopes of hydrogen) have nuclear spin 59. Consider following statements :
quantum number respectively as I: Rate of transfer of D+ from D20 is slower than
<4^ (b)ij.l that of H- from H2O
,, 1 . 3 111 II: Ka for CH3COOH CH3COO" + H+ is
C 2’^’2 d 2’2’2 smaller than that of Ka for
CH3COOD CH3COO- +D+
51. D20 has maximum density at III: Tritium is a radioactive isotope.
(a) 4°C (b) 11.6°C Select correct statements
(c) 0°C (d) OK (a) I, II (b) II, III
52. is the hydride of (c) I, III (d) I, II, III
(a) 02 (b) H^O 60. Which is hydrolysis reaction?
(c) both (a) and (b) (d) none of these (a) O2"(a<7)+ HjOfZ) - 20H-(aq)
(b) O^"(ag)+2H2O(Z) -> 02(g)+ 4OH"(aq)
53. can also be obtained by the partial oxidation of (c) 4O2 (aq')+ 2FLgO(Z) ->3O2(g) + 4OH“(a<7)
2-propanol. (d) All of the above
(CH3)2CHOH + 02 ---- > (CH3)2CO + HA 61. The hydride ion H" is stronger base than its
Intermediate in this reaction is hydroxide ion OH". Which of the following reactions
/OH will occur if sodium hydride (NaH) is dissolved in
(a) (CH3)2C water?
OH (a) 2H+(ag) ---- > A +2e~
(b) H+(aq) + I^OU) ---- > 0H"+H2
(b) (CH3)3C (c) H’+fLOC/) ---- > No reaction
(d) None of the above
OH
(c) (CH3)2C 62. Permanent hardness of water can be removed by
0—0—H adding
(a) NaHC03 (b) Na2CO3
(d) none of the above (c) CaOCl2 (d) Cl2
54. H2O2 exists as......... in alkaline medium. 63. The correct decreasing order of basic strength is
(a) AsH3 > SbH3 > PH3 > NH3
(a) HOJ (b) HO®
(b) SbH3 > AsH3 > PH3 > NH3
(c) (ft (d) 1^0® (c) NH3 > PH3 > AsH3 > SbH3
18
55. In the following reaction using I^C^, (d) PH3 > AsH3 > SbH3 > NH3
18 64. Which statement is not correct for hydrogen?
2MnO4- + SHgOa + 6H+---- > 2Mn'2+ + SHjO + 5O2 (a) It has a very high ionisation potential
18 0 goes with (b) It is always collected at cathode
(c) It can form bonds in +1 as well as -1 oxidation
(a) H^O (b) 02 state
(c) both (a) and (b) (d) none of these (d) It has same electronegativity as halogens
56. Water gas is a mixture of 65. The most stable is
(a) CO and ^0 (b) COandN2 (a) NaH (b) RbH
(c) CO and Ha (d) COandCH4 (c) KH (d) LiH
454 ] Essential Inorganic Chemistry

66. H2O2 is a reducing agent in the reaction 78. Hydrogen is produced by the reaction
(a) Ag2O + H2O2 ---- > 2Ag + H2O + O2 (a) Na2O2 + 2HC1 (b) Mg + ^0
(c) BaO2 +HC1 (d) HjSA +^0
(b) 2KI + H2O2 ---- > 2KOH + I-2 .
79. Which metal gives hydrogen gas on heating with hot
(c) H2O2 +SO2 ---- > H2SO4
concentrated alkali?
(d) PbS + 4HA ---- > PbSO4 + 4H2O (a) Ag (b) Ni
67. Which cannot be oxidised by H2O2 ? (c) Zn (d) Cu
(a) Na2SO3 (b) PbS 80. Hydrogen peroxide is prepared in the laboratory by
(c) KI (d) O3 (a) passing CO2 into BaO2
68. Dil. H2SO4 and oxide react to produce hydrogen (b) adding MnO2 to dil. H2SO4
peroxide. The oxide is (c) adding Na2O2 to cold water
(a) MnO2 (b) PbO2
(d) adding PbO2 into KMnO4
(c) TiO2 (d) Na2O2
81. The hardness of water is estimated by
69. Heavy water is
(a) EDTA method (b) titrimetric method
(a) water at 4°C
(c) conductivity method (d) distillation method
(b) water containing heavy ions
(c) D2O 82. Hydrogen peroxide is reduced by
(d) water containing sulphate of Ca (a) ozone
70. Adsorbed hydrogen by palladium is known as (b) barium peroxide
(c) acidic solution of KMnO4
(a) nascent (b) atomic
(d) lead sulphide suspension
(c) heavy (d) occluded
83. Hard water is not fit for washing clothes because
71. Pure water can be obtained from sea water by
(a) it contains impurities
(a) centrifugation (b) plasmolysis (b) it is acidic in nature
(c) reverse osmosis (d) sedimentation (c) it gives precipitate with soap
72. Zn gives Hj gas with HaSQ, and HC1 but not with (d) it contains Na2SO4 and KC1
HNO3 because 84. The acid, not suitable for the preparation of hydrogen
(a) Zn acts as an oxidising agent when reacts with by the action of metals, is
HNO3 (a) HC1 (b) CH3COOH
(b) HNO3 is weaker acid than and HC1 (c) HNO3 (d) H2SO4
(c) in electrochemical series Zn is above hydrogen
85. Maximum number of hydrogen bonding in H2O is
(d) NO3 is reduced in preference to hydronium ion
(a) 1 (b) 2
73. Mass of one atom is 6.66 x 10 g. Its percentage in (0 3 (d) 4
an hydride is 95.24. Thus, hydride is 86. Ag2O + H2O2 ---- > 2Ag + + O2
(a) MH (b) MH2
In an above reaction H2O2 acts as a/an
(c) MH3 (d) MH4
(a) oxidising agent (b) reducing agent
74. On burning hydrogen in air, the colour of flame is (c) bleaching agent (d) None of these
(a) green (b) light bluish 87. Ionic hydride reacts with water to give
(c) yellow (d) none of these (a) hydride ions (b) acidic solution
75. Triple point of water is (c) protons (d) basic solutions
(a) 273.16 K (b) 373.15 K 88. HA is manufactured these days by
(c) 203.12 K (d) 193.16 K
(a) the action of H2SO4 on BaO2
76. Hydrogen can be fused to form helium at (b) the action of H2SO4 on Na2O2
(a) high temperature and high pressure (c) electrolysis of 50% Na2O2
(b) high temperature and low pressure (d) burning hydrogen in excess oxygen
(c) low temperature and high pressure 89. Which of the following is heavy water?
(d) low temperature and low pressure (a) HjO18 (b) HjO16
77. Water gas is produced by (c) HA (d) D2O
(a) passing steam over red hot coke 90. The volume of oxygen liberated at NTP from 15 mL of
(b) passing steam and air over red hot coke 20 volume HA is
(c) burning coke in excess of air (a) 250 mL (b) 300 mL
(d) burning coke in limited supply of air (c) 150 mL (d) 200 mL
Chapter 10: Hydrogen and Its Compounds I 455

91. Spin isomerism is shown by 100. �' D2 and T2 do not differ in


(a) dichloro benzene (b) hydrogen (a) freezing point (b) boiling point
(c) dibasic acid (d) n-butane (c) critical temperature (d) bond energy
92. Select incorrect statement. 101. �02 is present as ...... in acidic medium.
(a) Ortho and para hydrogen are different due to
difference in their nuclear spins (a) HO2 ion (b) H 3O; ion
+
(b) Ortho and para hydrogen are different due to (c) H ion (d) �- ion
difference in their electron spins 102. Hydroxyl radicals are used in organic reactions.
(c) Para hydrogen has a lower internal energy than These are obtained from
that of ortho hydrogen (a) glycol (b) glycerol
(d) Para hydrogen is more stable at lower (c) �02 /Fe +
2
(d) �02 /Fe 3+
temperature
93. SiH4 is an example of which of the following type of 103. �02 is a reducing agent as shown by
(a) �C½ � 2H + 02 + 2e-
+
hydrides
(a) ionic (b) interstitial (b) H:zO2 + 2H+ + 2e- � 2�0
(c) metallic (d) covalent (c) �02 � H+ + HO2
94. Molar ratio ofltzO2 oxidised by one mole ofMnO; in (d) �02 + H+ � H30;
basic and acidic medium is
104. Industrial source of�O2 is
(a) 1 : 1 (b) 1 : 2
(c) 3: 5 (a) by oxidation of 2-ethyl anthraquinol in a cyclic
(d) 5: 3
process
95. Electrolysis of aqueous NaCl and NaH differs in
(b) by partial oxidation of 2-propanol
(a) formation of basic solution at the cathode by (c) by both (a) and (b)
NaCl only (d) none of the above
(b) formation of basic solution at the cathode by NaH
only 105. Fuel used for rocket propulsion is a mixture of
(c) formation of� gas at cathode and anode both by (a) hydrazine and hydrogen peroxide
NaH only (b) hydrazine and TNT
(d) formation ofH2 gas at cathode and anode both by (c) hydroxyl amine and TNT
NaCl only (d) hydroxyl amine and hydrogen peroxide
96. Select correct statement(s). 106. Select correct statement(s).
(a) Hydride ion is larger than any of the halide ions (a) H:zO2 reduces MnO; to Mn2 • in acidic medium
except iodide (b) H:zO2 reduces MnO4 to MnO2 in basic medium
(b) Hydrides ions are reducing agents (c) �02 can be used to bleach blackened oil
(c) Boranes are electron deficient hydrides paintings
(d) All of the above are correct statements (d) All the above are correct statement
97. Metallic hydrides
107. Electron-deficient hydride is/are
(a) are also called interstitial hydrides
(b) are non-stoichiometric, being deficient in (a) BH3 (b) AlH3
hydrogen (c) Be� (d) all of these
(c) are poor conductors of electricity, exhibit less 108.Perhydrol is
paramagnetism and have hydrogen as atom and (a) 30% solution ofl:IiO2
not as molecule (b) "100 volume" H:zO2
(d) have all the properties given above (c) both (a) and (b) are correct
98. Hydride ion is a (d) none of the above is correct
{a) strong conjugate acid of�
(b) strong conjugate base of�
109. 10 mL of Hi<l.? solution on treatment with KI and
(c) strong conjugate acid ofH+ titration of liberated Li, required 10 mL of 1 N hypo.
Thus �Cl.? is
(d) strong conjugate base ofH-
(a) lN (b) 5.6 volume
99. Select incorrect statement. (c) 17 gL- 1
(d) all are correct
(a) H+ can exist as H 9o; in water
(b) }'2 is thermally stable 110. In alkaline �02
(c) Ionisation of CH 3COOH is slower than that of (a) Mn2 + changes to Mn<l.?
CH3COOD (b) Cr3+ changes to cra:-
(d) Kinetic isotopic effect is observed when there is (c) both (a) and (b) are correct
retardation in the rate water �O is replaced by (d) none of the above is correct
D20.
456 | Essential Inorganic Chemistry

One or More Than One Options Correct 10. Industrially HA is obtained from
(a) 2-ethyl anthraquinol by oxidation and then
1. Select correct statements.
(a) Boiling point of HjO, NH3, HF are maximum in reduction in a cyclic process
their respective group due to intermolecular (b) H2SO5
H-bonding (c) H2S2O8
(b) Boiling point of CH4 out of CH4, SiH4, GeH4 and (d) BaO2
SnH4 is least due to lack of H-bonding 11. Density of H2O is maximum at
(c) Benzoic acid forms dimer by H-bonding
(a) 0°C (b) 100°C
(d) H2O exists as H9O| in acidic medium
(c) -273°C (d) 4°C
2. H2O'22 can be obtained when following reacts with 12. Temporary hardness and permanent hardness can be
HjSClj except with removed respectively by addition of
(a) PbO2 (b) BaO2 (a) CaO, CaCO3 (b) CaO, Na2CO'3;
(c) Na2O2 (d) SrO2 (c) Na2CO3, CaO (d) NaHCO3, CaCl2
3. How does H2O2 differ from O3 in its chemical action? 13. Decomposition of HA can be prevented by addition
(a) In oxidising PbS to PbSO4 of
(b) In liberating Lj from KI (a) ferrous sulphate (b) oxalic acid
(c) In decolourising acidified KMnO4 (c) ozone (d) glycerol
(d) In oxidising K4 [Fe(CN)6 ] to K3[Fe(CN)6 ] 14. Which is/are true statement(s)?
4. Which of the following statements about HjC^ are (a) The layer of ice on the surface of river in the
true? winter acts as a thermal insulator between the
(a) H2O2 is used to clean oil paintings water below and the air above
(b) H2O2 acts as oxidising as well as reducing agent (b) The fish and other marine organisms are enabled
(c) Two hydroxyl groups in H^O-j lie in the same to survive long periods of freezing weather due to
plane the fact that ice is lighter than water
(d) It retains same structure in liquid and solid form (c) When ice is formed volume decreases
5. H+, D+ and T+ differ in all except in
(d) Density of ice is maximum at 0°C
15. Hydrides of metals are named like
(a) number of electrons
(a) alkane (b) alkene
(b) number of neutrons
(c) ionic mass (c) alkyne (d) none is correct
(d) electronic configuration 16. In water, or in aqueous solutions of HC1 or HjSOi,
6. Which is/are radioactive isotope(s) of hydrogen? proton exists as
(a)H3O+ (b) HC^O);
(a) ]H (b) fH (c) H(H2O)* (d) [H3O+][OH-J
(c) jH (d) <H
17. HA is “5.6 volume”, then
7. Which is a source of nascent hydrogen?
(a) it is 1.7% weight by volume
(a) Zn + dil. HC1, (b) it is 1 N
(b) CH3OH + Na (c) it is 1 M
(c) Electrolysis of I^O (d) it is 5.6 M
(d) Silent electric discharge 18. can be obtained from
8. Select correct statement(s) about lime light (a) water gas (CO + H2) by liquefication of CO at low
(a) Oxy-hydrogen flame temperature under pressure
(b) Used in welding (b) water gas by oxidation of CO into C02 (by steam)
(c) Temperature of lime light is sufficient to melt which can be easily removed by dissolving in H>0
even platinum
(c) electrolysis of water or NaOH
(d) Produced in an endothermic reaction
(d) reaction of NaOH on Zn
9. “10 volume” H2O2 means
19. Out of the following metals which will give H2 on
(a) 1 mL HA gives 10 mL O2 at NTP
reaction with NaOH
(b) 1 g HA gives 10 mL O2 at NTP I: Zn, II: Mg, III: Al, IV : Be
(c) 1 mol HA gives 10 mL O2 at NTP (a) I, II, III, IV (b) I, III, IV
(d) 10 mL gives 1 mol A and 1 mol O2 at NTP (c) II, IV (d) I, III
Chapter 10: Hydrogen and Its Compounds I 457
20. Which is hydrolysis reaction? 1. Yellow colouredFeC1 3 solution changed to light green
( a) OZ-(aq) + H20(l) ---+ 2OH-( aq) (appeared as colourless) in the experiment of
(b ) �-(aq) + 2lii0<Z> ---+ O 2(g) + 4OH-(aq) �)A �)B
(c) 402 (aq) + 28iO(l) ---+ 3O 2 (g) + 4OH-(aq)
(c) both of these (d) none of these

(d) Fe 3+ + 3HiO ---+ Fe(OH) 3 + 3H+ 2. Select correct statements.


(a) Zn pieces liberate nascent hydrogen on reaction
Brain Twisters with acidic solution ofFeCl 3
(b) FeC1 3 solution is reduced to FeC1 2 in the
I. How many grams of barium hydride must be treated experiments of A and B both
with water to obtain 4.36 L of hydrogen at 20°C and (c) Blue colour complex is formed in both the
0.975 atm pressure? (Ba = 137). experiments on reaction with K4 [Fe(CN) 6 ]
2. An element X reacts with I¼ gas at 200°C to form
(d) A blood red colour is formed on reaction with
KCNS in experiment B
compound Y. When Y is heated to a higher
te mperature, it decomposes to the element X and Hi
gas in the ratio of 559 mL ofH2 (measured at STP) for
Passage 2
LOO g of X reacted. X also combines with Cl2 to form a Depletion of resources of fossil fuels will at some
compound Z, which contains 63.89 per cent by mass future time make hydrogen, either for use directly by
of chlorine. Deduce the identify of X, Y and Z. combustion and electrochemically in fuel cells or
indirectly via hydrogenation of coal, the major alternative
3. �02 is reduced rapidly by Sn2 +. H2 O 2 decomposes to nuclear energy; hence, arises the current interest in the
slowly at room temperature to yield 02 and ttiO. so-called hydrogen economy.
Calculate the volume of 02 gas produced at 20°C and A thermochemical cycle for hydrogen production
1 atm when 200 g "202 having 10% by mass H2O 2 is involves at least one element that can exist in two
treated with 100 mL of 2.00 M Sn 2 + and then the different oxidation states.
mixture is allowed to stand until no further reaction 2Ifi0 ---+ 2Hi + 02
occurs.
Read the above passage and answer the following
4. �02 is marked "22.4 volume". How much of it (in questions.
mL) are required to oxidise 3.4 g �S gas?
1. Production of Hi from �O requires a net input of
5. A 'hair-bleach' solution contains HzO 2• The amount of energy that would come from
�02 in a 12.5 g sample was determined by titration (a) nuclear source (b) solar source
withK.MnO 4 in acidic solution. If39.3 mL of 0.5045 N (c) both of these (d) None of these
KMn04 was required, how many equivalents of HiO 2
2. In the above th�rmochemical cycle,FeC12 is one of the
are contained in the solution ? What is the mass
percentage ofHiO 2 in the bleach solution? catalyst involved and thus regenerated at the end of
the reaction, after formed into
6. A 1.84 g sample of an alkaline-earth metal hydride (a) FeC1 3,FeO, Fe2O 3 (b) FeC1 3,Ff½O 3,Fe 3O 4
was treated with an excess of dil. HCl and the (c) FeC1 3 , FeO Cd) FeCl3 ,Fe3O 4
resulting gas was collected in 1.0 L container at 20°C.
The measured pressure of the gas was 750.0 mmHg. 8. Ha is produced in the following thermochemical
Identify the alkaline earth-metal. reaction
7. A 5.0 mL ofHiO 2 solution liberates 0.508 g � from an (a) FeCl2 + HiO ---+
acidified KI solution. Calculate volume strength of (b) Fe 3O4 + Cl2 + HCI ---+
Ii:z02 solution. (c) FeO + �O ---+
(d) FeC1 3 + HiO ---+
Passage Comprehension Questions
Passage 1 Assertion & Reason
Codes:
Read the following experiments and answer the (a) Both A and R are true and R is the correct
questions at the end of it.
explanation of A.
"Research scholar A added zinc pieces into aqueous (b) Both A and Rare true but Ris not the correct
FeC1 3 solution and performed some experiments with explanation of A.
resultant solution." Research scholar B passed Hi gas into (c) A is true but Ris false.
aqueous FeC1 3 solution and performed some experiments (d) A is false but Ris true.
With resultant solution." (e) A and Rboth are false.
458 | Essential Inorganic Chemistry

1. Assertion (A): On adding zinc pieces to FeCl3(ag) True & False


solution, colour changes from deep yellow to light
1. K02 is used as life supports in space crafts,
green.
submarines and emergency breathing apparatus
Reason (R): Aqueous FeCl3 solution is acidic and since it absorbs C02 and releases 02.
on adding zinc, nascent hydrogen is produced which 2. 02 is diamagnetic and K02 is paramagnetic.
reduces deep yellow FeCl3 solution to light green
FeCl2 solution. 3. Peroxides and oxides are decomposed by forming
2. Assertion (A): On passing H2 gas into aqueous 02 and their behaviour becomes alkaline.
Fe3+ solution, resulting solution gives test of Fe3+. 4. Due to very small size of H+, it has very high
polarising power and therefore, distorts the
Reason (R): Ho(g) reduces Fe3+ to Fe2+ butFe2+ is
electron-cloud on other atoms.
oxidised to Fe3+ by atmospheric oxygen.
5. H+ exists as H9O4+.
3. Assertion (A): Drinking of heavy water (D2O)
mixed water (H2O) could prove fatal. 6. Ionic size of H, H+ and H~ are equal.
7. Ionisation constant of CH3COOH and CH3COOD are
Reason (R): There is slower rate of transfer of D+
compared with that of H+ ion in acid-base reactions same.
involved in enzyme catalysis. 8. Spin isomerism is observed in H2.
4. Assertion (A): Ionisation constant of CH3COOH 9. On electrolysis of HCOONa(ag), H^ is obtained at
and CH3COOD are very equal. cathode and anode both.
Reason (R): The kinetic isotopic effect is not 10. Ionic hydrides conduct electricity in fused state.
observed in the ionisation of weak acid.
11. Hydride is a conjugate acid ofH2.
5. Assertion (A): In space crafts A gas is used in
12. Hydride is a Lewis base.
fuel cells for generating electrical energy and for
providing clean drinking water to the astronauts. 13. H2O2 is “10 volume” if 1 mL gives 10 mol O2.
Reason (R): A fuel cell may have an alkaline or 14. H2O2 behaves as an acid, oxidising agent and
acidic electrolyte. reducing agent.
6. Assertion (A): Hydride (H") is a conjugate base of 15. HA is used as fuel for rockets.
hydrogen (H^). 16. H2O2 oxidises Cr3+ to CrO|_ in basic medium.
Reason (R): Every negative ion is an electron-pair
H-
donor and is thus, a Lewis base. 0—>0.
17. True structure of HA is
7. Assertion (A): Ionic hydrides are the ready-made H
source of A.
18. Catalase present in blood interacts with HjOj
Reason (R): LiH is an ionic hydride.
forming 02.
8. Assertion (A): A^2 decomposes Na2CO3 to give 19. Density of H20 is not affected by change in
CO2. temperature.
Reason (R): H2O2 has two protons and is thus a 20. Tritium is the radioactive isotope of hydrogen.
strong acid.
9. Assertion (A): When blood is added to a solution Fill in the Blanks
ofH2O2, the solution bubbles furiously. 1. Aqueous solution of on electrolysis gives Hj
Reason (R): Catalase (an enzyme) present in at anode and cathode both.
blood decomposes AQj and produces bubbles of O2. 2 and are two spin isomers of Hj.
10. Assertion (A): KO2 is used as life supports in 3. At absolute zero there is only form ofH>.
space crafts. 4. Different atomic masses of three isotopes of hydrogen
Reason (R): KO2 absorbs CO2 and releases O2. is due to the difference in the number of
Chapter 10: Hydrogen and Its Compounds I 459
6. Hydrogen resembles ............... metals in the 2. Match the species in Column I with the
formation of ............... cation and ............... elements corresponding properties in Column II.
in the formation of ............... anion.
Column I Column II
6. Hydrogen is the ............... most abundant element in A. LiH 1. Reducing agent
the earth's crust.
B. H 20 2. Lewis base
7. Water gas contains CO and ............... . C. H 30 .. 3. Conjugate base
8. Very pure� is obtained by electrolysis of ............... . D. BH 3 4. Lewis acid
9. � is ............... rapidly adsorbed on to the surfaces E. H 5. Lowry Bronsted acid
than D2 • 6. Lowry Bronsted base

10. � reacts over 13 times faster with Cl2 than D2 ,


because � has a ............... energy of activation. Integer Answer Types
This section contains 8 questions. The answer to each
11 . Auto-protolysis constant of �O is ............... times of the questions is a single digit integer, ranging from Oto
more than that ofD20. 9. The appropriate bubbles below the respectively
12. Due to lower value of dielectric constant of D2O than question numbers in the ORS have to be darkened. For
�O, ionic compounds are ............... soluble in D2O example, if the correct answers to question number X, Y, Z
and W (say) are 6, 0, 9 and 2, respectively, then the correct
than in�O.
darkening of bubbles will like the following
13. B.E. of 0-D bond in D2O is ............... than that of
X Y Z W
0-H bond in �O. @ @ @ @
14. �Li +tn --,;He+............... G) (1) G) G)
15. When water is electrolysed, � is liberated much ® ® ® ®
@ @ @ @
............... thanD2 and thus remaining water becomes
@ @ @ @
enriched in ............... .
@ @ @ @
16• Tritium 1s · WI'deI y used as rad'1oact·1ve .............. . @ @ @ @
17, Considerable differences between the physical (J_) (J_) (J_) (J_)
properties of ortho and para forms of� is because of @ @ @ @
diflierences m
· their ............... energy. ® @I@ ®
18. LiH, NaH, KH are some examples of ...............
hydrides. Set I
19, Interstitial hydrides are made by ............... metals. 1. Number of neutrons in the heaviest isotopes of
+
20. H9 0t is ............... H ion and can be written as hydrogen is ............ .
··············· . 2. Number of peroxy linkage (s) in�S2O8 is ............ .
3. Two moles ofMnO4 reduces x mole(s) of�O2 in basic
Matrix-Match Type Questions medium. x is ............ .
I. One or More Correct 4. 68 g �02 will make ............ equivalent(s}.
1. Match the species in Column I with corresponding
properties in Column II. 5. �02 is "33.6" volume. Thus, is normality is ............ .

Column I Column II 6. Cr05 has peroxy linkage(s). x is ............ .


A. H20 1. Reducing agent 7. How many moles of �Oi are oxidised by 2 moles of
B. H20 2 2. Oxidising agent KMnO4 in acidic medium?
C. Aqueous Na 20 3. Paramagnetic
8. 5.0 mL of�Oi solution liberates 1.016 g ofli from an
0. Aqueous K02 4. Diamagnetic
acidified KI solution. Thus, volume strength (at STP)
E. Aqueous CaH 2 5. Turns red litmus blue
is approximately ............ .
6. Turns blue litmus red
460 | Essential Inorganic Chemistry

Set II (a) Write the formula of the simplest binary hydride


1. How many of the following make alkaline solution in of each element.
water? (b) Classify each binary hydride as ionic, covalent or
interstitial.
(c) Which of these hydrides are molecular?
2. How many of the following are oxidant? (d) What is the oxidation state of the element in
H2O, H2O2, KO2, Na2O2, Na2O, hydrides A, C and D?
3. Ionic product, [H+][HO2] of H^ was found *'• be 2. Arrange the compounds of hydrogen with elements
1 x 10“12 at a given temperature. Thus, pH of H^ is
E F G A
4. How many types of water can be formed out of
!H,?H,i3H ando60? £
D
5. H2 is formed by electrolysis of aqueous brine solution.
In an experiment 0.112 mL of Hj gas is formed at
NTP. What is the pH after electrolysis of brine
solution?
6. How many of the following undergo
disproportionation when dissolved in water? (a) A, B, C, D in increasing acid strength
CI2 > P2 > Na2O2, KO2, K2O (b) A, B, C, D in increasing boiling point
(c) E, F, G, A in increasing H-bonding strength
7. 74N undergoes nuclear reaction
(n, T) 3. In the Periodic Table given below
14N ------- >
What is atomic number of the atom formed?
8. 5.6 g of a unsaturated hydrocarbon occupies 2.24 L at
NTP. It also required 4.48 L of H2 gas for its
reduction. How many n bonds does it have?

Test Yourself on the Periodic Table


1. Look at the location of elements A, B, C and D in the
following Periodic Table
trace the element(s)
(a) that has maximum dissociation energy in
A C diatomic state
(b) that forms double bond in diatomic state
B D (c) that forms triple bond in diatomic state
(d) that forms compounds with +1 oxidation state
with hydrogen
Answers & Solutions
Master Exercises
Exercise 1
Short Answer Type Questions 8.
6-
&+- 6-
1. (c) H /F· ....fj/F· .... s. Metallic Property Used as
no. hydride
&- a+
,.O---H-O, 1. Low thermal neutron - Moderation in nuclear
(d) CHa-C( a+ o-_)C-CH3 (dimer) capture cross- section reaction
O-H---O area

(f)
CHa
6-
,o/H'
a+
5-.......
...
a+
H
2. TiH 2 High neutron- capture Shielding material in
cross-section area nuclear reaction

CHa Analytical Questions


2. Hydrogen is a non-metal in its natural gaseous state as 1. H2O2 = 1 g (impure)= � g pure
well as in liquid state. If a very thin layer of liquid 1 0
hydrogen (about 0.5 mm) placed in between two . X 1
Equivalent ofH2O2 = x
single-crystal alumina is subjected to shock compression 100 17
under a very high pressure (about 2 million atmopshere), Let normality ofKMn04 = N
then liquid hydrogen behaves like metal and its = Nx
conducting power is very comparable to Rb and Cs. This Equivalent ofKMnO4
1000
transition of a non-metal to a metal under very high
pressure is called metallisation of hydrogen. Nx
--=--
x
1000 1700
This phenomenon is taking place at the planet Jupiter.
=lO
3. Hydrides : LiH, BeH2 , BH3 N
17
4. In aqueous solution, hydrides are decomposed by H2O 2. Let normality of H2O2 = N
forming which show reducing nature. . 5N N
Equivalent of H2O2 = =
Ca� + 2H2O ---+ Ca(OH)2 +H2 1000 200
5. (a) boranes (b) silanes (c) alkanes (d) LiAIH4 . 0.508
Equivalent ofl 2 =
127
6. A:HCOONa
0.508
COONa Equ1v. alent ofH.,O2 =--
oxalate decolourises Mn�/W
M

127
B: I N 0.508
COONa -=-- =) N=0.8N H 2O2
c 2O:- ---+ 2CO2 + 2e­ 200 127
Mn04 + SH• + 5e- ---+ Mn 2 • + 4H2O If volume strength = x
C :H COOH HCOOH reduces Tollen's reagent then N=�
COOH 5.6
D :I x =Nx5.6
COOH = 0.8x 5.6 = 4.48volume
HCOOH+ Ag 2O � 2Ag +H2O+CO2 3. CA) : H202 ; (B) : H20
Tollen's reagent
4. 2Cr(OH>:, + 31¼02 + 2OH- ---+ 2Cr0}-+ Sf¼O
7. (a) SiCl4 + Li[AlH4 ] ---+ SiH4 + AlCI 3 + HCI yellow
(b) 4AgNO3 + SiH4 ---+ 4Ag + 4HNO3 + Si 2Cr0 J- + 2H +
� Cr2ot+ H:P
(c) 3Cu8O4 + 2PH3 ---+ Cu:12 +3Hf)O.1 orange

(d) SiH 4 + 4Cl2 ---+ SiCl4 +4HCl 5. pH= 5.91


(e) Al4C3 + 12H2O ---+ 4Al(OH)3 +3CH4 [H+ 1 = 10 -5.9l = 10-6 X 10°·09 = 1.23 X 10-6 M
462 | Essential Inorganic Chemistry

[HO2] = 1.23x IO-6 M a 613.1 mol H2 is obtained from = — x 613.1 g CaHj


/f(H2O2) = (H*][HO2] 2 2
= (1.23 x IO-6)2 = 12875.1g
= 1.51 x IO’12 M2 = 12.875 kg

6. React H2O with a reducing agent to produce H2 9. K is oxidised by water :


2K + 2H2O 2KOH + H2
xH2O+2Af ---- > Af2Ox+xH2
F2 is reduced by water :
(b) Al and (d) Ca could be used
2F9 + 2H,0---- > 4HF + O2
7. (a), (b), (d) are different kinds of water and have similar Cl2 and Br2 disproportionate when treated with H^O
properties.
Cl22 + H2O---- > HC1 + HOC1
v>i
(c) and (e) are different kinds of hydrogen peroxides and 10. Moles of D2 =^ =— 2x0.9— = 0.0736
have similar properties. 2
2 RT 0.0821x298
Properties of (a), (b) and (d) are quite different from H2O = 0.0736 mol = 0.0736 x 18 g ]
those of (c) and (e).
Actual water required with 0.015%D2 and 80% recovery
8. (a) LiH +H2O ---- > Li(OH)+ H2 _ 0.0736 x 18
8g 2i “0.015 80
1 mo 1 mol ------ x-----
100 100
CaH2 +2H2O ---- > Ca(OH)2 + 2H2 = 11040 g
42g 4g
1 mol 2 mol = 11.040 kg

8 g LiH gives = 2 g H2 11. A: CO; B:H2; C:CO2; P:CaCO3; £:Ca(HC03)j


1 g LiH gives = 0.25 g H2 12. A and B both are peroxo acids since, H2O2 is formed due
42 g CaH2 gives = 4 g H2 to their reactions with H2O.
1 g CaH2 = 0.095 g. O 0
Thus, LiH
T HO—J— oh+h2o,
HO—S—O—O—H+ H2O---->
(b) pV=nRT l ' X
n(H2) = ^_
2 RT
t linkage
0 peroxy 0

150x100 , O | O 0
=----------------=613.1 mol 1—6^0—J—OH + 2H2O 2HO-J-OH
0.0821x298 Hi ’2
X i
2 mol H2 is obtained from = 1 mol CaH2 = 42 g CaH2 0 0 0
+ 2HjO2

Exercise 2
Only One Option Correct
l.(c) Salt Ionisation At cathode At anode
(a) NaH Na* + H“ Na* + e“ ---- > Na
1
Na + H2O ---- > NaOH + - H2
2
(b) HCOONa HCOCT + Na* same 1 H, + CO2 + e"
HCOO­
2 2 2
te) NaCl Na* +ci­ same
cr ici2 + e"
2 2
(d) LiH Li* +H“ H~ ----> iH2 + e"
2 2
Li + H2O LiOH + - H2
2 2

Thus, (c)
Chapter 10: Hydrogen and Its Compounds | 463

2. (a) 55. (b) 56. (c) 57. (c) 58. (b) 59. (d)
3. (c) Force of attraction is minimum thus most volatile 60. (a) 61. (b) 62. (b) 63. (c) 64. (b)
4. (a) o-nitrophenol has intramolecular H-bonding thus 65. (d) 66. (a) 67. (d) 68. (d) 69. (c)
most volatile. 70. (d) 71. (c) 72. (d)
73. (b) Mass of one atom = 6.66 x 10'-“g
Mass of Nq atoms = 6.66 x 10r23 x6.02xl023 g
N-
= 40g
I
0 Thus, atomic weight of the element =40

Thus, (a) Element % %/at wt. Ratio


5. (d) 6. (d) M 95.24 2.381 1
H 4.76 4.76 2
7. (a) PbO2 does not have peroxy linkage
Thus, (a) Thus, hydride is AfH2
8. (a) 9. (b) 10. (d) 11. (c) 12. (b) Thus, (b)
13. (c) 03 does not decolourise KMnO,'4 74. (b) 75. (a) 76. (a) 77. (a)
A -- -
14. (c) 15. (d) 78. (b) Mg+H2O -=-» Mg(OH)2+H2. other are for H2O2
16. (b) 17. (a) 18. (a)
19. (d) 79. (c)
20. (b) Temporary hardness is due to dissolved HCO3 of 80. (a) BaO., + CO2 + H.,0 ---- > BaCO, + H,O,
Ca2+ and Mg2+ 81. (a) 82. (d) 83. (c) 84. (c) 85. (d)
Ca(HCO3)2 + CaO ---- > 2CaCO3+H2O 89. (d)
86. (b) 87. (d) 88. (c)
Permanent hardness is due to dissolved Cl and SO2 of
— n■ 90. (b) “20 volume” means
Ca2+ and Mg2+
1 mL H2O2 gives = 20 mL O2 at NTP
CaCL + Na„CO, — CaCO3X+2NaCl
thus, 15 mL H2O2 gives = 300 mL O2 at NTP
CaSO4 + Na2CO.*3 CaCO3X + Na^
91. (b) 92. (b) 93. (d) 94. (c) 95. (c)
21. (d) 22. (c) 23. (a) 24. (d) 25. (d)
96. (d) 97. (d) 98. (b) 99. (c) 100. (d)
26. (d) 27. (c) 28. (c)
101. (b) 102. (b) 103. (d) 104. (b) 105. (c)
29. (c) H2O2 9O + O2
* *H12'~
-1 -2 0 106. (d) 107. (d) 108. (b) 109. (c) 110. (d)
| reduction |
oxidation One or More Than One Options Correct
30. (a) 31. (d) 32. (a) 33. (d) 1. (a, b, c, d) 2. (a) 3. (c) 4. (a,b,d) 5. (a,d)
34. (b) FeCl3 solution is acidic due to hydrolysis when Zn is 6. (c) 7. (a, b) 8. (a, b, c) 9. (a) 10. (a)
added, nascent hydrogen is formed which reduces FeCl3 11. (d) 12. (b) 13. (d) 14. (a, b) 15. (a)
toFeCl2 and thus no reaction withK4[Fe(CN)6] 16. (a, b, c) 17. (a, b) 18. (a, b. c. d) 19. (b)
FeCl3 +K4[Fe(CN)6] ---- > KFe[Fe(CN)6] + 3KC1 20. (a, d)
blue
|H(Zn + HCl)
Brain Twisters
FeCl2 +K4[Fe(CN)6] ---- > No colour
1. BaH2+2H2O Ba(OH)2 + H2
FeCl3 + 3H2O ---- > Fe(OH)3 + 3HC1 139 g 2g = 1 mol
Zn + 2HC1 ---- > ZnCl2 + 2H
pV = nRT
gas is not effective in reducing FeCl3 to FeCl.2 0.975x4.36
Thus, (b) RT 0.0821x293
35. (b) 36. (c) 37. (d) 38. (d) 39. (d) = 0.1767 mol
40. (b) 41. (c) 42. (a) 43. (b) 44. (a) 1 mol H2 is obtained from = 139 g BaH2
45. (d) 46. (a) 47. (c) 48. (d) 49. (c) Hence, 0.1767 molH2 is obtained from = 139 x 0.1767
50. (a) 51. (b) 52. (a) 53. (c) 54. (a) = 24.56 gBaHj
464 | Essential Inorganic Chemistry

2. X + H2 Y This amount is present in 1 mL solution.


Y ---- > X + H2 H2S+H2O2 ---- > 2H2O + S
1.00 g 559 mL at STP 34 g 68 g
22400 mL H2 at STP = 2g H2 34 g HgS is oxidised by = 68 g H2O2
2 3.4 g is oxidised by = || x 3.4 = 6.8 g
a 559 mL H2 at STP =------- x 559= 0.05 g H„
2 22400 2
Also 559 mL H2 combines with = 1 g X 0.068 g H2O2 is present in = 1 mL
22400 mL H2 combines with = 40 g X 6.8 g H2O2 is present in = 100 mL
Thus, atomic mass of X =40 g mol"1 Or density of “x volume” H2O2 = g/mL
Thus, X is Ca 68x22.4
density of “22.4 volume” H2O2 = = 0.068 g/mL
Molecular formula of Y 22400

Element Amount % Molar ratio


Simplest 6.8 g 1^02 is in = 100 mL
ratio
5. 2MnO; + 5H2O2 + 6H+ ---- > 2Mn2+ + Slip + 5O2
Ca 1 g 95.24% 2.38 1
1000 mL of IN MnO; = 1000 mL of IN H2O2
H 0.05 g 4.76% 4.76 2 Or equivalent of H2O2 = equivalent of MnO<
Thus, Y is CaHj _ 39.3 x 0,5045 = q 0198
1000
Molecular formula of Z
a Amount of pure H2O2 = 0.0198 x 17 = 0.337 g
Element % Molar ratio Simplest ratio O 887
% ofH2O2 in bleach solution = —----- x 100 =2.69%
Cl 63.89% 1 80 2
12.5
6. Alkaline-earth metal hydride has M2+ (divalent cation)
Ca 36 11% 0.90 1
and H", hence, it can be represented as AfH2.
Thus, Z is CaC^. AfH2 + 2H2O Af(OH)2 + 2H2
3. 2H+ + H2O2 + Sn2+ -- > Sn4+ + 2H2O
z+ ---- ..(i) Number of moles of H2 collected
2H2O2 -- > 2H2O + O2 (ii) 750 1A
----- xl.0
pV 760
200 g H2O2 sample = 200 x RT 0.0821x293
= 20 g pure H2O2 - 0.041 mol
moles of 0.041
= 0.588 mol pure H22^2
O, Thus, number of moles of MH2 =
2 2
and 100 mL of 2M Sn2+ = 0.2 mol Sn2+
= 0.0205 mol
By reaction (i):
Let molar mass of M be = M
0.2 mol Sn2+ reduces = 0.2 mol H2O2
then molar mass of AfH9 = (Af + 2)
hence unreacted H2O2 = 0.588 - 0.200 1.84
given number of moles AfH2 = ——-
= 0.388 mol
By reaction (ii): 1 84
———=0.0205
0 ?8R Af + 2
O2 obtained from 0.388 mol H2O2 =-^— = 0.194 mol
1 84
Af + 2 = ——- = 89.7
nRT 0.194x0,0821x293 0.0205
Volume of O2 gas =
P 1 M = 87.7
= 4.67 L Thus, M is Sr and hydride is SrH2.
4. H2O2 is marked “22.4 volume" which means 1 mL H2O2 7. 0.508 g I2 = —equivalent I2
127
at NTP gives 22.4 mL of O2 at NTP.
_ 0.508
2H2O2 ---- > 2H2O + O2 equivalent H2O2 in 5 mL solution
127
(2 mol = 68 g) 22400 mL at NTP . 0.508 1000
MH,09) =-------x------- = 0.8N
22400 mL of O2 is from = 68 g H2O2 2 2 127 5
68 Volume strength = N x 5.6
22.4 mL of O2 is from = — — x 22.4 g H2O2 = 0.068 g
= 0.8 x 5.6
= 4.48 volume
Chapter 10 : Hydrogen and Its Compounds | 465

Passage Comprehension Questions Set II

Passage 1
Questions 1 2 3 4 5 £ 7_ 8
Answers 3 4 6 6 9 4 6 2
1. (a) 2. (a,d) ©I© ©
© © © © ©
Passage 2 ©. © © © © © © ©
1. (c) 2. (d) 3. (a) © © © © © ©
© © © © © ©._©. ©
Assertion & Reason © ® © © ® ©© ©
1. (a) 2. (c) 3. (a) 4. (e) 5. (b) 6. (b) © © © © © ©
7. (b) 8. (c) 9. (a) 10. (a) © © © © © ©
0 0 0 0 0 _0_0 0
True & False ® © ® ® ® © ® I®
1. T 2. F 3. T 4. T 5. T 6. F © © © © © ©
© ©
7. F 8. T 9. T 10. T 11. F 12. T
13. F 14. T 15. T 16. T 17. F 18. T
19. F 20. T Test Yourself on the Periodic Table
1. (a) A: NaH;
Fill in the Blanks
B:PdHx;
1. NaHorHCOONa 2. Ortho and para 3. para
4. neutrons 5. alkali, monovalent, Group VII, CjHjS;
monovalent
D: HI
6. tenth 7.H2 8. H2O (acidified) or aq. KOH 9. more (b) NaH — Ionic; PdHx — interstitial
10. lower H2S and HI—covalent
11.3.33 [KJH2O) = 1x10r14; K(D 2O) = 3xl0r15] 12. (c) HjS and HI—molecular
w
less NaH and PdHz—three dimensional crystals.
13. greater 14. ’T 15. faster, D2O 16.tracer
(d) Na = +1; S=-2; I=-l
17. internal 18. ionic 19. transition 2. (a) hydrogen compounds with A, B, C and D are
20. hydrated, H(H20)<
HF< HC1< HBr< HI
increasing acid strength
Matrix-Match Type Questions
(b) HC1 < HBr < HI < HF (increasing boiling point)
I. One or More Correct
(c) C— H<N—H<0—H <F—H
1. (A) —(4); (B)-(1,2,4,5); (C)-(4,5);
(A = F, B = Cl, C = Br, 0 = 1, E = C, F = N,
(D) — (2,3,5) (E) — (1,4,5) G = 0)
2. (A)-(1,2,3); (B)-(2,5,6); (C) —(3,6);
a
(D)-(l,4); (E)-(l) c b

Integer Answer Types 3.


Sell
Questions—► 1 2 3 4 5 6 7 8 LJ
Answers —► 2 1
© © ©
3
©
4
©
6 2
©I© ©
5 9
□«
© © ©. © © ©© © a=H
© © © © b=O
© © © © © _© _® ©
© © ®® © c=N
© © ©
© © © © ® © d - alkali metals
© © © © © _© _® ©
0 0 0 0 _0_0
® ® ® © © ®^_® ®
© © © © © ©I© ©
With Group 3 (boron family), we see the first evidence of
a change from metallic behaviour of the elements at the
left side of the Periodic Table to non-metal behaviour on
the right side of the Table. Boron is a metalloid, whereas
all the other elements of this group are metals.

p-Block Elements
-Boron Family (Group 13)

Shells up· to (n - 1) are completely filled and


differentiating electron (last filling electron) enters into
11p-suborbits - elements constitute what we call p-block
elements.
Group➔ 13 14 15 16 17 18 • Group 13(111A) (The Boron Family)
Period! (IIIA) (IVA) (VA) (VIA) (VIIA) (VIIIA)
• Compounds of Boron
2 B C N 0 F Ne
• Compounds of Aluminium
3 Al Si p s Cl Ar
4 Ga Ge As Se Br Kr
5 In Sn Sb Te Xe
6 Tl Pb Bi Po At Rn
ns 2np 1 ns 2
np 2 2 4 2 5
np ns np ns np
ns 2 3 ns 2np 6
• The p-block elements are characterised by the ns2 ni'
valence shell electronic configuration where x varies
from 1 to 6 and n from 2 to 6 but inner core for different
elements may be different.
• The highest oxidation state= (group number- 10)
Stability of this oxidation state (OS) decreases and
that of[(OS)- 2] state increases as we move down the
group-a case of inert pair effect. This is shown
below in Table 11.1
• Trends in properties of p-block elements have been
represented in Fig. 11.1
468 I Es�ential Inorganic Chemistry

Table 11.1 Common Oxidation States of p-Block Elements p,q,r


-------------+· ,_
x,y, t .. p,z �
Group
-------------------
13 14 15 16 17 18 B C N 0 F Ne q, r, �
Oxi d at ion
state
B C N
+4, -4 +5 to -3
O F
-1
Al Si p s Cl Ar s o'
+3 -1, -2
Ga Ge As Se Br Kr z
!ii
Al Si P,As S,Se, Cl, Br, I, Xe In Sn Sb Te I Xe
+3 +4 +3, +5, Te -1, +1, +2, +4, Tl Pb Bi Po At Rn
-3 -2,+2, +3 +5, +6,+8
+4,+6 +7
x, y, z (up to group 14)
Ga. In, Ge,Sn, Sb, Bi
Tl Pb +3,+5
where ... represents
+3,+1 +4,+2 x: covalent/van der Waals' radius
y: metallic character
z: heat of sublimation
The van Arkel-Ketelaar Triangle p : electronegativity
q: ionisation energy (N > 0, P > S)
Before looking at the compounds of the p-block r: oxidising power
elements in detail, it will be useful to consider the general s : stability of higher oxidation state within a group
types of compound we will expect and this, as we shall see, t : stability of lower oxidation state within a group
can often be rationalized on the basis of Fig. 11.1 Trends in properties of p-Block elements in the
electronegativities, X, and electronegativity differences, direction of arrow
Ax. Thus, compounds with a large t::,.x will tend to be 4.00
ionic and those with medium Ax, are likely to form
polar covalent bonds and therefore, have polymeric
or macromolecular structures. For compounds with a 3.00-
small AX, the type of compound will depend on whether the
elements have a low or high electronegativity. In the former
case, the compounds are likely to be metallic for much the a-,,2.00-
same reason that elements with low electronegativity are
metals. In the latter case, we shall expect relatively
non-polar molecular species.

DEEP Focus Cs
A useful way to visualize these ideas is with the so called 0.00
van Arkel-Ketelaar or element triangle, shown in Fig. 0.75 1.00 2.00
11.2 in which the vertices are labelled metallic, covalent Metallic covalent
and ionic in such a way that the vertical and horizontal axes Fig. 11.2 A van Arkel-Ketalaar triangle showing the
represent electronegativity, difference, ax, and absolute position of selected compounds and elements
electronegativity, :Ex, respectively and are defined such that, for
a molecule AB, LlX = X s - X A and :Ex= (xA + Xs)/2.

The metallic-covalent edge of the van Arkel-Ketelaar Group 13 (IIIA) (The Boron Family)
triangle comprises the elements themselves, since AX is This group marks the beginning of the p-b}ock
zero, and ranges from metals on the left to covalent species elements. Table 11.2 summarises the properties of this
such as F2 on the right. group, and trends shown in Fig. 11.3.
The ionic-covelent edge is characterized by decreasing
Ax, which is well illustrated with the fluorides of the DEEP Focus
elements. Thus, CsF and LiF are ionic with large Ax, Elements of group 13 are less metallic than those or
whereas NF3 and CF4 both have small llx, and are covalent; groups 1 and 2. Within the group, there is a variation in
metallic character. Boron, the smallest �lement in this
compounds with intermediate Ax, such as BeF2 are group is a non-metal. The other elements in this group are
polymeric or macromlecular solids. fairly reactive metals.
Chapter 11 : p-Block Elements - Boron Family (Group 13) I 469
The elements of group 13 have smaller atomic radii Ala+(g) + aq ---+ Al a+ ( aq) -4630
and higher electronegativities as compared to s-block 3Cl-(g) + aq ---+ 3Ci-(aq) -1155
ele-ments of the same period. Since, the size an the 3t
rJnisation energies of atoms depend on effective nuclear AIC1 3 (s)+ aq ---+ Al ( aq) + 3Cl-(aq)
1
charge of the atom, insertion of transition elements result Ill:{soln = - 296 kJ moJ-
m higher effective nuclear charge of the fourth row 3+
Born-Haber cycle for formation of Al (aq) ion
el�ments Ga, Ge, etc., than expected by simple
extrapolation from the second and third row elements. In Thus, despite the large ionisation energies of
other words, the nuclei of these fourth row elements aluminium, the enthalpy of solution of A1Cl 3 has a large
attract electrons more strongly than expected and negative value. Therefore, in aqueous solution, AICI 3
lb.is affects their properties. Thus, the atomic size of exsits as Al 3+( aq) and Cr(aq).BC1 3 hydrolyses in aqueous
solution to give boric acid and notB3+ (aq)
pllium is smaller; its electronegativity and
ionisation energies higher than expected. Atoms BC1 3 + 3:E-l.iO ---+ HaJ303 + 3HCI
lith d1° inner shell, in general, are smaller and have This group is the most metallic in nature of the p-block
higher ionisation energies. In a similar way, the inclusion elements and only boron is non-metallic.
of fourteen electrons in 4{-orbital further affects the size Boron and aluminium. show an oxidation state
and ionisation energy of Tl. As a result of this, of +3 in their compounds while gallium, indium and
irregularities in atomic radii, electronegativity and thallium also show +1 oxidation state (due to inert
ioD.isation energies are seen from B to Tl. pair effect).
Table 11.2 shows that densities of these elements • Both boron and aluminium are amphoteric.
5how a gradual increase while melting and boiling points
2B(s )+ 20H- {aq)+ 2:E-l.iO(l) ---+ 2B02 (aq}
fall in value. The exceptionally low melting point of metaborate ion
p.llium (303 K), however, has, so far, no simple + 3:E-l.i (g)
e1planation.

From Table 11.2 we observe that these elements 2Al(s) + 20H-(aq) + 2:E-l.i O(l) ---+ 2Al02 (aq)
form small ions of high charge density and the value metaaluminate ion
or the sum of their first, second and third ionisation + 3:E-l.i (g)
energies is very high. These properties lead us to the but the ions are better represented as complex ions.
conclusion that these elements will prefer to form covalent
rather than ionic compounds. Boron is always covalent [Al(OH)4 (:E-l.i0)2 f or [Al(OH)6 ] - 3

1
land does not form B3+ ions, because the energy • B and Al show +3 oxidation state. Ga, In and Tl show
!required to remove three electrons is very high. +3 and+1 oxidation state but stability order is
llany simple compounds of AI and Ga like AIC13 and Ga 3+ > In 3+ > Tl 3+
GaCl3 are also covalent when anhydrous. However, Ga + < In + <Tl +
in solution, the large amount of hydration energy due to inert-pair effect.
evolved compensates the high ionisation energy Thus, Ga + salts tend to stabilise by changing to
and all the metal ions exist in a hydrated state. Ga 3+ (Ga 3+ > Ga + ) and thus are better reducing
agents.
This can be explained with the help of Boron-Haber
qcle as given below. Ga + � Ga 3+ +2e-
lll:l(kJ) On the other hand·, Tl 3+ salts tend to stabilise by
3 changing to Tl + (Tl + > Tl 3+ ) and thus are better
A1Cl 3(s) ---+ Al(s) + - Cl2 (g) 704 oxidising agents.
2
TI 3+ + 2e- ---+ Tl +
Al(s) ---+ Al(g) 326
GaX, InX and TlX (X = Cl, Br, I) exist. GaX and InX
Cl2 (g) ---+ 3Cl(g) 366
2 disproportionate in water
Al(g) ---+ Al a+(g) + ae- 5137 3GaX(s} ---+ Ga(s)+ Ga 3+(aq)+ 3X-{aq)
3CI(g) + 3e- � 3Cl-(g) -1044 All the above reaction are spontaneous and AG 0 < 0
470 | Essential Inorganic Chemistry

Table 11.2 Physical Properties of the Elements of Group 13 (IIIA)


Property B Al Ga in Tl

Electronic structure [He] 2s2 p' [Ne] 3s23p1 [Ar] 3d'° 4s2 4p1
... 5s25p1 . 6s26p’
Electronegativity (Pauling) 2.0 1.5 1.6 1.7 I. 8
(IE),kJ mol-1 801 577 579 558 589
(/E)2kJ mol-1 2427 1816 1979 1820 1971
(/E)3 kJ mol-1 3659 2744 2962 2704 2877
m.p./°C 2180 660 30 157 303
b.p.fC 3650 2467 2403 2080 1457
Density (103 x kg m-3) 2.34 2.7 5.91 7.3 II. 8
Metallic radius/pm 88.5 143 141 166 170
Ionic radius/pm M3+ 27.0 53.5 62 80 88.5
M+ 120 140 150
Principal coordination number 3,4 3, 4,6 3.6 3.6 3.6
-1.66 -0.56 -0.38 +2.18
E°M*/M /V 0.55 -0.18 -0.34

* For Al and Ga in acidic solutions, others in basic solution.

5000-r 2.5-r high nuclear charge provide boron with non-metalhc


2-J properties.
4000--
U- • Except boron, other exist in aqueous solution in
tripositive (M3+) state. [Af ]3+,
K 3000"
IT
ro
1- [M(H2O)2(OH)4r and [M(0H)4]’ exist in aqueous
solution.
2000--
0.5-- • Stability of hydrides (MH3) decreases as we move
1000 0 down the group. A1H3 is a solid and is polymerised via
B 1 Al 'Ga' In' Tl1 B 1 Al 'Ga' In1 Tl1 Al—H—Al bridging units. Hydrides are
(a) (b) electron-deficients and thus act as Lewis acids (lone
pair acceptor) and form adducts with strong Lewis
900 -r
bases.
800-- A1H3 + H" --- > aih;
Lewis acid Lewis base adduct
kJ mol-1 700--- • They form halides of the type MX3, all of which are
known except Tl(III) iodide.
600--
- fluorides are ionic with high melting point
- chlorides, bromides and iodides are covalent with
500 B 1 Al *Ga' In1 Tl1 low melting point
(c) - halides have halogen bridged dimeric structures
Fig. 11.3 Some group trends of the group 13 elements : as shown.
(a) Boiling point, (b) Pauling electronegativity, and (c) The first
Cl <&,
ionisation energy. ><<UCI Cl
101* 7g. Al 118*
The term ‘inert pair effect’ is somewhat misleading.
The decrease stability of the higher oxidation state (+3) Cl Cl
with increasing atomic number arises because of the Cl
decrease in bond energy with size from Al to Tl. As a
result, the energy required to unpair ns2 electrons is not - MX3 are strong Lewis acids. Anhydrous A1C13 is
compensated by the energy released in forming the two used as a catalyst is Friedel-Craft’s reaction
additional bonds.
• Except boron, the other elements of group 13 (IIIA)
A1C13
show increasing metallic character from Al to Tl. + R—X + HX
Small size (atomic radius = 88.5 pm) and relatively
Chapter 11: p-Block Elements — Boron Family (Group 13) | 471

• They form oxides and hydroxides of the type Af2O3 and • Boron reacts directly with elements forming borides
Af(OH)3 and their basic nature increases as we move which are hard and refractory.
down the group. However, AL2O3 and Ga2O3 (and 3Mg + 2B---- > Mg3B2
their hydroxides) show amphoteric nature. Borides are decomposed by acids forming boranes.
2A1(OH)3 (s) +6HCl(aq)-----> 2A1013(aq) + 3^0(0 Mg3B2 + 6HC1---- > 3MgCl2 + B^
base acid diborane
A1(OH)3($) +NaOH(aq) -----> NaAl(OH)4(aq)
acid base
• They form octahedral aqua complexes of the type
DEEP Focus
(M(H2O)6]3+(M = Al,Ga,In,Tl) • Aluminium is far more reactive than boron but in air and
water it is stable. It is due to the formation of aluminium
• Aluminium sulphate forms double salt with alkali
oxide, A12O3 layer, that makes it passive for further attack.
sulphates of the type MA1(SO4)2 ■ 121^0 (M + = Na or HNO3 also makes Al passive by oxide layer formation.
KT). These are called alums.
The layer is so useful that in industry it is purposely
•’ (/E)j oi boron is relatively high and (ZS)
Vfijj of (lE^2 and (IE)
(1E)3 are increased by an electrolytic process called anodising. This
still higher. Thus, the total energy required to give B + is done by electrolysing dilute H2SO4 with Al as anode. This
ions is far more than that which would be produces much thicker layer of oxide on the surface (0.01
compensated by lattice energies of ionic compounds or mm). This layer can take up pigments; thus colouring the
hydration of such ions in solution. As a result, boron aluminium.
does not easily form a cation; however their covalent
compounds are known. • Reaction of Al with O2 is exothermic and is called
• The melting point of B is very high which suggests a thermite reaction.
strong bonding between individual atoms in solid 2Al(s) O2(g)-----> Al2O3(s), AH = -1670 kJ
state and is used as a semi-conductor.
The thermite reaction involves so much energy that it
Reactions of Elements can be dangerous. The Al becomes white hot and often
causes fires. For this precise reason, mixture of Al and
Pure crystalline boron is very unreactive. However, it
an oxide such as Fe^jj or SiO2 (a source of oxygen)
is attacked at high temperatures by strong oxidising
were used to make incendiary bombs during World
agents such as a mixture of hot concentrated H2SO4 and
HNO3 orNa202. War II. Warships are sometimes made of aluminium
alloys to reduce their weight. A thermite reaction can
2B(s) + 6HNO3(aq)-----> 2H3BO3(aq) + 6NO2(g) be started if the ship is hit by a missile. Thermite
In contrast, finely divided amorphous boron reaction also finds applications in the metallurgical
(containing about 2-5% impurities) is more reactive. extraction of other metals from their oxides :
Reactions are summarised in Table 11.3.
8A1 + 3Mn3O4 ---- > 4A12O3 + 9Mn
Table 11.3 Reactions of Group 13 (IDA) Elements 2A1 + Cr2O3 ---- > A12O3 + 2Cr
In recent years, Al has been under suspicion as a
Reaction Comment
possible course of Alzheimer's disease. This disease
• 4M + 3O2 —> 2M2O3 Reaction at high temperature; with induces senility in relatively young men and women. The
Al—exothermic, a protective oxide
layer is formed and Tl also forms disease causes them to lose their memory.
TI2O. • Most of the reactions of boron are at high
•2AI+N2 2 MN Only Al and (B) form nitrides. BN, a temperatures.
slippery white solid with layer BX3
structure similar to graphite.
• 2M+3X2—> 2MX3 All form MX3. Tl also form TIX. Iodide H3BO3 + NO2 MXBy
of Tl3* (Tll3) is not formed; but Til
x2
HNO3
reacts with l2 forming Tll3 (Tl* l3).
•2M+2NaOH + 2H2O —> B, Al and Ga only form NaMO2. N2 02
2NaMO2 + 3H2 BN BORON b2o3
•2M+6NH3 2M(NH2)3 All form amides except B which NaOH
MH3 •.base
- '2 forms BN.
+ 3H,
2B + 2NH3—> 2BN + 3H.'2 S
■’2M+6H*—> 2A43* + 3H2 bn + h2 Na3BO3 + H2
Metals liberate H2, but HNO3 makes
Al passive. b2s3
472 | Essential Inorganic Chemistry

Differences between Boron and the other Boron: Extraction


Elements Amorphous boron (called Moisasan boron) is obtained
Boron because of its small size has high ionisation by reduction of B2O3 with Na or Mg at a high temperature.
B.2O3 in turn is obtained by decomposing borax with acid
energy and electronegativity. It is always covalent and
and subsequent heating.
non-metallic. It differs other elements of its group in the
A KjO + BA
following respects : Na2B4O7 1011,0 —■> H3BO3
orthoboric acid
Boron Other elements A * 2B + 3Na2O
B2O3 + 6Na
• B,O3 is an acidic oxide (like SiO2) • AI2O3 is amphoteric
A, 2B + 3MgO
B2O3 + 3Mg -
• H3BO3 is acidic • AI(OH)3 is amphoteric
Boron obtained is 95-98% pure and black in colour.
• Simple borate and silicate ions can • Aluminates are formed but Pure crystalline boron is obtained by
polymerise forming isopolyacids by do not polymerise
sharing oxygen atoms
• reducing BC13 with
red hot W or Ta
• Boranes are gaseous compounds • Aluminium hydride (AIH3) n 2BC13 + 3H2 ------------------ > 2B + 6HC1
readily hydrolysed and is a polymeric solid
filament
spontaneously inflammable • pyrolysis of BI3
• Halides are easily hydrolysed • Halides are easily red hot W or Ta filament
hydrolysed 2BI3 --------------------------- -> 2B + 3Ij
(van Arkel method)
• thermal decomposition of I^Hg
A
BjHs 2B + 3H2

Target Practice 1
1. A and B with valence electrons are represented below
:a* • X
(A) ■*—Al —
What can be possible formula of the compound of A and B? Vessel

’HNO3
2. Write structure of ALCL.
Z 0

3. Do as directed (B) Glass-*-


Vessel
(a) Increasing order of stability of M3+ of Group 13 in 4th,
5th and 6th period.

(b) Hydrolysis reaction of BC13. Comment

(c) Intermediates formed when A1C13 reacts with CH3C1.


5. Disease due to Al metal is
This diseases causes
(d) Non-metal of group 13 6. Write the nature of the following
(e) T1C13 is unstable because (a) B2O3
(b) A12O3
4. NaOH and HNO3 are being manufactured in industries A (c) A1(OH)3
and B and stored as shown (d) B(OH)3
Chapter 11: p-Block Elements — Boron Family (Group 13) | 473

Answers
1. BasedI on number of valence electrons, A is more 4. Storage method in industry A is not correct for NaOH since,
electronegative than B. Thus, BA NaOH reacts with Al.
Cl\ /Clx /Cl 2NaOH + 2Al ---- > 2NaA102 + 3H2 + 2H2O
1 -Al Al'
Cl' However, HNO3 can be stored in Al vessel since, with HNO3,
Cl Cl
there is formation of oxide layer which makes Al passive for
3. (a)Tl3+<In3+<Ga3+ further reaction. Storage method in industry B is correct.
(b) BCl3+3H2O > H3BO3+3HC1 5. Alzheimer; it induces senility and causes loss of memory.
(c) A1C1,v + CH,C1 ---- > A1C1?O + CH®o 6. (a) B2O3 : Acidic
J
carbocation (b) A12O3 : Amphoteric
(d) B (c) A1(OH)3 : Amphoteric
(e) TIClj change to T1C1 due to greater stability of Tl+. (d) B(OH)3 : Acidic
Tl3+ + 2e~ ---- > Tl +

Practice Exercise 1
1. Third ionisation energy, (JEh of group 13 (IIIA) is generally (O (white ppt) dissolves in NaOH solution while (D) (gas)
high. Explain. gives white fumes in HC1. (A) has atomic number 13.
2. Boron shows amphoteric nature. Illustrate with chemical 7. Al is amphoteric, give chemical reactions to illustrate it.
reactions. 8. The following reaction is exothermic :
3. T1C13 is unstable while T1C1 is stable. Explain. 2Al(s) + |O2(^) -----> A12O3(s), AH = -1670 kJ
4. Al3+ ion is hydrated and exists as [A1(H2O)6]3+. It is said to
behave as an acid. Explain by chemical equation. What is this reaction called? Give two main applications.
5. X and Y have respectively 3 and 5 electrons in the valence 9. HN03 can be stored in Al vessel but NaOH can't be.
shell. Write the formula of the compound formed from X and Explain.
Y. 10. Why can the thermite reaction not be stopped by water like
6. Identify (A), (B), (C) and (P): an ordinary fire?
^>(C) + (D)
Metal (A) + N2 (B)

Answers
1. Due to increase in effective nuclear charge; this increases force A1(OH)3 + NaOH NaA102 +2H2O
of attractions hence IE increases; in general white ppt soluble
(IE)3 >(IE)2 >(IE)’i1 NH3+HC1---- > NH4C1
2. 2B + 2NaOH + 2H2O---- > 2NaBO2 + 3H2 white fumes
meta borate 7. Refer Q. (2)
2B + 6HC1---- > BC13 + 3H2 8. This is called thermite reaction. It is used (i) to reduce oxides
B reacts with NaOH (base) as well as with HC1 (acid) hence B is into metals:
amphoteric. Cr2O3 + 2A1---- > A12O3 + 2Cr
3. Due to ‘inert pair' effect, stability of higher oxidation state Fe2O3 + 2A1---- ■» A12O3 + 2Fe
decreases and that of lower oxidation state increases. Hence (ii) in welding.
Tl3+ < TT (stable) 9. Al turns passive withHNO3 due to formation of protective oxide
4. [A1(H2O)8]3+ +H2O ---- > [Al(H2O)5(0H)]2+ +h3o+ layer hence, reaction ceases. This makes Al-vessel suitable for
acid base base acid storing HNO3. Al dissolves in NaOH forming NaA102 hence it
5. XY can’t be stored in Al-vessel:
6. (A):Al (B): AIN (C): A1(OH)3 (D): NH3 2A1 + 2NaOH + 2H2O---- > 2NaA102 + 3H2
10. In an ordinary fire the reactions are kept going by the material
2A1+N2 ---- > 2A1N reacting with oxygen in the air. Water not only cools the fire,
(A) (B)
but also keeps oxygen out. In the thermite reaction, the oxygen
A1N + 3H2O---- > A1(OH)3 +nh3 involved is in the metal oxide, so keeping atmospheric oxygen
(O ID)
out has no effect. Also high temperature (about 1000°C) will
allow reaction between Al and H2O to give off H2.
474 | Essential Inorganic Chemistry

F
Compounds of Boron F
II
F
Boron is the first member of group 13 (IIIA) of the B B B,
Periodic Table and is the only non-metal of this group, F^ F F^ F F F
having three valence electrons. Because of its small size,
high ionisation energy and low electronegativity, boron • The empty 2pz orbitals on the boron atom in BF3 can
forms covalent compounds and in this respect resembles also be filled by a lone pair of electrons from donor
carbon and silicon. Since, compounds formed are molecules such as ether (E^O), NH3, (Me)3N orF and
electron-deficients, they behave as Lewis acids tetrahedral molecule or ion is formed.
(electron-pair acceptors) fXx,nh3
F\^^xOEt2
B B
Boron Halides F F F X ^F
• Boron halides are covalent. It is due to the extremely
small size ofB3+ ion (27 pm) and can polarise the anion R^NMe3 F^F
easily (smaller the size of cation, larger the size of B B
anion, greater the charge => greater the polarising F F F F
power hence, greater the covalent nature —Fajan's
rule) • Boron also forms halides of the formula B2X4. These
- BF3 is a gas decompose slowly at room temperature. BgC^ can be
made as follows :
- BC13 is a liquid
electric discharge
- BI3 is a solid 2BC13 + 2Hg ■> B2C14 + Hg2Cl2
low pressure
• When borates are treated with HF (or CaF2 and
concentrated H2SO4), the volatile compound BF3 is There is free rotation about the B—B bond, and in the
formed. If the BF3 gas produced is introduced into a gaseous and liquid states the molecule adopts a
bunsen flame, a characteristic green colouration is non-eclipsed conformation. In the solid state the
formed : molecule is planar, because of crystal forces and ease
of packing.
CaF2 +H2SO4 * CaSO4 + 2HF
Cone Ck .Cl CL Cl
>B—BC B—B<
3HF + HgBOg ■» BF3 + SHjO CK ^Cl Cl Cl
non-eclipsed planar
This can be used to test borates.
• BX3 is electron-deficient (octet of B incomplete) and • BF3 hydrolyses incompletely and forms fluoroborates.
thus behaves as a Lewis acid. The shape of the BF3 This is because the HF first formed reacts with the
H3BO3 :
molecule is a planar triangle with bond angles of 120°.
4BF3 + 12H2O---- > 4H3BO3 + 12HF
• The bond lengths in BF3 are 1.30A and are shorter
than the sum of the covalent radii (B = 0.80 A and 12HF + 3H3BO3 -» 3H+ +3[BF4]~ + 91^0
F = 0.72A). The bond energy is very high (646 kJ mol-1) Net: 4BF3 + SHjO -> H3BO3 + 3H+ +3[BF4r
and is higher than for any single bond. The shortness
and strength of (B—X) bond is interpreted in terms of The other halides hydrolyse completely, giving boric
a pn-pit interaction. The empty 2pz atomic orbital on acid.
B (which is not involved in hybridisation) is BC13 + 3H>0 » H3BO3+3HC1
perpendicular to the triangle containing the sp2
hybrid orbitals. This 2pz orbital may accept an CaF2 + H2SO4 + H3BO3 (or B2O3)
electron pair from a full pz orbital on any one of the A
three fluorine atoms. Thus a dative bond is formed H2O
and the B atom attains an octet of electrons. If one bf3 H3BO3 + BF4

localised double bond existed, there would be one


180°C NaH
short bond and two longer ones. But all B—F bonds
are equivalent and based on modem B2H6
explanation, the double bond is delocalised.
Chapter 11 : p-Block Elements - Boron Family (Group 13) I 475
Boron Hydrides • All the boranes act as Lewis acids and can accept
electron pairs. Thus they react with amines, forming
• The boron hydrides are sometimes called boranes by simple adducts. They also react with ammonia to give
analogy with the alkanes (hydrocarbons). variety of products depending on experimental
• There are two series : conditions:
- BnHn+t called nido-boranes, (8iH6) :RiH6 + 2MeaN ----+ 2[MeaN · BH3 )
- BnHn +s called arachno-boranes, (less stable), BiH6 + 2CO ----+ 2[BH3 • CO]
(B4 H10); A complex reaction occurs when BiH 6 is heated in a
- of all these, diborane is the simplest borane. sealed tubes. The BiH6 molecule probably breaks into the
reactive intermediate BH3 which combines with BiH6
Source forming another intermediate Balf 9 which can how further
reactions:
MgaBz
· + HJ>04 ----+ mixture of boranes --!.+ BiHs :RiH 6 ----+ 2BH3
(mainly B4H 10) diborane
BH3 + :RiH 6 ----+ B:i!f 9
150 C °
BzOa + 3J:r.
... "'2 + 2Al -n H6 + AI 2O3
.1.-'2 B3"9 ----+ Balf 1 + "2
750 atm
4B F3 + 3LiAIH ----+ 28.iH 6 + 2LiF + 3AIF3 BjI 7 + BH3 ----+ B4 H10
4
BiH6 splits into two symmetrical fragments (BH3 ) in
(industrial method)
presence of Lewis acid (L) to give borane-adducts BH3 • L.
180 °C
2BF3 + 6NaH BiH6 + 6NaF This is called symmetric or homolytic cleavage.
gas (industrial method) H H
2NaBH4 + � ---
ether
H)a< '>a( + 2L --+ 2L-B�:
H H'' H H
Properties
• Diborane is a colourless gas, highly reactive
• excess NH3
low temperature
(b.p. 180 K). It catches fire spontaneously in air and excess NHa
explodes with oxygen . (BN).r
high temperature boron nitr
ide
8iH6 + 3 02 ----+ 8i0 3 + 3"20, ratio: 2NH 3:IB.i H 6
BµJ-Is
Ml 0 = -2165 kJ mol-1 high temperature borazine
• It is hydrolysed by water : :BiH6 • 2NH3 is ionic and forms borazine on heating:
B.iH6 + 6"20 ---it 2H3B0 3 + 3"2
boric acid B2H6·2NH3 � [H3N ---+ BH2 � NH3r + [BH4r
• In alkaline solution, hypoborates are formed.

B4H10 + 4KOH ---+ 4KOBH3 + �


hypoborates

red heat
BiHs 2B + 3"2
• 8.iH6 + 2LiH ----+ 2Li[BH4]
28.iH6 + 2Na ----+ Na[BH4] + Na[Baff8]
Both LiBH4 and N aBH4 are used as reducing
agents in organic synthesis.
B.iH6 + HCI ----+ Bi�Cl + "2
Bi H6 + 3Cl2 ----+ 2BC1 3 + 6HC1

DEEP Focus
• B2H 6 selectively changes alkenes to alcohol with alkaline
H202 against Markownikoff s rule
(i) B.,Hs
CH 3CH=CH2 ) CH3CH2CH OH
(ii) H 20 2, OH- 2
476 | Essential Inorganic Chemistry

DEEP Focus enough electrons to fill all the available orbitals to form eight
normal—two-centre two-electron (2c-2e) or electron pair bonds.
Borazine, B3N3H6 (I) is called ‘inorganic benzene' because
The two bridging hydrogen atoms lie symmetrically above and
its structure shows some formal similarity with benzene, with below this plane (3c - 2e bonding). This has actually
delocalised electrons (II) and aromatic character (III).
multicentre bonds or odd-electron bonds and B—H bond
• Boron nitride is a white slippery solid. One B atom and one lengths (of terminal sides) is less than that of central B—H
N atom together have the same number of valence electrons bonds.
as two C atoms. Thus boron nitride has almost the same
structure as graphite, with sheets made up of hexagonal Ho H H
o
rings of alternate B and N atoms joined together (inorganic o
graphite). B B
o
I H° H
o
H
/C
N N N N C C C • Boron electron

B
I I I o Hydrogen electron
B
(Bridging groups are electron deficient).
I I I I l I 2c-2e B—H terminal bond : B—H
B /B ,B% %
N N c
C c H

boron-nitride graphite 3c - 2e B—H—B bridging bond : B •B


Mg3B2
• In borane, boron atom is sp2 hybridised with bond
B2O3. h3po4 NaBH4
angle of 120° and having triangular planar structure.
H2 + AI A
150°C^ / l2/ether
ttfi 2/ B2H5CI
750 atm
HCI
b2h6
NaH, 180°C
-BF3
or LiAIH4
B (ground state) O2
2s2
k\jJH
B2O3' h2o Na BCIj
Is2
nh3 LiBH4
H3BO3 NaBH4
/ h2o2/oh-
A
'V rch2ch2oh
B3N3H6 . B2H6-2NH3
(borazine)
B (excited state)
Is2 sp
2
[BH2(NH3)2]+ + [BHJ-

Similarity between Boron and Silicon


sp2 sp2 sp2 t (Diagonal Relationship)
• Both form volatile hydrides (called boranes and
they form o-bonds with silanes) which spontaneously catch fire on exposure to
H-atoms by (sp2—s) overlapping air and are easily hydrolysed.
• The molecular shapes of diborane is shown below : • Halides of both (except BF3) are easily hydrolysed:
BC13 + 3H2O-----> H3BO3 + 3HC1
H H
SiCl4 + 4HgO-----> H4SiO4 + 4HC1
• Boron forms binary compounds with several
metals—these are called borides. Silicon form
silicides. Borides and silicides are hydrolysed by H2O
forming boranes and silanes :
DEEP Focus
Mg3B2 + 6H2O-----> 3Mg(OH)2 + B,H6
The two boron atoms and the four terminal hydrogen
atoms lie in one plane; the two bridging hydrogen atoms Al4Si3 + 12^0-----> 4A1(OH)3 + 3SiH4
(encircled) lie symmetrically above and below the plane. There
are total 12 valence electrons—6 in two B and 6 in six H atoms • B2O3 and SiO2 are acidic in nature and are important
and they form in all eight B—H bonds. Evidently, there are not constituents of glass. Borate and silicates have
Chapter 11: p-Block Elements — Boron Family (Group 13) | 477

tetrahedral BO4 and SiO4 structural units, • Borax dissolves in water to give alkaline solutions :
respectively. Na2B4O7 + 2H2O-----> 2NaOH +
• Both are semi-conductors. Borax is therefore, used as a water softner and
’ • Boric acid (H3BO3) and silicic acids (H4SiO4) are weak cleansing agent.
acids.
DEEP Focus
• Both exist in two allotropic forms—amorphous and
• One mole of borax reacts with two moles of HC1 (or one mole
crystalline. HoS0,). This is because when borax is dissolved in water
• They do not dissolve in cold dilute acids but do so in both B(OH)3 and [B(OH)4]"are formed, but only the
alkalies—boron only in fused alkalies and silicon in [B(OH)4J" reacts with HC1. Each (B(OH)4J" has one
both fused and aqueous alkalies. neutralisable OH" ion.
[B4O5(OH)4]2" +5H2O 2B(OH)3+2[B(OH)J-
2B + 2NaOH + 2^0-----> 2NaBO2 + 3H2
2[B(OH).f+2H,O+ -> 2B(OH)_u + 4H„O
Si + 2NaOH +H20-----> Na2SiO3 + 21^ Z

• Borax on strongly heating forms B.2O3 which form


Borax :(Na2B4O7 10H2O) coloured glassy bead with coloured oxide (generally of
transition metals). It is called borax bead test.
Source , a,740°Cs
Na2B4O'7 * 2NaBO2 + BgO3
• Tincal {suhagd) occurs naturally and contains about (transparent bead)
45% borax. Tincal is dissolved in water and after
removing impurities (by filtration), aqueous solution BjOgte) + CoO(s) ■> Co(BO2)2
blue bead
is evaporated to get crystals of borax.
B^ls) + CuO(s) ■> Cu(BO2)2
• Orthoboric acid, H3BO'33 on neutralisation with red bead
Na2CO3 gives borax.
Na2CO3 + 4H3BO3 -----> Na2B4O7 + 61^0 + C02 DEEP Focus
• Colemanite, Ca2B6On is converted into borax by • Borax is used as a primary standard in titrations against
acids.
boiling it with concentrated solution of Na2CO3.
Na2B4O7 10H2O + 2HC1---- > 2NaCl + 4H3BO3
Ca2B6On + 2Na2CO3 -----> Na2B4O7 + 2CaCO3 + 5H2O
borax
One of the products H3BO3 is itself a weak acid. Thus, the
+2NaBO2 indicator used to detect end point of this reaction must be one
sodium metaborate that is unaffected by H3BO3. Methyl orange (pH range
(in mother liquor)
3.1-4.4) is normally used.
C02 is passed into mother liquor containing NaBO2 • Borax is actually made of two tetrahedra and two
when borax is formed. triangular units joined as shown and should be written as
4NaBO2 + C02 * Na2B4O7 + Na2CO3 Na2[B4O5(OH)4]-8H2O.
OH
Properties

• Boraxis 0 \ 0
- prismatic, Na2B4O7 • 10H2O obtained by crystallising HO—B< .0 B—OH
0 />0
salt at ordinary temperature.
~ octahedral, Na2B4O7 • SHjO obtained by fOH9
crystallising salt above 370 K. It is also called
jeweller's borax.
• It is used as a flux in welding and brazing metals for
~ borax glass, Na2B4O7, obtained by heating dissolving any oxide which may form on the surfaces
ordinary borax above its melting point. It is
to be joined; in the manufacture of high quality heat
colourless glassy substance which absorbs
resistant glass (borosil-mark) Jena and pyrex glasses
moisture from air and is converted into
contain boric oxide.
Na2B4O7 • lOI^O.
478 | Essential Inorganic Chemistry

hot
Na2CO3 DEEP Focus
Ca2B60n NaBO2
digest • Ortho boric acid, H3BO3 is soluble in water and behaves as a
weak monobasic acid. It does not donate protons like most
H3BO3 + NaOH ^0 C02 acids, but rather it accepts OH". It is therefore a Lewis and,
HCI and is better written as B(OH),.

z
NaCI + H3BO3 Na2B4O7
B(OH)3 + 2H„0
B(0H), H„0+ + [B(OH).F; pK0 =9.25
A ^<0/
NaBO2 + B2O3 A (BN) called inorganic
[H OH
I CuO, A graphite
Cu(BO2)2
blue H OH
SO2 Hu 'OH
plane triangle tetrahedral metaborate ion
H2O 1OO°C red hot • Polymeric metaborate species are formed at higher
H3O* + B(OH)4 H3BO3 HBO2 B2O3
A concentration, for example :
HF ,0^ 3B(OH)3 H30++ [B3O3(OH)4 ]“ + HjO
H3PO4
G^ pKa =6.84
bf3 bpo4 I"HO
OH
B—0
NaH O B
150°C, 75 atm
B—0
OH
B2He
Al, H2
Lho
• Orthoboric acid contains triangular BO33"units. In the
solid the B(OH)3 units are hydrogen bonded together
Boric acids and oxides into two-dimensional sheets with almost hexagonal
(i) HgBOg or B(OH)3 orthoboric acid. symmetry (Fig. 11.4). The layers are quite a larger
(ii) HBO2 or BO(OH) metaboric acid. distance apart (3.18A) and thus the crystal breaks
quite easily into very fine particles.
(iii) H^Oy pyroboric acid
Boric acid is the trivial name for ortho boric acid. In
aqueous solution meta and pyroboric acid are changed
into ortho boric acid.

Source
• Na2B4O7 + HjSQ, + SHjO-----> Na2SO4 + 4H3BO3
HgBOa HB02 + HjO

• On a commercial scale, boric acid is prepared from


colemanite, Ca2B6On -51^0. Mineral (colemanite) is
powdered finely and S02 is passed into hot aqueous
suspension. On cooling crystals of boric acid are
obtained.
Ca2B6On + 91^0 + 2SO2 -----> 2CaSO3 + 6H3BO 3
H
Excess ofSO2 dissolves CaSO3 asCa(HSO3)2 • Boron
CaSO3 + H20 + S02 ----- > Ca(HSO3)2 o Oxygen
insoluble soluble Fig. 11.4 Hydrogen bonded structure of orthoboric acid

Properties • It is used in the manufacture of enamels and glazes for


• Both form BgO;} (boron sesquioxide — sesqui means pottery and as disinfectant in medicine.
one and a half, so the oxide should have BO j orl^Og)
1—
2 Boron trioxide (B2O3)
• This (BgOg) is obtained when boron is burnt in oxygen
H-jBOg hbo2 21^ H2B4O7 B2O3
4B+ 3O2 -----> 2B2O3
Chapter 11: p-Block Elements — Boron Family (Group 13) | 479

• It is also obtained as a colourless glass when boric acid also unaffected by acids and is used as refractory to
is heated to redness line furnaces.
2H3BO3 ---- > B2O3 + 3H2O • A1(OH)3^P» Y-AJA
• B.2O3 is a white non-volatile glassy solid.
y-Al2O3 dissolves in acids, absorbs moisture and is
• It is an acidic oxide giving rise to boric acid, when used in chromatography.
dissolved in water
• Alumina is white in colour and addition of Ci2O3 or
B2O3 + 3H2O 2H3BO3 Fe2O3 makes it coloured.
• Boron trioxide reacts with various metallic salts at
high temperatures giving characteristic coloured DEEP Focus
beads. This is called borax-bead test of the • A1(OH)3 is amphoteric.
identification of coloured salts
(a) Al(OH), +3H* ---- > Al3* + 3H..0
CuSO4 +B2O3 -A> Cu(BO2)2 +SO3 acid base
blue Al3* exists as hydrated [(A1(H2O)6J3* ion.
• B2O3 also shows weakly basic properties as indicated (b) A1(OH)3 +NaOH---- > Na [A1(OH)41
by its reaction with ortho phosphoric acid base acid or NaA102-2H20
B2O3 + 2H3PO4 ---- > 2BPO4 + 31^0 sodium meta aluminate
In (a) it behaves as a base while in (b) it behaves as an
acid—thus amphoteric. However, Al3* is precipitated as
Compounds of Aluminium A1(OH)3 on passing C02, showing that the acidic properties are
very weak.
Aluminium Halides CO, + H,0---- > H,CO,
IL L> O
CO_2-+ 2H*
•>

A1X3(X = Cl, Br) exists as dimer, thus attaining an 2A13* + 3C0 2~ + 3H,0---- > 2A1(OH) + 3C0„
□ Z M «■

octet of electrons. The dimer formula is retained when the


halides dissolve in non-polar solvents (like ether, • Structure of aluminate ion changes with both pH and
benzene). concentration:
- between pH 8 and 12, the ions polymerise using
C1>A1<C1>AT Cl OH bridges and each Al is octahedrally
cr Cl coordinated.
- in dilute solution above pH 13, a tetrahedral
A1(OH)4 exists.
DEEP Focus
- in concentrated solutions above 1.5 M and pH
However, when the halides dissolve in H^O, the high
greater than 13, it exists a dimer
enthalpy of hydration is sufficient to break the covalent [(OH)3A1—0—Al(OH)3f".
dimer into [ M(£^O)S]3* and3X” ions.
Aqueous solution of A1C13 is acidic due to hydrolysis
A1CL + 3H,0---- > Al(OH), + 3HC1
OX V
Alums
A1C13, being electron deficient, finds use as Lewis acid An aqueous solution containing equimolar amounts of
catalyst in Friedel-Craft’s reactions of aromatic class. A12(SO4)3 andl^SQi crystallises as potassium aluminium
sulphate, KA1(SO4)2 121-1,0. This is generally called
alums with general formula
Aluminium Oxide and Hydroxide Af(I)M'(HD (SO4)2-12H2O
Source where Af(I) is almost any unipositive cation [except Li+,
• Alumina, A12O3 can be made by dehydrating A1(OH)3, because this ion is too small to meet the structural
or from elements. A12O3 is requirements of the crystal) and M'(III) is a tripositive
cation— Al3+, Ti3+, V3+, Cr3+, Fe3+, Mn3+, Co3+,
a-Al2O3 or corundum Ga3+, In3+, Re3+,Ir3+]. Alums contain the ions
y-Al2O3 [M'(HaOjg]3* and SO; ’in the ratio 1:1:2.
y-Al2O3 ^’100Q°- * a-Al2O3 Most common alums have Af(I) = K+ or NH4+ and
Mz(III) = A13+.
Properties Alums are used as mordant in dyeing. The fabric to be
dyed is dipped in a solution of the alum and heated with
• Corundum is extremely hard and is used as ‘jewellers
steam. Hydrolysis of [Ald^Olg ]3+ deposits A1(OH)3 into
rouge' to polish glass. Its impure form containing
FejOa and silica is called emery and is used to make the fibres of the material and the dye is adsorbed on the
A1(OH)3.
emery paper (sand paper) used to polish metals. It is
480 | Essential Inorganic Chemistry

Cement tiles and pottery. Another use of borax is to make


peroxyborates, e.g., NaBO2 H2O2 ■ 3H2O, which are
• Ca3Al2O6 (tricalcium aluminate) is important
useful cleaning and bleaching agents. In the
constituent of Portland and high alumina cements.
laboratory, borax is used for standardising acids and in
• General formula is Ca9[Al6O18] because it contains the borax bead test in qualitative analysis.
12-membered rings of Si—0—Si—0 made by joining
Aluminium exhibits useful properties of low density,
six A1O4 tetrahedra.
high thermal and electrical conductivity, good corrosion
• Portland cement is made by heating mixture of resistance with non-toxic nature of the metal and its
CaCO3, SiO2 and clay (aluminosilicate) at a compounds. Due to these properties, it is the most widely
temperature of 1450 - 1600° C in a rotary kiln. When used non-ferrous metal. Aluminium is used for making
mixed with sand and water, Portland cement sets to electrical conductors, cooking utensils and wrapping
give a concrete, a hard whitish solid (with appearance materials. Aluminium is extensively used for structural
similar to Portland cement). About 2-5% of gypsum purposes, either alone or alloyed, in aircraft, ship and
[CaSO4 ^HoO] is added to slow down the setting building industries. Large amounts of aluminium are
process to increase strength. converted into alloys, such as, duralumin and magnalium
Portland cement has: containing a few per cent of copper or magnesium. These
CaO 70% Fe9O3 3% alloys are harder and stronger than pure aluminium but
SiO2 20% CaSO4 -2H2O 2% possess almost the same properties of lightness and
A12O3 5% corrosion resistance which can be further increased by
• High alumina cement is made by fusing limestone and anodising process. Aluminium beryllium alloys are harder
bauxite with small amount of SiO2 and TiO2 at and lighter than other alloys of aluminium and are
1400-1500°C. It is better than Portland cement in extensively used in space-craft. However, toxic nature of
terms of: beryllium makes their handling difficult. Due to its strong
- it sets much quicker, affinity for oxygen, aluminium is a good reducing agent
- it develops high strength within one day, and is used in aluminothermic process for extraction of
- it has good resistance to sea water and dilute metals and in welding. Suspension of finely powdered
mineral acids. aluminium in oil is used as paint.
Charge Anhydrous A1C13 is an important catalyst, used in
1 organic synthesis and in the cracking of petroleum.
IL
A12(SO4)3 as such or as potash alum,
K2SO4 • A12(SO4)3 -24H2O is used for sizing paper, for
Rotary furnace tanning leather, for waterproofing cloth and as a mordant
-Gas
Clinker for dying cotton. It is extensively used for purification of
i i _ i _ r~T
IIIjT?
water and in sewage treatment.
Because of its extreme hardness, high melting point,
non-volatility, chemical inertness and good electrical
It has insulating properties, corundum (A^Og) finds many
CaO 40% SiO2 10% applications in abrasives, refractories and
A12O3 40% Fe^ 10%
ceramics. Large crystals of a-Al2O3 when coloured with
metal-ion impurities are prized as gemstones, e.g., ruby
DEEP Focus (Cr3+, red), sapphire (Fe2+/3+Tl4+, blue), oriental emerald
• When mixed with water, the setting of cement takes place. (Cr3+/V3+, green), oriental topaz (Fe3+, yellow), etc.
Chemically, it is hydration of the molecules of constituents
and their rearrangement. The adhesion of other particles to Aluminates are imporant constituents of Portland cement
each other and to the embedded aggregates is responsible Gallium has the longest liquid range (303-2343 K) of any
for the strength of the cement which is due, ultimately, to known substance and so finds use as a high temperature
the formation of Si—0—Si—0 bonds. thermometer liquid.
Some Gemsones Based on A12O3
Uses
Boron is used to increase the hardness of steels. Gem Impurity
Crystalline boron is used in transistors. Boron is a good White sapphire none
neutron absorber and is used as shields and control rods in Fe,Ti
Blue sapphire
nuclear reactors. Boranes are used as high energy
Green sapphire Co
fuels, for example, in rockets. Boric acid is used as an
Yellow sapphire Ni, Mg
antiseptic. Borax is used to make heat resistant
Star sapphire Ti
borosilicate glass. It is also used for vitreous
Ruby Cr
enamelling of baths, domestic appliances and for glazing
Chapter 11: p-Block Elements — Boron Family (Group 13) | 481

Target Practice 2
1. Boron finds application as semi-conductor. How is pure 9. (a) TIF dissolves in H2O forming
crystalline boron obtained? (b) T1F3 hydrolyses in H2O forming
(a)
10. Chemist ‘JC tried the synthesis of 1-propanol by following
(b) method but each time yield of 1-propanol was very low
(c)
H_O*
3’
2. Borax is made of two tetrahedra and two triangular units. CH,CH=CH, ——> CH-CH-CH-OH
Write this structure
(a) Suggest a suitable device to get better yield of
1-propanol.

3. How is Cu2+ identified by borax bead test?


(b) What was the main product of the method used by ‘ JC ?

4. Aqueous solution containing one mole of borax is treated


with H-SO..
Z 4
Write reaction 11. Write two resonance structures of inorganic benzene
(i)
Thus, H-SO
4
. used is
<1 (ii)
5. Write the names of following Ijjoric acids? 12. Write the common names of hybrides of
(a) HJBO- (a) Si (b) B (c) C
(b) HBO2 13. A1CLO dimerises to Al,CL.
2 O
L
This dimer exists as
(0 W7......... in ether and as in water.
(a) change to (b) by .................. 14. What is the role of A1CL3 in Friedel-Craft’s reaction?
(b) changes to (c) by
6. Fill in the blanks
B(OH)3 is acceptor. Thus, it is a 15. A12O3 dissolves in NaOH as well as in HC1. This property is
7. Write the structure of the species formed when H3BO3 called
reacts with H2O.
16. Write the percentage of each component in Portland
cement
8. Compare covalent nature of
(a) B2F4 TIF 17. Explain “Setting of Cement”.
(b) TIF,w TIF

Answers
1. Refer text. 10. (a) Oxidation ofCH3CH=CH2 by B2H6THF/H,O 2. OH"
2. Refer text. gives 1-propanol.
3. Refer text. (i) B,H6 THF
ch3ch=ch2 ----- —----- > CH33'CH2CH2OH
4. Na2B4O7+2H2O 2NaOH+ H2B4O7 (ii) H2O,,OH-
1 mol 2 mol weak acid
(b) CH3CHCH3
2NaOH + H2SO4 ---- > Na2SO4+2H2O
Thus,H2SO4 used = 1 mol OH
5. (a) Orthoboric acid, (b) Metaboric acid, 11. Refer text.
(c) Pyrobaric acid. 12. (a) Silane, (b) Borane, (c) Alkane.
h2b4o7 +h2o b 3H3BO3 13. Al2Cl6,[Al(H2O)6]3t and3Cl".
14. As Lewis acid, it helps in the formation of electrophile, without
H3BO3 100°Q HBO, 140°C -
H2-B4O7
(a) (b) (c) which electrophilic substitution at benzene nucleus does not
take place.
6. B(OH)3 is OH" acceptor. Thus, it is a Lewis acid.
CH3C1 + A1C13 CH3 + A1C1®
7. H3B03+2H,0 H3O++ [B(OH)4F
electrophile
8. (a) B2F4 > TIF (b) T1F3 > TIF
15. Amphoteric nature.
9. (a)TlF + H2O ---- > [Tl(H2O)rF"
lonic nature 16. Refer text.
17. Refer text.
(b) T1F3 + 3H2O > T1(OH)3+3HF
Covalent nature
482 | Essential Inorganic Chemistry

Practice Exercise 2
1. The end-point of a borax-acid titration is at about pH = 4. 9. BC1 3 or BI3 can be converted into pure B. Write chemical
What indicator would you use in the titration? reactions.
----- —> (B) 10. BF3 finds use as Lewis acid catalyst in Friedel-Crafts
2 4^7 + cone. H„SO
2. Na„B.O, 2 4 ----->(A)
(ii) ignite
reaction. Give example.
(B) is identified by the characteristic colour of the flame.
11. Identify (A), and (B) in the following:
Identify (A) and (B).
ch2
3. You should know that the oxides of non-metals are acidic and (i) B2H6
(A)
the oxides of the metals are basic. Would you expect boron to (ii) H2O2, OH"
be more acidic or more basic than aluminium in its
reactions? H3O+
(B)
4. Identify (A), (B) and (C).
12. BCN, BNO2 and BCHO can be reduced by B2H6. What are
H,0
Na„B.O-
2 4 7
+ NaOH ---- » (A) “2U (B) (C) the products due to reduction?

5. Borates are identified by “Flame Test”. Give the chemistry CH3


of this test. A1C13
13. + CH3CI
6. What happens when glycol reacts with orthoboric acid?
7. BqO3 is an acidic oxide. Give examples.
What is the function of A1C1, in this reaction ? Also name
8. Identify (A), (B) and (C). the reaction.
A
Na2B4O7 (A)+(B) 14. Aluminium salts turn blue litmus red. Explain.
15. A mixture contains ZnO and ALO,. Give a scheme to
(A) + NaOH -----> (B)
separate them.
— h2so4
(A) + HF * (C)

Answers
1. Methyl orange. H3BO3 + 2H2O---- > [B(OH)J‘ + H3O+
2. (A): H3BO3 (B): B(OC2H5 )3 : gives green-edged flame. /Ok
3. Being nearer the top of the group, we would expect boron to H2C /OH
[B(OH)J- + glycol ;B<
show more non-metallic nature than aluminium; so it should be h2c XOH
more acidic in nature than aluminium. 0
4. (A): NaBO2 (B):HBO2 (C):H3BO3 7. B2O3 + 6HC1---- > 2BC13 + 3H2O
Na2B4O7 +2NaOH---- > 4NaBO2 +H2O B2O3 + 2NaOH---- > 2NaBO2 + H2O
(A)
8. (A):B2O3 (B):NaBO2 (C):BF3
NaBO, + HC1 HBO2 + NaCl
IB) 9. 2BC13 + 3H2 redhotV( 2B + 6HC1
HBO2 + H2O---- > H3BO3 or B(OH)3 red hot W
(O 2BI3 ------------ > 2B + 3I2

5. Na2B4O7+H2SO4 Na2SO4 + 4H3BO3 + 5H2O ch3


borax
H3BO3+3C2H5OH B(OC2H5)3 + 3H2O
tnethyl borate
10. OJ +ch3ci bf3
o + HC1

Triethyl borate burns with green-edged flame when ignited. CH20H ch3
6. Crs-di-ol form very stable complex with [B(OH)4]“ which is 11. (A): (B): OH
formed when H3BO3 reacts with H2O :
Chapter 11: p-Block Elements — Boron Family (Group 13) | 483

12. BCN —> «CH2NH2 In the following reaction it behaves like an acid (H * ion donor)
KNO2 —> 7?NH2 [AKHjO^]3* +H2O (AKHjOVOH)]2* + h3o*
fiCHO —> BCH2OH acid base

11 Ills Friedel-Craft’s reaction. A1C13 acts as Lewis acjd that helps 15. ZnO and A12O3 both are amphoteric. They can be separated by:
in the formation of carbocation ion (in this case CH3 without
which SE (Electrophilic substitution) reaction is not possible: Step I: Dissolve in dil. HC1 when ZnCl2 and A1C13 are formed.

CH3C1 + A1C13 ---- > CH3 +A1C1; Step II: Add NH4C1 and then NH4OH dropwise.
H++A1C1; ---- > HC1 + A1C13 Al3+ is precipated as A1(OH)3 and Zn2+ is soluble.

ch3 A1C13 + 3NH4OH---- > A1(OH)31 + 3NH4C1


+ H+ ZnCl2 + 4NH4OH---- > [Zn(NH3)4]Cl2 +4H2O
soluble
14. Aluminium salts are acidic due to hydrolysis of Al3 +
Al3* + 3H2O---- > A1(OH)3 + 3H+
Aluminum salts also exist as [A1(H2O)6]3+.
-■ V-JR

. Z

Total Practice Set


(Read, Plan & Solved)

Net reaction is
Problem 1. Whereas A1C13 is always covalent, it
2A1 + 380^ + 12H+ ---- > 2A13+ + 3SO2 + 6H2O
ionises in solution inspite of high ionisation energy. Thus, E°el| is 2.83 V hence Al can reduce SO42- to S02.
Explain.
Given Al(^) - Al3+(#) + 3e~, AH = 5114 kJ mol Problem 3. The standard free energies of formation of
B2O3 and MgO are -1194 kJ mol-1 and -569 kJ mol'1
Al3+(g) +aq - * Al3+(ag). AH = -4665 kJ mol
respectively. Should it be possible to prepare boron by
Cl"(g)+aq - -> Cl'(aq), AH = -381 kJ mol-1 reacting B2O3 with magnesium.
Solution Al3+ will be formed from A1C13 in solution if
Solution B2O3 + Mg---- > 2B + 3MgO
heat of hydration of A1C13 is greater than total ionisation
energy of Al(g). AG° = AG°roduct - AGr°eactant
Al3+(g)+ 3Cl“(g) +aq---- > Al3+(aq) +3C\~(aq) =3AG;(MgO)-AG;(B203)
AH = -4665 + 3(-381) = -5808 kJ mol"1 = 3(-569)- (-1194)
Al(g) ---- > Al3+(g) + 3e~, = -513 kJ mol-1 [AGJ(element) = 0]
AH = 5114 kJ mol-1 The negative sign of AG° shows that the reaction
Thus, AH(hydration) > AH(ionisation) should be spontaneous. (However, the reaction is
initially endothermic when two reactants are heated.)
Thus, A1C13 ionises in solution.
Problem 4. Suggest synthesis (starting from
Problem 2. EA)34/Al ==-1.66V;E°
-1.66 V;E, 2
’ SO42-/SO2 = 1.17 V. deuterium oxide as the only source of deuterium) for:
(a) B2D6 (b) B(OD)3 (c) B3N3D6 (d) B(CH2CH2D)3
What, if anything, would happen if you reacted
aluminium with sulphuric acid? If you think a reaction
Solution (a) P2O5 + 3D2O -> 2D3PO4
would take place, write down the equation.
Solution The given reaction (say dissolution of Al into + 2D3PO4 • * Mg3(PO4)2+ BjPg
HgSC^) is possible if Ec°ell is positive. (b) + 3D2O---- > 2B(OD).'3
Al---- > Al3+ +3e (c) Mg3N2+D2O---- > ND3 + Mg(OD)2
E°Al/Al 3+ = - ^Al3+ ,*,= 166V 2ND3 + 62^6 ♦ [D2B(ND3)2 ][BD4 ] —BjNjD,
so42~ + 2e’ + 4H+ —> SO2 + 2H2 (d) 6CH2=CH2 + BjDg --- > 2B(CH2CH2D),
E°SO4227SO2 = 1.17 V.
Chapter 11: p-Block Elements—Boron Family (Group 13) | 485

Explanation (c) Al and Ga are trivalent in their compounds but


in case of Tl, monovalent compounds are the
(a) Boron does not usually form a cation : most stable. Why ?
B has
(ZE)j = 801 kJ mol’1, (lE'h = 2427 kJ mol"1, - It is due to ‘inert pair' effect due to which stability of
(ZE)3 = 3659 kJ mol-1 +3 state decreases and that of +1 state increases
” Thus, the total energy required to give B3+ ions is far while going down the group (in this case group 13).
more than that which would be compensated by
lattice energies of ionic compounds or hydration of Al3* > Ga3* > In
In 3* > Tl3+
(most stable)
such ions in solution. Thus formation of cation (like
B3t) is not possible. Al* < Ga* < In* < Tl*
(most stable)
(b) The polarity ofB—X bonds is in the order: B—F > (d) BF3 reacts with F" to give BF4", but ALF3 reacts
B—Cl > B—Br but Lewis acidicity shows the with F" to give AIF^".
sequence: BF3 < BCI3 < BBr3.
- Boron has only 2s and 2p valence orbitals and can
“ With increase in polarity of B—X bond, acidity also form maximum of four bonds. Aluminium has
increases and should be thus in the order :
available 3d-orbitals and can use octahedral
BF3 > BC13 > BBr3 sp3^2-hybrid orbitals to bond to sixF" ions.
But Lewis acidity is in reverse order : (e) Aqueous solutions ofaluminium salts are acidic
BF3 < BC13 < BBr3 [AKHaOJg]3* +H2O H3O* + [A1(H2O)5OH“ ]2*
There is lateral overlap of the vacant 2p-orbi tai of B with - Due to attractive power of the small, highly charged
I one completely filled orbital of F leading to pn-pit bonds Al3+ ion for electron, an 0—H bond in a ligand H2O
between B and F. This B—F bond thus acquires double molecule breaks thus releasing a proton to an HoO
bond character. This also leads to compensate electron molecule outside the coordination sphere. Thus, HjO
deficiency of boron and thus Lewis acid character of BF3 is ligand is converted to OH", and the complex ion to
reduced. This pit-pit bonding decreases going from BF3 to (AKH^OH-f*
BBr3 and thus Lewis acidic nature increases in the order :
[AKHjOjfHOH)]3* [Al(H2O)5(OH-)f* + H*
BF3 < BC13 < BBr3

B 1| excite 111 One ofH2O ligands releases H +


(which goes to H2O solvent) and OH”
ground state 2p.1 2p2
(which remains are ligand in complex)
2s1
excited state H* +H2O H3O*
Thus, net reaction is
F F [A1(H2O)6]3*+H2O [Al(H2O)5(OH-)]3++H3O+
B—F Thus aqueous Al3* salts are acidic.
F F'
.. J

Master Exercises
Exercise 1
(Stage 1: Learning)
Short Answer Type Questions 10. Hot A12O3 will catalyse the decomposition of ethanol.
1. Rewrite the equation : A colourless gas is given off, which decolorises Bi,
B4O27"(ag) + 2H+(ag) + 5H2O(Z) ---- > 4H3BO3(ag) water. What are the products of the reaction?
showing the reaction as one that involves the 11. A1C13 can be converted into A1C1. What are the
B4O5(OH)2“ ion instead ofB4O^".Does it matter very necessary conditions?
much which equation is used? 12. (A) NaQH’ gas (B) giving white fumes with HC1
2. Aqueous solution of borax containing one mole of it alongwith white ppt (D) which
reacts with two moles of acid. Explain. dissolves in excess of NaOH to give (E)
aq BaCl2
3. Borax is also used as a buffer. Explain. ------- —» white ppt (C)
4. HjBO3 can't be titrated satisfactorily with NaOH, 0.453g of (A) gave 0.466g of (C). H2O content in (A) is
why? 47.68%.
5. (a) The formula of boric acid does not represent its Identify (A), (B), (C), (D) and (E), and explain
structure. What is its formula? reactions.
(b) The formula could be written in a different way. 13. Which type of cations are capable of replacing
What is it? aluminium in alums? Give examples.
(c) How might you classify iDoric acid’ if this formula
were used?
Analytical Questions
6. Complete the following reactions :
1. Explain:
(a) BC13+NH4C1 2140 °C > (A)
4°°C (i) BC13 exists as a monomer whereas A1C13 is a
c6h5ci dimer.
red heat (ii) Al forms [A1F6 ]3“ but [BF6 ]3- is not formed.
(b) Na2B4O7 +NH4C1 --------- > (O) (iii) Boric acid is a weak acid but it can be titrated as
(c) BCl3+LiAlH4 ---- > (D) a strong acid in presence of ethylene glycol.
(d) BC13+(C2H5)4N+C1- ---- > (E) 2. Some baking powders contain the solids NaHC03 and
(e) BC13 + H, +C0bre 1700-1800°C, (F) NaAl(SO4)2. When water is added to this mixture of
■ compounds, CO2(g) and Al(OH)3(s) are two of the
7. What are bond order of B—H bonds (X) and (Y) in products. Write the chemical equations to represent
diborane? this change.
3. In the purification of bauxite ore as a preliminary
step in the production of aluminium, (Al(OH)4r(o?)
can be converted to Al(OH)3(s) by passing COjfg)
8. Boron-nitride is also called ‘inorganic graphite and through it. Write an equation for the same. Can HC1
borazine is also called inorganic benzene'. Why? be used instead ofCO2?
9. A1(OH)3 finds uses in ‘fire extinguisher’ and in 4. Which of the following compounds are molecular, and
‘dyeing1. Explain. which have an extended three-dimensional
structure?
Chapter 11 p-Block Elements—Boron Family (Group 13) | 487

(a) B2H6 (b) KAlSi3O6 2. Why are boron halides are diborane referred to as
(c) SO3 (d) GeCl4 “electron deficient compounds?
5. Which oxide in each of the following pairs is more 3. Write the structure of diborane and explain the
acidic?
nature of bonding in it.
. (a) A12O3 or P4O10 (b) B^ or Ga2O3
(c) SO2 or SnO2 4. What happens when a borax solution is acidified?
6. Identify the group 13 (IIIA) element that best fits Write a balaned equation for the reaction.
each of the following descriptions: 5. Describe what happens when boric acid is heated?
(a) has an unusually low melting point,
(b) is the most electronegative, 6. Describe briefly how elemental boron can be
(c) is the extremely toxic, prepared?
(d) forms an acidic oxide, 7. Describe the shapes of BF3 and BH4. What type of
(e) is the most abundant element of the group,
(f) is the most stable in +1 oxidation state, hybridisation can we assign to boron in each of these
(g) is a semiconductor, compounds?
(h) forms a molecular fluoride. 8. Why do boron halides form “addition compounds”
with amines?
NCERT Problems
9. By means of a balanced chemical equation show how
1. Write balanced equations for the reaction of B(OH)3 behaves as an acid in water.
elemental boron with elemental chlorine, oxygen and
nitrogen at high temperature.

Exercise 2
(Stage 2: High Skill Problem Solving)
Only One Option Correct 5. Consider the following boron halides
1. BF3 2. BC13
1. Select incorrect statements.
3. BBr3 4. BI3
(a) BgHg has all B—H bond equal
(b) A1H3 is colourless solid and is polymeric The Lewis acid characters of these halides are such
containing Al—H—Al bridges that
(c) LiAlH4 is ionic compound (a)l<2<3<4 (b)l<3<2<4
(d) AlXj(X= Cl, Br, I) have low melting point, are (c) 4<3<2<1 (d) 4<2<3<1
covalent and have the halogen-bridged dimeric 6. The correct structural representation of diborane is
structure
2. Select correct statements.
(a) [BH2]++[BH4F
ff
(b) H B-B H
(a) Hydrides of B and Si are volatile and catches fire
on exposure to air
(b) Oxides of B and Si(B2O3 and SiO2) are acidic in H H' H H H
nature (0 B (d) H,
(c) Borates and silicates have tetrahedral BO4 and H R H H H
SiO4 structural units
(d) All the above are correct statements 7. Consider the following statements for diborane
3. NaOH can't be stored in 1. Boron is approximately sp3 hybridized.
(a) Al vessel (b) Zn vessel 2. B—H—B angle is 180°
(c) Both (a) & (b) (d) None of these 3. There are two terminal B—H bonds for each boron
atom.
4. The dipole moments of the given molecules are such 4. There are only 12 bonding electrons available.
that
Of these statements:
(a) BF3 > NF3 > NH3 (b) NF3>BF3>NH3 (a) 1,2 and 4 are correct (b) 1,2 and 3 are correct
(c) NH3 > NF3 > BF3 (d) NH3 > BF3 > NF3 (c) 2,3 and 4 are correct (d) 1,3 and 4 are correct
488 | Essential Inorganic Chemistry

8. Molecule(s) possessing three-centre-two electron 18. General formula of aluminium alums is


bonds and three-centre-four electron bonds would (M1 is monovalent cation)
include
(a) B2H6 and SiF4 (b) XeF6 and BF6 (a) [M'IHjO^MAKHjOJbXSOA
(b) [Mi(H2O)4][A1(H2O)6](SO4)2
(c) B2H6 and XeFg (d) B2H6 alone
(c) (Mi(H2O)4][A1(H2O)4](SO4)4
9. Which of the following has the regular tetrahedral (d) [MI(H2O)6J[Al(H2O),iKSO4)4
structure?
(a) BF; (b) SF4 19. Aqueous solution of borax reacts with 2 moles of
(c) XeF4 (d) [Ni(CN)4f- acids. This is because of
(a) formation of 2 moles of B(OH)3 only
10. H^O3 and HBO2 do not differ in (b) formation of 2 moles of [B(OH)4 ]~ only
(a) oxidation number (b) basicity (c) formation of 1 mole each of B(OH)3 and
(c) melting point (d) structure [B(OH)4]-
11. Light-emitting diodes (LED), laser-diodes and (d) formation of 2 moles each of [B(OH)4 Land
B(OH)3, of which only |B(OH)4 ]" reacts with acid
memory chips of computers are made of
(a) gallium (b) aluminium 20. Borax is used as a buffer since
(c) arsenic (d) gallium-arsenide (a) its aqueous solution contains equal amount of
. weak acid and its salt
12. Indium is used
(b) it is easily available
(a) to dope crystals to make p-n-p transistors
(c) its aqueous solution contains equal amount of
(b) in thermistors strong acid and its salt
(c) in low melting point solder (d) statement that borax is a buffer, is wrong
(d) all the above are correct
21. While testing BO|“, there is green-edged flame on
13. When boron is fused with NaOH, products formed are
heating the salt with cone. H2SO4 and CH30H. Green
(a) Na2B4O7 and H2 (b) NaBO2 and H2
colour is of
(c) NagBO3 andHj (d) BA and Hg
(a) (CH3)3B (b) (CH3O)3B
14. Thermite welding uses Al because of (c) B2O3 (d) H3BO3
(a) its low melting point 22. Which is insoluble in excess of NH4OH?
(b) its lightness
(c) its greater affinity for oxygen (a) A1(OH)3 (b) Cu(0H)2
(c) Zn(OH)2 (d) Cd(OH)2
(d) all the properties given above
23. Borax is converted into B by steps
15. Consider following reactions
Borax -U H3BO3 BA ii B
I: ZnO + C---- > Zn + CO
II: Cr,O3 + 2A1---- > A12O3 + 2Cr I and II reagents are
III:Al2O3+2Cr---- > C^Og + 2A1 (a) acid, Al (b) acid, C
(c) acid, Fe (d) acid, Mg
In this, possible reactions are
(a) I, III (b) II, III 24. Boron carbide is used
(c) I, II (d) I, II, III (a) in nuclear reactor to absorb neutrons
(b) as an abrasive
16. Diborane is a Lewis acid forming addition compound (c) as both (a) & (b)
BgHg • 2NH3 with NH3, a Lewis base. This (d) as none of the above
(a) is ionic and exists as [BH2(NH3)2]+ and [BH4]~
ions 25. Borax is used
(b) on heating, is converted into borazine, B3N3H6 (a) as a flux in brazing and in silver soldering
(b) in making enamel
(called inorganic benzene)
(c) in leather tanning
(c) both (a) and (b) are correct
(d) in all given above
(d) none of the above is correct
26. Soda free glass fibre are made of
17. A12(SO4)3 is used in the following but not (a) H3BO3 BA and Ca3(BO3)2
(a) as a coagulant and precipitant in treating (b) H3BO3,BA
drinking water and sewage (c) H3BO3
(b) in paper industry (d) BA
(c) as a mordant in dyeing 27. Na3AlF6 is added to A12O3 to
(d) in plastic industry
(a) improve the electrical conductivity of the cell
Chapter 11 : p-Block Elements-Boron Family {Group 13) I 489

(b) increases rate of production (c) borazole is electron deficient compound


(c) increase the melting point (d) of localised electrons in it
(d) decrease the electrical conductivity 38. Arsenic drugs are mainly used for the treatment of
(a) cholera (b) typhoid
�. Na2B4 07 • l0ffiO � NaBO2 + (A) + RzO
(c) jaundice (d) syphilis
(A)+MnO-!.+ (B) 39. B(OH) 3 + NaOH ¢ NaBO2 + Na[B(OH)4 ] + liiO
(A) and (B) are How can this reaction is made to proceed in forward
(a) Na 3BO 3 , Mn 3(BO3 )2 direction?
(b) Na2 (BO2 )2 , Mn(BO2 )2 (a) Addition ofcis-1,2-diol
(c) B.z03 , Mn(BO2 )2 (b) Addition of borax
(c) Addition of trans-1,2-diol
(d) none ofthe above is correct
(d) Addition ofNa2 HPO4
29. Mixture ofAI(OH) 3 andFe(OH) 3 can be separated by 40. H;BO3 is
(a) HCl (b) NH4 OH (a) monobasic acid and weak Lewis acid
(c) NaOH (d) HNO3 (b) monobasic and weak Bronsted acid
30. Boric acid is the trivial name for (c) monobasic and strong Lewis acid
(a) orthoboric acid (b) metaboric acid (d) tribasic and weak Bronsted acid
(c) pyroboric acid (d) none ofthese 41. AICI 3 fumes in moist air because it
31. In RzO, Ga + changes to (a) is covalent (b) is volatile
(c) is hygroscopic (d) forms HCI in moist air
(a) Ga 3+ (b) Ga
42. The solution of sodium meta aluminate on boiling
(c) Ga and Ga 3+
(d) Ga2+ and Ga 3+ with ammonium chloride gives a white precipitate of
32, Al 3+ ofKAI(SO4 )2 • 12ffiO (alum) can be replaced by (a) Al20 3 (b) AlC1 3
(a) Cr3+ (c) Al(OH) 3 (d) (NH4 )2 SO4 • Al2 (SO4)3
(b) Co a+
(c) Fe3+ (d) all ofthese 43. The incorrect statement is
(a) Anhydrous AIC1 3 is a covalent substance
33. BoraxNa2B4O7 • l0I-IiO is actually
(b) Ionic compounds generally have low melting
(a) Na 2 [B4O5 (OH)4 ] • 8liiO point and boiling point.
(b) N32[B4O4 (OH) 6 l · 7ffiO (c) A molecule shows a more stable forms as
(c) Na2[B4O3(OH) 8 ] • 6ffiO compared to individual atom
(d) Na2 [B4O2 (OH ) 10 ]·5liiO (d) CC14 is a non-polar solvent
34. Which reactions can be used to prepare diborane 44. Which ofthe following has the highest first ionisation
I. NaBH4 + BF3 (in ether) � energy?
II. NaBH4 + � � (a) Lithium (b) Beryllium
III. BF3 + NaH � (c) Boron (d) Carbon
(a) I, III (b) I, II
(c) II, III (d) I, II and III 45. Fusion ofborane with NaOH produce
35. Which ofthe following statements aboutH:t8O3 is not (a) BiO3 + �O (b) NaBO2 + ffiO
co rrect? (c) NaaB4 O7 +lliO (d) Na;BO3 +�O
(a) It is strong tribasic acid 46. Radius of Ga is less than Al because of
(b) It is prepared by acidifying an aqueous solution (a) lanthanoid contraction
of borax (b) greater screening effect
(c) It has a layer structure in which planar BO3 units (c) inert pair effect
are joined by hydrogen bonds (d) None of the above
(d) It does not acts as a Lewis acid by accepting 47. In the reaction �0 3 + C + Cl2 � A + CO
hydrox yl ion
36. The type ofhybridisation of boron in diborane is A is
(a) sp (b) sp2 (a) BC12
(c) sp 3 3
(d) sp tf (c) CCI2
37. Reactivity ofborazole is greater than that ofbenzene 48. Which ofthe following compounds is/are possible?
because (a) BiC14 , BC12 , Cl2B (b) GaCl2 , GaS
(a) borazole is polar compound
(c) Ga[GaCl4 ] · Ga2S 2 (d) All of these
(b) borazole is non-polar compound
490 | Essential Inorganic Chemistry

49. Diamagnetic species can be 59. The thermal disproportionation of I^C^ gives
(a) B2C14 (b) GaCl2 (a) BC13 (b) B4C14 ■
(c) GaS (d) all of these (c) B8C18 (d) all of these
50. Which of the following are used as transition 60. Boric acid is a weak acid. Thus, it behaves as
materials and light-emitting devices (LED)? (a) proton donor as in
I: Gallium-phosphides H3BO3+H2O H3O++H2BO3
II: Indium-phosphides (b) an electron acceptor as in
III: Gallium-arsenides
IV : Indium-arsenides B(OH)3+H2O [B(OH)4F+H+
(a) All (b) Except I all (c) proton acceptor as in
(c) Except I, II all (d) Only IV H3BO3 +H2O H4BOJ +OH’
51. Select correct statemet(s). (d) an electron donor as in
(a) Aluminium dissolves in dilute mineral acids but H3BO3 H3BO3 + e~
is made passive by concentrated nitric acid
(b) Aluminium vessel can be used as storage of 61. The decrease stability of higher oxidation state in
NaOH p-block with increasing atomic number is due to
(c) Both of the above (a) decrease in bond energy as going down the group
(d) None of the above (b) energy required to unpair ns2 electrons is not
compensated by the energy released in forming
52. Thallium and lead are classified as chemially soft the two additional bonds
because (c) both (a) and (b) are correct
(a) they have higher affinities for soft anions as (d) none of the above is correct
I" and S2" ions
62. Select correct statement(s) about group 13 (IIIA)
(b) they are soft in reactivity
elements.
(c) they are stable in lower oxidation state (Tl+, Pb2+)
(a) Stability of hydrides (AfH3) decreases as we move
(d) they are stable in higher oxidation state (Tl3+,
down the group
Pb4+)
(b) MX2 is a strong Lewis acid
53. The relative stability of the different oxidation state (c) [AKI^Olg ]3+ exists in aqueous solution
Tl+> Tl3+,>Ga3+>Ga+ (d) All the above are correct statements
Tl+>In+>Ga+ 63. A1O2 is known as
is an example of
(a) aluminium dioxo ion (b) meta aluminate ion
(a) redox potential (b) disproportionation
(c) dioxo aluminium ion (d) aluminium oxide ion
(c) inert pair effect (d) electron-affinity
64. In aqueous solution GaCl disproportionates to
54. Thermal stability of boron compounds is in order
(a) GaCl2 andGaCl3 (b) Ga and GaCl3
(a) B2F4 <B2C14 <B2(O/?)4 <B2(N2^)4
(c) GaCl2 and Ga (d) GaCl3 andGaCl5
(b) B,C14 <B.2F4 <Br(O2?)4 <B>(N??2)4
(c) B,(NE,)4 <B2(O/?)4 <B2C14 <b2f4 65. Boron does not form B3+ cation easily. It is due to
(d) B2(N/?2)4 <B2(O7?)4 <B2F4 <B2C14 (a) energy required to form B3+ ion is for more than
that which would be compensated by lattice
55. Anion containing both three-and four-coordinated
energies or hydration energies of such ion
boron is
(b) boron is non-metal
(a) [B(OH)4r (b) B3O|- (c) boron is semiconductor
(c) [B4O5(OH)4f- (d) BOjBOJ- (d) none of the above
56. Which is spontaneous in forward side? 66. An alloy of boron and aluminium is allowed to react
(a) Al3++2e~ ---- > AI+ (b) Ga3++2e~ - Ga+ with NH3. Products formed are
(c) In3++2e" ---- > In+ (d) Tl3++2e’ - * Tl+ (a) BN, AIN and Ha
57. Light elements B, Al, C and Si are (b) B(NH2)3,A1(NH2)3 and EL,
(a) oxophiles (high affinity for oxygen) (c) BN, AKNH^andEL,
(b) fluorophiles (high affinity for fluorine)
(c) both type (d) B(NH2)3,A1N andHj
(d) none of the types given A H2O
67. Element M + N2 NH3
58. Stabilities of adducts formed with boron-halides is in Element Af belonging to group 13 can be
order (a) B or Al (b) Ga or Al
(a) BF3 < BC13 < BBr3 (b) BBr3 < BC13 < BF3 (c) Mg or Al or B (d) Mg or B
(c) BC13 < BBr3 < BF3 (d) BF3 < BBr3 < BC13
Chapter 11: p-Block Elements-Boron Family (Group 13) I 491
S8. N2 gas is passed or heatedMg and B. Products formed 78. Orthoboric acid Hal303 behaves as weak monobasic
are acid giving H30+ and
(a) MgaN2, BN (b) MgaN2, BN andMgaBz (a) �BO; (b) �B02
(c) MgN, BN (d) no reaction (c) [B(OH)4 r (d) [B(OH)4 r
69. Ano dising can be done by electrolysing dilute �S04 79. Boron sesquioxide is
with Al as anode. This results in (a) B.?Oa (b) B03
(a) the formation of protective oxide layer (Al 203 )
(c) Bo;- (d) BG;
(b) the formation of Al2 (S04 )a and S02 gas
(c) the formation of AIH 3 and S02 gas 80. Select correct statement(s) about Hal303
(d) the formation of Al(HS 03)3 and� gas (a) It has triangular BOf units
70. Alzheimer's disease is caused due to Al interaction (b) In solid states, molecules are hydrogen bonded
with internal organs of the body if food is (c) Both (a) & (b) are correct
contaminated with Al. This diseases (d) None of the statements is correct
(a) induces senility in young persons 81. The structure of BF3 is
{b) causes memory loss (a) tetrahedral (b) pyramidal
(c) both (a) and (b) are correct (c) planar triangular (d) T-shaped
(d) none of the above is correct
82. In BF3
71. Pure boron is best prepared by
(a) all the B-F bond are single covalents in nature
(a) heating B.?03 with�
(b) boron-fluorine bond has some double bond
(b) heating B.?03 with Na or K character and this bond is delocalised
(c) heating KBF4 with Na or K (c) bond energy and bond length of B-F bond
(d) heating BBr3 with� in presence of a catalyst indicate its single bond character
72. The borax bead test can be used to detect the (d) all the bonds are ionic
presence of 83. BF3 behaves as a
(a) Ala+ (b) Mg2+
3 (a) Lewis acid and lone pair from the Lewis base is
(c) Fe + (d) Na +
filled into empty 2 Pz orbital
'73, Water softener is (b) Lewis base and lone pair on B is donated to empty
(a) borax (b) zeolite orbital of the Lewis acid
(c) both (a) and (b) (d) none of these (c) amphoteric species due to interaction with acid
74. N82B4 07 • lOHiO is correctly represented as and base both
(d) polymer substance
(a) Na2 [B4 05 (0H)4 ] • 8f¼O
84. B.?Cl4
{b) 2NaB02 • Na2�03 • 10�0
(a) can be formed by passing BCI 3 over Hg at low
(c) Na2 [B4 (�0)4 07 ]·6�0
pressure in an electric discharge
(d) all of the above
(b) can exist on non-eclipsed and planar structure
75. B can be obtained from halide by van Arkel method. (c) decomposes at low temperature
This involves reaction (d) can show all the facts given above
red hot W or Ta
(a). 2BCla + 3lii 2B + 6 HCl 85. BF3 on hydrolysis forms
filament
(a) Hal30 8 (b) HBF4
2B + 3 ½
red hot W or Ta
{b) 2Bia
(c) both (a) and (b) (d) None of these
(c) both (a) and (b)
(d) none of the above 86. Which of the following is false statement?
(a) BH3 is a Lewis acid
76. Which of the following statements regarding ortho (b) All the B-H distance in diborane (l\H6 ) are
boric acid (Hg803 ) is false? equal
(a) It acts as a monobasic acid (c) Boranes are easily hydrolysed
{b) It is soluble in hot water (d) LiAIH4 reduces BC1 3 to borane
(c) It has a planar structure
(d) It acts as a tribasic acid 87. ln l\H6
(a) there is direct boron-boron bond
'17. When orthoboric acid is heated to red heat the
(b) the B-H bonds are ionic
residue is
(a) boron (c) it is isostructural to GiH6
(b) boric oxide
(d) boron atoms are linked through hydrogen bridges
(c) metaboric acid (d) pyroboric acid
492 | Essential Inorganic Chemistry

88. From B2H6 all the following can be prepared except (b) dye is adsorbed on KOH formed due to hydrolysis
(a) H3BO3 (b) NaBH4 (c) of both (a) and (b)
(c) B2(CH3)6 (d) BaOg (c) of none of the above
89. Boron nitride has the structure of the type 99. Alum is found to contain hydrated monovalent cation
(a) both diamond and graphite [M(H2O)6r, trivalent cation [M'CE^Olg]3* andSO;"
(b) graphite in the ratio of
(c) diamond (a) 1:1:1 (b) 1:2:3
(d) NaCl (c) 1:3:2 (d) 1:1:2
90. Which of the following is the false statement? 100. About 2 to 5% of gypsum (CaSO4 • 2H2O) is added to
(a) Reducing action of NaBH4 and LiAlH4 are Portland cement. It is just to
specific (a) increase the setting rate of the cement during
(b) B2H6 reduces alkene to primary alcohol hydration
(c) Both (a) and (b) are the false statements (b) slow down the setting rate to increase strength
(d) None of the above is false statement. during hydration
91. Beryllium and aluminium have similar properties (c) both (a) and (b )are correct
because (d) none of the above is correct
(a) they belong to same group
(b) they have different electronegativity One or More Than One Options Correct
(c) they have same electronegativity
(d) they have same ionisation energy 1. Which fumes in moist air?
92. The solubility of A1(OH)3 in strong NaOH solution is (a) A1C13 (b) BF£3
best explained by the formation of (c) CuSO4 (d) NH; 3
(a) A1O3 ion 2. Select correct statement(s).
(b) double salt (a) BF3 fumes strongly in moist air and is partially
(c) a peptised colloidal sol hydrolysed by excess of water
(d) a complex hydroxyl aluminate ion (b) BF3 is converted into the adducts BF3 • HjO and
93. On passing CO2 into aqueous solution containing BF3 • 2H2O with small amounts of water at low
Al3+ temperature
(a) A12(CO3)3 is formed (c) HgBOg is a weak acid but HBF4 is a very strong
(b) A1(OH)3 is precipitated acid
(c) [A1(OH)41- is formed (d) KBF4 is sparingly soluble in water
(d) colloidal A1(OH)3 is formed 3. Select correct statement(s).
94. Addition of sodium carbonate to a solution of an oxide (a) BF3 forms adducts with NH3, BOH and BNHj
in water produces CO2. This experiment indicates
that (b) BC13 form adducts with NH3, H^O and BNHj
(a) the oxide is that of a non-metal (c) BC13 reacts with liquid NH3 and ethanol forming
(b) the oxide is basic B(NH2)3 and B(OEt)3
(c) the oxide is amphoteric (d) [BC14 ]", [BBr4 ]" and [B^ ]" ions can be made only
(d) the oxide is neutral in non-aqueous media
95. Dimer formation can take place with 4. Select correct statement(s).
(a) BH3 (b) A1C13 (a) Boron trimethyl is a weaker Lewis acid than the
(c) NO2 (d) all of these boron trihalides or monoborane
(b) Lewis acid property when MegN is a donor is in
96. The major constituent in cement is order:
(a) silica (b) magnesium oxide
BBr3 > BC13 > BF3 ~ BH3 > BMe3
(c) calcium carbonate (d) iron oxide
(c) Lewis acid property when Me^ is a donor is in
97. Hardening of cement is due to order:
(a) dehydration
BBr3 > BC13 ~ BH3 > BF3 ~ BMe3
(b) hydrolysis
(c) hydration and hydrolysis (d) Lewis acid property when CO is a donor : no
(d) polymerisation adduct is formed with halides and Lewis acid
property of
98. Alums are used as mordant is dyeing because
(a) dye is adsorbed on A1(OH)3 which is deposited on BH3 » BF3 • BMe3
fibre in the hydrolysis process
Chapter 11: p-Block Elements-Boron Family (Group 13) I 493

5. Select correct statement(s). III :Sodium peroxoborate is an important constituent


(a) A saturated solution of boric acid in water is of washing powder.
neutral to the indicator bromocresol green Select correct statement.
(b) AlF3 is insoluble in anhydrous HF (a) I, II (b) II, III
(c) Aqueous solution of KHF2 is basic to the indicator (c) I, III (d) I, II, III
bromocresol green 12. Select correct alternate(s).
(d) Aqueous solution of boric acid and potassium (a) The group BN is isoelectronic with�
hydrogen difluoride is alkaline to bromocresol (b) No species with B!!!!!:N group is formed
indicator
(c) Common form of boron nitride has an ordered
6. Select correct statement(s). layer structure closely resembling that of
(a) Diborane is an electron-deficient compound graphite
(b) Diborane is stoichiometrically analogous to (d) AlN has the wurtzite structure
ethane (CzH 6 )
(c) Total electrons involved in bonding are 12 in
diborane and ethane Passage Comprehension Questions
(d) Total electrons involved in bonding are 12 in
diboroane and 14 in ethane
Passage 1
7. Boric acid is a weak Lewis acid proton comes from The tetrafluoroborate anion BF4-, is a Lewis base. The
(a) by its ionisation other tetrahaloborate anions, BC14 and BB14-, can be
(b) when it forms complex with �O prepared in non-aqueous solvents.
(c) when it forms borate esters with alcohol Based on the above short write-up, answer the
(d) all of the above following questions :
8. Select correct statement(s). 1. BF4- is
(a) On the basis of Lewis acidity, an oxygen atom is (a) F- donor
more effective than an fluorine atom as a n-donor (b) a lone-pair acceptor
towards boron
(c) a lone-pair donor
(b) 1,2-diols have a strong tendency to form borate
(d) solvated in aqueous solution
esters on account of chelate effect
(c) Borate esters are stable due to chelate effect 2. BCl4 and BB1f are stable in
(d) None of the above is correct statement (a) RzO °
(b) alcohol
(c) both (a) and (b) (d) None of these
9. Actual source of protons in boric acid is based on the
following
(a) B(OHMaq) + 2�(Xl) � H p (aq)
+ Passage 2
+ [B(OH)4 f(aq) Certain aspects of the aqueous chemistry of
+
(b) 3B(OH) 3(aq) � [B3O3(OH)4 J-(aq) + H (aq) aluminium compounds derive from the nature of the
in neutral or basic solution + �O(l) hydrated aluminium ion-[Al(R.zO) 6]3+ .
(c) B(OH) 3 ¢ �Bo;- + H+
[Al(R.zO)6]3+ +�O � [Al(H.P)5 (OH-rf + +H3O+
(d) all of the above
10. Select correct statement(s). Read the above write-up and answer the following
(a) Aluminium monohalides are formed by the questions.
reactions of the trihalides with aluminium at 1. Above equilibrium indicates that
1300K followed by rapid cooling {a) �O is a Lowry-Bronsted acid
Cb) Aluminium monochloride (red) is formed when
aluminium reacts with HCI at 1200 K (b) �O is a Lowry-Bronsted base
(c) Aluminium monohalides disproportionates at (c) �O is a Lowry-Bronsted acid as well as a base
room temperature (d) HiO is neither acid nor base
(d) Ga2Cl4 exists as Ga +[GaC14 t and is diamagnetic 2. Above equilibrium is repressed in
and has crystalline structure. (a) strongly acidic solution
11. Consider following statements (b) alkaline solution
I: Boric acid is a mild antiseptic and is used as a (c) aqueous solution
food preservative. (d) any of the above
II : Borax and other borates are used in
3. Ionisation of the above equilibrium is favoured in
water-treatment, timber preservation, glass
manufacture. (a) strongly acidic solution
(b) alkaline solution
494 | Essential Inorganic Chemistry

(c) aqueous solution 1. A is


(d) any of the above (a) NaBO2 (b) Na2B4O7
4. Aluminate ion can be written as (C) HgBOg (d) HBO3
(a) A1O2 (b) A1(OH)4- 2. Bis
(c) [Al(H2O)2(OH)4r (d) all of these (a) NaBO2 (b) BA
(c) Na3BO3 (d) H3BO3
Passage 3 3. C and D respectively are
Diborane is an electron deficient compound. (a) Cu2BO2,CuO (b) Cu(BO2)2,CuBO2
Stoichiometrically, it is analogous to ethane, C^Hg. (c) CuAO^CuO (d) Cu(BO2)2,Ci^BA
However, the total number of electrons involved in 4. Select correct statement(s) about E
bonding and less in diborane. (a) E is a weak acid (H3BO3)
Read the above short note and answer the following (b) It is not a protonic acid but electron-pair acceptor
questions. (c) E shows following equilibrium
1. Bonding electrons involved B(OH)3+H2O [B(OH)4r+H3O+
(a) in B.2H6 is 14 and in C2H6 16 (d) All the above are correct statements
(b) in B2H6 is 12 and in (^Hg 14
(c) in B2H6 is 10 and in C^Hg 12
(d) in B^jHg is 16 and in CgHg 18 Passage 5
The chemical properties of boron, carbon, silicon and
2. Octet is complete in germanium are distinctly those of non-metals. The light
(a) boron in diborane elements, boron, aluminium, carbon and silicon are strong
(b) carbon in ethane oxophiles and fluorophiles, in the sense that they have
(c) boron in diborane and carbon in ethane high affinities for oxygen and fluorine, respectively.
(d) None of the above Based on the above short write-up, answer the
3. In diborane there is following questions.
(a) two-electron-three-centre bonds 1. Oxophilic character is observed in
(b) three-electron-three-centre bonds (a) borates, aluminates, carbonates, silicates
(c) two-electron-two-centre bonds (b) boranes, alkanes, silanes, alanes
(d) three-electron-two-centre bonds (c) both of the above
4. Consider the following statements for diborane (d) none of the above
I. Boron is approximately sp3 hybridised 2. The similar oxophilicity and fluorophilicity concludes
II. B—H—B angle is 180° that diagonal relationship is between
III. There are two terminal B—H bonds for each (a) boron and aluminium
boron atom (b) boron and silicon
IV. There are only 12 bonding electrons available (c) boron and carbon
Of these statements (d) aluminium and silicon
(a) I, III and IV are correct
(b) I, II and III are correct
(c) II, III and IV are correct Assertion & Reason
(d) I, II and IV are correct Codes:
(a) Both A and R are true and R is the correct
Passage 4 explanation of A
Study the following sequence of reactions : (b) Both A and R are true but R is not the correct
A1, ,. H2O 4Z __ x x A explanation of A.
Alkaline <---- A(white crystalline) * B (transparent (c) A is true but R is false.
bead) (d) A is false but R is true.
Cu2+,A 1. Assertion (A): Boron forms BF4”.

h2so4 C(green-coloured bead) Reason (R): BF3 is an electron deficient species.
A in reducing flame 2. Assertion (A): B2C14, GaCl2, GaS are diamagnetic.
Reason (R): They show +2 oxidation state.
E (white crystalline D (red-coloured bead)
3. Assertion (A): NaBH4 has NaCl structure.
compound on cooling)
Reason (R): NaBH4 is an ionic salt.
Chapter 11 : p-Block Elements-Boron Family (Group 13) I 495
4. Assertion (A) : HBF4 is a very strong acid. Reason (R) : The high enthalpy of hydration is
sufficient to break the covalent dimer into
Reason (R) : B(OH) 3 behaves as an acid in the [M(E½0)6 13• and c1- ions.
following reaction :
+
r
B(OH) 3 + 4HF � H 30 + [BF4 + 2Jti0
16. Assertion (A) : In water, orthoboric acid behaves
as a weak monobasic acid.
5. Assertion (A) : Cryolite is added to bauxite in Reason (R) : In water orthoboric acid acts as a
metallurgical extraction of aluminium. proton donor.
Reason (R) : Cryolite, Na 3 [AIF6] contains an
17. Assertion (A): Boron always forms covalent bond.
octahedrally coordinated aluminium atom.
Reason (R) : The small size of B3+ favours
6. Assertion (A) : AIC13 exists as [Al(£½0) 6 ]Cl3 in
formation of covalent bond.
dilute aqueous solution.
Reason (R) : When water is dropped to a solid
A1Cl 3, vigrous hydrolysis ensues. True & False
7. Assertion (A) : B(OH)3 is a good lubricant and 1. Borosilicate glass has a lower coefficient of thermal
slipp ery. expansion.
Reason (R): B(OH)3 has a layer structure. 2. H:i,803, B.z03 and Ca 3(B03)2 are used to make
soda-free glass fibre.
8. Assertion (A) : Boric acid does not act as a proton
donor but an electron-acceptor. 3. Al203 is a good conductor.
B(OH)3 +JtiO � [B(OH)4 +H+ r 4. When boron is fused with sodium hydroxide, sodium
Reason (R) : Boric acid is a very weak acid. metaborate (NaB02) and hydrogen are formed.
9. Assertion (A) : The group BN is isoelectronic with 5. H:f02 is a weak monobasic acid.
c;.
6. Borax is also a good buffer.
Reason (R): No species containing a B!!!!!!!N group
has yet been discovered. 7. Metaborate beads of many transition metals have
characteristic colours.
10. Assertion (A) : BaN 3H12 (similar to cyclohexane)
exists as chair conformation. 8. HaB03 is not a Bronsted acid but Lewis acid and
accepts OH-.
Reason (R) : Borazine (BaN'aff6) is much more
reactive than benzene. 9. Na�P02 and NaHP03 are acid salts.
11. Assertion (A) : Aluminium trimethyl is a dimer 10. Borazine BaN 3H6 is called inorganic benzene.
having structure like that of A1Cl 3 •
11. In diborane all the B-H bonds are equivalents
Reason (R) : There is three-centre two-electron
bonding in Al-C-Al bridge. 12. Structure of aluminate ion changes with both pH and
12. Assertion (A) : Borazine is called inorganic concentration.
benzene. 13. Al(OH)3 is used as mordant in dyeing.
Reason (R) : Borazine undergoes addition and
14. In diborane (B.zH 6 ) there are (2c-2e) (3c-2e) type of
substitution reactions with HCI and BJii etc.
bonds.
13. Assertion (A) : The bond lengths in BF3 are 1.30 A
and are shorter than the sum of the covalent radii. 15. Aqueous solution ofAIC1 3 is acidic due to hydrolysis.
Reason (R) : pn-pn interaction exists and all B-F
bonds are equivalents are based on modern Fill in the Blanks
explanation, the double bond is delocalised. 1. Correct representation of the formula of borax is
14. Assertion (A) : AIC1 3 forms dimer Al2 Cl 6 but is
dissolves in �O forming [Al(Jti0} 6 ]3• and c1- ions. 2. Conversion of BI3 into B by pyrolysis is called
Reason (R) : Aqueous solution of A1Cl 3 is acidic
due to hydrolysis. S. Cryolite ............ the electrical conductivity of Al20 3
15. Assertion (A) : In aqueous solution AIC1 3 forms in the extraction of Al.
dimer Al2 Cl 6• 4. Stability order of Ga + < In + < Tl + is described by
496 | Essential Inorganic Chemistry

5. Concentrated HN03 renders Al passive due to Matrix-Match Type Questions


formation of
I. Only One Correct
6. A12O3 is an oxide.
1. Match the compounds (in List I) with their use
7. H3BO3 behaves as
(in List II).
8. H3BO3 does not donate protons but accepts
Compound Use
ion. (List I) _______ (List II)
9. In diborane, boron atom is hybridised. A. Boric acid 1. In Friedel-Craft acylation
B. Borax 2. Mild antiseptic
10. S2 is since, it has electrons.
C. Aluminium chloride 3. Washing powder
11. A1O is formed when aluminium powder is heated in D. Sodium peroxoborate 4. Buffer
oxygen at 3260°C and is called
12. When ethereal solutions of A1C13 and LiAlH4 are 2. Match the types of borates (in List I) with their
formula (in List II).
mixed, is formed.
Formula
13. AICI3 fumes in moisture due to Types of borates (List I) (List II)
14. When A1I3 is heated with CC14, and A. Metaborate 1. NaBO3-4H2O
are formed.
B. Orthoborates 2. Mg2B2O5
15. Aluminium chloride forms dimer A12C16 but in
C. Pyroborates 3. K3(BO2)3
aqueous solution [A1(H2O)6]3+ and Cl" ions are
D. Perborate 4. Ca3(BO3)2
formed since, covalent bonds of dimer are broken by

16. p-block elements are characteristed by valence shell 3. Match the items (in List I) with their formula
electronic configuration as (in List II).
17. Boron and aluminium both are amphoteric and form Items Formula
with NaOH. (List I) (List II)
18. Halides of group 13(IIIA) have halogen bridged A. Tetraboric acid 1. B3N3H6
structures.
B. Borax 2. Ga(OH)2
19. Reaction of Al with O2 is ... and is called
C. Borazine 3. H2B4O7
reaction.
D. Amphoteric 4. Na2B4O7-10H20
20. Use of Al causes disease called
21. Borax is used as a standard in titrations
against acids. II. One or More Correct
22. B.2H6 changes CH3CH—CH2 with alkaline H2O2 into 1. Match the compounds in Column I with their

23. Diborane (B2H6) has electron bonds. Column I Column II

24. Cu2+ salts form coloured bead with borax of the A. Borax 1. Electron deficient
formula B. Ortho boric acid 2. spz hybridisation
25. Corundum is C. Borane 3. Coloured beads with CuO
26. Gypsum is added to cement to slow down the D. Alums 4. Mordant in dyeing
to increase strength.
E. Boron nitride 5. Triangular planar
27. Be^jC and A14C3 are called .... because they react
6. Changes CH3CH=CH2 into
with H^O yielding methane. alcohol by anti-Markownikoffs
28. SigN4,B4Cand SiC are ceramics. addition
Chapter 11: p-Block Elements—Boron Family (Group 13) | 497

2. Match the species in Column I with their products formed are to be neutralised by HC1.
characteristics in Column II. Number of moles of HC1 required per mole of borax
hydrolysed is
Column I Column II
3. Total number of B and N atom in borazine is
A. Benzene 1. Six-membered ring
4. B(OH)3 can accept hydroxide ion(s) from
B. Borazine 2. Delocalisation of n-electrons HjO.
C. Inorganic graphite 3. sp2-hybridised carbon
5. Aluminium methanide contains .
0. Borane 4. Mole fraction of hydrogen 0.5 atom(s).
5. Mole fraction of nitrogen and 6. AG is ngative for the following reaction
boron equal rp|3+ ____ rp)X+

6. Mole fraction of boron 0.25


when x =
7. Out of B2C14, GaCl2, GaS and S2 number of
Integer Answer Types diamagnetic substances are
This section contains 8 questions. The answer to each 8. Number of tribasic acid out of the following is
of the questions is a single digit integer, ranging from 0 to
9. The appropriate bubbles below the respectively
H3BO3,H3PO3,H3PO2,H3PO4
question numbers in the ORS have to be darkened. For
trample, if the correct answers to question number X, Y, Z
and W (say) are 6,0,9 and 2, respectively, then the correct
Set II
[darkening of bubbles will like the following : 1. Alum is found to contain hydrated monovalent
cation, trivalent cation and sulphate in the simplest
X Y IV ratio x: y: z where x + y + z -
® ® ®]
2. Borax is found to have tettrahedral unit(s).
® ® ® 3. B2H6 has B—H terminal bonds and B—H bridging
® ® ® ® bonds which is equal to
4. Total electrons involved in all the B—H bridging
® ® ® ® bond is
® ® ® 5. Total electrons involved in all the B—H terminal
0 bonds is
® ® ® ®
® ® ® ® 6. A x—membered ring is formed in (BN)rt where x is

Set I 7. Al(s) is to be converted into Al3+(aq) ion. How many


steps are involved? (Assume ionisation in one step)
1. Maximum covalency of aluminium is
8. Cl2(g) is to be converted into Cr(04?) ion. How many
2. In the hydrolysis of borax
steps are involved?
Na2B4O7+7H2O ---- > 2Na[B(OH)4 ] + B(OH)3
Answers & Solutions
Master Exercises
Exercise 1
Short Answer Type Questions (c) 4BC13 + 3LiAlH4 3LiCl + 3A1C13 + B^
1. B4O5(OH)2“(aq) + 2H+(aq) + 4H2O(Z) Diborane
4H3BO3(aq) (P)
No, it does not; the mole ratio of the ion to the H+ ions is 1700-1800°C n r
(d) 4BC13 + 6H2 + C(flbre) --------------------- > h’4b(fibr«)
the same in both cases. (F)
2. When borax is dissolved in water, both B(0H), and V
+ 12HC1
B(0H)4]- are formed, but only [B(OH)4]~ reacts with 7. Bond order : bond X = 1 (formed by two electrons)
HC1. [B(OH)3, called orthoboric acid, does not react with
HC11 ¥ = 0.5 (formed by one electron)
(Also see text)
Na.>B4O7 + 7H2O---- > 2Na[B(OH)4] +2H3BO3
1 mol 2 mol 2 mol 8. Refer text.
2Na[B(OH)4] + 2HC1---- > 2NaCl + 2H3BO3 + 2H2O 9. Fire extinguishers can be made that contain aluminium
or Na2B4O7+2H2O ---- > 2NaCl + H2B4O7 sulphate together with the usual carbonate and acid.
weak acid When the extinguisher is used, A1(OH)3 is produced:
2NaOH + 2HC1 ---- > 2NaCl + 2H2O 2Al3+(ag)+ 3CO3-(ag)+ 3H2O---- > 2Al(OH)3(s)
3. Borax can be used as a buffer since, its aqueous solution + 3CO2(£)
contains equal moles of weak acid (H.jBO3) and its salt
Na[B(OH)4l. The hydroxide has the virtue of stabilising the foam,
which covers the fire.
B4O^ + 7H2O---- > 2[B(OH)4r + 2H3BO3
salt weak acid Particles of Al(OH), are able to attach themselves to the
fibres in cloth. The hydroxide is able to adsorb dye
4. Since, B(OH)3 only partially reacts with H2O to form
molecules much more efficiently than the fibres alone. A
H3O+ and [B(OH)4]_, it behaves as a weak acid. Thus,
substance like A1(OH)3 that assists in dyeing of a fabric
sharp end point is not obtained when it is titrated with
NaOH: is called a mordant.
A> hit ptt . tr n
B(OH)3 + 2H2O---- > [B(OH)4r + H3O+ 10. C^OH \ CH2 =CH2 + H2O
5. (a)H3BO3 (b) B(OH)3 (c) it looks like a hydroxide ethene
O V

11. A1C13 forms A1C1 in the gas phase at elevated


6. (a) 3BC13 + 3NH4C1 . 140°c-»+ b N33h
B33n C1.3 -aBH4>
H33ci
C6H5C1 3 3 3
3 (A) temperatures:
high
Bo3NJ3HO6 A1C1,0 + 2A1 4 3A1C1
Borazine temperature
(inorganic benzene)
(B) 12. (A): NH4A1(SO4 )2 ■ 12H2O (B): NH3 (C): BaSO4
H
I (D):A1(OH)3 (E): NaA102
H- B ■n/h

NH4A1(SO4)212H2O nh3t +ai(oh)3


(A) <b) m
Hr H NaOH
I BaCl2
H
BaSO4 NaA102
(b) Na2B4O7 + 2NH4C1-red hea4 2BN (C) (£)
+
Boron nitride 13. Ti3+,Cr3+,Mn3+,Fe3+,Co3+, etc. (of approximately
(inorganic graphite)
same size as that of Al3+) can replace Al3+ in alums.
+ 2NaCl + 4H2O
Chapter 11:: p-Block Elements—Boron Family (Group 13) | 499

B(OH)3 + 2H2O H3O+ + [B(OH)4f; pffa =9.2


Analytical Questions Above equilibrium can be shifted largely towards
1. (i) BC13 is stable due to stability of boron-chlorine right by forming a stable chelate with
n-bonding. When there is dimer formation, this cis-dihydroxy compounds (as glycol)
bonding is disrupted as there would be no vacant
orbital for n-bonding. It is also likely that the boron C—OH
atom is too small to form the stable 4-membered ring [B(OH)4r + 2 |
system in such dimers. C—OH
A1C13 is electron-deficient in Al (octet-incomplete)
thus behaves as Lewis acid and Al completes its + 4H2O
octet by taking electron-pair from Cl-atom The proton can now be titrated against strong alkali
Cl\ /Ci:x /Cl solution using phenolphthalein indicator.
Al Al NaAl(SO4)2 Na++ Al3++ 2SOJ-
2.
Cl :ci Cl

The crystalline solid has a layer-lattice with Na+ + SO*' + Al3+ + H2O A1(OH)3 + NaOH + H^
six-coordinated Al. On melting it changes to the NaHCO3+H+ Na++H2O + CO2
halogen-bridged dimer A12C16 in which coordination 3. [Al(OH)4r+H2O + CO2 A1(OH)3 +H2O + HCO3
number drops to four. This is accompanied by a
[Al(OH)4r +H+ A1(OH)3 + HjO
dramatic increase in volume-density of the melt is
nearly 45% lower than that of the solid at the aici3
melting point. The electrical conductivity also
HC1 converts it into A1C13.
approaches zero. The dimers also dominate in gas
phase. 4. (a) , (c) and (b): molecular
(ii) Boron has only 2s- and 2p-valence orbitals and can (b) : three dimensional
form maximum of four bonds. Aluminium has 5. (a) P4O10 (b) B2O3 (c) SO2
available 3d-orbitals and can use octahedral
sp3d2-hybrid orbitals to bond to six F" ions. 6. (a) Ga (b) B (c) T1 (d) B (e) Al (D T1 (g) B
(in) Orthoboric acid, HgBOg, is soluble in water and (h) B
behaves as a weak monobasic acid. It does not
donate protons like most acids, but rather it accepts NCERT Problems
OH’. Refer Text

Exercise 2
Only One Option Correct
5. (c) 6. (c) 7. (d) 8. (d) 9. (a) 10. (a)
1. (a) 2. (d) 3. (c) 4. (c)
15. (c) 16. (c) 17. (d) 18. (a) 19. (d) 20. (a)
11. (d) 12. (d) 13. (c) 14. (c)
25. (d) 26. (a) 27. (a) 28. (c) 29. (c) 30. (a)
21. (b) 22. (a) 23. (d) 24. (c)
35. (a) 36. (c) 37. (d) 38. (d) 39. (a) 40. (a)
31. (c) 32. (d) 33. (a) 34. (d)
45. (d) 46. (b) 47. (d) 48. (d) 49. (d) 50. (a)
41. (c) 42. (c) 43. (b) 44. (d)
55. (c) 56. (d) 57. (c) 58. (a) 59. (d) 60. (b)
51. (a) 52. (a) 53. (c) 54. (b)
65. (a) 66. (c) 67. (a) 68. (b) 69. (a) 70. (c)
61. (c) 62. (d) 63. (b) 64. (b)
75. (b) 76. (d) 77. (b) 78. (c) 79. (a) 80. (c)
71. (b) 72. (c) 73. (c) 74. (a)
85. (c) 86. (b) 87. (d) 88. (c) 89. (b) 90. (d)
81. (c) 82. (b) 83. (a) 84. (d)
95. (d) 96. (c) 97. (c) 98. (a) 99. (d) 100. (b)
91. (0 92. (d) 93. (b) 94. (a)

One or More Than One Options Correct Passage Comprehension Questions


1. (a,b) 2. (a,b,c,d) 3. (a,c,d) 4. (a,b,c,d) Passage 1
5. (a,b,d) 6. (a,b,d) 7. (b) 8. (a,b,c) 9. (a) 1. (c) 2. (d)
10. (a,b,c,d) 11. (d) 12. (a,b,c,d)
500 | Essential Inorganic Chemistry

Passage 2 Matrix-Match Type Questions


1. (c) 2. (a) 3. (b) 4. (d) I. Only One Correct
Passage 3 1. (AM2); (B)—(4); (C)-(l); (D)—(3)
1. (b) 2. (b) 3. (a) 4. (a) 2. (AH3); (B)—(4); (CM2); (DM2)
Passage 4 3. (AH3); (B)—(4); (C)—(1); (D)—<2)

1. (b) 2. (b) 3. (b) 4. (d) II. One or More Correct


1. (A)—(3); (B)—(1,2,3,5); (C)—(2,5,6); (D)—(4)
Passage 5
(EM2)
1. (a) 2. (b) (C)—(1,2,3,5);
2. (A)—(1,2,3,4); (B)—(1,2,3,4,5,6);
(DM3,6)
Assertion & Reason
1. (a) 2. (b) 3. (b) 4. (c) 5. (b) 6. (b) Integer Answer Types
7. (a) 8. (b) 9. (b) 10. (b) 11. (b) 12. (a) Set I
13. (a) 14. (b) 15. (d) 16. (c) 17. (a) Questions —> £7_ 8
1 2 3 4 5
Answers —► 6 2 6 1 3 1_3 1
True & False © © © © © ©I© ©
1. T 2. T 3. F 4. F 5. T 6. T O O O © 0
7. T 8. T 9. F 10. T 11. F 12. T © © © © ©
© © © © © ©_© ©
13. T 14. T 15. T
© © © © © © (4) ©
© © © © © ©
Fill in the Blanks © © © © © © © ©
1. Na2[B4O5(OH)4]-8H2O 2. van Arkel method
0 0 0 0 0 ©
3. increases 4. inert-pair effect © © © © © >_® ©
5. protective layer of oxide on the surface 6. amphoteric © © © © © ©I© ©
7. weak monobasic acid 8. OH" 9. sp3
Set II
10. paramagnetic, two unpaired 11. aluminium suboxide Questions —> 1 2 3 4 5 £ 7 8_
12. A1H3 13. hydrolysis 14. AlCl3andCl4 Answers —► 4 2 8 4 8 6 3 3
15. high enthalpy of hydration 16. ns2np1-6 © © © © © ©|© |@
©. © O © O ©I© (D
17. sodium metaborate and meta-aluminate respectively
© © © © © ©
18. dimeric 19. exothermic, thermite 20. alzheimer © ©
© © © © ©
21. primary 22. CH□3CH2CH-22OH 23. odd © © © © © _©_© ©_
24. Cu(BO2)2 25. Al90, 26. setting process © © © © © _©_© ©.
© © © © © © ©
27. methanides 28. non-oxide
© 0 0 0 0 ©_©
© © © © © © © ©.
© © © © © ©I® ©
14 Group
(IVA-The Carbon Family)
Carbon is a non-metal; silicon and germanium are classified as
11

metalloids; tin and lead are metals. As a result, the elements of group
14 (Carbon family) hale a broad range of chemical behaviour."

Group 14 (IVA-The Carbon Family)


Study of Properties
Table 12.1
! • Group 14 (IVA-The Carbon Family)
Element z Electronic configuration Oxidation state
Study of Properties
Carbon C 6 [He] 2s 2 2p 2 +2,+4 • Compounds of Carbon
. con
c:.·
Si 14 [Ne] 3s 2 3p 2 +4
■ Compounds of Graphite
G:-nnanium Ge 32 [Ar] 3d 0 4s 2 4p 2
1
+2,+4
fn Sn [Kr] 4d1° 5s 2 5p 2 +2,+4
• The Carbon Cycle and Green-House
50
Effect
Lead Pb 82 [Xe] 4f14 5d10 6s 2 6p 2 +2,+4
• Silicon
Group 14 (IVA, carbon family) has ns2 np2 configuration of ■ Compounds of Silicon
the valence shell, their most common oxidation state is +4, as in
CCI4, SiC1 4, GeCl4, SnCl4 and PbCl4• These are volatile, • Compounds of Tin
J!1olecu1ar liquids in which atom uses tetrahedral sp3 hybrid • Compounds of Lead
1Qb1tals to form covalent bonds to the Cl atoms.
Carbon is a non-metal. Silicon and germanium are
ill!HalJoids and are used in semiconductors. Tin and lead are
l!l�tals.
Carbon and silicon have tetravalent nature in most of the
irr.mpounds when ns2 pair is also involved in bonding.
�nnanium, tin and lead show +2 and +4 oxidation states
ICEpending on whether ns2 pair is inert (and is not involved in
hnding) or ns2 pair is involved in bonding-a case of inert pair
�ffect. As we go down the group, inertness of ns2 pair increases
ltence tendency to exhibit +2 oxidation state increases and that
=' +4 oxidation state decreases. Thus, stability of
502 | Essential Inorganic Chemistry

Ge4+ > Sn4+ >>PbPb4+ and Ge2+ < Sn2+ < Pb2+
2+
The +2 oxidation state occurs for tin and lead and is
Also Sn2+ <<Sn
Sn4+4+ and Pb i2+ >Pb4 + the most stable oxidation state for lead. Both Sn2+(a<?) and
Thus, AG° < 0 for the reactions : Pb2+(ag) are common solution species, but there are na
Sn2+ ---- > Sn4++2e~ simple Af4+(ag) ions for any of the group 14 (IVA)
Pb4++2e~ ---- > Pb2+ elements. Instead Af4+ species exist in solution a-
Ge2+ ---- > Ge4+ +2e~ covalently bonded complex ions, e.g., Sil^", GeClg’,
Thus, Sn2+ salts are better reducing agents and Pb4+ Sn(OH)|“ andPb(OH)g".In general, the +4 oxidation state
salts are better oxidising agents. Physical properties of compounds are covalent, and the compounds with the Sn
group 14 (IVA) are summarised in Table 12.2 and and Pb in the +2 oxidation state are largely ionic.
Fig. 12.1.

Table 12.2 Physical Properties of Carbon Family [Group 14, (IVA)]


Property C Si Ge Sn Pb
Covalent radius/(pm) 77 117 122 140 146
Ionic radius M(IV)/(pm) 40 53 69 78
M(ll)/(pm)
73 118 119
(/E),kJ mol- 1086 786 760 707 715
(/E)2 kJ mol-1 2354 1573 1534 1409 1447
(7E)3 kJ mol"1 4622 3232 3300 2943 3087
(/E)4 kJ mol-1 6223 4351 4409 3821 4081
Electronegativity (Pauling) 2.5 1.8 1.8 1.8 1.8
Melting point/°C 4100 1420 945 232 327
Boiling point/°C 3280 2850 2623 1751
Electrode potential E°
A42*+2e"---- > M(s)
-0.14V -0.13V
M4++ 2e~---- > M2+
0.15 V 1.46V
Abundance in earth’s crust 0.020 28.2
(mass%)
0.0015 0 0002 0.0013

Density (g/cm3) 3.51 (diamond) 2.33 5.35 7.28 (white) 11.3


Occurrence Free state and Found in rocks as Rare element Found as tinstone Found as galena PoS
combined state silicon or silicate SnO2
Allotropic forms Crystalline (diamond Crystalline and Two crystalline forms White grey rhombic None
and graphite) and amorphous
amorphous
M-M bond energy 348.0 297.0 260.0 240
(kJ mol-’)
Nature Non-metal Metalloids Metalloids Metal Metal

5000-r 3~r 1100-r

4000 4-
10004- -
2— ▼p
3000------- ? Pauling o
K units E
900 4-
2000--------- 2
1+
iooo4- 8004—

0 0 700
C ' Si 'Ge'Sn' Pb' C Si 'Ge'Sn'Pb' C 1 Si 'Ge'Sn1 Pb1
(a) (b) (c)
Fig. 12.1 Some group trends of the group 14 elements; (a) Boiling point, (b) Pauling
•• •• • • • —• - -
electronegativity and (c) First ionisation energy.
Chapter 12: Group 14 (IVA — The Carbon Family) | 503

Occurrence On the other hand, both the 3rd and the 4th ionisation
energies vary irregularly, their sequence is
• Carbon is the seventeenth, (0.8%) and silicon the C > Ge > Si > Pb > Sn.
second most (27.6%) abundant element by mass in The decrease in the 1st and 2nd ionisation energies
the earth’s crust. Crude oil and coal are the most from C to Sn is because of the increase in atomic size down
common forms of combined and free carbon the group. The 3rd and 4th ionisation energies increases
respectively. Flakes of graphite occur in from Si to Ge because of poor shielding of 4s-electrons
metamorphased sedimentary rocks like quartz. offered by the 3d-electrons against the increased nuclear
Diamond occurs embedded in other rocks charge in Ge. All the ionisation energies are slightly
(as Kimberlite) Germanium is about 1-2 ppm in tin higher in case of Pb. This increase can again be attributed
about 2.1 ppm and lead about 13-16 ppm. to the poor shielding effect of the 4f and 5d orbital
Carbon 0.8% electrons. Further, the s-electrons being more
Silicon 27.6% penetrating, experience higher nuclear charge which
Germanium 1-2 ppm makes it difficult to remove them.
Tin 2.1 ppm • On account of high ionisation energies, simple Af4+
Lead 13-16 ppm ions of the group are not known.
• The tendency to form negative ions by gaining
Some stable isotopes electrons is only in carbon. For C and Si, stable
Carbon 12C, 13C,14 C divalent compounds MX2 are very rare.
Silicon 28Si, 29Si, 30Si
Germanium 70 Ge,72 Ge, 73Ge,74 Ge,76Ge Size
The covalent radii of the elements, as expected,
Tin Ten stable isotopes of mass numbers :
increase with increase in atomic number. This increase,
112,114,115, 116, 118, 119, 120, 122, 124 however, is not uniform. When we move from carbon to
Lead 204Pb, 206Pb, 207 Pb, 208 Pb silicon the covalent radius increases sharply, and then
increases in gradual.
Ionisation Energy
I The ionisation energies (Table 12.2 indicate that a Explanation
very large amount of energy is required to form a MA+ ion, A large increase in the size of silicon is due to the
therefore, not many ionic compounds are formed by introduction of the 3rd shell. When we go down to
froup 14 elements in the +4 oxidation state. On the other germanium, the 4th shell is introduced, no doubt, but the
hand, the promotion of one of the ns electrons to the np effective nuclear charge resulting from addition of
vacant orbital, as shown below, can allow a eighteen protons to the nucleus out balances the effect of
iftracovalency. In fact, most of the tetravalent compounds the additional shell leading to only a marginal increase in
formed by elements of this group are covalent in nature. size. Similarly, in case of Pb, the increase in covalent
Electronic configuration of carbon in the ground and radius is very small in comparison to that in Sn. This is
excited state is given as an example. further reflected in the ionisation energies and electron
Is 2s 2p affinities of these elements.
Electronic configuration of | j | | P | i
C in ground state
Electronegativity
Carbon has higher electronegativity as compared to
C in excited state
nnm [wn
Four unpaired electrons can form
other elements of the group. As a result, it can accept
(gain) electrons and can form negative ions of the
kind C^“ in acetylides and C4- in methanides, while
four equivalent covalent bonds — a the other elements in the group are unable to do so.
case of sp3 hybridisation. When we see the electronegativity trends in the group, we
find that the value decreases down the group as expected,
with Ge as cm exception whose electronegativity is slightly
DEEP Focus higher than that of Si. As explained above, this could be '
Since, the sum of the first two ionisation energies also due to the higher effective nuclear charge in case of Ge.
•Increases down the group, it becomes increasingly favourable
thermodynamically, for Sn and Pb to form compounds in
oxidation state +2 which are more ionic in character. Catenation
Catenation is a property by virtue of which elements
If we go through the data for ionisation energies given form long chain compounds by single or multiple bond
an Table 12.2, we find that the first and second ionisation formation between atoms of the same element. Carbon
energies decrease down the group from C to Sn as shows this tendency to a remarkable extent because of its
Expected, but they increase slightly when we move to Pb. small size and high bond energy (Table 12.2), Si and Ge to
504 | Essential Inorganic Chemistry

a lesser extent and Sn and Pb to a very small extent. In therefore, hard and non-conducting. Graphite
case of carbon we have alkanes, e.g., ethane propane (Fig. 12.2(b) and (c)) is more thermodynamically
butane, pentane, etc. These are chains of carbon hydrides stable allotrope.
having the general formula C„H2n+2, where n can be a
large number, alkanes with n = 100 are known. Carbon DEEP Focus
also forms a number of catenated halides, e.g., polyvinyl Graphite consists of planar hexagonal layers of carbon
chloride (PVC). Silicon forms silanes having the general atoms held together by relatively weak van der Waals’ forces.
formula SinH,,, +2, but n does not exceed 8. Its halids also Within each layer, a carbon atom is bonded to three other
shows catenation and have a general formula SinX2n + 2, carbon atoms by sp2 hybrid bonds and the additionalp electron
where if X = F, n = 1 to 14; if X = Cl, Br or I, n = 1 to 10. from each carbon forms an extended delocalised n-bondmg
Germanium forms germanes, with general formula system encompassing the entire layer. This makes graphite a
GenH2n+2, where n = l to 9. Germanium halides form good conductor of electricity in directions along the planes of
dimers, e.g., Ge2Cl6. Hydrides of Sn and Pb are much less carbon atoms. The covalent bonds in graphite account for its
stable, while SnH4 and Sn2H6 have been prepared, hardness; however, because the layers can slide over one
analogous methods fail for the preparation of PbH4. No another, graphite is slippery to touch and is effective as a
catenated halides are known for Sn and Pb. lubricant.
• There is steady decrease in the M—M bond energy as Silicon exists only in the diamond structure due to
we go down the group. tendency to form pit-pit bonds to itself. Fullerene,
[Fig. 12.2(d)] a carbon allotrope discovered in 1990 as
a constituent of soot, consists of spherical CK
Multiple Bonding molecules with the extraordinary shape of a soccer
In group 14, carbon is the only element capable of ball. The C60. ball has 12 pentagonal and 20 hexagonal
forming comparatively stable multiple bonds with another faces with each atom sp2-hybridised and bonded to
C atom and also with N, O, etc. Examples are alkenes three other atoms.
(>C=C<), alkynes (—C=C—), imines (^>C=N—), C60 was first prepared in 1985, by laser vaporisation of
nitriles (—C=N), ketones 0 C=O), etc. None of the other graphite (at about 10000°C) followed by sweeping of
elements of the group forms stable compounds containing the carbon atoms by sweeping of the carbon atoms by a
multiple bonds. Recently, some success has been achieved blast of high pressure helium gas and rapid cooling in
in synthesising compounds with />Si=Si< and ^>Si=C<C a vacuum chamber (Richard Smaller, Rice university
double bonds. Stoichiometric analogues like SiO2, SnO2 Texas and Harry kroto, University of Sussex, U.K
are(Me2SiO)n are known which are giant macromolecules Noble Prize Winners (1990).)
containing M—O—M linkages.
The tendency of formation of multiple bonds O
easily, in case of carbon, is because of the high
bond energy (Table 12.3) and a small atomic
radius of carbon as compared to other elements
of the group. So its p-orbitals can approach the
p-orbitals of the other C, 0 or N atom for an
effective overlap essential for n-bond formation. p-orbitals
(a) The crystal structure of diamond (b) The structure of a graphite sheet
Table 12.3 Bond Energies of Group 14 Elements (kJ mor1)
Bond c Si Ge Sn Pb
E—E 347 226 118 150
E=E 611 318
E=E 837
E—H 414 318 285 251
360 464 360
5=0 736 640
E* = Element C, Si, Ge, Sn or Pb.

Allotropes
• Carbon and graphite are two allotropes of carbon. In
diamond (Fig. 12.2(a)) each carbon atom is linked (c) Structure of a-graphite (d) Structure of Buckminster fuCere-'*
tetrahedrally to four, other carbon atoms by sp3 bonds
and a giant covalent network is formed. It is, Fig. 12.2 Allotropes carbon
Chapter 12: Group 14 (IVA — The Carbon Family) | 505

DEEP Focus Carbon hydrides will not ignite in air unless a flame is
put to them. Apart from silane, SiH4, the silicon
* Silicon also uses d-orbitals in multiple bonding, In case of
N(CH3)3, geometry is pyramidal, but in case of hydrides are less well behaved. SijjHg is
spontaneously flammable in air :
N(SiH3)3 it is planar. It is due to the fact that in the
SiaHg(Z) + 5O2(g)---- > 3SiO2(s) + 4H2O(Z)
latter, the lone pair of N-atom is transferred to the
empty d-orbitals of silicon (pn-dn overlapping): Alkane as well as silane are not hydrolysed however,
traces of alkali converts silane into hydrated silica
SiO2 • ziHjO.
• As we descend the group, acidic nature of the oxides
h3 ch3 SiH3
decreases and basic nature of the oxides increases.
ch3 SiH3 Oxides Nature Oxides Nature
pyramidal planar
(no jt-bonding) (prt-cfrt bonding) CO, co2 acidic SnO, SnO2 amphoteric
SiO2 acidic PbO, PbO2 basic
• The normal valency of the elements is four, but apart GeO, GeO2 amphoteric Pb3O4 amphoteric
from carbon, the elements can make more than four
bonds. This is because they make use of a set of Si, Ge, Sn and Pb form oxides MO2 • SiO2 is an infinite
d-orbitals in bonding. Thus availability of cZ-orbitals three-dimensional network solid of silicon and oxygen
(empty but electrons of ns2 and np2 can be excited into
atoms connected by single covalent bonds. GeO2, SnO2 and
this) is responsible for the ability of the elements,
except carbon, to make complex ions such as SiF6 PbO2 are all solids and exist in several modifications.
(Fig. 12.3). Illustration 1 Why is graphite a better electrical
Energy conductor than diamond and why does the conductivity of
3d graphite depend on direction?
very large Solution Graphite has a two-dimensional sheet like
energy structure in which each carbon atom uses sp2 hybrid
gap, 3d
can not orbitals to form trigonal planar o bonds to three
use 3d orbitals relatively small neighbouring carbon atoms. In addition, each carbon atom
in bonding > energy gap, can uses its remainingp-orbital, perpendicular to the plane of
use 3d orbitals
2p the sheet, to form a it bond. Because each carbon atom
3p J in bonding must share its k bond with its three neighbours, the n
carbon silicon electrons are delocalised and are free to move in the plane
of the sheet. As a result, the electrical conductivity of
Fig. 12.3 Energy difference between 2pand 3d energy levels
graphite in a direction parallel to the sheet is about
in carbon and between 3p and 3d energy levels in silicon
IO20 times greater than the conductivity of diamond. The
• The main chemical differences between silicon and conductivity of graphite perpendicular to the sheets of
carbon are carbon atoms is lower because electrons must hop from
“ Si does not form stable double bonds, one sheet to the next. The substance or crystal that show
~ Si does not form very stable Si—Si bonds, different properties in different directions is called
~ Si has vacant 3d-orbitals in its valence shell into Anisotropic.
which it can accept electrons from donor atoms.
• Some carbon compounds are less reactive than the Chemical Reactivity
corresponding compounds of the other members of the Elements in this group are relatively unreactive, but
group. CC14 has no interaction with I^O (hydrolysis) reactivity increases down the group. Lead often appears
whereas SiCl4 does by use of empty 3d-orbitals.
more noble (unreactive) than expected. It is due to a
• All the elements give hydrides. Carbon, of course gives
surface coating of oxide and partly due to high
an immense number, but silicon and germanium also
show a wide variety. overpotential for the reduction of to at a lead
CH4 methane (alkane) surface. The production ofHj fromH+ at a lead electrode is
SiH4 stannate kinetically unfavourable, hence a much larger potential is
GeH4 germane required than the standard reduction potential. Table 12.4
SnH4 stannane summarises reactions of the elements with different
PbH, plumbane reagents:
506 | Essential Inorganic Chemistry

Table 12.4 Reactivity of the Elements of Group 14 (IVA)


Uses
Reagent Reactivity 1. Diamonds are used in jewellery on account of their
H2O • C, S, Ge unaffected byH2O. high refractive and dispersive power. Due to their
• Sn reacts with steam forming SnO2 and H2; exceptionally hard nature, inferior quality
Sn + 2H2O ------- > SnO2 + 2H2 diamonds are used for industrial purposes mainly
(steam) in the form of grit and powder in cutting, drilling,
• Pb is unaffected by H2O probably because of grinding and polishing tools.
protective oxide film.
Graphite electrodes are used in the extraction of
Dilute acids • C. Si, Ge unaffected by dilute acid. aluminium. Graphite is also used for making
• Sn dissolves in diluteHNO3 forming Sn(NO3)2. crucibles, as a lubricant in heavy machines and as
• Pb dissolves slowly in dilute HCI forming PbCI2 pencil lead, etc. Very pure graphite is used as a
(sparingly soluble) and quite readily in dilute
HNO3 forming Pb(NO3)2 and oxides of neutron moderator in nuclear reactors.
nitrogen. Coke is mainly used in metallurgy. Carbon black is
• Pb does not dissolve in dilute H2SO4 due to used to strengthen rubber, as a pigment in inks,
formation of PbSO4 coating. paints, paper and plastics. Activated carbon has
Concentrated • Diamond is unaffected by concentrated acids,
enormous surface area and is used extensively in
acids but graphite changes to mellitic acid sugar industry as a decolourising agent. It is a very
(C12H6O12) also called benzene efficient adsorbent it absorbs organic pollutants
hexa-carboxylic acid (A) from drinking water, noxious gases from the air. It
COOH is also used in the treatment of sewage water and as
a catalyst. Amongst the compounds of carbon, CO^
HOOC COOH finds an important use as a fire extinguisher and in
A: refrigeration as dry ice.
HOOC COOH 2. Silicon finds uses in both elemental and combined
COOH forms. Very pure silicon is used in making
with hot concentrated HNO3 and to graphite semiconductors and in solar cells which are
oxide with hot concentrated HF/HNO3. becoming increasingly important. Silicon becomes
• Si is oxidised and changes to SiF4 by hot p-type semiconductor when doped with a group 13
concentrated HNO3/HF. (Al, Ga) and an n-type semiconductor when doped
• Ge dissolves slowly in hot concentrated H2SO4 with group 15 (P, Ajs) Hyperfine Si is one of the
and HNO3.
purest materials used- in the production of
• Sn also dissolves in concentrated acids. transistors.
• Pb does not dissolve in concentrated HCI due
to formation of PbCI2 coating. Silicates are used in glass and cement
Alkalies manufacture.
• Carbon is unaffected by alkalies.
• Si reacts slowly with cold aqueous NaOH Silicones are also widely used in lubricants. Their
solution, and readily with hot solution forming inert nature makes them ideal for use in gas
SiO4". chromatography, e.g., methylphenyl silicones.
• Sn and Pb are slowly attacked by cold alkali, Germanium is used largely in transistor
and rapidly by hot alkali, giving stannates technology. However, its use in optics is growing;
Na2[Sn(OH)6] and plumbatesNa2[Pb(OH)6].
germanium is transparent in infrared region and is,
Halogens • Diamond is unreactive, but graphite reacts therefore, used in infrared windows, prisms and
forming (CF)^. lenses.
• Si and Ge form volatile SiX4 and GeX4.
• Sn and Pb are less reactive. Sn reacts with Cl22 • Silicon is extensively used in steel making as a
and Br2 in cold, and with F2 and l2 on warming. deoxidiser. Ferrosilicon is the usual form. Steels
Lead reacts with F2 in cold and with Cl2 on with high Si content are corrosion resistant.
heating forming PbX2.
3. Due to its resistance to corrosion, tin is used to coat
Air • Graphite burns above 7OO°C (CO) Diamond mild steeel plate. Tinning of copper and brass
burn above 900°C (CO). Si, Ge, Sn, burn on utensils has been an age old practice in India. Tin is
strong heating (At 1400°C, Si also forms SiN also used for making various alloys (Refer Appendix)
and Si3N4) Finely divided lead powder is
pyrophoric (combines highly exothermically
like bronze, pewter, gun metal, etc. Tin-lead alloys
with oxygen) but the surface is practically are fusible and low melting. Solders, which are low
inactivated by a thin protective layer of oxide, melting alloys, are used to join pieces of metal
chloride, sulphate. PbO is formed above together. Soldering is a very common practice for
600°C. joining components of an integrated circuit. Sn—Pb
alloys are also used as bearing metal or type metal.
Chapter 12: Group 14 (IVA — The Carbon Family) | 507

Lead is used as an inert material, e.g., for building Dehydration of formic acid with cone. gives CO
roofs, and for cable sheathing. Lead gets covered laboratory method).
with a layer of PbSO4, which is insoluble and does 0
not get attacked by common reagents. Lead pipes II Cone. SOa4.A
Ho2SO
Cone. H ,A
were used in municipal water supply at one time. H—C-OH —------ 2 4 > HjO + CO
Soft water, however, tends to dissolve lead and may • CO can be detected because it bums with a blue flame.
cause lead poisoning. Therefore, galvanised iron
• CO reduces I2O5 to Ig
(Gl) pipes are now used for water supply.
5CO + IgOg -----> I2 + 5CO2
Due to their low chemical reactivity, lead coated
chambers are used in many industrial preparations Jg thus liberated is determined by titration with
like that of sulphuric acid. In view of its high Na2S2O3 (hypo)
density, lead is also used in making shielding 2Na2S2O3 + Ig -----> 2NaI + Na2S4O6
blocks for radioactivity experiments. Organolead This is a quantitative method of estimation of CO.
• compounds, e.g., tetraethyllead, Et4Pb and PdCl2 is also reduced to Pd
tetramethyllead, Me4Pb, are used as anti-knock
additives to petrol for internal combustion engines. PdCl2 + CO + HgO-----> Pd + 2HC1 + CO2
However, in recent times, because of their polluting Following reduction takes place in blast furnace :
nature, many countries have reduced or altogether CuO + CO---- > Cu + CO2
stopped their use.
FegOg + 3CO-----> 2Fe + 3CO2
One of the major uses of lead still is in the storage • CO is toxic because it forms a complex with
batteries. Storage batteries also called lead-acid haemoglobin in the blood, and this complex is
batteries have a supporting grid of lead-tin alloy
more stable than oxy-haemoglobin. This
(91:9) with PbO2 as the oxidising agent and spongy
prevents the haemoglobin in the red blood
Pb as the reducing agent. The electrolyte is dilute
corpuscles from carrying oxygen round the
H^.
body. This causes an oxygen deficiency, leading
Following table summarises uses of elements of group to unconsciousness and then death. The blood of
14 (IVA): victims of carbon monoxide poisoning is a
Elements
brilliant red-pink colour rather than the dull
_________________ Uses_________________
red of normal blood.
Carbon Diamond in jewellery and glass cutter, graphite as
lubricant and electrode Hb—O2 + CO Hb-CO + O2
Slicon As a semiconductor, in silicone waxes and polymers, oxyhaemoglobin carboxyhaemoglobin
as silicates in the ceramic and glass industries.
Germanium As a semiconductor. • Although CO has no acidic or basic properties
Tn In alloys, as a protective coating for iron and steel. (in terms of proton H+), it can behave as a Lewis
Lead In alloys, in car batteries.__________________ base—electron-pair donor, and thus forms
complexes with transition metals (mostly with zero
oxidation state):
Ni(CO)4, Fe(CO)5, Cr(CO)6
Compounds of Carbon These are called carbonyl compounds and can be made
Carbon is unique in the variety and complexity of
by:
compounds that it can form. The molecules that are 28°C 230°C
responsible for the growth and development of living Ni + 4CO Ni(CO)4 ------- > Ni + 4CO?
organisms, such as vitamins, proteins, enzymes, This is Mond process of the purification of Ni metal.
hormones and DNA, are structures built from chains of
carbon. 200°C
Fe + 5CO 4 Fe(CO)5
under pressure
Oxides of Carbon 2Fe(CO)5
photolysis
Fe2(CO)9 + CO
• In oxides of carbon there are px-px multiple bonds
between carbon and oxygen. • Bond order between C and 0 is three and metal
stable oxides : CO and CO2 carbonyl bond is dative
less stable : C3O2 (carbon suboxide), CsO2, C12O9 M <-C=O
least stable : C^O (graphite oxide), Q2O3 • CO is sparingly soluble in water and is neutral oxide.
• CO is a colourless, odourless, poisonous gas. It is • CO is an important fuel
formed when carbon is burnt in limited supply of air.
co+iq,
&
—> co,, AH0 = -282.25 kJ mol’1
2C + O2 ---- > 2CO
508 | Essential Inorganic Chemistry

It is associated with following fuels : pressure. This property is used to carbonate drinks
Water gas : Equimolecular mixture of CO and H2 (Coca-Cola etc.)
CO2 + H2O----- > H2CO3
C + H2O -red 11u-> ™
1—hot. co +. h
TT2, AH° = +13lkJ
H2CO3 H+ + HCO3 (salt: NaHCOJ
Producer gas : A mixture of CO and N2 HCO3 H+ + coj- (salt: Na^Op
C +O2 + 4N22 -----> CO2 + 4N2 and thus forms two series of salt.
red hot '----- *-
coke air NaHCO3 : an acid salt
+c
Na2CO3 : normal salt
A hydrate CO2 • 8H2O can be crystallised at 0° C under
2CO + 4N2
(producer gas) a pressure of 50 atm CO2.

Producer gas is less efficient than water gas as only


DEEP Focus
part of the gas will burn. The overall reaction is • Free carbonic acid has not been isolated but its acid salt
exothermic: (NaHCO3) and normal salt (Na2CO3) have been isolated.
2C + O2 ----- > 2CO, AH° = -221kJ An etherate (A) of probable composition OC(OH)20(CH3)2
Coal gas : A mixture of CO, H2 CH, and CO2. is obtained by reacting Na2CO3 with HC1 in presence of
dimethyl ether at very low temperature (-50°C).The white
Water gas Co. H2 crystalline solid decomposes at 5°C. H2CO3 is believed to be
present in the gaseous phase by thermal decomposition of
Producer gas Co, N2 nh4hco3
Coal gas Co, H2, CHj, CO2 NaoCOq + 2HC1---- > 2NaCl +H„CO,
(A)
• CO + S -> COS (carbonyl sulphide)
OH OH
CO + Cl2 -> COC12 (carbonyl chloride) o=c + (CH3)2O---- > O=C •O(CH3'23)2
• CO2 is a colourless and odourless gas and is generally OH OH
recovered as by-product from (A)
— the manufacture of H2 NH.HCO„ A NH3(£)+H2CO3(g)
4 O

co + H2O CO2 + H2
• Biologically, CO2 is important in photosynthesis
CH4 + 2H2O-----> CO2 + 4H2
where the green parts of plants manufacture glucose
— the fermentation processes in breweries sugar
C6H12O6 2CO2 + 2G2H5OH sunlight
6CO2 + 6H2O ■> 6O2 +CgH,2Og
• CO2 gas can be liquefied under pressure between glucose
-57° C and +31° C. Solid CO2 called ‘dry ice' sublimes The reverse reaction occurs during the process of
directly to the vapour state (without going through the respiration, where animals and plants release energy:
liquid state) at -78° C under atmospheric pressure. C6H12O6 + 6O2 ----- > 6CO2 + 6^0 + energy
Dry ice (also called cardice) is used to freeze meat, • CO2 turns lime water milky which changes to
frozen foods and ice cream, and in the laboratory as a colourless on passing excess of CO2 in it
coolant.
Ca(OH)2 +CO2 ----- > CaCO3X + H2O
• CO2 is used in many fire extinguishers:
lime water milky
- Water based extinguishers have a sealed bottle of (white ppt)
acid immersed in a solution of HCO3 (say
NaHCO3). When the extinguisher is struck, the CaCO3 + HgO + CO2 ----- > Ca(HCO3)2
milky soluble
bottle bursts, and reaction between acid andHCO3
produces CO2 instantly in the form of watery spray • Carbon suboxide C3O2 is a foul smelling gas with
(or in a foam if other chemicals have been added). boiling point 6° C.
HCO3 + H+ ----- > HgO + CO2 CH2(COOH)2 0==c=c==c==0 + 2H,0
- Other extinguishers use pure CO2 under pressure. malonic acid 150°C
The gas given out covers the fire and prevents linear molecule stable at -78°C.
oxygen in the air from reacting with hot materials.
C3O2 + 211,0 ■> CHXCOOH),
• CO2 is an acidic oxide and reacts with bases, forming malonic acid
salts:
C3O2 + 2HC1 > CH2(COC1)2
Ca(OH)2 + CO2 ----- > CaCO3 + H^O malonyl chloride
Solution of CO2 in water behaves as weak dibasic acid,
C3O2 + 2NH3 ----- > CH/CONH^
the solubility in water increases remarkably under
malonic amide
Chapter 12: Group 14 (IVA — The Carbon Family) | 509
Fe(CO)5
I Carbides
Ni(CO)4
230°C
\n! ZCr • Solid compounds of carbon with elements other than
28°C
cone H2SO4
hydrogen are generally called carbides.
C + O2 (limited)------ CO HCOOH • Ionic or salt-like carbides are formed by the most
I2O5/
\ ^\Pe2O3
electropositive metals such as alkali and alkaline
earth metals and aluminium:
Cl2 PdCI2
I? + co2 H2O Fe + CO2 2BeO + 2C 200°IC> Be2C + 2CO
02 \
COCI2 4A1 + 3C electric furnace
CO2 + Cu *1 o
Pd + CO2
A nh31a Both Be^ and A14C3 are called methanides because
CaCO3 CO2 NH2CONH2 (urea)
they react with H^O yielding methane:
%
Be^ + 41^0---- > 2Be(OH)2 + CH4
h2o \ H2O A14C3 + 12H2O---- > 4A1(OH)3 + 3CH4
CeH^Oe
Na2CO3 CaC-j (calcium acetylide) is obtained by strongly
H2CO3 HCO3 cof" heating lime and coke:
-50°C HCI
(CH3)2O CaO + 3C 2000°C
2000°C> CaC, + CO
H2CO3(CH3)2O It is used to manufacture acetylene and calcium
cyanamide:
Compounds of Graphite CaC, + 21^0---- > Ca(OH)2 +CHe=CH
CaC^ + N2 n0QCC> CaCN2 + C
Type I: Electrically Conducting Compounds
Compounds in which the n-electron system of graphite Calcium cyanamide, Ca(NCN), is used as nitrogenous
is retained and thus electrical conductivity is retained due fertiliser, and to make urea:
to mobile electrons (or even many increase). In such CaCN2 + HrjO + CO2 ---- > CaCO3 + NH9CN
compounds separation between layers increases by as
much as 10A. Such compounds are called Lamellar
compounds.
N^CN+H^° NH£°NH>
The cyanamide ion [N =C=Nf “ is isoelectronic with
Preparation CO2 and has the same linear structure.
• (i) Such Lamellar compounds are formed by insertion of (?~ (acetylide) has multiple bonding of the type
various atoms (as alkali metals), molecules or ions "C^C-
(asFeCl3) between the graphite layers. while CaCj has NaCl type of lattice with Ca2* replacing
K)
Na+ andC^- replacing Cl".
C+K —> C60K C24K c8k Mg2C3 has (-C=C—C3-) ion and gives propyne on
(graphite) deep blue steel blue bronze
reaction with H^O.
(ii) By electrochemical method Mg2C3 +4H2O---- > 2Mg(OH)2 + CH3C=CH
Reduction: nC + M+ + e“ M+ + Cn • Interstitial carbides are formed by transition
Oxidation : nC + X~ C* + X metals in which carbon atoms occupy tetrahedral
holes in the close-packed arrays of metal atoms. These
The high electrical conductivity may arise from are generally very hard, electrically conducting and
transfer of electron(s) from the metal to the n-orbitals of have very high melting points (tungsten carbide
the carbon plane, where they are free to move under an (WC)). Carbides of Cr, Mn, Fe, Co and Ni are
electric field.
hydrolysed by water or dilute acids.
Type II : Electrically non-conducting • Covalent carbides are formed by metalloids like Si
and B. SiC and B4C (and more correctly as B^C,) are
Compounds extremely hard, infusible and chemically inert. SiC
Graphite changes to (CF)n — a white compound when has a diamond like structure (hence also called
it is treated withF2 at 400-600°C. It is found by X-ray study artificial diamond) in which C and Si atoms are
that in (CF)n layers separation is about 6.6 A. Also carbon each tetrahedrally surrounded by four of the other
atoms in the layers have sp3 hybridisation with boat or kind of atoms under the name carborundum, it is
chair conformation and (C—F) covalent bond exists. Due used in cutting tools and abrasives. It is produced by
to loss of x-electron, compound (CF)„ is non-conducting.
510 | Essential Inorganic Chemistry

heating quartz or sand with an excess of coke in an Illustration 1 Explain coal-gasification.


electric arc furnace at 2000—2500° C.
Solution One of the most promising methods for
SiO2 + C-----> Si + 2CO
making coal a more efficient and cleaner fuel involves the
Si + C-----4 SiC
conversion of coal to a gaseous form, called syngas for
SiC reacts with NaOH in presence of air and with Cl2 “synthesis gas”. This process is called coal gasification.
at 100° C. In the presence of very hot steam and air, coal decomposes
SiC + 2NaOH + 2O2 4 Na2SiO3 + CO2 + HjO and reacts according to the following simplified scheme
SiC + 2C12 4 SiCl4 + C C(s) + HjXXg)----- 4 CO(g) + ^(g)
Boron carbide is even harder than silicon carbide and C(s)+2112(g)-----4 CH4(g)
is used both as an abrasive and as a shield from radiation.
— red solid The main component of syngas is methane. Under
*
suitable conditions, CO and H2 combine to form
A14C3 — pale yellow solid
methanol
CaQz — colourless
SiC — pale yellow (pure) CO(g) + 2112(g)-----> CH3OH(Z)
Syngas is easier than coal to store and transport. It is
Cyanides not a major source of air pollution because sulphur is
• The cyanides contains the anion group *C=N’. The removed in the gasification process.
cyanide ions are extremely toxic because they bind
almost irreversibly to the Fe(III) ion in cytochrome
oxidase, a key enzyme in metabolic processes involved The Carbon Cycle and Green-House
in the oxidation of food molecules. With CN" attached
to the iron, the enzyme is unable to function, cellular
Effect
energy production comes to a halt, and rapid death • Carbon cycle is shown by the following scheme
follows. HCN being volatile (b.p. 26°C is more (Fig. 12.4)
dangerous and can cause death within minutes. CO2 in the atmosphere
Dissolves
• Cyanide ions are used to extract gold and silver:
Photosynthesis
4Au(s) + 8CN-(ag) + O2 (g) + 2H2O(Z)----- 4 | Sea |
| Green plants | | Animals]
(in mineral) I
Marine
4[Au(CN)2 ]~(a<7) + 4OH (aq) creatures Microorganism
2[Au(CN)2r +Zn(s)-----4 [Zn(CN)4f“ + 2Au(s) Decay Decay in soil
| Decay|
• Fe2+(aq) combines with CN" forming ferrocyanide (aq)
ionJFefCN)^-.
Burn
Fe2+(aq) + 6CN-(ag) 4 (FefCN^Maq) Fossil Fuels

Fig. 12.4 Carbon cycle


DEEP Focus Energy from the sun reaches the earth in the form of
• Mixture ofCu2+ (ag)andCd2+ (07) can be separated by using light. Neither CO2 nor HjO vapour absorbs the visible
CN’(aq). Cu2+ form stable [Cu^CNjJ3- and unstable
light so they do not prevent it from reaching the
[Cd(CN)4l2- complexes. On passing H2S gas yellow ppt of
CdS are obtained without affecting [Cu(CN)4]3“ complex. surface of the earth.
Cu2+ + 2CN- ---- > Cu(CN)21 • The CO2 as well as HgO absorbs strongly in the
yellow ppt infrared region, and its presence in the atmosphere
2Cu(CN)2 2CuCN I + (CN)2T decreases the loss of heat from the earth by radiation.
white ppt cyanogen This global warming is called the ‘green house
(poisonous gas)
effect’ (other gases, including the oxides of nitrogen
I CuCN +3CN"
excess
[Cu(CN)4]3-
stable
from car exhausts, freons from aerosols and
refrigerators and methane from bacteria in the soil
Cd2+ + 4CN“ [Cd(CN)4]2- and in the rumen of cows, also add to the green house
unstable effect). The concentration of atmospheric C0.2 has
[Cd(CN)4]2- Cd2+ + 4CN~ increased by 10%. This is resulting in the increase in
Cd2+ + S2’ CdSi the mean temperature of the earth by 2.5° C, varying
yellow ppt from 2° C at the equator to 4°C at the poles (Global
Chapter 12: Group 14 (IVA — The Carbon Family) | 511

Warming). This could have dramatic effects on the (a)


climate. The increased temperature would cause more
evaporation of water and hence more rain, flooding
and tropical storms in certain parts of the world. CO2
may dissolve in water and pH of the soil is affected.
(b)
CO|- + CO2 + HjO---- > 2HCO3
earth
Fig. 12.5 The ‘Green-House Effect’ (a) Sunlight received by earth, (b)
Infrared radiation reradiated by the surface of earth, (c) Infrared
radiation emitted to space, following absorption of some of this

Target Practice 1
1. Tetra covalent nature of the element with following valence 6. In the laboratory where you perform chemistry practical,
shell electronic configruation is explained on the basis of you would have noticed NaOH and Ca(OH)2 containing
reagent bottles with a white coating inside. This coating is
of and formed by reaction
__1B 7. Ni is metal but dissolved in CO gas due to formation ofA. A
2p2 decomposes into metal
(a) A is
2s2
(b) A is formed at a temperature
2. Covalent radii of (c) A decomposes into metal at a temperature
C Si Ge 8. What are the components in water based fire-extinguisher
77 pm 117 pm 122 pm (a) (b) and reaction releasingCO2 is
L 40 pm —IL 5 pm I 9. Mention two biological/medical uses of CO2
Show that there is difference of 40 pm while going from C to (a)
Si but there is difference of only 5 pm while going from Si to (b)
Ge. This is due to
10. Carbide of magnesium yields propyne on reaction with
H2O. Write bonding structure of this carbide ion.

3. (a) Write the element with maximum catenation property.


11. Silver containing ore can be converted into silver in
(b) Write the formulae of compounds/ions in which carbon cyanide process, with following steps. Write the species
forms with nitrogen. formed in
Ag +CN’ — (a)
(i) single bond
(a) + Zn ---- > Ag+ (b)
(ii) double bond
(iii) triple bond 12. In the complex [Fe(CN) J3’ and [Fe(CN)RT
D O

4. You have seen electrican joining two points in a circuit by (a) Write number of
using a wire which is low melting and immediately unpaired electrons
solidifies. (b) What is magnetic
(i) What is its name? moment
(ii) What is the composition?
13. When KCN (excess) is added to aCu2+ final species formed
5. Write name/formulae and use of the following is and on passing H„S gas into it
Name/Formula Use 14. pH of neutral water is considered as 7. When green house
(i) Dry ice gases dissolve in water, pH will
(ii) Semi conductor 15. There is strong warning due to global warming “New
(iii) Solder York City will submerge”. Comment in three or four lines
(iv) TEL
(v) CNG
(vi) LPG
512 | Essential Inorganic Chemistry

Answers
1. Excitation of 2s-electrons to 2p-orbital. 6. Na2CO3,CaCO3

excite
mu
jjap’
2NaOH + CO2 —> CO2 ---- > Na2C03+H20
Ca(OH)2+CO2 ---- > CaCO3+H2O
There is also reaction between silicates (of glass) and alkali.
1lj2p2 2s1 7. (a) Ni(CO)4 (b) 28°C (c) 230°C
8. (a) Acid (like CH 3COOH)
2s2 (b) NaHC03
2. Refer text HC03+H+ ---- > H2O + CO2 I
3. (a) Carbon 9. (a) Photosynthesis
I (b) CO2 (as HCO3) ’s an important component of buffer of
(b) (i) —C—N-H blood which controls pH.
H CO2+H2O H2CO3
(ii) >C=N—H h2co3 H+ + HCOj
(iii) —C=N 10. -C=C—C3’
4. (i) Solder (ii) Sn and Pb 11. Ag++2CN“ ---- > [AgfCN)2]“
2[Ag(CN)2r + Zn ---- > Ag £&n(CN) J
5. Name/ Formula Use
__ . O’)
(i) Dry ice Referigerant
12. [Fe(CN)6]2‘ [Fe(CN)6]4-
for
preserving food (a) 1 0 i

(ii) Semi Ge Electronic diode triode in (b) J3BM 0


conductor computer 13. [Cu(CN)4]3-
(iii) Solder Sn/Pb joining circuits no precipitation takes place being a stable complex.
(iv) TEL Tetra ethyl lead Anti-knocking compound 14. Decrease.
for petroleum products 15. Due to global warming, glaciers are melting and due to this
(v) CNG Compressed Eco-friendly fuel for sea-level will be certainly increasing which may cause flooding
natural gas automobiles of cities situated at the bank of sea. New York is one of them.
(vi) LPG Liquefied Cooking gas
petroleum gas
i

Practice Exercise 1
1. (a) Which theory other than valence bond theory could be (B) + H2O ---- > (C) -^4 (D) (A) excess^ (£)
used to explain the bonding in SiF2“ ?
(b) Predict the shape of this ion. (A)+NH, (F)
V

2. Use E° values given in Table 12.2 to decide whether A


following reactions are feasible or not? 9. h2c2o4 gas (A) + gas (B) + liquid (C)
(a) Pb4+ +Sn2+ ---- > Sn4+ +Pb2+ oxalic acid
(b) Sn4+ + Pb2+ ----
—>»Pb Pb4+ + Sn2+ Gas (A) burns with blue flame and is oxidised to gas (B). Gas
3. Explain: (B) turns lime water milky.
A NH3’A
(a) (CH3)3N is pyramidal in shape while (SiH3)3N is Gas(A) + CI2 ---- > (D) nh3> -> (B) < 33’ - (B)
planar.
(b) Graphite is a good conductor and diamond is Identity (A) to (E) and explain reactions.
non-conductor. 10. A binary salt (A) on reaction with H2O gives B(aq) and C(g).
4. Sn and Pb are amphoteric. Explain. (C) gas on passing into ammoniacal AgNO3 gives white ppi
5. Sn reacts with HN03 under different conditions. What are D(aq). CO2 gas turns B (aq) milky. Identify (A), (B), (Q and
various products? (D) .
6. Sn and Pb dissolve in hot NaOH solution. Write chemical 11. CaCO3(s) A(s)+B(g)
reactions. carbon, A
7. Every year some people die by being in a room containing a A(s) C(s)+D(g)
faulty heater that uses coal, gas or oil. How might the death (C)+N2 ---- > E(s)
occur?
(B) + H2O + (B) ---- > (F) + CaCO3
8. Identify (A) to (F) in the following:
(F) + H2O ---- > (G)
CaCO, ---- » (A)+ (B) Identify (A) to (G).
(g) (solid)
Chapter 12: Group 14 (IVA - The Carbon Family) I 513
12. Give th e number of cr and 1t bonds in (i) �- (ii) c:- and 15. 10 L of a mixture of CO and CO 2 gas is passed over heated
(iii)CN;-. coke when volume increases by 2L. What is % composition
13. (i)BaC2 + N 2 � (A) of the mixture ?
16. CO 2 goes to the atmosphere (greenhouse effect) and thus
Ui) CaC2 + N 2 � (B)
gets dissolved in H 2 0. pH of the water increases or
Identify (A) and (B). decreases and what is the effect on the fertility of the soil?
14. Is CO 2 the only atmospheric gas responsible for the 17. Which of the following gases qualify as a greenhouse gas :
greenhouse effect. CO , NO, N02 1 Cl 2 1 H 2 , Ne?

Answers
1, (a) Molecular orbital theory (b) Octahedral E : CaCN 2 (calcium cyanamide nitrolim)
2. Only (a)feasible due to the fact that stability ofPb 2+ > Pb 4+ and F: NH 2CN (cyanamide)
Sn .. > Sn 2•
G: NH 2CONH 2 (urea)
Eo
12. (i) -c�c-
one a, two 1[ (ij) -c�c-e3 - two a, two 1[
AJso, Sn 2+
--+ Sn 4+ + 2.e- ---0.15 V (iii) -N=C=N- two a, two 1t
Pb.,. + 2,e- --+ Pb 2 + 1.46 V 13. A: Ca (N =C=N) Calcium cyanamide
Pb +Sn
.,. 2+
� Sn • 4
+ Pb 2+
E:u = + 1.31 V Ba cc-
N)2 barium cyanide.
Thus, spontaneous. 14. There are several gases that are even stronger infrared
3. See text. absorbers than CO 2 [for example CH ,c , 0 3, N 20, chlorofluoro
(. Both metals react with NaOH (base) as well as with HCI (acid), carbons (CFC)]. About one-half of the anticipated greenhouse
hence are amphoteric : effect is due to CO 2 and the remainder of these other gases.
Furthermore, the atmospheric concentration of some of these
Sn+2NaOH+H 20 --+ Na 2Sn03 + 2H 2
other gases (especially CFC) have been growing at a faster rate
Sn + 2HCI --+ SnCl 2 + H 2 than CO 2 •
Similarly with lead. A
15. CO 2 +C--. 2CO
5. 4Sn + 10HNO 3 ---. 4Sn(N03 )2 +NH 4 N03 + 3H 20
dilute let CO 2 = XL
Sn +4HN03 H 2Sn0 3
---. +H 20 + 4N0 2 CO= (10 - x) L
cone meta stanbic acid CO due to passage over heated coke= 2r (from CO 2 , no effect on
6. Sn+ 2NaOH + H 20 --+ Na 2Sn03 + 2H 2 CO, already in mixture)
Or Sn+2NaOH+4H 2 0 --+ Na 2 [Sn(OH)6] + 2H 2
Thus, lOx + 2r = 12
Similarly with Pb.
x=2L
7. Hthe flue (chimney) to the heater becomes blocked, or there is
Thus, CO 2 = 2L,20%
insufficient ventilation, the fumes of CO and CO 2 build up in
the room. Unless the person lies on the floor, it is the CO that CO =8L,80%
kills. 16. CO 2 +H 20 � H 2C0 3
8. CA): CO 2 (B): CaO (C): Ca(OH)2 (D): CaC03
H 2COa --+ W + HC03
CE):Ca(HC03)2 (F): NH 2CONH 2
9. CA ): CO (B): CO 2 (C): H 20 (D) : COC1 2 (E): NH 2CONH 2 m•J increases hence pH decreases. Fertility of the soil also
binary) (B): Ca(OH)2 (C):C 2H 2 decreases.
10. CA) :CaC2 (Ca 2•, �-
17. CO, NO and N0 2
W ):C 2Ag 2
11. A: CaO B : CO 2 C : CaC2 D : CO

Silicon The silicon used for making solid-state semiconductor


devices such as transistors, computer chips, and solar cells
Isolation of Silicon must be ultrapure, with impurities at a level of less than
Silicon is a hard grey, semiconducting solid that melts
10-7 % (1 ppb). Silicon is purified by converting it to SiCl4 , a
at 1410°C. It crystallises in a diamond like structure but
does not form a graphite allotrope because of the relatively
volatile liquid (b.p. 58 °C) that can be separated from
poor overlap of silicon n orbitals. In nature, silicon is impurities by fractional distillation and then converted
generally found combined with oxygen in Si02 and in back to elemental silicon by reduction with hydrogen.
Various silicate minerals. Si(s)+ 2Cl2 (g) � SiC14 (l)
It is obtained in elemental form by reduction of silica
sand(Si02 ) with coke (C) in an electric furnace: SiC14 (g)+ 2f4(g) heat, Si(s)+ 4HCI(g)
Si02 (l) + 2C(s) heat• Si(s) + 2CO(g)
514 | Essential Inorganic Chemistry

CO
graphite is good electrical conductor but silicon
SiO2
C.A
Si J is a semiconductor.
Diamond Insulator
Cl2
Graphite Conductor
SiCI4 fractional Silicon Semiconductor
+ pure SiCI4
Impurities | distillation
• Silicon is one of the most abundant of elements, being
ultrapure zone H2,A the essential ingredient of a large number of minerals
Si Si
refining that make up the earth’s surface.
The silicon is purified further by a process called zone Silicates
refining (Fig. 12.6). In this process, a heater melts a
• The majority of the compounds consist of silicates, in
narrow zone of a silicon rod. Impurities concentrate in the
molten zone and as the heater sweeps slowly down the rod, which silicon and oxygen make giant three
ultrapure silicon crystallises at the trailing zone of the dimensional structures. They are all based on the
molten zone. same base building block made from one silicon atom
silicon rod
and four oxygen atoms at the corners of a tetrahedron.
furnace
These SiO4 tetrahedra may be :
1 - separate units,
sample drawn - joined into chains or rings in groups of 2,3,4 or 6,
slowly through
fa?:::::::: the furnace ~ joined together into long single or double chains,
- arranged in sheets,
/ furnace
_ linked into three dimensional network.
zone where less impurity at
after many passes, melting occurs • The name ‘silica’ covers an entire group of minerals,
most of the impurity the end
collects here which have the general formula SiO2, the most common
Fig. 12.6 Zone-refining. The sample is passed through the
of which is quartz. Quartz is a framework silicate with
furnace many times. The impurities collect at one end of the SiO4 tetrahedra arranged in spirals. The spirals can
turn in a clockwise or anti-clockwise direction—a
feature that results in there being two mirror image,
Compounds of Silicon optically active, varieties of quartz. Quartz is used as a
• A silicon atom, like a carbon atom, can form four bonds piezoelectric material (crystal oscillators and
simultaneously. It uses all of its valence electrons transducers). Silica get and fumed silica are
(3s2 3 p2) in an sp3 hybridisation scheme, and amorphous forms of silica. Silica gel is formed when
crystallises in an fee structure similar to diamond. sodium silicate is acidified, and its dehydrated form is a
There is no allotrope of silicon equivalent to the drying agent in chromatography and catalyst support
graphite. Diamond is an electrical insulator, Some silicate minerals are summarised in Table 12.5.

Table 12.5 Some Representative Silicate Minerals


Type______________
Anion type Mineral Composition
1. Ortho silicate (neso-silicates) SiOf zircon olivine ZrSiO4,Mg2SiO.'4
2. Pyrosilicate (soro-silicates) Si2Of- thortveitite Sc2Si2O7
Si6o^- beryl Be3AI2Si6O,8
3. Pyroxene Si—0 chains spodumene LiAI(SiO3)2
4. Amphibole Si—O double chains hornblende (Ca, Na, K)2_3 (Mg, Fe. Al)5 (Si, Al)2 SigO^OHh
5. Mica Si—O sheets muscovite KAI2(AISi3O10)(OH)2
6. Zeolite Si—0 three-D network natrolite Na2AI2Si3O12 -2H2O
7. Single-chain silicate SiO4" diopside [CaMg(SiO3)2]„
8. Double-chain silicate SiO4" asbestos [Ca2Mg5 (Si 40^)2 (OH)2]„
9. Sheet-like silicate SiO4“ talc [Mg3Si4O10 (OH)2]n
Chapter 12: Group 14 (IVA-The Carbon Family) I 515
• The vast majority of silicates are insoluble in water; Ring anions : (1) Si 3o;- -three SiO4 tetrahedra
only the simple alkali metal silicates are soluble. linked together as in benitoite (BaTiSi 3O9) and in
• The extreme stability of the silicates is due to the wollastonite (Ca:,'Si 3O9).
donation of extra electrons from O atoms into vacant (2) Si 6O!:--six SiO4 tetrahedra linked together as in
3d orbitals of Si. beryl (Be 3Al2SisO1a)
• In many common minerals, called alum.inosilicates,
Al atoms replace some Si atoms with very little
structural change. In such cases univalent cations :'
,..:......
...-..\

such as K + or Na + are also joined to balance positive l/
................
charge of Si.

DEEP Focus
Following points are helpful in understanding the
structural principles in silicates :
(a) The strong Si-0 bond (t:,11 = 466 kJ moi-1) may be
considered 50% ionic and 50% covalent from electronegativity
I. difference (which is 3.5 - 1.8 = 1.7).
Cb) The radii ofSi4+ ion and 0 2- ion are 0.40 and 1.40 A
respectively. The radius ratio 0.29 concludes coordination
oumber 4 around Si which is in agreement with sp 3 hybridized
tetrahedral arrangement around Si. Chain-Silicate Anions
(c) The radius ofAl 3+ ion (0.53 A) is close to that oftheSi 4+ The minerals having silicate chains are collectively
ic>n and thus Al 3+ can replace Si4+ in many silicates.
Monovalent cation as Na + or K+ maintains electrical
called pyroxenes: MgSiO3, CaMg(SiO 3)2•
+
o.eutrality. A pair ofions like (Si4 + Na , total charge +5) may
+ Double chains result by joining two simple chains by
alc:o be replaced by isoe1ectrical ion pairs like (Al 3+ + Mg 2+ ) or sharing oxygens. Examples are :
()J3 • +Ca 2+ ).
Amphiboles having anions like Si4 Of1
(d) The SiO tetrahedral units may occur singly or may
share one, two, three or four O-atoms through corners giving Asbestos mineral as tremoliteC32 Mg5 (Si4 O11 )2 (OH)2
me to cyclic groups, chains, infinite layers or infinite /0\
three-dimensional frameworks.
,: "-_
(e)SiO!- unit is represented as I • \ -. l • \

.. . . ,..
..o..
.... �... ...............: .:;·····:·····:.... :.. ................:• -.:;�····:····)
...... .T../ . . .....
/� • Si
\ .. \ ..

&/\
� ........Q 0 oxygen
\'(j/

Taking above facts we consider following cases (also lliustration 1 The silicate anion in the mineral
refer Table 12.5). kinoite is a chain of three SiO4 tetrahedra that share
corners with adjacent tetrahedra. The mineral also
Case I: No oxygen atom shared
contains Ca2 + ions, Cu2+ ions, and water molecules in a
(orthosilicate) / \ 1 : 1 : 1 ratio.
Unit: SiO!- ./ • \ (a) Give the formula and charge of the silicate anion
.................
(b) Give the complete formula for the mineral
Case II: One oxygen atom shared (pyrosilicate)
Solution (a) The silicate anion has three SiO4

�o�i
O···· ····· ·6
tetrahedra that share corners with adjacent tetrahedra
thus silicate is Si3O10, hence it can be represented as with
charge as= 3 x 4 + 10 x (-2) = - 8
8-
Unit: Si2 0;- ion, two SiO4 units joined by sharing an 0 0 0
�rygen atom. I I I
Case Ill : Two oxygen O-Si-O-Si-O-Si-O
atoms shared (closed ring : I I I
cyclosilicates and continuous 0 0 0
chains: inosilicates) (b) Ca2 +, Cu2+ and �O are in the ratio ofl: 1 : 1 and to
Each Si04 tetrahedron shares balance (- 8) charge of silicate anion, ( +8) charge is
an 0-atom with each of two
required thus there are two units each ofCa2 + ,Cu2 + and
l!Eighbouring tetrahedra resulting
�O thus, kinoite has formula Ca2Cu�isO10 · 2H20.
ring or chain anions
516 | Essential Inorganic Chemistry

Silicones Step I Chlorosilanes may be prepared by the


(i) reaction of Grignard reagent with SiCl4
Silicones are synthetic organo-silicon polymers
containing repeated R2SiO units. Since the empirical RMgCl+SiCll44 ----- > flSiCl3+MgCl2
formula is that of a ketone (ft,CO), the name silicone has (R = alkyl, aryl)
been given to these materials. Silicones can be made into RSiCl3 + RMgCl -----> flaSiClj,+MgCl2
oils, rubbery elastomers and resins. They find a variety of flsSiCLj + RMgCl ----- > R3SiCl+MgCl2
applications because of their
(i) chemical inertness (ii) reactions of alkyl halides with silicon (Rochow
(ii) water-repelling nature direct method)
Cu ,
(iii) heat-resistance flCl+Si > 7t^SiCl2
573 K
(iv) good electrical insulating property
(v) high thermal stability Nature of chlorosilanes may be changed by changing
(vi) resistant to UV rays molar concentration of flCl.
(vii) anti-foam characteristics Above reaction is exothermic hence in the course of the
reaction, removal of heat is necessary to make better yield.
(viii) non-toxicity
(ix) non-stick property Free radicals are involved as the intermediates.
(x) physiological inertness flCl+2Cu ----- > [CuR] + [CuCl]
The wide range of chain, cyclic and cross-linked [CuR] ----- > Cu+ [R’]
silicone polymers are based on structural patterns similar alkyl free radical
to those in silicates. For example, the O” in silicates have [CuCl] + Si -----> Cu + [SiCl]
been replaced by the formally isoelectronic CH3(Me) [7?*] + [SiCl] » RSiCl
groups in the dimethyl silicones
[CuR] -> Cu + [R* ]
O’ 0" 0~ Me Me Me
I I I [CuCl] * cu+cr
— Si—0 —Si—O—Si— — Si—0—Si—0—Si — [/?*] + [Cl*] + [flSiCl] -» ttjSiC^
I I I I Step II Hydrolysis of chlorosilanes to silanol
o- o- o- Me Me Me
R3SiCl+H2O -----> R3Si(OH) + HC1
Pyroxene Poly dimethyl siloxane
(silicone oil) R^SiCl2 +2H2O -----> 7?2Si(OH)2 +2HC1
OL Me K /O. .^Me RSiCl3+3H2O ----- > RSi(OH)3+ 3HC1
0-> Si' Me^’ ^Me Case I Silanol, R3Si(OH) undergoes dimerisation
I ^o-
o. 0 o. .0
Si' Si' R R R R

0’ o Me Me R—Si—|0H + H 0 —Si— R * fl_Si—O-Si-R


Cyclic metasilicate Cyclic dimethyl siloxane I
R R R R
Both silicones and silicates are formed due to the Dimer
reluctance of silicon to form silicon to oxygen double Case H Disilanol, fl^Si(OH)2 produces long-chain
bonds. But the silicates are all solids while the silicones polymers
may be oils; greases, rubbers or resins depending upon the fl R
extent of polymerisation.
Commercial silicon polymers are usually methyl HO —Si-|OH + H 0—Si—OH •>
derivatives and to a lesser extent phenyl derivatives and I
are synthesised by the hydrolysis of chlorosilanes. R R
R R
R2SiCl2 [J? = methyl (Me) or phenyl ($)]
Me Me Me
I
HO—Si—0—Si—OH
MegSiClg 0—Si—O—Si—O—Si—O—
R R
I
Me Me Me
R R
Thus, synthesis of silicones involves two steps I
__.O—Si—0—Si—0--
Step I Preparation of chlorosilanes Long-chain polymer
Step II Hydrolysis of chlorosilanes to silanols and
then to silicones R R
Chapter 12 : Group 14 (IVA - The Carbon Family) I 517
Case m Trisilanol, RSi(OH)3 produces cross-linked peroxide. These rubbers retain their quality from -100 ° C
l
po ymers to 250 ° C. Such rubbers find extensive uses as industrial
R R sealants and adhesives, space suits, medical tubing, etc.
I Silicone resins are used in electrical insulations, as
HO-Si OH+H O-Si-OH �
I I laminates for printed circuit boards or in encapsulation
resistors, integrated circuit, etc. They are used in coating
OH OH
cooking ware and industrial tyre moulds.
R R
I I
-O--Si-O-Si-O---
1
R R Glass
I I I Solids are most stable in crystalline form. However, if
0 0 HO-Si-O-Si-OH
I I I I a solid is formed rapidly (for example, when a liquid is
R--Si-0-Si-R OH OH cooled quickly), its atoms or molecules do not have time to
I I align themselves and may become locked in positions
0 0 other than those of a regular crystal.
iI iI
Cross-linked polymer If we mix sodium carbonate and calcium carbonate
with sand and fuse the mixture at about 1500° C, we get a
As the degree of polymerisation increases, silicones liquid mixture of sodium and calcium silicates. When this
may be changed to solid state from liquid state. mixture is cooled, it becomes viscous and eventually
• Liquid (oils) ceases to flow; it becomes solid-called glass. Glass is
• Semi-solids (greases) thus super-cooled liquid (solid solution). Amorphous
• Elastomers (rubbers) solids, such as glass, lack regular three dimensional
• Solids (resins) arrangement of atoms. Glass thus refers to an
optically transparent fusion product of inorganic
Some special adhesives are made by m1xmg two materials that has cooled to a rigid state without
silirones-based materials, which react together when
crystallising.
mixed to give a cross-linked polymer.
The hydrolysis is usually initiated by a small quantity
of concentrated �SO4 but KOH may also be used to
prepare certain silicone rubbers. Hydrolysis of PhSiCl 3
By varying the proportions of the three basic
{Ph = phenyl) and PhSiC12 in toluene produces silicone
ingredients, and by adding other substances, we can alter
resins. The polymerisation is allowed to proceed to a stage
the properties of the glass as given:
when the products is still soluble in toluene. It is shaped or
moulded in this form and heated to 200° C in presence of a • Calcium-alkali silicate glass - normal domestic glass
catalyst (a quaternary ammonium compound) when for windows
cross-linking develops the final 3-D network by Sodium silicate - soda glass used in cheap
condensing the remaining OH groups. laboratory glassware
Given properties of the silicones may be related to Potassium silicate - potash glass
their silica like (Si-O-Si-O) skleton surrounded by PbO (if CaO in glass is - lead glass has a higher
alkyl groups which make them water-repelling. replaced by PbO) refractive index used in
Silicone fluids containing striaght chain polymers making optical parts and
glass ornaments.
(20-50 units) are used in making cement work and fibres 4+
3+
water-repellant, as dielectric insulating media (in high B replaces some Si - borosilicate glass-used
voltage transformer), hydraulic oils and light duty in the tetrahedral in heat resistant
lubricating oil. Mixed with lithium stearate soaps, they glasswares (Pyrex)
make quality grease for use in high vacuum apparatus. • Fining agents such as NaNO3 or As 2O3 are added to
Silicone oils are also useful broad spectrum anti-foaming remove bubbles.
agents. • Decolourising agents may be added to eliminate
Silicone rubbers are made by reinforcing straight impurities and to obtain colourless glass. CaF2 is
thain polymers (having 6 x 103 to 6 x 1()5 Si units) with sometimes added to make opal glass.
fillers like SiO2 or graphite. They can be 'vulcanised' by • The colour of glass is due largely to the presence of
increasing cross links through oxidations with benzoyl metal ions (as oxides)
518 | Essential Inorganic Chemistry

Colour of glass Metal oxide Table 12.6 Properties of Some Ceramic and Metallic
Green Fe2O3, CuO
Materials
Yellow UO2 Material M.P/°C Density/g cm
Blue CuO. CoO Oxide ceramics
Red Colloidal Cu, Au Alumina, AI2O3 2072 4.0
Beryllia, BeO 2530 3.0
Following table shows the composition and properties Zirconia, ZrO2 2700 5.9
of quartz, Pyrex, and soda-lime glass. Non-oxide ceramics
Boron carbide, B„C 2350 2.5
Name Composition Properties and uses Silicon carbide, SiC 2700 3.2
Pure quartz glass 100%SiO2 Low thermal expansion, Silicon nitride, Si3N4 1900 3.4
transparent to wide range of Metals
wavelengths; used in Aluminium 660 2.7
optical research. Plain carbon steel 1515 7.9
Pyrex glass SiO2 :60-80% Low thermal expansion;
B2O3 : 10-25% transparent to visible and
infrared, but not to UV
AI2O3 : small%
radiation; used mainly in Compounds of Tin
laboratory and household
cooking glassware. Tin oxides
Soda-lime glass SiO2 :75% Easily attacked by Two types of oxides are known :
Na2O: 15% chemicals and sensitive SnO and SnO2
CaO: 10% thermal shocks. Transmits Both are amphoteric. As we know, basic character of
visible light, but absorbs UV oxides increases as we go down the group.
radiation; used mainly in
windows and bottles. CO, CO2, SiO-J2 GeO, GeOGeO<2 SnO, SnO2 PbO, PbOj
acidic amphoteric amphoteric basic
Glass is attacked by HF and this property is used to
make marking on the glass (etching) (A) SnO + 2H+ - Sn2+ + H2O
basic
Na2SiO3 + 6HF---- > Na2SiF6 + SHjO (B) SnO +4OH- + HgO Sn(OH)46
(in glass) acidic
CaSiO3 + 6HF---- > CaSiF6 + Sl^O In (A), SnO reacts with H+, hence SnO is basic oxide.
In (B), SnO reacts with OH-, hence SnO is an acidic
HF, however does not attack wax. The portion of the oxide.
glass on which etching is to be done is covered with wax These two reactions confirm that SnO is an
layer. Markings are carved on the wax layer with a pin amphoteric oxide.
and HF is poured on the carved parts and left for [Sn(OH)6]4- or[Sn(OH)3r is called stannite ion and is
sometime. After washing with water, wax is removed and also written as HSnO2 and SnO^-. [Sn(OH)6f" is called
permanent marking is obtained. stannate ion and is also written as SnO^- andSnO4“.

Ceramics Tin Chlorides


Ceramics are inorganic, non-metallic, non-molecular SnCl2 [tin (II) chloride] is obtained by dissolving tin in
solids, including both crystalline materials such as quartz concentrated HC1; on cooling crystals of SnCl2 • 21^0
(SiO2) and amorphous materials such as glasses. separate out. Anhydrous SnCl2 is formed by heating Sn in
Traditional silicate ceramics, such as pottery and a current of HC1 vapour.
porcelain are made by heating aluminosilicate clays to SnCl2 and SnCl4 are common chlorides but due to
high temperatures. Modem, so-called advanced ‘inert-pair effect’, Sn(II) is less stable than Sn(IY), hence
ceramics materials that have hightech engineering, the reaction
electronic and biomedical applications include oxide Sn22++ ---- > Sn4+ + 2e~ AG° = -ve
ceramics, such as alumina (A12O3), and non-oxide is spontaneous. Thus Sn2+ salts are reducing agents.
ceramics, such as alumina carbide (SiC) and silicon • SnCl2 reduces HgCl2 first to Hg2Cl2 and then to Hg.
nitride. Additional examples are given in Table 12.6 which
SnCl2 + 2HgCl2 ---- *> Hg2Cl21 + SnCl4
compares properties of ceramics with those of aluminium white ppt
and steel.
Hg2Cl2 + SnCl2 ---- > 2Hg + SnCl4
grey
Chapter 12: Group 14 (IVA -The Carbon Family) I 519
• SnCl 2 reduces FeC1 3 to FeC12 • PbC14 decomposes very readily into PbC1 2 and Cl2 due
2FeC1 3 +SnC12 ----+ 2FeC12 +SnC14 to greater stability ofPb2• than Pb4 + (inert-pair effect)
PbC14 (s)----+ PbCl2 (s) +Cl2 (g )
• It also reduces CuCI2 to CuCI
SnC12 +2CuC1 2 ----+ SnC14 +2CuCl.l, Pb4+ + 2.e- ----+ Pb2 +
white ppt 2Ci- ----+ Cl2 +2.e-
• SnCl 2 (and also SnF2 ) dissolve in solutions containing 4+
Thus, Pb salts are better oxidising agents.
c1- (or F-) ions • Lead shows the ability to make a complex with
SnC12 +c1- ----+ [SnC1 3 r chloride ions. If ice cold cone. HCl is added to lead
' SnCl 2 partly hydrolyses in water forming the basic dioxide [lead (IV) oxide], Pb02 , yellow hexachloro
chloride Sn(OH)Cl plumbate (IV) ions, PbCl�- are made. PbCl!- are made
SnC12 +�O ----+ Sn(OH)Cl from PbO
+
• SnCl4 also hydrolyses in dilute solution but it is Pb02 (s)+ 6Cl-(aq)+ 4H (aq)----+ PbCl�- (aq)
incomplete and can be repressed in presence of + 2�0 (l)
+
halogen acid : PbO (s)+ 4Cl-(aq) + 2H (aq) -1 PbCl !-(aq)
SnC14 +4�0 ----+ Sn(OH)4 +4HC1 + �O(l)
Sn(OH)4 +4HC1 ----+ SnC14 +4�0
SnC14 +2HC1 ----+ �SnC1 6 Lead oxides
SnS04 Three oxides of lead are known
(a) PbO - lead (II) oxide called litharge (a yellow-
!J.
orange solid)
(b) Pb02 - lead (IV) oxide also called lead dioxide
HCI
(a dark brown solid)
FeCl3 HgCl2 SnCl2
(c) Pb 304 - dilead (II) lead (IV) oxide. It is a
-·�v �er
Fecl2 . SnCl 2 · Sn Cl 4 + HQ2Cl2 • Hg
(grey) combination of (a) and (b) : 2Pb0 • Pb�.
�ppt)
It is also called red lead.
/ H20 HCI O
CuCI • PbO is obtained by roasting of PbS (galena):
(SnCl3) [H 2SnCls]

Sn(OH)CI Sn(OH)4 2PbS +302 --+ 2Pb0 +2S02
PbO is reduced to Pb by heating with carbon:
Compounds of Lead PbO +C-� Pb +CO
PbO +CO ----+ Pb +CO2
Lead Chlorides
PbO is amphoteric, being soluble both in acid and
• Lead reacts with dil. HCI producing insoluble PbC12 alkali:
which protects the metal from further attack. But due +
PbO +2H -: Pb2+ +f¼O
to the formation of the soluble complex ion [PbC1 3 r, PbO +20H- +�O ----+ [Pb(OH)4 f-
PbC1 2 dissolves in cone. HCL Thus lead dissolves in
cone. HCl. Crystal structure shows that it is predominantly
Pb +3HC1 (cone.) ----+ [PbCI 3 +H+ +H2(g) r covalent character.
• Pb can also be converted into PbC12 on reaction with • Pb02 is an oxidising agent and reduced to PbO since
Cl 2: stability of Pb (II)> Pb (IV) based on 'inert pair' effect:
Pb +Cl2 ----+ PbC12 Pb4+ +2.e- ----+ Pb2+
• PbC1 2 is partially hydrolysed by �O : Pb02 +4HCI --+ PbC12 +2�0 +Cl2
PbC12 +f¼O ----+ Pb(OH)Cl +HCl Pb� + S02 ----+ PbS04
or Pb2+ + 2�0 ----+ [Pb(OH>]+ + H30+ Pb� + 2�804 --+ 2PbS04 +2li.i0 + �
• Aqueous hot solution of PbC1 2 gives black ppt of PbS 2Pb� � 2Pb0 + 0i
on passing �S gas:
It is insoluble in an acid but soluble in concentrated
PbC12 +�S --+ PbSJ, + 2HC1 alkali thus indicating its acidic nature:
black ppt
Pb� + 20H- + 2�0 --+ [Pb(OH) 6 f-
520 | Essential Inorganic Chemistry

• Pb3O4, a double oxide, is obtained by heating lead (II) Lead oxides are used in the manufacture of lead-acii
oxide in air: (storage) batteries, glass, ceramic glazes, cements (PbO,
3PbO+^O2 Pb3O4 metal protecting paints (Pb3O4), matches (PbO2), ani
explosives (PbO2).
Its crystal structure shows that it is predominantly
covalent and composed of a combination of two types of Pb + HC1
lead atom. One type is bonded to six oxygen atoms, Cl2
the other to three oxygen atoms. But in the reactions, K2CrO4
PbCI2 PbCrO4|
it behaves like a mixture of PbO and PbO2 :
(yellow)
H2O KI
2Pb3O4 6PbO + 02 h2s/hci
k Pbl4|
Pb(OH)CI
O4 can be written as :
Pb33^4 PbS (yellow)
(black)
2(2PbO PbO2)=4PbO + 2PbO2 O2
2PbO2 ----- > 2PbO + O2 H* C, A
Pb2+ •* PbO *Pb
and out of two oxides only PbO2 is affected by heat:
OH”
When it reacts with K^SQi, PbSO4 is formed along O2
with O2 only: [Pb(OH)4]2”
Pb3O4 = 2PbOPbOz
Pb3O4 + 3H2SO4 3PbSO4 + 31^0 +- 02 A HNO3
2 H2SO4

DEEP Focus PbSO4 + H2O + O2 Pb(NO3)2 + PbOz

With HN03, PbO2 is precipitated and soluble Pb(NO3)2 is SO2 HCI


PbSO4 PbO2 *- PbCI2 + H2O + Cl2
also formed:
Pb_O, + 4HNO, ---- > 2Pb(NO,)2+PbO2X +2H2O OH”
This reaction clearly indicates that Pb(NO3)2 is from PbO [Pb(OH)6]‘
while PbO2 remains insoluble rnHNO3 and thus gets precipitated:
(2PbO PbO„) + 4HN0,---- > 2Pb(NOJ2 + PbO2l + 2H2O

Target Practice 2
1. SiO4 unit is represented as 4. SnCl2 can be volumetrically determined by titration with
FeCL V

• Si 2Fe3++Sn2+2+ ---- > 2Fe2++Sn4+


O oxygen Can you write the suitable indicator based on previous
sections.
Represent Si2O® similarly such that one oxygen atom is
shared between two units ofSi04. 5. SnO is called an amphoteric oxide. You are assked to
prove this statement then chemicals required are
(a) (b)
2. (a) Silicones are 6. I. [Sn(OH)6]4" can be represented as or
(b) They are formed when reacts with (b)
(a)
(c) They are used as and is called
(d) Write structure of simple silicone. (0
II. [Sn(OH)6]2" can be written as or
(a) (b)
3. A lab assistant was employed in a chemical industry and is called
manufacturing NaOH, HF and HN03. He was given three (c)
vessels for storing these chemicals. Can you help in exact 7. Write the products when Pb3O4 reacts with
matching without any loss to the vessel.
Chemicals: NaOH, HF,HN03. (a) HCI ---- >
(b) H2SO4---- >
Vessles made of: Al, glass, wax
(c) HN0„O ---- >
Matching is : (i) NaOH
8. Following reactions are spontaneous not.
(ii) HF (a) PbO2 ---- > PbO + O2 Yes/No
(iii) HNO,
o (b) PbCl4 ---- > PbCl2+Cl2 Yes/No
Chapter 12: Group 14 (IVA-The Carbon Family) I 521
(c) SnC1 4 +PbCI � PbCl 4 + SnC1 2 Yea/No (a) oxide is ......... .
2
(d ) 2FeCl� + SnCI Yes/No (b) insoluble oxide is .........•
2 ---+ SnCI4 +2FeC12
9. An oxide of lead changes to another insoluble oxide and (c) soluble salt is ......... .
soluble salt on reaction with HN03

Answers
1. Si20;- ion with two (SiO ◄ ) units joined by sharing as oxygen 3. (i) NaOH : Glass
atom. (ii) HF:Wil
(iii) HNO 3 : Al
4. KCNS, its gives red colour with Fe 3 + ion.
5. (a) HCl (b} NaOH
6. I : (a) HSnO2 (b) SnOf (c) stannite ion
1 (a) Synthetic polymers containing repeated l½SiO. Il: (a) Sn�- (b) SnO4 - Cc) stannate ion
(b) �SiCl 2 ,H 20 7. (a} Pb 3 O ◄ + 4HCI --+ 2PbC1 2 +PbO 2 +2H 2O
(c} Refer text PbO 2 + 4HCI --+ PbCl 2 +2H 20+Cl 2

R R R (b) Pb 3 O4 +3H 2SO 4 --+ 3Pbso .. +3H 2O+-0 2


2
I
(d} -O- i-O- i-O- i-O-
I I (c) Pb 3 O 4 +4HNO 3 � 2Pb(NO 3 )2 + PbO 2 +2H 20
, , ,
8. (a) Yes (b) Yes (c) No (d} Yes
R R R 9. (a) Pb 3 O 4 (b) Pb0 2 (c) Pb(NO 3 )2

Practice Exercise 2
1, SnCb + 2NaOH ---+ (A) (white ppt) NaOH (excess) (B) 5. You are given a colourless solution and was told to add a
Identify (A) and CB). little NaCl solution to it. A cloudy white precipitate was
2. FeC13 can be reduced to FeC1 by SnC1 as well as by the made. What conclusion should you draw about the nature of
2 2
the colourless solution?
addition of Sn into it. Explain.
3. FeC1 gives blood red colour with NH 4CNS. But after FeC1 8. If you were to attempt to dissolve lead (II) oxide in acid,
3 3
has been treated with SnC12, there is no colour with which acid would you choose?
NH 4 CNS. Explain. 7, Explain why the reactions with HCl and S02 show the
4, Identify (A) and (B) based on following results: oxidising nature of Pb02 ?
Colourless salt (A) + dil. HCl ---+ white ppt (B) soluble in
8. Predict the result of reacting Pb 30 4 with cone. HCI.
hot water.
ln this reaction vinegar smell is also observed.

Answers
1. (A ): Sn(OH)2 (B): Na 2SnO 2 (sodium stannite) (B): PbC1 2
2 . 2FeC1 3 +SnC1 2 --+ SnCl 4 +2FeC1 2 5. White ppt can be ofAgel or PbCI 2 • To make further distinction,
This reaction is possible since Sn 4+ is more stable than Sn +
2 warm the solution. PbC1 2 dissolves in hot water while AgCI
(inert-pair effect) remains insoluble. Agel dissolves in aq NH 3•
Also FeCl 3 solution is acidic due to hydrolysis : 6. HNO 3 (since PbC1 2 and PbSO 4 formed by using HCl or H 2SO 4 ,
are insoluble).
FeC1 3 + 3H 20 � Fe(OH)3 + 3HC1
7. The lead'changes from Pb (M to Pb(Il); this represents gain or
Sn added reacts with HCl forming nascent hydrogen that can electrons, i.e., reduction hence PbO 2 is an oxidising agent. Also
reduce FeC1 3 to FeC12 : stability of Pb 2+ > Pb 4+, hence the reaction
2HCI +Sn --+ SnC1 2 + 2H Pb 4++�- � Pb
2+
(nascent)
is spontaneous in forward side.
FeC1 3 +H --+ FeC1 2 +HCI
8. Pb 3O 4 +8HC1 � PbC1 2 +4H 2O+Cl 2
3. SnCl 2 reduces FeCl 3 to FeC1 2 hence no colour with NH 4CNS.
r-
L. . ...d

Total Practice Set


(Read, Plan & Solved

Increase in volume will be due to C02. By measuring


Problem 1. What would you expect to happen when: total volume after passing over heated charcoal,
(a) Tin is heated with a concentrated aqueous solution composition of mixture of CO and C02 can be
of sodium hydroxide. determined.
(b) Sulphur dioxide is passed over lead dioxide.
(c) Carbon disulphide is shaken with aqueous sodium Problem 3. A mixture of CS2 vapour and nitrogen
sulphide. monoxide, NO, gives a vivid blue flame when ignited. A
(d) Dichlorosilane is hydrolysed by water. yellow solid is left afterwards. Predict the products of the
(e) 4 moles ofClCI^SiClg react with 3 moles ofLiA!H4 reaction, and write the equations.
in diethyl ether solution?
Solution You should have predicted that a yellow solid
Solution (a) Sn + 20H" + 4H2O ♦ [Sn(OH),f- is sulphur formed by oxidation of CS2 and thus NO is
reduced to N2 which appears as a vivid blue flame :
+ 2H,
2N0 + CS2 ---- > 2S + N2 + C02
(b) PbO2 + S02 ---- > PbSO4 (yellow solid)
(c) CS2 + Na2S---- > Na2CS Problem 4. CO and CN“ ions are highly poisonous. A
sodium thiocarbonate sign that someone has been poisoned by either of these two
H OH chemicals is that the person's blood changes colour. They
(d) SiH^ + 21^0 ■>

+ 2HC1 both have the effect of preventing oxygen being taken up


H OH by the blood stream. What might happen to haemoglobin

L» H'
Si=0 + HaO
to stop oxygen being taken up?

Solution CO and CN~ ions are able to bond more


strongly to the iron atom in haemoglobin than can oxygen.
If you were to breathe in CO for any length of time, your
(e) 4ClCH2SiCl3 + 3LiAlH4 * 4ClCH2SiH3
haemoglobin would become saturated with CO molecules
+ 3LiCl + 3A1C13 rather than oxygen. As a result, all the body processes that
rely on a supply of oxygen gradually cease. Death is the
Problem 2. If you had a mixture of CO and C02, how
result.
would you find out relative proportions of the two gases in
the mixture ?
Problem 5. Comment on each of the following
Solution Method I : Mixture of CO and C02 is run observations:
into Ca(OH)2 solution. CO^ is absorbed by Ca(OH)2 and (a) When an aqueous solution of potassium cyanide is
thus residual volume will be that of CO. added to aluminium sulphate, A1(OH)3 is
precipitated.
Ca(OH)2 + C02---- > CaCO3 + H^O
(b) The carbide Mg2C3 liberated propyne (CH3C=CH)
Method II : Mixture of CO and C02 is passed over on treatment with water.
heated charcoal: (c) Magnesium silicide reacts with ammonium
C02 +C 2C0 bromide in liquid ammonia to form silane.
Chapter 12: Group (14) IVA — The Carbon Family | 523

SiMe2 Me C3-—C=C" +4^0 > CH3—C=CH + 40H~


is hydrolysed by 2Mg2++4OH“ ---- > 2Mg(OH)2
SiMe2 H
Net: Mg2C3 + 4H2O---- > 2Mg(OH)2
aqueous alkali at the same rate as the
+ CH3—C=CH
corresponding Si-D compound.
(c) Solutions of NH4Br (in general ammonium halide)
Solution (a) Aqueous solution of KCN is alkaline due
in NH3 may be used as acids and SiH4 (silane) is
io hydrolysis of CN" ion.
formed when NH4Br in NH3 reacts with Mg2Si
CN- + H2O * HCN + 0H~ (magnesium silicide)
weak acid
Mg2Si + 4NH4Br SiH4 +2MgBr> + 4NH3
OH" formed causes precipitation of Al2(S04 )3 as A1(OH)3 magnesium silane
silicide
Al3+ +30H" ---- > A1(OH)3I
(d) Si—H or Si—D bond is not broken in rate
(b) Mg2C3 ---- > 2Mg2+ + C*3" determining step, it is probable that there is attack
Cj" ion is decomposed by H^O giving CH3C=CH of OH- on Si.
524 | Essential Inorganic Chemistry

Thus CC14 is inert towards water and is not


Explanation hydrolysed but SiCl4 can coordinate with 2 moles a
(a) CO2 is a gas but SiO2 is a solid H20 molecules and is thus readily hydrolysed:
- Carbon and silicon (Group 14, IVA) have four valence SiCl4 + 21^0----- > Si(OH)4 + 2HC1
electrons. We might expect carbon and silicon to form
oxides with similar properties. In CO2 the ability of C Cl. .Cl H Cl. (ci Hj
and 0 atoms to form n bonds through the side-wise
overlap of their 2p orbitals is swung. The result is H2O + Si + OH2 0 Si 0
strong C— to —O double bonds and a very stn’Je
triatomic molecule. Cl Cl (H ci; Cl H

o=c=o Silicon with covalency six


Silicon, being in third period, would have to use 3 p -2HC1
orbitals to form double bonds with oxygen. The
side-wise overlap of these orbitals with th'* v. bitals of (H OH ,H Cl
CO
oxygen is too limited for 7c bond formation. From an 2H2O I
0 Si 0 OH—Si—OH
energy stand point, a stronger bonding arrangement
results if the Si atoms form four single bonds with 0 H Cl
HO (Cl
atoms (bond energy : 464 kJ/mol) rather than two
double bonds (bond energy : 640 kJ/mol) since each 0 Silicon with covalency six
atom must be simultaneously bonded to two Si atoms, OH
the result is a network of —Si—O—Si— bonds and thus
a hard (giant) solid : HO—Si—OH
-2HC1
O 0 OH
0 I
O—Si—0—Si—0
I (d) Glass made ofSiO2 is attacked by HF with the
SL formation of SiF^~ anion. The analogous Clf”
0 \ O 0 0
0 does not exist:
0—Si—0—Si—0
I I - Silicon has 3d-orbitals and can use octahedral
0 o sp3^2-hybrid orbitals to bond to six F" ions. Carbon
lacks 2d-orbitals (2d does not exist) and with just 2s
(b) (SiH3)N is planar while (CH3)3N is pyramidal
and 2p valence orbitals, carbon can form maximum of
- In case of(CH3)3N, lone pair on N and three covalent only four bonds.
bonds with CH3 groups results in sp3 hybridisation
but lone pair-bond pair repulsion causes distortion
hence gives pyramidal structure :
Sii4 [Ne] 11111 3d

0 (in excited state)

3s
3p

ch3 1
CH3
CH3
C6
(in excited state)
[He] mu 2p
2s
But in case of (SiH3)3N, lone pair on the nitrogen is
transferred to the empty d-orbitals of silicon (pn-dn (e) SnCLj • 2H2O readily loses 1 molecule of H2O at
overlap) thereby causing planarity of unit. 80°C.
(c) SiCl4 is hydrolysed butCC\A is inert to hydrolysis - Pyramidal SnCl2 • B^O molecule has a lone pair on
- SiCl4 and CC14 are covalent compounds. Carbon has Sn. The other H^O, hydrogen-bonded to the
maximum covalency of four while silicon has six. coordinated H^O is easily lost.
r -

Master Exercises
Exercise 1
(Stage 1: Learning)
Short Answer Type Questions 11. List three commercial uses for CO2, and relate each
1. Carbon has coordination number (C.N.) of four while use to one of carbon dioxide’s properties.
other elements of this group have C.N. of six. Explain. 12. Is it wise to write
2. Identify the group 14A element that best fits each of A12C3 ---- > 4A13++3C4~?
the following descriptions: 13. Draw a Lewis structure for the (%~ and C3“ ions.
(a) prefers the +2 oxidation state
(b) forms the strongest n bonds 14. Why are CO and ON" harmful?
(c) is the second most abundant element in the 15. Give an example of an ionic carbide. What is the
earth’s crust oxidation state of carbon in this compound?
(d) forms the most acidic oxide 16. Which of the following is a more effective greenhouse
3. Give the formula of a compound in which carbon gas CO or H>0?
exhibits an oxidation state of: 17. Instead of monitoring carbon dioxide, suggest another
(a) +4 (b) +2 ' gas that scientists could study to substantiate the fact
(c) -4
that CO2 concentration is steadily increasing in the
4. List three properties of diamond, and account for atmosphere.
them in terms of structure and bonding. 18. The equilibrium constant (Kp) for the reaction
5. Identify (A), (B), (C), (D) and (E): 2CO(g) + O2(g) 2CO2(g)
HNO3 /HF (A)_HF, (B)
cone, A is 1.4 x IO90 at 25°C. Given this enormous value, why
h2o doesn’t CO convert totally to CO2 in the troposphere?
(B) + (C) 19. What is glanzkohlenstoff?
~ , hot cone HNOi 20. Talc, Mg3Si4O10(OH)2 is the softest material. It has a
(b) Graphite--------------- > (D)
smooth, greasy touch. It is used in talcum powder.
(c) Na2CO3 +Si---- > (E) Muscovite, KAl2(Si3Al)O10(OH)2, is one of the micas,
fused
which split into thin layers extremely easily. Both
6. Solution ofCO^- (say Na2CO3) is alkaline. Explain. minerals have layer structures. What type of force
7. Explain why a solution containing Ca2+(aq) and holds the layers together?
HCO^ag) ions is said to be hard? 21. Silica will react in a similar way with metal sulphates
(and phosphates) as it does with carbonates, giving a
8. What happens if solution ofNa2CO3 is heated with Al?
solid residue and a gas. 1 mole of silica reacts with
9. A mixture ofCS2 vapour and nitrogen monoxide, NO, 1 mole of sodium sulphate, Na2SO4. Write the
gives a vivid blue flame when ignited. A yellow solid is equation.
left afterwards. Predict the products of reaction, and 22. What would be the structure of the silicone molecule
write the equation. made from Si(CH3)3Cl?
10. A student placed a pellet of NaOH on a watch glass. A 23. One type of silicone adhesive uses a polymer that has
few days later, she noticed that the pellet was covered two OH groups left on the ends of the chain :
with a white solid. What is the identity of this solid? HO—Si(CH3)2[O—Si(CH3)2 ]n—0—Si(CH3)2 —OH
526 | Essential Inorganic Chemistry

It has been found that this kind of polymer reacts


easily with an organo-silicon molecule that contains Diamond B
acetate (CH3COO) group. Which of these three Liquid
molecules
2x104
CH3COOSi(CH3)3, (CH3COO)2Si(CH3)2, Graphite A
Vapour
(CH3COO)3SiCH3
would you choose to add to the hydroxy polymer in 3300 „
order to make a strong adhesive? G°C)
24. The following pictures represent various silicate
(c) Synthetic diamond can be made from graphite.
anions. Give the formula and charge of each anions.
Using the phase diagram, how would you go about
making diamond?
3. Consider the following list of elements: C, Se, B, Sn,
Cl. Identify the element on this list that
(a) has the largest atomic radius,
(b) is the most electronegative,
(c) is the best electrical conductor,
(a) (b) (c) (d) has a maximum oxidation state of +6,
(e) forms a hydride with the empirical formula XH3
25. Draw the structure of the silicate anion in :
4. Carbon is an essential element in the molecule on
(a) K4SiO4
which life is based. Would silicon be equally
(b) Ag10Si4O13
satisfactory? Explain.
26. A colourless solution (X) gives black ppt (Y) on passing 5. The name dilead(II) lead(IV) oxide and plumbous
H^S gas. (X) also gives white ppt (Z) with SnCl2. (Z) orthoplumbate have both been used to describe red
changes to grey. Identify (X), (Y) and (Z). lead, Pb3O4. Show that these names are consistent
27. One problem with making crystals of tin (II) chloride with this mixed oxide.
is that they are easily hydrolysed by warm water. 6. Write chemical equations to represent the following
Instead of dissolving tin in HCl(a^) how else might reactions:
the anhydrous chloride be made? (a) The dissolving of PbO in NaOH (aq).
(b) The disproportionation of SnO to Sn andSnO2.
28. HF can’t be stored in a glass vessel. Explain. (c) The oxidation of HCl(ag) to Cl2(g) by PbO2(s).
29. Fill in the blanks : (d) The production of basic lead carbonate (white
(a) Glass is a............... liquid. lead) 2PbCO3 • Pb(OH)2 by the action ofHjO and
CO2 on PbO.
(b) Markings on the glass plate are made by use of
7. One natural source of atomospheric C02 is
(c) Glass with high refractive index contains precipitation reaction such as the precipitation of
silicates in the oceans :
(d) Pyrex glass can withstand heat and contains Mg2+(aq) + SiO2 (dispersed) + 2HCO3(aq) —*
MgSiO3(s)+ 2CO2(g) + H20(/)
(e) Opal glass contains How many grams of magnesium silicate would be
precipitated during the formation of 100 L ofCO2 at
Analytical Questions 30°C and 775 Torr?
(Mg = 24, Si = 28, C = 12,0 = 16)
1. Explain :
(i) Why silicon is not expected to form an allotrope 8. The organic solvent acetone has the molecular
with graphite like structure? formula (CH3)2CO. The silicon analogue, a thermally
(ii) Contrary to (CH3)2O, (SiH3)2O does not form stable lubricant, is a polymer, [(CH3)2SiO]„. Account
addition compound with BF3. for the difference in structure.
2. Given below the phase diagram of carbon, answer the 0 [ CH3 ch3 CK3 1
following questions: I
c
(a) How many triple points are there and what are
the phases that can coexist at each triple point? h3c/^ch3 nL 3 >i—0—Si—0—Si-04

ch ch3 shs3 IJ„


CH
(b) Which has a higher density, graphite or Acetone
diamond? Polymethylsiloxane (silicone oil)
Chapter 12: Group 14 (IVA — The Carbon Family) | 527

NCERT Problems 3. What is the oxidation state of carbon in each of the


following compounds?
1. Write balanced equations for the following reactions. (a) CO (b) HCN
(a) Combustion of C4H10 in a limited supply of (c) H2CO3 (d) CaCj
oxygen to form carbon monoxide and water.
4. What is dry ice? Why is it so called?
(b) The reaction of calcium carbide with water to
form acetylene. 5. What is the state of hybridisation of carbon in
(c) The preparation of calcium cyanamide from (a) COt <b) HCN
calcium carbide and nitrogen. (c) diamond (d) graphite
(d) Formation of hydrogen cyanide from methane 6. Explain the differences in the properties of diamond
and ammonia. and graphite on the basis of their structures.
2. Write resonance structures forCO^" andHCO3. 7. What are fullerenes? How are they prepared?

Exercise 2
(Stage 2: High Skill Problem Solving)
Only One Option Correct II: Graphite has planar hexagonal layers of carbon
atoms held together by weak van der Waals’
1. Stability of hydrides of carbon family is in order
forces.
(a) CH4 > SiH4 > GeH4 > SnH4 > PbH4
Ill: Silicon exists only in diamond structure due to
(b) CH4 < SiH4 < GeH4 < SnH4 < PbH4 its tendency to form pit-pit bonds to itself.
(c) CH4 > SnH4 > GeH4 > SiH4 > PbH4
In this
(d) None of the above
(a) Only I and II are correct
2. Select correct statement. (b) Only I is correct
(a) oxides of carbon family (AfO2) are all network
(c) Only II and III are correct
solids with octahedral coordination
(d) All the above are correct statement
(b) Silicon dioxide (silica) is a network solid with
tetrahedral coordination and is a giant molecule 7. OutofC02, SiO2,GeO2,SnO2 andPbO2
(c) GeO2, SnO2 andPbO2 are all acidic oxides (a) CO2 and SiO2 are acidic, SnO2 is amphoteric and
(d) None of the above appears correct PbO2 is an oxidising agent
3. Dry ice is (b) PbO2 is converted to Pb(NO3)2 on reaction with
(a) H,O(s) (b) NH3(g) HNO3
(c) CO2(s) (d) PH3(g) (c) both (a) and (b) are correct
4. Select correct statements. (d) none of the above is correct
(a) Hydrides of B and Si are volatile and catch fire on 8. Which is incorrect statement about silicones?
exposure to air (a) They are repeating units (SiO4) in silicates
(b) Oxides of B and Si(B2O3 and SiO2) are acidic in (b) They are synthetic polymers containing repeated
nature 7?2SiO2 units
(c) Borates and silicates have tetrahedral B04 and (c) They are formed by hydrolysis of J^SiCl,
Si04 structural units (d) All the above are incorrect statements
(d) All the above are correct statements 9. Which one of the following bonds has the highest
5. Select correct statements. bond energy?
(a) Catenation is maximum in carbon (a) C—C ' (b) Si—Si
(b) Carbon has pronounced ability to form pit-pit (c) Ge—Ge (d) Sn—Sn
multiple bonds to itself and to other elements like 10. Which of the following species have undistorted
0 and N octahedral structures?
(c) Both (a) and (b) are correct 1. SF6 2.PF£
(d) None of the above is correct
3. Sil^" 4.SeF6
6. Consider following statements,
I: In diamond, each carbon atom is linked Select the correct answer using the codes given below
tetrahedrally to four other carbon atoms by sp3 (a) 2, 3 and 4 (b) 1, 3 and 4
bonds. (c) 1, 2 and 3 (d) 1, 2 and 4
528 | Essential Inorganic Chemistry

11. Which of the following statements are true for 22. In which case hydrolysis is faster
zeolites? /SiMe2 Me
1. They are formed by the replacement of some of the (a) Me2Si
silicon atoms of the SiO2 lattice, by say, SiMe2 H
aluminium.
2. They have a more closed structure than feldspar. SiMe2 Me
3. They can absorb C02, NH3 and EtOH. (b) M^Si Si'
4. They can separate straight chain hydrocarbons SiMe2 D
from a mixture containing both straight, chain
(c) at equal rate
and branched chain hydrocarbons.
(d) no hydrolysis
Of these statements
(a) 1, 2 and 3 are correct (b) 1, 3 and 4 are correct 23. Which of the following is the ionic carbide?
(c) 1, 2 and 4 are correct (d) 2, 3 and 4 are correct (a) Fe3C (b) SiC
(c) CaCrj (d) Cu^C
12. Artificial gem used for cutting glass is
(a) graphite (b) diamond 24. The silicate anion in the mineral kinoite is a chain of
(c) SiC (d) CaCN2 three SiO4 tetrahedra that share comers with
adjacent tetrahedra. The mineral also contains Ca2’
13. Red lead used as primer for iron to prevent it from ions, Cu2+ ions, and water molecules in a 1:1:1
rusting is ratio. The mineral is
(a) Pb3O4 (b) PbO (a) Ca2Cu2Si3O10 ^HjO
(c) PbO2 (d) Pb2O
(b) CaCuSi3O10 ■ HjO
14. Lead as impurity in the extraction of silver is (c) CagCt^S^Og ^HjO
removed by (d) Ca2Cu2Si3O6
(a) Parke’s process (b) Solvay process
(c) Cyanide process (d) None of these 25. The best oxidising agent of the following oxides is
(a) SnO2 (b) PbO2
15. Which involves breaking of covalent bonds?
(c) HgO (d) MgO
(a) Boiling of H2S (b) Melting of SiO2
26. Dipole moment of CO is very small (0.1 D) (inspite of
(c) Melting of KCN (d) Boiling of CC14 the larger difference in electronegativity between C
16. Which is silane? and 0) and negative end of the dipole lies near the
(a) SiO2 (b) SiO3 less electronegative C-atom. It is due to
(0 SiH4 (d) Si (a) occupation of anti-bonding molecular orbitals
17. Holme's signals can be given by using (b) occupation of empty d-orbitals
(c) occupation of lone-pairs
(a) CaCg + CaCN2 (b) CaC^ + Ca3P2 (d) occupation of the lone pairs as well as empty
(c) CaQ, + CaCO3 (d) Ca3P2 + CaCN2 d-orbitals
18. Which is/are true statements? 27. Carbon and silicon belong to (IV) group. The
(a) Diamond is unaffected by cone acids but graphite maximum coordination of carbon is commonly
reacts with hot cone. HN03 forming mellitic acid, occurring compounds is 4, whereas that of silicon is 6.
C6(COOH)6 This is due to
(b) CO is toxic because it forms a complex with (a) large size of silicon
haemoglobin in the blood (b) more electropositive nature of silicon
(c) C3O2, carbon suboxide, is a foul-smelling gas (c) availability of d-orbital in silicon
(d) All the above are true statements (d) both (a) and (b)
19. Carborundum is 28. In graphite, additional electrons are
(a) CaQ> (b) Fe3C (a) localised on each C-atom
(c) CaCO3 (d) SiC (b) localised on every third C-atom
(c) present in anti-bonding orbital
20. Gas that strikes in thundering of light is (d) delocalised forming extended %-bonding system
(a) CO (b) NO
(c) C02 (d) Hj 29. PbO2 ---- > PbO AG298 < 0
21. NO2is not obtained when following is heated SnO2 ---- > SnO AG298 > 0
Most probable oxidation state of Pb and Sn will be
(a) Pb(NO3)2 (b) AgNO3 (a) Pb4+,Sn4+ (b) Pb4+,Sn2+
(c) LiNO3 (d) KN03 (c) Pb2+,Sn2+ (d) Pb2+,Sn4+
Chapter 12:Group 14 (IVA — The Carbon Family) | 529

30. When vapours of SiCl4 are passed over hot Mg, then 43. Graphite is a good conductor of heat and electricity
because it contains
the product formed is (a) layers of carbon atoms
(a) SiCl2 + MgCl2 (b) MgSi + C^ (b) sheet like structure
(c) MgSiCl6 (d) Si+MgCl2 (c) free electrons
31. Which one of the following is not a green house gas? (d) pn-dit bonding
(a) CO2 (b) H^O 44. (Me2)2SiCl2 on hydrolysis will produce
(c) N2 (d) O3
(a) (Me)2Si(OH)2 (b) (Me)2Si=O
32. Which of the following has the highest calorific value? (c) [—0—(Me)2Si—0—]„ (d) Me^iCKOH)
(a) Water gas (b) Producer gas
(c) Carbonium oxide (d) Coal gas 45. I. PbO2 ---- > PbO AG298 < 0
33. Carbogen is II. SnO2 ---- > SnO AG298 > 0
(a) mixture of CO + CO2 Which of the above is/are true?
(a) I (b) II
(b) mixture of O2 + CO2
(c) Both (a) & (b) (d) None of these
(c) pure form of carbon
(d) unsaturated organic compound 46. Which one of the following statements about the
34. Carbon monoxide is a poisonous gas, the antidote zeolite is false?
(a) They are used as cation exchangers
used for this poisoning is (b) They have open structure which enables them to
(a) pure oxygen (b) carbonic acid take up small molecules
(c) carborundum (d) carbogen (c) Zeolites are aluminosilicats having three
35. Method used for obtaining highly pure silicon, used dimensional network
as a semiconductor material, is (d) Some of the SiO4" units are replaced by A1O^“
(a) oxidation (b) electrochemical and A10|~ ions in zeolites
(c) crystallisation (d) zone refining 47. Bonding present between the carbon atoms in
36. Carbon and silicon belong to group 14. What is graphite is
nature of carbide of silicon? (a) metallic (b) ionic
(a) Covalent (b) Ionic (c) covalent (d) van der Waals’ forces
(c) Interstitial (d) None of these 48. Which of the following cuts ultraviolet rays?
37. Which of the following shows electrical conduction? (a) Soda glass (b) Crooke’s glass
(c) Pyrex (d) None of these
(a) Potassium ion (b) Graphite
(c) Diamond (d) Sodium ion 49. In diamond crystal each carbon atom is linked with
carbon atoms. The number of carbon atoms linked is
38. Water glass is
(a) Glass made up of water (a) 2 (b) 4
(b) Sodium silicate (c) 3 (d) 1
(c) Water gas 50. Which of the following has the highest first ionisation
(d) Crystal carbonate energy?
39. Name of the structure of silicates in which three (a) Lithium (b) Beryllium
oxygen atoms of [SiO414- are shared is (c) Boron (d) Carbon
(a) pyrosilicate 51. Which gas is used in aerated water?
(b) sheet silicate (a) C02 (b) S02
(c) linear chain silicate (c) CO (d) Water vapours
(d) three dimensional silicate
52. In which of the following the angle between the two
40. Marsh gas primarily contains covalent bond is maximum?
(a) CH4 (b) QA
(a) HjO (b) C02
(c) I^S (d) QA
(c) CH4 (d) NH3
41. Litharge is chemically
53. Percentage of lead in lead pencil is
(a) PbO (b) PbO2
(c) Pb3O4 (d) (CH3COO)2Pb (a) 30 (b) 20
(c) 10 (d) zero
42. The gas evolved by heating potassium ferrocyanide
crystals with cone. ASO4 is 54. Which of the following is not an acid anhydride?
(a) CO (b) CO.'2 (a) CO2 (b) CO
(c) SO2 (d) SO3 (c) SO2 (d) SO3
530 | Essential Inorganic Chemistry

55. The compound having maximum carbon percentage 68. The variation in element-element bond energy
is follows the order
(a) peat (b) bituminous (a) C—C > Si—Si > Ge—Ge > Sn—Sn > Pb-Pb
(c) lignite (d) anthracite (b) Si—Si > C—C > Pb—Pb > Sn—Sn > Ge-Ge
(c) C—C > Ge—Ge > Sn—Sn > Pb — Pb > Si-Si
56. At room temperature the solid compound is
(d) C—C > Sn—Sn > Pb—Pb > Ge—Ge > Si-Si
(a) CO2 (b) SiO2
(c) NO2 (d) SO2 69. Thermodynamically most stable form of carbon is
(a) graphite (b) diamond
57. By passing air over red hot coke the gas obtained is (c) ionsdaleite (d) chaoite
(a) coal gas (b) water gas
(c) oil gas (d) producer gas 70. The crystal structures of both chaoite and carbon (VI)
are based on
58. White lead is (a) — C=C—C=C— (b) — C=C—c=c-
(a) PbSO4 PbO (b) PbCO3 PbO
(c) PbCO3 (d) Pb(OH)2 PbCO3 I I
(c) -C-C- (d) all of these
59. Silicon is an important constituent of
(a) rocks (b) amalgams
(c) chlorophyll (d) haemoglobin 71. Elements of group 14 used in semiconductors are
(a) C, Si, Ge (b) Si, Ge, Sn
60. When chlorine is passed through molten tin, the (c) Si, Ge (d) B, Si, Ge
product obtained is
72. Select correct statement about stability of cations.
(a) SnCl4 (b) [SnCl6f-
(a) Ge4+ > Sn4+ > Pb4+
(c) [SnCl4f- (d) SnCI2
(b) Ge2+ < Sn2+ < Pb2+
61. In making casting of metal, silicon is used as (c) Pb2+ >Pb4+,Sn4+ >Sn2+
(a) oxidiser (b) semiconductor (d) All the above are correct statements
(c) deoxidiser (d) None of these 73. In which case geometry of the molecule is pyramidal?
62. Sindoor is represented by (a) N(CH3)3 (b) N(SiH3)3
(a) Pb(NO3)2 (b) PbCO3 • Pb(OH)2 (c) Both (a) and (b) (d) None of these
(c) Pb(OH)2 • 4PbCO3 (d) Pb3O4 74. pit-pit multiple bonds occur between
63. Lead is soluble in (a) C and C (b) C and 0
(c) C and N (d) in all cases
(a) CH3COOH (b) H2SO4
(c) HC1 (d) HNO3 75. Buckminster fullerene is
(a) pure graphite (b) C-60
64. Quartz is a crystalline variety of (c) diamond (d) C-90
(a) silicon carbide (b) sodium silicate
76. Lead pencil contains
(c) silica (d) silicon (a) lead
65. Which of the following have highest melting points? (b) graphite
(a) p-block elements (b) s-block elements (c) alloy of lead and tin
(c) d-block elements (d) None of these (d) alloy of lead and graphite
66. Which of the following halides is least stable and 77. Standard reduction electrode potential of lead
suggests that it is reactive metal but it appears more
doubtful existence?
noble (unreactive). It is due to
(a) CI, (b) SnI,
(a) surface coating of oxide
(c) Gel, (d) Pbl, (b) high over potential of reduction of H+ to H2
67. Graphite is a soft solid lubricant extremely difficult (c) both (a) and (b)
to melt. The reason for this anomalous behaviour is (d) None of the above
that graphite 78. Hot cone. HNO3 converts graphite into
(a) is a non crystalline substance (a) graphite oxide
(b) is an allotropic form of diamond (b) benzene hexacarboxylic acid
(c) has molecules of variable molecular masses like (c) both (a) and (b)
polymers (d) None of the above
(d) has carbon atoms arranged in large plates of 79. Silicon reacts with hot solution of NaOH forming
rings of strongly bound carbon atoms with weak (a) Si(OH)4 (b) Si(OH)2
interplate bonds (c) SiO2 (d) Na4SiO4
Chapter 12: Group 14 (IVA — The Carbon Family) | 531

80. Select incorrect statement. (b) absorption of heat radiation by atmospheric


(a) Mellitic acid is benzene hexa-carboxylic acid gases particularly water vapours
(b) Pb dissolves in cone. HC1 forming PbCl2 (c) both (a) and (b)
(c) Pb dissolves in hot NaOH forming Na2[Pb(OH)6] (d) None of the above
(d) Diamond is unreactive but graphite forms (CF)rt 91. Methanides are
withF2 (a) Mg2C3, Be2C, A14S3 and CaCj
81. When a mixture of carbon monoxide and chlorine is (b) Mg2C3, Be^ and A14C3
exposed to sunlight the product formed is (c) BegC, A14C3 and CaGj
(a) thionyl chloride (d) Be2CandAl4C3
(b) phosgene 92. Select correct statements(s).
(c) phosphine (a) Cyanamide ion (CN^“) is isoelectronic with CO2
(d) carbon tetrachloride and has the same linear structure
82. Stable oxides of carbon are (b) Mg2C3 reacts with water to form propyne
(a) CO,CO2,C3O2 (b) CO2, C3O2, CgO (c) CaC^ has NaCl type of lattice
(c) CO,CO2 (d) GACaQ, (d) all the above are correct statements
83. CO behaves as 93. Select incorrect statement(s).
(a) Lewis acid (b) Lewis base (a) Interstitial carbides are formed by metalloids
(c) amphoteric oxide (d) None of these like Si and B
84. Which reaction takes place during respiration? (b) Covalent carbides are formed by metalloids
(a) 6CO2 + 6H2O---- > 6O2 + C6H12O6 (c) CO and CN" both are fatal due to complex
formation with Fe(III) present in blood
(b) C + O2 ---- > CO2 (d) SiC is called carborundum
(c) C+-O2 ---- > CO 94. Which is/are used as ligand in complexes?
2 2
(a) CN’ (b) CO
(d) C6H12O6 + 6O2 ---- > 6CO2 + 6H.0 (c) Both (a) and (b) (d) None of these
85. C02 in water behaves as O2
95. Sulphide mineral + CN" [Af(CN)2T
(a) weak dibasic acid HzCOg
2[Af(CN)2f+Zn---- > [Zn(CN)4f‘ +2M
(b) weak monobasic acid HO-CO2H
M can be
(c) weak diacid base CO(OH)2
(a) Mg, Ag, Au (b) Ag, Au
(d) weak monoacid base HO-CO2H (c) Pt, Au (d) Pt, Ag
86. A certain compound on burning in air forms three 96. When zeolite is treated with hard water, there is
oxides. One of the oxides turned lime water milky, exchange reaction between calcium/magnesium
the other turned anhydrous CuSO4 blue and third ion/and.....
formed a solution of pH = 9. Compound is formed of (a) aluminium ion (b) sodium ion
(a) S, N and H (b) S, N and C (c) water of hydration (d) sulphate ion
(c) S, C and H (d) S, H and Na
97. Select correct statement(s).
87. Concentrations of the atmospheric CO2 have been (a) Si uses all of its valence electrons in an
rising because of sp3-hybridisation and crystallises in an fee
(a) use of fossil fuels (b) acid rain structure similar to diamond.
(c) photochemical smog (d) ozone depletion (b) Diamond is an electrical insulator, graphite is an
88. Principal cause of ozone depletion is the electrical conductor but silicon is a
(a) presence of fluorocarbons semiconductor.
(b) acid rain (c) Most common silica is quartz.
(c) photochemical smog (d) All of the above are correct statements.
(d) green-house effect 98. Silicones are
89. Global warming is called (a) synthetic polymers containing repeated Z^SiO
(a) photochemical smog (b) green-house effect units
(c) acid rain (d) respiration (b) silicates with common SiO4 unit
(c) ketones with silyl group (SiH3) similar to alkyl
90. Green house effect is due to (SiH3)2CO
(a) disturbance of the earth’s delicate thermal (d) Zircon (neso silicates)
balance
532 | Essential Inorganic Chemistry

99. Formula of the following silicate anion is 111. Out of Si, Ge, Sn and Pb which forms stable ionic
compounds in divalent state?
(a) Pb (b) Sn
(c) Ge (d) Si
112. Select correct statement about PbO.
(a) SiO," (b) Si2O," (a) Red oxide is called litharge
(c) SijOf- (d) Si3Ot (b) Yellow oxide is called massicot
100. The minerals having silicate chains are collectively (c) Both (a) and (b) are correct
called (d) None of the above is correct
(a) olivine (b) zircon 113. Pb3O4 reacts with HNO3 forming nitrate and oxide
(c) pyroxenes (d) natrolite which are
101. Glass is best described as a (a) Pb(NO3)4 fromPbO2, and PbO remains unreacted
(a) solid (b) liquid (b) Pb(NO3)2 from PbO, andPbO2 remains unreacted
(c) super cooled liquid (d) colloidal sol (c) Pb(NO3)2 and Pb(NO3)4, and PbO remains
102. Which among the following statements is false unreacted
(d) Pb3O4 remains insoluble inHNO3
regarding glass?
(a) it absorbs ultra-violet light 114. Unstable lead compounds are
(b) it consists of silicates of sodium and alkali (a) PbCl4,PbBr4 andPbl, (b) PbCl2,PbBr2 andPbI2
(c) HF causes marking on glass (c) PbO, PbO2 andPb3O4 (d) PbCl?’,PbCl26’
(d) all the halogen acids (HF, HC1, HBr and HI) are
stored in glass bottles 115. PbCl2 is soluble in
(a) dil. HC1 (b) CH3COOH
103. The colour of blue glass is due to the presence of oxide
(c) hot water (d) diLHjjSt^
of
(a) chromium (b) cobalt
(c) gold (d) silver One or More Than One Options Correct
104. Non-oxide ceramics can be 1. Select correct statement(s).
(a) B4C (b) SiC (a) Graphite is composed of planar, two dimensional
(c) SigN4 (d) all of these sheets of sp2 hybridised carbon atoms.
105. Glass having higher retroactive index is prepared (b) Each sheet is a network of fused, hexagonal rings
using of carbons.
(a) NiO (b) CoO (c) The layers in graphite are held together by
(c) PbO (d) CaO relatively weak van der Waals forces of
attraction.
106. Amphoteric oxides are
(d) Graphite is thermodynamically the most stable
(a) SnO, SnO2 (b) GeO, GeO2
allotropic form of carbon.
(c) both (a) and (b) (d) None of these
2. Select correct statement(s).
107. Compound used on thermal insulator is
(a) In graphite, only three electrons (out of four) are
(a) silica gel (b) zeolites involved in o bonding.
(c) silicones (d) asbestos (b) n-bonding electrons in graphite are delocalised
108. SnCl4 can be used as over the whole structure.
(a) reducing agent (c) Conduction of electricity in graphite is due to
(b) oxidising agent mobile electrons.
(c) catalyst in Friedel Craft’s reaction (d) Diamond is insulator due to absence of mobile
(d) a base electrons.
109. Aqueous solution of SnO liberates CO2 withNaHCO3. 3. Cone. H2SO4 reacts with carbon forming
This shows that SnO is (a) CO2 (b) SO2
(a) an acidic oxide (c) f^COg (d) J^SOg
(b) a basic oxide
4. Carbon suboxide (C3O2) is formed when
(c) an amphoteric oxide
(d) a Lewis acid (a) carbon reacts with cone. E^SOi
110. Which silicon compound is used as lubricant? (b) malonic acid reacts with P4O10
(a) Asbestos (b) Silicones (c) carbon reacts with strong alkali solution
(c) Zeolite (d) Mica (d) oxalic acid is heated strongly
Chapter 12: Group 14 (IVA — The Carbon Family) | 533

5. SiCl4 is hydrolysed but CC14 is not hydrolysed 13. Silicones


because (a) are synthetic polymer containing repeating
(a) Si has vacant d-orbitals and can accommodate (7?2SiO2) units
lone-pair of electrons from oxygen of water (b) are formed by hydrolysis of T^SiC^
(b) Si has relatively large size and can increase its (c) are natural occurring repeating (SiO4) units
coordination number from four to five (d) single B^SiO2 unit
(c) Si has relatively smaller size and thus 14. Select correct statement(s).
interaction with H2O is spontaneous (a) COC12 is called phosphine gas
(d) Si has oxidation number of six while carbon has (b) C3O2 (carbon suboxide) has sp2 hybridised
oxidation number of four in all its compounds. carbons
6. Select correct statement(s). (c) CO is toxic because it forms a complex with
(a) In a-graphite, the layers are arranged in the haemoglobin in the blood
sequence ABAB (d) Diamond is unaffected by concentrated acid but
(b) In |3-graphite, the layers are arranged in the graphite is oxidised to mellitic acid with
sequence ABCABC concentrated HNO3.
(c) In a-graphite, the layers are arranged in the 15. Gases responsible for “green-house effect” are
sequence ABCABC (a) CO2 (b) H>0
(d) In P-graphite, the layers are arranged in the (c) O3 (d) N2
sequence ABAB
7. Select corrrect statement(s). Passage Comprehension Questions
(a) The enthalpy difference between a- and
P-graphite is very large and thus two forms are
Passage 1
not interconvertible Questions are based on following medical report.
(b) The enthalpy difference between a- and p- In the first biological application of buckyball, chemists
graphite is very small and two forms are at the University of California at San Francisco and Santa
interconvertible Barbara made a discovery in 1993 that could help in
(c) a-graphite changes to P-graphite at about 1300 K. designing drugs to treat AIDS. The human
immunodeficiency virus (HIV) that causes AIDS
(d) P-graphite changes to a-graphite at about 1300 K
reproduces by synthesising a long protein chain, which is
8. Select ionic carbides cut into smaller segments by an enzyme called
(a) Al4C3,CaC2,Mg2C3 (b) A14C3, SiC, B4C HIV-protease. One way to stop AIDS, then, might be to
(0 CaC^MgA (d) CaG2,SiC,Mg2C3 inactivate the enzyme. When the chemists reacted a
water-soluble derivative of buckyball with HIV-protease,
9. Silly putty they found that it binds to the portion of the enzyme that
(a) is a silicone polymer would ordinarily cleave the reproductive protein, thereby
(b) has a composition intermediate between silicone preventing the HIV virus from reproducing. Consequently,
oils and silicone rubbers the virus could no longer infect the human cells they had
(c) is an asbestos grown in the laboratory. The buckyball compound itself is
(d) is not related to any chemical. not a suitable drug for use against AIDS because of
10. The great thermal and chemical stability of silicones potential side effects and delivery difficulties, but it does
is attributed to high strength of provide a model for the development of such drugs.
(a) silicon-carbon bond (b) silicon-oxygen bond 1. Buckyball is the allotrope of
(c) silicon-silicon bond (d) carbon-carbon bond (a) phosphorus (b) sulphur
11. Correct statement(s) out of the following is (c) carbon (d) titanium
(a) N(CH3)3 has pyramidal structure 2. What is the formula of the buckyball?
(b) N(SiH3)3 shows planar arrangement (a) P4 (b) S8
(c) SiC is highly volatile (c) Ti3 (d) CgQ
(d) SiO2 is called silane 3. In buckyball each atom is
12. Group 14 (carbon family) elements have the following (a) sp2 hybridized element with extensive
properties delocalised molecular orbital
(a) They are all network solids with octahedral (b) sp2 hybridised element with localised molecular
network orbital
fl)) Only SiO2 is a network solid with tetrahedral (c) sp3 hybridised element with delocalised
coordination and is a giant molecule molecular orbital
(c) SnO2 and PbO2 are amphoteric oxides. (d) sp3 hybridised element with localised molecular
(d) All oxides are acidic orbital
534 | Essential Inorganic Chemistry

4. Consider following statements about buckyball (a) SiO|- S13O7


A : It is also called fullerence (b) SiOj- Si 3^10
B : It is also called buckminster fullerence
C: Bucky tubes (made of fullerenes) are several (c) SiOj- SigO*'
times stronger than steel wires (d) SiO43“ Si3o?-
D : Buckyball is a plastic polymer 2. Si3O|~ (having three tetrahedra) is represented as
Select correct statement(s).
(a) A, C, D (b) A, B, C
(d) A, B, D (d) B, C, D
(a)
Passage 2
Read the following discovery and answer the questions
at the end.
“Elemental carbon appears in many structural forms
or allotropes. Three of these forms are
crystalline—diamond, graphite and the recently (b)
discovered fullerence (buckyball)—while more than 40
others, including coke and carbon black, are amorphous.
Now there seems to be set a fourth crystalline allotrope of
carbon, reported in 1995 by Richard Lagow at the (c) both (a) and (b)
University of Texas.” (d) none of the above
3. The silicate anion in the mineral kinoite is a chain of
1. Newly discovered allotrope of carbon has the form three SiO4 tetrahedra that share comers with
(a) polyyne (b) fullerene adjacent tetrahedra. The mineral also contains Ca ’
(c) buckyball (d) none of these ions, Cu2+ ions, and water molecules in a 1:1:1 ratio
2. Structures of different allotropes of carbon have mineral is represented as
been compared. Which represents incorrect (a) CaCuSi3O10 • H2O (b) CaCuSi3O10 • 2^0
comparison? (c) CagCiLjSigOiQ • 21^0 (d) None of these
(a) Allotrope discovered sp-hybridised carbon
in 1995 Passage 4
(b) Buckyball sp-hybridised carbon Based on following short report, answer the questions
(c) Graphite sp2-hybridised carbon given at the end.
(d) Diamond sp3-hybridised carbon In some foam-type fire extinguishers, the reactants
are Al2(SO4)3(aq) and NaHCO3(ag). When the
Passage 3 extinguisher is activated, these reactants are allowed to
Read the following short write-up and answer the mix producing Al(OH)3(s) and CO2(g). The A1(OH)3—COj
questions at the end of it. foam extinguishes the fire. ,
The name ‘silica’ covers an entire group of minerals,
which have the general formula SiO2, the most common of 1. CO2 is formed as a result of
which is quartz. Quartz is a framework silicate with SiO4 (a) reaction between Al3+ and HCO3
tetrahedra arranged in spirals. The spirals can turn in a (b) reaction between hydrolysis product of Al3+ and
clockwise or anti-clockwise direction—a feature that HCO3
results in there being two mirror images, optically active, (c) reaction between hydrolysis product of NaHCOj
varieties of quartz. and A12(SO4)3
(d) direct reaction between A12(SO4)3 and NaHCOj
1. The following pictures represent various silicate 2. Net ionic reaction of the above chemical change is
anions. Their formulae are respectively: (a) Al3+ + HCO3 + H2O----> A1(OH)3 + CO2
(b) Al3+ + HCO3 + OH“---- > A1(OH)3 + CO2 + HjO
• Silicon (c) [A1(H2O)6]3+ + HCO3---- > A1(OH)3 +C02 + Ho0
O Oxygen (d) Al2(SO4)3+NaHCO3 ---- > Na2SO4 + A1(OH)3
+ HjO j
Chapter 12: Group 14 (IVA — The Carbon Family) | 535

3. Addition ofNa2C03 to a solution of an oxide in water Passage 6


produces CO2. This experiment indicates that Read the following write-ups and answer the questions
(a) the oxide is that of a non-metal at the end of it.
(b) the oxide is basic Silicones are synthetic polymers containing repeated
(c) the oxide is amphoteric T^SiO units. Since, the empirical formula is that of a ketone
(T^CO), the name silicone has been given to these
(d) the oxide is neutral materials. Silicones can be made into oils, rubbery
elastomers and resins. They find a variety of applications
Passage 5 because of their chemical inertness, water repelling nature,
Questions given below are based on following technical heat-resistance and good electrical insulating property.
vport. Commercial silicon polymers are usually methyl
The C02 as well as H20 absorbs strongly in the derivatives and to a lesser extent phenyl derivatives and
ufrared region, and its presence in the atmosphere are synthesised by the hydrolysis of
iecreases the loss of heat from the earth by radiation. This 7?2SiCl2[jR = methyl (Me) or phenyl (0)]
global warming is called the ‘green house effect’ (other Me Me Me
pies, including the oxides of nitrogen from car exhausts, h2o
I
Me2SiCl2 ■> —0—Si—0—Si—0—Si—0—
freons from aerosols and refrigerators and methane from
bacteria in the soil and in the rumen of cows, also add to I I I
Me Me Me
the green house effect). The concentration of atmospheric
CQj has increased by 10%. This is resulting in the increase 1. If we mix SiMe3Cl with SiMe2Cl2, we get silicones of
in the mean temperature of the earth by 2.5°C, varying the type
from 2“C at the equator to 4°C at the poles. This could Me Me Me Me
have dramatic effects on the climate. I
(a) Me—Si—0—Si—0—Si—0 Si Me
1. As a result of green house effect, there can be I
A: increase in rate of evaporation of water, thus, Me Me Me Me
untimely more rain, flooding, etc. Me
Me
B : tropical storms in certain parts of the world I
C: decrease in pH of the soil —O—Si— 0—Si— O—
D: increase in pH of the soil ! I
Select correct alternate (b) 0O <j>
(a) B, C, D (b) A, C, D —0—Si— 0—Si— 0—
(c)A,B,D (d) A, B, C
Me Me
2. Which of the following is growing at a faster rate than
C02 thus responsible for the green house effect? (c) both (a) and (b)
(a) CFC (b) N20 (d) None of the above
(c) 03 (d) CH4 2. If we start with SiMeCl 3 as the starting material,
silicones formed is
3. Instead of monitoring carbon dioxide, suggest Me Me
Me Me
another gas that scientists could study to
substantiate the fact that C02 concentration is
I I
(a) Me—Si—0—Si—0—Si—0—Si—Me
steadily increasing in the atmosphere I I
(a) N20 (b) 02 Me Me Me Me
(c) CH4 (d) 03 Me Me
4. The equilibrium constant (Kp) for the reaction I
—0—Si— 0—Si—0
2C0(g)+ 02(g) 2CO2(g) I
(b) 0 0
is 1.4 x IO90 at 25°C. Given this enormous value, why
doesn’t CO convert totally to C02 in the troposphere? —O—Si— 0—Si—0—
(a) CO forms complex with haemoglobin
(b) CO has low solubility in H20 Me Me
(c) CO has high activation energy (c) both (a) and (b)
(d) CO is toxic in nature (d) None of the above
536 | Essential Inorganic Chemistry

Assertion & Reason 11. Assertion (A): Diamond is insulator and graphite
is conducting.
Codes:
Reason (R): C-atom is sp3 hybridised in both.
(a) Both A and R are true and R is the correct
explanation of A. 12. Assertion (A): Carbon dioxide is a gas but silicon
(b) Both A and R are true but R is not the correct dioxide is an infusible solid.
explanation of A. Reason (R): Carbon dioxide has a linear n-bonded
(c) A is true but R is false. structure.
(d) A is false but R is true.
1. Assertion (A): SiF^- is known but SiCl|" is not True & False
known. 1. N(CH3)3 and N(SiH3)3 both have sp3-hybridised
Reason (R) : Interaction of F lone pair electrons N-atom and pyramidal structure.
with Si is stronger than that of chlorine and due to
2. Cu2+ and Cd2+ both form soluble complex with CN'
smaller size of F steric repulsion will be less.
but are decomposed by H2S.
2. Assertion (A): If a metal exhibits more than one
3. Quartz is framework silicate with SiO4 tetrahedra
oxidation state, the halide in the higher oxidation
state will be more covalent than the one in lower arranged in spirals.
oxidation state. 4. Silicones are synthetic polymers containing repeated
Reason (R): SnCl4, PbCl4, SbCl5 and UF6 are more R^SiO units.
covalent than SnCl2, PbCl2, SbCl3 and UF4. 5. Fe2O3 and CuO impart green colour to the glass.
3. Assertion (A): CO is toxic. 6.
V* PbO
X 9 'is called red-lead.
Reason (R): CO forms complex with haemoglobin 7. Pb3O4 on reaction with HNO3 forms Pb(NO3)2 and
in the blood, and this complex is more stable than
PbO2.
oxy-haemoglobin.
8. Ultra pure silicon is prepared by reduction of Si02
4. Assertion (A): Lime water turns milky when the
gas liberated from an acidified carbonate solution is with carbon.
passed into it. 9. Si becomes a p-type semiconductor when doped with
Reason (R): The liberated CO2 reacts with lime group 13 element (Al, Ga).
water to give calcium bicarbonate as the precipitate. 10. Si becomes an n-type semiconductor when doped
with group 15 element (P, As).
5. Assertion (A): PbCl2 is more stable than PbCl4.
11. Graphite is the least stable form of carbon.
Reason (R): PbCl4 is powerful oxidising agent.
12. Pb(II) salts are easily oxidised to Pb(IV) salts.
6. Assertion (A): SiCl4 is easily hydrolysed as 13. SnCl2 is better reducing agent than HgCl2.
compared to CC14.
14. Finely divided lead powder is pyrophoric.
Reason (R): Silicon can extend its coordination
number beyond four. 15. Pb does not dissolve in hot concentrated HC1 or
H2SO4.
7. Assertion (A): N(CH3)3 is pyamidal but N(SiH3)3
is planar.
Fill in the Blanks
Reason (R): In N(SiH3)3 lone-pair on N-atom is
1. Silicon is the abundant element by mass in
transferred to the empty d-orbitals of silicon (pn-dn the earth’s crust.
overlapping).
2. Graphite consists of planar hexagonal layers of
8. Assertion (A): Between SiCl4 and CC14, only SiCl4 carbon atoms held together by
reacts with water.
3. Silicon exists only in the structure due to
Reason (R): SiCl4 is ionic and CC14 is covalent. tendency to form pn-pn bonds to itself.
9. Assertion (A) : Large number of organic 4. In case of N(CH3)3, geometry is and in case
compounds are formed with long carbon-chain. ofN(SiH3)3 geometry is
Reason (R) : Maximum catenation occurs in 5. Dry ice is
carbon atoms. because they react
6. Be2C and A14C3 are called....
10. Assertion (A): Graphite cannot be converted into with H-jO yielding methane.
diamond.
7. The cyanide ions are toxic because they bind almost
Reason (R): Graphite is conducting in nature. irreversibly to the Fe(III) ion in
Chapter 12 . Group 14 (IVA — The Carbon Family) | 537

8. The CO2 as well as H>O absorbs strongly in the Set I


infrared region and its presence in the atmosphere 1. Total number of atoms of carbon and oxygen in
decreases the loss of heat from the earth by radiation. graphite oxide
This global warming is called 2. When malonic acid is dehydrated using P4O10 at
9. SijN4, B4C and SiC are .... ceramics. 150°C, species formed has total number of atoms
10. SiO2 is , GeO2 is .... SnO is
and PbO is 3. Number of isotopes of carbon
11. Concentrated HNO3 converts graphite into 4. How many of the following oxides are basic oxides?
12. CC14 is kinetically stable towards CO2, SnO2, PbO, PbO2, Pb3O4, A12O3
13. SnCl2 • 2H.0 changes to on heating at 80°C. 5. For the half-cell reaction,
14. CWK is an M2++2e~ ---- > M
15. Graphite fluoride and graphite oxide are E° (in volts) are given for different metals
Cu Zn Fe Pb Sn
-0.14
Matrix-Match Type Questions 0.34 -0.76 -0.48 -0.13

I. One Or More Correct How many of these metals can reduce Cu2^ to Cu?
1. Match the oxide of Column I with their corresponding 6. Out of the elements C, S, Ge, Sn and Pb only +4
property (ies) in Column II. oxidation state is shown by how many element(s)?
Column I Column II 7. More than four bonds are made with how many
1. oxidation state +2 elements in carbon family?
A. Pb3O4
B. SnO2 2. oxidation state +4 8. How many elements in group 14 are used as
C PbO2 3. amphoteric semiconductor?
D. CO2 4. basic
Set II
E. GeO 5. acidic
1. How many of the following have two o and two n bonds?
2. Match the species in Column I with their C*3-,C^-,CO2,Ci-,C!H4,CN-
corresponding property (ies) in Column II.
2. How many of the fuel gases have CO and Hj both as
Column I Column II combustible gases?
A. CO2 1. Dry ice LPG, water gas, Producer gas, Coal gas, CNG
B. AI2(SO4)3 2. Fire extinguisher
3. A silicate mineral has three SiO4 tetrahedra that
C. KCN 3. Acidic
share comers with adjacent tetrahedra. Mineral also
D. CaC2 4. Ligand
contains Ca2+, Cu2+ and HjO in 1:1:1 molar ratio.
E. Mg2C3 5. sp-hybridisation
Thus, total number of atoms of Cu, Si and Ca per unit
6. Green house effect
of mineral is
4. A mixture contains 3 moles of CO and CO2. On
Integer Answer Types passing the mixture over heated charcoal, volume
This section contains 8 questions. X yw z increases to 5 moles. Thus, CO2 in the mixture is
The answer to each of the questions is © © 1] ®] mole(s).
a single digit integer, ranging from 0 to
5. Total number of o and it bonds in carbon suboxide
9. The appropriate bubbles below the © © ©
respectively question numbers in the © © ® © (C3O2)is
ORS have to be darkened. For © 6. What is total negative charge on the silicate [Si4On ] ?
example, if the correct answers to © © © ® 7. CaCO3(s) CaOUJ + CO^g), A' = 10'9 at 300 K
question number X, Y, Z and W (say) © ® ® ®
are 6,0,9 and 2, respectively, then the 0 0. 0 0 What is pC02 ?
correct darkening of bubbles will like ® ® ® ® 8. Inert pair effect is shown by how many elements? Ga,
the following: i® © ® ® Al, Tl, Pb, Sn, Ge
Answers & Solutions
Master Exercises
Exercise 1
Short Answer Type Questions 12. No, as such a high charge separation is unlikely.
I. Due to available d-orbital in which (n -l)p electrons.
can be excited to make C.N. of six. Carbon lacks 13 . . .. . ••
e·c== c· e e ••••
c ••C ••
•••• c ••
9

d-orbital.
2. (a) Pb (b) C (c) Si (d) C 14. Refer text.
3. (a) CC1 4 , CO2 (b) CO (c) CH4 15. Be 2C and Al4C3 are ionic carbides; oxidation state of
carbon =-4
4. Diamond is very hard, high melting solid. It is an
electrical insulator. Diamond has a covalent network 16. H 20 17. 02
structure in which each C-atom uses sp 3 -hybrid orbitals 18. Due to high activation energy.
to form a tetrahedral array of a-bonds. The interlocking,
three-dimensional network of strong bonds makes 19. Highly lustrous form (in layer) of carbon is called
diamond the hardest known substance with highest glanzkohlenstoff. It consists of very small crystals of
m.p. for an element. Because the valence electrons are graphite. It is found to have greater adsorption power
localised in the a-bonds, diamond is an electrical and greater hardness than graphite. This crystalline
insulator. form is obtained when the flame of burning methane is
impinged on glazed porcelain placed at 900 K
5. (a) (A) A : SiF4
20. van der Waals' forces
(B): "2SiF6 -,
(C): H4SiO4 J
21. SiO2 (s)+ Nai'Ois) ---+ Na�iO3{s)+ SO3(g)
3SiF4 + 4H2O ---+ H4SiO4 + 2H�iF6 9H3 CH3
(b) (D) C 6(COOH) 6 , mellitic acid (benzene I
22. H3C-Si-O-Si-CH3
I
hexacarboxylic acid)
(c) (E): (Na�iO3 + C)
I I
CH3 CH3
6. co:- + 2"20 � H2CO3 + 2OH- 23. (CH3COO)3 SiCH3• This would give the cross-linking for
co;- solution is alkaline due to hydrolysis. necessary strength.
7. The Ca 2 + ions give precipitates (scum) with soaps. On 24. (a) SiOt (b) Si3q30 (c) Si40�-
heating the HCO3 ions revert to co;- which gives a
precipitate with Ca 2+ ions :
25. (a)�
Ca(HCOa) 2 ---+ CaCO3 + H:P + CO2
8. Al reacts with NaOH forming H2 gas. NaOH is formed
due to hydrolysis of Na 2CO3 (Q. 6 above) smt S.14010-
13

2Al + 2NaOH+ 2H2O � 2NaA1O2 + 3H2


26. (X) : HgC1 2
9. 2NO(g) + CS2 ---+ 2S(s) + N2 (g) + CO2(g) HgS
(Y):
yellow solid
(Z): Hg 2Cl2
10. Na 2CO3 formed when atmospheric CO2 reacts with
27. Sn is treated with HCl gas to make anhydrous S1iC12•
NaOH.
28. HF attacks silicates of glass
11. (a) To provide a "bite" in soft drinks. rcaSiO3 + 6HF ---+ CaSiF6 + 3H2O
CO2 {g)+ H2O(l) � H2CO3 (aq) I_Na�i03 + 6HF ---+ Na�iF6 + 3H20
(b) As fire extinguisher; CO2 is non-flammable and 1.5
times more dense than air. 29. (a) supercooled (b) HF (c) PbO (d) B2O3
(c) As refrigerant; dry ice sublimes at -78°C. (e) CaF2
Chapter 12 . Group 14 (IVA—The carbon Family) | 539

6. (a) PbO+ NaOH+ H2O Na[Pb(OH)3]


Analytical Questions
(b) 2SnO---- > Sn + SnO2
1. (i) There is feasibility of p-p it bond in silicon.
(c) PbO9 + 4HC1---- > PbCl2 + 2H2O + Cl2
(ii) n-bonding between oxygen and silicon involving
p- and d-orbitals decreases availability of lone-pairs (d) 3PbO + ILjO + CO2 --- > 2PbCO3 Pb(OH)2
pV
of electrons on oxygen. 7. Moles of C09L formed = n = jfrp
RT
2. (a) Triple points : Two Phase 775
=---- x atm x 100 L
Liquid Vapour 760
Point A : Graphite=
= 0.0821 L atm mol"1 K x303K
Point B : Diamond Graphite Liquid
= 4.0992
(b) Diamond If 2 moles CO2 are formed then
(c) Apply high pressure at high temperature
MgSiO3 precipitated is = 1 mol
3. (a) Sn (b) Cl (c) Sn (d) Se (e) B and when 4.0992 moles CO2
4.0992
4. Carbon forms various stable compounds with itself and and formed MgSiO3 would be =
2
other elements such as N. 0, H, S. Bimolecules contain
chains and rings with many C—C bonds. Si—Si bonds = 2.0496 mol
are much less stable and chain of Si atoms are (molar mass of MgSiO3 = 100 g mol"1)
uncommon. = 2.0496 x100 g
5. Pb3O4 on reaction withHNO3 formsPb(NO3)2 andPbO2. = 204.96 g

Pb3O4 + 4HNO3 ---- > 2Pb(NO3)2 + 2H2O + PbO2 8. Refer text.

2 moles ofPb(N03)2 are from dilead(II) oxide while PbO2 NCERT Problems
is from lead(IV) oxide. Thus it is 2PbO PbO2.
Refer Text

Exercise 2
Only One Option Correct
6. (d) 7. (c) 8. (a) 9. (a) 10. (d)
1. (a) 2. (b) 3. (c) 4. (d) 5. (c)
16. (c) 17. (b) 18. (d) 19. (d) 20. (b)
II. (b) 12. (c) 13. (a) 14. (a) 15. (b)
26. (a) 27. (c) 28. (d) 29. (d) 30. (d)
21. (d) 22. (c) 23. (c) 24. (a) 25. (b)
36. (a) 37. (b) 38. (b) 39. (b) 40. (a)
31. (d) 32. (d) 33. (b) 34. (d) 35. (d)
46. (d) 47. (c) 48. (b) 49. (b) 50. (d)
41. (a) 42. (a) 43. (c) 44. (b) 45. (c)
56. (b) 57. (d) 58. (d) 59. (a) 60. (a)
51. (a) 52. (b) 53. (d) 54. (b) 55. (d)
66. (d) 67. (d) 68. (a) 69. (a) 70. (a)
61. (c) 62. (d) 63. (d) 64. (c) 65. (a)
76. (b) 77. (c) 78. (b) 79. (d) 80. (b)
71. (c) 72. (d) 73. (a) 74. (d) 75. (b)
86. (d) 87. (d) 88. (a) 89. (b) 90. (b)
81. (b) 82. (c) 83. (b) 84. (d) 85. (a)
96. (a) 97. (d) 98. (a) 99. (b) 100. (c)
91. (d) 92. (d) 93. (a) 94. (c) 95. (b)
106. (c) 107. (d) 108. (a) 109. (a) 110. (b)
101. (c) 102. (d) 103. (b) 104. (d) 105. (c)
III. (a) 112. (c) 113. (b) 114. (a) 115. (c)

Die or More Than One Options Correct Passage Comprehension Questions


1. (a,b,c,d) 2. (a,b,c,d) 3. (a,b,c,d) 4. (b) Passage 1
7. (b,d) 8. (a) 9. (a,b) 1. (c) 2. (d) 3. (a) 4. (b)
5. (a,b) 6. (a,b)
□0. (a,b) 11. (a,b) 12. (b,c) 13. (a,b) Passage 2
□4. (c,d) 15. (a,b) 1. (a) 2. (b)
540 | Essential Inorganic Chemistry

Passage 3 2. (A)—(1,2,3,5,6); (B)—(2,3); (C)—(4,5); (DM5);


1. (b) 2. (b) 3. (c) (E)—(5)
Passage 4
Integer Answer Types
1. (b) 2. (c) 3. (a)
Sell
Passage 5
Questions 1 2 3 4 5 £_7 8
1. (d) 2. (a) 3. (b) 4. (c) Answers 3 5 3 2 4 4
1_ 2
Passage 6 © © © © © ©|© ©
1. (a) 2. (b) © © © ©
© © © ©. © ©^© ©
Assertion & Reason © © © © © ©
1. (a) 2.2. (b)
(b) 3. (a) 4. (c) 5. (b) 6. (a) © © © © ©
7. (a) 8.8. (c)
(c) 9. (a) 10. (b) 11. (0 12. (b) © © © © © ©
© © © © © © © ©
True & False © © © ©
© © © © © ©
1. F 2. F 3. T 4. T 5. T 6. F 7. T
© © © © © © © ©
8. F 9. T 10. T 11. F 12. F 13. T
14. T 15. T Set II
Questions—> 1 2 3 4 5 £ 7_ 8
Fill in the Blanks Answers —► 3 2 7 2 8 6 9 5
1. Second most 2. van der Waals’ © © © © © ©
forces 3. diamond © © © © ©0. 0
4. pyramidal, planar 5. solid CO2 6. methanides © © © © © ©
7. cytochrome oxidase 8. green house effect © © © © © ©__© ©
9. non-oxide 10. acidic, amphoteric, amphoteric, basic © © ©. © © J® ©
11. mellitic acid C6(COOH)|i6 12. hydrolysis © © © © © ©
13. SnCl4 H2O 14. electrically conducting compound © © © © © ©© ©
15. electrically non-conducting compounds © © ©. © ©
© © © © © ©
Matrix-Match Type Questions © © © © © © © ©
I. One or More Correct
1. (A)—(1,2,3); (B)—(2,3); (CJ—(2,4); (DH2,5);
(E)-(l,3);
"Whether it may be laughing gas (which does not
make you laugh) or Holme's signal or flickering light
(called ‘will-o-the-wisp’) are clues from Nitrogen family"

Group 15
(VA—The Nitrogen Family)

The elements in this group (N, P, As, Sb, Bi) are also called
Pnictogen (pnicogen) or pnictides from the Greek synonym for
choking (as nitrogen does not support combustion). The
elements N and P are essential constituents of the living
system. Heavier members of the group, particularly arsenic are ■ Group 15 —Periodicity 7n
extremely toxic. Study of Nitrogen and
15P
Nitrogen is highly abundant (about 78% by volume and 75% ■ its Compounds
by mass) in air as N2. However abundance of nitrogen in rocks 33AS
■ Study of Phosphorus and
and soils is low (19 ppm) as KNO3 and NaNO3. ■ its Compounds 51Sb
Naturally occurring nitrogen consists of14N stable isotope,
83®'
15N also occurs slightly, relative abundance being 14N/15N
*272:1
Phosphorus is highly abundant in crystal rocks
Is 1120 ppm) most as orthophosphate. It also occurs in plants
and animals where it is an essential constituent of life.
31P is the only stable isotope of phosphorus. Among six
radioactive isotopes, 32P(p-emitter) is most important.
The elements As, Sb, Bi have very low abundances
As < 2 ppm
Sb 0-2 ppm
Bi 0-01 ppm
They occur as sulphides (As2S3, Sb2S3, Bi2S3) and are thus
chalcophilic.
542 | Essential Inorganic Chemistry

Group 15 —Periodicity
Physical properties of the elements in nitrogen family [Group 15 (VA)] are summarised in Table 13.1 and Fig. 13.1.

Table 13.1 Physical Properties of the Elements of Group 15 (VA)


n p As Sb Bi

Electronic configuration [He] 2s2 2p3 [Ne] 3s23p3 [Ar] 3d104s24p3 [Kr] 4d105s25p3 [Xe] 4f'45d106s26p3
Electronegativity (Pauling) /EkJmol 3.0 2.1 2.0 1.9 1.9
(/E)i 1403 1012 947 834 703
(/f)2 2857 1897 1950 1590 1610
(IE)3 4578 2910 2732 2440 2467
Melting point/0 C -210 44 (white) 597 816 631 271
(red)
Boiling point/0 C -195.8 281 (white) 615 1587 1564
431 (red) (sublimes)
Atomic radius/ pm 74 110 121 141 152
Principal oxidation number +3,0,+5,-3,-2,—1 +1, +3, + 5 +3, + 5 +3, + 5 +3
+2,+4
Abundance 0.0002% 0.00002% 0.00002%
Physical state (1 atm, 25°C) gas solid solid solid solid

3.5-r 2000-r 1500-r,


3

2.5--
1000------
Pauling 2--
units K1000-- kJ mol-1
1.5--
500 -
1—
0.5--
0 0 0
N ' P As Sb1 Bi N ' P ‘As'Sb1 Bi1 N 1 P 'As’Sb1 Bi'
(a) (b) (c)
Fig. 13.1 Some group trends of the group 15 elements: (a) Pauling electronegativity, (b) Boiling point, and the
(c) First ionisation energy

Physical State nature. In keeping with this trend, the electronegativity of


Group 15 elements are sometimes referred to as these elements decreases gradually on moving down the
pnictogens. All the elements are important and bismuth group. The density, melting and boiling points of the
is notable for being the heaviest element with elements increase with increase in atomic number.
non-radioactive isotope.
Elements of this group show a systematic gradation of DEEP Focus
properties (Table 13.1) Nitrogen is gaseous in nature and
exists as a diatomic molecule N =N. This is due to the very
Radii, IE, Oxidation State
high bond energy of the N =N bond, 946 kJ mol-1, which is Atomic radii, as expected, increase with increase in atomic
nearly six times the N—N bond energy, 160 kJ mol1. On number. As a consequence, ionisation energies decrease on
descending the group. Nitrogen shows an exceptionally high
the contrary P=P bond is quite weak, its energy of
ionisation energy which is due to its small atomic size. As
490 kJ mol-1 is only ~ 2.5 times the P—P bond energy, discussed above, all the elements of this group have fire
209 kJ mol-1. Therefore, phosphorus exists as electrons in their valence shell, they exhibit a highest oxidation
tetra-atomicP4 molecules in the gaseous state. In the solid state of +5 by using all five electrons in forming bonds. As one
state, it exists in different allotropic forms. goes down the group, the tendency of the pair of s electrons to
The change from non-metallic to metallic character remain inert increases and only p-electrons are used in
with increase in the atomic number is well illustrated in bonding, resulting in an oxidation state of +3. The stability of
this group. The first two elements, nitrogen and +3 state relative to the +5 state thus increases on moving down
phosphorus, are non-metals, arsenic and antimony are the group. In the case of nitrogen, avery wide range of oxidation
metalloids whereas bismuth is predominantly metallic in state exists:
Chapter 13: Group 15 (VA—The Nitrogen Family) | 543

• Metallic nature increases as we go down the group, N


Compound Oxidation state of N
and P are typical non-metals, As and Sb are
NH3 -3 metalloids and Bi is a metal.
n2h4 -2 • Phosphorus and the other members of the group can
NH4OH
make use of d-orbitals in their bonding, thus
compounds MX3 as well as MX5 are formed (PC13 PC15
n2 o etc.), butNCl5 is not formed; N does not have d-orbital.
n2o +1
NO +2 DEEP Focus
no2 +4 • Stability of+3 state increases and that of +5 state decreases
hno3 +5 going down the group due to “inert pair effect”, thus
Bi3* > Sb3* > As3*
The negative oxidation state of nitrogen arise because Bi5* < Sb5* < As5*
of its higher electronegativity (3.05) than that of hydrogen Bi3* salts are very rare and are good oxidising agents :
(2.1). As you can see from the Table 13.1 the energy Bi5* + 2e----- > Bi3*
required to produce M5+ is so immense that it is nevery
As3* salts are good reducing agents :
formed. The compounds formed by these elements in +5
As3* ---- > As5* + 2e“
oxidation state are thus predominantly covalent.
However, in case of Sb and Bi, the sum of the first three
ionisation energies is just low enough to form M3+ ions. Nature of Compounds
But fluorine is the only element which gives an • As we go down the group, basic nature of oxides
electronegativity difference enough to permit the increases and acidic nature decreases. Oxides of N and
formation of ionic bonds. Thus, SbF3 and BiF3 exist as ionic P are acidic, those of As and Sb are amphoteric and
solids. that of Bi are basic.
Element Oxides Trend
DEEP Focus N N2O.NO.N2O3.N2O4.NO2.N2O5 acidic
Since, it is not easy from energy considerations to gain
three electrons to achieve stable noble gas configuration, the P P4O6. P4O10
formation of the tri-negative ion (Af3-) is rare. But, nitrogen As As4O6. AS4O10 amphoteric
being the smallest and the most electronegative of all the Sb Sb4O6, Sb4O10
members of the group, forms trivalent anion with reactive basic
i metals which have low ionisation energies. Thus, nitrogen Bi Bi2O3
forms ionic nitrides such asLi3N,Be3N2, Mg3N2,Ca3N2. As the
electronegativity of the other elements of the group, i.e., P, As,
Sb and Bi, is low, most of their compounds are covalent in
DEEP Focus
nature. They form trihalide and pentahalide (except N, which does
not form NC16 etc). As we go down the group, covalent
Nitrogen is not able to extend its coordination number nature decreases and ionic nature increases; thus NC13
beyond four, the other elements have coordination and PC13 are covalent while BiCl3 is ionic. This is also in
accordance with Fajan’s rule : Smaller the size of cation,
numbers of five as well as six. The extension of
larger the size of anion, greater the charge then greater the
coordination number is due to the availability of d-orbitals polarising power and hence greater the covalent nature.
in P, As, Sb and Bi, using which these elements can attain
ip3d and sp3d2 hybridisation, e.g., in PC15 and PCI6, Halides are hydrolysed in water forming oxyacids or
respectively. Thus, nitrogen does not function as a oxychlorides:
Lewis acid whereas phosphorus, arsenic, antimony PC13 + 3H2O-----> H3PO3 + 3HC1
-and bismuth do so. The hydrides of this group and
SbCl3 + H2O-----> SbOCl 4- + 2HC1
"their alkyl derivatives act as Lewis bases, formming (orange ppt)
a number of addition complexes, e.g., (NH3‘ BF3),
BiCl3 + H20-----> BiOCl + 2HC1
:CaCl2 • 4NH3), etc. (white ppt)
• N and P show the typical properties of non-metals : • Trihalides (PF3) have a pyramidal structure in the
- are poor conductors of heat and electricity, gaseous state.
• Nitrogen is relatively unreactive, which is why it has
- form acidic oxides,
accumulated in such large amounts in the
- form covalent compounds predominantly.
atmosphere.
544 | Essential Inorganic Chemistry

White phosphorus catches fire when exposed to air, Solution The angle required by P4 is 60°. The strain
burning to form P4O10. It is stored under water to would not be reduced by using sp3 hybrid orbitals because
prevent this. Red phosphorus is stable in air at room their angle is - 109°.
temperature, though it reacts on heating.
Arsenic is stable in dry air, but tarnishes in moist air,
giving first a brown and then a black tarnish. White Study of Nitrogen and its Compounds
arsenic is As2O3. • N2 makes up almost 78% of the atmosphere, and is
Antimony is less reactive, and is stable towards water therefore the most common gas. N2 is unreactive
and to air at room temperature. (inert), however it reacts with some metals forming
metal nitrides which are ionic with N3" ion :
Hydrides 3Mg + N2 ---- > MgjN2 (magnesium nitride)
• Elements of group 15 (VA), all form volatile hydrides • The most important compounds of nitrogen are in
of formula MH3, which are all poisonous, foul smelling living things, e.g., amino acids and proteins. Among
gases. On descending the group from NH3 to BiH3 : other things, the high electronegativity of nitrogen
- the hydrides become increasingly difficult to allows it to take part in H-bonding and thus helps to
prepare keep the strands of DNA together.
- their stability decreases • Some microorganisms capture nitrogen from the
- their reducing power decreases atmosphere and convert it into useful compounds—a
- the ease of replacing hydrogen atoms by other case of nitrogen-fixation. It involves a complicated
groups such as Cl or Me (methyl) decreases system of biochemical reactions involving an enzyme,
- Lewis-base nature (tendency to donate lone pair of nitrogenase and adenosine triphosphate (ATP).
electrons) decreases Some plants fix nitrogen by use of Rhizobium
- PH3 is a much weaker base than NH3; AsH3; SbH3 bacteria, which are often found in the root system.
and BiH3 do not show any basic properties. Nitrogen is also fixed by chemical reactions in the
- hydrides have a pyramidal structure atmosphere, particularly in thunderstorms.
- boiling point increases but NH3 is the N2(g) + O2(g) lightnin^ 2NO(g)
exception due to intermolecular H-bonding
2NO(g) + O2(g) -> 2NO2(g)
PH3 < AsH3 < NH3 < SbH3
-87°C -55°C -33.5°C -18°C 2NO2(g) + H2O(Z) ■ * HNO2(ag) + HN03(a<?)
(BiH3 is very unstable)
— bond angle decreases DEEP Focus
NH3 > PH3 > AsH3 > SbH3 • Hence, the energy released by lightning converts nitrogen
into nitrogen oxides and nitric acid, which falls to earth in
• The ionisation energy to produce M5+ is so large for N the rain. However, the proportion of nitrogen in the
that it does not show +5 oxidation state. Nitrogen also atmosphere is not continually decreasing because that lost
differs from other elements in that it can form strong by fixation is replaced by the decomposition of nitrogenous
pit-pit multiple bonds and hence it produces a compounds by bacteria. This process is called
number of compounds which have no counter parts in denitrification. Currently, the total rate affixation
other elements. Thus, NO3, NO2, N3 (azide), C=N", exceeds the rate of return of nitrogen to the
N2O, NO, NO2, N2O4, N2O3, N=N, azo and diazo atmosphere which means that nitrogen compounds
compounds are formed. Oxides N2O3 and N2O5 are are accumulating in soil, ground water, surface
monomeric while trioxides and pentoxides of other water and the oceans with subsequent environmental
elements are dimeric as inP4O6 andP4O10.Phosphorus effects, e.g., NOj occurs in ground water in higher
and the heavier elements of this group show dn-pit concentration as a result of fertilisers runoff and drainage
bonding. from cattle feedlots. When this water is drunk, nitrate
reducing bacteria in the gastro-intestinal tract can convert
Illustration 1 Could the strain in the P4 molecule be NO3 into NOj. NOj interferes with the oxygen transport
reduced by using sp3 hybrid orbitals in bonding instead of ability of blood, which can lead to a condition known as
pure p-orbitals? Explain. methemoglobinemia. This order can be fatal in infants.
Chapter 13: Group 15 (VA—The Nitrogen Family) | 545

Nitrogen cycle is represented in Fig. 13.2. •Ona commercial scale, N2 is obtained by the fractional
evaporation of liquid air. Liquid air usually contains
50-60 per cent oxygen instead of 23 per cent in
Atmospheric gaseous air. Since N2 and differ widely in their b.p.
1 nitrogen j
(N2=-195°C, O2=-183oC) hence separation of N2
J and (^ can be easily affected by fractional distillation.
Plant
and
N2 thus obtained has some traces of rare gases like He
animal and Ne.
protein
Nitrates
Nitrogen Oxides
• The oxides show the wide range of oxidation numbers
decay
bacteria of N, from +1 to +5.
• All the oxides are covalent.
• All are gases at room temperature (except N2O5 which
bacteria Ammonia
is unstable solid at room temperature).
Nitrites
• The lower oxides are neutral and the higher ones are
acidic. Oxides are summarised in Table 13.2.
Fig. 13.2 Nitrogen cycle in nature

Table 13.2 Oxides of Nitrogen


Oxidation Magnetic
Name Formula Oxyacid Preparation
no. properties
1 Dinitrogen oxide (nitrous N2O h2n2o2 Diamagnetic
+1 NH4NO3 N2O + 2H2O
oxide) (hyponitrous acid)
2. Nitrogen monoxide (nitric NO Paramagnetic
+2 3Cu + 8HNO3 —3Cu(NO3)2
oxide) dil.
+ 2ND + 4H2O
3 Nitrogen dioxide no2 +4 2Pb(NO3)2 2PbO + 4NO2 + O2 Paramagnetic
4. Dinitrogen tetraoxide N2O4 +4 Cool NO2, N2O4 2NO2 Diamagnetic
----- cool--- 1

5. Dinitrogen trioxide N2O3 hno2 +3 NO+NO2----- >N2O3 Diamagnetic


(nitrogen sesquioxide)
2HNO2 N2O3 + H2O

6 Dimtrogen pentoxide n2o5 HNO3 +5 2HNO3 n2o5 + h2o Diamagnetic

Nitrous oxide (N20) • It forms sodium azide on reaction with NaNH2 :


• NH4NO3 A,250°C) N2O + 2H2O NaNH2 + N2O---- > NaN3 + H2O
(molten) NaNH2 + H2O---- > NaOH + NH3
• NH20H + HN02 > N2O + 2H2O 2NaNH2 + N2O * NaN3 + NaOH + NH3
hydroxyl amine
• It is stable, relativity unreactive colourless gas, it is a • N2O resembles molecular oxygen (Q>) in that in
neutral oxide. supports combustion. It does so because it decomposes
to molecular nitrogen and molecular oxygen when
• It is soluble in water (1:1 by volume), but solution is
heated
neutral probably the following equilibrium lies in
left-hand side 2N2O---- > 2N2 + 02
N2O + H2O H2N2O2 • The molecule is linear as would be expected for
hyponitrous acid a triatomic molecule with 16 outer shell electrons
It may be taken as anhydride of H2N209 (as for COo).
546 | Essential Inorganic Chemistry

:n=n—o: <—> :n=n=o: • It can be represented by the following resonance


structures:
N N 1.86 A 0 N=O <—* e'N=0®
• The largest use of N20 is as a propellant for whipped Due to one unpaired electron (total 11 valence
ice-cream. Because it has no taste, and is non-toxic, it electrons) it is paramagnetic. In solid and liquid state
meets the strict food and health regulations. it dimerises forming N2O2 and paramagnetism
• N20 is also used as an anaesthetic, particularly by disappears.
dentists. It is also called laughing gas, because small O—N—N—0 N— 0
amounts cause euphoria. (I)
0—N—0—N 0—N
NaNO3 + NH4CI
(ID dimer
II is asymmetrical and is formed as a red solid in the
NH4NO3
presence of HC1.
N2 + O2 HNO2
A
dil HNO3 CD
NaNH2 \Cu or
NaN3 * N2O NH2OH + HNO2
,g>
\Hg
H+ H2O A
NO
n3h h2n2o2 n2 +o2 02^
Cb Fe2+
NO2 30-50°C
high pressure [Fe(H2O)5N0f
Nitric Oxide (NO) NOCI
N2O + NO2 (brown ring)
• N2 "F 02 ---- * 2N0
. • 3Cu +8HNO3 * 3Cu(NO3)2 + 2N0 + 4H2O
dil Nitrogen dioxide (N02), dinitrogen tetraoxide
• 2HNO2 + 2F + 2H+ ---- > 2N0 +12 + 2H2O (N204)
A
• NO can be prepared in the laboratory by reacting Cu + 4H+ + 2NO3 Cu2+ + 2H2O + 2NO2
sodium nitrite (NaN02) with a reducing agent such as 2Pb(NO3)2 -A» 2PbO + 4NO2 + 02
Fe2+ in an acidic medium
On condensation, N02 is obtained as a pale liquid
N02" (aq) + Fe2+(aq) + 2H+(aq) -» NO(g)
and ultimately a colourless solid (N204 due to
+ Fe3+(aq) + H2O(Z) dimerisation).
• 2NO3 + 6Hg + 8H+ ---- > 3Hg2++ 2N0 + 4H2O • N02 is paramagnetic due to unpaired >z0

• Lighting also contributes to the atmospheric electron but in dimer formation


concentration of NO. Exposed to air, nitric oxide paramagnetism disappears due to pairing
0
quickly forms brown nitrogen dioxide gas: of unpaired electrons
2NO(g) + O2(g)---- » 2NO2(g) 2NO2 N2O4
(brown fumes) ,0
0K
Nitrogen dioxide is a major component of smog. N ■N
0 0
DEEP Focus This reaction occurs in both the gas phase and the
• NO is an odd-electron molecule; alternately we can say it is
a free-radical. Hence it is highly reactive : liquid phase.
2N0 + Cl2 ---- > 2N0C1 • It is represented by resonance structures:
3N0 v 30-50°C_> n o + no
high pressure *• *
:o\
N- -<—> •N-
NO readily forms coordination complexes with transition :o: :o:
metal ions. These complexes are called nitrosyls.
[Fe(H2O)5NOl2+ (brown ring) • N02 molecule is angular with an 0—N—0 angle of
132° and N—0 bond length of 1.20A (which is
• Bond-order is 2.5 (as described by M.O. theory) and intermediate between a single and a double bond).
bond length is 1.15A which is intermediate between a
double bond and a triple bond.
Chapter 13: Group 15 (VA—The Nitrogen Family) | 547

DEEP Focus Dinitrogen Trioxide (N203)


• N2O4 has planar structure studied by X-ray • It is obtained by condensing equimolar amount of NO
Ok , /O andNO2 together, or by reacting NO with O2 in a fixed
jqr 1-64 A N ratio.
o 0 NO + NOz n2o3
N-N bond length (1.64 A) in N2O4 is about 20% larger than 4NO + O2 ■> 2N2O3
the usual N—N single bond. We can think of the positive • It is an acidic oxide and is the anhydride of HNO2
charge formed on the adjacent N atoms which causes
N2O3 + H2O--- > 2HNQ,
repulsion and thus increases the bond length.
N2O3 + 2NaOH---- > 2NaNO2 + H20
• N2O4 is a mixed anhydride as indicated by its reaction sodium nitrite
with H2O when HNO2 (nitrous acid) andHNO3 (nitric • It is an intensely blue liquid or a pale blue solid. It
acid) are formed
exists only at low temperature and readily dissociates
n2o4 +h2o hno2 +hno3 to give NO and NOz
u
2NQj
N2O3 NO + NO2
• The N2O3 molecule has an QjN—NO structure in the
TheHNOz formed decomposes to give NO andNOz gas phase and at low temperature, with an extremely
long (1.86A) N—N bond which is consistent with its
2HNO2 -----> NOa + NO + H2O
easy dissociation.
Thus moist NO2 and N2O4 gases are strongly acidic. 0. 128.6" xO
• Liquid N2O4 is useful as a non-aqueous solvent and
self-ionises as
n204 NO+ +NO3 1.86A p
.105*
acid base N
Thus, ZnCl2 + 2N2O4 -----> 2NOC1 + Zn(NO3)2
• N02—N2O4 system is a strong oxidising agent Two interconvertible structures (in presence of light
2NO2 2NO + O2 radiations of a particular wavelength) are assigned to it.
^0
2NO2 + F2 ------4 2NO2F ■N—N: O=N N=O
nitryl fluoride 0
2NO2 +4HC1 4 2NOC1 +C12 +2H2O NO + NO2
nitrosyl
chloride
NO2 +CO-----> COa +NO nh4oh h2o
NH4NO2 N2O3 - hno2
2NO2 N2O4 A NaOH
N2O4 solidifies at -11.2° C and is colourless.
N2 + H2O NaNO2
As the temperature increases, proportion of brown
SOi increases and the colour of the liquid darkens. At
around 140° C, the mixture is almost entirely NO2. Dinitrogen Pentoxide (N205)
• 2HNO3 -?2-5 > N2O5 + H2O
Pb(NO3)2 Cu + HNO3 (cone) low temp.
A I Thus N2O5 is the anhydride of HNO3.
• It is colourless deliquescent solid and highly reactive.
h2o
no2f* F2 no2 N2O4 hno2 + HNO3 • In the gaseous phase :
A N2O5 NQz 4-NO + Oz
CO
NOCI + Cl2 N2O3 + O3 -----> 2NO3 + Oz
no2 + no + h2o
CO2 + NO NO+ + NO3 • In solid state :
tacid base,
NA + H2O-----> 2HNO3
|znCI2
N2O5 + Na---- > NaNO3 + NC^
Zn(NO3)2 + NOCI N2O5 +NaCl-----> NaNO3 +NOzCl
548 | Essential Inorganic Chemistry

• With cone. H2SO4, it forms N02 (nitronium ion) which Solution N02 is an acidic oxide; it reacts rapidly wii
can attack benzene nucleus forming nitrobenzene cold water to form both nitrous acid, HN02, and nitric ad
N2O5 + 3H2SO4 4 2NO2 + 3HSO4 + H30+ HN03
C6H6 + N02+ 4 C6H5NO2 + H+ 2NO2(g) + H2O(Z) 4 HNO2(g) +HNO3(j
• X-ray diffraction shows that solid N2O5 is ionic Oxidation T ?
+3
T
+5
+4
N02 N03 (nitronium nitrate). In the solution and number
gaseous phase, it is covalent with structure :
This is a disproportionation reaction in which th
0> ,0
oxidation number of nitrogen changes from +4 (in NO;
N—O—N to +3 (in HN02) and +5 (in HN03). This reaction i
cr 0 quite different from that between C02 and H20 i
Each N atom is sp2-hybridised. which only one acid (H2CO3, carbonic acid) is forme
hno3 I without change in oxidation number of carbon
hno3 H20
C02 + H20 H2CO3
P2O5 A N02 + NO + 02
NaNO3 + NO2 Na (in solid phase) N20j (in gaseous phase)
NaCI (N02 +N03)
A
^no; + o2
Ammonia (NH3)
H2SO4 • NH3 is formed by decomposing NH4 salts with a basi
NaNO3 + NO2CI (radical)
A 4 NH3+H2O + X’
no2 no2 NHtX + OH'
catalyst
N2 + 3H2 2NH3 AH = —40 kJ mol
Nitrogen Oxides and Air Pollution
■&298 “ a^IID
Collectively, nitrogen oxides are often given the
symbol NOt. Two of them, NO and N02 are of special This is the basis of the Haber process and is carriec
importance in environmental chemistry, causing out at 400-500°C and at a pressure of H^-IO3 atm in th<
considerable amount of air pollution. They are given off in presence of a catalyst.
car exhaust fumes and when fossil fuels are burnt, as well
as being made during thunderstorms. In each case NO is CaNCN + 3H2O-----» 2NH3 + CaCO3
calcium cyanamide
made and thenNO2.
• Ammonia is extremely soluble in water. Although
N2 + 02 4 2N0 aqueous solution is generally referred to as solution ci
weak base NH4OH, undissociated NH4OH does noi
DEEP Focus exist. The solutions are best described as NH3(ag)with
N02 is more unpleasant and gives rise to acid rain and also equilibrium :
takes part in photochemical reactions forming smog. The NH3(ag) + H20 NH4 + OH'
photons making up sunlight can cause the N02 molecules to
fall apart, releasing oxygen atoms: [NH|][QH~]
^6 = = 1.8 x KT5 mol L"1
N02 -^4 NO + 0 [NH3]
Oxygen atoms are extremely reactive and will attack other • NH3 is colourless, pungent and quite poisonous gas
molecules, especially those in car exhaust fumes. For examples (b.p-33.35° C)
hydrocarbons that escape combustion can be changed into
aldehydes. The oxygen atoms can also react with 02 to form 03. DEEP Focus
N02 is also involved in the production of one of the most
unpleasant of all chemicals present in photochemical smog. It Oxidation
is known as PAN (peroxyacetyl nitrate):
/O
• 4NH, + 30, ---- 4 2N, + 6H,0
v £• L L

(PAN) CH,—C This reaction is thermodynamically favoured under normal


V

xo—0—no2 conditions. However, at 750-900°C, in the presence of Pi


catalyst, NO is formed.
Its presence in the atmosphere irritates eyes and lungs
and generally makes breathing difficult. 4NH, + 50, Pt, A 4N0 + 6H,0
Thus NO can be converted into N02 which in turn intoHNOj
Illustration 1 What is difference in the nature of the 2N0 + 02 ---- > 2NO2
products obtained when CO2(g) and NO2(g) are dissolved
3NO2 + H20---- 4 2HNO3 + NO
separately in water?
Chapter 13: Group 15 (VA—The Nitrogen Family) | 549

Thus the sequence in industrial utilisation of atmospheric


nitrogen is as follows:
Test
N2 ___ Sa——, NH ------- --------- > NO • 2K2Hgl4 + NH3 + 3K0H H2NHgOHgU
Haber process Ostwald process Nessler's iodide of Millon’s base
reagent (brown ppt)
O2+H2O?
HNOo V
+ 7KI + 2H2O
CaSO4 + 2NH3 + CO2 + H20 CaCO3
• Liquid NH3 resembles water in its physical behaviour,
+ (NH!)2SO4
being highly associated via strong hydrogen bonding.
A
LikeH2O it also self-ionises: • 2NH3 + CO2 NH2COONH4 NH2CONH2
ammonium urea
H2O + H2O H3O+ + OH" carbamate
nh3 + NH3 nh; + nh2 + h2o
or NH3 H+ + NH2 [Ag(NH3)2]+
NH>- N2 + H2
CaCN2
^\AgCi
A OH"
As a Ligand ■/h2o
O2
• Because NH3(Z) has a much lower dielectric constant n2 NH3 —► nh2 + nh;
normal
\H2O2
than water, it is a better solvent for organic conditions
<^^hno2
compounds but generally a poorer one for ionic Ostwald O2
Pt, A ^*lN2 (if NaClO in excess)
inorganic compounds. But complex formation by NH3
N2H4 (if NH3 in excess)
(as ligand) is better than that by H2O. AgCl is insoluble nh4ci + n2 O2
in H2O but soluble in NH3 due to complex formation : (if NH3 in excess) NO M no2 b---- 1
NCI3 + 3HCI
AgCl + 2NH3 -----> [Ag(NH3)2]Cl (if Cl2 in excess ) H2O O2

HNO3 + NO
Reducing Nature
• NH3 can reduce oxides into metal:
3CuO + 2NH3 A 3Cu + N2 +3H2O
DEEP Focus
• Liquid ammonia functions as a good solvent for many
A substances and many types of reactions. All those
3PbO + 2NH3 3Pb + N2 + 3H2O
substances which dissolve in liquid NH3 to give NH;
» «
8NH3 + 3C12 > N2 + 6NH4CI ions are acids and those which give amides, NHj ions are
excess bases. NH4C1, NH4NO3 are acids in liquid NH3 due to
NH3 + 3C12 > NC13+3HC1 existence of NH; ions.
excess nh4.ci A nh4 : + Cl"
NH3 + 3H2O2 ---- > HNO2 + 4H2O NH.NO, nh:4 + noJ;
4 O x

• 2NH3 + 3NaC10 N2 + 3NaCl + 3H2O


sodium hypochlorite NaNH2 is a base in liquid NH3 due to existence ofNH2
excess ion:
2NH3 + NaClO--- > NH2—NH2 + NaCl + H2O NaNH2 Na+ + NH2
excess hydrazine
Acid-base neutralisation reaction in liquid NH3 can be
2NH3 +3CaOCl2 ■> 3CaCl2 + N2 + 3H2O thus represented as,
bleaching NH3
powder 2NH4C1 +PbNH > PbCl2 + 3NH3
acid base
nh3
3NH4C1 + AIN A1C13 + 4NH3
Solvated Electrons
• Acids which are weak in water will be highly acidic in
• 2NH3 + 2Na---- > 2NaNH2 + H2
sodamide liquid NH3:
Dilute solutions of alkali metals in liquid NH3 are ch3cooh + h2o CH3COO" + h3o+
weak acid
dark blue in colour and the main species are solvated
metal ions and solvated electrons. CH3COOH + NH3 ch3coo- + nh;
strong acid
550 | Essential Inorganic Chemistry

Hydrazine (N2H4) Hydrazinium salts of N2Hs are stable in water but


those of N2H26+ are hydrolysed in aqueous solution.
Preparation Such salts as N2H|+ are less soluble than those of
Raschig Process N2Hg and may be crystallised in presence of acid.
It is obtained by controlled oxidation of NH3 (in These series of salts are formed with H2SO4
excess) by alkaline solution of sodium hypochlorite (N2Hs)2SO4 withN2H£ cation
(NaOCl):
(N2H6)SO4 withN2H@+ cation
2NH3+NaOCl -----> N2Hj + NaCl + H2O
(N2H6)(HSO4)2withN2H6+ cation
Reaction involves rapid formation of chloramine
• Reducing Nature : N2H4 is oxidised to N2 and
(NH2C1)
thus it is a better reducing agent.
NH3+OCI- -----> nh2ci+oh-
In acidic medium
NH2C1 changes to N2H4 by path I or path II
N2Hs+ ----- > N2 +5H+ + 46-
Path I It involves nucleophilic attack of NH3
In basic medium
followed by neutralisation with alkali
NH2C1+NH2—H -^4 [NH3— NH2]*+C1- N2H4 + 4OH- -----> N2 + 4H2O + 4e"
r, . , x molar mass
fo cf Equivalent mass =-------- -------
[NH3—NH2r+OH- -^4 NH2— NH2+H2O
32 o _ , ,-i
= — = 8.0 g equivalent
Path II There is formation of chloramide ion eNHCl
(in fast step) followed by nucleophilic attack of NH3
- AuC13 is reduced to N2
(in slow step)
4AuC13+3N2H4 ----- > 4Au+3N2+12HC1
nh2ci+oh- ^4 "NHC1+H2O - CuO is reduced to Cu2O (red ppt)
slow 4CuO+N2H4 -^4 2Cu2O + N2 + 2H20
-nhci+nh3 -^4 n2H4 +cr
A side reaction decreases yield of NijHj - IO3 is reduced to I+
N2H4 +2NH2C1 -^4 N2 +2NH4C1 N2H4 + K1O3 + 2HC1-----> N2 + KC1 + IC1
T t
The reaction is catalysed by traces of heavy metal ions (O.N. =+5) (O.N.=+1)
(e.g., Cu2+). Addition of gelatin or glue suppresses the side + 3H2O
reaction by complexing the metal ions and yield of
hydrazine is increased. - AgNO3 is reduced to Ag
4AgNO3+N2H4 ----- > 4Ag + 4HNO3+N2
Industrially and laboratory method involve the same
type of reactants. Urea may replace ammonia. - PtCl4 is reduced to Pt
PtCl4 +N2H4 -----> Pt + 4HC1+N2
Properties N^ is oxidised to NH3 and N2 or N3
• Colourless fuming liquid with faint ammoniacal Medium
odour.
Acidic N2Hs+ -----> NH; + -N2 +H++e"
Melting point : 20°C 2
Boiling p[oint : 113.5°C Basic N2H4 + OH" -----> NH3 +1N2 + H2O + e
Density : 1 g/cm3 at 25°C 2
Dielectric : 51.7at25°C Equivalent mass = molar mass
constant 1 15
AH/ : 50.6 kJ mol-1 (an endothermic compound) Acidic -----> -NHJ + -N3H + -H*+2e’
2 2 2
ag; : 149.0 kJ mol"1 Basic N2H5++-OH" ----- > -NH3 + iN3 +^H,0
^2 2 2 2
• Pure hydrazine and its aqueous solution are
+2Z
kinetically stable inspite of its endothermic nature.
molar mass
• Basic Nature It is a weaker base than NH3 but its Equivalent mass =
2
behaviour is bifunctional.
Fe3+ is reduced to Fe2* Fe3+ + e- —> Fe2+
NaH^+HaO N2Hs+ (aq) + OH"^ = 8.5 x IO"7
N2H5+(^) + H2O NaHV+OH" Kb = 8.9 x IO"16 MnO4 is reduced to Mn2+ MnO4 + 8H+ + 5e“ —>
Mn2t+4H30
NH3+H2O NHl+OH" Kb = 1.8 x IO"5
Chapter 13. Group 15 (VA—The Nitrogen Family) | 551

Oxidising Nature Hydrogen Azide (Hydrazoic acid) N3H


In presence of a powerful reducing agent (as Zn/HCl), Preparation
it is an oxidising agent 1. By oxidation of aqueous hydrazine with nitrous acid
N2H4 + Zn+2HC1 ---- > ZnCl2 +2NH3 N2H4+HNO2 ---- > N3H + 2H2O
?
-4 -6
? Oxidising agent can be H2O2, HN03, CIO3 etc.
2. By careful distillation of sodium azide (NaN3) with
molar mass
Equivalent mass of N2H4 = dilute H2SO4
2 NaN3+H2SO4 NaH + NaHS04
As a Rocket Fuel Hydrazine being an endothermic NaN3 is formed by
compound burns rapidly and completely in <air — with
----
N20 + 2NaNH2 ---- > NaN3 + NaOH + NH3
evolution of heat
NaN03 + 3NaNH2 ---- > NaN3 + 3Na0H + NH3
N2H4 + 02 ---- > N2 +2H2O tsH = - 620 kJ mol
air 3N2O + 4Na+NH3 ---- > NaN3+3NaOH+ 2N2
Thus, N2H| is used as a rocket fuel in guided missile, Properties
space shuttles and lunar missions. (i) Colourless liquid, boiling point 37°C. The liquid and
Structure vapour are deadly poisonous and explode on heating
H
. 0
N—N—H
2N3H ---- > H2 + 3N2
(ii) Weak acid, stronger than acetic acid
N3H + H20 H30+ + N3 Ka = 2.8 x IO’5
H H CH3COOH+H2O h3o*+ch3coo-
Ka' = 1.8 x 10’5
Each N-atom is sp3-hybridised and tetrahedrally
surrounded by one nitrogen, two hydrogen and one lone (Ka>Ka')
pair of electrons. The molecule has a gauche conformation (hi) N3H dissolves Mg forming H2
with an angle of rotation of 95° from a purely eclipsed Mg+2N3H ---- > Mg(N3)2 +H2
position.
(iv) Metals like Cu, Zn, Fe form NH3 and N2 with N3H
Uses CU + 3N3H ---- > Cu(N3)9 +n2 +nh3
• As a rocket fuel; methyl derivatives MeNHNH2 and (v) Reducing nature
Me2NNH2 are also used in combination with it. N2 + N20 + H20
(a) HNOz + N3H
Oxidants are 02 (liquid), N2O4 or H^. ? T T T
• As a reducing agent to prepare copper and silver +3 _1 0 +1
mirror; to prepare platinum metal. 3
• Isonicotinic hydrazine (C^HjNCONHNHu) is an i oxidation |
antitubercular drug. reduction
N3H may be estimated by titration with NaNQj
Similarity Between Hydrazine and Hydrogen solution using FeCl3 as an indicator NjH gives red
Peroxide colour with FeCl3.
N2H4 Hath (b) HOCI+2N3H 3N2 +HC1+H2O
1. Structures H H FT ? T T J
0
N —N- O—O
H +1 "l oxidation
2. Association Form liquid and solid through reduction
hydrogen-bonding
O-atom is Structure The azide ion has very much similarity with
3. Hybridisation N-atom is
sp3-hybridised sp3-hybridised halide ions and cyanide ion and is termed as pseudo halide.
4. Decomposition 3N2H4 ---- > N2 2H2O2 2H2O+ O2 a^24A_n^-N
+ 4NH;'3
H2O2 + 2H* + 2e" H' 120*
5. Oxidising nature N2Hj + 3H+ + 2e_
---- > 2NH4+ ---- ♦ 2H2O
H2O2 —> o2 n—n=n: <—> n=n=n:
6. Reducing nature N2Hj —> N2
+ 5H++ 4e“ + 2H+ + 2e H H
552 | Essential Inorganic Chemistry

Hydroxyl Amine (NH20H) SO2 + 2H2O SOT+4H++2e'


Preparation 2NH3+2H++SO?" (NH4)2SO4
It is stable in the form of acid salt as [NH3OH]+C1“ or (c) HI is oxidised to I2
[NH3OH]+HSO4- NH2OH + 2HI ----- > NH3+H2O+l2
1. NH2OH in the form of salt is formed when (d) Fe(OH)2 is oxidised to Fe(OH)3 in basic medium
ammonium nitrite (Nh^NOjj) reacts with
2Fe(OH)2 +NH2OH + H2O —2Fe(OH)3+NH3
ammonium hydrogen sulphite (NT^HSO^ in
presence of sulphur dioxide (SO2). (e) Na3AsO3 (sodium argenite) is oxidised to Na3AsO4
N^NOa +NH4HSO3 +SO2 +2H2O ----- > (sodium argenate) in basic medium

[NH3OH]+HSO; +(NH4)2SO4 Na3AsO3 +NH2OH ---- > Na3AsO4 +NH3

2. Methyl nitrite (CH3NO2) changes to 6. NH2OH as a reducing agent


[NH3OH]+HSO4 on reaction with H2SO4 Medium
ch3NO2+h2so4 —> [nh3ohj+hso;+co Acidic 2NH2OH N2O + H2O + 4H++ 4e’
Properties
Basic 2NH2OH + 4OH’ ---- > N2O + 5H2O +de­
1. Pure NH2OH is colourless, odourless deliquescent molar mass
crystals — melting point = 33 0 C Equivalent mass =
4
boiling point = 58° C at 72 mmHg
(a) CuO(II) is reduced to Cu2O(I)
2. Dielectric constant = 78
4CuO + 2NH2 OH ------>> Cu2OX + N20 + 3H2O
highly soluble in water and alcohol but slightly in red ppt
ether or benzene.
(b) Fe3+(III) is reduced to Fe2+(II)
3. Its aqueous solution is less basic than NH3 and
2NH2OH + 4FeCl33-----> N2O + 4FeCl3 + H20 + 4HC1
N2H4
Uses
NH2OH + H2O NH3OH+ + OH- Kb = 6.6 x IO-9
(i) As a reducing agent-antioxidant in photographic
4. In absence of H2O, it also behaves as a weak acid
developers and reduction of Cu(II) in the dyeing of
NH2OH nh2o- +H+ acrylic fibres.
NH2OH + Ca(OH)2 ----- > Ca(OH)(ONH2) (ii) In the preparation of caprolactam (as intermediate
acid base salt
in the manufacture of nylon-6)
5. NH2OH as an oxidising agent
NH2OH can be reduced to NH3 hence it can be used OH ,
nh2oh oleum - / 1
------------------►
as an oxidising agent ( NH
Medium
Caprolactam j
Acidic NH2OH + 2H+ +2e“ ----- > NH3 +H2O
Structure
Basic NH2OH + H20 + 2e~ NH3 + OH"
N-atom is sp3-hybridised with one lone pair of
molar mass
Equivalent mass = electrons. The N—O bond distance is '1.47A which
2 "
corresponds to single bond.
(a) SnCl2 is oxidised to SnCl4 in acidic medium
/H H H
SnCl2 + NH2OH + 2HC1 ----- > SnCl4 + NH3 + H2O N- H N-
(b) SO2 is oxidised to SC^" and (NH4)2SO4 is the end •0• •P'
product.
H
NH2OH + 2H++ 2e” ’----- > NH3+H2O cis- trans-
Chapter 13: Group 15 (VA—The Nitrogen Family) | 553

N2 + N20
N2 + HCI

Reducing nature

explode
n3h n2 + h2
nh3
Raschig NaOCI^/
eft
n3 + h3o+
Oxidising nature Basic H2O
NH3 + ZnCI2 nh^nh2 N2Hs + OH"
Zn/H* nature
H20
2+
N2H6 + OH"
Reducing nature
N2 + 4e“
AuCI3 AgNO3
Au Ag

w
PtCI4 IO3
Pt
<<V
CuC^ Mn.2+
FeCI2
Cu2O

CH3NO2 + H2SO4 NH4NO2 + so2 + NH4HSO3


\ NaOH
NaOH
NH2OH

2H* + 2e~
NzO + 4e" NH3 + H2O
Reducing nature Oxidising nature

AgNO3 FeCI3 CuO


F7 JhI |S02 |snCI2

Ag FeCI2 Cu2O Fe'.3+ l2 S04" SnCI4

Target Practice 1
1. Boiling points of different (a) What do you conclude about basic radical?
hydrides (MH,) of group 15 B.P. (b) Turbidity dissolves when we add
elements have been represented 3. Write full form of ATP
in the graphical form 4. When there is thunderstorm accompanying lighting and
Hydrides (MH,) are rain, some oxoacids are formed.
A B Oxoacids are
C D and chemical reactions taking place are
mh3
Bond angle (H 'H)is
maximum in.. 5. Nitrite interferes in the oxygen transport ability of blood
causing a disease known as
Basic nature is maximum in
6. NO is a free radical. What is
2. In qualitative inorganic analysis of basic radicals, solution (a) bond order?
is prepared in dil. HC1 and then diluted with H2O beforeH2S
(b) number of unpaired electrons?
gas is passed. Orange turbidity appeared in one case on
dilution. (c) bond length between N and 0?
554 | Essential Inorganic Chemistry

7. [Fe(H2O)5NO]2+ is formed by charge-transfer mechanism. 10. Consider following equilibrium


Explain it. NEL + NH- <5
NH* + NH-
Q

(a) it is called............................
What is magnetic moment of iron in it?............................... (b) based on the above equilibrium, NH3 is...........
8. N2O5 is said to be ionic in solid state. Write the cation and (c) ionic product of above equilibrium is written as ....
anion in it and hybridised state of each N atom.
11. Cl2 is passed into NH3. What are the products
Species Hybridisation (a) when Cl2 is in excess /
Cation ..?
(b) when NH3 is in excess •>
Anion •/....................................
12. What is type of following reaction?
9. In presence of cone. H2SO4, N2O5 can be used as the 3HNO2 ---- > 2N0 + HNO3+H2O
nitrating agent. Explain the formation of the intermediate.

Answers
1. A : PH3 B : AsH3 C : NH3 D: SbH3
NH3 (bond angle maximum) Thus, Fe+ : ], 1!k
NH3 (basic nature maximum)
2. (a) Sb3* Thus, in [Fe(H2O)5 NO]2+ there is positive charge on Fe as well
SbCl3 +H20 ---- > SbOCl + 2HC1 as on NO.
orange turbidity Unpaired electron in Fe+ = 3 = N
(b) HC1 Thus, magnetic moment = + 2) BM
SbOCl + 2HCl ---- > SbCl3+H2O = •715 BM
3. Adenosine tri-phosphate 8. Cation : NO2 — sp
4. HNO2,HNO3 Anion : NO3 —spz
N2+O2 ---- > 2NO
9. N2O5 + 3H2SO4---- > 2NO2 + 3HS04+H30*
2NO + O2 ---- > 2NO2
2NO2+H2O ---- > HNO2+HNO3 Nitronium ion
used in nitration of
5. Methemoglobinemia. benzene nucleus
6. (a) 2.5 (b) 1 (c) 1.15 A
10. (a) Self ionisation (autoprotolysis in which NH3 is proton
7. Fe: [Ar] 4s23d6
donor as well as proton acceptor).
Fe2+: 1 (b) Amphiprotic.
(c) Knh., = [NH4][NH2]
3d6
NO donates its electrons to Fe2 * (charge transfer) 11. (a) NC13, HC1 (NH3 + 3C12 ---- > NC13 + 3HC1)
NO ---- > NO* + e~ (b) NH4C1, N2(8NH3 + 3C12 ---- i 6NH4C1 + N2)
Fe2* + e~ ---- > Fe* 12. It is a disproportionation reaction.

Practice Exercise 1
1. Give examples of compounds of N with oxidation number -3 6. N2 can be obtained by
to +5. (a) passing NH3 over red-hot copper (II) oxide
2. Nitrogen does not form NC15 or NF5 but phosphorus can. (b) heating (NH4)2Cr2O7.
Explain. (c) passing NH3 into suspension of bleaching powder
3. P4 exists but N4 does not; N2 exists butP2 does not. Explain. Write a balanced equation for each preparation.
4. What happens when PH3 reacts with 7. Some farmers feel that lightning helps produce a better crop.
(a) Aqueous Ag+ (b) Aqueous Cu2+
2* (c) O2 What is the scientific basis for this belief?
5. There are two colourless solutions (A) and (B) both give 8. NO often acts as a three electron donor in contrast to most
white ppt with AgNO3; ppt dissolves in aq NH3. Also (A) on ligands which donate two electrons. What happens when
reaction with H2O gives orange turbidity while (B) gives NO replaces three CO in [Fe(CO)5] and six CO in [Cr(CO)-]?
white turbidity. Identify (A) and (B). Suggest the reagent 9. Mixture of KN03 and KNO2 can oxidise Cr2O3 to CrO*'.
that can dissolve turbidity.
Write chemical equation to represent change.
Chapter 13: Group 15 (VA—The Nitrogen Family) | 555

10. Complete the following: 13. NH3+O2 (A)


(a) MnO; + NO + H* -----> (b) SO2 + H* + NO - (A) + O2 -—» (B) (brown fumes)
A
(c) Cr2+ + H* + NO ---- >
H*+NO (d) NaNO3 + NH4C1 (B) + H2O --- > (C) + (D) (both oxy acid)
11. NH3 gas is dried by CaO and not by P2O5 an^ H2SO4.
(a + r -—> (E) (violet vapours)
Identify (A), (B), (C), (D) and (E).
Explain.
12. Identify (A), (B), (C) and (D) in the following:
Al + N2 (A) Hz H2O
-> (B) -^4 (P) (brown ppt).

Answers
6. (a) 3CuO + 2NH3 - 3Cu + N2 + 3H2O
1. NH3 : -3; N2H4 : -2; NH2OH : -1
NO (b) (NH4)2Cr2O7 — N2 + 4H2O + Cr2O3
N2 : 0(zero); H2N2O 2 ; +1; NO : +2 (c) 3CaOCl2 +2NH3 —> 3CaCl2 + 3H2O + N2
7. _. o------ „ forms NO which is converted to ”
Lightning NOO2, HNO2 and
N2O3 : +3; NO2 : +4; N2O5 : +5
HN03 under atomospheric conditions. Thus, NO3 is a source of
hno2 N 2O 4 hno3 nutrients (fertilisers) to the plant. Thus, there are chances of
2. Nitrogen (Is2 2s2 2p3) lacks 2d-orbitals in which electrons of2s better crops when lightning occurs.
can be excited to make five orbitals unpaired. Only three N2+O2 ll8l,unin8> 2NO
electrons remain unpaired hence only NX3 is formed. In case of
2NO + O2 ---- > 2NO2
P(1 s 2 2s 2 2p 6 3 s 2 3 p 3), 3 s electron can be exci ted into 3 d va can t
2NO2 + H2O---- > HN02 + HN03
orbitals thus making five unpaired electrons, thus PX5 is
possible. 8. Fe(CO)5+2NO---- > [Fe(CO)2(NO)2] + 3CO
------ * . t

CO donates two electrons while NO three electrons, hence


3. P^>P electrons lost (due to CO) = six
(white phosphorus : P4) electrons gained (due to NO) = six
\p/ Similarly, Cr(CO)6 + 4NO [Cr<NO)4l + 6CO
A 2K2CrO4 + 4N0
9. 3KNO2 + KN03 + Cr2O3
N=N 946 kJ mol-1, P=P 481 kJ mol
(butKNO2 andKNO3 are in molten states)
N-N 160 kJ mol-1, P—P 215 kJ mol"1 —*---- > 5NO3 + 3Mn2* + 2H2O
10. (a) 3MnO4 + 5NO- + 4H*
Greater the B.E., greater the stability [compare BE of N = N SO 2 + 2NO + H2O---- > N2O + SOt + 2H+
(b)
and P=P and N—N (in N4) and P—P (in P4)]
(0 3Cr2* + NO + 3H* ---- > NH20H + 3Cr3*
4. (a) PH3 + 6Ag+ + 3H2O---- > H3PO3 + 6Ag + 6H+
(b) PH3 + 4Cu2+ + 4H2O---- > H3PO4 + 6Cu + 8H+ (d) NaN03 +NH4C1---- > NH4NO3 +NaCl
(c) 4PH3 + 8O2 ---- > P4O10 + 6H2O N2O + 2H,O
5. (A):SbCl3 (B):BiCl3 11. Moist NH3 can react with P2O5 forming phosphate and with
SbCl3 orBiCl3 + AgNO3 AgCl X (due to Cl’) cone. H2SO4 forming sulphates :
white ppt
6NH3 + P2O5 + 3H2O 2(NH4)3PO4
AgCl + 2NH3 (aq) [Ag(NH3)2]Cl (NH4)/2ov
2NH3+H2SO4 ---- > u'in SO 4
soluble
hence can’t be used for drying NH 3. CaO being basic would only
SbCl3 + H2O---- > SbOCl I + 2HC1 cause drying of NH 3.
orange
12. (A): AIN (B): NH3 (C): K2HgI4OH- (Nessler’s reagent)
SbOCl + 2HC!---- » SbCl3 + H2O (D): H2N—Hg—0—Hg—I (Iodide of Millon's base)
soluble

BiCl3 +H2O---- > BiOCll +2HC1 or 0 ’NH.,I


white
Hg
BiOCl + 2HC1---- > BiCl3 + H2O
soluble 13. (A): NO (B):NO2 (C):HNO2 (D):HN03 (E):I2
Turbidity can be dissolved in dil. HC1.
556 | Essential Inorganic Chemistry

Oxyacids of Nitrogen 3HNO2 --- > 2NO + HN03 + H2O


+3 +2 +5
Acid Formula Ox. no. Anhydride | redn. t
Hyponitrous acid H2N2O2 +1 N2O* oxidn.
Nitrous acid hno2 +3 n2o3 • As an oxidising agent:
Nitric acid hno3 +5 n205 HN02 + H+ +e~ - -> NO + H2O .(i)
* A - it oxidises I- to I2 /
h2n2o2 h2o + no
21" - -> I2 + 2e~
N2O + H2O ---- > (no acid formation)
H2N2O2 gives N2O (anhydride) only on heating butN2O
- it oxidises H2S to S I
when dissolved in H2O will not form H2N2O2. H2S- > 2H+ + S + 2e‘
- it oxidises SO2 to SO|-
Hyponitrous Acid (H2N2O2) SO2 + 2H2O---- > SOt + 4H+ + 4e“
• It is obtained starting from NaNO2 : - it oxidises Fe2+ to Fe3+
Na—Hg ' XT_ XT
NaN02 ———~NaoNoOo
AgNO3 A _XT
* Na2N2O2 ----------- AgN2O2 Fe2+ —> Fe3+ 3+ +e~
reduction 2 2 2 6 2 2 Note : Oxidation half-reaction may be added to reduction half
HC1 v u nt n reaction (I) to cancel electron in final reaction.
—-—HoNoOo
-AgCl 2 2 2 • As a reducing agent:
• Colourless leaflets hygroscopic crystals. HN02 + H2O---- > NO3 + 3H+ + 2e" ...(H)
• In aqueous solution it decomposes : - It reduces acidified MnO4 (purple) to Mn2’
H2N2O2 ---- > H2O + N2O (colourless)
• Very weak dibasic acid but a strong reducing agent: MnO4 + 8H+ + 5e~ ---- > Mn2+ + 4H2O
- I2 is reduced to HI Note : Mn2+ salt are pink-coloured but appears colourless in
aqueous solution.
H2N2O2 + 3I2 + 3H2O---- > HN03 + HNO2 + 6HI - it reduces acidified Cr2C>7“ (orange) to Cr3+ (green)
- MnO4 is reduced toMn2+ Cr2O7- + 14H+ + 6e~ ---- > 2Cr3+ + 7H2O
8MnO4 + 5H2N2O2 + 24H+ 10HNO3 + 8Mn2+ - it reduces H2O2 to H2O
+ 12H2O H2O2 + 2H+ + 2e" ---- > 2H2O
• It has zero dipole moment which is based on its trans • Aliphatic amines and amides on reaction with HNQ
structure: give N2 gas
.OH
7?NH2 + HN02 - flOH + Na +H20
-N=N 2N2 + C02 + 3H2O
NH2CONH2 + 2HNO2
HO urea
This provides the basis of Duma’s method of
estimation of nitrogen quantitatively in organic
Nitrous Acid (HNO2) compounds.
• Nitrous acid is unstable except in dilute solution. It is
• However, aromatic amines form diazonium salts at
easily made by acidifying a solution of a nitrite :
low temperature (diazotisation)
Ba(NO2)2 +H2SO4 ---- > BaSO4 + 2HNO2
NaN02 + HC1---- > NaCl + HNO2
It appears as a pale blue solution (due to N2O3) when
<S ■NH2HC1 + HNO2
o°c QVN2C1 + 2H2O

nitrite is dissolved in acid. To prevent its • It behaves as a monobasic acid and has structure
decomposition, it must be kept below 5° C.
H—0—N=O
• N2O3 + H2O---- > 2HNO2
• NH3 + 3H2O2 ---- > HNO2 + 4H2O Microwave spectroscopic studies have shown that
nitrous acid in the gaseous state has trans
• It is a weak acid (Ka = 4.5 x 10'5) and configuration :
HNO2 + NaOH---- > NaNO2 + H2O
Za = 110.7°
• It disproportionates to HNO3 (oxidation) and NO 4.18 A
MX Zp = 102.1°
(reduction) even in cold solution X)
Chapter 13: Group 15 (VA—The Nitrogen Family) | 557

NO^ has planar triangular structure, with N at the — 2N0 + Q2 2NO2


centre, two comers occupied by 0 atoms, and the third 2NO2 + H20---- > HN03 + HN02------
comer occupied by the lone pair. 2HNO2 ---- > H2O + NO2 +N0+

0 3NO2 + H20---- > 2HNO3 + NO

-0 zNV0 Overall reaction being


NH3 + 2O2 ---- > HN03 + H2O
Nitrite ion can exist in the following resonance Acid is 60% by weight and concentrated HN03 (98% by
structures: weight) is obtained by dehydration with cone. H2SO4.
& Fuming HN03 contains dissolved N02 in concentrated
zN<
:0: 6 :0 0:
nitric acid.
• Anhydrous HN03 is colourless fuming and pungent
smelling liquid, soluble in water; boiling point = 84.1° C
NaNO2 nh3 and freezing point = 41.55° C. On account of
Ba(NO2)2 photochemical decomposition (in sunlight), it is often
O'h .N2O3
x\h2so4 HCI of yellow colour.
HzO
NH2CONH2|— 2HNO3 ■AX, 2N0, +H2O+1O2
C02 + N2 hno2 ROH + N2
rnh2
• Its auto-ionisation givesNO^ (nitronium ion) andNO3
// 2HNO3 ---- > N02 + NO3 + H20
H +e~ + h2o % • It is a strong monobasic acid and forms nitrates with
I
NO NO3 + 2e“
X
\ $
carbonates, oxides and hydroxides.

VA
/
v- • HN03 is a powerful oxidising agent:
Fe2+ Mn>2+

/O Ih2o2\^ 2NO3 + 4H+ + 2e" ■> 2NO2 + 2H2O
/ OT
concentrated
S0^“ Fe3+ h2o Cr3* N03 + 4H+ + 3e“ ■> NO + 2H2O
dilute
N03 + 10H+ +8e" ---- > NHJ +3H2O
Nitric Acid (HNO3) very dilute
• KNO3 +h2so4 A
khso4 + HN03 Thus oxidising power of HN03 depends on the extent
cone of dilution.
On condensation, vapours of HN03 changes to liquid N03 ---- ■>> N0
N022 ---- > NO ---- > NH3
+5 +4 +2 ”3
state (a laboratory method).
• Non-metals are oxidised to their oxy acids by
• Birkeland-Eyde process involves electric sparking conc.HNO3.
of N2 and 02 and passing of the gas into H20. C---- > H2CO3 (carbonic acid)
spark S----» H2SO4 (sulphuric acid)
N2 + 02 =T■<—2N0, AH = 181 kcal mol
P---- > H3PO4 (o-phosphoric acid)
2N0 + 02 ---- > 2NO2 I2 ---- > HIO3 (iodic acid)
2NO2 + H20---- > HN02 + HNO3 One reaction is given to explain, others may be
3HNO2 ---- > HN03 + H20 + 2N0 written similarly.
• Ostwald process depends on the catalytic oxidation 2NO3 + 4H+ + 2e~ ---- > 2NO2 + 2H2O]x 3
ofNH3 to NO, followed by oxidation of NO to NO^ and S + 4H2O ---- > SOj“ + 8H+ + 6e~
conversion of N02 with H20 to HN03. S + 6NO3 + 4H+ ---- > 6NO2 + SO^’ + 2H2O
Pt/Rh,5 atm
4NH3 + 50a ■> 4NO + 6H2O
850°C Metals are oxidised forming nitrates and NO?, NO,
N2O,N2 orNH3 depending on the nature of the metal, the
The NO and air are cooled and the mixture of gases is
temperature and the concentration of the acid. Reactions
absorbed in a counter current of water.
of metals with HN03 are summarised in Table 13.3.
558 | Essential Inorganic Chemistry

DEEP Focus HNO3 + 3HC1 2H2O + 2C1 + N0C1


• Noble metals like Au, Pt, Rh and Ir are not attacked by aqua-regia
nitric acid. Aqua regia, which is a mixture of 75% cone HC1
and 25% cone. HNO3 can dissolve Au and Pt. The enhanced Au + 3C1----- > AuC13
ability to dissolve metals shown by aqua-regia arises from AuC13 + HC1-----> HAuC14
the oxidising power ofHNO3 coupled with the ability of Cl- to chloro auric acid
form complexes with the metal ions.
Pt + 4C1-----> PtCl4
PtCl4 + 2HC1-----> H2PtClg
chloro platinic acid
Table 13.3 Reactions of Elements (Metals/Metalloids with HN03)
Element Nature of HNO3 HNO3 changes to Reactions

(A) Metals placed above H


in electrochemical series (ECS)
1. Mg, Mn dilute M(NO3)2 M + 2HNO3 — M(NO3)2 + h2

2. Zn, Fe (a) very dilute nh4no3 4Zn + 10HNO3 4Zn(NO3)2 + NH4NO3 + 3H£

(b) dilute n2o 4Zn + 10HNO3 -» 4Zn(NO3)2 +N20 + 5H20


(c) cone no2 Zn + 4HNO3 ----- > Zn(NO3)2 + 2NO2 + 2H2O
Fe becomes passive with 80% HNO3
3. Sn (a) dilute nh4no3 Refer 2(a).
(b) cone no2 Sn + 4HNO'33 ----- > H2SnO
HjSnOg3 + 4NO2 + H2O
meta stannic acid
4. Pb (a) dilute NO 3Pb + 8HNO3 3Pb(NO3)2 + 2NO + 4H2O
(b) cone no2 Refer 2(c).
(B) Metals below H in ECS

5. Cu, Ag (a) dilute NO Refer 4(a), Hg forms Hg2(NO3)2

Hg (b) cone no2 Refer 4(c).


(C) Metalloids

Sb, As cone NO2


Sb + 5HNO3 H3SbO4 + 5NO2 + H20
antimonic acid

• HNO3 can also be used to oxidise compounds : NH3 + O2 KNO3 + H2SO4 (cone)
2HNO3 ---- > ^>0 + 2NO2 + 0 \Ostwald NOj

6
Birkeland-Eyde
H2S----- > S + H2SO4
H2O
SO2 --- > soj- n2 + o2->no-* no2 hno3 no2*
cone
Fe2+ -----> Fe3+
HzCOs^C Fe2*
FeS------ > FeSO4 S hv
SO2 Fe3*
H2SO4*|5 H2S
C^^Oh -----» (COOH)2
P X no2 + 02 NO2+NO3
cane sugar oxalic acid s
HIO3 I SO4I?"
2 FeS
• Structure can be represented as:
H3PO4
PeSO4
H- 1.41 A
•o:

Chapter 13.Group 15 (VA—The Nitrogen Family) | 559

Phosphorus exists in three common allotropic forms


Aqua Regia
HNO3 + HCI (i) white phosphorus
cone cone (ii) red phosphorus
1 : 3 (iii) black phosphorus
(i) White phosphorus In earlier days, white
phosphorus was obtained by the distillation of
2CI + NOCI
Au Pt metaphosphoric acid (HPO3) with carbon at high
AuCI3^ HgS ^PtCI4 temperature. HPO3, in turn was obtained by decomposing
bone-ash [Ca3(PO4)2 ] with 60%
HgCI2 H2PtCI6
HAuCI4 NiCI2 Ca3(PO4)2+H2SO4 -----> CaSO4 X + H3PO4
Reduction of * HP03+H,0
cone
h3po4
NO- HNO3 - NO2
and N2O
dilute 4HPO3 + IOC-----> P4 + 10CO + 2H,0
| very dilute

nh3—«-nh; 60% H2SO4 H3PO4


Ca3(PO4)2 Filter

NaN02 and NaNO3 as Food Additives CaSO4


The brown colour of “old” meat is the result of Evaporate H2O
oxidation of blood and is objectionable to many consumers.
Nitrites and nitrates are added to food to retard this
CO
oxidation and also to prevent growth of botulism bacteria.
NOj ions are reduced to NO2 ions, which are then
converted to NO. This in turn reacts with the brown
L P4
C, A
HPO3

oxidised form of the heme in blood. This reaction keeps In modem electrochemical process, white phosphorus
meat red longer. However, there are doubts that nitrites is manufactured by heating a mixture ofCa3(PO4)2, sand
may combine with amines under the acidic conditions in and coke at 1500°C in a furnace by means of an electric arc
the stomach to give carcinogenic nitrosoamines. (Fig. 13.3). The reactions are:
HN02 +^NH /ySf— N^O + HjjO 2Ca3(PO4)2 +6SiO2 6CaSiO3 + P4O10
nitrosoamine P4O10 + IOC P4 +10COT

Study of Phosphorus and its i------------ ------ —----- 7-]


Ca3(PO4)2 + SiO2 + C
In electric arc
1500’C
P4 + CO

Compounds CaSiO3 CO

Isolation of Phosphorus 250°C, A


P4 in H2O H2O
Red phosphorus
Phosphorus is the most abundant element of air absent
group 15(VA), accounting for 0.1% of the mass of the
earth’s crust. Phosphorus is also important in living
systems and is the sixth most abundant element in the
human body. Our bones are mostly Ca3(PO4)2, and tooth
enamel is almost pure Ca5(PO4)3OH. Phosphate groups
are an integral part of the nucleic acid DNA and RNA, the P4 + CO
molecules that pass genetic information from generation
to generation.
Ca3(PO4)2
+ sand + coke
Minerals
Phosphorite Ca3(PO4)2
Fluorapatite 3Ca3(PO4)2 CaF2 Mlg;:

PFC0
: "• - ^-P
|
orCa5(PO4)3F — 4
|ll l
Chlorapatite 3Ca3(PO4)2 CaCl2 CaSiO3
Aluminium phosphate A1PO4 Fig. 13.3 Manufacture of phosphorus by electrochemical
Ferric phosphate FePO4 process
560 | Essential Inorganic Chemistry

The calcium silicate forms a slag and is run off Table 13.4 Comparison of White, Red and Black Phosphorus
through an outlet at the bottom of the furnace. Black
White Red
Phosphorus vaporus issuing through an opening at the top Properties phosphorus
phosphorus phosphorus
of the furnace is condensed under water. Black
1. Colour White Reddish-violet
(ii) Red phosphorus Red phosphorus is prepared 2. Smell Garlic-like Odourless Odourless
by heating white phosphorus in the absence of air at 250°C 3. Specific gravity 1.83-1.85 2.06-2.39 = 3.9
for a few hours with a trace of iodine at catalyst in a closed Insoluble
4. Solubility in Soluble Insoluble
iron vessel fitted with a safety valve. At the end of CS2
conversion, the product is ground under water and treated 5. m.p./°C 44 -600 -610
with hot caustic soda solution to eliminate any unchanged 6. Ignition 30 260 - Stable
white phosphorus. It is finally washed with hot water and temperature/°C
dried by heating with steam. 7. Action of air Phosphoresc- No No
(iii) Black phosphorus It is obtained by heating ence phosphoresc-e
nee
white phosphorus at 470 K under high pressure. A series
8. With Cl2 Reacts at room Reacts only Atm.p
of phases of black phosphorus are formed and one of these temperature when heated
phases consists of an extended layer structure in which No reaction
9. With NaOH PH3 is formed No reaction
each phosphorus is bonded to three neighbours by single
10. Physiological Poisonous Non-poisonous Non-poisonous
covalent bonds. Thermodynamically, black phosphorus is effect
the most stable form.
White phosphorus, the form produced in the Illustration 1 Explain the high reactivity of white
industrial synthesis, is a toxic, waxy, white solid that phosphorus.
contains discrete tetrahedral P4 molecules. Red
phosphorus, by contrast, is essentially nontoxic and has a Solution The high reactivity of white phosphorus is
polymeric structure: due to an unusual bonding that produces considerable
R R strain in the P4 molecules. If each P atom uses 3p orbitals
to form three P—P bonds, all the bond angles should be
—R '—R P— R
90°. The geometry of P4, however, requires that all the
•P' ^P P bonds have 60° angles, which means that the p-orbitals
White phosphorus Red phosphorus cannot overlap in a head-on-fashion. As a result, the P—P
bonds are “bent”, relatively weak, and highly reactive.
I
R R Note : White phosphorus was once used in matches, but
—P P P— because of its toxicity it has been replaced by}\S3.
I I
P R P—
—p p- p Compounds of Phosphorus
—p. R .p.
p DEEP Focus
Yp P—
I
P—
• Phosphorus and the other members of the group can make
p use of d-orbitals in their bonding, and thus +5 oxidation
state is shown beside +3 which is common in all the
compounds. Thus,PCI. is formed while NC15 is not formed.
Black phosphorus

As expected for a molecular solid that contains small, ng n 11111111111


3s2 3p2 „ 3d
Three electrons
unpaired in 3p,
hence PCI3 is
non-polar molecules, white phosphorus has a low melting Excite formed
point (44°C) and is soluble in non-polar solvents such as
CS2. It is highly reactive, bursting into flames when
exposed to air, and is thus stored under water. fflWIIH Illi]
3s1 3jd3 3rfi
Five electrons unpaired
in outershell thus, PCs
is formed
Consistent with its polymeric structure, red Three electrons unpaired in 2p,
phosphorus is higher melting (m.p. ~ 600°C), less soluble, N’ Me] [t] |1|1|1 , hence NCI3 is formed, since 2d
less reactive than white phosphorus, and it does not ignite does not exist, hence electrons
2s2 2p3 of 2s can't be made unpaired,
on contact with air. Comparative properties of white red thus, NC15 is not formed
and black phosphorus have been summarised, in
Table 13.4.
Chapter 13: Group 15 (VA—The Nitrogen Family) | 561

Phosphine (PH3) The oxides of phosphorus react vigorously with HjO to


give acidic solution :
• Phosphine can be formed by :
PtO6 + 6H^O > 4H3PO3
Ca3P2 +6H2O---- > 2PH3 + 3Ca(OH)2 o-phosphorous acid
calcium phosphide
?1^10 + 6H2O * 4H3PO4
P4 + 3NaOH + 3H2O PH3 + 3NaHzPO2 o-phosphoric acid
It is necessary to exclude air from the apparatus This takes place in stages :
because, although pure PH3 will not burn unless ignited, it P4O10 + 2H2O---- > 4HPO3
is often accompanied with P2H4 and Hz which are 4HPO3 + 2H9O---- > 2H4P2O7
spontaneously flammable.
2H4P2O7 + 2H2O---- > 4H3PO4
2P + 2NaOH + 2H2O 4 2NaH2PO2 + Hz, ?
6P + NaOH + 4H2O » 4NaH2PO2 + P2H4 DEEP Focus
• P4O10 is a white flocculent powder that has greater affinity
• 2A1P+3H2SO4 * 2PH3 +A12(SO4)3 for water hence, is used as dehydrating agent in:
dil.
4HNO3 + P4O10 ---- > 4HPO3+2N2O5
• It is a colourless, highly reactive and extremely toxic
2CH3CONH2 + P4O10 ---- > 2CH3CN + 4HPO3
gas, which smells slightly of garlic or bad fish.
4CH3COOH + P4O10 ---- > 2(CH3CO)2O + 4HPO3
• It is better reducing agent than NH3
2HzSO4 + P^Oio -----> 2SO3+4HPO3
“ Ag+ is reduced to Ag metal 6CaO + P4O10 ---- > 2Ca3(PO4)2
6Ag+ + PH3 + 3H2O---- > H3PO3 + 6AgX + 6H+ IOC + P4O10 ---- > 4P+10COT
~ Cu2+ is reduced to Cu • P4O10 reacts with alcohols and ethers forming
4Cu2+ + PH3 + 4H2O * HgPO4 +4Cui +8H+ phosphate esters:
OH OEt
• PH3, unlike NH3, is not very soluble in water. Its I
aqueous solution is neutral. P4O10 + 6EtOH 2O=P—OH + 2O=P—OH
• It is more soluble in CS2 and other organic solvents.
OEt OEt
• Anhydrous PH3 and anhydrous HCI (not in aqueous OEt
solution) combine to form PH4C1" I
• Pure PH3 is stable in air, but it catches fire when P4O10 + GEtjO 2O=P—OEt
heated to about 150 C.
OEt
PH3 + 2O2 ---- > H3PO4
(Formation of esters confirms that H3PO4 has
• In liquid NH3, phosphine dissolves to give NH4PH^ OH
NH3 +PH3 — nh;ph2
base acid structure 0=P—OH)
Ca3P2
AIP P4 + NaOH
OH
H2O h2o • The basic building block of both of them is a
tetrahedron of P-atoms. In P4O6; oxygen atoms act as a
nh3 Ag1
nh4ph2 PH3 Ag + H3PO3 bridge between the comers of the tetrahedron. In
Cu2+ P4O10, each P-atom is also bonded to an extra oxygen
HCI
atom. Each oxide (P4O6,P4O10) has six POP bonds.
h3po4 ph4cp Cu + H3PO4

O
Oxides of Phosphorus b-.. N
Two important oxides of phosphorus are known :
0
Oxides Oxoacids Oxidation number

h3po3 +3
P4O10 h3po4 +5
o
(P4O6) (P4O10)
562 | Essential Inorganic Chemistry

h2o
P4O6 H3PO3 DEEP Focus
N2O5 + HPO3 • In all of these acids, P is four-coordinate and tetrahedrally
x\hno3 02
surrounded wherever possible pn - pn back bonding gives
rise to P=O bonds. The hydrogen atoms in 0—H
H2SO4 h2o groups are ionisable and are acidic, but P—H bonds
so3 + HPO3 P4O10 h3po4
found in phosphorous acids have reducing and not
£aO the acidic properties.
C • Also basicity = number of ionisable H (protonic) = H
Ca3(PO4)2 attached to 0-atoms
P + CO (H encircled are acidic and H squared indicates reducing
CH3CN + HPO3
properties)

Oxoacids of Phosphorus
• Following oxoacids of phosphorus are commonly
known :
tribasic
O-<: ■:H—O I.
V'O-h;
■Hi
dibasic
Table 13.5 o
Acid
h^po2
Basicity
1
Name
Phosphinic acid
(hypophosphorous acid)
Oxidation no.
+1 ;H- i
•H;
monobasic
0-4®

H3PO3 2 Phosphonic acid +3


(o-phosphorous acid)
• Thus:
h3po4 3 Phosphoric acid +5
(o-phosphoric acid) - all P atoms in the oxoacids and oxoanions
HPO3 1 Metaphosphoric acid +5 (conjugate base) are 4-coordinate and contain at
H4P2O7 4 Pyrophosphoric acid +5 least one P—0 unit,
H5P3O10 5 Triphosphoric acid +5
- all P atoms in the oxoacids have at least one
P—OH group. In every case, the H atom is
Wet Process
ionisable as H+.
• H3PO4 is obtained when phosphate rock reacts with
H2SO4. - some oxoacids have one or more P—H bonds. This
H atom is not ionisable as H+.
Ca3(PO4)2 + 3H2SO4 ----- » 3CaSO4 + 2H3PO4
[3Ca3(PO4)2CaF2] + 10H2SO4----- > 6H3PO4 - Polymerisation can occur by P—0—P bond
formation to give both linear and cyclic species.
+ 10CaSO4 + 2HF
Two P atoms are never joined by more than one
Furnace Process P—O—P bridge.
• P4 + 5O2 ----- > P4O10
- When P atom is surrounded by 0 atoms (as in
P4O10 + 6H2O----- > 4H3PO4 H3PO4) its oxidation number is +5. For each
• H3PO2 andH3PO3 disproportionates toPH3 andH3PO4 P—OH that is replaced by P—H bond, its oxidation
on heating: number drops by 2.
3H3PO2 ----- > 2H3PO3 + PH3 Illustration 2 Write balanced equations for the
4H3PO3 ----- > 3H3PO4 + PH3 reactions o/‘H3PO4 and B(0H)3 with water. Classify each
• P4 + 20HNO3 ----- » 4H3PO4 + 20NO2 + 4^0 acid as Bronsted-Lowry acid or a Lewis acid.

• H3PO3 is formed by hydrolysis of PC13: Solution


PC13 + 31^0----- > H3PO3 + 3HC1 HaPO^aq) + HjOli) HjPO^ag) + H3O+(aq)
• H3PO2 is obtained by alkaline hydrolysis ofP4: Since, H3PO4 is proton-donor hence, HjPClf is a
P4 + 30H" + SHjO----- > PH3 + 3H2PO2 Bronsted-Lowry acid and H>0 which is a
proton-acceptor is a Bronsted-Lowry base.
• Table 13.6 gives distinction between H3PO4,H3PO3
and H3PO2. B(0H)3(aq) + 2H2O(Z) B(0H)4“ + H30*
• Phosphoric acid, H3PO4, is hydrogen bonded in B(0H)3 has electron-deficient B (octet incomplete)
aqueous solution, and because of this the hence, it accepts electron-pair from HjO. Thus, B(OH)S
‘concentrated acid’ is syrupy and viscous. is a Lewis acid.
Chapter 13: Group 15 (VA—The Nitrogen Family) | 563

Table 13.6 Distinctive Properties of Oxoacids of Phosphorus


Properties HsPO2 HsPOa H3PO4

1 Basicity Monobasic Dibasic Tribasic


H3PO2 H+ + H2PO2 hence only one H3PO3 FT + H2PO3 3PO4 4 :
HH3PO H* + H2PO;
type of salt (like NaH2PO2) is formed. H2PO3 H4 + HPO|“ Two
Two types
types of
of HH2PO
2PO4 4 = H+ + HPO2'
NaH2PO2 is normal salt (which does not
further react with base or which is without salts, NaH2PO3 and Na2HPO3 are formed. HPO4 H + PO4 Three series of
protonic H+). NaH2PO3 is an acid salt butNa2HPO3 is sa|ts NaH2PO4, Na2HPO4 and Na3PO4
normal salt. are formed. NaH2PO4 and Na2HPO4 are
acid salts butNa3PO4 is normal salt.
2. Effect of A A
2H3PO2 —H3PO4 + PH3 4H3PO3 3H3PO4 + PH3 h3po4 600° d HPO3 + H2O
heat 2OO°C
disproportionation disproportionation dehydration
3. Reducing Most reducing agent Does not reduce salts into metals.
properties AgNO3 ----- > Ag AgNO3 Ag
HgCI2 ----- > Hg HgCI2 Hg2CI2
CuSO4 —> Cu2H2 CuSO4 —> Cu
Cl2 ----- > HCI MnO4 ----- > Mn2*

Phosphorus Halides • 3B0H + PCI, ---- >> 3BC1 + H,PO3


Two series of halides are known PX3 and PX5, where • BOH + PC15---- > BC1 + POC13 + HCI
X = halogen. Among these PI5 is doubtful, whereas iodine • SO2(OH)2 + PC15 ---- > SO2C12 + 2POC13 + 2HC1
forms a stable di-iodide P^. In all the oxo-acids and hydroxy compounds, —OH
P4 +6X2 ---- > 4PX3 (in the limited supply of X2) group is replaced by CL
P4 + 10X2 ---- > 4PX5 (in excess amount of X2) HNO3(HO-NO2) + PC15 ---- > NO2C1 + POCI3 + HCI
All these halides are gases, volatile liquids or • In carbonyl compounds, 0 is replaced by two Cl atoms.
low-melting solids. BCHO+ PC15 ---- > BCHC12 +POCI3
• PC13 is a colourless Equid (sp. grav. = 1.6, b.p. = 76° C). BCOB+ PC15 --- > BCCl2B+ POCI3
PC15 is an off-white solid m.p. = 167°C). POCI3 is the chloroanhydride of phosphoric
It fumes strongly in moist air and reacts strongly with acid.
HjO giving phosphorous acid. • In the liquid and vapour states, PC15 has the trigonal
PCI3 + Cl2 ---- > PC15 bipyramidal structure, P atom being sp3d-hybridised.
PC13 + H2O---- > H3PO3 + SHjO
t SO3 + PC13 ---- > POCI3 + SO2
SO3 + PC15 ---- > SO2C12 + POCI3 [Ne] |-||,
11111 3d
3p3
S2C12 +3PC13 ---- > 2PSCI3 +PC15 3s2
(phosphorus)
PC13 + 3NH3 ---- > P(NH2)3 + 3HC1
PCI3 + Bifc ---- > PClgB^ 1
PC13 + 3HI---- > PI3 + 3HC1 Excite
11111 3d1
[Ne] 1 3p3
PC13 + S---- > PSCI3 sp3d
3?
2PCla +0,
• '-'2 ---- » 2POCI3
PC13 + 3AgNC0 > 3AgCl + P(NCO)3 11111111111
I______________________________ I
3BCOOH + PC13 » 3BCOCI + H3PO3 sp3d hybrid orbitals forming
* BCOOH + PCL□ > BCOC1 + POCI3 + HCI <T-bonds with Cl atoms
564 | Essentia/ Inorganic Chemistry

Phosphorus Cl2 (limited)


(red or white) pci3 ROCI
C02
h2o/ SO3
Cl2 RCOOH
POCI3 + SO2
H3PO3
O Cl
H2O ROH
H3PO4 PCI5
Cl2 (excess)
in solid
H2SO4 state
2HCI + SO2CI2 + POCI3

PCtf+PCIi
rci

Table 13.7 Comparison between Nitrogen and Phosphorus


Properties Nitrogen Phosphorus
1. Physical state N2 is a gas which can be condensed to a liquid at very low P4 is a solid at room temperature,
temperature.
2. Bond energy N= N (multiple bonds) are present in N2 Multiple bonds are absent in P
N=N B.E. . 941.4 kJ mol"1 P—P B.E.: 79.08 kJ mol"1
Due to high value of bond energy (N= N), Due to low value of (P—P) bond energy P4 is less stable and
it is less reactive and more stable more reactive
N2+O2 33WC* 2N0 P4 + 50 2 ----- * P4O1Q

This reaction takes place at room temperature.


3. Formation of NCI3 is formed but NCI5 is not formed since N cannot Both PCI3 and PCI5 are formed due to expansion of octet,
compounds expand its octet due to lack of d-orbital.
4. Hydride NH3 is more stable than PH3. Also PH4 is less stable than NH4.
NH3 is highly soluble in water due to H-bonding with water, PH3 is sparingly soluble in water and the solution is
giving alkaline solution. neutral-the hydrogen bond in latter is very weak.
5. Oxyacids Oxyacids of nitrogen are stronger than those of H3PO4 is weaker acid than HNO3.
phosphorus.
HN03 is a strong oxidising agent. H3PO4 is not oxidising agent.

Fertilizers • Phosphate rocks such as fluoroapatite


• Nitrogen, phosphorus and potassium are three most (Ca3(PO4)2 • CaF2) are very insoluble, and thus are not
essential plant nutrients consumed by them. Fertility used as fertilisers. Superphosphate is made by
of soil is maintained by using these nutrients in the treating phosphate rock with cone. H2SO4. The acid
form of fertilizers. salt Ca(l4PO4)2 is more soluble, and over a period of
• Fertilizer used should : weeks this would dissolve in the soil water. CaSO4 is
be water soluble. waste product of this fertiliser :
not disturb alkaline nature (pH). 3[Ca3(PO4)2 CaF2] + *
be stable. 3Ca(H2PO4 )2 + 7CaSO4 + 2HF
V

• NHoCONHj (urea) superphosphate


NH4NO3 (ammonium nitrate) • *Triple superphosphate’ is made using H^PC^ and
(NH4)2SO4 (ammonium sulphate)
thus waste product CaSO4 is not formed.
NH4H2PO4 (mono ammonium hydrogen phosphate)
These are either used on their own or in conjugation [Ca3(PO4)2-CaF2] + 6H3PO4 t 4Ca(H2PO1)2
with other chemicals, often to make NPK fertilisers. triple superphosphate
These are designed to release nitrogen (N), phosphorus (P) + 2HF
and potassium (K) into the soil. If the NPK fertiliser is
marked 4-8-2, it means it has : Uses
4% N2 A major use of phosphoric acid is to impart corrosion
8% P2O5 resistance to metal objects such as nuts, bolts, tools and
2% KjO car engine parts by plunging the object into hot acid bath.
Car bodies are similarly treated with phosphoric acid
Chapter 13: Group 15 (VA—The Nitrogen Family) | 565

Disodium hydrogen phosphate, Na2HPO4, is widely used


containing metal ions such as Zn2+, and aluminium trim is
as an emulsifier in the manufacture of pasteurised cheese.
'polished” by treating it with acid.
Sodium dihydrogen phosphate, Naf^PC^, is used as a
Red phosphorus is used in matches, fire works and as constituent in the undercoat for metal paints. Calcium
a component of phosphor-bronze alloys used for making phosphates are used in baking powders and tooth-pastes.
bearings. The main use of elemental arsenic, antimony Synthetic Ca6(PO4)3OH (1-2%) is added to table salt
and bismuth is in the production of various alloys. to impart free flowing properties. Sodium
Besides the use of elemental nitrogen, compounds of tripolyphosphate, Na5P3O10, is used in detergents
nitrogen have extensive applications in various fields. and in softening water. Phosphorus compounds like
Ammonia is used in various forms as a fertiliser, e.g., as
melathion, parathion, etc., are used as pesticides.
NH4NO3, (NH4)2SO4, (NH4)2HPO4, etc. Its applications
also include refrigeration, as a pH controller in food and Arsenic compounds find extensive use in agriculture
beverage industry, pharmaceuticals and in water as herbicides for weed and pest control, e.g., monosodium
purification. Nitric acid is another important compound of methyl arsenate, MSMA and disodium methyl arsenate,
nitrogen. Its major uses include manufacture of NH4NO3, DSMA. Arsenic acid is used as a wood preservative.
explosives like nitroglycerine, nitrocellulose an TNT. Sodium arsenite is used for aquatic weed control. Arsenic
Hydrazine, NgH4 and its methyl derivatives, oxide, As2O5, is used to decolourise bottle glass. Antimony
MeNHNH2 and MegNNELj, are used as rocket fuels. compound, antimonial lead, is used in storage batteries.
Nitrous oxide, N2O, is used as an anaesthetic. Compounds of antimony with aluminium, gallium
and indium, AlSb, GaSb and InSb, And applications
Sulphides of phosphorus, P4S3 and P4S10 are used
in infrared devices and in diodes.
in match industry. Phosphoric acid is used is fertilisers,
deaning of metals, rust proofing, pharmaceuticals, etc.

Target Practice 2
1. Nitrite ion (NO2) is said to interfere in the ring-test of Ca3(PO4)2 + ®+Cj 1500°C [P4 + CO]
nitrate ion (NOZ). Name the chemical that can decompose
NO'.
y

7. Compare white and red phosphorus in the following


2. CH3CH2CH2NH2 on reaction with HN02 changes to properties.
2-propanol as the main product. Explain the formation of White Red
2-propanol. 1. Smell
2. Solubility in CS2
3. With NaOH

3. Write the species of aqua-regia. 8. Write formula of phosgene and phosphine

What is the product when Of these


(a) gold is dissolved in it? (a) Which is poisonous?
(b) Which is reducing agent
(b) platinum is dissolved in it? Write reactions of their formation.
4. Nitrites and nitrates are added to food to keep meat red for Phosgene
longer time. But there are harmful effects. Phosphine
Harmful effects are 9. NH3 is more basic than PH3. It can be proved by

5. Anhydrous HN03 is colourless but it is often yellow in 10. Many hundred years ago, a ship was transporting calcium
carbide and calcium phosphide (stored in different
colour due to container). By chance sea-water entered sea and luminous
flame appeared. Passengers travelling in ship ran here to
6. White phosphorus is manufactured by modem there assuming a ghost would have come. However, a
chemist travelling in it explained it based on reactions.
electrochemical process represented in the flow-sheet. Fill These reactions are
in the blank space X, Y and Z.
566 | Essential Inorganic Chemistry

11. 1 mole each of H„P0„, H„P0., and H..PO. were given in 13. Write full form of
three test tubes. In a volumetric titration, standard 1 M (a) DSMA
Ca(0H)2 solution was provided. Volume of Ca(OH)2 used (b) MSMA
by (c) NPK
h3po2 ...............
H3PO~
14. Write formulae of
(a) Superphosphate
H3P°4 ...................... (b) Triple superphosphate
12. H3P0o is represented by 15. Write applications of the following
O (a) NH
I Z

(b) N2O
4

O—H (0 P4S3 .........


(d) Ca5(PO4)3OH

A
Specify the nature of different H (acidic^basic/reducing)
(e) Na5P3O10
(f) MSMA, DSMA

Answers
1. Urea 9. NH3+PH3 —> NH*PH2
2HNO2 + NH2CONH 2 ---- > 2N2 + CO2 + 3H2O NH3 is H+ ion acceptor thus, NH3 is more basic than PH3
which isH+ ion donor.
2. CH3CH2NH2 +HNO2 —> ch3ch2ch2n®
intermediate 10. CaC2 + 6H2O Ca(OH)2+ C2H2
Calcium carbide Acetylene
CH3CH2CH2N® CH3CH2CH® + N2
1° carbocation Ca3P2 +6H
+ 6H2O ---- > 3Ca(OH)2 + 2PH3
Calcium phosphide Phosphine
1,2-hydride shift
ch3ch,ch® ■* ch3chch3 CaC2 and Ca 3P2both were stored in containers but when water
2° carbocation
reacted with them PH3 and C2H2 were formed. Some highly
CH3CHCH3 —> ch3chch3 spontaneously flammable gases (P2H4 andH2) are also formed
and as a result of thus, C2H2 gas also bums (which people
OH assumed as ghost)
2-propanol
(2° carbocation is more stable than 1° carbocation hence, 11. 2H3PO2 +Ca(OH2) ---- > Ca(H2PO2)2+2H2O
2 mol 1 mol
1,2-hydride shift takes place)
1 mol 0.5 mol s 500 mL of 1 M Ca(OH)2
3. 75% cone. HC1 and 25% cone. HNO3 H3PO3 +Ca(OH)2 ---- > CaHP03+2H20
HNO3+3HC1 ---- > 2H2O + 2C1 + 2NOC1 1 mol 1 mol
(a) HAuC14 (chloro auric acid) 1000 mL of IM Ca(OH)2 solution
h 1

(b) H2PtCl6 (chloro platinic acid) 2H3PO4 + 3Ca(OH)2 ---- > Ca3(PO4)2+3H2O
2 mol 3 mol
4. They cause formation of carcinogenic nitrosoamines.
1 mol 1.5 mol = 1500 mL of 1 M Ca(OH)2
5. Photochemical decomposition (in sunlight)
Thus, H3PO2 = 500 mL 1 M Ca(OH)2
2HNO3 H2O + 2NO2 + |O2
H3PO3 s 1000 mL of 1 M Ca(OH)2
H3PO4 s 1500 mL of 1 M Ca(OH)2
6. X:SiO2 Y:P4 Z: CO 7. Refer text 0
8. Phosgene COC12
Phosphine PH3 12.
I
H- 0—H-<— | acidic | !
(a) both (b) PH3
Formation Phosgene is formed by air oxidation ofCHCl3
chci3 + |o2 ---- > COC12 + HC1
reducing
Phosphine is formed by action of NaOH on 13. Refer text
P4+3NaOH + 3H2O ---- » PH3+3NaH2PO2 14. Refer text
15. Refer text
Chapter 13: Group 15 (VA—The Nitrogen Family) | 567

Practice Exercise 2
1. HNO, can oxidise I" to I2; suggest a suitable reagent that 9. Arrange NH3, PH3, AsH3, SbH3 in increasing:
can estimate liberated I2. Also determine equivalent weight (a) covalent nature (b) reducing nature
ofHN02 in this estimation. (c) bond angle (d) thermal stability
2- Fe2+ as well as HNO2 can be used to estimate MnO4 in 10. What is the origin of the flickering light in marshes ?
acidic medium. If 100 mL of 1 M Fe2+ can be oxidised by 11. What is the chemistry of Holme’s signal?
V mL of 1 M MnO;/H+ solution, calculate volume of 1 M 12. What is the type of the following reaction?
HNO2 that can be oxidises by V mL of 1 M MnO4/H+ P4 + 3NaOH + 3H2O PH3 + 3NaH2PO2
solution.
3. Vessels made of like Al, Fe, Cr, Ni etc., can be used to store 13. Explain reducing action of H^C^ and H.jPO3 with some
cone. HNO3. Explain. examples.
4. Make distinction between NO" and NO".
14. Classify acids and normalsalts out of:
5. What is effect of heat on NH 4.NO,J and NH 4.NO,?
z (a) NaH2PO3 (b) NaH,PO4
6. NO" interferes in the ‘ring test’ of NO;. How is it removed (c) NaH,PO2 (d) Na^POj
from the mixture? (e) Na2HPO4
7. What is hybridisation of N-atom in HNO2 and HNO3? 15. How is phosphorus estimated?
8. Complete the following in acidic medium : 16. (a) 4H,PO, ---- > 3H_PO 4 + PH
JO J ••
(a) Fe2++NO3 ----->
(b) SO2" + NO; ---- > (b) 2H,PO,
J L
---- > H,PO,+PH,
J *> J

(c) SO2J " + NOIZ ---- >


Determine equivalent weights of the underlined.

Answers
1. HN02+H*+e" ---- > N0 + H20 8. (a) 3Fe 2+ + 4H+ + NO; ---- > 3Fe3+ + 2H2O+NO
21" ---- > I2 +2e" (b) SO^" + 2NO; + 2H+ ---- > SOf + 2NO, + H2O
Na2S2O3 (hypo sodium thiosulphate) is used to estimate (c) SO^" + 2NO; + 2H* ---- > SO^" + 2NO + H2O
liberated I2.
9. (a) Covalent NH3 > PH3 > AsH3 > SbH3
2Na2S203 +I2 ---- > 2NaI + Na,S4O6 (b) Reducing nature NH3 < PH3 < AsH3 < SbH3
HN02 = N0 or HN02 = le" (c) Bond angle NH3 > PH3 > AsH3 > SbH3
Equivalent weight of HN02 = molecular weight
107° 48' 93°36' 91°48' 91°18'
2. MnO; + 8H+ + 5e" -- > Mn2+ + 4H20 Eq. wt. = M/5
Fe2+ ---- > Fe3+ + e" = M/1 10. PH3 frequently contains traces of diphosphine (P2H4) which
causes it to catch fire spontaneously. This is the origin of the
HN02 + H,0---- > NO; + 3H+ + 2e“ = M/2
flickering light called ‘will-o-the-wisp’, which is sometimes
V mL of 1 MnO; = 100 mL of 1 M Fe2+
seen in marshes.
or V mL of 5N MnO; s 100 mL of IN Fe2+ —(i)
Also 11. Mixture of CaC2 and Ca3P2 placed in the container with holes
VmLoflMMnO; s V'mLoflMHNO2
is thrown into the sea. Both react with H,0 :
V mL of 5 N MnO; s V' mL of 2N HNO2 ...(ii)
CaC2 +2H,0---- > Ca(OH), +C.jH2
Thus, from (i) and (ii) V' = 50 mL
Ca3P, +6H,0---- > 3Ca(OH)2 +2PH3
3. There is formation of a protective oxide layer on the surface of
the metal when it reacts with HNO3. Hence, vessels of such Some P2H4 is also produced, which catches fire instantly,
metals can be used to store HNO3. which in turn ignites C2H2 (acetylene) and then luminous
4. Add KI solution. NO; would oxidise I" to I2. NO; would give flame is produced. This guides the approaching ship. This
‘ring-test’. signal is called Holme’s signal.
5. NH4NO3 N2O + 2H2O 12. In this reaction P4 (ox. no. = 0) has been reduced toPH3 (ox. no.
NH4NO2 n2 +2H2O = - 3) as well oxidised toH,PO; (ox. no. = + 1). Hence, this is a
disproportionation reaction.
6. If NH4C1 or urea is added to a solution containing NO; and
NO;, NO; is decomposed as N2 and thus removed : 13. H3PO2 + H,0---- > H3PO3 + 2H
NaNO2 + NH4C1---- > NaCl + N2 + 2H2O H3PO3 +H,0---- > H3PO4 +2H
2NaNO2 +NH2CONH22 ---- > 2N2 +Na2CO3 +2H,0 H produced causes reduction of salts into metals :
H-O-N^ AgNO3 +H---- > HNO3 +Ag
7. H—0—N=O
CuSO4 + H---- > H2SO4 + Cu
O
2HgCl, + 2H---- > Hg,Cl2 + 2HC1
N-atom is sp2 hybridised in both.
Hg2Cl, + 2H---- > 2Hg + 2HC1
568 | Essential Inorganic Chemistry

14. Salts with ionisable H (attached to 0-atom) are acid salts, 16. (a) H3PO3 ---- > H3PO4 Equivalent weights
others are normal salts : +3 +5
„ mol.*t
0 0 oxidn E, =--------
1 2
I I moL»i
(a) H—O (b) H—OK ^0—H H3Pp3 —4 PH3 £2=“F
O—Na O—Na I—redn —I
acid salt acid salt
Eq. wt. ofH3PO3 in disproportionation reaction
0 0 = Er + E2
I 2
3
2''
2
= — mol. wt. = -x 82
3
(0 H (d) Na—O
= 54.67
O—Na 0—Na
(b) —>* h
2H3PO2 ---- -.4
n33po +ph3
normal salt normal salt +1 +5 —3
I—oxidation -I
0
-------- reduction-----
/ ’^0—H
(e) Na—0
E, (oxidation) = —
O—Na 4
acid salt
E2 (reduction) = —
4
15. Phosphorus can be estimated by converting it into phosphates
Eq. wt. (H3PO2) = + E2 = M = 33
and then precipitating as MgNH4PO4 (by adding NH40H and
Mg2*).
Eq. wt. of (X) when it undergoes disproportionation
HNO3, A
p 4 H3PO4 = Ej (oxidation) + E2 (reduction)
H3PO4 +NH*4 + Mg2+
2*---- > MgNH4PO4 + 3H+ where, Er (oxidation) is the eq. wt. in oxidation reaction andfj
On ignition, white ppt of Mg2P2O7 is obtained : (reduction) that of reduction reaction.
2MgNH4PO4 Mg2P2O7 +2NH3 +H20
[ .. .- iZ-* z. <1 .-i- *- — — •

Total Practice Set


(Read, Plan & Solvel)
SRMEHHHBRHBBOHBM__ ___ ;__

Solution NE^CONH^ (Urea) reacts with cone. H2SO4


Problem 1. Given the following bond dissociation
forming XCH^OgSJ.X contains one ionisation H (since, X
enthalpies (kJ mol"1):
is a monobasic acid). Thus,
P=P 490 0 0
P-P 209 I I
NH,—C—NH,+H—0—S—O—H ---- >
N=N 946 Urea Sulphuric acid
N —N 160
NHj — SO3H + NH3 +CO
Compare the enthalpy changes for the process X
(i) 2P2(g) ---- > P4(g)
X contains free —NHg as well as free —SO3H.
(ii) 2N2(g) ---- > N4(g) (NaNO2 +HC1) converts free —NH2 into free —OH
Solution The process involves replacement of group liberating N2.
2(X=X) units by 6 X—X bonds in the X4 tetrahedron. NaNO2 +HC1---- > HNO2 +NaCl
Energy is released when bonds are formed and thus the
species formed is stable. NHa-SOaH +HNO2 ---- > N2 ? +H>O+H2SO4
(i) 2P2(g) ---- > P4(g) 1 mol X 1 mol X 1 mol
/P H2SO4 +BaCl2 BaSO4 +2HC1
/
2(P=P) P 1 mol 1 mol
•> P-
P Thus, X is NH2—SO3H (amino sulphonic acid), 1 mole of
which liberated 1 mol of N2 on reaction with
AHf = - 6 x BE (P — P) + 2BE (P=P) (NaNO2 + HC1) at 0°C and in turn resulting solution
= - 6 x 209 + 2 x 490 gives 1 mole ofBaSO4 on reaction withBaCl2.

= - 274 kJ mol"1 Problem 3. Suggest synthesise for,


Formation of P4 is exothermic thus possible. (a) Na15NH2, 15N2 fromK15NO3.
(ii) 2N2(g) ---- > N4(g) (b) 32ph3,h3l3: 2PO3fromCa3(32PO4)2.
= - 6BE (N — N) + 2BE (N=N) Solution (a) Isotopic 15 N compounds are to be
= -6x10 + 2x946 synthesised from K15NO3.
15
= + 932 kJ mol"1 KNO3 + NaOH + Zn * 15
NH3 (unbalanced)
Formation of N4 is endothermic, thus is not feasible. Na
15
Problem 2. The action of concentrated sulphuric acid 15N2 NaNH2
on urea (NHjCONK^) results in the production of a white
crystalline solid X of formula HgNOyS. This is a monobasic 32 A 32
acid. On treatment with sodium nitrate and dilute (b) Ca3(PO4)2 + SiO2 + C *P4
hydrochloride acid at 0°C it liberates 1 mole of N2 per mol NaOH
of X, and on addition of aqueous barium chloride the
resulting solution yields 1 mole of BaSO4 per mol of X 32 H20 32
h3po3 PI3 ph3
taken.
Deduce the structure of X.
570 | Essential Inorganic Chemistry

24.7 x 0.1007
Problem 4. Deduce what you can about the nature of Equivalent of Cr2O^ =
the following reactions: 1000
(a) 1 mole of NH2OH reacts with 2 moles of Ti(III) in = 0.0025
the presence of excess alkali, and the Ti(III) is
converted into Ti(IV). Equivalent ofFe3+ orFe2+ =0.0025
(b) Silver phosphite, Ag2HPO3 is warmed with water, Equivalent of NH^OH = 0.0025
and the silver is precipitated as the metal. 1 rxnr att /. i \ 25x 0.0497
(c) When 1 mole of hypophosphorous acid is treated Moles of NH20H (taken) =--------------
2 1000
with excess of iodine in acidic solution, 1 mole of
iodine is reduced; on making the solution alkaline, = 0.00125
another mole of iodine is consumed. Moles (NH2OH) = Change in oxidation number
Solution (a) Based on change in oxidation number of x Equivalent of NH^OH
two moles Ti(III) from 3 to 4, NH2OH is reduced to NH3. . nxTTT ATT 0.0025
Change m oxidation number of NH2OH = qq^
2Ti(III) ---- > 2Ti(IV)
+3j .j ..
oxidation +4| =2
two units change Oxidation number of N is NHjOH = -1
NHaOH------- >NH3 Oxidation number of N is oxidised produced
-1 J
reduction -3 =-l+2=+l
two units change
Thus, compound formed = N2O
Thus, NHjOH is reduced to NH3
Problem 6. Give a scheme of reactions that do not
2Ti3+ + NH20H + H2O--> 2Ti‘ + NH3 + 2OH“ involve disproportionation of the oxides of nitrogen.
(b) Ag+ (in Ag2HPO3) is reduced to Ag
Solution
2Ag’ —reduction^ 2A
* 2Ag Oxidation state of
two units change
nitrogen
Hence, HPO^"(oxidation number of P=+3) should be V n2o5 HNO3 NO3
oxidised to PO|“ (oxidation number of P=+5) giving a "d
change of oxidation number of two units. c
2Ag+ + HP0|_ + H,2O ---- > PC3’ + 2Ag + 3H+ IV no2 N2O4
| oxidation a
III
hno2
reduction
(0 H3PO2 + + HjO---- > H3PO3 + 21" + 2H+ II NO
H3PO3 +12+ 2OH" --- > H3PO4 + 21- +H2O
0
Problem 5. 25.0 cm3 of 0.0497 M solution of NI^OH
was boiled with excess of ferric alum solution in B^SC^.
The Fe(II) formed required 24.7 cm3 of 0.1007 NKaCtjO.? -III NH3
solution. Identify the product formed by oxidation of
NHjOH. Reactions:
Solution Fe3+ is reduced to Fe2+ which in turn is (a) 2NO2 + O3 ---- > N2O5 + O2
oxidised to Fe3+ by Ci^O?- in acidic medium. (b) 2NO2 +H2O2 ---- > 2HNO3
NHjOH + Fe3* ■» Fe2+ + (oxidation product) (0 2NO2 N2O4
6Fe2++Ci2O27-+14H+ -> 6Fe3+ + 2Cr3+ + 71^0 (d) N2O4+Cu ---- > Cu(NO2)2
Chapter 13: Group 15 (VA — The Nitrogen Family) | 571

(e) 2HNO2 +2HI ---- > la +2NO+2H2O Solution


(0 Fe2+ + HNO2 + H+ > Fe3+ + NO + l^O Oxidation state of
nitrogen
(g) 2NO + O2 2NO2
V HNO3q NO3 NO2
(h) 2Cu + NO2 -> Ci^O + NO b'
c
(i) c+no2 CO2+iN2 IV I—NO2 d N2O4

(j) NO2 +2H2 iN2 +2H2O III e


1“ I
HNO2—I
b c

NO+

(k) 2NO2 +7H2 > 2NH3 + 41^0 II


ld
*— NO
1 |e
(I) N2 +3H2 > 2NH;3
I n2o
(m) 4NH3+3O2 -> 2N2 +6H2O
Reactions:
(n) 4NH-33+5O
* ov2 ---- > 4NO + 61^0
(a) 2NO2 +H20 ---- > HN03 +hno2
(0) N2 +02 ---- > 2N0 (b) n2o4 4 N0+ +N03

Problem 7. Give a scheme of reactions which involve (c)


disproportionation of the oxides of nitrogen. N2O4+3H2SO4 ---- > N0++N02+3HSO4+H30+
(d) 3HNO2 > HNO3+2NO + H2O
(e) 3N0 » N2O+NO2
572 | Essential Inorganic Chemistry

Explanation However, ionic halides are quite stable. NaN,


decomposes smoothly (but not explosively) at about
(a) (SiH3)N is planar while (CH3)3N is pyramidal 600 K
- In case of (CH3)3N, lone pair on N and three covalent 600 K
2NaN3 2Na+3N2
bonds with CH3 groups results in sp3 hybridisation
but lone pair-bond pair repulsion causes distortion It is due to linear structure of N3 which is having
hence gives pyramidal structure : resonating structures is stable.
2- 2-

0 :n=n=n: <—> :n=n ■n: » :n-n=n:

(d) H;jPO2 is a powerful reducing agent butTL^C^ u


ch3 ch3
not.
ch3 H-jPO, h3po2
oxidation number of P +5
+5 +1
But in case of (SiH3)3N, lone pair on the nitrogen is H3PO2 is a monobasic acid with oxidation number of
transferred to the empty d-orbitals of silicon (pn-dn P as +1 which can be further increased to +3 and tn
overlap) thereby causing planarity of unit. +5. Thus, H3PO2 can be used as a reducing agent
(b) NF3 is stable while NCLj and Nig are explosive H3PO4 is already in oxidised state (oxidation number
ofP = + 5) Thus, it is not a reducing agent HgPO4 with
- In NF3 the electronegativity difference (N = 3.0; all the ionisable to hydrogens is a tribasic acid and
F= 4.0) permits strong pit-pit bonding and the H3PO2 with only one ionisable hydrogen is a
molecule is stable. But in NC13 and NI3, the monobasic acid. Two other hydrogen directly
electronegativity difference is nil or small and the attached to P provide reducing nature to H3PO2.
molecules are unstable with pit- pit bonding weak. 0 Reducing
Hence, NF3 is a stable gas, NC13 an explosive liquid I nature / H
and NI3 an explosive solid. H—0—P—0—H
iR—P=O
(c) Hydrogen azide is explosive, while sodium azide
is not. O-H

- 2N3H
slightly heat
> 3N2 +H2 t
ionisable H
Due to increase in volume, there is explosion. Thus, ionisable H
N3H is used as detonators and explosives. I h2po;+h+
Pot+ 3H+
7-

Master Exercises
Exercise 1
(Stage 1: Learning)
Short Answer Type Questions 12. Write balanced equation to account for each of the
1. Identify the group 15 (VA) elements (s) that best fits following observations:
each of the following descriptions: (a) HNO3 turns brown when exposed to air
(a) makes up part of bones and teeth (b) HNO3 turns yellow-brown on standing
(b) forms stable salts containing M3’ ions (c) Ag dissolves in dilute HNO3, yielding a colourless
(c) is the most abundant element in the atmosphere gas
(d) forms a basic oxide. (d) N2H4 (hydrazine) reduces iodine to I" and in the
process is oxidised to N2 gas
2. Draw an electron-dot structure for N2, and explain
13. What substances form when concentrated HNO3 is
why this molecule is so unreactive?
reacted with As2S3?
3. In the ‘ring test’ of NO3 ion, there is formation of 14. Arrange oxides (M2O3) of nitrogen family in
brown colour ring. What is its formula? increasing acidic nature.
4. What is the nature of the nitride ion? 15. POC13, like P2O5, is also used as dehydrating agent.
5. Apart from N2O, which other colourless gas supports Write reaction.
combustion? 16. How many (P—0) bonds are there inP4O6 and P4O10?
6. Write an equation for the reaction of burning Mg with (Coordinate bonds may also be included.)
N02. 17. One of the oxoacid salts on heating with cone. HN03
7. Automobiles are now provided with catalytic and ammonium molybdate gives yellow ppt. What is
converters. What are their role in reducing this ion? Give chemical reactions.
air-pollution?
18. What are structures of metaphosphoric acid and
8. Colourless salt (A) + NaOH gas (B) giving white trimetaphosphoric acid?
fumes with HC1 + alkaline solution (O 19. H4P2P7 + H>0 ---- > 2H3PO4
(C) + Zn---- > gas (B) (A) — gas (D) + liquid (E) What conclusion can you make from this reaction
both triatomic regarding structure of H4P2O7?
Identify (A), (B), (C), (D) and (E). 20. The structures of various phosphate and silicate
9. A few drops of concentrated ammonia solution added anions have many similarities. Give examples.
to a calcium bicarbonate solution causes a white 21. Starting with elemental phosphorus, P4, show how
precipitate to form. Write a balanced equation for the
reaction. you would prepare phosphoric acid.
10. What is the type of the following reaction? 22. The product Catf^PQ,^ is known as triple
NH3+NH3 NH4 +NH^ phosphate but it is said to be richer in phosphorus
11. Identify salts (A) and (B) based on following than other superphosphates. Explain.
reactions: 23. What happens when urea slowly hydrolyses in the
(A) or (B) —a0H' A)NH3 <■ H2O soil?
24. Which has maximum% of (P) out of
(A) -A-> (C) (gas) (D)
(a) superphosphate
(B) —A% (E)(gas) (b) triple superphosphate
(c) fluoroapatite rock
(E) is used as an anaesthetic.
574 | Essential Inorganic Chemistry

Analytical Questions 7. Determine the oxidation number of nitrogen in


(a) N2O (b) NO2
1. Explain : (c) HNO3 (d) NH3
(i) PC15 is common butNCl5 is unknown.
(ii) The N—O+ ion has a shorter bond length than 8. Write the resonance Lewis structure for N2O, N02
does NO, even though the latter has an extra andNO3.
electron 9. Illustrate how a metal such as copper can give
(Hint: compare bond order) different products with nitric acid? Give balanced
2. Predict the structures of the following species: equations.
(a) NF4+ (b) N2F3+ 10. Write balanced equation(s) for the manufacture of
(c) NH2OH (d) SPC13 nitric acid by the oxidation of ammonia.
(e) PF3C12.
11. Describe the uses of ammonia and nitric acid.
3. It has been claimed that NH4NO3 fertiliser can be
rendered unexplodable by a process that involves 12. Classify the following compounds into acidic, basic
additives such as diammonium hydrogen phosphate and amphoteric oxides
(NH4)2HPO4. Analysis of such a “densensitised” SrO, SiO2, A12O3, N2O5, SO2, P4O6, C12O7. Write
sample of NH4NO3 showed the mass per cent balanced equations for the reaction of each of these
nitrogen to be 33.81%. compounds with water, a base or an acid as the case
Assuming that the mixture contains only NH4NO3 may be.
and(NH4)2HPO4, what is the mass per cent of each of 13. How far do you agree with the phenomenon concept
these two components? of ‘inert pair effect’? Justify your answer.
4. A mixture of the sulphides of As2S3, Sb2S3 and Bi2S3 14. Write balanced equations for the following reactions
was treated with a Na2S solution. Which sulphide (a) Ca3P2 +H2O ---- >
remained undissolved? Write the equations of the (b) P4O10 + ^0 ---- >
sulphide dissolving reactions. (c) As4 + Cl2 (excess) ----- >
5. Hyponitrous acid and nitramide both have the (d) P4 +KOH+H2O ---- >
formula H2N2O2. Hyponitrous acid is a weak diprotic 15. Describe the method for the isolation of elemental
acid; nitramide contains the amide group (—NH/j). phosphorus from Ca3(PO4)2.
Write structures of both of these compounds.
16. What is the oxidation state of phosphorus in the
6. Peroxonitrous acid is an unstable intermediate following?
formed in the oxidation of HNO2 by H^. If has the (a) H3PO3 (b) PC13
same formula as nitric acid, HNO3. Show how you (c) Ca3P2 (d) Na3PO4
would expect these two acids to differ in structure? (e) POF3 ?
17. Calculate the volume of 0.1 M NaOH solution
NCERT Problems required to neutralise the solution produced by
1. Give examples of compounds in which nitrogen dissolving 1.1 g of P4O6 in water.
exhibits oxidation states of -3, +3 and +5. Hint: P4O6 + GI^O---- > 4H3PO3
2. Discuss the conditions required in the Haber process
HgPO3+2NaOH ---- > Na2HPO3 + 2^0] x 4
for the manufacture of ammonia.
P4O6 s 4H3PO3 = 8NaOH (HgPO3 is a dibasic acid)
3. How is dinitrogen prepared in the laboratory?
1.1 g P4O6 = ~ = 5 x 10-3 mol P„O6
4. Write the products of the following reactions
(give balanced equations) = 4 x 10-2 mol NaOH
Mg3N2 +H2O ---- >
Let NaOH = VmL
I2 + HNO3 (concentrated) ---- > Vx0 1
Moles of NaOH = • -- -
Cu + HNO3 (concentrated) ---- > 1000
Li+N2 ---- > VxO.l
= 4 x 10"2
HNO3 + P4O10 ---- > 1000
5. How would you prepare a sample of deuterated V = 400mL
ammonia, ND3 ? = 400 mL of 0.1 M NaOH
6. Illustrate how nitrogen compounds provide good 18. Compare the structures of white phosphorus, P4O6
examples of multiple bonding and resonance? and P4O10 .
Chapter 13: Group 15 (VA — The Nitrogen Family) ] 575

Exercise 2
(Stage 2: High Skill Problem Solving)
Only One Option Correct 10. Which is/are correct statements about P4O6 andP4O10?
t Of the following acids (a) Both form oxyacids H3PO3 and H3PO4
I : hypo phosphorous acid respectively
II : hydrofluoric acid, (b) In P4O6 each P is joined to four 0 and in P4O10
III: oxalic acid each P is joined to six 0 atoms
IV: glycine (c) Both (a) and (b)
(a) I, II are monobasic; III dibasic acid and IV (d) None of the above
amphoteric 11. 1 mole each of H3PO2, H3PO3 and H3PO4 will
(b) II monobasic; I, III dibasic acid; IV amphoteric neutralise x moles of NaOH,y moles ofCa(OH)2 and
(c) I monobasic, II, III dibasic, IV amphoteric z moles of Al(0H)3 (assuming all as strong
(d) I, II, HI dibasic; IV amphoteric electrolytes), x, y, z are in the ratio of
2. Following metals are soluble in aqua-regia (a) 3 :1.5 :1 (b) 1: 2 : 3
(a) Pt (b) Au (c) 3 : 2 :1 (d) 1: 1: 1
(c) Ag (d) all of these 12. The dipole moments of the given molecules are such
3. Which forms pit-pn multiple bonds with itself and that
with C and 0? (a) BF3 > NF3 > NH3 (b) NF3 > BF3 > NH3
(a) P, As (b) N, As (c) NH3 > NF3 > BF3 (d) NH3 > BF3 > NF3
(c) N,P (d) N
13. The electronegativities of N, C, Si and P are such that
4. In P4 (tetrahedral) (a) P < Si < C< N (b) Si < P < N < C
(a) each P is joined to four P (c) Si < P < C< N (d) P < Si < N < C
(b) each P is joined to three P 14. Glacial phosphoric acid is
(c) each P is joined to two P (a) HP03 (b) H3PO3
(d) P4 does not exist (c) HgPO4 (d) H4P2O7
5. Maximum number of covalent bonds formed by N and 15. Phosgene can be obtained when
P respectively are (a) white phosphorus reacts with alkali
(a) 3, 5 (b) 3, 6 (b) calcium phosphide reacts with water
(c) 4, 5 (d) 4, 6 (c) chloroform reacts with air
6. For the hydrides of nitrogen family going down the (d) bone comes in contact with water
group 16. Which are acid salts?
(a) stability decreases (a) NaH2PO2,NaoHPO3 (b) Na2HPO3, Na2HPO4
(b) reducing activity increases (c) NaHCO3,Na2HPO4 (d) All of these
(c) bond angle HMH decreases
(d) all the above variations followed 17. Which of the following statements are true?
(a) Cold and very dilute HN03 forms NH4NO3 with
7. Which of the following compounds do not exist? Zn or Sn
n4, p4, pci5, nci5, nci3, p2o5, no2, P02 (b) Concentrated HN03 forms l^SnCXj with Sn
(a) N4, NC15, N02 (b) N4, NC15, P02 (c) Cold and more concentrated HN03 forms N02
(c) PC15, NC15, P02 (d) All of these with Cu
8. Which of the following is/are paramagnetic? (d) All the above are correct
NO2,NO,N2O4,N2O2,N2O5 18. N02 is not obtained when following is heated
(a) OnlyNO2 (b) N02,N0 (a) Pb(NO3)2 (b) AgNO3
(c) NO, N02, N2O5 (d) All are paramagnetic (c) LiN03 (d) KN03
9. Which of the following dibasic acids shows 19. Which is least basic?
geometrical isomerism? (a) NF3 (b) NC13
(a) Hyponitrous acid (c) NBr3 (d) NI3
(b) Maleic acid C4H4O4 20. Which is/are used as an anaesthetic?
(c) Both (a) and (b)
(a) N20 (b) C,H4
(d) None of the above (c) CHC13 (d) All of these
576 | Essential Inorganic Chemistry

21. Which is a set of acid salts and can react with base? (a) cone. H2SO4 (b) P2O5
(a) NaH9PO2,Na2HPO3,NaH2PO4 (c) CaCl2 (d) Mg(C104)2
(b) Na2HPO3, NaH2PO3, Na2HPO4 32. N2O is formed when
(c) NaHCO3, NaH2PO3, Na2HPO4
(d) All of the above (a) moist Fe reacts with NO
(b) Sn2+ reacts with cone. HNO3 in presence of HC1
22. Which is used as a rocket fuel? (c) cold dil. HNO3 reacts with Cu and Zn
(a) N2H4 (b) Polybudiene (d) by all the reactions
(c) Both (a) and (b) (d) None of these
33. Which of the following compounds has the greatest
23. NH3 can't be obtained by ionic character?
(a) heating of NH4NO3 or NH4NO2 (a) PC15 (b) SiCl4
(b) heating of NH4C1 or (NH4)2CO3 (c) CC14 (d) BC13
(c) heating of NH4NO3 with NaOH 34. Fe3+ oxidises NI^OH to
(d) reaction of AIN or MggN2 or CaCN2 with KjO
(a) NO2 (b) N2O
24. Red and white phosphorus will differ but not in (0 N2 (d) NO
(a) smell
(b) solubility in CHC13 35. Fe2+ reduces NI^OH to
(c) exhibiting phosphorescence (a) NH3 (b) N3H
(d) reactions with HNO3 (c) N2H4 (d) N2
25. Choose correct statements. 36. All of the following are bases except
(a) Superphosphate is [3Ca(H2PO4)2 + 7CaSO4] (a) N2H4 (b) NH20H
(b) triple superphosphate is 4Ca(H2PO4)2 (c) NH3 (d) NgH
(c) Both (a) and (b) are correct 37. An example of an “ortho” acid is
(d) None of the above is correct (a) HAsO2 (b) H3AsO4
26. N2O (laughing gas) finds use in the following except (c) (HP03)n (d) I^AsA
(a) as a propellant for whipped ice-cream 38. Compound used in safety matches is
(b) as an anaesthetic (a) P4S3 (b) P4
(c) for the preparation of NgH (c) P2O5 (d) PC13
(d) as fuel for rockets 39. Repeated use of which one of the following fertilizers
27. Following are neutral oxides except would increase the acidity of the soil?
(a) NO (b) N2O (a) Urea
(c) CO (d) NO2 (b) Superphosphate of lime
28. Select incorrect statement about N2O4. (c) Ammonium sulphate
(d) Potassium nitrate
(a) It self ionises as NO+, NO3
(b) Substance containing NO+ is said to be acid and 40. Arsenic drugs are mainly used for the treatment of
that containing NO3 is said to be base (a) cholera (b) typhoid
(c) N2O4 is paramagnetic (c) jaundice (d) syphilis
(d) NO2 dimerises to N2O4 with disappearance in 41. The wrong statement about N2O is
paramagnetism (a) it is nitrous oxide
29. Acid rain may cause (b) it is least reactive oxide of nitrogen
(a) rusting easier (c) it is not a linear molecule
(b) stone-cancer in Taj Mahal (d) it is known as laughing gas
(c) non-fertility of soil 42. The true statement for the acids of phosphorus.
(d) all of the above H3PO2, H3PO3 and H3PO4 is
30. Select correct statement. (a) their acidic nature is : H3PO4 < HgPO3 < H3PO2
(a) Mixture ofNH4 Cl and NaN03 on heating gives N2 (b) all of them are reducing in nature
gas (c) all of them are tribasic acids
(b) CFC is used as refrigerating fluid and as (d) the geometry of phosphorus is tetrahedral in all
propellant in aerosols the three
(c) Phosgene is formed when P4 reacts with NaOH
43. The oxide which is not a reducing agent is
(d) Phosgene dissolves in water forming P2O5
(a) CO2 (b) NO2
31. ‘Anhydrone' is a very effective desiccant (water (c) SO2 (d) C1O2
absorber) used in ‘dry batteries'. It is
Chapter 13: Group 15 (VA — The Nitrogen Family) | 577

44. Nitrogen is obtained by the thermal decomposition of (a) 25 (b) 33


(a) NH4NO2 (b) NH4NO3 (c) 50 (d) 75
(c) AgNO3 (d) Pb(NO3)3 56. Which blue-liquid is obtained on reacting equimolar
amounts of two gases at -30°C?
45. The substance used as a fast drying agent in the
(a) N2O (b) N2O3
laboratory is
(0 N2O4 (d) N2O5
(a) Na2PO4
57. Calcium cyanamide on treatment with steam
(b) P2O5
produces
(c) charcoal (a) NH3 + CaO (b) NH3 + CaHCO3
(d) anhydrous calcium chloride (c) NH3 + CaCO3 (d) NH3 + Ca(OH)2
46. Laughing gas is prepared by heating 58. Which gas cannot be collected over water?
(a) NH4C1 +NaNO3 (b) NH4C1 (a) O2 (b) PH3
(c) (NH4)2SO2 (d) NH4NO2 (c) N2 (d) SO2
47. The sides of safety matches contain 59. NaOCl reacts with NH3 to produce
(a) red phosphorus + sand powder
(b) P4S3 (a) NHjOH (b) NH^
(c) Ca3(PO4) + glass pieces (c) N2 (d) NO
(d) KC1O3,KNO3, sulphur + antimony 60. Which one of the following acid possesses oxidising,
reducing and complex forming properties?
48. Cold solution of barium nitrite on mixing with
(a) HN03 (b) HC1
sulphuric acid produces
(c) H2SO4 (d) HNO2
(a) BaSO4 +HNO2 (b) BaSO4 +HNO3
(c) BaSO4 + NO2 (d) BaSO4 + N2 + O2 61. On hydrolysis, PC13 gives
49. 4HNO3 + P4O10 ---- > 4HPO3 + X (a) H3PO3 (b) H3PO4
(c) POC13 (d) HPO3
In the above reaction the product X is
(a) NO2 (b) N2O3 62. On photochemical decomposition of HN03 the
(c) N2O4 (d) N2O5 products formed are
(a) NO and O2 (b) NO2 and O2
50. Which of the following oxides of nitrogen is
(c) N2O4 and NO2 (d) N2O5 andNO2
paramagnetic?
(a) N2O (b) N2O5 63. Nitric oxide is formed by the reaction of
(0 NO2 (d) N2O3 (a) C + N2O (b) Cu + HNO3
(c) Cu + N2O (d) Cu + NO2
51. The structural formula of hypophosphorus acid is
64. On heating lead nitrate the product formed is
0 0
(a) NO (b) NO2
I I (c) N2O (d) N2O5
(a) (b) P-
OH H OH
65. The number of P—0—P bonds in cyclic
H OH metaphosphoric acid is
0 0 (a) zero (b) two
I I (c) three (d) four
(0 HO^p (d) HO /P- OH
OH 66. Polyphosphates are used as water softening agents
H OH because they
(a) form soluble complexes with anionic species
52. Among the following, the paramagnetic compound is
(b) precipitate anionic species
(a) Na2O2 (b) P2O5
(c) form soluble complexes with cationic species
(c) N2O (d) KO2
(d) precipitate cationic species
53. Which of the following is most acidic?
67. Solid crystalline PC15 has structure which of the
(a) N2O5 (b) P2O5
(c) As2O5 (d) Sb2O5 following?
(a) Bi-pyramidal moieties
54. Which of the following phosphorus is the most (b) Octahedral and tetrahedral ions
reactive? (c) Square-pyramidal moieties
(a) Red phosphorus (b) White phosphorus
(d) Pentagonal moieties
(c) Scarlet phosphorus (d) Violet phosphorus
68. Hydrazine is not
55. The percentage ofp-character in the orbitals forming (b) an oxidising agent
(a) a reducing agent
P—P bonds in P4 is (d) a base
(c) an acid
578 ] Essential Inorganic Chemistry

69. Which of the following oxides of nitrogen is solid? (b) It increase pH of the water
(a) NO2 (b) N2O (c) It removes Ca2+ and Mg2+ ions from water that
(c) N2O3 (d) N2O5 causes hardness
70. The element which forms oxides in all oxidation (d) It increases its solubility in water
states +1 to +5 is 81. Among the VA(15) group elements one of them forms
(a) N (b) P a compound sold under the name ‘Pearl White’. The
(c) As (d) Sb element used is
71. Which of the following is arranged in the increasing (a) N (b) P
order of enthalpy of vaporisation? (c) Sb (d) Bi
(a) NH3, PH3, AsH3 (b) AsH3, PH3, NH3 82. White arsenic has the chemical composition
(c) NH3, AsH3, PH3 (d) PH3, AsH3, NH3 (a) As2 (b) As4
72. The statement true forN3 is (c) As2O5 (d) As2O3
(a) It has non-linear structure 83. Both NF3 and NC13 are covalent but they differ in the
(b) It is called pseudohalogen extent of hydrolysis because
(c) the formal oxidation state of nitrogen in this (a) NF3 is more stable than NC13 and hydrolysis
anion is +1 product ofNF3, HOF, does not exist
(d) It is isoelectronic with N2O (b) dipole moment ofNF3 is greater than that ofNCl3
73. Metallic tin and cone. HNO3 reacts to form (c) electronegativity of F is greater than that of Cl
(a) metastannic acid (b) stannic nitrate (d) Cl can expand its octet by using d-orbitals
(c) stannous nitrite (d) stannous nitrate 84. Nitrogen differs from P, As, Sb and Bi in the following
74. PI3 upon hydrolysis give properties
(a) It forms diatomic molecule
(a) monobasic acid and dibasic acid
(b) It is not able to extent its coordination number
(b) monobasic acid and tribasic acid
beyond four
(c) monbasic acid and a salt
(c) Nitrogen does not function as a Lewis acid
(d) dibasic acid and tribasic acid
whereas P, As, Sb and Bi do so
75. Common name of phosgene is (d) In all the properties given above
(a) carbon tetrachloride (b) phosphoryl chloride
85. dn- pit bonding is shown in
(c) carbonyl chloride (d) phosphorus
trichloride (a) NO5,NO2-,N3-,CN- (b) P2O„PA,PQ?-
76. The bond angle in AsH3 is greater than that in (c) NH3,PH3,BiH3 (d) CO, NO, CO,, NO,
(a) NH3 (b) H2O 86. In plants, atmospheric nitrogen is converted into
(c) BC13 (d) None of these (a) carbohydrate (b) proteins
77. When PC13 reacts with sulphuric acid sulphuryl (c) amino acids (d) fats
87. Pure N2 is prepared in the laboratory by heating a
chloride is formed as the final product. This shows
that sulphuric acid mixture of
(a) has two hydroxyl groups in its structure (a) NH4C1 and NaOH (b) NH4OHandNaCl
(b) is a derivative of sulphur dioxide (c) NH4C1 andNaNO2 (d) NH4ClandNaNO3
(c) is a dibasic acid 88. Essential constituents of plants and animal tissues
(d) has greater affinity for water are
78. Pentavalence is phosphorus is more stable when (a) N and P (b) N and As
compared to that of nitrogen even through they (c) Cu and Mg (d) Ca and Mg
belong to same group is due to 89. Major minerals containing nitrogen are
(a) reactivity of phosphorus (a) DNA (b) RNA
(b) inert nature of nitrogen (c) KNO3,NaNO3 (d) all of these
(c) presence of d-orbital 90. Most abundant uncombined element present in
(d) large size of phosphorus atom atmosphere is
79. Bond angle of 109°28/ is found in (a) Ca (b) Mg
(c) Al (d) N
(a) NH3 (b) HjO
91. Substances burn more readily in N2O than in air
(c) CHg (d) NH4 because N2O
80. What is role of phosphate ion in a detergent? (a) is reative at high temperature
(a) It reduces pH of the water (b) dissociates to give O2 that supports combustion
Chapter 13: Group 15 (VA — The Nitrogen Family) | 579

(c) the activation energy is increased on increasing (a) N2O5 (b) NH3
temperature (c) N2O3 (d) HNO3
(d) acts as a catalyst 102. pit- pit multiple bonding between nitrogen atoms is
92. Nitric oxide is paramagnetic in present in
(a) gaseous state (b) liquid state (a) hyponitrous acid (b) nitrous acid
(c) solid state (d) polymeric state (c) nitric acid (d) in all of these
93. Nitrosonium ion (NO+) is isoelectronic with 103. In which case geometrical isomerism is not exhibited?
(a) carbon dioxide (b) carbon monoxide (a) Hyponitrous acid (b) 2-butene
(c) nitrogen dioxide (d) nitric oxide (c) 1-butene (d) 2-pentene
94. pit-pit multiple bonding is present in 104. Ambidentate ligands are
(a) oxides of nitrogen (b) oxides of phosphorus (a) NO2 (b) CN"
(c) halides of nitrogen (d) halides of phosphorus (c) CNS" (d) all of these
95. Paramagnetic species are 105. Mn04/H+ oxidises NO2 to
(a) NO,NO2,O2 (b) N2O2,N2O4,O3 (a) N2 (b) NH3
(c) NO+,NO2+,O2+ (d) O2,CO,CO2 (c) NO3 (d) N2O
96. With excess of Cl2 ammonia forms 106. A colourless salt gives a white ppt (soluble in
(a) ammonium chloride (b) nitrogen trichloride ammonium acetate) and a brown coloured pungent
gas on reaction with cone. E^SO^ Salt is
(c) nitrosyl chloride (d) nitrogen
(a) Ba(NO3)2 (b) Pb(NO3)2
97. The dipole moment of NF3 is less than that of NH3 (c) NaNO3 (d) NH4NO3
because 107. Nitration of aromatic compounds is done using a
(a) F is more reactive than H mixture of cone. HaSC^ and cone. HNO3. Intermediate
(b) NH3 forms associated molecules formed in the nitration process is
(c) the resultant of the individual bond polarities is (a) NO2 (nitronium ion)
less (b) NO3 (nitrate ion)
(d) the resultant of the individual bond polarities is (c) NO+ (nitrosonium ion)
opposed by the polarity of lone pair (d) NO2 (nitrite ion)
98. NH4C1(s) is heated in a test tube. Vapours are 108. NaNO2 and NaNO3 are used as food additives and
brought in contact with red litmus paper, which generally added to meat to
changes to blue and then to red. It is because of (a) retard oxidation of meat to brown coloured
(a) formation of NH40H and HC1 material
(b) formation of NH3 and HC1 (b) prevent growth of botulism bacteria
(c) greater diffusion of NH3 than HC1 (c) impart red colour to oxidised meat by NO by
(d) greater diffusion of HC1 than NH3 reaction of heme in blood
99. Select incorrect statement about hydrides of group 15 (d) make all of the functions given above effective
elements. 109. Nitrites in water may react with amines under acidic
(a) The central atom in the hydride is sp2 hybridised conditions in the stomach to
(b) NH3 readily form NH4 salts with H+; PH4 salts (a) give carcinogenic compounds
are formed with H+ under anhydrous condition (b) give proteins
(c) The tetrahedron is distorted due to repulsion (c) keep stomach acidity free
between the lone pair of electrons and the bond (d) produce free amino acids
pairs 110. HNO3 in pure state is colourless but it is often of
(d) The bond energy of the M—H bond decreases
yellow colour. It is due to
from NH3 to BiH3 because of increase in the size
(a) unstable structure of HNO3 which immediately
of the element
changes to NO?
100. In liquid NH3 (b) photochemical decomposition in presence of
(a) NH4C1 is an acid sunlight
(b) NaNHj is a base (c) interaction of atmospheric gases with HNO3
(c) CH3COOH behaves as strong acid (d) conversion of HN03 into NOJ
(d) all of the above facts are true 111. Among these ores the highest phosphorus content is
101. A gaseous substance dissolves in water giving a pale in
blue solution which decolorises KMnO4 and oxidises (a) chlorapatite (b) phosphorite
KI to Ij. Gaseous substances is (c) fluorspar (d) equal
580 | Essential Inorganic Chemistry

112. A person working with phosphorus suffers from a 123. PC15 +CH3CH2OH ---- >
disease in which bones decay. It is known as x y z
(a) arthrities (b) phossyjaw In the above reaction Cl replaces one of the H-atoms
(c) rickets (d) cancer in CH3CH2OH. This H is of
113. In the preparation of red phosphorus from white (a)C(inX) (b) C (in Y)
phosphorus (c) 0 (in Z) (d) any ofX, Y andZ
(a) MnO2 is used as a catalyst 124. The high reactivity and low volatility of white
(b) the white phosphorus is treated in an electric phosphorus is due to
furnace (a) tetrahedrally arranged P4 units
(c) a little iodine is used as catalyst
(b) bond angle of 60°
(d) the gas P4 is released (c) weak van der Waals’ force of attraction
114. Compound A undergoes hydrolysis to produce a (d) increased steric (strain) factor
colourless gas with rotten fish smell. The gas gives a 125. Stability of pentahalides is in order
vortex ring. The gas is (a) P > Sb > As > Bi for given X
(a) PH3 (b) P2O3 (b) F > Cl > Br for given element
(0 P2O5 (d) P2S5 (c) both (a) and (b)
115. One of the acid listed below is formed only from P2O3; (d) None of the above
the rest are formed from P2O5. Acid formed from P2O3
is One or More Than One Options Correct
(a) HPO3 (b) H4P2O7
(0 HgPO, (d) HgPOjj 1. N2H4 and H2O2 show similarity in
(a) hybridisation of central atoms
116. Consider the reaction
(b) oxidising nature
(NH4)2HAsO3+CuSO4 ■>CuHAsO3 +(NH4)2SO4 (c) reducing nature
green ppt (d) molar mass
The green precipitate is also known as 2. In acidic medium N2H4
(a) Pair’s green (b) Scheele’s green (a) is oxidised to N2
(c) Verdigris’ green (d) Rinmann’s green (b) loses four electrons
117. A substance which gives an yellow precipitate when (c) has equivalent mass 8
boiled with an excess of nitric acid and ammonium (d) reduces AuCl3 to AuCl
molybdate, and red precipitate with AgNO3 is 3. N2H4 reduces IO3/H+
(a) orthophosphate (b) pyrophosphate (a) tol+
(c) metaphosphate (d) hypophosphate (b) with 1^ as an intermediate indicated by violet
118. This salt of oxoacids of phosphorus is used as colour in CC14 layer
washing detergent (d) indicated by blue colour with starch
(a) wavellite (b) microcosmic salt (d) to!’
(c) calgon (d) chlorapatite 4. N2H4
119. Calgon (sodium metaphosphate) finds application as (a) is an endothermic compound
(a) artificial jewel (b) paint (b) burns in air with evolution of heat in an
(c) a resin (d) washing powder exothermic compound
120. With FeCl3, a neutral solution of orthophosphate (c) is kinetically stable
(d) reduces FeCl3 to FeCl2 in acidic medium
gives an yellow precipitate insoluble in
(a) CH3COOH (b) HC1 5. Thermodynamic tendency of PH3(#) to react with
(c) H2SO4 (d) NaOH HX(^) would be favoured by
(a) low H—X bond energy
121. Phosphorus trichloride, PC13, undergoes hydrolysis (b) high electron affinity of X
to produce an oxoacid. It has the formula (c) high lattice energy of PH4X(s)
(a) HPO3 (b) H3PO3 (d) high electronegativity of P
(c) H3PO4 (d) H3PO2 6. The following side reaction in the production ofN2H4
122. The solid PC15 exists as N2H4 +2NH2C1 ---- > N2 +2NH4C1
(a) PC13 (b) PC14+ (a) is catalysed by traces of heavy metals as Cu2+
(c) PClg (d) PC14+ andPClg (b) is suppressed by addition of gelatin or glue
Chapter 13: Group 15 (VA — The Nitrogen Family) | 581

(c) is made reversible by removing N2 2. Bond energy of


(d) is made reversible by adding NaOH (N=N)= 946 kJ mol'1
7. NH2OH can be used as (P=P)= 481 kJ mol-1
(a) on oxidising agent (N—N)= 160 kJ mol"1
(b) a reducing agent (P—P)= 215 kJ mol-1
(c) an autioxidant in photographic developers
which molecules do not exist?
(d) oxime forming reagent (with carbonyl compound)
(a) P4 (b) N4
8. Select correct statement(s). (c) N2 (d) P2
NHjOH forms syn and anti isomer with
3. Following table compares properties of red
(a) benzaldehyde (b) acetone
phosphorus and white phosphorus. Which is
(c) acetaldehyde (d) benzophenone
incorrect comparison?
9. Nylon-6 is formed from caprolactam which is formed
from Red White
(a) cyclohexanone and hydroxyl amine (a) Nontoxic and has polymeric Toxic, waxy discrete
(b) cyclohexanone and hydrazine structure tetrahedral P4 molecule
(c) benzophenone and hydrazine (b) Discrete tetrahedral P4 Polymeric structure
molecule
(d) benzophenone and hydroxylamine
(c) High m.p., less reactive Low m.p., more reactive
10. Select correct statement(s). (d) Shows no phosphorescence Shows phosphorescence in
(a) SbCl3 is hydrolysed to orange SbOCl soluble in in air air
dil.HCl
(b) BiCl3 is hydrolysed to white BiOCl soluble in 4. In P4 (tetrahedral)
dil. HC1
(a) each P is joined to four P
(c) PC15 can change toPClg andPCl4 with no change (b) each P is joined to three P
in hybridisation of P.
(c) each P is joined to two P
(d) NC15 and PC15 are stable compounds.
(d) P4 does not exist
5. High reactivity of white phosphorus is due to
Passage Comprehension Questions (a) unusual bonding that produces considerable
Passage 1 strain
Consider the six second and third-row elements in (b) high solubility in water
group 15-17 of the Periodic Table. Possible molecular (c) oxide layer made on it
structures for common allotropes of these elements are (d) all of the above
shown below: 6. White phosphorus reacts with NaOH forming PH3
and H>PO2. This is an example of
N OF (a) neutralisation
P Sd (b) thermal decomposition
(c) redox reaction
(d) disproportionation

Passage 2
Question given below is based on the following
structures:

0
,N
0
,N
h3c CH3 H3Sr SiH3
I II III ch3 SiH3
pyramidal planar
1. Following is the matching of the structure of
molecular form. Which is not the correct matching? 1. Structures ofN(CH3)3 andN(SiH3)3 are different. It
(a) Nitrogen I (b) Phosphorus II
(c) Sulphur III (d) Ozone I is due to the fact that
(a) silicon also uses d-orbitals in multiple bonding
582 | Essential Inorganic Chemistry

(b) in case of N(SiH3)3, lone pair of N-atom (a) 6 + 2x (b) 6-2x


is transferred to the empty d-orbitals of silicon (c) 6-x (d) 6 + x
(pn-dit overlapping)
(c) both of the above
(d) none of the above
Passage 5
The following flow diagram represents the industrial
preparation of nitric acid from ammonia:
Passage 3 (A) (T>\
NH3 + O2 ----- -> NO -^4 (C) ------- > HN03+N0
Consider following scheme involving oxides and (excess 900°C * air water
oxy-acids of nitrogen. Based on this answer the questions air) [___________
given at the end.
Answer the questions given below
hno3 no3”
1. Which line of entry describes the undefined reagents,
n2o5 (2) (4)
products and reaction conditions?
(1)
(3) A B C
(9) / no2 ^=r n2o4 HNO;2 (a) Catalyst cool (- 25°C) no 2
^^(10) x>^(8) (b) Catalyst cool (- 25°C) n20
(7? (5), (6)
n2 — NO (c) Catalyst high pressure NO,
(11)
^(12) N2O3
(14) (d) High pressure catalyst
(13) nh3
2. Formation of HNO3 when (C) is dissolved in H,0
1. Out of the following which reactions are takes place through various reactions. Select the
disproportionations? reaction not observed in this step.
(a) 2,10 1 (b) 2, 3 (a) NO2 + H2O---- > HNO3 + HNO2
(c) 1,11 i (d) 13,14 (b) HNO2 ---- > HjO + NO + NO2
2. In which case change in oxidation number is (c) NO2 + H,0---- > HNO3 + NO
maximum? (d) None of the above
(a) 1 (b) 10
(c) 5 (d) 11 Passage 6
Trimethylamine oxide and trimethyl phosphine oxide
Passage 4 can be represented as
Questions given below are based on the following (CH3)3P-O
(CH3)3N-O-
experiment. (I) (ID
An oxyacid of phosphorus has the following properties
Complete neutralisation of the acid with sodium Answer the following questions based on these
hydroxide solution gives an aqueous solution of sodium 1. Select correct structure.
ions and oxy-acid anions in the ratio 2 :1. When a solution (a) (CH3)3N=O (b) (CH3)3P=O
of the acid is warmed with silver nitrate solution metallic (c) both (a) and (b) (d) None of these
silver is deposited. 2. Trimethyl phosphine oxide can be represented by two
1. What is the structure of the oxy-acid? canonical forms but trimethylamine oxide can’t be
OH OH H OH (CH3)3P-O <—> (CH3)3P=O
HO—P—OH H—P—OH HO—P—»OHO—P—>0 It is due to
X
O
I I (a) phosphorus accepts charge density from oxygen
H OH and accommodates it in its vacant d-orbitals
1 2 3 4 (b) nitrogen does not possess d-orbitals and thus
(a) 1 (b) 2 can’t from the 7t-bonded structure
(c) 3 (d) 4 (c) both (a) and (b)
(d) None forms canonical forms
2. This oxy-acid is converted into . on reaction
with acetic anhydride 3. Trimethyl phosphine oxide is stable because
(a) (CH3CO)2HPO4 (b) (CH3CO)2HPO3 (a) it has two resonating structure
(c) P2O3 (d) P2O5 (b) phosphorus-oxygen double bond has high
dissociation energy
3. pH of 0.01 M solution of the sodium salt of this acid (c) both (a) and (b)
formed (assume pKai = 2x) is (d) None of the above
Chapter 13: Group 15 (VA — The Nitrogen Family) | 583

Passage 7 5. Assertion (A): N2H4 is a weaker base than NH3.


N2H4 reduces IO3 to I+ in acidic medium • Reason (R): Its behaviour is bifunctional.
N2H4 +IO3 +H+ ---- > N2 +I+ + ^0 6. Assertion (A) : Equivalent mass of N2H4 is 8 in
Answer the following questions acidic medium or basic medium.
1. Equialent mass of N2H4 (molar mass = 42 g mol-1) is Reason (R) : It is oxidised to N2 with loss of four
(a) 42 (b) 21 electron per N2H4 unit.
(c) 10.5 (d) 14 7. Assertion (A) : IO3 is reduced to I+ but
2. Equivalent mass ofKIO3 (molar mass =214 gmoP1) intermediate gives violet colour in CC14 (taken as
is medium).
(a) 71.33 (b) 107.0 Reason (R): is formed as the intermediate.
(c) 214.0 (d) 53.5 8. Assertion (A) : Each N-atom is sp3-hybridised in
3. Intermediate in the above reaction is which can be N2H4.
detected by Reason (R): It has gauche conformation.
(a) taking solvent CC14 in which Lj is soluble and
9. Assertion (A) : and N2H4 have oxidant and
thus layer becomes violet
(b) starch as indicator which turns blue reductant nature.
(c) both (a) and (b) Reason (R): Both have equivalent mass of 8.
(d) None of the above 10. Assertion (A) : N3H is a weak acid and NH3 is a
4. N2H4 is oxidised to N2. Both N-atom in N2 are from weak base.
N2H4. It can be verified by Reason (R) : There is proton-donor acceptor
(a) taking 15N enriched N2H4, equilibrium
(b) taking2 D enriched N2H4 NgH+NH3 N3 +NH4+
(c) taking both (a) & (b)
(d) taking none of the above
True & False
Assertion & Reason 1. H3PO2 is a weak monobasic acid.
Codes: 2. N3H is a stronger acid than acetic acid.
(a) Both A and R are true and R is the correct 3. NgH is thermodynamically stable.
explanation of A.
(b) Both A and R are true but R is not the correct 4. N3 has linear structure.
explanation of A. 5. N2H4 is thermodynamically endothermic compound
(c) A is true but R is false.
but is kinetically stable.
(d) A is false but R is true.
1. Assertion (A): On cooling, the brown colour of 6. P4O10 changes malonic acid into CO2.
NO2 disappears. 7. NaHjPO-j andNaHPO3 are acid salts.
Reason (R) : On cooling NO2 forms dimer (N2O4)
8. NagPO4 andNaHPO3 are normal salts.
resulting in the pairing of the odd electrons in NO2.
2. Assertion (A) : Both HgPO3 and HgPO4 have the 9. NH3,PH3 and AsH3 have sp3-hybridised central atom
same number of hydrogen atoms but H3PO4 is a and thus each has bond angle of 109°28'.
tribasic acid and H3PO3 is a dibasic acid. 10. NH4C1, NH4NO3 and (NH4)2CO3 give NH3 gas on
Reason (R) : 1 mole of H^Og is neutralised by heating.
2 moles of NaOH while 1 mole ofHgPO4 is neutralised 11. N2O resembles molecular O2 since, it supports
by 3 moles of NaOH.
combustion.
3. Assertion (A) : N2H4 is kinetically stable
12. Structure of N2O is similar to that of CO2.
compound.
Reason (R) : Two N-atoms are bonded by triple 13. N—0 bond length in N2O4 is greater than expected
bonds. due to charge repulsion.
4. Assertion (A) : N3H explodes violently on slight 14. H2N2O2 exists as cis and trans geometrical isomer.
heating.
15. NaNO2 and NaNO3 are used as food additives and
Reason (R): N3 has linear structure and is stable
due to resonance. generally added to meat.
584 | Essential Inorganic Chemistry

Fill in the Blanks 2. Match the reactions of metals with dilute HNOj
1. When mixture of NH4C1 and NaN02 is heated, (in List I) with the nitrogen compounds (obtained by
gaseous product formed is oxidation/reduction (in List II).
2. Formation of N2H4 is an reaction but List I List II
combustion is an reaction.
A. Mg + dil. HNO3 1. NO
3. Out of NO, NO2, N2O4, N2O2, paramagnetic species
B. Zn + dil. HNO3 2. H2
are
4. N2H4 is C. Sn + dil. HNO3 3. N2O
stable compound.
D. Pb + dil. HNO3 4. NH4NO3
5. N2H44 is a base than NH3 and NgH is a
acid than CH3COOH. 3. Match the compounds (in List I) with their oxidation
6. N2H4 is oxidised to in acidic medium and to number of N (in List II).
in basic medium.
List I List II
7. N3 is stable due to
8. HgPO4 is a A. N2H4
acid,
HgPOa is a B. NH2OH 2. 3
acid and
HgPO2 is a acid C. HNO2 3. 4

9. Reducing nature of HgPO4, HaPC^ and HaPOa is in D. NO2 4. 5


order E. HNO3 5. -2
10. N2O, NO are AA 4O
s4sO106,are
Bi2O3 is a
11. Out of PC13, PC15, NC15 and NC13, least stable is II. One or More Correct
1. Match the reaction in Column I with their
12. As3+ salts are white Bi5+ salts are corresponding characteristic(s) in Column II.
13. IfMC13 is hydrolysed and there is an orange turbidity Column I Column II
then MC13 is and when there is white
turbidity then MC13 is A. 3H3PO2 2H3PO3 + PH3 1. Disproportion­
ation
14. N3 resembles and is also called
B. 2NO2 + H2O---- > HNO3 + HNO2 2. Reducing nature
15. N2H4 can be prepared by of any reactant

C. H3PO4 + Ca(OH)2 —» CaHPO4 3. Dibasic acid


Matrix-Match Type Questions + 2H2O
D. 2NH2OH + 4CuO---- > Cu2O i 4. Equivalent weight
I. Only One Correct (reactant)
+ N2O + 3H2O
mol, wt
1. Match the acids (in List I) with number of acid salts
4
formed by them (in List II).
5. Reactant is
List I List II paramagnetic
A. Hypophosphorus acid 1. Two
6. Oxidising nature
B. Orthophosphorus acid 2. Zero of any reactant
C. Orthophosphoric acid 3. One
D. Mellitic acid 4. Five
Chapter 13: Group 15 (VA — The Nitrogen Family) | 585

2. Match the half-reactions in Column I with the Set I


corresponding characteristic(s) in Column II. 1. How many of the species are isoelectronic?
Column I Column II N2O, CO2, CN2“, Ng, NHjOH, N3H
A. N2H+5 ---- > N2 + 5H* 1. Oxidation 2. How many of the species are paramagnetic?
N2O, NO, NO2,02, NO2+, NO+, CO+
B. N2H< + OH"---- > N2 + H2O 2. Reduction
C. 2N3 ---- > 3N2 3. Equivalent weight 3. In the following, equilibrium partial pressure ofNH3,
molar mass N2 and Hj gases are 4, 1 and 2 atm respectively at
4 300 K. What is value of Kp ?
D. NH2OH —> NH3 4. Equivalent weight ’ N2(g) +3112(g) 2NH3(g)
molar mass
2 4. In the oxidation of N2H4 to N2, equivalent weight of
E NH2OH---- > N2O 5. Exchange of 4 N2H4 would be
electrons 5. Total number of o bonds in N3H is
6. Exchange of 2
electrons 6. NO3 is reduced to NH4. What is change in oxidation
number?
3. Match the species in Column I with their
7. A phosphate mineral has formula CaJ^O^F. Thus x
characteristic(s) in Column II.
is
Column I Column II
8. How many species out of the following have reducing
A. NH2OH 1. Reducing agent properties?
B. N3H 2. Oxidising agent NH3, PH3, H3PO2, HgPO3, HjjPO, , LiAlH4, BH3
C. NH2—NH.2 3. Weak base
D. NH3 4. Molecule or anion with linear Set I!
structure 1. How many POP bonds are there in P4O10 ?
E. N2O 5. Isoelectronic of CO2
2. How many P—H bonds are there in HgPO2 ?
3. In PC15, how many (P—Cl) are inclined at 120° with
Integer Answer Types one each.
This section contains 8 questions. The answer to each
of the questions is a single digit integer, ranging from 0 to 4. 25.0 mL of 0.05 M solution ofNHgOH was boiled with
9. The appropriate bubbles below the respectively excess of Fe3+ in acidic medium. The Fe2+ formed
question numbers in the ORS have to be darkened. For required 25.0 mL of 0.1 N Ci^O^" in acidic medium.
example, if the correct answers to question number X, Y, Z What is oxidation number of the N in new product?
and W (say) are 6, 0, 9 and 2, respectively, then the correct 5. What is pH of 0.01 M Na2HPO4 if pKa is 6?
darkening of bubbles will like the following :
6. Consider following reactions :
X Y Z W
hno3
© © © © nh2oh hno2
© © O ©
© © © N2O5- no2 * n2o4
© © © © H3N
© © © NO
© © © © N2H4N2
© © © ©
In how many reactions NO2 has been oxidised?
© © ©. ©.
© © © © 7. In how many reactions NO2 has been reduced in Q. 7?
© © © © 8. Maximum change in oxidation number of nitrogen is
Answers & Solutions
Master Exercises
Exercise 1
Short Answer Type Questions 16. In P4O6 : 12 (P—0) bonds '
Refer structure
1. (a) P (b) Sb, Bi (c) N (d) Bi In P4O10 : 16 (P—0) bonds.
2. :JN:;N%
n::n:» N2 is unreactive since, bond energy is very 17. It is P04~ (phosphate) ion.
high. H3PO4 + 21HNO3 + 12(NH4)2MoO4 ---- >
3. [Fe(H2O)5NO]2+ (NH4)3PO4 • 12MoO3 +21NH4NO3 + 12H2C
4. N3(ag)+ 3H2O(Z)---- > NH3(g)+ 30H-(ag)
N3 accepts proton hence, it is a strong Bronsted Lowry
18. HO—Pf
^0
z metaphosphoric acid
base and reacts with water to produce NH3 and OH"
ions. H0\z° zPx
5. 02 6. 4Mg + 2NO2 ---- > 4MgO + N2
0 0

K
7. Hydrocarbons are oxidised into C02 and H20 in the 0, OH trimetaphosphoric acid
presence of an oxidation catalyst (Pt or Pd) and NO is P
HO
reduced to N2 by reduction catalyst. Catalytic converter O
has dual catalyst system.
<yi, + (x + ^o, 19. H4P2O7 has two units of H3PO4 joined by (—0—) formed
xCO2 + -H,0
2 2 2 2 due to loss of water :
0 0 0 0
2N0+ 2C0---- > N2+ 2CO2 I I
I
HO—P—O-kH H0}-P—OH
X I HO—P—0—P—OH
8. (A):NH4NO3 (B):NH3 (C): (NaN03 + NaOH)
(D): N20 (E): H20 OH OH OH OH
9. Ca(HCO3)2 + 2NH3 ---- > CaCO3 X + (NH4)2CO3 20. In silicate and phosphate anions both Si and P are
white ppt
surrounded by tetrahedra of O-atoms, which can link
10. It is an acid-base reaction in terms of Bronsted Lowry together to form chains and rings.
protonic concept
0
nh3 + NH3 NH- + NH4
Acid (I) Base (II) Base (I) Acid (II) /Si.
0 \ ^0
11. (A):nh4no'2. (B):nh4no3 (O:n2 (D):ain
0
(E): n2o
h9o
12. (a) 2HNO3 ---- > H20 + 2N0 + 3(0) 21. P4(s)+ 5O2(g)---- > P4O10(s) H^O,
2NO + O2 ---- > 2NO2 22. It is
JO without CaSO. hence
.. richer in P-content.
(b) 4HNO3 ---- > 4NO2 + 02 + 2H2O
23. NH2CONH2 + 2H2O (NH4)2CO3
(c) 3Ag + 4H+ + N03 ---- > 3Ag+ + NO + 2H2O
(d) N^ + 2I2 ---- > N2 + 4H+ + 4F 24. (b) maximum % of P
13. AsjjSg + 10HN03 ---- > 2H3AsO4 + 10NO2 + 3S + 2H2O
Analytical Questions
14. Bi2O3 < Sb2O3 < As2O3 < P2O3 < N2O3
1. Refer text
15. “3
ch TT — nh2
3 —c
---- > CH3CN + H2O 2. (a) Tetrahedral (b) Planar
(c) Pyramidal at N and bent at 0 (d) Tetrahedral
0
(e) Trigonal bipyramidal with apical Cl atom.
POC13 + 3H2O H3?O4 + 3HC1
Chapter 13: Group 15 (VA — The Nitrogen Family) | 587

3. Let (NH4)2HPO4=x% 4. BigS-j remains undissolved

NH4NO3 =(100-x)% As^ +3Na2S - 2Na3[AsS3]


OQt
soluble
Nitrogen in (NH4)2HPO4 = — Sb^ + 3Na.>S 2Na3[SbS3]
JLoZ
soluble
28(100-x)
Nitrogen in NH4NO3 = N-O-H H ^0
80 N— N
5.
28x 28(100-x) N-O—H H 0
-------- F = 33.81
132 80 cis hyponitrous acid nitramide
(cis and trans)
x 100-x
132 + 80 = 1.2075 ^0
6. H—0— N H—0—0—N=O
x = 8.63%
0
(NH4)2HPO4 = 8.63% nitric acid peroxo nitrous acid

NH4NO3 = 91.37%
NCERT Problems
Refer text

Exercise 2
Only One Option Correct
1. (c) 2. (d) 3. (d) 4. (b) 5. (a) 6. (d) 7. (b) 8. (b) 9. (c) 10. (c)
11. (d) 12. (c) 13. (c) 14. (c) 15. (c) 16. (c) 17. (d) 18. (d) 19. (a) 20. (d)
21. (c) 22. (c) 23. (a) 24. (d) 25. (c) 26. (d) 27. (d) 28. (c) 29. (d) 30. (a)
31. (d) 32. (d) 33. (b) 34. (b) 35. (a) 36. (d) 37. (b) 38. (a) 39. (c) 40. (d)
41. (c) 42. (d) 43. (a) 44. (a) 45. (d) 46. (a) V 47. (b) 48. (a) 49. (d) 50. (c)
56. (b) ‘ 57. (c) 58. (d) 59. (b) 60. (d)
51. (a) 52. (d) 53. (a) 54. (b) 55. (d)
61. (a) 62. (b) 63. (b) 64. (b) 65. (c) 66. (c) 67. (b) 68. (c) 69. (c) 70. (a)
71, (d) 72. (d) 73. (a) 74. (a) 75. (c) 76. (d) 77. (a) 78. (c) 79. (d) 80. (d)
81. (c) 82. (d) 83. (a) 84. (d) 85. (b) 86. (b) 87. (c) 88. (a) 89. (c) 90. (d)
91. (b) 92. (a) 93. (b) 94. (a) 95. (a) 96. (b) 97. (d) 98. (c) 99. (a) 100. (d)
101. (c) 102. (a) 103. (c) 104. (d) 105. (c) 106. (b) 107. (a) 108. (d) 109. (a) 110. (b)
111. (b) 112. (b) 113. (c) 114. (a) 115. (d) 116. (b) 117. (a) 118. (c) 119. (d) 120. (a)
121. (b) 122. (d) 123. (c) 124. (d) 125. (c)

One or More Than One Options Correct Passage 4


1. (a,b,c) 2. (a,b,c) 3. (a,b,c) 4. (a,b,c,d) 1. (b) 2. (c)
5. (a,b,c) 6. (a,b) 7. (a,b,c,d) 8. (a,c) 9. (a) - , 1QgC
3. (d) pH = 7+^-
10. (a,b) 2 2
= 7 + —-l = (6 + x)
2
Passage Comprehension Questions
Passage 5
Passage 1
6. (d) 1. (a) 2. (d)
1. (d) 2. (b,d) 3. (b) 4. (b) 5. (a)
Passage 6
Passage 2
1. (b) 2. (a) 3. (c)
1. (0
Passage 7
Passage 3
1. (c) 2. (d) 3. (c) 4. (a)
1- (a,c) 2. (d)
588 | Essential Inorganic Chemistry

Assertion & Reason 2. (A)-(1,3,5); (B) —(1,3,5); (C) — (1,4,6);

1. (b) 2. (b) 3. (c) 4. (b) 5. (b) 6. (a) (D) — (2,4,6); (E) —(1,4,6)

7. (a) 8. (b) 10. (a) 3. (A) —(1,2,3); (B) —(1,4,5); (O- (1,2);
9. (c)
(D) — (1,3); (E) —(1,2,4,5);
True & False
1. T 2. T 3. F 4. T 5. T 6. F 7. F Integer Answer Types
9. F 10. F 11. T 12. T 13. F 14. 'T Set I
8. T
15. T Questions—► 1 2 3 4 5 2 7_ 8
Answers —► 5 4 2 8 3 8 5 6
Fill in the Blanks © ©
© ©I©
© © ©
2. endothermic, exothermic 3. NO, NO2
©. ©
0 © ©
1. n2
© ©
© © (D
© ©
4. kinetically 5. weaker, stronger 6. N2, b<2 © ©
© ©®
© © ©
7. resonance 8. tribasic, dibasic, monobasic
© ©
© © ©
© ©
© © © ©
9. H3PO4<H3PO3<H3PO2 © © ©._©
® © ©
©
10. acidic, amphoteric, basic 0 0.0
12. reducing agents, oxidising agents © © © ® © ©
11. NC15
© © © © © © © ©
13. SbCl3,BiCl3 14. halide, pseudo halide
Set II
15. Raschig method
Questions —► 1 2 3 4 5 £ 7 _8_
Answers —► 6 2 3 1 9 2 6 7
Matrix-Match Type Questions © ©I© ©.
© © © ©
I. Only One Correct © © ©
1. (A)-(2); (B)-(3); (C)-(l); (D) —(4) © © © © © @^_© ©.
2. (A)-(2); (B) — (3); (C)-(4); (D)-(l); © © © © © ©>®
© © © © © ©_
3. (A)-(5); (B)-(l); (C) — (2);
© © © © © ©__© ©_
(D)-(3); (E) —(4); © © © © © © ©
®
II. One or More Correct 0 0 0 0
1. (A) —(1,2,6); (B)-(1,2,5,6); (O —(3); © © © © © © © ©_
(D)-(2,4,6) © © © © © ©I© ©.
■fl 4
Group 16
(VIA—The Chalcogens)
"I am king" of acids. I can dehydrate every thing even skin. I
can cause boiling of water with violent splashing of added
into my concentrated form" Guess me.

Oxygen is the most abundant element in the earth’s crust,


representing slightly less than 50% of it by weight. It is present ea <
X.

as elemental oxygen in the atmosphere and is combined with


other elements in water and in many minerals. Sulphur,
I laptei
seventeenth in abundance in the earth’s crust occurs as
sulphide (HgS, PbS etc) in natural gas and oil, as disulphide ion ■ Periodicity in Group 16
(FeS2 —“fools’ gold”) as sulphate (CaSO4) (VIA—The Chalogens)
Selenium and tellurium are comparatively rare on earth’s ■ Ozone (03)
crust, having abundance as selenide/telluride of Cu, Ag, Fe, As.
■ Allotropy and Polymorphism of Sulphur
■ Study of Compounds of Group 16 (VIA)
Periodicity in Group 16
(VIA—The Chalcogens)
• The first four elements are called chalcogens meaning ore
forming (oxides, sulphides, selenides, tellurides).
• Some representative properties of group 16 (VI A) elements
are given in Table 14.1 and Fig. 14.1.
Similar to the groups discussed earlier, the metallic
character in group 16 increases with increasing atomic number.
Oxygen and sulphur are entirely non-metallic in their chemical
behaviour. Selenium and tellurium, though essentially
non-metallic, assume increasing metallic character and are
termed as metalloids. Polonium is most metallic in the group.
Density, melting and boiling points covalent and ionic radii,
show a regular increase from oxygen to polonium. The large
difference in melting and boiling points between oxygen and
sulphur can be explained on the basis of their structure. Oxygen
exists mostly as a diatomic molecule held together by weak
590 | Essentia/ Inorganic Chemistry

van der Waals’ forces while others exist as polyatomic view of the fall in electronegativity, metallic
molecules, e.g., S8, Se8, etc., where the atoms are bonded character within the group increases with
by covalent bonds associated with high dissociation increasing atomic size.
energy. Their existence as diatomic and polyatomic Oxygen, the second most electronegative element,
molecules can be explained as follows: The 0=0 bond fluorine being the first, has a strong tendency to accept
more than three times as strong as the O—O bond. By two electrons and give O2- ion. Thus, almost all metal
comparison the S=S bond is less than twice as strong as oxide are ionic and contain O2- ions.
the S—S single bond. This results in catenated r- -
—O—O—0— chains begining unstable relative to 0=0; DEEP Focus
but catenated —S—S—S— chains being stable relative to
Usually oxygen exhibits as oxidation state -2 in its other
the molecule S=S. compounds also. It exhibits positive oxidation state only
Bond Bond energy kJ mol in a few compounds formed with fluorine, i.e., 0F2 and
0=0 498 O2F2. The tendency for the formation of divalent anions
0—0 142 decreases from sulphur downwards because of the increasing
s=s 431 size and decreasing electronegativity of the elements. Sulphur,
s—s 265 selenium an tellurium show a tendency for covalency with
formal oxidation states +2, +4 and +6 in compounds in which
The elements of group 16 are characterised by high they are combined with more electronegative elements such as
ionisation energies. (Fig 14.1) decreasing gradually from oxygen and halogens. In the higher oxidation state of +4 and +$
oxygen to polonium. The high values indicate reluctance of of these elements electrons are unpaired and promoted to
vacant d-orbitals.
these elements to from cations. Their electronegativities
decrease with increasing atomic number. This, is

Table 14.1 Physical Properties of the Element in Group 16 (VIA)


Qa S,,I6______ _______ $e34_____ ______ Te52______ _____ P°M____
Electronic configuration [He] 2s22p4 [Ne] 3s23p4 [Ar] 3d104s24p4 [Kr] 4d'°5s25p4 [Xe]4/,45d106s26p4
Covalent radius (pm) 74 104 114 . 137 168
Ionic radius (M2~, pm) 140 184 198 221
(/E),/kJ mol-1 1314 999 941 869 813
Electronegativity 3.5 2.5 2.4 2.1 2.0
Melting point (°C) -229 114' 221“ 452 254
Boiling point (°C) -183 445 685 1087 962
Density (103 xkg m"3) 1.14 2.07 4.79 6.25 9.4
Oxidation states -2, -1, +1,+2 -2, +2, +4, +6 -2, +2, +4, +6, -2, +2, +4, +6 +2. +4
Electron affinity (kJ mol-1) -141 -200 -195 -190 -183
E7Vfor 1.23 0.14 -0.40 -0.79
X + 2H+ + 2e~ -*H2X

*Rhombic sulphur, a-Grey Se


1250-1 1263 4-r 1500-r
1050- C 1235
* 958 b.p.
3--
£ 850- 1000----
□ Pauling
718 723 units
2 650- 2-- kJ mol-1
8. Z\m.p.
| 450- / >490 \
//392GrBV 527 500--
1--
250- / Monoclinic
90
55
50 i i i l 0 0
O^s’lSel Te1 Po1 O ' S 'Se’Te'Po1
0 S Se Te Po
(a) (b) (c)
Fig. 14.1 Some group trends of the group 16 elements : (a) Boiling point and melting point,
(b) Pauling electronegativity, and (c) The first ionisation energy
Chapter 14: Group 16 (VIA—The Chalcogens) | 591

DEEP Focus Oxidation States


• One of the points about oxygen is that as the first member of • With valence electronic configuration ns2npi, the
a group it differs in some important ways from other group group 16 (VI A) elements are just two electron short of
members due to: an octet configuration, and -2 oxidation state is
~ small size, therefore, a common one. The stability of -2 state
“ high electronegativity, and decreases with increasing metallic character as
~ non-availability of d-orbitals for bonding. indicated by E° (redox) values. Oxygen is powerful
Table 14.2 summarises similarities and differences oxidising agent and l^Se and I^Te are reducing
between oxygen and sulphur. agents (E° values of Se and Te are negative). Because
S, Se, and Te are much less electronegative than
Table 14.2 Comparison of Oxygen and Sulphur oxygen, they are commonly found in positive oxidation
states as +4 in SF4, S02 and H^SOg and +6 in SF6, S03,
Oxygen Sulphur H2SO4.
• Exists as 02(g) at 298 K and • Exist as Se(s) rings at 298 K • Unlike oxygen, the maximum coordination number of
1 atm and 1 atm. At higher sulphur and other elements can exceed four and
temperature S2 predominates. valency is not limited to two because of available
• Two allotropes, 02(g) and 03(g) • Two solid polymorphic forms d-orbitals for bonding. Stability of +6 state decreases
and many different molecular and that of +4 state increases going down the group
species in liquid and gaseous
states. due to inert-pair effect. It is:
•02(9) and 03(g) are very good • S(s) is a poor oxidising agent, Se4+ < Te4+ < Po4 +
oxidising agents. Se6+ > Te6+ > Po6+
• Metallic oxides are generally • Metallic sulphides are ionic
ionic. (Na2S, K2S...) as well as
covalent insoluble in water Hydrides
(CuS, NiS, CdS... generally of • H^Se and H.2Te are unpleasant gaseous hydrides like
d-block) HoS but less stable than HgS, order of thermal stability
• 02 paramagnetic • S2 paramagnetic being H20 > H2S > HjSe > HgTe > H2Po.
• Can form only two and three • Can form molecules with up to • Oxyacids of S, Se and Te exist and HoSO^ and H2SeO4
atom chains as in H2O2 and 03. six S atoms per chain in
Compounds with 0—0 bonds compounds such as H2Sn. are considered as strong acids.
decompose readily. Na2S„,H2SnO6.
• Maximum coordination number • Maximum coordination Catenation
of oxygen is four, as in water: number of sulphur is six, as in • The tendency for catenation decreases markedly as we
H-O-H SF6:
go down the group. This property is prominently
F displayed by sulphur (S8 ).The S—S bond is important
-H
H'" F\ I in biological systems and is found in compounds such
F—S— F
H as cysteine, some proteins and enzymes. Stable
0—H F polyoxides and polysulphides are known. Examples
F
are:
•H20: • H2S
H—0—O—H, H—S—S—H, H—S—S—S—H,
- is a liquid at 298 K and -is a (poisonous) gas at
1 atm 298 K and 1 atm
-is extensively hydrogen - is not hydrogen bonded
bonded - has a small dipole Metallic Nature
- has a large dipole moment moment • The metallic character increases as we descend the
- is a good solvent - is a poor solvent group. S is non-metal and insulator. Se and Te are
- is oxidised with difficulty - forms no complexes metalloids an are semiconductors. Po is a metal.
-forms hydrated and aqua - is easily oxidised.
complexes.
Multiple Bonding
• Selenium has unique property of photoconductivity • The tendency to form multiple bonds to C, N and 0
and is used in photocopying (xerox) machines and also decreases as we descend the group from Se to Te. Thus
a decoloriser of glass. Te and Po are highly toxic, Po is S=C=S is moderately stable; Se=C=Se
higher due to its intense radioactivity. Se and Te are decomposes readily whereas Te=C=Te does not
found as selenides and tellurides in sulphide ores exist.
(Ci^Se, Ch^Te). The colour of the red glass in the • Acidic nature of hydride increases as we go down the
group:
traffic signals is due to CdSe.
H20 < HoS < HjSe < H2Te < H2Po
592 | Essential Inorganic Chemistry

Halides • Some 30 km above the earth’s surface oxyge-


molecules can be split apart by UV light from the sur
• The form halides of different oxidation number +1, +2, Some of the atoms join with other oxygen molecules t-
+4 and +6. make ozone :
Halides X Oxidation numbers 02(g) + 0(g) ---- > 03(g)
S2X2 F. Cl, Br, I +1 • 03 is an unstable, dark blue diamagnetic gas, boilin.
sx2 F. Cl +2 point - 112°C. The colour is due to intense absorptioi
sx4 F +4 of red light. It also absorbs strongly in the UV region
This is particularly important since there is a layer o
sx6 F +6
03 in the upper atmosphere which absorbs harmful
Se2X2 Cl. Br +1 radiation from the sun, thus protecting people on th.
SeX2,TeX2 Cl, Br +2 earth. The use of chlorofluorocarbons (CFO ifi
SeX4.TeX4 F. Cl. Br +4 aerosols and refrigerators, and their subsequen
TeX4 I +4 escape into the atmosphere, is blamed for making hole:
SeX6.TeX6 F +6 in the ozone layer over the Antarctic and Arctic. It l
feared that this will allow an excessive amount of
The stablity of the halides decreases in the order • light to reach the earth which will cause melanomz
F > Cl > Br > I. +6 oxidation is observed in fluoride, while (skin cancer) in humans. Oxides of nitrogen (from cai
iodide of TeCTel^) is formed. exhausts) and the halogens can also damage the 0.
layer.
SF6 is thermodynamically inert, non-toxic gas. Ozone destroying reactions:
Interness of SF6 is due to the presence of sterically — based on oxides of nitrogen
protected sulphur atom. SF6 is used as a gaseous insulator
in high voltage generators. SF4 an SeF4 are good NO + 03 ---- > N02 + 02
fluorinating agents and —COOH group can be converted 03 +/iv —> 02 + 0
into CF3 and ^>C=0 and —P=0 into CF2 and —PF2 N02 +0 —> no + o2
groups. Net reaction : 2O3 hv 3O2
• Oxides of sulphur are more stable than the
corresponding oxides of other elements. - based on reactive chlorine species from CFC
C10 + 0
Cl + 03 ---- ■»> CIO 0;2
Reactions 03 + hv -> 0 + 02
Reactions of the group 16 (VIA) elements are C10 + 0 -> Cl + O2
summarised below: 203o+hv ---- > 3O2
Net reaction : 2O
General equations Remarks
• 03 is thermodynamically unstable, and decomposes te
02:
xE + yM ---- > With many metals 2O3 ---- > 3O2, AG = -163 kJ mol'!
zE + MXEV ---- > MxEv ., Especially with S and Se
E + H2 ---- > H2E Decreasing in the series 02, S, Se.
• 03 is an extremely powerful oxidising agent:
Te O3+2H++2e" O2+H2O
E + 3F2 ---- » EFe With S, Se, Te and excess F2 03 + H20 + 2e~ 02 + 20H"
2E + Cl2 ---- > E2CI2 With S, Se (Te gives TeCI2); also
with Br2 3PbS + 4O3 > 3PbSO4
E2CI2 + Cl2 ---- > 2ECI2 With S, Se, also with Br2 6NO2 + 03 > 3N2O5
E + 2CI2 ---- > ECI4 With S, Se, Te and excess Cl2; also S + HgO + 03 ---- > H2SO4
with Br2
2K0H + 5O3 ---- > 2KO3 + 5O2 + H>0
E + 02 ---- * E02 With S (with Se, useO2 and NOZ)
O3+2KI + H2O---- > 1.+2KOH + 02
(E = 0, S, Se, Te, Po and M metal). 3SnCl2+6HC1+O2 ---- » 3SnCl4+3H2O
2[Fe(CN)6]4- + H2O + 03--- > 2[Fe(CN)6]3’ + 02 + 20H
Ozone(03) • Potassium ozonide K03 is an orange colours
• In the laboratory, 03 can be made by passing oxygen solid and contains the paramagnetic 03 ion.
through a strong electric field. An equilibrium is set
up: DEEP Focus
3O2(s) 2O3(g) • The amount of 0? in a gas mixture can be determined bj
It is only a metastable allotrope, always having a passing the gas into a KI solution (at a constant pH 9^
tendency to convert back into oxygen. using borate buffer). The iodine that is liberated is titrate
with sodium thiosulphate solution
Chapter 14 . Group 16 (VIA—The Chalcogens) | 593

O3^I2=2I=2S2O23-
• 03 can also be decomposed catalytically and change in
t, . 1 * • r/A molecular weight
volume measured : Equivalent weight of 03 =-------------------------
2
2O3 ----- > 3O2
kcio3
(2 volume) (3 volume)
• 03 also adds to unsaturated organic compounds at A I MnO2
room temperature forming ozonides which can be electrolysis
cleaved to aldehydes and ketones in solution : Air o2 h2o

Z°\CH,
CH, H20/Zn» 2HCH0
strong
electric field
CH^CH, +O3 ■>

ch2=ch2 __JA---
0---0 HCHO O3 (HCI + SnCI2)
H2O/Zn A^N0^SnCI<
H20/Zn [Fe(CN)6f>^'
CH3—CH=CH—CH3 + 03 —------ > 2CH3CHO <es(f^ay^O2 + NO2
PbS \ y'n9r^ctiOns
[Fe(CN)6]3- / s h2o no2'
DEEP Focus / t
AXkOH
To derive products of ozonolysis, break (C=C)bond, place PbSO4 H2SO4 N2O5
0 at breaking point, h2so4 ko3
break ? CH,CH0 + CH_CH0
CH,CH=f=CHCH, O d The molecules 03 is bent with an 0—0—0 angle
O 1 v
Place 0
116.8° and equal 0—0 distance of 128 pm and may be
• 03 is used as a disinfectant; it is used to purify described 90° canonical forms.
drinking water since, it can destroy bacteria and
viruses. It is better than Cl2 since, it (03) avoids the
unpleasant smell and taste of Cl2 and any excess 03
soon decomposes to 02.
O3 + 21 + H20 + la + 20H" + 02
0 ------
218 pm
01 A - ..A,
xo
&

:o
•• •
.o •
o. ••
o:
I2 + 2S2O| ■> 21 + S40g

Target Practice 1
1. Write the formulae of the compounds with given oxidation 4. 2O3 ---- > 30,; AG° = -163 kJ mol*1
number of the given elements. Is this reaction spontaneous. Yes/No
Element O.N. Compounds
~0 5. Write reaction representing 03 as an oxidising agent
(a) +2
(b) 0 (a) in acidic medium
(c) 0 -0.5 (b) in basic medium
(d) S -2 Equivalent weight in (a)
(e) S +4 Equivalent weight in (b)
1 (I) S +6 6. Compare oxygen and fluorine in following properties

2. Based on following reaction identify alkene A (a) Electronegativity 0 F


O,/H,O/Zn TTZ>TT^ 9H0 (b) Electron affinity 0 F
A -—--------> 2HCH0+ I (c) Reactivity 0, F,
CHO
(d) Bond energy to form 0, F,
3. What is bond length (0—0) in 03 gaseous atoms
(e) Covalent radius 0 F
and bond angle 0 OinO, (f) Ionic radius O 2- F
V

Answers
48 1
1. (a) 0F2 (b) H2O2 (c) KO 2 (d) H2S (e) SO, (f) so3 Equivalent weight in (a) = — = 24 g equivalent
2
2. CH2=CH—CH=CH2
48 i
3. Bond length = 128 pm, Bond-angle = 116.8° Equivalent weight in (b) = — = 24 g equivalent .
4 Yes (since AG °< 0)
5. (a) O3+2H+ + 2e~ O2 +H2O 6. (a)O<F (b)0<F (c)O2>F, (d) 0, > F2 (e) 0 > F
(b) 03+H20 + 2e' 02+20H" (0 02">F"
594 | Essential Inorganic Chemistry

Practice Exercise 1
1. Explain. 6. O3 is diamagnetic and O to O bond lengths are equal
(a) O„ is paramagnetic Explain.
(b) H.,0 is liquid while H2S is gas under given conditions 7. O3 is a powerful oxidising agent. Write equations to
(c) Boiling point of H.,0 is abnormally high
represent oxidation of
2. What is coordination number of oxygen in H2O(/)? (a) I" to I2 in acidic solutions,
3. From the trend of (IE)l as in Table 14.1, what do you (b) sulphur to sulphuric acid in the presence of moisture,
conclude about the nature of polonium? (c) [Fe(CN)G]4- to [Fe(CN)6]3“ in basic solution.
4. Polonium also exhibits +2 oxidation state, explain. 8. It has been estimated that if all the O3 in the atmosphere
5. Identify the group 16 (VI A) element that fits each of the were brought to sea level at STP, the gas would form a
following description in: layer 0.3 cm thick. Estimate the number of03 molecules in
(a) the most electronegative the earth’s atmosphere.
(b) a semimetal (radius of the earth is 4000 mile, 1 km = 0.6214 mile)
(c) radioactive 9. Alkene (CgH12) °3/H2°> 2CH_COCH 3
Zn 33 3
(d) the most abundant element in the earth’s crust.
Identify alkene.

Answers
1. (a) It is due to presence of two unpaired electrons in 5. (a) 0 (b) Te (c) Po (d) 0
anti-bonding molecular orbitals
(ols)2(a * 1s)2(g2s)2(o *2s)2(o2px)2(7t2pJ,)2(n2pI)2 6.
(n *2py)l(jt *2pI)1. •O,» .0.
(b) H2O molecules have intermolecular H-bonding due to
greater electronegativity of oxygen hence, H2O is liquid; Due to resonance, two O—O bonds are equivalent and have a
H2S lacks H-bonding hence, gas. bond order of 1.5 (bond length = 128 pm, angle = 116.5°)
(c) Due to intermolecular H-bonding. All electrons appear paired hence diamagnetic.
2. Four. 3. It is least, hence metallic. 7. (a) O3 + 2r + 2H+ ---- > O2 +12 + H2O
4. It is due to inert-pair effect of ns-pair as well as np-pair (b) 3O3 +S + H2O ---- > H2SO4 +3O2
(c) O3 + 2[Fe(CN)4]4" + H2O---- > 2(Fe(CN)6]3’
n ? T + 20H'+0j
ns np 8. 4 x 10 37 ozone molecules.
H3C\ /CH3
(both are inert due to shielding effect hence 9. c=c
difficult to unpair—hence +2 oxidation state) H3C ch3

Allotropy and Polymorphism of up and join together


O o ---- O
s S2
Sulphur into long spiral-chain
molecules, resulting in
Many different molecular species are possible for the
elemental sulphur, and this accounts for the existence of
• liquid sulphur (Sp)
the large number of physical forms in which the element
may appear as shown in Fig. 14.2. Allotropy is more which is a dark in
complex for the element sulphur than for any other colour and very thick
element of group 16(VI). Sulphur may exist as: and viscous. The chain
• rhombic sulphur (Sa) which has sixteen S8 rings in length and viscosity of
the liquid reach a
a unit cell and at 95.5°C converts to:
maximum at about
• monoclinic sulphur (Sp) which has six S8 rings in 180°C. At higher
its unit cell. It melts at 119°C, yielding : temperatures the
• liquid sulphur (Sz) comprised of S8 molecules. This chains break up and
viscosity decreases
is yellow, transparent, mobile liquid. At 160°C, a again. At 445°C, this
remarkable transformation occurs. The S8 rings open liquid boils, producing: Sn (n = 2000 - 5000)
Fig. 14.2 Different molecul’
forms of sulphur
Chapter 14: Group 16 (VIA—The Chalcogens) | 595

• Sulphur vapours, which consists of molecules liquid (by tank car, ship, or pipeline) or allowed to solidify
ranging from S2 to S10, but predominantly S8. At to a solid.
higher temperatures, S2 predominates. S2 is Although the Frasch process was once the principal
paramagnetic. source of elemental sulphur this is no longer the case. The
• Plastic sulphur forms if liquid sulphur (p) is poured change has been brought, about by the need to control
into cold water. It consists of chain-like molecules and sulphur emissions from industrial operations. In one
has rubber-like properties when first formed. On process to remove H2S from a gaseous mixture, the gas
standing, it becomes brittle and finally converts to stream is split into two parts. One part (consisting of about
rhombic sulphur. one-third of the stream) is burnt to convert HgS to SO2. The
Above transformations can be written as: streams are rejoined in a catalytic converter at 200 to
300°C, where this reaction occurs.
q 95.5°Q o 119°C c 160°C Q 445°C o ,
\ * Sp ------- > SA,r------- > S„^^S
\F* - 8(g)
, 2H2S(g)+ SO2(g) 3S(g)+2H2O(g)
S ^ooor
2000°C s2 4000^
1000°C s< Se
• If rhombic sulphur is heated rapidly, it fails to convert
Study of Compounds of Group 16 (VIA)
into monoclinic sulphur and melts at 113°C. Hydrides (H2S)
• In cyclo-S6, the ring adopts the chain form chain
• All form covalent hydrides. Water (HjO) is liquid at
polymers, catena-Sn are also known. room temperature but others are all colourless, foul
smelling toxic gases.
Production of Sulphur • A comparative study of hydrides of this group are
given in Table 14.3.
Frasch Process
Sulphur occurs abundantly in the earth’s crust as • H^S is prepared by reaction of FeS on dil. HgSC^:
elemental sulphur, as mineral sulphides and sulphates, as FeS + HjSO* -----> FeSO4 + EL2S
HjS in natural gas, and as organic sulphur compounds in
oil and coal. The sulphur is mined in an unusual way,
known on the Frasch process (Fig. 14.3). DEEP Focus
• Now a days, H?S used in qualitative analysis is prepared by
compressed air—TN the hydrolysis of thioacetamide:
o
A liquid sulphur CH-—C
NH2
+ 2H2O + H*
OH
superheated thioacetamide
water +nh; i-KgS
Use of thioacetamide is useful as it produces E^S
in situ, and thus causes no pollution.
• H2S is soluble in water and burns in air with a blue
0 0° 0 0
0/ sulphur-bearing
.rock
flame :
HaO + HzS H3O+ +HS‘
^o^x^lquid OOO 0 HS" + l^O H3O* + S2-
OOO °o0 2H2S + 3O2 ---- > 21^0 + 2SO2
OOO
J1A OOO
////////ZvAO OOO
• A saturated solution of H^S is used as a laboratory
reagent for qualitative analysis of basic radicals
7/777777777/ (cations):
7//////////t - in acidic medium (HC1), Cu2+, Pb2+ .... of II
group are precipitated as sulphides
Fig. 14.3 Frasch process for production of sulphur
CuSO4 +H2S-----> CuS4, +H2SO4
A mixture of superheated water and steam (at about black
160°C and 16 atm) is forced down the outermost of three CdCl2 +H2S CdSi +2HC1
yellow
concentric pipes into an underground bed of sulphur
containing rock. The sulphur melts and forms a liquid - in basic (ammoniacal) medium, Zn2+,Ni2+ of
pool. Compressed air (at 20-25 atm) is pumped down the IV group are precipitated as sulphides :
innermost pipe and forces liquid sulphur up the remaining ZnCL + ELS NH4°H> ZnS i + 2HC1
pipe. The liquid sulphur is either stored and shipped as a white
596 | Essential Inorganic Chemistry

• H2S is a weak dibasic acid and forms two series of Source


salts :
• SO2 is produced commercially on a vast scale:
H2S + NaOH---- > NaHS + H2O
- by burning sulphur and H2S in air
NaHS + NaOH---- > Na2S + H2O
s + o2 —> so2
• O3 can oxidise H2S to H2SO4:
2H2S + 3O2 ---- > 2H2O + 2SO2
H,S + 2O3 ---- > H2SO4 + O.'2
- by roasting metal sulphide ores with air
• H2S decolourises acidified KMnO4 : 2ZnS + 3O2 ---- > 2ZnO + 2SO2
5H2S +2MnO; + 6H+ ---- » 5S 1 + 2Mn2+ + 8H2O • In the laboratory, it can be prepared by the action cl
purple yellow colourless
dilute acid on a sulphite ion,
• In alkaline medium MnO4 is reduced to MnO2 (black). Na2SO3 + 2HC1---- > 2NaCl + 11,0 + S02
2MnO; + 3H,S---- > 2S1 + 2MnO2 1 + 21^0 + 2OH’
Or by the action of concentrated H2SO4 on Cu.
• H2S also turns acidified K2Cr2O7 to green : Cu + 2H2SO4 ---- > CuSO4 + 2H2O + S02
3H2S +Cr2O27" + 8H* ---- > 3S 1 +2Cr3+ + 7H2O
orange yellow green
• H2S is also oxidised by Cl2 : Properties
H2S + Cl2 ---- > 2HC1 + S1 • SO2 is a colourless gas (b.p. -10°C, m.p. -75.5’C)
which has a choking smell, and is very soluble in
• FeCl3 (yellow in aqueous solution) changes to FeCl2 water. SO2 in solution is almost completely present as
(light green) SO2 GHgO and only traces of H2SO3 (sulphurous acid)
2FeCl3 + H2S---- > 2FeCl2 + 2HC1 + S may be present. SO2 levels above 5 ppm are
poisonous to man, but plants are harmed at
Table 14.3 Some Physical Properties of Hydrides of
Group 16 (VIA) appreciably lower levels.
• An acidic oxide, it reacts with water as follows:
Property HjO H2S HjSe HJe
SO2 + H2O HSO3 + H*
Melting point/K 273 188 208 222
Boiling point/K 373 213 232 269
M— H Bond length (pm) 96 134 146 169 Reducing Properties
HMH angle (°) 104.5 92 91 90 • SO2 is reducing agent and this property is due to
Bond energy (kJ mol"1) 463 347 276 238 nascent H:
Ka (dissociation constant) 1.8x 10" 16 1.3x 10‘ 1.3x 10-4 2.3 x 10-3
in aq solution. 2H2O + SO2 H2SO4 + 2H
or to ionic reaction :
sulphide ppt sulphide ppt
(CuS, CdS... As2S3) (ZnS, NiS, MnS, CoS) SO2 + 21^0---- > SOj“ + 4H* + 2e”
- it reduces acidified Ci^O^- to green Cr3+
Cr2O^“ + 14H* + 6e~ ---- > 2Cr3+ + 7H>0
s
- it reduces acidified MnO4 to colourless Mn T
FeS + H2SO4----------- h2s
H II ...
- CH3—c—nh2 MnO; + 8H+ + 5e” ---- > Mn2+ + 4H>0
(dil) thioacetamide
03^ - it turns starch iodate paper blue (due to ^)
1 o O/y.
1 rJ 2IO3 + 12H* + 10e' ---- > \ + 611,0
FeCk oc O
h2so4 ^>2 MnO2 + S
S This reaction can also be used to determine SQ
X HCI + S quantitatively. liberated is titrated using hypo
FeCI2 + S
S + Mn2* Cr3* + S (Na2S2O3).
• SO2 is oxidised to SO3 and thus used to manufacture
H2SO4
Oxides of Sulphur 2SO2 + O2 2SO3
• Sulphur forms two main gaseous oxides : SO3 + ---- > H2SO4
502 (anhydride of H2SO3)
503 (anhydride ofH^SOJ Oxidising Properties
• Oxides of other elements (SeO2, TeO2, PoO2, SeO3»
: HgS is oxidised to colloidal sulphur
TeO3, TeO and PoO) are solids; oxides of Se are
21^5+SO2 ---- > 21^0+ 3S
volatile.
Chapter 14: Group 16 (VIA—The Chalcogens) | 597

• S02 is used to make other products : At room temperature, S03 is solid and exists as :
2SO2 +Na2CO3 +11,0 -> 2NaHSO3 + CO2 0 0 0
2NaHSO3 +Na2CO3 -> 2Na2SO3 +11,0 + CO2
I I I
—0—8—0—8-0—S—0—
Na2SO3 + S ■» Na2S2O3 (hypo) I I I
0 0 0
a-SO3 (chains)
DEEP Focus
• Liquid SO2 has also been used as a non-aqueous solvent and 0. O
is a useful reaction medium for organic and inorganic 7S
compounds: 0 0
2SO2 SO2* + SO2- Gk I 1^0
Radioactive isotopic study shows thatSOCl2 ionises as o> <0
SOC12 SOC1*+Gl­ 0
and not as SOC12 so2*+2cr y-SO3 (cyclic, ice-like)
Thus, formation of SO2* in liquid S02 is not favourable. SO2* + so3_ S + O2 H2SO3
(Cu + H2SO4) h2o/
• The S03 is not usually isolated, and practically all of it
(cone)
is converted to HgSQj. /Cr2O?_ /H*
bum Cr3* (green)
• H2SO4 saturated with SO, is called oleum h2s + 02 so2 MnO; /H* Mnz*
(pyrosulphuric acid)
H2SO4 + S03 ---- > H2S2O? I2

DEEP Focus ZnS + O2 Na2CO3/H2O Na2CO3


NaHSO3 —-—4- Na2SO3
• S03 is used for the sulphonation of long chain alkyl benzene
H2SO4 O2 S
compounds (like dodecyl benzene). The sodium salt of these
H2O
alkyl benzene sulphonic acid are anionic surface-active SO3 Na2S2O3
agents, and are the active ingredients of detergents: (hypo)
H2S2O7 SO3
(oleum)
SO3Na (ABS)
so3h
S3O9
“ 012^25 (no S—S bond) detergent
ABS (Alkyl Benzene Sulphonates)
• S03 + H2S04 +NH2CONH2---- > 2^80^ + CO2 Oxoacids of Sulphur
sulphamic acid • The oxoacids of sulphur are more numerous and are
summarised in Table 14.4.
Structure • Acids ending in -ous have S in the oxidation state
• S02 gas forms discrete V-shaped molecule and this (+IV), and form salts ending in -ite. Acids ending in -ic
structure is retained in solid state. have S in the oxidation state (+VD and form salt
• The sulphur atom is spz hybridised but bond angle ending in -ate.
ZOSO is 119°30'. It is due to VSEPR (valence shell • In peroxy acids, there is a peroxy linkage (with
oxidation number of each oxygen in peroxide -1) hence
electron pair repulsion):
oxidation number of sulphur is (+VI).
• In H2S2O3 one sulphur is attached to central S, hence,
•0
•• x•0:
• :o
•• 0: its oxidation state is (-11) as in oxygen.
• In the gas phase S03 has a plane triangular structure. • (oxidation number of S = VI) is stronger than
The bonding is best described as S forming three HzSQi (oxidation number of S = IV).
0 bonds, giving rise to plane triangle, and three
delocalised rc-bonds: Sulphurous Acid (H2S03)
• The acid is obtained by dissolving S02 gas in H20:
:0
II SOa +H20 HjSOa
:0^ ^0 It actually exists as S03 ■ H20 and this acid is known
•• O:
only in solution.
598 | Essential Inorganic Chemistry

• A strong diprotic acid: Table 14.4 Oxoacids of Sulphur


H2SO3 H+ + HSO3 1. Sulphurous acid series
HSO3 H++S02-
(a) H2SO3 S (IV) sulphurous acid HO
NaHS03 is thus an acid salt andNa2SO3 is normal salt •S=O
and both liberate S02 gas on reaction with acid : HO'
NaHS03 + HCI----- > NaCl + H20 + S02 (b)H2S2O5 S(+V) disulphurous acid or 0 0
Na2SO3 + 2HC1----- > 2NaCl + H20 + S02 S (+111) pyrosulphurous acid II I
H—0—S-S-OH
• The acid is a good reducing agent: I
0
• HjSOg + H20----- > H2SO4 + 2H+ + 2e“
(c)H2S2O4 S (+111) dithionous acid 0 0
- X2(C12, B12,12) is reduced to HX II I
HO—S-S-OH
I^SOg + 12 + H20----- > HjSQ, + 2HI
- Fe3+ salts are reduced to Fe2+ salts 2. Sulphuric acid series
2FeCl3 + H^SOg + H20----- > 2FeCl2 + H2SO4 + 2HC1 0
- Mn04 (violet) is reduced to Mn2+ (colourless) II
(a) H2SO4 S (VI) sulphuric acid HO—S-OH
2Mn0; + 5S0|’ + 6H+ ----- > 5S0j" + 2Mn2+ + 3H2O I
0
- (orange) is reduced to Cr3+(green)
0
CrjO2/ + 3S0g— + 8H+ -----> 3SO2’ + 2Cr3+ + 4H2O S (VI) thiosulphuric II
(b)H2S2O3 OH—S—OH
- HA is reduced to H20 S(-ll) acid
I
HjSC^ + H2O2 ----- > + H20 0
0 0
- It combines with S forming thiosulphuric acid I
(c) H2S2O7 S (VI) pyrosulphuric acid HO— S—0— S-OH
H2SO3 + S----- > H2S2O3
II I
• It behaves as a good bleaching agent and this 0 0
action is due to nascent H (reducing action) 3. Peroxo acid series
H2SO3 + HjO-----> H2SO4 + 2H 0
(a) H2SO5 S (VI) peroxomono­
sulphuric acid II
HO—S-O-OH
DEEP Focus (Caro's acid)
I
0
Structure (b) H2S2O8 S (VI) peroxodisulphuric 0 0
• The sulphite ion exists in crystal and has a pyramidal acid II I
HO—S-O-O-S-OH
structure, with one position occupied by lone pair. S-atom is (Marshall’s acid)
sp3 hybridised but bond angle 0—S—0 is 106° (slightly II I
0 0
distorted due to lone pair) and the bond lengths are 151A.
The (dn - pre) bond between S and 0 is delocalised, and
hence, the S—0 bonds have a bond order of 1.33 Sulphuric Acid
/ O’ .. /O’ o • Contact process (a commercial method)
o=s 4—> 0-S 4—> O—S Involves four stages:
0 A
0" ■0-
Stage I : S + 02 * so2
Fe2+ Mn2+
Cr3*
v2o5
HCI Stage II : 2SO2 + 02 —2SO3
Cl2 Fe3+ MnO4/H*
Stage HI: S03 + HgO-----> H2SO4
\ h202
H2SO4 + 2H* + 2e~ —► H20 This is a violent reaction; instead S03 is passed into
i
reducing action of H2SO3 98%H2SO4
H20
H^SC^ + S03 H2S2O7
i oleum
h2o S
H2SO4 + 2H h2so3 H2S2O3 (pyrosulphuric acid)
bleaching
action Some trisulphuric acid E^SgO^ is also formed.
NaOH
Stage IV: H2S2O7 + H^O-----> 21^864
NaOH _
NaHSO3 *■ Na2SO3 H^Ojo + 21^0---- > SHjjSO, i
Chapter 14 . Group 16 (VIA—The Chalcogens) | 599

Zn), thereby liberating molecular hydrogen (H2) in a


Lead chamber process (which is no longer in use)
displacement reaction; Mg reacts in the same way
Involves three stages:
Mg + 2H3O+ -----> Mg2+ + + 21^0
Stage 1: 2N0 (catalyst) + 02 ---- > 2NO2
This is a typical reaction of an active metal with an
Stage n: 2SO2 + 2NO2 ---- > 2SO3 + 2N0 acid.
2SO2 + 02 2SO3
DEEP Focus
Stage III: S03 + H^O ---- > H2SO4 The strength of sulphuric acid as an oxidising agent is
greatly enhanced when it is both hot and concentrated.
r—N02 + S02 + HjO---- > HjSC^ + NO (recycled) In such a solution, the oxidising agent is actually the
sulphate ion rather than the hydrated proton, H+ (aq).
I Thus,
2N0 + 02 -----> 2NQ2
SO*-
4
+ 4H+ + 2e" ---- > SO,Z + 2H,0 Z

Cu + 2H,SO.
L 4
---- > CuSO.4
+ 2H,0Z
+ SO,Z
• Pure HgSC^ is strongly hydrogen bonded and in S + 2H2SO4 ---- > 2H2O + 3SO,*2
the absence of water it does not react with C + 2H9SO4 ---- —> Cw CO2 + 2H2O + 2SO2
metals to produce H2 2HBr + H2SO4 ---- —> Br2 + SO2 + 2H,0
K4[Fe(CN)6] + 6H2SO4 + 6H2O 0---- > 2K,SO,

X
Z 4
■-HO>SZ-HO>SZ-HO + FeSO4. + 3(NH4. )„SOZ
. + 6CO
4
-H0Z H0Z X>—HO Depending on the nature of the reducing agents, the
sulphate ion may be further reduced to elemental sulphur or
• If pure H2SO4 is heated, a little S03 is evolved, and an
the sulphide ion
azeotropic mixture of 98.3% H2SO4 and 1.7% water is
8HI T+ H,SO.
0X11 1 k^J --------- T H2S + 4I2 + 4H,0
produced.
• As a dehydrating agent:
DEEP Focus Concentrated HzSC^ is also a dehydrating agent:
• Anhydrous H2SO4 and concentrated H2SO4 mix with water C6H12O6 conc' 6C + GHjO
in all proportions, and evolve a great deal of heat (880 kJ
mol'1). If water is poured into concentrated sulphuric acid, CHgC^OH C°nC16^^4 CH^CH* + BjO
QC^~^ CH2=CH2 HaO
the heat evolved leads to hoiling of the drops of water and
causes violent splashing. The safe way to dilute strong
acid is to carefully pour the acid into the water with 2CH3CH2OH
stirring.
+ 1^0
• It is a diprotic acid. It is colourless viscous liquid (m.p. • A mixture of cone. HN03 and cone. HaSQt is used as a
10.4°C). The concentrated sulphuric acid we use in the
nitrating mixture in the nitration of aromatic
laboratory is 98 per cent by mass (density
compounds :
1.84 g/cm3) which corresponds to a concentration of
HN03 + 2H2SO4 -----> N02+ + H30+ + 2HS0;
18 M. nitronium ion
U a strong acid : Nitronium ion is a sufficiently good electrophile to
In aqueous solution, it is regarded as a strong acid attack a benzene ring :
onising in two stages :
KjSO, + ^0---- > HSO; +H3O+ + no2 QY-NO2 + H+
HSO; + HjO sot +H3O+
nitrobenzene
Only the first dissociation is complete; the second is
fartial. • Cone. HjSQ! is also used for sulphonation of organic
When it is dilute, it shows all the usual properties of compounds :
-rids.
Zn + H2SO4 ---- > ZnSO4 + H2
Zn + 2H3O+ ---- > Zn2+ + IL, + 2^0
Q + H2SO4 so3h + h2o

benzene sulphonic acid


• As an oxidising agent:
• HO—SO2 —OH + 2PC15 -----> SO2C12 + 2POC13
The oxidising strength of sulphuric acid depends on
sulphuryl
its temperature and concentration. A cold, dilute chloride
sulphuric acid solution reacts with metals above
+ 2HC1
hydrogen in the activity series (as shown above with
600 | Essential Inorganic Chemistry

• When cone. H2SO4 is subjected to electrolysis there is • 2Na2S3 + 3O2 -heat


neatin
M airr) 2Na2S2O3 + 2S
formation of peroxy disulphuric acid and H2O2 at the
anode • Following reactions called Spring’s reactions, art
also used to convert Na2S or S into Na2S2O3:
H2SO4 H+ +HSO4-
at anode Na2S + Na2SO3 + Lj ----- > Na2S2O3 + 2NaI
2HSO; ■> H2S2O8 + 2e 2Na2S + 4SO2 + Na2CO3 -----> 3Na2S2O3 + C02
H2S2O8 + 2H2O 4 2H2O + 2H2SO4 6NaOH + 4S Na2S2O3 + 2Na2S +
at cathode • Hydrated sodium thiosulphate, Na2S2O3-5H2O, is
2H+ + 2e~ * H, called ‘hypo’. It forms very large colourless hexagonal
crystals, melting point 48°C.
Structure • It is highly soluble in water.
0 0 • It is an efflorescent substance and at 215° C, it is
I II completely dehydrated to Na2S2O3.
f^O-H • Dilute acids like HgSC^, HC1 decompose it into S02 and
o o colloidal sulphur (yellowish-white turbidity):
0—H 0“
Na2S2O3 + 2HC1-----> 2NaCl + SO2 + S +1^0
S-atom is sp3 hybridised. SO^- ion is tetrahedral. The
This is a disproportionation reaction.
bond lengths are all equal (1.49A) and are all rather short.
The bond order of the S—O bonds is approximately 1.5.
There are four o-bonds and two n bonds between S and 0
DEEP Focus
• L oxidises NaoSo0, into Na0S ,OC:
atoms (with two rr-bonds delocalised over the S and the
four 0 atoms). 2Na2S2O, + 1,2 ---- > Na2S4O6 + 2NaI
sodium tetrathionate
! f This reaction is used for the iodometric estimation of
: NO + O2 Lead-chamber
process
oxidising agents like CuSO4, CaOCl2, MnO2, K2Cr2O7,KMnO4
etc., which on reaction with KI liberate I2 which in turn is
i NO2 no determined by titration using Na2S2O3.
I |so2 2CuSO4 +4KI---- > 2K2SO4 + Cu^X +I2
: SO3------- White ppt
Contact process ""|h2'o...
• Na2S2O3 is used in the bleaching industry to destroy
O2r SO3 r
s + o2 —► so2 so3-^ H2SO4 H2S2O7 any excess Cl2 on fabrics :
V2O5 h/5 L
Na2S2O3 + 4C12 + 511,0-----> 2NaHSO4 +8HC1
o %
CD
2FeCl3 + 2Na2S2O3 -----> Na2S4O6 +2FeCl2 +2NaCl
6
% • AgNO3 reacts withNa2S2O3 giving white pptchanging
8
H2 f 4-
to yellow, then to brown and finally black:

SO3H
PCI5 SO2CI2 '^Sq 2AgNO3 + Na2S2O3 ----- > Ag2S2O3 + 2NaNO3
c \Cu white ppt
SO2 + CO2' s % SO2 + C11SO4 Ag2S + HjSO,
black ppt
so2 • Hypo is used in photography for ‘fixing’ films and
FeSO4 + (NH4)2SO4 + CO
Br2 + SO2 prints. It dissolves unreacted AgBr from the
photographic emulsion by complexation :

Sodium Thiosulphate (Na2S203 5H20 — Hypo) AgBr + 2Na2S2O3 -----> Na3[Ag(S2O3)2 ] + NaBr
soluble
• H2S2O3, thiosulphuric acid, because of its low stability,
can’t be prepared. However, its salts called The thiosulphate ion is structurally similar to the
thiosulphates can be prepared. sulphate ion; one oxygen atom has been replaced by
sulphur atom. Based on Lewis structure oxidation number
boiling
• Na2SO3 + S Na2S2O3 +6 has been assigned to central sulphur and -2 for the
neutral or sodium
alkaline thiosulphate terminal sulphur. Thus, average oxidation state is +2.
Chapter 14.Group 16 (VIA—The Chalcogens) | 601

In tetrathionate ion S4O^“, two of the sulphur atoms


o
present in the ion are in oxidation state 0 (zero) and two
I
are in oxidation state +5. Alternately one can say that S' O-
S$- is the adduct formed by binding of the Lewis base. 0"
^'toSO3 sp^hg’SReact^

Si- + 2SO3 S4O^ NaOH + S


Lews Lewis adduct I

base acid
0 Na2S + Na2SO3 + l2 Na2S + SO2 + Na2CO3

0"
I A

8>f s^s=° Na2SO3


\S
A A Na2S3
0- 6<
0
SO2 + S
HCI
Isotopic tracer technique has been used to verify the Na2S2O3 Na2S4O6 + Nal
t Cl2
existence of two types of sulphur in S2O^-. Sulphite ion NaHSO4 + HCI
h2o
(S0^“) is treated with radioactive isotope (35S) and the AgBr
AgNO3
thiosulphate ion (S2O^“) is decomposed by acid. 35S is
found to be ony in precipitated sulphur. This indicates
FeCI2 + Na2S40g Ag2S2O3| Na3[Ag(S2O3)2]
that sulphur (35 S) is not distributed with other sulphur is
I (white) (soluble)
s2tf3-
SO23'+S*(35S) S*SO23"
Ag2S|
S*SO23’+2H+ HjSO-j+S* (black)

Target Practice 2
1. What are following sulphur allotropes called? 7. Write the formulae of
(a) Pyrosulphuric acid
(a) Sa (b) Sp
(b) Peroxosulphuric
(c) Sy (d) S, (c) Peroxodisulphuric acid
2. H2S changes to sulphur in Frasch process by reaction
8. Equivalent weight of SO2 is found to be 32 in a reaction
taking place in acidic medium,
represented as.................................
3. Differentiate H2O and H22S in following properties
9. A gaseous mixture ofCO2 and SO, was to be identified in a
(smaller/larger) laboratory. Mr X and Mr Y. performed following
H,0 h2s experiments with common reagents but at different stages.
(a) Boiling point I------- 1 MrX|cO2 + SO^ | Thus, Ca(OH)2 turns
(b) Bond angle I I
(c) Acid-strength L ...J
and K2Cr2O2 turns
4. H2S gas is passed into aqueous solution
(a) CuCl2 solution (b) ZnCl2 solution
Ca(OH)2
K2Cr20r
H2S H2S
I II
CuS is precipitated ZnS is precipitated
Yes/No |Yes/Np] Mr^CO2+SO2 —usingK,CroO7 firstand thenCa(OH)2
Thus, K,CrO7 turns
5. Oxide of sulphur are part of acid rain. Represent
phenomenon of acid rain by chemical reaction

6. 2SO2 |SO2V |SQg-| 10. Sulphuric acid is produced by lead chamber process with
following exothermic reaction
NO H,0
2SO22+O2 2SO.3 * H2SO4
Name the cation and anion.
602 | Essential Inorganic Chemistry

(a) What is the role of NO? 11. Hypo dissolves unreacted AgBr of the photographic film by
reaction
(b) What is the intermediate?
12. In the following
(c) Formation of H2SO4 is favoured under following 2Cu 2+ + 4F ---- > Cu2I2+I2
conditions (higher/lower) -2S„O:
C,‘J2'^3 +^2 -------* $4^6
4
pressure
(a) equivalent mass of Cu2+is
temperature
(b) equivalent mass of S2O2-is
concentration of SO2,02.

Answers
1. (a) Rhombic (b) Monoclinic (c) Liquid (d) Liquid 8. SO2+2H2O ---- > SO3-+ 4H++ 2e~
2. 2H2S(g)+SO2(g) ---- > 3S(g) + 2H2O(g) 9. Mr X: Both CO2 and SO2 react with Ca(OH)2 and lime water
3. Refer text turns milky. No effect onK2Cr2O7.
4. (a) Yes, because acidic (HC1) medium is formed in which CuS MrY :SO2 turns K2Cr2O7 green andCO2 does not react at this
is insoluble. stage. CO 2 reacts at stage II with Ca(OH)2 and it turns milky.
CuCl2+H2S ---- > CuS + 2HCl 10. (a) Catalyst
(b) NO,
(b) No, because acidic (HC1) medium is formed in which ZnS is
soluble (c) Higher pressure
Lower temperature (theoretically)
ZnCl2 + H2S ZnS + 2HC1 '
Higher concentration ofSO2 andO2
5. SO22+H2O ---- > H,SO
— 2OM3
rain acid rain 11. AgBr + 2S2O|- ---- > [Ag(S2O3)2]3‘+ Br"
12. (a) 2Cu2+s I2 s 21
SO3+H2O ---- > H2SO4 .♦. Cu2+= 1
rain acid rain
equivalent mass ofCu3 + s ionic weight
6. SO2 * : Sulpharyl (b) 2S2O3‘ sl2 = 21
SO3~ : Sulphite S2O^=I
7. (a) H2S2O7 (b) H2SO5 (c) H2S2O8 Equivalent mass ofS2O3~ = ionic weight.

Practice Exercise 2
1. You have been given an inorganic mixture with a 6. Give examples of reactions for preparing SO2 that
knowledge that group II is absent, but when pass H2S gas (a) are attended by a change in oxidation state of sulphur
into the mixture solution acidified with dil. HC1, yellowish
(b) are not attended by a change in its oxidation state
turbidity appears. Explain.
2. FeCl3 solution gives blood red colour with NH4SCN. If H2S 7. Sodium salt (A) of a dibasic acid HC1
nui > gas (B)
gas is passed and filtered then filtrate does not give red Gas (B) turns K2Cr2O7 green and also lime water milky-
colour with NH4SCN. Explain. Identify (A) and (B).
3. SO2 and Cl2 both are used as bleaching agent. What factors 8. How will you estimate NaHSO3 volumetrically?
cause bleaching ?
9. What is structure of SO: ion?
4. When H2S gas is passed into SO2 in aqueous solution,
colloidal yellowish white turbidity appears. Write reaction. 10. Cl2 also behaves as bleaching agent like H2SO3 (aq S0t).
Calculate weight of this turbidity if 11.2 L of each gas Explain.
reacts at NTP. 11. How will you identify SO3’ in presence of CO2"?
5. 4FeS + 7O2 ----- > 2Fe2O3+4SO2
Cr2O2-+ 2H++ 3SO2 ----- > 2Cr3+ +3S0J- +H2O 12. Which of nitric acid and sulphuric acid is the stronger
Bronsted acid, i.e., which donates proton to the other?
1 g sample of FeS was roasted and SO2 thus formed
required 10 mL of 0.05 M K2Cr2O7 solution in acidic 13. Zinc reacts with dil H2SO4 to give H2. It also reacts with
medium. Calculate percentage of FeS in the given sample. concH2SO4 to give SO2. Make difference between these two
(Fe = 56,S=32) reactions.
Chapter 14: Group 16 (VIA—The Chalcogens) | 603

Answers
1. It is due to presence of some oxidising agents (acidic radicals 7. (A): Na2SO3 (B): SO2
like NO2, NO3) which oxidise H2S to colloidal sulphur 8. NaHSO3 is treated with excess (known) amount of I2 and
(yellowish turbidity) unreacted I2 is determined by back titration using Na2S2O3
H2S + O---- > H2O + S (colloidal yellowish turbidity) (hypo) iodometrically.
2. FeCl3 +NH4SCN---- > [Fe(SCN)]Cl2 +NH4C1 I2 + NaHSO3 + H2O NaHSO4 + 2HI
FeCl3 changes toFeCl2 on pa5lAigH2S hence, no colour with I2 + 2Na2S2O3 Na2S4O6+2NaI
NH4SCN. 9. See text
3. SO, +2H2O---- > H2SO4 +2H 10. C12+H2O ---- > 2HC1+(O)
Cl2 +H2O---- > 2HC1 + O It is due to nascent oxygen.
Bleaching action ofSO2 is due to H (that causes reduction) and 11. (CO3“+SO2") + dil. HC1 ---- > CO2+SO2
that ofCl2 is due to O (that causes oxidation).
Both gases turn lime water milky. Hence, proceed :
4. SO2 4-2H2S ---- > 2H2O + 3S
1 mol 2 mol 3 mol (colloidal turbidity) (a) First pass the gaseous mixture into acidified K2Cr2O7
solution. Only SO2 reacts and turns it green :
1 1 . 3
- mol - mol - mol = 24 g
2 Cr2O7" + 3SO2 + 2H+ ---- » 2Cr3+ + 3SO2" + H,0
(112 L of each at NTP = i mol) green
2 (b) Gas unreacted is CO2. Pass it into lime water. It turns
Q
H2S is limiting reactant, hence S formed = “~ 3J Uivl
mol milky.
4
6. 10 mL of 0.05 MK2Cr2O7 = 10 x 0,05 mol = 5x 10"4 mol Ca(OH)2 + CO2 CaCO3 + H2O
1000 milky

SO2 - 3x 5x IO"4 mol 12. h2so4+hno3 —> hso’;4 + h,no+3


H2SO4 is proton donor.
pure FeS = 3x 5x 10"4 mol
13. (a) Zn + H2SO4 ---- > ZnSO4 + H2
= 15 x 10"4 x 88 g dil
= 0.132 g in 10g impure FeS In this reaction H + is reduced by Zn
Zn + 2H* ---- > H2 + Zn 2>
= 1.32%
8. (a) S + O22 ---- > SO2 ] change in oxidation state of sulphur (b) Zn + 2H,SO4 ---- > ZnSO4 + SO, + 2H2O
cone.
0 +4
(b) Na2SO3 + 2HC1---- > 2NaCl + H2O +SO2 -| Zn reduces SO2- to SO2
+4
no
change SO2" + 4H* + Zn Zn2* + SO, + 2H2O
J

Total Practice Set


(Read, Plan & Solve')
__ __ ____ '

FeS2 > 2SO2 * 2SO3


Problem 1. Zinc dissolves in dil HC1 to liberate H2(g), ---- > 2H2SO4
but from cone H2SO4, SO2 is evolved. Explain.
Thus, 1 mol of FeS2 gives = 2 mol HjSQj
Solution (A)Zn+ 2H+ ---- > Zn2+ +H, E° = 0.76 V
dil = 2 x 98 g H2SO4
(B) Zn + SO42" + 4H+---- > 2H2O + SO2 + Zn2+ Hence, 1.00 kgFeS2 = 1000 g FeS2 =~ mol FeS2
X^uU
E° = 0.93 V 1000 o 2 x 98 x 1000
E° of reaction (B) is large than that of (A). H2SO4 from ------ mol FeS9 = 120 g
120
In cone. H9SO4, SO42"is a better oxidising agent than
= 1.63 kg
H+. Hence, with cone. H2SO4, reaction (B) predominates.

Problem 2. EAl3*/Al = -1.66


-1.66V;E°
V;E, 2 Problem 4. The action of concentrated sulphuric acid
’ SO427SO2 = 1.17 V. on urea (NH2CONH2) results in the production of a white
What, if anything, would happen if you reacted crystalline solid X of formula H3NO3S. This is a monobasic
aluminium with sulphuric acid ? If you think a reaction acid. On treatment with sodium nitrate and dilute
hydrochloride acid atO° Cit liberates 1 mol ofN2 per mol of
would take place, write down the equation. X, and on addition of aqueous barium chloride the
resulting solution yields 1 mol of BaSO4 per mol of X
Solution The given reaction (say dissolution of Al into
taken.
J^SQj) is possible if Ec°ell is positive. Deduce the structure of X.
Al---- > Al3+ +3e"
Solution NHaCONHj (Urea) reacts with cone. HjSO^
E°ai/ai 34 ai3+/ai
= 1.66 V forming X(H3NO3S).X contains one ionisation H (since, X
so42- +2e~ + 4H+---- > SO2 + 2H2 is a monobasic acid). Thus,
E° 2_lcn = 1.17 V. 0 0
so/ /SO2 I I
Net reaction is NH^—C— NH, + H—0—S— 0—H —>
Urea Sulphuric acid
2A1 + 3SOJ- + 12H+ ---- > 2A13+ + 3SO2 + 6H2O
Thus, Ec°ell is 2.83 V hence, Al can reduce SO42- to SO2. NHj— SO3H + NH3 +C0
X
Problem 3. What mass of I^SQ, could be produced in X contains free —NH2 as well as free —SO3H.
(NaNO2 + HC1) converts free —NHg into free —OH group
the process given below if 1.00 kg of FeS2 is used? The liberating N2.
unbalanced equations for the process are: NaNO2 + HC1---- > HN02 + NaCl
FeS2(s)+ O2(g)---- > Fe2O3(s) + SO2(£)
NHg—SO3H +HNO2 ---- > N2? +H2O+H2S04
SO2(g)+O2(g)---- > SO3(g) lmolX ImolX 1 mol
SO3(g) + H2SO4(Z) HaS2O7(Z) RjSQj + BaCl2 ---- > BaSO4 + 2HC1
Imol 1 mol
H2S2O7(Z)+ HjOU) JLjSOjm/XFe = 56, S = 32)
Thus X is NH2—SO^H (amino sulphonic acid), 1 mol of
Solution Sulphuric acid contains one sulphur atom which liberated 1 mol of N2 on reaction with
per molecule, and each FeS2 molecule gives two atoms of (NaNO2 + HC1) at 0°C and in turn resulting solution gives
sulphur. Thus, 1 mol ofBaS04 on reaction withBaCl2.
Chapter 14: Group 16 (VIA—The Chalcogens) | 605

(b) The S—OH and S—O distance in the tetrahedral


Explanation species are as follows:
(a) When metallic copper is heated with cone,
sulphuric acid, in addition to copper(U) S—OH (A) s—o (A)
sulphate, CuSO4, and sulphur dioxide SOg, some H2SO4(g) 1.54 1.42
copperdi) sulphide, CuS, is also formed. hso; 1.56 1.47
“ When cone H2SO4 is used, Cu is oxidised tn Cu2+ and
SOt is reduced to SO2. sot 1.49

Cu * Cu2+ + 2e Comment on the increase in S—O bond length.


SOt + 4H+ + 2e" * SO2 + 211,0 - As the Jt-bonding is distributed over a larger number
There is also side reaction assisted by very high [H+] of atoms, bond order decreases resulting in an
and very low solubility of CuS. increase in bond length.
SOt + 8H+ + 8e" ---- - S2_ + 4H2O
fc._ . i. .

Master Exercises
Exercise 1
(Stage 1: Learning)
Short Answer Type Questions 10. Which of the following desiccants can be used to
1. Give the names and formulae of the compounds in remove moisture from SO2: cone. H2SO4; KOH, P2Qs,
which sulphur exhibits an oxidation state of: - K2CO3?
(a) -2 (b) +4 (c) +6 11. Cone. H2SO4 can’t be used for drying upH2S andNH3.
2. Explain why SCI 6, SBr6 and SI6 cannot be prepared? Explain.
3. Give applications ofO3.
4. What is the change in oxidation number for the Analytical Questions
following reaction? 1. O3(g) is a powerful oxidising agent. Write equations
3O2 -----> 2O3 to represent oxidation of:
5. One of the steps involved in the depletion of ozone in (a) I" to 1% in acidic solution.
the stratosphere by nitric oxide (NO) may be (b) [FetCNlg]4- to [Fe(CN)6]3- in basic solution.
represented as: (c) S in the presence of moisture to E^SO4.
NO(g)+ O3(g)-----> NO2(g)+ O2(g) 2. It has been estimated that if all the O3 in the
What is change in oxidation number of N and O? atmosphere were brought to sea level at STP, the gas
6. After the ozonisation of a certain volume of oxygen, would form a layer 0.3 cm thick. Estimate the number
the volume of the oxygen was found to diminish by of O3 molecules in the earth's atmosphere, (radius of
500 mL. What volume of ozone was formed? What the earth = 4000 miles; 1 km = 0.6214 mile)
amount of heat was absorbed in its formation if is 3. The redox reaction involving the reducing power of
144.2 kJ mol-1 for ozone? Assume values at STP. hydrogen sulphide is :
7. Black (A) + EI^SO,----- > gas (B) + (C) S + 2H+ +2e“-----> HgS,. ^s/h2s = +
Two other half equations are :
(B) + (CH3COO)2Pb----- > black ppt (D)
Fe3+ +e" -> Fe2+ ^Fe3+/Fe2+
: = +0.77 V
(O+ K3[Fe(CN)6] ----- > blue(E)
Bi^ +2e~ -----> 2Br~ E° .-1.07 V
Br2/Br"
(O also decolorises acidified KMnO4. Identify (A) to (E).
Under standard conditions :
8. What properties does E^S show when it reacts with
(i) will hydrogen sulphide react with iron (III) ions?
aqueous solution of KMnO4, HjO-j and NaOH?
(ii) will it react with bromine?
heat HC1 n /rr. Cr2O?7H+
9. Acid salt (A) (B) —* (C) --------- ► Green (G)
► Gas (O If you decide a reaction should take place, write the
Molar mass equation.
104 BaC12 „Ca(OH)2
4. Explain:
White ppt (D) Milky (i) Oxygen is never more than divalent, but S, Se
soluble in HC1 and Te can form four or six bonds.
Gas (C) (ii) SO2 is gas but SeO2 is a solid at room
temperature.
White ppt (H) Soluble (F) (iii) SF6 is unreactive but TeF6 is readily hydrolysed
Identify (A) to (H) in the above sequence of reactions. by water.
Chapter 14: Group 16 (VIA—The Chalcogens) | 607

(iv) The melting point and boiling point show the 6. How many litres of SO2 taken in standard conditions
sequence : have to be passed through a solution of HC1O4 to
H>0» H2S < HaSe< HaTe reduce 16.9 g of it to HC1?
(v) Acidity increases in the sequence : 7. Which of the compounds listed below enter into a
reaction with NaaS2O3?
H2O < HaS < HaSe< H^Te (a) HC1 (b) NaCl
(vi) The bond angle decreases in the order: (c) Nal (d) I,
H^O > HaS > HaSe (e) KMnO4 if:
(1) the initial substance becomes decolorised,
(vii) Liquid oxygen sticks to the poles of a magnet but (2) a precipitate forms,
liquid nitrogen does not. (3) a gas evolved.
[Hint: By MO theory, O2 is paramagnetic)
8. Oxides and peroxides are not the only types of
(viii) The 0—0 bond in O2F2 is much shorter than that compounds that metals can make with oxygen,
inHaO2. superoxides are also possible. 1.15 g of sodium on
(ix) The two sulphur atoms in thiosulphate ion are reaction with oxygen at high pressure gives 2.75 g of a
not identical. white powder, sodium superoxide.
(i) What itiass of oxygen was combined with the
5. Account for the following facts: sodium?
(a) HaSO4 is a stronger acid than HaSO3 (ii) How many moles of oxygen atoms in this?
(b) SF4 exists, but OF4 does not. (iii) How many moles of sodium were used?
(c) TheS8 ring is non-planar. (iv) What is the formula of the oxide and superoxide
ion?

Exercise 2
(Stage 2: High Skill Problem Solving)
Only One Option Correct (c) the tendency to form multiple bonds with C, N
and 0 decreases as going down the group from S
1. Oil of vitriol is
to Te.
(a) HNO3 (b) HC1
(d) all of the above are correct statements
(c) HaSO4 (d) H3PO4
6. Which does not have (S—S) bond?
2. Which of the following has peroxy linkage?
(a) HaS2O3 (b) H>S2O5
(a) HaS2O3 (b) HaSO5
(c) (d) S3O9
(c) HaS2O7 (d) HjSA
7. Which has maximum pKa value?
3. Which of the following can be used as dehydrating
agents? (a) HaO (b) H2S
(a) Cone. HaSO4 (b) POC13 (c) HaSe (d) HaTe
(c) P2O5 (d) All of these 8. Consider the following species :
4. The thermal stability of hydrides of oxygen family is 1. [O2f‘ 2. [CO1+
in order 3. [O2]+
(a) HaPo < HaTe < H2Se < HaS < 11,0 Among these species sigma bond alone is present in
(b) HaPo < HaO < HaTe < HaSe < (a) 1, 2, and 3 (b) 2 alone
(c) 2 and 3 (d) 1 alone
(c) < H2O < H2Te < HaSe < H2Po
(d) HaO < HaS < HaTe < HaSe < H>Po 9. Which of the following species have undistorted
octahedral structures?
5. In case of oxygen family (Group 16)
1. SF6 2. PF6-
(a) the tendency for catenation decreases markedly
as we go down the group 3. SiFf" 4. SeF6
(b) maximum coordination of oxygen is four due to Select the correct answer using the codes given below
lack of d-orbital but that of other elements is six (a) 2, 3 and 4 (b) 1, 3 and 4
due to presence of d-orbital (c) 1, 2 and 3 (d) 1, 2 and 4
608 | Essential Inorganic Chemistry

10. Consider the following compounds 21. Oxalic acid on heating with cone. H2SO4 produce
1. Sulphur dioxide (a) H2O and CO2 (b) CO, CO2 and HjO
2. Hydrogen peroxide 3. Ozone (c) CO2 andHjjS (d) CO andCO2
Among these compounds, those which can act as 22. Which of the following is not a chalcogen?
bleaching agents would include (a) Se , (b) 0
(a) 1 and 3 (b) 2 and 3 (c) S (d) Na
(c) 1 and 2 (d) 1, 2 and 3
23. Which of the following gas is insoluble in water?
11. A considerable part of the harmful UV rays of the sun (a) SO2 (b) Ha
does not reach the surface of the earth. This is (c) NH3 (d) CO2
because high above the earth's atmosphere, there is a
layer of 24. Which one of the following acid possesses oxidising,
(a) O3 (b) CO2 reducing and complex forming properties?
(c) SO2 (d) NO (a) HNO3 (b) HC1
12. Hypervalent ion is/are (c) H2SO4 (d) HNO2
(a) SOt (b) PO43’ 25. Na2CO3 reacts with SO2 in alkaline medium to
(c) SO42' (d) cio; produce
*
13. Which is/are true statements? (a) Na2SO4 (b) Na2SO3
(a) Sulphur trioxide exists as cyclic trimer in solid (c) NaHSO4 (d) NaHSO3
state, S3O9 26. The number of S—S bonds in sulphur trioxide trimer,
(b) Selenium trioxide solid is a cyclic tetramer, (S3O9)is
Se4O12 (a) three (b) two
(c) TeO3 is a solid with a network structure in (c) one (d) zero
which TeO6 octahedra share all vertices
27. Sulphur dioxide levels in the atmosphere can be
(d) All of the above are correct reduced by using
14. Which is thermocromic? (a) catalytic converters in industry
(a) S4N4 (b) B3N3H6 (b) static eleectricity to attract it in factory chimneys
(c) S8 (d) None of these (c) more efficient car-engines
(d) low-sulphur fuels
15. The best reducing agent of the following is
28. What does Green Chemisry in terms of environment
(a) HgS (b) Cl" mean?
(0 SO?" (d) O3 (a) Greenhouse effect
16. All of the following have a tetrahedral shape except (b) Reactions related to depletion of ozone layer
(a) SO?' (b) XeF„ (c) Photosynthetic reactions in plants
(d) Reduction in the use and production of hazardous
(c) CIO; (d) XeO4
chemicals
17. The term ‘thio’ is used in the names of all of the
29. Which one of the following compounds is not a
following compounds except
protonic acid?
(a) Na2S2O3 (b) NaCS3
(a) SO(OH)2 (b) SO2(OH)2
(c) NaSCN (d) Na2SO3
(c) B(OH)3 (d) Po(OH)3
18. Cold solution of barium nitrite on mixing with
30. Which of the following gas mixture is used by the
sulphuric acid produces
divers inside the sea?
(a) BaSO4 +HNO2 (b) BaSO4 +HN03
(a) O2+He (b)O2+Xe
(c) BaSO4 +NO2 (d) BaSO4+ N2 +O2
(c) O2 + Ar (d) O2 + N2
19. Which of the following product is formed by the
31. A group 16 element exists in the monoatomic state in
reaction of sulphur dioxide with chlorine in presence
metallic lattice. It also exists in two crystalline forms.
of sunlight?
The metal is
(a) SO2C1 (b) SO2C12 (a) sulphur (b) polonium
(c) SOC12 (d) SO3C1 (c) selenium • (d) tellurium
20. Which of the following bonds has the least energy? 32. To a piece of charcoal sulphuric acid is added. Then
(a) Se—Se (b) Te—Te (a) there is no reaction
(c) S—S (d) 0-0 • (b) water gas is formed
Chapter 14: Group 16 (VIA—-The Chalcogens) | 609

(c) SO2 and CO2 are evolved 42. An experiment involving absorption of oxygen and its
(d) CO and SO2 are evolved quantitative estimation would involve use of
33. The correct order of 0—O bond length in O2, and (a) caustic soda (b) pyrogallol
(c) cone sulphuric acid (d) tupentine oil
O3 is
(a) HgOjz > O3 > O2 (b) O2 > > O3 43. Maximum coordination number of oxygen in liquid
(c) O3 > HA > O2 (d) O2 > O3 > H2O2 water is
(a) two (b) three
34. Which of the following is used in vulcanization of (c) four (d) five
rubber?
(a) SF6 (b) CF4 44. The colour of the red glass is traffic signals is due to
(0 C12F2 (d) (a) Sb2S3 (b) Hgl,
(c) CdS (d) CdSe
35. Aqueous solutions of hydrogen sulphide and sulphur
dioxide when mixed together yield 45. There is pit-dn multiple bonding in
(a) sulphur trioxide and water (a) CS2 (b) CO2
(b) presence of d-orbital (c) NO2 (d) NO
(c) sulphur and water 46. Ozone reacts with all the following reagents except
(d) hydrogen preroxide and sulphur one of the following
36. Which of the following have highest melting points? (a) K^MnQj
(a) p-block elements (b) s-block elements (b) moist iodine
(c) d-block elements (d) None of these (c) potassium ferrocyanide
(d) mercury
37. Electron affinity is positive, when
(a) 0 changes into 0" 47. The absorption of UV radiation by O3
(b) 0” changes into O2- (a) protects the inhabitants of our planet from
(c) 0 changes into O+ injurious radiation
(d) electron affinity is always negative (b) maintains an equilibrium between the
concentrations of O2 and O3
38. Consider the following compounds (c) makes both the function effective
1. Na2S2O3 (d) makes no function effective
2. Na2S4O6 48. Ozone layer is being depleted. This is due to
3. CaOCl2 (a) No emission from supersonic jets
Among these compounds identify those that have (b) chlorofluorocarbon used as aerosols
different oxidation states for same type of atoms (c) both (a) and (b)
(a) 1 and 3 (b) 1 and 3 (d) None of the above
(c) 1 and 2 (d) 1, 2 and 3 49. Estimation of ozone can be made quantitatively by
(a) decomposition into O2 and absorption of O2 into
39. Select correct statement(s). pyrogallol
(a) O3 molecule is resonance stabilised (b) volumetric method using KI and titration of the
(b) There is 3-centre, 4-electron rc-bond system in O3 liberated iodine using hypo solution
(c) Ozone layer is being depleted by fluorocarbons (c) oxidative ozonolysis method
(d) All the above are correct statements (d) all methods given above
40. Alkali ozonides are 50. Oxidative ozonolysis of 2-butene in presence of Zn or
(a) paramagnetic in nature dimethyl sulphide forms
(b) hydrolytically unstable and decompose to (a) CH3CHO (b) CH3COOH
superoxides (c) CO2 (d) (CH3)2CO
(c) decreasing in stability as size of alkali metal
51. The process of obtaining sulphur by the borehole
decreases
(d) having all the above statements as correct method is called
(a) the Frasch process
41. Among CH4, NH3 and l^O, acidicity of HjO is (b) the Lablanc process
maximum because (c) the Calcaroni process
(a) oxygen contains two lone pairs of electrons (d) the Mannheium process
(b) bond angle in H^O is less than 109°28' due to 52. When molten sulphur is suddenly cooled by pouring
VSEPR into water, it takes the form of
(c) oxygen is more electronegative than C or N (a) milk of sulphur (b) colloidal sulphur
(d) HgO is an associated liquid (c) flower of sulphur (d) plastic sulphur
610 | Essential Inorganic Chemistry

53. FeS+HCl ---- > A+B(g) 65. In SOI’


Gas B is passed into aqueous solution of C to form A (a) dn-pn bond between S and O is delocalised
Select correct statements based on the above facts (b) bonds between S and O are equivalents
(a) Colour of C changes from yellow to very light (c) there is sp3 hybridised sulphur atom
green (d) all of the facts given above are true
(b) Gas B turns lead acetate paper black 66. Liver of sulphur is a mixture of
(c) A form deep blue colour with K3[Fe(CN)6] (a) potassium pentasulphide (A) and potassium
(d) All the above facts are true thiosulphate (B)
54. Yellow oils of sulphur is/are (b) potassium carbonate (C) and (B) above
(a) HoS (b) H2S,,H2S3 (c) potassium disulphide (£>) and (B) above
(c) H2SO4 (d) CS2“NH2CSNH2 (d) (B) and (B) above
55. H2S precipitates aqueous ZnCl2 solution in 67. Bleaching of a fabric cloth is done using A and excess
(a) acidic medium (b) neutral medium of chlorine is removed using B is
(c) both (a) and (b) (d) None of these (a) CaOCl2,Na2SO3 (b) Na2S2O3, CaOCl2
56. In presence ofH2O,SO2 acts as (c) CaCl2,Na2S2O3 (d) CaOCl2,Na2S203
(a) an oxidising agent (b) a reducing agent 68. Na2S2O3 is used in photography for fixing the
(c) a hydrolysing agent (d) a redox reagent negative. It removes the AgBr by
57. SO2 behaves as a reducing agent when (a) complex formation Na3[Ag(S2O3)2 ]
(a) passed over hot CuO (b) oxidation of AgBr to Br2
(b) mixed with moist H2S (c) reduction of AgBr to Ag
(c) passed through acidified KMnO4 solution (d) formation of double salt
(d) passed through FeSO4 solution 69. Aqueous hyposolution on reaction with aqueous
58. When an article is bleached by SO2 it loses its colour. AgNO3 gives
The colour can be restored by (a) yellow ppt changing to black
(a) exposure to air (b) heating (b) white ppt changing to black
(c) dilution (d) None of these (c) orange ppt changing to blue
(d) no ppt
59. The compound of sulphur used as a solvent in rubber
industry is 70. S2O1~ has
(a) SO2(OH)C1 (b) SO2 (a) S—Slinkage (b) S—0 linkage
(c) SO3 (d) S2C12 (c) both (a) and (b) (d) None of these
60. S—S bond is present in
(a) a-(SO3)„ (b) y-(S3O9) Brain Twisters
(c) H2S2O3 (d) H2S2O8 1. Is the same amount of sulphuric acid needed io
61. The compound which has no reaction with KMnO4 is dissolve 40 g of nickel if we take concentrated acid in
(a) perdi sulphuric acid (b) sulphurous acid one case and dilute acid in another? How many grains
(c) hydrogen sulphide (d) hydrogen peroxide of sulphuric acid will go to oxidise the nickel in each
62. A bottle completely fillled with cone. HgSQj is left case?(Ni= 59)
unstoppered for several days and we observe 2. What mass of could be produced in the process
spontaneous overflow of acid. It is due to
given below if 1.00 kg of FeS2 is used? The unbalanced
(a) change in temperature
(b) hygroscopic nature of equations for the process are:
(c) absorption of air by H2SO4 FeS2 + O2 ---- > Fe2O3 + S02
(d) dehydration ofH2SO4
SO2 + O2 ---- > SO3
63. Out of H2S2O3, H2S4O6, H2SO5 and H^Og peroxy
acids are SO3 + H2SO4 ---- » H2S2O7
(a) HL^O^SA (b) H2S4O6)H2SO5 H2S2O7 + H^O---- > H2SO4
(c) H2SO5,H2S2O8 (d) H2S2O3 andH^Og
3. What is equivalent weight of Na2S2O3 • SHjO ?
64. Which of the following can convert acidified to
green? 4. Calculate number of moles of Na2S2O3 that will react
(a) SO-j/HjSOg/HjSO,'4 (b) SOa/HaSOg/HgS with Ij obtained when 1 mole of reacts with
(c) SO^/H^/Fe2* (d) S2O3"/SO3/Fe3+ excess of KI in acidic medium.
Chapter 14: Group 16 (VIA—The Chalcogens) | 611

5. Colourless salt (A) decolorises solution and gives Passage 3


white ppt (changing to black) with AgNO3 solution. Sodium thiosulphate is the laboratory reagent used in
(A) also changes FeCl3 (yellow) solution to FeCl2 iodometric and iodimetric titration. This also finds used in
(green). Identify (A) and explain reactions. photography.
6. A 1.1 g sample of copper ore is dissolved and Cu2+(aq) Based on the above passage, answer the following
questions.
is titrated with excess KI. The liberated requires
12.12 mL of 0.1 M Na2S2O3 for its titration. What is 1. In the following reaction
the % copper by mass in the ore? ^2 + 2S20g ---- > S40g + 21
equivalent mass ofS2O^- is (molar mass = M)
Passage Comprehension Questions (a) M (b) —
2
Passage 1 , v M ... M
(c) — (d) —
Peroxydisulphuric acid and its salts are powerful 3 4
oxidising agents, thus can be used to estimate reducing 2. If I2 is the titre in a titration A and is formed in situ
agents and also to study the kinetics of the reaction. in a titration B, then these are respectively
Answer the following questions. (a) iodometric, iodimetric
1. Peroxydisulphuric acid (b) iodimetric, iodometric
(a) is also called Marshall’s acid (c) both iodimetric
(b) has two peroxy linkages (d) both iodometric
(c) liberates L, with KI in a fast reaction 3. Sodium thiosulphate is called...... i and is used as ii
(d) has oxidation number of sulphur as +7 and removes unreacted light sensitive AgBr by
2. As compared to H2SO5, HjSA forming the complex
(a) has two peroxy linkage I ii III
(b) liberates Lj from KI solwly and thus kinetics of (a) hypo developer [AgS2O3]
oxidation of KI by S20|- can be studied
(c) has different oxidation number (b) hypo fixer [Ag(s2o3)2r
(d) reacts with KMn04 (c) hypo fixer [Ag2(S2O3)J
(d) none of the above is correct
Passage 2
Two acids have been derived from H2O2 by replacing H
Passage 4
by(SO2OH) group. Both the acids have one peroxy linkage
When metallic copper is heated with cone, sulphuric
H-O-O-H H—O-O—SO2-OH
H2SO5 acid, various types of products are formed depending on
sttandard reduction potential.
HO-SO2—0—0—S02 —OH Answer following questions based on above study.
H2S2O8
1. Which reaction is possible?
Based on the above study, answer the following (a) Cu + 2H+ ---- > Cu^+H,
questions. (b) Cu2++H> ---- > Cu + 2H+
1. Which is called Marshall’s acid and which is called (c) Both (a) & (b)
Caro’s acid? (d) None of the above
(a) I^SOg, H^A (b) HAOg, H^Og 2. When Cu reacts with cone. H^SC^
(c) H2SO5,H2S2O7 (d) H2S2O3,H2SO5 (a) Cu is oxidised to Cu2+
(b) S0^~ is reduced to S02
2. HjSjjOg is obtained at anode by the electrolysis of...... (c) both of the above reactions take place
(a) H2SO3 (b) HjSA (d) none of the above reaction take place
(c) H>S04 (d) HA andHjSC^ 3. There is also formation of CuS which is formed by
(a) SO^" +8H+ +8e" ---- > S2’ +^0
3. Which reacts with KMn04?
(b) S02 +6H+ +6e" ---- > H2S +2H2O
(a) H2SO3 (b) I^SOs (c) both (a) & (b)
(c) H>S04 (d) HAOg (d) None of the above
612 | Essential Inorganic Chemistry

Assertion & Reason 12. Assertion (A) : Structure of SC^“ is identical ’z


S2O| .
Codes:
(a) Both A and R are true and R is the correct Reason (R) : Sulphur atoms in S20^“ are ir
explanation of A different oxidation states.
(b) Both A and R are true but R is not the correct
explanation of A
(c) A is true but R is false.
True & False
(d) A is false but R is true. 1. Sulphur is an insulator while selenium and telluriuir
(e) A and R both are false. are semiconductor.
1. Assertion (A): Oxygen is the most abundant 2. Maximum catenation among non-metals is ii
element on the surface of earth. sulphur.
3.02, O2 are paramagnetic while O3, Oj' an
Reason (R) : Oxygen is paramagnetic with double
bond character. diamagnetic.
2. Assertion (A): Of all the elements of the Group-16, 4. There is one (S—S) bond in H^Og.
sulphur has the largest tendency to catenation. 5. S4N4 is thermochromic.
Reason (R) : Sulphur-sulphur bond energy is very 6. S4N4 is kinetically stable but endothermic compouni
high.
3. Assertion (A): S3O|” (trithionate) has S—S bond.
7. Peroxy linkage is present in oleum HjSgO?.
8. Caro’s is an oxidising agent.
Reason (R): It is a conjugate base of HS3Og.
9. SO3 is an electrophile in the sulphonation of benzene.
4. Assertion (A): O2 gas is effected in magnetic field.
10. Highest fluorides of oxygen is OF2 whereas sulphui
Reason (R): It has north and south pole. forms SF6.
5. Assertion (A): SO3 is a nucleophile. 11. KO2 is paramagnetic.
Reason (R): SO3 has lone-pair of electrons. 12. Bond order of O2 is 1.5.
6. Assertion (A) : SO2 turns acidified K^Ci^O? green. 13. KO2,BaO2,Na2O2 are superoxides.
Reason (R): is reduced to Cr3+ (green)salt 14. SO2 and CO2 both turns lime water milky.
bySO2.
15. Oxygen is by far the most abundant metal on th:
7. Assertion (A): SO2 is a bleaching agent. earth’s crust.
Reason (R): Nascent oxygen causes bleaching.
8. Assertion (A) : One mole of SO2C12 in aqueous Fill in the Blanks
1. Paramagnetic behaviour is observed in OU
solution is neutralised by 3 moles of NaOH.
ofO3,O2, NO and NO2.
Reason (R) : 3 moles of H+ are formed due to
reaction with water. 2. Highly electrical conducting behaviour is present ii
anhydrous H2SO4 due to
9. Assertion (A): Anhydrous sulphuric acid has very
high viscosity. 3. Anhydrous HjSO,! has high viscosity due to.........
Reason (R): H-bonding makes it viscous. 4. Peroxy linkage is present in acid
10. Assertion (A) : Anhydrous H2SO4 undergoes 5. When Marshall’s acid is hydrolysed there is formatioi
autoprotolysis of and
H2SO4 +H2SO4 h^o; +HSO; 6. (S—S) bond is present in the acid
Reason (R) : Anhydrous HjSC^ has very high 7. Tetrasulphur tetranitride is
electrical conductivity. 8. In group 16 maximum catenation is in
11. Assertion (A) : In aqueous solution, SO2 reacts 9. Common oxidaton states of group 16 elements aj
with H^S forming sulphur.
Reason (R): SO2 is an effective reducing agent. 10. Unlike sulphur which is an selenium ac
tellerium are
Chapter 14: Group 16 (VIA—The Chalcogens) | 613

H. Superoxide ion is and is in Set I


character. i.O2 is paramagnetic due to unpaired
12. Bond orders of O2, O2,0^", O2 and 0^+ in increasing electrons.
order are
2. SO2 reduces to Cr3+; change in oxidation
number of Cr is
Matrix-Match Type Questions
3. Maximum oxidation state shown by sulphur in its
I. One or More Correct compounds is
1. Match the species in Column I with their 4. There are equal number of sulphur atoms in
corresponding properties in Column II. thiosulphuric acid, Marshall’s acid and oleum. This
number is
Column I Column II
5. SO2 reduces MnO4 to Mn2+. Number of electrons
A. so2 1. Oxidant involved in the reduction ofMnO4 toMn2+ is
B. so3 2. Reductant
C. O3 3. Resonance stabilised
6. SO2 turns lime water milky, 3.2 g of SO2 gives
D. h2s 4. sp2 hybridisation milkyness g.
E. h2o 5. sp3 hybridisation 7. One mole of SO2C12 on hydrolysis gives acids which
6. Maximum boiling point can be neutralised by moles of NaOH.
7 Minimum boiling point 8. Ozonolysis of 2,5-dimethyl-2,4-hex-diene gives total
number of carbonyl compounds
2. Match the acids in Column I with their corresponding
properties in Column II.
Set II
Column I Column II
1. 4.35 g of pyrolusite (MnO2 mineral) is digested with
A. H2SO5 1. Hydrolysed to give H2O2 cone. HC1 and KI added. Mixture required 5 mL of 1M
B. h2so4 2. Hydrolysed to give H2SO4 Na2S2O3 solution for complete reaction of KI3 formed.
C. H2S2O7 3. S—S bond present What is % of pure MnO2 in pyrolusite? (Mn = 55)
D. H2S2O3 4. Peroxy linkage present
2. How many S—S bonds are there in S3O9?
E. H2S2O8 5. S in different oxidation
state 3. There are two types of sulphur with different
6. O in different oxidation oxidation states in thiosulphate ion. Difference in
state oxidation states is
7, Hygroscopic
4. Acid rain is due to dissolved non-metallic oxides
(SO2, NO2, P2O5, etc). 10 mL of acid rain sample
Integer Answer Types required 5 mL of 0.01 M Ca(OH)2 for making it
This section contains 8 questions. X Y z w neutral. What is pH of acid rain?
The answer to each of the questions is a ® © ©j 5. How many SOS angles are there in S3O9?
single digit integer, ranging from 0 to 9. ® O
The appropriate bubbles below the 6. How many of the following have peroxy linkages?
® ®
respectively question numbers in the ® ® ®_ H2S2O3, H^Og, H2SO5, CrO5, BaO2, KO2
ORS have to be darkened. For example,
if the correct answers to question
® ® 7. There is also formation of insoluble CuS (black) when
number X, Y, Z and W (say) are 6, 0, 9
® © ® ®_ copper reacts with cone. HoSO4. What is change in
and 2, respectively, then the correct ® ® ® oxidation number of sulphur?
darkening of bubbles will like the 8. How many of the following can be the oxidising agent?
following ® ® ® ®
H2S, H2SO4, H^SO^j, H2S2O3
® ® ®
Answers & Solutions
Master Exercises
Exercise 1
Short Answer Type Questions Analytical Questions
1. (a) HjS (b) SO2,H2SO3 1. (a) O3 + 21" + 2H+ ---- > O2 +1■22 + H2O
(c) H2SO4,H2SO5,H2S2O8,SO3 (b) 2[Fe(CN)6]4" + O3 + H2O---- > 2[Fe(CN)6]|3-
:
2. Cl, Br and I atoms are two bulky around the sulphur + 20H-+0,'2
atom. (c) S + 3O3 + H2O---- > H2SO4 + 30.2
3. It absorbs UV light. It is also used as a disinfectant to 2. radius = 4000 miles
purify drinking water, since it destroys bacteria and .. 4000 . 4xl06
viruses. Its advantage over Cl2 for this purpose is that it 4000 miles =-------- km =-------- - m
0.6214 0.6214
avoids the unpleasant smell and taste of chlorine since
any excess O3 soon decompose to O2. thickness =0.3 cm =0.003 m
Volume of O3 molecules at the surface = 4nr2 thickness
4. No change
22 r4xl06V x 0.003 m3
5. N O +O3 ---- > N O2 +O. =4 x — x
T2 7 0.6214,
+2-2 0 +4-2 o
V = 1.56273 xlO12 m3
6. 1000 mL 03»
o, 6.44 kJ P = 1 atm = 1.01325 x 105 Pascal
,, i Pv
3xO2 2xO3 Moles = n = ——
PT
(3x -2x) = x = 500 mL
_ 1.01325 x 105 x 1.56273 x 1012
Volume of formed O, = 1000 mL = mol “ 8.3143 x273
3 22400
= 0.0446 mol = 6.976 xlO13
Molecules = nN0 = 6.976 x 1013 x 6.02 x 1023
Heat absorbed = 144.2 x 0.0446 kJ = 6.44 kJ
= 4.2 x 1037
7. (A): FeS (BJrHjS (C):FeSO4 (D): PbS
3. (i) Yes, Fe3+ ions will oxidise H2S to S
(E): K Feu[FenI(CN)g] Turnbull's blue
2Fe3+ + H2S---- > 2Fe2+ + 2H+ + S
MnO4 + 5Fe2+ + 8H+ ---- > 5Fe3+ + Mn2+ + 4H2O
colourless Since, = 0.77-0.14 = 0.63 V
8. MnO4 + H+ + HjjS---- > S + Mnz+, HjS is a reducing and thus &G° = -nFE°M = -ve
agent (ii) Yes, Br2 also oxidises H2S to S
+ H2O2 ---- > H2O + H2SO4, H2S is a reducing agent Br2 + H2S---- > 2Br + 2H+ + S
HgS + NaOH---- > NaHS---- » NagS, H2S is a diprotic Since, E° = 1.07 -0.14 = 0.93V
acid. and AG° = - ve
9. (A): NaHSO.l3 (B):Na2SO3 (C): SO2 (P):BaSO3u Note : A cell reaction is spontaneous
(E): CaSO3 (F):Ca(HSO3)2 (G):Cr3+ (H):BaSO4 if E°ell = + ve and AG° = - ve

10. P2O5
(Take values as per half-reactions)
11. HgS is oxidised to S by (which is reduced to SO2) 4. Refer Text.
H2>S + ---- > 2H2O + SO2 + S 5. (a) Acid strength increases as the number of 0-atoms
while NH3 forms salt with H2SO4 increases.
(b) In comparison with S, O is much too electronegative
2NH3+H2SO4 ---- > (NH4)2SO4
to form compounds of O in +4 oxidation state. Also
hence, can’t be used for drying up these gases. uses sp3d2-hybrid orbitals for bonding in SF4 but 0
Chapter 14: Group 16 (VIA—The Chalcogens) | 615

does not have valence d-orbitals and it cannot form 7. (a) NajSA +2HC1 2NaCl + SO2 + S + H2O
four bonds to F. If NaoS203 reacts with HC1, there is formation of
(c) Each S is sp3-hybridised with two lone pair of precipitate (S) — thus (2) and formation of gas (SO2) —
electrons. The bond angles are 109.5°. A planar ring thus (3)
would require bond angles of (a) — 2, 3
= (180-—"| = 135;o
(b) None
I 8 ) (c) None
6. cio;4 Cl Change in oxidation number (d) Na^A + I2 > .2NaI + Na_2S4O6
+7 -1 8 units violet colourless
SO4 —> SO*" Thus, initial substance is decolourised (1)
+4 4 +6 4 2 units Thus, (d) — 1
Thus, CIO; 4SO.2 ---- > 4SO4" + Cl"
cio; + 4SO (e) KMnO4 (pink-violet) is also decolourised.
Thus, (e) — 1
Since, 1 mol HC1O4 is reduced by = 4 mol SO2
8. Na + O2 NaO2
16.9 superoxide
hence, 16.9 g = mole HC104 is reduced by
100.5 1 mol 1 mol 1 mol
4 x 16.9 23 g 32 g 55 g
mol SO2 1.15 g = 0.05 mol 0.05 mol
100.5
9 75
2.75 g =-— = 0.05 mol
= 0.673 mol 55
1 mol of STP = 22.4 L SO2 (i) O2 combined = 0.05 mol = 1.6 g
(ii) O2 = 0.05 mol = 0.10 mol O-atoms
Thus, 0.673 mol at STP = 22.4 x 0.673
(hi) sodium used = 0.05 mol
= 15.067 L (iv) superoxide = NaO2

Exercise 2
Only One Option Correct
1. (0 2. (b) 3. (d) 4. (a) 5. (d) 6. (d) •7. (a) 8. (d) 9. (d) 10. (c)
11. (a) 12. (d) 13. (d) 14. (a) 15. (a) 16. (b) • 17.(d) 18. (a) 19. (b) 20. (b)
21. (b) 22. (d) 23. (b) 24. (d) 25. (b) 26. (d) 27. (d) 28. (d) 29. (c) 30. (a)
31. (b) 32. (c) 33. (a) 34. (a) 35. (c) 36. (a) 37. (b) 38. (d) 39. (d) 40. (d)
41. (c) 42. (b) 43. (c) 44. (d) 45. (a) 46. (b) 47. (c) 48. (c) 49. (b) 50. (a)
51. (a) 52. (d) 53. (d) 54. (b) 55. (b) 56. (b) 57. (c) 58. (a) 59. (d) 60. (c)
61. (a) 62. (b) 63. (c) 64. (c) 65. (d) 66. (d) 67. (d) 68. (a) 69. (b) 70. (c)

Brain Twisters 4[SO3 +H2SO4


1. Ni +H2SO4 NiSO4 + H2 41H.S A + H,0 2H2SO4]
(dil.) Thus, 2FeS2 h4H2SO4
59 g 98 g
2[56 + 64] g =4 x 98 g
40 g= 66.44 g ITSO4 FeS2
Ni + 2H,SO
2
4---- > NiSO44 + 2H2*
4
2O+ SO2 4x98
(cone.) 1000 gFeS2 give = x 1000 g
59 g 2 x 98 g 240
= 1633.33 g
40 g= 132. 88 g H2SO4
= 1.6333 kg
Thus, amount of dil. H2SO4 and cone. to dissolve
40 g Ni is different. 3. 2Na2S2O3 +12 —> Na^A + 2NaI
2. 2FeS2 + O2 ---- > Fe2O3 + 4SO2 2Na2S2O3 =I2 =21

4SO2 + O2 ---- » 4SO3 Thus, I = Na^A


616 | Essential Inorganic Chemistry

Equivalent weight of Na^Og -5H2O a molar mass Passage 3


= 248 g 1. (a) 2. (b) 3. (b)
3. 2Na2S2O3 +12 2NaI + Na^Og Passage 4
2Na2S3O3 = I2 = 2I 1. (b) 2. (c) 3. (a)
Or NajjSgOg = I
Assertion & Reason
Thus equivalent weight of (Na^Og -5H2O) = mol. wt.
1. (b) 2.2. (a)
(a) 3. (b) 4. (c) 5. (e) 6. (a)
= 248.0
2S2O2" ---- > S4O2- + 2e~ 7. (c) 8.
8. (e)
(e) 9. (a) 10. (b) 11. (b) 12. (b)
Or in terms of electrons :
S V,23 ’ = le“
°29O True & False
molecular weight
equivalent weight = 1. T 2. Carbon has maximum catenation 3. T
electrons gained/lost
4. there is (O—0) bond in H2S2O8
= molecular weight
4. Cr2O2’ + 14H+ + 6r ---- > 2Cr3+ + 7H2O + 3I2 5. T 6. T
7. H2S2O7 (oleum) is SO3 saturated H2SO4 (no peroxy
---- > SNa^Og + 6NaI
3I2 + 6Na2S2O'33 ~
linkage)
1 mol K2Cr2O7 KI 3 mol I2 == 6 mol Na^Og
8. T 9. T 10. T 11. T 12. T
5. (AlcNa^O-j 13. Only KO2 is superoxide BaO2, Na2O2 are peroxides
Na-jSjOg + 2AgNO3 AgrjSjOgi + 2NaNO3 14. T
white ppt
15. it is not metal, it is non-metal.
A&2®2®3 '*'^2^ H^+AggSX
black Fill in the Blanks
2FeCl3 + 2Na2S2O3 2FeCl2 +Na2S4O6 +2NaCl 1. O2, NO and NO2
yellow green
2. Autoprotolysis (HgSO,, + H2SO4 h3so; +hso;i

6. 2Cu (-'u2^2 + ^2 3. Strong hydrogen bonding

I22 + 2S2O!" ---- > S4O!" + 21" 4. H2SO5 and H2S2O'8(

Equivalent mass of Cu2+ = molar mass 5. H2SO4,H2O2


Equivalent mass of S2O3“ = molar mass 6. H2S2O3 (thiosulphuric acid)
Thus, millimoles of Cu2+ = millimoles of S2O3“
7. thermochromic
= 12.12x0.1
8. sulphur
= 1.212 millmol
9. -2,+4,+6
= 1.212 x 10'3 mol
10. insulator, semiconductors
= 1.212 x 10-3 x 63.5 g pure Cu
11. O2, paramagnetic
= 0.076962 g in 1.1 g Cu - ore
0.076962 12. O2’(l)<O2(1.5)<O2(2) <0,(2.5)<O2+(3)
Thus, % of copper = xlOO
1.1
= 7.0% Matrix-Match Type Questions
I. One or More Correct
Passage Comprehension Questions 1. (A) —(1,2,3,4); (B) — (1,3,4); (O-(1,3,4);
Passage 1 (D)-(2,5,7) (E) —(5,6)
1. (a) 2. (b) 2. (A) — (1,2,4,6); (B) — (7); (C) —(2,7);
Passage 2 (D) —(3,5); (E)- (1,2,4,6)
1. (b) 2. (c) 3. (a)
Chapter 14: Group 16 (VIA—The Chalcogens) | 617

Integer Answer Types Set II


Set I Questions 1 2 3 4 5 6 7_ 8
Answers 5 0 8 2 3 4 8 2
Questions 1 2 3 4 5 _6 7_ 8
Answers
© © © © © ©I© ©
2 6 6 2 5 6 4 3
© 0 O G) GW
© © © © © ©I© © © © © © © ©© ©
© 0 © O G) © © © © © ©
© © © © © © © © © © © ©
© © © © © © © © © ©© ©
© ©
© © © © © (4) (4) © © © © © © ©
© © © © © ©©, © 0 0 0
© © © © © © © © © © © f® ©
© © 0 0 0 © © © © © © © ©
© © © © © >_© ©
© © © © © ® © © Hints
1. MnO2+4HCl MnCl2 + Cl2
Hints
Cl2 + 2KI ---- >
2. Cr2O3- ---- > 2Cr3+
? charge I2 + 2Na2S2O3 ---- > 2NaI + Na^Og
T 6 6 units MnO2 =C12 sl2 ^Na^Og
12

6. SO2 + Ca(OH).2 CaSCL Moles of Na^jOjj = — = 5xlOJ


64g 120 g3 1000
3.2 g 6g Moles ofMnO2 =2.5 xlO-3
Pure MnO2 = 2.5 x 10”3 mol
7. S02C12+2H20 2HC1 + H2SO4
= 2.5 x 10'3 x 87 g
Total4H+ = 0.2175 g in 4.35 g impure
s 4NaOH sample
8. C-C=i=C—C^=C—c —> % of pure MnO2 = 2^x100
4.35
ch3| |ch3 = 5%
these bonds break 4. 10 mL of (H+)s5 mL of 0.01 MCa(OH)2
hence, three products.
= 5 mL of 0.02 NCa(OH)2
2V1(H+) = 10-2 N
pH = -log[H+] = 2
= -log 10’2=2
"A unique group of non-metals in different molecular
forms. F2,CI2 (gas), Br2 (liquid) and l2 (solid)."
I
Group 17
(VII A—The Halogen Family)

Gradation of Physical Properties


• The name halogen comes from the Greek, and means salt 11 giniewoi
former. All the elements react with metals to form salts, and
they are also very reactive with non-metals. Fluorine is the most I hapter’SBM
reactive element known.
■ Gradation of Physical Properties 9F
Stable isotope
1*F, 19F (tracer) ■ Study of Halogens (X2)
Fluorine 17ci
■ Interhalogen Compounds
Chlorine 35 Cl (75%), 37 Cl (25%)
35Br
Bromine 79Br (50.7%), 81Br (49.3%) ■ Pseudohalogens and
______ Iodine 127l Pseudohalides 53*

• All the elements have seven electrons in their outer shell and ■ Anomalous Behaviour of
85At
thus complete their octet by gaining one electron (in ionic Fluorine
compounds as in NaCl) or by sharing electrons (thus forming ■ Halogen Acids
covalent bonds as in CC14). Physical properties of the elements ■ Oxoacids of Halogen
are summarised in Table 15.1 and Fig. 15.1 ■ Oxides of Halogen
Physical properties like melting and boiling points are related
to the size and mass of the molecules as well as intermolecular
attraction. The effect of size and mass, which gradually increase as
we go down the group, is easy to understand. As for the
intermolecular attraction in non-polar homonuclear diatomic
molecules like halogens, which do not have any permanent polarity,
the only forces of attraction are the weak van der Waals’ forces. The
polarisability of halogens increases as we go down the group, being
maximum in iodine and least in fluorine. So, van der Waals’ forces of
attraction are maximum in iodine and least in fluorine, with
bromine and chlorine coming in between. As a consequence of this,
fluorine and chlorine are gases at ordinary temperature, bromine is
a liquid and iodine a solid. This is also reflected in the trends
observed in their enthalpies of fusion and vaporisation.
620 | Essential Inorganic Chemistry

As we move along a period, the effective nuclear the fact that fluorine atom is very small and the
charge increases reaching a maximum at the noble gases. non-bonding electrons on fluorine are nearer to each
Halogens which immediately precede the noble gases have other, resulting in a much greater lone pair-lone par­
a very high effective nuclear charge coupled with small repulsion, which weakens the covalent bond and lower*
its dissociation energy. This repulsion is not so great
size and thus have the highest ionisation energies in the relatively larger halogen molecules like chlorine,
respective periods, next only to the noble gases. Like the bromine and iodine where the lone pairs are at a greater-
trend in other groups, ionisation energy of halogens also average distance from each other.
decreases in going down the group from fluorine«o iodine.
• The halogens show very close group similarities
g
£- Fluorine differs in several ways from the rest of ths
£ a2 I3* Is-
fl
co
J £ fl o group due to :
Electrons 2 *f11- - £ sE
o
- smaller in size,
in
Size of
tn
■§
Q *5
*Ii o„ - non availability of d-orbitals which may be usee
valence aI § * fa

s a
'fl 12
shell x X* <>2 § <2 (EN) (EA) M K X w O for bonding.
F 7 Fluorine is the most reactive of all the halogens.
*0
5- fa N

Cl 7 N A 6 The difference in reactivity between fluorine and chlorine


'S
S! is greater than between chlorine and bromine.
c § a
7
Br
7
tSD
a
J
M
§ B. a £3
• HF has a high boiling point (19.5°C) as a result of
I £ strong intermolecular hydrogen bonding,
Halogens have seven electrons in their valence shell, whereas all other hydrogen halides have much lower
they have a very strong tendency of gaining an electron to boiling point (Fig. 15.2)
acquire a stable noble gas configuration. Therefore, they
have very high electron affinities. In fact, their electron Table 15.1 Physical Properties of the Halogens
affinities are the highest in their respective periods. Their I
Properties F Cl Br I
electron affinity follows the order Cl > F > Br > I. As
Atomic number 9 17 35 53
explained earlier, the smaller electron affinity of
fluorine than that of chlorine is a consequence of its Electronic [He] [Ne] [Ar] [Kr]
configuration 3d10-4s24p5 4d'°5s25p5
small size. 2s22p5 3s2 3p5
As we go across the p-block elements in a period, 4.0 3.0 2.8 2.5
Electronegativity
the electronegativity increases reaching a maximum (Pauling)
at the halogen group. Thus, halogens are the most 1009
(/E)/kJ mol- 1681 1256 1143
electronegative elements in their respective periods.
EA/kJ moi -333 -349 -325 -296
Electronegativity decreases on moving down a group,
making fluorine the most and iodine the least Hydration energy of -515 -381 -347 -305
electronegative among the halogens. X‘/kJ mol-
Atomic radius/pm 72 99 114 133
220
DEEP Focus Ionic radius(X-M>m 133 184 196

In going down the group from chlorine to iodine, the X—X Enthalpy of atomisation 79.1 122 111 106
bond dissociation energy gradually decreases. This is easily (kJ mol-1)
explained by considering once again the size factor. In the X2 + 2e~ ->2X~E°/V 2.87 1.40 1.09 0.62
chlorine molecule, which is the smallest of the three, viz. Cl2,
Br2,12, the two bonding electrons are nearer to both the nuclei Bond energy (X—X)/kJ 126 210 158 118
and are held strongly, while in bromine and iodine, the distance mol-
of the bonding electrons from the nuclei gradually increases Melting point (X)/°C -219 -101 -7 114
resulting in lesser attraction and consequent weakening of the
bond. Furthermore, as the size of the atom increases, it results Boiling point (X)/°C -188 -34 60 185
in a less effective overlap of the orbitals and therefore, Density of solid X2 1.3 1.9 3.4 4.93
progressively weaker bonds are formed as we go down the (g/cc)
group. The bond dissociation energy increases in the order
I2 <Br2 <Cl2 and if this trend was to continue, we should Oxidising power (X) most least

expect the F—F bond dissociation energy to be greater Common oxidation -1 -1, +1. -1.+1.+3, -1.+1.+3.
than the bond energy of chlorine, 210 kJ mol’1. But this is state +3, +5, +5, +7 +5. +7
not so. The actual bond dissociation energy of fluorine +7
molecule is, however, surprisingly low and has the value Physical form (X2) at pale yellow dark red violet-black
of 126 kJ mol’1 only. The anomalously low bond room temperature yellow green liquid solid
dissociation energy of fluorine molecule is attributed to gas gas
Chapter 15: Group 17 (VIIA—The Halogen Family) | 621

2000-r
3]^ +6H2SO4 • SO3 ■> 2I3 + 2HS3Or0 + 5H2SO4
i
oleum
3- —
1 + SO2
Pauling kJ moP1 \
1
units 2 • Electrical conductivity of molten iodine is due to the
1000------- presence of(I3 + I3) species produced by self ionisation
‘I
of iodine:
1-- 312 I3+ + I3

0 0
F 'Cl 1 Br' I 1 At' F 'Cl 1 Br1 I 1 At'
(a) (b)
Study of Halogens (X2)
• F2 is extremely reactive, and this causes great
200-r difficulties in the preparation and handling of the
element.
160-- - Moissan (1886) prepared it from CaF2 :
120---- CaF2 + H2SO4 -----> CaSO4 + 2HF
kJ mol' KF + HF---- > KHF2
80-- electrolysis^
KHF2 H2 + F,
at cathode at anode
40--
Electrodes : Pt-Ir alloy
0 Working temperature : -40° C to -23° C (using
F 'Cl' Br' I ' At'
boiling CH3CI,-23° C)
(c) - Dennis and others (1931) also prepared F2 by using
Fig. 15.1 Some group trends of the group 17 elements (a) fused KHF2 with graphite electrodes in a copper
Pauling electronegativity, (b) The first ionisation energy, and vessel.
(c) Bond enthalpy in the halogens X2. - Modem method uses graphite anode and steel
• HF is a weak acid, whereas all other hydrohalic acids cathode in a steam heated steel tank.
(HC1, HBr and HI) are strong acids. - If F2 were formed by the electrolysis of aqueous
fluoride solution, it would immediately oxidise
• F2 reacts with cold NaOH solution to produce OF2
(oxygen difluoride) gas water to Qz
F2 + 2H2O---- > 4HF + Oa
2F2 + 2NaOH---- > 2NaF + H2O + OF2
Most recent method of the preparation of F2 involves
The same reaction with chlorine or bromine produces
reaction
a halides (X") and hypohalite (XO“)
K2MnF6 +2SbF5 2KSbF6 +MnF3+iF2
X2 + 2NaOH---- > NaX + NaXO + H2O
X stands for Cl or Br. Iodine (I2) does not react under • The commercial production of Cl2 is by electrolysis,
the same condition. either of concentrated aqueous NaCl or of molten salt
• AgF is soluble in water while other silver halides such as NaCl(Z) or MgCl2(Z):
(AgCl, AgBr and Agl) are insoluble.
• Chlorine, bromine and iodine occur as halides in 2NaCl + 2H2O electrolysis> 2NaOH + H2 + Cl2
sea-water, and fluorine occur in the mineral fluorite at cathode at anode
(CaF2) and cryolite (Na3A1F6)
• Fluorine is the most electronegative element and has This process is called chlor-alkali process
no basic properties. Chlorine shows a slight tendency
to from cations. C1F ionises to form Cl+ and F“ (due to DEEP Focus
higher electronegativity of fluorine) This reaction yields two useful by-products, NaOH and H,.
C1F---- > Cl++F‘ The cells are designed to separate the molecular chlorine
• Electrolysis of ICN in pyridine solution gives iodine at from the sodium hydroxide solution and the molecular
hydrogen to prevent side reactions such as
the cathode. This indicates that ICN ionises to give I*
2NaOH + Cl2 ---- > NaOCl + NaCl + H2O
andCN".
H2 + Cl” ---- > 2HC1
• Iodine dissolves in oleum giving a bright blue solution
The reaction between H2 and Cl2 is explosive.
which has been shown to have I2 and I3.
2NaCl(Z) electrol.vsis> 2Na + Cl2
2l2+6H2SO4 SO3---- > 2I2 + 2HS3O[0 + 5H2SO4 + SQj
(molten) at cathode at anode
oleum
622 | Essential Inorganic Chemistry

MnO2
• Cl2 is also prepared (in laboratory) by heating cone.
I cone. HCI
HCI with KMnO4 or Mn0.2 (both oxidising agent): --------- electrolysis
cone. HCI
KMnO4 Cl2 <----------- NaCI (aqueous or molte*-
2MnO4 + lOCl" + 16H+ ----- > 2Mn2+ + 5C12 + 8H2O
Br"
MnQ2 + 4H+ + 2C1- ----- > Mn2+ + 2H2O + Cl2 OH"
Br2 Br“ + BrOa
• Br2 is obtained from sea water and brine lakes. Sea
H*
water contains about 65 ppm Br". Br2 is isolated in I"
following steps : l2
- First sea water is adjusted to pH 3.5 and treated “107
HSO3
with Cl2 which oxidises Br- to Br2 r ■ 105

Cl2 + 2Br- ----- > Br2 + 2C1- in chile salt petre

- Br2 is quite volatile hence is removed by a stream of • Oxidising power is in order :


air. F2 > Cl2 > Br2 > I2
The oxidation potential is the energy change between
the elements in its standard state, and its hydrated
DEEP Focus
ions and summarised in the Table 15.2.
~ Br2 is absorbed into aqueous Na2CO3 solution when
X2 (solid, liquid, gas) ---- > X"(hydrated)
NaBr and NaBrO3 are formed :
3Na2CO3 + 3Br2 ---- > 5NaBr + NaBrO3 + 3CO2
Or 3Br2 + 6OH- ---- > 5Br- + BrO" + 3H2O Table 15.2 Free Energy (AG°) Values for X2 —> X'
~ Finally the solution is acidified and distilled to give (hydrated) (All values in kJ mol-1)
pureBr2 :
1/2 Free 1/2 Free Free
5NaBr + NaBrO3 + 3H2SO4 ---- > 5HBr + HBrO3 1/2 Free Sum of
i energy of energy of Electron
energy of energy of
+ 3Na2SO4 evaporati dissociat affinity AG’
fusion hydration
on ion
5HBr + HBr(X V
3Br2 + 3H2O
F2 — 126/2 -333 -460 -730
or 5Br" + BrOZ + 6H+ 3Br2 + 3H2O
0
Cl2 — 210/2 -349 -348 -592
Br2 31/2 158/2 -325 -318 -548.5
• I2 is obtainable in small quantities from dried -486.5
h 15/2 44/2 118/2 -296 -279
seaweed, since certain marine plants absorb and
concentrate I- selectively in presence of Cl" and Br".
Low concentration ofl" are also found in some natural DEEP Focus
F2 has the most negative AG° value, hence is strongest
brines (salt solutions) associated with oil fields.
oxidising agent and can oxidise Cl", Br" and I" to respectively
Chile saltpetre is mainly NaNO3 but it contains Cl2,Br2 andl2. F2 can also oxidise H2O to O2
traces of sodium iodate NaIO3 and sodium F2 +H2O---- > 2HF+|O2
periodate NaIO4. is recovered by reducingIO3
to 1“ by NaHSO3 and then oxidation ofl- to Lj by SimilarlyCl2 will oxidiseBr" andl", andBr2 will oxidise I".
In general any halogen (X2) of lower atomic number will
IO3 itself:
oxidise halide ions (X") of higher atomic number.
IO3 + 3HSO3 ----- > r + 3SC^- + 3H+
Some reactions of the halogens are summarised in
5F + IO3 + 6H+ ----- > 31a + 3H2O Table 15.3.
Natural brine is treated with Cl2 water to oxidise I- to DEEP Focus
I2 and the solution is passed through an ion-exchange • Cl 2 is used as a bleach or as a disinfectant (as in public
resin which adsorbs I2 as I3.13 is removed from the water supplies). It reacts slowly with H2O to form HCI and
resin by treatment with alkali. HOC1. The hypochlorous acid then decomposes into HCI
and 0 radicals, which kill bacteria.
- Reactivity of halogens is based on bond-energy (BE) Cl2 +H2O HC1 + HOC1
value: HOC1---- > HC1+ :o-
F2 > Cl2 > Br2 > I2 These oxygen radicals are very strong oxidising as well as
Greater the BE, smaller the reactivity. effective bleaching and disinfecting agent in aqueous
solution ofCl2 or hypochlorite salts.
Chapter 15: Group 17 (V1IA—The Halogen Family) | 623

• Iodine is necessary for the normal functioning of the and thus for reverse reaction AG° = -105 kJ mol-1,
thyroid gland. Insufficient iodine in the diet leads to hence reverse reaction occurs spontaneously and even
Goitre (enlargement of the thyroid gland) ‘Iodised atmospheric O2 oxidises 1“ to I2.
salt’ contains KI/KIO3.
• For I2 + H2O 2H+ +2F 4 O2, 2HI+5O2 * H2O +I2
2 2
AG° = 105 kJ mol

Table 15.3 Some Reactions of Halogen


Reaction Comment
• With HjO
2F2 + 2H2O > 4H* + 4F" + O2 Vigorous reaction with F2; atmospheric O2 can oxidise r to l2 hence,
2X2 + 2H2O > 4H* + 4X“ + O2 reverse reaction Cl2 > Br2 > l2 (F2 does not disproportionate).
X2 + H2o x- + HOX+ H*
• With Hj
H2 + X2 -----> 2HX All the halogens, with Br2 photochemical reaction, with l2 very slow
even at high temperature.
• With metals
2M+nX2 ----- > 2MXn Most metals form halides.
• With CO
CO+X2 —> COX2 Only Cl2, Br2 form carbonyl halide
• With P
2P + 3X2 -----> 2PX3 For F, Cl, Br, I
2P + 5X2 -----> 2PX5 For F, Cl, Br
• With S
2S + X2 ----- > S2X2 Cl. Br
S + 2CI2 -----> SCI4
S + 3F2 -----> SF6
• With H2S
H2S+X2 —> 2HX +S All the halogens oxidise H2S (S2*) to S
• WithSO2
SO2 + X2 SO2X2 F and Cl
• WlthNH3
8NH3 + 3X2 ----- > N2 + 6NH4X F, Cl. Br
»With halogens
X2 + X'2 2XX' Interhalogen compounds
• With cold NaOH
2NaOH + 2F2 -----> 2NaF + OF2 + H2O
2NaOH +X2 —> NaX + NaOX + H2O (X = Cl, Br) l2 does not react under the same condition.
Clj reacts with dry slaked lime forming bleaching powder
Ca(OH)2 + Cl2 —> CaCI(OCI) + H2O
• With hot NaOH
6NaOH + 3X2 5NaX + NaXO3 + 3H2O (X=CI, Br, I)

Colour of Halogens All these colours may be assigned to transition of an


In gaseous phase, colours of the halogens vary electron from the highest occupied it* molecular orbital to
remarkably: the lowest occupied o * molecular orbital. As one moves
F2 — pale yellow from F2 to I2, the difference between the two energy levels
decreases and probability of this transition also increases
Cl2 — pale greenish yellow at the same time. Hence, the absorption maximum
Br2 — deep red-brown gradually shifts from the near UV to the red region of the
La — deep violet
624 | Essential Inorganic Chemistry

visible spectrum; the intensity of absorption also increases H2O


ic: ■> Cl + IO3
side by side. Hence, we observe the complimentary colour iodate ion
with increasing intensity. h2o io-
* cr + hypoiodite ion
Interhalogen Compounds - Interhalogen compounds can fluorinate ma:
• The halogens react with each other to form metal oxides, metal halides and metals :
interhalogen compounds of the type AX’, AX& AX5 3UO2 + 4BrF3 ---- > 3UF4 + 2Br2 + 302
and AX7 of which A is of higher atomic number as UF4 + C1F3 ---- > UF6 + GIF
shown : Some interhalogen compounds are summarised belc
AX ax3 AXS ax7 in Table 15.4
CIF CIF3 cif5 Table 15.4 Interhalogen Compounds and their Properties
BrF BrF3 BrF5
Type of Physical state
ICI ICl3 (l2CI6) if5 if7 Formulae and
inter­ and colour at Melting Bolling
IBr names of ordinary point (K) point (K
halogen
compounds temperatures
BrCI compound
XY CIF Gas, colourless 119 163
• There are never more than two different halogens in a
molecule. The bonds are covalent because of the small Chlorine
fluoride
electronegativity difference. Ternary combinations
BrF Liquid below 240 293
occur in polyhalide ions (e.g., IBrCD
Bromine 293 K,
- Neutral interhalogen compounds are of the type fluoride Pale-brown gas
AXn where n is an odd number and X is always above 293 K
lighter halogen when n > L Because n is odd the BrCI Bromine Liquid, reddish 207 283
compounds are diamagnetic, their valence chloride yellow
electrons are present as bonding pairs or as 310
ICI Solid, rubby red
unshared pairs. Iodine chloride
- Interhalogen ions can be either cations or anions.
IBr Solid, brown 232
Halogen fluorides react with fluoride ion Iodine bromide like iodine
acceptors,
XY3 CIF3 Chlorine Liquid below 197 261
2C1F + AsF5 ---- > FC£AsF6- 261 K,
trifluoride
or with fluoride ion donors, colourless gas
above 261 K
IF6 + CsF---- > Cs+IF6-
- The pale yellow triiodide ion (I3) is formed by BrF3 Bromine Liquid, 282 400
trifluoride colourless
dissolving I2 in aqueous KI.
i +r i3 BrCI3 Bromine Solid, orange 374
trichloride (decomp
Other ions are not usually stable in aqueous solution oses)
although they can be obtained in CH3OH or CH3CN
+ xys CIF5 Chlorine Gas, colourless 170 259
and as crystalline salts of large cations as Cs+ or _R4N. pentalfluoride
Cl3 is formed only in concentrated solution BrF5 Bromine Liquid 213 314
Cl2 + Cl" Cl3 pentafluoride colourless
— The electrical conductance of molten I2 is ascribed IF5 Liquid 283 314
to self-ionisation Bromine colourless
pentafluoride
31a I* +I3
— The interhalogens are generally more reactive XY7 if7 Gas, colourless 279.5
Iodine
than the halogens (except F2). This is because the heptafluoride
A—X bond in interhalogens is weaker than the
X—X bond in the halogens.
— Hydrolysis of interhalogens gives halide and Structure of Interhalogen Compounds
oxohalide. Oxohalide is always formed from The interhalogen compounds are generally covalent in
larger halogens present: which the larger atom lies in the centre. Their shapes can
be predicted by VSEPR theory (Chapter 7) Shapes of some
BrF5 5F" + BrO3 of the interhalogen compounds are given below
bromate ion
Chapter 15: Group 17 (VIIA—The Halogen Family) | 625

Xatom in excited state pp FTT"! F ZJ Z i


Total number of Bond Lone
Molecule electron pairs Shape sp3d2 hybridisation-octahedral structure
pairs pairs
around X ---- with one of the axial sites occupied by lone pair
XY 4 1 3 Linear
XY3 5 3 2 T-shaped
W5 6 5 1 Square
pyramidal
xr7 7 7 0 Pentagonal
y^ANy
bipramidal
Square pyramidal structure of interhalogen
Structure of interhalogens can be explained with the molecules of the type XY5
help of Valence Bond Theory also in the following manner: (iv) Type XY7 In these molecules, the central X
(i) Type XY As expected, the compounds of the atom uses all its seven valence electrons in forming seven
type XY are linear. Thus, C1F, BrF, BrCl, IBr and IC1 all X—Y bonds. Formation of this type of compounds can be
I have a linear sturcture. In these compounds both the explained on the basis of sp3d3 hybridisation of the X atom.
halogen atoms have an unpaired electron in the pz orbital. These compounds have a pentagonal bipyramidal
structure with two axial X—Y bonds and five equatorial
Axial overlap of the pz orbitals of the two halogen atoms X—Y bonds. Bond angle is 72° for five sp3d3 orbitals and
results in a linear interhalogen molecule. other two are at 90° with first the sp3d3 orbitals.
(ii) Type XY3 Structure of the interhalogens of this
X atom in excited state 1\ i j| : j ; r i i ?
type can be explained on the basis of sp3d hybridisation of
the central halogen atom X. Three of the hybrid orbitals sp3d3 hybridisation-pentagonal
bipyramidal structure
having an electron each are used in making three covalent
bonds with three atoms of the other halogen atom. The Y
remaining two orbitals accommodate the two lone pairs of
A
the central atom. In order of minimise lone pair-lone pair
and lone pair-bond pair repulsions, the lone pairs occupy
the two equatorial sites around the central halogen atom,
thereby giving a T-shape to the molecule. Thus, ClF3,BrF3
andICl3 have a T-shaped structure.
Y^>fF/Y
Xatom in excited state |~j[j f1 111 ' H i Y
Pentagonal bipyramidal structure of interhalogen
sp3d hybridisation-trigonal bipyramidal molecules of the type XY7
structure with two equatorial sites
occupied by lone pairs
,Y Polyhalides and Polyhalonium Ions
Halide ions often associate with molecules of halogens
or with interhalogen compounds to form polyhalide ions.
For example, on addition of potassium iodide, the
Y
solubility of iodine in water is greatly enhanced due to the
T-shaped structure of interhalogen
molecules of the types XY3 formation of the triiodide ion, I3 :
KI+ 12 ---- > KI3
(in) Type XY5 This type of compounds have a More complex ions of iodine, such as pentaiodide, I5,
square pyramidal structure (instead of octahedral due to heptaiodide, I7 and noniodide, I9 are also known.
VSEPR). The central atom X uses five of its seven valence These are generally found in the form of salts of large
electrons in forming five bonds with Y and two electrons metal jions,
onS) as Rb++Ig
as Rb 2C66H
Ig •• 2C or large complex cations, as
H66 or
remains as a lone pair. Thus, X-atom in XY5 molecule is nh^I§ , A number of mixed polyhalide ions
ip3c? hybridised. The lone pair occupies the axial hybrid containing two or three different halogens are also known,
orbital. as IClj, IC1; and IBrF" and IBrCl".
626 | Essential Inorganic Chemistry

All the polyhalides are hydrolysed to some extent in • Fluorine is the strongest oxidising agent and oxidise
aqueous solution. The ease of hydrolysis increases with the elements to their highest oxidation state, ft.
the introduction of the more electronegative halogen, into example in IF7, iodine has oxidation number seven.
the ion. Thus, the triiodide being the most stable and CI3 • The reactions of fluorine are also different from othe
halogens. Fluorine fumes in moist air and decompose
the least. In non-aqueous solutions, polyhalides behave as water to give oxygen, whereas other halogens ar
mild halogenating agents. sparingly soluble in water and react partly to gir
Polyhalonium cations, e.g., IC14 and BrF2 are formed hydrohalic and hypohalous acids.
as a result of autoionisation of IC13 and BrF3. Other cations 2F2 +2H2O ---- > 2HF+O2
like Br2+, I2, Ij, C1F2+, IF2+, IBr2+, IBrCl+, C1F4+, BrF4+, IF4+, C1F6+, X2 +H2O HOX + HX
BrFg, IFg, etc. are also known. These are stabilised by Fluorine reacts with alkalies to yield the oxide, 0F2:
large anions like BF^PFg, AsF6“, A1C14 .SbClg, etc. 2F2 +2NaOH ---- » OF2 + 2NaF + H2O
The other halogens react with alkalies to yield ;
Pseudohalogens and Pseudohalides solution of hypohalite ions, which may disproportionate
Some molecules like cyanogen, (CN)2, thiocyanogen, The oxoacid of fluorine, HOF, prepared recently is ver
(SCN)2 and selenocyanogen, (SeCN)2, have properties unstable. The oxides of fluorine are not acidic.
similar to those of the halogens. Therefore, these are Cl2+2NaOH ---- > NaClO+NaCl+H20
called pseudohalogens. Pseudohalogens form hydro acids
similar to the hydrohalic acids, e.g., H—CN. They also Uses of Halogens
form anions such as cyanide, CN", cyanate, OCN", Halogen Uses Remark
thiocynate, SCN”, selenocyanate, SeCN-, tellurocyanate.
TeCN" and azide, N3, which resemble the halide ions to F2 In the preparation of UF6 used in the gaseo.s
UF6 diffusion process for separate
some extent. isotopes of uranium (U-235 ara
U-238)
DEEP Focus In the preparation of Teflon is used in electrics
The best known pseudohalide is the cyanide, CN“, which teflon insulators, high temperature
resembles Cl”, Br” and I” in the following respects : 4-CF2 — CFg% plastic and cooking utensils
• It can be oxidised to form a molecule, (CN)2. Cl2 CF ion is the principal —
• It forms an acid HCN similar to HC1, HBr, etc. anion in intracellular
• It forms insoluble salts with Ag+, Pb2+ and Hg+. and extracellular fluids
• Interpseudohalogen compounds C1CN, BrCN and ICN can Cl2 as bleaching Household laundry bleacr
also be formed. agent contains the active ingrede"
NaOCI (sodium hypochlorrs
• Like AgCl, AgCN is insoluble in water but soluble in liquor about 5% by mass)
ammonia.
• It forms a large number of complexes similar to the halide Cl2 used to purify Cl2 + H2O —> HO + HOCi
complexes, e.g., [Cu(CN)4]2' is similar to [CuCl4]2- and water (as disinfectant) qiq- jQns destroy bacteria L
oxidising life-sustaining
[Co(CN)6]3' is similar to [CoCl6]3-.
compounds within them
As a chlorinating CHCI3, CCL, CH2=CHCI. BHC
Anomalous Behaviour of Fluorine agent to prepare DDT
Fluorine differs from other elements of the group various organic
compounds
because of its exceptionally small atomic and ionic size
and low fluorine—fluorine bond dissociation energy. The Br2 To prepare ethylene BrCH2CH2Br is used as a-
dibromide insecticide and as a scavenge
result of these differences is that fluorine is the most BrCH2CH2Br for lead in gasoline to keep lea:
electronegative element in the Periodic Table and is a deposits from clogging engnes.
powerful oxidant. Some difference between fluorine and To prepare AgBr AgBr is used in photographs
other halogens are : plate
• It is more reactive than other halogens because of low •2 To prepare tincture of 50% (by mass) alcohol soluter
F—F bond energy, also due to its high iodine —an antiseptic of iodine is called tincture 0
electronegativity the bond between fluorine and other iodine
elements is very strong so its compounds are more In iodised salt Iodised salt contains 0.01 pe
stable. Some of them are inert, e.g., SF6. cent KI or Nal which is more ths
• Fluorine is almost invariably monocoordinate sufficient to satisfy 1 mg iodr-
per week required for th
(coordination number = 1) and is never more than formation of thyroxine in th
monocovalent. human body.
• Fluorides are more ionic. To prepare Agl Used in cloud seeding-,
• Fluorine forms strong hydrogen bonds resulting in the process for inducing rainfall.
properties of hydrogen fluoride being anomalous.
Chapter 15: Group 17 (VIIA—-The Halogen Family) | 627

Target Practice 1
!• Cl, gas is passed aqueous NaOH solution under given 3. AG/ (standard free energy of formation) of
I conditions HF(g) =-273.20 kJ mol’1
(a) Cl2 + NaOH(cold) -----> HI(g) = + 1.72 kJ mol-1
(b) Cl2 + NaOH(hot) -----> Based on the above data, select which reaction is most
Write balanced ionic reactions favourable in each case
Equivalent mass of Cl2 in (a) is (a) H2+I2 ---- > 2HI
Equivalent mass of Cl2 in (b) is or 2HI ---- > H2 +12
2. (a) (CN)2 cyanogen is also called (b) H2+F2 ---- > 2HF
(b) CN- cyanide ion is also called or 2HF ---- ■> H2 + F2
(c) Bond energy of F2, Cl2, Br2 and I2 in increasing order is 4. Out of HF, HC1, HBr and HI, select
i
(a) strongest acid
(d) Electron-affinity is maximum (out of F, Cl Br and I) for (b) weaker acid
r (c) acid with maximum boiling point
(e) 0F2 is formed by the reaction (d) acid that attacks glass
(e) acid with maximum dipole moment
(f) acid which has been used as non-aqueous solvent

Answers
1. (a) 2NaOH(cold) + Cl2 ---- > NaCl + NaOCl + H2O Equivalent mass ofCl2 in (b)
L reduction | 1d reduction^4 cr Equivalent mass = —
2 o’ -1 2
— oxidation---- 1
1 71
---- > 5NaCl + NaC103 + 3H2O
(b) 6NaOH(hot) + 3C12 — - Cl2---- > C1O3 Equivalent mass = —
2 0 +5 10

t reduction—
— oxidation------
Equivalent mass ofCl2 in (a)
71 71
Net equivalent mass = — + — = 42.6 g equivalent

2. (a) Pseudohalogen (b) Pseudohalide


1

1 ril reduction (c) I2 < F2 < Br2 < Cl2 (d) Cl


- Cl2 --------------> Cl Equivalent mass =
2 0 -1 (e)2F2+2NaOH ---- > 2NaF+H,O + OF2
1 oxidation oc|-
..M, 3. (a)H2+I2 ---- » 2HI AG; = + ve
Equivalent mass = ~
2 o +1 Decomposition of HI is kinetically controlled, (rather than
thermodynamically controlled)
Net equivalent mass = in reduction part + in oxidation part
(b) H2+F2 ---- > 2HF
Very fast since, AG° = - ve
2 2
4. (a) HI (b) HF (c) HF (d) HF (e) HF (0 HF
= 71.0 g equivalent-1

Practice Exercise 1
1. (a) IC1 is found to be partially ionised in fused state. Give BF3+HF H++[BF4r
reaction. Identify nature of HF and fluorides.
(b) IC1 is also found to form addition compound with NaBr
and KI. Write the formulae of the addition compounds. 5. CN" resembles Cl-, Br- and I- in many of its properties.
2. Identify (A) and (B). What is it called ? Give some other examples which
resemble halides.
Br, +OH" ---- > (A)+(B)
hot 6. Select the species which are soluble in H2O and which are
(A)+(B) + H+ ---- > Br2
soluble in NH3 (aq) AgF, AgCN, AgCl, Agl, CaF2, CaCl2.
(A) gives yellow ppt with AgNO3.
7. Identify the products (which can be in the form of ions) in
3. When I2 is estimated by Na2S2O3 iodometrically, blue
the following:
colour (due to starch) changes to colourless when I2 has
(a) KF+IF, — (b) CsF + IF5 ---- >
completely reacted
2SZ2O|"+L
O Z
---- > S44O*'
D
+ 2r (c) KC1 + IC1 — (d) Csl3
but after sometimes blue colour reappears. Explain A (f) KIBrICI] -A*
(e) Rb[ICl2]
4. SbF5, AsF5 and BF3 accept F- from HF as :
A
AsF5+HF H++[AsF6r (g) I2 +IC1 + A1C13
628 | Essential Inorganic Chemistry

8. H2SO4 is a weaker acid than HCI, yet HCI is evolved when 9. HF can be prepared by the action of H^O^ on NaF.
cone. H2SO4 is added to NaCI. Explain Explain why HBr cannot be prepared by the action of the
same acid on NaBr.

Answers
1. (a) 2IC1 ---- > F + [IC12]- 5. CN“ is called pseudohalide. Like X~:
(b) NaBr + ICl-- > Na+[BrIClF - it forms HCN
KI + IC1--- > K+[I2C1F - it can be oxidised to (CN)2
2. Method is for enriching Br2 - it forms insoluble salts with Ag+, Pb2+ andHg|+.
3Br2 + 6OH" ---- > 5Br’ +BrO3 Other pseudohalides are:
(A) (B) SCN“ (thiocyanate), OCN" (cyanate),
5Br" + BrO3 + 6H* ---- > 3Br2 +3H2O NCN2- (cyanamide), SeCN" (selenocyanate)
3. This is because some atmospheric oxidation has taken place, ONC” (fulminate), N3 (azide)
forming I2, which reacts with starch to give the blue colour 6. H2O soluble : AgF, CaCl2
again: NH3 (aq) soluble : AgCN, AgCl
21“ +|O2 + 2H+ ---- > I2 +H2O Agl and CaF2 are insoluble in H2O and NH3 (aq) both.
7. (a)K+[IF6F (b)Cs+[IF6F (c)K+[ICl2F (d)CsI + I2
4. SbF5, AsF5, BF3 etc., are F~ acceptors thus they are behaving (e)RbCl + ICl (f)KCl + IBr (g) [I3riAlCl4F
as an acids and HF behaves as a base : 8. NaCI + H2SO4 ---- > NaHSO4 + HCI
SbF5 + HF ---- > [SbF6F + H+ The HCI gas escapes taking the equilibrium to the right
acid base 9. H2SO4 oxidises HBr formed to Br2.

Halogen Acids to some extent. However, because of the high


• All the halogen acids (HF, HCI, HBr, HI) are known. activation energy the dissociation occurs only very
In aqueous solution, they are typical mineral acids slowly in the absence of a catalyst. As a result, HI(g)
whose acid strengths decrease in the direction of is quite stable at room temperature. Thus, at room
increasing bond energy, that is temperature, decomposition of HI(g) is kinetically
controlled (rather than thermodynamically
HI > HBr > HCI > HF
controlled).
strong acids weak acid
• HX can be prepared by direct combination of H2 and • CaF2 + H2SO4 CaSO4 + 2HF
X2
H2(g) + X2(£) ----- > 2HX(g) • CaF2 has also SiO2 as impurity otherwise HF formed
would react with SiO2 forming SiF4 and H2SiF6:
- reaction between H2 and F2 is very fast,
- reaction between H2 and Cl2 is also rapid, h2+x2
- but reaction between H2 and Br2 or I2 is very slow.
Free energies of formation of HX are given :
HX
HX AG}, kJ mol-1 NaCI + cone. H2SO4 CaF2 + H2SO4
HF(g) -273.20
HCI(g) -95.30 HCI Na2SiO3 (glass)
HF
HBr(g) -53.43
Hl(g) + 1.72 NaCI + SO2 + H2O + O2 HBr
Hargreaves method or
- thus we see that AGy of HF, HCI and HBr are large HI NaBr or Nal + H3PO<
and negative suggesting that for them reaction
H2 +X2 ----- > 2HX SiO2 + 4HF----- > SiF4 + 2H2O
goes to completion. SiF4 +2HF----- > H2[SiF6]

DEEP Focus NaCI +H2SO4 ^^4 HCI+NaHSO4


cone. salt cake
For HI(g), AGy is small and positive suggesting that HI should
dissociate 550°C
NaHSO4 + NaCI --------> HCl + Na2SO4
2HI(g) ---- > H2(g) + I2(g)
Chapter 15: Group 17 (VIIA—The Halogen Family) | 629

This method called ‘salt-cake’ method involves DEEP Focus


Jnnation ofNaHS04 (salt-cake). Because of the tendency to form hydrogen bond, metal
NaCl is now replaced by NHjCl since NH4HSO4 fluorides are solvated by HF giving species of the type HF2
intermediate) is water soluble and reaction goes to etc. KHF2 (KF HF), KH,F3 (KF-2HF) and KH/4
ompletion: (KF -3HF) are known examples.
2X1^01+ H2SO4 -----> (NH4)2SO4+2HC1 • HF attacks glass, other halogen acids do not. Glass is
silicates of sodium and calcium and are converted into
The Hargreaves process is a variant of the salt-cake complex fluorides:
mess in which NaCl is reacted with a gaseous mixture of Na^iOa + 6HF Na^iFg + 3H2O
)0> and H2O: CaSiO3 + 6HF---- ? CaSiF- + 3Ho0
2NaCl + SO2 + H2O +- O2 Na2SO4 + 2HC1. Hence, HF can't be stored in a glass vessel. Instead, it
2 is stored in a wax-vessel which does not react with HF.
• HBr and HI are made by the reaction of concentrated This unique property of HF of attacking glass is used for
making permanent marking on glass-wares.
phosphoric acid on metal bromides and iodides :
NaBr + H3PO4 ---- > NaH2PO4+HBr • HF also reacts with CC14 to form freons (refrigerant)
NaI + H3PO4 -----> NaH2PO4 +HI CC14 + HF---- > CFC13 + HC1
CFCI3 + HF---- > CF2C12 + HC1
ALERT • In the gaseous state, HX are essentially covalent. In
Concentrated H2SO4 (a strong oxidising agent) can't aqueous solution, they ionise to formH3O+ and X~ but
be used since it would oxidise HBr to Br2 and HI to 1?. not H+ and X~
H2O +HX“ H3O+ + X~
These are also formed from red phosphorus and base acid acid base
X2 (Br2,12) which give PX3 and its hydrolysis gives HX: HF has more ionic character (due to greater
2P +3Br2 -----> 2PBr3 electronegativity difference) than HC1, HBr and HI but
red
acid strength varies as :
2PBr3 + 6H2O---- > 6HBr + 2H3PO3 HF <HCl<HBr<HI
2P + 3Br2 + 6H2O-----> 6HBr + 2HaPO3 weak strong
• HF is only just liquid at room temperature and HC1,
HBr and HI are gases. HF has abnormally high DEEP Focus
boiling point due to intermolecular H-bonding which Acid strength is dependent not on the electronegativity
is not possible in other halogen acids due to their difference of H and X but on the tendency of the hydrated
smaller electronegativity values. molecule to form hydrogen ions :
Boiling point HC1 < HBr < HI < HF HX (hydrated)---- > H+ (hydrated) +X~ (hydrated)
The acid strength is equal to the sum of all the energy terms
40 in:
20 AH
= free energy of dehydration
tI -200 + free energy of dissociation
*1
x2
bp. -40 + ionisation energy of H(g) *3
+ electron affinity ofX(g) *4
-60
+ free energy of hydration ofHT and X~ *5
-40 HX (hydrated)---- > HX (g) +aq AH = Xj
-100 HX(g) > H(g) + X(g) = x2
molecular weight----- ►
H(g) ---- > H’(g)+e" = X3
Fig. 15.2 Boiling point of HX X(g)+e’ ----- » X’(g) = x4
H*(£) + X"(g)+aq H* (hydrated) + X (hydrated)
Some H-bonding also occurs in the gas, which consists
______________________________ =X5
of a mixture of cyclic (HF)6 polymers, dimeric (HF)2
and the monomeric HF. HX (hydrated)---- > H+ (hydrated) +X” (hydrated)
8- AH=xl + x2 + x3 + x4 + x5
„-F' 8+ Due to maximum value of Xj and x2 for HF due to
H-'"' H-^ x- 8 +
'F 8+ 134° H-bonding, net AG (free energy change) for HF comes out to be
8- positive which disfavours dissociation; (-ve sign of AG favours
spontaneous dissociation, +ve sign disfavours dissociation).
(Hydrogen-bonded chain in solid HF)
AGZ’ HI < HBr < HC1 < HF
' +ve -ve
630 | Essential Inorganic Chemistry

• Dipole moment of HF is maximum and is in order HOX series


HF > HC1 > HBr > HI • +1 oxidation state in HOF appears very doubtful bi
1.91D 1.05D 0.80D 0.42D this has been prepared by trapping F2 and H2O in a
• Liquid HF has been used as a non-aqueous solvent. It unreactive matrix of solid N2 (at very lc
undergoes self ionisation. temperature) and photolysing the gases :
2HF [H2F]+ + F~ F2 + H2O HOF + HF
It has been recently prepared by passing F2 over ice a
DEEP Focus 0°C and removing the product into a cold trap:
Acid-base reactions occur in this solvent system. However, -40°C
the solvent itself has a very strong tendency to donate H2O + F2 HOF + HF
protons. Mineral acids like HNO3, HgSO^, and HC1, when
dissolved in HF, behave as bases in HF as they accept HOCI, HOBr and HOI are not very stable and ar
proton from HF: known only in aqueous solution.
HC1 + HF H2C1+ + F“ - HOCI is formed when Cl2 is dissolved u
base acid acid base H2O. Cl2, Br2 and I2 all dissolve to some extent ii
H2SO4 + HF H3so; +F" water forming hydrated X2 molecules, and X' ant
base acid acid base OX~ ions:
HNO3 + HF H2NO* + F“ H2O + Cl2 ---- > HOCI + HC1
base acid acid base
HC1O4 (perchloric acid) however behaves as an acid in (Bleaching action of chlorine water is due to fonnatior
liquid HF which acts like a base : of HOCI)
HC1O4 +HF H,>F+ +C1O; - HOX are also formed by shaking X2 with freshlj
add base acid base precipitated HgO in water :
2HgO + 2C12 + H2O---- > HgO ■ HgCl2 + 2HOC1
The solution, after filtration, is distilled whereby the
Oxoacids of Halogen dilute acid passes over. The maximum concentration
Five series of oxoacids are known and are summarised which can be obtained is only 25 per cent. In the
in Table 15.5. process of distillation, the acid decomposes into its
anhydride, C12O.
Table 15.5 Oxoacids of Halogen 2HOC1---- > H2O + C12O
• Salts of HOX are called hypohalites and are
HjXOe
HOX HXO2 HXO4 bleaching agent. NaOCl is formed due to electrolysis
HXO3 para
hypohalo halous perhalic
us acid acid
haiic acid
acid
perhalic of cold NaCl (brine) solution :
acid NaCl(aq) Na+ + Cl
HOF at anode : at cathode :
HOCI HCIO2 HCIO3 HCIO4 2C1“ ---- > Cl2 + 2e~ : Na+ +e‘ ---- > Na
HOBr HBrO3 HBrO4 : 2Na + 2H2O---- > 2NaOH + H2
HOI HIO3 HIO4 H5IO6
If anode and cathode are not separated, NaOH and Cl>
Anhydride X2O X2O3 X2O5 .. • X2C>7 l2O7
mix forming NaOCl.
Oxidation +1 +111 +V +VII +VII
state of X 2NaOH + Cl2 ---- + NaCl + NaOCl+ H20
Oxidising strong weak * Bleaching powder CaOCl2 is formed when Cl2 is
power passed into moist lime CaO. CaOCl2 has Ca2+,OC1' and
Acid weak strong Cl~ ions
strength*
(CaO + H2O) + Cl2 ■> CaOCl2+H2O
Thermal weak ■> good
stability
Hydrolysis of max min DEEP Focus
anion * OCr disproportionates in hot solution
Structure
anion
of (OCI-) (CIO;) (Cio;) (CIO;)
ON of Cl
3ocr
+i
2cr +cio;
-1 +53
linear angular trigonal tetrahe­
pyrami­ dral
dal
I--- red. —f
*Acid strength of one type oxoacid increases as size of halogen
---------- oxid.--------
atom increases
* Fe2+ is oxidised to Fe3+
HOCI + 2Fe2+ + H+ — + Cr+2Fe3++H20
Chapter 15: Group 17 (VI1A—The Halogen Family) | 631

* 0C1 also oxidises I to Ij in acidic medium, • IO3 oxidises I- to I2 :


[
ocr + 2r + 2H+ —> h2o + cr +12 IO; + 51" + 6H+ -----> 312 + 3H2O
• CIO; oxidises SO^" to SC^~ :
' HX02 series
• Only stable acid known is HC1O2 and exists in
cio; + 3S0J- —> cr + 3so?-
solution: 2KCIO3 400-500^ 2KC1 + 3O2
MnO2
Ba(C102)2 + H2SO4 -----> BaSO4 + 2HC1O2
Following disproportionation of CIO; also takes place
• Salts are called chlorites and are made from :
as side reaction at low temperature and in the absence of
2C1O2 + 2NaOH-----> NaClOfc +NaClO3 + H2O catalyst:
2C1O2 + Na2O2 -----> 2NaC102 + O2
4KC1O3 -----> KC1 +3KC1O4
i • It is stable in alkaline solution but disproportionates +5 -1 +7
in acid solution :
• Chlorates are used to make fireworks and matches.
5HC1O2 -----> 4C1O2 +HC1 + 2H2O
+3 +4 -1 HX04 series
and 4HC1O2 —> 2C1O2 + HC1O3 + HC1 + H2O • HC1O4 and HIO4 and their salts are known.
HX03 series NH|C1O4 + HNO3 -----> HC1O4 + NH4NO3
A
• HCIO3, HBrO3, HIO3 are known of which most stable 4HC1O3 3HC1O4 + HC1
isHIO3.Salts likeNaC103, NaBrO3,NaIO3 are stable: HC1O4 is commercially available as 70% HC1O4 and is
Ba(C103)2 + H2SO4 -----> BaSO4 + 2HC1O3 dihydrate HC1O4 which can be dehydrated to
After reaction, BaSO4 is removed by filtration, and the HC1O4 by oleum :
filtrate is evaporated in vacuum till 40 per cent solution is HC1O4-2H2O +H2S2O7 HC104 + 2H2SO4 + H2O
obtained. However, further concentration by evaporation oleum
leads to decomposition: NaC103 + H2O Electrol?s4 NaC104 + H2
3HC1O3 -----> HC1O4 + Cl2 + 2O2 + H2O
Oxidation of CIO; occurs at a Pt anode through the
Similarly for HBrO3.
half-reaction :
HIO3 is formed by oxidation of I2 with concentrated CIO; + H2O-----> CIO; + 2H+ + 2e~
HN03or03.
• HC1O4 is the strongest of all other oxoacids of Cl:
8H+ + IONO3 + Iz -----> 210; + lONQz + 4H2O
HC1O < HClOa < HCIO3 < HC1O4
Chlorates, etc. may be made :
~ by passing Cl2 into hot alkali solution. DEEP Focus
Thus, 2NaOH + Cl2 NaCl + NaOCl + H2O • The dissociation of an oxoacid involves two energy terms :
breaking an 0—H bond to produce an anion and H* ion
HOX H+ + OX~ and
6NaOH + 3C12 hot, 5NaCl + NaC103 + 3H2O
- hydrating both ions.
- CIO; ion is larger than OC1" so the hydration energy of
2OH“ +C12 cr + ocr+H2o OC1" is larger than CIO;. Also electronegativity of
hot oxygen is greater than that of chlorine. In H0C1,
6OH“ + 3C12 5C1" + CIO; + 3H2O HC1O2, HC1O3, HC1O4, an increasing number of oxygen
In this Cl2 undergoes disproportionation. atoms are bonded to the chlorine atom. The more
oxygen atoms that are bonded, the more the
“ by electrolysing hot halide solutions (with vigorous electrons will be pulled away from the O—H
stirring) bond, and the more this bond is weakened. Thus,
electrolysis^ greater the number of oxygen bonds attached to
2Cr+2H2O Cl2 + 2OH" + H2 halogen, least is the energy required to break
at anode at cathode O—H bond, hence greater the acid strength
6NaOH + 3C12-----> 5NaCl + NaClO3 + 3H2O HC1O < HC10, < HC1O, < HC1O.
£• •> •»

• HCIO3 on evaporation gives C1O2 : • Metals like Zn, Mg and Fe dissolve in dilute solutions
4HC1O3 -----» 4C1O2 + 2H2O + 02 of the acid forming their perchlorates:
• HCIO3 oxidises SO2 to SO3 Zn + 2HC1O4 -----> Zn(C104)2 +H2?.
HCIO3 + 3SO2 -----> HC1 + 3SO3
632 | Essential Inorganic Chemistry

Hot concentrated acid (73%) behaves as a remarkable DEEP Focus


oxidising agent: “Available chlorine” content is defined as the weight cf
4HC1O4 ---- > 2C12 + 7O2 + 2H2O Cl2 which liberates the same amount of I2 from HI as does the
• Mg(C104)2 is used in dry batteries and is also a given weight of the compound; it is expressed as a percentage.
effective desiccant called anhydrone. KC1O4 is used Available chlorine can be determined by treating
in fireworks and flares. aqueous solution of bleaching powder with KI and
HgO + X2 + H2O titrating liberated I2 using Na2S2O3 solution (hypo).
CH3COOH is also added to neutralise Ca(OH)2 formed.
Fe2* CaOCl2 + H2O---- > Cl2 + Ca(OH)2
HOX Fe3*
Cl2 + 2KI---- > 2KC1 + Ig
I2 + 2Na2S2O3 ---- > 2NaI + Na2S4O6
hot
ox- X-+XO3 Thus, CaOCl2 s=Cl2 seIj, =2Na2S2O3
r molecular weight^
Eq. wt. of CaOC12 =
l2
2 J
• Pale yellow solid with odour of Cl2; in aqueous
CIO2 + NaOH solution it gives Cl-, OC1" and Ca2+ ions :
H0
H2SO4
HCIO2
H*
NaCIO2
CaOCl2 Ca2+ +cr +ocr
Ba(CIO2)2
• It undergoes disproportionation on standing:
CIO2 + Na2O2
H+ 6CaOCl2 ---- > Ca(C103)2+5CaCl2
CI2O7 SO2
HCI + SO3 • When CO2 is passed into suspension of bleaching
P2O5 powder in water, HOC1 is formed
A ■ CIO2 +| HCIO31+ HCI
hcio4 2CaOCl2 + H2O + CO2 ---- > CaCl2 + CaCO3 + 2HOC1
+ HCI
SO23" KOH
• It is a bleaching agent as well as oxidising agent
/evaporate due to oxygen formed in acidic medium/neutral
KCIO3
medium:
A L sol" CIO2
CaOCl2 + H2O---- > Ca(OH)2 + Cl2
73% solution A, catalyst
Cl2 + H2O---- > 2HC1 + 0
Cl2 + O2 KOI + O2
KOI + KCIO4 CaOCl2 + HCI---- > CaCl2 + HOC1
HOC1---- > HCI + 0
• CaOCl2 oxidises
Bleaching Powder - HjjS into S
• Commonly it is written as CaOCl2 with one Cl" and CaOCl2 + H2S---- > CaCl2 + H20 + S
one OC1" (hypochlorite) ions (Cl with two different - FeSO4 into Fe2(SO4)3
oxidation number) and is thus calcium chloro CaOCl2 + 2FeSO4 +H2SO4 ---- > Fe^SO^ +CaClj
hypochlorite + H20
(-1) ____ __ 1
Cl - NH3 into N2
Ca 3CaOCl2 +2NH3 ---- > 3CaCl2 +3H2O + N2
OC1 - KNO2 into KNO3 and NasAsOs into NajAsO^
(+1)
KNO2 + CaOCl2 ---- > KNO3 + CaCl2
• Its actual composition is o 0
Ca(OCl)2 • CaCl2 • Ca(OH)2 • 2H2O containing 35%
- Carbonyl compounds (CH3—C—H, —C—CHJ
I I
available chlorine. It is obtained by action of Cl2 gas
and alcohols (—CH—CH3) are oxidised to
on slaked lime :
3Ca(OH)2 + 2C12 ---- > OH
chloroform.
Ca(OCl)2 • CaCl2 • Ca(OH)2 -2H2O
Chapter 15: Group 17 (VIIA—The Halogen Family) | 633

CaOCl2 + H2O Ca(OH)2 + Cl2 • It is soluble in water giving orange yellow coloured
solution of HOCI
-> 2HC1 + 0
Cl2 + I^O
0 C12O + HzO---- » 2H0C1
CH3CH2OH CH3CHO +1^0
• It forms salt with alkali solution indicating its acidic
CH3CH0 + 3C12 CC13CHO + 3HC1 character.
2CCI3CHO + Ca(OH).,2 ---- , (HCOO)2Ca + 2CHC13 C12O + 2K0H — 2K0C1 + HjO
• It is an oxidising agent
Similarly for CH3CHO, 2?COCH3. This test, called
‘Halfonn test*, is given by 3C12O + 10NH3 ---- > 2N2 + 6NH4C1 + 31^0
• Structure is angular with sp3 hybridised 0-atom and
- CH3CHO
is related to a tetrahedron with two positions occupied
- CH3CH2OH by lone pairs of electrons.
- all 2° alcohols (at Q,) —CH—CH3
<0
. •• 1.71A
OH 111' XC1
Cl
” all ketones (with carbonyl group at Cj as
CH3—C—) DEEP Focus
• The bond angle varies in the order FOF < CIOC1 < BrOBr.
0 This is because electrons in the case of OF2 are nearer to
fluorine due to high electronegativity of F compared to Cl
Ca(OH)2 I2
to Br. The bonded electron pairs in C12O and Br2O are
S oV\ CHCI3 closer to oxygen making the repulsion between them more
\h2s Cl2 KI
and thereby reducing the lone pair-lone pair repulsion on
"h2o oxygen to some extent. Also due to the bulkiness of Cl and
Fe3*- CaCI(OCI) Ca(OH)2 + Cl2 (available)
NH3 Br, the angles C1OC1 and BrOBr increase to such an extent
n2 KNO^/
HCI thatl09°28', (the tetrahedral angle), is approached.
H2O
HOCI C1O2 (Chlorine dioxide)
KNO3 03(0103)2 + OaCI2 TZ -»| HCI + 0 • C1O2 (as yellow-coloured gas with unpleasant smell) is
Bleaching action obtained when KC103 reacts with cone. HjSO^ or
oxalic acid
3KC1O3 + 3H2SO4 HC104 + 3KHSO4 + HjO
Oxides of Halogen + 2C1O2?
Following oxides of chlorine are known. 60°C
2KC1O3 + 2H2CA ■> K^C^ + 21^0 + 2CO2
Oxidation state oxalic acid
Formula Name Oxyacld of chlorine + 2C1O2T
Chlorine monoxide CI2O HOCI +1 90°C 2AgCl 1 +2C1O2? + 02?
2AgC103 + Cl2
Chlorine dioxide CIO2 HCIO2 +3
• Commercially it is prepared by passing S02 gas into a
Chlorine hexoxide CI2O6 HCIO3 +5
mixture of NaC103 and HjSO^j having NaCl in traces.
Chlorine heptoxide CI2O7 hcio4 +7
2NaC103 + S02 + H2SO4 2C1O2 + 2NaHSO4
C12O (Chlorine monoxide)
• Aqueous solution of gas is neutral and is a strong
• When Cl2 gas is passed over HgO at 300-500°C, C12O oxidising agent:
is formed.
10HI + 2C1O2 2HC1 + 41^0 + 512
2C12 +2HgO---- > HgOHgCl2 +C12O?
Reaction may also be carried out with chlorine
dissolved in CC14. DEEP Focus
• C1O2 reacts with KOH forming KC1O3 and KC102.
• C12O is a brownish-yellow gas which condenses to an 2K0H + 2C1O2 ---- > KC1O2 + KC1O3 + H2O
orange liquid (b.p. 2°C)
This reaction indicates that C10o is a mixed anhydride of
• It is thermally unstable and explodes on heating HC1O„ and HC1O-.
2C12O-----> 2C12 + 02
634 | Essential Inorganic Chemistry

• It is strong oxidising agent and chlorinating agent: Solid has the ionic composition CIO* C104 confirmed by
Mg + 2C1O2 ----- > Mg(C102)2 X-ray study. The liquid may have either of two structures :
°\ Z° 0\ /Ox /O
Al + 3C1O2 ----- > A1(C1O2)3 oAci—Cl^-0 Cl- Ch
These metals in powdered form in aqueous suspension o' ^0 0 0 0
are oxidised.
• Ozone oxidises C102 to C12O6( CL)O7 (Chlorine heptoxide)
2C1O2 + 2O3 —-> C12O6 + 2O2
• When HC104 is added to P2O5 at -10°C, C12O7 is
• Alkaline H2O2 is oxidised to 02 obtained which separates on distilling the mixture at
2C1O2 + 2NaOH + H2O2 ----- > 2NaC102 + 02 + 2H2O 85°C.
• Cl atom is sp3 hybridised in the angular molecule with
2HCIO4 + P9O5 -----** C12O7 +• —
2HPO3
bond angle of 118° and Cl—0 bond length of 1.47A. It
has odd number of electrons and is thus • C12O7 is colourless, oily liquid (b.p. 82°C)
paramagnetic and the stability is due to the • C12O7 is extremely explosive.
presence of 3-electron bonds stabilised by resonance.
• C12O7 + H2O 2HC1O4
Molecules containing odd number of electrons are
reactive and generally dimerise in order to pair the • C12O7 reacts with alkali forming perchlorates.
electrons, but C102 is the exception.
C12O7 + 2K0H----- > 2KC1O4 + H2O
-Ck 1.47A
X Cl x •• • C12O7 has a bent structure with a bridging O-atom.
•• x
•o- •o-
X .
•• 0 118°
The terminal Cl—O bonds are substantially shorter
C^Og or C1O3 [Chlorine hexoxide (trioxide)] than the bridge Cl—O bonds, indicating double bond
character.
• When C102 reacts with ozonised 02 at 0°C, C12O6 is
obtained. 0
2C1O2 + 2O3 ----- > C12O6 + 2O2 0 cr 119° C1H 115° 0
• C12O6 is unstable red oil (m.p. 3.5°C) and explodes on
heating. 0 /
0 0
• It is a strong oxidising agent
C12O6 + 2Na0H + ----- > 2NaC103 + 02 + 21^0 Cl2
• Anhydrous HF reversibly gives FC102 and HC104 HgO
C12O6 + HF FC102 + HC104 h2o A
H0C1* CI2O Cl2 + 02
• N0 + Cl206 ----- » C102+[N0]+[C104F
no2 +ci2o6 —> cio2 +[No2]+[cio4r KOH

KOC1
DEEP Focus
• When CLO-Zoo
(or C1O„) reacts with KOH, KC1O„o and KC104.
are formed indicating that C12O6 is a mixed anhydride of
HC1O,o andHClO.
4 Cl2 NaCIO3 + S02 + H2SO4
AgCIO3
2K0H + Cl„0 fi ---- > KC1O,O +KC1O.4 +H„O 90-C j
Z O Z commercial
• In the liquid state and in aqueous solution, there is HI S' -__________ .*»&■ CIO3
paramagnetism indicating existence ofClO3 formed due to h CI2Og
dissociation of CLO„
Z o
Cl20fi T--- 2C1O.o
z o
NaOH\NO
Paramagnetism is due to unpaired electrons inC103,Cl206 + h2o2
is diamagnetic in solid state which is due to paired electrons. ,, CI02 + [NO)‘(CX)J'
y ; ° 0 ________ ,
HCIO4 p-
u*
KCIOs + KCIO4 NaCIOj

0—ci-4-ci—o 20—ci • H2O P2O5

: 0
I CI2O7 [
KOH
KCIO4
0 0
Chapter 15: Group 17 (VIIA—The Halogen Family) | 635

Target Practice 2
L Arrange HOC1,HC1O2,HC1O3 andHC104 in increasing (a) Cl2 thus, obtained is called
(a) oxidation state of halogen atom (b) Equivalent weight ofCaOCl2 is
(b) oxidising power (c) What is the type of above reaction?
(c) acid strength 6. Which of the following pairs can be distinguished by
iodoform test?
(d) thermal stability
(a) CH3CHOandCH3COCH3
(e) s-character in hybridised state
(b) CH3CHOandCH3CH2OH
2. What are the anhydrides of
(c) CH3OHandCH3CH2OH
(a) HOC1
(d) HCHO and CH3CHO
(b) HC1O2
7. Calculate magnetic moment ofCl2O8 and C1O3.
(c) HC1O3
(d) HC1O.4 (a) C12O6 BM
3. Mixture ofKBr andKBrO3 in acidic medium can be used as (b) CIO. BM
8. Consider bond lengths x and y and bond angles a and p in
brominating agent. Explain by chemical reaction
the following and compare these values
Q
4. KC1O3 is used
o^ y,0
/Cl a Cl^yO
KC1O4 is used as 0 / 0
0
Mg(C104 )2 is used as
5. In aqueous solution bleaching powder reacts as Bond lengths: *□ y
Bond angles: aQ p
CaOCl,+H„O
Z Z
---- > CL+Ca(OH)
Z
2
4

Answers
1. (a) HOC1 < HC1O2 < HCIO3 < HC1O4 CaOCl2 b 2C1
(b) HOC1 < HC1O2 < HCIO3 < HC1O4 Molecular weight
Equivalent weight =
(0 HOC1 < HC1O2 < HCIO3 < HC1O4 2
(d) HOC1< HC1O2 < HCIO3 < HC1O4 (c) There are two types of Cl in CaOCl2
(e) Equal (25%) each Cl-atom is sp3 hybridised. Cl” (oxidation number = -1) oxidised to Cl2
2. (a) C12O (b) C1O2 (c) C12O6 (d) C12O7 CIO” (oxidation number = + 1) reduced toCl2
3. 5Br~ + BrOj + 6H+ ---- > 3Br3+3H2O Thus, it is a intramolecular redox reaction
4. (a) an explosive, (b) fireworks and flares 6. (a) No (b) No (c) Yes (d) Yes
(c) dry batteries 7. (a) Zero BM (no unpaired electron)
5. available chlorine (b) -73 BM (with one unpaired electron)
(b) CaOCI2 s 2C12 s 2C1 8. (a) x > y (b) a > P

Practice Exercise 2
1. Complete following reactions : 5. 10 g of bleaching powder is dissolved in water containing
(a) CIO’ + I" + 6H+ ---- > (b) CIO’ + Fe2+ + H+ - some CH3COOH and diluted to 1 L. 20 mL of this solution
on reaction with KI required 20 mL of 0.1 N Na2S2O3
(c) Zn(C103)2 (d) o3 +i2+h2o --- > solution. Calculate percentage of available chlorine in it.
(e) KBrO,«3 + F„z + KOH 6. CaOCl2 in aqueous solution changes to Cl2. What is the
2. Which is stronger acid in each pair ? type of this change ?
(a) HC1O HC1O2 (b) H2SO4 H2SO3 7. Give structure of oxoacids of halogens.
(0 HjPOg H.PO. (d) HNO3 hno2 8. What are the corresponding anhydrides of the following
(e) HF HC1 (f)HjjPOg
H3PO4 oxyacids
3. C4H8O gives iodoform test. Identify it. HC1O4,HC1O3,HC1O2 and HOC1
CaOCl*22/H2O 9. Identify A, B and C in the following
4. C8o Hso O(A) 2 2 CHCL + (B) HC1O4 + P2O5 ------- * A + B
o
A
KOH
(B) H*
COOH KOH
C + H2O
What are (A) and (B)?
636 | Essential Inorganic Chemistry

Answers
1. (a) C1O3 + 61“ + 6H+ ---- > CP + 3I2 + 3H2O 5. CaOCl2 =CI2 =I2 =2Na2S2O3
(b) CIO3 + 6Fe2* + 6H+ ---- > CP + 6Fe3+ + 3H2O CaOCl2 =C12 =I2 =0.1N (NjVi = JV2V2)
(c) 2Zn(C103)2 2ZnO + 2C1, + 50, Cl2 = 3.55 g L-1 in 10 g L"1 bleaching powder.
Available Cl2 =35.5%
(d) 3O3 + I2 + H,0 ---- > 2IO3 + 2O2 + 2H*
(-1)
(e) KBrO3 + F2 + 2K0H ---- > KBrO4 + 2KF + H,0 >C1 H2O
■2
2. (a)HC102 (b)H2SO4 (c) almost same (d)HNO3 6. Ca1 —> Cl, + Ca(OH),
0C1 0
(e) HC1 (f) almost same (+1)
In H3PO2, H3PO3 and H3PO4, terminal O is one, and
CP (alone) is oxidised to Cl2 and Cl (of 0C1) is reduced toClr
electronegativity values of P and H are almost same, hence, Hence, it is a redox reaction.
acid strengths of these acids are same. 7. (a) H — O— Ci: (b) H—0—ci:--- > 0
0 0
I T
3. CH,— C—CH,— CH, (it should have CH3—C— group of (c) H—0— Cl:-> 0 (d)H — 0— ci:—>0
I
0 I 1
the given carbonyl formula) 0 0
0 0 In all cases Cl atom is sp3 hybridised.
I
c—ch3 (B):
8. HC1O4:C12O7; HC103 : C12O6; HC102: C102;
4. (A): HOC1 :C12O
9. (A):C12O7 (B):HP03 (C):KC104.
Total Practice Set
(Read, Plan & Solved)

Problem 1. An important oxidation-reduction


reaction used to determine trace amounts of O3 and other Problem 3. In neutral solution, one mole of periodate
oxidants in polluted air involves oxidation of iodide ion : ion (IO4) reacts with excess of iodide ion to produce one
mole of iodine; on acidification of the resulting solution, a
2F + O3 + H2O---- > 2OH" ++ O2 further three moles of iodine are liberated. Derive
equations for the reactions which occur under these
The solution is acidified and the is titrated with conditions.
sodium thiosulphate solution using starch indicator. One
Solution IO4 + 2F + H2O + IO3 + I2 + 2OH"
litre of a mixture of O3 and O2 at STP was allowed to react
with an excess of acidified solution of KI and liberated IO; + 5F + 6H*
V
-> 31a + 3H2O
required 40 mL of 0.1 M sodium thiosulphate solution.
What is the percentage of ozone in the mixture? Problem 4. Describe in outline how you would
attempt to determine the equilibrium constant and
standard enthalpy change for the reaction in aqueous
Solution 21" + O3 + H2O 20H + I2 + 02
solution.
+ 2Na2S2O3 Na2S4O6 + 2NaI Cl2 + H>O HC1+HOC1
From the above reaction :
Solution This question is based on the study of
2Na2S2O3=l2 =O3 physical aspects of the equilibrium; equilibrium constant
moles ofNa2S2O3 =4Qx — = 0.004 is determined by using equation,
2 2 3 1000 K _ [HC1HHOC1]
moles of I2 = moles of O3 = 0.002 IC12]

volume of O33 at STP = 0.002 x 22.4 L (EUO as reactant is not considered in equilibrium
constant expression).
= 0.0448 L present in 1 L mixture
When equilibrium has been attained [HC1] = [HOC1]
Percentage of O3 =4.48% (by volume) Concentration of acid is determined by pH
measurement:
Problem 2. Although fluorite, CaF2, is its chief pH = -log[H*]
mineral source, fluorine can also be obtained as a
To determine concentration of Cl2, reactions mixture is
by-product of the production of phosphate fertilisers.
treated with KI (adding some NaHCO3 to neutralise acid)
These fertilisers are derived from phosphate rock and titrating liberated L> against standard Na2S2O3
[3Ca3(PO4)2 •CaF2]. What is the maximum mass of (sodium thiosulphate) using starch indicator.
fluorine that could be extracted as by-product from
1000 kg of phosphate rock? (Ca = 40, P = 31, F = 19) Cl2 +2KI ---- > 2KC1+I2
I2 + 2Na2S2O3 ---- > 2NaI + Na2S4O6
Solution 3 Ca3(PO4)2 • CaF2 = 1 F2 sodium sodium
thiosulphate tetrathionate
3(120 + 190) + (40 + 38) = 1x 38
Based on the above reactions,
1008 g phosphate rock gives = 38 g F2
2Na2S2O3 = I2 = Cl2
1000 g phosphate rock gives = 37.698 g F2
Hence, form the concentration of HC1, HOC1 and Cl2
1000 kg phosphate rock gives = 37.698 kg F2 equilibrium constant can be determined.
638 | Essential Inorganic Chemistry

To determine AH (standard enthalpy change) we Solution Let oxidation product of 1“ by MnO4 = 10’
measure value of equilibrium constant K at different
temperature and using van’t Hoff equation, Oxidation number of I in IO“ = n = (2x - 1)
2.303#711T2 . K2 MnO4 is reduced to MnC^“ in the oxidation of I“. Thus,
AH =---------- — log —-
(T2 - Tx) B K.
1“ + MnO4 ----- > MnO^“ + IO“
T T ? T
Problem 5. Describe in outline how you would -1 +7 +R
+6 V* - D
attempt to show that oxide X,O9, reported to be formed by
the action of ozone on iodine, reacts with water according reduction
to the equation, change in ON = 1
51,09 + 9^0 ----- > 18HIO3 + Lj oxidation
Solution Neutralise the solution of a weighted amount change in ON = 2x
of the oxide with NaHCO3 and titrate the I2 against
Na2S2O3 (sodium thiosulphate) solution, Thus, I" + 2x MnO4 ----- > 2x MnO^“ + IO'

12+2Na2S2O3 -—-» 2NaI + Na2S4O6 1 mol I- = 2x mol of MnO4


There is consumption ofNa2S2O3 due to formation of^, 0.01587 1T_ 2rx 0.01587
mol r = mol MnO4
hence, ^Og reacts with HjO to give I2. 127 127
10x0.1
Problem 6. In a strongly alkaline solution containing
excess of barium ions, a solution containing 0.01587 g off 1000
was treated with 0.1 M MnO4 until a pink colour persisted x=4
in the solution. 10.0 mL of the MnO4 solution were
required. Under these conditions MnO^ is converted into Thus, oxidation product of I is IO4 and
the sparingly soluble BaMnO4. What is the product of r + 8MnO; + 8OH“ ----- > 8MnOj“ + IO4“ +4H2O
oxidation of the iodide? (I = 127)
Chapter 15: Group 17 (VIIA—The Halogen Family) | 639

dissolves in excess of thiosulphate. If the


Explanation precipitate is heated with water it turns black,
(a) The polarity ofB—X bonds is in the order: B—F and the supernatant liquid then gives a white
> B—Cl > B—Br but Lewis acidicity shows the precipitate with acidified barium nitrate
sequence: BF3 < BClg < BBr3. solution.
“ With increase in polarity of B—X bond, acidity also - When sodium thiosulphate (Na2S2O3) solution is
increases and should be thus in the order : added to silver nitrate (AgNO3) solution, there is a
BF3 > BC13 > BBr3 white precipitate due to the formation silver
thiosulphate (Ag2S2O3); these precipitate dissolve in
But Lewis acidity is in reverse order: excess of sodium thiosulphate due to the formation of
BF3 < BC13 < BBr3 the complex [AgfS2O3)213-.
There is lateral overlap of the vacant 2p orbital of B
with one completely filled orbital of F leading to pn-pit 2AgNO3 + Na2S2O3 •> Ag2S2O3X + 2NaNO3
bonds between B and F. This B—F bond thus acquires silver thiosulphate
(white precipitate)
double bond character. This also leads to compensate
electron deficiency of boron and thus Lewis acid character Ag2S2O3 + 3Na2S2O3 2Na3[Ag(S2O3)2 ]
of BF3 is reduced. This pit-pit bonding decreases going sodium argentothiosulphate
fromBEj toBBr3 and thus Lewis acidic nature increases in
the order: If the precipitate of Ag2S2O3 is heated, there is
disproportionation giving black precipitate of Ag2S and
BF3 < BC13 < BBr3 soluble F^SOi which remains in the upper (supernatant)
layer which gives white precipitate (of BaSO4) with
B excite 1HI Ba(NO3)2 solution.
ground state 2/ 2p2
Ag2S2O3 + H2O AgS X + H2SO4
2s1 white precipitate black precipitate
2s1
excited state
H2SO4 + Ba(NO3)2 BaSO4 X + 2HNO3
white precipitate
F F.
B----- F B: :F (e) Silver chloride and silver iodide are soluble in
F F saturated aqueous potassium iodide, but
insoluble in saturated aqueous potassium
chloride.
(b) NF3 is stable while NClg and Nig are explosive. - Iodide complex of silver must be more stable than
~ In NF3 the electronegativity difference chloride complex.
(N= 3.0; F= 4.0) permits strong - pK bonding and (f) Although the hydrogen bonding in hydrogen
the molecule is stable. But in NC13 and NI3, the fluoride is stronger than that in water, water
electronegativity difference is nil or small and the has much the higher boiling-point.
molecules are unstable with pK - pK bonding weak.
Hence, NF3 is a stable gas, NCI3 an explosive liquid - Hydrogen fluoride vapour is polymeric, hence, the
and NI3 an explosive solid. hydrogen bonds are not broken on volatisation; those
in water are broken thus requiring higher enthalpy.
(c) In the series HXO, the order of stability is : Hence, boiling point ofH2O is higher than that of HF.
HOC1 > HOBr > HOI
(g) Fluorine combines with other elements in the
~ HOX and OX" are unstable and undergo higher oxidation states.
disproportionation:
- Following factors contribute the unique ability of
ocr —> cr +cio3 fluorine to combine with other elements in their
+1 -1 +5
higher oxidation states.
The instability is due to decrease in bond energy from (i) the highest oxidising power
chlorine to iodine.
(ii) high lattice energy of fluoride
(d) Silver nitrate gives a white precipitate with (iii) high enthalpy of hydration of fluoride
aqueous sodium thiosulphate; the precipitate
(iv) lower F—F bond energy of F—F bond
640 | Essential Inorganic Chemistry

(h) The bond dissociation energy increases from Jg acid. On hydrolysis CIO" forms HC1O (weak acid) and
to Cig. Thus, bond dissociation energy of F2 free OH" ions which makes the solution alkaline.
should be higher than that ofCl^.But it is not so. C1CT + H2O ■> HC1O + OH"
(weak (free)
- Fluorine has the smallest size of all the halogens. acid)
There is considerable repulsion in the non-bonding
(k) Tap water requires fluoridation.
electrons which weakers the F—F bond. Thus, bond
dissociation energy ofF2 is less than that ofCl2. — The main constituent of tooth enamel is
Ca5(OH)(PO4)3 called hydroxyapatite, which in
(i) HF is the weakest acid
presence of fluoride ions is converted into harder,
- HF has the highest bond dissociation enthalpy acid-resistant fluorapatite, Ca5F(PO4)3
amongst all the halogen acids. The hydration
enthalpy of the fluoride ion is also highest, the former Cas(OH)(PO4)3 +F- ---- > Ca5F(PO4)3+OH"
predominates making HF the weakest of the halogen Thus, addition of fluoride makes the teeth harder and
acids. prevents their decay. If we use tap water deficient in
(j) Salts ofhypohalous acids are alkaline in nature fluoride, it can cause formation of cavity.
due to hydrolysis. Imp Paradoxically, consuming 2 ppm or more
- NaClO (sodium hypochlorite) is a salt of HC1O fluoride also causes mottling of teeth.
(hypochlorous acid) and NaOH. HC1O is a fairly weak
...a

Master Exercises
Exercise 1
(Stage 1: Learning)
Short Answer Type Questions 9. Name the halogen
1. Sea water contains around 65 x IO"3 g of bromide ions (a) has two complete electron shells below the
valence shells
in 1 dm 3. If all the bromide ions are converted into Bi^, (b) is solid at room temperature
how many dm3 of sea-water are needed to produce (c) has highest electronegativity value
1 kg of bromine Bi^ ? (d) lowest ionisation energy
2. Complete the following reactions: (e) lowest formation of X(g)
(a) CH3CH=CH2 +IC1 ---- > 10. Complete the following reactions
(b) Cl2+NaOH ---- > (a) 1, +IF3 ---- > (b) Cl2 + 1^0 ->
hot (c) Cl2 +H2S ---- >
(c) Cl2+S2- —> 11. Write the names of the following oxoacids and deduce
(d) SO2 + L, + H2O -> the oxidation number of halogen atom in each
(e) SnCl2 + HC1 + Ij (a) HOC1 (b) HOC1O
(c) HOBr (d) HOBrO2
3.13 is formed when^ combines with I". Which of them is (e) HOIO3 (f) HOCIO3
a Lewis base?
4. Which is. stronger base in each pair? Analytical Questions
(a) C1O2" CIO3 (b) OH" CN"
1. The redox reaction involving the reducing power of
(c) I" cr (d) CH3CH^ CH2=CH" hydrogen sulphide is :
(e) NOj no2 (f) H2PO2" HaPOg
S + 2H+ + 2e"---- > HjS, ^S/HaS = +0.14V
5. Iodine pentoxide, IjOg, is sometimes used to remove Two other half equations are :
CO from the air by forming CO2 and Ij. Write a Fe3+ + e" ---- > Fe2+ E'Fe3*/Fe'2+ = + 0.77 V
balanced equation for this reaction and identify
species that are oxidised and reduced. Bi^ + 2e“ ---- > 2Br E°Br2/Br“ = 1.07 V
6. HOC1 can also be prepared by passing CO2 through a Under standard conditions:
suspension of bleaching powder in water. Write (i) will hydrogen sulphide react with iron (III) ions?
balanced equation for this reaction. (ii) will it react with bromine?
7. C12O6 is diamagnetic but in liquid state, it shows If you decide a reaction should take place, write the
paramagnetism. Explain. equation.
8. Identify A to F in the following : 2. Explain:
(i) Electron affinity of Cl is the highest among the
AgC103+A ----- ► Ct + Dt halogens; yet,F2 is the strongest oxidising agent.
(ii) C1O2 is a free radical with one unpaired electron
cone. HC1 O3 KOH A but has no tendency to dimerise likeNO2.
MnO2
(iii) HCl(g) and HC1 (aq) have different behaviour
D + CI2O6 E + F + H2 0 towards litmus.
642 | Essential Inorganic Chemistry

3. 0.01 M solutions of iodine in n-hexane, benzene, n-hexane, benzene and ethanol, all of these beconu
ethanol and pyridine are violet, purple, brown and yellow, Explain.
yellow respectively. When 0.001 mol of pyridine is
added to 100 mL of each of the solutions of iodine in

Exercise 2
(Stage 2 : High Skill Problem Solving)
Only One Option Correct 9. Which is/are true statement(s)?
1. Of the following acids (a) Basic nature of X"is in order F" > Cl" > Br” > I*
I: Hypophosphorus acid (b) HI is strongest acid of HF, HC1, HBr and HI
II: Oxalic acid (c) The ionic character of M—X bond decreases in
III: Glycine the order M—F > M—Cl > M—Br > M—I
(a) I, II are monobasic; III dibasic and amphoteric (d) All the above are correct statements
(b) II monobasic; I, dibasic acid; III amphoteric 10. Select incorrect statement.
(c) I monobasic, II, dibasic, III amphoteric (a) C1O2 and C12O are used as bleaching agents for
(d) I, II dibasic; III amphoteric paper pulp and textiles
(b) OC1" (hypohalites) salts are used as detergent
2. Which incorrect statement?
(c) OC1” disproportionates in alkaline medium
(a) All halogens form oxyacids
(d) BrO3 is oxidised to Br2 by Br" in acidic medium
(b) All halogens show -1, +1, +3, +5, +7 oxidation
states 11. Consider the following boron halides:
(c) Hydrofluoric acid forms KHF2 and and I.BF3 II.BCI3
attacks glass II. BBr3 IV.BI3
(d) Oxidising power is in order F2 > Cl2 > Bi^ > The Lewis acid characters of these halides are such
that
3. HC1O4, HNO3 and HC1 are all very strong acids in (a) I<II<III<IV (b) I<III<II<IV
aqueous solution. In glacial acetic acid medium, their (c) IV<III<II<I (d) IV<II<III<I
acid strength is such that 12. Which of the following compounds possesses Lewis
(a) HC1O4 > HNO3 > HC1 acid character?
(b) HNO3 > HC1O4 > HC1 I. BF3 II. SiF4
(c) HC1 > HC1O4 > HNO3 III. PF5
(d) HC1>HC1O4 ~HNO3 Select the correct answer using the codes given below
4. Tincture of iodine is Codes
(a) CHI3 in alcohol (b) Ij in alcohol (a) I alone (b) I, II and III
(c) la in KI (d) CHI3 in KI (c) II and III (d) I and III
13. Inter-halogen compounds can be
5. “Chlorine-type" laundry bleaches are in reality
(a) ICI3 (b) BrF5
aqueous solution of (c) IF7 (d) all of these
(a) HOC1 (b) HCIO3
14. Oxidation of hydrogen halide, HX affords a method
(c) NaC103 (d) NaOCl for the industrial and laboratory preparation of the
6. The active constituent of bleaching powder is halogen, X2, in the free state in respect of all of the
(a) Ca(OCl)2 (b) Ca(OCl)Cl following except
(a) fluorine (b) chlorine
(c) Ca(C102)2 (d) Ca(ClO2)Cl
(c) bromine (d) iodine
7. Which one of the following is the weakest base as per 15. Which of the following statements are correct for all
the Bronsted concept? three halogens (X), Cl, Br and I?
(a) [C1O41" (b) [C103r (a) They all need to gain one electron to acquire
(c) [cio2r (d) tcior stable configuration
8. Which one of the following is not a true “per acid”? (b) They all form strong acid of the type BX
(a) HC1O4 (b) HgSOs (c) Both (a) and (b) are correct
(c) H2S2O8 (d) H3PO5 (d) None of the above is correct
Chapter 15: Group 17 (VIIA—The Halogen Family) | 643

16. Which of the underlined atoms in oxyacids have sp3 28. An important product in the ozone depletion by
hybridised atoms? chlorofluorocarbons is
(a) HCIO4.H2SO4.HNO2 (a) Cl2 (b) OC1
(c) F2 (d) O2F2
(b) H2SO4.H3PO4.HNO3
29. Which one is the strongest bond?
(c) HC^.H^.HjSOg (a) Br—F (b) F—F
(d) HCIO4, HNO3, HCIO3
(c) Cl—F (d) Br—Cl
17. In the following
30. The interhalogen compound not obtained is
CsL3 will ionise
(a) Csl ionise* to trivp Cs
t.n give Cs+ and T7
I3
(a) IC1 (b) IF5
(b) RbICl2 will give RbCl and IC1
(c) BrF5 (d) BrCl7
(c) KBrICI will give KC1 and IBr
(d) all of the above are correct 31. Which of the following will displace the halogen from
the solution of the halide?
18. Which of the following atoms has the highest
(a) B^ added to Nal (b) B12 added to NaCl
ionisation energy? (c) Cl2 added to KC1 (d) Cl2 added to NaF
(a) F (b) Cl
(c) Br (d) I 32. Which is the strongest oxidising agent out of the
following?
19. Which one of the following arrangements does not
(a) I2 (b) Cl2
truly represent the property indicated against it?
(c) B12 (d) F2
(a) Be, < Cl2 < F2 Oxidising power
(b) Br<Cl<F Electronegativity 33. The high oxidising power of fluorine is due to
(c) Br<F<Cl Electron affinity (a) high electron affinity
(d) Bij < Cl2 < F2 Bond energy (b) high heat of dissociation and low heat of
hydration
20. The mixture of concentrated HC1 and HNO3 made in (c) low heat of dissociation and high heat of
3:1 ratio contains hydration
(a) C1O2 (b) NOCI (d) high heat of dissociation and high heat of
(c) NC13 (d) N2O4 hydration
21. The formula for calcium chlorite is 34. When Cl2 is passed through cold dil. NaOH, the
(a) Ca(C104)2 (b) Ca(C103)2 products are
(c) CaC102 (d) Ca(C102)2
(a) NaCl, NaOCl and H2O
22. Which one of the following oxoacids of chlorine is the (b) NaCl, NaC102 and l^O
-least oxidising in nature?
(c) NaCl, NaCIO3 and H2O
(a) HC1O (b) HC1O2
(c) HCIO3 (d) HC1O4 (d) NaCl,NaC104 and 1^0
23. Least stable oxide of chlorine is 35. The greater reactivity of F2 is due to
(a) C12O4 (b) C1O2 (a) lower electron affinity of F
(c) C12O7 (d) C12O6 (b) lower bond energy of F—F bond
(c) higher electronegativity of F
24. Which reaction is not feasible?
(d) gaseous state of F2
(a) 2KI + BI2 ---- > 2KBr + l2
36. Dissociation of HI into Hj and I2 is
(b) 2KBr + l2 ---- * 2KI + Bi2
(a) kinetically controlled
(c) 2KBr + Cl2 ---- > 2KC1+Bi2
(b) thermodynamically controlled
(d) 21^0 + 2F2 ---- > 4HF + O2 (c) kinetically as well as thermodynamically
25. Shape of O2F2 is similar to that of controlled
(a) q,F2 (b) H>02 (d) neither kinetically nor thermodynamically
controlled
(0 H.F2 (d) C^Hjj
37. In the preparation of HBr or HI, NaX (X = Br, I) is
26. Iodine test is shown by
(a) glucose (b) starch treated with H3PO4 and not by cone. and since
(c) glycogen (d) polypeptide (a) HgSC^ makes the reaction reversible
(b) H0S04 oxidises HX to X2 (Br, I)
27. Which one will liberate Bi^ from KBr?
(c) Na3PO4 is water soluble
(a) 12 (b) SO2 (d) Na2SO4 is water soluble
(c) HI (d) Cl2
644 | Essential Inorganic Chemistry

■38. Acid strength ofHX(X = F, Cl, Br, I) is dependent on 48. 10 g of bleaching powder on reaction with KI required
(a) the electronegativity differences of H and X 100 mL of 1 N hypo. Thus, % of pure bleaching
(b) the tendency of the hydrated molecule to form H+ powder is
(c) both (a) and (b) (a) 100% (b) 80%
(d) None of the above (c) 63.5% (d) 35.5%
39. Select correct statement(s) regarding behaviour of 49. CaOCl2 + HgO + CO2 - ► CaCO3+CaCl2+H0C1
HF as non-aqueous solvent. This reaction is simply
(a) HC1 behaves as an acid and HF as a base (a) oxidation
(b) HC1O4 behaves as a base and HF as an acid (b) reduction
(c) HN03 and H2SO4 behave as base and HF as an (c) redox
acid (d) hydrolysis in presence of CO2
(d) All of the above are correct statements 50. CH^H^OH on reaction with bleaching powder forms
40. Dipole moment and ionisation constant are (a) CHC13 (b) CC13CHO
maximum in case of (c) CHjClCHO (d) CC14
(a) HF, HF (b) HF, HI 51. Cl2O,Br2O, have positive value of AG (free energy)
(c) HI, HF (d) HI, HI
indicating that
41. The tetrahedral arrangement of perchlorate ion is (a) these oxides are stable
due to (b) these oxides are unstable and changes to X2 and
(a) presence of a lone pair of electrons O2
(b) trigonal bipyramidal shape of the ion (c) these disproportionate into X~ and XO
(c) sp3 hybridisation (d) these oxides can form interhalogen compounds
(d) sp2 hybridisation
52. Select correct statement(s).
42. Acid strength of oxoacids of chlorine is in order (a) C12O and C1O2 are used as bleaching agents and
(a) HOC1 < HC1O2 < HC1O3 < HC1O4 as germicides
(b) HC1O4 < HC1O3 < HC1O2 < HOC1 (b) is used in the quantitative estimation of CO
(c) Bond angle XOX varies in the order
(c) HC1O3 < HC1O4 < HC1O2 < HOC1
(d) HC1O2 < HC1O3 < HOC1 < HC1O4 FOF < C1OC1 < BrOBr
(d) All of the above are correct statements
43. Acid strength of oxoacids of halogen is in order
(a) HOF > HOC1 > HOBr > HOI 53. The bond angle in OF2 out of OF2, Cl20, Br2O is
(b) HOI > HOBr > HOC1 > HOF minimum. It is because in case of OF2
(a) electrons are nearer to fluorine due to high
(c) HOI >HOC1> HOBr > HOF electronegativity of F compared to Cl to Br
(d) HOC1 > HOBr > HOI > HOF (b) lone pair-lone pair repulsion decreases bond
44. Which has peroxy linkage? angle
(a) Perchloric acid (b) Hypochloro acid (c) both (a) and (b) are correct
(c) Para perchloric acid (d) None of these (d) None of the above is correct
45. SO2 can reduce 54. C1O2 is the anhydride of
(a) HC1O3 to HC1 (b) toCr3+ (a) HOC1 (b) HC1O2
(c) MnO4-/H+toMn2+ (d) all of these (c) HC1O3 (d) HC1O2 andHC103
46. Bleaching powder contains 55. C1O3 is the mixed anhydride of
(a) CaO and Cl2 molecules (a) HC1O2 andHC103 (b) HC1O3 andHC104
(b) Ca2+ and OC12" ions (c) HC1O2 andHC104 (d) HC102 andHC103
(c) Ca2+,Cr andOCl" ions 56. Chlorine dioxide
(d) Ca2*,©2" ions and Cl2 molecule (a) is paramagnetic in nature
(b) has odd-electron bond
47. Bleaching powder is disinfectant for purification of
(c) is stable
water when water born germs are killed. But
(d) has all the above properties true
disinfectant activity is destroyed. It is due to its
disproportionation into 57. Select incorrect statement(s).
(a) CaCl2 and Cl2 (a) C12O6 is diamagnetic in solid state
(b) CaCl2 and Ca(C1033'2
)2 (b) C1O3 is paramagnetic and exists in liquid state
(c) CaO and Cl2 (c) C12O6 is anhydride of HC1O3
(d) CaO, Cl2 and CaCl2 (d) C1O3 forms dimer due to unpaired electron
Chapter 15: Group 17 (VIIA—The Halogen Family) | 645

58. Cl2 gas is passed into aqueous solution of KBr and KI 5. In the conversion ofBrO3 toBrO4
andCHCl3 added. It is observed that there is (a) there is no change in oxidation number
(a) violet colour in CHC13 layer (b) reduction takes place in basic solution
(b) yellow colour in CHC13 layer (c) reaction also takes place by XeF2
(c) yellow colour in aqueous layer (d) equivalent mass ofBrO3 is one-half of ionic mass
(d) violet colour in CHC13 layer
6. Select correct statement(s).
59. Consider following reactions in which atoms have (a) Chile saltpetre (NaNO3) is a main source of Ij
been labelled with isotopes (indicated by *) (b) IO3 is reduced to 1“ and which in turn oxidised to
(I) S*SO|- +2H+ ---- > H2SO4 +S
Ig by IO3 in acidic medium
(II) NagHgl^ +2AgNO3 ---- > 2AgI + Hgl2
(c) Anhydrous HF is a non-conductor of electricity
+ 2NaNO3 (d) F2 is obtained by electrolysis of molten KHF2
In which case isotopes are equivalent in the
7. Select ocrrect statement(s).
products?
(a) I (b) II (a) Colours of the halogens (F2, Cl2, Br,, ) in the gas
(c) Both I and II (d) None of these phase are complimentary colours.
(b) Colour is due to transition of an electron from the
60. In very recent Sunami in Japan, food materials were
highest occupied n * molecular orbital to the
banned as they were contaminated with
radioactivity. Which of the following was found to lowest unoccupied a * molecular orbital
have radioactive beyond prescribed limit? (c) C1O2 is paramagnetic due to unpaired electrons
(a) F (b) Cl (d) C1O2 dimerises toC12O4 asNO2 dimerises to N2O4
(0 I (d) At 8. Base on E° values gives
-F2 +e* + F" E° = 2.87 V
One or More Than One Options Correct 2 2
1. Which are correct statements? -Cl2 +e~ ■> cr E° = 1.40 V
2 2
(a) All halogens form oxoacids.
(b) All halogens show -1, +1, +3, +5, +7 oxidation | B^ + e~ ---- > Br" E° = 1.09 V
states.
(c) Hydrofluoric acid forms KHF2 and and -L+e"
22
> r E° = 0.62 V
attacks glass.
(d) Oxidising power is in order F2 > Cl2 > > Ij (a) Oxidising power of F2 is maximum
2. Select correct statement(s). (b) Oxidising power of L, is maximum
(a) C1O2 and C12O are used as bleaching agents for (c) Oxidising power is in order F2 > Cl2 > B^ >
paper pulp and textiles (d) Oxidising power is in order F2 < Cl2 < B^ <
(b) OC1~ salts are used as detergents 9. CHCI3 and H2O form two separate immiscible layers
(c) OC1" disproportionates in alkaline medium B% (orange) and L> (violet) are soluble in organic
(d) BrO3 is oxidised to Br2 by Br" in acidic medium layer due to a greater extent. In test tube A
3. In the structure of (KI + CHC13-t-H^O) and test tube B
(a) IF2+, bond angle is smaller than the tetrahedral (KI + KBr + CHC13 +H2O) Cl2 gas is passed then
angle because of lone pair-lone pair repulsion (based on E° values in Q. 8)
(b) IF4+, there is sp3 hybridisation with bond angle of (a) organic layer in A appear violet
109°28' (b) organic layer in B appear orange
(c) IFg, there are six bonding electrons and no lone (c) organic layer in B appear violet
pairs. (d) organic layer in B first appears violet then orange
(d) IF4+, two F-atoms one at equatorial positions and
10. Select correct statement(s).
two are inclined at an angle less than 120° due to
lone pair-bond pair repulsion (a) pKa value of HI (strongest halogen acid) is most
negative
4. In the conversion of Br" to BrO2 (b) high H—F bond strength makes H—F a weak
(a) there is anodic oxidation is basic solution acid in dilute aqueous solution
(b) equivalent mass of Br" is 20 (atomic weight of (c) Halogen forms clatharates by freezing solution in
Br = 80)
water
(c) there is anodic oxidation is acidic medium
(d) there is cathodic reduction in basic solution (d) Ka values of HZ is in order HF < HC1 < HBr < HI
646 | Essential Inorganic Chemistry

Passage Comprehension Questions 4. Equivalent mass of Br2 is

Passage 1 (a) y (b)


3
In the gas phase, colours of the halogens very , , 2M M
(d)
remarkably:
By­ 6
Fluorine — Pale yellow
Chlorine — Pale greenish-yellow Passage 3
Bromine — Deep red brown This question concerns the elements of Group 17 -
Iodine — Deep violet fluorine, chlorine, bromine and iodine.
Answer the following questions When sodium chloride is treated with concentrate
sulphuric acid, a colourless gas, X, which fumes in moisi
1. This colour is due to
air, is formed. When sodium iodide is treated in the saint
(a) transition of an electron from the highest
way a coloured vapour, 7, is product.
occupies 7t* molecular orbital to the lowest
If 90% phosphoric(v) acid is used instead of sulphuht
unoccupied o * molecular orbital acid, a colourless gas is produced in each reaction.
(b) occupied o * MO to the lowest unoccupied n* MO
(c) Occupied o MO to the lowest unoccupied m MO 1. Gases X and Y are respectively
(d) None of the above (a) Cl2,12 (b) HCl,^
(c) HC1, HI (d) HClO.Ij
2. As one moves from F2 to Lj, difference between the two
2. With 90% phosphoric acid (HgPO4), colourless gases
energy levels
(a) decreases and probability of transition increases formed are
(b) increases and probability of transition increases (a) HC1, HI (b) Cl2, Ij
(c) remains unchanged and probability of transition (c) (d) C12,HI
remains unchanged 3. Behaviour of H2SO4 and HgPC^ is different towards
(d) depends on the temperature Nal because
3. Due to the above transition, absorption maximum (a) HgSC^ is stronger acid than H^PC^
gradually shifts from (b) H2SO4 is a strong oxidising agent and oxidises
(a) red region of the visible spectrum to near UV colourless gas to coloured vapour 7
region (c) H3PO4 is tribasic acid
(b) the near UV region to the red region of the visible (d) HjSC^ is a dibasic acid
spectrum
(c) the near UV region to the far IR region
(d) the near IR region to visible region Passage 4
Water pollution can be by dissolved gases. Bad smeD
of water is due to H^S and can be removed by treatment
Passage 2 with bleaching powder.
Consider following change,
CaOCl2 + 1^0---- > Ca(OH)2(ag) + Cl2(ag)
BrO2 ---- > B^O + A bleaching powder
Bi^O and A are formed by the change of equal number HySCog) + Cl2(aq) ---- > 2HCl(ag) + S(s)
oxidation number. Based on this change, answer the
Based on the above treatment answer the following
following questions.
questions:
1. Oxide A could be
1. If H^S content of contaminated water is 22 ppm by
(a) B^Og (b) Br>O5
mass, Cl2 required to remove all the H2S from 200
(c) B^O-j (d) BrO gallons of water (1 gallon = 3.785 L) is
2. Balanced equation is (a) 30.00 g (b) 71.00 g
(a) 4BrO2 ---- > Bj^O+B^Ot (c) 17.5 g (d) 35.00 g
(b) 8BrO2 ---- > B^O + SB^Og 2. 25.0 L of a natural gas measured at 25°C and
(c) 6BrO2 ---- » Bi^O + 2Bi2O3 740 mm Hg is bubbled through Pb2+(a<?), a precipitate
weighing 0.3938 g is obtained. %o(ppt—parts per
(d) None of the above
thousand) of HgS by volume in the natural gas is,
3. Bi^O has structure (Pb=206)
(a) Br—Br=O (b) Br—0—Br (a) 0.225%o (b) 2.25%o
(c) Br=O=Br (d) None of these (c) 5.25%o (d) 0.525%o
Chapter 15: Group 17 (VIIA—The Halogen Family) | 647
1

Assertion & Reason True & False


Codes: 1. FC1O3 (perchloryl fluoride) has the highest dielectric
(a) Both A and R are true and R is the correct constant.
explanation of A. 2. HF is a weak acid but H (FHF) is a strong acid.
(b) Both A and R are true but R is not the correct
explanation of A. 3. Solid CaF2 could emit light when heated.
(c) A is true but R is false. 4. When NaCl(aq) is electrolysed, sodium metal is
(d) A is false but R is true. formed at the cathode and chlorine gas is formed at
1. Assertion (A): CaF2 has been given the name the anode.
fluorspar. 5. It very recent Japan Sunami, there has been increase
in radioactive iodine content is salt.
Reason (R): Solid CaF2 emits light when heated.
6. When Cl2 gas is dissolved in water, there is
2. Assertion (A): HF is stored in the vessel made of disproportion to HC1O and HC1.
wax.
7. Solid propellent in the rockets is aluminium powder
Reason (R): HF is a weak acid. with ammonium perchlorate as reducing agent.
3. Assertion (A) : Fluorine has only -1 oxidation 8. Br> oxidised KC1 into Cl2 and KI into Jg.
state in compounds.
9. C1O3 has structure with sp2 hybridisation having
Reason (R) : Fluorine has maximum
bond angle of 120°.
electronegativity and electron-affinity.
10. IF6+ has regular octahedron structure.
4. Assertion (A): Cl2 or Br2 changes KI into (violet
in CC14 layer), Cl2 changes KBr into Bifc 11. Interhalogen compounds are strong oxidising agents.
(orange-yellow in CC14 layer). Thus, Cl2 would 12. Interhalogen compounds are strong reducing agents.
change mixture of KI and KBr into violet layer in 13. X—Ybond is more polar than X—X and Y—Y bonds.
CC14.
14. Interhalogen compounds are ionic.
Reason (R) : Oxidising power is in order
15. XY3 types of molecules have square pyramidal
\ <Br> <C12 <F2.
structure.
5. Assertion (A): Fluorine is not super halogen.
Reason (R): It is most reactive of all the halogen. Fill in the Blanks
6. Assertion (A): Molten is conducting in nature. 1. FC1O3 is an inert and non-corrosive gas with
Reason (R): Bond dissociation energy increases in structure.
the order 2. Perchloryl fluoride (FC1O3) is used as a rocket
Br2 < Cl2 < as well as an in high-voltage
7. Assertion (A): Hydrofluoric acid forms two series system.
of salts KHF2,K2F2. 3. BrF3 has shaped structure.
Reason (R) : It is a dibasic acid due to hydrogen 4. C1F3 is a in the reaction
bonding.
3 +AsF3 ---- > [ClF2]+[AsF6F
ClF3
8. Assertion (A): CN" can be oxidised to (CN)2. and is a in the reaction
Reason (R): CN" is pseudohalide. C1F3+NOF ...-> [NO]+[ClF4r '
5. IF7 has structure.
9. Assertion (A) : Bond dissociation energy of F2 is
smaller than that ofCl2. 6. Decreasing order of reactivity of F2, C1F and C1F3 is
Reason (R) : Bond dissociation energy is in order
F2 < Cl2 < Bib < I2. 7. CN, CNO’.CNS’ are called
10. Assertion (A) : Aqueous solution of NaOX is 8. Out of chloric acid (HC103) and perchloric acid
alkaline. (HC104) more stable is
Reason (R) : It is a salt of weak acid (HOX) are 9. Chlorous acid is unstable and decomposes into
and
strong base (NaOH).
648 | Essential Inorganic Chemistry

10. Ammonium perchlorate is used as the in the Set I


solid booster rockets for the space shuttle.
1. pCl and f7Cl are two isotopes of chlorine in mas
11 displaces three halogens from their
compounds. ratio of3 :x where x =
12. Cl~ can be oxidised by 2. CaOCl2 (bleaching powder) has two types of chlorine
13. Br2 can be reduced by with different oxidation numbers. Sum of oxidation
number is
14. Fluorine and can oxidise water to oxygen.
3. Number of series of salts formed by hydrofluoric acid
15. Oxidation of and by Cl2 will give
is
Br2 and L respectively.
4. Cl2 disproportionates to Cl" and species A such that
Matrix-Match Type Questions equivalent mass ofCl2 is 71. Thus, oxidation number
of chlorine in species A is
I. One or More Correct 5. Number of electrons forming bond between Cl and 0
1. Match the oxides in Column I with their is C1O2 is
corresponding properties in Column II.
6. Maximum acidic character is shown by the oxyacids
Column I Column II of halogens with oxidation number of halogens as
A. CI2O 1. Anhydride of HOCI
B. CIO2 2. Anhydride of HCIO2
7. Br and BrO~ in acidic medium forms Bi^ where x =
C. CI20g 3. Anhdride of HCIO3
D. CI2O7 4. Anhydride of HCIO4 8. 3C12 4-xNaOH NaCl +NaC103. xis
5. sp3 hybridization
6. Paramagnetic in gaseous state Set II
7. Paramagnetic in liquid state
1. How many lone-pairs and bond-pairs are present in
2. Match the species in Column I with the IF7?
corresponding properties in Column IL 2. How many lone-pairs are present in C1I3 ?
Column I Column II
3. C1O2 has odd-electron bonds. How many electrons are
A. SO 2 1. Turns lime water milky
B. CO2 2. Turns Cr2O7‘/H+ solution green involved in (Cl—0) bond?
C. Cl2 3. Turns Kl/starch blue 4. C1O2 is paramagnetic due to unpaired
D. Br2 4. Disinfectant for drinking water electron(s).
E. O3 5. Green house effect
5. How many of the following have underlined atoms m
6. Acid rain
7. Controls pH of blood
different oxidation states?
I^Hg^, CaOClg, Na2 SA, Na2 SA, H2 S05, H2S2O^,
Integer Answer Types CrO5, I3
This section contains 8 questions. 6. How many of the following have Cl atom in spJ
X Y z IV
The answer to each of the questions is a hybridised state?
single digit integer, ranging from 0 to 9.
© © © ©
The appropriate bubbles below the
CD 0 cio2, ci2o, ci2o7, ci2o6, cio;
respectively question numbers in the © © © © 7. How many of the are water insoluble?
ORS have to be darkened. For ® ® ® AgF, AgCl, AgBr, Agl, CaF2,BaSO4
example, if the correct answers to ® ©
question number X, Y, Z and W (say) © © © © 8. 6.35 of impure bleaching powder paste is mixed with
are 6, 0, 9 and 2, respectively, then the © © ® ® KI and I3 formed required 50 mL of 0.1 M hypo to
correct darkening of bubbles will like © © © neutral medium. Thus % purity of bleaching powder
the following ® ® ® ® is
© © © ©
Answers & Solutions
Master Exercises
Exercise 1
Short Answer Type Questions 11. (a) hypochlorous acid +1
1- Br-—4Br2+e- (b) chlorous acid +3
(c) hypobromous acid +1
65x 10"3 g Br“ = 32.5 xlO-3 g Br2 is in 1 dm3 (d) bromic acid +5
1000 g Br2 is in =—1000-^ = 30769 dm3 (e) periodic acid +7
2 32.5 x 10"3
(0 perchloric acid +7
2. (a) CH3—CH—CH2 (IC1 ---- » I+ + CD

Cl I
Analytical Questions
(b) 3C12 + 6NaOH---- > 3NaCl + NaClO3 + 3H2O 1. (i) Oxidation of H^S by Fe 3+ can be written as
(c) ci2 + s2----- > 2CP + S Oxidation
(d) SO2 +12 + 2H2O---- > H2SO4 + 2HI
2Fe3+ + > 2Fe2+ + 2H+ + S
(e) SnCl2 + 2HC1 + L ---- > SnCL + 2HI Reduction t
I" behaves as a Lewis base.
HjS ---- > S + 2H+ + 2e~ E°x=-0.14 V
4. If acid is strong, its conjugate base is weak and
vice-versa. 2Fe3+ + 2e~ ---- > 2Fe2+ £°d= 0.77 V
(a) C1O2 (b) OH­ (c)Cr (d)CH3CH2 (e)NO- ( = 0.63 V
TO
Thus, E''cell
IO almost same
Thus, AG° = - nFEc°el] = - ve.
5. I2O5 + 5CO---- > I2 + 5CO2
Thus, oxidation of HgS by Fe3' is spontaneous.
I2OS is reduced to I2, CO is oxidised to CO2
(ii) Oxidation of by Br2 can be written as
6. 2Ca(OCl)Cl + H2O + CO2---- > CaCl2 + CaCO3 + 2HOC1
Br-j+H^ ---- > 2H+ + S + 2Br-
7. C12O6 has paired electrons. In liquid state or in aqueous [ Oxidation |
solution Cl—Cl bond breaks giving one unpaired Reduction
electron hence, diamagnetic nature of C12O6 changes to
paramagnetic nature in C1O3. 2H* + S + 2e“ =-0.14 V
0.0 0
I Br2 + 2e" - 2Br~ ^°ed = 1.07 V
0—Cl—Cl—0 O-Cl*
Thus, E°u =0.93 V
O O 0 AG° = -nFE°ce]1 = -ve
8. (A):Cl,12 (B):AgCl (C):C1O2 (D):O2 (E):KC1O2
Hence, oxidation of H^ by Br2 is spontaneous.
(F):KC1O,. V
2. Refer text
9. (a) chlorine (b) iodine (c) fluorine (d) iodine
3. Solution in n-hexane contains uncomplexed I2 molecule;
(e) chlorine
charge-transfer complex formed with benzene; probably
10. BIF
0— and N—donor complexes with ethanol and pyridine;
—> HC1 + HOC1 pyridine complex being the most stable.
—> 2HC1+ S
650 | Essential Inorganic Chemistry

Exercise 2
Only One Option Correct
1. (0 2. (b) 3. (a) 4. (b) 5. (a) 6. (b) 7. (a) 8. (a) 9. (d) 10. (b)
11. (a) 12. (a) 13. (d) 14. (a) 15. (c) 16. (c) 17. (d) 18. (a) 19. (d) 20. (b)
21. (d) 22. (d) 23. (a) 24. (b) 25. (b) 26. (b) 27. (d) 28. (b) 29. (c) 30. (d)
31. (a) 32. (d) 33. (c) 34. (a) 35. (b) 36. (a) 37. (b) 38. (c) 39. (c) 40. (b)
41. (c) 42. (a) 43. (b) 44. (d) 45. (d) 46. (c) 47. (b) 48. (c) 49. (d) • 50. (a)
51. (b) 52. (d) 53. (a) 54. (d) 55. (b) 56. (d) 57. (c) 58. (a) 59. (b) 60. (c)

One or More Than One Options Correct 9. HC1,C1O2 10. oxidises 11. F2 12. F2
1. (c,d) 2. (a,c,d) 3. (a,c,d) 4. (a,b) 13. r 14. chlorine 15. Br-,1’
5. (a,b,c,d) 6. (a,b,c,d) 7. (a,b,c) 8. (a,c)
9. (a,c) 10. (a,b,c,d) Matrix-Match Type Questions
I. One or More Correct
Passage Comprehension Questions 1. (A)-(1,5); (B) — (2,3,5,6); (C) - (3,4,5,7);
Passage 1 (D) —(4,5)
1. (a) 2. (a) 3. (b) 2. (A) —(1,2,6); (B) —(1,5,7); (O —(3,4);
Passage 2 (D) —(3); (E) — (3,4)
1. (0 2. (a) 3. (b) 4. (c)
Passage 3 Integer Answer Types
1. (b) 2. (a) 3. (b) Set I
Passage 4 Questions—► 1 2 3 4 5 £ 7 8
Answers —► 1 0 2 1 3 7 3 6
1. (d) 2. (b)
© © © © © ©T© ©
Assertion & Reason © © ©. © ©
1. (a) fluorspar name is given after fluorescence (meaning
© © © © © ©_©
emitting light) © © © © © ©© ©
© © © © © ©
2. (b) © © ©
© © © © ©
3. (c) Cl has maximum electron-affinity © © © © © ©
4. (a) Cl2 oxidises Br~ toBr2, and Br2 oxidisesI“tol2 (violet) © © © © © ©
© © © © © © © ©
5. (a) I+ and I3 are formed, order is Cl2 < Br2 < I,2
© © © © © © © ©
6. (c) 7. (a) 8. (b) 9. (0 10. (b)
Set II
Questions—► 1 3 4 5 6_ 7 8
True & False 2
Answers —► 7 2 3 1 6 5 5 5
1. T 2. T 3. T 4. F 5. T 6. T
© © © © © ©I© ©
7. F 8. F 9. T 10. T 11. T 12. F ©. © O © O
13. T 14. F 15. F ©. © © © © © © ©
© © © © © ©©
Fill in the Blanks © © © © © ©
1. tetahedral 2. propellant, insulator 3. T © © © © © ©
©. © © © © ©_© ©
4. Lewis base, Lewis acid
© © © © © © 0
5. pentagonal bipyramidal structure © © © © © ©_© ©
6. C1F3 > C1F > F2 7. pseudohalides 8. HC1O4 © © © © © © © ©
It
he Transition and Inner
Transition Elements
"If elements of s- and p-blocks make two hands of the Periodic
Table, then elements of d- and f-blocks are its heart and kidney"

Quick Points
• Electrons in various subshells of a given shell are filled
based on Aufbau rule (ascending energy rule) and last filling
electron decides block. ■ Transition Elements (d-Block)
• Chemical reactivity is based on the position of the element ■ Physical Properties
in electrochemical series (ECS). A particular > Compounds
electrochemical reaction is possible of reducing agent in
■ Inner-Transition Elements (f-Block)
ECS is above oxidising agent, i.e., more negative value of
standard reduction potential (E° n +...)indicates the better
reducing nature of the metal.
• Equivalent mass in a redox reaction
molar mass
change in oxidation number
molar mass
electrons exchanged in a redox reaction
Thus, for (MnO" Mn2+)
molar mass
Equivalent mass of MnO" =
5
i
and for, (Cr2O*- +6e~ ■» 2Cr3*)

molar mass
Equivalent mass ofCr2O2- =
6 ■I
652 | Essential Inorganic Chemistry

Transition Elements (d-Block)


Group III B IV B VB VI B VII B VIII B IB IIB
3 4 5 6 7 8 ~9 10 11 12
Period
4th Sc Ti V Cr Mn Fe Co Ni Cu Zn
5th Y Zr Nb Mo Tc Ru Rh Pd Ag Cd
6th La Hf Ta W Re Os Ir Pt Au Hg
7th Ac Rf uzb Sg Bh Hs Mt Ds

On August 16, 2003 IUPAC approved the name for the element
of atomic number 110, as DARMSTADTIUM with symbol Ds. It is
a super heavy element which was first produced in 1994

Physical Properties Physical properties of 3d-series [Sc(21)—Zn(30)J have


As discussed in the Periodicity, three series of been summarised in Table 16.1.
elements are formed by filling the 3d, 4d and 5d-shells of
electrons. Together these comprise the d-block elements. Comparison of the First Row Transition Metals
They are often called transition elements because their
position in the Periodic Table is between the s-block and Through the d-Electron Configuration
p-block elements. The electrons are added to the The d° configuration Of the simple ions, only Sc3’
penultimate shell, expanding it from 8 to 18 electrons. is known to have this configuration. This configuration
then occurs for those metals in which the formal oxidation
DEEP Focus states equal the total number of 3d and 4s-electron. This is
Thus, transition metals typically have incompletely filled true for Ti(IV), V(V), Cr(VI), and Mn(VII) but for Fe(VIII)
d-subshells or readily give rise to ions with incompletely is unknown.
d-subshells. The group 12 (IIB) metals—Zn, Cd and Hg do not The d1 configuration Except vanadium(IV), all
have this characteristic electron configuration, and so although others with this configuration are either reducing or
they are some times called transition metals, they really do not undergo disproportionation. For example,
belong in this category. disproportionation occurs for Cr(V) an Mn(VI) as follows:
• The elements with a partially filled /-subshell in 3CrOj- +8H+ -----> 2CrOj- +Cr3+ +4^0
common oxidation states are known as /’-block 3MnOj- + 4H+ ----- > 2MnO4’ +MnO2 +2H2O
transition elements or inner-transition The d2 configuration This configuration ranges
elements. They are the two rows elements at the from Ti11 which is very strongly reducing, to Fe171, which is
bottom of the Periodic Table. The elements in the first very strongly oxidising. Vanadium (III) is also reducing.
row are called the lanthanides or rare earths. The
The d3 configuration Chromium (III) is the
elements in the second row are called actinides.
important species with this configuration which is stable
All of the actinides are radioactive, and from
and well known for complex formation. In other cases, this
neptunium (Z = 93) onwards, are synthetic. They have
configuration is relatively unimportant.
been produced in nuclear reactor or by using particle
accelerators. The d4 configuration There are really no stable
• The lowest-energy configuration of the lutetium atom species with this configuration. The chromium (II) is
is [Xe] 4/14 5c?1 6s2, and in its only common oxidation strongly reducing and manganese(III) disproportionates.
state (+3), the configuration is [Xe] 4/14. Although The d5 configuration The two important species
both configurations have a filled /-subshell, lutetium with this configuration are Mn2+ and Fe3+, the latter may,
is usually considered a lanthanide element. however, be reduced to Fe2+.
The d6 configuration Iron(II) and cobalt(III) are
DEEP Focus important species with this configuration. Iron(II) is quite
stable although a mild reducing agent and cobalttfll) is
• Nuclear charge increases from Sc to Cu, but electrons are
being added to the inner 3d-subsheH. These 3d electrons stable in the presence of strong complexing reagents.
shield the 4s-electrons from the increasing nuclear charge The d7 configuration The species with this
somewhat more effectively than outer-shell electrons can configuration is cobalt(II) which is stable in aqueous
shield one another, hence the atomic radii decrease less solutions but gets oxidised to form Co(III) complexes in the
rapidly. For the same reason, electronegativities and presence of strong ligands.
ionisation energies increase only slightly from Sc across to The d8 configuration Nickel(II) is the most
Cu compared with the increase from Na to Ar.
important species with this configuation.
Chapter 16: The Transition and Inner Transition Elements | 653

The d9 configuration The configuration is found oxides because of most electronegative nature of F
inCu2T compounds and is by far the most important in the and O. The different oxidation states of 3d-series have
chemistry of copper. Otherwise, this configuration is of been given below (very rare oxidation states are provided
little importance. in parenthesis).
The d10 configuration The two species Cu+ and Sc Ti V Cr Mn Fe Co Ni Cu Zn
Zn2+ are important with this configuration. Whereas,
I I
copperU) is easily oxidised to copper(II), zinc(II) is the only
(II) (II) II II II II II II II II
state known for zinc.
III III III III (III) III III (III) (III)
IV IV IV IV (IV) (IV) (IV)
Metallic Character V (V) (V) (V) (V)
Nearly all the transition metals have the simple hep, VI VI VI
cep or bcc lattice characteristics of true metals. They VII
have relatively high densities. They are malleable, ductile
and have high tensile strength, thermal and electrical • The lesser number of oxidation states in the beginning
conductivity and lustre. Melting points of 3d-series of the series can be due to the presence of too few
(shown in Fig. 16.1) rise to a maximum and fall as the electrons to loose or share, towards the end of the
atomic number increases except in Mn where melting series it can be ascribed to the presence of too many
point is minimum. High melting point and high heat electrons and thus fewer empty orbitals to share
of atomisation indicate that the atoms in these electrons with the ligands.
elements are held together by strong metallic bonds • Except Sc, the most common oxidation state of the
that are present in the molten states as well. first row transition elements is II which arises from
Greater the number of valence shell, stronger is the the loss of two 4s-electron. This means that after Sc,
resultant bonding. d-orbitals become more stable than the s-orbital.
• In the II and HI oxidation states, bonds formed are
m.p.
mostly ionic. In compounds of the higher oxidation
states, the bonds are formed by the removal or sharing
of d-electron hence, the bonds formed are essentially
covalent (as in MnOj).
• Within a group, the maximum oxidation state
increases with atomic numbers.
Fe II, III
Ru, Os II, III, IV, VI, VIII
T 1 i i r
• Transition metals also form compounds in low
Z-*
oxidation states (1,0). Such compounds are expected to
Fig. 16.1 Melting point of 3d-series elements
be unstable except in cases where vacant d-orbitals
(not according to scale)
are used for accepting lone-pairs from 7t-bonding
The periodic trends in the boiling points are similar to ligands e.g., Ni(CO)4,[Ag(CN)2r,[Ag(NH3)2r.
those in the melting points. As the process of boiling
requries almost complete breaking of bonds and such DEEP Focus
metallic bonding exists in the liquid state to some extent, • When an element exists in more than one oxidation
high temperatures are necessary. Therefore, the boiling state, standard electrode potential data provide a
points of metals are much higher than their melting clue to the relative stabilities of different states w.r.t.
points. oxidation. Cr2+ is unstable in water w.r.t. oxidation
whereas Cr3+ is stable in water; Fe2+ is unstable in
Oxidation States aerated water (and changes to Fe3+) w.r.t. oxidation.
A characteristic property of the d-transition metals is • Another feature is the reduced tendency of higher
their ability to exhibit several oxidation states differing oxidation states towards the end of the series. This
usually by units of one. This is due to the fact that could be due to steady increase in the effective nuclear
(n-l)d electrons may get involved along with charge along the series thus pulling the d-orbitals into
ns-electrons in bonding, as electrons in (n - l)d the electron core and not making them readily
available for bonding. Thus, only oxidation of zinc is
orbitals are in an energy state comparable to
Zn(II) where no d-orbital is involved. Also, it is
ns-electrons. The stability of a given oxidation state
difficult to form species that do not utilise the
depends on the nature of the element with which the
d-electrons, i.e., Sc(II) is virtually unknown and Ti
transition metal is combined. The highest oxidation
(IV) is more stable than Ti (II).
states are found in the compounds of fluorides and
654 | Essential Inorganic Chemistry

Table 16.1 Selected Properties of Elements of the First Transition Series


Properties Sc (21) Ti (22) V (23) Cr (24) Mn (25) Fe (26) Co (27) Ni(28) Cu (29) Zn (30)

Electron configuration [Ar] 3d14s2 3d2 4s2 3d3 As2 3d5 4s1 3d5As2 3d64s2 3d7As2 3d6As2 3d’°4s1 3d10 4s2
Radius/pm 161 145 132 127 124 124 125 125 128 133
M+ 90 88 85 80 77 75 69 72
M2+ 81 77 74 64 66 60 64 74
Covalent radius/pm 144 132 122 117 117 117 116 115 117 125
(/F)i 631 656 650 652 717 762 758 736 775 906
(/C)2 1235 1309 1413 1591 1509 1561 1645 1751 1958 1733
(/E)3 2389 2650 2828 2986 3250 2956 3231 3393 3578 3833
Abundance ppm by weight 25 6320 136 122 1060 60000 29 99 68 76
Melting point/0 C 1397 1672 1710 1900 1244 1530 1495 1455 1083 420
5° 2+ /I/ -2.08 -1.63 -1.18 -0.91 -1.19 -0.44 -0.28 -0.23 -0.34 -076
m2+/m
(Sc3+ + 3e“ -> Sc)
Density g/cm3 2.99 4.49 5.96 7.20 7.20 7.86 8.90 8.91 8.92 7.14
Electrical conductivity [w.r.t. Ag 2 3 10 2 17 24 24 97 100
which is 100 (say)]
Common oxidation +3 +4 +3 +3 +2 +2 +2 +2 +1 +2
state

+4 +6 +4 +3 +3 +2
+5 +7
Sublimation energy/kj mol 305 429 459 349 220 351 382 372 305 115
Electronegativity (A/R) 1.20 1.30 1.45 1.55 1.60 1.65 1.70 1.75 1.75 1.65

Table 16.2 Reactivity of Some Oxidation States of Even though the ground state of the atom has a
3d-series Elements in Aqueous Solution d10-configuration, Pd and the coinage metals Cu, Ag and Au
behave as typical transition elements. This is because in their
Reducing agent Most stable Oxidising agents most common oxidation states, Cu (II) has a
Sc3+ d9-configuration, and Pd (II) and Au (III) have
Ti2+, Ti3+ Ti3 + d8 -configuration (incomplete d level). However, Zn (II), Cd (DO
and Hg (II) have a d10 configuration. Because of this, these
V2*, V3+ V4 + V5*
elements do not show the properties characteristic of transition
Cr2+ Cr3+ Cr6+(CrO2‘) elements.
Mn2 + Mn3\ Mn4+(MnO2)
Mn7+(MnO;)
Ionisation Energies
Fe2+ Fe3+
• The magnitude of ionisation energies provide ar
Co2+ Co3+
indication of the energy needed to raise the metal to a
Ni2+ particular oxidation state in a compound.
Cu+ ■ Cu2+
• (IE) of 5d-elements are higher than those of the
3d and 4<Z-elements. This is due to greater
DEEP Focus effective nuclear charge acting on outer valence
• The electronic structure of the atoms in the second and electrons because of the weak shielding of the
third rows do not always follow the pattern of the first row. nucleus by 4/-electrons. The ionisation energies of
The structures of the nickel group are : the 3d and 4</-elements are irregular.
Ni 3d8 4s2
Pd 4d105s° DEEP Focus
Pt 5d9 6s1 • It is important to note that though the 4s-orbitals are
Since a full shell of electrons is a stable arrangement, the occupied before 3d-orbitals, we cannot say that they are­
place where this occurs is of importance. always more stable. In fact, the ionisation of transition
Thed-levels are complete in : elements takes place by the loss of ns-electrons first What
Ni Cu 3d10 4s1 Zn 3d10 4s2 happens actually is that when the electron is ionised from
Pd 4d10 5s0 Ag 4d10 5s1 Cd 4d105s2 any transition element, say from iron (Z = 26) of3d-series.
Pt Au 5dlQ6sl Hg 5d 10 6s2 the effective nuclear charge experienced by the3d-electrons

I
Chapter 16: The Transition and Inner Transition Elements | 655

is greatly enhanced over that of any 4s-electron as a direct Electrode Potential


consequence of the greater stability attained by the
3d-orbitals. Thus, 3d-orbitals are expected to drop in energy When a metal is placed in a solution of its ions a
below 4s-orbitals. Thus, ionisation of two or more electrons potential difference is set up between the metal and the
from an atom of a transition element will take place with solution. There is a tendency for the metal ions to leave the
the removal of s-electrons in preference to the d-electrons. metal lattice and go into the solution thus leaving an
excess of electrons and hence a negative charge on the
* Common oxidation states for each element include +2 metal; there is also a reverse tendency for the metal ions
and +3 or both. The +3 oxidation states are more from the solution to deposit on the metal leading to a
stable at the beginning of the series, whereas towards positive charge on the metal. In practice one of these
effects is greater than the other, bringing about a potential
the end of +2 oxidation states are more stable. This
difference between the metal and the solution. The value
can be explained by study of variation of (IE) plots. IE of this potential difference for a particular metal depends
increases gradually from left to right. However, the upon the nature of metal, the concentration of the metal
third (IE), when an electron is removed from ions in solution an the temperature. By convention, the
3d-orbital, increases more rapidly than the second and potential difference set up in a 1 M solution of metal
third (IE). Because it takes more energy to remove the ions at 298 K is called the standard electrode
potential, (measured w.r.t. to reference half-cell)
third electron from the metals near the end of the row
In general, we can say that more negative the value of
than from those near the beginning, the metals near the electrode potential for the couple Mn*IM, more is the
the end tend to form M2+ ions rather than M3+ ions. reducing power of the element. Similarly, more positive is
(Fig. 16.2) the value of electrode potential for the couple Mn+IM,
more is the oxidising power of the element. The values of
| 4000- some standard electrode potentials for the elements of
Third first transition series are given in Table 16.1. Electrode
0 potential is a measure of the electropositive character and
JC
3000- the reactivity of the metals. In general along a period,
there is a decrease in electropositive character. The
E5 reactivity of metals also decreases along a period and
2 2000- Second down a group. All the elements of the first transition
c
o series, except copper, have negative values and can react
S 1000- with acids (H+) producing hydrogen. A plot of variation of
First
co ♦ -•------ the electrode potential of the transition elements of

3d-series is shwon in Fig. 16.3.
0 L T T t-
0.5-1
Sc 71 V Cr Mn Fe Co Ni Cu
Element—►
Fig. 16.2 Variation of the first, second, and third 0-
ionisation energies for the first transition metals. M3*+3e~^ M
-0.5-

DEEP Focus
• It is possible to determine thermodynamic stability of the LU
M2*+2e~ M
transition metal compounds based on the values of (IE) of
the metals. (IE)V (IE)2, (IE)3 and (ZE)4 of Ni and Pt in mJ
mol"1 are compared.
(LE)l+(IE)2 (IE\ + (1E)4 Total
Ni 2.49 8.80 11.29 I i i I | | iiii
Pt 2.66 6.70 9.36 20 21 22 23 24 25 26 27 28 29 30
Atomic number
Thus, nickel (II) compounds tend to be thermodynamically
more stable than platinum (ID while platinum (TV) Fig. 16.3 Trends in electrode potentials of transition metals of
compounds are relatively more stable than nickel (IV). 3d-series
IV
K2 PtCl6 is well known but such type of nickel compound is A qualitative treatment of the stability of transition
metal ion in different oxidation states in aqueous medium
not formed. Following factors also decide the stability of the
compounds: is obtained from the electrode potential data. The value of
~ the energy of the sublimation, E° for M2*IM denotes the emf of the cell in which the
“ the lattice energy, reaction
~~ the solvation energy. 2H+(aq) + M(s) M2+(aq) +H2(g) + 2e~
656 | Essential Inorganic Chemistry

occurs under reversible conditions (values given in added to an inner shell which provides relatively goo«
Table 16.1). The total energy AH of process shielding to the outer electrons from the nucleus. Th
M(s) ---- > M2*(aq)+ 2e~ increasing electronegativity from Sc to Cu means that th
elements become slightly less metallic and this is reflect#
can be represented as equal to sum of the energy of the
following steps : in the increasing positive electrode potentials of their ion
M2+and (Table 16.1)
M(s) ----> M(g); AHsub (enthalpy of sublimation)
M(g)---- > M2+(g) + 2e-; (IE). + (IE)2 Magnetic Properties
M2*(g)---- >M^(qg);AHhyd (enthalpy of hydration) When you place an iron piece near a magnet, you wil
M(s) ---- » M2+(aq) +2e~ see that it is immediately drawn towards the magnet
However, some elements are repelled by the magnets. Th
+ (!£), + (ZE)2 + AHhyd.
property of an element to be attracted or repelled by a
magnet differs from element to element. Substance
DEEP Focus which are weakly repelled by the magnetic field are called
• Although the transition metals are less electropositive (or
more electronegative) than the alkali and alkaline-earth diamagnetic (with no unpaired electrons) while the
metals, their standard reduction potentials suggest that all substances which are weakly attracted by the magnetic
of them except copper should react with strong acids such as field and lose their magnetism when removed from the
dil. HC1 to produce H2 gas. However, most transition metals field are called paramagnetic. If the force of attraction is
are inert towards acids or react slowly with them because of
a protective layer of oxide. In case of Cr, despite a negative very large and the permanent magnetisation is retained,
standard reduction potential, it is quite inert chemically the substance is said to be ferromagnetic, e.g., iron and
because of the formation of chromium (III) oxide, Cr2O3 on some iron compounds.
its surfaces. This fact find application of Cr to use as a
protective and noncorrosive plating on other metals. The magnetic properties of any individual atom or ioa
will result from the combination of the two properties,
Complexes I. inherent spin moment of the electron and
The transition elements have an unparalleled II. the orbital moment resulting from the motion of
tendency to form coordination compounds with Lewis the electron around the nucleus.
bases that is with groups which are able to donate an The magnetic moment is usually expressed in unit
electron pair (called ligands). A ligand may be a neutral called Bohr magnetons (BM). The general eequation for
molecule such as NH3, or an ion such as Cl" or CN“.
the magnetic moment is given by :
Co3++6NH3 ---- > [Co(NH3)6]3+
g(S „1) = ;4S(S + 1) + L(L + 1)
Fe2++ 6CN" ---- > [Fe(CN)6f-
Cr3+ + GH^O---- > [Cr(H2O)6]3+ In above expression, S is the sum of the spin quantum
numbers and L is the sum of orbital angular momentum
This ability to form complexes is in marked contrast to quantum numbers of all the electrons. In many
the s— and p-block elements which form only a few compounds including those of the first row transition
complexes. Tendency to form complexes by transition elements, the orbital contribution is compensated by the
metal is due to :
electric fields of the surrounding atoms and as an
- their smaller size, approximation, the observed magnetic moment may be
- higher nuclear charge, considered to arise only from unpaired spins. Putting
- presence of low energy vacant orbitals to accept L = 0 in the above expression, you can get the spin-only
lone pair of electrons donated by ligands. magnetic moment ps.
Complexes where the metal is in the (III) oxidation Thus, ps = y]4S(S +1)
states are generally more stable than those where the
The spin-only magnetic moment, jis can also be
metal is in the (II) state. Thus, [Fein(CN)6 ]3- is more
related to the number of unpaired electrons, N, in anj
stable than [Fen(CN)6 J4- (Detailed study of the complexes
is given in Chapter 18 of the Text. N
species, as the total spin quantum number S = —

Electronegativity Hence, |is = 74S(S + 1)


Transition elements have fairly low values of = l4N{N
electronegativity. It increases from Sc to Cu with a fall at —+ 1
2 2
Mn and Zn. However, this increase in electronegativity is
much slower because the additional electron is being jis=7M2V + 2)
Chapter 16: The Transition and Inner Transition Elements | 657

Above expression gives the values of magnetic


moment in Bohr magnetons which can be converted into
SI unit of Ampere square meter (Am2) by the following 4
CM
relationship: +
1 BM= 9.274 x IO"24 Am2 2

Magnetic moment increases as number of unpaired


electrons increases. For 3d-series elements, variation of
magnetic moment is given in Fig. 16.4. This clearly shows
magnetic moment is maximum for chromium (with six i------- 1------- 1 i------- 1------- 1------- 1------- 1------- r-
0 21 22 23 24 25 26 27 28 29 30
unpaired electrons). Z—
Fig. 16.4 Magnetic moment of 3d-series elements (on
Sc Ti V Cr Mn Fe Co Ni Cu Zn arbitrary scale)
Z 21 22 23 24 25 26 27 28 29 30 In some cases (e.g., Mn2+, or Fe3+, in which all the
2 1 0 d-orbitals are occupied singly by electrons for which
ml = 2,1,0, -1 and -2, giving L - 0) the observed magnetic
P '8 '3 0
moment values agree very well with the spin-only value as
given in Table 16.3. But generally, experimental values
differ from the spin-only values. This is because the orbital
motion of the electron also makes some contribution to the
moment.

Table 16.3 Predicted and Observed Magnetic Moment Values of Some Transition Metal Hydrated Ions
Magnetic moment (BM)
Ion Electronic configuration Unpaired electrons
[Calculated] [Experimental (ps)]

[Ti(H2O)6]3+ 3d1 T 1 1.73 1.75


[V(H2O)6]3+ 3d2 T T . 2 2.84 2.75
IV(h2O)6]2+ 3d3TTT 3 3.87 3.86
[Cr(H2O)6]2+ 3d4 T T ? T 4 4.90 4.80
[Mn(H2O)6]2+ 3d5 T T ? ? T 5 5.92 5.96
[Fe(H2O)6]2* 3d6 UTTT1 6 4.90 5.00
[Co(H2O)6]2+ 3d7 UTiTTT 7 3.87 4.40
[Ni(H2O)6]2+ 3d8 nnntr 8 2.84 2.90
[Cu(H2O)6]2+ 3d9 TinUUT 9 1.73 1.80

Colour d-level to another d* level of higher energy (called d-d*


Many ionic and covalent compounds of transition transition). This corresponds to a fairly small energy
elements (and also inner transition elements) are difference, and so light is absorbed in the visible region.
coloured. In contrast, compounds of the s- and p- block The colour of a transition metal complex is
elements are almost always white. Colour may arise from dependent on how big the energy difference is
an entirely different cause in ions with incomplete d or between the two d-levels. This in turn depends on
/•shells. In a free isolated gaseous ions, the five d-orbitals the nature of the ligand, and the type of the complex
are degenerate, that is, they are identical in energy. The formed. The colour which we see is the colour of the
surrounding groups, which can be solvent molecules in transmitted wavelength and is the complimentary
solution or ligands in a complex or other ions in a crystal colour absorbed by the compounds
lattice, affect the energy of some d-orbitals more than [Ni(NH3)6f‘ blue
others. Thus, the d-orbitals no longer degenerate and at
[NKHjO),?* green
their simplest they form two groups of orbitals of
different energy. Thus in transition element with a partly [NKNCVsf- brown-red
filled d- shell, it is possible to promote electrons from one Energy 2 d 2
x - y,2
658 | Essential Inorganic Chemistry

Size of Atoms and Ions


ligand field • The covalent radii of the elements (summarised in
splitting Table 16.5) decrease from left to right across a row in
set of five d-orbitals the transition series, until near the end when the size
increases slightly. On passing from left to right, extra
without ligands d d d protons are placed in the nucleus and extra orbital
xy yz xz electrons are added. The orbital electrons shield the
two sets of d-orbitals with nucleus charge incompletely (order being d<p<s}-
different energies in an Thus the nuclear charge attracts all the electrons more
octahedral complex. strongly, hence, a contraction in size occurs.
• ZnSO4 (Zn24 with d10 configuration) and TiO2 (Ti4+
with d° -configuration) are white—in both of them DEEP Focus
d-d* spectra are impossible and they are colourless. • The radii of the elements from chromium to copper,
however, are very close to one another. This is due to
the fact that successive addition of d-electrons screen
DEEP Focus the outer electrons (4s) from the inward pull of the
nucleus. As a result of this, the size of the atom does
• In the series Sc (+III), Ti (+IV), V (+V) Cr (+VI) and not alter much in moving from chromium to copper.
Mn (+VII), all have empty d-orbitals, hence d-d* spectra are
impossible and they should be colourless. But following ions • The relative constancy in covalent radii for the later
in aqueous solution are: elements is partly responsible for the similarity in
VO2’ (oxidation state +V) — pale yellow properties of the Groups 8, 9 and 10 (VIIIB) elements
(which are Fe, Co, and Ni).
CrO2- (oxidation state +VI) — deep yellow
• Atoms of the transition elements are smaller than those of
Mn07•1 (oxidation state +VII) — intense purple
the Group I or II elements (s-block) in the same horizontal
The colours arise by charge transfer, for example, in
period. This is partly because of the usual contraction in
MnO^ an electron is momentarily transferred from 0 to the
metal and thus oxygen changes from O2- to O" and manganese size across a horizontal period and partly because the
from Mn(+VII) to Mn(+VI). (Also refer Chapter 18 of this Text). orbital electrons are added to the penultimate d-shell
Table 16.4 gives the colour and oxidation states of the metal rather than to the outer shell of the atom.
ions present in some hydrated ions of transition elements. However, it is important to emphasise here that
shielding of the outer ns-electron(s) by (n -1) d-electron(s)
is more efficient than the shielding of a ns-electron by
Table 16.4 Oxidation States and Observed Colours for another ns electron (or that of a np-electon by another
Some Aqua Species np-electron). This is why the decrease in atomic
radius from sodium to chlorine is greater than that
Element +2 +3 +6 +7
from scandium to copper. The elements which occur
Sc Colourless immediately after the transition elements are smaller
[Sc(H2O)6]3t than expected from simple.
The rate of decrease in size along the lanthanide
Ti Violet
series is even less than that in the transition series since
[Ti(H2O)6]3+
in the lanthanides the electrons are added to the
V Violet Green penultimate [(n - 2)f] shell and these shield the outer
[V(H2O)J!- [V(H2O)6]3+ electrons much more effectively. The presence of
Cr Blue Violet/green Orange yellow
4/-electrons in the lanthanides affects the atomic size and
[Cr(H2O)6]2* [Cr(H2O)6]3+ Cr2O3’,CrO2“ therefore, the chemistry of the elements following the
lanthanides. The atomic radii of the elements of third
Mn Pink Red Green Purple transition series are much smaller than expected. This is
[Mn(H2O)6)2+ [Mn(H2O)6]3* MnO2- MnO4 due to the effect of the greater than expected effective
Fe Pale green Yellow/ brown nuclear charge felt by the electrons of the elements of the
[Fe(H2O)6]2* [Fe(H2O)6]3+ third row transition series, hafnium to gold, owing to the
insertion of lanthanides.
Co Pink Blue
[Co(H2O)6]2+ [Co(H2O)6]3+ Metallic radii (pm) of some elements of Group 1-13
Ni Green 1 2 3 4 5 6 7 8 9 10 11 12 J3_
[Ni(H2O)6]2+
K ~Ca~ ~Sc~
Ti V Cr Mn Fe C? ~nF Cu. Zn Ga]
Cu Blue 235 197 164 147 135 130 135 126 125 125 128 137 1411
[Cu(H2O)6]2+ Rb Sr Y Zr Nb Mo Tc Ru Rh Pd Aq Cd In j
Zn Colourless 248 215 178 160 146 139 136 134 134 137 144 154 I6oi
[Zn(H2O)6]2+ Cs Ba La Hf Ta W Re Os Ir Pt Au Hq n i
267 222 188 160 149 141 137 135 136 139 146 157 171;
Chapter 16 : The Transition and Inner Transition Elements | 659

Data in Table 16.1 and Fig. 16.5 show that the general Atomic volume of an element is directly related to its
trend of decreasing size is reversed towards the end of the size and therefore, atomic volumes follow the same trend
series. This could be due to an increase in inter-electronic as the atomic size. Similarly density is also related to the
repulsion after the addition of sufficient number of size of the element. The smaller the size, the higher the
electrons in the d-orbitals leading to the gradual increase density of the element as shown in Fig. 16.6. For 4d and 5d
in size. elements, this increase is not as regular as the increase in
densities for 3d-elements.
Table 16.5 Covalent Radii of the Transition Elements (pm) Os
K Ca Sc Ti V Cr Mn Fe Co Ni Cu Zn
20F
203 174 144 132 122 117 117 117 116 115 117 125
Zr Nb period 6
145 134____________ ____________ E 15-
Hf Ta u> ^H9
X
144 134_____________ ___________ r> Rh
1.10-
There is a gradual decrease in size of the 14 2*
wc period 5Co o Cd
lanthanide elements from cerium (58) to lutetium (71). La
0)
This is called Lanthanide Contraction. The □ 5b © Zn
lanthanide contraction cancels almost exactly the period 4
normal size increase on descending a group of
transition elements. Thus covalent and ionic radii of Rbsr^L Sc
K
Nb are the same as the values for Ta. Same is the
case for Zr and Hf (also refer Table 16.6 and Fig. 16.5). 1 2 3 4 5 6 7 8 9 10 11 12
Group number
Table 16.6 Effect of Lanthanide Contraction on Ionic Radii (pm) Fig. 16.6 Trend in densities of the alkali, alkaline earth and
transition metals of the fourth, fifth and sixth periods.
Ca2+ 100 Sc3+ 74.5 Ti4 + 60.5 V3+ 64.0
Sr2+ 118 Y3+ 90.0 Zr4 + 72.0 Nb3+ 72.0
Ba2+ 135 La3+ 103.2 Hf4+ 71.0 Ta3* 72.0 Non-stoichiometry
The effects of lanthanide contraction are less Transition elements have the property of forming
pronounced towards the right of the d-block. compounds of indefinite structures and proportions. These
are called Non-stoichiometric compounds. Iron (H)
170 L oxide FeO is written as FeO to indicate that ratio of Fe
and O atoms is not exactly 1:1. Analysis shows that the
160 formula varies between Fe094O and Fe0g4O. Vanadium
and selenium form a series of compounds ranging from
t
E 150 -
VSe0 98 to VSe2 :
a VS^ (VSe09g —» VS12)
■6
E 140 F V2Se3 (VSe12 > VSe16)
♦rf

<0
V2Se4 (VSe16 ---- > VSe^
> Period 6
0 ,
O 130 b CuS (insoluble) is actually a mixture of CuS and CujS.
Period 5 Non-stoichiometry is shown particularly among transition
metal compounds of the Group VI elements (O, S, Se, Te)
120
Period 4 and is mostly due to variable valency of transition
elements, and also sometimes due to defects in solid
Period 4: Sc Ti V Cr Mn Fe Co Ni Cu Zn structures.
Periods: Y Zr Nb Mo Tc Ru Rh Pd Ag Cd
Period6: La Hf Ta W Re Os Ir Pt Au Hg
Elements—►
Catalytic Properties
Fig. 16.5 Comparison of the covalent radii for transition elements Many transition metals and their compounds have
of different periods. Note that elements in the same column of the catalytic properties.
fifth and sixth periods have approximately the same radii.
660 | Essential Inorganic Chemistry

DEEP Focus Alloy Formation


These metals can function as catalysts because they can An alloy is a blend of metals prepared by mixing the
utilise both d and s-electrons for the formation of bonds
between reactant molecules and the surface catalyst atoms. components. Alloys (refer Appendix VI) may be
This increases the concentration of the reactants at the catalyst homogeneous solid solutions in which the atoms of one
surface and weakens the bonds in the reactant molecules with metal are distributed randomly among the atoms of the
the result that the activation energy is lowered. Compounds of other. Such alloys are formed by atoms with metallic radii
transition metals are able to act as catalysts because of the ease that are within about 15 per cent of each other. Because of
with which the metal can adopt different oxidation states and
also because of their ability to form complexes. similar radii and other characteristics of transition
metals, alloys are readily formed by these metals. The
Some of the common catalysts used for important alloys so formed are hard and have often high melting
reactions are summarised in Table 16.7. points. The best known are ferrous alloys; chromium,
Table 16.7 Transition Metals and their Compounds as vanadium, tungsten, molybdenum, and manganese are
Catalyst used for the production of variety of steels and stainless
steel. Alloys of transition metals with non-transition
Catalyst Function
metals as brass (copper-zinc), and bronze (copper-tin) are
TiCI4 Used as Ziegler Natta catalyst in the production of also of considerable industrial importance.
polythene
FeSO4 + H2O2 Used as Fenton's reagent in the oxidation of
alcohols to aldehydes. Interstitial Compounds
Ni Raney nickel in reduction processes :
• Transition metals can trap some small atoms like
—manufacture of hexamethylenediamine,
—production of H2 from NH3,
hydrogen, boron carbon, nitrogen, etc., in vacant
—reducing anthraquinone to anthraquinol in the spaces in their crystal lattice forming interstitial
production of H2O2 compounds. Carbon and nitrogen always occupy
Ni complexes Reppe synthesis (polymerisation of alkynes to give octahedral holes; hydrogen is smaller and always
benzene, cyclooctatetraene). occupies tetrahedral holes. As only transition metals
Cu Used in manufacturing of (CH3)2SiCI2 being used form such compounds, the d-electrons are, therefore,
to make silicones. presumably involved in the bonding. The structure of
Cu/V Used in the oxidation of cyclohexanol/
cyclohexanone mixture to acetic acid which is
the metal often changes during the formation of such
used to prepare Nylon-66. compounds. The composition of these compounds is
CuCI2 Deacon process of making Cl2 from HCI. generally non-stoichiometric, e.g., TiHx 73, PdH056,
MnO2 Used in the decomposition of KCIO3 to KCI andO2. VH0 56, but may approach regular stoichiometry and a
v2o5 Used in contact process of H2SO4.
regular structure, e.g., TiC and VN. The later
Fe/Mo
transition elements of the first series form
Promoted Fe used in the Haber-Bosch process of
non-stoichiometric carbides with irregular structures,
NH3
such as Cr7C3, which are more reactive than the
FeCI3 Used in the production of CCI4 from CS2 and Cl2
interstitial carbides of the early transition elements.
PdCI2 Wacker process for converting C2H4 into
CH3CHO.
These interstitial compounds are of much importance,
Adams catalyst in reduction.
e.g., carbon steels are interstitial iron-carbon
PVPtO
Pt In three stage convertors for cleaning car exhaust
compounds in which the interstitial carbon prevents
fumes. the iron atoms from sliding over one another, making
Pt/Rh In Ostwald process for HNO3 from NH3 iron harder, stronger but more brittle.
ZnO/CuO In synthesis of CH3OH from CO and H2
Co/Fe In synthesis of hydrocarbons by CO and H2 in
Fischer-Tropsch process. Nature of Oxides/Hydroxides
Pt/PtO Adians Catalyst for reduction While moving along a period from Sc to Zn basic
nature decreases and acidic nature increases. Behaviour
Enzymes are catalysts that enhance the rates of differs when oxidation states are different
specific reactions. Some enzymes require the presence of
metal ions as cofactors called as metalloenzymes which Sc2O3 TiO2 V2O5 Cr2O3 MnO2 Fe2O3 CoA NiO |CuO| ZnO
generally contain transition metal, e.g., Mo in xanthine A A A A A B B B B A
oxidase (metabolism of purines).
A cobalt atom lies at the centre of the vitamin B12 A : Amphoteric, B : Basic
coenzyme. Iron atoms are importantly involved in With increase in the oxidation state of a given
haemoglobin of blood and in the ferredoxins of transition metal, the covalent character of its compound
photosynthetic process. Fe and Mo are present in nitrogen increases and thus acidic character also increases.
fixing enzyme.
Chapter 16 . The Transition and Inner Transition Elements | 661

MnO Mn3O4 Mn2O3 Mn02 Mn03 Mn20'7 CrO3 + 2K0H ■» K2CrO4 +


Qxdation state of Mn +2 +8/3 +3 +4 +6 +7 2CrO3 + H2O > H2Cr>07
future of oxide Basic Weakly Strongly
(dichromic acid)
Amphoteric
acidic acidic 2CrO3 + 2K0H---- >
Characteristics of oxides and ions of vanadium and potassium dichromate
chromium have been summarised in Table 16.8. Mn2O7 + H2O---- > 2HMnO4
The basic and amphoteric oxides dissolve in (permanganic acid)
non-oxidising acids and form hexaaquo ions [M(T^Ole ]n+. Mn2O7 + 2K0H---- > 2KMnO4 + HjO
The acidic oxides like CrO3 and Mn2O3 dissolve in water to
form the oxyacids These oxides also react with bases to As we move along a given transition series, +2
form oxosalts. oxidation state of the metals becomes more and more
stable and hence the tendency of these metals to lose
CrO3 + HgO H2CrO4 electrons increases hence, reducing nature of the metal
(chromic acid) oxide increases. VO and CrO are strong reducing agents.
Table 16.8 Chracteristics of Oxides and Some Ions of V and Cr
o.s. Oxide/ Hydroxide Behaviour Ion Name of ion Colour of ion
+2 VO basic V2* vanadium (II) (vanadous) violet
+3 VZO5 basic v3+ vanadium (III) (vanadic) green
+4 vo2 amphoteric VO2* oxovanadium (IV) (vanadyl) blue
hypovanadate (vanadite)
v4or brown
+5 v2o5 amphoteric vo^ dioxovanadium (V) orthovanadate yellow
VO^- colourless
+2 CrO ’ basic Cr2* chromium (II) (chromous) light blue
Cr(OH)2
+3 Cr2O3 amphoteric Cr3* chromium (III) (chromic) violet
Cr(OH)3 chromite
Cr(OH);
green
+6 CrO3 acidic CrO2- chromate yellow
CrO2(OH)2 dichromate orange
Cr2O2
H2Cr2O7

Table 16.9 Fourth Period Transition Elements Essential to Human Nutrition


Element Some biochemical substances Functions
Chromium Glucose tolerance factor Utilisation of glucose
Cobalt Cobalamin (vitamin B12) Development of red blood cells
Copper Ceruloplasmin Synthesis of haemoglobin
Cytochrome oxidase Cell energetics
Iron Haemoglobin and myoglobin Transport and storage of oxygen
Cytochrome C Cell energetics
Catalase Decomposition of hydrogen peroxide
Manganese Isocitrate dehydrogenase ' Cell energetics
Zinc Carbonic anhydrase Elimination of CO2
Carboxypeptidase A (Pancreatic juice) Protein digestion
Alcohol dehydrogenase Oxidation of ethanol
662 | Essential Inorganic Chemistry

Target Practice 1
1. What are the types of the following reactions? (b)
(a) 3CrO74 + 8H+ ----- > 2CrO42- + Cr3+ + 4H„O
Z
(0
12. State whether the following complexes are coloured or not
(b) 3MnO2" + 4H+ 2MnO4 + MnO2 + 2H2O (a) [Sc(H2O)6]3+ Yes/No
(b) [Cu(CN)4]3- Yes/No
2. Write EC of (c) [Cu(H2O)4]2+ Yes/No
(a) Cu(II) ...................
(d) [Zn(H2O)4]2+ Yes/No
(b) Cr(0)
13. Consider following complexes
(c) Co(III)
I: [Fe(H2O)6]2+ II: [Fe(CN)6]4'
3. Name the element with maximum oxidation state in
d-block elements III: [Ni(CO)4] IV: [Ni(H2O)4]2+
V: [Ni(CN)4]z“
4. (a) Cr2+is unstable in.... Out of these select the complex(es) with.
(b) Fe2+ is oxidised toFe'.3+ in (a) Maximum number of unpaired electrons
5. Why a transition series contains ten elements? (b) Maximum number of magnetic moment
(c) Equal number of unpaired electrons
(d) Equal magnetic moment
6. (JE\ of 5d-elements are higher than those of the 3d and 4d
(e) Square planar geometry
elements. It is due to (f) Octahedral geometry
7. Stepwise (ZE)5 of Ni and Pt are given below (g) Tetrahedral geometry
(IE\ + (IE)2 (IE)3 + (IE)4 14. Convert 3.00 BM into SI unit (Am2)
Ni 2.49 8.80
15. Calculate magnetic moment of V3+ (Z = 23)
Pt 2.66 6.70
BM
Compare thermodynamic stability of Ni and Pt salts in (II) 16. What is oxidation state of iron in [Fe(H„O),N0]2+ ?
and (IV) oxidation state z

Ni(II) | | Pt(II)
17. Give one example in which ion is coloured due to charge
Ni(IV) I I Pt(IV)
transfer.
8. E° values of reactions
Fe2+ +2e" ----- > Fe; E° = - 0.44 V
Cu2+ +2e’ ----- > Cu; E° = + 0.34 V 18. Give oxidation of the underlined and give the colour of the
TheE°ofFe + Cu2+ Fe2++Cu is .... .... and thus, ion
reaction is spontaneous/non-spontaneous. Oxidation state Coloured/coloufless
9. Select ions which can form complexes out of Fe>12+, Ni2+, VO2+
Ca2+, K+, Mg2+, Na+, Cu2+, Zn2+ CrO2’ ........

19. Give examples of coloured ions with following oxidation


10. Which complex is more stable in each pair?
state
(a) [Fe(CN)6]4’ | | [Fe(CN)6]3" (a) Cr(VI)
(b) [Ni(CN)4]2- |--- 1 [Pt(CN)4]2- (b) Cu(II)
(c) [NiCl6]2~ | | [PtCl6]2- (c) Mn(VI)
11. Mention three factors which favour formation of complexes (d) Ni(II)
by d-block elements (e) V(III)
(a) (f) Co(III)

Answers
1. (a) CrO;(V)isoxidisedtoCrO^(VI)andreducedtoCr3+(III) 4. (a) water (b) aerated water (CO2 mixed water)
(b) MnO2-(VI) is oxidised to MnO<(VII) and reduced to 5. In transition series, the electrons enter the d-orbitals which
MnO2(IV) can accommodate ten electrons and therefore, there are ten
Thus, (a) and (b) both disproportionation reactions. elements in one transition series.
2. (a) [Ar] 3d9 (b) [Ar] 3dB 4s1 (c) [Ar] 3d6 6. This is due to greater effective nuclear charge acting on outer
valence electrons because of the weak shielding of the nudeus
3. Os, Ru(VIII)
by 4f-electrons.
Chapter 16: The Transition and Inner Transition Elements | 663

7. Ni(II)>Pt(H) NO change to NO* by loss of electron and this electron is


Ni(IV) < Pt(IV) gained by Fe2 ’ which change to Fe *
NO ---- > NO* + e~
= -(-0.44)+ 0.34=0.78 V Fe2* + e' ---- > Fe*
Spontaneous. Thus, oxidation state of iron = + 1
9. Fe2*,Ni' Ca2 + ,Cu2+, Zn2+, Mg2*
Fe2+ (Ar) 4s°3d6 U ? ? ? T
(Ca2*andMg2 + form complexes with EDTA)
10. (a) [Fe(CN)6]4- < [Fe(CN)6]3-
(b) [Ni(CN)4]2‘ > [Pt(CN)4]2-
Fe+ (Ar) 4s°3d7 UH? T ?
(c) [NiCl6]2- < [PtCl6]2- Three unpaired electrons
11. (a) Smaller size,
+1 oxidation state is confirmed by magnetic moment of iron
(b) Higher nuclear charge,
= J15 = 3.87 BM and also by diamagnetic character of NO.
(c) Presence of low energy vacant orbitals to accept lone pair
(which is only possible when it has no unpaired electron)
of electrons donated by ligands.
12. (a) No (b) No (c) Yes (d) No 17. MnO;
Mn is in +7 oxidation state,
(Complex with unpaired electrons in d-orbital in the atom/ion
is coloured) Mn :[Ar] 4s23d5
13. (a) I (b) I (c) None (d) None Mn 2 *: [Ar] with no unpaired electrons; oxygen atom transfers
(e) V (01, II (g) HI, IV electrons to Mn (in its d-orbital) and thus MnO4 is coloured.
14. 3 x 9.274 x IO"24 A m2 Oxidation state
15. V(Z = 23) :[Ar] 4s23d3 18. vol; +5 coloured
V3*:[Ar]4s°3d2 CrO2- +6 coloured
Magnetic moment = ^N(N 4- 2) BM = V8 BM 19. (a) CrO2 (b) [Cu(NH3)4]2+ (c) MnO2’
16. This is a typical case of complex formed by electron exchange. (d) [Ni(H2O)4]I2* (e) [V(H2O)6J3‘ (f) [Co(H,O)6]3*

Practice Exercise 1
1. What is the oxidation state of the underlined atoms in 7. Which of the following compounds in each pair is more
(a) Ni(CO)4 (b) MnO2' stable?
(c) CiO2Cl2 (d) CrO5 (a) K2[PtCl6] K2[NiCl6]
(e) [Cr(CN)6]3- (f) [Fe(CN)5NO]2’ (b) [Ni(CN)4]2- [Pt(CN)4]2"
2. Give the electron configuration of the metal in each of the (c) PtCl4 NiCl4
following atoms or ions:
(a) Ni (b) Cr3+ (d) PtCl2 NiCl2
(c) FeO2- (ferrate ion) (d) MnO2 8. Why does chromium seem to be less reactive than its
3. Arrange the following hydroxy compounds in order of standard reduction potential suggests?
increasing acid strength, and account for the trend 9. Write a balanced net ionic equation for reaction of each of
(a) CrO2(OH)2 (b) Cr(OH)2 the following metals with HC1 in the absence of air. If no
(0 Cr(OH)3 reaction occurs, indicate N.R.
4. Explain howCr(OH)3 can act both as an acid and as a base. (a) Cr (b) Zn
5. E° = 0.34 V, E° 2+ =-0.76 V, E° = 0.80 V (c) Cu (d) Fe
Cu2*/Cu ’ Zn2+/Zn Ag+/Ag
Calculate E° values of following reactions : 10. [Fein(CN)6]3- ion has magnetic moment of 1.73 B.M.,
(a) 2Ag+ 4-Cu---- > 2Ag4-Cu2+ while [Fe111 (H2O)6]3+ has a magnetic moment of 5.92 B.M.
(b) Cu 4- Zn2+ ---- > Zn + Cu2+ Explain.
(c) 2Ag4-Zn2+ ---- > 2Ag+ 4-Zn 11. Which of the complexes use d-orbital of the metal/metal ion
Are they spontaneous. in hybridisation ?
6. Following arbitrary values are given for the conversion of (a) MnO4 (b) Ni(CO)4
Af(s)into M2+(aq) (d) [Fe(H2O)g]3+
(c) [Ni(CN)4]2-
Mts) ----- > Af(g); AH =1000 kJ mol
(e) [Fe(CN)6]3-
Af(g) ----- > Af(g)4- e~; AH = 950 kJ mol"1
12. What is the EAN (effective atomic number) of the
Af+fg) ----- > AH =1100 kJ mol
underlined atoms in the following complexes?
+aq---- > AH =-750 kJ mol’ 4- 3+
(a) [Fe(CN)6] (b) [Cr(en)3J ,
Calculate AH for (c) Ni(CO)4 (d) [Fe(H2O)6]2+
M (s) ---- > AT*(aq) + 2e“
664 | Essential Inorganic Chemistry

[Hint :EAN =Z(atomic no.) - oxid no. + 2 x (CN), CN is (a) Ni(CO)4 (b) [Fe(CN)6]4"
coordination number] (c) [Fe(H2O)6]3+ (d) [Fe(H2O)6]2+
13. Match the geometry (given in column A) with the 15. Explain the term ‘Lanthanide contraction.’
complexes (given in column B) in :
16. Which is larger in each pair ?
A B (a) Fe2* and Fe3* (b) Ni and Ni2*
(c) Nb3* and Ta3* (d) Y3* and La3*
(I) octahedral (a) [Ni(CN)4]2
(e) Cu and Zn (f) Cu and Ni
(II) square planar (b) [Fe(CN)6]
(g) I” and I*
(III) tetrahedral (c) Ni(CO)4 17. Due to ‘lanthanide contraction*, second and third row
transition elements (called 4d and 5d series respectively)
have similar radii. What are the other properties do you
14. Calculate magnetic moment [=^N(N +2) BM where
think would be similar?
N= number of unpaired electrons] of the underlined
atoms in

Answers
1. (a) 0 (b) + 6 (c) + 6 (d) + 6 (e) + 3 (0 + 2 11. (a) MnO; sp3
2. (a) [Ar]3d8 4s2 (b)[Ar]3d3 (c) [Ar]3d2 (d) [Ar] 3d3 (b) Ni(CO)4 sp3
3. Cr(OH)2 <CrfOH)3 <CrO2(OH)2 (c) (NilCN),]2- dsp2
Acid strength increases with polarity of the bond which (d) [Fe(H2O)6]3+ sp3d2
increases with increase in oxidation state. (e) [Fe(CN)6]3" dj2 sp 3
4. Cr(OH)3 + 3HC1---- > CrCl3 + 3H2O hence (c), (d), (e) use d-orbital in bonding.
base
12. (a) 36 (b) 33 (c)36 (d) 36
Cr(OH)3 + NaOH ---- > Na[Cr(OH)4]
acid 13. A B
I (b)
5. (i) E° = 0.46 V spontaneous
II (a)
(ii) E° = - 1.10 V non-spontaneous
(iii) E° = - 1.56 V non-spontaneous
in (0

6. AH = 2300 kJ mol"1 (Add all the reactions to get the desired Unpaired Magnetic moment
reaction) 14. Complex
electrons (N) = 7W+ 2) BM
7. (a)K2PtCl6 (h)[Ni(CN) 4]2"- (c) PtCl.4 (d) NiCl2
(h)[Ni(CN)J
(a) 0 0
8. because of formation of oxide layer
(b) 0 0
9. (a) 2Cr+ 6H* ---- > Cr3* +3H2
(b) Zn+ 2H* ---- > Zn2* +H2 (c) 5 735 BM
(c) Cu+ 2H* ---- > no reaction (d) 1 73 BM
(d) Fe+ 2H* ---- > Fe2*+H2
15. There is a steady decrease in the radii as the atomic number o(
10. [Fe(CN)6]3- has one unpaired electron (d2sp 3 hybridised Fe34-) the lanthanide element increases. It is because, for every
Magnetic moment = ^N(N + 2) BM additional proton in a nucleus, the corresponding electron goe<
into 4/-shell which is too diffused to screen the nucleus as
= 71 (1 + 2) BM
effectively as more localised inner shell. Hence, the attraction
= 73 = 1.73 BM of the nucleus for the outermost electrons increases steadily
while [Fe(H2O)6]
3+
has five impaired electrons (sp3d2 with the atomic number of the lanthanides. This contraction in
size is quite regular. This is called Lanthanide contraction.
hybridised Fe34-).
16. (a) Fe2* > Fe3* (b) Ni > Ni2* (c) Nb3+ = Ta3*
Magnetic moment = ^5(5 + 2) = 35 (d) Y3* < La3* (e)Cu<Zn (f) Cu > Ni (g) I" >F
= 5.92BM 17. Lattice energies, solvation energies and ionisation energies
Note : Magnetic moment = jN(N + 2) BM, where N = would be similar.
number of unpaired electrons
Chapter 16: The Transition and Inner Transition Elements | 665

Compounds
Potassium Dichromate (K2Cr207)
Preparation
K2C^O7 is obtained from chromite ore (FeCi^OJ in following steps:
Step I: Chromite ore is fused with molten Na2CO3 in 4FeCr2O4 + 8Na2CO3 + 7O2 1100°C
8Na2CrO4 + Fe2O3 + 8CO2?
the presence of air when Na2CrO4 and Fe2O3 water soluble water insoluble
are formed. Fe2O3 is water insoluble while
Na2CrO4 is water soluble hence, separation is
done.
Step II: Aqueous solution of Na2CrO4 is acidified when 2Na2CrO4 +H2SO4 Na2Cr2O7 + Na2SO4 + H2O
Na2Cr2O7 is formed. dil.

Step III ■ Sodium dichromate (Na2Cr2O7) is more Na2Cr2O7 + 2KCI K2Cr2O7 X + 2NaCl
soluble and less stable than K2Cr,O7 hence, orange
Na2Cr2O7 changes to K2Cr2O7 on reaction crystals
with KCI On coohng concentrated solution of
K2Cr2O7, orange crystals are obtained.

Properties Chromium (VI) oxide (also called chromium trioxide)


is red, crystalline compound. It precipitates when
Solubility conc.H^SC^ is added to cone solution ofK^C^O?.
• It exists as orange-red crystals (m.p. 398° C) and is
NaCl +H2SO4 NaHSO4 + HC1
moderately soluble in cold water but freely soluble in cone.
hot water.
CrO3 + 2HC1 —CrO2Cl2 + H^O
Behaviour in acid and base CrO2Cl2 (chromyl chloride) when passed into aqueous
• In alkaline solution (pH > 7), orange colour of Ci^Of “ NaOH solution, yellow colour solution of Na2CrO4 is
(dichromate ion) changes to yellow colour due to obtained. This on reaction with (CH3COO)2Pb/CH3COOH
formation of CrO2" (chromate ion) and again yellow gives yellow ppt of PbCrO4:
colour changes to orange colour in acid medium (pH < 7)
CrO2CI2 + 2NaOH —> Na2CrO4 + 2HC1
Na2CrO4+(CH3COO),Pb---- > PbCrOj
C^O2" + 2OH" ---- > 2Cr(^“ + ^0
orange-red yellow + 2CH3COONa
2CrO42“+2H+---- > Cr2O72"+HjO This test called ‘Chromyl-Chloride Test' is used to
Thus CrO^- and Cr2O^~ exist in equilibrium and are identify Cl" in inorganic salt-analysis.
interconvertible by altering the pH of the solution : CrO2Cl2 is an acid chloride and is analogous toSO2Cl2
2CrO42“+2H+ 2HCrO4 Ce,O72- + H2O (sulphuryl chloride.
Effect of heat Oxidising agent
a
• 4K2Cr2O7 K2CrO4 + 2^0., + 3O2 • Cr2O^“ is a good oxidising agent in acidic medium
(NH4)2Cr2O7 N2 +Ci^O3 + 4^0 Cr2O^“ + 14H+ + 6e ---- > 2Cr3+ + 71^0
green orange green
Once this reaction starts, it will keep going at a £°=1.33 V(A)
reasonable rate. Also a small amount of it gives a much
larger volume ofN2(g) along with Ci^C^ (green). Both features The reaction is spontaneous< E° >0) and Cr2O^“ is used
of the reaction lead to it being used in indoor fireworks. as a primary standard in volumetric (titrimetric) analysis.
Cr,O72-= 6e“; /. eq. wt. (Cr,O72-) = moL
Chromyl chloride test 6
• K^Ci^O? when heated with cone. H.2SO4 and soluble Following reactions are affected by Cr,O^“/H+ and
chloride (like NaCl, KCI), gives deep red vapours of solution turns green due to formation of Cr3’ (green):
CrO2Cl2 (chromyl chloride). (a) I- is oxidised to I2
KoCrA +2H2SO4 — 2KHSO4 + 2CrO3 + H2O 61“ 31, +6e"
cone.
666 | Essential Inorganic Chemistry

(b) H2S is oxidised to S Cr2O^ + 14H+ + Qe~ -> 2Cr3++7H2O


3H2S -—> 6H+ + 3S+6e" _______________ 6I~ -» 3^ +6e~_______
(c) Fe2+ is oxidised to Fe 3+ CigO?" + 14H* + 6I~ ■+ 2Cr3++711,0 + 312
6Fe2+ ----- > 6Fe3+ + 6e~ Iz + 2S2O^ + S.O2- + 2T
(d) S02 is oxidised to S0^~ Thus, Cr2O^" = 3^ = 61“ = 6e~
3SO2 + 6H2O----- > 3SO42’ + 12H+ + 6e"
(Use equation (A) to write complete reaction) 6
„ _ . molecular weight
(e) When H2O2 is added to an acidified solution of a Equivalent weight of K^C^Oy =---------- - ---------
6
dichromate [or any other Cr(VI) species], a
complicated reaction occurs. The products depend = ??4 = 49
6
on the pH and the concentration of Cr(VI) ion :
Cr2O2- + 2H+ + 4H2O2 ----- > 2CrO(O2)2 + 5H2O 2S20g = Ig — 21 = 2e
SoOq- = I = 1 e~
A deep blue-violet coloured peroxo compound 248
CrO(O2)2, called chromic peroxide is formed. This Equivalent weight of Na2S2O3 • = —1- ’
decomposes rapidly in aqueous solution into Cr3+ and
oxygen. The peroxo compound can be extracted into ether, • PbCrO4 — Chrome yellow
where it is stable; it reacts with pyridine forming the • PbCrO.4 PbO — Chrome red
adduct: Py. CrO(O2)2 PbCrO4+ [FeFe(CN)6r — Chrome green
0 Prussian blue
O. 0 Zn(OH), • ZnCrO4. H„O — Yellow
Cr, Cr2O3-2H2O — Guigrete's green
0 XO CrO3 — Orange (called chromic acid)
(chromic peroxide) K3[CrO8] — Red-brown
[Cr(H2O)6]2+ — Bright blue
[Cr(H2O)6]3+ — Violet
DEEP Focus • In Ci^O^- two CrO3 units have been joined by 0
In less acidic solution, K2Cr2O7 and H2O2 give salts
which are violet coloured and diamagnetic due to formation of 0 0
[CrO(O2)(OH)J-. Cf 1230 ^Cr
In alkaline solution with 30% H2O2, a red-brown 0 | 1.62A 0
compound K3CrO8 is formed. It is tetraperoxo species O 0
[Cr(O2 )4 J3- and thus contains Cr (V) ion. | FeCr2O4]
In ammoniacal solution (NH„ ),CrO., 4’ a dark red-brown
compound with Cr (IV) ion is formed. A Na2CO3 + air
C02
K^SC^ Cr2(SO4)3 24H./) — Chrome alum water soluble -»] Na2Cro7~|—
+H20 |

It is a deep purple compound used for leather tanning. H*


I
Fe2O3 (water insoluble)
• Chrome alum is obtained when acidified K2Cr2O7 solution | Na2Cr2O7|— K2Cr2O7 |
is saturated with S02:
K„Cr907 + H„SO, + 3SO9 A K„SO>4 + Cr2(SO4)3
NH4CI
Z Z I Z 4 Z Z ‘
| (NH4)2Cr2O7 [■ A
Nzf+ Cr2O3
crystallises as (green)

+ H2° | K2CrO4 [
(NH3)3CrO4 CrO5 (deep violet soluble in eJerj
KCr(SO4 )2 ■ 12H2O OH- H* j
12 x
s ^J+S r A K2Cr04 + Cr203 + 02
Fe^ cone. H2SO4
Fe3% KHSO4 + Crth
Estimation of Ci^O*" s2>- ■oxidising
K2Cf2O7
H2SO4 + SO;
gg2- KCr(SO4)2-12H2o|
I?" — nature
KjCrA can be estimated volumetrically by a chrome alum
+ cH^green) KCI
iodometric titration. A
cone. H2SO4
Acidifiedis treated with KI and liberated^ is K3CrOg CrO2CI2 NaOhl Na2CrO4 CH3C0_^ PbOO,|
(CH3C00)2Pb
titrated using hypo (Na2S2O3) solution with starch (chromyl chloride test)
indicator.
Chapter 16: The Transition and Inner Transition Elements | 667

Potassium Permanganate (KMnO4)


Preparation
KMnO4 is prepared from Mn02 (pyrolusite) in following steps :
s,eP1 • MnO2 is fused with KOH and KC103 or KNO3 when potassium 3MnO2 + 6K0H + KCIO3 A
3K2MnO4 + KCI + 3H2O
manganate (K2MnO4) is formed as indicated by its green colour. green
KCIO3/KNO3 acts as oxidising agent.
Step ||: The fused mass is extracted with water and solution is green (MnO42*)
(a) when Cl2 gas is passed. K2MnO4 is oxidised toKMnO4 (purple). 2K2MnO4 + Cl2 —> 2KCI +2KMnO4
green purple
(b) MnO2- can be oxidised to MnO4 electrochemically at anode. at anode : MnO2’ —♦ MnO4 + e~
at cathode: 2IT + 2e' ---- > H2

• Strong oxidising agents such as PbO2 or sodium mol. wt.


Eq. wt. of KMnO4 in acidic medium =
bismuthate (NaBiO3) can also oxidise Mn2+ to Mn04. 5
• MnO42“ is unstable in acidic medium and Some oxidation reactions (using Mn04/H+) are :
disproportionates: (a) Fe2+ is oxidised to Fe3+
SMnO2- + 4H+ ■» MnO2 + 2MnO4 + 2H2O Fe2+ ---- > Fe:3+ +e-
• KMnO4 crystallises as deep purple rhombic prisms. (b) Oxalate is oxidised to C02,
QjO/----- > 2CO2 + 2e"
DEEP Focus (c) I" is oxidised to
MnO“ (VII) has 3d electronic configuration in Mn and thus
MnO“ would be expected to be colourless, but colour arises due to 21“ ---- > 12 +2e“
charge transfer by oxygen to Mn and thus one of the oxygen is 0“ (d) Cl" is oxidised to Cl2
and Mn changes from +VII to +VI (with one unpaired electron in 2C1“ -----> Cl,2 + 2e"
3d) hence, coloured.
(e) HA is oxidised to 02
* It is isomorphous with KC104, potassium perchlorate;
highly soluble in water :
HA 02 + 2H+ +2e“
(f) S02 (S0^“) is oxidised to SQ;”
Effect of heat HoO + SO32“-----> S02“ + 2H+ + 2e“
• 2KMnO4 I^MnC^ + MnO2 + O2 (g) H^S is oxidised to S
S + 2H+ + 2e~
Oxidising agent (h) CHgCHjOH is oxidised to CH3CHO
I. KMn04 is a good oxidising agent; it finds use in CHgCHjOH CH3CHO + H^O
volumetric analysis as secondary standard in the
estimation ofFe2+, CjO2-, HA etc. in acidic medium: II. In dilute alkaline solution, Mn02 is formed.

MnO4 + 8H+ + 5e“ - > Mn2+ + 4^0 Mn04 + 21^0 + 3e“ ---- +5 MnO2 + 4OH
(purple) (colourless very- (mol. wt.')
Eq. wt. (in basic medium) =
very light pink) I 3 )
The purple colour of Mn04 acts as its own indicator HA is oxidise to O2
(self) in titrations
2MnO4 + 3H2O2 ---- > 2MnO2 + 302 + 30H“ + 2^0
Mn04 =5e"
Thus, equivalent weight of Mn04 HI. In very strong base, and with an excess of
mol. wt. . .,. ,. Mn04, however manganate ion (Mn0^“) is
=---------- m acidic medium.
5 formed.
Alternately, 4Mn0; +40H“ ---- > 4MnO42“ + O2 +2^0
2KMnO4 + 3H2SO4 K2SO4 + 2MnSO4 + 3^0 MnO^+e" ---- > MnO^“
+ 5(0) mol wt.
and eq. wt. =—-—
2KMnO4 s 5(0) taking weight as 16.
= 10 (0) taking weight as 8. Thus, equivalent weight of KMn04 varies with
medium.
™n°4 = 1(0)
5
668 | Essential Inorganic Chemistry

Alkaline KMnO4 is called Baeyer’s reagent and is With cone. H2S0d


used to test unsaturation in an organic compound, when • If a small amount of KMnO4 is added to concentrated
it decolorises KMnO4. H2SO4, a green solution containing MnO3 ions is
CH2=CH2 + H2O + O CH2—CH<l2 formed:
alkaline KMnO4 KMnO4 + 3H2SO4 K+ +MnOg + 3HSO4 + H3O4
OH OH green
In dilute alkali, water or acidic solutions, MnO^~ MnO3 changes to MnO3 with Na2CO3;
again disproportionates: 2MnO3 + Na2CO3 2MnO3+C02+|o2+2Na’
3MnO42-+4H+ ---- > 2MnO4 + Mn02 + 2H2O
3MnC^~ + 2^0---- > 2MnO4 + Mn02 + 40H" If KMnO4 is in excess, an explosive oil Mn2O7 is
formed:
Stability in acidic medium 2KMnO4 +H2SO4 Mn2O7 + i-l^O
• Permanganate solutions are intrinsically unstable is cone.
acidic solution, and decompose slowly. MnO^ has following structure
Decomposition is catalysed by sunlight, so
0
KMnO.4 solutions should be stored in dark
bottles and they must be standarised at the /Mn
I
moment of its use in volumetric analysis.
4MnO^ + 4H+ ---- > 4MnO2 + 3O2 + 21^0
0 I
0
0

MnO2

A KOH + KCIO3

H+ Cl2
MnO2 + KMnO4 K2MnO4 KMnO4

electrolysis
(at anode)
K2MnO4 + MnO2 + O2
H2O2 A
O2 H* MnO2 + O2 (if placed in sunlight)
C2O4
CO2
Fe2* H* + OH"
Fe3+ KMnO4 Baeyer’s reagent
sof~ oxidising nature
SO4'
Mn2+
t
decolourises (C=C)
MnO2 cone. H2SO4 compounds
X2
s
h2s r
Mn2O7 (explosive oil) MnO3 (green) Na2CO3
if acid is in excess H2O
if KMnO4 is in excess MnOs KMnO4 + MnO2
t H;0

Zinc Oxide (ZnO) Zinc white Zinc on burning in air gives ZnO (commercial method)
2Zn + 02 ---- > 2ZnO
Preparation
• ZnO is formed when ZnS is oxidised : Properties
2ZnS + 3O2 ---- > 2ZnO + 2SO2 • ZnO is white when it is cold, a property that has given it
a use as a pigment in paints. However, it changes
Zn(0H)2 on strongly heating gives ZnO :
colour, when hot, to a pale yellow. This is due to
Zn(OH)2 ZnO + HjO change in the structure of lattice.
Chapter 16: The Transition and Inner Transition Elements | 669

• ZnO is soluble both in acid and alkali and is thus Properties


amphoteric in nature :
• Colourless crystalline compound soluble in H/2O and
ZnO + 2H+ ■» Zn2+ +HaO alcohol; m.p. 212°C
base acid
• When exposed to light, it decomposes hence, stored in
ZnO + 20H“ + HaO---- > [Zn(0H)4 f" or ZnOg- a brown coloured bottle :
acid base zincate ion
A. red hot A,T>212°C
Or 2HC1 ---- > ZnCl2 + H2O
ZnO + 2HC1 2Ag + 2NO2 + O2 <----------- 2AgNO3 -------------- >

ZnO + 2NaOH---- > Na2ZnO2 + HaO 2AgNO2 + O2


sodium zincate • It is reduced to metallic Ag by more electropositive
A metals like Cu, Zn, Mg and also by PH3.
• ZnO + C ---------- > Zn + CO
>1000°C 2AgNO3 + Cu-----> Cu(NO3)2 + 2Ag
ZnO + CO Zn + CO2 6AgNO3 + PH3 + 3^0-----> 6Ag + 6HNO3 + H3PO3
• It is preferred to white lead as it is not blackened by • It dissolves in excess of KCN :
HoS. It is also used in medicine and in the preparation ———>
AgNO3------- > AgCN------- K[Ag(CN)2]
ofRinmann’s green (ZnCo2O4). white soluble
PPt potassium argentocyanide
• AgNO3 gives white precipitate with Na2S2O3; white
line Sulphate (ZnS04) precipitate changes to black.
Preparation 2AgNO3 + Na2S2O3 ---- > Ag2S2O3 + 2NaNO3
• ZnSO4 • THjO (also called white vitriol) is formed: white ppt
“ by decomposing ZnCO3 with dil. H2SO4 Ag2S2O3 + H2O Ag2S + H2SO4
black
ZnCO3 + H2SO4 ---- > ZnSO4 +1^0 + C02
“ by heating ZnS (zinc blende) in air at lower
temperature and dissolving the product in
DEEP Focus
dil.H2SO4 • It possesses powerful corrosive action on organic tissues,
which it turns black especially in presence of light. The
A
2ZnS + 3.5 02 ---- > ZnO + ZnSO4 + S0'22 blackening is due to finely divided metallic silver, reduced
by the organic matter.
ZnO + H2SO4 ---- > ZnSO4 + HaO
• Ammoniacal AgNO3 is called Tollen's reagent and is used
Properties to identify reducing sugars (including aldehydes):
ECHO + 2Ag+ + 3OH- -£-> flCOO' + 2Ag i + 2H2O
• Highly soluble in water and solution is acidic in
nature due to hydrolysis It is called ‘silver mirror test' of aldehydes and reducing
sugar (like glucose, fructose).
ZnSO4 + 2HaO Zn(OH)2 +H2SO4
100°C ^3 ZnSO4 Ag
• ZnSO4 -7^0 -------- > ZnSO4 -6HaO
A A 4
T > 760° „ _ A HNO3
---------- > ZnO + SO3
A, red hot A,T>212*C
It slowly effloresces when exposed to air. O2 + Ag + N02 AgNO3 AgNO2 + O2

• It is isomorphous with Epsom salt and used in the S'T Cu


manufacture of lithophone (which is a mixture of
Ag2S2O3 KCN Cu2* + Ag
BaS + ZnSO4 and is used as white pigment). PH3
white
H2O Ag Ag + H3PO3 + HNO3
K[Ag(CN)2]
Silver Nitrate (Lunar Caustic) AgN03 Ag2S
Preparation black

• When Ag is heated with dil. HNO3 AgNO3 is formed.


Crystals separate out on cooling the concentrated Ferric Chloride (FeCI3)
solution of AgNO3.
A
Preparation
3Ag + 4HNO3 3AgNO3 + NO + 2H2O
• 2Fe + 3C12 2FeCl3 (anhydrous)
670 | Essential Inorganic Chemistry

• When Fe is dissolved in aqua-regia or FegOg is in ii


or FeCl3 +K4[Fe(CN)6]----- > KFe[Fe(CN)6] + 3KCl
dissolved in HC1, hydrate FeCl3 ■ GHjO is formed : Prussian blue
HNO3 + 3HC1----- > NOCI + 2H2O + 2C1 • A deep blue colour is also produced when Fe2+ reacts
Fe + 4HC1 + 2C1----- > 2FeCl3 + 2H2 with K3[Fe(CN)6 ], potassium ferricyanide :
Fe2O3 + 6HC1----- > FeCl3 + 31^0 II in II III
FeSO4 + K3[Fe(CN)6] KFe[Fe(CN)6]
Turnbull's blue
Properties
• Anhydrous FeCl3 is black but hydrate FeCl3 • GHgO is
yellowish brown, highly deliquescent crystalline solid. DEEP Focus
• It sublimes at 300°C giving a dimeric gas : Recent X-ray work, IR and other spectroscopic
methods have proved that Turnbull's blue is identical to
Cl\ /Cl Cl Prussian blue. The intense colour arises from electron
2FeCl3 300 °C) Fe Fe transfer between Fe(+II) and Fe(+III).
Cl Cl^ XC1
Fe3* partly oxidises [Fe(CN)6J4“ to [Fe(CN)6J3- and itself is
(FeCl^ gas dimer
111
reduced to Fe2*, and also Fe2* partly reduces [Fe(CN)6l' to
• FeCl3 dissolves in both ether and water giving II , II in
solvated monomeric species : [Fe(CN)6J4- and itself is oxidised to Fe3*. Thus, Fe[Fe(CN)6f
III II
andFe[Fe(CN)Dc]- are identical.
O •> FeCl3, [FeOV^ClJCl^O

solvated FeCLo hydrate FeCl3


Ferric Oxide (Fe2O3)
Preparation
• It normally consists of trans [Fe(H2O)4Cl2]Cl 2H2O Hydrolysis of FeCl3 actually gives a red-brown
but in strong HC1 it forms tetrahedral (FeCl4)“ ions. gelatinous precipitate of the hydrous oxide Fe2O3(H2O)1
which on heating at 200°C gives red-brown a-Fe2O3 (which
Hydrolysis occurs as the mineral haematite).
• Aqueous solution is acidic due to hydrolysis :
• On oxidation ofFe3O4,y-Fe2O3 is formed.
FeCl3 + SHjO Fe(OH)3 + 3HC1
or [FetHjOJJ3* +H2O [FeOIsOMOHtf* +H3O* Properties
acid base base acid • a-Fe2O3 has hexagonally close-packed lattice of 02'
ions with Fe3* ion in two thirds of the octahedral hoies
Oxidising agent
while y-Fe2O3 has cubic close-packed arrangement of
• FeCl3 is also an oxidising agent: O2" ions with Fe3* ions randomly distributed in both
2FeCl3 + HgS----- > 2FeCl2 + 2HC1 + S the octahedral and tetrahedral sites.
2FeCl3 + SnCl2 ----- > 2FeCl2 + SnCl4
Yellow colour of aqueous Fe (III) changes to light
Effect of heat
green aqueous Fe (II). 1400°C, A
* 6Fe2O3 4 4Fe3O4 + O2
n m
Detection of Fe3* ion Fe3O4 is a mixed oxide FeO Fe2O3.
• Fe3* solution gives blood red colour with SCN” ions : O3 and xFe
FeO, Fe;2vr O4d all tend to be
v3qv,
Fe3* + SCN" ----- > (Fe(SCN)f * non-stoichiometric. Fe3O4 occurs in nature as
This red colour also contains [Fe(SCN)(H2O)5f *, magnetite.
[Fe(SCN)3] and [Fe(SCN)4 ]“. However, when F~ is added,
colour fades due to formation of [FeF6]3- .
Solubility in water
• Fe3* solution also gives deep blue precipitate of • Fe2O3 is predominantly basic. Freshly precipitated
Prussian blue with K4[Fe(CN)6], potassium Fe^g-tf^O) dissolves in acid giving pale violet
ferrocyanide : [Fetf^Og ]3* ion. This [Fe2O3 • ] also dissolves in
m n in 11 concentrated NaOH forming [Fe(OH)gf". This shows
4FeCl3 +3K4[Fe(CN)6]----- > Fe4[Fe(CN)6]3 + 12KC1
Prussian blue that Fe2O3 is slightly amphoteric.
Chapter 16:The Transition and Inner Transition Elements | 671

Fusion with Na2C03 Oxidation


• Fusion of Fe2O3 with Na2CO3 gives NaFeO2 (sodium • Acidified Mn04 andC^O?" oxidise Fe2* toFe3* :
ferrite) which is hydrolysed to Fe2O3 and NaOH : MnOj + 8H* + 5Fe2* ---- > 5Fe3* + Mn2+ + 41^0
Na2CO3 + Fe2O3 ---- > 2NaFeO2 + C02 CijjO?- + 14H* + 6Fe2* ---- > 6Fe3+ + 2Cr3* + 70,0
2NaFeO2 + HjO---- > 2NaOH + Fe^
Thus, MnO^ and Ci^O^" can be used to estimate Fe2*
(In earlier days this method, called Lowig Process, salts by volumetric method in acidic medium.
was used to manufacture NaOH).
MnO; Mn2*
With Cl 2 Fe2* -- » Fe3* + e~
• If Cl2 gas is passed into an alkaline solution of MnO; = 5Fe2*
hydrated ferric oxide, a red purple solution is formed Similarly, Cr^O2/=6Fe2*
VI
containing the ferrate ion [FeO4 r.
Detection
Fe2O3 + 2NaOH---- > 2NaFeO2 + 1^0
• Fe2* salts form blue precipitate of Turnbull's blue with
2NaFeO2 + Cl2 + 4NaOH--- > 2Na2FeO4 + 2NaCl
K3[Fe(CN)61.Turnbull's blue is used as pigment in ink
+ 2H2 and paint (also refer FeCl3)
Fe2+ +[FeII1(CN)6J1- ---- > Fen[Feni(CN)6T
DEEP Focus Tumbull's blue
Na^eC^ can also be obtained by oxidation of Fe2O3 with
NaOCl (sodium hypochlorite). Na^eO* has Fe(+VI) and is Ring test (for N03)
a strong oxidising agent (like KMnO4). • The complex [Fe(H2O)5NO]2 * is formed in the brown
ring test for nitrates when freshly prepared FeSO4
Ferrous Sulphate (FeS04 7H20- Green Vitriol) solution is added to aqueous solution ofN03 followed
by addition of cone. HaSC^. The colour is due to charge
Preparation
transfer. Its magnetic moment is approximately
• FeS + H2SO4 ---- > FeSO4 + T 3.873 BM confirming the presence of three unpaired
Fe +H2SO'44 FeSO4 + H2 T electrons in iron. Thus iron exists as Fe(+I) and
dil. nitrosyl as NO*.
FeSO4 solution is concentrated when pale green NO ■> NO* + e~
crystals are obtained. J Fe2+ + e~ ---- > Fe*

Effect of heat TJ, T T ? ? n|u|(t | t | t)


A
• 2FeSO4 + SO2 + SO3
gained three unpaired
electron from electrons in
Atmospheric oxidation NO Fe*

• Anhydrous FeSO4 is colourless/white. On exposure to Thus, magnetic moment = y]N(N + 2) BM


atmosphere it turns brownish- yellow due to formation = 715
of basic ferric sulphate : = 3.873 BM
4Fe(0H)SQ4
4FeSO4 + 21^0 + 02 ---- ■>> 4Fe(OH)SO
• Mixture of FeSO4 and H9O2 is used as Fenton's
• FeSO4 • 7H2O is isomorphous with ZnSO4 • 7H2O reagent for producing hydroxyl radicals and for
(white vitriol) and MgSO4 • 71^0 (Epsom salt). oxidising alcohols to aldehydes.
• Saturated solution of FeSO4 and (NH4)2SO4 on
Hydrolysis concentration and cooling gives crystals of Mohr's salt,
• Aqueous solution of FeSO4 is acidic due to hydrolysis FeSO4 (NH4)2SO4 6H2O which is widely used as
ofFe2* : primary standard for standardisation of KMnO4 and
Fe2* + 2H2O Fe(OH)2 + 2H+ I^C^O? solutions.
672 | Essential Inorganic Chemistry
Fe(OH)SO4
f H2O/O2

h2so4 A
Fe FeSO4 Fe2Os + SO2 + SO3

Cl2
,, K3[Fe(CN)6]
HCI aqua regia
Fe2O3 FeCI3 Fe ’ll III
KFe[Fe(CN)6]
H2S KSCN
HCI + FeCI2 + S Turnbull's blue
SnCI2
SnCI4 + FeCI2
,_ L
oxidising nature
K4[Fe(CN)6]
[Fe(SCN)]CI2
red
MnO4 /H*

Cr2O7-/H*
*Mn2+ + Fe3*

* Cr3* + Fe3+
III II
KFe[Fe(CN)6] NO3 cone. H2SO4
[Fe(H2O)5NO]2+
Prussian’s blue brown ring
H2O (Ring Test)

A A
Fe(OH)3 Fe2C>3 Fe3O4 + 02
140°C
(FeOFe2O3)
H* V/
[Fe(H2O)6]3+' o°7
&
NaOCI
pale violet
H2O NaOH + Cl2
NaOH + Fe2O3 NaFeO2 - —► Na2FeO4
(sodium ferrite) sodium ferrate
(oxidising agent)

Target Practice 2
1. Iron(II) oxide is written as FeO. What does it mean? 5. Cr2O3 is an amphoteric oxide. Thus,
it dissolves in acid forming........................
and dissolves in base forming...................
2. Mention catalysts in the following 6. Cr2O2" PH > 7 > CrO2’ pH < 7 > Cr2O2’
(a) KC1O3 -----»> KCl
KC1 + O,
O,
(b) CS2+C19 ----
---- >> CC1
CC1/ Explain the change based on reactions
H\ ch3
(c) CH,C=CCH,o O
C=C
h3c H 7. Name the compound which imparts odour in fire-works
.................................................
09
(d) C2H4 CH3CHO 8. Write the formulae of
(e) CO + H„ ---- > CH,OH (a) Chromyl chloride
4. o
(b) Chrome yellow
3. When CO., is passed into aqueous K2CrO4 (yellow) (c) Chrome green
solution, this changes to orange. This is explained by (d) Chrome alum
following reactions 9. What is oxidation state of Cr in
(a) CrO5
(b) [CrO3(OH)r
4. Which of the following pairs is more stable? (c) [Cr(O2)4]3-
(a) CrO.. I I WO, (d) CrO2Cl2
(b) MnO;4 I I ReOT4 10. Consider structure of Cr2O7
(c) Cr2* I_____ I Cr3+ Ox ^0. ,0
Cr cT Cr<
(d) Mn2* MnOT4 0 |y I
(e) CrO42- MoO24 ’ 0 0
(f) Mn2O7 Re2O7 State (a) Angle Za =
(b) Bond length x =
(g) V2+ VO43’
(c) Bond length y =
Chapter 16: The Transition and Inner Transition Elements | 673

IL Alcohol content in a drunkard person can be determined by (a)


K4Cr2O7. Explain in their lines. (b)
15. (a) ZnO is white when cold and is yellow when hot. This
colour change is due to ..........
(b) ZnSO4 • 7H2O loses H2O on exposure to air. This
12. (a) Lithophone is property is called
(b) Rinmann’s green is 16. Calculate equivalents in one mole each of in
(c) Zinc blende is following reactions
(d) White vitriol is one mole =
(a) Cr2O2- ---- > 2Cr3* equivalents
13. ZnO is converted into . in basic medium and into
in acidic medium. (b) MnO” ---- > MnO2- equivalents
14. ZnO is amphoteric. Name two other oxides of d-block (c) MnO; ---- > Mn2* equivalents
elements which are amphoteric (d) MnO" ---- > MnO2 equivalents

Answers
1. It is to indicate that ratio of Fe and 0 atoms is not exactly 1:1. 7. (NH4)2Cr2O7
Analysis shows that formula varies between Fe094O and 8. (a) CrO2Cl2 (b) PbCrO4 (c) PbCrO4 +FeIII(Fe1,(CN)6r
(d) [K2SO4 Cr2(SO4)3 -24H2O]
2. (a) MnO 2< (b) FeCL3 (c) Li/NH3 (d) PdCl2 9. (a) Cr = + 6 (b) Cr = + 4 (c) Cr = + 5 (d) Cr = + 6
(e) ZnO/CuO 10. (a) Za = 123° (b) x = 1.77 A (c) y = 1.62 A
3. C02 + H20 H2CO3 11. Drunkard person takes a breath, alcohol content is oxidised to
H2CO3 x— n hco;3
+ nw
H* * CH;1CHO by K2Cr2O7/H* which in turn is reduced to Cr3*
(green). Formation of green colour indicates alcohol content.
HT change K2CrO4 intoK2Cr207
12. (a) Mixture of BaS and ZnSO4 (b) ZnCo2O4
2CrO4"+2H* ---- > Cr2O2" + H2O (c) ZnS (d) ZnSO4-7H2O
yellow orange
13. [ZntOH).,]2" or ZnO2" (basic medium); Zn 2* (acidic medium)
4. (a) WOi3 (b) ReO" (c) Cr3* (d) Mn2* (e) MoO4 14. (a) TiO2 (b) V2O5
(0 Re2O7 (g) VO3"
15. (a) Change in structure of lattice (b) efflorescence
5. Cr3 * (acid medium), Cr(OH)4 (basic medium)
g pH > 7 (basic medium) 1pH ” < 7 (acidic medium) 16. . (a) 6 (b) 1 (c) 5 (d) 3
Cr2O7~ changes to CrO2" CrO4" changes to Cr2O7- Change oxidation number per mole
Cr2O7"+2OH" > 2CrO2"+H2O
2CrO2" + 2H* ---- > Cr2O?"+H2O

Practice Exercise 2
1. V02+ is oxidised to VO2 by MnO" in acidic medium, which 5. Identify A, B, C... G.
v KOH / 4 \ Pb(CH3COO).
in turn is reduced to Mn2*. Write balanced equation for K2Cr2rx
O7 ------- > (A) (B)
this reaction. CH-COOH M

2. One of the following oxide is also called chromic acid. r


NH.C1
«
Al, A
Select that one : CrO2, Cr2O3, CrO3, CrO (C) (D) + (E), (E) ------ > (F), (metal)+(G), (oxide)
gas green
3. The red oxide CrO3 dissolves in water to produce a strongly
6. Green coloured compound (A)
acidic solution to form orange coloured solution. Solution |A,NaOH + H2O2 J .
changes to yellow on making it alkaline. Explain these 1---------------- ±—=-> (C) yellow coloured solution
observations in terms of chemical equations.
A KoCr2O7 + cone. H2SO4
4. What is oxidation number of Cr in deep-red coloured gas (B) yellow coloured solution (C)
(a) CrO2Cl2
(CH3COO)2Pb + CH3COOH. zr^ „
(b) CrO6 orCrO(O2)2 -—------------------- --------- > CD) yellow ppt

(c) (NH4)2Cr2O7 Identify A, B, C and D.


(d) (NH3)3CrO. or(NH,),Cr(O2)2 7. Explain by reactions.
(e) [Cr(O2)4]3’ (a) Mn3* disproportionates to Mn2* and MnO2 in acidic
medium
674 | Essential Inorganic Chemistry

(b) MnO2- also disproportionates to MnO2 and MnO; in 13. You have been given A1C13 and ZnCl2 solution. To each
alkaline medium.
NaOH and NH4OH are added in excess. What happens?
(c) MnO; oxidises Mn2+ into MnO2 in acidic medium.
14. AG^ (standard free energy of formation) of ZnO and CO are
8. MnO42- changes to MnO2 and MnO; in acidic medium.
Calculate equivalent weight of MnO42-. -318.2 kJ mol-1 and -137.3 kJ mol-1. Is reduction of ZnO by
carbon possible ?
9. (a) A qualitative analysis test for Mn 2+ involves its
15. Colourless salt (A) gives white ppt (B) with NaOH; ppt (B)
oxidation to MnO; in acidic solution by sodium
dissolves in excess of NaOH to form (C). On passing H^S
bismuthate, NaBiO3. The bisulphate ion is reduced to
gas into (C) white ppt (D) appears. (A) also gives white ppt
Bi3*. Write a balanced equation for this test.
(E) with AgNO3 solution. Identify (A) to (E).
(b) What happens ifPbO2 is used instead of NaBiO3 ?
16. Colourless salt (A) —(B) + (C) gas. (B) dissolves both in
10. Give structures of MnO?4 and CrO42- ions.
acid and alkali solution. Gas (C) turns lime water milky and
11. H902 can be oxidised to O2 by MnO; in acidic as well as in acidified K2Cr2O7 solution green. Ammoniacal solution cf
basic medium. (A) gives white ppt (D) with H2S gas. Identify (A), (B), (0
(a) Write reactions. and CD).
(b) If 100 mL of IM MnO; is used in acidic medium, 17. What is the advantage of using ZnO in paints instead cf
calculate volume of IM MnO; to be used in basic Pb2+ salts?
medium by sameH2O2.
18. Compare thermal stability of ZnO, CdO and HgO.
12. What happens when CO2 gas is passed into MnO2-
solution?

Answers
1. 5VO2* + MnO; + H,0---- > 5VO2 + Mn2+ + 2H+ 9. (a) 5BiO»O + 2MnO2+ + 14H* ---- > 2MnO;+5Bi3++7H2O
j.

2. CrO 3 (b) 5PbO2 + 2Mn + 4H* ---- > 2MnO; + 5Pb2++ 2H2O
3. 2CrO3 + H2O---- > Cr2O2- + 2H* 0 0
Cr2O2- + 2OH- ---- > 2CrO2- I I
10. Cr. -Mn-
4. (a) VI (b) VI (O = - 2, diperoxo = - 4) (c)VI (d)IV(e)V o' I 0 O' 0
(oxidation number of 0 in peroxide = - 1 and in oxide = - 2) O 0
5. (A):K2CrO4 (B):PbCrO4 (C):(NH4)2Cr2O7 (D): N2 2+ +5O2 +8H20
11. (a) 2MnO; +5H2O2 +6H* ---- > 2Mnz+
(E): Cr2O3 (F): Cr (G): A12O3
2MnO; + 3H2O2 ---- > 2MnO2 + 3O2 + 2OH" + 2H,0
6. (A):CrCl3 (B): CrO2Cl2 (by chromyl chloride test of Cl-)
(C) : Na2CrO4 (due to oxidation ofCr 3+ by NaOH/H2O2 and (b) In acidic medium MnO; ---- > Mn2*
due to dissolution ofCrO2Cl2 in NaOH) 1M = 5N
(D) : PbCrO4
H2O2 s 100 mL of 5N KMnO4
7. (a)2Mn3+ +2H2O ---- > Mn2+ +MnO2 +4H+ In basic medium MnO; ---- > MnO2
+3 +2 +4 2
(b) 3MnO2- + 2H2O ---- > MnO2 +2MnO; + 4OH- .. IM = 3N
+6 +4 +7
H 2O2 = V mL of 3N KMnO4
(c) 2MnO; + 3Mn2+ + 2H2O ---- > 5MnO2 + 4H+
NV = 3 V
8. 3MnO2- + 2H* ---- > MnO2 +2MnO; + 2H2O
+6 +4 +7 3V = 500
It is a disproportionation reaction. In disproportionation V=—= 166.67 mL
reaction, equivalent weight of the species undergoing 3
disproportionation = Eq. wt. (in oxidation) + Eq. wt. (in 12. Aqueous CO2 releases H+ which can convert MnOJ' ink
reduction)
MnO2 and MnO; in a disproportionation reaction.
MnO2- ---- > MnO2 Eq. wt. (redn.) = mol. wt.
+6 +4 2 CO2 + H2O H+ + HCOj
MnO2- ---- > MnO; Eq. wt. (oxidn.) = moL wt- 3MnO2- +4H* ---- > MnO2 + 2MnO;+H20
+6 +7 1 reduction | 1'
Eq. wt. of MnO2- =— +— =—M oxidation
2 12
Chapter 16: The Transition and Inner Transition Elements | 675

13. AICI3 ZnCI2


(a) NaOH added:
AI(OH)3 white precipitate which dissolves in excess of it. Zn(OH)2 white ppt dissolved in excess of it
AICI3 + 3NaOH —> AI(OH)3 i + 3NaCI ZnCI2 + 2NaOH —> Zn(OH)21 + 2NaCI
AI(OH)3 + NaOH —> NaAIO2 + 2H2O Zn(OH)2 + 2NaOH —> Na2ZnO2 + 2H2O
sodium meta aluminate sodium zincate
(soluble) (soluble)
(b) NH4 OH added: Zn(OH)2 white ppt. but precipitate dissolves in excess of it
AI(OH)3 white ppt; precipitate remains insoluble in excess of it Zn(OH)2 + 4NH4OH---- > [Zn(NH3)4]2* + 4H2O + 2OH"
soluble

14. ZnO + C---- > Zn + CO (E):AgCl


AG° = &G°f (CO)-AG7(ZnO) 16. (A): ZnSO3 (B): ZnO (C):SO2 (D): ZnS
= - 137.3- (- 318.2)= 180.9 kJ mol'1 17. ZnO in white paint by atmospheric H2S gives ZnS (white) but
Pb2+ salts in white paint turns black by atmospheric H2S due
Positive value of AG0 indicates that reaction is non-spontaneous.
to formation of PbS.
(However, it is spontaneous above 1000°C).
18. ZnO > CdO > HgO
15. (A):ZnCl2 (B): Zn(OH)2 (C):Na2ZnO2 (D): ZnS

Copper Sulphate (CuS04 5H20 - Blue Vitriol) 2Cu2* + 4CN" ■> 2CuICN + (CN)2
cyanogen
Preparation
CuCN + 3CN" ---- > [Cu’fCNJJ3-
• CuO + ---- > CuSO4 + HjO
[Cu(CN)4 J3- is a stable complex. Cd24 also forms
2Cu + 2H2SO4 + O2 2CuSO4 + 2^,0 complex [Cd(CN)4]2" but unstable, which ionises to
On concentration and cooling, blue crystals of form Cd2* and CN". Thus if Cd2* and Cu2+ both are
CuSO4 • SHjO separate. present, Cu2^ and Cd2* both are complexed by CN~.If
gas is passed, Cd2* (in unstable [Cd(CN4^" ]#ives
Properties
• The Cu(II) ion or cupric ion is written more precisely yellow ppt of CdS.
as [Cud^O^f* when in aqueous solution. It has a
bright blue colour. In CuSO4 • four of the water Complex formation with NH3
molecules are associated withCu2*, and the fifth • Addition of aqueous NH3(aq) to Cu2* salts give blue
ppt of Cu(OH)2 which dissolves in excess of NH3(a<?)
is hydrogen - bonded to the sulphate ion as well due to the formation of the deep blue complex
as to the water molecules on the copper ion, [Cu(NH3)4f* :
hence, this it is written as [Cu(H2O)4 ]SO4 • t^O.
CuSO4 + 2NH4OH---- > Cu(OH)2 X + (NH4)2SO4
Effect of heat Cu(OH)2+4NH3 ---- > [Cu(NH3)4f* + 2OH"
• When this is heated, it loses its water of hydration in deep blue (soluble)
stages:
CuSO4 -5H2O ^3 CuSO4 -^O ^3 CuSO4
With hypo
blue bluish-white white • CuSO4 solution on reaction with Na^Og gives
CuO + 5O2 + O2 colourless (white) NaCuS2O3 due to reduction of Cu(II)
to Cu(I).
CuSO4 + Na2S2O3 ---- > CuS2O3 + Na2SO4
DEEP Focus
2CuS2O3 + 2Na2S2O3 ---- > 2NaCuIS2O3 + Na^Og
It is the complexing of water molecules to the copper ion
that is responsible for the blue colour of the pentahydrate. On boiling or in acid solution, NaCuS2O3 changes to
Cu^S.
Complex formation with KCN 2NaCuS2O3 + HjO---- > Na2S2O3 + CujS + H2SO4
• Addition of KCN to CuSO4 solution first causes • When the copper (II) ion (complexed with tartrate ion)
reduction and then precipitates CuCN (cuprous is heated in basic solution with a reducing sugar, such
cyanide). This reacts with excess CN", forming a as glucose), the ion is reduced to CtijO (brick red
soluble four coordinate complex [Cu(CN)4 J3- which is precipitate). Mixture of alkaline CuSO4 and sodium
tetrahedral in shape: potassium tartrate is called Fehling's solution and
676 | Essential Inorganic Chemistry

this test called Fehling’s test of aldehydes and Hg2Cl2 is also formed when aqueous HgCl2 and
reducing sugars: aqueous SnCl2 are mixed in 1:1 molar ratio, orHgOj
jRCHO + 2Cu2+ + 5OH" ■> jRCOO + Cll2O 4- is reduced by Hg.
+ 3H2O HgCl2 + SnCl2 ----- > Hg2Cl2 + SnCl4
• Benedict’s solution contains Cu(II) ion complexed to Excess of SnCl2 would reduce Hg2Cl2 to Hg :
citrate ion. When it is heated with reducing sugar (as Hg2Cl2 + SnCl2 ----- > 2Hg + SnCl4
glucose) Cu(II) is reduced to Cu(I) oxide (CU2O) as in
the Fehling’s test. HgCl2 + Hg Hg2Cl2

This test called Benedict’s test is used to test glucose Properties


in urine sample.
• If treated with aqueous NH3, it turns black due to
Estimation formation ofHg(NH2)Cl Hg

• If excess of KI is added to an acidified solution of Hg2Cl2 + 2NH3----- > HgNH2Cl + Hg + NH4CI


CuSO4(Cu2+ ions), is produced by oxidation of I" black
which reduces Cu2+ to Cui (or Cu^): • In Hg2Cl2 two Hg atoms are bonded together using the
2CuSO4 + 4KI----- > 2CuU +12 6s orbitals.
white ppt
L> + KI KI3 DEEP Focus
• Mercurous ion is diamagnetic; this indicates that there is
produced can be determined by titration with hypo
no unpaired electron, hence, mercurous ion is notHg’ but
solution (Na2S2O3) Hg22+ :
I2 + 2Na2S2O3 ----- > 2NaI + Na2S4O6 Hg2Cl2 + Cl2 2HgCl2
Thus CuSO4 (Cu2+ salts) can be determined Hg2Cl2 A HgCl2+Hg
iodometrically:
2CuSO4 == I2 = 2Na2S2O3 • It dissolves in aqua-regia forming HgCl2
CuSO4 = I == Na2S2O3 Hg2Cl2 + 2C1----- > 2HgCl2
Eq. wt. of CuSO4 = mol. wt. It is used in medicine as purgative.
Eq. wt. of Na2S2O3 = mol. wt. HgCI2 HgCI2 + Hg

aqua-regia Cl2 A
CuO + H2SO4
SnCI2 r*- NaCI
Cu + H2SO4 + Oj Hgci2 *„• Hg2Cl2 •<--------- Hg2(NO3)2
(limited) >-r-
SnCI2 NH3
I2 + Cu2l2 KI KCHO +OH- ' reducing (excess),,
CuSO4 Cu2O|
(white ppt) property of Cu.2+
: HgNH2CI + Hg
red ppt . (as Fehling’s Hg
KCN NH3(aq) Na2S2O3 solution) black

K3[Cu(CN)4] *™CuCN Cu(OH)2


CuS2O3
soluble
(stable)
white ppt
excess NH3(aq) Mercuric Chloride (HgCI2—Corrosive sublimate)
Na2S2O3
[Cu(NH3)4]2+ NaCuS2O3
Preparation
deep blue (soluble)
H2O,A . Hg + Cl2 HgCl2
Cu2S + Na2S2O3 • Hg + 2C1 (aqua regia) —-+ HgCl2
• Hg2Cl2 + Cl2 ----- > 2HgCl2
Mercurous Chloride (Hg2CI2—Calomel) • HgSO4 +2NaCl Mn°2A HgCl2 + Na2SO4
Preparation
Properties
• Hg2Cl2 (calomel) is formed as white precipitate when
soluble chloride (say NaCI) is added to soluble MnO2 prevents formation ofHg2Cl2.
mercurous salt (say Hg2(NO3)2 : • On account of corrosive property, it is callee
corrosive sublimate.
Hg2(NO3)2 + 2NaCl----- > Hg2Cl2 1 + 2NaNO3
• Highly soluble in hot water, ether and alcohol.
Chapter 16: The Transition and Inner Transition Elements | 677

• With aqueous NH3, white ppt of HgNB^Cl is formed : TiOSO., <°nC-H*S0* cone- NaOH
TiO2 (H2O)„
HgCl2 +2NH3 ---- > HgNI^Cl + NH4C1 titanyl sulphate
• SnCl2 reduces it to Hg2Cl2 and then to Hg: Na2TiO3(H2O)n
SnCl2 + 2HgCll22 ---- > Hg2Cl2 I + SnCl4 sodium titanate
white ppt • TiO2 is used as white pigment in paint. It has three
Hg2Cl2 + SnCl2 ---- > 2Hg + SnCl4 advantages over lead paints
• When treated with KI, it forms orange ppt of HgL>, - it covers better
precipitate dissolves in excess of KI forming soluble - it is non-toxic and
I^HgXj (Nessler's reagent): - it is not blackened by H2S
HgCl2 + 2KI -4 Hgljj 1 + 2KC1 • TiO2 is also used for whitening paper and as a filler in
HgL>+2KI 4 KjHg^ (soluble) plastics and rubber.
Radioactive tracer technique indicates that in
| Hgl|* all the iodine atoms are identical. Titanium (IV) Chloride (TiCI4)
• It dissolves AgCl, Preparation
Hgci2 Hgcr + Gi­ • It is prepared by passing Cl2 gas over heated TiO2 and
nger + or (from AgCl) HgCl2 C.
• HgCl2 + 2NaOH---- > HgO + 2NaCl + H^O
TiO2 + Cl2 + 2C TiCl, +2COT
• It is used in medicine as an antiseptic and dressing
skins, furs etc.
Properties
Hg + aqua-regia
• It is colourless, diamagnetic, covalent fuming liquid.
Hg + Cl2 Hg2CI2 + Cl2 • It is hydrolysed vigorously by water, giving fumes in
moist air
aq NH3 TiCl4 + 41^0 ■» Ti(OH)2 + 4HC1
HgNH2CI HgCI2
+ NH4CI KI
SnCI2
^Hgl2f (orange) TiO2
Hg2CI2 + SnCI2 NaOH
KI • If hydrolysed in presence of aqueous HC1, TiOCl2 is
SnCI2 HgO formed
K2Hgl4 (soluble) HC1
Hg Nessler's reagent TiCl4 + H2O TiOCl2 + 2HC1
• It acts as a Lewis acid (electron-pair acceptor) forming
Titanium (IV) oxide octahedral complexes:
Preparation TiCl4 + cone. HC1---- > [TiCl6f-
(very unstable)
Rutile (TiO2) is natural occurring titanium (IV) oxide
HoO [Ti(H2O)6]Cl3
and is invariably coloured by impurities. Pure TiO2 is TiCl4 650°C* TiCl3 —-—>
obtained commercially by chloride process violet [Ti(H2O)5Cl]Cl2
Ti
TiO2 + 2C + Cl2 TiCl4 + 2CO T A | disproportionates
rutile
TiCl4 is volatile and can be separated from impurities. TiCl2 TiCl4 + TiCl2
TiCl4 is heated with O2 at about 1200°C forming TiO2 and
C12.C12 is recycled for further conversion of TiO2 into TiCl4 DEEP Focus
TiCl4 + O2 ---- > TiO2 + 2C12 • Solution of AlEt3 (triethyl aluminium) and TiCl4 in a
hydrocarbon solvent (as cyclohexane) react exothermically
I to form a brown solid called Ziegler-Natta catalyst. This
Cl2 + C catalyst is used for polymerising ethene into polythene.
TiO2 TiCI4
A
, °2
TiO2
Photography
Ordinary black-and-white photographic film consists
Properties of a celluloid strip that has been coated with a gelatin
emulsion containing silver halides, usually AgBr.
• TiO2 is amphoteric and thus dissolves in both bases Photography involves a series of redox reactions (in
and acids stages) as given:
678 | Essential Inorganic Chemistry

Exposure By carefully monitoring the time period allowed for


When exposed to light, the surfaces of the AgBr grains reduction of the AgBr with hydroquinone it is possible to
turn dark because of light-induced redox reaction in which amplify the latent image on the exposed film and make it
Br" transfers an electron to Ag+, producing atoms of visible. At the desired point, further developing is stopped
elemental Ag. (The Br2 produced in the reaction reacts
by transferring the film from the basic hydroquinone
with the gelatin emulsion)
2AgBr(s) hght solution to a weakly acidic one.
* 2Ag(s)+Br2(Z)
Fixing
Developing Once the image is fully formed, the film is fixed by
The exposed film is subjected to developing by adding
washing away the remaining unreacted AgBr with a
an organic reducing agent (probably hydroquinone in
basic medium) when more AgBr is reduced solution of sodium thiosulphate (hypo)
OH AgBr(s)+ 2S2O23-(aq)----- > [Ag(S2O3)2]3'(aq)+ Br‘M
The negative is now ready and can be processed in
2AgBr(s) + 20H (aq) + 2Ag(s) + 2H2O(Z) day-light. Negative is converted to positive by allowing the
light to pass through it on to AgBr containing
photographic paper which is then treated in a similar way
OH
hydroquinone as the negative.
0
+ 2Br7aq)+||^jj

O
quinone

Target Practice 3
1. Name two substances that can reduce AgNO.3 to Ag 7. Name two reagent used for testing Fe'.3+ and the
(a) (b) compounds formed with them
Reagent Compound formed
2. Silver coin contains mainly silver. Pure silver is obtained
by first dissolving it in HNO3 and then other steps I:
Step I: Ag + HNO3 A AgNO3 II:
8. Following tests are performed with aq FeCl3 solution.
• Step II: AgNO. B
What is the observation and conclusion?
A
Step III: B Ag + CO2 + O2 Test Observation Conclusion
Name A (a) NaHCO3 is added '
NameB
(b) Blue litmus is added
3. AgNO3 is a common reagent of testing halides.
(c) Red litmus is added
(a) AgCl is of ppt soluble in
(b) AgBr is of ppt partially soluble in 9. Write reaction of manufacturing NaOH in earlier days by
Lowig process.
(c) Agl is of ppt insoluble in
4. Silver-mirror test is for testing aldehydes reagent used is ....... ..............................................................................
called and contains 10. Name the compound of iron in the +6 oxidation state. How
5. Identify conjugate acid-base pair in the following acid-base is it prepared.
reaction
[Fe(H2O)6l2* + H2O [Fe(H2O)56OHl2+ +H,O+
11. Name three non-stoichiometric oxides of iron
A: B: C: D:
8. It is said that FeCl3 • 6H2O consists of frans-complex. This
trans-complex is 12. Fe2O3 • (H2O)4 (freshly precipitated) is dissolved in acid as
well as in base in two different test tubes. Write the
compounds formed
Chapter 16: The Transition and Inner Transition Elements | 679

(a) in acid 19. Compare stability of


(b) in base (a) [Cu(CN)4l2- CZI [Cu(CN)4]3-
13. Name the compounds with which green vitriol is (b) [Cd(CN)4]2- | | [Cu(CN)4J3-
isomorphous
(I) (ID 20. Specify the temperature for the following changes
A . K o . K
14. FeSO4 is generally used freshly prepared. It is due to CuSO,-5H,0 ----- :---- > CuSO. H„O CuSOx
i 15. FeSO4 or FeSO4(NH4)2SO4 -6H2O is used an volumetric K
CuO
reagent for the estimation of oxidising agent like
(I) (ID 21. Give two chemical tests to identify Cu2* in laboratory
16. Mohr’s salt can be estimated by MnO'/H*, with MnO” as (a)
self-indicator. How does it work? (b)
17. Mohr’s salt can be estimated by Cr2O2-/H*. Name the 22. Calculate moles of Na2S2O3 to react with I2 obtaiend when
indicator used... 0.5 mol ofCuSO4 reacts with excess of KI. Write balanced
equations.
18. Name the contents of
(a) Fehling solution ...
(b) Benedict’s solution
(For more problems refer Target Practice 4)
(c) Fenton’s reagent...

Answers
1. (a) Cu (b) HCHO io. Fe2O3+2NaOH ---- > 2NaFeO2+H2O
2. A: Na2CO3 J3 : Ag2CO3 2NaFeO2 + Cl2 + 4NaOH ---- > 2Na.>FeO4 + 2NaCl + 2H 2
3. (a) White, aqueous NH3 (b) Yellow, aqueous NH3 Net: Fe2O3 + 6NaOH + Cl2 ---- > 2Na,FeO4 + 2NaCl
(c) Yellow, NH3 red purple
4. Tollen’s reagent, ammoniacal AgNO3
+ 2H2 +H2O
5- A: Acid B : Base C: Conjugate base of A
D: Conjugate acid of B Na2FeO4 (a strong oxidising agent like KMnO4) has Fe in +6
6. [Fe(H2O)4Cl2]Cl-2H2O state.
11. FeO, Fe2O3 andFe3O4
Cl 12. (a) [Fe(H2O)6]3+ pale-violet (b) [Fe(OH)6]3"
H2Of; ;iOH2
13. I: ZnSO4 • 7H2O II: MgSO4 • 7H2O
(white vitriol) (epsom salt)
Fe
14. The fact that on exposure to atmosphere it turns brownish-
H2Ok OHo yellow due to the formation of basic ferric sulphate :
Cl 4FeSO4 + 2H,0 + 0,2 ---- > 4Fe(OH)SO4
15. (i) KMnO4 (ii) K2Cr2O7
7. Reagent Compounds formed 16. MnO4 changes to Mn 2 * (very light pink but appears colourless
I. K4[Fe(CN)6] in aqueous solution) when Fe2 * is completely oxidised. One
FeUI[Fen(CN)6r
Purssian blue additional drop of MnO4 imparts its own purple-violet colour
n. KCNS at the end point.
[Fe(SCN)]2* 17. K3[Fe(CN)6] as external indicator. Fe2+solution is tested at
with some
the intermediate stages. K3[Fe(CN)6] gives blue colour with
[Fe(SCN)(H2O)5]2* Fe2 * but when it is completely oxidised there is no blue colour
blood red
Fe2* + [Fe(CN)6]3- ---- > blue
8. Observation Conclusion Fe3*+[Fe(CN)6]3- ---- > no colour
(a) CO 2 is evolved FeCl3 is acidic 18. (a) Mixture of alkaline CuSO4 and sodium potassium
(b) turns red FeCl3 is acidic tartrate.
(c) no effect (b) Cu(II) complexed to citrate ion.,
9. (c) FeSO4 andH2O2
Na2CO3 +Fe2O3 ---- > 2NaFeO2 + CO2
19. (a) (Cu(CN).]2- < [Cu(CN)4]3-
2NaFeO2 +H2O ---- > 2NaOH + Fe2O3 (b) (Cd(CN)2-] < [Cu(CN)J3-
Net: Na2CO3 + H2O ---- > 2NaOH + CO, 20. A : 373 K B : 503 K C: 1023 K
680 | Essential Inorganic Chemistry

21. (a) Add aqueous NH3 in excess. Formation of deep blue I2+KI KI3
coloured complex indicates Cu 2 +
I2+2Na2S2O3 ---- > 2NaI + Na2S4O6
CuSO4 + 4NH3 ---- > [Cu(NH3)4JSO4
deep blue Deep brown colour is due to KIy but when hypo is added, I2
disappears andCu2I2 appears in the form of white precipitate.
(b) Add KI solution. Deep brown colour which change to white 22. Based on chemical reaction in 21(b)
precipitate on addition of hypo (Na2S2O3)
2CuSO4 = 2Na2S2O3
2CuSO4 + 4KI ---- > Cu2I2 +I,+2K2SO4
while ppt Na2S3O3 = 0.5 mol

Practice Exercise 3
1. AgNO3 is soluble in excess of hypo. Give reaction. 12. Complete
2. AgNO3 solution can be used to identify following radicals (a) Fe2O3H2O + NaOCl
by the colour of the ppt obtained : (b) Fe2O3+LiOH (c) Fe2O3 + H2C2O4
(rust)
(a) Cl" (b)Br~ A
(d) Fe2O3+CO A (e) Fe2O3 +H2
(c)T (d)CrO2-
(e)S2- 13. What happens when :
Specify the colour of the precipitate (a) Fe2(SO4)3 reacts with Fe.
3. Unreacted AgBr in photography is removed by hypo used (b) FeCO3 reacts with dilH2SO4.
as ‘fixer1. What is the reaction ? (c) FeSO4 reacts with K2Cr2O7/H+ and then K4[Fe(CN)sl
4. Ag2S ■ aq NaCN > (A) (B) is added.
(d) FeSO4 is strongly heated.
Identify (A) and (B).
14. FeSO4 can be determined volumetrically by titration with
5. Give the scheme of extraction of silver from silver coins.
K2Cr2O7 usingK3[Fe(CN)6] as an external indicator. What
6. How is AgNO3 determined volumetrically? is the end-point ?
7. (a) When H2S gas is passed into FeCl3 solution, yellow 15. 2.78 g ofFeSO4 • xH2O is treated with excess ofBaCl2 when
colour of FeCl3 changes to light green. Explain 2.33 g of white ppt are obtained. Calculate the value ofr.
(b) Same behaviour is also observed when SnCl2 solution (Ba = 137, S = 32, Fe = 56, H = 1, 0 = 16)
is added. Explain. !-
16. Give chemical tests for making distinction between Fe"
8. K3[Fe(CN)6], potassium ferricyanide, is used as external andFe3+ ions.
indicator in the volumetric estimation of Fe2+ salts by 17. Aqueous FeSO4 gives test for Fe2+ and SO2- but after
K2Cr2O7/H* solution How does it indicate end-point? excess of KCN is added, solution does not give test for Fe2'.
9. Aqueous FeCl3 forms acidic solution due to hydrolysis and Explain.
there is evidence of [Fe(H2O)5OH]2+ ion. This forms dimer 18. Copper (II) ion in basic solution is reduced by
at pH 4-5. What is structure of dimer ? formaldehyde, HCHO to copper (I) oxide. Formaldehyde is
oxidised to formate, HCOO". Write the balanced equations
10. Identify (A) to (F) in the following:
for this reaction.
FeSO4 (A) (B) + (C) + (D) 19. You are supposed to be ‘pathologist’ in a medical lab. You
are given a sample of urine of a diabetic patient. How will
(C) and (D) are gases, (B) is red-brown solid, (C) can be
you detect the presence of sugar in it ?
oxidised to (D). (C) also turns Cr2O7“/H+ solution green.
20. A blue coloured mixture is supposed to contain Cd1* ion
(B) dissolves in HC1 forming deep yellow solution (E). (E)
also along with Cu2+ ion. How is the presence of Cd:*
gives blue colour (F) with K4[Fe(CN)6]. (D) dissolves inH2O detected ?
forming a well known mineral acid called “’king of acids'”.
21. Cu2+ ion gives deep violet colour with glycine
11. In earlier days Fe2O3 was used to prepare NaOH in which
CH2(NH2)COOH. Give its equation.
Fe2O3 was recycled. Give scheme of this method.
(For more problem refer Practice Exercise 4)

Answers
1. 2AgNO3 + Na2S2O3 Ag2S2O3 + 2NaNO3 (b) Ag+ + Br" ---- > AgBr fl­
white ppt
yellow ppt. partially soluble in NHj (g?)
Ag2S2O3 + 3Na2S2O3 ---- > 2Na3[Ag(S2O3)2] (c) Ag* +1" ---- > Agl fl­
sodium argento thiosulphate yellow ppt. insoluble in NHj (at?)
(soluble)
2. (a) Ag+ +C1- —> AgCl 1
white ppt. soluble in NH3 (aq)
Chapter 16: The Transition and Inner Transition Elements | 681

(d) 2Ag* + CrO42 Ag2CrO4 X (c) Fe2O3 + 6H2C2O4 2[Fe(C2O4 )3]3-+ 3H2O + 6H*
red ppt water soluble complex

(e) 2Ag* + S2~ Ag2S X (d) Fe2O3 + 3CO 2Fe + 3CO2


black ppt (e) Fe2O3+3H2 2Fe + 3H2O
3. AgBr + 2Na2S2O3 Na3(Ag(S2O3)2] + NaBr
Reaction 12(c) has wide application in removing iron rust
4. Ag2S + 4NaCN 2Na[Ag(CN)2] + Na2S stains (Fe2O3) from fabric. If oxalic acid is used, then Fe(+III)
(A) forms complex [Fe(C2O4 )]3' which can be washed with water.
|zn 13. (a) Fe2(SO4)3+Fe---- > 3FeSO4
Ag + Na2[Zn(CN)4] (b) FeCO3 + H2SO4 ---- > FeSO4 + H2O + CO2
(B)
(c) 6Fe2* + Cr2O2- + 14H* ---- » 6Fe3* + 2Cr3* + 7H2O
5. Silver coin has silver and copper metals (in the form of alloy).
Fe3*+[Fen(CN)6]4- ---- > Feni[Fen(CN)6r
Coin ■HN°3,A) Cu(NO3)2 +AgNO3 Prussian blue

(i) NaCl (d) 2FeSO4 ---- > Fe2O3 + SO3 + SO2

Filtrate Cu 2 * <---- • (ii) Filter 14. Fe2* salts give blue colour with K3[Fein(CN)6l being used as
an external indicator. After Fe2’ has been completely oxidised
K[Ag(CN)2] white ppt (AgCl) to Fe3*, there is no colour change when tested with
| Zn K3(Fe(CN)6].

Ag (also refer Q.4) 15. FeSO4xH2O + BaCl2 ---- > BaSO4


6. AgNO3 can be determined volumetrically by titration using 152+18x 233
NH4SCN (ammonium sulphocyanide) with ferric alum as 2.78 2.33
indicator: => x=7
Ag* + SCN" AgSCN X
16. Reagent Fe2* Fe3*
white ppt.
When precipitation is complete, additional drop ofSCN- gives NaOH green ppt red-brown ppt
red colour with Fe3* of ferric alum: K4[Fe(CN)6] white ppt turning blue Prussian blue ppt
rapidly
Fe3* + SCN- ---- » (Fe(SCN)]2*
red colour K3[Fe(CN)6] Turnbull's blue ppt red brown colour (no
KSCN no colour ppt.)
7. In both cases Fe2 * ions are formed due to reduction. Hence,
deep red colour
solution appears light green:
(a) 2FeCl3 + H2S * 2FeCl2 + 2HC1 + S 17. It is due to formation of the complex K4[Fe(CN)6J in which
(b) 2FeCl3 + SnCl2 ---- > 2FeCl2 + SnCl4 CN" has been coordinated toFe2’
8. Fe2++ [Fera(CN)6]3“ ---- > blue colour FeSO4 +6KCN---- » K2SO4 +K4(Fe(CN)6]
Fe3* + [FeIU(CN)6]3- ---- > no blue colour 18. 2Cu 2* + 5OH' + HCHO HCOO~ + Cu2O X + 3H2O
Initially when Fe2* is present, indicator K3[Fe(CN)6] gives 19. Urine sample of the diabetic patient is supposed to contain
blue colour. After Fe2* is completely oxidised to Fe3 *, there is glucose. Hence, when it is heated with Fehling's solution, (see
no colour with indicator. text) a brick red ppt. appears.
hydrolysis A
2Cu2*+5OH’ + RCHO 7?COO’+Cu2O1 +3H2O
9. [Fe(H2O)6]I3: * + h2o - [Fe(H2O)5OH]2* + H3O* (in urine sample) brick red
OH 20. If KCN in excess is added to the mixture, Cu 2 * forms a stable
2[Fe(H2O)6OH]2* >Fe(H2O)5 complex [Cu^CN^]3- while Cd2* forms unstable complex
(H2O)5Fe
[Cd(CN)4]2-. This complex ionises :
\)HZ
dimer [Cd(CN)4]2- Cd2*+4CN"
10. (A):FeSO4 (B):Fe2O3 (C):SO2 (Z)):SO3 CE):FeCl3 When H2S gas is passed, yellow ppt. of CdS appears if Cd2 ‘ is
(F) :KFeIn[Fen(CN)6], Prussian blue present.
11. —> Fe2O3 + Na2CO3 -- > 2NaFeO2 + CO2 21. Cu2* + 2CH2(NH2)COOH---- > [Cu{CH2(NH2)COO’}2]
2NaFeO2 + H2O---- > 2NaOH + Fe2O3 + 2H*
(Lowig method) i CH2NH2 NH2-CH2
---------------------------- recycled---------------- 1
or I u
12. (a) Fe2O3 H2O + 4NaOCl ■ 2Na2FeIVO4 + 2HC1 + Cl2 0=C-0 0-C=0
A
(b) Fe2O3 + 2LiOH 2LiFeO2 + H2O
’■JE.'W.'.'V-Jr
682 | Essential Inorganic Chemistry

Inner-Transition Elements (f-Block) orbital exactly balances the additional unit of nuclear
charge, and thus effective nuclear charge (Zef)
As we have specified earlier, the elements with a remains unchanged. Hence, it would be predicted that
partially filled /-subshell in common oxidation states are the atomic radii should also remain constant
known as /-block transition elements or Table 16.10. shows that there is slight decrease in the
inner-transition elements. They are the two rows value of atomic radius along the series (with exception
elements at the bottom of the Periodic Table. (4/ and that of Eu and Yb. These two mecals crystallise with
5/-series). cubic lattices and attain oxidation state of +2. It is to
• Fourteen elements of the first row (4/-series) from be noted that the 4/-orbitals are half-filled (if1) and
cerium (Z = 58) to lutetium (Z=71) which follow completely filled (4/14).
lanthanum (Z = 57) in the Periodic Table are called 119 o La3*
Lanthanides or Lanthanoids or Lanthanones. U3*
These elements are characterised by successive filling
of 4/-orbitals in their atoms. Lanthanides along with to NP3+
k Ce3* X. Pu3*
yttrium (Z = 39) and lanthanum (Z = 57) were
originally called as rare earths since their oxides 114 - to Pr3* b Am3*
(earths) were rare.
Nd3* to Cm3*
Fourteen elements of the second row (5/-series) from E
Q.
thorium (Z = 90) to lawrencium (Z = 103) following \ Pm3*
W
actinium are known as Actinides or Actinoids or X. Sm3* *h Bk3*
•\> Cr3*
Actinones. These are characterised by successive filling S 109 - V Eu3*
o
of 5/-orbital. Actinides from Z = 93 (after uranium Z = 92) o Gd3*
have been produced artificially and are known as \jb3+
transuranium elements. y3+
XgDy3*
104 - 0
H°3+Er3*
The Lanthanoids
90 - X^Tm3*
Electronic Configuration
• Shells n,(n -1) and(n - 2) are incomplete and(n - 2)f, ©
Sc3* Yb3tlu*
X\
88 -
i.e., 4/'-orbital is being successively filled.
1 J___ [_ i i i i i i i i i i i

• Lanthanum (Z = 57) has electronic configuration 1 2 3 4 5 6 7 8 9 10 11 12 13 14


[XelSd^s2. Fig. 16.7 Ionic radii (CN = 6) of Sc3*, Y3+,La3+,Ln3t and
• There is an electron in 5c?-orbital only in Ce(Z =58) An3* ions
Gd(Z = 64) and in Lu(Z = 71). This is just to explain
extra stability of half-filled or completely filled When the trivalent ion Ln3+ is formed, then screening
4 /-orbitals. power of 4/-electron is no longer equal to unity (but 0.85
• Lanthanides mostly from ionic compounds in +3 since 4/ is now [(n - 1), penultimate shell], and each time
oxidation state, the electronic configuration of M3+ an electron is added to 4/-orbitals, the effective nuclear
ions from Z = 58 to Z = 71 varies in a regular manner charge (Zeff) is increased by 0.15 and thus ionic size
/
from [XeH/1 to [Xe]4/14. Electronic configuration 1 )
of Ln3+ decreases continuously from Ce3’ to
have been summarised in Table 16.10. \
Atomic and Ionic Sizes: Lu3+.
(The Lanthanide Contraction)
DEEP Focus
The observed values of the atomic radii and ionic radii • As a result of lanthanide contraction, the normal increase
of the trivalent lanthanide ion (Ln3+) have been in size from Sc—> Y—> La disappears after the lanthanide
summarised in Table 16.10. Like the ions of the transition and the pairs of elements such as Zr and Hf, Nb and Ta,
elements, the ionic radii show a progressive decrease with Mo and W, etc. have nearly same size. The properties of
increasing atomic number. The decrease in size ofLn3+ ion these elements are very similar.
in 4/-series is known as lanthanide contraction. • The similarities in properties within these pairs make their
(Fig. 16.7) separation very difficult.
• In lanthanides, electrons in 4/-orbitals are being • Due to lanthanide contraction, the elements of 5d and
filled progressively. Since addition of an extra electron 4d-transition series resemble each other much more closely
in if (screening constant of (n - 2)f electron = 1) than do the elements of 4d and 3d series.
Chapter 16: The Transition and Inner Transition Elements | 683

Table 16.10 Electronic Configuration and Other Physical Constants of Lanthanum and Lanthanides
Electronic configuration outside [Xe] Core Radlus/pm
Atomic Oxidation E°IV /1 + /a + /3
Name Symbol
number state M3*/M kJ mor1
Ln Ln2* Ln3* Ln4* Ln Ln3*
Lanthanum 57 La III 5d’6s2 5b1 ~4f° 187 106 -2.52 3455
Cerium 58 Ce III, IV 4f15d’6s2 4f2 4f' 4f° 183 103 -2.48 3524
Praseodymium 59 Pr III (IV) 4f36s2 4f3 4f2 4f' 182 101 -2.46 3631

Neodymium 60 Nd III 4f46s2 4f* 4f3 4f2 181 100 -2.43 3692
Promethium 61 Pm III 4f56s2 4fs 4f* 181 98 -2.42 3728
Samarium 62 Sm 111(H) 4f66s2 4f6 4fs 179 96 -2.41 3871

Europium 63 Eu lll(ll) 4f76s2 4f7 4f6 204 95 -2.41 4057


Gadolinium 64 Gd III 4f75d16s2 4f75d1 4f7 180 94 -2.40 3763

Terbium 65 Tb lll(IV) 4f96s2 4f9 4f8 4f7 178 92 -2.39 3798


Dysprosium 66 Dy lll(IV) 4/106s2 4fi° 4f9 4f* 177 91 -2.35 3918
Holmium 67 Ho III 4f”6s2 4f” 4f’° 176 89 -2.32 3920
Erbium 68 Er III 4f126s2 4f12 4/11 175 88 -2.30 3930
Thulium 69 Tm lll(ll) 4/136s2 4f3 4f12 174 87 -2.28 4044
Ytterbium 70 Yb lll(ll) 4f146s2 4f14 4f13 194 86 -2.27 4193
| Lutetium 71 Lu III 4r145d16s2 4f145d1 4f14 174 85 -2.26 3887

Oxidation States Colour of Ions


The sum of the first three ionisation energies of the Ions of lanthanides are coloured in the solid state as
lanthanides is comparatively low, so the elements are well as in aqueous solution (except La34-, Ce3+, Gd3+, Yb3+
highly electropositive. They readily form Ln3+ ions by and Lu3+). Colour of these ions arise due to electronic
losing both the 6s and one5d-electrons. In the absence of5d transition in 4/'-orbitals(f - f*). Colours of Ln34 ions have
electron, one of the electrons present in the (n-2)/ been summarised in Table 16.11.
orbitals is lost. Besides the +3 state, some of the
lanthanides show other oxidation states. Following stable Magnetic Properties
ions also exist as:
Paramagnetism is associated with the presence of
4f° La3+
unpaired electrons in a substance. Ln 3+ ions other than f°
V7 Eu2+,Gd3+ ,Tb4+
type (La3+ and Ce4+) and the f14 type (Yb2+ and Lu3+) are
4f14 Yb2+,Lu3+
Stability of these ions depend on all paramagnetic. The values of magnetic moment have
— ionisation energy been given in Table 16.11
— sublimation energy
— lattice energy.
684 | Essential Inorganic Chemistry

Table 16.11 The Colours and Observed and Calculated Magnetic Moment of La3+ and Ln3+ Ions
Magnetic moment of N unpaired
electrons/p BM
Colour Electronic N
Ion Calculated
(aquo complex) configuration (unpaired electrons)
(only total spin Observed
JN(N + 2)
La3* colourless 4/° 0 0.0 0
Ce3* colourless 4f1 1 1.73 2.3- 2.5
Pr3* green 4f2 2 2.83 3.4- 3.6
Nd3* lilac 4f3 3 3.87 3.5- 3.6
Pm3* yellow 4/4 4 4.90 2.70
Sm3* .5
yellow 5 5.92 1.5- 1.6
Eu3* pale pink 4f6 6 6.93 3.4- 3.6
Gd3* colourless 4f7 7 7.94 7.8-8.0
Tb3* pale pink 4f8 6 6.93 9.4- 9.6
Dy3* yellow 4f9 5 5.92 10.4-10.5
Ho3* pale yellow 4/’° 4 4.90 10.3-10.5
Er3* rose-red (pink) 4/11 3 3.87 9.4- 96
Tm3* pale-green 4/12 2 2.83 7.1-7.4
Yb3* colourless 4f13 1 1.73 4.4- 4.9
Lu3* colourless 4/14 0 0.00 0

Colours ofLn4+,Ln2+ ions are given below • The hydroxides are ionic and basic. They are less bask
Ce4+ —orange-red 4f° isoelelctronic ofLa3+ than Ca(OH)2 but more basic than amphoteric
Sm 2 + — blood-red 4 f 6 isoelectronic ofEu3+ A1(OH)3. The base strength decreases from
Eu2+ — pale greenish yellow 4f7 isoelectronic ofGd3+ Ce(OH)3 to Lu(OH)3.
Yb2+ — yellow 4f14 isoelectronic ofLu3+ • Non-availability of d-electrons for K-bonding does not
favour the formation of carbonyls by most
Chemical Reactivity lanthanides. However, an unstable Nd(CO)6 has been
prepared.
• The lanthanide elements (as well as La and Y) are all
• Since E° for Ln3+ + 3e~ ---- > Ln is negative, they
silvery white, soft and malleable with high electrical
conductivity. Eu and Yb are pale yellow. react directly with water, rapidly on heating, to
liberate H2 Reactions with acids are more vigorous,
• They are highly electropositive and reactive. All of but the metals do not dissolve in alkalies. They are
them burn in air to form oxides Ln2O3 and nitrides also stable in HF and H3PO4 owing to the formation of
LnN (cerium gives CeC^). Powdered cerium is even a protective film of insoluble salts.
pyrophoric. Yttrium usually forms a protective oxide
coating on it and remains unattacked by air even at DEEP Focus
1273 K. Europium and ytterbium dissolve in liquid ammonia at 195
• The hydrides, formed in stable in H2 and LnH3 phases, K forming blue solutions which give spectra characteristic
usually have a defect lattice and are of Af2* and ammoniated electrons. The solution
decolourises slowly but the concentrated solution appear
non-stoichiometric. They react with H2O golden.
liberating H2 gas.
• An alloy of lanthanide metal is mischmetal. It has
LnH3 + 3H2O---- > Ln(OH)3 + 3H2 lanthanide metal (95%) and iron about 5% with traces
The hydrides are remarkable stable to heat up to of S, C, Ca and Al and is used in the production of
1200 K bullets, shells and lighter flint. Mixed oxides of
• They form trihalides with halogens but Ce and Tb give lanthanides are used as catalysts in petroleum
CeF4 and TbF4 respectively. Complex fluorides of cracking. Oxides are also used as phosphorus in
Pr(IV), Nd(IV) and Dy(IV) are also formed. television screens.
Chapter 16: The Transition and Inner Transition Elements | 685

Tso = 23.5 min


LnCz
£9u ---------------- > 239kt« + -l
93 Np °„e

Ln(CO3)3 C, 2500°C 239 TT T50 = 2.3 day


L^Oa (Ce02) 93 u --------------- > 29349Pu + ?e
\O,
n2.a • At present, transuranium elements with atomic
LnX3 Ln LnN number upto 110 have been reported. Recently, on
August 16, 2003, IUPAC approved the name for the
LnH2, LnH3 element with atomic number 110 as
Ln(OH)3 + H2
7 S, A DARMSTADTIUM (Ds)
Ln3* +H2 The IUPAC proposed a system for naming elements
10283
with Z > 100.
Digits Name Abbrevatlon
The Actinoids 0 nil n
The actinoids are the fourteen elements from Th to Lr 1 un u
following actinium in the seventh period of the Periodic 2 bi b
Table. They are radioactive, and therefore of the greatest 3 tri t
importance in the study of nuclear chemistry and of the 4 quad q
principles which underline the conversion of mass into 5 pent P
energy. 6 hex h
7 sept s
Electronic Configuration 8 oct 0

Three shells (n, (n - 1) and (n - 2)) are incompletely 9 enn e

filled and differentiating electron enters into 5/'-orbital


[Le., (n - 2)/]. Electronic configurations (EC) of actinoids Table 16.12 IUPAC Nomenclature of Elements with Atomic
and their ions have been given in Table 17.13. The Number above 100
variation in the (EC) of the actinoids like those in Atomic IUPAC official IUPAC
lanthanoids is due to extra stabilities of f°, f1 and f14 EC Name Symbol
number name symbol
of the 5f-orbitals. Thus, EC of Am and Cm are with 5/"7
101 Un-nil-unnium Unu Mendelevium Md
arrangements.
102 Un-nil-bium Unb Nobelium No
Am(Z = 95) : [Rn]5/-77s2
103 Un-nil-trium Unt Lawrencium Lr
Cm(Z = 96) : [Rn]5f76dl7s2 Un-nil-quadium Unq Rutherfordium Rf
104
Similarly Th(Z = 90) has 5fQ and No (Z = 102) and 105 Un-nil-pentium Unp Dubnium Db
LhZ = 103) have 5f14 arrangements 106 Un-nil-hexium Unh Seaborgium Sg
Th(Z = 90) [Rn]5/'°6d27s2 107 Un-nil-septium Uns Bohrium Bh
No(Z = 102) [Rn]5/147s2 108 Un-nil-octium Uno Hassnium Hs
LrfZ = 103) [RnJS/^GdW 109 Un-nil-ennium Une Meitnerium Mt
110 Un-un-nillium Unn Darmstaditium' Ds
DEEP Focus 111 Un-un-unnium Uuu
The allocation of electrons to the 5f or 6d-orbitals in the 112 Un-un-bium Uub
actinide atoms is more uncertain than the allocation to the 4f or
5d-orbitals in the lanthanide atoms, because the external 113 Un-un-trium Uut

shielding ofthe5f-electrons is less than that of the ^-electrons. 114 Un-un-quadium Uuq
115 Un-un-pentium Uup

Transuranic Elements 116 Un-un-hexium Uuh

Elements with atomic numbers greater than 92 117 Un-un-septium Uus


(atomic number of uranium) are called the transuranium 118 Un-un-octium Uuo
elements. All these elements are man-made, i.e., they are
produced in the laboratory through artificial nuclear •Elements yet to be discovered,
reactions. **IUPAC names yet to be announced.
***IUPAC approved the name on August 16, 2003.
29328U+U +Y
686 | Essential Inorganic Chemistry

Atomic and Ionic Sizes • The maximum oxidation state increases from +4 in Th
to +5 in Pa, +6 in U and +7 in Np and Pu, an then
The radii of the trivalent (Af3*) and quadrivalent
decreases in succeeding elements.
(M4*) ions have been given in Table 16.13. The ionic radii
for the M 3+ and Af4* ions contract slightly with increasing • The actinoids resemble the lanthanoids in having
more compounds in +3 state than in +4 state but they
atomic number. There is thus an actinide contraction
corresponding to the lanthanide contraction. (Fig. 16.7). tend to hydrolyse.
There are two observed differences between the two series • U3* in aqueous solution evolves hydrogen on standing;
of ions: aqueous solutions of Np3* and Pu3+ are stable, but are
(1) although the actinide contraction initially easily oxidised to Np4+ and Pu4*. All the other
parallels that of the lanthanides, the elements from
trivalent ions up to No3+ are stable.
curium on are smaller than might be expected, probably
resulting from poorer shielding by 5/’-electrons in these • U4* and Np4+ are stable to water, but are slowly
elements; oxidised by air to UC^* and NpC^+.
(2) the lanthanide curve consists of two very shallow
arcs with a discontinuity at the spherically symmetrical DEEP Focus
Gd3* (4/“7) ion. A similar discontinuity is not clearly seen • Pu4* disproportionates to Pu3+ 3* and PuO2+ when the
at Cm3*. solution is strongly acidic
Pu4* ---- > Pu3* + PuO2*
+4 +3 +6
Oxidation States I reduction I
The actinoids in general show +3 oxidation state. oxidation
There is also a greater range of oxidation states due to
comparable energies of 5f, 6d and 7s levels, and have been UOj disproportionates to U4* and UO2* .
shown in Table 16.14. UOj* is stable and difficult to reduce.

Table 16.13 Some Properties of Actinium and Actinoides


Atomic Electronic configuration outside [Rn] core Ionic radlus/Pm
Name Symbol
number An An4*
An3* An An3*
89 Actinium Ac 6d'7s2 5f° 111
90 Thorium Th 6d27s2 5f1 5f° 99
91 Protactinium Pa 5f26d'7s2 5f2 5f' 96
92 Uranium U 5f36d'7s2 5f3 5f2 103 93
93 Neptunium Np 5f46d'7s2 5f4 5f3 101 92
94 Plutonium Pu 5f&7s2 5fs 5f4 100 90
95 Americium Am 5f77s2 5f6 5f* 99 89
96 Curium Cm 5f76d'7s2 5f7 5f6 99 88
97 Berkelium Bk 5f97s2 5f& 5f7 98 87
98 Californium Cf 5f'°7s2 5f9 5f6 98 86
99 Einsteinium Es 5f117s2 5f'° 5f9
100 Fermium Fm 5f127s2 5f11 5fw
101 Mendelevium Md 5f137s2 5f'2 5f11
102 Nobelium No 5f147s2 5f'3 5f'2
103 Lawrencium Lr 5f'46d'7s2 5f14 5f13
110 Darmstadium Ds 5f'46dQ7s2 5f'4Gd7 5f'46d6

Table 16.14 Oxidation States of Actinium and Actinoids


Ac Th Pa U Np Pu Am Cm Bk Cf Es Fm Md No Lr Di
3 3 3 3 3 3 3 3 3 3 •3 3 3 3 3
4 4 4 4 4 4 4 4
5 5 5 5 5
6 6 6 6
7 7
Chapter 16 . The Transition and Inner Transition Elements | 687

Magnetic Properties Physical and Chemical Properties


The magnetic properties of the actinoids is again • The actinoids metals are all silvery in appearance
dependent on unpaired electrons (2V) and are given by with a variety of structure.
p. (magnetic moment) = ^N(N + 2) BM • The dominant feature of the actinoids is their nuclear
but observed values are more complex than those of instability as they have tendency to undergo
lanthanoids and have been summarised in Table 16.15. spontaneous fission and decay becomes more
pronounced with the heavier elements. The
Table 16.15 Magnetic Moment of Actinoids_________ radioactivity of Th and U is probably responsible for
much of the earth’s internal heat.
Element p = JN(N + 2) BM (only total
Z N • The actinoids metals are more electropositive and are
spin)
reactive, reactivity increase with increase in atomic
Th 90 2 2.83
number.
Pa 91 3 3.87
• They tarnish rapidly in air, forming an oxide coating
U 92 4 4.90 which is protective in case of Th but less for the other
Np 93 5 5.92 elements.
Pu 94 6 6.93 • The metals react with most non-metals especially on
Am 95 7 7.94 heating, but resist alkali attack and are less reactive
Cm 96 8 8.94 towards acids than might be expected. Concentrated
Bk 97 5 5.92 HC1 react rapidly but concentrated HN03 makes Th,
Cf 98 4 4.90 U and Pu passive, which can be prevented in presence
Es 99 3 3.87 ofF".
Fm 100 2 2.83 • With boiling water or steam, H2 is obtained and metal
Md
is converted into oxide.
101 1 1.73
No 102 0.00
• Metals react rapidly with H2 forming hydrides which
0
are decomposed by H2O.
Lr 103 1 1.73
h20 UO2
H2 + U(OH)4 uh4
h2
Ionisation Energy O2
• Ionisation energy of the early actinoids are smaller
H2O Cone. HNQ3[» passjve
than those of early lanthanoids. When electrons are UO2 + H2 An
Steam to oxide layer
being filled into 5/’-orbitals they will penetrate less
Th
into the inner core of the electrons and thus these
90
occupied 5/'-electrons will be more effectively shielded
-a
from the nuclear charge than are the 4/‘-electrons of X UCI4, UCI6
the corresponding lanthanoids. 88

Target Practice 4
Note: This also includes problems from d-block elements. (b) He took two colourless solutions. On adding one into
1. Write formulae of another dropwise, orange precipitate appeared which
(a) Calomel...................................................................... dissolved on adding excess of that. It was explained as
(b) Corrosive sublimate.................................................
(c) Nessler’s reagent........................................................ (c) He placed some white substance with 10 paise coin on
(d) Iodide of Millon’s base.............................................. the palm of a child and asked to close it tightly. After
2. Mercury (I) chloride is written as Hg2Cl2 and not as HgCl sometime, palm got hot and child cried. It was due to
because .........................................................................

3. A magician showed following performances in a magic


show. But you immediately disclosed their secrets by 4. TiO, can be converted into TiCl4 and vice-versa. Explain
chemical reactions. based on reactions
(a) He rubbed a white substance on a copper coin which
was covered with a silvery layer. It is due to
688 | Essential Inorganic Chemistry

5. What are the contents of the Ziegler-Natta catalyst? 9. Lanthanides does not form carbonyl because
What happens when europium or ytterbium is dissolved in
6. Select species which are amphoteric or amphiprotic? liquid ammonia?

Species Amphoteric Amphiprotic


I TiO2 10. ■ Write maximum oxidation states of the following
II h2o (a) U (b) Np (c) Bk
4+ disproportionates. Write
III AI2O3 11. (a) In acidic solution Pu
chemical reation
IV ZnO
Pu4+ +
V HSO4 Oxidation
VI H2PO2 L
(b) U02 disproportionates
T
Reduction
__ J
to......... ..... and...
VII H2PO3
VIII HPOf 12. Balance following nuclear reactions
(a) 2|® U -----> A + 4He
7. What are the contents of mischmetal?
(b) 238 U -----> 2°6Pb + B + C
8. Mention ion of following electronic configuration in 13. Write IUPAC symbol for the elements with atomic number
valence shell (a) 100 (d) 112
(a) 4f° (b) 101 (e) 114
(b) 4f7 (c) 110
(0 4f14.........................................................................

Answers
1. (a) Hg2Cl2
(b) HgCl2 6. Species Amphoteric Amphiprotic
(0 K^gl, I TiO2 X
Hg II H2O . x ✓
(d) nh2 0 I III AI2O3 ✓ X
Hg IV ZnO X
2. Mercurous ions is diamagnetic indicating the absence of any V HSO4 ✓ ✓
unpaired electrons. Hence, mercurous ion is notHg * butHgf *. VI H2POi X X
3. (a) Formation of Hg layer by reduction of HgCl2 to Hg by VII H2PO3 ✓ ✓
copper. VIII HPOr ✓ ✓
HgCl2 + Cu CuCl2 + Hg
silvery 7. 95% lanthanide, 5% iron with traces of sulphur, carbon,
calcium and aluminium.
(b) HgCl2 + 2KI ---- > Hgl2 + 2KC1 8. (a)4f°:La3 +
orange (b) 4/7 :Eu2+,Gd3*, Tb
(c) Exothermic reaction (c) 4f14 :Yb2*, Lu2*
3HgCl2 + 2A1 2A1C13 + 3Hg + Heat 9. of non-availability of ^-electrons for rt-bonding.
(coin) 10. (a) U = + 6 (b) Np = + 7 (c) Bk = + 4
1173 K 11. (a) Pu4* ---- PuO|*+ Pu3*
--- > > PuO|*
4. TiO2 + 2C12 + 2C ------- » TiCl4 + 2CO T +4 +6+3
+6

TiCl4 + 4H2O ---- > Ti(0H)4 + 4HC1 TiO2 I oxid


5. Triethyl aluminium and titanium (IV) chloride. red
(b) U4* and U02 +
.. 12. (a) A: |o4Th (b) B: 8a C:6p
13. (a) Unn (b) Unu (c) Uun (d) Uub (e) Uuq
Chapter 16: The Transition and Inner Transition Elements | 689

Practice Exercise 4
1- Mercurous ion has zero magnetic moment. It is 8. TiCl4 can be converted to pure Ti by reduction with Na or
represented as Hg+, Hg2* or Hg2*, explain. Mg. Write chemical equation.
2. What is van’t Hoff factor of Hg2 (N0)2 if it is 100% ionised 9. Explain lanthanide contraction
in aqueous solution? 10. Write EC of (a) Gd, (b)Gd3* (c) Lu3*
3. Give the scheme of separation of mixture of insoluble AgCI, Also determine magnetic moment (assuming total spin
Hg2Cl2 andPbCl2. only).
4. When KJlgl* reacts with NH3, brown ppt is formed. 11. Compare base strength ofLn(OH)3, Ca(OH)2 and A1(OH)3
Explain the formation of brown ppt 12. Compare lanthanoids and actinoids in terms of:
5. Make distinction between Hg2* and Hg:2: * salts. (a) EC (b) ionic size
6. (a) What happens when a solution ofHgCl2 is boiled with a (c) radioactivity (d) oxidation state
solution of HgO? 13. Complete the following reactions:
(b) What happens when HgCl2 is dissolved in cone. HCI?
(a) Pu4* (b) UO2* H+
7- Ilmenite (FeTiO3) can also be converted into TiCl4 by
H2O/O2
heating with carbon and chlorine. Write a balanced (0 u4+ ->
equation for this reaction.

Answers
1. Hg2 * has no electron unpaired hence, its magnetic moment is
zero, but it is called mercuric [Hg(II)] ion. Hg+ has one 4. 2K2HgI4+NH3 + 3Na0H 0/ \nh 2 I +4KI
XHg/
unpaired electron, it has magnetic moment of V3 BM, but
mercurous ion has zero magnetic moment, it is only possible if (iodide of Millon’s base)
brown ppt.
it has all paired electrons. Thus it is Hg22*.
+ 3NaI + 2H2O
2. Hg2(NO3)2 Hgl+ +2NO3-
Total ions = y = 3 5. Reagent Hgr___________ Hg2*
i (van't Hoff factor) = (1 + (y - 1 )x] • KI green ppt. which orange ppt. which
= 1 + 2x (x = degree of ionisation - 1) dissolves in excess of dissolves in excess of
KI KI (K2Hgl4)
=1+2=3 • HCI white ppt (Hg2CI2) no ppt.
3. (a) PbCLl2 is soluble in hot water; Hg2Cl2 and AgCI are • H2Sgas no ppt. black ppt (HgS).
insoluble in hot water.
(b) AgCI is soluble in aq NH3. 6. (a) Mercury oxychlorides are formed:
(c) Hg2Cl2 is blackened by aq NH3 2HgCl2HgO (red)
PbCI2 + AgCI + Hg2CI2 HgCl2-2HgO (black)
HgCl2 -3HgO (yellow)
hot water, filter
HgCl2 • 4HgO (black)
white ppt. (b) H2HgCl4 is formed which crystallises on cooling.
AgCI + Hg2CI2 7. 2FeTiO3 + 6C + 7C12 ---- > 2TiCl4 + 2FeCl3 + 6C0
8. TiCl4 + 2Mg---- > Ti + 2MgCl2
PbCI2 in hot water aq NH31 filter
TiCl4 + 4Na---- > Ti + 4NaCl

r
[Ag(NH )2]CI
3 (HgNH2CI + Hg)
9. Refer text
11. Refer text
10. Refer text
12. Refer text
(soluble) (black) 13. Refer text
Total Practice Set
(Read, Plan & Solved)

Hence, in Cr O | (O2)2| <- four oxygen of peroxide.


Problem 1. In an atmosphere with industrial smog,
Cu corrodes to a basic sulphate Cu2(OH)2SO4 and basic
carbonate Cu/OH^CC^. Propose a series of chemical
t tt
x-2-4=0
reactions to describe this corrosion.
x=+6
Solution 2Cu + H2O + SO3 + O2 ---- > Cu2(OH)2SO4
O 0
2Cu + HjO + CO2 + O2 ---- > Cu2(OH)2CO3 I
0 0
Fortunately this corrosion product forms a tough
adherent coating that protects the underlying
Problem 4. Both Ci2O72"and Mn04 solutions can be
metal. Overall reaction is a combination of
oxidation-reduction, acid-base and precipitation used to titrate Fe2+ in acidic medium.
reaction. Suppose you have available 0.1 M solutions of each. For
a given sample of Fe2* solution, if a given titration
Problem 2. When Zn is added to an acidified solution requires 24.5 mL of 0.1M C^O^- solution, how many mL of
0.1 M MnO4 solution would have been required if it had
of K^O,, the colour of the solution changes from orange
been used instead ?
to green, then to blue and over a period of time, back to
green. Write equations for this series of reactions. Solution CijjO2- + 14H* +6Fe2+ + 2Cr3*+6Fe5*
+12 +2 +6 +3
Solution Zn reduces CigOf -(orange)/H* to Cr3+ (green)
+ 7H.0
and then to Cr2* (blue), but atmospheric O2 oxides Cr2*
(blue) back to Cr3* (green): Cij>02~==6e ]gain of 6e by two Cr atoms.
Zn---- > Zn2* + 2e~ .-. 0.1M CijA2- = 0.6 N C^O72-
Cr2O^“ + 14H* + 6e~ ■> 2Cr3* + 7H2O MnO; + 8H* +5Fe2* + Mn2* +5Fe3*+4H20
+7 +2 +2 +3
Cr3* + e~ + Cr2*
MnO4 = 5e“
4Cr2+ +02 + 4H* + 4Cr3* + 21^0
(ak) 0.1 M MnO4 = 0.5 N MnO4
Fe2* = CijjO^B MnO4"
Problem 3. In qualitative analysis, a confirmatory 24.5 mL of 0.1 M C^O?’ = V mL of 0.1 M MnO;
test for chromium involves the formation of a blue
transient peroxo species, CrO5, which is better 24.5 mLx 0.6 N C^O^b V mL of 0.5 N MnO;
represented as CrO(O2)2. It is formed when is added T7 24.5 x 0.6 o_ . T
V =------------ = 29.4 mL
to on acidic solution containing Cr>07 “. What is oxidation 5
state of Cr is CrO(O)2? Write the equation of formation of
CrO5. Problem 5. Nickel can be determined by the
precipitation of nickel dimethyl glyoximate. What is the
Solution Cr2O2"+2H* + 41^02 > 2CrO(O2)2 + SHp reaction?
Oxidation state of oxygen in peroxide = - 1 Solution (a) Ni2+ salts (in basic medium) on reaction
Oxidation state of oxygen in oxide = - 2 with dimethyl glyoxime give cherry red ppt of nickel
dimethyl glyoximate :
Chapter 16 . The Transition and Inner Transition Elements | 691

H33'C—C=NOH The extreme toxicity of cyanides is due to CN~


NiCl2 + 2 i +2NHl44 OH complexing with metals in cytochrome-enzymes and
H3C—C=NOH haemoglobin in the body, thus preventing normal
{in alcohol) metabolism.
O— H 0 (b) 2Ag++S20g-- 2Ag2* + 2SO2-
t Ag2* + 4Py [AgfPy^f*
h3cXc^n N^C ch3 red
Ni I + 2NH4CI + 2H2O Problem 8. Explain the terms:
h3c^
.C, ,c. (a) Rusting, (b) Passivity.
W N ch3
I Solution (a) Rusting : When exposed to air and
0........ H—0 moisture, iron gets rapidly oxidised to a hydrated oxide,
(cherry red ppt.) Fe2O3 • xH^O, known as rust of iron. Actual composition is
not known, but it consists mainly of 2Fe2O3 • SHjO
Problem 6. (a) When Mn(OH)2 is made by adding an together with a small quantity of FeCO3.
alkali to a solution containing Mn2+ ions, the precipitate Rusting is an electrochemical phenomenon. When
quickly darkens, and eventually goes black. What might impure iron comes in contact with water containing CO2,
be the chemical giving the black colour, and how is it an internal voltaic cell is set up. The iron and the other
made? metals present as impurities constitute the electrode
(b) Dimercury (I) iodide, Hg^ is a greenish colour and while aerated water (having CO2) acts as an electrolyte. As
iron is more electropositive than many other metals, it
is precipitated if iodide ions are added to a solution of
dimercury (I) sulphate, Hg2SO4. Likewise the red mercury passes into solution as iron (II) ion more readily than the
(II) iodide, Hglj, is precipitated from a solution of mercury rest of the metals.
(II) sulphate, HgSO4. However, both precipitates dissolve Anode (Fe) Cathode
in excess iodide solution. What might be the reason for ; (Impurities/Noble metals)
this? Fe(-^U Fe!:2++2e" ! HjO + COa H2CO3+H+
Solution (a) The black colour is due to the manganese (Fe dissolves) + hco33
(IV) oxide, MnO2. It is made by the Mn(OH)2 being oxidised
by oxygen in the air: Fe2* -%FeO OH ! 2H+ + 2e" ■> Hj
H2O !
Mn(OH)2 ---- > MnO + HgO Fe2O3 I Fe(OH)2 + CO2 — FeCO3 + H9O
MnO +- O2 ---- > MnO2 Fe3O4 ! Fe2(CO3)3 + H2O -> F^Ojj • H2O
2 . black
air + 3CO2
(b) It is due to formation ofHgl^a soluble complex) in Iron is protected from rusting by:
both the cases with Hg^ : - galvanisation (by a coating of Zn)
Hgl2+2r ---- > Hgtf- - electroplating with Ni or Cr
But in HggLj, first there is oxidation of Hg(I) to Hg(II) - heating the iron to redness in steam when a
and then complex formation takes place; it is by protective coating of Fe3O4 (ferrosoferric oxide) is
following disproportionation reaction: formed - Barf Process.
Hgf+41’ ---- > Hgl2- +Hg - painting with Pb3O4, PbCrO4, ZnCrO4 etc.
+1 +2 0
(b) Passivity : When a piece of iron is dipped in
Problem 7. (a) CO and CN“ are toxic. Explain by a conc.HN03, a slight reaction takes place which then
suitable example. ceases completely. There is no change in the appearance of
(b) S2O|" (peroxy disulphate ion) oxidises Ag+ in the the iron, but this does not displace Cu from CuSO4 solution
presence of excess pyridine. Write chemical reaction. which is however a common property of Fe. This behaviour
is also observed with Cr and Al when in contact with
Solution (a) CO is toxic because it forms a complex HNO3. These metals in this form are said to be passive.
with iron of haemoglobin in the blood, and this complex is
more stable than oxy-haemoglobin. This prevents the Passive iron returns to normal state when scratched or
haemoglobin in the red blood corpuscles from carrying heated in a reducing atmosphere of Hg or CO or immersed
oxygen round the body. This causes an oxygen deficiency, in a cone, alkaline solution.
leading to unconsciousness and then death.
692 | Essential Inorganic Chemistry

Passivity of iron is believed to be due to the formation of Solution


an extremely thin layer ofFe3O4 on the surface. Passivity H2S
of iron prevents it from rusting.
(a) FeS
h2so4
FeSO4 J
Problem 9. In acidic solution silver (II) oxide first NaOH + H2O2
dissolves to produce Ag2+.This is followed by oxidation of
H20 to 02 and the reduction of Ag2* to Ag+. Write chemical Fe(OH)3
equations for the dissolution and redox reactions.

Solution C02

Dissolution : AgO + 2H+-----> Ag2+ + H20 HCI t_


(b) Baco3 BaCI2
BaCrO4
Redox : 4Ag2+ + 2H2O----- > 4Ag+ + 4H+ + 02
KOH
K2Cr2O7 K2CrO4

Problem 10. How are the following prepared?


(a) Ferrochrome (b) Chrome alum h2o, n2
(c) Mohr's salt (d) Ferromanganese
(0 (NH4)2Cr2O7
A
Cr2O3 J
A
Solution (a) FeCr2O4 + 4C —> Fe + 2Cr +4C0T cone. HCI
chromite
ferrochrome CrCI3
Ferrochrome is an alloy (of Fe and Cr) and can be added
directly to iron, together with other metals, to produce Cl2
steel.
(d) MnO2
A, cone. HCI
MnCI2 — t
(b) + H2SO4 + 3SO2
Na2CO3
K2SO4 + Ci2(SO4)3 + H,0
MnCO3
crystallises as
KCr(SO4 >2-12^0
Problem 12. Write chemical reactions for the
(chrome alum) following :
It is used in tanning hides, dyeing textiles and fixing (a) Mn3+ disproportionates toMn2+ andMnO2 in acidic
baths in photography. medium
(c) FeSO4 +(NH4)2SO4 + H20 (i) concentrate (b) MnO^" disproportionates to Mn02 and MnOj in
(ii) cool acidic medium
(c) MnO^- is obtained if Mn02 reacts with MnO4 in
FeSO4(NH4)2SO4-6H2O
basic medium.
Mohr's salt (d) Zn reduces VO2* to V3+ in acidic medium
(d) Mn02 + Fe2O3 + 5C Mn + 2Fe +5C0?
ferromanganese Solution
(a) 2Mn3* +1^0-----> Mn2* + Mn02 + 4H*
Problem 11. Suggest a series of reactions, using (b) 3MnOj‘ +4H* ----- > Mn02 + 2Mn0; +2H2O
common chemicals, by which each of the following
(c) Mn02 + 2MnO4 + 40H‘ ----- > 3MnOj" +21^0
syntheses can be performed?
(d) Zn + 2VO2* + 4H* ----- > Zn2+ + 2V3+ + 21^0
(a) Fe(OH)3 from FeS
(b) BaCrO4 from BaCO3 and KjCi^O? Problem 13. Calculate the sum of the first two
(c) CrCl3 from(NH4)2Ci2O7 ionisation energies for the first-series transition elements,
(d) MnCO3 from MnO2 from Table 16.1 and account for the general trend.
Chapter 16: The Transition and Inner Transition Elements | 693

Solution (From Table 16.1) (3) The solution from (2) is treated with NaOH(aq) and
Element Z (/g)2+(/£)2
A dark precipitate separates from the
(/H)i (/g)2
Sc~ solution. The solution is colourless.
21 631 1235 1866
Ti 22 656 1309 1965 (4) The precipitate from (3) dissolves in dil. HC1 to
V 23 650 1413 2063 produce a coloured solution.
Cr 24 652 1591 2243 (5) The solution from (4) is treated with NH3(aq). A
Mn 25 717 1509 2226
dark precipitate forms.
Fe 26 762 1561 2323
Co 27 758 1645 2403 (6) The precipitate from (5) is soluble in HCKck?) and
Ni 28 736 1751 2487 the solution develops an intense red colour when
Cu 29 775 1958 2733 treated with SCN" (aq).
Zn 30 906 1733 2639
Solution
Across the first transition element series, Zefr increases Scheme Conclusion
and there is an almost linear increase in the sum of first (1) and (2) A dark precipitate may be due to NiS, Cr2S3,Fe2S31MnS.
two ionisation energies. This is what is expected if the two (ZnS is colourless)
electrons are removed from the 4s-orbital. Higher than the Cr3* is not possible since this treatment makes Cr3* as
(3)
expected values for the sum of first two ionisation energies Na2CrO4 soluble. Possibility of Ni2* and Mn2* is
are observed for Cr and Cu, because of their anomalous rejected since solution (in upper layer) is colourless. A
dark precipitate is due toFe(OH)3.
electron configurations (Cr^c^ds1 and Cu :3d10 4s1). An
increasing Zefr affects 3d-orbitals more than the 4s-orbital (4) Fe(OH)3 is soluble in dil. HCI forming FeCI3 (coloured)
and the second ionisation energy for an electron from the (5) FeCI3 is precipitated asFe(OH)3 on addition ofNH3(aq).
3d-orbital is higher than expected. (6) Fe(OH)3 is again soluble in dil. HCI, and FeCI3 formed
gives intense rod colour with SCN"
2800 Fe3* + SCN’ —> (Fe(SCN)]2*
red
2600
Probably present : Fe3*
CM
2400 Probably absent : Cr3*
~ 2200
Uncertain : Ni24, Zn2*, Mn2*
£ 2000 Problem 15. Predict the products of the following
reactions:
1800 (a) Zn is added to a solution of C^tSO.^ in acid.
Atomic number
(b) TiCl3 is added to a solution containing an excess of
CrO^" ion.
Problem 14. A solution may contain any of the
(c) Cr2* is added to CrO^“ is acid solution.
following ions: Fe2*, Ni2*, Cr3*, Zn2*, Mn2*. The solution is
(d) Mn is heated with CrO3
treated as outlined below, with the results indicated.
(e) CrO is added to HNO3 solution
Which of the ions is (are) probably present? Are there some
ions about which you have insufficient information to (f) CrCl3 is added to NaOH solution
reach a decision? Explain. (g) Fe is heated in an atmosphere of steam
(h) NaOH is added to solution of Fe(NO3)3
(1) The original unknown solution is treated with
(i) Fe(NO3)2 is added to an acidic solution of K2CrO4
(NH4)2S(aq) in a buffered basic solution. A dark
precipitate is obtained. (j) A solution of Fe(NO3)2 and HNO3 is allowed to
stand in air
(2) With the exception of a small residue of sulphur, (k) FeCO3 is added to a solution of HC1O4
the precipitate from (1) is completely soluble in
(l) Fe is heated in air.
aqua-regia (HNO3 + HC1)
694 | Essential Inorganic Chemistry

Solution (a) 2Zn(s) + 2Cr3+(aq) + 2H3O+(aq) (h) Fe(NO3)3(az7) + 3NaOH(ag) 4 Fe(OH)3(s)


+ 3NaNO3(<M?j
2Zn2+(ag) + ^(g) + 2H2O(Z) + 2Ci^+(ag)
(b) 3TiCl3(s) + CrOj-(a?) + 5H+(ag) ---- > 3Ti4+(ag) (i) 2CrO^"(ag) + 2H+(ag) ---- > Cr2O^“(ag) + H^CXZ)
+ Cr(OH)3(s) + 9CF(ag) + H2O(Z) 6Fe2+(ag) + Cr2O27-(ag) + 14H+(ag) —> 6Fe3W
(c) 3Cr2+(ag) + CrOj'(a^) + 8H3O+(ag) ---- » 4Cr3+(agf) 4-2Cr3+(ag) + 7H2afr
+ 12H2O(Z) (In presence of H+, CrO^- changes to Ci^O^- which
oxidises Fe2+ toFe3+)
(d) 8CrO3(s) + 9Mn(s) ---- > 4Cr2O3(s)+ 3Mn3O4(s)
(j) 4Fe2+(ag) + O2(g) + 4HNO3(ag) 4Fe3+(a?)
(e) CrO(s) + 2H3O+(ag) + 2NO3(ag) ---- > Cr3+(ag)
+ 211,0(0+ 4N0;M
+ 2NO3(ag)+ 3H2O(Z)
(k) FeC03(s) + 2HC104(ag) ■» Fe(C104)2(ag)
(0 CrCl3(s) + 3NaOH(a<7) ---- > Cr(OH)3(s) + 3NaCl(ag)
+ H20(Z) + C02(^
(g) 3Fe(s) + 4^0^) ---- ■> Fe3O4(s) + 411,(g) (1) 3Fe(s) + 2O2(£) ---- > Fe3O4(s)
Chapter 16: The Transition and Inner Transition Elements | 695

(e) Cu+ is a di0 ion and colourless but Cu20 is red


Explanation and Cu-jS is black.
(a) Explain why a green solution of potassium
- Cu+ is a d10 ion so that there is no d—d * transition
manganate (VI) K2MnO4 turns purple and a and thus the ion is colourless. ButCuaO and Cu^S are
brown solid is precipitated when CO2 is bubbled coloured because of charge transfer of electrons from
into the solution. O2" orS2- to the vacant orbital ofCu*.
~ C02 in aqueous solution is acidic (f) Prussian blue and Turnbull's blue have
CO2 + HaO H2CO3 H* + HCO3 different chemical formula, yet they are
identical.
In presence of H*, MnO42-disproportionates to MnO4
- Prussian blue is formed as an intense blue
(purple, by oxidation) and MnO2 (brown solid, by
reduction) precipitate when a solution of ferric salt is treated
withK4[Feu(CN)6l
3MnO2- + 4H* ---- > 2MnO4‘ +MnO2 + 21^0 (I) 4FeCl3 + 3K4[Fe(CN)6] Fe4[Fe(CN)6]3 X
+VI +VII +IV
Cb) When mercury is oxidised with a limited amount + 12KC1
of oxidising agent (i.e., an excess of Mg) then Hg1 Turnbull's blue is formed as a deep blue precipitate
compounds are formed. If there is an excess of when a solution of ferrous salt is treated with
oxidising agent, then Hgu compounds are K3[FeUI(CN)6].
formed. (II) 3FeCl2 + 2K3[Fe(CN)6]---- > Fe3[Fe(CN)6 X
+ 6KC1
Hg2* + 2e“ ---- > Hg E° = 0.85 V
X-ray studies have established that Prussian blue and
Hg2* + 2e~ ---- > 2Hg E° = 0.79 V Turnbull's blue are identical. When either of the
“ The standard reduction potentials are so close that reactions (I) and (II) is performed, a redox reaction also
oxidising agents like HN03 will convert Hg to Hg2* takes place partially:
rather than Hg(I) if the oxidising agent is present in
Fe3++[FeII(CN)6]4- ---- > Fe2*+[FeIII(CN)6l3"
excess.
The reduction potential diagram shows that Hg|* is Fe2* + [Fem(CN)6]3- ---- > Fe3* + [Feu(CN)6 J4"
stable to disproportionation by a small margin under Thus, both of these compounds have a mixture of
standard conditions : Fe2*, Fe3*, [Fe(CN)6]‘- and [Fe(CN)6 ]3"
Hg2*+Hg^ Hg2* E° = 0.06 V hence we write Prussian blue and Turnbull's blue as
Fe[Fe(CN)6r that can be due to Fe2* and
(0 The mercurous ion is written as Hg^* while the
[Feni(CN)6]3" shown as Feu(Fein(CN)6] or due to
cuprous ion is written as Cu*.
Fe3* and [Fen(CN)6 ]*~ shown asFeIU[Feu(CN)6r.
-Hg80 : [Xel.44/-145d1()6s2
(g) [Co(NH3)6]3* is diamagnetic and [CoFg]3- is
Hg* : [Xe] 4/14 5d10 6s1
strongly paramagnetic.
In Hg1, there is one unpaired electron and we expect
3d 4s 4p 4d
Hg1 (mercurous) salt to be paramagnetic. But
magnetic moment of mercurous salt is zero indicating Co3* |tl|t|t|t|f|
that it is diamagnetic which is only possible when 6s
electron has been used by two Hg atoms in bonding. [CofNHalJ3* tl|tl|tljX|X| |x| Ixlxlxl diamagnetic due
to paired electrons
Hence, mercurous ion is Hg2*. d?sp3

Cua9 : [Ar]3d104s1 [CoFg]3- tlltltltltl ig Ixlxlxl Ixlx


Cu* : [Ar] 3d10 sp3d2
paramagnetic due to four unpaired electrons
Cu1 (cuprous) salt is diamagnetic by experiment and
thus it is not dimeric and is thus Cu*. Co3+ has 3d6-configuration with four unpaired
(d) ZnO is yellow when hot, but white when cold. electrons in ground state. In presence of NH3 (strong
~ Zn2* has 3d10 EC hence all electrons paired thus ligand) all the unpaired electrons in Co3’ get paired
form white salt. The yellow colour of hot ZnO is due to and thus [Co(NH3)6]3+ has (Psp3 hybridisation
defects in the solid structure which increases with (octahedral structure), thus it is diamagnetic (no
temperature.
electron unpaired). F“ is a weak ligand hence, six lone
696 | Essential Inorganic Chemistry

pairs of six F“ are filled in outer d-orbitals of Co3+ this. This also explains why the +3 state ofMnisd
which has now four electrons unpaired. Thus CoFg“ little importance.
has sp3d2 hybridisation in Co3+ and is thus (j) Although Mn2+ ion has five unpaired electrons,
paramagnetic due to unpaired electrons. [Mn(H2O)6 ]2+ is not intensely coloured.
(h) For the first row transition metal the E° values - Spin multiplicity is defined as = (2s +1)
are where, s = sum of spin.
E° V Cr Mn Fe Co Ni Cu
Mn(25) = [Ar] 4s33d5
(M2 + /M) -1.18 -0.91 -1.18 -0.44 -0.28 -0.25 +0.34
Mn2+ = [Ar] 3d5 = [Ar] 11 | t 111 t~Tt
Explain the irregularity in the above values.
The E° (M2+ /M) values are not regular which can be c 1 5
Thus, s=5x-=-
explained from the irregular variation of ionization 2 2
energies (ZEj+ZEj) and also the sublimation Hence, multiplicity = (2s + 1) = 6
energies which are relatively much less for
manganese and vanadium. (240 kJ mol”1 for Multiplity of excited state can’t be six.
manganese and 470 kJ mol-1 for vanadium). Thus, electronic transition in Mn2+ are spin
(i) Explain as to why the E° value for the forbidden, hence [Mn(H2O)6f+ salts appear
Mn3+/Mn2+ couple is much more positive than colourless.
that for Cr3+/Cr2+ or Fe3+/Fe2+. Also, [Mn(H2O)6 f+ has centre of symmetry and in
- Much larger third ionization energy to Mn (where the such cases electronic transitions are expected to be
required change is d5 to d4) is mainly responsible for lap orate forbidden.
Master Exercises
2

Exercise 1
(Stage 1: Learning)
Short Answer Type Questions (b) Cr tT~] Mn
1. How many ions are obtained in the following (c) Cu I I Zn
complexes due to ionisation in aqueous solution ? (d) Cu I I Ni
•. (a) PtCl4 -6H2O (b) CuCl2 • 2^0 (e) Zr |----- 1 Hf
. (c) CrClg SHjO (d) CuC12 -4NH3
(0 Ca I I Sc
(g) Cr I I Mo
2. Calculate magnetic moments of M2+ ions (divalent
[B] Calculate effective nuclear charge of 3d-series
cations) of 3d-series and arrange them in the
elements (Sc—Zn)
increasing order.
3. Magnetic moment of [Fe(CN)6l4~ is zero but that of Screening Effective nuclear
constant(s) charge Z*=Z-S
[Fe(H2O)6l2+ is V24 BM Explain.
4. Arrange [FeCCN)^’, [Fe(CN)6]3", [Ni(CN)4f" and (a) Sc(21)
[Nitf^O^ f + in increasing order of magnetic moment. (b) Ti(22)
5. Magnetic ofFex+ ion is V35 BM. Determine number of (c) V(23)
unpaired electrons. Also write its electronic (d) Cr(24)
configuration. (e) Mn(25)
6. s- and p-block elements form colourless ions. (f) Fe(26)
Explain. (g) Co(27)
7. Which of the following ions are coloured/colourless ? (h) Ni(28)
(a) Zn2+ (b)Cu+ (i) Cu(29)
(c) Cu2+ (d)Sc3+ (j) Zn(30)
(e) Ti2+ (0 V3+
(g) Mn2+ (h)Ni2+ 13. (a) CrO5 forms addition compound with pyridine in
(i) Na+ (j)Ca2+ ethereal layer. What is structure of the addition
product?
8. Assuming Aufbau rule is not known then which of the
(b) In ammoniacal solution, dichromate in presence
following ions would be coloured? of forms (NH3)3CrO4 in which Cr is in
(a) K+ (b) Cu+ +IV state. What is structure?
(c) Ca+ (d) Sc2+ (c) CrO3 is used as cleansing agent for glassware.
9. Hg2+ salts are colourless. Explain. How is it prepared?
(d) Give structure of Ci^O2-ion.
10. Which of the following are paramagnetic, coloured,
and coloured and paramagnetic both? 14. What happens when (NHj )2 Ci2O7 is heated? What is
N02,Cu+,02,03,Hg^+, Cd2+, Al, C (graphite) the type of reaction?
11. Compounds, containing the Sc3+ ion are colourless, 15. Gas (A) and gas (B) both turns K2Ci2O7/H+ green.
whereas those containing the Ti3+ ion are coloured. Gas (A) also turns lead acetate paper black. When gas
Explain. (A) is passed into gas (B) in aqueous solution,
12. [A] Compare covalent size of the following yellowish white turbidity appears. Identify gas (A)
(a) Nb I 1 Ta and (B) and explain reactions.
698 | Essential Inorganic Chemistry

16. IfCO2(g) under pressure is passed intoNa2CrO4(ag), (A) also gives white ppt (E) with AgNO3 solution.
yellow colour solution changes to orange-red solution. Identify (A) to (E).
What is it due to ? What is the function of CO2(g) ? 29. Colourless salt (A) (B) + (C) gas. (B) dissolves
17. Arrange the following ions in order of increasing
strength as a reducing agent, and account for the both in acid and alkali solution. Gas (C) turns lime
trend: water milky and acidified KjCigO? solution green.
(a) Ci^+ (b) Cr3+ Ammoniacal solution of (A) gives white ppt. (D) with
(c) CigO^- H2S gas. Identify (A), (B),(C) and(D).
18. (a) Zn2+ salts appear acidic due to hydrolysis. 30. What is the advantage of using ZnO in paints instead
Identify acid and base in the following : ofPb2+ salts?
[Zn(H2O)4f+ +H2O [Zn(H2O)3OH]++ H3O+ 31. Compare thermal stability of ZnO, CdO and HgO.
(b) Which other metal ion (other than Zn2+) behaves 32. Match the compounds in column (A) with their uses
like [Zn(H2O)4 f + in giving an acidic solution? in column (B):
19. What happens if NaHCO3 is added to aqueous ZnSO4 Column A Column B
solution? ZnO (a) Metallurgical extraction of Ag
(I)
20. Calculate magnetic moment of Mn in MnO4 (ID ZnO (b) Zerox printing.
.............................unpaired electrons on Mn in MnO4 (HI) ZnSO4 (c) Luminous paints.
........................... BM (IV) Zn (d) Lithophone, eye lotion.
21. Determine number of moles of MnO4 which are
33. An impure metal is allowed to react with carbon
required to oxidise one mole of ferrous oxalate. Write monoxide at 50°C and the volatile gas thus formed is
reactions. collected and heated further to about 200°C. This
procedure gives the metal of 99.99% purity. What is
the metal?
22. Rust stains contain Fe3+ salts (Fe2O3 • H2O). These
stains from fabric can be removed by the addition of Analytical Questions
oxalic acid. Explain.
1. (a) Calculate number of moles ofMnO4 required in
23. FeCl3 solution gives CO2 with NaHCO3 solution. acidic medium to oxidise :
Explain. (i) 1 mol of ferrous oxalate,
24. How might you separate the following pairs of ions by (ii) 1 mol of ferric oxalate.
addition of a single reagent? (b) Calculate number of moles of Cr2C^“ required in
(a) Fe3+ andNa* (b) Cr3+ andFe3+ acidic medium to oxidise :
(d) Fe3+ andCu2+ (i) 1 mol of ferrous oxalate,
(ii) 1 mol of ferric oxalate.
25. (A) is a blue coloured hydrated salt giving white ppt.
KNOo + KOH
(B) with BaCl2.2.495 g of (A) gave 2.33 g of white ppt. 2. Black coloured solid (A) ------- 3--------- > green coloured
(B). (A) gives deep reddish-brown solution on reaction A
with KI but after addition of hypo, reddish-brown solution (fl)
solution appears in the form of white ppt. (C). (A) also n C02
gives chocolate coloured ppt. (D) with K4[Fe(CN)6]. (C) pink + (A)
Identify (A) to (D) and explain reactions. (C) is decolorised by Fe2+. Identify (A), (B) and (O
26. When aqueous KCN is added to aqueous CuSO4 and explain reaction.
solution, a white precipitate soluble in an excess of 3. Mineral (A) + dil. H2SO4 (B) + (C)+(D), (gas)
KCN, is formed. No precipitate is formed when HjS soluble
gas is passed through the solution this point. (B) or (C) + BaCl2 solution---- > white ppt. (B)
Explain.
(B) + K3[Fe(CN)6]---- > blue ppt (F)
27. AG^ (standard free energy of formation) of ZnO and
(C) +aq NH3---- > deep blue solution (G)
CO are -318.2 kJ mol-1 and -137.3 kJ mol-1. Is (excess)
reduction of ZnO by carbon possible. (O + (D)gas -> black ppt. (H) soluble in dil. HNOj
28. Colourless salt (A) gives white ppt. (B) with NaOH;
ppt. (B) dissolves in excess of NaOH to form (C). What are (A) to (H) and explain reactions?
On passing H^S gas into (C) white ppt. (D) appears.
Chapter 16: The Transition and Inner Transition Elements | 699

4. A student prepared three coordination compounds 12. Identify A, B, C, D and E in the following :
containing chromium with the following properties: CO2
K2CQ3, O2 (air)
Cl' Ions In solution A 8 —
Formula Colour per formula unit A
soluble in 1Fe2O3
(a) CrCI3-6H2O Violet 3 water
(yellow)
(b) CrCI3-6H2O Light green 2
(c) CrCI3 • 6H2O Dark green 1 H2SO4
B C_

Write modern formulae for these compounds and orange solution


suggest a method for confirming the number of Cl“ NH4CI
C D
ions present in solution in each case. N2
5. Titanium, used in making jet aircraft engines, is D E J
much harder than potassium or calcium. Explain. green
6. The highest oxidation state for the early transition
metals Sc, Ti, V, Cr and Mn is the periodic group
number. The highest oxidation state for the later NCERT Problems
transition elements Fe, Co, and Ni is less than the 1. Write down the electronic configuration of
periodic group number. Explain. (a) Cr3* (b) Cu*
7. (a) Which is stronger oxidising agent, Cr2* or Cu2*? (c) Co2* (d) Mn2*
(b) Which is stronger reducing agent, V2* or Co2+? (e) Pm3* (f) Ce4*
(g) Lu2* (h) Th4*
(c) Which is more easily oxidised, Cr2* orNi2*?
(d) Which is more easily reduced, Ti3* or Co3*? 2. Why are Mn2* compounds more stable than Fe3+
towards oxidation to their +3 state?
8. Arrange the following substances in order of
increasing strength as an oxidising agent, and 3. Explain briefly how +2 state becomes more and more
account for the trend: stable in the first half of the first row transition
(a) Mn2* (b) MnO2 element with increasing atomic number?
(c) MnO4 4. To what extent do the electronic configuration decide
9. What product is formed when dilute H2S(}l4, a non the stability of oxidation states in the first series of
the transition elements? Illustrate your answer with
oxidising acid is added to each of the following ions or example.
compounds?
(a) CrO2’ (b) Cr(OH)3 5. What may be the stable oxidation state of the
(0 Cr(OH)2 (d) Fe(OH)2 transition element with the following d-electron
(e) Cu(0H)2 configurations in the ground state of their atoms :
3d3,3d5,3d8, and 3d4?
10. Write a balanced net ionic equation for each of the
following reactions: 6. Name the oxometal anions of the first series of the
(a) A CrO^~ solution turns from yellow to orange transition metals in which the metal exhibits the
upon addition of acid. oxidation state equal to its group number.
(b) Fe3*(aq) reacts with aqueous KSCN to give a deep 7. What is lanthanoid contraction? What are the
red solution. consequences of lanthanoid contraction?
(c) Copper metal reacts with HNO3 to give NO gas 8. What are the characteristics of the transition
and a blue solution. elements and why are they called transition
(d) A deep green solution of Cr(OH)3 in excess base elements? Which of the d-block elements may not be
turns yellow on addition ofH^. regarded as the transition elements?
(e) Dissolution of CuS in hot HN03 give NO gas, a 9. In what way is the electronic configuration of the
blue solution and a yellow solid. transition elements different from that of the
11. For each of the following reactions in acidic solution, non-transition elements?
predict the products and write a balanced net ionic 10. What are the different oxidation states exhibited by
equation: the lanthanoids?
(a) Co3*(aq) + 1^0(1)
11. Explain giving reason
(b) Cr2*(aq)+ O2(g) - (a) Transition metals and many of their compounds
(c) Cu(s)+ show paramagnetic behaviour.
700 | Essential Inorganic Chemistry

(b) The enthalpies of atomisation of the transition (b) Cobalt(II) is stable in aqueous solution but in the
metals are high. presence of complexing reagents it is easily
(c) The transition metals generally form coloured oxidised.
compounds. (c) The dl configuration is very unstable in ions.
(d) Transition metals and their many compounds act 23. What is meant by ‘disproportionation’? Give two
as good catalyst. examples of disproportion reaction in aqueous
12. What are interstitial compounds? Why are such solution.
compounds well known for transition metals? 24. Which metal in the first series of transition metals
13. How is the variability in oxidation states of transition exhibits +1 oxidation state most frequently and why?
metals different from that of the non-transition 25. Calculate the number of unpaired electrons in
metals? Illustrate with examples. following gaseous ions : Mn3+, Cr3+, V3+, and Ti3*.
14. Describe the preparation of potassium dichromate Which one of these is the most stable in aqueous
from iron chromite ore. What is the effect of solution?
increasing pH on a solution of potassium dichromate? 26. Give example and suggest reason for the following
15. Describe the oxidising action of potassium features of the transition metal chemistry:
dichromate and write the ionic equations for its (a) The lowest oxide of transition metal is basic, the
reaction with highest is acidic.
(a) iodide (b) iron(II) solution (b) A transition metal exhibits higher oxidation
(c) H2S states in oxides and fluorides.
16. Describe the preparation of potassium (c) The highest oxidation state is exhibited in
permanganate. How does the acidified permanganate oxoanions of a metal.
solution reacts with (a) iron(II) (b) SO2 (c) oxalic acid? 27. Indicate the steps in the preparation of
Write the ionic equations for the reactions. (a) I^CijO? from chromite ore
17. ForAf2+/M andM3+/Af2+ systems that E& values for (b) KMnO4 from pyrolusite ore
(c) Copper sulphate from metallic copper
some metals are as follows : (d) Calomel from corrosive sublimate
Cr2+/Cr -0.9 V Cr3+/Cr2+ -0.4 V
2+
28. What happens when aqueous ammonia reacts with
Mn2+/Mn -1.2 V Mn3+/Mn +1.5 V (a) silver chloride
Fe2+/Fe -0.4 V Fe3+/Fe2+ +0.8 V (b) mercury (I) chloride
Use this data to comment upon : (c) mercury (II) chloride?
(a) The stability ofFe3+ in acid solution as compared 29. Describe the two uses of each of the following
to that of Cr3+ or Mn3+ (a) Copper sulphate (b) Silver nitrate
(b) The ease with which iron can be oxidised as (c) Silver bromide
compared to the similar process for either 30. What are alloys? Name an important alloy which
chromium or manganese metal. contains some of the lanthanoid metals. Mention its
18. Predict which of the following will be coloured in uses.
aqueous solution? 31. What are inner transition elements? Decide which of
Ti3+,V3+,Cu+,Sc3+,Mn:2+,Fe3+ ,Co2+ andMnO4. the following atomic numbers are the atomic
Give reason for each. numbers of the inner transition elements: 29,59,74,
19. Compare the stability of +2 oxidation state for the 95, 102, 104.
elements of the first transition series. 32. The chemistry of the actinoid elements is not so
20. Describe the chemistry of the three stages of smooth as that of the lanthanoids. Justify this
photography, i.e., exposure, developing, and fixing. statement by giving some examples from the
oxidation state of these elements.
21. Compare the chemistry of actinoids with that of the
33. Which is the last element in the series of the
lanthanoids with special reference to
actionoids? Write the electronic configuration of
(a) electronic configuration
these elements. Comment on the possible oxidation
(b) oxidation state
state of this element.
(c) atomic and ionic sizes and
(d) chemical reactivity 34. Use Hund’s rule to derive the electronic configuration
of Ce3+ ion, and calculate the magnetic moment on
22. How would you account for the following the basis of‘spin-only’ formula.
(a) Of the cf4 species, Cr3+ is strongly reducing while
35. Name the members of the lanthanoid series which
manganese(III) is strongly oxidising.
exhibit +4 oxidation states and those which exhibit
Chapter 16.The Transition and Inner Transition Elements | 701

+2 oxidation states. Try to correlate this type of 37. What is lanthanoid contraction? What is its effect on
behaviour with the electronic configuration of these the chemistry of the elements which follow the
elements. lanthanoids?
36. Compare the chemistry of the actinoids with that of 38. Write the electronic configurations of the elements
lanthanoids with reference to : with the atomic number 61, 91, 101, and 109.
(i) electronic configuration
(ii) oxidation states, and
(iii) chemical reactivity.

Exercise 2
(Stage 2: High Skill Problem Solving)
Only One Option Correct 9. InNa2[Fe(CN)5NO], sodium nitroprusside,
1. When KCN is added to CuSO4 solution (a) oxidation state of Fe is +2
(a) KCN acts a reducing agent (b) this has NO+ as ligand
(b) KCN acts as complexing agent (c) both (a) and (b) correct
(c) K3[Cu(CN)4 ] is formed (d) none of the above is correct
(d) all of the above are correct 10. FeSO4 solution gives brown colour ring in testing
2. Out of AgNO3, AgF and AgC104, water soluble salts nitrates or nitrites. This is
are (a) [FelHjOJjNOf (b) [Fe(H>O)5NO2f’
(a) AgF (c) [Fe(H2O)4(NO)2f * (d) [Fe(HjO)4NOf*
(b) AgF, AgNO3 11. FeCl3 -6H2O is actually
(c) AgF, AgN03,AgC104
(d) None of the above (a) [Fe(H2O)6]Cl3
(b) [Fe(H2O5Cl]Cl2 H2O
3. Stability ofCu+ and Ag+halide complexes are in order
(c) [FefH^ClJCl^O
(a) I > Br > Cl > F (b) F > Cl > Br > I
(c) Cl > F > I > Br (d) Br > I > Cl > F (d) [Fe^O)^ 1-30,0

4. Which of the following exists as white salt in 12. Ni2+, in traces, can be tested using
(a) sodium nitroprusside
anhydrous state?
(b) dimethyl glyoxime
(a) CuF2 (b) CuSO4
(c) ammonium sulphocyanide
(c) Both (a) and (b) (d) None of these
(d) potassium ferrocyanide
5. Which oxides will not give metal on heating? 13. CuSO4 can be estimated volumetrically
(a) HgO (b) ZnO
(a) by reaction with KI followed by reaction with
(c) Ag2O (c) All of these
Na2S2O3
6. When KI (excess) is added to (b) by reaction with BaCl2
I:CuSO4 II:HgCl2 (c) by reaction with K4Fe(CN)6
III:Pb(NO3)2 (d) none of the above is correct
(a) a white ppt. of Cui in I, a orange ppt. Hglj in II 14. Which is correct statement?
and a yellow ppt. PbLj in III (a) Ammoniacal CuCl is used to measure the amount
(b) a white ppt. of Cui in I, an orange ppt. dissolving of CO in gas samples
toHgI42~in II, and a yellow ppt. ofPb^ in III (b) Ammoniacal CuCl gives red ppt. with CH=CH
(c) a white ppt. of Cui, Hg^ and Pblg in each case (c) Both (a) and (b) are correct
(d) None of the above is correct (d) None of the above is correct
7. Most common oxidation state of lanthanides is 15. Which is not true statement about FeO?
(a) +2 (b) +3 (a) It is non-stoichiometric and is metal deficient
(c) +4 (d) +5 (b) It is basic oxide
(d) Its aqueous solution changes to Fe(OH)3 and
8. The purest form of Fe is
then to Fe2O3 • (Hz,O)n by atmospheric oxygen
(a) stainless steel (b) steel
(d) It gives red colour with KCNS
(c) cast iron (d) wrought iron
702 | Essential Inorganic Chemistry

16. Match List I with List II and select the correct answer 23. Match List I with List II and select the correct answer
using the codes given below the lists: using the codes given below the lists:
List I List II List I List II
(Metals) (Ores) (Alloys) (Constituents)
A. Zinc 1. Azurite A. Gun metal 1. Lead + tin
B. Tin 2. Carnallite B. German silver 2. Copper + tin + zinc
C. Copper 3. Calamine C. Brass 3. Copper + zinc
D. Magnesium 4. Cassiterite 4. Copper + zinc + nickel
D. Solder
Codes
A B c D Codes
4 2 1 A B C D
(a) 3
(b) 3 4 1 2 (a) 1 3 4 2
(c) 4 1 3 2 (b) 4 2 1 3
(d) 4 3 2 1 (0 2 4 3 1
(d) 3 1 2 4
17. Haemoglobin and chlorophyll contain respectively
(a) Fe, Co (b) Fe, Mn 24. An acidic solution contains Cu2+, Pb'i2+ and Zn2t.lf
(c) Mg, Fe (d) Fe, Mg hydrogen sulphide gas is passed through this
solution, the precipitate will contain
18. Due to lanthanide contraction (a) CuS and ZnS (b) PbS and ZnS
(a) Fe, Co, Ni have equal size (c) CuS and PbS (d) CuS, PbS and ZnS
(b) Zr and Hf have equal size
(c) all /-block ions have equal size 25. Ti2+ is purple while Ti4+ is colourless, because
(d) all isoelectronic ions have equal size (a) there is no crystal field effect in Ti4+
(b) Ti2+ has 3d2 configuration
19. In Ci2O72"every Cr is linked to (c) Ti4+ has 3d2 configuration
(a) two O atoms (b) three O atoms (d) Ti4+ is a very small cation when compared to Ti‘
(c) four O atoms (d) five O atoms and hence, does not absorb any radiation
20. For Ni and Pt different IP in MJ mol-1 are given 26. Silver ornaments turn black by atmospheric
below (a) O2 (b) N2
(IP)! + (IP).Z. (IP)3+(IP)4 (c) Cl2 (d) H^S
Ni 2.49 8.80 cro4422-^4
27. Cr2O7z- -^-4 CrO ’ c^o,2-
Pt 2.60 6.70 pH values x and y can be
hence: (a) 4 and 5 (b) 4 and 8
(a) nickel (II)
(ID compounds tend to be (c) 8 and 4 (d) 8 and 9
thermodynamically more stable than platinum
(ID 28. In dilute alkaline solution MnO^ changes to
(b) platinum (IV) compounds tend to be more stable (a) MnO^“ (b) MnO2
than nickel (IV) (c) Mn2O3 (d) MnO
(c) both (a) and (b) are correct 29. Cl2 gas is obtained by various reactions but not by
(d) none of the above is correct
(a) KMnO4 + cone. HC1 —
21. Paramagnetism is given by the relation p = 2^/s(s +1) A
(b) KC1 + + cone.
magnetons where ‘s' is the total spin. On this basis,
the paramagnetism of Cu+ ion is (c) MnO2 + cone. HC1
(a) 3.88 magnetons (b) 2.83 magnetons (d) KC1 + F2 ---- >
(c) 1.41 magnetons (d) zero
30. Which is not true statement about KMnO4 ?
22. The oxygen carrying pigment, oxy-haemocyanin, (a) Its solution is unstable in acidic medium
containing two copper ions is diamagnetic, because (b) Its small quantity added to cone. H^SC^, a green
(a) the two copper ions are in +1 oxidation state coloured solution containing MnO3 ions is formed
(b) one of the copper ions is in +1 oxidation state and (c) MnO4 changes to Mn2+ in basic solution
the other is in +2 oxidation state (d) It is self-indicator in Fe2+ or titration
(c) there are strong anti-ferromagnetic interactions 31. Pyrolusite in MnO2 is used to prepare KMnO4.
between the two copper ions
Steps are:
(d) there are ferromagnetic interactions between the I
two copper ions MnO2 MnO42" i MnO4’
Chapter 16: The Transition and Inner Transition Elements | 703

I and II are 40. FeCi2O4 (chromite) is converted to Cr by following


(a) fuse with KOH/air, electrolytic oxidation steps:
(b) fuse with KOH/air, electrolytic reduction in
(c) fuse with concHNO3/air, electrolytic reduction Chromite I Na2CrO4 > C12O3 Cr
(d) all the above are correct I, II and III are
32. Fe is made passive by I II III
(a) dil. H2SO4 (b) dil. HC1 (a) Na2CO3/air, A c C
(c) aqua-regia (d) cone. H2SO4 (b) NaOH/air, A C,A Al, A
(c) NaOH/air, A C,A Mg, A
33. Maximum magnetic moment is shown by (d) cone. H^Oj, A NH4C1, A C, A
(a) d5 (b) d6
(0 d7 (d) d8 41. When HjO., is added to an acidified solution of
34. Magnetic moment of Cr (Z = 24), Mn+ (Z = 25) and K2Ci2O7
(a) solution turns green due to formation of Ci^Og
Fe2+ (Z =26) are x, y, z. They are in order
(b) solution turns yellow due to formation ofK2Cr04
(a) x<y < z (b) x~y< z
(c) a deep blue-violet coloured compound CrO(O2)2 is
(c) z<x = y (d) x = y = z formed
35. Interstitial compound is formed by (d) solution gives green ppt. ofCriOHjg
(a) Fe, Co (b) Co, Ni 42. Out of SiCl4, TiCl4, PO43; SO42", CrO42", CC14
(c) Fe, Ni (d) all of these isostructural are
36. A jeweller is selling 22-carat gold articles with 95% (a) SiCl4,TiCl4 .(b) SO42-, CrO2"
purity, it is approximately (c) both (a)and (b) (d) None of these
(a) exact (b) 3.5% higher 43. Maximum oxidation state is shown by
(c) 3.5% lower (d) 5% lower (a) Os (b) Mn
37. Which is not true statement? (c) Cr (d) Co
(a) Ions of d-block elements are coloured due to d—d 44. Following elements do not show the properties
transition characteristic of d-block elements
(b) Ions of /-block elements are coloured due to f—-f
(a) Cu, Ag, Au (b) Zn, Hg, Cd
transition
(c) Sc, Ti, V (d) Fe, Co, Ni
<0 [Sc(H2O)6]3’,[Ti(H2O)6]u are coloured
complexes 45. The ability of d-block elements to form complexes is
(d) Cu+ is colourless ion due to
38. Match the catalysts in column X with their uses in (a) small and highly charged ions
column Y. (b) vacant low energy orbitals to accept lone pair of
electrons from ligands
Column X Column Y
(c) both (a) and (b) are correct
A TiCI4 1. Adams catalyst in reduction
(d) None of the above is correct
B. PdCl2 2. in preparation of (CH3)2 SiCI2
C. Pt/Pto 3. Reppe synthesis 46. AgCl and NaCl are colourless. NaBr and Nal are also
D Cu 4. used as the ziegler-Natta catalyst in colourless but AgBr and Agl are coloured. This is due
polythene production to
E. Ni-complexes 5. Wacker process for converting C2H4 (a) Ag+ polarises Br~ and I~
to CH3CHO (b) Ag+ has unpaired d-orbital
A B C D E (c) Ag+ depolarises Br- and I"
(a) 4 5 • 1 2 3 (d) None of above is correct
(b) 4 5 2 1 3 47. Which is correct statement?
(0 5 4 1 3 2 (a) In less acidic solution KoC^O.? and H2O2 give
(d) 2 1 3 5 4
violet coloured diamagnetic [CrO(O2)(OH)r ion
39. When (A)NH4VOI3: is heated. (b) In alkaline FUCX, K3CrO8 (with tetraperoxo
(B) (NH4)2Ci2O7 is heated species) [Cr(O2)413- is formed
(a) in both cases N2 is formed (c) In ammoniacal solution, K^Ci^O-j gives
(b) in both cases NH3 is formed (NH3)3CrO4
(c) in (A) NH3 and in (B) N2 are formed
(d) in (A) N2 and in (B) NH3 are formed (d) All of the above are correct statements
704 | Essential Inorganic Chemistry

48. Match the compounds of column X with oxidation 56. In 3d transition series, if nuclear charge increases,
state of column Y. the screening effect
Column X Column Y (a) increases
A. [Cr(H2O)6]Cl3 5 (b) decreases
B. CrO5 4 (c) first decreases and then increases
(d) first increases and then decreases
C. K3CrO8 6
57. Maximum oxidation state of Os is
D. (NH3)3CrO4 3
(a) +6 (b) +7
A B C D (c) +8 (d) +5
(a) 3 6 5 4
(b) 3 4 5 6 58. Which forms interstitial compound?
(0 4 5 6 3 (a) Fe (b) Co
(c) Ni (d) All of these
(d) 6 5 4 3
59. Which shows maximum magnetic moment among the
49. For CrO3 following is not true statement bivalent ions of the first transition series?
(a) it is called chromic acid (a) Fe2+ (b) Co2+
(b) it is colourless due to 3d10 configuration (c) Ni2+ (d) Mn2+
(c) it is bright orange solid and colour arises due to
60. Select incorrect statement(s).
charge transfer (a) Ionisation energies of 5d-elements are greater
(d) it is toxic and corrosive than those of 3d and 4d elements
50. Only +2, +3, +4 oxidation states are shown by (b) Cud) is diamagnetic while Cu(II) is paramagnetic
(a) Sc, Ti (b) Fe, Ni e (c) [Ti(H2O)6]3+ is coloured while [Scd^Olg]3* is
(c) Ti, Ni (d) Zn, Ni colourless
51. Coagulation of blood takes place by (d) Transition elements cannot form complexes
(a) ferric alum (b) potash alum 61. Guigret’s green is
(c) both (a) and (b) (d) None of these (a) NiO • 21^0 (b) Cr2O3 • 2H2O
(c) CuSO4 • 2H2O (d) CrO ■ 21^0
52. When KCN comes in contact with blood, one dies
immediately, it is due to 62. Which of the following metals has more than one
(a) CN" forms stable complex with iron of oxidation state?
(a) Zn (b) Mn
haemoglobin of blood
(c) Sr (d) Ca
(b) CN" combines with H2O of blood causing it poison
(c) both (a) and (b) are correct 63. Which of the following metals have only one
(d) None of the above is correct oxidation state?
(a) Co (b) Fe
53. MnO4 is of intense pink colour, though Mn is in (+7) (c) Sc (d) Al
oxidation state. It is due to 64. Which of the following compounds is amphoteric?
(a) oxygen gives colour to it (a) Cr(OH)2 (b) Fe(OH)2
(b) charge transfer when Mn gives its electron to (c) Cr(OH)3 (d) Fe(OH)3
oxygen 65. Increasing basic properties of TiO2, ZrO2 and HfO,
(c) charge transfer when oxygen gives its electron to are in order
Mn making it Mn (+VI) hence, coloured (a) TiO2 < ZrO2 < HfO2 (b) HfO2 < ZrO2 <TiO,
(d) None of the above is correct (c) HfO2 < TiO2 < ZrO2 (d) ZrO2 < TiO2 < HfO,
54. Out of[Fe(CN)6]*",[Ni(CN)4f“ and[Ni(CO)4] 66. Which is called chromic acid?
(a) all have identical geometry (a) CrO (b) Cr>03
(b) all are paramagnetic (c) CrO3 (d) CrO2
(c) all are diamagnetic 67. What are the species A and B in the following
(d) [FeCCNJg]4- is diamagnetic but [Ni(CN)4 and CrO3 + H2O A 2H; b?
[Ni(CO)4 ] Eire paramagnetic
(a) HjjCrC^, (b) H^CejO? , Cr2O3
55. In [Fe(CN)5(NO+)f“, Fe has +3 state. It can be (c) CrO^CtjO2,- • (d) H^O^CrOj’
decided by
68. Very recently (in Aug 2003), IUPAC has approved the
(a) magnetic measurement
name of the element of atomic number 110. Name of
(b) colligative property
the element is
(c) colour
(a) Darmstadtium (b) Mountanium
(d) hybridisation
(c) Rhenium (d) Bhorium
Chapter 16 . The Transition and Inner Transition Elements | 705

69. The element Ds lies in - 79. Which one of the following characteristics of the
(a) s-block (b) p-block transition metals is associated with their catalytic
(c) d-block (d) /-block activity?
70. The element Ds lies below... in the Periodic Table (a) Colour of hydrated ions
(a) W (b) Hg (b) Variable oxidation states
(c) Pt (d) Au (c) High enthalpy of atomization
(d) Paramagnetic behaviour
71. Which one of the following statements is not correct?
80. The basic character of the transition metal
(a) Zinc dissolves in sodium hydroxide solution
(b) Carbon monoxide reduces iron (III) oxide to iron monoxides follows the order
(Atomic number Ti = 22, V = 23, Cr = 24, Fe = 26)
(c) Mercury (II) iodide dissolves in excess of
potassium iodide solution (a) TiO > FeO > VO > CrO
(d) Tin(IV) chloride is made by dissolving tin (b) TiO > VO > CrO > FeO
solution in concentrated hydrochloric acid (c) TiO > CrO > TiO > FeO
(d) CrO> VO > FeO > TiO
72. Which of the following statements concerning
lanthanides elements is false? 81. Voids are
(a) Lanthanides are separated from one another by (a) 14 elements in the sixth period (atomic no. = 90 to
ion-exchange method 103) that are filling 4/-sublevel
(b) Ionic radii of trivalent lanthanides steadily (b) 14 elements in the seventh period (atomic no. =
increase with increase in the atomic number 90 to 103) that are filling 5/-sublevel
(c) All lanthanides are highly dense metals (c) 14 elements in the sixth period (atomic no. = 58 to
(d) More characteristic oxidation state of 71) that are filling 4f-sublevel
lanthanides elements is +3 (d) 14 elements in the seventh period (atomic no. =
58 to 71) that are filling 4/-sublevel
73. The mercury is the only metal which is liquid at 0°C.
82. Among K, Ca, Fe and Zn, the element which can form
This is due to its
(a) weak metallic bond (b) high ionization energy more than one binary compound with chlorine is
(a) Fe (b) Zn
(c) high vapour pressure (d) both (a) and (b)
(c) K (d) Ca
74. A transition element X has a configuration [Ar] 3c?4,
83. Philospher’s wool on treatment with cobalt nitrate,
in its +3 oxidation state. Its atomic number is produces
(a) 25 (b) 26 (a) CoBaO2 (b) CoZnO2
(c) 22 (d) 19 (c) CoSrO2 (d) CoMgO2
75. Which of the following elements is responsible for 84. Which of the following does not react with AgCl?
oxidation of water to O2 is biological processes? (a) NH4OH (b) NaNO3
(a) Fe (b) Cu (c) Na2S2O3 (d) Na2CO3
(c) Mn (d) Mo
85. Which of the following is used as purgative?
76. Which one of the following ionic species will impart (a) HgS (b) Hg9Cl2
colour to an aqueous solution? (c) HgCl2 (d) ZnSO4
(a) Ti4+ (b) Cu+
(c) Zn2+ (d) Cr3+ 86. The reactivity of transition elements decreases with
(a) the decrease in the atomic number
77. Which of the following statements is not correct? (b) the increase in the atomic number
(a) La(OH)3 is less basic than Lu(OH)3 (c) low heat of hydration
(b) In lanthanide series ionic radius of Ln3+ ion (d) None of the above
decreases
(c) La is actually an element of transition series 87. Fulminating gold is
rather lanthanide (a) AuC13 (b) Au2S
(d) Atomic radius of Zr and Hf are same because of (c) Au(NH2)=NH (d) H[Au(C1)41
lanthanide contraction 88. An extremely hot copper wire reats with steam to
78. The correct order of ionic radii of Y3+,La3+,Eu3+ and produce
Lu3+ is (a) Ci^O (b) CuO9
(c) Cu2O2 (d) CuO"
(Atomic number ofY = 39, La = 57,Eu = 63,Lu = 71)
(a) Lu3+ <Eu3+ < La3+ < Y3+ 89. Ag is obtained from AgNO3 with
(b) La3+< Eu3+< Lu3+< Y3+ (a) NH3 (b) PH3
(c) Y3+< La3+< Eu3+< Lu3+ (c) AsH3 (d) Na2CO3
(d) Y3+ < Lu3+ < Eu3+ < La3+
706 | Essential Inorganic Chemistry

90. The incorrect statement for d-block element is 103. Which among the following is consumed by humans
(a) it shows magnetic property in the elemental form?
(b) it has variable valency (a) Cu (b) Pb
(c) it has tendency of formation of coloured ions (c) Ag (d) Hg
(d) it has complete c?-orbitals 104. Anhydrous ferric chloride is prepared by
91. Which of the following transition element shows the (a) heating hydrated ferric chloride at a high
highest oxidation state? temperature in a stream of air
(a) Mn (b) Fe (b) heating metallic iron in a stream of dry chlorine
(c) V (d) Cr gas
92. To protect iron against corrosion, the most durable (c) reaction of feric oxide with HC1
metal plating on it, is (d) reaction of metallic iron with HC1
(a) nickel plating (b) tin plating 105. WhenMnO2 is fused with KOH andKN02, a coloured
(c) copper plating (d) zinc plating compound is formed, the product and its colour is
93. KgC^O? on heating with aqueous NaOH, gives (a) K2MnO4,green (b) KMnO4,purple
(a) CrO2- (b) Cr(OH)3 (c) Mn2O3, brown (d) Mn3O4, black
(c) CigO2,- (d) Cr(OH)2 106. (NH^C^O? on heating gives a gas which is also
94. The lanthanide contraction is responsible for the fact given by
that (a) heating NH4NO'22 (b) heating NH4NO3
(a) Zn and Y have about the same radii (c) Mg;jN2 + H2O (d) Na + HA
(b) Zr and Nb have similar oxidation state 107. The pair of compounds having metals in their highest
(c) Zr ahd Hf have about the same radii oxidation state is
(d) Zr and Zn have the same oxidation state
(a) MnO2,FeCl3
95. The electronic configuration of (Gd) (At. no. 64) is (b) [MnO4F,CrO2Cl2
(a) [Xe] 4/’8,5c?9,6s2 (b) [Xe] 4f\ 5d\ 6s2
(c) [Xe] 4/'6,5c?2‘ ,6s.22 (d) [Xe] 4f3,5d3,6s2 (c) [Fe(CN)6]’-,[Co(CN)3]
(d) [NiCl4f-,[CoCl4r
96. Which one of the following elements shows maximum
number of different oxidation states in its 108. The compound having tetrahedral geometry is
compounds? (a) [Ni(CN)4f- (b) [Pd(CN)4f-
(a) Eu (b) La (c) [PdCl4f- (d) [NiCl4f-
(c) Gd (d) Am
109. Spin only magnetic moment of the compound
97. Oxidation state of Fe inFe3O4 is Hg[Co(SCN)4 ] is
z x 3 ,, x 4
(a) - (b) - (a) V3 (b) V15
2 5
(c) V24 (d) V8
(c) 7 (d) I8 110. Which pair of compounds is expected to show similar
4 33
98. Bell-metal is an alloy of colour in aqueous medium?
(a) Cu + Pb (b) Cu + Sn (a) FeCl3 and CuCl2 (b) VOC12 and CuCl2
(c) Cu + Zn (d) Cu + Ni (c) VOC12 andFeCl2 (d) FeCl2 andMnCl2
99. Of the following transition metals, the maximum 111. A solution of a metal ion when treated with KI gives a
number of oxidation states are exhibited by red precipitate which dissolves in excess KI to give a
(a) chromium (Z = 24) (b) manganese (Z = 25) colourless solution. Moreover, the solution of metal
(c) iron (Z = 26) (d) titanium (Z = 22) ion on treatment with a solution of cobalt(II)
100. Philospher’s wool on heating with BaO at 1100oC thiocyanate gives rise to a deep blue crystalline
produce precipitate. The metal ion is
(a) Ba + ZnCl.2 (b) BaCdO2 (a) Pb2* (b) Hg.2‘ +
(c) BaZnO2 (d) BaO2 + Zn (c) Cu2* (d) Co!.2*
101. The nature of Fe2O3 is 112. Consider a titration of potassium dichromate
(a) acidic (b) basic solution with acidified Mohr’s salt solution usinj
(c) amphoteric (d) None of these diphenylamine as indicator. The number of moles d
102. The metal present in vitamin B12 is Mohr’s salt required per mole of dichromate is
(a) 3 (b) 4
(a) cobalt (b) iron
(c) 5 (d) 6
(c) manganese (d) magnesium
Chapter 16: The Transition and Inner Transition Elements | 707

113. Consider following statements, 121. When MnO2 is fused with KOH and KC1O'3, : a
I. The sizse of the lanthanide M 3+ ions decreases as coloured compound is formed, the product and its
the atomic number of M increases. colour is
II. Electronic spectra of lanthanides show very (a) K/jMnO^ green (b) KMnO4, purple
broad bands.
III. As with transition metals, coordination number (c) Mn2O3, brown (d) Mn3O4 black
six is very common in lanthanide complexes. 122. CrO2Cl2 is formed while testing
Which of the statements given above is/are correct? (a) NO3 (b) Ci­
(a) I only (b) I and II
(c) I and III (d) III only
te) Cr3+ (d) Fe3*
114. A red solid is insoluble in water. However, it becomes 123. The most abundant element is
soluble if some KI is added to water. Heating the red (a) Cu (b) Hg
solid in a test tube results in liberation of some violet (c) Cd (d) Fe
coloured fumes and droplets of a metal appear on the 124. Which represents correct comparison of the stability
cooler parts of the test tube. The red solid. of ions?
(a) (NH4)2Ci2O7 (b) HgLj (a) MnO; < Mn2+ (b) Cr2* < Cr3+
(c) HgO (d) Pb3O4 (c) CrOj- < Cr3+ (d) All of these
115. What would happen when as solution of potassium 125. On August 2003, IUPAC approved the name of the
chromate is treated with an excess of dilute nitric element with atomic number 110 as
acid? (a) Ds (b) Mt
(a) Cr3+ and C^O?- are formed (c) Uun (d) Nnu
(b) CijO^- and HjO are formed
126. Different (variable) oxidation state is shown by
(c) CrO^" is reduced to +3 state of Cr transition elements. It is due to the fact
(d) None of the above (a) (n - l)d electrons may be excited to ns orbital
116. Which one of the following nitrates will leave behind (b) ns electrons may be excited to (n - l)d orbitals
a metal on strong heating? (c) (n - l)d-electron may get involved along with ns
(a) Ferric nitrate (b) Copper nitrate electrons in bonding
(c) Manganese nitrate (d) Silver nitrate (d) none of the above facts is correct
117. Cerium (Z = 58) is an important member of 127. Select correct statement for the lesser number of
lanthanides. Which of the following statements about oxidations states in 3d-series.
cerium is incorrect? (a) In the beginning of the series it can be due to the
(a) The common oxidation states of cerium are +3 presence of too few electrons to loose or share
and +4 (b) Towards the end of the series, it can be ascribed
(b) The +3 oxidation states of cerium is more stable to the presence of too many electrons and thus
than the +4 state fewer empty orbitals to share electrons with the
(c) The +4 oxidation state of cerium is not known in ligands
solutions (c) Both (a) and (b) are correct
(d) Cerium (IV) acts as an oxidising agent (d) None of the above is correct
118. Calomel (Hg2Cl2) on reaction with ammonium 128. In which case(s) there is change in oxidation number
hydroxide gives (a) Aqueous solution ofCrC^- is acidified
(a) HgO (b) Hg2O
(c) NHj-Hg—Hg—Cl (d) HgNBLjCl (b) SO2 gas is passed into C^O^/H*
(c) C^O^- solution is made alkaline
119. Which of the following factors may be regarded as the
main cause of lanthanide contraction? (d) CrO2Cl2 is dissolved in NaOH
(a) Greater shielding of 5d-electron by 4/-electrons 129. Factors which affect the stability of the compounds
(b) Poorer shielding of 5d-electron by 4/-electrons are
(c) Effective shielding of one 4 /-electrons by another (a) the energy of sublimation
in the subshell (b) the lattice energy
(d) Poor shielding of one of 4/-electrons by another (c) the solvation energy
in the subshell (d) all of the above
120. Amongst the following, identify the species with an 130. Which forms protective and non-corrosive oxide
atom in +6 oxidation state layer?
(a) MnO4 (b) Cr(CN)t (a) Cr (b) Ni
(c) NiP?" (d) CrO2Cl2 (c) Zn (d) Cu
708 | Essential Inorganic Chemistry

131. First IE of 5d-elements are higher than those of 3d 138. There are three electrons unpaired in [Cotf^O^F*
and 4d-elements. This is due to and calculated value of magnetic moment is 3.87 BJl
(a) greater effective nuclear charge acting on outer which is quite different from the experimental valus
valence electrons of 4.40 BM. This is because of
(b) greater effective nuclear charge is experienced (a) increase in number of unpaired electrons
because of the weak shielding of the nucleus by (b) some contribution of the orbital motion of the
4/-electrons electron to the magnetic moment
(c) both (a) and (b) (c) change in orbital spin of the electron
(d) None of the above (d) d-d* transition
132. NH4C1 is used to clear metal surfaces because 139. Cu2+ has a stronger polarising power than that d
(a) it dissociates into NH3 and HC1 on heating Ca2+ because
(b) NH3 forms soluble complexes with the metal (a) Cu2+ ion is smaller than Ca2+ ion
(b) Ca2+ has inert gas configuration whereas Cu2*
(c) HC1 forms a volatile
ion does not
(d) None of the above (c) copper shows variable valency, calcium does not
133. Most transition metals (d) Cu2+ is smaller than Ca2+ ion and the d-electrons
I: forms sets of compounds which display different in Cu2+ ion shield the nucleus poorly
oxidation states of the metal. 140. Atoms of the transition elements are smaller than
II: form coloured ions in solution those of the s-block elements. This is because of
III: bum vigorously in oxygen (a) usual contraction in size across a horizontal
IV : replace H2 from dilute acids period
of these (b) orbital electrons added to the penultimate d-shell
(a) I, II, III are correct rather than to the outer shell of the atom
(b) II, III, IV and correct (c) both (a) and (b)
(c) I, II are correct (d) none of the above
(d) all are correct
141. Select correct order of sizes of different species.
134. A blue solution of copper sulphate becomes darker (a) Zr = Hf; Nb = Ta; Fe = Co = Ni
when treated with excess of ammonia. This is (b) Zr4+ < Zr; Nb3+ < Ta3+, Fe3+ < Fe2+ <Fe
because (c) Zr4+=Hf4+; Nb3+=Ta3+; Fe<Co<Ni
(a) ammonia molecules replace water molecules in (d) Zr4+ < Hf4* Nb3+ = Ta3+, Ni < Cu < Co
the solution
(b) ammonia is stronger ligand than water 142. The radii of the elements from chromium (Z = 24) to
(c) ammonia forms a stable complex ion copper (Z = 29) are very close to one another. This is
[Cu(NH3)4]2+ withCu2* ions. due to
(d) all of the above are correct (a) lanthanide contraction
135. Effective atomic number (EAN) of Fe in brown ring (b) the fact that successive addition of d-electrons
complex [Fe(H2O)5NOf+ screen the outer electrons (4s) from the inward
(a) 36 (b) 37 pull of the nucleus
(c) 38 (d) 39 (c) increase in radii due to increase in n is
compensated by decrease in radii due to increase
136. Select the correct relative stability. inZ
(a) [CuCCN),]3- >[Cd(CN)J3- (d) atomic radii do not remain constant but decrease
(b) PtClJ- > NiClJ- in a normal gradation
(c) [NiCl6f- > [PtCl6f- 143. The lanthanoid contraction is due to
(d) [Cd(CN),f- >[Cu(CN)4]3- (a) filling of 4f before 4d
(b) filling of 4d before 4f
137. Magnetic moment of [Ni(CN)4 f~ is zero but that of
(c) filling of 4f before 5d
[NifHjO^f* is 2.83 BM. (d) filling of 5d before 4f
It is because of
(a) ■ different oxidation state of Ni in two complexes 144. Magnetic moment of [Titf^O^ ]3+ is
(b) CN" is a strong ligand making two unpaired (a) 1.73 BM (b) 2.83 BM
electrons in Ni2+ paired while in [Ni(H2O)4f+, (c) 3.87 BM (d) 4.90 BM
two electrons remain unpaired H2O being weak 145. Magnetic moment of Fe is similar to that of
ligand (a) [Fe(H2O)6:r (b) [CrfHjOV* ,
(c) both (a) and (b) (c) both (a) and (b) (d) None of these
(d) none of the above
Chapter 16: The Transition and Inner Transition Elements | 709

146. The atomic numbers of vanadium (V), chromium 156. Which catalyst is matched according to its name and
(Cr), manganese (Mn) and iron are respectively 23, function?
24,25 and 26. Which one of these may be expected to Catalyst Name Function
have the highest second ionization enthalpy? (a) TiCI4 Fenton's reagent Oxidation of alcohols
(a) V (b) Cr (b) FeSO4/H2O:2 Ziegler-Natta Polythene preparation
(c) Mn (d) Fe (c) Pd/C Lindlar Hydrogen to give
147. There are three unpaired electrons in [Cod^Ogf* c/s-alkene
(d) Pt/PtO Adam Synthesis of CH3OH
and calculated value of magnetic moment is 3.87 BM
which is quite different from the experimental value 157. Select correct statement.
of 4.40 BM. This is because of (a) Fe and Mo atoms are present in nitrogen fixing
(a) increase in number of unpaired electrons enzymes
(b) some contribution of the orbital motion of the (b) A cobalt atom lies at the centre of the vitamin B12
electron to the magnetic moment coenzyme
(c) change in orbital spin of the electron (c) Fe atoms are involved in the ferrodoxins of
(d) d-d* transition photosynthetic process
(d) All of the above are correct statements
148. Increasing value of magnetic moments of (I) Ni(CO)4,
II: [TitHjOJef \ III: [VIHjO),]^, IV: [VIHjOIjf* is 158. Which is the set of nonstoichiometric compounds?
(a) NaCl, FeO, MgCl2
(a) I < II < III < IV (b) IV < III < II < I (b) FeO, CuS, VSe
(c) II < III < I < IV (d) II < I < III < IV (c) Fe3O4, NaCl, CuS
149. Which of the following is colourless? (d) CuCl, CuS, MgO
(a) (Zn(H2O)6f+ (b) [VIHjOIjf* 159. Which one of the following elements is the main
(c) [MnlHjOlg]3* (d) [Fe(H2O)6f+ metallic constituent of haemoglobin?
150. Which is the coloured ion? (a) Al (b) Cu
(a) [Cu(CN)4]3- (b) [SclHjOle]3* (c) Fe (d) Mn
(c) [MnlHjOlsJ3* (d) All of these 160. KMnO4 is the oxo salt of
151. Oxidation state of Mn in MnO4 is +7 indicating all (a) MnO2 (b) Mn2O7
electrons paired in Mn butMnO4 is coloured. It is due (c) MnO3 (d) Mn2O3
to 161. CrO3 liberates CO2 withNa2CO3 solution. Thus, CrO3
(a) charge transfer
(b) presence of unpaired electron in d-orbital in is
oxygen (a) an acidic oxide
(c) both (a) and (b) (b) a basic oxide
(d) none of the above (c) an amphoteric oxide
(d) a nonstoichiometric oxide
152. Which of the following is paramagnetic as well as
coloured ion? 162. VO2 is an amphoteric oxide and in acidic medium it
(a) Cu+ (b) Cu2+ forms
(c) Sc3+ (d) Ti4+ (a) VO2" (b) VO2
153. Select correct statement(s). (c) V3+ (d) VO^+
(a) Coloured the ion arises due to d-d* transition 163. CrO^+ is the cation from
(b) Colour we observe is the complimentary colour
absorbed by the compound (a) CrO l(b) C^Og
(c) Both (a) and (b) are correct (c) CrO5 I(d) CrO3
(d) None of the above is correct 164. The oxide C&Og is
154. Nonstoichiometry is shown (a) acidic I(b) amphoteric
(a) due to variable valency of transition elements (c) basic l(d) ozonide
(b) due to defects in solid structures
(c) due to both (a) and (b) 165. At pH = 4, CrjO^- exists as
(d) due to none of the above (a) CrO^“ (b) CrO3
155. Cementite is (c) CrO£+ (d) CioO2;-
(a) interstitial compound of iron and carbon 166. At pH = 12, Ci^O^- changes to
(b) an alloy of Fe and Cr
(c) a compound resembling cement (a) CrO3 (b) CrO;+
(d) an ore of iron (c) CrOj" (d) no change
710 | Essential Inorganic Chemistry

167. In acidic medium H2O2 changes K2Cr2O7 to CrO5 177. ZnO in CH3COOH gives bluish white ppt. with X Xis
(deep violet solution in ether) having two peroxy (a) K4[Fe(CN)6] (b) K3[Fe(CN)6]
linkage. Oxidation number of Cr in CrO5 is (c) Na2[Fe(CN)5NO] (d) Fe(CO)5
(a) +10 (b) -10 178. Select correct statement.
(c) +4 (d) +6 (a) ZnSO4 • THjO is called Epsom salt
168. In alkaline H2O2, Cr2C£_ changes to tetraperoxo (b) ZnSO4 • 7H2O is isomorphous with Glauber’s salt
species ... having oxidation number of Cr as ... (c) ZnSO4 • 71^0 with BaS is called lithophone
(a) CrOj" 6 (d) All the above are correct statements
(b) CrO5 6 179. Silver nitrate solution is kept in brown bottles in the
(c) CrOj" 5 laboratory because
(d) CrO|’ 11 (a) it reacts with ordinary bottles
(b) ordinary bottles catalyst the decomposition
169. MnO^- can be converted to MnO4 (c) the materials of brown bottles do not react with it
(a) by oxidation with Cl2 (d) brown bottles cut the passage of light through it
(b) by electrochemical oxidation at anode
180. On addition of AgNO3 to four different test tubes
(c) by both (a) and (b) methods
(d) by none of the above methods containing different solutions, one of them gave a
170. In acidic medium MnO^" white precipitate. It may be
(a) CHC13 (b) CaCl2
(a) disproportionates to MnO2 and MnO4
(c) KNO3 (d) CC14
(b) is oxidised to MnO4
(c) is reduced to MnO2 181. Select correct statement.
(a) PH3 reduces AgNO3 to metallic Ag
(d) is reduced to Mn2+
(b) Organic tissues turn AgNO3 black by reducing it
171. Alkaline KMnO4 (Baeyer’s reagent) can be used to to Ag
test unsaturation in (A). (c) AgCN is soluble in KCN
^X/CH=CH2 (d) All of the above are correct statements
182. Hypo (Na2S2O3)
(a) dissolves AgBr in photographic plate
(A) (b) gives white precipitate with AgN03; white
In this case precipitate changes to black on dilution
(a) unsaturation in side-chain is affected (c) gives both reactions
(b) unsaturation in benzene nucleus is affected (d) gives none of the above reactions
(c) unsaturation in both is affected 183. Sugar in urine sample can be detected by
(d) Baeyer’s reagent cannot be used I: Fehling’s solution
172. KMnO4 spot can be bleached by II: Benedict’s solution
(a) HA/H* (b) SO2 /H+ III: Tollen’s solution.
(c) C20j“/H+ (d) all of these Select correct alternate
173. Traces of MnO4 in cone. HjSC^ may change to (a) I, II, III (b) I, III
(c) I, II (d) II, III
(a) Mn2O7 (b) MnO3
184. If a person is asked to prepare the blue print of a
(c) MnO2 (d) MnO2-
building plan, he can use
174. A white solid Y, on heating gives off a gas which turns (a) FeCl3+K4[Fe(CN)6]
lime water milky; the residue is yellow when hot;
(b) FeCl2 +K3[Fe(CN)6]
white when cold. The solid Y is probably
(a) ZnCO3 (b) PbCO3 (c) FeCl2 +K4[Fe(CN)6]
(c) ZnSO4 (d) Zn(NO3)2 (d) Fe(Ct) + K3[Fe(CN)6 ] Ct is citrate
175. Aqueous ZnO can neutralise HC1 as well as NaOH 185. If FeCl3 • GHjO would exist as a complex, one moled
solution. Thus, ZnO is an it in aqueous solution on reaction with excess d
(a) acidic oxide (b) basic oxide AgNO3 forms
(c) amphoteric oxide (d) amphiprotic oxide (a) 1 mole of AgCl (white precipitate)
176. Which is not blackened by atmosphere I^S? (b) 2 moles of AgCl
(a) Pb(CH3COO)2 (b) Zn(CH3COO)2 (c) 3 moles of AgCl
(c) Cu(CH3COO)2 (d) Hg(CH3COO)2 (d) no reaction
Chapter 16: The Transition and Inner Transition Elements | 711

186. K3[Fe(CNj6 ] is used as an external indicator in the 195. [FetHjOgNOf * is brown-ring complex inNO3.In this
dichromate estimation of Fe2+. Following change is complex
observed (a) NO transfers its electron toFe2+ so that there are
(a) colourless to blue (b) blue to red three electrons unpaired making iron as Fe(I)
(c) colourless to red (d) blue to colourless and nitrosyl as NO+
(b) magnetic moment of Fe is 3.87 BM
187. If zinc pieces are added yellow FeCl3 solution, it turns (c) the colour is due to charge transfer
to very light green. It is because (d) all of the above are correct statements
(a) FeCl3 solution is acidic due to hydrolysis and zinc 196. K2Ci2O7/H+ changes to green by
added produces nascent hydrogen causing
reduction of FeCl3 to FeCl2 (a) Fe2* (b) SOt
(b) FeCl3 solution is basic and zinc added produces (c) both (a) and (b) (d) none of these
H2 which reduces FeCl3 toFeCl2 197. Some of the following reagents are used as primary
(c) Zinc reduces FeCl3 to FeCl2 standard.
(d) none of the above is correct I: KMnO4; H : NaOH; III: K2C^O7;
188. At300°C,FeCl,
7 o
IV:FeSO4-(NH4)2SO4-6H2O; V : H,G2O4 -2^0
(a) decomposes into FeCl2 and Cl2 Select primary standard
(b) decomposes into Fe and Cl2 (a) all except II, IV (b) all except I, II
(c) sublimes to give liquid FeCl3 (c) all except I, II, III (d) only IV
(d) sublimes to give gaseous dimer (FeCl3)2 198. One mole of FeCjQj is oxidised by KMnO4 in acidic
189. The compound that get oxidised even on exposure to medium. Number of moles of KMnO4 used are
air is (a) 0.6 mol (b) 1.2 mol
(a) Fe2O3 (b) FeCl2 (c) 0.4 mol (d) 1 mol
(c) FeCl3 (d) Fe2(SO4)3 199. Fehling’s solution consists of two separate alkaline
solutions. One solution contains CuSO4. The other
190. Rust contains Fe^ • HjO. Rust spots can be removed solution contains.
by (a) NaHC03 (b) KNaC4H4O6
(a) Na2S2O3 (hypo) (b) SO2 (c) KHCO3 (d) KoCOa
(c) HjCjOt (oxalic acid) (d) KMnO4
200. If HjS gas is passed into a solution of Cu2+, Cd2+
191. Select correct statement(s). having excess of KCN
(a) a-Fe2O3 has hexagonally close-packed lattice of (a) CuS and CdS both are precipitated
O2" ions with Fe3+ ions in two-thirds of the
(b) soluble complex [Cu(CN)4 J3- and [Cd(CN)4 f" are
octahedra holes.
formed and no effect of passing H2S gas
(b) y-Fe2O3 has cubic close-packed arrangement of
(c) soluble complex [Cu(CN)4 J3- and [Cd(CN)4 f" as
O2’ ion with Fe3+ ions randomly distributed in
are formed, of which CdS is precipitated as yellow
both the octahedral and tetrahedral sites.
ppt.
(c) Fe3O4, Fe2O3 and FeO all and tend to be
(d) soluble complex [Cu(CN)4 ]3“ and [Cd(CN)4 f" are
non-stoichiometric.
formed of which CuS is precipitated as black ppt.
(d) All the above are correct statements
201. When K^CrQj is added to CuSO4 solution, there is
192. On passing Cl2 gas into alkaline hydrated Fe2O3
formation ofCuCrO4 as well as CuCi^O?. Formation of
solution CuC^O7 is due to
(a) red purple solution of Na2FeO4 is formed (a) basic nature of CuSO4 solution which converts
(b) colourless NaFeO2 is formed due to dissolution of CrO^~ toCi^O}
Fe2O3 into NaOH (b) acidic nature of CuSO4 solution which converts
(c) Fe(OH)3 is formed CrO^~ to Ci^O^-
(d) no reaction takes place (c) CuSO4 has the typical property of converting
193. Fe3O4 is called CuCrO4 is formed to CuC^O?
(a) haematite (b) magnetite (d) no CuCloO; is formed
(c) casserite (d) dolomite 202. In an atmosphere with industrial smog, Cu corrodes
194. Isomorphous salts are to
(a) green vitriol, blue vitriol and Epsom salt (a) basic sulphate Cu2(OH)2SO4
(b) green, vitriol, white vitriol and blue vitriol (b) basic carbonate Cu2(OH)2CO3
(c) green vitriol, white vitriol and Epsom salt (c) both (a) and (b)
(d) blue vitriol, white vitriol and Epsom salt (d) none of the above
712 | Essentia/ Inorganic Chemistry

203. Na2S2O3 (b) TiCl„, [Ni(CN)4 f ’, CO, [NMHjO), f'


(a) reduces Cu2+ to Cu+ (c) TiCl4,[Ni(CN)4“], CO
(b) reduces I2 to I" (d) TiCl4,[Ni(CN)4f-,O2
(c) complexes AgBr as [Ag(S2O3)213-
212. Which are not blackend by atmospheric HaS?
(d) undergoes all the above are correct
(a) TiO2 (b) ZnO
204. The element which forms ions in dimeric state is (c) ZnSO4 + BaS (d) All of these
(a) iron (b) mercury
(c) cadmium (d) nickel 213. Elements after atomic number 92 are called
(a) lanthanoids
205. Mixture is supposed to contain nitrates of Pb2+ and (b) actinoids
Hg2+. On adding dil. HC1 both are precipitated as (c) inner-transition elements
white precipitate (d) transuranium elements
Select correct statement(s).
(a) PbCl2 (white precipitate) is soluble in hot water 214. Ln3+ (trivalent lanthanides ions) have EC
and formation of yellow precipitate on adding KI (a) [Xe]4/'1to[Xe]4/'14
to hot solution confirms Pb2+ (b) [Xe] 4d14/'1 to [Xe] 4d14/'14
(b) Hg2Cl2 is blackened by NH3, and changes to (c) [Xel4d24/° to[Xe]4d14f14
HgCl2 by aqua-regia. HgCl2 can be detected by KI (d) [Xe]4f° to[Xe]4/14
when orange precipitate soluble in excess of KI is 215. A certain metal will liberate hydrogen from dilute
formed. acids. It will react with water to form hydrogen only
(c) both tests are conclusive when the metal is heated and the water is in the form
(d) none of the tests is conclusive of steam. The metal is probably
206. Hg2Cl2 ionises as ........ and cation has ........ (a) iron (b) potassium
unpaired electron(s) (c) copper (d) mercury
(a) 2Hg+ and2Cl",two (b) Hg|+ and 2C1 , two 216. The magnetic moment p, of transition metals is
(c) Hg^+and2Cr, one (d) Hg2+ and2Cr, no related to the number of unpaired electrons, n as
207. Alkaline NH4C1 solution reacts with KgHg^ (a) p. = n(n + 2)2 (b) p = n2(n + 2)
(Nessler’s reagent) forming (c) I1 = (d) I1 = +
(a) (HgNHaCl + Hg) — black (n + 2)
F /Hg
(b) O<
H 217. Misch metal is
I — brown iodide of Millon’s (a) an alloy of lathanide and copper
XHg H
base (b) an alloy of lanthanide and nickel
(c) an alloy of lanthanide, iron and carbon
(c) KJO—Hg—NHjCl] I — brown iodide of (d) an alloy of calcium and copper
Millon’s base 218. Which of the following have completely filled 5/
(d) none of the above
orbitals?
208. HgCl2 is soluble in (a) No (b) Lr
(a) cold water (c) Both (a) and (b) (d) None of these
(b) NaCl solution due to formation of HgCl2’ 219. An3+ (trivalent actinoid ions) have configuration
(c) both (a) and (d) (a) 5f° toS/44 (b) 5/’1to5/'14
(d) none of the above (c) S^toS/14 (d) 5f° to 5f13
209. In the following reaction 220. Consider following radioactive decay of
nCH2=CH2 > [—CHz—CHa—]71 1: 92 U > 90Th
Catalyst is
II: qn
90 Th > 88
(a) Adam (b) Ziegler-Natta
(c) Ni/Pd (d) Fe/Mo In which case group of the parent and daughter
210. TiCl4 is a element remains unchanged?
(a) in I (b) in II
(a) Bronsted-Lowry acid (b) Bronsted-Lowry base (c) in both (a) and (b) (d) in none of these
(c) Lewis acid (d) Lewis base
221. Select correct statement(s).
211. Which of the following are sets of diamagnetic (a) Pu4+ disproportionates to Pu3+ and PuO^ in
species? strongly acidic solution
(a) TiCl4,02,[Ni(CN)4f[Ni^O),f+ (b) Maximum oxidation state of Np is+7
Chapter 16 . The Transition and Inner Transition Elements | 713

(c) UO^+ is stable 230. Ferric sulphate is represented by which formula?


(d) All of the above are correct statement (a) FeSO4 (b) FeSO3
222. Uranium reacts with cone. HNO'3 (c) Fe(SO4)2 (d) Fe2(SO4)3
(a) to give U(NO3)4 and H2 231. The transition metal with least atomic number
(b) to give U(NO3)2 and NO2 (a) Os (b) Zr
(c) UO2(NO3)2 andH-j (c) Pt (d) Ru
(d) to make metal passive 232. Lanthanide and actinides resemble in
223. Which of the following statements is not correct? (a) electronic configuration
(a) La(OH)3 is less basic than Lu(OH)3 (b) oxidation state
(b) In lanthanide series ionic radius of Ln3+ ion (c) ionisation energy
decreases (d) formation of complexes
(c) La is actually an element of transition series 233. Stainless steel is an alloy of
rather lanthanide
(a) Cu (b) Ni and Cr
(d) Atomic radius of Zr and Hf are same because of
lanthanide contraction (c) Mn (d) Zn
224. Lanthanoids are 234. Solder is an alloy of
(a) 14 elements in the sixth period (atomic number = (a) 70% lead 30% tin (b) 30% lead, 70% tin
90 to 103) that are filling 4/-sublevel (c) 80% lead, 20% tin (d) 90% copper, 10% tin
(b) 14 elements in the seventh period (atomic no. = 235. Which of the following is a highly corrosive salt?
90 to 103) that are filling 5/-sublevel
(a) FeCl2 (b) PbCl2
(c) 14 elements in the sixth period (atomic no. = 58 to
71) that are filling 4/-sublevel (c) Hg2Cl2 (d) HgCl2
(d) 14 elements in the seventh period (atomic no. = 236. Which of the following is potassium ferricyanide?
58 to 71) that are filling 4/-sublevel (a) K4[Fe(CN)6)] (b) K3[Fe(CN)6]
225. Four successive members of the first row transition (c) K3[Fe(CN)3] (d) K3[Fe(CN)4]
elements are listed below with their atomic numbers.
Which one of them is expected to have the highest 237. German silver is an alloy of
third ionization enthalphy? (a) Fe, Cr, Ni (b) Ag, Cu, Au
(a) Vanadium (Z = 23) (b) Chromium (Z = 24) (c) Cu, Zn, Ni (d) Cu, Zn, Sn
(c) Iron (Z = 26) (d) Manganese (Z = 25) 238. The Mohr’s salt is shown by
226. The number of moles of KMnO4 reduced by one mole (a) FeSO4(NH4)2SO4 GHjO
of KI in alkaline medium is (b) FeSO4(NH3)2SO4
(a) one fifth (b) five (c) K2SO4Al2(SO4)3 • 241^0
(c) one (d) two (d) FeSO2(NH2)4SO4
227. The main reason for larger number of oxidation state 239. The composition of duralumin is
exhibited by the actinides than that corresponding (a) Al 94%, Mg 6%
lanthanides, is (b) Cu 56%, Zn 24%, Ni 20%
(a) lesser energy difference between 5f and 6d (c) Cu 95, Al 5%
orbitals than between 4/ and 5d-orbitals (d) Al 95%, Cu 4%, Mn 0.5%, Mg 0.5%
(b) larger atomic size of actinides than the
240. Lanthanide for which +11 and +III oxidation states
lanthanides
(c) more energy difference between 5f and 6d are common is
orbitals than between 4/ and 5d-orbitals (a) La (b) Nd
(d) greater reactive nature of the actinides than the (c) Ce (d) Eu
lanthanides 241. The colourless species is
228. The common oxidation state of the elements of (a) VC13 (b) VOSO4
lanthanide series is (c) Na3VO4 (d) [Y(H2O)6]SO4 HjO
(a) +1 (b) +3 242. MnQj- (1 mol) in ineutral aqueous medium
(c) +4 (d) +6
disproportionates to
229. The metal that does not displace hydrogen from an 2 1
acid is (a) - mol of Mn07 and - mol of MnO2
3 4 3 2
(a) Al (b) Ca 1 2
(c) Zn (d) Hg (b) - mol of MnO4 and - mol of MnO2
3 3
714 ] Essential Inorganic Chemistry

(c) - mol of Mn2O7 and - mol ofMnO2 255. The outer electronic configuration of transitional
3 7 elements is
2 1 (a) (n - 1) s2nd1-2
(d) - mol of Mn2O7 and - mol of Mn02
3 3 (b) (n + l)s2nd1-5
243. An aqueous solution of CoCl2 on addition of excess of (c) (n - l)s2p6(n - l)d1-10 , ns
(d) ns2(n + l)d1-10
concentrated HC1 turn blue due to formation of
(a) [Co(H2O)4C12] (b) [Co(H2O)2CI4f- 256. The highest magnetic moment will be shown by
(c) [CoCl4f- (d) [Co(H2O)2C12] (a) Ni (b) Co
(c) Fe (d) Sc
244. The diamagnetic species is
(a) [Ni(CN)4f‘ (b) [NiCl,f- 257. Which of the following is not a member (T
(c) [CoCl4f- (d) [CoFj- 3d-transition series?
(a) Fe (b) Co
245. Among the following pairs of ions, the lower oxidation
(c) Au (d) Cu
state in aqueous solutions is more stable than the
other in 258. Which is most soluble in water?
(a) Ti+,Ti3+ (b) Cu+,Cu2+ (a) AgBr (b) AgCl
(c) Cr^Cr3* (d) V2+,VO2+ (c) AgF (d) Agl
246. The colour imparted by Co(II) compounds to glass is 259. Addition of SnCl2 to HgCl2 give precipitate
(a) green (b) deep blue (a) white turning to red
(c) yellow (d) red (b) white turning to grey
247. Given below, catalyst and corresponding process/ (c) black turning to white
reaction are matched. The mismatch is (d) None of the above
(a) [RhCl(PPh3)2]: Hydrogenation 260. The stability of ferric ion is due to
(b) TiCl4 + A1(C2H5)3 : Polymerisation (a) half filled /-orbitals
(c) V2O5 : Haber-Bosch process (b) half filled d-orbitals
(d) Nickel: Hydrogenation (c) completely filled /-orbitals
248. KI and CuSO4 solutions on mixing produce (d) completely filled d-orbitals
(a) Ct^L, + K2SO4 (b) Ct^^ + KI3 + K2SO4 261. Which of the following lanthanide is commonly used?
(c) Cui, +K2SO4 (d) CuL, +KI3 + K2SO4 (a) Lanthanium (b) Nobelium
249. The reactivity of transition elements decreases with (c) Thorium (d) Cerium
(a) the decrease in the atomic number Heat■» 4K2CrO4 +3O2 + X; in the above
262. ^C^
(b) the increase in the atomic number
(c) low heat of hydration reaction X is
(a) CrO3 (b) CijO7
(d) None of the above
(c) Ci^O3 (d) CrO5
250. Catalyst used in making I^SQj in contact process is
263. The colour of FeSO4 ■ (NH4)2SO4 ■ 6^0 is
(a) V2O5 (b) FeA
(c) Cc>03 (d) CrO3 (a) blue (b) green
(c) white (d) red
251. The incorrect configuration is
(a) K = [Ar] 4s1 (b) Cr = [Ar] 3d5, 4s1 264. Which of the following is most stable among Cu+,Fe\
(c) Cr = [Ar]3d4,4s.22 (d) Cu = [Ar]3d10, 4s1 Fe2+andFe3+?
252. By annealing, steel (a) Cu+ (b) Fe+
(a) becomes soft (c) Fe2+ (d) Fe3+
(b) becomes liquid 265. Brown glass and cement have, which element
(c) becomes hard and brittle common in them?
(d) is covered with a thin film of Fe3O4 (a) Fe (b) Al
253. Transition metals show paramagnetism due to (c) Na (d) All of these
(a) characteristic configuration
266. The atomic size of cerium and promethium is quite
(b) high lattice energy
close, due to
(c) variable oxidation states
• (a) they are in same period in Periodic Table
(d) unpaired electrons
(b) they electronic configuration is same
254. The oxides, CrO3, MoO3 and W03 are strongly (c) /'-electrons have poor shielding effect
(a) neutral (b) acidic (d) nuclear charge is higher on cerium than
(c) basic (d) None of these promethium
Chapter 16: The Transition and Inner Transition Elements | 715
■1 267. The trivalent ion having largest size is 279. Which of the following chloride is water insoluble?
(a) Ti (b) Zr (a) HC1 (b) AgCl
(c) Hf (d) La (c) Both (a) and (b) (d) None of these
268. The transition metal used as a catalyst is 280. Which of the following dissolves in hot cone. NaOH
(a) nickel (b) platinum solution?
(c) cobalt (d) all of these (a) Fe (b) Zn
269. Transition elements does not show (c) Cr (d) Ag
(a) paramagnetism (b) colour 281. Reason of passivity of iron is
(c) fixed valency (d) all of these (a) Fe2O3 (b) Fe3O4
270. Aluminium appears like gold when mixed with (c) FeO (d) Fe2O3 -3H2O
(a) 90% Cu (b) 75% Ni 282. Which is wrongly matched?
(c) 80% Sn (d) 80% Co (a) Duralumin — Al + Cu + Mg + Mn
271. Iron is dropped in very dil. HNO3, it gives (b) Alnico — Fe + Al + Ni + Cu
(c) German silver — Cu + Zn+Ni
(a) ferric nitrate (d) Monel metal — Cu + Zn + Sn
(b) ferric nitrate and NO2
(c) ferrous nitrate and ammonium nitrate 283. Formula of green vitriol oil is
(d) ferrous nitrate and nitric oxide (a) FeSO4 ■ 7^0 (b) MgSO4 • 711,0
(c) ZnSO4 ■ 7^0 (d) CuSO4 ■ SHjO
272. Stainless steel contains
(a) Fe+Cr + Cu (b) Fe+C + Ni 284. Which of the following is called white vitriol?
(c) Fe + Cr + Ni (d) Fe + Ni + Cu (a) ZnCl2 (b) MgSO4-711,0
(c) ZnSO4 -7H2O (d) A12(SO4)3
273. Which of the following alloys is used for making '
magnets for hearing aids? 285. Which of the following is not an actinide?
(a) Alnico (b) German silver (a) Curium (b) Californium
(c) Invar (d) Monel metal (c) Uranium (d) Terbium
274. Cuprous ion is colourless while cupric ion is coloured 286. Europium is
because (a) s-block element (b) p-block element
(a) both have half-filled p- and d-orbitals (c) d-block element (d) /-block element
(b) both have unpaired electrons in d-orbitals 287. Which of the following is not an actinoid?
(c) cuprous ion has incomplete d-orbital and cupric (a) Am (b) Cm
ion has a complete d-orbital (c) Fm (d) Tm
(d) cuprous ion as a complete d-orbital and cupric ion
288. Most oxidising agent is
has an incomplete d-orbital
(a) [WO4f- (b) [CrO4f-
275. Stainless steel does not rust because (c) [CeO4f- (d) [MnO4f-
(a) chromium and nickel combine with iron
289. Which of the following mixture is chromium acid?
(b) chromium forms an oxide layer and protects iron
from rusting (a) KoC^O? +HC1 (b) K2SO4 +conc.H2S0'44
(c) nickel present in it, does not rust (c) K2Ci20'7 +conc.H2SO4 (d) H,SO4 +HC1
(d) iron forms a hard chemical compound with
290. In reduction of dichromate by Fe(II), the number of
chromium present in it electrons involved per chromium atom is
276. Which of the following pairs of element cannot form (a) 2 (b) 3
an alloy? (0 4 (d) 1
(a) Zn, Cu (b) Fe, Hg
291. Which alloy contains Cu, Sn and Zn?
(c) Fe, C (d) Hg, Na (a) Gun metal (b) Solder
277. Which of the following types of metals form the most (c) Type metal (d) Bronze
efficient catalysts? 292. Finely divided iron combines with CO to give
(a) Alkali metals (b) Alkaline earth metals
(a) Fe(CO)5 (b) Fe^CO),,
(c) Transition metals (d) All of these
(c) Fe2(CO)12 (d) Fe(CO)6
278. Impure metal forms volatile compound (X) with CO
293. Green vitriol is formed by
and then (X) gives pure metal on heating. Metal is
(a) FeS2 + H,0 + O2 (b) FeS2 +1^0 + 002
(a) Cu (b) Fe
(c) Ni (d) Pt (c) FeS2 + CO + CO2 (d) FeS2+CO
716 | Essential Inorganic Chemistry

294. Thermite is a mixture of (c) In ammoniacal solution, (NH3)3CrO4 is formed


(a) MgO + Al (b) Fe2O3 + Al (d) CrO^- changes to Cr2O^“ by oxidation
(c) Zn + CaCO3 (d) Zn + P2O5
■ 9. For CrO3 following is not true statement.
295. Which plays a major role in the formation of complex (a) It is called chromic acid
compound? (b) It is colourless due to 3d° configuration
(a) Transition metal (c) It is bright orange solid and colour arises due to
(b) Lanthanides and actinides charge transfer
(c) Representative elements (d) It is toxic and corrosive
(d) p-block element
10. Lanthanoids are
(a) 14 elements in the sixth period (atomic no. = 90 to
One or More Than One Options Correct 103) that are filling 4/-sublevel
1. When KCN is added to CuSO4 solution (b) 14 elements in the seventh period (atomic no. =
(a) KCN acts as a reducing agent 90 to 103) that are filling 5/-sublevel
(b) KCN acts as a complexing agent (c) 14 elements in the sixth period (atomic no. = 58 to
(c) complex K3[Cu(CN)4 ] is formed 71) that are filling 4/-sublevel
(d) complex K, [Cu(CN)4 ] is formed (d) 14 elements in the seventh period (atomic no. =
58 to 71) that are filling 4/-sublevel
2. Which will not give metal on heating?
(a) Ag2CO3 (b) ZnCO3 11. WhenCO2 is passed into aqueous
(c) HgO (d) CuO (a) Na2CrO4 solution, its yellow colour changes to
3. Electron transfer from Fe(H2O)g+ to FedtjO)!* is orange
(b) KaMnO.j solution, it disproportionates to KMnO,
likely to occur via
(a) d-d transition and MnO2
(b) inner sphere electron transfer (c) Na2Ci2O7 solution, its orange colour changes to
(c) SN1 mechanism green
(d) outer sphere electron transfer (d) KMnO4 solution, its pink colour changes to green
4. Pyrolusite isMnO2 used to prepare KMnO4. Steps are 12. Select incorrect statement(s).
(a) Nickel (II) compounds tend to be
MnO2 —U MnO2- — ■> MnO4 thermodynamically more stable than
Steps I and II are platinum (II)
(a) fuse with KOH/air, electrolytic oxidation (b) Platinum (IV) compounds are relatively more
(b) fuse with KOH/air, electrolytic reduction stable than nickel (IV) compounds
(c) fuse with cone. HNO3/air, electrolytic reduction (c) KjPt^Clg exists
(d) dissolve in HjO, oxidation (d) K2NiIVCl6 exists

5. Fe is made passive by 13. Standard reduction electrode potential atZnz+/Znis


(a) dil.HjSO, (b) dil. HC1 -0.76 V. This means
(c) aqua-regia (d) cone. H2SO4 (a) ZnO is reduced to Zn by H2
6. N2 gas can be obtained by heating (b) Zn liberates with cone, acids
(c) Zn is generally the anode in an electrochemical
(a) (NH4)2Ci2O7 (b) NH4NO2
cell
(c) NH4VO3 (d) NH4NO3
(d) Zn is generally the cathode in an electrochemical
7. Which are correct statements about KMnO4? cell
(a) Its solution is unstable in acidic medium 14. Select correct statement(s) about [CoCig]4- complex
(b) Its small quantity added to cone. IH^SC^, a green
(a) It is paramagnetic
coloured solution containing MnO3 ions is formed
(b) It is a low-spin complex
(c) MnO4 changes toMn2+ in basic solution
(c) Oxidation number of cobalt is -4
(d) It is self-indicator in Fe2+ or C^O^- titration
(d) The coordination number of cobalt is 6
8. Which are correct statements?
15. Why does aqueous Fe(III) ion develop intense red
(a) In less acidic solution and HjQj gives
colour when it reacts with SCN“ ion while Fe(II) ion
violet coloured diamagnetic [CrO(O2)(OH)r ion
does not?
(b) In alkaline HjC^, K^CrOg (with tetraperoxo
species [Cr(O2)4 ]3-) is formed (a) Fe(III) ion forms a charge transfer complex with
SCN" ions
Chapter 16: The Transition and Inner Transition Elements | 717

(b) Fe(III) is reduced to Fe(I) which is deep red in 3. When a black powder (A) was heated strongly, it
colour gave off a colourless non-inflammable gas. When (A)
(c) SCN" ion is oxidised to CN~ which forms red was boiled with excess dil. H2SO4 and a little ,
coloured complex with Fe(III) ion a non-inflammable odourless gas was evolved. The
(d) SCN" does not form any complex with Fe(III) ion solution formed gave no precipitate when saturated
with HjS, but this saturated solution gave a light
16. Select correct statement(s). coloured precipitate when NH4OH solution was
(a) Stabilities of variable oxidation states can be added to it. A small sample of (A) was warmed with
explained by standard electrode potential excess of cone. HC1 and the gas liberated was passed
(b) Cr2+ is unstable and Cr3+ is stable in aqueous into a cone. KI solution. This solution was made up to
solution 200 mL; 20 mL of this solution was found to required
(c) Fe2+ is stable and Fe3+ is unstable is aerated 180 mL of 0.1 N Na2S2O3 solution iodometrically.
water f- Identify (A) and calculate the weight of (A) taken.
(d) All of the above are correct statements Also write necessary reactions.
17. Select correct statement(s). 4. A monomeric compound of cobalt gives the following
data on quantitative analysis :
(a) When FeCl3 solution is added to K4[Fe(CN)6]
Co3+ :21.24% Cl" : 12.81% NH33 :24.77%
solution, in addition to Fein[Fen(CN)6r, SOj": 34.65% 1120:6.53%
FeII[FeIII(CN)6]“ is also formed due to side redox What is empirical formula of the complex?
reaction
5. One mole of the aqueous solution of the complex
(b) When FeCl2 solution is added to K3[Fe(CN)6] CoClj SHgO when treated with excess of aqueous
solution, in addition to Fen[Fen(CN)6]~ AgNO3 solution gave one mole of white precipitate of
Feni[Fen(CN)6r is also formed due to side redox AgCl. What is the complex? (For this you should
reaction know that how many Cl" ions are not coordinated to
(c) Feni[Fen(CN)6]" is paramagnetic while Co3+— naturally one which is precipitated as AgCl)
Fen[FeUI(CN)6r is diamagnetic 6. For the following cell
(d) Feni[Fen(CN)6]“ is diamagnetic while 0.005 M 0.05 M
Fen[Fein(CN)6]“ is paramagnetic Hg mercurous nitrate mecurous nitrate Hg
in 0.1 MHNO3 in 0.1 MHN03

Brain Twisters EMF is 0.0295 V. What is charge on mercurous ion?


1. A 0.81 g sample of pyrolusite ore (impure MnO2) is 7. A solution made by dissolving 0.875 g ofCo(NH3)4Cl3
in 25.0 g of water freezes at -0.56°C. Calculate the
treated with 1.651 g of oxalic acid (f^CjC^ I^O) in
number of moles of ions produced when 1 mole of
an acidic medium. Following this reaction the excess Co(NH3)4C13 is dissolved in water, and suggest a
oxalic acid is titrated with 0.1 M KMnO4, 30.6 mL structure for the complex ion present in the
being required. What is the % of MnO2 in the ore? compound. Molal depression constant for water
(Mn = 55)
is l.SOKkgmol-1 (molar mass of solute = 235.5 g mol"1)
H^CA +MnO2 +2H+ ---- > Mn2+ +2H2O + 2CO2
+2MnO4 +6H+ ---- > 2Mn2++8H2O Passage Comprehension Questions
+ 10CO9
Passage 1
2. A steel sample is to be analysed for Cr and Mn
Read, the following report from a science journal and
simultaneously. By suitable treatment the Cr is answer the questions at the end of it.
oxidised to Ci^O^" and Mn to MnO4. A 10.0 g of sample CNIC (Commission on Nomenclature of
of steel is used to produce 250.0 mL of solution Inorganic Chemistry) appointed by IUPAC in 1994,
containing Ci^O^" and MnO4. A 10.0 mL portion of approved official names for elements with atomic number
this solution is added to a BaCl2 solution, and by 104 to 109. With Z > 109, IUPAC recommended a
proper adjusting the acidity, the Cr is precipitated as nomenclature to be followed for naming these elements
BaCrO4; 0.0549 g is obtained. A second 10 mL of the until their names are officially recognised. This
nomenclature is based on the Latin words for the atomic
solution requires exactly 15.95 mL of 0.0750 M numbers of the elements.
standard Fe2+ solution for its titration is acid Very recently, on August 16, 2003, IUPAC approved
solution. Calculate the % Mn and % Cr in the steel the name for the superheavy element which was first
sample. (Ba = 137, Cr = 52, Mn = 55) produced in 1994.
718 | Essential Inorganic Chemistry

1. Atomic number of the recently discovered element is 3. From the table, atomic radii of Zr and Hf and those
(a) 107 (b) 108 Nb and Ta are same. It is due to
(c) 109 (d) 110 (a) lanthanide contraction
(b) the fact that increase in size due to increase in ®
2. Name and symbol given to this new element is is compensated by decrease in size due
(a) Darmstadtium Ds
(b) Bohrium Bh increase in Z based on equation rn =
(c) Hassnium Hs (c) both of the above
(d) Meitnerium Mt (d) none of the above
3. IUPAC nomenclature of element with atomic number
110 is Passage 3
(a) Unu (b) Uun Observe the following data and answer the question!
at the end of it:
(c) Unn (d) Uum
A characteristic property of the d-transition metals is
their ability to exhibit several oxidation states. The
Passage 2 stability of a given oxidation state depends on the nature
From the data given in the table, answer the following of the element with which the transition metal is
questions: combined. The highest oxidation states are found in the
compounds of fluorides and oxides because of most
Covalent Radii of the Transition Elements (pm) electronegative nature of F and O. The different oxidation
states of 3d-series have been given below (very rare
K Ca Sc Ti V Cr Mn Fe Co Ni Cu Zn oxidation states are provided in parenthesis).
203 174 144 132 122 117 117 117 116 115 117 125 Sc Ti V Cr Mn Fe Co NI Cu Zn
Zr Nb I I
145 134 (ID (ID II II II II II II II II
III III III III (HI) III III (III) (III)
Hf Ta
IV IV IV IV (IV) (IV) (IV)
144 134 V (V) (V) (V) (V)
VI VI VI
1. Atoms of the transition elements are smaller than VII
those of the s-block elements. This is because of
1. Nickel also shows (0) oxidation state. This oxidation
(a) usual contraction in size across a horizontal state is in
period (a) nitriles (b) carbonyls
(b) orbital electrons added to the penultimate d-shell (c) cyclopentadienyl (d) in all of these
rather than to the outer shell of the atom 2. These elements do not show the properties
(c) both of the above characteristic of transition elements
(d) none of the above (a) Fe, Co, Ni (b) Sc, Y, La
2. The radii of the elements from chromium (Z = 24) to (c) Cu, Ag, Au (d) Zn, Cd, Hg
copper (Z = 29) are very close to one another. This is 3. In which case(s) there is change in oxidation number?
due to (a) aqueous solution ofCrO^' is acidified
(b) SO2 gas is passed into C^O^/H*
(a) lanthanide contraction
(b) the fact that successive addition of d-electrons (c) Ci^O^" solution is made alkaline
(d) CrO2Cl2 is dissolved in NaOH
screen the outer electrons (4s) from the inward
pull of the nucleus 4. Most transition metals are inert towards acids or
react slowly with them because of
(c) increase in radii due to increase in n is
(a) negative standard reduction potential
compensated by decrease in radii due to increase (b) protective layer of oxide
inZ (c) both of the above
(d) all of the above (d) none of the above
Chapter 16: The Transition and Inner Transition Elements | 719

r 5. For Ni and Pt, different I.P. in MJ mol are given Passage 5


below: Following diagram represents variation of magnetic
(IP)! + (IP)2 (IP)3 + (IP)4 moment (p) with atomic number Z for 3<Z-series elements:
Ni 2.49 8.80
I Pt 2.60 6.70
hence
(a) nickle (II) compounds tend to be
thermodynamically more stable than
platinum (II)
(b) platinum (IV) compounds tend to be more stable
than nickel (IV)
(c) both (a) and (b)
(d) none of the above is correct
6. Match the compounds of column X with oxidation
state of chromium in column Y.
Column X Column Y
Z —►
I [Crd^OeJClg 5 Magnetic moment of 3d-series elements (on arbitrary scale)
II CrO5 4
Based on this, answer the following questions.
HI K3CrO8 6
1. Consider following statements:
IV (NH3)3CrQ'4 3 A: The magnetic moment arises from the spin and
I II III IV orbital motions. It is measured in the unit of BM.
(a) 3 6 5 4 B : Every coloured ion is paramagnetic and every
(b) 3 4 5 6 paramagnetic ion is coloured.
(0 4 5 6 3 C : Fe, Co and Ni exhibit ferromagnetism.
(b) 6 5 4 3 D : Magnetic moment of N unpaired electron is
N(N + 2)
Passage 4 Select correct statements.
(a) A, B (b) B, C
Read the following short write-up and answer the (c) C, D (d) A, C
questions given:
2. Maximum magnetic moment is 6.93 for chromium.
“The transition elements have an unparalleled This means number of unpaired electrons is
tendency to form coordination compounds with Lewis (a) 4 (b) 5
bases that is with groups which are able to donate an (c) 6 (d) 7
electron pair (called ligands).” 3. Magnetic moment of Fex* ion is v35. Thus, x, number
1. The tendency to form complexes by transition metal of unpaired (N) electrons and its electronic
compared to s and p-block elements is due to configuration are
x N EC
(a) their smaller size
(b) higher nuclear charge (a) +3 5 [Ari 3d5
(c) presence of low energy vacant orbitals to accept (b) +2 4 [Ari 3d6
lone pair of electrons donated by ligands (0 +1 3 [Ari 3d7
(d) all of the above (d) 0 4 [Ar] 3d5 4s1

2. Out of [Fe(CN)6 ]*”, [Ni(CN)4 f ~ and [Ni(CO)4 ] 4. The oxygen carrying pigment, oxy-haemocyanin,
containing two copper ions is diamagnetic, because
(a) all have identical geometry (a) the two copper ions are in + 1 oxidation state
(b) all are paramagnetic (b) one of the copper ions is in + 1 oxidation state and
the other is in + 2 oxidation state
(c) all are diamagnetic (c) there are strong anti-ferromagnetic interactions
(d) [Fe(CN)6J4- is diamagnetic but [Ni(CN)4f“ and between the two copper ions
[Ni(CO)4 ] are paramagnetic (d) there are ferromagnetic interactions between the
two copper ions
720 ] Essential Inorganic Chemistry

Passage 6 Passage 7
Read the following passage and answer the questions Taking into account the following experimental fa-
given at the end of it: answer the questions given at the end.
Many ionic and covalent compounds of transition “When metallic copper is heated with concentrate
elements (and also inner transition elements) are sulphuric acid, in addition to copper(II) sulphate an
coloured. In contrast, compounds of the s- and p-block sulphur dioxide, some copper (II) sulphide is also formed.'
elements are almost always white. Colour may arise from
an entirely different cause in ions with incomplete d or 1. In which reaction, SO2 is formed?
f-shells. In a free isolated gaseous ions, the five d-orbitals (a) SO*- + 2e- ---- > (b) SO?" +8e~ —>
are degenerate, that is, they are identical in energy. The (c) SO?" + 6e- ---- > (d) in all of these
surrounding groups, which can be solvent molecules in
solution or ligands in a complex or other ions in a crystal 2. Copper (II) sulphide is formed because following sid
lattice, affect the energy of some d-orbitals more than reaction is favourable due to its low solubility ii
others. Thus, the d-orbitals are no longer degenerate and acidic medium
at their simplest they form two groups of orbitals of (a) SO?" +4H+ +2e" *
different energy. (b) SO?’ +8H++8e"
1. Two sets of d-orbitals with different energies in an (c) both of the above
octahedral complex are (d) none of the above
(a) d and di-■> - y.2
(b) rfx2_ v2 and dxy Passage 8
(c) d?2 > d^2 _ 2 and d^, Read the following short write-up and answer the
questions given at the end.
(d) d^, dyz and dxz Recent X-ray work, IR and other spectroscopic
2. Consider following statements methods have proved that Turnbull’s blue is identical to
A : Colour of a transition metal complex is dependent Prussian blue.
on energy difference between two d-levels 1. What is the common formula of Turnbull’s blue and
B : Colour of the complex is dependent on nature of Prussian blue?
the ligand and the type of complex formed (a) Fe3[Fe(CN)6t (b) Fe4[Fe(CN)6]3
C : ZnSO4 and TiO2 are white-in both d—d* spectra (c) KFe[Fe(CN)6] (d) KFe2[Fe(CN)6]
are impossible
Select correct statements. 2. Intense blue colour arises as a result of
(a) A, B, C (b) A, B (a) electron transfer between Fe(II) and Fe(I)
(c) B, C (d) A, C (b) electron transfer between Fe(II) and Fe(III)
3. Out of the following, select colourless complexes (c) d—d* transition
(d) spin magnetic moment
Ni(CO)4, [Ni(CN)J-l [Ni(NH3)4f* ,
A B C
[Ni(H2O)4f+, VO2*, CrO?", MnO; Passage 9
D E F G Based on the following statements answer the
(a) D, E (b) A, B, E questions given at the end.
(0 A, E (d) A, B Statement: The mercurous ion is written as Hg|'
4. MnO^ is of intense pink colour, though Mn is in (+ 7) while the cuprous ion is written as Cu+.
oxidation state. It is due to Statement: Cu+ is d10 ion and colourless but Cu>0
(a) oxygen gives colour to it is red and CujS is black
(b) charge transfer when oxygen gives its electron to
1. Mercurous ion is written as Hg2+ because
Mn making it Mn (+ VI) hence, coloured
(c) charge transfer when Mn gives its electron to (a) magnetic moment of mercurous ion is zero and
oxygen thus, 6s unpaired electron has been used in
(d) none of the above is correct bonding to make Hg2+
(b) in aqueous solution two Hg+ ions are solvated
5. Which is not true statement? forming Hg2+
(a) Ions of d-block elements are coloured due to d—d (c) HgCl2 disproportionates to Hg2Cl2 and Hg
transition (d) none of the above is correct
(b) Ions of /‘-block elements are coloured due to f—f
transition 2. There is no d—d* transition in Cu+(I) ion, but Cu2(XI)
(c) [Scd^Og ]3+, [Tid^Og J4 * are coloured complexes and CujSd) are coloured because of
(d) Cu+ is colourless ion (a) presence of electronegative ions O2-, S'i2-
Chapter 16: The Transition and Inner Transition Elements | 721

(b) presence of divalent anions 2. When exposed to air and moisture, iron gets rapidly
(c) charge transfer of electrons fromO2- orS2- to the oxidised to
vacant orbitals of Cu+ (a) FeO (b) FeA-xI^O
(d) all of the above (c) Fe3O4 (d) FeO- Fe2O3
3. Select some facts about passivity and corrosion
Passage 10 (rusting) of iron:
Based on the following experimental facts answer the A : Rusting is an electrochemical phenomenon
questions given at the end'. B : Passivity of iron prevents it from rusting
uGreen solution of potassium manganate (VI), turns C : Passivity and rusting are reversible processes
purple and a brown solid is precipitated when CO2 is Select correct statements.
bubbled into the solution”. (a) A, B (b) B, C
1. Purple colour is due to formation of (c) A, C (d) A, B, C
(a) manganese dioxide (IV)
(b) potassium permanganate (VII) Passage 12
(c) manganese (II) ion Based on the following qualitative test of chromium,
(d) none of the above answer the questions given at the end of it.
2. Brown solid that is precipitated is of In qualitative analysis, a confirmatory test for
(a) manganese dioxide (IV) chromium involves the formation of a blue transient
(b) potassium permanganate (VII) peroxo species, CrO5. It is formed when H2O2 is added to on
(c) manganese oxide (II) acidic solution containing C^O^-.
(d) manganese trioxide (III) 1. Select correct statement about CrO5.
3. Above change is observed when CO2 is passed into (a) It has two types of oxygen-peroxide and oxide
aqueous solution of the given ion. Function ofCO2 is (b) Oxidation number of Cr is + 10
(a) it makes solution acidic due to formation of (c) It is formed by oxidation ofCigO^"
EjCOg (d) All of the above are correct statements
(b) it makes the solution basic due to formation of 2. CrO5 can be represented as
C(£“ 0
(c) it is simply the medium of the reaction 0
(d) none of the above is correct (a) (b) I I
0
4. Given experiment involves
(a) acid-base reaction 0 0 0
(b) a redox reaction 11/
(c) O=Cr=O
T
(d) 0<-Cr->0
(c) disproportionation reaction
(d) decomposition reaction I
0 (f %
5. What happens when CO2 is passed into aqueous 3. When Ci^O^- is heated with Cl“ and cone. H2SO4 deep
chromate (VI) solution red vapours of chromyl chloride are formed. Deep red
(a) CrO3 is formed vapours have formula
(b) CrO5 is formed (a) CrOCl2 (b) (CrO)2Cl2
(c) Cr3+ is formed (c) CrO2Cl2 (d) CrO3Cl
(d) is formed
Passage 13
Passage 11 Read the following observation and answer the
Read the*following passage and answer the questions questions given at the end.
at the end of it, In an atmosphere with industrial smog, Cu corrodes to
When a piece of iron is dipped in cone. HNO3, a slight a basic sulphate Cu2(OH)2SO4 and basic carbonate
reaction takes place which then ceases completely. There Cu2(OH)2CO3.
is no change in the appearance of the iron, but this does
not displace Cu from CuSO4 solution which is however a 1. Basic sulphate is generally formed during acid rain
common property of Fe. This behaviour is also observed with a reaction
with Cr and Al when in contact with HNO3. (a) 2Cu + 2H2O + SO3 ---- > Cu2(OH)2SO4
(b) 2Cu + H.,0 + SO3 + O2 ---- >“Cu2(OH)2SO4
1. Above phenomenon is called
(a) corrosion (b) rusting (c) 2Cu + H2O + SO2 +O2 ---- > Cu2(OH)2SO4
(c) passivity (d) reduction (d) all of the above
722 | Essential Inorganic Chemistry

2. Formation of basic carbonate and basic sulphate 6. Assertion (A): Ionic radii of Ta and Nb are same.
involves some of the steps Reason (R): The lanthanide contraction canceb
A : a redox reaction almost exactly the normal size increase on
B : acid-base neutralisation reaction descending a group of transition elements.
C : precipitation reaction
D : decomposition 7. Assertion (A): Aqueous solution of ZnO is purely
Select the steps actually taking place basic in nature.
(a) A, C, D (b) B, C, D Reason (R): ZnO cannot be dissolved in dil. acid
(c) A, B, D . (d) A, B, C
8. Assertion (A): CrO3 is an acidic oxide.
3. This type of corrosion
(a) forms a tough adherent coating that protects the Reason (R): CrO3 liberates CO2 with Na^Oj.
underlying metal 9. Assertion (A): When CrO3 is dissolved in NaOH,
(b) causes loss of copper and thus the articles made
of copper CrO3 goes in the cationic part.
(c) causes poisoning of the atmosphere Reason (R): Na2CrO4 (yellow) is formed.
(d) gives a direct method of preparing basic copper
carbonate and basic copper sulphate 10. Assertion (A): Cu2+(aq) and Cd2+(ag) both form
stable complexes with CN-(aq).
Assertion & Reason Reason (R): [Cu(CN)4 ]3~ is not decomposed by
Codes: HjS.
(a) Both A and R are true and R is the correct 11. Assertion (A): Green solution of potassium
explanation of A. manganate (VI), K2MnO4 turns purple and brown
(b) Both A and R are true but R is not the correct solid is precipitated when CO2 is bubbled into it.
explanation of A. Reason (R): K^MnC^ changes to KgMnOg by COj.
(c) A is true but R is false.
(d) A is false but R is true. 12. Assertion (A): Prussian blue and Turnbull’s blue
(e) A and R both are false. are formed by different reagents, yet they are
identical.
1. Assertion (A): Most of the compounds of the
transition elements are coloured in the solid and Reason (R): They have formula as Fe[Fe(CN)6F•
solution state. 13. Assertion (A) : Copper(I) chloride ic white, white
Reason (R) : During d-d transition the d-electrons Cud) oxide is red.
absorb certain radiation from the visible region of the Reason (R) : In Cu(I) oxide, crystal structure has
spectrum.
excess copper (I) ions making it defective.
2. Assertion (A): The purple colour of KMnO4 is due
to the charge transfer transition. True & False
Reason (R): The intense colour, in most of the 1. On heating MnO2 with Fe2O3 and carbon,
transition metal complexes is due to d-d transition.
ferromanganese is formed.
3. Assertion (A): Nickel (II) compounds tend to be
2. Zn reduces VO2* to V3+ in acidic medium.
thermodynamically more stable than platinum (II).
3. Cu+ and Hg^+ both are paramagnetic.
Reason (R): Platinum (IV) compounds are
relatively more stable than nickel (IV) compounds. 4. ZnO is yellow when hot, but white when cold.
4. Assertion (A): Chromium can be used as a 5. When KCN is added to CuSO4 solution, a stable
protective and non-corrosive plating on other metals. complex K3[Cu(CN)4 ] is formed.
Reason (R): Due to atmospheric oxidation, 6. Cast iron is the purest form of iron.
chemically a inert layer of CejO3 is formed. 7. When excess of AgNO3 solution is added to 1 mol of
5. Assertion (A): [Fe(CN)6 ]3“ is more stable than aq. FeCl3 • GH^O, 2 mol of AgCl is formed.
[Fe(CN)6]4-. 8. Aqueous solution of CO2 changes CrO^- (yellow) into
Reason (R): Complexes where the iron is in the C^O^- (orange).
(III) oxidation state are more stable than those in (II) 9. Chromium and iron and made passive (inert) by
oxidation state. HN03.
Chapter 16: The Transition and Inner Transition Elements | 723

10. Due to lanthanide contraction, Zr and Hf have equal 34. Maximum oxidation state of Np is +6.
size. 35. When CrO2Cl2 is dissolved in NaOH, yellow colour is
11. Oxidation state of Cr in CrO5 is +10. due to the anion.
12. Maximum oxidation state of osmonium is +6.
13. In [Fe(H2O)5NO]2+, magnetic moment of Fe is
Fill in the Blanks
715 BM. 1. Coagulation of blood takes place by use of
solution.
14. Equivalent weight of MnO4 in acidic medium is
2. Rust stains can be removed by using which
one-fifth of the molecular weight.
forms water soluble complex .with ion of
15. Stains of blue ink on the cloth can be removed by rust.
oxalic acid.
3. Mercurous ion is and cuprous ion is
16. When gas is passed into KJCdtCN^], no ppt. is (magnetic behaviour).
found since Cd2+ is in the complex form. 4. ‘Brown ring’ in ring test of nitrate is due to the
17. When H2S gas is passed into aqueous ZnCI2 solution, formation of [FedlgOgNOf * in which magnetic
white ppt. of ZnS are formed. moment is found to be BM due to presence
18. MnO4 has Mn in +7 state thus it is colourless due to of impaired electrons.
no unpaired electron in d-orbital. 5. In ‘ring test’, NO transfers its electron to Fe2+ (charge
19. Nickel(II) compounds tend to be thermodynamically transfer) thus there are and ions
more stable than platinum(II) while platinum (IV) in[Fe(H2O)5NOf+.
compounds are relatively more stable than
nickel(IV). 6. Measurement of magnetic moment (78 BM) of
20. [Fe(H2O)6]3+ and [Fe(CN)6]3- have same magnetic Na2[Fe(CN)5 NO] indicates that Fe is state
and NO exists as
moment and identical structure.
21. The colour of the d-block ions is due to d-d* 7. [FetCNJg]4- and [Fe(CN)6]3" differ in their
transition. while their structures are similar.
22. ZnSO4 and TiO2 are colourless (white) due to absence 8. [Ni(CN)4f“ and [Ni(CO)4] are but
of d-d* transition.
[Ni(CN)4 f~ is and [Ni(CO)4 ] is tetrahedral
23. CrO3 is also called chromic acid. in structure.
24. KMnO4 solution is used as a primary standard 9. Titanium is called the metal.
solution in volumetric titration.
10. Rusting of iron is a special case of
25. Mohr’s salt is isomorphous with white vitriol and
epsom salt. 11. In case ofCu+, all the d10 electrons are paired, hence
26. Correct formula of blue vitriol is [Cu(H2O)5]SO4. it is and it does not give
transition.
27. Hg2Cl2 is blackened by addition of aq. NH3 due to
12. In MnO4, Mn is in +7 state with d° electronic
formation of[Hg2(NH3)2]Cl2.
configuration but it is coloured ion due to
28. HgCl2 is soluble in excess of KI due to formation of
from to
W.
13. Due to charge transfer from O2- to Mn(VII), O is
29. HgCl2 is called calomel and Hg2Cl2 is called corrosive and Mn(VII) changes to hence
sublimate. MnO4 is
30. Solution of AftC^H^g and TiCl4 in cyclohexane react 14. Magnetic moment of Fe2+ (24 electrons ) is
to form Ziegler-Natta catalyst. and is thus than Cr that of (24 electrons).
31. AgBr is insoluble in Na2S2O3 due to formation of 15. FeO is a compound and is
white precipitate of Ag2S2O3. deficient.
32. The element (Unn) has atomic number of 100. 16. Fe, Co and Ni have approximately size.
33. The actinoids are more electropositive and are 17. Due to Zr and Hf, and also Nb and Ta have
reactive, reactivity increases with increase in atomic equal size.
number.
724 | Essential Inorganic Chemistry

18. V2O5 is used as catalyst in process for the 37. Fe3O4 is a oxide
manufacture of
38. NaFeO2 on hydrolysis gives and
19. [Fe(H2O)6 ]3+ reacts with H2O forming and
39. Fehling solution contains and sodium
H3O+ and is thus in nature.
potassium tartrate (Rochelle salt).
20 is used to make many ferrous alloys 40. CuSO4 is determined iodometrically by its reactions
including stainless and hard chromium steel.
with and then titration with -
21. Fe2+ salts can be determined volumetrically using solution.
and in acidic medium.
22. Ci2O^_/H+ and H2O2 gives a deep blue-violet Matrix-Match Type Questions
compound in which oxidation number of Cr I. Only One Correct
is 1. Match the alloys (in List I) with the constituents
23. Oxidation number of Cr in K3CrO8 with [Cr(O2)4]3- metal (in List II).
ion is List I List II
24. In (NH3)3CrO4, oxidation number of Cr is +4 hence, it A. Gun metal 1. Pb.Sn
is B. German silver 2. Cu, Sn, Zn
25. Chromyl ion is C. Brass 3. Cu, Zn
26. (NH4)2Cr2O7 (orange) changes to green due to D. Solder 4. Cu, Zn, Ni
formation of by and 2. Match the natural coordination compounds (in List I)
gas is obtained by with metals therein (in List II).
27. MoO3 and CrO3 are strongly hence dissolve
List I List II
in
A. Nitrogenases 1. Cu
28. CrO^- changes to Cr2O^~ in solution and
B. Cytochrome oxidase 2. Mo
C^O^- changes to CrO^“ in solution.
C. Cytochrome C 3. Zn
29. Mn3+ disproportionates in H^O/H* to and
D. Carboxy peptidase 4. Fe

30. In acidic medium, equivalent weight of MnO4 is 3. Match the lanthanide ions (Ln3+, in List I) with their
calculated magnetic moments (in List II).
of molecular weight and in dilute alkaline
medium it is of molecular weight. List I List II
31. When CO2 gas is passed into K2MnO4 solution, there A. Ce3+ 1. 7.94 BM
is formation of and by B. Nd3+ 2. 4.90 BM
32. CO is toxic since when inhaled, it forms complex with C. Gd3+ 3. 1.73 BM
of present in blood. D. Pm3+ 4. 3.87 BM
33. When a small amount of KMnO4 is added to cone.
4. Match the catalysts in List I with their uses in List II
H2SO4, a green solution containing ion is
formed. List I List II
34. When excess of KMnO4 is added to cone. H2SO4, an A. TiCI4 1. Adams catalyst in reduction
explosive oil is formed. B. PdCI2 2. In preparation of (CH3)2 SiCI2
35. Fe(OH)2 dissolves in concentrated solution of NaOH C. Pt/PtO 3. Reppe synthesis
forming a blue-green complex D. Cu 4. Used as the Natta catalyst in
polythene production
36. Turnbull’s blue and Prussian blue are identical with
E. Ni 5. Wake process for converting CjHa
formula ......... to CH3CHO
Chapter 16: The Transition and Inner Transition Elements | 725

5. Match the alloys (in List I) with their constituents (in 10. Match catalyst is (List I) according to its name and
List II). funtion.
List I List II List I List II List III
Catalyst Name Function
A. Invar 1. 70%Cu + 30%Zn
A. TiCI4 1. Fenton’s P. Oxidation of
8. Brass 2. 66% Ni + 33% Cu
reagent alcohol
C. Monel 3. 36% Ni
B. FeSO4/H2O2 2. Ziegler-Natta Q. Polythene
D. Coinage metal 4. 60% Ni + 20% Fe + 20% Cr preparation
E. Nichrome 5. 75% Cu + 25% Ni C. Pd/C 3. Lindlar R. Hydrogenation of
alkynes to give
6. Match the compounds of List I with oxidation state of cis-alkene
List II. D.^Pt/PtO 4. Adam S. Synthesis of
CH3OH
List I List II

A- [Cr(H2O)6]CI3 1. 5 11. Match the complex ion (List I) with its spin-only
spin-only magnetic moment (List II).
8. CrO5 2. 4
C. K3CrO8 3. 6 List I List II

D. (NH3)3CrO4 4. 3 A. [Co(NH3)6]3* 1. 1.73 BM


B. [Fe(CN)6]3- 2. 5.92 BM
7. Match the compounds in List I with their uses in
List II. C. [Mn(H2O)6]2+ 3. 0.0 BM
D. [Ni(H2O)6]2* 4. 2.83 BM
List I List II

A. Hg2CI2 1. Metallurgial extraction of Ag


II. One or More Correct
B. ZnO 2. Electrode
1. Match the species in Column I that can react (oxidise,
C. ZnSO4 3. Luminous paints
reduce, give ppt) species in Column II.
D. Zn 4. Lithophone
Column I Column II
8. Match the underlined atoms in List I with oxidation A. Fe2* 1. Cr2O2'
number in List II.
B. C2O4 2. CrOf
List I List II
C. Pb2* 3. [Fe(CN)6]
A. MnO4 1. 1
D. Ag* 4. MnO;
B. CrO2" 2. 2
E. SO2 5. S2O§’
C. Fe [Fe"(CN)6]' 3. 3
D. ZnO2’ 4. 6 2. Match the species in Column I with their
E. [Ag(CN)2]- 5. 7 characteristics in Column II.
Column I Column II
9. Match the alloys (in List I) with their constituents
(in List II). A. Cu* 1. Paramagnetic

B. Cu'.2* 2. Diamagnetic
List I List II
C. Hg2* 3. Corrosive sublimate
A. Invar 1. 70%Cu + 30%Zn
D. Hg2* 4. Calomel
8. Brass 2. 66% Ni + 33% Cu
C. Monel 3. 36% Ni E. Zn2* 5. Colour with SON"

D. Coinage metal 4. 60% Ni + 20% Fe + 20% Cr. 6. d'°


E. Nicrome 5. 75% Cu + 25% Ni 7. Reacts with KI
726 | Essential Inorganic Chemistry

Integer Answer Types 2. Cr+ (23 electrons) has electrons in (n -1)<


This section contains 9 questions. The answer to each orbitals.
of the questions is a single digit integer, ranging from 0 to 3. Maximum oxidation state shown by manganese L
9. The appropriate bubbles below the respectively
question numbers in the ORS have to be darkened. For
example, if the correct answers to question number X, Y, Z 4. Magnetic moment of Mn in MnSO4 is 5.91 BM. Thtu
and W (say) are 6, 0, 9 and 2, respectively, then the correct it has......... electrons in 3cf-orbitals.
darkening of bubbles will like the following.
X Y Z W
5. CrO5 has............. peroxy linkage(s).
© © © © 6. 2.674 g of CoCl3 • 6NH3 (molar mass = 267.4 g molJ1
© © © © in aqueous solution gave 4.305 g of white precipitate
© © © © of AgCl (molar mass = 143.5 g mol"1). On reaction
© © © ©
with excess of AgNO3 solution. Thus Cl atoms in
© © © ©
© © outer sphere of the complex is.............
© ©
© © © © 7. Number of oxygen atom(s) between two chromium
© 0 © © atoms in Ci2O^~ is.............
© © © ©
8. When CrO5 is treated with H2O2 and KOH, a
© © © ©
red-brown compound K3[Cr(O)x] is formed, r is
Answer of this integer type question are in the form of
colours given in number.
1. Violet 6. Orange Set III
2. Indigo 7. Red
3. Blue 8. Prussian blue This section contains 8 questions
4. Green 9. Cherry Red 1. 2.78 g of FeSO4 xH2O on treatment with excess of
5. Yellow 0. Colourless (white)
Colour of the BaCl2 solution gives 2.33 g of white precipitate of
BaSO4. What is value of x? (Ba = 137, S = 32, Fe = 56.
Set I 0 = 16)
1. Complex when KCNS is added to FeCl3 solution. 2. How many of the following also forms dimer?
NO2,AlCl3,FeCl3,N2O5,CuSO4, PC13
2. Complex when NH3 is added to CuSO4 solution.
3. How many of the following are paramagnetic as well
3. Compounds whenK^C^O? solution is made alkaline. as coloured species?
4. Compound when K2CrO4 solution is made acidic. O2, NO2, Cu2*, Hgf*, Fe2*, Fe3*, [FefCN),/',
5. Compound when Cl2 gas is passed into K2MnO4 [Fe(CN)6]3~, [Ni(H2O)6 f *, [Ni(CN)4 J2-
solution. 4. What is van’t Hoff factor of Hg2(NO2)2 if it is lOO^
6. Compound when SO2 gas is passed into acidified ionised in aqueous solution?
KjCijjO, solution. 5. How many of the following have underlined atoms in
7. Compound when CuSO4 is treated with KI and then different oxidation states?
titrated with Na2S2O3 solution. Hg^ ,S2O| , S4O^ , CrO5, Fe3O4, Pb3O4
8. Complex when dimethyl-glyoxime (in alcohol) added 6. How many of the following pairs have approximately
to ammoniacal nickel chloride solution. equal radii
9. Complex when K4[Fe(CN)6] is added to FeCl3 (Zr,Hf), (Nb, Ta), (Cr,Mn), (Mn,Fe), (Nb3+, Ta3+),
solution. (Zr4+,Hf*+)
7. How many of the following are amphoteric?
Set II
ZnO, A12O3, VO2, V2O5, P2O5, Ci^Oa*
This section contains 8 questions
1. Fe2+ (24 electrons) has ...... electrons in 8. How many H^O molecules are in coordination sphere
3d-orbitals. inFeCl3-6H2O
Chapter 16 . The Transition and Inner Transition Elements | 727

Test Yourself on the Periodic Table


1. Locate with each of the following electron
configuration on the Periodic Table given the
transition elements. Identify each element.
(a) [Ar] 3d7 4s2 (b) [Ar] 3d5 4s1
(c) [Kr]4d25s2 (d) [Xe]4/'36s2

4. On the Periodic Table locate


(1) Coinage metals (2) Lanthanides
(3) Actinides (4) Alkaline earth
elements
(5) Chalcogens

2. Locate the transition elements, on the Periodic Table,


with following specifications :
(a) Element in 3d series giving +7 oxidation state
(b) Element in 5th period whose divalent cation has
d10 configuration
(c) Element which has been recently named as Ds
(d) Elements which forms colourless ion in M+ state

5. On the Periodic Table, locate the elements


(a) in 3d series which form insoluble sulphides
(b) in 4d series which form insoluble sulphides
(c) with s2 p6 configuration of valence shell
(d) with [Ar] configuration in +3 state

3. Locate the period, group and block of the element Ds


recently named by IUPAC. Also write its electronic
configuration.
Answers & Solutions
Master Exercises
Exercise 1
Short Answer Type Questions level is much greater and corresponds to ultraviolet ligt
1. In all cases CN of the metal [Cu = 4, Cr = 6,Pt= 6)istobe being absorbed. Thus compound will not be coloured.
satisfied. 7. Colourless — (a), (b) (d), (i), (j) Coloured — (c), (e), (0, (g)
(a) [Pt(H2O)6]Cl4 x [Pt(H2O)6]4* + 4C1"; five ions, (h).
(b) [Cu(H2O)2C12]^ no ionisation; exists as molecule. 8. (a), (b) — colourless (c), (d) — coloured.
(c) [Cr(H2O)5Cl] Cl.2 [Cr(H2O)2Cl]2* + 2C1-; 9. Hg2* has all filled 5d-orbitals (no unpaired electron i
three ions. 5d), hence no d—d* transition, hence colourless.
(d) [Cu(NH3)4]C12 [Cu (NH 3 )4 f + + 2C1~; three ions.
10. NO2 — coloured and paramagnetic

2. Ion (M 2*) Electrons EC Unpaired Magnetic Cu* — colourless


Z
(M 2*) (M 2*) electron moment °2 — paramagnetic
(BM)
Sc2* 21 19 3d1 1 73
o3 — diamagnetic
Hg2+ — colourless
Ti2* 22 20 3d2 2 78 Cd2* — colourless
V2* 23 21 3d3 3 7t5 Al — paramagnetic
Cr2* 24 22 3d4 4 724 C (graphite) — paramagnetic.
Mn2* 25 23 3ds 5 735 11. Sc(21): [Ar] 3d1 4s 2
Fe2* 26 24 3d6 4 724 Sc3* : [Ar]3d° no unpaired electron in d-orbital
Co2* 27 25 3d7 3 715 hence no (d-d*) transition hence,
Ni2* 28 26 3d3 2 78 colourless
Cu2* 29 27 3d9 1 73 Ti(22): [Ar]3d2 4s 2
Ti3* rtArJSd1 due to unpaired electron in d-orbital
Zn2* 30 28 3d'° 0 0
(d-d*) transition possible hence,
Zn2* < Sc2* = Cu2* < Ti2* = Ni2* <V2* = Co2* < Cr2* coloured.
= Fe2* < Mn2* 12. A. (a) Nb Ta (b) Cr Mn
3. [Fe(CN)6]4-— d2sp3 hybridised Fe2* has all the paired (c) Cu Zn (d) Cu > Ni
electrons hence magnetic moment = zero.
(e) Zr s Hf (0 Ca > Sc
[FeCHgOg]2* — sp3d2 hybridised Fe2* has four unpaired
(g) Cr < Mo
electrons, hence magnetic moment =724 BM

z
4. [Fe(CN)t. J4- unpaired electron (TV) = 0,
p =7MTV + 2) BM = 0
B.
(a)
Element (Z)
Sc(21)
S
18.0
z*=z-s
3.0
[FeCCNJJ3- N=1 = 73 (b) Ti(22) 18.85 3.15
[Ni(CN)4]2- N =0 =0 (c) V(23) 19.70 3.30
[Ni(H2O)4]2+ N =2 = 7s (d) Cr(24) 20.55 3.45
(e) Mn(25) 21.40 3.60
5. t]N(N +2) =735 => TV =5 (unpaired electrons)
(f) Fe(26) 22.25 3.75
=> x=3(Fe3*)
Fe3* = [Ar] 3d5 (g) Co(27) 23.10 3.90
(h) Ni(28) 23.95 4.05
6. The s- and p- block elements do not have a partially filled
(i) Cu(29) 24.80 4.20
d-shell so there cannot be any d-d* transition. The
energy to promote an s- or p- electron to a higher energy (j) Zn(30) 25.65 4.35
Chapter 16: The Transition and Inner Transition Elements | 729

0 NH3 21. FeC2O4 Fe2* + C92044"


ferrous oxalate
13. (a) (b)
b 5Fe2+ + MnO; 4 8H* 5Fe3* + Mn2+ + 4H2O
Pentagonal pyramid 5C2O?" + 2MnO7 + 16H* 10CO2 + 2Mn2* 4 8H2O
nh3
Pentagonal bipyramid 5FeC2O4 + 3MnO4 + 24H+ 5Fe3+ 4 10CO2 4 3Mn2+
4 12H2O
(c)Na2Cr2O7 +H2SO4 Na2SO4 4 2CrO3 4 H2O
saturated cone. 5FeC2O4 = 3MnO4
t aq solution .*. lFeC22O44 =0.6 MnO44

/K
fdi -n—Cr—0—Cr^_n-
(Note Both Fe2+ and C2O4" are oxidised by MnO4. Ask your
teacher if one takes one mole of ferric oxalate)
22. Oxalic acid forms a dark-coloured complex with rust
stains (Fe3+). This complex is water soluble. The rust
i stains can be removed by treating the fabric with oxalic
14. (NH4)2Cr2O7 A acid and then washing with water.
N2 + Cr2O3 + 4H2O
It is a redox reaction Fe3+ + 3C22O44 " [Fe(C2O4)]3“
— NH4+ ion is oxidised to N2 soluble
— Cr2O2- ion is reduced to Cr3+ 23. FeCL + 3H2O---- > Fe(OH)3 + 3HC1
15. (A): H2S, oxidised to S by Cr2O727 H+ NaHCO3 + HC1---- > NaCl + H2O + CO2
(B): SO2, oxidised to SO2- by Cr2O7"/H+ 24. (a) Excess of KOH separates Fe3+as Fe(OH).'3
2H2S + SO'22 ---- > 3S +2H22O Fe3+ + 3OH“ ---- > Fe(OH)3 I V

yellowish (b) Excess of KOH precipitates Fe3+ as Fe(OH)3 while


white
Cr3+ remains soluble KCr(OH)4
16. C02 + H2O * h2zcoo3 h+ + hco3 (c) Excess of NH3(ag) precipitates Fe3* as Fe(OH)3
2CrO2' + 2H+ ---- > Cr2O7“ +H2O while Cu2+ remains soluble as [Cu(NH3)4]2+
yellow orange White ppt. with BaCl2 =*>SO2-
Choclate coloured ppt with K4(Fe(CN)6] =>Cu2t
Aqueous CO2 is acidic and changes yellow CrO2- into
Thus, hydrated salt is : CuSO4 xH2O
orange Cr2O2-
CuSO4 xH2O ---- > BaSO4
17. Cr2O2" <Cr3+<Cr.2+ 159.5 + 18x 233
Smaller the oxidation state of the element in the ion, 2.495 2.33
greater the reducing nature. x =5
25. (A) :CuSO4 -5H2O (B): BaSO4 (C):CU2I2
18. (a)[Zn(H2O)4]2++H+ H2O [Zn(H2O)3OH]++H3O+
acid base base acid (D) :Cu2[Fe(CN)6]
(b) [A1(H2O)6]3+ Reddish brown solution is due to formation of
19. ZnSO4 solution is acidic due to hydrolysis. When KI3(KI+I2 KI3). On adding hypo, I2 is reduced to I"
and white ppt. ofCu^ appears.
NaHCO3 is added to it, CO2 is given off and ZnCO3 is also
precipitated: 26. White precipitate is due to CuCN

[Zn(H2O)4]2+ +2HCO; ZnCO3 >1 + 5H2O 4 CO2 2Cu2+ + 4CN" ---- 4 2CuCN + C2N2
cyanogen
white ppt.
White precipitate of CuCN dissolves in excess of KCN
H2O
due to formation of a soluble and stable complex
ZnSO.4 [Cu(CN4]3- and even in the presence ofH^Cu2* is not
20. Oxidation number of Mn in MnO4 = + 7 precipitated as CuS.
CuCN + 3KCN---- >K3[Cu(CN)4] 3K* +[Cu(CN)4]:
It is expected that (Mn in) MnO4 should have [Ar] stable
configuration and should be colourless. But one electron
27. ZnO + C---- > Zn + CO
is transferred to Mn from oxygen and thus Mn in MnO4
has one unpaired electron making it paramagnetic. AG° =AG7(CO)-AG7(ZnO)
= - 137.3-(-318.2)
Magnetic moment = y]N(N + 2) = 73 BM = 180.9 kJ mol-1
730 | Essential Inorganic Chemistry

Positive value of AG° indicates that reaction is 3. (A):CuFeS2 (B): FeSO4 (C): CuSO4 (P): KJS
non-spontaneous. (However, it is spontaneous above
1000°C). (E) :BaSO4 (F): KFe[Fe(CN)e], Turnbull’s blue.
(G):[Cu(NH3)4]|2+
: (H): CuS.
28. (A): ZnCl2 (B): Zn(OH)2 (C): Na2ZnO2 (D):ZnS
4. (a) [Cr(H2O)6]Cl.I3 (b) [Cr(H2O)5Cl]Cl2 H2O
(E):AgCl
Na2ZnO2 +H2S ---- > ZnS + 2NaOH (c) [Cr(H2O)4Cl2]Cl-2H2O
(C) (£>) 1 mol of each is treated with AgNO3.
29. (A): ZnSO3 (B):ZnO
yuj. uuv (O:SO.
y^j. (D) : ZnS (a) gives 3 mol AgCl (b) given 2 mol AgCl
ZnO is amphoteric soluble in NaOH and HC1 (c) gives 1 mol AgCl
Thus, nature of the complex can be decided.
30. ZnO in white paint by atmospheric HgS gives ZnS
(white) but Pb2+ salts in white paint turns black by 5. Due to sharing of s-as well as d-electrons there is
atmospheric H2S due to formation of PbS. formation of stronger metallic bonding.
31. ZnO > CdO > HgO 6. Refer text (oxidation state)
32. A
A ___ 7. (a) Cu2+ (b) V2+ (c) Cr2+ (d) Co3+
I (b) 8. Mn2+ < MnO2 < MnO;4
II (c)
III (d) 9. (a) 2CrO4~ + 2H+ ---- > Cr2O2" + H2O
yellow orange
IV (a)
(b) Cr(OH)3 + 3H+ ---- > Cr3+ + 3H2O
33. Ni (obtained by Mond’s process)
(c) Cr(OH)2 + 2H+ ---- > Cr2+ + 2H2O
Analytical Questions (d) Fe(OH)2 + 2H+ ---- > Fe2+ + 2H2O
1. (a) MnO4 oxidises Fe2+ to Fe3+and C2O4" to CO2 in (e) Cu(OH)2 + 2H+ ---- > Cu2+ + 2H2O
ferrous oxalate (i)and only C2O4“ to CO2 in (ii) In cases (b) to (e), metal hydroxides (insoluble) dissolve
5Fe2+ + MnO4 + 8H+ ---- » 5Fe3+ + Mn2+ + 4H'22O in dil. H2SO4.
5C2O42“ T
UV>2^ 2MnO44 T
+ ilTUlU 16H+ ---- > 10CO2 + 8H2O
+ J.VXX 10. (a) 2CrO4” + 2H+ ---- > Cr2O72- + H2O
yellow orange
5(Fe2++ C2O4-) + 3MnO4 + ... ---- > ...
Thus, 5Fe2+ = MnO4 (b) Fe3+ + SON’ ---- > [Fe(SCN)]2+
deep red
5C2O2" = 2MnO4
5FeC2O4 s3MnO4 (c) 3Cu + 2NO- + 8H+ 3Cu2+ +2NO + 4H20
blue
1 FeC2O4 = 0.6 MnO4
(d) Cr(OH)3 + OH" ---- > [Cr(OH)4F
Fe2(C2O4)3 ---- >2Fe3++3C'2^4 2O;
2[Cr(OH)4F +3H2O2 ---- > 2CrO2’ +6^0 + 2^
Ferric oxalate yellow
2
1 mol of ferric oxalate = 3 mol of CqO?" = - x 3 = 1.2 mol
5 (e) 3CuS + 8H+ + 2NO:o 3Cu2+ + 3S +2N0
of MnO44 blue yellow
(b) As above + 4HjO
6Fe2+ + Cr2O2- + 14H+ ---- > 6Fe2+ + 7H2O + 2Cr3* 11. (a) 4Co3+(ag) + 2H2O(Z) 4Co2+(a^)+ O2(g)
6C2O2" + 2Cr2O2~ + 28H+ ---- > 12CO2 + 4Cr3+ + 14H2O
+ 4pr(a4)
Thus, 6(Fe2+ + C2O42’) = 3Cr2O2"
(b) 4Cr2+ (aq)+ O2(g) + 4H+(ag) 4Cr3+(ag)
1 mol FeC2O4 = 0.5 mol Cr2O2-
+ 2H2O(D
Thus, 1 mol ferric oxalate = 3 mol oxalate
(c) 3Cu(s)+ Cr2O2 (ag)+ 14H+(ag) -> 3Cu2+(a<?)
= 1.0 mol ofCr2O2-
+ 2Cr3+(ag) + 7H,O(0
2. (A): MnO2 (B): K2MnO4 (C): KMnO4
K2MnO 12. A:FeO Cr2O3 B:K2CrO4 C : K2Cr2O7
MnO2 + KOH + O2(KNO3) 4 (B) D : (NH4)2Cr2O7 E : Cr2O3
(A)
green
CO2 + HLO H2CO3 H+ + HCO’
At At At O O
NCERT Problem
3MnO^“ + 4H+ -----> MnO2 + 2MnO4"+ 2H2O
Refer Text.
Chapter 16: The Transition and Inner Transition Elements | 731

Exercise 2
Only One Option Correct 21. (d) Cu* :[Ar]3d10
1. (d) Cu2+ is reduced to Cu+ by ON" No unpaired electron, thus s = 0
Thus, (a) is true Thus, (d)
Cu+undergoes complexion with CN" forming 22. (a) 23. (c)
K3[Cu(CN)4], Thus, (b) and (c) are true
24. (c) II group (of cation-analysis) contains Pb2+ and Cu2+
Thus, (d)
and are precipitated as sulphides on passing in
2. (c) acidic medium. ZnS is precipitated in presence of
3. Larger anion stabilises to a greater extent. NH4OH and NH4C1.
Thus, (a) Thus, (c)
4. (c) 25. (b) Ti(22): [Ar]4s23d2 T T
5. (b) Metals lying above H in electrochemical series are Ti2+ : [Ar] 3d2
not obtained from their oxides by heating Ti4+ :[Ar]3d°
HgO ----> Hg+O2 Ti2+has two unpaired electrons in 3d and thus d-d*
ZnO 0.2
> Zn + O transition is possible due to absorption of light in visible
Ag2O ---- region.
—> Ag + O2
Ag+O.
26. (d) 2Ag+ -fILjS - Ag^ (black)
6. 1: 2CuSO4 + 4KI ---- > Cu^ +12 +
white ppt. 27. (c) Cr2O2- + 2OH- -» 2CrO2-+H2O
II: HgCl2 +2KI 2KC1+ HglJ pH>7(x>7)
orange ppt. 2CrO4~+2H+ ---- > Cr2O2-+H2O
HgI2+2KI ---- > K.>HgI4 pH<7(y<7)
soluble Thus, (c)
III: Pb(NO3)2 +2KI PbLj +2KNO3 28. (a) MnOj ---- > MnO2- + e~
yellow ppt'
Thus, (b) 29. (b) In this case CrO2Cl2 is formed

7. (b) 8. (d) 30. (c) MnO4 charges to MnO2- or MnO2 in basic medium.
9. (c) NO ---- > NO+ + e" 31. (a) 32. (c)
Fe3+ + e~ ---- > Fe2+ 33. (a) Maximum unpaired electrons T f T T T d',5
Oxidation state of Fe is +2 and NO+ is ligand 34. (c) Magnetic moment p = y]N(N + 2) BM.
Thus, (c)
where, N = unpaired electrons
10. (a) Ring is formed by charge transfer iV = 6, h = 748BM=x
Cr(24) [Ar]3d54s1
NO ---- > NO* + e~ Mn+ [Ar] 3d54sl N - 6, |1 = <48 BM = y
Fe2+ + e~ ---- > Fe2+ Fe2* [Ar] 3d6 N = 4, |1 = <24 BM = z
Thus, (a)
Ti T T I T | T
11. (c)
N=4
1 +DMG ---- > [Ni(DMG)2]2+
12. (b) Ni2+
cherry red ppt. Thus, z < x = y
13. (a) 2CuSO4 +4KI ---- > +CU2I2 +I2 Thus, (c)

I2+2Na2S2O3 ---- > 2NaI + Na2S4O6 35. (d)


Thus, (a) 36. (b) 24 carat gold is 100%
Thus, 22-carat gold is 91.67%
14. (c)
Thus, 3.5% higher
15. (d) Fe(III) gives red colours with KCNS
Thus, (b)
hence, (d) is incorrect.
37. (c) 38. (a) 39. (c) 40. (b) 41. (c)
16. (b) 17. (d) 18. (b) 19. (c) 20. (c)
42. (c) 43. (a)
732 | Essential Inorganic Chemistry

44. (b) generally salts are colourless due to d10-configuration. 141. (a) 142. (b) 143. (c) 144. (a) 145. (c)
45. (c) 46. (a) 47. (d) 48. (a) 49. (b) 146. (b) Cr3+ :[Ar]3d5 stable
50. (c) 51. (c) 52. (a) 53. (c) 54. (c) 147. (a) 148. (a) 149. (a) 150. (c)
55. (a) 56. (a) 57. (c) 58. (a) 151. (a) 152. (b) 153. (c) 154. (c) 155. (a)
59. (d) Mn 2+ (d5) ? T T ? ? 156. (c) 157. (d) 158. (b) 159. (c) 160. (b)

60. (d) 61. (b) 62. (b) 63. (d) 64. (c) 161. (a)
65. (a) 66. (c) 162. (a) VO2 + 2H+ ---- > VO2+ + h2o
OH- 4 CrO2- 163. (d) CrO3 + 2H+ CrO2+ + h2o
67. (d) CrO3 + H20 H2CrO4
(A) (B)
164. (b)
Thus, (d)
165. (d) 2CrO4~ + 2H+ Cr2O2' + H20
68. (a) 69. (c) 70. (c) 71. (d) 72. (b)
74. (a) 75. (c) 76. (d) 77. (a) 166. (c) Cr2O2- + 2OH’ 2CrO4+H2O
73. (a)
78. (d) 79. (b) 80. (b) 81. (c) 82. (a) 167. (d) 168. (c) 169. (c)
83. (b) 84. (b) 85. (b) 86. (b) 87. (c) H+ Mn09 + MnO.
170. (a) 2MnO4~ -2-4 2 4
88. (d) 89. (b) 90. (d) 91. (a) 92. (d) 175. (c)
171. (a) 172. (d) 173. (b) 174. (a)
93. (a) 94. (c) 95. (b) 96. (d) 97. (d)
176. (b) ZnS is white.
98. (b) 99. (b) 100. (c) 101. (b) 102. (a) 181. (d)
177. (a) 178. (c) 179. (d) 180. (b)
103. (c) 104. (b) 105. (a)
182. (c) 183. (a) 184. (a)
A
106. (a) (NH4)2Cr2O.7 Cr2O3 + N2 + 4H2O 185. (a) [Fe(H2O)4Cl2]Cl-2H2O
NH4NO2 —-4 N2+2H2O 186. (d) Fe2+ +[Fe(CN)6]3" ---- > blue
107. (b) 108. (d) | Fe3*
4 no colour
109. (b) [Co(SCN)4 ]2“, cobalt is +2 with three unpaired (by oxidation)
electrons in 3d. Thus, p = V15 BM. 187. (a) 188. (d) 189. (b)
110. 0)) 111. (b) 190. (c) Fe3+ forms soluble complex with oxalic acid.
6Fe3+ + 2Cr3+
191. (d)
112. (d) 6Fe2+ + Cr2O2-
192. (a) Fe2O3 + OH" + Cl2 ---- > [FeO4]2‘
113. (b)
193. (b) 194. (c)
114. (b) Hgl2 +2KI ---- > KjjHg^
195. (d) NO ---- > NO+ + e~,
Hgl22 ---- > Hg +12 (violet)
Fe2+ + e~ Fe+ (three unpaired electrons)
115. (b) CrO2" + 2H+ - Cr2O2"+H2O 3d6 3d7

116. (d)
. 2O
AgNO3 -^4 Ag _ —A—4 -■
2Ag + O2 U T T T T nut ? t
117. (c) Magnetic moment = yjN(N + 2) = V15 BM
118. (d) Hg2Cl2 +2NH3 ---- > HgNH2Cl+ Hg +NH4C1 196. (c) 197. (b)
white black
198. (a) Fe2+ as well C2O2- is oxidised by Mn04 in acidic
119. (d) 120. (d) 121. (a) 122. (a) 123. (b) medium
124. (d) 125. (a) 126. (c) 127. (c) FeC2O4 Fe2+ +c2o2-
128. (b) Cr2O2“ + SO2 4 SO2"+Cr3+ 5Fe2+ + MnO4 Mn2+ + 5Fe3+
+6 +4 +3
5C20j- + 2MnO4 Mn 2+ + 10CO2
129. (d) 130. (a) 131. (b) 132. (b) 133. (c)
5FeC2O4 = 3MnO4
134. (d)
1 FeC2O4 = | MnO4
135. (b) NO ---- > NO+ + e~ 25 + 12 37
Fe2+ + e~ ---- > Fe+(25e) = 0.6 mol
136. (a) 137. (b) 138. (b) 139. (d) 140. (c) Thus, (a)
Chapter 16 . The Transition and Inner Transition Elements | 733
I
, 199. (b) 200. (c) 254. (b) CrO3 + H2O H2CrO4 (acidic)
201. (b) CuSO4 + 2H2O Cu(OH)2 h-H^O,4 255. (c)
2CrO2' + 2H+ Cr2O2" + H2O 256. (c) Fe has 4 unpaired electrons
Thus, (b) 257. (c) 258. (c)
SnCl2
202. (c) 203. (d) 204. (b) 205. (c) 206. (d) 259. (b) SnCl2 + HgCl2 Hg2Cl2 4 Hg
white grey
207. (b) 208. (b) 209. (b) 210. (c) 211. (c)
260. (b) 261. (d) 262. (c) 263. (b)
212. (d) 213. (d) 214. (a) 215. (a) 216. (d)
264. (a) Fe 3+:[Ar]3d5; Cu+:[Ar]3d10
217. (c) 218. (c) 219. (b)
265. (a) 266. (c) 267. (d) 268. (d) 269. (c)
220. (a) Element (90-103) are in TUB
270. (a)
221. (d) 222. (d) 223. (a) 224. (c)
271. (c) Nascent H reduces NO to NH3 which changes to
225. (d) Third electron which is removed in third ionisation NH.NO
4 3
o
potential enthalpy belongs to 3d3 subshell in V. 272. (c) 273. (a) 274. (d) 275. (b) 276. (b)
In 23V = ls22s22p6 3s23p63d34s2 278. (c)
277. (c) 279. (b)
Third electron which is removed in third ionisation 280. (b) Zn + 2NaOH —> Na2ZnO2 +H2
potential enthalpy belongs to 3d4 subshell.
281. (b) (d) 283. (a)
282. (d)
282. 284. (c) 285. (d)
24 Cr = ls22s22p63s23 p63d54s1
286. (d) 287.
287. (d)
(d) 288. (b) 289. (c)
Third electron which is removed in third ionisation 290. (b) Cr2O2’ + 6e~ + 14IT ---- > 2Cr•;3* + 7H2O
potential enthalpy belongs to 3d6 subshell.
electron per Cr atom = 3
26Fe = ls22s22p63s23 p63d64s2
291. (a) 292. (a) 293. (a) 294. (b) 295. (a)
Third electron which is removed in third ionisation
potential belongs to 3d5 subshell.
One or More Than One Options Correct
In all elements shell and subshells are same. Required
amount of energy is based upon the stability of 1. (a,b,c) 2. (b,d) 3. (b) 4. (a) 5. (c)
d-subshell. Hence, Mn shows highest third ionisation 6. (a,b) 7. (a,b,d) 8. (a,b,c) 9. (b) 10. (c)
potential or enthalpy.
11. (a,b) 12. (a,b,c) 13. (c) 14. (a,d)
25 Mn = ls22s22p63s23p63d54s2
15. (a) 16. (a,b) 17. (a,b,d)
226. (d) In alkaline solution, KMnO4 is reduced to MnO2
(black).
Brain Twisters
2KMnO4+2H2O ---- > 2MnO2 + 2KOH + 3[O]
MnO,4 used = 30.6 x 0.1 millimol
KI + 3[O] ---- > KIO3
= 3.06 xlOr-33 mol
2KMnO.4 + 2H,0
Z
+ KI ---- > 2MnO,Z + 2KOH + KIO,«j
2 mol MnO4 = 5 mol (H2C2O4 H2O)
Hence, 2 mol of KMnO4 are reduced to MnO2 by 1 mol of
KI. thus, 3.06 x 10“3 mol MnO4 = 7.65 x 10'3 mol unreacted
(HjCA^O)
227. (a) 228. (b) 229. (d) 230. (d) 231. (b) ^^ = 1.53 xlO’2 mol
H2C2O4 HjO taken
232. (b) 233. (b) 234. (b) 235. (d) 236. (b) 108
237. (c) 238. (a) 239. (d) 240. (d) 241. (c) Used H2C2O4 H2O by MnO, = 7.650 x IO-3 mol
242. (a) 3MnO44~ ---- > MnO,2 + 2MnO74 1 mol H,C2O4 H2O = 1 mol MnO2
.-. 7.64 x 10’3 mol H,C2O4 H,0
243. (c) 244. (a) = 7.64 x IO"3 mol MnO,
245. (b) Cu+ :[Ar]3d10 all electrons paired. = 7.64 x 10"3 x 87 g pure MnO2
246. (b) 247. (c) = 0.6644 g in 0.81 g impure MnO2

248. (b) 2CuSO4 + 4KI ---- > Cu^ +K2SO4 +I2 Percentage of pure MnO2 = 1^x100
0.81
I2+KI —♦ ki3 = 82.02%
249. (b) 250. (a) 251. (c) [Ar]4s13d5 2. Cr is in the form of Cr2O}~ and Mn is in the form MnO4
252. (a) 253. (d) Steel content (as MnO4 and Cr2O2“) = 10 g in 250 mL
734 | Essential Inorganic Chemistry

100 . ,A T Cone in g/L = 0.9 x 43.5 g/L


=----- g in 10 mL
250 6
= 39.15 g/L
= 0.4 g
39.15
Thus, = 7.83 g
Cr lCr2Or —> BaCrO,4
MnO2 in 200 mL =
5
1 mol 0.5 mol 1 mol
4. lonic/molar Simplest
Species % Mole ratio
2.17 x 10-4 mol 1.085 x 10"4 mol 0.0549 g mass ratio
= 0.011284 g in 10 mb = 2.17 x 10 mol Co3 + 59.0 21.24 0.36 1
in 0.4 g steel = 0.01085 M (mol L-1)
cr 35.5 12.81 0.36 1
0.011284
Thus, percentage of Cr =
0.4
X100 sor 96 34.65 0.36 1
NH3 17 24.77 1.45 4
= 2.821%
h2o 18 6.53 0.36 1
Mn ----- > Mn074
1 mol 1 mol
Thus, empirical formula is : [Co(NH3)4(S04)(C1)(H2011
MnO;+5Fe2+ ----- -> 5Fe + Mn2*
5. Coordination number of cobalt = 6
Cr2O2“ + 6Fe2+ ----- > 6Fe3* + 2Cr3*
MnO4 and Cr2O2- both oxidises Fe2+ toFe3* Since, one mole of the complex gives one mole of AgClon
15.95x0.075 reaction with AgNO3, only one Cl is outside coordination
TV (Cr2O2- + MnO“) =
16 sphere, and two Cl satisfy primary and secondary
= 0.119625 N valency. Out of fine water molecules, four are as ligand
since, [Cr2O2-] = 0.01085 M and remaining one water unit makes the compla
= 0.0651 N hydrated. Thus comples is [CofHjO^ClglClHjO
(N = 6 x M as Cr2O2- is reduced to Cr3*) 6. Let charge on the mercurous ion = n
N (MnO4) = 0.119625 - 0.0651 = number of electrons
= 0.054525 N exchanged
= 0.010905 M (mol L"1) Reactions E°
(N = 5xM as MnO4 is reduced to Mn2*)
Mn = 0.010905 mol L-1
At anode 2Hg Hgr + ne~ E°z -x
0.005
= 1.0905 x 10-4 mol in 10 mL solution
At cathode Hg,+ + ne~ ----- > 2Hg
= 0.006 g Mn in 10 mL solution 0.05
containing 0.4 g steel
Hgr Hgr cell
=o.oov
Percentage of Mn = x 100
0.05 M 0.005 M
= 1.50% K _ 0.005 1
“ 0.05 “10
3. (A): MnO2
0.0501 1
-> Mn2* + H2O + CO2 EcellI. = E°,i - log —
MnO2 + C2O4“ + H* cell
n 10
Mn2* + -> MnSX 0.0591
0.0295=0 + log 10
MnO2 + 4HC1 -> MnCl2 + 2H2O + Cl2 n
Cl2 + 2KI -----■>, 2KC1 -r
+ xI.2 n=2
I■22 + 2Na2S2O3 ----- > 2NaI + Na2S4O(6
1. Using equation
MnO2 = C12 = I2 ^NagSgOg lOOO^w/i)
(Mn = 55) (depression in freezing point) =

20 mb of I2 containing solution = 180 mL of 0.1 N


where, Kf = molar depression constant = 1.86° mol'1 kg
NagSgOg
180x0.1 w1 = mass of solute =0.875 g
MI2) = w2 = mass of solvent =25.0 g
20
= 0.9 N A2} =0.56°
mol. wt. 87 e i = van’t Hoff factor = Number of ions from one mo’.e
Equivalent weight of MnO2 = — =43.5
2 2 complex (solute)
Thus, MnO2 = 0.9N = molar mass of solute
Chapter 16: The Transition and Inner Transition Elements | 735

1000xl.86x0.875(i)
0.56° = Fill in the Blanks
233.5x25
1. FeClL 2. oxalic acid, Fe3*
i=2
3. diamagnetic, diamagnetic
Thus, only one Cl“ is outside coordination sphere (NH3 is
4. "15, three 5. Fe* and NO+ 6. +2, NO*
not ionisable).
7. magnetic moment 8. diamagnetic, square planar
[Co(NH3)4C12]C1 [Co(nh3)4ci2]+ +cr
9. ‘wonder’ 10. corrosion.
Two ions 11. colourless (white), d-d*
12. charge transfer, O2-, Mn (+VII)
Passage Comprehension Questions 13. O“, Mn(VI), coloured.
Passage 1 14. 724 BM, smaller (Fe2* has 3d64s°, Cr has ScPAs1 EC)
1. (d) 2. (a) 3. (b) 15. nonstoichiometric, metal
Passage 2 16. equal 17. lanthanide contraction
1. (c) 2. (b) 3. (a) 18. contact, H2SO4 19. [Fe(H2O)5OH]2+, acidic
Passage 3
20. Ferrochrome 21. MnO4, Cr2O72-
1. (b) 2. (d) 3. (b) 4. (b) 5. (c) 6. (a)
Passage 4 22. CrO(O2)2 (a diperoxo compound), +6. 23. +5
1. (d) 2. (c) 24. (NH3)3Cr(O2)2 (with two peroxide) 25. CrO2*
Passage 5 26. Cr2O3, reduction, N2, oxidation.
. 1. (d) 2. (c) 3. (a) 4. (c) 27. acidic, alkali (say NaOH) 28. acidic, alkaline
Passage 6
29. Mn2*, MnO9 30. - th, - rd
1. (0 2. (a) 3. (d) 4. (b) 5. (d) 2 5 3
Passage 7 31. MnO2, MnO4 disproportionation
1. (a) 2. (b) 32. Fe, haemoglobin
Passage 8 33. MnO: 0.7
34. Mn,.29U 35. Na4[Fe(OH)6]
1. (c) 2. (b) ii in
36. Fe[Fe(CN)6F 37. mixed, FeOFe2 0.3
Passage 9
38. Fe2XO,, NaOH 39. CuSO4 40. KI, hypo
1. (a) 2. (c) O'

Passage 10
1. (b) 2. (a) 3. (a) 4. (c) 5. (d) Matrix-Match Type Questions
Passage 11 I. Only One Correct
1. (c) 2. (b) 3. (a) 1. (AM2); (B)—(4); (C)—(3); (DMD;
Passage 12 2. (AM2); (BMD; (CM4); (DM3);
1. (a) 2. (b) 3. (c) 3. (AM3); (B)-(4); (C)—(1); (DM2);
Passage 13 4. (AM4); (BM($); (CMD; (DM2); (E>—(3)
5. (AM3); (B>—(1); (C)—(2); (DM5); (E)—(4)
1. (b) 2. (d) 3. (a)
6. (AM); (B>—(3); (CMD; (DM2);
Assertion & Reason 7. (AM2); (BM3); (C>—(4); (DMD;
1. (a) 2. (b) 3. (b) 4. (b) 5. (b) 6. (a) 8. (AM5); (BM4); (C)—<3); (DM2); (E)—(1)
7. (e) 8. (b) 9. (d) 10. (d) 11. (c) 12. (b) 9. (AM3); (BMD; (CM2); (DM5); (E)—(4)
13. (a) 10. (AM2-Q); (BM1-P); (C)—<3-R); (DM4-S);
II. (AM3); (B>—<1); (C)-<2); (DM4)
True & False 11. One or More Correct
1. T 2. T 3. F 4. T 5. T 6. F
1. (AMI,3,4); (B)—(1,4); (C>—<2); (DM2,5)
7. F 8. T 9. T 10. T 11. F 12. F
13. T 14. T 17. F 18. F (EMI,4)
15. T 16. F
19. T 20. F 21. T 22. T 23. T 24. F 2. (A)—(2,6); (B)—(1,5,7); (C>-(2,3,6,7);
25. T 26. F 27. F 28. T 29. F 30. T (DM2,4,6,7) (E)—(2,6)
31. F 32. T 33. T 34. F 35. T
736 | Essential Inorganic Chemistry

Integer Answer Types


d
Set I 1
2.
Questions—► 1 2 3 4 5 £_ 7 8 9 a
Answers —► 7 2 5 6 1 4 0 9 8 b
© © © © © © © © © c
0 © © (D <3> © CD ©
© © © © © ©I© © ©
© © © © © ©I© © ©
© © © © © © © ©
© © © © © © © © © 10
© © © © © © © © © 3. I
® ® ® ® ® ® ® ® ®
© © © © © © © © ©
© © © © © © © © © 7->
Setn
Questions—► 1 2 3 4 5 £ 7 8
Answers —► 6 0 7 5 2 3 1 8
© © © © © ©|© © Period : 7th
© CD CD © CD Group : 10 (VIIIB)
© © © © © © © © Atomic no.: 110
© © © © © ©T© © (It lies below Pt)
6c?-series element of d-block
© © © © © ©
Electronic configuration:
© © © © © j©® ©
© © © © [Rn]5/'146d97s1
© ©
® ® ® ® ®.® ®
© © © © © ®|® © R?
Set III
© © © © © © © ©
4. £
Questions—► 1 2 3 4 5 £ 7 8 i

Answers —► 7
©
3
©
7
©
3
© © ©|©
5 6 5
©
4 o
© © © (©_©
© © © © © ©^_© ©
© © © © © (©_© ©
© © © © © ©|© ©
© © © © © © © ©
© © © © © © © © 5.
® ® ® ® ® ®r® ® XiX
© © © © © ©Lj® ©
© © © © © © © ©
Test Yourself on the Periodic Table
(a) | insoluble sulphides of 3d-series
i. a : Co (b) [X] insoluble sulphides of 4d-series
b a b : Cr (c) p7! s2p6 configuration
c
c : Zr (d) M3+ with [Ar] configuration
d : Pr

d
"Gold, silver, copper, tin and iron were probably the
metals discovered by our ancestors"

Metallurgical Extraction

Metallurgy is the science and technology of separating metals


from their ores and of compounds alloys. An alloy is a solid solution
either of two or more metals, or of a metal or metals with one or more
non-metals.
Recovery of a metal from its ores involves three principal steps
(three P’s) ■ The Occurrence and Isolation of the
Elements
Preparation of Production of Purification of
ore the metal the metal ■ Extraction of Copper
■ Extraction of Magnesium
■ Extraction of Lead
■ Extraction ofTin
■ Extraction of Silver
reparation I
reduction, ' ■ Extraction of Aluminium
urification;/y
• Extraction of Iron
■ Extraction of Zinc

The Occurrence and Isolation of the Elements


• Most metals occur in nature as minerals; silicates and
aluminates are the most abundant minerals (Table 17.1), but
they are difficult to concentrate and reduce and therefore
generally unimportant for commercial source of metals. Other
minerals are oxides as haematite (Fe2O3) and rutile (TiO2) and
sulphides as cinnabar (HgS) and galena (PbS). Principal ores of
some important metals have been summarised in Table 17.2.
738 | Essentia/ Inorganic Chemistry

DEEP Focus • Other very abundant elements are nitrogen (78% ■


• The early transition metals on the left hand side of the the atmosphere) and hydrogen, which occurs as wate
d-block generally occur as oxides, and the more in the ocean.
electronegative, late transition metals on the right hand
side of the d-b)ock occurs as sulphides. Less Table 17.1 The Most Abundant Elements
electronegative (more electropositive) metals can
Element % of Earth's crust
form ionic compounds by losing electrons to oxygen
(and thus oxides are possible). More electronegative 1. Oxygen 46.4
(less electropositive) metals tend to form covalent 2. Silicon 28.2
sulphide minerals with less electronegative sulphur.
3. Aluminium 8.32
• We might also expect to find oxide ores for the s-block 4. Iron 5.63
metals and sulphide ores for the more electronegative 5. 4.15
Calcium
p-block metals. In fact, sulphide ores are common for
6. Sodium 2.36
the p-block metals, except for Al and Sn.
7. Magnesium 2.14
• Oxides of the s-block metals are strongly basic hence,
form carbonates and silicates with acidic oxides as 8. Potassium 2.09
CO2 andSiO2 9. Titanium 0.57
10. Hydrogen 0.14
CaO +CO2 * CaCO3
basic acidic 11. Phosphorus 0.112
12. Manganese 0.106
MgO +SiO2 ■» MgSiO3
basic acidic P (1.1%) [ Mg (0.50%)
' K (2.095%) K(0.20%)
Ca(1.4%)
• Only gold and platinum-group metals (Ru, Os, Rh, Ir, Mg (2.14%) . Ti (0.57%)
N (2.6%) \ J S (0.20%)
Na (2.36%) \ H (0.14%) .Na (0.14%)
Pd and Pt) are sufficiently unreactive to occur as free
Ca(4.15%) Vat H (10%)
metals (in natural state). Fe (5.63%j^2^\\\
• The most abundant elements in the earth’s crust (by Al (8.32%)^A?\V
weight) are shown in Table 17.1 and Fig. 17.1. This Oxygen Oxygen
(46.4%) Carbon (61%).
indicates : (23%)
Silicon
- oxygen is the most abundant element, (28.2%)
- aluminium is the most abundant metal,
- first five elements comprise almost 92% by weight,
(a) (b)
and
Fig. 17.1 Ten most abundant elements (a) In the earth's crust
- first ten elements make up over 99.5% of the
earth's crust. (b) In the human body

1 18
2 13 14 15 16 17

Na Mg 3 4 5 6 7 8 9 10 11
K Ca Ti V Cr Mn Fe
:. y. ;•.
Sr Zr Nb Mo Tc Ru Rh Pd Sn
Ba La Hf Ta W Os Ir Pt Au

.■iW/.-..?;

Chlorides Silicates Carbonates Phosphates Oxides Free metals Sulphides


Fig. 17.2 Primary mineral sources of metals. The s-block metals occur as chlorides, silicates, and carbonates. The d- and p-block
metals are found as oxides and sulphides, except for the group 3 metals, which occur as phosphates, and the platinum-group metals
and gold, which occur in uncombined form. There is no mineral source of technitium (Tc in group 7), a radioactive element that is made
in nuclear reactors.
Chapter 17: Metallurgical Extraction | 739

t
Table 17.2 Principal Ores of Some Important Metals Type

Metal Ore Formula Oxides AI?O3 • 2H,0 (bauxite), Cu2O (cuprite), MnO2
(pyrolusite), SnO2 (cassiterite) AI2O3 (corundum),
Aluminium Bauxite AI2O3 ■ x H2O
Fe2O3 (haer,.alite), TiO2 (rutile), ZnO (zincite), Fe3O4
Chromium Chromite FeCr2O4 (magnetite)
Copper Chalcopyrite CuFeS2 Phosphates Ca3(PO4)2 (phosphate rock), Ca6(PO4)3OH
Iron Haematite Fe2O3 (hydroxyapatite), 3Pb3(PO4)2 -PbCI2
Lead PbS (pyromorphite)
Galena
Manganese Pyrolusite MnO2 Silicates NaAISi3O8 (albite), Mg3(Si4O10)(OH)2 (talc)
CaMg3Si4O12 (asbestos) K2OAI2O3-6SiO2
Mercury Cinnabar HgS
(felspar) 2ZnO-SiO2 H2O (hemimorphite)
Tin Cassiterite SnO2 KAI3H2Si3O12 (mica)
Titanium Rutile TiO2 Sulphides Ag2S (argentite), CdS (greenockite), Cu2S
Ilmenite FeTiO3 (chalcocite), FeS2 (pyrite), HgS (cinnabar), PbS
Zinc Sphalerite ZnS (galena), ZnS (sphalerite) CuFeS2 (chalcopyrite),
NiS (nickel blende)
Table 17.2 and Fig. 17.2 summarise principal type of Sulphates BaSO4 (barite), CaSO4 (anhydrite), MgSO4'7H,O
minerals as anion. Other minerals have been given in (epsomite). PbSO4 (anglesite) CaSO4 ■ 2H2O
Appendix I (gypsum)

Table 17.3 Principal Types of Minerals Representing as


Anions Benefication of Ores
Type Most of the ores available in nature contain large
amount of impurities, i.e., gangue. The pretreatment of
Uncombined Ag, Au, Bi, Cu, Pd, Pt
metals
ores, based on physical properties and without bringing
(Native ores)
out any major chemical change in the ore, is known as
Carbonates CaCO3 (calcite, limestone), MgCO3 (magnesite),
benefication or concentration of the ore or
CaCO3 MgCO3 (dolomite) PbCO3 (cerussite),
ZnCO3 (smithsonite), Cu(OH)2 -CuCO3 (malachite), ore-dressing.
FeCO3 (siderite) Table (17.4) summarises the various methods adpoted
Halides CaF2 (fluorite), NaCI (halite or rock salt), KCI (sylvite),
for the ore-dressing
Na3AIF6 (cryolite), KCI-MgCI2 • 6H2O (carnallite),
AgCI (horn siliver)

Table 17.4 Methods of Concentration of the Ore


Method Ores specification Process

, 1. Gravity separation Based on the difference in the specific gravities Crushed ore is kept on top of a slope table, which is made to
(Tabling) by hydraulic of the ore and gangue. Cassiterite or tin-stone, vibrate. A stream of water is passed in the direction
washing chromite and
chromite and pitchblende are concentrated
pitchblende are concentrated perpendicular to the slope. The lighter particles are thrown up
by vibration and are removed by water stream
2. Magnetic separation Based on the difference in magnetic properties The pulverised mineral is passed oyer^a rubber belt which
moves on a pulley in a magnetic field. The non-magnetic
of minerals. If the ore but not the gangue is
gangue particles fall off in a vertical position when the belt
attracted by a magnetic field, it can be passess over a pulley, but the magnetic ore clings to the belt.
concentrated to yield a sample which is rich in When the belt is out of magnetic field, the ore separates,
metal. Magnetite (Fe3O4), haematite (Fe2O3), (Fig. 17.3)
wolframite (FeWO4), chromite (FeO- Cr2O3) and
ilmenite (FeOTiO2) separated from Powdered ore
non-magnetic impurities Electromagnets

Magnetic belt
Gangue
Magnetic

.sSSfciL.
—.Sr.7.7.V

Fig. 17.3 Magnetic separation of ores


740 | Essential Inorganic Chemistry

Method Ores specification Process


3. Froth flotation process Used to concentrate sulphide ores, based on the Refer Fig. 17.5. Sometimes depressants are used to preve<«
of different
difference in wettabilitycertain type minerals ...........
of particles from forming the froth with the bubbles
NaCN is used as a depressants in the separation of CuS and
PbS. NaCN acts a depressant for ZnS but does not prevent Pb$
from the formation of the froth, due to water soluble compie/
formation.
ZnS + 4NaCN---- > Na2[Zn(CN)4] + Na2S
Some additives increase the efficiency of this process. Cresos
(froth stabiliser) increase the life time of froth.
4. Leaching Useful in case the ore is soluble in a suitable Leaching of bauxite
solvent. AI2O3(s)+2OH"(aq) + 3H2O —> 2AI(OH);
Pure Al2O3 is obtained from the bauxite ore in the dilution Al/n|_|\ + OH’ a > Al-O-
Baeyer’s process by leaching, also employed in nn.co2 y -c j
concentration of silver and gold ores when
leaching is done using NaCN in presence of air. Fe2O3 if present as impurity is not affected.
Leaching of silver and gold
4M+8CN- + 2H2O + O2 [M(CN)2]~
soluble
(M = Ag or Au)
Ag2S + 4CN” » 2[Ag(CN)2f + S2'
5. Calcination Used when concentrated ore is in the form of 2AI(OH)3 A
AI2O3 + 3H2O
hydroxide or carbonate; volatile matter is burnt
away. It is done in a reverberatory furnace. A
CaCO3 CaO+CO2
It also drives out any organic matter that remains with the ore
after concentration. It also removes moisture or water d
crystallisation
2Fe2O3 • 3H2O---- > 2Fe2O3 + 3H2O
6. Roasting Sulphide ore is heated in a regular supply of air at 2ZnS + 3O2 2ZnO + 2SO2
a temperature below the melting point of the
metal. It converts sulphide into oxide or sulphate; 2Cu2S + 3O2 2Cu2O + 2SO2
part of sulphide may also act as a reducing
agent. 2Cu2O + Cu2S A 6Cu + SO2
Process is carried out in a reverberatory furnace. The impurities
which form volatile oxides are also driven off during roasting
P4 + 5O2 > P4O10
As4 + 5O2 ) As4O10
C + O2 —> CO2 T
S + O2 —> SO2 T

Separation and Extraction of Elements • Diamonds are found in the earth and are obtained by
The different methods for separating and extracting mechanical separation of large amount of earth and
elements may be divided into five classes : rock.
Mechanical Separation : A surprisingly large Thermal Decomposition Method : A few
number of elements occur in the free elemental state. They compounds will decompose into their constituent elements
have remained in the native form because they are simply by heating:
unreactive as Cu, Ag, Au and Pt. 2AsH3 redhottube* 2As + 3^
• Au is found in the native form, as grains in quartz, as
nuggets and in the silt of river beds because of very 2NaN3 2Na + 3N2
sodium azide
high density (19.3 gem-3). Ag and Cu are sometimes 230°C
found in the native form as nuggets. All the three In Mond's process : Ni(CO)4 ------ > Ni + 4C0
metals lie below hydrogen and other reactive metals
in electrochemical series and thus are easily displaced Zrl, W, A -> Zr + 2^
to the metallic form. • 2BI3 W, A 2B + 3Ij
• Palladium and platinum are also found as native
metals. • In van Arkel-de-Boer process: 2HgO —2Hg + O2
• Liquid droplets of Hg are found associated with • Ag2CO3 Ag2O + CO2 2Ag+^a1
cinnabar, HgS.
Chapter 17: Metallurgical Extraction | 741

DEEP Focus a large amount of energy (1675 kJ mol-1) on oxidation to


Oxides of the metals lying below Hg (and that of Hg) in A12O3. This is the basis of the thermite process :
electrochemical series decompose on heating giving metal. 3Mn3O4 + 8Al---- > 9Mn + 4A12O3
A reaction is spontaneous if reducing agent is above I^Og + 2A1---- > 2B + A12O3
oxidising agent in ECS.
Zn and Mg can reduce Fe2* to Fe (in prevention of Ci2O3 + 2A1---- > 2Cr + A12O3
rust) since both are above Fe in ECS. Magnesium is used in a similar way to reduce oxides
in certain cases where the oxide is too stable to reduce,
Displacement of Element by Another : In
electropositive metals are used to reduce halides :
principle any element may be displaced from solution by
another element which is higher in electrochemical series TiO2 + 2Mg--- > Ti + 2MgO
(ECS).
TiO2 + 4Na---- > Ti + 2Na2O
• Zn displaces Cu from CuSO4 solution since Zn is above iTL-fil nw 1000-1150°C rrv zii
Cu in ECS: TiC14 + 2Mg----- - --------- > Ti + 2MgCl2
Kroll process
Cu.2+
2+ + Zn---- > Zn2+ + Cu
IMI process
• Cu displaces Ag from AgN03 solution due to the same TiCl4 + 4Na - Ti + 4NaCl
fact: (IMI—Imperial Metal Industries)
Cu + 2Ag+ ---- > Cu2+ + 2Ag • Reduction of oxides with H2:
• Sea water has Br“. Cl2 can oxidise Br" to B^ CuO + Ha ---- > Cu + HjO
2Br" + Cl2 ---- > 2d' + Big Co3O4 + 4HJ -----> 3Co + 41^0
This method is quite applicable when displaced metal This method is not widely used, because many
is costlier than the other metal which is oxidised. ECS is in metals react with H2 at elevated temperatures
order: forming hydrides. Also there are chances of
Li > K > Ca > Na > Mg > Al > Mn > Zn > Fe > Cu > explosion of H2 with O2.
Hg > Ag > Au
• Self reduction:
High Temperature Chemical Reduction
Sulphides of Cu, Pb and Sb when roasted in air are
Methods : Once an ore has been concentrated, it is converted partially into oxides. On further roasting in the
reduced to the free metal, either by chemical reduction absence of air, self reduction takes place:
or by electrolysis as given in Tables 17.5 and 17.6. The Cug roast in airv CuQ CuS, roast in the
■> Cu + SO2
method used depends on the activity of the metal as absence of air
measured by its standard reduction potential. The most It is carried out in a Bessemer converter or
active metals have the most negative standard reduction Pierce-Smith converter.
potentials and are the most difficult to reduce; the least
active metals have the most positive standard reduction Electrolytic Reduction
potentials and are the easiest to reduce.
To get metal of high purity, electrochemical refining is
• Reduction by carbon:
done making impure metal as anode.
Fe2O3+3C bla5tfumace> 2Fe+ 3CO • In an aqueous solution
1200°C> Zn+CO It is used in cases when the electrolyte is soluble in
ZnO + C
water and products do not react with it. Copper, Zn, Ag
electric furnace p and other metal are obtained by the electrolysis of aqueous
2Ca3(PO4)2 + 10 C+ 6SiO2 ------------------ * r4
solution of their salts (ZnSO4, AgNO3, CuSO4). Impure
+ 6CaSiO3 + 10CO metal is oxidised at anode and is deposited at the cathode
2000°C ; (of pure metal).
MgO + C * Mg+ CO
electric furnace • In fused melts:
PbO + C---- > Pb+CO Na, K, Ca, Mg etc., react with H2O hence, these metals
• Reduction by another metal: are extracted by electrolysis of fused halides.
If the temperature needed for carbon to reduce an - Na and Cl2 are obtained by electrolysis of fused
oxide is too high for economic or practical purpose, the NaCl.
reduction may be affected by another highly - Al is obtained by the electrolysis of fused mixture of
electropositive metal such as aluminium, which liberates A12O3 and Na3[AlF61 (cryolite).
742 | Essential Inorganic Chemistry

• In other solvents:
Electrolysis can be carried out in solvents other than water. F2 reacts violently with hence, it is produced by
electrolysis of KHF2 in HF.

Table 17.5 Reduction Methods for Producing Some Common Metals


Metal Reduction method
Least active Au. Pt None; found in nature as the free metal

I
Cu, Ag, Hg Roasting of the metal sulphide
V, Cr, Mn. Fe Reduction of the metal oxide with carbon, hydrogen or a more active metal
Al Electrolysis of molten A12O3 in cryolite
Most active Li, Na, Mg Electrolysis of the molten metal chloride_________________________

Table 17.6 Some Specific Reduction Processes


Metal Compound (ore) Reduction process Comments
Mercury HgS (cinnabar) Roast reduction; heating of ore in air
HgS + O2 Hg + SO2
Copper Sulphides such as Cu2S Blowing of oxygen through purified molten Cu2S Preliminary ore concentration and
(chalcocite) purification steps required to remove FeS
Cu2S + O2 2Cu + SO2 impurities
Zinc ZnS (sphalerite) Conversion to oxide and reduction with carbon Process also used for the production of
lead from galena PbS
2ZnS + 3O2 2ZnO + 2SO2
ZnO + C —Zn + CO
Iron Fe2O3 (haematite) Reduction with carbon monoxide
2C(coke)+ O2 2CO
Fe2O3 + 3CO 2Fe + 3CO2
Titanium TiO2 (rutile) Conversion of oxide to halide salt and reduction with an Also used for the reduction ofUF4 obtained
active metal from UO2, pitchblende
TiO2 + 2CI2 + 2C TiCI4 + 2CO
TiCI4 +2Mg Ti+2MgCI2
Tungsten FeWO4 (wolframite) Reduction with hydrogen Used also for molybdenum
WO3 + 3H2 W + 3H2O
Aluminium AI2O3 • nH2O Electrolytic reduction (electrolysis) in molten cryolite.
(bauxite) Na3[AIF6] at 1000°C.
A 4AI+3O2
2AI2O3 -A>
Sodium NaCI (sea water) Electrolysis of molten chlorides Also for calcium, magnesium, and other
active metals in groups IA(1) and IIA(2)
2NaCI 2Na + Cl2
Chapter 17: Metallurgical Extraction | 743

Table 17.7 Refining of Crude Metals Methods Metals purified

Methods Metals purified 5. Vapour phase In Mond process for the refining of nickel
refining
1. Distillation Ni + 4CO Ni(CO)a
Zinc, mercury and cadmium. This impure
methods is used in cases where metal
Ni +4COT
has lower boiling point than impurities. pure
Impure metal is heated to the boiling point In van Arkel method for zirconium
in a closed retort when vapours formed Zr + 2I2 -^21^ Zrl4
are condensed to the pure metal. impure
Non-volatile impurities are left unreacted. 1800°C
--------- > Zr + 2I2
2. Electrolysis Copper, gold, silver, lead, zinc pure
aluminium
Impure metal is made anode and the pure ORE
Flotation and Removal of
metal is cathode. The net result is the gangue
^other methodsy ,f \
transfer of pure metal from anode to
cathode _____ Partially concentrated ore
At anode: Leaching/other Separation of
Cu —> Cu2* + 2e~ operations other minerals
(impure) Concentrated ore
At cathode: < Roasting/
Removal of
Cu2* + 2e‘ Cu calcination/smelting
(pure) other metals
Cu (impure) —> Cu (pure)
I Impure metal
<ReductiorO
3- Liquation A low melting metal like tin can be made
to flow on a sloping surface and thus Removal of
Electrorefining, Pure metal other elements
separated from higher melting impurities. zone refining^

Charge
\ | / Impurities
.Sloping Factors Influencing the Choice of Extraction
' hearth
Screen
Process
Pure The type of process used commercially for any
5E molten
* metal
particular element depends on a number of factors :
- Is the element unreactive enough to exist in the
III!"
~|~ free state?
II I ~
I I I
I l­
li
T^T
_i~
- Are any of the compounds unstable to heat?
Furnace
- Does the element exists as an ionic compound, and
is the element stable in HgO?
4. - Is there a cheap element above it in the ECS which
Zone refining Metal of high purity are obtained. Silicon,
can be sacrificed to displace it from solution?
germanium, boron, gallium, indium are
- Does the element occur as sulphide ores which can
purified. (which are used in
be roasted, or oxide ores which can be reduced by
semiconductors). It is based on the fact
carbon (which can be the cheapest) or by Al, Mg
that impurities are more soluble in the
and Na (which can be still costlier)?
melt than in the pure metal. Impure metal
- If all other methods fail, electrolysis is preferred
(in the form of rod) is moved through a
(in aqueous solution which is the best to get metal
circular heater. When melting point is
of highest purity or in fused state which is again
reached, pure metal melts and impurities
costlier).
are concentrated in a particular zone and
pure metal in the other zone and
crystallises out. Process is repeated
Thermodynamics of Extractive Metallurgy
several times. It is interesting to think of the reduction of ZnO with C
as involving a competition between Zn and C for 0 atoms.
744 ] Essential Inorganic Chemistry

We consider following values at 1100°C for the Main source is copper pyrites. Some copper is also
oxidation of Zn and C : found in native state.
1: 2Zn + O2 ---- > 2ZnO AG°=-360kJ
II: 2C + O2 ---- > 2CO AG° =- 460 kJ Pyrometallurgical Method (Dry Method)
(I)—(II) gives Sulphide ore is converted into copper in following
2Zn + 2CO 2ZnO + 2C, AG° = + 100 kJ steps :
or (III): Zn + CO---- > ZnO + C, AG° =+50 kJ mol"1 Step I : Concentration by Froth-Flotation
(IV): ZnO + C---- > Zn + CO, AG° =—50 kJ mol-1 process (Fig. 17.5)
For reaction III, in which Zn reduces CO, AG° is air
positive indicating that this reduction is not spontaneous
(in forward direction), but for the reaction IV with
Cl agitator

negative value of AG°, in which C reduces ZnO is □ froth


spontaneous, (in forward direction).
Why must we carry out the reduction of ZnO with C at o o 0O O
1100° C? From an energy stand point it would certainly be °°° oo o
o a o
more economical to carry it out at a lower temperature.
The graphical representation of Fig 17.4 (Ellingham
diagram) is useful tool to help answer such questions. The
lines (2C + O2 ---- > 2CO) and (2Zn + O2 ---- > 2ZnO)
cross at about 950°C. At about this temperature Zn and C /gangue.
have equal affinities for O atoms : AG° for reaction (TV) is
zero. Above this temperature AG° is negative, and below
this temperature AG° is positive. To make reaction (IV) go Fig. 17.5 Concentration of an ore by flotation
essentially to completion we need a temperature
somewhat in excess of 950°C. Powdered ore is suspended in water in a large vat,
together with suitable additives, and the mixture is
0 ---------------------------
2Zn + O2—»2ZnO agitated with air. Particles of ore become attached to air
bubbles, rise to the top of the vat, and are collected in the
-200 2C + O2 2CO overflow froth. Particles of the undesired waste rock
t -400 -~?z<-950oc (gangue) fall to the bottom.
2 The success of this method depends on the use of
b.p. /
0 -600 -
< f
m.p.
proper additives—a material that will produce a stable
-800 ■ 2Mg + O2—►2MgO foam (frother) and a substance (collector) that coats the
particles of ore but does not “wet” the particles to be
-1000 -
t
b.p. rejected. Pine oil is widely used as a frother and sodium
f
m.p. ethyl xanthate (CjHgOCSgNa) as a collector.
i i __ l_
0 500 1000 1500 2000 Step II: Roasting is done in multiple hearth furnace.
Temperature, °C —► Volatile impurities are removed as thin oxide (As2O3,
Fig. 17.4 The points noted by arrows are the melting and Sb2O3, P2O5). It also converts iron sulphide to iron oxide.
boiling points of the metals zinc and magnesium. AG0 as a The copper remains as the sulphide if the temperature is
function of temperature for some reactions of extractive kept below 800°C.
metallurgy. A
2FeS + 3O2 2FeO + 2SO2 T
Step III : Smelting of the roasted ore is done in a
Extraction of Copper furnace at 1400°C. In this furnace charge melts and
separates into two layers. The bottom layer is
Occurrence
copper-matte, consisting chiefly of the molten sulphides of
• Cuprite Ct^O
copper and iron. The top layer is a silicate slag formed by the
• Malachite CuCO3Cu(OH)2
reaction of oxides of Fe, Ca, and Al with SiO2 (which either is
• Azurite 2CuCO3 • Cu(0H)2
present in the ore or is added)and is removed.
• Copper pyrites or
Chalcopyrites FeO + SiO2 ---- > FeSiO3
CuFeS2
Chapter 17 .Metallurgical Extraction | 745

Step IV: Bessemerisation (or Converting) occurs Hydrometallurgical Process


in another furnace where air is blown through the molten
copper matte. First, the remaining iron sulphide is (Wet Process)
converted to oxide (as in step II) followed by slag formation Step I : Leaching with HjSC^ converts salts into
(as in step III). The slag is poured off and air is again sulphates:
blown through the furnace. Now the following reactions CuCO3 + HgSQj -±> CuSO4 + HaO + CO2
occur, yielding a product that is about 98 to 99% Cu.
ZC^S + 3O2 ---- > 2Cu2O + 2SO2 Cu(OH)2 + H2SO4 CuSO4 + 21^0
A
2Cu2O + CujS-----> 6Cu + SO2 CuFeS2 + 211,804 CuSO4 + FeSO4 + 21^8
The product of this step is called Blister copper Step II: Electrochemical Displacement : Fe is
because of frozen bubbles of SO2. It can be used where high above Cu in ECS, hence if Fe scrap is added to CuSO4
purity is not required. solution, Cu is displaced :
Step V: Electrorefining : CuSO4 + Fe FeSO4 + Cui
Anode : impure copper
CuFeSj Froth Flotation Roasting SO2
Cathode : pure copper
Electrolyte : 3.5% CuSO4 + 15% HjSQi
Cu2S + FeO
Voltage : 0.3 to 0.5 V
Temperature : 50.55°C Blister Cu — Bessemerisation Matte
Cu from impure copper anode goes into solution and is
deposited at the cathode :
T T Smelting FeSiO3
Electrolysis FeSiO3 so2 SiO2
Anode „ Cathode I
Cu------- > Cu.22++ + 2e~ Cu2+ + 2e" ------- » Cu
Pure Cu.
Anode-mud (anode-sludge) contains Au, Pt, Ag and
other precious metals (noble metals).

Target Practice 1
1. NaCl solution is neutral. If it undergoes electrolysis, pH (iii) Chromite
increases/decreases/remains constant (iv) Copper pyrites
2. When Cu turnings are added to AgNO3 solution, it turns 8. A mineral is rich in ZnS and PbS. Give scheme to
blue. It is due to formation of concentrate it by froth flotation process.
3. We have CuSO4 solutions (blue) in two beakers A and B. In
beaker A we add Ag pieces and in B we add Zn pieces. What
will happen? 9. What are the main steps of pyrometallurgy?
InA
In B
4. Name the process (unbalanced reactions) 10. Explain the term hydrometallurgy with example.
(a) 2Cu2S + 3O2 ---- > Cu2O + SO2
Cu2O + Cu2S ---- > Cu + SO2
(b) Ni + 4CO 200°C, Ni(COk Ni What are the criteria of selection of a good reducing agent
11.+ 4COT
•1 for extraction of metals?
(c) Zn + 2L 600°C Zrl.4
a
Zn + 2L*
5. Write three minerals (in each case) which occur as
(a) Sulphide 12. Why is carbon reduction not used to obtain certain metals
(b) Oxide from their ores?
(c) Oxosalts
13. From the Ellingham diagram (Fig. 17.4) compute the
temperature of reduction of MgO, CaO, TiO, and A12O3 by
6. You are living very near to sea-coast. What types of carbon
minerals can be present in direct supply of tap water?
(i) (ii)
(iii) (iv)
14. Define the terms
7. What method can be used for benefication of the following? (a) Copper matte
(i) Haematite
(b) Blister copper
(ii) Cassiterite
(c) Anode-mud
746 | Essential Inorganic Chemistry

15. Minerals are rich in the following as given (I)


Mineral I: Rich in AgCl, CuS, FeS (ID ......................................................................................
Mineral II: Rich in ZnS, CuS, FeS (Ill) ..............................................................................
Mineral III: Rich in Ag2S, ZnS, FeS 16. Chemical reactions of the hydrometallurgical extracton of
If concentration of the above minerals are done separately copper are
without knowng the presence of their contents, metals
obtained in the native state from

Answers
1. Increases (NaCl in neutral; due to electrolysis NaOH is formed powdered ore is first treated with an aqueous solution of i
which being basic has pH > 7). suitable chemical whereby the metal is obtained in the fonncf
2. Cu*’(aq) its soluble salts leaving behind gangue particles, (leaching)
3. In A : no reaction (Ag is not oxidised) CuO + H2SO4 ---- > CuS04+H20
In B :Cu2 + is reduced to Cu (which is displaced) by Zn which is AgCl + 2NaCN ---- > Na[Ag(CN)2] + NaCl
oxidised. Solution becomes colourless or fades.
Ag2S + 4NaCN 2Na[Ag(CN)2] +NajS
4. (a) Bessemerisation (b) Mond’s process (c) van Arkel
5. (a) Sulphide Ag2S (argentite), HgS (cinnabar). PbS (galena) 11. A good reducing agent for extraction of metals should posses
(b) Oxide Cu2O (cuprite), Fe3O4 (magnetite), TiO2 (rutile) the following properties
(c) Oxo salts BaSO4 (barite), Ca3(PO4)2 (phosphate rock). (i) It should be cheaper than the metal being produced.
CaCO3 (calcite) (ii) It should be able to reduce the metal compounds to fra
metal in a pure form at a suitable temperature.
6. (i) NaCl (ii) NaBr (iii) Nal (iv) MgCl2
(iii) It should not combine with the metal being produced.
7. (i) Magnetic separation
(ii) Gravity method 12. Carbon reduction is not used to obtain certain metals frtta
their ores because it can combine with them to produce metal
(iii) Magnetic separation or gravity method
carbides.
(iv) Froth flotation process
13. MgO : 1935 K
8. NaCN is added benefication by froth flotation process (FFP). CaO :2445K
ZnS is converted to soluble complex Na2[Zn(CN)4] and PbS
TiO2 :1865K
goes into froth by FFP.
A12O3 :2570 K
9. In pyrometallurgy, the concentration ore is heated to a high
temperature and reduction is done with a suitable reducing 14. Refer text
agent. Steps involved are 15. I: Ag II: Cu ni: Ag
(a) Calcination, (b) Roasting, (c) Smelting 16. CuO + H2SO4 > CuS04+H20
10. Hydrometallurgy is generally used in the case of low grade CuCO3+H2SO4 > CuSO4+H2O +C02
ores, which cannot be concentrated economically. The
Cu2S + 2H2SO4 + 2O2 ---- > 2CuSO4 +S02+2H20

Practice Exercise 1
1. Look at the location of elements A, B, C and D in the 4. Imagine a planet with an atmosphere that contains 02 an!
following periodic table: SO2 but no CO2. Give the chemical composition of the
minerals you would expect to find for the alkaline earth
metals on such a planet.
5. Na metal can’t be obtained by electrolysis of aqueous NaCl
A B solution. Explain.
C 6. (A) A12O3 can’t be reduced by carbon to get Al metaL
D Explain
(B) NaCN is sometimes added in the froth flotation process
as a depressant when ZnS and PbS minerals are
expected because
(a) Pb(CN)2 is precipitated while no effect on ZnS
(b) ZnS forms soluble complex Na2[Zn(CN)4] while PbS
Predict whether these elements are likely to be found in
forms froth
nature as carbonates, oxides, sulphides or in uncombined (c) PbS forms soluble complex Na2[Pb(CN)J while ZnS
state. forms froth
2. List three metals that are found in nature as oxide ores. (d) NaCN is never added in froth flotation process
3. Explain, why early transition metals occur in nature as 7. Cu can reduce Ag+ to metallic Ag but Ag can't reduce Cu3'
oxides but the late transition metals occur as sulphides. to metallic Cu. Explain.
Chapter 17: Metallurgical Extraction | 747

Among the methods for extracting metals from their ores 11. Write chemical equations for metallurgical processes to
are (i) roasting a metal sulphide, (ii) chemical reduction of represent:
a metal oxide, and (iii) electrolysis. The preferred method (a) roasting of galena (PbS),
depends on E° value for the reduction half-reaction: (b) reduction ofCu2O using charcoal as a reducing agent,
Mn+(aq) + ne~ ---- > M(s) (c) deposition of pure silver from an aqueous solution of Ag”,
(a) Which method is preferred for the metals with the most 12. Write chemical equations to represent the most probable
negative E° values? outcome in each of the following. If no reaction is likely to
(b) Which method is preferred for the metals with the most occur, state so:
positive E° values? (a) CdCO3 (b) MgO
9. The roasting of an ore of a metal usually results in (c) SnO2 +CO A (d) CdSO4(aq) ^^'y318)
conversion of the metal to the oxide. Why does the roasting
of cinnabar, HgS, produce metallic mercury rather than an (e) HgO (0 MgO + Zn
oxide of mercury? 13. Using data in Fig. 17.4, predict whether the reduction of
10. A metal is in combined state as sulphide. Identify the steps MgO with C is spontaneous or not at 1500°C?
(A), (B), (C) 14. Name two metals and two non-metals obtained by
electrolysis.
Sulphide A Oxide B Impure metal
15. The following reactions take place during the extraction of
copper from copper ore:
(a) 2Cu2S(Z)+ 3O2(g)---- > 2Cu2O(Z) + 2SO2(g)
Pure metal (b) 2Cu2O(/)+ Cu2S(Z)---- > 6Cu(Z) + SO2(g)
Identify the oxidising and reducing agents.

Answers
1. A: Oxide B : Sulphide C: Carbonate D : Free metal A
12. (a) CdCO3 CdO + CO2
2. Fe asFe3O4, Ti as TiO2, Sn asSnO2 3. Refer text (b) No reaction.
4. MSO4,MSO3 (c) SnO2 + 2CO Sn + 2CO2
5. When aqueous NaCl solution is electrolysed, Cl2 is formed at
the anode and Na at the cathode. But Na is very reactive with (d) CdSO4(aq) H2SO4 + O2 + Cd
H2O forming NaOH and H2. Thus, Na can’t be prepared by at anode at cathode
electrolysis of aq NaCl (e) 2HgO -±> 2Hg + O2
NaCl(aq) Na+ + Cl“
(f) no reaction
At anode: 2d" ---- > Cl2 + 2e” 13. From Fig. 17.4 at 1500°C
At cathode: 2Na+ +2e~ ---- > 2Na 2C + O2 ---- > 2CO, AG° « -530 kJ
2Na + 2H2O---- > 2NaOH + H2 2MgO---- > 2Mg + O2, AG° => + 730kJ
6. A. Al has greater affinity forO2 that carbon has, hence C can't
be used to reduce A12O3 to Al. Thermodynamically for this 2MgO + 2C---- > 2Mg + 2CO, AG° =+ 80 kJ
reaction:
Or MgO + C —> Mg + CO, AG0 = + 40 kJ mol
A12O3 + 3C---- > 2AI + 3CO
non-spontaneous AG0 being positive.
ACy is positive hence, reaction is non-spontaneous.
Remember AG° = AG°/- (products) — AG0^ (reactants)
B. (b)
7. Cu + 2Ag+ ---- > 2Ag + Cu2+ AG ° = - ve forward reaction is favoured
2Ag + Cu2* ---- > X (no reaction) = + ve backward reaction is favoured '
In ECS, Cu is above Ag hence Cu can reduce Ag+ to metallic The positive value of AG° indicates that the reduction of MgO
Ag. with C does not occur to a significant extent at 1500°C.
8. (a) Electrolysis (b) Roasting a metal sulphide •14. Metals : Na (from fused NaCl), Al (from fused
A12O3+Na3AlF6) *
9. HgS —> HgO---- > Hg.
Since, Hg lies much below H in electrochemical series hence, Non-metals : Cl2 (from fused NaCl), F2 (from fused KHF2 in
HgO is further reduced to Hg. HF)
10. (A): roasting 15. Oxidising agent Reducing agent
(B) : reduction by carbon or more electropositive metal.
(C) : electrolysis (a) O2 (itself reduced toO2-) S2' (in Cu2S) oxidised to SO2
11. (a) 2PbS + 3O2 —> 2PbO + 2SO.2 (b) Cu2O oxidises S2‘ of Cu2S to Cu2S, reduces Cu2O to Cu,
(b) Cu2O + C—2Cu + CO SO2 and itself also reduced to Cu.
(c) Ag+ + e" —Ag (at cathode)
748 ] Essential Inorganic Chemistry

Extraction of Magnesium (+)


H! graphite anode
Occurrence
r ^*CI2(g)

r
Magnesite : MgCO3 »//////////////////////////////a I Z/////////ZZ/Z//ZZ/ZZZZ//ZZZ////Z Sji

Dolomite : MgCO3-CaCO3
: MgSO4-H2O P\ Mg(/)z'^j^p>Mg(/) I steel vessel

I
Kieserite
serves as
Epsomite : MgSO4-7H2O cathode
Sea water : Mg2+ as SOj“ and Cl" \ MgCI2(/)
Carnallite : MgCl2 • KC1 • 6H2O
Heating chamber H
Main sources are dolomite and sea water.
Fig. 17.7 The electrolysis of molten MgC^
Dow Sea-Water Process
Sea water contains about 0.13% Mg2+ ions, and the
extraction of Mg depends on the fact that Mg(OH)2 is Extraction of Lead
very much less soluble than Ca(OH)2. Slaked lime,
Ca(OH)2 is added to sea water, and calcium ions dissolve
Occurrence
and Mg(OH)2 is precipitated. This is filtered off, treated • Galena PbS
with HCI to produce MgCl2, and electrolysed. • Anglesite PbSO4
Ca(OH)2 + MgCl2 ---- > Mg(OH)2 X + CaCl2 • Cerussite PbCO3
Mg(OH)2 + 2HC1 -A> MgCl2 + ZHjO • Lanakite PbO PbSO4
The process has been summarised in Fig. 17.6. and 17.7. • Matlockite PbCl2 PbO
Main source is PbS.

DEEP Focus
The actual electrolyte is a mixture of molten NaCl, Self Reduction Method (Air Reduction Process)
CaCl2 and MgCl2- This mixture has a lower melting point Step I: Concentration by froth flotation process
and higher electrical conductivity than does MgCl2 (Fig 17.5)
alone, but only Mg2* is reduced under the conditions Step II: Roasting in a limited supply of air:
employed in the electrolysis.
PbS + 3O2 ---- > 2PbO + 2SO2
Oxidation: 2C1 ■» Cl2(g) + 2e PbS + 2O2 ---- » PbSO4
Reduction : Mg2* + 2e" 4 Mg(Z)
---1 Step III: Smelting is done along with step II. Whe:
Ca(OH)2 |HCI hci ; 50% of the PbS has been oxidised, the air supply is cut of
Sea water Precipitation Dissolving and the temperature is increased. PbS reduces PbO an:
Mg(OH)2
containing Mg2* + 2OH- —► Mg(OH)2(s) + 2H* + 2CI* -►
Mg2* Mg(OH)2(s)
PbSO4 to Pb :
Mg2* + 2CP + 2H2O
PbS + 2PbO-- > 3Pb + SO2
PbS + PbSO4 -- > 2Pb + 2SO2
Evaporation Electrolysis ■ HC1 Production
I
The metal contains a number of metallic impurities
Mg2*(aq) + 2CI~(aq) MgCI2 Mg2’ ♦ 2Cr -A>. 2CI2(g) + 2H2O(g)-^- !
I
Cu, Ag, Sn, As, Sb, Bi and Zn. These are removed b;
molten Mg(/) ♦ Cl2(g)
MgCI2(s) 4HCI(g)*O2(g) ;
cooling to near the freezing point of Pb, when first Cu an
then Zn containing most of the Ag and Au solidify (Parke
Product method). Preferential oxidation converts As, Sb and Sn t
Mg
As2O3, Sb2O3 and SnO2 which float on the surface of th
Fig. 17.6 The Dow process for the production of Mg. The main molten metal and may be skimmed off.
reaction sequence is traced by solid arrows. Recycling of Cl^g) is
Further refining is done by electrolysis.
shown by broken arrows.
Chapter 17: Metallurgical Extraction | 749

Carbon Reduction Process Cyanide Process


Roasted ore as obtained in step II above is mixed with (Based on Hydrometallurgy)
coke and melted.
Mac Arthur —Forrest Method
PbO + C Pb + CO Step I: Complexation of Ag/Ag^ /AgCl
The powdered ore is treated with NaCN solution and
Extraction of Tin O2 is bubbled through the solution. Ag if any present in
native state is oxidised to Ag* • Ag* is complexed by CN"
Occurrence forming soluble [Ag(CN)2 f
• Cassiterite (tin stone) : SnO2 4Ag + 8CN" + 2^0 + O2 ---- > 4[Ag(CN)2 T + 4OH"
It contains FeWO4 • MnWO4 (Wolframite) as a AgCl + 2CN“ ---- > [Ag(CN)2r +C1-
magnetic impurity. Ag2S + 4CN" ---- > 2[Ag(CN)2r+S2-
Step I: Concentration is done in stages : Last reaction is reversible due to very low solubility of
~ Electromagnetic separation : To remove Ag2S in water. Air oxidises S2" to SO2-, S and S2O^"
magnetic impurities (FeWO4 MnWO4) S2- + 2O2 '•---- > SO42’
- Gravity separation : To remove gangue, ore is
washed with current of water. Step II: Precipitation of Ag
~ Roasting : To remove volatile impurities as Zn is added to soluble [Ag(CN)2 fwhen Ag is
oxides: displaced.
S + O2 ---- > SO2 Zn + 2[Ag(CN)2T ---- > 2AgX+[Zn(CN)4f"
2As2S3 + 9O2 ---- > 2As2O3 + 6SO2 Step III: Refining by electrolysis
Step II: Reduction (Smelting): Concentrated ore Cathode pure Ag
from step I is mixed with coal, lime stone and sand and Anode impure Ag
heated in a reverberatory furnace at 1200°-1300°C Electrolyte AgNO3(aq)
SnO2 + C---- > Sn + 2CO When electric current is passed, impure Ag (anode) is
oxidised to Ag* and is deposited at pure Ag (cathode).
Step III: Refining: Sn obtained above has some
SnO2.It is refined by : At anode : Ag---- > Ag* + e~
At cathode: Ag* +e' ---- > Ag
(i) Poling process (molten tin is stirred with poles of
green wood which reduces SnO2 to Sn). In case of gold, O2(g) oxidises free metal to Au*, which
complexes with CN"
(ii) Liquation (bars of tin are heated on the hearth of
a reverberatory furnace when the readily fusible tin flows 4Au(s)+ 8CN" + O2(g) + 2HoO---- > 4[Au(CN)2T
leaving the impurities like Fe, Cu, W, As etc., (behind). + 4OH"
Step IV: Electro-refining The pure metal is then displaced from the solution by
Cathode : sheet of pure tin an active metal (as Zn).
Anode : blocks of impure tin 2[Au(CN)2 T + Zn---- > 2Au(s) 1 + [Zn(CN)4 f "
Electrolyte : tin sulphate (SnSO4) in dil. HjSC^
and some phenol sulphonic acid
Pure tin from the blocks (anode) is deposited at the
Lead Process
cathode. The process depends upon the fact that when ores of
silver are smelted down with lead, an alloy of lead and
silver is formed; from this alloy lead is removed by
oxidation.
Extraction of Silver smelted with
ores of silver -----;—;--- ; + (Pb- Ag) alloy
Occurrence lead
• Argentite (silver glance) : Ag2S cupellation
• ------------ > silver
• Hom silver (chlorargyrite) : AgCl
• (Pb-Ag) alloy rich in silver : Cupellation process
• Argentiferrous galena : PbSAg2S is used
• Pyragyrite or Ruby silver • 3Ag2SSb2S3
• (Pb-Ag) alloy poor in silver : Parke’s or Pattinson
Main source is argentite.
process is used
750 | Essential Inorganic Chemistry

Mexican Amalgamation Process Felspar or orthoclase on weathering gives rise to


porcelain-clay, China-clay or Kaolin.
Mercury and magistral (burnt pyrites sulphates and
oxides of copper and iron) are added to the powdered
mineral containing H2O and NaCl. The mixture is kept for Purification of Bauxite
several days. Silver is formed in the amalgam form. On Baeyer’s Process
washing, drying and subsequent distillation, silver is Step I : Bauxite contains Fe2O3 and TiO2 as
obtained. impurities. A12O3 is amphoteric and dissolves in NaOH
CuSO4 + 2NaCl---- > CuCl2 + Na2SO4 The other oxides are basic oxides and therefore, insoluble
in NaOH.
CuCl2 + Ag2S---- > Cu2Cl2 + 2AgCl + S
A12O3 + 2OH" + 3H2O -> 2(Al(OH)(r
Cu^C^ + Ag2S---- > CuoS + 2AgCl soluble
2AgCl + 2Hg---- > Hg2Cl2 + 2Ag Insoluble oxides (as Fe2O3) are removed by filtration
Ag + Hg * Ag(Hg) -distil^. AgJ,+HgT Step II : Soluble [A1(OH)4F is diluted and then
acidified when A1(OH)3 is precipitated which on ignition
Desilverisation of Lead: When lead-silver alloy is at 1250°C gives pure alumina :
poor in silver (such as argentiferrous lead or galena),
Al(OH); + H3O+ ---- > A1(OH)3 i + 21^0
desilverisation of lead is affected by Parke’s process. It
depends upon the fact that: 2A1(OH)3 1250°^ A12O3 + 3^0
(i) when zinc is added to a melted alloy of silver and The complex is decomposed into A1(OH)3 by passing
lead, zinc takes away silver from the alloy and itself CO2 through it
forms an alloy with silver and not with lead 2A1(OH)3 +CO23' +H2O
2[Al(OH)4r+CO2
(ii) alloy of zinc and silver melts at a higher
temperature than lead Serpeck's Process
A
(iii) the alloy of silver and zinc is lighter than lead and A12O3 + 3C + N2 2A1N + 3COT
forms two separate layers A1(OH)3+NH3
AIN + 3H2O
(iv) on distillation of silver and zinc alloy, zinc A
separates 2A1(OH)3 A12O3+311,0

Pattinson’s Process Electrolysis of Al203 (Hall-Heroult Process)


It depends upon the fact that the alloy of lead and Al is usually extracted by this process. A12O3 is melted
silver containing less than 1.8 per cent of silver, melts at a * with cryolite Na3[AlF6 ] and electrolysed in a graphite
lower temperature than pure lead. If this type of alloy is lined steel tank, which serves as cathode. The anodes are
melted, and the melted alloy is allowed to cool, lead first also made of graphite. The cell runs continuously and at
separates and repeating the process, one becomes richer in intervals molten Al (m.p. 660°C) is drained from the
silver. Further recovery is done by cupellation process. bottom of the cell and more bauxite is added (Fig. 19.8)
Cryolite improves the electrical conductivity of the cell as
A12O3 is poor conductor. This also serves as an added
Extraction of Aluminium impurity and lowers the m.p. of the mixture to about
950°C.
Occurrence
Bauxite : A12O3-2H2O Al of 99.6 to 99.8% purity is obtained. Probable net
reactions are.
Kaolin : Al2O3-2SiO2 -2H2O
Cryolite : Na3AlF6 Oxidation at anode:
Felspar : K2O • A12O3 ■ 6SiO2 C + 202" ---- > CO2 +4e~]x3
Corundum : A12O3 Reduction at cathode:
Orthoclase : KAlSi3O8 Al3+ + 3e“ 4 Al]x4
Beryl : Be3Al2Si6O18 3C + 4A13+ +6O2' * 4A1 + 3CO2
Main source is bauxite.
Chapter 17: Metallurgical Extraction | 751

Carbon Reduction Method


+ Step I: Concentration by
C(s) C(s) C(s) - gravity separation method (which removes
gangue)
C(s) - magnetic separator (which enriches it by removing
non-magnetic impurities)
liquid cryolite + bauxite
Step II: Roasting is done
liquid aluminium; - to decompose carbonates to Fe2O3 and also to
remove moisture:
4FeCO3 + O2 ---- > 2Fe2O3 + 4CO2
- to oxidise volatile impurities :
Fig. 17.8 Electrolysis cell for aluminium production. The cathode
is a carbon lining in a steel tank. The anodes are also made of 4P + 50% ---- > 2P2O5
carbon. Liquid aluminium is more dense than the electrolyte 4As + 302 ---- > 2As2O3
medium and collects at the bottom of the tank. A crust of frozen S + 02 ---- > SO2
electrolyte forms at the top of the cell.
Step III: Reduction or Smelting
Reaction can also be written as It is done in a blast furnace (Fig. 17.9), the reactions
Anode (oxidation) taking place in a blast furnace (along with working
temperature) are:
2O2- 02 + 4e"]x 3
• Formation of reducing agents, principally CO(g) and
Cathode (reduction)
H2(g):
Al3+ + 3e" ---- > Al]x4
2C + 02 ■> 2CO (1700° C)
Overall 2A12O3 ---- > 4A1 + 3O2 C + COa 4 2CO (> 1000° C)
Oxygen gas reacts with carbon anodes (at elevated C + H2O » CO + H2 (> 600° C)
temperature) to form CO • Reduction of iron oxide :
Bauxite NaOH solution 3CO + Fe2O3 ---- > 2Fe + 3CO2 (900° C)
3H2 + FejjOa ---- > 2Fe + 3H2O (900° C)
• Slag formation to remove impurities from ore :
DIGESTER CaCO3 ---- > CaO + COjj (800-900° C)
The bauxite dissolves
in NaOH solution under CaO + SA ---- > CaSiO3(Z) (1200° C)
pressure 3CaO + P9O5 ---- > Ca3(PO4 )2 (/) (1200° C)
• Impurity formation in iron :
MnO + C---- > Mn + CO (1400° C)
SETTLING TANK PRECIPITATORS
Undissolved solids AI(OH)3 added to encourage SiOs + 2C---- > Si + 2CO (1400° C)
removed from the solution precipitation of the solution P2O5 + 5C---- > 2P + 5CO (1400° C)
• Iron obtained from a blast furnace is called pig iron.
It contains about 95% Fe, 4% C and varying quantities
AI2O3 fed into KLIN of other impurities. Cast iron can be obtained by
Al electrolytic cell AI(OH)3, ppt. roasted
at high temperature pouring pig iron directly into molds of the desired
shape. Cast iron is very hard and brittle and can be
used where it will not be subjected to mechanical or
Extraction of Iron thermal shock.
Occurrence • Wrought iron is prepared by “Pudding process” in
which cast iron is fused in a reverberatory furnace, the
Haematite : Fe2O3 hearth of which is lined with haematite. Reactions
Magnetite : Fe3O4
taking place are:
Limonite : FeO(OH) orFe203-3H20
Siderite FeCO3 3C + Fe2O3 ---- > 2Fe + 3CO
Pyrites (fool's gold) : FeS2 3Mn + Fe2O3 ---- > 2Fe + 3MnO
Pyrrhotite : FeS 3Si + 2Fe2O3 ---- > 4Fe + SSiC^
Ilmenite : FeTiO3
MnO + SiCL ---- > MnSiO3 (slag).
Oxide-ores are main sources of iron.
752 | Essentia/ Inorganic Chemistry

• Pig iron is converted into steel by following changes: molten pig iron. The reactions that occur accomplish the
1. Reduce the carbon content from 3-4% in pig iron to first two objectives are:
0-1.5% in steel. 2C + O.’2 4 2CO
2. Remove, through slag formation, Si, Mn, and P 2FeO + Si + 2Fe + SiO2
(each present in pig iron to the extent of 1% or so), FeO + Mn -> Fe + MnO
together with other minor impurities.
FeO + SiO2 FeO-SiOa
3. Add alloying elements (such as Cr, Ni, Mn, V, Mo, slag
and W) to give the steel its desired end properties.
The most important method of steel making today is MnO + SiO2 4 MnO SiOa
the basic oxygen process (BOP). The process is carried slag
out in a steel vessel with a refractory lining (usually made 4P + 5O2 ■> 2P2O5
of dolomite, CaCO3 • MgCO3). Oxygen gas at about 10 atm
3CaO + P2O5 » Ca3(PO4)2
pressure and a stream of powdered limestone are fed slag
through a water-cooled lance and discharged above the

i . .... x Waste gases


------------------ 200°C----------------
3Fe2O3 + CO----- * 2Fe3O4 + CO2
CaCO3----- * CaO + CO;CO2
V)
■O Fe3O4 + CO----- * 3FeO + CO; CO2
0) 700°C--------------
o 0) C + CO2----- * 2CO
(A ca
0) ’u.
FeO + CO----- • Fe + CO2
TJ tn
(V <D ------------ 1200°C -------------
CT V)
k. ra Iron melts, molten slag forms
ro ct
o ------------ 1500°C --------------
o 2C + O2----- * 2CO
X
o — 2000°C --------------
w Hot air

Slag
Iron
rzzzzzzzzzzzzzzzzzzzzzzzzzzzzz/

Fig. 17.9 Iron ore, coke,'and limestone are added at the top of the furnace, and hotair is introduced through the bottom.
Maximum temperatures are attained near the bottom of the furnace where molten iron and slag are drained off.

The properties of steel depend on its chemical DEEP Focus


composition but also on heat treatment. At high
• Heating the steel to appropriate temperature for a short
temperatures, iron and carbon in steel combine to form time and then cooling it rapidly in order to give it the
iron carbide, Fe3C, called cementite: desired mechanical properties is called tempering. In this
3Fe(s) + C(s) Fe3C(s) way, the ratio of carbon present as graphite and
cementite can be varied within rather wider limits.
The forward reaction is endothermic, so that the
formation of cementite is favoured at high temperatures.
When steel containing cementite is cooled slowly, the Based on % of carbon, steels are named
above equilibrium shifts to the left, and the carbon Name %C
separates as small particles of graphite, which give the
Mild 0.15-0.3
steel a gray colour. If the steel is cooled rapidly,
equilibrium is not attained and the carbon remains large in Medium 0.3-0.6
the form of cementite, Fe3C. Steel containing cementite High-Carbon 0.6-0.8
is light in colour, and is harder and more brittle than Tool 0.5-1.4
that containing graphite.
Chapter 17: Metallurgical Extraction | 753

Table 17.8 shows the composition, properties, and SO2 is the important side product and is used to
uses of various types of steel. manufacture H2SO4.
0
2H2O
2SQ2 +02 ---- > 2SO3 —?-> 2H2SO4
Table 17.8 Types of Steel
Type-mass%
Step III: Reduction of ZnO in a smelter
Composition 1200CC
Plain High strength Stainless ZnO + CO <==± Zn + CO2
C 1.35 0.25 0.03-1.2 Above reaction is reversible and thus high
Mn 1.65 1.65 1.0-10 temperature is required to shift the equilibrium towards
P 0.04 0.04 0.04-0.06 formation of Zn. Thermodynamically :
AG° = AH0 - TaS°
S 0.05 0.05 0.03
To make AG° negative (for spontaneous reaction)
Si 0.06 0.15-0.90 1-3 A
Ni 1.22
TAS > AH and T>~
0.4-1.0 AS°
Cr 0.3-1.3 4.0-27 If we use standard thermodynamic data, T should
Others Cu (0.2-0.6) Cu (0.01-0.08) exceed 1200°C. Zn and CO2 both are in gaseous state at
Uses sheet products, construction, kitchen utensils, this temperature. Reversible nature of the above reaction
tools steel turbines razor blades can be minimised :
(a) by having excess of carbon so that CO? formed is
converted to CO
Extraction of Zinc C + CO2 -----> 2CO
Zinc occurs in the earth’s crust to the extent of (b) by cooling the gaseous products so that equilibrium
132 ppm by weight. It is the twenty-fourth most abundant is not attained immediately.
element. Zinc obtained in this way is of 99% purity.
Step IV: Refinement by Electrolysis
ZnO is dissolved in dil. H2SO4 and electrolysed.
Occurrence
Anode : Impure zinc plate
• Zinc blende (Sphaelerite) : (ZnFe)S
Cathode : Pure zinc plate
• Smithsonite (Calamine) : ZnCO3
Electrolyte: ZnSO4 /H2SO4
• Hemimorphite : Zn4(OH)2(Si2O7) H2O
Main source is zinc blende and is extracted in Zinc from impure zinc plate is oxidised at anode, goes
following steps: into solution and reduced to pure zinc at cathode.
Step I: Concentration by froth-flotation process Anode: • Zn ---- > Zn2++2e"
Step II: Roasting Cathode : Zn2+ + 2e” Zn
ZnS ---- > ZnO + SO2

Target Practice 2
1. Sulphide ore is concentrated by the method 7. Based on following Ellingham diagram, indicate the
2. Name the following process temperature favourable for the reduction of metal oxide to
metal.
SnO2+C 1500 Sn+2CO

3. Ag is precipitated when Zn is added to [Ag(CN)2]". Write AG’ (M, MXO)


the reaction 0
<
4. In the extraction of silver, some lead is also present as
impurity in final step. This lead can be removed by
(i) (ii)
5. Carbon can’t be used to reduce A12O3 to Al while ZnO can
be reduced by carbon. It is due to AG* (C, CO)
6. What is the name of the process involving following reaction:
----- ► Temperature
CaOMgO(s) + Fe/Si — Mg(Z) + Fe + Ca2SiO4
754 | Essential Inorganic Chemistry

8. What is the principle of the following reaction? 10. Match the following properties of Al with the uses to which
Ca(OH)2 + MgCl2 ----- > Mg(OH)2 + CaCl2 the metal or its compounds are put.
(i) Good thermal conductivity (a) Building ships
and aircrafts
(ii) Good electrical conductivity (b) Utensils
9. Alumina (A12O3) and silica (SiO2) get dissolved in a
(iii) Low density and resistance (c) Electric wires
concentrated solution of NaOH under pressure. to corrosion
(a) In what forms are these soluble? Write reactions. (iv) Non-toxicity (d) Adsorbent
(v) Gel nature of A1(OH)3 (e) Food-packaging
11. In the production of iron, intermediate are oxides. Name
(b) A1(OH)3 is precipitated when CO2 is passed or by
the oxides of iron
seeding with some freshly precipitated A1(OH)3.
(a) ...................... (b) ....................
Explain.
12. Basic impurities are removed as slag in the production cf
iron. Write various formulae of slags.
(a) ...................... (b) ....................
(c) Is silica precipitated by treatment in (b).
(0 ......................

Answers
1. Froth flotation process 8. Solubility of Mg(0H)2 is lower than that ofCa(OH)2.
2. Smelting (reduction) 9. (a) A12O3 + 2NaOH + 3H2O ---- > 2NaAl(OH)4
3. 2[Ag(CN)2r + Zn ---- > [Zn(CN)4]2" + 2Ag
SiO2+2NaOH ---- > Na2SiO3+H2O
4. (i) Cupellation process if rich in silver
(ii) Parke’s or Pattinson process is used. (b) 2NaAl(OH)4 + CO2 ---- > 2A1(OH)3 X + Na2CO3 +H,0
5. Al has greater reactivity with oxygen hence, can’t be reduced (c) Silica is more acidic oxide than alumina hence, silicate
by carbon remains in solution.
6. Pidgeon process.
10. (i) (b) (ii) (c) (iii) (a) (iv) (e) (v) (d)
7. The formation of carbon monoxide from carbon can reduce the
metal oxide to metal at a temperature higher than the point of 11. (a) FeO (b) Fe2O3 (c) Fe3O4
intersection of the two lines. 12. (a) CaSiO3 (b) MgSiO3 (c) MnSiO3
(To make AG = - ve, TAS > AH)
Sine AG = AH - TAG

Practice Exercise 2
1. Pb can also be obtained by reduction of roasted ore with 5. Scrap tin plate can be recycled by dissolving the tin in
coke. Outline the process. NaOH solution, followed by electrolysis. Write equations
2. Considering 100% pure, which of the following has for the reactions involved.
maximum percentage of Pb? 6. To prevent the air oxidation of aqueous solutions ofSn2.
2' u
(a) PbS (b) PbSO4
Sn4+, sometimes metallic tin is kept in contact with the
(c) PbCO3 (d) PbO : PbSO4
Sn2+. Suggest how this helps to prevent the oxidation?
3. SnO2 can be precipitated as Sn(OH)4. Can this be used to
7. Gold is also extracted by cyanide process as in case of
concentrate tin stone.
silver. Outline the reactions.
4. (A) Tin stone is amphoteric. Explain.
(B) SnO2 is reduced to metallic Sn on smelting oxide with 8. When 1 mol of pure Ag2CO3 is strongly heated, calculate
anthracite, limestone and sand. weight of residue obtained.
In this, function of sand is: 9. How is Ag extracted from silver coin?
(a) it acts as a flux 10. (Ag + Pb) alloy melt and add zinc> (Ag + Pb + Zn) melt
(b) it removes basic impurities as slag
Cool Layer 11
(c) both (a) and (b) are correct
Layer Y j
(d) none of the above is correct
(C) Which is not the correct process-mineral matching in Select correct statement based on above scheme:
metallurgical extraction? (a) Layer X contains zinc and silver
(a) Leaching : Ag (b) Layer Y contains lead and silver but amount of silver in
(b) Zone-refining : Sn this layer is smaller than in the layer X.
(c) Liquation : Sn (c) X and Y are immiscible layers
(d) van Arkel : Zr (d) All of the above are correct statements
Chapter 17:Metallurgical Extraction | 755

11. In the purification of bauxite ore as preliminary step in the 19. Identify (A) to (F) in the following:
production of Al, [A1(OH)4]‘ can be converted to A1(OH)3 by
passing CO2 through it. Write an equation for the reaction FeSO4 (A) + (B) + (C)
solid gag
that occurs.
O2
12. Addition of NH4C1 can also precipitate A1(OH)3 from (B) (C)
soluble Na[Al(OH)4]. Explain. h2Q]
(A) (D) brown ppt
13. Coke does not cause reduction of A1■22O3.
A
Explain. (D) (A)
AG° (in kJ mol-1) for HC1
(A) ■> deep yellow solution (E)
O3 : -1582
A122^3 K4[Fe(CN)6l
CO : -137.2 (E) 6 > blue ppt. (F)
14. What happens when A1(NO3 )3 is heated? 20. It is possible to identify an “Iron cycle” of the same kind
as a water or nitrogen cycle. Suggest some stages in the
15. In the Baeyer*s process:
cycle.
(a) A12O3 goes into solution as soluble A1(OH)4 while other
basic oxides as TiO2 and Fe2O3 remain insoluble. 21. Al is costlier than Fe but Al is sacrificed to convert Fe2O3
(b) A1,O3 changes to AIN which in turn decomposed by into Al. Why this change is possible? Name this process.
h2o AG0,. (Fe2O3) = - 742.2 kJ mol’1
(c) A12O3 changes to A12(CO3)3 which changes to A1C13
AG°/. (A12O3) = - 1582 kJ mol-1
(d) none of the above is correct
16. In electrolysis of A12O3 by Hall-Heroult process: 22. When iron ore is reduced in a blast furnace, some of the
SiO2 impurity is also reduced by reaction with carbon to
(a) Cryolite Na3[AlF6] lowers the melting point of A12O3
give elemental silicon and carbon monoxide. The silicon is
and increases its electrical conductivity.
(b) Al is obtained at cathode and probably CO2 at anode subsequently reoxidised in basic oxygen process, and the
resulting SiO2 reacts with CaO, yielding slag, which is
(c) both (a) and (b) are correct
then separated from the molten steel. Write balanced
(d) none of the above is correct
equations for the three reactions involving SiO2.
CO
♦ H2O
23. Although iron is only about two-third as abundant as
17. Bauxite +cr AIN A1(OH)3 — A12O3 aluminium in Earth’s crust, mass for mass it costs only
+
N2 about one-quarter as much to produce. Why?
This flow-sheet is for 24. Dolomite (MgCO3-CaCO3) can also be treated to get
(a) Baeyer’s process (b) Serpeck’s process MgCl2 which in turn is electrolysed to get Mg. (Dow
(c) Hall’s process (d) Kroll’s process natural brine process). Give reactions of this process
18. Which mineral has been named incorrectly? [CaCO—
--------------
25. 3 • MgCO3] A. CaO-MgO Fe/S^ Mg+Fe
(a) Bauxite : ALO,-2H,0
Z O z
1150°C
(b) Corundum : A12O3 + Ca2SiO4
(c) Cryolite : 3NaF AlF3 Name this process.
(d) Felspar : Be3Al2Si6O18

Answers
H2O
1. PbS roastini PbO, PbSO4. This mixture is smelted with 3. SnO2 4 Sn(OH)4 SnO2
O2 A
coke,Si02 andCaCO3 This process will not precipitate impurities hence,
SiO^ precipitation of Sn(OH)4 and subsequent heating will
(PbO + PbSO4) PbSiO3
silica enrich SnO2.
PbO + C —> Pb + CO T 4. (A) SnO2 is soluble in acid and alkali both hence, amphoteric.
PbO + CO —> Pb + CO2 ? SnO2 +4HC1---- > SnCl4 +2H2O
basic
CaCO3 ---- > CaO + CO2
SnO2 +2NaOH---- > Na2SnO3 +H2O
CaO + PbSiO3 ---- > CaSiO3 + PbO acidic
slag
(B)(c) (C)(b)
2. Ore % of Pb 5. Sn + 2NaOH---- > Na2SnO2 + H2
la) PbS 86.55 (maximum)
Na2SnO2 e^ectrofysis) Sn (at cathode)
(b) PbSO4 68.21
(c) PbCO3 77.44 6. Sn4* +Sn---- > 2Sn2*
(d) PbOPbSO4 78.62 Sn reduces Sn4+ toSn2*.
756 | Essential Inorganic Chemistry

7. In the cyanide process, crushed rock containing traces of gold = -3(137.2)+ 1582
is treated with 0.1-0.2% solution of NaCN and aerated. O2 = +ve
(air) oxidises free metal to Au * which complexes with CN"
hence, reaction is not spontaneous.
4AU + 8CN'+2H2O + O2 ---- > 4[Au(CN)2r + 4OH-
14. 2A1(NO3)3 A12O3 + 6NO2 + 1.5O2
The pure metal is displaced from solution by an active metal.
15. (a) 16. (c) 17. (b) 18. (d)
2[Au(CN)2r + Zn ---- > [Zn(CN)4]2’ + 2Au 1
19. (A):Fe2O3 (B):SO2 (C):SO3 (D):Fe(OH)3 (E):FeClj
8. Ag2CO3 -£-> Ag2O + CO2? (F): KFeni[Feu(CN)6], Prussian blue.
A 2Ag+|O2T 20. The cycle might be :
Ag2O
Iron oxide---- > Fe---- > iron and steel products
Residue is metallic Ag. ---- > rusting —> iron oxide.
1 mol Ag2CO3 = 2 mol Ag 21. 2 Al + Fe2O3 ---- > 2Fe + A12O3
= 216 g Ag AG° = AG7<A12O3)- AG°z(Fe2O3)
9. Silver coin (Cu + Ag) c°nc HN0». (Cu2* + Ag-)no; = -1582 + 742.2
[NaCl = -ve
AgCl X (white ppt) hence, this reduction of Fe2O3 by Al is possible. This is called
Thermite welding.
2AgCl + Na2CO3 ---- > Ag2CO3+2NaCl
Ag2CO3 2Ag J- + CO2 T +| O2T
DEEP Focus
Or use cyanide process with AgCl. Remember:
* AG° = AG 7 (products) — (reactants)
10. (d) * AG° = + ve, then backward reaction
11. Aqueous solution ofCO2 is acidic. Hence, CO2 when passed = -ve, then forward reaction
into [A1(OH)4]‘ solution makes it acidic hence, A1(OH)3 is
precipitated. 22. SiO2(s)+ 2C(s)---- > Si(s)+ 2C0(g)
CO2 + 2H2O---- > H3O+ + HCO3 Si(s) + O2(g)---- > SiO2(s)
[Al(OH)4r +H3O+ ---- > A1(OH)3 I +2H2O CaO(s)+ SiO2(s)---- > CaSiO3(Z) (slag)
12. NH4C1 solution is acidic due to hydrolysis : 23. Iron production does not require electrolysis.
NH4C1 + H2O NH40H + HC1. 24. MgCO3 CaCO3 catenation Mg0 + Ca0 + 2CO21
A
hence addition of NH4C1 solution precipitates A1(OH)3 when HC1
added to NaAJ(OH)4 ^CO2
Na[Al(OH)4] + HC1---- > NaCl + A1(OH)3 X + H2O MgCl2 + CaCO31 MgCl2 + CaCl2

13. A12O3 + C---- > 2A1 + 3CO [On passing CO2 intoaq MgCl2 andCaCl2 solution, CaCOj is
AG° =3AG °f (CO)-AGor (A12O3) precipitated leaving MgCl2 soluble].
25. It is called Pidgeon process.
r —jr* ----- _ .e—* --------

Total Practice Set


(Read, Plan & Solve1
.)

Problem 1. Following method of extracting Zn is • large quantities of waste material generated in


concentrating low grade ores;
based on thermodynamics:
(A) 2ZnS + 3O2 ---- > 2ZnO + 2SO2 • high energy consumption to maintain high
(B) ZnO + C---- > Zn + CO temperatures necessary for roasting and reduction of
AG°, (standard free energies of formation, in kJ mol ) ores;
of • gaseous emissions, such as SO2 in roasting, that must
be controlled.
ZnS = -205.4; ZnO = - 318.2 In hydro-metallurgy, the materials handled are
SO2 = - 300.4; CO = -137.3 water and aqueous solutions at moderate
Calculate free energy changes of the above reactions temperatures rather than dry materials at high
and comment on the result. temperatures. It involves :
• leaching with acid/water;
Solution For reaction (A)
• purification and/or concentration;
AG° = 2AG} (ZnO) + 2AG°f (SO2) - 2AG°a (ZnS) • precipitation.
= 2[- 318.2 -300.4 + 205.4] Low grade oxide, carbonate and sulpohide ores of copper
are treated with dilute sulphuric acid in the presence of
= - 826.4 kJ (AG’y- (element) = 0) oxygen:
For reaction (B) CuO + HjSC^ ---- > CuSO4 + H>O
AG° = AG°f (CO) - AG°f (ZnO) CuCOjj + H2SO4 ---- > CuSO4 + CO2 + H,0
= -137.3 + 328.2 Cu^S + 2H,SO4 + 2O2 ---- > 2CuSO4 +SO2 +2H2O
When the silver ore, AgCl, is treated with an aqueous
= + 180.9 kJ solution of sodium cyanide, AgCl dissolves in it due to
For reaction (A), AG° is negative hence, reaction is the formation of Na[Ag(CN)2 ]:
spontaneous; reaction (B) is non-spontaneous AG° AgCl+2NaCN ---- > Na[Ag(CN)2 ] + NaCl
being positive for it.
Sulphide ore, Ag2S, dissolves only slowly as the reaction
is reversible :
DEEP Focus
Again remember: Ag2S + 4NaCN 2Na[Ag(CN)2 ] + Na2S
*AG°=aG° (products) - AG(reactants)
* AG° =+ve indicates non-spontaneous reaction (in forward If air is passed through this solution, sodium sulphide is
direction) oxidised to sodium sulphate and the forward reaction
* AG° = - ve indicates spontaneous reaction (in forward goes to completion dissolving all the sulphide ore. In the
direction) presence of air, native silver is also leached out in the
form of Na[Ag(CN)2 ]
Problem 2. Explain the difference between hydro­ 4Ag + 8NaCN + 2ILO + O2---- > 4Na[Ag(CN)2 ] + 4NaOH
metallurgy and pyro-metallurgy. The leached out metals are recovered from the solution
either by precipitation on treatment with a more
Solution The metallurgical procedure based on the
electropositive metal or by electrolysis. For example,
roasting of an ore, followed by its reduction (usually with copper can be recovered from its solution by adding
carbon), is known as pyro-metallurgy. A key
metals like Fe, Al, etc. Silver is obtained from its
consideration in pyro-metallurgy is the temperature at
solution by treatment with Zn or Al:
which reaction becomes spontaneous. As the term “pyro”
suggests, often this temperature is quite high. Some of the CuSO4 +Fe ---- > Cu+FeSO4
characteristics of this process are : 2Na[Ag(CN)2] + Zn ---- > 2Ag+Na2[Zn(CN)4]
758 | Essential Inorganic Chemistry

Problem 3. In some foam-type fire extinguishers, the Solution When Pb3O4 is treated with HNO3, PbCNO^
reactants are A12(SO4)3 and NaHCO3 solutions. When the is formed and PbO2 is precipitated :
extinguisher is activated, these reactants are allowed to
mix, producing A1(OH)3 andCO2.The A1(OH)3—CO2 foam Pb3O4 + 2HNO3 ---- > 2Pbn(NO3)2 + PbIVO21
extinguishes the fire. Write the net ionic equation to + 2HjO
represent this reaction.
This shows that red lead has PbO and PbO2 in 2 :1
Solution A12(SO4)3 solution is acidic due to hydrolysis molar ratio and thus it is 2PbO • PbO2 and is thus dilead
(II) lead (IV) oxide. Only PbO reacts with HN03 forming
Pb(NO3)2 while PbO2 remains unreacted and is
Al3+ +4H2O---- > [Al(OH)4r +4H+ precipitated.
[A1(OH)4]" reacts with HCO3 forming A1(OH)3—CO2
foam: Problem 6. The following are two reaction schemes
involving Mg.
[A1(OH)4 T + HCO3 ---- > A1(OH)3 + HaO + CO2
HC1?
Scheme I Mg airA > colourless solid A
Problem 4. Use the relationship AG° = - nFE°cell to
NagCOs -> white pptC
estimate the minimum voltage required to electrolyse colourless solution B
A12O3 in the Hall-Heroult process.
AG°f (A12O3) = - 1520 kJ mol"1
white ppt F <Ca(0Hk colourless gas E J1
D
AG°y (CO2) =-394 kJ mol"1
Show that the oxidation of the graphite anode to CO2 Scheme II Mg > colourless solution G
permits the electrolysis to occur at a lower voltage than HNO3) colourless solution
if the electrolysis reactions were white pptH
j evaporate |
A12O3 ---- > 2A1 + 3O2 D + O2 + gas J A
Solution Net reaction in Hall-Heroult process is: Identify A to J
3C + 2A12O3 ---- > 4A1 + 3CO2
Solution A : MgO; B : MgCl2; C: MgCO3;
Or 4A13+ + 12e" ---- > 4A1,
D: MgO; E:
F: CO2; F: CaCO3;
Number of electrons (n) = 12, AG° (element) = 0 G:MgSO4; ZZ:Mg(OH)2;
AG°=3AG°Z (CO2)-2AG°r(Al2O3) Z: Mg(NO3)2 J: NO2
= -3x 394-2(- 1520)
Problem 7. Explain following terms
= 1858 kJ (a) Electrometallurgy
AG° = -nFE ocell (b) Zone refining
170 — AG° _ 1858 x 1000
(c) van Arkel de-Boer process
& cell “
nF ’ 12 x 96500 (d) Mond’s process
= 1.60 V Solution
For the reaction A12O3 ---- > 2Al + 3O'22 (a) Electrometallurgy
The two metallurgical processes, namely
AG° = 1520 kJ pyrometallurgy and hydrometallurgy can be used in the
2A13+ + 6e - —> 2A1, n = 6 extraction of a fairly large number of metals. These
_ AG _ 1520 x 1000 methods, however, cannot be used in cases:
-E°
ceU " nF 6 x 96500 Where the metal is highly reactive, e.g., Na, Li, etc
There are no any chemical-reducing agent strong enough
= 2.62 V to prepare these metals.
Thus Hall-Heroult process takes place at lower Where the oxide gets reduced only at very high
voltage. temperature at which formation of carbides also takes
place, e.g., Al, Mg, etc.
Problem 5. The names dilead (II) lead (IV) oxide and In these cases, metals can be extracted by electrolysis of
plumbous orthoplumbate have been used to describe red their salts in molten state. Thus, sodium and magnesium
lead Pb3O4. Show that these names are true. are prepared by electrolysis of fused chlorides, where the
Chapter 17: Metallurgical Extraction | 759

aetals are liberated at the cathode and chlorine gas is ... . „_____ _ molten zone
recrrystallised—1
volved at the anode. Following reactions take place metal

V Lv\ v
luring electrolysis :
Anode: 2C1" -----> Cl2(g) + 2e"
Cathode: 2Na+ + 2e" > 2Na(Z) heater' metal rod
Mg2++2e" > Mg(Z) Zone refining

Anode Cathode (c) van Arkel de-Boer Process

■n 1'-; This method is based on the thermal decomposition of a


volatile metal compound like an iodide. In this method,
first a metal iodide is formed by direct reaction of iodine
and the metal to be purified at a temperature of475-675 K

+
II in an evacuated vessel. The vapours of metal iodide, thus
formed are heated strongly on a tungsten or tantalum
IB filament at 1300-1800 K. The metal iodide decomposes to
Na
£ yield the pure metal, as in the case of zirconium.
cr 473-673 K „ , , x 1300-1800 K
2CGEClif2!?nHNal*<iSNa: Zi<s)+ 21,(5) * Zrl^g) —---------- —------ >
Tungsten filament
Electrolysis of molten sodium chloride
> Zr(s) + 212(g)

In theory, aluminium metal could be made the same Titanium is also purified by this method. The impure
*ay. But, aluminium trichloride is covalent and it does not metal is heated with iodine and Til^ thus formed is
conduct electricity. Aluminium is obtained by electrolytic decomposed by heating at 1700K over tungsten filament.
reduction of alumina in fused cryolite at 1100-1300 K 1700 K
using carbon anode and iron cathode. Electrolysis yields Ti(s) + 2l2(g) * Til^g)
Tungsten filament
aluminium at cathode and O2 at anode which reacts with
carbon to produce CO2. The reactions at electrodes are : Ti(s) +212(g)
Anode: 2O2' -----> O2(g) + 4e~ The regenerated iodine is used over and over again.
This process is very expensive and is employed for the
C(s) + O2(g) ---- > CO2(g)
preparation of limited amounts of very pure metals for
Cathode : Al3++ 3e~ ---- > Al(s) special uses.
I
(b) Zone Refining (d) Mond Process
This method is used to obtain metals of very high Some metals are purified by obtaining their volatile
purity. The basic principle involved in this process is carbonyl compounds which on heating strongly decompose
similar to fractional crystallisation. A small heater is used to yield pure metal. Purification of nickel is done by this
10 heat a bar of the impure metal. The heater melts a small method. Impure nickel is reacted with carbon monoxide at
band of metal as it is slowly moved along the rod. As small 325 K to give volatile nickel carbonyl leaving solid
bands of metal are thus melted sequentially, the pure impurities behind. Pure nickel is obtained by heating
metal crystallises out of the melt, while impurities pass nickel carbonyl at 450-475 K.
into the adjacent molten zone. The impurities thus collect 450-475K .... x
at the end of the bar. This end can be cut off and removed. Ni(s) + 4CO(g) Ni(CO)4(g) ------------ > Ni(s)
High grade germanium and the silicon are obtained by
purifying them by zone refining. + 4CO(g)
Master Exercises
z~sa

Exercise 1
(Stage 1: Learning)
Short Answer Type Questions (a) In what form is silver present in the solutions
after developing and fixing photographic film?
1. Is it possible to reduce a sulphide ore directly with
(b) How is silver recovered from such solution?
carbon.
(c) What is undesirable pollutant?
2. Consider following reactions
(a) 2ZnS + 3O2 •» 2ZnO + 2SO2 7. Following reaction is reversible
ZnO + C ---- > Zn + CO Ag2S + 4NaCN 2Na[Ag(CN)2 ] + Na2S
(b) 2ZnS + C ---- > 2Zn + CS2 Suggest a method to make it spontaneous.
(i) Which reaction is more spontaneous? 8. ZnS is always contaminated with PbS. Outline a
(ii) Which reaction has more negative value of AG ? method that can enrich PbS by froth flotation process.
(iii) Which reaction has more positive value of AG ?
9. Consider following facts
3. Which reactions are favourable? I : Zinc and lead are immiscible in molten state;
(a) MgO + Si ---- > zinc is lighter than lead.
(b) SiO2 +Mg ---- > II: Solubility of silver in molten zinc is higher than
4. Specify magnetic impurity present in tin stone.......... that in molten lead.
Ill: Zinc is volatile.
5. Pure tin is obtained by electro-refining. Specify the
(a) Outline method of separation of silver from lead
indicated (by arrow) contents.
based on above facts.
(b) Name the method
10. (a) Write full from of BOP process of making steel.
(b) Write various forms of iron.
(c) Which is the purest form?
c 11. Consider following equilibrium
3Fe(s) + C(s) Fe3C(s), AH’>0
What is the effect of temperature on this
Ii equilibrium?
*
B (anode)
C (cathode) Analytical Questions
1. (a) Calculate the free energy changes for these
reactions
A (electrolyte) I: 2CuS + C 4 2Cu + CS2
II : 2CaS + C -> 2Ca + CS2
6. The recovery of silver from the solutions used in III: 2CuS + 3O2 ■» 2CuO + 2SO2
developing and fixing photographic film and prints
Given AG°y (in kJ mol-1) for
presents just such a problem. Silver is an expensive
metal, and the recovery is profitable. Moreover, if not CuS = -49, CS2 =63.6
recovered, the silver ions would constitute and CaS = - 1320; CuO = -127.2
undesirable pollutant in water supplies. SO2 = - 300.4
Chapter 17: Metallurgical Extraction | 761

(b) Which of them are spontaneous under standard Explain, why roasting of ZnS gives ZnO but roasting
conditions? of HgS gives elemental Hg. What product would you
(c) What do you notice about the value of A G°,(CS2) expect from roasting CdS?
compared to A G°r (SO2)? 8. When an inert atmosphere is needed for a
(d) Why are the reactions not carried out at standard metallurgical process, nitrogen is frequently used.
conditions? However, in the reduction of TiCl4 by magnesium,
2. A student suggested that calcium should be made if helium is used. Explain, why nitrogen is not suitable
calcium oxide is reacted with aluminium powder. for this process?
Was the student correct? 9. The following equations represent reactions used in
A G°f (CaO) = - 604.2 kJ mol"1 some important metallurgical processes:
A G°/.(A12O3) = - 1582 kJ mol"1 (a) Fe3O4 +CO(g)---- > Fe(Z)+CO2(g)
3. Although Au is soluble in aqua regia, Ag is not. What (b) MgCO3(s)+SiO2(s)---- > MgSiO3(Z) + CO2(g)
is the likely reason for this difference ? (c) Au(s)+CN"(aq)+H2O(Z)+O2(g)---- »
4. FeCr2O4 (chromite) can be used to extract Cr. Outline [Au(CN)2F(aq)+OH-(aq)
scheme of extraction.
5. Lead metal is purified by electrolysis in a manner Balance the equations. Which one (s) represent
similar to that for copper; the electrolyte is lead (II) reduction to a free metal?
hexafluorosilicate PbSiF6. Describe the process. 10. Assuming complete recovery of metal, which of the
6. (a) Pure iron is prepared for special purposes by following ores would yield the greater quantity of
precipitating iron (III) oxide and reducing the dry copper on a mass basis?
oxide with gas. Write the balanced equation. (a) an ore containing 3.30% (by mass) azurite
(b) HC1 can’t used to precipitate A1(OH)3 from Cu(OH)2 • 2CuCO3 or
soluble Na[Al(OH)4] but addition of NH4C1 can (b) an ore containing 4.95% (by mass) chalcopyrite
cause precipitation. Explain by reactions. CuFeS2.
(c) AgCl (Horn silver) is converted into Ag by 11. When 1.164 g of a certain metal sulphide was roasted
pyrometallurgical method. Describe reactions. in air, 0.972 g of the metal oxide was formed. If the
7. Zinc, cadmium and mercury are found in nature as oxidation number of the metal is +2, calculate the
sulphides. Values of standard reduction potentials molar mass of the metal.
for the half-reactions are : 12. How much aluminium can be produced by a current
M2+(aq) + 2e~ ---- > M(s) of 94.0. A flowing through a Hall-Heroult cell for
l.OOh?
M E°
13. Sea-water contains 1272 g of Mg2* per metric ton
Zn -0.76 V
(1 mega gram). How much of slaked lime must be
Cd -0.40 V added to 1.0 metric ton of sea water to precipitate all
Hg + 0.79 V of the Mg2* ion? (Mg « 24, Ca = 40)

Exercise 2
(Stage 2: High Skill Problem Solving)
Only One Option Correct 2. Main source of lead is PbS. It is converted to Pb by:
1. Si of high purity to be used in semiconductor can be I. PbS -^4 PbO + SQj
prepared by following methods: I C Pb + CO2
I. SiO2 + 2C----> Si + 2CO
II. Si + 2C12 --- > SiCl4 Il.PbS PbQ + PbS
A 1
SiCl4 + 2Mg---- > Si + 2MgCl2 Pb + SQj
Better method is Self-reduction process is
(a) I , (b) II (a) I (b) II
(c) both (a) and (b) (d) None of these (c) both (a) and (b) (d) None of these
762 | Essential Inorganic Chemistry

3. Match the compounds (A) with their applications/ 11. When FeCr2O4 (chromite) is reduced with C in an
colour in (B) electric-arc furnace
A B (a) Cr and Fe2O3 are formed
I. Pb3O4 W. white lead (b) Fe and Cr2O3 are formed
II. (PbCO3)2 • Pb(OH)2 X. rust proofing sheets (c) Fe and Cr (ferrochrome) are formed
III. Ca2PbO4 Y. road signs and markings (d) FeCrO4 is formed
IV. PbCrO4 Z. red lead • FeCr2O4 (chromite) is a good source of Cr and its
Choose the correct alternate
I II III I compounds like Na2CrO4, Na2Cr2O7. Questions 12
(a) Y Z X W to 14 are based on following reactions:
(b) X W Y z I. FeCr2O4 + NaOH + air---- > (A) + Fe2O3
(0 Z W X Y II. (A) + (B)---- > Na2Ci2O7
(d) X Y W Z
III. Na2CE>07 +X C%O3
4. P4O10 is converted to white phosphorus by
(a) Al (b) C IV. C^ +Y Cr
(c) Na (d) all are correct
12. Compounds (A) and (B) are
5. Phosphine, PH3, is prepared by
(a) Na2CrO4, H2SO4
(a) Ca3P2 + H2O---- > (b) P4 + NaOH---- >
(b) Na2C^O7, HC1
(c) both are correct (d) None is correct
(c) Na2CrO5, H2SO4
6. Formation of Ni(CO)4 and subsequent its
(d) Na4[Fe(OH)6], ^SO,
decomposition into Ni and CO (recycled) makes the
basis of Mond's process 13. (X) and (Y) are
Ni + 4CO Ni(CO)4
r2 Ni + 4CO
(a) C and Al (b) Al and C
(c) C in both (d) Al in both
7] and T2 are 14. Na2CrO4 and Fe2O3 are separated by dissolving in
(a) 100° C, 50° C (b) 50° C, 100° C
(c) 50° C, 230° C (d) 230° C, 50° C (a) cone. HaSQj (b) NH3
7. Oxides of the various metals are converted into metal (c) H2O (d) dil. HC1
by heating but not in 15. Match the reactions taking place in blast furnace
(a) Ag2O (b) CuO with temperature-range of operations:
(c) HgO (d) all are correct
Reactions Temperature range
8. Of the following reduction processes
I. Fe2O3 + C---- > Fe (A) : Fe2O3 + CO —> CO2 + Fe (I) 1200°C
II. ZnO + C---- > Zn (B) : C + CO2 ---- > CO (II) 900’C
III. Ca3(PO4)2 +C---- > P (C) : C + H2O---- > CO + H2 (III) 600° C
IV. PbO + C---- > Pb
(D) : CaO + SiO2 ---- » CaSiO3 (IV) 1000°C
Correct processes are
(a) all of these (b) all but III
Choose the correct alternate
(c) all but IV (d) II and IV
A B C D
9. Following reaction is not involved in Thermite (a) IV III II I
process (b) II IV III I
(a) 3Mn3O4 + 8Al---- > 9Mn + 4A12O:3 (0 I II IV III
(b) Ci^O3 + 2A1---- > A12O3 + 2Cr (d) I II III IV
(c) 2Fe + A12O3 ---- > 2A1 + Fe2O3
(d) B2O3 + 2A1---- > 2B + A12O3 16. Match column (A) (process) with column (B)
10. Consider the following steps (electrolyte).
Qug roast in air^ roast without air■* (B) A (process) B (electrolyte)

Which is not the correct statement? (I) Downs cell (W) Fused MgCI2
(a) It is self-reduction (II) Dow sea water process (X) Fused (AI2O3 + Na3AIF6)
(b) It involves disproportionation (III) Hall-Heroult (Y) Fused KHF2
Ct^S---- > Cu + CuS
(IV) Moissan (Z) Fused (40% NaCI + 60%
(c) (A) is a mixture of CuO and CuS and (B) is a CaCI2)
mixture of Cu and SO2
(d) All are incorrect statements
Chapter 17. Metallurgical Extraction | 763

Choose the correct alternate 25. In the metallurgy of iron, when CaCO3 is added to
I II III IV I II III IV blast furnace, calcium ion appears as
(a) Z W X Y (b)X Y Z w (a) CaO (b) metallic Ca
(c)W Z X Y (d)X Z W Y (c) gangue (d) slag
17. In the Pidgeon process, Mg is produced by 26. Iron obtained from blast furnace is
(a) electrolysis of fused MgCl2 (a) wrought iron (b) cast iron
(b) reducing calcined dolomite with ferrosilicon at (c) pig iron (d) steel
high temperature under pressure 27. Four metals and their methods of refinement are
(c) both (a) and (b) given:
(d) none of the above (i) Ni, Cu, Zr, Ga
18. Boron can be obtained by various methods but not by (ii) Electrolysis, van Arkel process, Zone refining,
(a) thermal decomposition of B.2H6 Mond's process
(b) pyrolysis of BI3 (van Arkel) Choose the right method for each:
(c) reducing BC13 with (a) Ni: Electrolysis, Cu : van Arkel process, Zr: Zone
(d) electrolysis of fused BC13 refining, Ga : Mond's process
(b) Ni : Mond's process, Cu : Electrolysis, Zr : van
19. Following method is not used for extraction of Al Arkel process, Ga : Zone refining
(a) van Arkel (b) Serpeck (c) Ni : Mond's process, Cu : van Arkel process,
(c) Bayer (d) Hall-Heroult Zr : Zone refining, Ga : Electrolysis
20. B4C (boron carbide) is used except (d) Ni : Electrolysis, Cu : Zone refining, Zr : van
(a) to extract boron Arkel process, Ga : Mond's process
(b) as an abrasive for polishing 28. On ignitingFegOg at 1400° C, the product obtained is
(c) for making bullet-proof clothing (a) Fe2O3 melt (b) FeO
(d) for making diborane (c) Fe3O4 (d) metallic iron
21. In the extraction of nickel by Mond’s process, the 29. Consider the following metallurgical processes
metal is obtained by I. Heating impure metal with CO and distilling the
(a) electrochemical reduction resulting volatile carbonyl (b.p. 43° C) and finally
(b) thermal decomposition decomposing at 150° to 200° C to get the pure
(c) chemical reduction by aluminium metal
(d) reduction by carbon II. Heating the sulphide ore in air until a part is
22. Which one of the following statements is not correct? converted to oxide and then further heating in
(a) Nickel forms Ni(CO)4 the absence of air to let the oxide react with
(b) All the transition metals form monometallic unchanged sulphide
carbonyls 3. Electrolysing the molten electrolyte containing
(c) Ni(CO)4 is volatile smd decomposes into Ni Emd approximately equal amounts of the metal
CO chloride and CaCl2 to obtain the metal
(d) Transition metals form complexes The processes used for obtaining sodium, nickel and
copper are respectively
23. Match List I with List II and select the correct answer
(a) I, II and III (b) II, III and I
using the codes given below the lists:
(c) III, I and II (d) II, I and III
List I List II
30. Extraction of aluminium from aluminium oxide
A. Carnallite 1. Zinc (A12O3) is best done by
B. Calamine 2. Titanium (a) electrolytic reduction of A12O3
C. Ilmentite 3. Magnesium (b) reduction of A12O3 with carbon
D. Chalocopyrite 4. Copper (c) reduction of A12O3 with sodium
(d) reduction of A12O3 with CO
A B C D
(a) 1 31. Consider the following statements:
3 2 4
Roasting is carried out to
(b) 1 3 4 2 I. convert sulphide to oxide and sulphate
(c) 3 1 4 2 H. remove water of hydration
(d) 3 1 2 4 III. melt the ore
24. Which of the following is not an ore? 4. remove arsenic and sulphur impurities
(a) Malachite (b) Calamine Of these statements
(c) Stellite (d) Cerussite (a) I, II and III Eire correct
764 | Essential Inorganic Chemistry

(b) I and IV are correct 42. Match the method of concentration of the ore in
(c) I, II and IV are correct column I with the ore in column II and select the
(d) II, III and IV are correct correct alternate:
32. The element which could be extracted by electrolytic Column I Column II
reduction of its oxide dissolved in a high temperature
X: Magnetic separation A: Ag2S
melt is
(a) sodium (b) magnesium Y: Froth flotation B : FeCr2O4
(c) fluorine (d) aluminium Z: Gravity separation C: Al2 (81'03)3
33. Chemical leaching is useful in the concentration of X Y Z
(a) copper pyrites (b) bauxite
(c) galena (d) cassiterite
(a) A B c
(b) B A c
34. Carbon cannot be used in the reduction of A12O3 (0 C A B
because (d) B C A
(a) it is an expensive proposition 43. Which of the following does not contain Mg?
(b) the enthalpy of formation of C02 is more than (a) Magnetite (b) Magnesite
that of A12O3 (c) Asbestos (d) Carnallite
(c) pure carbon is not easily available
(d) the enthalpy of formation of A12O3 is too high 44. Calcination is the process of heating the ore
(a) in inert gas
35. In which of the following isolations no reducing agent (b) in the presence of air
is required? (c) in the absence of air
(a) Iron from haematite (d) in the presence of CaO and MgO
(b) Aluminium from bauxite
(c) Mercury from cinnabar 45. Ag2S + NaCN + Zn---- > Ag
(d) Zinc from zinc blende This method of extraction of Ag by complex formation
36. Which one of the following is not a method of and then its displacement is called
concentration of metals? (a) Parke's method
(a) Gravity separation (b) Mac Arthur-Forrest method
(b) Froth flotation process (c) Serpeck method
(c) Electromagnetic separation (d) Hall’s method
(d) Smelting
46. The slag obtained during the extraction of copper
37. Blister Cu is about from copper pyrites is composed of
(a) 60% Cu (b) 90% Cu (a) CujS (b) CuSiO3
(c) 98% Cu (d) 100% Cu (c) FeSiO3 (d) SiO2
38. There are following extraction process of silver but 47. In zone-refining method the molten zone
not (a) consists of impurities only
(a) as a side product in electrolytic refining of copper (b) contains more impurity than the original metal
(b) Parke's process in which Zn is used to extract (c) contains the purified metal only
silver by solvent extraction from molten lead (d) moves to either side
(c) by reaction of silver sulphide with KCN and then
reaction of soluble complex with Zn. 48. Reduction of a metal oxide by excess carbon at high
(d) by heating Na[Ag(CN)2 ] temperature is a method for the commercial
preparation of some metals. This method can be
39. Ag2S + NaCN---- > (A) successfully applied in the case of
(A) + Zn---- > (B) (a) BeO and A12O3 (b) ZnO and Fe2O3
(B) is a metal. Hence, (A) and (B) are (c) CaOandCr^Og (d) BaOandU3O8
(a) Na2[Zn(CN)4], Zn (b) Na[Ag(CN)2 ], Ag 49. Match List I with List II and select the correct answer
(c) Na2[Ag(CN)4],Ag (d) Na3[Ag(CN)J,Ag using the codes gives below the lists :
40. Refining of silver is done by
(a) liquation (b) Poling List I List II
(c) cupellation (d) van Arkel method A. van Arkel method 1. Manufacture of caustic soda
B. Solvay process 2. Purification of titanium
41. Bessemerisation is carried out for
I: Fe, II: Cu, HI: Al, IV : silver C. Cupellation 3. Manufacture of Na2CO3
(a) I, II (b) II, HI D. Poling 4. Purification of copper
(c) III, IV (d) I, III 5. Refining of silver
Chapter 17: Metallurgical Extraction | 765

Codes (c) 2CU2S + 3O2 ---- > 2Cu2O + 2SO2


A B C D (d) 2FeS+3O2 ---- > 2FeO + 2SO2
(a) 2 1 3 4 56. Sodium is made by the electrolysis of molten mixture
(b) 4 3 2 5 of about 40% NaCl and 60% CaCl2 because
(c) 2 3 5 4 (a) CaCl2 helps in conduction of electricity
(d) 5 1 3 4 (b) Ca2+ can reduce NaCl to Na
50. Formation of metallic copper from the sulphide ore in (c) Ca2+ can displace Na from NaCl
the normal thermo-metallurgical process essentially (d) this mixture has a lower melting point than NaCl
involves which one of the following reaction? 57. Calcium is obtained by the
(a) CuS+|o2 ---- > CuO + SO2; (a) roasting of limestone
(b) electrolysis of solution of calcium chloride in HjO
(b) CuS+|o2 - -> CuO + SO2; (c) electrolysis of molten anhydrous calcium chloride
2 z (d) reduction of calcium chloride with carbon
2CuO + CuS —» 3Cu + SO2 58. Which one of the following elements constitutes a
(c) CuS + 2O2 — ► CuSO4; major impurity in pig iron?
W) CuS +1 O, - * CuO + SO2; (a) Silicon (b) Oxygen
2 2 (c) Sulphur (d) Graphite
CuSO4 + CuS * 2Cu + 2SO2 59. Purification of aluminium by electrolytic refining, is
known as
CuO + CO---- ) Cu + CO2
(a) Hall’s process (b) Baeyer’s process
51. Annealing of steel is the process of heating steel (c) Hoope’s process (d) Serpeck’s process
(a) to a bright red hot and then cooling it slowly 60. Cassiterite is an ore of
(b) to a bright red hot and then cooling it suddenly (a) Mn (b) Ni
(c) to a temperature much below redness and cooling (c) Sb (d) Sn
it slowly
(d) none of the above 61. The process used for purification A12O3 • 2^0 is
(a) magnetic separation (b) froth flotation
52. Out of CujS, HgS, Ag2S and ZnS, roasting will convert
(c) leaching (d) liquation
the minerals into metal in case of
(a) CujS, ZnS (b) HgS, ZnS 62. Froth flotation is a process of
(c) Cu>S, Ag2S (d) HgS (a) oxidation (b) reduction
(c) refining (d) concentration
53. The method of zone refining of metals is based on the
principle of 63. Malachite is an ore of
(a) greater noble character of the solid meted than (a) silver (b) mercury
(c) magnesium (d) copper
that of the impurity
(b) greater solubility of the impurity in the molten 64. Flux is used to remove
state than in the solid (a) basic impurities
(c) greate mobility of the pure metal than that of (b) acidic impurities
impurity (c) all types of impurities
(d) higher melting point of the impurity than that of (d) acidic and basic both impurities
the pure metal 65. Mac Arthur process is used for the extraction of
54. Splitting of silver is (a) Al (b) Ag
(a) reduction of ammoniacal silver nitrate solution (c) Cu (d) Zn
by tartarate 66. The chemical process in the production of steel from
(b) extraction of silver from its ore Ag2S, by haematite ore involves
hydrometallurgy (a) reduction
(c) making of silver amalgam during filling of teeth (b) oxidation
(d) cooling of molten silver with the evolution of (c) reduction followed by oxidation
oxygen causing violent spurting (d) oxidation followed by reduction
55. In the extraction of Cu, the reaction takes place in 67. Electrolytic reduction of alumina to aluminium by
bessemer converter is Hall-Heroult process is carried out
(a) 2Cu2O + Ci^S ---- > 6Cu + SO2 (a) in the presence of NaCl
(b) 2CuFeS2 + O2 ---- > Ci^S + FeS + SO2 (b) in the presence of fluorite
766 | Essential Inorganic Chemistry

(c) in the presence of cryolite which forms a melt 75. Main process for extracting nickel in the pure forn
with lower melting temperature uses which one of the following?
(d) in the presence of cryolite which forms a melt (a) Vapour phase refining
with higher melting temperature (b) Zone refining
68. Which of the following ore is concentrated by both, (c) Electrolysis
magnetic and gravimetric separation? (d) Solvent extraction
(a) Dolomite (b) Tinstone 76. Which represents correct matching of metals with
(c) Galena (d) Bauxite their minerals?
69. Which of the following processes is used in extractive Mg Na Cu Al
metallurgy of magnesium? (a) chloride chloride sulphide silicate
(a) Fused salt electrolysis (b) carbonate chloride sulphide oxide
(b) Self reduction (c) carbonate carbonate oxide phosphate
(c) Aqueous solution electrolysis (d) oxide chloride sulphide oxide
(d) Thermite reduction 77. Which of the following metals are found in native
70. In the process of extraction of gold, state?
O2 Ag, Pt, C, Si, N, O, Mg, Na, Pb.
Roasted gold ore + CN“ + H2O [X] +OH-
(a) Ag, Pt,C, N, 0 (b) Ag, Pt, Mg
[X] + Zn ---- > [Y] + Au (c) Ag, Pt, Pb, Mg (d) Ag, Pt
Identify the complexes [X] and [V].
78. Most abundant metal in earth crust is
(a) X = [Au(CN)2],Y = [Zn(CN)4f- (a) Al (b) 0
(b) X = [Au(CN)4 ]3", Y = [Zn(CN)4 (c) Fe (d) Si
(c) X = (Au(CN)2r, Y = [Zn(CN)6]4- 79. Followed by oxygen which element is the most
(d) X = [Au(CN)4]“, Y = [Zn(CN)4f- abundant in human body
71. The methods chiefly used for the extraction of lead (a) C (b) H
and tin from their ores are respectively (c) Ca (d) P
(a) self reduction and carbon reduction 80. Select correct statement.
(b) self reduction and electrolytic reduction (a) Based on reactivity series, occurrence of certain
(c) carbon reduction and self reduction elements takes place in native state
(d) cyanide process and carbon reduction (b) Due to basic nature of oxides of alkaline earth
72. Extraction of zinc from zinc blende is achieved by elements, they combine with atmospheric acidic
(a) electrolytic reduction oxides giving salts
(b) roasting followed by reduction with carbon (c) Both (a) and (b) are correct
(c) roasting followed by reduction with another (d) None of the above is correct
metal 81. Which is not the correct statement?
(d) roasting followed by self-reduction (a) Cassiterite, chromite and pitchblende are
73. Which one of the following statements is correct? In concentrated by hydraulic washing (Tabling).
the metallurgy of sodium by electrolysis, excess of (b) Pure A12O3 is obtained from the bauxite ore by
calcium chloride is mixed with sodium chloride to leaching in the Baeyer’s process.
(a) make the latter a good conductor (c) Sulphide ore is concentrated by calcination
(b) make the latter soft method.
(c) generate more energy for the electrolytic cell (d) Roasting can convert sulphide into oxide or
(d) assist liquefication of the latter at a much lower sulphate and part of sulphide may also act as a
temperature reducing agent.
74. Consider the following statements 82. NaCN is sometimes added in the froth flotation
I: Pig iron is malleable and ductile. process as a depressant when ZnS and PbS minerals
II: Pig iron can be melted to produce cast iron of are expected because
desired shapes. (a) Pb(CN)2 is precipitated while no effect as ZnS
Ill: Wrought iron is obtained by heating pig iron (b) ZnS forms soluble complex Na2[Zn(CN)4] while
with iron oxide in a furnace. PbS forms froth
Which of the statement(s) given above is/are (c) PbS forms soluble complex Na2[Pb(CN)4] while
correct? ZnS forms froth
(a) I only (b) I and II (d) NaCN is never added in froth flotation process
(c) II and III (d) III only
Chapter 17: Metallurgical Extraction | 767

i 83- Which method is not correct given for refining of (c) both (a) and (b) are correct
crude metals? (d) none of the above is correct
(a) Distillation : zinc and mercury 93. Froth flotation process used for the concentration of
(b) Liquation: tin sulphide ore
(c) van Arkel: Zirconium (a) is based on the difference in wettability of
(d) Mond process : lead different minerals
84. Self-reduction of CuS to Cu can be carried out in: (b) uses sodium ethyl xanthate, C^HgOCS^a as
(a) Bessemer converter (b) Pierce-Smith converter collector
(c) both (a) and (b) (d) None of these (c) uses NaCN as depressant in the mixture of ZnS
and PbS when ZnS forms soluble complex and
85. Select incorrect reduction process PbS forms froth
(a) 2[Ag(CN)2F +Zn---- > [Zn(CN)4f~ +2Ag (d) all of the above are correct
(b) CuO + ---- > Cu + HjO
(c) ZnO + ft, ---- > Zn +11,0 94. Select correct statement(s) about differentiation
(d) MgO + C---- » Mg + CO flotation
(a) is used to concentrate mixture of sulphides at
86. Ellingham diagram represents different stages
(a) change of AG with temperature (b) zinc sulphate is used to depress zinc sulphide
(b) change of AH with temperature and PbS forms froth
(c) change of AG with pressure (c) copper sulphate is used to activate depressed
(d) change of (AG - TAS) with temperature lead sulphide and O2 is blown to make froth
87. Selection of temperature to carry out a reduction (d) all of the above are correct statements
process depends so as to make 95. Which step is not involved in hydrometallurgical
(a) AG negative (b) AG positive process?
(c) AH negative (d) AH positive (a) Ci^S + 2CU2O ——» 6Cu + SO2
88. The reduction of an oxide by aluminium is called (b) CuFeS2 + ZlijSCXl4---- > CuSO4 + FeSO4 + 2^8
(a) Ellingham process (c) CuSO4 + Fe---- >• FeSO4 + Cu
(b) Goldschmidt’s aluminothermite process (d) CuCO3 + H2SO4 ---- > CuSO4 + HjO + CO2
(c) Kroll’s process
96. When alloy of silver and lead is rich in silver,
(d) van Arkel process
(a) cupellation process is used
89. Select correct statement. (b) Parke’s method is used
(a) In the decomposition of an oxide, into oxygen and (c) Pattinson’s method is used
gaseous metal, entropy increases. (d) any of the above methods can be used
(b) Decomposition of an oxide is an endothermic
change. 97. Magistral is the burnt pyrites containing
(c) To make AG° negative, temperature should be (a) sulphates of iron and copper
high enough so that T AS0 > AH0. (b) sulphates and oxides of iron and copper
(d) All of the above are correct statements (c) oxides of iron and copper
(d) sulphides of silver and lead
90. Which does not represent correct method?
(a) TiCl2 + 2Mg---- > Ti + 2MgCl2 : Kroll 98. In the leaching of Ag2S with NaCN, a stream of air is
(b) Ni(CO)4 ---- > Ni + 4CO : Mond also passed. It is because of
(c) Ag2CO3 ---- > 2Ag + CO2 +i1 O2 : van Arkel (a) reversible nature of reaction between Ag2S and
2 NaCN
(d) Zr^ ---- > Zr + 2^ : van Arkel (b) to oxide Na2S formed into Na2S2O3 and sulphur
91. Pyrometallurgical method used for the extraction of (c) both (a) and (b)
copper from sulphide ore (d) none of the above
(a) is a dry method melt and zinc .» 17 x - ix
99. (Ag + Pb) alloy ----------- —» (Ag + Pb + Zn) melt
(b) involves concentration by leaching the sulphide is added
ore with dil. HjSQi cool LayerX
------------ 7
(c) involves concentration of the sulphide ore by LayerY
froth flotation process
(d) statement given in (b) is incorrect Select correct statement(s) based on above scheme
(a) layer X contains zinc and silver
92. Blister copper is (b) layer Y contains lead and silver but amount of
(a) impure copper silver in this layer is smaller than in the layer X.
(b) obtained in self-reduction process during (c) X and Y are immiscible layers
bessemerisation (d) all of the above are correct statement
768 | Essential Inorganic Chemistry

A 111. Black tin is


100. AgCl + Na2CO3 ---- > Ag2CO3 X,Xis
(a) an alloy of Sn (b) an allotrope ofSn
(a) Ag2O and CO2 (b) Ag, O2 and CO2
(c) 60-70 per cent SnO2 (d) 100 per cent SnO2
(c) Ag2O2 and CO2 (d) no effect
101. Purest form of iron is 112. SnO2 is reduced to metallic Sn on smelting oxide will
(a) cast iron (b) wrought iron anthracite, limestone and sand.
(c) pig iron (d) none of these In this, function of sand is
102. The most important method of making steel is (a) it acts as a flux
(a) BOP process (b) BOD process (b) it removes basic impurities as slag
(c) pudding process (d) smelting (c) both as in (a) and (b)
(d) none of the above
103. Cementite is
(a) predominant content of cement 113. Poling process
(b) a mineral of calcium (a) reduced SnO2 to Sn
(c) a carbide of iron in steel (b) oxidises impurities like iron and removes as scum
(d) none of the above (c) uses green poles
(d) involves all of the above
104. Phosphorus separates, in the extraction of iron, as
(a) slag, Ca3(PO1)2 (b) volatile, P2O5 114. Refining of tin cannot be done by
(c) slag, Mg3(PO4)2 (d) Ca3P2 (a) cupellation (b) liquation
105. Tempering of steel (c) poling (d) electrorefining
(a) is the heating the steel to appropriate 115. Which is not the correct process-mineral matching in
temperature and then cooling it rapidly metallurgical extraction?
(b) increases mechanical strength (a) Leaching Ag
(c) changes ratio of carbon in cementite (b) Zone-refining Sn
(d) all of the above (c) Liquation Sn
106. Mg can be obtaineds (d) vanArkel Zr
(a) by heating MgCl2 (anhydrous) with Na in the 116. Softening of lead means:
atmosphere of coal gas (a) melting pure lead at high temperature
(b) by electrolysis of fused carnallite (b) removal of impurities, except silver, present in
(c) by both the methods commercial lead
(d) by none of the above methods (c) formation of lead alloy
107. During the electrolysis of carnallite, MgCl2 is (d) formation of 100% pure lead
decomposed and not KC1. This is because of 117. Softened lead is desilverised by
(a) lower decomposition voltage of MgCl2 than that (a) Parke’s method (b) Pattinson’ method
ofKCl (c) both (a) and (b) (d) none of these
(b) reverse reaction MgCl2 + 2K---- > Mg + 2KC1 if
118. Self reduction of the sulphide ore takes place during
KC1 is decomposed under other experimental
condition (a) roasting (b) smelting
(c) both (a) and (b) (c) calcination (d) cupellation
(d) none of the above 119. Lead pencil contains
108. NaCl andCaCl2 are also added to fused MgCl2 in the (a) graphite (b) softened lead
electrolysis of MgCl2 since (c) galena (d) anglesite
(a) melting point is decreased and conductivity is 120. Select incorrect statement.
increased (a) When the lead-silver alloy is rich in silver, lead is
(b) melting point is increased and conductivity is removed by the cupellation process
decreased (b) When the lead-silver alloy is rich in lead, lead is
(c) melting point and conductivity both are removed by Parke’s or Pattinson’s process
decreased (c) Zinc forms an alloy with lead from which lead is
(d) melting point and conductivity both are separated
increased (d) Zinc forms an alloy with silver, from which zinc is
109. Which is the sulphate ore of Mg? separated by distillation
(a) Dolomite (b) Carnallite
(c) Magnesite (d) Kieserite 121. In the Baeyer’s process:
(a) A12O3 goes into solution as soluble Al(0H)4 while
110. Which is the chloride ore of Mg? other basic oxides as TiO2 and Fe^ remain
(a) Kieserite (b) Carnallite insoluble.
(c) Epsomite (d) Dolomite
Chapter 17: Metallurgical Extraction | 769

(b) A12O3 changes to AIN which in turn decomposed 130. Calamine is


byHgO (a) ZnS (b) PbCO3
(c) A12O3 changes to A12(CO3)3 which changes to (c) ZnCO3 (d) MgCO3
5 A1C13 131. The methods chiefly used for the extraction of lead
(d) none of the above is correct and tin from their ores are respectively
122. In electrolysis of A12O3 by Hall-Heroult process (a) self reduction and carbon reduction
(a) Cryolite Na3[AlF6 ] lowers the m.p. of A12O3 and (b) self reduction and electrolytic reduction
increases its electrical conductivity. (c) carbon reduction and self reduction
(b) Al is obtained at cathode and probably CO2 at (d) cyanide process and carbon reduction
anode 132. Which ore contains both iron and copper?
(c) both (a) and (b) are correct (a) Cuprite (b) Chalcocite
(d) none of the above is correct (c) Chalcopyrite (d) Malachite
CO 133. Extraction of zinc from zinc blende is achieved by
I h2o A
123, Bauxite +C AIN A1(OH)3 AI2O3 (a) electrolytic reduction
I (b) roasting followed by reduction with carbon
N2
(c) roasting followed by reduction with another
This flow-sheet is for metal
(a) Baeyer’s process (b) Serpeck’s process (d) roasting followed by self-reduction
(c) Hall’s process (d) Kroll’s process 134. The method of zone refining of metals is based on the
124. Corundum is........... mineral of Al. principle of
(a) silicate (b) oxide (a) greater noble character of the solid metal than
(c) double salt (d) sulphate that of the impurity
125. Which mineral has been named incorrectly? (b) greater solubility of the impurity in the molten
state than in the solid
(a) Bauxite A12O3-2H2O (c) greater mobility of the pure metal than that of
(b) Corundum A12O3 impurity
(c) Cryolite 3NaFAlF3 (d) higher melting point of the impurity than that of
(d) Feldspar Be3Al2Si6O18 the pure metal
135. A12O3 can be converted to anhydrous A1C13 by
126. The chemical process in the production of steel from heating
haematite ore involves
(a) A12O3 with HC1 gas
(a) reduction
(b) oxidation (b) A12O3 with NaCl in solid state
(c) reduction followed by oxidation (c) a mixture of A12O3 and carbon in dry Cl2 gas
(d) oxidation followed by reduction (d) A12O3 with Cl2 gas
127. Electrolytic reduction of alumina to aluminium by 136. Flux is used to remove
Half-Heroult process is carried out (a) silica (b) metal oxide
(a) in the presence of NaCl (c) silica and metal oxide (d) impurities from ore
(b) in the presence of fluorite 137. The process of heating and suddenly cooling of steel is
(c) in the presence of cryolite which forms a melt known as
with lower melting temperature (a) tempering (b) annealing
(d) in the presence of cryolite which forms a melt (c) hardening (d) quenching
with higher melting temperature
138. The metal which cannot be extracted by smelting
128. The chemical composition of‘slag* formed during the process
smelting process in the extraction of copper is (a) Zn (b) Al
(a) CujO + FeS (b) FeSiO3 (c) Pb (d) Fe
(c) CuFeS2 (d) Ct^S+FeO
139. Which of the following processes is related with the
129. Which of the following processes is used in extractive removal of sulphur by heating in the air?
metallurgy of magnesium? (a) Smelting (b) Calcination
(a) Fused salt electrolysis (c) Annealing (d) Roasting
(b) Self reduction
(c) Aqueous solution electrolysis 140. In electrorefining of copper, some gold is deposited as
(d) Thermite reduction (a) cathode (b) electrode
(c) cathode mud (d) anode mud
770 | Essential Inorganic Chemistry

141. Bessemer converter is used for preparation of 154. The silver is extracted by Parke’s process. The basL
(a) steel (b) wrought iron of this method is
(c) pig iron (d) cast iron (a) silver is immiscible in molten Zn
142. Colemanite is (b) Ag is miscible in NaCN
(c) Ag is more miscible in molten zinc than in molten
(a) Ca[B3O4(OH2)] -2H2O
Pb
(b) Ca2B6On ■ 5H2O (d) Ag is more miscible in molten Pb in comparison tc
(c) Ca(OH)2 molten zinc
(d) Na2B4O7 -2H2O 155. Railway wagon axles are made by heating iron rods
143. Mac Arthur process is used for the extraction of embedded in charcoal powder. This process is known
(a) Au (b) Ag as
(c) Cu (d) Zn (a) tempering (b) annealing
(c) sherar dising (d) case hardening
144. Which of the following is not an iron ore?
(a) Caniterite (b) Magnetite 156. Electric refining is used for refining of
(c) Limonite (d) Copper pyrite (a) Cu (b) Fe
(c) Pb (d) Al
145. Thermite is a mixture of iron oxide and
(a) aluminium powder (b) zinc powder 157. To dissolve argentite ore which of the following is
(c) potassium metal (d) sodium metal used
(a) Na[Ag(CN)2] (b) NaCN
146. In the extraction of copper from its sulphide ore, the
(c) NaCl (d) HC1
metal is formed by reduction of Cu^O with
(a) FeS (b) CO 158. In aluminothermic process, Al acts as a
(c) Ci^S (d) SO2 (a) flux (b) oxidising agent
(c) reducing agent (d) solder
147. Which of the following is a carbonate ore?
(a) Pyrolusite (b) Malachite 159. Which of the following is not an ore of magnesium?
(c) Diaspore (d) Cassiterite (a) Carnallite (b) Magnesium
(c) Dolomite (d) Gypsum
148. Purification of alumina takes place by
(a) Bosch process (b) Hall’s process 160. The formula of malachite ore is
(c) Hoope’s process (d) Quartation process (a) Fe3O4 (b) Mn3O4
(c) NiAs (d) Cu(OH)2CuC03
149. The hardest naturally occurring substance is
(a) iron (b) graphite 161. Which of the following has maximum percentage of
(c) diamond (d) astatine copper?
(a) CugS (b) CU2O
150. Wolframite ore is separated from tinstone ore by the
(c) CuFeS2 (d) CuCO3 Cu(0H)2
process of
(b) roasting (b) electromagnetic 162. The chief impurity present in red bauxite is
(c) smelting (d) calcination (a) SiO2 (b) Fe2O3
(c) K2SO4 (d) NaF
151. By annealing, steel
(a) becomes soft 163. The extraction of which of the following metals
(b) becomes liquid involves bessemerisation?
(c) becomes hard and brittle (a) Fe (b) Ag
(d) is covered with a thin film of Fe3O4 (c) Al (d) Cu
152. The process of purification of metals is represented 164. Which of the following metal is correctly matched
by the following scheme with its ore?
1400°C
Ti+212 Til, Ti +L Metal Ore
pure
(a) Zinc Calamine
(a) cupellation (b) poling
(b) Silver lllmenite
(c) electrolytic refining (d) van-Arkel method
(c) Magnesium Cassiterite
153. Among the following statements, the correct one is
(d) Tin Azurite
(a) zinc blende and iron pyrites are sulphides
(b) calamine and siderite are carbonates 165. A cuprous ore among the following is
(c) malachite and chalcocite are the copper ore (a) cuprite (b) malachite
(d) argentite and cuprite are oxides (c) chalcopyrites (d) azurite
Chapter 17: Metallurgical Extraction | 771

166. Aluminium oxide is not reduced by chemical 177. In Mac-Arthur Forrest method, silver is extracted
reactions since from solution of Na[Ag(CN)2 ] by the use of
(a) reducing agent contaminates (a) Fe (b) Mg
(b) the process pollutes the environment (c) Cu (d) Zn
3
(c) aluminium oxide is highly stable 178. Which of the following process is not involved in
(d) aluminium oxide is reactive manufacturing of steel?
) 167. Identify the reaction that does not take place during (a) Bessemer process (b) Open hearth process
smelting process of copper extraction (c) BOP (d) Thermite process
j (a) 2FeS+3O2 ---- > 2FeO+2SO2 179. Bauxite is concentrated by
i (b) CujO + FeS ---- > Ci^S+FeO (a) chemical method
(b) roasting
(c) 2Cu2S+3O2 ---- > 2Cu2O + 2SO2 (c) magnetic separation
(d) FeO + SiO2 ---- > FeSiO3 (d) froth flotation process
168. Gold is extracted by hydrometallurgical process, 180. The ore carnallite is represented by structure
based on its property (a) Na2Al2O3 (b) Na3AlF6
(a) of being electropositive (c) KClMgCl2 -eHjO (d) Fe3O4
(b) of being less reactive 181. Which of the following statement is incorrect?
(c) to form complexes which are water soluble (a) Pure aluminium oxide is obtained by heating
(d) to form salts which are water soluble aluminium hydroxide.
169. In blast furnace, iron oxide is reduced by (b) Cryolite lowers down the melting point of bauxite
(a) hot blast of air (b) carbon monoxide in electrolytic cell for extraction of aluminium.
(c) Carbonate ores are converted into oxides by
(c) carbon (d) silica
roasting ore in air.
170. During the extraction of Fe the slag obtained is (d) Mercury cannot be produced by roasting the
(a) FeSiO3 (b) FeO cinnabar ore in air.
(c) CaSiO3 (d) MgSiO3 182. A carbonate ore is
171. The final step for the extraction of copper from copper (a) carnallite (b) limonite
(c) siderite (d) horn silver
pyrites in Bessemer converter involves the reaction
(a) Ci^S + 2FeO ---- > 2Cu + 2Fe + SO2 183. Chalcopyrites is an ore of
(a) gallium (b) copper
(b) CujS + 2Cu2O ---- > 6Cu + SO2 (c) calcium (d) magnesium
(0 4Cu2O + FeS ---- > 8Cu + 2FeSO4
184. Thomas slag is
(d) 2Cu2O + FeS ---- > 4Cu + Fe + SO2 (a) Ca3(PO4)2 -2H2O (b) Ca3(PO4)2 CaSiO3
172. Lead as impurity in the extraction of silver is (c) MgSiO3 (d) CaSiO3
removed by which process? 185. Flux in the smelting process is added to
(a) Cyanide process (a) decrease the solubility of impurities
(b) Solvay process (b) increase the fusion temperature of roasted ore
(c) Parke’s process (c) convert impurities into slag
(d) Froth flotation process (d) all of the above
173. The ore of aluminium is
(a) carnallite (b) malachite One or More Than One Options Correct
(c) galena (d) bauxite 1. Of the following reduction processes, correct
174. The process of zinc plating on iron sheet is known as processes are:
(a) annealing (b) roasting (a) Fe2O3 + C —> Fe
(c) galvanization (d) smelting (b) ZnO + C - ■> Zn
175. A metal used to recover copper from a solution of (c) Ca3(PO4)2 +C---- > P
CuSO4 is (d) PbO + C---- > Pb
(a) Fe (b) Hg
(c) Na (d) Ag 2. Following reaction(s) is/are not involved in Thermite
process
176. Gravity separation process is used for the (a) 3Mn3O4 + 8A1---- » 9Mn + 4A12O3
concentration of (b) Ci2O3 + 2A1---- > A12O3 + 2Cr
(a) calamine (b) haematite (c) 2Fe + A12O3 ---- > 2A1 + Fe.2O3
(c) chalcopyrites (d) bauxite (d) B2O3 + 2A1---- > 2B + Al263
772 | Essential Inorganic Chemistry

3. Consider the following steps: AG° for CuO = 129.7 kJ mol-1, CO = -137.2 kJ mol’"
(JUS roas1, in a*r) (yjj roast without air >
H2O = - 237.2 kJ mol-1
Which are the correct statements? 3. Copper pyrites, CuFeS2, is the important source c
(a) It is self-reduction copper. 10 g of it was leached with dil. H^SO^ ab­
(b) It involves disproportionation solution diluted to 1 L.
CugS---- > Cu + CuS (a) 10 mL of this solution required 10 mL of 0.02
(c) (A) is a mixture of CuO and CuS and (B) is a KMnO4 in acidic solution
mixture of Cu and SO2 (b) 10 mL of this solution was treated with excess u
(d) (A) is a mixture of Cu and SO2 and (B) is CuSO4 KI and liberated Ig required 10 mL of 0.02 V-
4. Phosphine is obtained by Na2S2O3 solution. (Cu = 63.5, Fe = 5.6)
(a) Ca3P2 + H2O ---- > (b) P4 + NaOH---- > What is % of Fe and Cu in CuFeS2 ?
(c) CHC13 + O2 ---- » (d) CC14 + O2 ---- > 4. Use the relationship AG° = - nFEc°eI1 to estimate the
5. Carbon cannot be used in the reduction of A12O3 minimum voltage required to electrolyse A12O3 in the
because Hall-Heroult process.
(a) It is an expensive proposition AG;(A12O3) = - 1520 kJ mol'1
(b) The enthalpy of formation of CO2 is more than AG^(CO2) = -394kJmol-1
that of A12O3 Show that the oxidation of the graphite anode to CO,
(c) Pure carbon is not easily available permits the electrolysis to occur at a lower voltage
(d) The enthalpy of formation of A12O3 is too high than if the electrolysis reactions were
6. In which of the following isolations no reducing agent A12O3 ---- > 2A1+3O2.
is required?
5. Ferrochrome, an iron-chromium alloy used in
(a) Iron from haematite
making stainless steel, is produced by reducing
(b) Aluminium from bauxite
chromite (FeCigO^ with coke
(c) Mercury from cinnabar
(d) Zinc from zinc blende FeCi2O4 (s) + 4C(s) ---- > Fe(s) + 2Cr(s) + 4C(Xg)
7. Which of the following are methods of concentration ferrochrome
of metals? (a) How many kilograms of chromium can be
(a) Gravity separation obtained by the reaction of 236 kg of chromite
(b) Froth flotation process with an excess of coke?
(c) Electromagnetic separation (b) How many litres of carbon monoxide at 25°C and
(d) Smelting 740 mm Hg are obtained as a by-product?
8. Boron can be obtained by various methods but not by (Cr = 52, Fe = 56, C = 12,0 = 16)
(a) thermal decomposition of I^Hg 6. Complete the following puzzle-crossword
(b) pyrolysis of BI3 (van Arkel)
Metals and activity series crossword. 1. Displaces
(c) reducing BC13 with Hg
silver but not lead. 2. Honorary metal in many
(d) electrolysis of fused BC13
versions of the activity series. 3. Unreactive metal, a
9. Following method is not used for extraction of Al salt of which is used in the chloride test. 4. Most
(a) van Arkel (b) Serpeck abundant transition metal in Earth’s crust. 5. This
(c) Baeyer (d) Hall-Heroult metal forms a nitrate which is hard to decompose.
10. Roasting is carried out to 6. Metal used in sacrificial protection of iron from
(a) convert sulphide to oxide and sulphate corrosion. 7. First member of Group IA element.
(b) remove water of hydration 8. This metal does not react with water, but reacts
(c) melt the ore with acid.
(d) remove arsenic and sulphur impurities _1_
2
Brain Twisters 3
1. ^ 2+/v = 0.44V,E° 2+in = 0.34V,’ EAg
A +. /Ag = 0.80 V 4
Fez+/Fe ’ Cuz+/Cu
5
Will Fe displace Cu from CuSO4 solution? 6
Will Ag displace Cu from CuSO4 solution? 7
2. CuO can’t be reduced by carbon but can be reduced by 8
Hg. Explain in terms of thermodynamics, given :
Chapter 17: Metallurgical Extraction | 773

i 7. 18.4 g of a mixture of CaCO3 and MgCO3 on strongly 2ZnS + 3O2 2ZnO + 2SO2
heating gave 8.8 g of CO2 gas. Calculate mole% of The ZnO is then treated with dilute H2SO4
each in the mixture.
1 ZnO + H2SO4 ZnSO4 + ^O ...(ii)
d to produce ZnSO4(a<?) which produces Zn metal on
Passage Comprehension Questions
electrolysis.
4 Passage 1 98.2%
2ZnSO4 + 2H2O ------- > 2Zn + 2^804 + O2 ...(iii)
Read the following short write-up about recovery of
f magnesium and answer the questions at the end of it. 1. What mass of Zn will be obtained from an ore
1 containing 225 kg of ZnS? Efficiencies of the
Magnesium is a valuable, light weight metal used as a
structural material as well as in alloys, in batteries, and in process have been indicated above the arrow
chemical synthesis. Although magnesium is plentiful in mark. (Zn= 65,S= 32,0= 16,H= 1)
Earth’s crust, it is cheaper to mine the metal from (a) 134 kg (b) 112 kg
ft sea-water. Magnesium forms the second most abundant (c) 102 kg (d) 130 kg
cation in the sea (after sodium); there are about 1.3 g of 2. What amount of current is required (with 100%
magnesium in a kilogram of sea-water. The process for efficiency) in step (iii) if it takes one month?
obtaining magnesium from sea-water employs all three (a) 10.2 A (b) 15.4 A
types of reactions precipitation, acid-base, and redox (c) 17.0 A (d) 154.0 A
reactions. 3. ZnO in step (ii) can also be dissolved in NaOH
1. Precipitation reaction involves formation of forming
(a) insoluble MgCO3 by adding Na2CO3 (a) Zn(OH)2 (b) Na2ZnO2
(b) insoluble Mg(OH)2 by adding Ca(OH)2 (c) Zn2O3 (d) NaZn(OH)4
(c) insoluble MgSO4 by adding Na2SO4 4. How many kilomoles of NaOH are required to
(d) insoluble MgCl2 by adding NaCl dissolve all the ZnO of step (i) assuming 100% yield?
2. Acid-base reaction involves reaction between (a) 2.32 (b) 1.16
(a) MgCO3 and HC1 (b) Mg(OH)2 andH2SO4 (c) 4.64 (d) 9.28
(c) Mg(OH)2 andHCl (d) MgCO3andH2SO4 5. What volume of 98% H2SO4 (by weight, density
3. Redox reaction takes place (in the extraction of Mg) 1.8 g/mL) is required in step (ii) assuming 100% yield
(a) in the electrolytic cell when fused MgCl2 is of steps (i) and (ii)
subjected to electrolysis (a) 130 L (b) 140 L
(b) when fused MgCl2 is heated (c) 120 L (d) 150 L
(c) when fused MgCO3 is strongly heated
(d) none of the above Passage 3
4. Instead of calcium oxide, why don’t we simply add Questions given below are based on the following
sodium hydroxide to sea-water to precipitate sequence of reactions:
magnesium hydroxide? At high temperature carbon reacts with water to
(a) Solubility of Ca(OH)2 is smaller than that of produce a mixture of carbon monoxide (CO) and hydrogen
NaOH so thatMg(OH)2 is precipitated (H,).
(b) NaOH may dissolve Mg(OH)2 formed C+ HjO redCO+H2
(c) NaOH may also precipitate other species, being a
strong electrolyte CO is separated from H2, and then used to separate
(d) NaOH, being a weak electrolyte will not nickel from cobalt by forming a volatile compound, nickel
coagulate Mg(OH)2 tetracarbonyl, Ni(CO)4.
5. Which is the best source of the given process in the Ni + 4CO---- > Ni(CO)4 (Ni = 59)
metallurgical process?
(a) Magnesite (b) Kieserite 1. How many moles of Ni(CO)4 could be obtained from
(c) Epsomite (d) Dolomite the CO produced by the reaction of 75.0 g of carbon?
Assume 100% reaction and 100% recovery in both
steps.
Passage 2 (a) 6.25 (b) 1.563
Following passage describes extraction of zinc from (c) 3.125 (d) 25.0
zinc sulphide. Answer the questions at the end of it. 2. Formation of volatile Ni(CO)4 and its subsequent
The chief ore of zinc is the sulphide, ZnS. The ore is
concentrated by flotation process and then heated in air, heating gives pure Ni. This process is called
which converts the ZnS to ZnO. (a) Hall (b) Dow
(c) Serpeck (d) Mond
774 | Essential Inorganic Chemistry

Passage 4 (b) their much low electronegativity values as a


FeCr2O4 (chromite) is a good source of chromium and result of which ionic sulphides are formed
its compounds like Na2CrO4, Na2Cr2O7. Questions are (c) high melting point of sulphides
based on the following reactions. (d) low melting point of sulphides
P :FeCr2O4 + NaOH + air---- > A + Fe2O3 3. s-block elements generally occurs as carbonates and
Q: A + B ---- > Na2Cr2O7 silicates (as MgCO3, CaSiO3) because
R: Na2Cr2O7 + X Cr2O3 (a) these elements react with CO2 and SiO2
S: Cr2O3 + Y ---- > Cr (b) these elements are decomposed by acid reacts
(c) their oxides are basic and oxides react with acidic
1. Compounds (A) and (B) are oxides in an acid-base reactions
(a) Na2CrO4,H2SO4 (b) Na2Cr2O7,HCl (d) their oxides are acidic and oxides react with basic
(c) Na2CrO5,H2SO4 (d) Na4[Fe(OH)6],H2SO^'4 oxides in an acid-base reaction
2. (X) and(Y) are 4. Elements found in native state are
(a) Al and C (b) C and Al (a) Pb and Bi as they are in p-block
(c) C in both (d) Al in both (b) Hg and Os as they are in liquid state
(c) C and Si as they are non-metals
3. Na2CrO4 and Fe2O3 are separated by
(d) Pt and Au as they are at the bottom of the
(a) dissolving in cone. H2SO4 electrochemical series
(b) dissolving in NH3
(c) dissolving in H20
(d) dissolving in dil. HC1 Passage 6
Questions given below are based on the natural
occurring minerals I ores.
Passage 5 Look at the location of elements A, B, C and D in the
Questions given below are based on the Periodic Table following Periodic Table and answer the questions given
showing different ores I minerals. below
1 18
2 13 14 15 16 17
Li Re
Na Mg 3 4 5 6 7 8 9 10 11 12 Al
A B
K Ca Sc I Ti! V Cr [Mid Fe Co Ni Cu Zn Ga
C
Rb Sr. Y 'l ;Xb|Mo|CS)|Ru|Rh| Pd I Ag| Cd In So
D
CsjBa La Hi- Ta, W Re Os Ir Pt Au Hg T1 Pb Bi

I | W
Chlorides Silicates Carbonates Phosphates Oxides Free metals Sulphides
Primary mineral sources of metals. The s-block metals occur as
chloride, silicates, and carbonates. The d- and p-block metals
1. Which of the elements indicated by A, B, C and Dis
are found as oxides and sulphides, except for the group 3,
expected to be found in native state?
metals, which occur as phoshates, and the platinum group (a) A (b) B
metals and gold, which occur in uncombined form. There is no (c) C (d) D
mineral source of technitium (Tc in group 7), a radioactive 2. Which is found as its sulphide?
element that is made in nuclear reactors. (a) A (b) B
(0 C (d) D
1. The early transition metals on the left hand side of
the d-block generally occur as oxides because 3. Which is found as its carbonate?
(a) they are above hydrogen in electrochemical (a) A (b) B
series (c) C (d) D
(b) they are more electropositive and thus form ionic 4. Which is found as its oxide?
compounds (in the form of oxides) by more (a) A (b) B
electronegative oxygen by exchange of electrons (0 C (d) D
(c) these oxides are soluble in water 5. Imagine a planet with an atmosphere that contains
(d) these oxides remain insoluble in water 02 and S02 but no C02. What is the chemical
2. Transition metals on the right hand side of the composition of the mineral you would expect to find
d-block occurs as sulphides because of for the alkaline earth metals (M) on such a planet?
(a) their high electronegativity values as a result of (a) MO2,M2O2 (b) MSO3,MSO4
which covalent sulphides are formed (c) M(HSO3)2, Af(HSO4)2 (d) all of these
Chapter 17: Metallurgical Extraction | 775

Passage 7 (b) use of sodium ethyl xanthate as a collector that


coats the particles of ore
Read the following cyclic process of recovery of copper
(c) both (a) and (b)
from a copper wire and answer the questions at the end ofit.
(d) none of the above
The following cycle of copper experiment is performed
in some general chemistry laboratories. The series of 2. Smelting process is carried at about 1400°C. Two
reactions starts with copper and ends with metallic layers are formed
copper. The steps are as follows: (1) A piece of copper wire (a) Bottom layer contains copper-matte
of known mass is allowed to react with concentrated nitric (b) Upper layer contains copper-matte
acid [the products are copper(II) nitrate, nitrogen dioxide, (c) Copper-matte remains in both the layers
and water}. (2) The copper (II) nitrate is treated with a (d) Copper-matte is not present in any of the layers
sodium hydroxide solution to form copper (II) hydroxide 3. Some of the reactions taking place during
precipitate. (3) On heating copper (II) hydroxide bessemerisation are
decomposes to yield copper (II) oxide. (4) The copper (II) A: Fe0
JC + OIW2
tiky T Si02 ---- > FeSiO3
oxide is reacted with concentrated sulphuric acid to yield
copper (II) sulphate. (5) Copper(II) sulphate is treated B: 2FeS + 3O2 ■» 2FeO + 2SO2
with an excess of zinc metal to form metallic copper. C: 2Cu2S + 3O2 » 2Cu2O + 2SO2
(6) The remaining zinc metal is removed by treatment D: 20^0 + CujS -> 6Cu + SO2
with hydrochloric acid, and metallic copper is filtered, Select reactions actually taking place
dried, and weighted. (Cu = 63.5) (a) A, B (b) B, C
1. Assuming that a student started with 65.6 g of (c) A, C (d) C, D
copper, calculate the theoretical yield of copper 4. In the electrorefining method
sulphate (a) anode is pure copper
(a) 165 g (b) 82.4 g (b) anode is impure copper
(c) 90 g (d) 100.2 g (c) anode mud contains noble metals
2. Copper obtained at the end of the cycle is............. (d) cathode mud contains noble metals
amount originally taken 5. This method is called
(a) less than (a) pyrometallurgical (b) hydrometallurgical
(b) greater than (c) parametallurgical (d) semimetallurgical
(c) nearly equal
(d) only copper salts are obtained Passage 9
3. CuSO4 solution thus formed is subjected to Questions given below are based on the given diagram
electrolysis. To deposit all the copper formed at the for extractive metallurgy.
end, a current of 50 A will have to be used for 0
approximately
(a) 10 h (b) lh -200
(c) 5h (d) 0.5 h
t -400
2
Passage 8 •0 -600
Given flow-sheet is for the extraction of copper. Based <
on this, answer the questions given at the end -800
CuFeS2 Froth Floatation Roasting so2 -1000

Cu2S + FeO

Temperature, °C —►
Blister Cu Bessemerisation Matte ♦
The points noted by arrows are the melting and boiling points of
Smelting the metals zinc and magnesium. AG° as a function of temperature
Electrolysis SiO2 FeSiO3 for some reactions of extractive metallurgy.

Pure Cu 1. At what approximate temperature, zinc and carbon


have equal affinity for oxygen?
1. Froth-flotation process can be made more effective by (a) 1000°C (b) 1500°C
(a) use of pine oil which produces stable froth (c) 500°C (d) 1200°C
776 | Essential Inorganic Chemistry

2. At this temperature AG° of the reaction is (d) reduction is highly endothermic and heat
ZnO + C---- > Zn + CO absorbed makes the process spontaneous.
(a) -ve (b) +ve 6. Following reaction is not involved in Thermite
(c) zero (d) nothing can be said process
3. To make the following reduction process (a) 3Mn3O4 + 8A1---- > 9Mn + 4A12O3
spontaneous, temperature should be (b) C^O3 + 2A1---- > A12O3 + 2Cr
ZnO + C ---- > Zn + CO (c) 2Fe + A12O3 ---- > 2A1 + Fe2O3
(a) < 1000°C (d) B.2O3 + 2A1---- > 2B + A12O3
(b) > 1100°C
(c) < 500°C Passage 11
(d) > 500°C but < 1000°C
The Ellingham diagram for a number of metallic
4. At 1100°C, which reaction is spontaneous to a sulphides is reproduced below.
maximum extent?
(a) MgO + C---- > Mg + CO
(b) ZnO + C---- > Zn + CO
(c) MgO + Zn ■» Mg+ZnO
0/
t
(d) ZnO + Mg -> MgO + Zn 0- */ X

X
6/
Passage 10 o

From the following flowsheet for the extraction ofpure


I "50~
0
metal, answer the questions at the end of it. <
-100-
x /
Sulphide A Oxide B Impure metal 7/
-150

Pure metal C
T “i i I
1. Step A is 500 1000 1500 2000
(a) roasting (b) smelting Temperature/'C
(c) calcination (d) bessemerisation
Answer the questions given below
2. Step B (reduction) can be carried out using
(a) carbon 1. Formation of which of the sulphides is most
(b) more electropositive element spontaneous?
(c) both (a) and (b) (a) HgS (b) Bi2S3
(d) none of the above (c) PbS (d) CS2
3. Impure Cu metal is also obtained 2. Which occurs to minimum extent in nature?
(a) by self reduction during roasting of CuS (a) HgS (b) H2S
(b) by reduction of CuO with Hj (c) Bi2S3 (d) CS2
(c) by reduction of CuO with Al 3. Which of the following sulphides can be reduced to
(d) by electrolysis of Cu2+solution metal by H^ at about 1000°C?
4. Some of the following metals are obtained by (a) HgS (b) PbS
electrolysis of their fused salts (c) Bi2S3 (d) All of these
Al, Na, Cu, Ag, Mg, Ca. These metals are
(a) Cu, Ag (b) Al, Na, Cu Assertion & Reason
(c) Ag, Mg, Ca (d) Al, Na, Mg, Ca
5. Reduction of oxides to elements with carbon Codes:
generally takes place at high temperature, hence Al (a) Both A and R are true and R is the correct
is used in aluminothermite process. It is because explanation of A.
(a) Al is more electropositive than the other metals (b) Both A and R are true but R is not the correct
(to be formed by reduction) explanation of A.
(b) Al has maximum affinity for oxygen (c) A is true but R is false.
(c) reduction is higly exothermic and heat liberated (d) A is false but R is true.
makes the process spontaneous (e) A and R both are false.
Chapter 17: Metallurgical Extraction | 777

1. Assertion (A): Gold occurs in native state. 17. Sulphide ore is converted into pure metal (say Cu) by
Reason (R): Gold dissolves in aqua-regia. following steps.
2. Assertion (A) : Froth-flotation process is used to Bessemerisation---- > Roasting---- > Electrolysis
concentrate sulphide ores. 18. Dow sea-water process is used for the extraction of
Reason (R): There is difference in the wettability Mg.
of different minerals. 19. FeWO4 andMnWO4 are magnetic impurities.
3. Assertion (A) : A12O3 is converted into Al by 20. Calcium silicate is called cementite.
reduction with carbon.
Reason (R): Carbon (graphite) has greater affinity Fill in the Blanks
for oxygen than aluminium has.
1. When(NH4)2Ci2O7 is heated, green coloured residue
4. Assertion (A) : Metals of high impurity are
is along with gas
obtained by zone-refining.
Reason (R): Impurities in the ore are more soluble 2. The reducing agent used in thermite process is
in the melt than in the pure metal.
5. Assertion (A) : Reduction of ZnO with carbon is 3. In cyanide process, Zn is added to soluble to
done at 1100°C. get Ag.
Reason (R): At this temperature, AG° is negative 4. Temperature required in pyrometallurgical process
thus process is spontaneous. is than that required in
6. Assertion (A) : Desilverisation of lead is done by hydrometallurgical process.
Parke’s method. 5 iron is the purest form of it.
Reason (R) : When lead-silver alloy is poor in 6 iron is brittle.
silver, zinc is added to molten ore.
7. Most abundant element in the earth's crust is
True & False
8. Most abundant metal in the earth's crust is
1. Lunar caustic is KOH.
2. KCN forms soluble complex with silver glance. 9. Sulphur is extracted by
3. Zinc can displace silver from [Ag(CN)2]~. 10. Zrl< is converted to Zr by the
4. Main source of aluminium is corundum. 11. Mg can be used to reduce TiCl4 into Ti in
5. Wrought iron is prepared by Pudding process. 12. Self-reduction process is used in the extraction of
6. Pig iron is the purest form of iron. and lead.
7. Steel is mostly made by BOP method. 13. Dow sea-water process is used in extraction of
8. Heating the steel to appropriate temperature for a
short time and then cooling it rapidly in order to give 14. In Serpeck process, bauxite is converted into
it the desired mechanical properties is called the which is hydrolysed to precipitate
tempering.
9. Oxides of the s-block metals are strongly acidic and
15. Cryolite, Na3AlF6 is added to A12O3 in
form salts with basic oxides.
process to m.p. and to electrical
10. Oxygen is the most abundant metal on the earth’s
crust. conductivity.
11. Mond’s process is used for the extraction of Mg. 16. Parke’s process is used to separate from
argentiferrous lead.
12. In the bessemerisation of sulphide ore of copper,
there is self-reduction giving copper metal. 17. Be^ and A14C3 are also called . because they
13. Na is obtained by the electrolysis of aqueous NaCl react with HjO yielding
solution. 18. CO is toxic because it forms complex with
14. Al is obtained by the electrolysis of fused mixture of in the blood.
A12O3 andNa3A!F6.
19. When FeCi^ is heated with carbon an alloy called
15. Metals of high purity (to be used in semiconductors) is obtained.
are obtained by zone-refining.
20. FeS2 is also called because it has been
16. Concentration of sulphide ore is done by calcination
method. mistaken for gold.
778 | Essential Inorganic Chemistry

21. ‘18 carrat’ gold is pure. 4. Match ores in List I with metals in List II.
22. ‘Puddling’ process was used originally to make List I List II
A. Carnallite 1. Zinc
23. Puddling process involved mixing molten B. Calamine 2. Titanium
iron with haematite and burning off all the C and C. Ilmenite 3. Magnesium
other impurities to give iron. D. Chalcopyrite 4. Copper
24. BOP is the modem process to prepare
5. Match the extraction process listed in List I with
25. Siemen open-hearth process is used in making metals listed in List II.
List I List II
26. Mond’s process is used in the extraction of
A. Self reduction 1. Zirconium
27. Cr^S is called B. Carbon reduction 2. Silver
28. Malachite (green) is C. Complex formation and 3. Copper
displacement by metal
29. Zinc is used to extract silver by solvent extraction D. Decomposition of iodide 4. Boron
from molten lead in process
30. Paris green is made from basic copper 6. Match the method in List I with related reaction in
acetate, arsenious oxide and acetic acid. List II.
List I List II
Matrix-Match Type Questions A. Mond’s process 1. Cr2O3 +2AI—> AI2O3 +2Cr
B. van Arkel-de-Boer „ 1000-1150°C
I. Only One Correct 2. TiCI4 + Mg-----------
process
Ti + 2MgCI2
1. Match the compounds (in List I) with their
applications/colours (in List II). C. Thermite process 3. Ni(CO)4 Ni + 4CO

D. Kroll process 4. 2CuO + CuS —> 3Cu + S02


List I List II
E. Self reduction 5. Zrl4 Zr + 2I2
A. Pb3O4 1. White lead
B. (PbCO3)2 Pb(OH)2 2. Rust proofing sheets
C. Ca2PbO4 3. Road signs and markings
II. One or More Correct
D. PbCrO4 4. Red lead 1. Match the species in Column I with then corresponding
methods of extraction/concentration in Column II.
2. Match the reactions taking place in blast furnace . ___ List I List II
(in List I) with temperature-range of operations A. Copper pyrites 1. Electrolysis
(in List II). B. Galena 2 Mac-Arthur Forrest method
List I List II C. ‘ Silver glance 3. Extraction by complex formation
D. Corundum 4. Magnetic separation
A. Fe2O3 + CO---- > CO2 + Fe 1. 1200°C
E. Ilmenite 5. Concentration by gravity separation
B. C + CO 2 > CO 2. 900°C
6. Concentration by froth-flotation
C. c + h2o —> co + h2 3. 600°C process
D. CaO + SiO2 —■> CaSiO3 4. 1100°C
2. Match the elements in Column I with their
corresponding mineral formation in Column II.
• 3. Match process (in List I) with electrolytes (in List II).
List I List II
List I List II 1. Oxide
A. Calcium
A. Downs cell 1. Fused MgCI2 B. Copper 2 Sulphide
C. Magnesium 3. Phosphate
B. Dow sea-water process 2. Fused (AI2O3 + Na3AIF6)
D. Silver 4. Carbonate
C. Hall-Heroult 3. Fused KHF2 E. Aluminium 5. Chloride
i
D. Moissan 4. Fused (40% NaCI + 60% 6. Fluoride
CaCI2) 7. Sulphate
Chapter 17: Metallurgical Extraction | 779

Integer Answer Types 2. If reduction ofCu2+ by Al is possible then E°cl| will be


This section contains 9 questions. X Y z W volt, (based on Q. 1)
The answer to each of the questions is © © © ©] 3Cu2++2A1 ---- > 2A13++3Cu
a single digit integer, ranging from 0 to 0 © © ©. 3. How many of the following ores can be enriched by
9. The appropriate bubbles below the © © © © FFP?
respectively question numbers in the ©.© © ©
OflS have to be darkened. For dolomite, cerussite, argentite, pyrites, anhydrite,
© © © © cinnabar, chalcopyrite, galena, greenockite,
example, if the correct answers to ©
© © ©. chalcocite
question number X, Y, Z and W (say)
are 6,0,9 and 2, respectively, then the © © © © 4. Phosphate rock is represented asPb10(PO4)xCl2 what
correct darkening of bubbles will like © 0 0 0
© © © is value of x?
the following
© © © © 5. In the electrolysis of AgNO3 solution, 97.2 g of Ag is
Set I deposited on passing current for 9650 s. What is the
value of current in amperes?
1. Bauxite is A12O3 ■ xHjO, where x =
6. A12O3 is converted to AIN on heating with carbon in
2. Epsomite is MgSO4 • xH2O, where y = the atmosphere of N2. How much carbon is required
3. When one mole of Ag2CO3 is strongly heated, residue to convert 23 g of A12O3 into AIN?
is mole(s). 7. A1(OH)3 is soluble in alkaline solution of pH 8.3010.
4. In the following reaction slag CaSiO3 is formed Thus [NaOH] = y x 10"6 what is value of y?

Ca3(PO4)2 + C + SiO2 ---- > P4 +CaSiO3 +CO 8. Gold is found to be 37.5% pure. What is its grading in
terms of carrat?
If one mole of P4 is formed, then CaSiO3 formed is
moles.
Test Yourself On The Periodic Table
5. One unit formula of azurite contains copper
atoms 1. Locate the following metals in the Periodic Table and
predict whether they are likely to be found in nature
6. Total number of atoms in one unit of chalcopyrites as oxides, sulphides, carbonates or in uncombined
are state
7. One unit of corundum has .... oxygen atoms.
8. Mineral CuFeS2 is 50% pure.
9. 5 moles ofCuFeS2 is dissolved in Mixture will
require mole(s) KMnO4 to oxidise Fe2+ to
Fe3+ .

Set II
This section contains 8 questions.
A: Copper B : Calcium
1. values are :
C : Sodium D : Chromium
E° E : Mercury F : Gold
Ag+ + e" ---- > Ag + 0.80 V 2. Name the final process to obtain the metals indicated
Zn2++2e“ ---- > Zn -0.76 V in the Periodic Table below:
Fe2++2e~ ---- > Fe -0.44 V
Cu2+ +2e” ---- > Cu + 0.34 V A B
+ 0.79 V D
Hg2++2e- ---- > Hg
E
Mg2+ + 2e~ ---- > Mg - 2.37 V C
Al3++3e“ ---- > Al -1.66 V
How many of the metals can displace Cu from CuSO4
solution?
Answers & Solutions
Master Exercises
Exercise 1
Short Answer Type Questions 7. Air is passed when S2 is oxidised to SO2'
1. In practice, sulphide ore is first roasted in air to the S2' + 2O2 ---- > SO2'
corresponding oxide which in turn is reduced to metal
8. Add NaCN. ZnS changes to soluble complex
with carbon.
ZnS + 4NaCN ---- > Na2[Zn(CN)4] + Na^
2ZnS + 3O2 ---- > 2ZnO + 2SO2?
PbS can now be enriched by froth flotation process
ZnO + C ---- > Zn + COT
without affecting ZnS.
2ZnO + C ---- > 2Zn+CO2T
9. (a) Mineral containing silver and lead is melted, and
CO, CO2 and SO2 being gases escape from the reaction molten zinc is added. Ag is distributed between zinc
and thus, entropy change is positive and reaction is and lead layers (zinc being lighter forms upper
thermodynamically favourable. layer). Since, solubility of Ag in molten zinc is higher
If ZnS is directly reduced with carbon, CS2 (liquid) is that in molten lead, hence, most of the silver goes
formed which remain in reaction mixture. Entropy from lead layer to zinc layer. Two layers are
change is less positive and reaction is separated. Ag containing zinc layer is distilled when
thermodynamically less favourable. zinc separates leaving behind silver.
2ZnS + C ---- » 2Zn + CS2 (liquid) (b) Parke’s method.
2. (i) Reaction (a) since gaseous species (CO, SO2) escape 10. (a) Basic oxygen process
from the mixture (b) (i) pig iron (ii) cast iron (iii) wrought iron
(ii) Reaction (a) (c) wrought iron
(iii) Reaction (a) 11. Forward reaction is endothermic so that formation of
3. In the Ellingham diagram, below 2000 K magnesium cementite (Fe3C) is favoured at high temperatures.
curve is below the silicon curve. Above 2000 K, the When steel containing Fe3C is cooled slowly, equilibrium
silicon curve lies lower than the magnesium curve. is displaced in left hand side and the carbon separates. If
Thus, magnesium can reduce silicon dioxide to silicon the steel is cooled rapidly, equilibrium is not attained
below 2000 K. and the carbon remains in the form of Fe3C.
SiO2 + 2Mg ---- > Si + 2MgO
Analytical Questions
Silicon can reduce magnesium oxide above 2000 K
1. (a) AG° = AG^ (products) - AG^ (reactants)
2MgO + Si ---- > SiO2 + 2Mg
(b) MnW04 (I) AG° (element) = 0
4. (a) FeWO4
AG° = AG° (CS2) - 2AG° (CuS)
5. A : SnSO4 in dil. HgSO4 = 63.6-2(-49)
B : Blocks of impure tin = 161.6 kJ mol-1
C : Sheet of pure tin (II) AG° =2703.6 kJ mol'1
6. (a) Silver is present as (b) If AG° = - ve then spontaneous (->)
(i) soluble [Ag(S2O3)2] = + ve then non-spontaneous (<-)
(ii) insoluble AgBr (III) AG/=-757.2 kJ mol'1
H9O (c) Formation of SO2 is more spontaneous
(b) [Ag2(S2O3)]3“ —-—> .AgjS
(d) AG = AH - TAS
Ag2S + Na2CO3 ---- > Ag2CO3 + Na^
If T is low, AS is also low such that TAS < AH making AG
Ag2CO3 2Ag + C02 + io2
Lt
positive hence, non-spontaneous.
(c) AgBrandAgjS At higher temperature, reactions are spontaneous.
Chapter 17: Metallurgical Extraction | 781

2. 3CaO + 2A1---- > A12O3 + 3Ca (c) 2Au(s)+ 4CN‘(ag) + H2O(/) 2[Au(CN)2]-(ag)
AG° = AG}(A12O3) - 3AG°z(CaO) + 2OH“(aq)
=-1582+ 1812.6 (c) is used to convert mineral to a free metal
=+ve 10. (a) Cu(OH)2-2CuCO3s 3Cu
hence non-spontaneous.
Student was wrong. 344.5 g= 3x 63.5 g Cu
3. Au dissolves in aqua-regia (1 part cone. HNO3 and 3 x 63.5 x 0.033 g Cu =
0.033 g =
3 parts cone. HC1) forming soluble HAuC14, but Ag forms 344.5
insoluble AgCl. (b) CuFeS2 = Cu
air, Na9CO A 183.5 g= 63.5 g
4. FeCr2O, Na2ZCrO44 Na2Cr2O7 nn.or 63.5x0.0495
0.0495 g =---------------- gCu = 0.0171 g
heat NH4C1 183.5
Cr Al Cr2O3 <- (NH4)2Cr2O7 <----------------------------- ■---------------

A (a) gives greater quantity of ore


5. The impure lead metal serves as the anode and pure 11. Sulphide is = MS (oxidation number of metal ion being*
lead serves as the cathode. Impurities either remain at +2)
the anode or go into solution. i/n . 3
MS + - 0,2 ---- > MO + SO.2
Anode : Pb(s)---- > Pb2+ (aq) + 2e~ 2
Cathode : PbSiF6(aq) + 2e~ ---- > Pb(s) + SiF/-(ag) (M + 32) (M + 16)
6. (a) Fe2O3 + 311, ---- > 2Fe + 3H2O 1.164 0.972
Af+ 32 M + 16
(b) Na[Al(OH)J + HC1---- > NaCl + A1(OH)3 + H2O 1.164" 0.972
but strong acid like HC1 will further dissolve A1(OH)3 M =64
which is amphoteric. Aqueous solution of NH4C1 is
12. W = zit, Al'3* +3e"---- > Al
slightly acidic and this acidic solution will cause
precipitation of A1(OH)3 from Na[Al(0H)4 ] where, z = electrochemical equivalent
nh; + 2h2o nh4oh + h3o+ atomic mass 27
[A1(OH)4]- + H3O+ A1(OH)3 X + 2H2O electrons x 96500 3 x 96500
(c) AgCl + Na2CO3 -A+ Ag2CO3 Ag2O + CO2 i = current (A) = 94.0 A
t = time (seconds) = 1 h = 3600 s
9x94x3600
1
2Ag+|O2 w =----------------
96500
= 31.56 g
7. Since, E° <0 for Zn2+; the reduction of Zn2+ is not
favoured. Since, E° > 0 for Hg2+, the reduction ofHg2‘r is 13. Mg2++Ca(OH)2 Mg(OH)2 + Ca2*
favoured. The roasting of CdS should give CdO since 24 g 74 g
£°<0forCd2+.
1272 g ?
8. Since, N2 can also combine with Mg giving magnesium 24 g of Mg2* is precipitated by = 74 g Ca(OH)2
nitride, and inert atmosphere no longer remains 74 x1272
.*. 1272 g of Mg2* is precipitated by =------------ g
3Mg + N2 ---- > Mg3N2 24
9. (a) = 3922 g
Fe3O4(s) + 4CO(g)+ i O2(g)---- > 3Fe(Z) + 4CO2(g) = 3.922 kg

(b) balanced
782 | Essential Inorganic Chemistry

Exercise 2
Only One Option Correct
1. (b) 2. (b) 3. (c) 4. (b) 5. (c) 6. (c) 7. (b) 8. (a) 9. (c) 10. (b)
11. (c) 12. (a) 13. (a) 14. (a) 15. (b) 16. (a) 17. (b) 18. (d) 19. (a) 20. (d)
21. (b) 22. (b) 23. (d) 24. (c) 25. (d) 26. (c) 27. (b) 28. (d) 29. (c) 30. (a)
31. (c) 32. (d) 33. (b) 34. (d) 35. (c) 36. (d) 37. (c) 38. (d) 39. (b) 40. (c)
41. (a) 42. (b) 43. (a) 44. (c) 45. (b) 46. (c) 47. (b) 48. (b) 49. (c) 50. (b)
51. (a) 52. (d) 53. (b) 54. (d) 55. (a) 56. (d) 57. (c) 58. (d) 59. (a) . 60. (d)
61. (c) 62. (d) 63. (d) 64. (d) 65. (b) 66. (d) 67. (c) 68. (b) 69. (a) 70. (a)
71. (a) 72. (b) 73. (a) 74. (b) 75. (a) 76. (b) 77. (d) 78. (a) 79. (a) 80. (c)
81. (c) 82. (b) 83. (d) 84. (c) 85. (c) 86. (a) 87. (a) 88. (b) 89. (d) 90. (c)
91. (d) 92. (c) 93. (d) 94. (d) 95. (a) 96. (a) 97. (b) 98. (c) 99. (d) 100. (b)
101. (b) 102. (c) 103. (c) 104. (a) 105. (b) 106. (c) 107. (c) 108. (a) 109. (d) 110. (b)
111. (c) 112. (c) 113. (d) 114. (a) 115. (b) 116. (b) 117. (c) 118. (b) 119. (a) 120. (c)
121. (a) 122. (c) 123. (b) 124. (b) 125. (d) 126. (d) 127. (c) 128. (b) 129. (a) 130. (c)
131. (a) 132. (c) 133. (b) 134. (b) 135. (c) 136. (d) 137. (d) 138. (b) 139. (d) 140. (d)
141. (a) 142. (b) 143. (b) 144. (a) 145. (a) 146. (c) 147. (b) 148. (b) 149. (c) 150. (b)
151. (a) 152. (d) 153. (d) 154. (c) 155. (d) 156. (a) 157. (b) 158. (c) 159. (d) 160. (d)
161. (b) 162. (b) 163. (d) 164. (a) 165. (a) 166. (c) 167. (a) 168. (c) 169. (b) 170. (c)
171. (b) 172. (c) 173. (d) 174. (c) 175. (a) 176. (b) 177. (d) 178. (d) 179. (a) 180. (c)
181. (c) 182. (c) 183. (b) 184. (b) 185. (d)

One or More Than One Options Correct More negative value of AG° for reaction (II) indicates
1. (a,b,c,d) 2. (c) 3. (a,c,d) 4. (a,b) 5. (d) that H2 is better reducing agent.

10. (a,c,d) 3. CuFeS2 + 2H2SO4 ---- > CuSO4 +FeSO4 +2H2S


6. (c) 7. (a,b,c) 8. (d) 9. (a)
(a) Fe2+ — MnO4
Brain Twisters 10 mL of Fe2+ = 10 mL of 0.02 M MnO;
1. Fe + Cu2+ ---- > Fe2+ + Cu = 10 mL of 0.1 N MnO4
£'° = ^Fe/Fe22+* + Fe 2+ =0.1 N
E\ 2+/Cu
+ ^Cu
= 5.6 g L"1 in 10 gL-1 mineral
= 0.44+ 0.34 = 0.78 V
= 56% Fe
This reaction (a) is spontaneous
(b) 2CuSO4+4KI ---- > Cu^J^K^ +I2
2Ag + Cu2+ ---- > 2Ag+ + Cu
I2 = Na2S2O3
jE° = E .. + + -E 2+
E° =
Ag/Ag+ Cu2 /Cu Cu2+=Na2S2O3
= -0.80 + 0.34 10 mL of Cu2+ = 10 mL of 0.02 M Na^Og
=-0.46 V = 10 mL of 0.02 N Na^Og
This reaction (b) is not spontaneous. Cu 2+ =0.02N
2. CuO + C ---- > Cu + CO = 1.27 gL-1 in 10 gL-1 mineral
AG° = AG/(CO) - AGf°(CuO) = 12.7% Cu
= -137.2+129.7 4. Net reaction in Hall-Heroult process is
= -7.5 kJ 3C + 2ALO,
£ O
---- > 4A1 + 3CO.*2
CuO + H2 ---- > Cu + H2O Or 4A13+ + 12e" ---- > 4A1,
AG° = AG/(H2O) - AG/(CuO) Number of electrons (n) = 12
0(
= -237.2 + 129.7 AG° = 3 AG/(CO2) - 2AG;(A12O3) •
= -107.5 kJ = -3 x394-2(-1520)
=1858 kJ
Chapter 17: Metallurgical Extraction | 783

——lO
AG° = -nFE',
------ 'cell This gives x = 10 g
-AG° 1858 x1000 CaCO3 = 10 g = 0.1 mol
“u nF 12x96500 MgCO3 = 18.4 -10 = 8.4 g = 0.1 mol
= 1.60 V Total moles ofCaCO3 and MgCO3 =0.2 mol
For the reaction Al90, ----- > 2A1 + 3O, CaCO3 = 50 mol%

AG° = 1520 kJ MgCO3 = 50 mol%

2A13+ + 6e“ - -> 2A1, n=6


Passage Comprehension Questions
1520 x1000
Passage 1
6x96500
1. (b) 2. (c) 3. (a) 4. (c) 5. (d)
= 2.62 V
Passage 2
Thus, Hall-Heroult process takes place at lower 100% 98.2%
1. (a) ZnS = 90.6% = ZnO ZnSO4 Zn
voltage.
1 mol ZnS gives 1 mol Zn but based on % yield, 1 mol
5. (a) FeCr2O4(s) + 4C(s) ----- > Fe(s) + 2Cr(s) + 4CO(g) ZnS gives = 0.906 x 1.0 x 0.982
1 mol 2 mo1 4 mol
224 g 104 g 112 g = 0.8897 mol Zn
225
225 kg ZnS = kmol ZnS = 2.32 kmol ZnS
224 kg of chromite gives = 104 kg of Cr
104x236 = 2.06 kilomol Zn
Thus, 236 kg of chromite gives = kg of Cr
224 = 134 kg Zn
= 109.57 kg Cr 2. (d) w = zit
(b) 224 kg chromite gives = 112 kg CO . w
i-—
112x236 zt
Thus, 236 kg chromite gives = kg CO 134x 103 g
224
65
= 118 kg CO Ux 96500 (30x24x60x60)
= 4214.2857 mol
= 154 A
740
Using equation pV = nRT; T = 298 K, p = atm where, z = electrochemical equivalent
_ f atomic mass
V^nRT 4214.2857x0.0821x298 lelectronsx 96500 J
P (740/760)
= time in seconds
= 105.892 xl03L CO
3. (b)
6. (1) Copper (2) Platinum (3) Silver (4) Iron 4. (c) ZnO + 2NaOH Na2ZnO2 + HjO
(5) Barium (6) Zinc (7) Lithium (8) Magnesium
5. (a) 98% by weight HjSO^ (d = 1.8 g/rnL) is 18 M
7. LetCaCO,=xg
100 g solution has = 98 g
then, MgCO3 = (18.4 - x) g
mL HjSC^ solution has = 98 g ELjSO*
CaCO,□ ----- > CaO + CO.'2 1.8
1 mol 1 mol 1 mol mol (98/98)
100 g 56 g 44g molarity =
litre 100
xg 44r
1.8 x 1000
100 g
MgCOg MgO + CO2 = 18M
1 mol 1 mol 1 mol ZnO = 2.32 kilomol in step (i) assuming 100% yield
84 g 40 g 44 g
= 2.32 x 103 mol
(18.4 - x) g — (18.4-x)g
84 18 mol H2SO4 are in = 1L
44x 44(18.4-x) 2.32 x 103 mol are in = 129 mol
Total CO2 100 + = 8.8 (given)
84 »130
784 | Essential Inorganic Chemistry

Passage 3 True & False


1. (b) 2. (d) 1. F 2. T 3. T 4. F 5. T 6. F
Passage 4
7. T 8. T 9. F 10. F 11. F 12. T
1. (a) 2. (b) 3. (a) 17. F 18. T
13. F 14. T 15. T 16. F
Passage 5 19. T 20. F
1. (b) 2. (a) 3. (c) 4. (d)
Passage 6 Fill in the Blanks
1. (d) 2. (b) 3. (c) 4. (a) 5. (b) 1. Cr2O3, N2 2. Al 3. [Ag(CN)2r 4. higher
Passage 7 5. Wrought 6. Pig 7. oxygen 8. aluminium
1. (a) 2. (c) 9. Frasch process 10. van Arkel-de-Boer process
3. (b) w = zit Cu 2+ + 2e~ ---- > Cu 11. Kroll process 12. copper 13. magnesium
63 5 14. A1N,A1(OH)3 15. Hall-Heroult process, lower, increase
Equivalent weight = 18. haemoglobin
16. silver 17. methanides, methane
19. ferrochrome 20. fool's gold 21.75% 22. steel
where z = electrochemical equivalent
equivalent weight 23. pig, wrought 24. steel 25. steel 26. nickel
" 96500 27. chalcocite 28. CuCO3 Cu(OH)2 29. Parke’s
i = current (A) 30. an insecticide
t = time (s)
63.5 x50 x t Matrix-Match Type Questions
65.6 =
2x96500 I. Only One Correct
2x96500x65.6 1. (AM4); (BM(1); (CM2); (D>—(3)
t =---------------------- s
63.5x50 2. (AM); (B)—<4); (CM3); (DMD
= 1.1 h 3. (AM4); (BM(1); (CM2); (DM3)
Passage 8 4. (AM3); (B)—(1); (CM2); (DM4)
1. (c) 2. (a) 3. (d) 4. (b,c) 5. (a) 5. (AM3); (BM(4); (CM2); ■ (DMD
Passage 9 6. (AM3); (BM(5); (CHI); (DM2)
(EM4)
1. (a)
II. One or More Correct
2. (c) ZnO Zn+-O2 AG** = x
2 2 1. (AMI,6); (BM); (CM(1,2,3,6);
C+-O, CO AG°2 = -x (DM1,5) (EH4.5)
2 2 2. (AM(3,4,6,7); (BM(1,2,4); (CM(4,5,7);
ZnO + C---- > Zn + CO, ^g°3=q (DM2,5) (EM(1,6)
System is at equilibrium;
AG° = - AG2° since zinc and carbon have equal affinity Integer Answer Types
for oxygen. Set I
3. (b) This will be above a temperature when two curves
intersect.
Questions—► 1 2 3 4 5 £ 7 8
Answers —► 2 7 2 6 3 4 3 1
4. (d) AG° = most negative.
© © © © © ©I© ©
Passage 10 0 © 0 O 0 G)
1. (a) !2. (c) 3. (a,b) 4. (d) 5. (a,c) © © © © © ©© ©
© © © © © ©
6. (c)
© © ©
Passage 11 © © © © © © © ©
1. (c), AG° being more negative 2. (d) 3. (a,c) © © © © © © © ©
0 0 0 0 0 0 _® 0
Assertion & Reason © © © ©. © ©_© ©
1. (b) 2. (a) 3. (e) 4. (b) 5. (a) 6. (b) © © © © © © © ©
Chapter 17 : Metallurgical Extraction | 785

Set II E: Sulphide F: Uncombined form


Questions 1 2 3 4 5 6_ 7 8
Answers 4 2 7 6 9 9 2 9
C
© © © © © ©I© ©, B D A
0 •® CD CD CD O
© © © © © © F E
© © © © © ©
CD (4)
© © © © © ©
© © © © © ©
0 0 0 0 0 © 2. A (Mg): Electrolysis of molten MgCl2
® © © © ® ® _® © B (Al) : Electrolytic reduction of(Al2O3 + Na3AlF6)
© © © © © ® © © C (Au): Cyanide reduction process
2 (Au(CN)2r + Zn--- > [Zn(CN)4]2" + 2Au X
Test Yourself on the Periodic Table D (Cu): Bessemerisation followed by electrolysis of
aq. CuSO4 solution using impure Cu as anode
1. Minerals are
E (Sn): Reduction of SnO2 by charcoal
A: Sulphide B: Carbonate C: Chloride D: Oxide

I
!
1

Coordination Compounds
"Even when, to judge by the valence number, the combining power of certain atoms
is exhausted, they still possess in most cases, the power of participating further in
the construction of complex molecules with the formation of very definite linkages.
The possibility of this action is to be traced back to the fact that, besides the affinity
bonds designated as principal valences, still other bonds called auxiliary valences,
may be called into action." Alfred Werner

Quick Points I. UH
• For the stepwise formation of ML^ from M and L
M+L ML *1

ML + L ML, K2 Double Salts and Coordination


M + 2L ML, k3 Compounds
■ Ligands
K2 = kxk2
■ Coordination Number
log K3 = log Kx + log K2 • Nomenclature
P^-3 = P^l + P^-2 ■ Effective Atomic Number
• 02 is called superoxide ion. ■ Isomerism in Complexes
• Anions outside coordination sphere ionise, greater the
• Werner Theory of Coordination
number of ions larger the conductance
Compounds
[Co(H2O)6]C13 [Co(H2O)e]3+ + 3cr
■ Bonding in Complexes
• van’t Hoff factor i
■ Stability Constants and Stability of
i = [1 + (y -1) x]
■ Coordination Compounds
where, y = number of ions per unit of the complex/species
■ Applications of Coordination Chemistry
x = degree of ionisation
i = (1 + 3x) in the above case since, y = 4
• Electron-pair donor is a Lewis base and electron-pair
acceptor is a Lewis acid.
788 | Essential Inorganic Chemistry
i

K4[Fe(CN)6] K+ is other ion


Double Salts and / ,
Coordination Compounds complex anion
• When Fe(CN)2 solution is treated with KCN solution, [Pt(en)3]Cl4 Cl- is other ion
species formed, no longer, gives tests of complex cation
Fe2+andCN".
[Pt(en)2C12]
• When K2SO4 solution is treated with A12(SO4)3 neutral complex
solution, species formed gives tests of K+, Al3+ and Many important biological substances are
sot. coordination compounds. Haemoglobin and chlorophyll
Fe(CN)2 +4KCN *Fe(CN)2 -4KCN are two such examples. Haemoglobin is a protein that
(A)
carries 02 in blood. It contains iron (II) ions bound to large
K2SO4 + A12(SO4)3 + 24H,O---- > porphyrin rings. Chlorophyll is necessary for
K2SO4A12(SO4)3-24H2O photosynthesis in plants. It contains magnesium ions
(B) bound to porphyrin rings. Vitamin B-12 is a large complex
We call them addition compounds of of cobalt.
Type I : those which lose their identity in solution
(double salt) as alum (B).
Type II: those which retain their identity in solution
Ligands
(complexes) as potassium ferrocyanide (A). A ligand is a species that is capable of donating an
Most d-transition metal ions have vacant d-orbitals electron pair(s) to a central metal or metal ion. It is a
that can accommodate electron pairs. Many act as Lewis Lewis base. In accepting electron pairs, the central ion
acids by forming coordinate covalent bonds in acts as a Lewis acid.
coordination compounds (coordination complexes • Ligand is said to be unidentate if it has only one pair
or complex ions).
of electrons that it can donate (Table 18.1).
A complex ion is a metal ion with Lewis bases (ligand)
attached to it through coordinate covalent bonds (also • Ligand is said to be bidentate, tridentate,
called coordination). A complex (or coordination tetradentate etc. depending on the number of
compound) is a compound consisting either of complex electron-pairs that it can donate. (Table 18.2).
ions with other ions of opposite charge.

Table 18.1 Some Common Unidentate Ligands


Neutral Anions (replacing ide by o) Anions (replacing e by o)

Formula Name as ligand Formula Name as ligand Formula Name as ligand

qh2 aqua F" (fluoride) fluoro SOf sulphate

nh3 ammine cr chloro s2oi- thiosulphate

co carbonyl Br" bromo CQ2- carbonate

NO nitrosyl r iodo no2 nitro*

nh2ch3 methylamine 0 (oxide) oxo ONO" nitrito*

NC5H5 or Py pyridine 0 H" (hydroxide) hydroxo SCN" thiocyanate’


N(C2H5)3 triethylamine CN" (cyanide) cyano* NOS' isothiocyanate*
N(CH3)3 trimethylamine NC isocyano*
P(C6H5)3 triphenyl phosphine S2" sulphido

ph3 phosphine H" hydrido

* underlined atoms are electron-donating, ambidentate.


Chapter 18: Coordination Compounds | 789

Table 18.2 Some Common Multidentate Ligands (Chelating Agents)


Name Formula Charge
Abbreviation Multi
ethylenediamine ch2nh2 0
en bidentate
ch2nh2

oxalate ion COO- -2


ox bidentate
COO-
glycinate ion ch2nh2
9’y bidentate
COQ-
CH3—C=NO'
DMG bidentate dimethylglyoximate ion
I
CH3 —C = NOH

diethylene-triamine | H2N(CH2)2-NH-(CH2)2NH2 0
dien tridentate
EDTA hexadentate ethylenediamine tetraacetate ion
f-OOCCH2 CH2COO‘
n-ch2—ch2-n
-OOCCH2- ch2coo*

la) INH3, one electron-pair available for DEEP Focus


donation-unidentate. These are called multidentate ligands (Table 18.2). When
the bonding of a multidentate ligand to a metal ion produces a
CHgNHa ring (usually five- or six-membered), we refer to the complex as
(b) I , two electron-pairs available for a chelate (pronounced-keylate). The multidentate ligand is
CRjNHa called a chelating agent and the process of chelate formation
is called chelation. The number of such ligating groups
donation- bidentate. indicate the denticity (unidentate, bidentate ) of the
• Some ligands can be coordinated to the metal or metal ligand.
ion through either of the sides. They are called 0- H— 0
“ambidentate” ligands. t I
If the nitrite ion is attached through N atom (—N02), CH3—C=N /N— C—CH3
it is nitro; if attached through 0 atom (—ONO), then Ni
CH3—C=N/’ XN=C—CH3
nitrito.
• Ligands can be 0—H---- 0
— neutral molecules such as HgO nickel (II) dimethyl glyoximate (chelate)
— anions such as CN", C^' fbis-(dimethylglyoximato) nickel (II)]
— cations [but in rare cases since an electron pair on a
cation is held firmly by the positive charge and CH2NH2\ 0—c=o
I >Cu
would not be involved in coordinate bonding.]
o=c—xNH2CH2
• Ligands forming complex with metal do not ionise.
copper (II) glycinate (chelate)
• In some cases it is not possible to identify donor atoms, [trans-bisglycinato copper (II)]
because the bonding electrons are not localised on
specific atoms. Ethylene (CIL^CHj) bonds to
transition metal through the electrons in their double
bonds. Coordination Number
The coordination number (C.N.) of a metal atom in a
Chelates complex is the total number of bonds the metal atom forms
Some ligands are capable of donating more than a with ligands. Some common coordination numbers of
single electron pair, from different atoms in the ligand and metal ions are summarised in Table 18.3.
io different sites in the geometric structure of a complex.
790 | Essential Inorganic Chemistry

Table 18.3 Coordination Number (C.N.) of Metal Ions ‘ide’ are changed to ‘o’
Metal ion C.N. ‘ite’ are changed to ‘ito’
‘ate’ are changed to ‘ato’ (Table 18.1 and 18.2)
Ag+ 2
(e) Many ligands that are molecules carry th
Cu* 2,4
unmodified name (Table 18.1 and 18.2)
Cu2* 4,6 (f) Positive groups end in —ium
Au* 2,6 NHj —NH® hydrazinium.
Ca2* 6
Fe2*, Fe3* 6
DEEP Focus
Co2* 4.6
(g) When there are several ligands of the same kind, we
Co3* 6 normally use the prefixes di, tri, tetra, penta and
Ni2* 4,6 hexa to show the number of ligands of that type. An
Zn2* 4 exception occurs when the name of the ligands includes
Al3* 4,6 a number, for example as in ethylenediamine (en). To
avoid confusion in such cases, bis, tris and tetrakis are
Sc3* 6 used instead of di, tri, and tetra, and the name of the
Cr3* 6 ligand is placed in brackets.
Pt2* 4 bis (ethylenediamine)
Pt4* 6
(h) If anion is a complex, then metal ends with ‘ate’
Examples of complexes of various C.N. are given in [Ni(CN)4f-
Table 18.4. tetracyanonickelate (II) ion
lead — plumbate
Table 18.4 Examples of Complexes of Various Coordination gold — aurate
Numbers zinc — zincate
Complex C.N. tin — stannate
silver — argentate
[Ag(NH3)2]* 2
cobalt — cobaltate
[Hgi3r 3 iron — ferrate
PtCI2', Ni(CO)4 4 aluminium — aluminate
Fe(CO)5, [Co(CN)5]3' 5 manganese — manganate
[Co(NH3)6]3+, W(CO)6 6 copper — cuprate
[Mo(CN)7]3_ 7 chromium — chromate
[W(CN)8]4- 8
platinum — platinate
(i) If the complex contains two or more metal atoms, it
is termed polynuclear. The bridging ligands which
Nomenclature link the two metal atoms together are indicated by
the prefix p —.
The definite source for the naming of inorganic
compounds is Nomenclature of Inorganic (j) Ambidentate ligands may be attached through
Compounds 2005 (N.G. Connelly and T. Damhus, Sr. different atoms
Eds. Royal Society of Chemistry 2005) M—NO2 (NO2 joined to metal M through
Following rules are adopted for naming a complex ion; N; it is nitro)
M—ONO (NO 2 joined to metal M through
(a) Cations are named before anions
(b) Oxidation state (O.S.) of the central metal ion is O; it is nitrito)
denoted by Roman numeral.
Similarly, the SCN group may bond M—SCN
Cation O.S. anion (thiocyanato) or M—NCS (isothiocyanate). They may be
CuCI Copper (I) chloride
named
thiocyanato-S [to indicate S coordinated to
CuCI2 Copper (II) chloride
metal M ]
FeCI2 Iron (II) chloride thiocyanato-N [to indicate N coordinated to
FeCI3 Iron (III) chloride metal M ]
(k) When writing (not naming) the formula of the
(c) The names of ligands are given first followed by the
complex:
name of the central metal ion.
(d) The names of ligands that are anions and ending • complex ion should be enclosed by square
with brackets and
Chapter 18: Coordination Compounds | 791

• ligands are placed after metal in the (0 K4[Fe(CN)6]


alphabetical order but first negative ligands, Potassium hexacyanoferrate (II)
then neutral, then positive. (d) Na3[Ag(S2O3)21
[P«H20)4(NH3)21u — wrong Sodium dithiosulphatoargenate (I)
aqua ammine (e) Na2[OsCl5N]
[Pt(NH3)2(H2O)4]4+ — right Sodium pentachloronitridoosmate (VI)
[Cr(CN)4(NH3)2r — right (f) KJPtClJ
Potassium hexachloroplatinate (IV)
negative neutral
(g) " K2[Cr(CN)2(O2)2(O2)NH3]
Examples Potassium amminedicyanodioxoperoxo chromate (VI)
(1) metal ligands (h) [Zn(NCS)4f-
Tetrathiocyanato-N-zincate (II) ion
[Fe(CN)6] (N indicates that ligand joined to metal ion
' V

cation anion through N-side)


potassium hexacyanoferrate (ID 3. In the following examples, organic ligands have
cation anion been used:
• anion is a complex hence its metal ends with (a) [Pt(Py)2][PtCl4]
ate (ferrate), Tetrapyridineplatinum (II)
• ligands are named before metal ion of the tetrachloroplatinate (II)
complex.
(b) [Cr(en)3]Cl3
(2) [Co(NH3)6]Cl3 d-or-l tris (ethylenediamine) chromium (III) chloride
cation anion [Pt(en)2f*
(c)
hexaamminecobalt (III) chloride Bis (ethylenediamine) Platinum (II) ion
• cation is a complex [Cr(C6H6)2]
(d)
• anion will not change its name (since it is not Bis (benzene) chromium (0)
the ligand)
(3) (e) [Fe^H^l
[Co(en)3]Cl3 Bis (cyclopentadienyl) iron (II)
tris (ethylenediamine)cobalt (III) chloride
• tris is used to indicate three ligands en 4. In the following examples, bridging groups have
(ethylenediamine) since it also carries di. been used:
(4) [(en)2Co<^
[PtCl2(NH3)4]Cl2
(a) Co(en)2
tetraamminedichloroplatinum (IV) chloride
• NH3 (ammine) and Cl (chloro) are two types of Tetrakis(ethylenediamine)-p- hydroxo-
ligands named in alphabetical order: p-imidodicobalt (III) ion.
(5) [(NH3)5 Co-NH, —Co(NH3)5 xno3)5 (b) [(CO)3Fe(CO)3Fe(CO)3]
bridging ligand will start with p Tri-p-carbonyl-bis (tricarbonyliron (0))
p -amido bis [pentaamminecobalt (III)] nitrate (C6H5)3Px /C1V'- ZC1
1. In the following examples, complex ion exists as (c) Pd •Pd
cation: Cl Cl P(C6H5)3
(a) [Fe(NH3)6]Cl3 (.... line used to indicate symmetry)
Hexaammineiron (III) chloride
7>ans-bis(chlorotriphenylphosphine-p-chloropalladium(I))
(b) [CoCl(NH3)5f* or
Pentaamminechlorocobalt (III) ion Chlorotriphenylphosphine palladium (II)
(c) [Co(SO4(NH3)4)]NO3 p-dichlorochlorotriphenylphosphine palladium (II)
Tetraamminesulphatocobalt (HI) nitrate
(d) [Co(Cl)(NO2XNH3)4r
Tetraamminechloronitrocobalt (III) ion Effective Atomic Number
2. In the following examples, complex ion exists as Each ligand donates an electron pair to the metal
anion ion, thus forming a coordinate bond. Transition
(a) [ZnCl4f-
metals form coordination compounds very readily because
Tetrachlorozincate (II) ion
they have vacant d-orbitals which can accommodate these
(b) [aih4]- electron pairs. Metal ion in the complex tends to attain
Tetrahydridoaluminate (III) ion
792 | Essential Inorganic Chemistry

nearest stable inert gas configuration by gaining electrons (where O.N. is oxidation number, C.N. the
from ligands. Effective atomic number (EAN) of the coordination number and Z the atomic number).
metal in a complex is given by EAN can also be given by
EAN =Z - (O.N.) + 2(C.N.) EAN =Z - (O.N.) + 2(Ligands)
assuming that each ligand is unidentate.
EAN of some metals in complexes is summarised in Table 18.5.
Table 18.5 Effective Atomic Numbers of Some Metal in Complexes
Atomic ~m Electrons lost In ion Electrons gained by EAN
Atom number (Z) Complex C.N. formation (O.N.) coordination
Cr 24 [Cr(CO)6] 6 0 12 36
Ni 28 Ni(CO)4 4 0 8 36
36
Fe 26 [Fe(CN)6]4- 6 2 12 [Kr]
36
Fe 26 [Fe(NH3)6]2* 6 2 12
36
Cu 29 [Cu(CN)4]3" 4 1 8
36
Co 27 [Co(en)3]3+,(en) is bidentate 6 3 12
Pd 46 [Pd(H2O)6]4+ 6 4 12 54[Xe]
Pt 78 [PtCI6]2- 6 4 12 86[Rn]
Fe 26 [Fe(CN)6]3" 6 3 12 35
Ag 47 (Ag(CN)2]~ 2 1 4 50
Al 13 [AI(C2O4)3]3”. ox is bidentate. 6 3 12 22

18-Electrons System
If there are total of 18-electrons attained due to gain of (i) [Co(NH3)6(SO4)]Br [Co(NH3)5Br]S04
red violet
coordination electrons of ligands including (n - l)dxnsy
electrons of metal/metal ion, the complex is said to attain Pentaamminesulphatocobalt Pentaammine bromo cobalt
(III) bromide (III) sulphate
18-electrons configuration.
• SO^“ inside coordination • Br" inside coordination
In[Fe(CN)6]4" sphere and Br“ outside it sphere and S04’ outside
Fe: [Ar] 3d6 4s2 Electron gained from6CN” = 12 it,
Fe2+:[Ar]3d6 • gives yellow ppt. with Ag* • gives white ppt. with Ba**
ionisation being as ionisation being as
Thus, 3d6 + 12e“ = 18e~
[Co(NHj)6SO4]++ Br" [Co(NH,)sBrf+SO^
outside coordination sphere
Isomerism in Complexes [Pt(NH3)4 Cl2]tr2
[Pt(NHJ),Br.|Cl,
(ii)
Compounds that have the same chemical formula but
different structural arrangement are called isomers. t__ _______ f
inside coordination sphere
A. Structural Isomerism Other examples are:
Structural isomers are isomers that differ in how the [Pt(NH3)4SO4](OH)2 and [Pt(NH3)4(OH)2]S04
atoms are joined together—that is, in the order in which [Co(NH3)5NO2]SO4 and [Co(NH3)5S04]N02
the atoms are bonded to one another.
[Cr(NH3)5SO4]Br and [Cr(NH3)5Br]SO4
(H—N=C=O and N=C—0—H are structural
isomers)
These are of the following types: (b) Coordination Isomerism
When both the positive and negative ions are complex
(a) Ionisation Isomerism ions, isomerism may be caused by the interchange of
This type of isomerism occurs when there is an ligands between the anion and cation. This arises only if
interchange of groups between the coordination sphere of cationic and anionic metals have same C.N. and charge.
the metal ion and the ions outside this sphere. Examples Examples are:
are:
Chapter 18: Coordination Compounds | 793

same charge Illustration 1 One mole of the complexCrC\3 SHgO on


G) [Co(NH3)6]3+ [Cr(CN)6]3- reaction with excess of aq. AgNO3 gives two moles of white
same C.N. = 6 precipitate of AgCL What is the complex?
Hexaamminecobalt (III) hexacyanochromate (III)
Solution CrClg bHjO +AgNO3---- > AgCl
[Cr(NH3)6]3* [Co(CN)6]3- 1 mol excess 2 mol
Hexamminechromium (HI) hexacyanocobaltate (III) This reaction indicates that two Cl are outside the
(ii) [Co(en)3][Cr(CN)6] coordinate sphere and one Cl inside it. Coordination
[Cr(en)3][Co(CN)6] number of six of Cr ion is satisfied by five H20 and one Cl
(iii) [Pt(NH3)4Cl2][PtCl4] as ligands. Thus, this is monohydrate complex
[CrdiaOsClJClj, HoO
[Pt(NH3)4][PtCl6]
[CrdigOsCUC^ [CrGLOgClf+ +2C1-
(c) Linkage Isomerism 1 mol 2 mol

This type of isomerism arises when the ligand may 2d- + 2Ag+ 2AgCU
attach to the central metal ion of a complex in different 2 mol
ways. (Such ligands are called ambidentate ligands).
Nitrite ion has electron pairs available for coordination
both on N and 0 atoms, (refer Table 18.1) (f) Electronic Isomerism
e The complex [Co(NH3)5NO]C12 exists in two forms.
:s• •—s=n: :o^xo:9 One is black paramagnetic while the other is pink and
’ ! /
diamagnetic. Strucutral studies show that the black
isomer is a Co(II) complex containing neutral NO group
bind to metal ion using' whereas the pink one is a Co(III) complex with NO" ligand.
either lone pair This type of isomerism is called electronic isomerism.

Examples:
and [Co(NH3)5NO21C1.2
(g) Ligand Isomerism
(i) [Co(NH3)5ONO]C12
pentaamminenitritocobalt (III) penta amminenitrocobalt Isomerism may also arise due to isomerism in the
(no ligand set. The effect of changing an alkyl chain on a
chloride chloride
(red), decomposes in acids (yellow), stable in acids ligand from a linear to a branched structure is sufficient to
induce quite profound strucutral changes in the complex.
(ii)[Mn(CO)5(SCN)]+ and [Mn(CO)5(NCS)]+
pentacarbonylthiocyanato- pentacarbonylthiocyanato- Consider reaction of A, &zs-(salicylaldehyde) nickel
S-manganese (II) ion N-manganese (II) ion (II) with propyl amine to give B,
his-(N-propylasalicylidene aminato) nickel (II).
(d) Coordination Position Isomerism The use of n-propyl amine or isopropylamine gives two
In polynuclear complexes an interchange of ligands isomers in this reaction. The more stearically hindered
between the different metal nuclei gives rise to position
isopropyl amine results in a complex distorted away from
isomerism.
Example: the square planar geometry displayed by the n-propyl
complex towards a tetrahedral geometry.
(NH3)4Co Co(NH3)2C12 Cl2 H\
>=Q 0
2PtNH2
Ni
C1(NH3)3Co Co(NH3)3C1 Cl2 o'
2

(A)
(e) Hydrate (Solvate) Isomerism
Hydrate isomers are isomers of a complex that differ
Ni
in the placement of water molecules in the complex.
CrCl3- 6^0 have following hydrate isomers:
[Cr(H2O)6 ]C13 — violet (anhydrous)
[CrfHjOJg C1]C12 -HgO — light green (monohydrate) (B)
[CrtfljO^Clg ]C1 • 21^0 — dark green (dihydrate)
794 | Essential Inorganic Chemistry

(h) Polymerisation Isomerism h3n ci


This term is used to describe compounds haivng same Pt
empirical formula but different molecular compositions.
The difference from normal polymerisation is that here we Br nh3
have differences in arrangements of group as trans
multiplication of molecular weight. Thus, value of n in the Mabcd Pt(NH3)(C5H5N)(CI)(Br) H3N NC5H5
empirical formula \MLm ]n varies.
[Pt(NH3)2Cl2]n gives Pt
(A) [Pt(NH3)2 Cl2 ] if n = 1 and
(B) [Pt(NH3)4][PtCl4]if n = 2 Br Cl
and thus (A) and (B) are polymerisation isomers, H3N Cl
[Co(NH3)3(NO2)3] gives following isomers
Pt
Formula Molecular weight
[Co(NH3)6][Co(NO2)6] double Br NC5H5
[Co(NH3)4(NO2)2][Co(NH3)2(NO2)4] double H3N nc5h5
[Co(NH3)5(NO2)][Co(NH3)2(NO2)4]2 triple
[Co(NH3)6][Co(NH3)2(NO2)4]3 quadruple Pt
[Co(NH3)4(NO2)]3[C0(NO2)6] quadruple
Cl Br
[Co(NH3)5(NO2)]3[Co(NO2)6]2 quintuple

(i) [Pt(NH3)2Cl2 ] can exist as two geometric isomers:


B. Stereo Isomers Cl NH3 Cl NH3
These are the isomers in which the atoms are bonded to
each other in the same order but that differ in the precise Pt
arrangement of these atoms in space. These are of the
following types: Cl nh3 h3n Cl
cis trans
(orange yellow) (pale yellow)
(a) Geometric Isomerism
Geometric isomers are isomers in which the atoms are (ii) [Pt(Gly)2]
joined to one another in the same way but that differ CH2— NH* -NHa— CH2
because some atoms occupy different relative positions in Pt
space. G—0 ^0—CO
cis
Geometric isomerism of metals of C.N. 4 CO—0^ -NHa-CHa
For square planar complex Ma4, Ma3b or Mab3 where a •Pt
and b etc., are monodenate ligands, geometrical isomerism CHj—NH^ O-CO
is not possible. However, square planar complexes of the trans
type Mo^dc, Mabcd and M(AA)2, M(AB)2—when
AA and AB represent symmetrical and unsymmetrical Geometric isomerism of metals of C.N. 6
chelating agents to give geometrical isomers. (i) [Co(NH3)4Cl2]+ can exist as
Type Compound Isomers Cl Cl
M^2^2 Pt(NH3)2CI2 cis and trans as in (i) h3n Cl h3n. nh3
NH3 Cl Co Co,
Ma2bc Pt(NH3)2CIBr
h3n< nh3 h3n nh3
nh3 Cl
H3N cis trans
(violet) (green)
Chapter 18: Coordination Compounds | 795

(ii) [Pt(NH3)2Cl2Bi2] can exist as Cl


Cl Cl
Br, Cl h3n> Br
en C?7 en
12
pt; pt: Cl
Br nh3 Br NH3 trans-(no optical isomer possible)

NH3 Cl Mirror
cis trans Cl Cl
There are so many other arrangements also.
-H-7CI : Cl/ T-- A
Another type of geometrical isomerism occurs in
octahedral coordination entities of the type [ACYjY,] like
en. Co3y : I 9° //n
lCo(N02)3(NH3)3]. If each trio of donor atoms occupy
adjacent positions at the comers of an octahedral face, it is en ■ en
called facial (fac-) isomer. When the positions are enantiomorphic pair d and I
around the meridian of the octahedron the isomer is called cis-dichlorobis (ethylenediammine) cobalt (Ill)ion
meridional (mer-) isomer.
J
• Optical isomers of [Co(en)3]3+ are:
Mirror
Y K X
!\ M 2 2

X \ en en
Y X X 5
3 ___en
4
Fac- Mer

Bridged binuclear complexes like [Pt(PEt3)Cl2 may • In [Fe(C2O4)3 is a bidentate ligand and that
exist in three isomers forms three ligands mean that the metal has a
Et3P\ XL /Cl Et3P\ /Ck /PEt3 coordination number of 6. Mirror images of complex
>pt< >pt< >pt< >pt/
[Fe(C2O4)3]3- are not superimposable; therefore two
cr xcr xPEt3 cr xcr xci optical isomers are possible.
trans cis
Mirror
Et3Px /CL ZC1 <0
>Pt< >Pt<
Et3Pz xcr xci
C\ I J3,
>Fev
i; (
T
a^Fe^.
unsymmetrical 0Z | \) I V I X)
As we increase the number of different ligands, the
possible number of isomers increases. Non-superimposable mirror images of [Fe(ox)3]3-

(b) Optical Isomerism Illustration 2 If A, B, C, and D are four different


If a molecule is asymmetric, it can’t be superimposed ligands,
on its mirror image. The two forms have the type of (a) how many geometric isomers will be found for square
symmetry shown by the left and right hands and are called planar [PtABCDf+?
(b) will tetrahedral [ZnABCD]2 + display optical
enantiomorphic pair. The two forms are called optical isomerism?
isomer. They are called either dextro or laevo (d or I). This
Solution (a) Following three different isomers are
depends on the direction they rotate the plane of polarised possible for the square planar [PtABCD]2+.
light in a polarimeter. (You may write others but they will be identical)
• Optical isomerism is common in octahedral complexes
involving bidentate groups. <s> ®. ©
• [Co(en)2Cl2]+ exists as cis- and trans-isomers. But Pt) Pt
cis-isomer can have d and /-optical isomers. R
III
796 | Essential Inorganic Chemistry

(b) There is chirality and enantiomers do not (a) Label the isomers as cis or trans.
superimpose hence, [ZnABCD]2* displays optical (b) Which isomers are identical and which are different.
isomerism. (c) Do any of these isomers exist as enantiomers.

t Cr: •
NH3:O
Cl: •
A J
III IV

Solution (a) I, IV cis; II, III trans


(b) I and IV are the same; II and III are the same
(c) None of the isomers exists as enantiomers because
Illustration 3 Consider the given isomers of their mirror images are identical.
[Cr(NH3)2Cl4T

Target Practice 1
1. Determine the oxidation state of the coordinated metal 5. Suppose the complex ion [Co(NH3)2(H2O)2Cl2J* is
atom in each of the following compounds : synthesised with two ammine ligands cis to each other, the
(a) K[Co(NH3)2(CN)4]
two aqua ligands cis to each other, and two chloro ligands
(b) Os(CO)5 cis to each other. Is this complex optically active.
(c) Na[Co(H2O)3(OH)3] 6. Give the formulae of the following coordination complexes:
2. Write formulas of (a) A Ni2+ ion is bound to two water molecules and two
(a) Sodium tricarbonatocobaltate (III) bidentate oxalate ions
(b) Diamminediaquadichloroplatinum (IV) bromide (b) A Co3+ ion is bound to one Cl" ion, one ammonia
(c) Sodium tetranitratoborate (III) molecule, and two bidentate ethylenediamine
3. How might one distinguish between the following isomers? (en) molecules
(a) [Co(NH,O kBr]SO . and[Co(NH„o LSOJBr 7. Identify the type of isomerism shown by
O 4 D 4
I II (a) [Co(NH3)4Cl2]+
(b) cis and irans-[CoCl2 (en)2]Cl (b) [Pt(NH3)2(CN)2] (square planar)
4. Give examples of the following types of isomerisation. (c) Zn(NH„)„Cl„ (tetrahedral)
(a) Hydrate isomerism u Z Z

(b) Ionisation isomerism (d) [Fe(C2O4)3]3-


(c) Geometrical isomerism (e) [Co(NH3)6(SCN)1,2+
:

Answers
1. (a) +3 (b) zero (c) +2 OH2 oh2
2. (a) Na3[Co(CO3)3] h 3n I Cl ci I nh3
(b) [Pt(NH3)2(H2O)2CL]Br2 5. Co Co
(c) Na[B(NO3)4J h 3n OH2 : h2o nh3
Cl C1
3. (a) I gives white precipitate with BaCl2(ag) while II gives cis
cis
yellow precipitate with AgNO3(aq).
Mirror
(b) Dipole moment of cis- > trans-
Mirror images are not superimposable hence the complex is
4. (a) [Co(H2O)3Cl3]-3H2O and[Co(H2O)61C13 optically active (being chiral)
(b) [Co(NH3)5BrlSO4 and[Co(NH3)5SO4]Br 6. (a) [Ni(H2O)2(C2O4)2]2“ (b) [Co(NH3)(en)2Cl]2+
(c) MCl2Br2 (square planar) will form cis- and trans- isomers 7. (a) geometrical cis, trans- (b) geometrical cis-, trans-
(c) only single structure is possible (d) optical and
(e) linkages—bonded SCN" N—bonded NOS'
Chapter 18: Coordination Compounds | 797

Practice Exercise 1
1. Classify double salts and coordination compounds out of (e) [A1ui(C2O4)3], (0 [Ag’(S2O3)2]
(a) 3NaF AlF3 which does not give tests ofF- and Al3+ ions
(b) 4KCN ■ Fe(CN)2 which does not give tests ofCN- and Fe.2+ (g) (en)2Fe,n< Fe,n(H2O)
------ 4 4
•NH;
(c) Alum which gives tests of Al3+ and SO2- ions
(d) Mohr’s salt which gives tests of NH4, Fe2+ andSO2’ ions. 9. One mole ofCoCl3 ■ 4NH3 on reaction with excess of AgNO3
2. What is the type of complex formed, cationic or anionic, in gives one mole of white precipitate. What is the complex?
the following? What is the coordination number of cobalt?
Also designate the coordinate ion in each case. 10. Give the oxidation number of the metals in the following
(a)FeSO4 + KCN---- » (b) CoCl2 + KN02 ---- > species:
(0 PtCl4 + KC1---- > (d) AgCl + NH3 ---- > (a) K3[Fe(CN)6] (b) K3[Cr(C2O4)3]
(e) AgBr + Na2S2O3---- > (0 Hgl2 + KI---- > (c) [Ni(CN)4]2- (d) Na,MoO4
3. Following complexes ionise in aqueous solution. What are (e) Fe(CO)5 (0 MgWO4
the ions in each case? 11. Complete the following statements for the complex ion:
(a) KFen[Feni(CN)6] (b) NaA102 [Cr(C2O4)2(H2O)2]-
(c) Ni(CO)4 (d) Na3AlF6 (a) The oxidation number of Cr is
(e) (NH.LPtCL
4 4 O
(f) Na2[Ni(CN)4] (b) The coordination number of Cr is
4. When aqueous KCN is added to a solution ofCuSO4, a white (c) is a bidentate ligand
precipitate, soluble in excess of KCN, is formed. No 12. Name the following compounds:
precipitate is formed when H2S gas is bubbled through the (a) [Co(NH3)4(NO2)2]NO3 (b) (Pt(en)Cl4l
solution at this point. Explain. (c) Na3[Ag(S2O3)2] (d) K_(C
•J
o(ONO)
o
6]
5. (a) Formation of [Ni(en)3]2+ from Ni2f and (e) Na2[Zn(EDTA)] (0 [Ru(en)2Br,]Br
ethylenediamine is a Lewis acid-base reaction. Explain. (g) [Pt(Py)4][PtF4]
(b) Identify the Lewis acid and Lewis base in the reaction of 13. Write the formulae of each of the following compounds :
oxalate ions (C2O2-) with Fe3+ to give [Fe(C2O4 )3]3-. (a) cesium trichlorofluoro iodate (III)
6. Classify the following ligands as unidentate, bidentate (b) tris (ethylenediamine)nickel (II) nitrate
polydentate: (c) potassium dibromodioxalatocobaltate (III)
(a) CH3COO- (b) ;0CN- (d) pentamminechlorochromium (III) sulphate
(0 !NCO- (d) CO: (e) ethylenediaminetetraiodoplatinum (IV)
(f) potassium tetrafluoroargentate (III)
(e) NH2-CH22CH(CH3)-NH2 (g) rubidium ethylenediaminetetraacetatocobaltate (III)
(propylene diamine written as pn), dihydrate
(0 CH3—jj^-CH =C— CH3 14. Calculate EAN of the metal in the following complexes, and
compare it with the atomic number of the nearest rare gas:
:o :o: (a) AuCl" (b) Fe(C2O4)3“
(acetyl acetonato written as acac) (c) Cdl2' (d) [Cr(H,O)4(NH3)2]2+
(e) Co(NOj|- (f) Fe(CO)5
(g)
p N XN^
(2,2'-bipyridine written as bipy) (g) (Ca(EDTA)]2"
(i) [Zn(OH)4]2-
15. Determine EAN of iron in
(h) [Ni(Py)(en)(NH3)3]2>
(j) Hgl2-

(h) Ph—P—Ph (triphenyl phosphine) (i) Fe(CO)5 (ii) Fe(C5H,)2U U 4

Ph 16. Predict value of x in


(a) HxCr(CO)5 (b) HxCo(CO)4
7. Write the formula of 17. Make hydrate isomers of:
(a) a complex ion having Co2+ as the central ion and two
(a) [Fe(Ho0)g]CL (b) [Cu(H2O)4]C12
NH3 molecules and four CP ions as ligands,
(b) a complex ion of manganese (III) having coordination Also arrange them in the increasing conducting power.
number of 6 and ON" as ligands, 18. One cationic complex has two isomers A and B. Each has
(c) a coordination compound comprised of two types of oneCo3*, five NH3, one Br“ and oneSO2- stoichiometrically.
complex ions; one is a complex of Cr(III) with A gives white ppt. with BaCl, while B give yellow ppt. with
ethylenediamine (en), having a coordination number of AgNO3. What are A and B? What is the type of isomerism
6 and the other is a complex of Ni(II) with CN' and shown?
having a coordination number of 5. 19. Two isomers of[Co(NH3)5(NO,)]Cl2 differ in colour. What
8. What are the coordination numbers of the underlined is the cause? If each reacts with excess AgNO3, what is
atoms in the following? Also specify the charge on the precipitated and how much per mole of the complex?
complex, oxidation number of the metal ion has been given: 20. Which of the following can show coordination isomerism?
(a)[Fell(CN)6], (b) [Ni°(CO)4J, (a) [Cu(NH3)4][PtCl4] (b) [Fe(NH3)6]2(Pt(CN)6]3
(0 [Ca11 (EDTA)], (d) [Pt™ (en) Cl.^] (c) [Co(NH3)6][Cr(C204)3] (d) (Pt(en)3](SO4)2
798 | Essential Inorganic Chemistry

21. Write the formula of ionisation isomer of each of the 24. A complex has a composition corresponding to the formula
following compounds: CoBr2Cb 4NH3. What is the structural formula i_
(a) [Ni(en)2(NO2)2]Cl2 (b) [Fe(NH3)5CN]SO4 conductance measurements show two ions per formula
22. Draw the structures of all the isomers of the square-planar unit? Silver nitrate solution gives ppt. of AgCl but no AgBrr
complexes: Write the structural formula of an isomer.
(a) Pd(NH3),BrI (b) Ni(en)Cl2 25. How many geometric isomers are in the following species?
23. For which of the following complexes are optical isomers (a) (Co(NH3)2Cl4r (b) [Co(NH3 )3C13]
possible? Explain your answers and draw structure of 26. Draw structures of all the geometric and optical isomers cf
enantiomers: each of the following cobalt complexes:
(a) [Cr(NH3)4(C2O4)]+ (b) [Cr(NH3)2(C2O4)2]- (a) [Co(NH3)4C12]+ (b) [Co(en)3]i3+
;
(c) [FeCl5F]3- (d) [Co(en)2NH3Cl]2+ 27. Which of the following complexes are chiral?
(e) [Cr(C2O4)3]3- (a) [Pt(en)Cl2] (b) cis-(Co(NH3)4Br2]*
(c) crs-[Cr(en)2(H2O)2]3+ (d) [Cr(C2O4)3]3-

Answers
1. (a)Na3A!F6; (b) K4[Fe(CN)6]; (d) potassium hexanitritocobaltate (III)
(c) K2SO4-A12(SO4)3-24H2O (d)FeSO4-(NH4)2SO4-6H2O (e) sodium ethylenediaminetetraacetatozincate (II)
2. (a) K4[Fe(CN)6] — anionic complex; (f) dibromo bis-(ethylenediamine)mthenium (III) bromide
(b) K3[Co(NO2)6] — anionic complex; (g) tetrapyridineplatinum (II) tetrafluoroplatinate (II)
(c) K,[PtCl6l — anionic complex; 13. (a)Cs[ICl3F] (b) [Ni(en)3](NO3)2 (c) K3[Co(C2O4)2Br2]
— cationic complex; (d) (Cr(NH3)5Cl]SO4 (e) Pt(en)I4 (I) K[AgF4]
(d) [Ag(NH3)2]Cl
(e) Na3[Ag(S2O3)2] — anionic complex; (g) Rb[Co(EDTA)]-2H2O
— anionic complex; 14. (a) Au = 82 (b) Fe = 35 (c) Cd = 54 [Xe] (d) Cr = 34
(f) K2HgI4
(e) Co = 36 [Kr] (f) Fe = 36 [Kr] (g) Ca= 30 (h) Ni = 38
3. (a) K*, Fe2*, [Fe(CN)6]3"; (b) Na*, A1O; (c) no ionisation
(d) Na*, AlFg“; (e) NH*4, PtCl|"; (f) Na'*, [Ni(CN)4]2" (i) Zn = 36[Krj (j)Hg = 86[Rn]
15. (i) EAN of Fe =26 + 10 (from five CO) = 36 = Kr
4. CuSO4+ 2KCN ---- > Cu(CN)2 +K2SO4 (ii) EAN of Fe = 24 + 10 (from two C5Hg ions) = 34
white ppt.
16. (a)H2Cr(CO)4
Cu(CN)2 +KCN---- > K3[Cu(CN)4] (b) HCo(CO)4 (in each case EAN of metal is 36]
stable
17. (a) (I)[Fe(H2O)5Cl]Cl2 H2O (II) [Fe(H20)4Cl2]Cl-2H20
K3[Cu(CN)4] 3K* + [Cu(CN)4]3’
(III) [Fe(H2O)3Cl3]-3H2O
Stable [Cu(CN)4]3- does not ionise to give Cu* hence no Conductance : III < II < I
precipitate with H2S gas. (b) (I)[Cu(H2O)3C1]C1H2O (II) [Cu(H2O)2Cl2]-2H2O
5. (a) Ni2* is electron-pair acceptor (a Lewis acid) and Conductance : II < I
en (ethylene diamine) is electron-pair donor (a Lewis B:[Co(NH3)5SO4]Br
18. A :[Co(NH3)5Br]S04
base). Hence, complex formation is a Lewis acid-base
A and B are ionisation isomers.
reaction.
ch2nh2 19. Difference in colour is due to difference in linkage of N02 that
Ni2* +3I [Ni(en)3]2* can show linkage-isomerism.
Lewis acid ch2nh2 A :[Co(NH3)5(NO2)]Cl2 B :[Co(NH3)5(ONO)]C12
Lewis base One mole of each gives two moles of AgCl (white ppt) with
(b) Fe3+ :Lewis acid; C2O2" :Lewisbase excess AgNO3.
6. (a), (b), (c), (d) and (h) — unidentate; (e), (f), (g) — bidentate 20. (a) and (c).
7. (a) [Co(NH3)4Cl2] (b) [Mn(CN)6]3" 21. (a) [Ni(en)2Cl2](NO2)2 (b) [Fe(NH3)5SO4]CN
(c) [Cr(en)3][Ni(CN)5] IR------ 71Br I nh3
8. (a) (b) (0 (d) (e) (f) (g) 22.(a) Pd '' Pd
C.N. 6 4 6 6 6 2 6
Charge -4 0 -2 0 -3 +4 h3n NH3 H3N Br
cis trans
9. Complex [Co(NH3 )4C12]C1; C.N. of cobalt is six.
(b) Only one isomer is possible as ethylenediamine cannot
10. (a) Fe = +3 (b) Cr=+3 (c) Ni = +2 (d) Mo=+6 span the frans-position.
(e) Fe= 0 (0 W = +6 (chelating agent like CH2NH2 —CH2NH2 with at least
11. (a) +3 (b) 6 (c) C2O; two coordination points is symbolised
12. (a) tetramminedinitrocobalt (III) nitrate by a curved line giving the symbol for
(b) tetrachloroethylenediamine platinum (IV) the ligand in the middle
(c) sodium dithiosulphatoargentate (I)
Chapter 18. Coordination Compounds | 799

ci ci ci ci 23. (b) and (e) for structure see text.


a
J 24. [Co(NH3)4BrCl]Cl, isomer [Co(NH3)4Cl2]Br
a Ni Ni 25. (a) 2 (b) 2
26. (a) Two geometical isomers
H2N nh■2. en- (b) Two optical isomer. For structures see text
\ /
CHj—CH2 27. (c) and (d)

Werner Theory of Coordination • In the complex [Co(NH3)6 ]C13, primary valency of Co


is three and secondary valency six.
Compounds • Primary valency is satisfied by negative ions (anions)
• Beginning of the coordination chemistry is not based while secondary valency is satisfied by ligands which
on the first formation of CoCl3 -6NH3 by Tassqert but can be neutral, and as well as negative (positive in
is actually considered based on the outstanding work rare cases)
of the Alsatian-Swiss chemist Alfred Werner in 1892. — In [Coni(NH3)6lCl3, six NH3 ligands satisfy
• Werner prepared and isolated many different secondary valencies while three Cl* satisfy
coordination compounds from C0CI3 and NH3 like primary valencies.
(a) CoCig ■ 6NH3 [Co(NH3)6]Cl3 orange-yellow — In[ColI(NH3)4(H2O)2 ]SO4, fourNH3 and two l^O
(b) CoClg • 5NH3 [Co(NH3)5C1]C12 violet (purple) (ligands) satisfy secondary and one SO^" satisfy
primary valencies.
(c) CoC13 -4NH3 [Co(NH3)4C12]C1 violet • Ligands/ions attached by secondary valency do not
(d) CoCl3 • 3NH3 [Co(NH3)3C13] green ionise while ions attached by primary valency ionise.
K4(Fe(CN)6] 4K* +[Fe(CN)6]'-
Treatment of these compounds with aqueous [Pt(NH3)6]Cl4 [Pt(NH,)e]u+4Cr
hydrochloric acid did not remove the ammonia, which
suggested that the ammonia was somehow closely bound • The secondary valencies are directional and are
with the cobalt ions. Treatment with aqueous silver responsible for the isomerism in complexes.
nitrate at 273 K, however gave interesting results. One • Primary valency in the complex is represented by......
mole each of the complexes (a), (b) and (c) gave while secondary valency by-------- .
respectively thus, two and one mole of white precipitates • In all cases, metal or metal ion should satisfy primary
ofAgCl, with excess of AgNO3. Complex (d) did not give the and secondary valencies both. Some negative ions
precipitate at all. may satisfy primary as well as secondary valencies
(dual nature). In the complexes, they are
represented as.........
Postulates
• There are two types of valencies shown by central
metal/metal ion in a compound :
Werner’s Representation
(a) primary (b) secondary Cunsiuer me case
Consider the ease of uuvig3 xNH3
ui CoCl
Primary valency corresponds to oxidation number and where maximum value of x=C.N. of Co(III) = 6
secondary valency corresponds to coordination and minimum value of x =C.N.-O.N. = 3
number. Various structures are summarised in Table 18.6.

Table 18.6 Werner’s Coordination Compounds


Werner Secondary valency satisfied
Modern notation Ionisation Primary valency satisfied by
_____ complex by
(A) CoCI3-6NH3 [Co(NH3)6]CI3 [Co(NH3)6]3* + 3CI" six (NH3) three (Ci-)
(B) CoCI3-5NH3 [Co(NH3)5CI]CI2 [Co(NH3)5CI]2+ + 2CF five (NH3) and one (CF) three (Cl-) including one (Cl-) with dual
nature
(C) CoCI3-4NH3 [Co(NH3)4CI2]CI [Co(NH3)4CI2r +cr four (NH3) and two (Cl-) three (Cl‘) including
two (Cl’)with dual nature
(D) CoCI3-3NH3 [Co(NH3)3CI3] [Co(NH3)3 Cl3] three (NH3) and three (CF) three (CF) all with dual nature
(single species)
800 | Essential Inorganic Chemistry

• From the Table 18.6, it is clear that conduction of the


3d 4s 4d
complexes which depends on the number of ions will
be in order: Fe'MAr] t f t t t
D < C<B <A
• They are represented as :
nh3 Cl hybridise
Cl
H3N-
Cl....
H3N'
Cd:
^nh3

\^nh3
h3n
Cl.....
H3bT ^NH3
1
■nh3
Fe'MAr]
3d
f f f f f
six sp3 d 2 hybrids 4d

Cl Cl
nh3 nh3
Each of the six H2O ligands donates one electron pair
(A) (B) into one of the six sp3^2-orbitals, forming six
[only Cl ions joined by ( ) will ionise]. coordinate-covalent bonds. These donated electrons (from
O of H2O ligands) have been represented as “x” rather than
Cl Cl “T”. These are the only bonding electrons; none of the
3d-electron of iron is involved in bonding.
h3n nh3 H3N-^ : ■nh3 A set of outer d orbitals is used in hybridisation, and
Cl..... ‘Co: -Co hence [Fetf^Og ]3+ is called an outer orbital complex.
h3: ‘NH3 Cl" •nh3
3d sp3d2 4d
Cl Cl [Fe(H2O)6]3+ [Ar] f f f t t XX XX XX XX XX XX
(C) (D)

In deciding whether the hybridisation at the central


metal ion of octahedral complexes is sp3cfi (outer orbital)
Bonding in Complexes or c?sp3 (inner orbital) we must know the results of
magnetic measurements. These indicate the number of
Valence Bond Theory (VBT) One way to think
unpaired electrons (2V). Magnetic moment is given by
about bonds between a metal ion and its ligands is based
on coordinate covalency. The central ion furnishes the p = 7MAT + 2)BM
orbitals and the ligands furnish electron pairs (refer (BM is Bohr Magneton)
Chapter-7). Although this view is now considered In [Fe(CN)6]3", Fe3+ is involved (a d5 ion). But
inadequate, it does account for the observed coordination magnetic measurements indicate only one unpaired,
number of the central ion, and it also describes the electron per iron atom. The 3d*2 an^ 3d^2 orbitals
geometric structure of the complex ion.
(rather than those in 4d-shell) are involved in (Psps
Let us study some complexes to see how the valence
hybridisation. All but one of the non-bonding 3 d-electrons
bond theory explains their properties. Take example of
of Fe3+ are paired in this complex.
[Fed^Ojg]3*. The complex is known to be paramagnetic,
with a magnetic moment corresponding to five unpaired 3d 4s 4p
4-------
electrons. This can be described if we follow:
Electronic
Fe26:
configuration of 3d 4s Fe'.3+ [Ar]
f t
[Ar] Ti ? L ? T U
Electronic configuration of Fe3* 3d 4s
(it is formed by loss of the
Sdjy 3dJ*
v2 sixd2sp3 hybrids
[Ar] ? ? T ? T
4s-electrons and one of the
3d-electrons) [FeCCNJe]3- [Ar] fl fl f
xx xx XX XX XX XX

To account for the experimentally observed fact that


[Fe(H^O)6 ]3+ has five unpaired electrons, each 3d-orbitals Only inner d-orbitals are used in hybridisation, hence
is assumed to have one unpaired electrons in the complex. [Fe(CN)6]3“ is called an inner orbital complex.
The vacant 4d^2 ^2 and4d*2 orbitals are hybridised with Additional illustrations have been summarized in
the vacant 4s-and 4 p-orbitals. Table 18.7 (refer Chapter-7)
Chapter 18 Coordination Compounds | 801

Illustration 1 Explain the paramagnetic behaviour of When hybridisation occurs, one of the 3d-electrons is
tCo{NH3)6f+. excited to 5s-orbital.
Solution Oxidation state of Co = + 2 The suggested bonding for [Co(NH3)6f+ is consistent
with the fact that this complex is easily oxidised to
3d 4s 4p 5s [Co(NH3)6 ]3+, an extremely stable complex ion.
Co (ground state) [Ar] fl tl t t t tl Since there exists one unpaired electron, this complex
ion is paramagnetic. Magnetic moment is given by:
3d j 4s 4p :5s ji = ylN(N+ 2) B.M. = V3 BM
Co2+ [Ar] _ti_ ___ 32
| hybridise

3dxy 3(1x2 3d, dV 5s


[Co(NH3)6]2+ [Ar] tl tl tl xx xx xx u xx xx t

Table 18.7 Bonding and Hybridisation in Some Octahedral Complexes

Metal Complex
Type Outer electron configuration
Ion Outer electron configuration ion

3d 4s 4p 4d 3d d2sp3 4d

Cr3*
(d’)
ttt [Cr(NH3)6]3+ inner orbital XXXXXXXXXXX>
(paramagnetic)

3d 4s 4p 4d 3d d2sp3 4d
Mn2*
(d5)
ttft t [Mn(CN)6]2’ inner orbital UH t XXX XX XX XX XXX
(paramagnetic)

3d 4s 4p 4d 3d d-sp1 4d
Fe2+ tlttt t [Fe(CN)6[ inner orbital fit! fl XXXXXXXXXXX>
(d6)
(diamagnetic)

^2^3
3d 4s 4p 4d 3d d sp 4d

Co3+ fltf f t [Co(NH3)6]3+ inner orbital fl fl fl JOOOOOOOOQOO


(d6)
(diamagnetic)

3d sp3d2 4d

[CoFs]3- outer orbital fl t t t t XXXXXXX XXXXX

(paramagnetic)

3d 4s 4p 4d 3d sp3d2 4d

Co2+ tltlttf (Co(OH2)6]2+ outer orbital fl fit t f XXX XX XX XX XXX


(d7)
(diamagnetic)

3d 4s 4p 4d 3d sp CT.2 4d
Niz+ fltltlt t [Ni(OH2)6]2+ outer orbital tlfltltt XXX XX XX XX XXX
(d6)
(paramagnetic)
802 | Essential Inorganic Chemistry

Crystal Field Theory relatively close approach of ligands. Electron in set


tends to destabilise octahedral complexes.
The valence-bond theory has some shortcomings: fl

• Based on valence bond theory, [CoF6] is a_


• It does not provide insight into the origin of the
paramagnetic outer orbital complex while
characteristic colour of the complex ions.
[Co(NH3)6]3+ is a diamagnetic inner orbital complex.
• It does not distinguish between weak and strong Both contain d6 Co3+ ions (four unpaired electron) in
ligands. its 3 d-orbitals.
• It does not give quantitative interpretation of the
3d
thermodynamic or kinetic stabilities of coordination
compounds. Co3+ (Ar) U T T T T
• There is no quantitative interpretation of magnetic Magnetic measurements indicate that [CoFg]3- also
data. has four unpaired electrons per ion. Thus, there must be
• It does not explain why some complexes are four electrons in t^g orbitals and two in eg orbitals.
inner-orbital complex and some complexes are
outer-orbital.
Most of the shortcomings of VBT of coordination
AU = small, weak
compounds were removed in Crystal Field Theory crystal field
(CFT). It attributes partial covalent character and partial Co3+ ion in \jl_t t ’ -
due to F-
ionic character to bonds. & Hl ti spherical [CoFg]3-}^ (not to scale)
O)
C
3d
~ ’
crystal field
M free Co3+ ion
Complexes with C.N. 6 (Octahedral Crystal
On the other hand, [Co(NH3)6 f + is diamagnetic, thus,
Field) all the six d-electrons must be paired in the orbitals.
• Five d-orbitals (d^, dyz, d^, dx2_ 2 and d 2) are of
z ■—
zVt = large; strong
equal energy and thus are degenerate.
crystal field
• The d^ y2and d^ orbitals are directed along a set of \Otl I
'metal ion in due to NH3
mutually perpendicular x, y and z-axes. As a group, spherical [CoCNHs^]3*)^•8 (not to scale)
these orbitals are called eg orbitals. <u 3d crystal field
c
w free Co3+ ion
• The d^, dyz and d^ orbitals, lie between the axes and
collectively called t2g orbitals. The difference in configuration between [CoF6 ]*" and
[Co(NH3)6 ]3+ is due to the relative magnitude of the crystal
• The ligand donor atoms approach the metal ion along field splitting, Aoct caused by the different crystal field
the axes to form octahedral complexes. Crystal field strengths of F" and NH3. The NH3 molecule interacts with
theory proposes that the approach of the six donor vacant metal orbitals more strongly than the F- ion does.
atoms along the axes sets up an electric field Aort for[Co(NH3)6]3+> Aoct forlCoFJ3-
(crystal field). Electrons on the ligand repel electrons
in eg orbitals on the metal ion more strongly than they The crystal field splitting for [CoF6]3- complex ion is
very small and thus, an energetically more favourable
repel those in t2g orbitals. This removes the condition may arise if two electrons remain unpaired in
degeneracy of the set of d-orbitals and splits them into the anti-bonding,eg orbitals. After all d-orbitals are
two sets, the eg set at higher energy and the t2g set at half-filled, additional electrons will pair with electrons in
^set. This is the case for [CoF6 ]3-, which is called a high
lower energy.
spin complex.
The energy separation between the two sets is called
For [CoFg]3" : F" is a weak field ligand, so A^ < P
Aoctahedral or Aoct • is proportional to the crystal field of
(electron pairing energy), thus, high spin complex.
the strength of the ligands.
A high spin complex in crystal field terminology
Aoct
corresponds to an outer orbital complex in VBT.
(not to scale) For [Co(NH3)6 ]3+ : NH3 is a strong field ligand so that
metal ion
& in spherical d^a^ av A oct > P, thus low spin complex.
v
c
dxy da dyz djjdj crystal field A low spin complex corresponds to an inner orbital
free metal ion complex.
(into crystal field) • Low spin configurations exist only for octahedral
complexes having metal ions with d4, d5, d6 and d'
• The d-electrons on a metal ion occupy the t2g set in configurations. For d1 - d3 and d8 - d10 ions
preference to the higher energy, eg set. Electrons that configuration is designated as high spin. All d*
occupy the eg orbitals are strongly repelled by the possibilities have been shown in Table 18.8
Chapter 18: Coordination Compounds | 803

Table 18,8 Electron Configurations and Crystal Field Stabilisation Energies for High and Low Spin Octahedral Complexes
Configuration d]
d d2 d3 d4 d5 d6 d7 d8 d9 d10

Examples Ti3+ Ti2+, V3* V2+. Cr3+ Mn3+, Re3+ Mn2+, Fe3+ Fe2+ , Pd4+ Co2+, Rh2+ M-2+
Ni ,Pt2+ Cu2+ Zn2+. Ag+

c eg
T f f fT- Tf t~T TF fl fl
£.
Q.
m kg f f f f f f f f f f f f H f t fl fl f till fl fl fl fl fl fl fl
□>
I CFSE 0 -|*o
-1*0 -1*0 0

eg f
I '29
fl f f fl fl f fl fl fl fl fl fl
1 CFSE Same as high spin
10 A 12 A
-T*o Same as high spin

Complexes with C.N. 4 (Tetrahedral Crystal Thus, greater the ionic charge on the central metal
Field) ion, the greater the value of A.
In tetrahedral complex formation, the d-orbital
splitting is inverted and smaller as compared to the Colour and the Spectrochemical Series
octahedral field splitting. If A t is the magnitude of energy A substance appears coloured because it absorbs light
separation in tetrahedral crystal field, then for the same that corresponds to one or more of the wavelengths in the
metal, the same ligands and metal-ligand bond distance, visible region of the electromagnetic spectrum (4000 to
*t ~ ~ y *oct
7000A) and transmits or reflects the other wavelengths. A
combination of all wavelengths in the visible region is
In such cases, low spin configuration are rarely called “white light”, sunlight is an example. The absence
observed. of all wavelengths in the visible region is blackness.
djy dyz Cb Most transition metal compounds are coloured. In
transition metal compounds, the d-orbitals in any one
energy level of the metal are not degenerate. They are
metal ion in'-._______ }e^ often split into two sets of orbitals separated by energies,
£
o
tetrahedral d^ d22 A^, that correspond to wavelength of light in the visible
region. The absorption of visible light causes electronic
c crystal field
W <4y dy, transition between orbitals in these sets. Colour of some
transition metal nitrates in aqueous solutions are given
• The magnitude of Aoct or the orbitals splitting below:
energy is decided by
— nature of the ligand : which have been Transition metal ion Colour of aqueous solution
arranged in increasing field strength as: Cr3* Deep blue
I~ < Br" < Cl" < F~ < OH" < HgO < < NH3 Mn2* Pale pink
< en < NOg < CN- Fe2* Pale green
— geometry of the complex: At value (of Fe3* Orchid
tetrahedral complex) is about 50% compared to A^ Co2* Pink
value (of octahedral complexes) Ni2* Green
— nature of the metal ion: going from Cr to W Cu2* Blue
(through Mo), Aoct increases 50% in each series
change (i.e., 3d to 4d to 5d). Thus, there is greater One transition of a high spin octahedral Co(III)
tendency of the second (4d) and third (5d) complex is represented below :
transition series to be low spin as compared to first
Group state Excited state Energy of light
(3d) transition series.
— oxidation state of the metal ion Absorption f Absorbed
(,.Ht t of light f f f | AE = hv depend on Axt
A(Fe3+) > A(Fe2+)
A(Cu2+) > A(Cu+)
804 | Essential Inorganic Chemistry

The frequency (v), and thus the wavelength and • Strong field 1i 'jands, such as CN", usually produce low spin
colour, of the light absorbed are related to Aocl. This, in complexes, where possible, and large crystal
field-splittings.
turn, depends on the crystal-field strength of the ligands.
• Weak field ligands, such as Cl", usually produce high spin
Hence, the colour and the visible absorption spectra of
complexes and small crystal field-splittings. Low spin
transition metal complexes, as well as their magnetic complexes usually absorb higher-energy
properties, provide information about the strengths of the (shorter-wavelength) light than do high spin complexes.
ligand-metal interactions. • In [Cr(NH,)fi]3+, the Cr(III) is bonded to six NH, ligands,
which produce a relatively high value of A^. This absorbs
Table 18.9 gives the relationship among colours
light in the blue and violet regions of visible light and thus
absorbed and colours transmitted or reflected in the orange-yellow complimentary colour is observed.
visible region. The spectral colour is the colour • H2O is a weaker field ligand than NH3, hence,
associated with the wavelengths of the light absorbed by ((Cr(H2O)G]3+)<Ao„ ([CrtNH,),]3*)
the sample. The complimentary colour is the colour
and thus [Cr(H2O)6]3+ absorbs lower-energy light and
associated with the wavelengths that are not absorbed by appears blue (complimentary colour)
the sample, and is seen when the spectral colour is
removed from white light. Complimentary colour of
orange is spectral colour blue and vice-versa and is
Crystal Field Stabilisation Energy
represented in the form of a wheel (to remember) Electrons in the t^ orbitals of an octahedral complex
are lower in energy, while electrons in eg orbitals are
Table 18.9 Complimentary Colours higher in energy, than they would be in a spherical field.
Complimentary The lower the total energy of a system, the more
Wavelength Spectral colour
absorbed (A) (colour absorbed) colour (colour stable it is.
observed)
The crystal field stabilisation energy (CFSE) of a
4100 violet lemon yellow
complex is a measure of the net energy of stabilisation of a
4300 indigo yellow
4800 metal ions electron’s. CFSE is zero for weak field (high
blue orange
spin) octahedral d°, cP and d10 complexes.
5000 blue-green red
5300 green purple Ion dn
5600 lemon yellow violet
5800 yellow
cT7 ~d°
indigo
Mn2 + d5 CFSE = O
6100 orange blue
6800 Zn2+ d10
red blue-green

Illustration 1 Draw a crystal field orbital energy level


Orange diagram, and predict the number of unpaired electrons for
/ each of the following complexes’.
(a) [Cr(en)3]3+ (6) [Mn(CN)6]3" (c) [CoOVV
Red | Yellow
Solution (a) Cr3+ ([Ar] 3d3) has three unpaired,
electrons. In the complex, they occupy the lower-energy
set of d-orbitals.
(b) Mn3+ ([ArJSc?4) can have a high-spin or low-spin,
configuration. Because CN" is a strong-field ligand,
Fig. 18.1 Spectral colour and complimentary colour all four d-electrons go into the lower-energy
are opposite to each other d-orbitals. The complex is low-spin with two
unpaired electrons.
DEEP Focus (c) Co2+([Ar] 3d7) has a high-spin configuration with
• By study of visible spectra of many complexes, it is possible three unpaired electrons because H2O is a weak-field
to arrange common ligands in order of increasing crystal ligand.
field strength: In the following orbital energy level diagrams, the
I" < Br" < Cl", F" < OH" < HZ9O < CI9O42" < NH,O relative values of the crystal field splitting A are in
< en < NO" < CN", CO accordance with the position of the ligands in the
This arrangement is called the spectrochemical series. spectrochemical series (HgO < en < CN"):
Chapter 18: Coordination Compounds | 805

E DEEP Focus
• For a given ion and ligand, greater the charge on the metal
ion, greater the stability. Thus, stability of
z2-y2 [FeI,l(CN)6]3- >[Fe1I(CN)6J4-
A
A • For the divalent ions of the first row transition elements
I (3d-series), the stabilities vary in the Irving-Williams
order:
xy yZ zx Mn11 < Fenu < Coii11 < Nin < Cu" < Zn I!
• Ligands containing N, O or F donor atoms form most stable
[Cr(en)3]3+ [Mn(CN)6]s* [Co(H2O)6]3+ coordination compounds when metal ions are:
— groups 1 and 2 elements, the inner transition elements
and transition elements of groups 3 to 6 (Sc, T, V, Cr)
called ‘class a’ acceptors.
Stability Constants and Stability of — Transition elements —Rh, Pd, Ag, Ir, Au and Hg having
Coordination Compounds relatively completely filled d-orbitals called ‘class b’
acceptors.
Consider the stepwise formation of the complex ML4 — The stability also depends on the formation of chelate
from the metal ion M4+ (Lewis acid) and ligand L~(Lewis rings and stabilisation due to chelation is called the
base). Let k^, and k4 be stepwise formation constants chelation effect. This effect is maximum for the five
and six-membered rings.
and P4(= kf) be the overall formation constant. Thus,
• If a multidentate ligand happens to be cyclic and there are
[ML3] no unfavourable steric effects, a further increase in
m4++l- ML3+,
[M44][r] stability occurs. This is termed the macrocyclic effect.

ml3+ +l~ =
Kinetic Consideration of Ligand Displacement
When we add NH3(aq) to a solution containing Cu2+,
MlV+L- MI£, k3 = we observe a change in colour from pale blue to deep blue.
[M4*][L-]
The reaction involves NH3 molecules displacing KjO
ml3 + l~ ml4, k4 =
(ml;i molecules as ligands.
[Af£$][L-] [Cu(H2O)4f+ +4NH33 -----> [Cu(NH3)4f* +4H2O
(pale blue) (very deep blue)
[ML,]
Af4++4L" ML4, 04 = [M4*)(r ]* This reaction occurs very rapidly as rapidly as the two
reactions can be brought together. The addition of HCl(aq)
to an aqueous solution of Cu2t produces an immediate
04 — colour change from pale blue to green, or even yellow if the
log p4 = log + log + log + log k4 HCl(aq) is sufficiently concentrated.
[Cu(H2O)4f+ +4Cr -----> [CuCl4f- +4H2O
If the complex is MLn, and pn the overall formation pale blue yellow
constant, then
Complex ions in which ligands can be interchanged
M + nL MLn (charges omitted) rapidly are said to be labile. [CutfljO^ f+, [Cu(NH3)4 f+,
and [CuCl4f" are all labile.
WLn]
0n = [M][L]n - ^^2^4 ••• Ki In freshly prepared CrCl3(aq), [Crd^O^C^l*
produces a green colour, but colour gradually turns to
log Pn =logfe1+log^ + log^+... + log violet. This colour change results from the very slow
exchange of FLO for Cl" as ligands. A complex ion that
• pn, the stability constant, is related to thermodynamic exchanges ligands slowly is said to be non-labile or inert.
stability when the system has reached equilibrium. In general, complex ions of the first transition series,
Most of the measurements have been made from except for those Cr(III) and Co(III) are kinetically labile.
aqueous solutions, which means that when complex is Those of the second and third transition series are
formed, stronger ligand L (say CN“, en) displaces generally kinetically inert.
weaker ligand H2O of the aqua complex of the metal These terms (labile/inert) are used to describe the
different kinetics of the reactions that are
ion.
thermodynamically allowed. Large activation energy
MOELjO)" + nL MLn + nH2O barriers for ligand substitution reactions of inert
806 | Essential Inorganic Chemistry

complexes make those reactions slow even though there while Cd2+ forms less stable complex:
may be thermodynamic tendency to proceed. In the Cd2++4CN’^ [Cd(CN)4f~
substitution reaction unstable
ICo(NH3)6]3+(ag) + 6H3O*(a9) [Co(H2O)6]3+(aq) When H2S gas is passed, [Cd(CN)4 f ~ is precipitated as
+ 6NH4 (aq) CdS without affecting [Cu(CN)4 ]3- complex.
The products are favoured by an enormous amount, • Electrolyte solutions used in commercial
yet the inert [Co(NH3)6]3+ complex ion last for weeks in electroplating are quite complex. A number of metals
acidic solution because there is no low-energy pathway for such as Cu, Ag and Au are generally plated from
the reaction. The cobalt (III) ion, [Co(NH3)6]3+, is solutions of their cyano complex ions. In the
thermodynamically unstable relative to [Co(H2O)6 ]3+ but electrolysis reactions below the object to be plated is
kinetically stable (inert) The closely related cobalt (II) made the cathode and the piece of copper metal is the
complex, [Co(NH3)6]2+, reacts with water in a matter of anode.
seconds. Anode: Cu + 4CN- ---- > [Cu(CN)4]3' +e"

[Co(NH3)6f+(ag) + 6H3O*(ag) * [Co(H2O)6f+(aq) Cathode: [Cu(CN)4]3-+e ---- >Cu + 4CN"


+ 6NH4(aq) The net change simply involves the transfer of Cu
The complex [Co(NH3)6f+ is thermodynamically metal from the anode to the cathode through the
unstable and also kinetically labile. formation, migration and decomposition of complex
Illustration 1 Formation constant for following ion[Cu(CN)4]3".

complexes have been given : • Purification of water is also through complex


[Ni(en)3f*. Kf = l.lx 1018 formation. One method of water treatment involves
chelation. Among the chelating agents widely
(Ni(NH3)6f4, Kf = 5.3x 108
employed are the salts of EDTA, usually the sodium
Explain the relative stability of these complexes. salt. The formation constants of [Ca(EDTA)f“ and
Solution Ethylenediamine (en) is a bidentate ligand [Mg(EDTA)f- are very large. Hence, on treating
and ammine (NH3) is an unidentate ligand. Complex ion water with sodium salt of EDTA, concentration of
[Ni(en)3]2+ with multidentate ligand has much larger Ca2+ and Mg2* can be reduced to the point that these
formation constant than do those with unidentate ligands.
ions will not precipitate with soap or detergent.
Thus, [Ni(en)3f + is more stable than [Ni(NH3)6 f+.
This additional stability of complex ions associated with Ca2+ + (EDTA)4" [Ca(EDTA)f“
chelate formation by multidentate ligands is known as Mg2+ +(EDTA)4’ [Mg(EDTA)f- Kz = 4xl08
the chelation effect.
• Chelation with EDTA can also be used in the
treatment of metal poisoning. If a person suffering
from lead-poisoning is fed (Ca(EDTA)]2-, the following
Applications of Coordination exchange occurs:
Chemistry Pb2++ [Ca(EDTA)]2- ---- > (Pb(EDTA)f“ + Ca2+
• The photographic “fixing” process removes unreacted
AgBr which forms soluble complex with sodium because Kf of [Pb(EDTA)f- is 1 x 1018 and is thus
thiosulphate: more stable than [Ca(EDTA)]2“. The lead complex is
excreted by the body and the Ca2+ remains as the body
AgBr(s) + 2S2O§" [Ag(S2O3)2]3- + Br" nutrient.
soluble
• The extraction of silver and gold involves the
• In qualitative analysis of group I, there is formation of
formation of cyanide complex:
white precipitate of AgCl, Hg2Cl2, and PbCl2.PbCl2 is
soluble in hot water. AgCl can be removed as soluble 4Au(s) + 8CN - (ag) + O2 (g) + 2H2O(Z) —>
[Ag(NH3)2]Cl on adding NH3(aq): 4OH"(ag) + 4[Au(CN)2 ](ag)
AgCl(s) + 2NH3(ag) Ag[NH3)2]Cl 2[Au(CN)2] (aq) + Zn(s) ■>[Zn(CN)4f-(ag) + 2Au(s)
soluble Mond’s process involves formation of carbonyl
In group II, Cd2+ can be precipitated as CdS in complex of nickel:
presence ofCu2+.By treating the solution ofCu2+ and
Cd2+ with an excess of CN", Cu2+forms a stable Ni(s) + 4CO(g) ---- > Ni(CO)4(g)
complex in a redox reaction : • Traces of Ni2+ salt can be detected by dimenthyl
2Cu2+ +10CN- ---- > 2[Cu(CN)4]3" + Q>N2(g) glyoxime when cherry red precipitate of
stable nickel-dimethyl glyoximate (chelate) is formed:
Chapter 18: Coordination Compounds | 807

HO—C=N/CH3 The s- and p-block organometallics are named


Ni2+ + 2 | — according to substituent names used in organic chemistry.
HO—C=Nk CH3Li methyl lithium
xch3 B(CH3)3 trimethyl boron
ZO—H-0 Si(CH3)4 tetramethyl silane
As(CH3)3 trimethyl arsane
h3c\^N CH3
The oxidation number of the metallic element in an
Ni- organometallic compound is based on the organic part
being considered to be anionic. Thus, CH3 group in
h3c CH3 Zn(CH3)2 has (-1) charge.
XO—H—0' • d- and /-block organometallics
• Cis-diammine dichloroplatinum (II), [Pt(NH3)2 Cl2 ], is W.C. Zeise was the first to prepare organometallic
H3N Cl compound of d-block. Examples are
- trichloroethene platinum (II): [PtCl3(CyH4 )]"
H3N Cl - tetracarbonyl nickel (0): Ni(CO)4
one of a number of platinum coordination compounds used - bis(cyclopentadienyl) iron (II) : FetCgHg^ also
in the treatment of cancer. Commonly known as cis-platin, called ferrocene.
this compound has the ability to block the uncontrolled Ferrocene is stable in air, forms orange crystals and is
division of cancerous cells that results in the growth of diamagnetic. It is soluble in organic solvents (alcohol,
tumours. Recent studies show that cis-platin can cause ether and benzene) and insoluble in water, NaOH solution
serious side effects, including severe kidney damage. It is and cone. HC1 solution.
being replaced by dinuclear platinum complex. It is thermally stable up to 500°C. The X-ray structure
2+ shows that this has a sandwich structure in which the
Cl NH3 HjjN Cl metal atom is sandwiched between two planar
cyclopentadienyl rings.
H3N NHjCCH,^ HgN NH3 In metal carbonyls, CO as a ligand binds itself to
trans-isomer (n = 2 to 6) metal atom through the carbon atom. It is a weak donor (a
weak Lewis base) and forms a weak o-bond to the central
• Organometallic compound must contain, at least one
atom. CO is also and acceptor (Lewis acid) ligand and
metal-carbon bond. Ni(CO)4 is also considered
forms n bond to the metal. This characteristic
organometallic compound, through CO is not
considered to be an organic compound. The suffix property of back bonding stabilises the metal-ligand
metallic often includes metalloids like boron, silicon, interaction.
arsenic and tellurium as well as true metals. n*
Metal-cyanides, though possessing an M—C bond are n:
not included in organometallics. c
M
• Main group organometallics
E.C. Frankland was the first chemist to synthesise an J n
organometallic compound.

Target Practice 2
1. The complex CoCl2- has a tetrahedral structure. [Ptci3]3* + cr —> [Ptcij2* #2 = IO’3

(a) How many d- electrons are on the cobalt? [Ptcy2* + cr —> [Ptci3r K. = IO"4
♦J

(b) What is its electronic configuration? [PtcLF + cr —» [Ptcij x4 = IO’5


2. Make complexes with varying NH3 ligands to satisfy 6. Dilute equimolar solutions of each of the following
primary and secondary valencies of platinum. compounds are prepared. Arrange the compounds in order
PtCl4xNH3 of increasing molar conductivity (which is proportional to
the number of ions) of these solutions :
!
(a) Na_[Co(NO„)J
J L O
(b) K{Co(EDTA)]
3. Arrange the complexes in (2) in increasing electrical (c) [Co(Py)3(CN)3] (d) [(MNH3),C1]C12
conductivity (e) [Pt(NH3)6]Br4
4. What is CFSE of high-spin d5 complex ofFe3+?
7. While practice in the laboratory, you have some yellow
5. What is pKvalue of the complex [PtCl4J if spot of FeCl3 on your white shirt. Suggest a suitable
Pt4+ + CP ---- > [PtCL]3f Kx = 10“2 reagent that can remove this yellow spot
808 | Essential Inorganic Chemistry

Answers
1. (a) 7 log K = log Ki + log K2 + log K3 + log K4 = -14
(b) f2/5)d2 4 dl n n ? - log K = 14
eJ2) d\ d\_y2 T pK = 14
6. (c) < (b) < (d) < (a) < (e), ionisation being as:
2. I. [Pt(NH3)6]Cl4
(c) does not ionise
II. [PttNH3)6Cl]Cl3
(b) K+ + (Co(EDTA)]" (two ions)
in. [Pt(NH3)4Cl2]Cl2
IV. [Pt(NH3)3Cl3]Cl (d) [Cr(NH3)5Cl]2+ + 2d" (three ions)
V. [Pt(NH3)2Cl4] (a) 3Na+ + [Co(N02)6]3~ (four ions)
3. V<IV<III<II<I [Pt(NH3)6]4+ + 4Br" (five ions)
(e)
4. Zero
7. Yellow spots can be removed by complexingFe3* by oxalate ion
5. On adding
and washing it with water.
Pt4* + 4C1" ---- > PtCl4
Fe3+ + 3C2O2" [Fe(C2O4)3]3-
Overall stability constant colourless (soluble)
K = KxK2K3K4 = 10 -14

Practice Exercise 2
|3*
1. PtCl4 ■ 6H2O on reaction with AgNO3 forms 2 moles of AgCl 9. The absorption maximum for the complex ion [Co(NH3 )61’
(white ppt.) per mol of the PtCl4 -6H2O. What is complex? occurs at 470 nm.
Give its Werner’s representation. (a) Predict the colour of the complex and
2. CuC12 xH,0 (x = 2,3, 4) can be represented by Werner’s (b) Calculate the crystal field splitting in kJ/mol.
formula. What are those? Arrange them in increasing 10. Draw a crystal field orbital energy level diagram, and
conducting power. predict the number of unpaired electrons for each of the
3. What are types of ions joined by primary valency and following complexes:
secondary valency of the metal in the following complexes? (a) [Fe(H2O)6]2* (b) [Fe(CN)6J
(a) FeCl3-6H2O (b) CuSO4 -5H2O (c) (VF6)3-
(c) PtCl4 -6NH3 (d) PtCl2 -2H2O 11. Explain why [CoCl4 J2- (blue) and [Co(H2O)6]2+ (pink) have
4. What is the true formula of the complex PtCl4 -5NH3 if
different colours? Which complex has its absorption bands
one mole of it on reaction with excess of AgNO3 yields
at longer wavelength?
three moles of white precipitate of AgCl?
5. Wnte modem formulae of the following complexes based on 12. What is chelation effect?
the number of free Cl” ions per formula unit: 13. Give the Irving —Williams order of stability of metal ions.
14. Stability constants of [Cd(CN)4]2- and [Ni(CN)4]2' are
Cl" ions in solution
Formula Colour 1.28 x 1017 and 1.0 x 1031 respectively. Which is more
per formula unit
stable?
(a) CrCI3 • 6H2O violet 3
15. Which of the following complexes is expected to be inert to
(b) CrCI3 • 6H2O light green 2
the ligand substitution?
(c) CrCI3 • 6H2O dark green 1 (a) [Ni(en)3]2+ (b) [Mg(EDTA)]2"
(c) [Sc(H2O)6]3+ (d) [Co(NO2)6]3'
6. Transition metal complexes containing, CN" ligands are
often yellow in colour, whereas those containing H2O (e) [Cr(H2O)6]2+ (f) [A1(OH)4]-
ligands are often green or blue. Explain. (g) [FeF6]3'
7. The complex [FeCl6]|3~ is more paramagnetic than 16. Give the classification of organometallic compounds.
[Fe(CN)6]3-. Explain. 17. Discuss the nature of bonding in metal carbonyls.
8. Give a valence bond description of the bonding in each of the 18. Draw the structure of iron oxalate complex [Fe(C2O4)3]5‘.
following complexes. Include orbital diagram for the free
metal ion and the metal ion in the complex. Specify type of Describe the coordination geometry and identify any
hybridisation and number of unpaired electrons. chelate ring. What is the oxidation number and the
(a) [Ti(H2O)6]3+ (b) [NiBr4]2- coordination number of iron?
(c) [Fe(CN)6]3- (d) [MnCl6]3- 19. What happens when [Ni(H2O)6]i2+ : is mixed with
ethylenediamine in excess?
Chapter 18. Coordination Compounds | 809

Answers
1. [PtCl2(H2O)4]Cl2-2H2O electron, hence, (FeCl6)3“ is more paramagnetic than
Cl [Fe(CN)6]3-. Also refer text.
.Cl 8. Refer text
h2o 9. (a) orange (b) 254.38 kJ mol
h2o -------;Pt—OH2-2H2O E
Cl''* : ^OH2

Cl A
10. A
Since two moles of AgCl are obtained per mole of the compound -- t
hence, two Cl are joined by primary valency and two by
secondary valency.
nJ
2. (Cu(H2O)4]Cl2, [Fe(H2O)6]2* [FeCCbDg]4- [VFg]3-
[Cu(H2O)3C1]C1, [Cu(H2O)2C12]
I II Unpaired (4) (0) (2)
III
electrons:
OH2 OH2 ci
I Cl CL 11- Atrt<Aort hence, different colour [Co Cl J has absorption
peak at higher wavelength.
H20---- Cu—OH2
Cl"" I
oh2
(I)
H 2O
r
zCuk

(ID
oh2
H2O— Cu —OH2

Cl
(HI)
12. Increase in stability due to presence of multidenate ligands
(which causes ring formation) is called chelation effect.
13. Mn2* < Fe2* < Co2* < Ni2* < Cu2* < Zn2*
14. [Ni(CN)4]2-
15. Only (d), [Co(NO2)6]3', due to very high value of formation
Conductance III < II < I
constant.
3. (a) [Fe(H2O)6]Cl3: Three Cl joined by primary
valency (PV) and six H2O joined 16. Refer text
by secondary valency (SV) 17. Refer text
(b) [Cu(H2O)4]SO4H2O: SO joined by (PV) and four 18. Oxidation number of Fe = + 3
H2Oby (SV) Coordination number of Fe = 6
(c) [Pt(NH3)6]Cl4 : Four Cl by (PV) and six NH3 by -13-

(d) [Pt(H2O)2Cl2]:
(SV)
Two Cl have dual nature joined
by (PV) as well as (SV) while
[ cf 0 «
«
oJ
f
two H2O by (SV).
// 0.
r

4. [Pt(NH3)6Cl]Cl3 0 Fe< Chelate ring


5. (a) [Cr(H2O)6]Cl3 (b) [Cr(H2O)6Cl]Cl2-H2O
(c) [Cr(H2O)4Cl2]Cl -2H2O
0
6. CN" is a strong ligand. Absorption near UV (blue), hence
appear yellow.
7. Based on VBT, [FeCl6]3- has Cl" as weak ligands, Fe is sp2d2 L J
hybridised (outer orbital complex) having five unpaired [Ni(H2O)6]2* + 3en ---- > [Ni(en)3]2* + 6H2O
electrons. [Fe(CN)6]3- has CN' as strong ligand. Fe is d2sp2
hybridised (inner orbital complex) having only one unpaired 19. Ethylenediamine (en) displaces H2O forming a chelate.
Total Practice Set
(Read, Plan & Solved
HMBVS

(t) potassium trisoxalatoaluminate (III)


Problem 1. The complex studied by Werner had a (u) tetrapyridineplatinum (II) tetrachloroplatinate (D)
composition corresponding to the formula PtCl4 -2KC1.
From electrical-conductance measurements, he Solution (a) Fe2+ + 6^0 + CP---- > [Fed^Oef'Cl’
determined that each formula unit contained three ions. Cation is a complex ion and carries + 2 charge, hence its
He also found that silver nitrate did not give a precipitate structure is
of AgCl with this complex. Write a formula for this (Fen(H2O)6]Cl2
complex that agrees with this information.
Oxidation state of the metal ion has been given in the
Plan Since Ag+ does not give precipitate with the structure. Students may omit it.
complex, it means all the Cl are in coordinate sphere.
Electrical conductance shows three ions per mol of (b) K+ + Ag3++4F~ ---- > K+[AgF4F
cation
complex, thus, formula can be derived.
anion is a complex
Solution Given complex does not give precipitate with K*[AglnF4]-
AgNO3. It shows the absence of Cl" ions. Thus, all the Cl (c) (Ti"Cl5]3-
ions are ligands and are in coordination sphere.
(d) [ComCl2(NH3)4]Cl
2KCl + PtCl4 KJPtClg]
(e) [Ag'(CN)2T
K2 PtCl6] 2K+ + [PtCl6f-
(f) [NiuCl4(NH3)2f-
AgNO3 three ions (g) [Cun(en)3]SO4
no precipitate (h) Na[AlIn(OH)4(H2O)2]
Thus, complex isK2[PtCl6] (i) (CrlnCl(NH3)(en)2]SO4
(j) K,[Znu(CN)4]
Problem 2. Write the structural formula
corresponding to each of the following IUPAC names : (k) (Cr,n(NH3)6J2[CunCl4]3
(a) hexaaquairon (II) chloride (l) [Pt1ICl2(NH3)2]
(b) potassium tetrafluoroargentate (III) (m) [Ni°(CO)4]
(c) pentachlorotitanate (II) ion
(n) [Pt“(NH3)4](Pt"Cl3NH3J2
(d) tetraamminedichlorocobalt (III) chloride
(e) dicyanoargentate (I) ion (p) Na3[Ag‘(S2O3)2]
(0 diamminetetrachloronickelate (II) ion (p) K4[Ni’(CN)4]
(g) tris (ethylenediamine) copper (II) sulphate
(h) sodium diaquatetrahydroxoaluminate (III)
(q) Felrf-QHjt
(i) amminechlorobis (ethylene diamine) chromium q5 means that all the five carbon atoms of
(III) sulphate cyclopentadienyl anion are coordinated to Fe
(j) potassium tetracyanozincate (II) the metal ion.
(k) hexaamminechromium (III) tetrachlorocuprate (II) (r) [Znn(NCS)4f-
(l) diamminedichloroplatinum (II)
(s) K[Mn™04]
(m) tetracarbonyl nickel (0)
(n) tetraammineplatinum (II) (t) K3(A1,ii(C2O4)3]
amminetrichloroplatinate (II) (u) [Ptu(Py)4 ](PtuCl41
(o) sodium bis(thiosulphato) argentate (I)
(p) potassium tetracyanonickelate (0) Problem 3. Write IUPAC names of the following:
(q) bis (cyclopentadienyl) iron (II)
(r) tetrathiocyanato-N-zincate (II) ion (a) Na2[Fe(CN)5NO] (b) [Ni(DMG)2]
(s) potassium tetraoxomanganate (VII) (c) NH4[Cr(SCN)4(NO)2] (d) [Pd(H20)2(ONO}2 IJ
Chapter 18: Coordination Compounds | 811

i (e) 1Co(NH3)4(NCS) Cl]* (f) KJFeOJ Problem 4. A compound Co(en)2(NO2)2Cl has been
(g)K2[Co(N3)4] (h)[Ni(PPh3)2Cl2] prepared in a number of isomeric forms. One form
(i) [Cr(acac)3] (j) [PWC^HsNlJlPtClj] undergoes no reaction with AgNO3 or (en) and is optically
inactive. A second form reacts with AgNO3 but not with
(k) [Co(NH3)6][CuCl5] (I) [VfHjCWc ]C13 (en) and is optically inactive. A third form is optically
(m) (NH4)3[Co(C2O4)3] active and reacts with both AgNO3 and (en). Identify each
(n) ^(NH3)4Co<(^>Co(en)2 Cl4 of these isomeric forms. Name and sketch each of the
structures.
Plan Based on reactions with AgNO3 and (en), and
OH
(o) (en)2Co Co(en)2 optical activity, isomers can be identified.
■NH
Solution First form : There is no reaction with
(p) Na2[SiF6 ] AgNO3, hence no Cl' ions outside coordination sphere.
(q) KJCrOJ Also there is no reaction with bidentate (en), hence these
(r) l(NH3)5Cr—OH—Cr(NH3)5]Cl5 ligands are trans to each other. Optical inactivity is also
due to trans structure. Thus, it may have structure :
(s) [Fe(en)3][Fe(CO)4]
(t) TiCl4(Et2O)2 Cl
(u) Mn2(CO)10
(v) [VO(acac)2]
en, Co en NO2
(w) Fe[Fe(CN)6]+
Solution (a) Sodium pentacyanonitrosoferrate (II)
(b) Bis (dimethylglyoximato) nickel (II) NO2
(c) Ammonium tetrathiocyanatodinitrosochromate (III) tra7is-chloronitrobis(ethylenediainme)cobalt (III) nitrite.
(d) Diaquadiiododinitritopalladium (IV)
Second form : In this Cl" is outside coordination
i (e) Tetraamminechloroisothiocyanatocobalt (IH) ion
sphere since it reacts with AgNO3. As in the first form NO2
I (0 Potassium tetraoxoferrate (IV)
ligands are trans to each other being optically inactive.
(g) Potassium tetraazidocobalt (II) [N3 is azide] This is represented as,
(h) Dichlorobis(triphenyl phosphine) nickel (II)
(i) Tris (acetylacetonato)chromate (III) no2
[(acac)'is CHgCOCHaCH^)]
(j) Tetrakis (pyridine)platinum (H) tetrachloroplatinate
(ID
en. Co en cr
(k) Hexaammine cobalt (III) pentachlorocuprate (II)
(l) Hexaaquavanadium (III) chloride NO2
(m) Ammonium tris(oxalato)cobaltate (III)
trans-bis (ethylenediamine) dinitrocobalt (III) chloride
(n) Tetraamminebis (ethylenediammine)-g-dihydroxo
dicobalt (III) ion Third form : In this case also, Cl' is outside
(0) Tetrakis (ethylenediamine)4i-hydroxo-p-imidodi- coordination sphere. Also it shows reaction with (en)
cobalt (III) ion hence monodentate ligands are cis to each other. Being
optically active, mirror image should not superimpose.
(p) Sodium hexafluorosilicate (IV)
Thus, it can have structure:
(q) Potassium tetraoxochromate (VI)
(r) g-hydroxodecaamminedichromium (III) chloride
(s) Tris(ethylenediamine)iron (HD tetracarbonyl iron (IH) no2
(metal in this complex can also be iron (II)) cr
(t) Tetrachlorobis (diethyl ether) titanium (IV)
(u) Decacarbonyldiamanganese (0) en_J
(v) Bis (acetylacetonato) oxovanadium (IV)
(w) Iron (III) hexacyanoferrate (H) ion cis-bis (ethylenediamine) dinitrocobalt (III) chloride

(also called Prussian blue)


812 | Essential Inorganic Chemistry

Problem 5. Each of the compounds (a) Pt(NH3)6Cl4, Plan Based on C.N., their correct formulae are written
(b)Cr(NH3)6Cl3,(c)Co(NH3)4Cl3 and (d)K2PtCl6 has been and number of ions per mol of complex are deterninetL
dissolved in water to make its 0.001 M solution. Rank Conductivity is proportional to the number of ions.
them in order of their increasing conductivity in solution
(assume 100% ionisation in each case).

Solution
Actual Total Ions
Complex C.N. of metal ion Ionisation of complex
presentation

(a) Pt(NH3)6CI4 6 [Pt(NH3)6]CI4 [Pt(NH3)6]4+ + 4Cr 5


(b) Cr(NH3)6CI3 6 [Cr(NH3)6]CI3 [Cr(NH3)6]3* + 3Cr 4

(c) Co(NH3)4CI3 6 [Co(NH3)4CI2]CI [Cr(NH3)4CI2]++Cr 2

(d) K2PtCI6 6 K2[PtCI6] 2K+ + [PtCI6]2’ 3

Greater the number of ions, greater the mole of AgCl obtained = 2^1=0.02
conduction hence, order of increasing conductivity 143.5
is (c) < (d) < (b) < (a)
mole of CrCl3 • 6H2O = = 0.01
266.5
Problem 6. Two compounds have the empirical 0.01 mole of complex gives = 0.02 mol AgCl
formula Cr(NH3)3(NO2 )3. In aqueous solution, one of these
conducts electricity while the other does not. Deduce their 1 mole of complex gives = 2 mol AgCl
probable structures. Thus, numbers of free Cl" = 2
Plan Complex responsible for conductance ionises in Also C.N. of Cr is six, hence complex is
aqueous solution. Hence, formula can be derived. [CrCKHjjOsJClj, H2O
Solution We know C.N. of Cr is six, hence the complex [CrCKHzOs ]C12 • J^O [CrCl (H^ f + • HjO + 2CT
that does not conduct electricity, would not ionise. Hence,
NO2 and NH3 ligands are in coordination sphere and the Problem 8. A solution containing 0.319 g of
complex is CrCl3 6H2O was passed through a cation exchange resin
[Cr(NH3)3(NO2)3] and acid coming out of the cation exchange resin required
28.5 mL of 0.125 M NaOH. Determine correct formula of
The other complex that conducts electricity would
the complex.
ionise. Thus, in such complex H2O should enter into
(mol. wt. of the complex = 266.5)
coordination sphere making at least oneNO2 available
outside coordination sphere. This complex can be Plan Cr = HCl=NaOH
[Cr(H2O)(NH3)3(NO2)2]NO2 From the number of moles o/NaOH, number of moles of
Cl" are determined. Hence, complex is known.
Problem 7. A solution containing 2.665 g of Solution Let the number of Cl" ions outside the
CrCl3 -6H2O is passed through a cation exchanger. The coordination sphere or number of chloride ions which can be
chloride ions obtained in solution react with AgNO3 and ionised be n. When the solution of the complex is passed
give 2.87 g of AgCl. Determine the structure of the through cation exchanger, nCl- ions will combine withHT (of
compound (Cr = 52, Cl = 35.5, H = 1, O = 16, Ag = 108, the cation exchanger) to form HC1
N = 14)
nCl’ + nH+ ---- > nHCl
Plan Free Cl" after passing through the ion exchange
Thus, 1 mole of the complex will form n moles of HC1
gives white precipitate with Ag+. From the given
stoichiometric data, number of free Cl" can be derived, 1 mole of complex = n mol HC1=nmoles NaOH
hence the formula.
mole of the complex = 5^-=0.0012 mol
Solution AgCl is obtained only with free (ionisable) 266.5
ions 28.5x 0,125
mole of NaOH used = = 0.0036 mol
CrCl3-6H2O + AgNO3 ■» AgCl 1000
Chapter 18 : Coordination Compounds | 813

i 0.0012 mol of complex = 0.0036 mol NaOH Problem 11. Give the name describing the type of
s 0.0036 mol HC1 structural isomerism displayed by each of the following
i i r i 0.0036 o . yr-,, pairs :
1 mol of complex =--------= 3 mol HC1
0.0012 (a) [Co(en)3][Cr(CN)6l and [Cr(en)3l[Co(CN)6]
n=3 (b) [Mn(CO)5(SCN)J and [Mn(CO)5(NCS)]
Thus, all the Cl“ ions are outside coordination sphere. (c) [Co(NH3)5(N03)]S04 and [Co(NH3)5(SO4)]NO3
Hence, complex is [Cr(H2O)6]Cl3. (d) [Co(NH3)4(H2O)CU Cl2 and [Co(NH3)4C12]C1 H,0
Solution (a) Coordination isomerism
Problem 9. Metal carbonyls can have formula
lf(CO)x where x is the number of carbonyl units (b) Linkage isomerism
coordinated to metal M. If EAN of each metal in the (c) Ionisation isomerism
carbonyl is 36, what are formulae of the carbonyl of (d) Hydrate isomerism
Fe(26), Cr(24) and Ni (28)?
Problem 12. Draw/write possible isomers of
Plan Based on EAN and atomic number, value of x in
(a) [Pt(NH3)2Cl2l (b) CrCl3 6H2O
tach complex is determined. Thus, their formulae are
inown. (c) [Co(NH3)5(SO4)]C1 (d) [Co(en)2NH3Br]SO4
(e) [Pt(NH3)(H2O)(C5H5N)(NO2)]Cl
Solution Carbonyl is neutral ligand, hence, oxidation
number of M =0. E AN of metal M (neutral) in the complex Cl nh3 Cl nh3
= 36. Each CO donates 2 electrons.
Solution (a) Pt Pt
(i) Fe(CO)x EAN = Z-(O.N.)+ 2x
36 = 26-0 + 2* Cl nh3 H3N Cl
cis trans
x=5
(b) [CrOL.OJslClg ] anhydrous
hence, complex is Fe(CO)6
[CriHgO^ClJC^ HjO
(ii) Cr(CO)x Z=24
[Cr(H2O)4Cl2]Cl-2H2O hydrate
EAN =36 =24 - 0 +2r
[CHHjjO^Clgl-SHiO
x=6 ionisation
(c) [Co(NH3 )5C1]SO4
hence, complex is Cr(CO)6 (d) [Co(en)2(NH3)S04 JBr ionisation
(iii) Ni(CO)v Z =28 (e) (i) [Pt(NH3)(H2O)(C5H5N)CllNO2 ionisation
EAN =36 =28 - 0+2x (ii) [Pt(NH3)(NO2)(C5H5N)Cl]H2O hydrate
x =4 (iii) [Pt(NH3)(ONO)(C5H5N)(H2O)]Cl linkage
hence, complex is Ni(CO)4
Problem 13. Write equations to represent the
Problem 10. The EAN of each Mn (Z=25) in its following observation :
carbonyl is 36. What is the structure of the carbonyl with When NaOH is added to CuSO4(aq), a pale blue
molecular formula Mn2(CO)10 ? precipitate forms. If NH3(a<7) is added, the precipitate
Plan Based on EAN and Z, structure can be derived. redissolves, producing a solution with an intense deep
blue colour. If this deep blue solution is made acidic
Solution Electrons from each Mn = 25 with HNO3(aq), the colour is converted back to pale
blue.
Electrons from five CO ligands = 2x5=10
Electron from (Mn—Mn) bond = 1 Solution Cu2+ + 2OH" Cu(OH)2(s)
pale blue
Thus, EAN =25+10+1=36
Thus, in the complex five CO (ligands) are coordinated Cu(OH)2(s) + 4NH3 + [Cu(NH3)4f+ +2OH-
deep blue
to each Mn atom and sixth coordination number is
attained by other Mn atom of (Mn-Mn) bond. Thus, it [Cu(NH3)4 f + + 4H3O+ ■> [Cu(H2O)4f++4NH4+
can have structure :
CO CO Problem 14. Write a series of equations to show the
co, oc stepwise displacement of HoO ligands in [FetfW^]3* by
oc -co ethylenediamine (en), for which log = 4.34; log = 3.31
and log = 2.05. What is the overall formation constant,
I CO co p3 (= kf) for [Fe(en)3]3*?
CO
814 | Essential Inorganic Chemistry

Plan Ifk^ k2 and are stepwise formation constants and Plan Rust is Fe2O3 xH2O. We are to study what happens
P3 is the overall formation constant, then to the rust when oxalic acid is used for cleaning.
P3 =
Solution Oxalic acid forms a dark green coloured
log p3 = log + log + log ^3 complex with rust stains (Fe3+ ions).
Thus, P3 can be calculated.
Fe3+ + SCgO3- ---- > [Fe(C2O4)3l3-
Solution Each ethylenediamine (en) being a bidentate This complex is water soluble. Thus, rust stains can be
ligand replaces two H2O ligands (unidentate) removed by treating the fabric with oxalic acid and then
washing with water.
[Fe(H2O)6]3+ + en---- > [Fe(en)(H2O)4 ]3+ + 2H2O, K
[Fe(en)(H2O)4]3+ + en---- > [Fe(en)2(H2O)2l3+ + 2H2O, Problem 16. Carbon monoxide (CO) is toxic when
[Fe(en)2(H2O)2]3+ +en---- > [Fe(en)3]3+ + 21^0, k3 inhaled. Explain.
[Fe(H2O)6]3* + 3 en---- > [Fe(en)3]3+ + 6H2O, P3 ~ kf Plan We are to study the effect of CO on the internal body
Thus, Ps = kf = system.
log p3 = log + log + log 63 Solution CO is toxic because it forms a complex with
= 4.34 + 3.31 + 2.05 = 9.70 haemoglobin in the blood and this complex is more stable
than oxy-haemoglobin. This prevents the haemoglobin in
P3 = 5.01 x 109
the red blood corpuscles from carrying oxygen round the
Thus, overall formation constant is P3 = = 5.01 x 10>9! body. This causes an oxygen deficiency, leading to
unconsciousness and then death. The blood of victims of
Problem 15. Oxalic acid is sometimes used carbon monoxide poisoning is a brilliant red-pink colour
to clean rust stains from sinks and bathtubs. Explain the rather than the dull red of normal blood.
chemistry underlying this cleaning action.
;---------

Master Exercises
Exercise 1
(Stage 1: Learning)
Short Answer Type Questions
1. Which of the following complexes obey the rule of 18 (a)
(EAN rule)?
(a) (Cu(NH3)4 f*. [Cu(en)3f *, [Cu(CN)4 J3"
OC <co
(b) [Fe(CN)6]*-,[Fe(CN)6]3-,Fe(CO)5
<c) [Ni(NH3)6 f*, [Ni(CN)4 f[Ni(CO)4 ] (b) Mn(CO)5-
(d) [Cr(NH3)6]3+, [Cr(CO)6]
(c)
(e) [Co(NH3)6]3*,[CoCl4f-
2. Suppose 0.010 mole of each of the following
Co
compounds is dissolved (separately) in 1.0 L water : ^CH2
KNO3, [Co(NH3)6C13], [Cu(NH3)2C12 ], [Na2(PtCl6)l I I
ch2 ch2
Rank the resulting four solutions in order of
conductivity, from lowest to highest. 9. The anion [NiCl4 f ~ is paramagnetic, but when CN“
3. Predict which of the following octahedral complexes • ions are added, the product (Ni(CN)4f‘ is
has the shortest Xmax : [FeF6]3", [Fe(CN)6]3-, diamagnetic. Explain the observations.
[Fe(H2O)6]3+? [NiCl4f-(ag) + 4CN-(ag) [Ni(CN)4 f-(ag)
4.1s the coordination compound [Co(NH3)6}C12 + 4Cl-(ag)
diamagnetic or paramagnetic. 10. In water, the titanium (III) ion [TKHgOlg]3* has a
broad absorption bond at about 500 nm. What colour
5. Will methylamine [CH3NH2 ] be a monodentate or a
light is absorbed by the ion?
bidentate ligand. With which of its atoms will it bind
to a metal ion? 11. How many unpaired electrons expected for high spin
and low-spin complexes of Fe2+?
6. Consider following octahedral complexes
I: [Fe(CN)6]3-
U: [Fe^OlJ3"
Analytical Questions
(a) Which has fewer impaired electrons? Coordination Number, Effective Atomic Number and
(b) How many unpaired electrons are present in Oxidation
I Number
each? 1. Determine EAN of the metal ion in the following
(c) What is CFSE in terms of Ao? complexes:
7. The chromium (III) ion in aqueous solution is (i)[Cu(NH3)4f* (ii) [CrCNH3)6]Cl3
blue-violet. (iii) [CrtenJJ3*
What is the complimentary colour to blue-violet? 2. What is coordination number and the oxidation state
of the central metal ion in each of the following
8. Which of the following molecules/ions satisfies the complexes?
EAN rule?
816 | Essential Inorganic Chemistry

(a) [Ni(NH3)6f* (b) [A1F6]3- 12. The complex ion; [Pt(NH3)(NH2OH)(NO2) (Py)]* has
(c) [Cu(CN)4f- (d) [Cr(NH3)3Br3] been found to be optically active. What is the
(e) [Ag(S2O3)213’ probable geometry of the complex?
3. Identify the ligands and give the coordination 13. How many different structures are possible for each off
number for the central atom or ion in each of the the following complex ions?
following (a) [Co(NH3)6H2O]3* (b) [Co(NH3)4(H2O)2)’’
(a) [Co(NH3)2(NO2)4r (b) K4[Fe(CN)6] (c) [Co(NH3)3(H2O)3]3+ (d) [Co(NH3)2(H2O))]j’
(c) [Cr(NH3)5Cl]Cl2 (d) [Pd(NH3)4]2+ 14. Indicate what type of isomerism may be found in each
(e) Na[Au(CN)2] (f) [Co(en)3]3+ of the following cases?
4. Only two-third of the chlorine contained in the (a) [Zn(NH3)4][CuCl4] (b) [Fe(CN)5SCN]4’
solution of the complex salt CoCl3-5NH3 were (c) [Ni(NH3)5Cl]+ (d) [Pt(Py)Cl3r
precipitated from it by silver nitrate. No cobalt or
(e) [Cr(NH3)3(OH)3r
free ammonia were detected in the solution of the
salt. The electrical conductivity of the solution shows 15. The structure of four complex ions are given.
that the salt dissociates into three ions. What is the Determine which, if any, of these complex ions are
coordination structure of the complex salt? isomers (geometric or optical); which, if any, are
5. Silver nitrate precipitates the entire chlorine as identical and which, if any, are distinctly different.
silver chloride from a solution of the complex salt OH2
h3n
I

PtCl4 -6NH3 and only one-fourth of the chlorine from


a solution of the salt PtCl4 -3NH3. Write the
coordination formulae of these salts, and determine (a) Co3+ ox
the coordination number of the platinum in each
case. H3N
OH2
6. A sufficient amount of AgNO3 solution was added to
a solution containing 23.35 g of the complex salt oh2
CoCl3 • 4NH3. The mass of the precipitated AgCl was nh3
14.35 g. Derive the coordination formula of the salt.
(Co = 59 g mol-1) (b) ox Co3+
7. Manganese carbonyl is a dimer, (C0)nMn—Mn(C0)n. NH3
If each Mn atom has an EAN of 36, deduce the value OH
of n, assuming single covalent bond between two Mn
nh3
atoms.

Nomenclature (c) ox ‘Co3+

8. Name the following: I OH2


(a) [Ag(NH3)2]+ (b) [Fe(H2O)3OH]2+ nh3
(c) [ZnCl4f- (d) [Pt(en)2f+
nh3
(e) [Co(NH3)4(NO2)C1]+ (f) [Co(NH3)5Br]SO4
h2o
(g) [Co(NH3)5SO4]Br (h) [Cr(NH3)6][Co(CN)6]
(i) Na3[Co(NO2)6] (j) [Co(en)3]Cl3 (d) Co3+ ox
9. Write IUPAC name of H2O
(a) [Cr(NH3)5CO3]Cl (b) [Co(NH3)5ONO]C12
(c) K3[Cr(CN)6] (d) [Co(NH3)6]C13 nh3
10. Write the formulae of the following complexes: 16. Consider the following ethylenediamine complexes
(a) Pentamminechlorocobalt (III) ion
(b) Lithiumtetrahydroaluminate (III) O

Isomerism
11. Draw the possible geometrical isomers for the
following:
(i) [Pt(gly)2] (ii) [PtWCWaKM
Chapter 18: Coordination Compounds | 817

(b) whether the square planar complex ion


Cr (Cu(py)4f+ is diamagnetic or paramagnetic,
O Cl (c) whether the octahedral complex ion [Mn(CN)6 ]3-
CH2NH2 or tetrahedral [FeCl4 ]" has the greater number
of unpaired electrons.
ch2nh2
IV 24. Of the complex ions, [Cod-^Og]3* and [Co(en)3]3+,
(a) Which complexes are chiral, and which are one has a yellow colour in aqueous solution and the
achiral? other blue. Match each ion with a expected colour and
(b) Draw the enantiomer of each chiral complex. state your reason for doing so.
(c) Which, if any, of the chiral complexes are 25. On the basis of the spectrochemical series, determine
enantiomers of one another? whether each of the following complexes is inner
17. Which of the following complexes can exist as orbital or outer orbital and diamagnetic or
diastereoisomers? paramagnetic.
(a) [Cr(NH3)2Cl4r (a) [CufHjOJjf* (b) [MnFJ3-
(b) [Co(NH3)5Br]2+ (c) [CotCNtJ1- (d) [Cr(NH3)6]3*
(c) [FeCl2(NCS)2f- (tetrahedral) 26. Draw a crystal field energy level diagram, for each of
(d) [PtCl2Bi^ f" (square planar) the following complex ions and predict the number of
18. Two different compounds of Cr(III) have the same unpaired electrons.
empirical formula, CrCl3 -GH/). One (A) is green and (a) (CrF6]3“ (b) [Y(H2O)6]3+
the other (B) is violet. (A) on reaction with excess (c) [Fe(CN)6]3-
AgNO3 gives 1 mole of AgCl per mol of (A) while (B) on 27. The[Fe(CN)g]3- ion has magnetic moment of 1.73 BM
reaction with excess AgNO3 gives 3 moles AgCl per mol (Bohr Magneton) while [FeiH^OJg J3* has a magnetic
of (B). Deduce formula of (A) and (B). What is affect of moment of 5.92 BM. Explain.
heating on each? Also compare their conductance. 28. Among the following ions which has the highest
19. A monomeric compound of cobalt gives the following paramagnetism?
data on quantitative analysis : (a) [CrtHiOJg]3* (b) [FelH/V
Co3+ : 21.24%; NH3 : 24.77% , Cl" : 12.81% ; (c) [CufHgOlgr (d) [ZnlHjOlgf
SOj" : 34.65% ; HgO : 6.53%. 29. Identify the complexes which are expected to be
What is empirical formula of the complex ? Write coloured, explain.
possible isomers. (a) Ti(NO3)4 (b) [Cu(NCCH3)]+BF;
20. A,BandC are three complexes of chromium (III) (c) [Cr(NH3)g]3*3Cr (d) K3[VF6]
with the empiricial formula H12O6Cl3Cr. All the three 30. Select correct answer.
complexes have water and chloride ions as ligands. (I) KF combines with HF to form KHF2. The
Complex A does not react with concentrated , compound contains the species :
whereas complexes B and C lose 6.75% and 13.5% of (a) K+, F“ and H+ (b) K+ , F and HF
their original weight, respectively, on treatment with (c) K+ and[HF2r (d) [KHF]+ and F"
concentrated H2SO4. Identify A , B and C. (II) Which compound is formed when excess of KCN
is added to aqueous solution of copper sulphate?
Bonding (a) Cu(CN)2 (b) K>[Cu(CN)4]
(c) K[Cu(CN)2] (d) K3[Cu(CN)4]
21. A solution containing 1 g of the complex
[Cr(H2O)5Cl]Cl2 H2O was passed through a cation 31. In nitroprusside ion the iron and NO exist as Fe11 and
exchanger. The acid liberated was made up to 1 L. NO+ rather than Fem and NO. These forms can be
Calculate the strength of the acid formed. differentiated by
(a) estimating the concentration of iron
22. The [Ti(NCS)6 J3- ion exhibits a single absorption (b) measuring the concentration of CN"
band at 544 nm. Calculate the crystal field splitting (c) measuring the solid state magnetic moment
energy A (in kJ mol"1)- Is NCS" a stronger or weaker (d) thermally decomposing the compound
field ligand than water? Predict the colour of
32. Draw the structures of
[Ti(NCS)6]3".
[Co(NH3)6f+, [Ni(CN)4f- and [Ni(CO)4].
23. Predict
33. A metal complex having composition Cr(NH3)4Cl2Br
(a) the number of unpaired electrons expected for the
tetrahedral complex ion [CoCl4 f has been isolated in two forms (A) and (B). The form
818 | Essential Inorganic Chemistry

(A) reacts with AgNO3 to give a white precipitate (i) Pentaamminenitrito-O-cobalt (III) ion
readily soluble in dilute aqueous ammonia, whereas (j) Pentaamminenitrito-N-cobalt (III) ion
(B) gives a pale yellow precipitate soluble in 4. Using IUPAC norms write the systematic names ol
concentrated ammonia. Write the formula of (A) and the following
(B) and state the hybridisation of chromium in each.
(a) [Co(NH3)6]C13
Calculate their magnetic moments (spin-only value).
(b) [CoC1(NO2)(NH3)4]C1
34. Deduce the structures of [NiCl4 and [Ni(CN)4 f" (c) [Ni(NH3)6]Cl2
considering hybridisation of the metal ion., Calculate (d) [PtCl(NH2CH3)(NH3)2 ]C1
the magnetic moment (spin only) of the species. (e) [Mn(H2O)6r
(0 [Co(en)3]3+
Applications (g) (Ti(H2O)6]3+
35. Ethylenediamine tetraacetic acid, EDTA, in the form (h) [NiCl4f-
of its calcium salt, is administered as an antidote for (i) (Ni(CO)«J
lead poisoning. Explain why this reagent might be an 5. How many geometric isomers are possible in the
effective medicine? Why is the calcium salt following coordination entities?
administered rather than the free acid? (a) [Cr(OX)3]3- (b) [CoC13(NH3)3]
36. Explain the following observations in terms of 6. Draw the structures of optical isomers of
complex ion formation: (a) [Cr(OX)3]3-
(a) A1(OH)3 is soluble in NaOH(aq) but insoluble in (b) [PtCl2(en)2f+
NH3(a<7); (c) [CrCl2(en)(NH3)2]+
(b) ZnCO3(s) is soluble in NH3(ag) but ZnS(s) is not;
(c) CoCl3 is unstable in water solution, being 7. Draw all the isomers (geometric and optical) of
reduced to CoCl2 and liberating O2(g). On the (a) [CoCl2(en)2]+
other hand, [Co(NH3)6]Cl3 can be easily (b) [CoCl(en)2(NH3)f+
maintained in aqueous solution. (c) [CoCl2(en)(NH3)2]+
37. Potassium alum, KA1(SO4)2 12H2O is obtained in the 8. Draw the structure of
(a) Cis-dichlorotetracyano-chromate (III)
form of octahedral crystals when a solution of K2SO4 (b) Afer-triamminetrichlorocobalt (III)
and A12(SO4)3 is concentrated by evaporation. (c) Fac-triaquatrinitro-N-cobalt (III)
Suggest specific experiments to specify whether it is
a complex or double salt. 9. Write the correct formulae for the following
coordination compounds
(a) CrC^ GHjO (violet, with 3 chloride ions/unit
NCERT Problems formula)
1. Complete the following statements for the (b) CrCl3 - 6H2O (light green colour, with 2 chloride
coordination entity (complex ion) [CrCl2(OX)2]3- ions/unit formula)
(a) OX is abbreviation for.......... (c) CrCl3 • 6HgO (dark green colour, with 1 chloride
(b) The oxidation number of chromium is.......... ion/unit formula)
(c) .......... is a bidentate ligand. [Hints : Some of these compounds may exist as hydrates].
2. Specify the oxidation numbers of the metals in the 10. Aqueous copper sulphate solution (blue in colour)
following coordination entities gives
(a) [Co(CN)(H2O)(en)2f+ (b) [PtCl4 f~ (a) a green precipitate with aqueous potassium
(c) [CrCl3(NH3)3] (d) [CoB^(en)2]+ fluoride, and
(b) a bright green solution with aqueous potassium
(e) K3[Fe(CN)6]
chloride.
3. Using IUPAC norms write the formulae for the Explain these experimental results.
following
11. Discuss the nature of bonding in the following
(a) Tetrahydroxozincate (II) ion
coordination entities on the basis of valence bond
(b) Hexaammine cobalt (III) sulphate theory
(c) Potassiumtetrachloropalladate (II)
(a) [Fe(CN)6]*- (b) [FeF6]3'
(d) Potassium tris (oxalato) chromate (III)
(c) [Co(OX)3]3- (d) [CoF6]3‘
(e) Diamminedichloroplatinum (II)
(f) Hexaammineplatinum (IV) ion 12. Write the valence bond description of
(g) Potassium tetracyanonickelate (II) (a) [Ni(CN)4[3“
(h) Tetrabromocuprate (II) ion (b) [NiCl4f"
Chapter 18: Coordination Compounds | 819

13. Write the formulae of the following (e) hexamethyldialuminium


(a) methyllithium (0 trimethylboron
(b) tetramethylsilane 14. Give IUPAC name and draw the structure of
(c) trimethylbismuth (a) Ni(CO)4 (b) Fe(CO)5
(d) trimethylarsane (c) [PtCl^Hpf (d) [Cr(CO)6]

Exercise 2 i

(Stage 2: High Skill Problem Solving)


Only One Option Correct 8. Which of the following complexes exhibit optical
isomerism?
1. A compound has the empirical formula CoC13-5NH3.
(a) Jrans-tetramminedithiocyanatochromium(III)
When an aqueous solution of this compound is mixed ion
with excess silver nitrate, 2 moles of AgCl precipitate (b) cis-diamminedicarbonatocobalt (III) ion
per mol of compound. On reaction with excess HC1, no (c) irans-diamminedicarbonatocobalt (III) ion
NH4 is detected. Hence, it is (d) cis-glycinatoplatinum (II)
(a) [Co(NH3)5C12 ]C1 (b) [Co(NH3)5C1]C12
(c) [Co(NH3)5C13] (d) [Co(NH3)4C12]C1NH 3 9. What is the ratio of uncomplexed to complexed Zn2+
ion in a solution that is 10 M in NH3, if the stability
2. In the above compound in (Q. 1), constant of [Zn(NH3)4 f + is 3 x 109 ?
(a) all the Cl show primary valency (PV) (a) 3.3 x 10‘9 (b) 3.3 x 10"11
(b) two Cl show (PV) and one Cl secondary valency (c) 3.3 x 10‘14 (d) 3 x 10"13
(SV)
(c) two Cl show (PV) and one Cl (PV) as well as (SV) 10. The formula of a carbonyl complex of cobalt,
(CO)„Co—Co(CO)n,in which there a single covalent
(d) all the Cl show (SV)
Co—Co bond is
3. If there is 100% ionisation of this complex in (Q. 1) (a) Co2(CO)4 (b) Coo(CO)6
above, its 1 M solution at 300 K will have osmotic (c) Co2(CO)8 (d) Co2(CO)10
pressure equal to
(a) 8.21 atm (b) 24.63 atm 11. If excess of AgNO3 solution is added to 100 mL of a
(c) 49.28 atm (d) 73.89 atm 0.024 M solution of dichlorobis (ethylene diamine)
cobalt (III) chloride. How many moles of AgCl be
4. Why is [Ni(en)3 f+, nearly 1010 times more stable than precipitated?
(Ni(NH3)6f*? (a) 0.0012 (b) 0.0016
(a) NH3 evaporates easily and causes instability to (c) 0.0024 (d) 0.0048
[Ni(NH3)6 f+ complex
12. Among Ni(CO)4 , [Ni(CN)4 f ’ and NiCl?'
(b) Six NH3 ligands cause steric hindrance around
the Ni2+ centre (a) Ni(CO)4 and [Ni(CN)4f’ are diamagnetic and
(c) ‘en’ is a chelating ligand and forms NiCl4“ is paramagnetic
thermodynamically more stable complexes (b) Ni(CO)4 and NiCl2- are diamagnetic and
(d) NH3 is the weakest ligand known [Ni(CN)4 f “ is paramagnetic
5. Which of the following complexes is diamagnetic? (c) Ni(CO)4 is diamagnetic and [Ni(CN)4f" and
(a) [Fe(CN)6]*- (b) [Cu(NH3)4f‘ NiCl2- are paramagnetic
(c) [Ti(H2O)6]3+ (d) [Ni(en)2f* (d) NiCl2- and [Ni(CN)4f- are diamagnetic and
Ni(CO)4 is paramagnetic
6. Which of the following cations is not paramagnetic?
(a) Sc3+ (aq) (b) Ti3+(ag) 13. [Cr(NH3)5Br]Cl and [Cr(NH3)5Cl]Br can be
(c) V3+(aq) (d) Cr3+(m?) distinguished by/and isomerism shown is
(a) BaCl2, ionisation (b) AgNO3, ionisation
7. Which of the following complexes is not a chelate? (c) AgNO3, coordinate (d)BaCl2, linkage
(a) bis (dimethylglyoximato) nickel (II)
(b) potassium ethylenediamine tetrathiocyanato 14. Which is true for[Ni(en)2f+,Z(Ni) = 28 ?
chromate (III) (a) Paramagnetism, dsp2 , square planar, C.N. of
(c) tetrammine carbonatocobalt (III) nitrate Ni = 2
(d) trans-diglycinatoplatinum (II) (b) Diamagnetism, dsp2, square planar, C.N. of
Ni = 4
820 | Essential Inorganic Chemistry

(c) Diamagnetism, sp3 , tetrahedral, C.N. of Ni = 4 27. Extraction of Ag from sulphide ore and removal tn
(d) Paramagnetism, sp3, square planar, C.N. of unreacted silver from photographic plate involv-
Ni = 4 complexes
15. Which is Prussian blue? (a) [Ag(S2O3)2 ]3" in both
(a) KFe,n[Fen(CN)6] (b) KFeni[Fein(CN)6] (b) [Ag(CN)2r in both
(c) K2[Fen(CN)6] (d) K3[Fe(CN)6] (c) [Ag(S2O3)2]3-, [Ag(CN)2r
16. [Ni(CN)4 f" and [NiCl4 f~ have similarity but not in (d) [Ag(CN)2F , [Ag(S2O3)2]3"
(a) magnetic moment (b) C.N. and O.N. 28. [Co(NH3)5(NO2)f+ and[Co(NH3)5(ONO)f+ will diffei
(c) structure (d) both (a) and (c) in
17. Which has maximum EAN of the underlined atoms? (a) colour (b) structure
(Cr = 24 , Co = 27 , Fe = 26 , Ni = 28) (c) hybridisation (d) magnetic moment
(a) [CrfEDTA)]’ ' (b) [Cp(en)3]3+ 29. In which case racemic mixture is obtained on mixing
(c) [Fe(Q>04)3]3- (d) [Ni(CN)4f- its mirror images in 1: 1 molar ratio?
18. Oxidation number of Cr in the following complex is (a) [Crtenlj]3* (b) [Ni(DMG)2]
/O2X q3+ (c) Cis-[Cu (Gly)2 ] (d) In all cases
(H2O)6Cr CrfHjOg 30. E AN of the elements (*) are equal in
0—0
(a) Ni(CO4)4,[Fe(CN)6j'-
(a) 3 (b) 6
(c) 4 (d) 5 (b) [Ni(en)2J2MFe(H2O)6f!*
19. Which has maximum coordinating (donor) points? (c) [Co(CN)6]3-, [Fe(CN)6]3’
(a) DMG (b) EDTA
(d) [Ni(en)2]2t,(Sc(H2O)6]3*
(c) en (d) py
20. Following ligands can show linkage isomerism 31. Consider the following complexes:
(a) CNS (b) NO2 (1) K2PtCl6 (2) PtCl4 -2NH3
(c) CN (d) all of these (3) PtCl4 -3NH3 (4) PtCl4 -5NH3
21. [Fe’^OaXCN^Cl]4- is named as Their electrical conductances in aqueous solutions are
(a) chlorotetracyanodioxoferrate (II) ion (a) 256, 0, 97, 404 (b) 404, 0, 97, 256
(b) chlorotetracyanoperoxoferrate (II) ion (c) 256, 97, 0, 404 (d) 404, 97, 256,0
(c) chlorotetracyanosuperoxoferrate (II) ion 32. Which one of the following is an example of
(d) none of the above is correct coordination isomerism?
22. Which has maximum conductance? (a) [Co(NH3)5Br]S04 and [Co(NH3)5SO4]Br
(b) [Co(NH3)5NO2]Cl2 and [Co(NH3)5ONO]Cl2
(a) PtCl4 • 6NH3 (b) PtCl4 -5NH3
(c) [Crtti2O)6]Cl3 and [Cr(H2O)5Cl]Cl2 l^O
(c) PtCl4 -4NH3 (d) Equal (d)
23. Primary and secondary valency of Pt in [Cr(NH3)6](Co(CN)6] and [Co(NH3)6][Cr(CN)6]
[Pt(en)2Cl2]Cl2 are 33. Which of the following compounds show optical
(a) 4, 4 (b) 4, 6 isomerism?
(c) 6, 6 (d) 4, 4 I. Cis- [Co(NH3)4Cl2]+ II. Trans-tCofen^Cbr
24. Which is used in cancer-chemotherapy? III. Cis- [Co(en)2Cl2]+ IV. [Co(en)3]3+
(a) Cis-platin (b) Zeisse’s salt Select the correct answer using the codes given below
(c) Both (a) and (b) (d) None of these (a) I and II (b) II and III
(c) III and IV (d) I, III and IV
25. Which one of the following complex species does not
obey the EAN rule? 34. Lead poisoning in the body can be removed by
(a) [Cu(CN)4]3~ (b) [Cr(NH3)6]3' (a) EDTA in the form of calcium dihydrogen salt
(b) cis-platin
<c) IFefCNlJ- (d) [NifCOJJ
(c) Zeisse’s salt
26. Which has aromatic ring in complex? (d) DMG
(a) DMG in dimethyl glyoximate 35. [Pt(NH3)(NO2)Py(NH2OH)]+ will form geometrical
(b) Cyclopenta-dienyl anion in ferrocene
isomers
(c) Both (a) and (b)
(a) 2 (b) 3
(d) None of the above
(0 4 (d) 5
Chapter 18 . Coordination Compounds | 821

36. The coordination number of Pt in the complex ion (c) It has magnetic moment of 3.87 BM confirming
[Pt(en)2Cl2f+is three unpaired electrons in Fe
(a) 3 (b) 4 (d) All the above are correct statements
(c) 5 (d) 6 46. While Ti3*,V3*.Fe3+ and Co2+ afford a large number
37. Of the following complex ions one exhibits optical of tetrahedral complexes, Cr3+ never does this, the
isomerism. That one is reason being
(a) cis- [Co(en)2Cl2]+ (b) [Co(NH3)4C12]+ (a) Cr3* forces h:gh crystal field splitting with a
(c) [Co(NH3)2Cl4r (d) frans-[Co(en)2Cl2]+ varieties of ligands
(b) crystal field stabilisation energy in octahedral
38. Of the following complex ions, one exhibits vis-a-vis tetrahedral Cr3*" system plays the
isomerism. That is deciding role
(a) [Ag(NH3)2]+ (b) [Co(NH3)5NO2f* (c) the ionic radius of Cr3+ is the largest among the
(c) [Pt(en)Cl2] (d) (Co(NH3)6Clf* other M3+ ions mentioned
39. The number of unpaired electrons expected for the (d) electronegativity of Cr3+ is the largest among
complex ion [Cr(NH3)6f+ is these trivalent 3d-metals and so chromium
(a) 2 (b) 3 prefers to be associated with as many ligands as
its ionic radius permits
(c) 4 (d) 5
47. Isomerisms exhibited by [Cr(NH3)2(H2O)2Cl2]+ are
40. Of the following complex ions, one is a
Bronsted-Lowry acid. That one is (a) ionisation, optical
(b) hydrate, optical
(a) [Cu(NH3)4f+ ' (b) [FeCl4r (c) geometrical, optical
(c) [Petting ]3+ (d) [Zn(OH)4f- (d) coordinate, geometrical
41. Of the following complex ions, the one that probably
48. The crystal field-splitting for Cr3+ ion in octahedral
has the largest overall formation constant, K?, is
field increases for ligands F, HoO, NH3, CN“ and the
(a) [Co(NH3)6]3+ (b) [Co(H2O)6]3+ order is
|3+
(c) [Co(H20)4(NH3)2]3+ (d) [Co(en)3]3+ (a) F<H2O<NH3< CN"
42. When concentrated HC1 is added to a, solution of (b) CN“ < I" < l^O < NH3
[CotH^Olg f+ ion, an intense blue colour develops due (c) CN"<NH3 <H20<F
to the formation which one of the following?
(d) NH3 < HgO < F < CN~
(a) [CoCig]4- (b) [CoCl4f-
49. Which of the following types of bonds are present in
(c) (CoC12(H20)4 ] (d) [CoCKHLjOs]+
CuSO4 SHoO?
43. Select the correct statement. I: Electrovalnt II: Covalent III: Coordinate
(a) Complex ion [MoC16]3" is paramagnetic Select correct answer out of
(b) Complex ion [Co(en)3]3+ is diamagnetic (a) I, II (b) II, HI
(c) Both (a) and (b) are correct (c) I, III (d) I, II, III
(d) None of the above is correct 50. Arrange the following in order of decreasing number
44. Match the geometry (given in column A) with the of unpaired electrons:
complexes (given in column B) in I: [Fe(H2O)6f* II: [FetCNlgl3-
III: [Fe(CN)6]*- IV: [FedloOg]3*
Geometry: A Complex: B
(a) IV, I, II, III (b) I, II, III, IV
I: Octahedral P: [Ni(CN)J2-
(c) III, II, I, IV (d) II, III, I, IV
II: Square planar Q: Ni(CO)4
51. In an octahedral complex, if ligands on one axis are
III: Tetrahedral R: [Fe(CN)6]4'
displaced little away from their ideal positions, the
I II III I II III crystal field splitting of d-orbitals for this complex is
(a) P as given below
Q R (b) R P Q
d2 2 do dn dV7 dr.
(c) R Q P (d) Q P R
45. The complex [Fe(H2O)5NOf+ is formed in the brown The ligands are displaced on which axis/ axes
ring test for nitrates when freshly prepared FeSO4 (a) x- and y (b) y-only
solution is added to aqueous solution of NO3 followed (c) x-only (d) z-only
by addition of cone. Select correct statement 52. Which one of the following compounds has
about this complex. tetrahedral geometry?
(a) Colour change is due to charge transfer (a) [Ni(CN)4f- (b) [NiCl4f-
(b) It has iron in +1 oxidate state and nitrosyl asNO+ (d) [Pd(CN)4f-
(0 [PdCl4f-
822 | Essential Inorganic Chemistry

53. Aqueous Fe(III) ion develops intense red colour with (a) Py, Br', NH3 (b> Br', Py, NH3
SCN- while Fe(II) does not. It is due to (c) Br", NHj, Py (d) NH3, Br", Py
(a) Fe(III) ion forms a charge transfer complex with 62. Consider the following statements in respect
SCN- ions [CoCig]4- complex ion
(b) Fe(III) is reduced to Fe(I) which is deep red in I: It is paramagnate
colour II: It is low-spin complex
(c) SCN- ion oxidises to CN“ ion that form red III: Oxidation number of cobalt is-4
complex with Fe(III) ion IV : The coordination number of cobalt is 6
(d) SCN- ion does not form any complex with Fe(II)
Select correct statements.
ion (a) all of the above (b) III and IV only
54. The intense blue colour of Prussian blue salts arises (c) I and IV only (d) I and II only
from which one of the following?
(a) d-d transition 63. The reaction between metallic silver and aquecc
(b) inter valence electron transfer NaCN forming a soluble complex occurs in th
(c) ligand to metal charge transfer presence of
(d) metal to ligand charge transfer (a) nitrogen (b) helium
(c) argon (d) oxygen
55. Which one of the following is the correct order of the
wavelengths of absorption for complexes? 64. What type of isomerism is present in the pair c
I: [NKHjO), f+ II: [Ni(NH3)6f+ complexes [Co(NH3)5Br]SO4 and [Co(NH3)5S04]Br?
III: [Ni(NO2)6f- (a) Linkage isomerism (b) Ligand isomerism
(a) I > II > III (b) II > I > III (c) Ionisation isomerism (d) Coordination
(c) III > II > I (d) III > I > II isomerism
56. What are the spin-only magnetic moments (in BM)
for Ni(II) ion in square-planar and octahedral One or More Than One Options Correct
geometry, respectively?
(a) 0, 2.83 (b) 2.83, 2.83 1. Which are correct statements?
(c) 2.83, 0 (d) 0, 0 (a) [Ag(NH3)2 F is linear with sp hybridised Ag’ ioc
(b) NiCl4-, VO43- and MnO4- have tetrahedra
57. What is CFSE of a free Co(II) ion on forming the
tetrahedral chloro complex [CoCl4 f ~ (in the units of geometry
(c) [Cu(NH3)4 f+, [Pt(NH3)4 f+ and [Ni(CN)4 f"
Ao)?
(a) 0.6 (b) 1.2 dsp2 hybridisation of the metal ion
(c) 1.8 (d) 2.4 (d) Fe(CO)5 have bipyramidal structure with dsp
hybridised iron
58. Which one of the following complexes is expected to
have lowest Ao values? 2. Which of the following are paramagnetic?
(a) [Co(NH3)6]3+ (b) [CoF6]3“ (a) [Ni(CN)4f- (b) [NiCl4f-
(c) [Rh(NH3)5]3* (d) [Ir(NH3)6]3+ (c) [C0FJ3- (d) [Co(NH3)6]3*
59. If Ao and Az represent crystal field splitting energies 3. Which of the following can show coordinate!
for d-orbitals for octahedral and tetrahedral isomerism?
geometries respectively, then for d6 (high spin in (a) [Cu(NH3)4][PtCl4]
both cases), what are CFSE (ignore the pairing (b) [Fe(NH3)612[Pt(CN)6]3
energy) respectively? (c) [Co(NH3)6][Cr(C^O4)3]
(a) 0.6 Ao and 0.6 At (b) 0.4 Ao and 0.4 A, (d) [Pt(en)3](SO4)2
(c) 0.4 Ao and 0.6 A, (d) 0.6 Ao and 0.4 Af 4. 0.0012 mole of CrCl3 OH^O was passed through 1
60. The complex [Co(NH3)5(NO2)f+ and cation exchange resin and acid coming out of 1
[Co(NH3)5(ONO)f+ are called required 28.5 mL of 0.125 M NaOH. Hence, compk
(a) ionisation isomer (b) linkage isomers is
(c) coordination isomer (d) geometrical isomer (a) [Crtf^OgCUClg I^O (b) [CrfH^AW
61. The complex (c) [Cr(H2O)6]Cl3 (d) [Cr^OJgClgW
Cl Br 5. Which is/are correct statement(s)?
(a) [Co(en)3][Cr(CN)6] will display coordinatia
H3N
isomerism
py
(b) [Mn(CO)5(SCN)] will display linkage isomerism
can be formally formed from K^lPtCl,, ] by which one (c) [Co(NH3)5(NO3)]SO4 will display ionisata
of the following sequences of substitution of the isomerism
chloride ions? (d) None of the above is correct
Chapter 18 : Coordination Compounds | 823

6. Identify the complexes which is/are expected to be Passage Comprehension Questions


coloured
(a) Ti(NO3)4 (b) [Cu(NCCH3)]+BF4- Passage 1
* (c) [CrtNH3)6]CI3 (d) K3[VF6] Three different compounds (A, B and C) are known to
7. Nitroprusside ion is have the empirical formula CrCl3 •
A: [FeII(CN)5NO+f+ and not When exposed to a dehydrating agent
B: [FeHI(CO)5NOf+.A and B can be differentiated by A loses 2 moles of water per mole of the compound (X).
(a) estimating the concentration of iron B loses 1 mole of water and compounds C loses no
(b) measuring the concentration of ON water. Answer the following questions.
(c) measuring the magnetic moment 1. Thus, A, B and C are
(d) thermally decomposing the compound A B C
8. The effective atomic number of Co(CO)4 is 35 and (a) [Cr(H2O)4Cl2]Cl-2H2O [Cr(H2O)5Cl]Cl2 H2O
[Cr(H2O)6]Cl3
1 hence, is less stable. It attains stability by
fl (a) oxidation of Co (b) reduction of Co (b) [Cr(H2O)2Cl3]-4H2O [Cr(H2O)Cl3]-5H2O
(d) tetramerization [CrCl3]-6H2O
(c) dimerization
(c) [CrtH2O)3Cl3]-3H2O [Cr(H2O)4Cl2l-2H2O
9. Bidentate ligands are [Cr(H2O)6]Cl3
(a) CgO^- (oxalate) [Cr<H2O)5Cl]Cl2H2O
i! (b) en (ethylenediamine) (d) [CrfH2O)6]Cl3
[Cr(H2O)4Cl2]Cl-2H2O
(c) DMG (dimethyl glyoxime)
(d) Gly (glycine) 2. Which is most conducting in aqueous solution?
10. Which of the following complexes are chelates? (a) A (b) B
(a) 6is-(dimethyl glyoximato) nickel (II) (c) C (d) Equal
(b) Potassium ethylenediaminetetracyanato 3. One mole each of A, B and C is treated with excess of
chromate (III) AgNO3. There is formation of white precipitate of
(c) Tetraamminedicyanocobalt (III) nitrate
(d) trans-diglycinatopalladium (II) AgCl.
Moles of white precipitates are
11. CN“ is a strong field ligand. This is due to the fact that A B C
(a) it carries negative charge (a) 1 2 3
(b) it is a conjugate base of weak acid (b) 3 2 1
(c) it can accept electrons from metal species 3 2
(0 1
(d) it forms high spin complexes with metal species 1 2
(d) 3
12. Considering H2O as a weak field ligand, the number
of unpaired electrons in [MnlHjOJgf+ will be (At. no. Passage 2
of Mn = 25) The octahedral complex ions [FeCl6]3- and
(a) three (b) five [Fe(CN)6 ]3- are both paramagnetic but the former is high
(c) two (d) four
spin and the latter is low spin.
13. Which of the following coordination compounds
Based on the above study, answer the following
would exhibit optical isomerism?
(a) Pentaamminenitrocobalt(III) iodide questions
(b) Diamminedichloroplatinum(II) 1. High spin complex [FeCl6 ]3- has the d-configuration
(c) 7>ans-dicyanobis (ethylenediamine)
as
chromium(III) chloride (b)
(d) Tris-(ethylenediamine) cobalt(III) bromide (a)
14. In which of the following compounds transition metal (c) fige3 (d) e5g
may have zero oxidation state? 2. Low spin complex [Fe(CN)6]3- has the
(a) [Fe(CO)5] (b) [Ni(CN)4f“ d-configuration as
(c) Fe^ (d) CrO5 (a) fige2g (b) fig
15. Which one of the following complexes will have four (c) fige3 (d) e5
different isomers?
3. Unpaired electrons in [FeCl6 ]3“ and [Fe(CN)6]3- are
(a) [Co(en)2Cl2]Cl
(b) [Co(en)(NH3)2Cl2]Cl respectively
(c) [Co(PPh3)2(NH3)Cl2]Cl (a) 5, 2 (b) 5,1
(c) 3, 2 (d) 2, 3
(d) [Co(en)3)Cl3
824 | Essential Inorganic Chemistry

4. CFSE for the complexes are (c) [Co(NH3)6]3+ is thermodynamically ai


[FeClg]3- [Fe(CN)6]3- kinetically unstable relative to [Co(H^O)6 J3*
(a) 0 0 (d) [Co(NH3)6 ]3+ is thermodynamically stable b
(b) -2A0 0 kinetically unstable relative to [Co^OJg]3'
(0 0 -2A0 2. Which is true in terms of AG°?
(d) -2A,o -2A0
(a) AGr° is more negative than that of II
Passage 3 (b) AGj° is less negative than that of II
The Alsatian-Swiss chemist Alfred Werner pioneered (c) AGj^AGjJcO
the field of coordination chemistry in the late nineenth (d) AGj^AGjpO
century. At that time, a number of compounds of cobalt 3. Which is thermodynamically unstable and ah
(III) chloride with ammonia were known, with general kinetically labile?
formulae
(a) (CotHjOIs ]3* (b) [CoffljOV*
CoCl3 • xNH3 x = 3 to 6
(c> [Co(NH3)6]3t (d) [Co(NH3)6f‘
Treatment of these compounds with aqueous HC1 acid
did not remove the ammonia. However treatment of the Passage 5
compounds with excess of AgNO3 at 273 K gave white The octahedral complex ion [FeCl6 ]3" and [Fe(CN)s I5
precipitate of AgCl in varying ratio depending on the
are both paramagnetic. Answer the following questions.
structure. Also each complex had electrical conductivity of
varying nature. Based on the above study, answer the 1. Select correct statements.
following questions. (a) [FeCl6]3- and [Fe(CN)6]3-both are high-spu
1. Which complex has the maximum electrical complex
(b) [FeCl6 ]3“ is high-spin and [Fe(CN)613" is low-spii
conductivity?
(a) CoCl3 -3NH3 (b) CoCl3 • 4NH3 complex
(c) [FeCl6 ]3- is low-spin and [Fe(CN)6 ]3- is high-spii
(c) CoC13 -5NH3 (d) CoCl3 -6NH3
complex
2. There is no reaction of the compound with (d) [FeCl6]3- and [Fe(CN)6]3- both are low-spii
aqueous HC1 indicating that there is no free NH3- complexes
Thus, NH3 ligand is bonded to cobalt (III) by
(a) primary valency (b) secondary valency 2. [FeCl6l3- has
(c) both of these (d) None of these (a) five unpaired electrons (^e2)
3. One mole of the compound gave three moles of white (b) one unpaired electron (4^)
precipitate of AgCl, on reaction with aqueous AgNO3. (c) three unpaired electrons (4#eP
Thus, complex is (d) all the above electrons paired
(a) CoCl3 ■ 4NH3 (b) CoC13 -5NH3 3. Select correct statement.
(c) CoC13 -6NH3 (d) all of these The CFSE for
(a) [FeCl6]3- is zero (b) [Fe(CN)6]3'is-2A,
Passage 4 (c) Both are correct (d) None is correct
Consider following reactions:
I. [Co(NH3)6]3+(ag> + 6H3O*(ag) [CoOijOJ.^Iag) Passage 6
+ 6NHJ(aq) Consider following exchange reactions :
The products are favoured in the above reaction k (rate constant)
IL [Co(NH3)6ft(a9) + 6H3O*(a9) ---- > • [FefHjOjJ* +Cr [Fed^OgCir+HgO 106MV
[Cod^Ogf \aq) + 6NH;(a?) [RutHsOtsf+Cr [RudljOgCir+HjO IO'2 mV
This, reactions (II) takes place with HgO in a matter of Now answer the following questions
seconds
Based on the above reactions, answer the following 1. Rate constant of the exchange reaction with iron
questions. complex is very high. It is attributed to that Fe'
complex involving a first-row transition metal is
1. Select correct statement. (a) high-spin d6 and labile
(a) [Co(NH3)6]3+ is thermodynamically and (b) low-spin d6 and kinetically inert
kinetically stable relative to [Co(H2O)6 ]3+ (c) paramagnetic and kinetically inert
(b) [Co(NH3)6 ]3+ is thermodynamically unstable but (d) diamagnetic and kinetically inert
kinetically stable relative to [Cod^Og ]3+
Chapter 18: Coordination Compounds | 825

2. Ru2+, like other second- and third-row transition H20 <C1


metal ions
(a) forms low-spin complexes (0 Cl—Cr ?— OH2-2H2O
(b) low-spin d6 species are kinetically inert h2o ''Cl
(c) both of the above
(d) none of the above 0H2
(d) None of the above
Passage 7 5. Conducting nature of these complexes in the
Read the following experiment and answer the increasing order is
questions at the end of it. (a) A<B<C (b) C< A<B
uk student in 1895 prepared three coordination (c) B < A < C (d) C<B<A
compounds containing chromium with the following
properties:
Passage 8
Cl Ions In solution per
Formula Colour formula units Read the following experimental method and answer
the questions at the end of it.
(A) CrCl3-6H2O Violet 3
A rose-coloured compound has the empirical formula
(S) CrCl3-6H2O Light green 2 CoCl3 • 5NH3 • 11,0. Two moles of this compound react with
(C) CrCl3-6H2O Dark green 1 concentrated sulphuric acid to form HCl(g) and one mole
1. Which complex has maximum molecules of of a new compound with empirical formula
hydration? Co2(SO4)3 • 10NH3 -SHjO. When this new compound is
(a) A (b) B dried at room temperature, it loses three moles of water
(c) C (d) Equally per mol of Co2(SO4)3 • 10NH3 -Sl^O.
2. IUPAC name of the complex A is 1. What is the formula of the rose-coloured compound?
(a) hexaaquotrichlorochromium (III) (a) [Co(NH3)5C1]C12-H20
(b) hexaaquochromium(III)chloride
(b) [Co(NH3)5H2O]C12
(c) pentaaquochlorochromium(III)chloride (c) [Co(NH3)3(^O)-C12]C1
(d) tetraaquodichlorochromium(III)chloride
(d) None of the above
3. Complexes A, B, C are represented as
(a) A: [Cr(H2O)6]Cl3 2. What is the structure of the new complex formed?
B: [Cr(H2O)5Cl]Cl2 H20 (a) [Co- (NH3)5 —Co—(NH3)5 ](S04 )3 -511,0
C: [CrOHL^ClJCl- 21^0 (b) [Co—(NH3)5 -Co-(NH3)5(SO4)](SO4)2 -511,0
(b) A: [Cr(H2O)3Cl3]-3H2O
B: [Cr(H2O)4Cl2]Cl- 2H2O ZS04\
C: [Cr(H2O)5Cl]Cl2-H2O (0 (NH3)6Co' Co(NH3)4 S04 -SHjO
(c) A: [Cr(H2O)6]Cl3 S04
B: [Crd^OelClg
C: [Crd^OgJClg (d) None of the above
(d) none of the above
4. Structural formula of B indicating primary and Passage 9
secondary valencies both is Mr. William isolated two complexes and in Chemical
Cl Absract, he reported
. /0H2 “A metal complex having composition Cr(NH3)4Cl2Br
H20 : />oh2 has been isolated in two forms (A) and (B). The form (A)
(a) H20- Cr<--Cl reacts with AgNO3 to give a white precipitate readily
soluble in dilute aqueous ammonia, whereas (B) gives a
H20z I
pale yellow precipitate soluble in concentrated ammonia.
oh2 Read the above report from Chemical Abstract and
Cl answer the questions at the end of it.
0H2
1. Complex A andB are respectively
h2o. Cl (a) [Cr(NH3)4Cl2]Br,[Cr(NH3)4Br]Cl2
(b) Ct- JCrll -OH2-H2O (b) [Cr(NH3)4Br]Cl2,[Cr(NH3)4Cl2]Br
H20z (c) [Cr(NH3)4ClBr]Cl,[Cr(NH3)4Cl2]Br
''Cl
(d) [Cr(NH3)4Cl2]Br,[Cr(NH3)4ClBr]Cl
oh2
826 | Essential Inorganic Chemistry

2. Cr atom is............. hybridised 1. Species formed in experiment A does not give test oT
(a) sp3cfi in both complexes Fe2+ and CN-. It is due to formation of
(b) d2sp3 in both complexes (a) K2[Fe(CN)4] (b) K3[Fe(CN)6]
(c) sp3cft in A and cfisp3 in B (c) K3[Fe(CN)6] (d) K4[Fe(CN)6]
(d) d2sp3 in A and sp3cft in B 2. Species formed in experiment (B) is
3. EAN and magnetic moment of Cr in A and B are (a) complex (b) double salt
(a) 33, 715 BM (b) 36, 715 BM (c) liquid crystal (d) None of these
(c) 33, 724 BM (d) 36, 724 BM 3. EAN of iron formed in (A) is
(a) 26 (b) 24
Passage 10 (c) 36 (d) 38
Study the following short write up and answer the 4. When the species formed in (A) is treated withFeCl3,
questions at the end of it. a blue colour is obtained. It is due to the formation of
A, B and C are three complexes of chromium (III) with (a) Fen[Feni(CN)6r (b) FeUI[Feu(CN)6r
the empirical formula H12O6Cl3Cr. All the three complexes (c) both (a) and (b) (d) None of these
have water and chloride ions as ligands. Complex A does
not react with concentrated H2SO4, whereas complexes B Passage 12
and C lose 6.75% and 13.5% of their original weight
respectively, on treatment with concentrated H2SO4. Consider the following isomers o/’[Cr(NH3)2Cl4r ond
answer the questions
1. Complex A is
(a) [Crdi.OeJCIg
(b) [CrdiaOsCllCla H2O
(c) [CtH^ClJCl^O
ri o ?^T > Cr: •
NH3:O
(d) None of the above
2. Loss in weight on treatment with concentrated
I J T• Cl: •

I II in IV
H2SO4 is due to
(a) Cl“ is lost as HC1 1. Out of these isomers
(b) Cr3+ is lost as C^(SO4)3 (a) I and IV are trans and II and III are cis
(c) H2O is lost due to absorption (b) I and IV are cis and II and III are trans
(c) I and II are cis and III and IV are trans
(d) Cl- is lost as Cl2
(d) I and II are trans and III and IV are cis
3. Which of the above complexes is maximum hydrated?
2. Select the correct statement.
(a) A (b) B
(a) Pair of I and IV are same and pair of II and III are
(c) C (d) Equally
also same
4. Select the correct statements). (b) All have chiral centres
(a) Conductance in these complexes is in order (c) I and III are enantiomers
C<B<A (d) II and IV are enantiomers
(b) EAN and nuclear spin of chromium are identical
in each complex
(c) Non-reactivity of A with cone. J^SQj is due to
Passage 13
absence of water molecules outside coordinated Cts-diamminedichloroplatinum (II), [Pt(NH3)2Cl2 ], is
sphere HoN
o Cl
(d) AU of the above are correct
HoN
o Cl
Passage 11 One of the number of platinum coordination
Consider following experiments and answer the compounds is used in the treatment of cancer. Commonly
questions at the end of it. known as cis-platm, this compound has the ability to
(A) When Fe(CN)2 solution is treated with KCN block the uncontrolled division of cancerous cells that
solution, species formed, no longer, gives tests of Fe2+ and results in the growth of tumours. Recent studies show that
CN". cis-platin can cause serious side effects, including severe
(B) When K^SQi solution is treated with A12(SO4)3 kidney damage.
solution, species formed gives tests of K+, Al3+ and SO^". Read the above medical report and answer the
following questions.
Chapter 18: Coordination Compounds | 827

1. Cis-platin is being replaced by 1. First form of the complex is


12 + (a) A (b) B
Cl nh3 /Cl (c) C (d) None of these
(a) /P\
2. One of the complexes turns Kl-starch paper blue in
HgN NH^CH^HX vNH3
acidic medium. This complex is
trans-isomer (n = 2 to 6) (a) C (b) B
HgN- /Cl Cl\ nh3 (c) A (d) all of these
(b) ■Pt’ Pt’
HgN' ^NH2(CH2)n HjN' nh3 Passage 15
(n= 2 to 6) A research-guide instructed his two students to
synthesize complex
HgN Cl HgN\ Cl
(0 Pt’
[Co(NH3)5(NO2)]C12
h3n NH2(CH2)n HjN’ NHg They synthesised the complexes with identical
molecular formula, molar mass, geometry conductance
(d) None of the above and spin, but they differed in colour. Based on the above
facts answer the following questions.
Passage 14 1. The difference in colour is due to
Following experiment has been given to identify (a) optical isomerism
isomer. Read the experiment and answer the questions. (b) geometrical isomerism
A compound Co(en)2(NO2)2Cl has been prepared in a (c) linkage isomerism
number of isomeric forms. One form undergoes no reaction (d) nuclear isomerism
with AgNO3 or (en) and is optically inactive. A second form 2. Which of the ligands can show ambident property?
reacts with AgNO3 but not with (en) and is optically (a) NO2 (b) NH3
inactive. A third form is optically active and reacts with
(c) HaO (d) CCff
both AgNO3 and (en), complexes are:
1— —1+ 3. Complexes synthesized can be
(a) (Co(NH3)5(NO2)1C12 (b) [Co(NH3)5(ONO)]Cl2
NO2 (c) [Co(NH3)5C12]N02 (d) all of these
(A) cr
Passage 16
en_J
Read the following short write-up and answer the
questions at the end of it.
cis-bis (ethylenediamine)dinitrocobalt (III) chloride
One cationic complex has two isomers A and B. Each
has one Co3+, five NH3, one Br“ and one SO^~
NO2
stoichiometrically. A gives white ppt with BaCl2 while B
give yellow ppt with AgNO3.
(B) en, Co en cr 1. A can be
(a) [Co(NH3)5]BrSO4
(b) [Co(NH3)5SO4]Br
L no2 (c) [Co(NH3)5Br]SO4
(d) [Co(NH3)4SO4]Br-NH;3
trans-bis (ethylenediamine)dinitrocobalt (III) chloride
2. B can be
Cl (a) [Co(NH3)5S04]Br
(b) [Co(NH3)5Br]SO4
(c) [Co(NH3)3Br(SO4)i-2NH;3
(C) en, Co en no2 (d) None of the above
3. Complexes A and B have similarity in the following
but not in
L no2 J (a) molar conductance (b) van’t Hoff factor
fra7w-chloronitrobis(ethylenediamine)cobalt (III) nitrite (c) EAN (d) colour
828 | Essential Inorganic Chemistry

Assertion & Reason 8. A cyclic multidentate ligand increases the stability of


the complex.
Codes: 9. [Fe(H2O)6]2+ is an inner d-complex.
(a) Both A and R are true and R is the correct
explanation of A. 10. H2O is a weaker field ligand than NH3 and
(b) Both A and R are true but R is not the correct A„t ([Cr(NH3)6 ]3+) < Aocl ([CrCFL.Og]3*)
explanation of A. 11. Orange is the complimentary colour of blue spectral
(c) A is true but R is false. colour.
4
(d) A is false but R is true. 12. At= —Aoct
1. Assertion (A): All the iron-carbon bond distances y
in ferrocene are equal. 13. The d^, dyz and d„ orbitals lie between the axes and
Reason (R): The n-electrons in the cyclopenta­ collectively called t^g orbitals.
dienyl group of ferrocene are delocalised. 14. Geometry of[Ni(CN)4f” is tetrahedral.
2. Assertion (A): The electronic absorption 15. In the complex CoCl3 -5NH3 one Cl” ion satisfies
spectrum of the tetrahedral complex, [CoCl4 f ~ shows primary and secondary valencies of cobalt(III).
bands which are more intense compared to the
16. cis[Co(en)2Cl2]+ can have d and /-optical isomers.
absorption bands for the octahedral complex
[CofHjOlJ*. 17. [Co(NH3)5(S04)]Brand[Co(NH3)5Br]S04 are linkage
Reason (R): The octahedral complexes possess a isomers.
centre of symmetry. 18. CN ofFe in[Fe(en)3]3+ is three.
3. Assertion (A): Ni(CO)4 is a diamagnetic complex. 19. Ferrocene is stable since n-electrons in
Reason (R): All the electrons in the complex are cyclopentadienyl ion are delocalised.
paired. 20. EDTA is hexadentate ligand and is used to remove
hardness in water.
4. Assertion (A): NO2 is an ambident ligand.
Reason (R): NO2 is a Lewis base.
Fill in the Blanks
5. Assertion (A): A stable complex is formed when 1. Ligands which can be coordinated to a metal or metal
ethylene diamine (en) is added to [Ni(H2O)4 f+. ion through either of the sides are called
Reason (R): (en) being a strong ligand replaces 2. Vitamin B12 is a complex of
weak ligand H2O and thus, chelate is formed.
3. Haemoglobin is a complex of Fe(II) bound to
6. Assertion (A): CoCl3 • 4NH3 gives two ions when rings.
ionised.
4 is the number of ligating groups in a
Reason (R): Complex is [Co(NH3)4 ]C13. multidentate.
7. Assertion (A): Spot of iron-rusting can be 5. Common name of bicyclopentadienyl iron (II) is
removed by addition of oxalic acid.
Reason (R): Water soluble complex is formed with 6. 1 mol of the monohydrate complex CrCl3 -61^0 00
oxalic acid. reaction with excess of AgNO3 solution gives I
mol of white precipitate of
True & False 7. [Pt(Gly)2 ] can exist as isomers.
1. Use of cis-platin in cancer treatment also causes
severe kidney damage. 8. [Fetf^OJg ]3+ is called orbital complex.
2. Lead poisoning can be removed by use of EDTA. 9. [Fe(H2O)5NOf+ has O.N. of Fe as and thus
3. Out of [Cu(CN)4]3- and [Cd(CN)4f”, [Cu(CN)4l3- is magnetic moment of Fe is
unstable and gives black precipitate of CuS on
10. The dx 2 -y 2 and dz 2 orbitals are directed along a set of
passing HjS gas.
4. AgBr is soluble in aq. NH3 as well as in hypo. mutually perpendicular x, y and z-axes and are called
orbitals.
5. [Cu(NH3)4f+ and [Cu(CN)4f“ are labile complexes.
11. The djy, dyz and dMorbitals lie between the axes and
6. [Cu(H2O)4f+ +4NH3 ---- > [Cu(NH3)4f+ +4^0 is
spontaneous in forward side, and AG = -ve collectively called orbitals.
4
7. [Fe(CN)g]3- is more stable than [FefCbOg]4- 12. A. = - —
' 9
Chapter 18: Coordination Compounds | 829

13. Blue-green is the complimentary colour of 4. Match the complex (in List I) with the oxidation
spectral colour. number of Co (in List II).
14. Strong field ligands, such as CN’, usually produce List I List II
spin complexes and crystal field
A. [Co(NCS)(NH3)5]SO3 1.
splitting.
B. Na(Co(CO)4] 2. 0
15. Reaction between NH3 and [CudljO^ f+ is
C. Na4[Co(S2O3)3] 3. +1
16. [Cud^O^f+, [Cu(NH3)4f+ and [CuCl4f" are called D. Co2(CO)5 4. +2
complexes. 5. +3
17. Stability of a complex with multidentate ligand is
increased due to 5. Match the complex (in List I) with the type of
13. Lead poisoning can be decreased by addition of isomerism (in List II).
Ust I Ust II
19. Gold and silver are extracted by complex formation
A. [Co(NH3)4CI2] 1. Optical
with
B. c/s[Co(en)2CI2] 2. Ionisation
20. Unreacted silver bromide in photographic plate is
C. [Co(en)2(NO2)CI]SCN 3. Coordination
removed by the addition of
D. [Co(NH3)5][Cr(CN)6] 4. Geometrical

Matrix-Match Type Questions 6. Match complex ion (in List I) with number of
I. Only One Correct impaired electrons (in List II).
1. Match the complex (in List I) with the hybridisation List I List II
of the central ion (in List II). A. [Fe(H2O)6]2* 1. 0
List I B. [Fe(H2O)6]3* 2. 1
List II
C. [Fe(CN)6]4- 3. 2
A. Ni(CO)4 1. sp3^2
3d‘ 4.
D. [Fe(CN)6]3- 3
B. [Ni(CN)4]2 2. sp3 E. [Ni(H2O)4]2 + 5. 4
C. [Fe(CN)6] 3. d2sp' .3 6. 5

D. [MnF6]4" 4. dsp2
II. One or More Correct
2. Match the complex (in List I) with the geometry 1. Match the complex (in Column I) with its
(in List II). corresponding property (ies) (given in Column II).
List I Ust II Column I Column II
A. [Ni(CN)4]2- 1. Tetrahedral A. [Ni(DMG)2] 1. Chelation
B. Na3[Fe(C2O4)3] 2. Paramagnetic
B. [ZnCI4]2- 2. Trigonal bipyramidal
C. [Ni(en)2]CI2 3. Diamagnetic
C. [Fe(CO)5] 3. Square planar
D. Ni(Co)4 4. Conducting
D. [Co(NO2)6] 4. Square pyramid 5. H-bonding
5. Octahedral
2. Match the complex (in Column I) with its
3. Match the complex (in List I) with the equivalent corresponding property (ies) (in Column II).
conductance (in List II). Column I Column II
List I List II A. [Cr(NO2)2(NH3)2CI2] 1. Geometrical isomerism
A. [Pt(NH3)5CI]CI3 1. 229 B. [Cr(NH3)4CI2]Br 2. Ionisation isomerism
B. [Pt(NH3)4CI2]CI2 2. 97 C. [Cr(en)3]3* 3. Optical isomerism

C. [Pt(NH3)3CI3]CI 3. 404 D. [Pt(NH3)2CI2][Cu(CN)4] 4. Linkage isomerism


5. Coordinate isomerism
D. [Pt(NH3)6]CI4 4. 523
830 | Essential Inorganic Chemistry

Integer Answer Types 7. Square-planar [Ni(CN)4 f “ uses d-orbital(sZ


This section contains 8 questions. The answer to each in hybridisation.
of the questions is a single digit integer, ranging from 0 to 8. CFSE of high-spin cf-Mn21 complex is
9. The appropriate bubbles below the respectively
question numbers in the ORS have to be darkened. For Set II
example, if the correct answers to question number X, Y, Z This section contains 10 questions.'
and W (say) are 6, 0, 9 and 2, respectively 1. lonisable chloride ions in CoCl5 • 4NH3 are
X Y Z W
2. Magnetic moment of a complex is 4.9 B.M. Thus,
r© © © © unpaired electron(s) may be
_© (3) ©
3. Nickel dimethylglyoximate is stable due to chelation.
_© © © © Chelate ring(s) is/are
.© ® © ®
© 4. Ethylenediamine displaces H2O in the complex
_© © © © [Fe(E^O)6 ]3+ in three steps
_© ® © ® [Fe(H2O)6]3++3en ---- > [Fe(en)3]3+ + 61^0
_© © © © Stepwise formation constants are
_© ® ® ® log = 4.34
© © © ©
log ^2 =3.31
log = 2.05
Set I Overall formation constant is A x 105
1. [Co(NH3)6]3+ has unpaired electron(s) in t^g A is
orbitals. 5. B in question (4) is
2. There are electron in eg orbitals is [CoF6 ]3- 6. Ambidentate ligands out of
Py, en, CNS, CN, CNO, C^NQ,, are
3. There are electron in t^g orbitals in [CoF6 ]3-
7. EDTA is a multidentate ligand. Its deniticity
4. In the high spin complexes of Cr3+ number of (multicity) is
unpaired electron in t^g orbitals is 8. CrCl3H12O6 loses 13.5% water on treatment with
5. Total number of ions due to ionisation of the complex cone. HaSO4. Number of water molecules lost is
CoCl3 bl^Ois 9. Number of aromatic rings in ferrocene is
6. Total number of orbitals used in hybridisation of 10. Effective atomic number of Ni in Ni(CO)x is 36.
octahedral complex [Fe(CN)6]3- is Thus, x =
1
Answers & Solutions
Master Exercises
Exercise 1
Short Answer Type Questions [Fe(CN)6 f" : red colour — absorb blue-violet light
1. To obey the rule of 18 (EAN), the sum of the number of [Fe(H2O)6]3+ : pale violet — absorb red light
electrons in the metal valence shell [(n -1) dsp] and [FeF6]3- : colourless — absorb light in visible
those donated by the ligands should be 18, filling the spectrum
low-energy metal orbitals.
4. [CofNHjjlg]2* is paramagnetic due to presence of one
CutArJSd^s1 Ni[Ar]3d8 4s2
electron unpaired in presence of strong field ligand
Cr(Ar)3d5 4s2 Co[Ar]3d74s2 (low-spin complex).
Fe(Ar)3d64s2 5. Monodentate
Cu+[Ar]3d10 Ni2+[Ar]3d8 N-atom is bonded to metal ion
Fe2+[Ar]3d6 Cr3+[Ar]3d3
6. Unpaired CFSE
Fe3+[Ar]3d5 Co3+[Ar]3d6 Ligand
electron (b) (0
(a) [Cu(NH3)4]2+ 9 + 8 = 17e" I. [Fe(CN)6]3- (d5) strong (low 1 10 A

[Cu(en)3]2+ 9+12 = 21e" spin complex)


[Cu(CN)4]3- 10+ 8 = 18e" II. [Fe(H2O)6]3* (d5) weak (high 5 zero
spin complex)
Thus, [Cu(CN)4]3~
(a) [Fe(CN)6]3“ has fewer unpaired electron
(b) [Fe(CN)6]4- 6 + 12 = 18e"
[Fe(CN)6]3- 5 + 12 = 17e“ 7. Yellow

[Fe(CO)s] 8 +10 = 18e~ 8. (a) The overall charge is -1, which is equal to the
charge on the C5Hj group. Thus, the Fe atom must
Thus, [Fe(CN)6]4- and [Fe(CO)5]
have no charge. Thus,
(c) [Ni(NH3)6]2+ 8+ 12 = 20e“ Fe =[Ar]3d64s2
[Ni(CN)4]2' 8 + 8 = 16e“
(6 electrons from C5Hg) + (8 electrons from Fe) +
[Ni(CO)4] 10 + 8 = 18e" (4 electrons from 2 CO groups) = 18e“
Thus,[Ni(CO)4] (b) Mn = [Ar] 3d54s2
(d) [Cr(NH3)6]3+ 3 + 12 = 15e" Mn" =[Ar]3d64s2
[Cr(CO)6] 6 + 12 = 18e' Valence electrons in Mn~ = 8
Thus, [Cr(CO)6] Electrons donated of 5CO = 10
(e) [Co(NH3)6]3+ 6 + 12 = 18e" Thus, 8 + 10 = 18e"
[CoC14]2“ 7+8 = 15e" (c) Co = [Ar]3d74s2
Thus, [Co(NH3)6]|3+
: Co+ =[Ar]3d74s1
2. [Cu(NH3)2C12] < KNO3 < Na2[PtCl6] <[Co(NH3)6]C13 Electrons of Co+ = 8
neutral two ions three ions four ions Electrons donated by C5Hj = 6
Larger the number of ions, greater the electrical Electrons donated by two C,>H4 = 4
conductivity. Thus, 8 + 6 + 4 = 18e"
3. [Fe(CN)6]3- has the strongest field ligands of the three 9. In [NiClJ2- Cl” is a weak ligand thus electrons are
complexes; thus, its energy levels are split by the unpaired in Ni2+ making it paramagnetic
greatest amount. The frequency of the light absorbed
should be greatest, and Xmax should be the shortest for sp3-tetrahedral structure
this ion.
832 | Essential Inorganic Chemistry

When strong ligand CN~ is added, substitution 17. Refer text


takes place and electrons are paired in [Ni(CN).]2-
18. (A) + AgNO3 AgCl
dsp2 — square planar. 1 mol 1 mol
10. Blue green
Thus, (A) has only one ionisable Cl atom. Thus, A is
11. Fe2+— [Ar] 3d6 high spin — four unpaired electrons [Cr(H2O)4Cl2]Cl-2H2O
low spin — none unpaired electrons (B) + AgNO3 ---- > AgCl
1 mol 3 mol
Analytical Questions Thus, (B) has three ionisable Cl-atoms. Thus, B is
1. (i) 35 (ii) 33 (iii) 33 [Cr(H2O)6]Cl3 — an anhydrous complex.
2. (a) (b) (c) (d) (e) A: [Cr(H2O)4Cl2]Cl-2H2O,
C.N. 6 6 4 6 2 B: [Cr(H2O)6]Cl3
O.N. 2 3 2 3 1 A will change to [Cr(H2O)4 C12]C1 on heating being
hydrated complex. B has no effect of heating, all HjO
3. Ligands C.N. O.N. molecules being ligands. A will ionise as [CifHgO^ClJ
(a) NH3, NO2; 6 +3 and Cl“
(b) CN 6 +2 B will ionise as [Cr(H2O)6]3+ and 3C1" hence conductance
(c) NH3,C1 6 +3 of B is greater than that of A.
(d) NH3 4 +2 19. A : [Co(NH3 )4C1(H2O)]SO4 B : [Co(NH3 ^(SO^XHjOUCl
(e) CN 2 +1 C: [Co(NH3)4(SO4)C1] H2O
(f) en 6 +3 A and B are ionisation isomers, C is hydrate isomer each
of A andB.
4. Only two-third of the chlorine is precipitated indicating
(You can also make geometrical isomers of each)
that two chlorine atoms are outside coordinate sphere.
Thus, complex can be [Co(NH3 )5Cl]Cl2. It also ionises as 20. A does not lose water with cone. H-jSC^ indicating that
[Co(NH3)5C1]C12 [Co(NH3)5Cl]2++2Cr H2O are in coordinate sphere.
5. [Pt(NH3)6]Cl4; C.N. ofPt = 6 Thus, A is [Cr(H2O)6]Cl3
[Pt(NH3 )3C13 ]C1; C.N. of Pt = 6 Molar mass of the complex = 52 + 108 +106.5
« w, , rxv 1 23.25 n, = 266.5 g mol-1
6. Moles of the complex =------- = 0.1 266.5x93.25
233.5 After loss with cone. H2SO4 in (B) =
100
Moles of AgCl ppt = _q j
143.5 = 249.18
Thus, 1 mole of the complex gives 1 mole of AgCl Net loss =266.5-249.18
indicating only one chlorine is outside coordinate = 17.32 g = 1 molH2O
sphere. Thus, complex is [Co(NH3)4Cl2]Cl. Thus, one water molecule is lost per molecule of the
7. n = 5 8. See text complex. Thus, complex (B) is
[Cr(H2O)6Cl2]Cl2 ^0
9. (a) Pentamminecarbonatechromium (III) chloride
(b) Pentamminenitritocobalt (III) chloride After loss with cone. molar mass of (C) is
(c) Potassium hexacyahochromate (III) 266.5 x 86.5
= 100
(d) Hexammine cobalt (III) chloride
10. (a) [Co(NH3)5Cl]2+ (b) Li[AlH4] = 230.52 g moF1
Net loss =35.98 g
11. See text 12. See text 13. See text 14. See text xt u r x i i i x 35.98

15. (a) and (b) are identical Number of water molecules lost =------ = 2. Thus, two
18
(a) and (d) are geometrical isomers H2O molecules are as hydrate molecules.
(c) is distinct Thus, (Qis [Cr(H2O)4Cl2]Cl-2H2O
16. (a) I,H, IV—Chiral 21. While passing through the cation exchanger there is
II — Achiral formation of HC1. Molar mass of the complex
(b) Refer text Cr(H2O)6Cl2 = 52 +108 + 106.5
(c) I, IV are enantiomers = 266.5 g mol-1
Chapter 18: Coordination Compounds | 833

[Cr(H2O)5Cl]Cl2 H2O ---- » 2HC1 34. Also text for structures.


1 mol 2 mol
Unpaired Magnetic
1 2 i Hybridisation
----- mol electron moment
266.5 266.5
[HC1] = 7.5 x IO-3 M
[NiClJ2- sp3, tetrahedral 2 V8BM
= 7.50 x 10-3 x 36.5 gL-1 [Ni(CN)4 J2- dsp2, square planar 0 0
= 0.24 gL’1
35. EDTA can form complex with lead present in the body.
22. AE = — Calcium salt is preferred otherwise calcium of the body
X
6.62 x IO-34 x3 xlO6 will also be removed as calcium-EDTA complex (that is
harmful).
544 x 10’9
36. (a) Al3+ forms a hydroxo compound such as
= 3.65 x 10-19 J mol-1 [Al(H2O)2(OH)4r but not an ammine complex;
= 3.65 x 10"19 x 6.02 x 1023 J mol-1 (b) Solubility product (Kgp) for ZnS is much smaller
= 219.79 kJ mol-1 than for ZnCO3, and a saturated solution of ZnS does
not produce a large enough [Zn2+1 for the complex
23. (a) 3 unpaired electrons; (b) paramagnetic; ion to form;
(c) [FeCl4]’ has the greater number of unpaired (c) [Co3+ ] is kept sufficiently low in a solution of the
electrons than [Mn(CN)6]3-. stable complex ion, [Co(NH3 )6 ]3+, that it is unable to
Cl" — a weak ligand oxidise water.
CN’ — a strong ligand
37. (a) One can determine conductance;
24. [Co(en)3]3+ — yellow (b) One can determine freezing point;
(Co(H20)6]3+ —blue (c) One can detect Al3+, SO4", K*.
Since (en) produces greater d-level splitting than does
H20. NCERT Problems
25. (a) inner, paramagnetic (b) outer, paramagnetic 1. (a) oxalate (b) six (c) oxalate
(c) inner, diamagnetic (d) inner, paramagnetic 2. (a) +3 (b) +2 (c) +3 (d) +3 (e) +3

26. t t --
A t
3. (a) [Zn(OH)4]2- (b) [Co(NH3)6](SO4)3 (c) K2[PdCl4]
(d) K3[Cr(OX)3] (e) [Pt(NH3)2Cl2]
4
XxJlllL xt (f) [Pt(NH3)6]4+ (g) K2[Ni(CN)4] (h) [CuBeJ2-
(i) [Co(NH3)5ONO12+ (j) [Co(NH3)5NO2]2+
[CrFJ3- tvtHjOy3* [Fe(CN)6] 4. Refer text.
Three unpaired Two unpaired One unpaired 5. Refer text.
electrons electrons electron
6. Refer text.
27. [Fe(CN)6]3- has d 2sp3 (inner d-complex) hybridisation 7. Refer text.
with one electron unpaired while [FetfLjOJg]3* has 8. Refer text.
sp3d2 (outer d-complex) hybridisation with five
9. (a) [Cr(H2O)6]Cl3 (b) [Cr(H2O)5Cl]Cl2 H2O
unpaired electrons
(c) [Cr(H2O)4Cl2]Cl-2H2O
magnetic moment-4N(N +2) BM
10. Aqueous CuSO4 is [Cutf^O^ ]SO4 • FUO
where, N is the number of unpaired electrons.
28. (b) 29. Only (c) is correct 30. (I) (c) (II) (d) (a) [Cu(H2O)4]2+ +KF ---- > [CuFJ2-
green ppt
31. (c) They differ in number of impaired electrons.
(b) [Cu(H2O)4]2+ +KC1 [CuCl4]2"
32. Refer text light green
33. A: [Cr(NH3)4ClBr]Cl 11. Refer text.
B: [Cr(NH3)4Cl2]Br 12. Refer text.
Cr is d2sp3-hybridised 13. (a) CH3—Li (b) Si(CH3)4 (c) Bi(CH3)3
unpaired electrons N = 3 (d) As(CH3)3 (e) A1(CH3)6 (f) B(CH3)3
magnetic moment = ^N(N + 2) BM = 415 BM
14. Refer text.
834 | Essential Inorganic Chemistry

Exercise 2
Only One Option Correct
45. Fe'2+
1. (b) 2. (c)
3. (d) Complex is [Co(NH3)6C1]C12 3d6
NO ---- > NO+ + e"
+ Charge transfer
Fe2+ + e~ ---- > Fe-
[[Co(NH3 )5C1]2+ + 2C1’] y = 3 ions per mol
i = van’t Hoff factor = l + (y-l)x=3 three unpaired
k = MSTi = 1 x 0.0821 x 300 x 3
electrons
= 73.89 atm.
Thus, (d) .-. Fe+

5. (a) 6. (a) 7. (c) 8. Oj) ,7


4. (c) / 3d
9. (c) Zn2+ + 4NH, t [Zn(NH3)4]2+ from NO
K JZn(NH3)4]2* Thus, (a), (b), (c) are true.
f [Zn2+][NH3]4 Hence, (d)
46. (b) 47. (c) 48. (a) 49. (d) 50. (a)
= 3 xlO9
[Zn(NH3)4]l2+ 51. (a) 52. (b) 53. (a) 54. (b) 55. (a)
— = [NH3]4 x3 xlO9
[Zn2+] 56. (a) 57. (b) 58. (d) 59. (c) 60. (b)

= 104 x3 xlO9 61. (d) 62. (c) 63. (d) 64. (c)
= 3 xlO13
One or More Than One Options Correct
[Zn2*]
= 3.3 xlO’14 1. (a), (b), (c), (d) 2. (b), (c) 3. (a), (c) 4. (c)
[Zn(NH3)4]2+
5. (a), (b), (c) 6. (c) 7. (c) 8. (c)
Hence, (c)
9. (a), (b), (c), (d) 10. (a),(b),(d) 11. (b) 12. (b)
10. (c)
13. (d) 14. (a) 15. (b)
11. (c) 100 mL of 0.024 M = 0.0024 mol of the complex
complex is [Co(en)2Cl2 ]C1 and thus, one Cl’ is formed per Passage Comprehension Questions
mol of the complex which gives 1 mole of AgCl.
Passage 1
Hence, AgCl formed = 0.0024 mol
1. (a) 2. (c) 3. (a)
Hence, (c)
Passage 2
12. (a) 13. (b) 14. (b) 15. (a) 16. (d)
1. (a) 2. (b) 3. (b) 4. (c)
17. (b) 18. (a) 19. (b) 20. (d) 21. (c)
Passage 3
22. (a) 23. (b) 24. (a) 25. (b) 26. (b)
1. (d) 2. (b) 3. (c)
27. (d) 28. (a) 29. (a) 30. (a)
Passage 4
31. (a) Greater the number of ions, greater the
conductance. 1. (b) 2. (b) 3. (d)
1. K2[PtCl6] ’ 2K+ + [PtCl6]2’ Passage 5
three ion 1. (b) 2. (a) 3. (c)

2. PtCl4 -2NH3 [Pt(NH3)2Cl4] ---- > no ion (least) Passage 6


3. [Pt(NH3)3Cl3]Cl two ions 1. (a) 2. (c)
4. [Pt(NH3)5Cl]Cl3 four ions (maximum) Passage 7
Hence, (a) 1. (c) 2. (b) 3. (a) 4. (b) 5. (d)
32. (d) 33. (d) 34. (a) 35. (b) 36. (d) Passage 8
37. (a) 38. (b) 39. (a) 40. (c) 41. (d) 1. (b) 2. (d)
42. (a) 43. (c) 44. (b)
Chapter 18: Coordination Compounds | 835

Passage 9 2. (AM3); (BMD; (CM2); (DM5);


1. (0 2. (b) 3. (a) 3. (AM3); (BMD; (CM2); (DM4);
Passage 10 4. (AM5); (BMD; (CM4); (DM2);
5. (AM4); (BMD; (CM2); (DM3);
1. (a) 2. (c) 3. (a) 4. (d)
6. (AM5); (BM6); (CMD; (DM2);
Passage 11
(EM3)
1. (d) 2. (b) 3. (c) 4. (c)
Passage 12 II. One or More Correct
1. (b) 2. (a) 1. (AMI,3,5); (BMl.2,4); (C)—(1,3,4); (DM3);
2. (AMI,4); (BM1,2); (CM3); (DM1,4,5);
Passage 13
1. (a)
Integer Answer Types
Passage 14
Set I
1. (0 2. (a)
Passage 15
Questions —► 1 2 3 4 5 £ 7 8
Answers —► 0 2 2 3 4 6 1 0
1. (c) 2. (a) 3. (a), (b)
© © © © © ©I© ©
Passage 16 © ©. © © <3)1® ©
1. (0 2. (a) 3. (d) © ©. © ©._©_© ©
© © ©. © ©__© ©
Assertion & Reason © © © © ©__© ©
1. (b) 2. (b) 3. (a) 4. (b) 5. (a) 6. (c) © © © © © _©_© ©
7. (a) © © © © © ©._© ©
© © _©_© ©
True & False © © © © © ©)_® ©
1. T 2. T 3. F 4. F 5. T 6. T 7. T © © © © © © © ©
8. T 9. F 10. F 11. T 12. F 13. T 14. F Set II
15. T 16. T 17. F 18. F 19. T 20. T Questions —> 1 2 3 4 5 £ 7_ 8 9 10
Answers —► 3 4 4 5 9 4 6 2 2 4
Fill in the Blanks © © © © © ©I© © © ©
1. ambidentate 2. cobalt 3. porphyrin 4. Denticity © © ©.©. © ©_© © ©
5. ferrocene 7. two, AgCl 8. cis-, and trans © © © © © ©l©~ © © ©
8. outer 9. +1, 715 B.M. 10. eg 11. t2g © © © © © ©__© © © ©
13. red 14. low, large 15. spontaneous © © © © © @^© © © ©
© © © © © © © ©
16. labile 17. chelation 18. EDTA 19. CN“ ©I© © ©
© © © © © ©
20. hypo(Na2S203) © © © © © ®_® © ©
© © © © ©__© © © ©
Matrix-Match Type Questions © © © © © © © © © ©
I. Only One Correct
1. (AM2); (BM4); (CM3); (DM1);
Salt Analysis
"Salt-analysis is the "Magic-show" of chemists playing with colours."

Quick Points [ I'illJ


• Common-ion Effect Ionisation of weak electrolyte or
sparingly soluble salt is decreased in presence of common ion.
I HtffifaiRWiHL,, 1
It is called common-ion effect.
~ Ionisation of weak acid (say CH3COOH) is decreased in ■ Physical Examination
presence ofCH3COONa (with CH3COO- as common ion) ■ Analysis of Acid (Anion) Radicals
~ Ionisation of weak base (say NH4OH) is decreased in
■ Analysis of Basic (Cation) Radicals
presence of NH4C1 (with NH4 as common ion)
“ Solubility of sparingly soluble salt (say AgCl) is decreased • Dry Tests
in presence of AgNO3 (Ag+ as common ion) or KC1 (Cl‘ as ■ One Reagent forTesting Different Ions
common ion)
• Solubility product of solute A~By is represented as
XBp=[F]r[O=//sx + ?
where, s is the solubility of solute A^By (in mol L-1)
For AgCl, K,p=s2
Mg(OH)2, K,„=4s3
A1(OH)3, Asp=27s4
Fe2(SO4)3, Kflp-108s5
• Any solute AB is precipitated if
[A+][B-] >7fsp
• Salts have been divided into groups based on common-ion
effect and solubility product.
• pH = - log[H3O+]
pH + pOH = 14 = pK^
Kw = [H3O+][OH-J = 1 x 10‘14 at 298 K
• Species is said to be amphoteric if it acts as an acid as well as
a base. It is amphiprotic if can donate H* ion as well as can
accept H+
H2° H++OH“ ; H„O + H+ H,O+
acid base
838 | Essential Inorganic Chemistry

Physical Examination
Solubility Rules for Common Compounds
The following solubility rules are not comprehensive, but they cover most of the compounds you encountered is
general chemistry. Compounds that dissolve in water to the extent of approximately 0.02 mol per litre (0.02 M) ar-
usually classified as “soluble” compounds, while those that are less soluble are classified as “insoluble” compounds. N-
gaseous or solid substances are infinitely soluble in water.
1. All common inorganic acids are soluble in water. Low molecular weight organic acids are soluble.
2. All common compounds of the group LA metal (Na, K, etc.) and the ammonium ion, NH4, are soluble in water.
3. All common nitrates NO3, acetates, CH3COO~, and perchlorates, CIO4, are soluble in water.
4. (a) All common chlorides, Cl-, are soluble in water except AgCl, Hg2Cl2, and PbCl2.
(b) The common bromides, Br", and iodides, F, show approximately the same solubility behaviour as chlorides,but
there are some exceptions. As the halide ions (Cl", Br", I"), increase in size, the solubilities of their slightly
soluble compounds decrease. Although HgCl2 is readily soluble in water, HgBi^ is only slightly soluble andHgl;
is even less soluble.
(c) The solubilities of the pseudo-halide ions, CN" (cyanide) and SCN" (thiocyanate), are quite similar to those oT
the corresponding iodides. Additionally, both CN" and SCN" show strong tendencies to form soluble complex
compounds.
5. All common sulphates, SO^", are soluble in water except PbSO4, Hg2SO4, and BaSO4; CaSO4, and Ag2SO4 are
sparingly soluble.
6. All common metal hydroxides are insoluble in water except those of the group LA metals and the lower members of
the group ILA metals, beginning with Ca(OH)2.
7. All common carbonates, CO^", phosphates, PO4, and arsenates, AsO|", are insoluble in water except those of the
group LA metals and NH4 • MgCO3 is fairly soluble.
8. All common sulphides, S.2- 2" are insoluble in water except those of the group LA and group IIA metals and the
ammonium ion. Solubility and colour of different salts are given in Table 19.1
Table 19.1 Solubility and Colour of Different Salts
Soluble
Salt/Complex Formula Colour Insoluble
in water
Ammonium carbonates (NH4)2co3 white soluble
Ammonium chloride nh4ci white soluble
Ammonium nitrate nh4no3 white soluble
Ammonium phosphate (nh4)3po4 white soluble
Ammonium oxalate (NH4)2C2O4 white soluble
Ammonium sulphate (NH4)2so4 white soluble
Ammonium dichromate (NH4)2Cr2O7 orange (due to Cr2O7" ion) soluble
Ammonium molybdate (NH4)2MoO4 white soluble in presence of NH4OH
Arsenic(lll) sulphide As2S3 yellow insoluble
Arsenic(V) sulphide As2S5 yellow insoluble
Arsenic(lll) iodide AsI3 red insoluble
Antimony sulphide Sb2S3 orange insoluble
Antimonyl chloride SbOCI orange insoluble
Antimony iodide Sbl3 red-yellow insoluble
Bismuth sulphide Bi2S3 black insoluble
Bismuth iodide Bil3 black insoluble
Bismuth hydroxide Bi(OH)3 white insoluble
Bismuthyl chloride BiOCI white insoluble
Bismuth(V) oxide Bi2O5 brown insoluble
Bismuthyl iodide BiOl red-orange insoluble
Barium chloride BaCI2 white soluble
Chapter 19 . Salt Analysis | 839

Soluble
Colour Insoluble
Salt/Complex Formula
in water
1 white soluble
Barium nitrate Ba(NO3)2
BaSO4 white insoluble
Barium sulphate
BaCrO4 yellow (due to CrO4") insoluble (also in CH3COOH) but
Barium chromate
soluble in dil. HCI

Cr2O3 green insoluble


Chromium(lll) oxide
Cr2(SO4)3 green partially soluble
Chromium sulphate
CrCI3 green soluble
Chromium chloride
CrCI3 • 6H2O violet soluble
Chromium chloride (mineral)
CrO2CI2 orange red gas soluble giving H2CrO4
Chromyl chloride
Cr(OH)3 dark green insoluble
Chromium hydroxide
CrO2' yellow soluble (salts of alkali and
Chromate ion ammonium ion)

Cu2S black insoluble


Copper(l) sulphide (cupric sulphide)
CuS black insoluble
Copper(ll) sulphide (cupric sulphide)
CuSO4-5H2O blue soluble
Hydrated copper (II) sulphate
white soluble (colour changes to blue)
Anhydrous copper(ll) sulphate CuSO4
Cu(OH)2 sky blue insoluble
Copper(ll) hydroxide
CuCI2 bluish green soluble
Copper(ll) chloride (cupric chloride)
Cu2O red insoluble
Copper(l) oxide (cuprous oxide)
Cu2l2 white insoluble
Cuprous iodide
Cu2S2O3 white soluble
Cuprous thiosulphate
[Cu(NH3)4]SO4 deep blue soluble (insoluble in ethanol)
Tetrammine copper(ll) sulphate
Cu2[Fe(CN)6] brown insoluble
Cupric ferrocyanide
CuO black insoluble
Cupric oxide, (copper(ll) oxide)
Cu(CN)2 yellow insoluble
Copper(ll) cyanide
CuCN white insoluble
Cpper(l) cyanide
Cu(SCN)2 black insoluble
Copper(ll) thiocyanate
CuSCN white insoluble
Copper(l) thiocyanate
CoS black insoluble
Cobalt sulphide
K3[Co(NO2)6] yellowish orange insoluble
Potassium cobalti nitrite
CoCI2 • 2H2O brown soluble
Cobalt(ll) chloride dihydrate
CoCI2 • 4H2O brown soluble
Cobalt(ll) chloride tetrahydrate
CoCI2 • 6H2O red soluble
Cobalt(ll) chloride hexahydrate
Co(CN)2 buff-coloured insoluble
Cobalt(ll) cyanide
CdO brown insoluble
Cadmium oxide
CdS yellow insoluble
Cadmium sulphide
CdCO3 white partly soluble
Cadmium carbonate
Cd(OH)2 white insoluble
Cadmium hydroxide
Cr2O2- orange soluble
Dichromate ion
ZnS white insoluble
Zinc sulphide
insoluble, soluble in acid as Zn2*.
Zn(OH)2 white
Zinc hydroxide
soluble in alkali as ZnO2_

ZnO white (-do-)


Zinc oxide
ZnCO3 white insoluble
Zinc carbonate
PbSO3 white insoluble
Lead sulphite
PbSO4 white insoluble
Lead sulphate
PbCI2 white soluble in hot water
Lead chloride
840 | Essential Inorganic Chemistry

Soluble
Salt/Complex Formula Colour Insoluble
In water
Lead bromide PbBr2 white soluble in hot water
Lead oxide (litharge) PbO orange-red insoluble
Lead double oxide 2PbO-PbO2(Pb3O4) red insoluble
Lead dioxide PbO2 blackish brown insoluble
I
Lead carbonate PbCO3 white insoluble
Lead chromate (basic) PbO-PbCrO4 red insoluble
Lead chromate PbCrO4 yellow insoluble (soluble inHNO3 as
PbCr2O7 and in NaOH as
[Pb(OH)2-]
Lead iodide Pbl2 yellow insoluble
Lead sulphide PbS black insoluble
Lead hydroxide Pb(OH)2 white insoluble (soluble in NaOH as
[Pb(OH)4]2’
Lead cyanide Pb(CN)2 white insoluble
Lead carbonate (basic) PbO-PbCO3 white insoluble
Lead phosphate Pb3(PO4)2 white insoluble
Manganese dioxide MnO2 brownish-red insoluble
Manganese sulphide MnS buff coloured insoluble
Manganese chloride MnCI2 light pink soluble
Manganese hydroxide Mn(OH)2 light pink insoluble
Permanganate (potassium) KMnO4 violet (purple) soluble
Manganate (potassium) K2MnO4 green soluble
Manganese oxide MnO green insoluble
Manganese carbonate MnCO3 brown insoluble
Calcium chloride CaCI2 white soluble
Calcium carbonate CaCO3 white insoluble
Calcium oxalate CaC2O4 white insoluble
Calcium nitrate Ca(NO3)2 white soluble
Calcium bicarbonate Ca(HCO3)2 white soluble
Calcium sulphate CaSO4 white insoluble
Magnesium chloride MgCI2 white soluble
Magnesium ammonium phosphate Mg(NH4)PO4 white insoluble
Iron(lll) oxide Fe2O3 red insoluble
Iron(ll) carbonate FeCO3 green soluble
Iron(ll) chloride FeCI2 green soluble
Iron(ll) sulphide FeS black insoluble
I FeS2 yellow insoluble
Iron(ll) phosphate Fe3(PO4)2 brown insoluble
Iron(lll) hexacyanoferrate(ll) (Prussian Fel'WlCN),]- blue soluble, Fe4[Fe(CN)6]3
blue) ion insoluble
Iron(ll) hexacyanoferrate(lll) ion Fe"[Fe(CN)6]- blue soluble
(Turnbull’s blue)
Complex of S2- with sodium nitroprusside Na2[Fe(CN)5NOS] purple soluble
Magnetite Fe3O4 red insoluble
[Fe(CNS)]2+ blood red soluble
Iron(lll) carbonate Fe2(CO3)3 red insoluble
Iron(lll) fluoride FeF3 white insoluble
Iron(ll) fluoride FeF2 green insoluble
Chapter 19: Salt Analysis | 841

Soluble
Salt/Complex Formula Colour Insoluble
i In water
li'on(lll) oxalate Fe2(C2O4)3 blackish-red insoluble
fron(lli) hydroxide Fe(OH)3 reddish-brown insoluble
Mercury(ll) sulphide HgS black insoluble
Mercury(l) oxide Hg2O black insoluble
Mercury(ll) chloride (corrosive sublimate) HgCI2 white soluble
Mercury(l) chloride (calomel) Hg2CI2 white insoluble
Mercury(ll) bromide HgBr2 white
HgNH2CIHgO black insoluble
Mercury(ll) oxide HgO yellow insoluble
Mercury(ll) iodide Hgl2 reddish orange insoluble
Iodide of Millon's base NH2 brown insoluble
Hg
0
Hg.
I
Mercury(l) chromate Hg2CrO4 red insoluble
Mercury(l) carbonate Hg2CO3 yellowish white insoluble
Mercury(l) hydrogen phosphate HgHPO4 white insoluble
Mercury(ll) cyanide Hg(CN)2 white soluble
Nickel sulphide NiS black insoluble
Nickel chloride NiCI2 green soluble
Nickel hydroxide Ni(OH)2 green insoluble
Nickel(lll) oxide Ni2O3 black insoluble
Silver chloride AgCI white insoluble (soluble in aq. NH3 as
(Ag(NH3)2]CI)
Silver bromide AgBr yellow insoluble
Silver iodide Agl yellow insoluble
Silver sulphide Ag2S black insoluble
Silver oxide Ag2O black/brown insoluble
Silver thiosulphate Ag2S2O3 white insoluble
Silver nitrate AgNO3 white crystals soluble
Silver phosphate Ag3PO4 yellow insoluble
Silver carbonate Ag2CO3 yellowish white insoluble
Silver arsenate Ag3AsO4 red insoluble
Silver chromate Ag2CrO4 red insoluble
Silver sulphite Ag2SO3 white insoluble
Silver arsenite Ag3AsO3 yellow insoluble
Silver arsenate Ag3AsO4 brownish red insoluble
Tin(IV) sulphide SnS2 yellow insoluble
Tin(ll) chloride SnCI2 white soluble
Tin(ll) sulphide SnS brown insoluble
Tin(ll) iodide Snl2 red insoluble
Strontium carbonate SrCO3 white insoluble
Strontium sulphate SrSO4 white (insoluble in dilute mineral acid)
Strontium sulphite SrSO3 white insoluble
Strontium oxalate SrC2O4 white insoluble in CH3COOH but soluble
in mineral acids
842 | Essential Inorganic Chemistry

A Convenient Classification of Bases Metal Hydroxides that are Soluble in Excess r


1. Strong soluble bases Bases that are soluble in water and Aqueous Ammonia
are completely, or nearly completely, Some metal hydroxides dissolve in an excess -
ionised (dissociated) in dilute
aqueous solutions. The only common aqueous ammonia to form soluble complex compouncL
strong soluble bases are the The common examples are listed in the Table 19.2.
hydroxides of the group IA metals and
the hydroxides of the lower members Table 19.2 Common Metal Ions that Forms Ammine
of the group IIA metals. Ca(OH)2 and
Sr(OH)2 have limited solubilities but
Complexes with an Excess of Aqueous AmmonL
are classified as strong soluble
Insolubles hydroxides Complex Ion formed
bases. Most metal hydroxides are Metal Ions formed by limiting by excess aq. NH,
“insoluble" in water.
amount aq. NH,
2. Weak soluble bases Bases that are soluble in water but [Co(NH3)612*
which ionise only slightly in dilute
Co2+(‘) Co(OH)2
aqueous solutions. Aqueous Co3* Co(OH)3 [Co(NH3)613’
ammonia and its low molecular weight Ni2* Ni(OH)2 [Ni(NH3)6l2+
organic derivatives, the amines, are
common weak soluble bases. Cu+(“) CuOH----- > 1 Cu2O [Cu(NH3)2r

Cu2+(‘) Cu(OH)2 [Cu(NH3)4]2’


3. Insoluble bases Bases that are very slightly soluble in
water. As a matter of common usage, Ag+(”) [Ag(NH3)2r
AgOH----- » - Ag2O
very slightly soluble bases are called
"insoluble" bases. With the exception Zn2+ [Zn(NH3)4]2+
Zn(OH)2
of those listed as strong soluble
bases, all common low oxidation state Cd2* Cd(OH)2 [Cd(NH3)4]2’
metal hydroxide are “insoluble". Hg2* Hg(OH)2 [Hg(NH3)4]2+
4. Amphoteric hydroxides Insoluble bases that react with both
(Bases) acids and strong soluble bases to *Co(OH)2 and Cu(OH)2 are only slightly amphoteric, i.e., a very
complex ions of the general formula large excess of strong soluble base is required to dissolve small
[M(OH)n]where "n" varies from 3 to amounts of these insoluble hydroxides.
6 and “x" is either 1 or 2 and metalloids **CuOH and AgOH are unstable and decompose to tie
which form amphoteric hydroxides corresponding oxides.
are listed together with the formula for
the amphoteric hydroxides and the
The common ions of Rh, Ir, Pd, Pt, and Au sho>
complex ions formed in an excess of similar behaviour, but since these ions are seldom
strong soluble bases. encountered in elementary courses, they are not included
here. Note that these metals and those in the table are aD
Insoluble Complex ion formed members of the Co, Ni, Cu, and Zn families of the Periodic
Metal ion or
metalloid
amphoteric in an excess of a Table.
hydroxide strong soluble base

Be2* Be(OH)2 [Be(OH)4]2“ Colour Change on Heating


Al34 AI(OH)3 [AI(OH)4r Certain oxides change colour on heating and this fact
Cr3* Cr(OH)3 [Cr(OH)4]- can be used to identify salt.
Zn2* Zn(OH)2 [Zn(OH)4]2' Colour
Oxides
Sn2+ Sn(OH)2 in cold on heating
[Sn(OH)3]-
Sn4 + Sn(OH)4 [Sn(OH)6]2- ZnO white yellow
SnO2, Bi2O3 yellow yellowish brown
Pb2* Pb(OH)2 [Pb(OH)4]2’
Fe2O3 brown black/red
As3* As(OH)3 (As(OH)4T
PbO yellow
Sb3* Sb(OH)3 [Sb(OH)4]-

Si Si(OH)4 SiO4“ and SiOf Observation Inference

Co2* Co(OH)2 [Co(OH)4]2- (i) Sublimate formed with smell of NH3 NHJ salts
(ii) Sublimation formed without smell of NH3 Hg2* salts
Cu2+ Cu(OH)2 [Cu(OH)4]2"
(iii) Brown fumes Some nitrates
Chapter 19: Salt Analysis | 843

i< Analysis of Acid (Anion) Radicals • Soluble carbonate gives white precipitate of silver
carobnate with silver nitrate
Salt or mixture is treated with dil. and also
2Ag* + COg" Ag2CO3
with cone. HgSQi separately and by observing the types of white ppt
1 gases evolved, confirmatory tests of anions are performed.
The precipitate is soluble in nitric acid and in
ammonia. The precipitate becomes yellow or brown upon
Observation with Dilute H2SO4 addition of excess reagent owing to the forming of silver
a S. no. Observation Acid radical Confirmatory test* oxide; the same happens if the mixture is boiled:

1. Brisk co2- Gas turns lime Ag2CO3X Ag2Oi+CO2T


effervescence with (carbonate) water (or baryta
evolution of water) milky but
colourless and milkyness DEEP Focus
odourless gas disappears on • Aqueous carbonate solution is alkaline due to hydrolysis
passing gas in
(withH2O)
excess (A)*
CO2- + 2Hz~2O ---- > H2CO3 +20H-
2. Brown fumes no; Add KI and starch
(nitrite) solution—blue Alkaline solution turns pink when a drop of
colour (B)* phenolphthalein is added to it.
3. Smell of rotten S2" Gas turns lead IfCO2 (liberated from the reaction of given carbonate with
eggs (H2S smell) (sulphide) acetate paper dilute H2SO4) is passed into above pink-coloured solution, then
on heating black (C) it is decolourised by HCO3 formed with C02.
Sodium carbonate
extract (SE)t + C0,+C02’+H'290 ---- > 2HC0Z
•» U

sodium
nitroprusside Nitrite
solution — purple
colour (D) (B) 2NaNO2 + HjSO, * Na^ + 2HNO2
4. Colourless gas
with pungent smell
sof- Gas turns acidified
K2Cr2O7 solution
HNO2 * NO (colourless)
of burning sulphur (sulphite) green [(E), different 2NO + O2(air) * 2NO2 (brown)
from C03 ] since 2KI + H2SO4 + 2HNO2 ► K2SO4 + 2H.2O + 2N0
gas also turns lime
water milky (F) + I2

Solution gives CH3COO’ Aqueous solution + + starch---- > blue colour


5.
smell of vinegar (acetate) neutral FeCI3 -> When the nitrite solution is added carefully to a
blood red colour
(G) changing to saturated solution of iron(II) sulphate acidified with acetic
brownish red ppt acid or with dilute sulphuric acid, a browk ring, due to the
on boiling
compound [Fe -NO]SO4, is formed at the junction of two
6. White or yellowish S2O32' Aqueous solution + liquids.
white turbidity on (thiosulphate) AgNO3 _> white ppt N02 +CH3C00H ---- > HNO2+CH3COO“
warming due to
changing to black 3HNO2 ---- > H20+ HN03 + 2N0
colloidal sulphur
(H) on warming
Fe2++SOj'+NO ---- > [FeNO]SO4
•(A), (B) indicates reaction number given separately. Iodides, bromides, coloured ions, and anions that give
t S E is prepared by boiling mixture and Na2CO3 (1:3) in H2O coloured compounds with iron(II) ions must be absent.
and filtering. Filtrate has soluble sodium salt of anions. N02 solution also decolourises Mn04/H+
5NO2 + 2MnO4 + 6H+ ---- > 5NO3 + 2Mn2+ + SHjO
Carbonate
(A) Na2CO3 + HjSO* ---- > Na2SO4 + HjO + CO2 N02 in CH3C00H solution on treatment with
thiourea forms SCN" which gives red colour with FeCl3
Ca(OH)2 + CO2 ---- > CaCO3i + and HC1.
lime water milky
(NH^CS +HN02 ---- > N2 +H+ +SCN’ +2H,0
Ba(OH)2 +CO2 BaCO3i + HjO thiourea
baryta milky
water FeCl3+SCN- ---- > [Fe(SCN)]Cl2 +CP
blood red
CaCO3 + + CO2 —> Ca(HCO3)2
soluble
844 | Essential Inorganic Chemistry

Sulphide [Fe(S2O3)2J- + Fe3+ 2Fe2+ + S4OJ-


(C) Na2S + H2SO4 -» H2S + Na2SO4 colourless tetrathionate ion

H2S + (CH3COO)2Pb * PbS + 2CH3COOH


(black) Observation with Concentrated H2S04
(D) Na2S+Na2[Fe(CN)5NO] Na4[Fe(CN)^NOS]
sodium S. Acid Confirmatory test
nitroprusside no. Observation
radical

Sulphite 1. Colourless pungent Cl” Add MnO2 in the same


A gas giving white fumes (chloride) test tube and heat —
(E) Na2SO3 + H2SO4 ■> Na2SO4 + H2O + SO2 with pale green Cl2 gas (A),
aq NH4OH SE + HNO3 + AgNOj
Cr2O^ + 3SO2 + 2H+ 4 2Cr3+ + 3SO2- + H2O solution —* while ppi
(green) soluble in aq NH3 (B)
Chromyl chloride test
Acidified KMnO4 solution is also decolourised by SO^~ (Q
2. Reddish brown fumes Br~ AddMnO2 and heat—
bSO2^ + 2MnO4 + 6H+ ----- > 2Mn2+ +5SC^-+3H2O (bromide) yellowish brown Br,
dark pink colourless gas (D). SE +
HNO3 + AgNO3
(F) Ca(OH)2 + SO2 ----- > CaSO3 solution — pale yellow
(milky) ppt partially soluble aq
NH3 (E). Layer test (F)
Acetate 3. Violet pungent F SE + HNO3 + AgNO3
vapours turning starch (iodide) —> yellow ppt
(G) 6CH3COO' + 3Fe3+ + 2H2O----- > insoluble in aq NH3 (G)
paper blue
[Fe3(OH)2(CH3COO)6]++ 2H+ Layer test (A7)
blood red 4. Brown pungent fumes NO3 Ring test (I)
intensified by the (nitrate)
[Fe3(OH)2(CH3COO)6]+ +4H2O addition of Cu-turnings
3[Fe(OH)2CH3COO]l + 3CH3COOH + H+ 5. Colourless gases C2O4 Acidified KMnO4
turning lime water (oxalate) solution is decolorised
brownish-red ppt
milky and burning with (J) SE +
Thiosulphate blue flame CH3COOH + CaCI2
solution — white ppt
S2O|-+2H+ ---- > Si+SCV+J^O decolorising acidified
SO2 gas turns Ci2O^-/H+ solution green. KMnO4 solution (K)

(H) Na2S2O3 + 2AgNO3 ----- > Ag2S2O3X + 2NaNO3 Chloride


white
(A) KC1+H2SO4 -----> KHSO4 + HC1
Ag2SU HjSO, cone.
black
HC1 + NH3(a<7)-----> NH4C1
Lj solution (in KI) is decolourised by thiosulphate
(white fumes)
solution
i3+2S2o^~ —> s4O|-+3r 4HC1 + MnO2 MnCl2 + Cl21+2H2O
S2O^“ gives white precipitate with Pb(NO3)2 or AgCli +KNO3
(B) KCl + AgNO3
(CH3COO)2Pb; white precipitate changes to black on white ppt
boiling:
AgCl + 2NH3(oq) ♦ [Ag(NH3)2]Cl
S2O^"+Pb2+ ■» PbS2O3i soluble
white
(C) Chromyl-Chloride Test
PbS2O3 H-HgO -> PbSX+2H++SO^
Chloride + (solid) + cone. H2SO4 -^1 reddish
black
When FeCl3 solution is added to S2C>^“ solution, a brown vapours of chromyl-chloride (CrO2Cl2). Pass these
dark-violet colour appears due to formation of the complex vapours into NaOH solution, when yellow Na2CrOt
2S2O^“ + Fe3+ ----- > [Fe(S2O3)2 ] solution is formed. On adding CH3COOH and
dark-violet (CH3COO)2Pb, yellow ppt of lead chromate (PbCrO4) is
(dithiosulphatoiron (III) complex) formed.
But on standing colour disappears rapidly due to the
following change KC1 +H2SO4 KHSO4 + HC1
cone.
Chapter 19: Salt Analysis | 845

A
K2Ci^O7 +2H2SO4 2KHSO4 + 2CrO3 +HA DEEP Focus
cone. The colour is due to charge transfer. This complex formally
CrO3 + 2HC1---- > CrO2Cl2 +HA contains Fe (+1) and NO* since Fe(Il) gains one electron from
NO which changes to NO*. Its magnetic moment is 3.87 B.M.
CrO2Cl2 + 4NaOH Na2CrO4 + 2NaCl + 21^0 confirming the presence of three unpaired electrons. Nitrites
give the brown colour before H^O^ is added.
Na2CrO4 +(CH3COO)2Pb - PbCrO4X
yellow ppt NO ---- > NO* + e~
Fe2* + e~ ---- > Fe*
+ 2CH3COONa
Bromide
(D) A The ring test is unreliable in the presence of bromide,
KBr +H2SO4 KHSO4 + HBr
cone. iodide, nitrite, chlorate and chromate, hence must be removed
before ring-test of NOj is performed.
4HBr + Mn02 Be, + 21^0 + MnBi^
Oxalate
(E) NaBr + AgNO33 ---- > AgBrX +NaN03
pale yellow ppt Na2C2O4 + H2SO4 ---- > Na2SO4 + H2O + CO + .CO,
CO bums with blue flame and C02 turns lime water
AgBr + 2NH3(ag)---- > [Ag(NH3)2 ]Br
partially soluble milky.

(F) Layer Test (J) 5CA42’ +2MnO4 + 16H* ---- > 10CO2
violet
shake
SE + Cl2 water + CHC13 ■ shak4 yellowish orange
+ 2Mn2+ + 8HA
colour in CHC13 layer (CS2 or CC14 can be taken instead of colourless
CHC13)
(K) CaCl2 + Na2CA -» CaCA I + 2NaCl
2NaBr + Cl2 4 2NaCl + Bd> white ppt
orange yellow CaCA decolourises acidified KMnO4.
soluble in CHC13
In case of I-, violet colour of in CHC13 layer (H)
Specific Tests in Solution
2NaI + Cl2---- > 2NaCl + (violet) (A) Sulphate
Iodide To SE add dil. HC1 (to decompose CO^- until reaction
ceases). Add BaCl2 solution. White ppt insoluble in
(G) KI +H2SO4 KHSO4 + HI conc.HNO3.
cone.
BaCl2 + Na2SO4 ■> BaSO4 X + 2NaCl
2HI + H2SO4 ---- > (violet) + 2H2O + S02 white ppt.
(H) Layer Test See (F) above.
Hg(NO3)2 gives yellow precipitate of basic mercury (II)
Nitrate sulphate
NaNO3 +H2SO4 ---- > NaHSO4 + HN03 sot+3Hg2++2H2O HgSO4 • 2HgO + 4H*
cone. yellow
4HNO3 4NO2 + O2 + 2H2O (B) Borate
brown fumes
Ignite the mixture containing borate, cone. E^SOj and
Cu + 4HNO3 Cu(NO3)2 + 2NO2 + 2H2O ethanol in a china-dish with a burning splinter—green
(I) Ring Test To water extract (all N03 are water edged flame of ethyl borate.
soluble) add freshly prepared FeSO4 solution and then 2NagBO3 +3H2SO4 ---- > 2H;jBO3 + 3Na2SO4
cone.
cone. HjSC^ carefully by the side of the test-tube. A dark A
HgBOa +3CA0H (CqHjOJjjB + 3H,0
brown ring of [Fe(H2O)5NO]2+SO42- at the interface
ethanol ethyl borate
between the two liquids is formed. bums with green flame
(volatile)
2NaNO3 + H2SO4---- > 2NaHSO4 + 2HNO3
In presence of Cu2*, perform this test in a test-tube
2HNO3 + 6FeSO4 + 3H,SO4---- > 3Fe2(SO4)3
since Cu2+ salts are not volatile.
+ 2N0 + 4H2O
AgNO3 gives white precipitate of silver metaborate
[Fe(H2O)6]SO4 +NO [Fe(HA)5NOf+SO42- + H,0 AgBO2 from fairly concentrated borax solution, soluble in
846 | Essential Inorganic Chemistry

dilute aq. NH3 and in CH3COOH. On boiling the Arsenate ion also gives this test. Hence, presence
precipitate with H2O, a brown precipitate of Ag2O is ofphosphate should also be checked after group IL
obtained due to hydrolysis.
(D) Fluoride
B4O27-+4Ag++H2O 4AgBO2l +2H+
white Sand + salt (F“) + cone. H2SO4; heat and bring a
water-wetted rod in contact with vapours at the mouth of
2AgBO2 + 3H2O ----- > Ag2Oi + 2H3BO3
the test-tube. A white deposit on the rod shows the presence
Barium salts also give white precipitate ofBa(BO2)2 ofF".
barium metaborate.
A
(C) Phosphate NaF + H2SO4 » NaHSO4 +HF

SE + HNO3 + ammonium molybdate solution. Heat; A


SiO2 + 4HF ■> SiF4 + 2H2O
yellow crystalline ppt confirms PO4~ .
A 3SiF4 + 4H2O * 2H2SiF6 + H4SiO4
Na3PO4 + 12(NH4)2MoO4 + 24HNO3
white
(NH4)3PO4-12MoO3+ 21NH4NO3 + (silicic acid)
yellow ppt
3NaNO3 + 12H2O

Target Practice 1
1. It is experimental facts that all common inorganic acids 5. MnO~/H+is decolourised by radicals
dissolve in water. What are the ions formed if acid is HC1? (i)
(ii)
■ (iii)
2. Specify the colour and soluble/insoluble character in water
of the following 6. Two different test tubes
---- ------------------ contain CH„COO"
---------------- — 'and S,0;
(thiosulphate). Specify one reagent which can detect both.
Salt Colour Soluble/lnsoluble Reagent
(i) CuCI Product Colour
(ii) CuCl2
I. . S2O3
(iii) KI CH3COO’
II.
(iv) h
(v) l2 in KI 7. Write the formulae of
(vi) KNO3 (a) Chromyl chloride

3. Specify the cases in which soluble nature of the salt (b) Brown-coloured complex (ring) in nitrate test
changes?
(a) KCN is added to AgCl (insoluble) ’............. (c) AgCl in the form of complex with aqueous NH3
(b) NH3 is added to AgCl (insoluble)
(c) KI is added to Hgl2 (insoluble) 8. Brown-coloured ring is formed in nitrate test
(d) H„O is added BiCL (soluble) in HC1 (a) Write EC. of iron in the complex
(e) H90 is added to SbCl3 (soluble) in HC1
4. Consider following activity with salt X (b) What is EAN of iron?
X + H,0 ---- > Aqueous solution turns red litmus blue
IL I (c) What is magnetic moment of iron?
+ phenolphthalein
+ dil. H9S0, I
—•> gas ---- > pink ---- > colourless
X ---------- 2
(d) What is charge on NO?

X contains (A) ......... as acid radicals. 9. Take solution oxalate in a test-tube and heat it strongly.
Solution ofX is (B) Take a burning match-stick at the mouth of the test-tube.
You observe that a gas is buring at mouth of the test tube
Formation of pink colour with phenolphthalein indicates
but on further heating gas is put-off itself.
that aqueous solution ofX is (C)
(a) gas bums due to
Colourless solution is formed due to CD) (b) gas is put-off due to
Chapter 19: Salt Analysis | 847

IS
I ’
I Answers
1. H3O+andCr
2. (i) White insoluble Product Colour
(ii) Blue soluble
I. S2O3 [Fe(S2O3)2] dark-violet
(iii) White soluble
I II. ch3coq~ [Fe2(OH)2(CH3COO)6r blood dark
(iv) Violet insoluble
(v) Brown (Ij) soluble 7. (a) CrO2Cl2 (b) [Fe(H2O)5NO]2+ (c) [Ag(NH3)2]Cl
(vi) White soluble 8. Fe is in (+1) state
3. (a) AgCl changes to K(Ag(CN)2] (soluble) (a) [Ar] 4s°3d7
(b) AgCl change to [Ag(NH3)2]Cl (soluble)
UU T T ?
(c) Hgl2 changes to K2HgI4 (soluble) (b) EAN = Z - (O.N.) + 2 x (ligands)
(d) BiCl3 is hydrolysed to BiOCl (insoluble) = 26 - 1 + 12=37
(e) SbCl3 is hydrolysed to SbOCl (insoluble) (c) = ylNlN + 2) = V15 = 3.87 BM due to three unpaired
4. (A) CO^ (B) Basic (C) Alkaline electrons.
(D) If CO2 (CO| + dil. H2SO4) is passed into above
(d) +1
pink-coloured solution, HCO j formed decolourises it
9. (a) CO (b) CO2
CO^+CO2+H2O ---- > HCOj Note: CO (molar mass = 28) diffuses more rapidly that CO2
5. (i) Fe2+ (ii) C2O2’ (iii) NOj (molar mass = 44) hence, (CO) gas bums/ After
There are so many others. further heating CO2 goes upwards which puts-off
• 6. Reagent FeCl3 burning CO.

Practice Exercise 1
1. Oil paintings turn blackish after sometime. What is the 8. (A), (black) + dil. H2SO4 ---- > (B) (g) + (O (light green
salt formed? Assume oil paint contains Pb2+.
colour solution). Gas (B) turns lead acetate paper black.
2. A metallic statue under ‘acid-rain’ attack is turned What are (A), (B) and (C)?
bluish-green. What can be the probable metal and the ion 9. NOj interferes in the “Ring Test’ of NOj. Suggest a chemical
formed?
3. Potassium permanganate is purple in colour. On adding method of removal of NOj.
with KOH, it turns green. What is the compound formed ? 10. I" also interferes in the ‘Ring Test’ of NOj. Suggest a
4. Aqueous solution of K2Cr2O7 (orange) changes to yellow on chemical reagent that can remove I".
adding alkali. Can you explain. 11. Arrange AgF, AgCl, AgBr and Agl in the increasing order of
5. FeCl3 is yellow in aqueous solution but on passing H2S gas, solubility in water.
12. While testing oxalate, gas obtained bums with a blue flame
solution turns green. Explain.
initially but is put off instantly even as gas appears coming.
6. CO2 and SO2 both turn lime water milky. How will you Explain.
detect the presence if both are present in a mixture?
13. Can you detect Br’ and I’ by ‘Layer Test’ if present together.
7. Identify (A), (B) and (C).
14. Colourless solid (A) —(B)(g) + (C)(g). Aqueous solutions
(A) + dil. H„SO . brown colour vapours turning
Z 4 of (A) and (B) turn blue litmus red while that of (C) turns red
(KI + starch) paper blue. litmus blue. Aqueous solutions of (A) and (B) also give white
(A)+NaOH NH3(g) ppt with AgNO3 solution soluble in aqueous solution of (C).
(A) -A-> (fl) (g) + (C) (g), (but (C) liquid at room Identify (A), (B) and (C).
temperature)

Answers
1. PbS 6. Pass the mixture of gases first into acidified K 2Cr2O7 solution
2. Probable metal : Cu then into lime water.
Salt formed : Cu2+ * IfK2Cr2O7 solution turns green —SO2;
3. 2KMnO4 + 2K0H---- > 2K2MnO4 + H2O + 1()2 * If lime water turns milky —CO2
green
<rrppn " 7. (A):NH4NO2 (B):N2 (C):H2O
8. (A):FeS (B):H.,S (C):FeSO4"
4. Cr2O2- + 2OH" ---- » 2CrO^“ + H2O
orange yellow 9. Boil the mixture with NH4C1, NOj is decomposed as N2:
5. 2FeCl3 +H2S---- > 2FeCl2 + 2HC1 + S NaNO2 + NH4C1 NaCl + N, T + 2H2O
green
848 | Essential Inorganic Chemistry

10. HgCl2 removes I" as Hgl2 21" + Cl2 i2 + 2cr


HgCl2 + 2r ---- > Hgl2 I + 2C1- violet
orange ppt 2Br" + Cl2 Br2 + 2Cr
11. Agl < AgBr < AgCl < AgF orange-yellow
A H2O(Z)+CO(g)+CO2(g)
12. H2C2O4 -±> Br2 will also oxidise I" to I2. Thus, first there will be violet
colour inCHCl3 layer. Extract aqueous layer (containingBr’)
CO bums with blue flame. It diffuses faster than CO2. Once and repeat addingCl2 water andCHCl3. Orange-yellow colour
CO bums with blue flame, it is put off by CO2 which diffuses in CHC13 layer confirms Br“.
later.
14. (A): NH4C1 (B): HCI
HC1 (C):NH3
13. Reducing power of I- > Br“ > Cl“ > F“. Add Cl2 water and
CHC13 into mixture of I" and Br". nh*4 +h2o NH40H + H*
acidic

Analysis of Basic (cation) Radicals Pb2+ + 2cr -


► PbCl21
Classification into Groups Hg|+ + 2C1- ■>Hg2Cl2X
Precipitates are separated. Filtrate is for group II.
Based on the values of solubility products (jK"8P), basic ; • .■ ........................................... ’ ' ............................ ' ;

radicals are classified into groups summarised in


Table 19.3
DEEP Focus
• PbCl2 (A) is soluble in hot water and thus separated.
• AgCl is soluble in aqueous NH3 as [Ag(NH3 )2]Cl
Group I
AgCl + 2NH3----- > [Ag(NH3 )2]C1
Ag+, Pb2+, Hg|+ are precipitated as insoluble diamine silver chloride
chlorides on adding dil. HC1 to aqueous solution. If no (soluble), (B)
precipitate, this group is absent. • Hg2Cl2 is blackened by NH3
Hg2Cl2 + 2NH3 - HgNH2Cl + Hg + NH4Cl ’
Ag+ + Cl"---- > AgCl X _ u .__ _..... . .. .. . . black

Table 19.3 Classification of Basic Radicals into Groups Based on Ksp Values
Group Basic radicals Group reagent Precipitate as Explanation
I Ag*,Hg2*(l),Pb2* dil. HCI chloride K. values of chlorides are low, hence precipitated.
(AgCl,
Hg2CI2.PbCI2) Others have higher Ksp values hence not precipitated.
II Cu2*, CdI2*, Pb•2+ : . Hg2*(ll), H2S gas in presence of sulphides Ksp values of sulphides are low hence precipitated by
(CuS,
Bi3*, Sb'•3*,Sn2* , As3* As2S3 etc) low [S2-] ion. HCI (with common H* ion) decreases
dil. HCI ionisation of H2S which gives low[S2-]. Hence. II group
is precipitated. Others with higher Ksp values are not
precipitated.
III AI3*,Cr3*,Fe3* NH4OH in presence of hydroxide, AI(OH)3 X
''sp values of AI(OH)3 etc are low. NH4CI (with
NH4CI etc. common NH4 ion) decrease ionisation of NH40H
giving low [OH-].
Hence, group III is precipitated.
IV Zn2*,Ni2* ,Mn2* , Co2* H2S in ammoniacal sulphides Ksp values of sulphides of group IV are high hence
medium (ZnS etc.) precipitation takes place in higher [S2-] Basic medium
increases ionisation of H2S, thus increasing [S2-]
hence precipitation of group IV.
V Ca2* ,Ba2* ,Sr2* (NH4)2CO3+ nh4ci carbonates (CaCO3 Ksp values of carbonate are less than that of group VI
etc.) (Mg2*) hence precipitation before Mg2*.
VI Mg2*, (Na*. K+) also included NH4OH+ Na2HPO4 white ppt(MgHPO4) —
(only for Mg2*)
0 NH4* tested independently from original solution.
(zero)

Pb2+ : A : soluble PbCl2 (in hot water) solution. HNO3 are NaOH dissolve the precipitate ofPbCrO4
(A) + KI---- > yellow ppt 2PbCrO4 +2H+ 2Pb2+ + Ci2O27" +1^0
PbCl2 + 2KI ---- > Pb^ X + 2KC1 PbCrO4 + 4OH" [Pb(OH)4f“ +CrOj"
yellow ppt Ag+ : B : soluble [Ag(NH3)2 ]C1
(A) + K2CrO4 yellow ppt (B) + KI---- > yellow ppt
PbCl2 + K2CrO4 PbCrO4i + 2KCl
yellow ppt
Chapter 19: Salt Analysis | 849

[AgCNH3)2 ]C1 + KI * AgU +KC1+2NH3 BiCl3 + 0,0 ► BiOCl + 2HC1


yellow ppt white turbidity
(B) + HN03---- > white ppt SbCl3 + HjO SbOCl + 2HC1
orange turbidity
[Ag(NH3)2]Cl + 2HNO3---- > AgCl I + 2NH4NO3
white ppt BiOCl + 2HC1 ---- > BiCl3 +H^O
soluble
(B) + K2CrO4 —> brick red ppt SbOCl + 2HC1 SbCl3 + HjjO
2[Ag(NH3)2]Cl +K2CrO4 — Ag2CrO4l +2KC1 + 2NH3 soluble
brick red ppt • PbS, CdS, CuS, Bi2S3 are soluble in HNO3(1:1) while
Hg^*: Black [HgfNl^ )C1 + Hg] + aqua-regia, heat to HgS is insoluble; HgS, however, is soluble in aqua
dissolve regia.
• Add SnCl2---- > white ppt changing to grey Pb2*:
• PbS (black) is soluble in HN03 (1:1)
HN03 + 3HC1 NOCI + 2H>0 + 2C1
cone cone • 3PbS + 8HNO3 3Pb(NO3)2 + 2N0 + 3S + 4^0
soluble (A)
aqua-regia
A
(A) + KI---- » yellow ppt (Pb^)
HgtNH^Cl +Hg + 2HC1 +2C1 2HgCl2 +NH4C1 CHoCOOH
• (A) + K2CrO4 —------- > yellow ppt (PbCrO4)
2HgCl2 + SnCl2---- > Hg2Cl2 X + SnCl4
white ppt Cd2*:
Hg3Cl2 SnCl2 ■> 2Hg 1 + SnCl4 • 3CdS (yellow) + 8HNO3 3Cd(NO3)2 + 2N0
(grey) (B)
+ 41^0 + 3S
nh3 „
• Distinction: Salt + K2CrO4 A --- ^B • (B) + CH3COOH + K4 [Fe(CN)6 ]----> blpish white ppt
Pb2+ Hgj* Ag* i
Cd(NO3)2 + 2CH3COOH---- > (CH3COO)2Cd
A Yellow PbCrO4 I Red Hg2CrO41 Red Ag2CrO4 4-
+ 2HNO3
B:no change black Hgl dissolves as
[Ag(NH3)2f 2(CH3COO)2Cd + K4 [Fe(CN)6] —> Cd2[Fe(CN)6] +
bluish white ppt
4CH3COOH
Group II
Cu2+ (salt is blue)
Filtrate of group I + dil. HC1+B^O. Pass B^S gas---- >
• 3CuS (black)+ 8HNO3---- > 3Cu(NO3)2 +2NO + 3S
• yellow ppt — CdS soluble (blue)
• black ppt — PbS, HgS, Bi2S3, CuS (salt is blue) (C)
• bright yellow ppt— As2S3 + 411,0
• dark brown ppt — SnS (C) + CH3COOH + K4[Fe(CN)6]---- > reddish brown ppt
• orange ppt — Sb2S3 Cu(NO3)2 + 2CH3COOH---- >(CH3COO)2Cu+ 2HNO3
• As2S3, SnS, Sb2S3 (IIB—Arsenic group) are soluble in
2(CH3COO)2Cu + 3K4[Fe(CN)61---- > Ci^ [Fe(CN)61
yellow ammonium sulphide (YAS) or NaOH.
reddish brown ppt
As2S3 +(NH4)2S2---- >(NH4)3AsS4 +S
YAS ammonium + 4CH3COOH
thioarsenate
(O + aqueous NH3(excess)---- > deep blue
Sb2S3 + (NH4)2S2---- > (NH4)3SbS4 +S
YAS ammonium Cu2* + 4NH3(aq)---- > [Cu(NH3)4f*
thioantimonate deep blue
SnS2 + (NH4)2S2 (NH4)2SnS3 + S
YAS ammonium C+KSCN -> black ppt '0
thiostannate
Cu2* + 2SCN“ ■> Cu(SCN), X
As2S3 + 6NaOH---- > Na3AsO3 + Na3AsS3 + 3H2O black ppt
• PbS, CdS, HgS, Bi2S3, CuS (II A—Copper group) are
Bi3*:
insoluble in YAS.
• If on dilution with H^O (before passing l^S), there is • Bi2S3 (black) + 8HNO3 2Bi(NO3)3 + 2NO +
white turbidity—Bi3* confirmed, or orange turbidity— CD)
Sb3* confirmed. Turbidity disappears on adding 4^0 + 38
dil. HC1.
850 | Essential Inorganic Chemistry

(D) + NH4OH--- > white ppt of Bi(OH)3 A1(OH)3 + NaOH ■» NaA102 + 2^0
Bi(NO3)3 + 3NH4OH----- > Bi(OH)3 + 3NH4NO3 sodium meta-aluminate
(A) (soluble)
Bi(OH)3 + dil. HC1----- > ppt dissolves forming BiCi 3 White ppt reappears if NH4C1 is added into soluble
BiCl3 + Na2SnO2 --9.^ black residue meta-aluminate and boiled.
sodium stannite NH4C1+NaA102+HgO -^NaCl +A1(OH)3 X +NH3
(A) white ppt
BiCl3 + 6NaOH + 3Na2SnO2 -> 3Na2SnO3 +2Bi i Al3+ + CH3COONa-----> no ppt in cold, neutral solution
black
+ 3H2O + 6NaCl ______ blue
____ > white ppt
excess CH3COONa
Hg2* : HgS (black) is soluble in aqua regia.
Al3+ + 3CH3COO" + 2H2O-----» Al(OH)2CH3COOl
3HgS +2(HNO3 +3HC1) -A> 3HgCl2 +2NO + 3S + 4H2O + 2CH3COOH
(E) Fe3+:
aqua regia
(E) + SnCl2---- ■> white ppt changing to grey Fe(OH)3 ppt are insoluble in NaOH but soluble in
cone. HC1.
2HgCl2 + SnCl2 ---- > 2Hg2Cl2 X + SnCl4
white ppt Fe(OH)3 + 3HC1-----> FeCl3 + 3H2O
soluble (B)
Hg2Cl2 + SnCl2 -----> 2Hg + SnCl4 • (B) + CH3COOH +K4 [Fe(CN)6 ] ---- > blue ppt
grey potassium
(E) + KI * orange ppt soluble in excess of KI ferrocyanide
HgCl2 + 2KI HgLj +2KC1 A large excess of the reagent dissolves it (Prussian
orange blue) partly or entirely when an intense blue colour is
obtained.
HgI2+2KI K^Hg],
soluble 4FeCl3 +3K4[Fe(CN)6] ■» Fe4[Fe(CN)6]3 +12KC1
(Nessler's reagent) ferric ferrocyanide
(Prussian blue)
• (E) + NaOH (in small amount) — -> brownish red ppt
which changes to yellow on adding NaOH in Fe4[Fe(CN)6]3 + K4[Fe(CN)6] 4KFe[Fe(CN)6]
stoichiometric amount. soluble
(B) + KCNS — ■> red colour
Hg2+ + 2OH‘ » HgOl+H^O
yellow FeCl3 + KCNS - -> [Fe(CNS)]Cl2 + KC1
potassium blood red
sulphocyanide colour
Group III Actual composition of blood red colour is
Boil off H2S gas from the filtrate of group II. Add [Fe(CNS)(H2O)5f+
NH4C1 and one drop of dil. HNO3; heat, cool and add
• B + sodium acetate solution---- > a reddish brown
NH40H----->ppt
colour
• Reddish brown ppt of Fe(OH)3 if Fe3* is present (salt 3Fe3* + 6CH3COO“ + 21^0 ----->
is brown).
[Fe3(OH)2(CH3COO)6r +2H*
• Dirty green ppt of Cr(OH)3 if Cr3+ is present (salt is
The reaction becomes complete only if the strong acid,
green).
which is formed, is removed by the addition of an excess of
• White ppt of A1(OH)3 if Al3+ is present (salt is reagent, which acts as a buffer.
colourless). Fe2+ does not give these tests, hence at the start of this
group, HN03 is added to convert Fe2* into Fe3*.

DEEP Focus Cr3* :


• dil. HNO3 is added to convert Fe2+ (light green) into Cr(OH)3 (green ppt) is insoluble in NaOH but
dissolves in presence of Big water (oxidising agent) giving
Fe3+ (deep yellow).
yellow coloured Na2CrO4 solution.
• NH4C1 by common-ion effect decreases ionisation of Big + HgO -----» 2HBr + 0
NH.OH.
4
4NaOH + 2Cr(OH)3 + 3(0) 2Na2CrO4 + 5^0
soluble (C)
Al3*:
C + CH3COOH + (CH3COO)2Pb-----> yellow ppt
• White ppt of A1(OH)3 (salt is colourless) + NaOH
solution (excess)--> ppt dissolves forming NaA102. Na2CrO4 + (CH3COO)2Pb----- > PbCrOj + 2CH3COONa.
yellow ppt
Chapter 19 . Salt Analysis | 851

Group IV NiS + 2HCl+0


cone.
NiCl2 + HaO + S
soluble (C)
Filtrate of group III +NH4OH + NH4C1 ■^-4 passHgS (green)
gas---- > A black
• (C) + NaHCO3 + Br2 water + NaOH
• white ppt (salt colourless) of ZnS residue
• buff coloured ppt (salt light pink) of MnS
NiCl2 +2NaHCO3 NiCO3 + 2NaCl + ^0 + CO2
• black ppt (salt green) of NiS
• black ppt (salt dark blackish brown) of CoS 2NiCO3 + 4NaOH + 0 (Br^ water) — Ni2O3 X
• Ammoniacal medium increases ionisation of HjS, black
hence higher [S2“] will precipitate Zn2+, Co2+, Ni2+ + 2Na2CO3 + HjO
andMn2+ as sulphides (having higher KBp values)
(C) + NH4OH + dimethyl glyoxime---- > cherry red ppt
Zn2+ : ZnS ppt (white) are soluble in dil HCL
NiCl2 + 2CH3—C=NOH
ZnS + 2HC1---- > ZnCl2 + HjS *
(A) (O
• (A) + NaOH white ppt soluble in excess of in NH4OH CH3—C=NOH
dimethyl glyoxime
NaOH.
0--------- H—0
ZnCl2 + 2NaOH ---- > Zn(0H)9 X + 2NaCl
(A) white ppt t I
H3C—C=N\ /N=C-CH;■3
Zn(OH)2 + 2NaOH ---- > Na2ZnO2 + 2H2O Ni
sodium zincate H3C—C=N^ \N=C-CH3
White ppt of ZnS reappears on passing E^S into
0—H--------- 0
soluble Na2ZnO2 solution.
cherry red ppt
Na2ZnO2 + ELS ---- > 2NaOH + ZnS X
(white PPt) (dimethyl glyoximate)
(A) + CH3COOH + K4[Fe(CN)6 ]--- > bluish white ppt Co2+ : Like NiS, CoS (black ppt) is also soluble in
2ZnCl2 +K4[Fe(CN)6] ---- > Zn2[Fe(CN)6]X + 2KC1 cone. HC1 in presence of oxidising agent KC103
bluish white ppt CoS +2HC1 +0-^4 CoCl2 + HoO + S
Zn2+ salts form white ppt of ZnNH4PO4 on reaction cone. (Z?)
withNa2HPO4 in basic buffer (NH4C1 +NH40H) soluble
• (D) + NH4CNS crystal + ether blue colour in
ZnSO4 +Na2HPO4 +NH4C1 +NH40H ---- >
etheral layer
ZnNH4PO4 X +Na2SO4 +NH4C1 +H20
(white ppt) CoC12 +4NH4CNS (NH4),[Co(CNS)4]
blue in ethereal layer
Mn2+: MnS (buff coloured ppt) are soluble in dil. HC1 + 2NH4C1
but addition of excess of NaOH gives Mn(0H)2 ppt which • (P) + CH3COOH + KN02---- > yellowish orange ppt
changes to brown/black by atmospheric 02.
Co(NO2)2
CoCl2 + 2KNO2---- > 2KC1 + Co(NO
MnS + 2HC1 MnCl2 + H,S
CH3COOH + KN02 CH3COOK + HNO2
MnCl2 + 2NaOH Mn(0H)2 X + 2NaCl
3KNO3+Con(NO2)2 +2HNO2 ---- >
Mn(OH)2 + 0 MnO2X + H20
air brown or black K3[Com(NO2 )6 ] X + H^O + NO T
Dissolve Mn02 or Mn(0H)2 ppt into HN03 and then potassium cobaltinitrite
add oxidising agent KC103 or PbO2; heat to boil—purple (yellowish orange ppt)
solution (B). A green solution
• (D) + NaOH + NaHC03 +6^ water —^-4
MnO2 + 2HNO3 Mn(NO3)2 +H2O+io2
z CoCl2 + NaHC03 + NaOH 2NaCl + CoC03 + ^0
2Mn(NO3)2 + 5PbO2 + 6HNO3 2HMnO4 2NaHCO3 ^-4 Na2CO3 + ^0 + C02
permanganic acid (B)
(purple) v>uvvr3 + 4na
CoC0 CO3 -A > Na4[Con(CO3)3]
2Na2vu
+ 2H2O + 5Pb(NO3)2 2Na4[Con(C03)3] + 2NaHCO3 +0-^4
5PbO2 +2Mn2+ +6H+---- > 5Pb2+ +2HMnO4 +2H00
2Na3[CoUI(CO3)3] +2Na2CO3+H2O
Ni2+ : NiS (black ppt) is soluble in cone. HC1 in (green)
presence of oxidising agent like KC1O3. sodium cobalticarbonate
852 | Essential Inorganic Chemistry
Hg.
Group V NH3 + 3NaOH + 2K2HgI4 + 0 nh2i
(i) boil off H2S Nessler’s
Filtrate of group IV ----------------------------- > white ppt reagent (A)
(ii) NHjOH, (NR, )2CO3
• BaCO3,CaCO3,SrCO3 appear as white ppt + 4KI + 3NaI + 2H2O
• Flame test Perform flame test with these group ppt Above reaction can also be written as :
Green flame — Ba2+ : NH4++2[HgI4f-+4OH- -----> HgOHg(NH2)Il
Crimson red - Sr2*
Brick red — Ca2* + 7P +3H2O
• Dissolve the ppt in CH3COOH.
MCO3 + 2CH3COOH (CH3COO)2M +H2O + CO2
(A)
Dry Tests
(M = Ca,Ba,Sr)
(A) + K2CrO4 - ■» yellow ppt(BaCrO4) Borax Bead Test
(CH3COO)2Ba + K2CrO4----- > BaCrO4 i If borax, Na2B4O7 • 10H2O is heated on the platinum
yellow ppt
loop, a transparent colourless glass like bead of sodium
+ 2CH3COOK
metaborate (NaBO2) and boric anhydride (B2O3) is formed.
• (A) + (NH4)2SO4 + white ppt(SrSO4)
A
Na2B4O7 2NaBO2 + BA
(CH3COO)2Sr + (NH4)2SO4 ----- > SrSO4 i
white ppt Characteristics coloured beads are produced with
+ 2CH3COONH4 salts of copper, iron, chromium, manganese, cobalt and
• (A) + (NH4)2C2O4----- > white ppt(CaC2O4) nickel.

(CH3COO)2Ca + (NH4)2C2O4 ----- > CaC2O4 X CuO + B2O3----- > Cu(BO2)2


white ppt copper (II) metaborate
+ 2CH3COONB4 Results have been summarised in Table 19.4.
white ppt + H+ + MnO4 — ■> pink colour is discharged
pink Table 19.4 Colour in oxidising and Reducing Flames in
Borax-Bead Test
CaQA + 2H+— + Ca2* + HAO,
5C2O42-+ 2MnO4 + 16H+ 2Mn2* +10CO2 Oxidising flame Reducing flame
pink colourless Metal
Hot Cold Hot Cold
+ 8H2O
Green Blue Colourless Opaque Copper
red-brown
Group VI(Mg2+)
Yellowish- Yellow Green Green Iron
Filtrate of group V + NHjOH + Na2HPO4----- > a fine brown
crystalline ppt on scratching the side of the test tube.
Yellow Green Green Green Chromium
MgCl2 + NHjOH+Na2HPO4 + MgfNHj )PO4 >L Violet Amethyst Colourless Colourless Manganese
white ppt (amethyst)
+ 2NaCl+H2O Blue Blue Blue Blue Cobalt
Violet Reddish Grey Grey Nickel
Group Zero (NH4+) -brown
• Salt + NaOH —gas giving white fumes with HC1 Yellow Colourless Brown Brown Mdybderun

NKfCl + NaOH ^>NH3(g) + NaCl + H2O Rose-violet Rose-violet Red Violet Gold
Yellow Colourless Yellow Yellowish- Tungsten
NH3(#)+HC1----- > NHA brown
white fumes
Yellow Pale yellow Green Bottle-green Uranium
. A add Nessler's reagent ,
• Salt + NaOH----- > gas------------------------- > brown ppt Yellow Greenish Brownish Emerald Vanadium
or brown or yellow colouration (oxydimercuri -yellow -green
ammonium iodide) called iodide of Millon's base (A). A Titanium
Yellow Colourless Grey Pale violet
is also called basic mercury (II) amido iodine.
NHjCl + NaOH -A>NH3 Orange-red Colourless Colourless Colourless Cerium
Chapter 19: Salt Analysis | 853

Microcosmic Salt Bead Test produced. If this is moistened, dipped into a little KNO3
and then into a small quantity of a manganese salt (for
A test similar to borax bead test is used for example) and the whole heated in the oxidising flame, a
identification of coloured cations if microcosmic salt, green bead of sodium manganate (Na2MnO4) is formed.
Na(NH4)HPO4 • 4H2O, is used.
MnO + Na2CO3 + A Na2MnO4 + CO2
Na(NH4)HPO4 NaPO3 + H2O?+NH3T
transparent A yellow bead is obtained with chromium salt due to
bead
formation of sodium chromate (Na2CrO4)
NaPO3 + CoO---- > NaCoP04
(blue bead) 2Cr2O3 + 4Na2CO3 + 3O2---- > 4Na2CrO4 + 40^
Results have been summarised in Table 19.5.
Flame Test
Table 19.5 Microcosmic Salt Bead Test Paste of the salt and cone. HC1 is taken into the lower
Oxidising flame Reducing flame Metal oxidising zone and colour imparted to the flame by salts is
Green when hot, blue Colourless when hot, red Copper observed; salts, particularly of group V (Ba2+, Ca2+, Sr2+),
when cold when cold are identified by colours of the flame and summarised in
Yellowish-or Yellow when hot, Iron Table 19.6.
reddish-brown when hot, colourless to green when
yellow when cold cold
Table 19.6 Flame Tests
Green, hot and cold Green, hot and cold Chromium
Violet, hot and cold Colourless, hot and cold Manganese Colour Cation
Blue, hot and cold Blue, hot and cold Cobalt Golden yellow Na*
Brown, hot and cold Grey when cold Nickel
Violet K*
Yellow, hot and cold Green when cold Vanadium
Carmine-red Li*
Yellow when hot, Green, hot and cold Uranium
yellow-green when cold Brick-red Ca2*
Pale yellow when hot, Green when hot, blue Tungsten Apple-green Ba2*. Mo2*
colourless when cold when cold
Green Cu2*,(BO33-),TI3’
Colourless, hot and cold Yellow when hot, violet Titanium
when cold Crimson-red Sr2*

The yellow colouration due to sodium masks that of


Sodium Carbonate Bead Test potassium. In such cases view the flame through cobalt
The sodium carbonate bead is prepared by fusing a glass (Table 19.7), the yellow sodium colour is absorbed
small quantity of sodium carbonate on a platinum wire, and the potassium flame appears crimson.
loop in the Bunsen flame; a white, opaque bead is

Target Practice 2
1. One reagent can be used to detect more than one ion. Give 3. Consider following experiments
colour of the precipitate/solution formed. (i) Aqueous solution of
Colour of the precipitate with
ZnCl2 + H,jS gas ---- > no precipitate
Reagent (ii) Aqueous solution of
Ag + Pb2* Hg2+ (CH3COO)2Zn + H^S gas ---- > white precipitate
(a) KI (iii) Aqueous solution of
(b) K2CrO4 (CH3COO)2 Zn + diL HC1 + H,S gas no precipitate
(c) HCI Observation in case of
2. Thioacetamide (CH3CSNH2) serves as a source of H2S. (i) is due to ...................
Write reactions. (ii) is due to ...................
(iii) is due to ...................
854 | Essential Inorganic Chemistry

4. Write the compounds formed/ separated from a mixture 6. Filtrate of group II has been given to you for analyaia of
containing Ag*, Hg2* and Pb2 r in the following scheme. group III (which may contain Fe2*, Cr3+ and Al3*! ’!
Mention systematic steps you would follow
Ag+, Hgj* Pb2+ Step I: .........................................
Step II: ..................................................................
+ dil. HC1 Step III:
7. Two students (X and Y) have salts with common cation
Fe2* but different anion. Each of them dissolved the salt in
A water, added dil. HCI, boiled and added NH4C1 and NH40H.
Student X: Light green precipitate.
Student Y: Brown precipitate.
< Add hot
Anion in X can be
.water and filter;
Anion in Y can be
Light-green precipitate in X is due to
Brown precipitate in Y is due to
8. Give the reagents to identify Fe3*
Hot water soluble Insoluble in hot water (I)
® • © (ID
(HI) ~
9. A reagent can identify Fe3*, Zn2*, *,CdCd2* and Cu2*; reagent
Add NH3 is and coloured formed with
(a) Fe3*
(b) Zn2* ........................ ;
(c) Cd2*
Soluble Insoluble (d) Cu2*
® ® 10. Write the names of reagents to identify
(a) Co2* .........
5. Write following reactions (b) Ni2*
(a) Dissolution of CuS precipitate in dil. HNO.3 11. Mn2* is identified by its oxidation to a salt giving pink
colour in solution.
(a) Pink colour is due to formation of
(b) Dissolution of HgS precipitate in aqua regia
(b) Oxidising agent used is
(c) Oxidised Mn2* changes to by KOH.
(c) Dissolution of Ag2S3 in aqueous NaOH
12. KCNS can identify following basic radicals
(I)
(d) Stannite [Sn(OH)3]_ ion disproportionates on long (ID
exposure to air
(Ill)..........................................................................•

Answers
(ii) CH3COOH does not dissolve ZnS which remains insoluble.
i. Ag* Pb2* Hg2* (iii) Medium is HCI and ZnS is dissolved.
(a) KI Agl Pbl2 Hgl2 4. (A): AgCl,PbCl2,Hg2Cl2 (B): PbCl2 !
(yellow) (yellow) orange soluble in (O: AgCl,Hg2Cl2 (D): [Ag(NH3)a]Cl
excess of KI forming (E): HgNH2Cl + Hg
K2Hgl4
5. (a) 3CuS + 8HNO3 ---- > 3Cu(NO3 )2 + 2NO + 4H2O + 3S
(b) K2CrO4 Ag2CrO4 PbCrO4 (b) 3HgS + 2(HNO3 + 3HC1) A . 3HgCl ™ . own . oq
2 + 2N0 + 3S
black red (yellow) aqua regia
(C) HCI AgCI PbCI2 HgCI2 , + 4H2O
(white) (white) (soluble)
(c) As2S3 + 6NaOH ---- > Na3AsO3 + Na3AsS3+3H2O
S O (d) 2[Sn(OH)3F ---- > Sn(s) + [Sn(OH)6]2’
II I _________ redn. |
2. CH3CNH2+2H2O ---- > CH3CO‘ + NH*4 +H2S
oxidn.
3. (i) ZnS is not precipitated in presence of HCI which is formed
in following reaction : 6. Step I: Boil offH2S
ZnCl2+H2S ---- > ZnS+2HCl Step II: Add a drop ofHNO3 (to oxidise Fe2* toFe3*)
Step III: Add NH4C1 boil, cool and add NH4OH.
Chapter 19: Salt Analysis | 855

7. Anion in X can be all soluble anions except NO3. (b) Zn2[Fe(CN)6] bluish white
Anion in Y can be NO3 (c) Cd2[Fe(CN)6] bluish white
Light green precipitate inX due toFe(OH)2, (d) Cu2[Fe(CN)6] reddish brown
Light green precipitate in Yis due toFe(OH)3. 10. (a) NH4CNS
(HNO3 formed in Y oxidises Fe2 * to Fe3+ hence, Fe(OH)3 is (b) dimethyl glyoxime (DMG) in NH4OH/alcoholic solution.
formed) 11. (a) MnO;
8. I:K4Fe(CN)6 II: KCNS III:CH3COO- (b) Br2 water or PbO2/H‘
9. Reagent :K4[Fe(CN)6] (c) K2MnO4 (green)
(a) Fe4(Fe(CN)6)3 Prussian blue 12. I:Fe3+ II:Co2* III:Cu 2 +

Practice Exercise 2
1. Identify (A), (B), (C) and (D) in the following: A gives brick red colour in flame and decolourises
Colourless salt (A) + cone. H2SO4 ---- > brown fumes MnO4/H+. Gas (O bums with blue flame. Identify (A), (B),
intensified on adding Cu turnings. (C) and (£>).
Aqueous solution (A) + Cu---- > blue coloured solution (B) 8. MC12 + K2CrO4 ---- > Yellow ppt
and metal (C). What can be AfCl2
Aqueous solution (A) + HC1 ---- > white ppt soluble in (a) if it is soluble in hot water?
aq eous NH3 forming (D)
(b) if it gives green colour in flame?
2. Colourless salt (A) white ppt (B) 9. CaSO4 is insoluble but is not precipitated when excess of
soluble in H2O soluble in hot water
(NH4),SO4 is added toCaCl2; explain.
10. Identify (A), (B), (C), (£>) and (£).
(i) FeSO4 KI
1' (ii) cone. H2SO4 (A) + NaOH ---- > B(g) + C(aq)
yellow ppt (O
brown colour ring (D) (slightly alkaline)
Identify (A), (B), (C) and (D). ,,A ■ HC1 Zn
3. Salt (A) makes part of electrode and is insoluble in water. H2O + Z)(g) white fumes B(g)
(A) is blackened by NH3 forming (B). (B) is soluble is aqua | Al, A H,0 |
regia forming (C). (C) gives orange ppt with KI but ppt -> E
dissolves in excess of KI forming (D). Identify (A), (B), (C) A
B(g) + residue (C) (green) + H20
11. A (orange)
and (D).
4. Identify (A), (B), (C) and (D) and explain reactions. NaOH
(A) (green coloured salt) + K2Cr2O7 + cone. H2SO4 —(B) ,,A

(B) (reddish brown gas) + NaOH---- > (C) (yellow coloured D(g) JHC1 > white fumes
solution) H2O
CH3COOH B(g) (E) gas (B)
(C)+(CH3COO)2Pb > (D) (yellow ppt) A
cone. HC1 NaOH, H,O.
(A) + NaOH + Br2 water (C) residue (C) ------------ > (F) G (yellow)
A
CH3COOH CH,COOH
(O + (CH3coo)2Pb —.3—(p) ------- ----------- > H (yellow)
(CH.,COO)9Pb
5. Light green solution of (A) does not give blue coloured ppt
withK4[Fe(CN)6] but on adding a drop of HNO3, blue ppt Identify (A) to (H) and explain reactions.
(B) appears. However, (A) gives blue colour (C) with 12. Colourless salt (A) gives apple green flame with cone. HC1.
K3[Fe(CN)6], Explain the formation of (B) and (C). Identify (A) on reaction with dil. HoS04 gives light brown fumes (D)
(A) if it, also gives white ppt with AgNO3 solution. turning Kl-starch paper blue.
6. (A) (yellow coloured solution) changes to light green
(A) + CH3COOH + K2CrO4 ---- > yellow ppt (B)
coloured solution (B) on passing H2S gas. (A) and (B) both
give white ppt with BaCl2 solution, insoluble in cone. HC1. (A) + H2SO4---- > white ppt (C) + (D)
A gives blue coloured ppt (C) with K4[Fe(CN)6l (B) does (D)+CH3NH2 ---- > CH3OH + H2O +gas (£)
not. What are (A), (B) and (C)? (F)+Mg -±> (F)
7. (A) (colourless) —(B) (residue) + (O (gas) + (D) (gas)
|h2O (F) + H,0 ---- > NH3
(D) Identify (A) to (F) and explain reactions.
solution of (B) ■> milky
856 | Essential Inorganic Chemistry

Answers
1. (A): AgNO3 (B):Cu(NO3)2 (C):Ag (D) :[Ag(NH3)2]Cl 7. (A):CaC2O4 (B):CaO (C):CO (P):CO2
2. (A):Pb(NO3)2 (B):PbCl2 (C):PbI2 8. (a) PbCl2 (b) BaCl2
(D):[Fe(H2O)5NO]2+ 9. CaSO4 +(NH4)2SO4 (NH4)2[Ca(SO4)2J
3. (A) :Hg2Cl2 (calomel) (B): (HgNH2Cl + Hg) soluble
(C):HgCl2 (B):K2HgI4 10. (A):NH4NO 2 (B): NH3 (C): NaNO2 (D):N2
4. (A): CrCl3 (B): CrO2Cl2 (by chromyl-chloride test of CP) (E): AIN
(C):Na2CrO4 (D):PbCrO4 11. (A): (NH4)2Cr2O7 (B):N 2 (C):Cr2O3 (B):NH3
5. (A):FeCl2 (B) :KFe111 [Fe11 (CN)6] Prussian blue, (E):A1N (F):CrCl3 (G):Na2CrO4, (H):PbCrO4
(C): KFen[Fein(CN)6], Turnbull’s blue 12. (A):Ba(NO2)2 (B):BaCrO,'4 (C):BaSO4 (P):HNO2
6. (A):FeCl3 (B):FeCl2 (C):KFem[Fen(CN^]. Prussian (E):N2 (F):Mg3N2
blue

One Reagent For Testing Different [Cu(CN)4]3 no ionisation


Ions (e) Cd2+ : Cd2+ salts form white precipitate of
Cd(CN)2 white dissolves in excess of KCN forming soluble
1. KCN tetracyanocadimate(II) ions
(a) Pb2*: Pb2+ salts form white precipitate of Cd2+ +2CN- Cd(CN)24
Pb(CN)2 with KCN which is insoluble in excess of KCN. white
Pb2+ +2CN“ ----- > Pb(CN)2>L Cd(CN)2 +2CN- ----- > [Cd(CN)4 in­
(b) Hg^ : Hg2+ salts form soluble Hg(CN)2 and a soluble
precipitate of Hg [Cd(CN)4 f ~ is unstable forming free Cd2+ which is
Hgi‘ +2CN- ----- > Hg+Hg(CN)2 precipitate as CdS (yellow) on passing H2S gas
Above reaction is an example of disproportionation [Cd(CN)4f" +H2S -----> CdS-L +2H+ + 4CN“
reaction. yellow
(c) Ag+: CN" dissolves insoluble AgCl forming
Thus, mixture of Cu2+ and Cd2+ can be identified by
soluble [Ag(CN)2 T complex formation with CN“ and then passing H2S gas.
AgCl+2CN~ ----- > [Ag(CN)2r + ci-
dicyano argentate(l) ion Cu2+ + Cd2+ [Cu(CN)4r+cd(cro,r
(soluble)
(d) Cu2* : KCN dissolves the precipitate of CuS, HS
when colourless tetracyanocuprate(I), [Cu(CN)4]3-, and Yellow precipitate Yellow ppt <—-—*
disulphide S2“ ions, are formed. (a) confirms Cd2+ if original solution is colourless
2CuS+8CN“ ----- > 2[Cu(CN)4 ]3-+ S2- (b) confirms Cd2+ andCu2+ both if original solutionis
This is an oxidation-reduction process. Cu2+ is blue.
reduced to Cu+ and sulphur is oxidised coupled with (f) Fe2+ : KCN forms yellowish-brown precipitate of
formation of a complex. Fe(CN)2 with Fe2+, which dissolves in excess of KCN
Soluble Cu2+ salts react with KCN forming yellow forming KJFefCNle ] a pale yellow coloured solution.
precipitate of copper(II) cyanide, Cu(CN)2 which quickly
decomposes into white precipitate of copper(I) cyanide, Fe2+ +2CN- -----> Fe(CN)2 4-
yellowish-brown
CuCN, and a highly poisonous, (CN)2, cyanogen gas
Cu2++2CN" ----- > Cu(CN)2i Fe(CN)2 + 4CN- (Fe(CN)6]*-
yellow hexacyanoferrate(II) ion
CN (pale yellow solution)
Cu(CN)2 2CuCN + | ?
white cn (g) Fe3* : KCN forms reddish-brown precipitate of
cyanogen Fe(CN)3 with Fe3+, which dissolves in excess of KCN due to
Excess of KCN dissolves the precipitate of CuCN formation of K3[Fe(CN)6 ] — a yellow coloured solution
forming colourless [Cu(CN)4 ]3- Fe3++3CN’ -----> ■>
Fe(CN)3 4
CuCNX+3CN' ----- > [Cu(CN)4]3~ reddish-brown
The complex is so stable that no precipitate- of CuS is • Fe(CN)3+3CN_ [Fe(CN)s]3-
hexacyanoferrate(III) ion
formed on passing H2S gas
Chapter 19: Salt Analysis | 857

(h) Co2+ : KCN forms reddish-brown precipitate of (b) Hg^* (Mercurous ion): Potassium iodide gives
Co(CN)2 with Co2+ ions; precipitate dissolves in excess of green precipitate of Hg2^ (mercury(I) iodide or mercurous
KCN forming a brown solution ofKJCoCCNJg] iodide)
Co2++2CN“ ---- > Co(CN)21 Hg|++2r Hg2I2 1
reddish-brown green
Co(CN)2+4CN“ ---- > [Co(CNL]*- If excess of KI is added, a disproportionation reaction
brown (soluble) takes place, soluble tetraiodo mercurate(II) ions and a
black precipitate of finely divided mercury being formed
If dilute/cold HC1 is added Co(CN)2 reappears as
reddish-brown precipitate Hg2fcJ.+2r --- > [Hgl,f-+Hgl
[Co(CN)6 J4- + 4H* ---- > Co(CN)21 + 4HCN Disproportionation of Hg2l2 also takes place to Hgl2
and Hg on boiling it with H2O
Brown solution of KJCotCN)6] changes to yellow on A, H2O . -
boiling in air or by addition ofH2O2. Hg2l2 ) Hgl^+HgJ,
orange
4[Co(CN)4l4- +O2 +2H2O---- > 4[Co(CN)6]3- + 4OH’
yellow (c) Ag+: KI gives yellow precipitate Agl with Ag*
2[Co(CN)6]4- +H2O2 ---- > 2[Co(CN)6]3-+2OH" ions.
(i) Ni2* KCN forms green precipitate ofNi(CN)2 with ■> AgU
Ni2* salts; precipitate dissolves in excess of KCN forming a yellow
yellow coloured solution of [Ni(CN)4f~. The precipitate is soluble in dilute or concentrated
Ni2+ +2CN“ ---- > Ni(CN)2 ammonia, butdissolves readily in KCN and inNa2S2O3
green
Agi+2CN"—> ■>

[Ag(CN)2r +r
Ni(CN)2 +2CN- ---- > [Ni(CN)4f- soluble
dicyanoargentate(I) ion
yellow
Agl + 2S20a [AgfS2O3)2]3- +r
In presence of dil. HC1, Ni(CN)2 is again precipitated thiosulphate soluble
from soluble [Ni(CN)4 f~. dithiosulphato argentate(I)
[Ni(CN)4f“ +2H+ ---- > Ni(CN)2l+2HCN ion
(d) Hg2* (Mercuric ion) : Hg2* gives orange
A black precipitate of Ni(OH)3 is formed if soluble
yellow solution of [Ni(CN)4f- is heated with sodium precipitate of Hg^ with KI. Precipitate dissolves in excess
hypobromite solution of KI forming Hg^~
2[Ni(CN)4 f~ + OBr" + 4OH" + H2O ---- » HgCl2 +2KI HglJ +2KC1
hypobromite orange
2Ni(OH)34-+ 8CN" +Br" Hgl2 +2KI ---- > K2HgI4
black soluble
(e) KI is oxidised by Cl2 or Br2 forming I2 which
2. KI appears as violet colour in CCLj or CHC13 layer
(a) Pb2+ : Yellow precipitates of lead iodide are 2F+C12 ---- > I2 +2CF
formed. violet in
CC14
Pb2++2I" ---- > Pbl2 X
yellow 21 + Bib 4 I2 + 2Br"
violet in
The precipitate is moderately soluble in boiling water CCLt
to yield a colourless solution, from which it separates as (f) Cu2+ : Cu2+ (CuSO4) gives white precipitates of
golden yellow plates on cooling.
Cu2I2 along with which gives dark brown coloured
An excess of a more concentrated solution (6 M) of the solution
reagent dissolves the pecipitate and tetraiodoplumbate(II) CuSO4 +2KI ---- » Cul2 +K2SO4
ions are formed.
Pbla +2F [Pbl*f- 2CuI2 ---- > Cu2I2 X +1^
white ppt
The action is reversible, on diluting with water the i2+ki KI3
precipitate reappears. dark brown
858 | Essential Inorganic Chemistry

If Na2S2O3 (sodium thiosulphate-hypo) is added, Il22 Both reactions are reversible; by buffering the-
disappears and white precipitate of Cu2I2 appears solution with NH3(ag) or CH3COOH respectively, PbCrO4
I2(I3)+2Na2S2O3 ----- > Na2S40g +2NaI reprecipitates.
(g) Bi3+ : KI when added dropwise gives black (b) Hg2+ (Mercurous ion): Hg2+ salts form red
precipitate of Bil3
crystalline precipitate of Hg2CrO4 (mercury(I) chromatte)
Bi3+ + 31“ ■> Bil3 -L
Hg|++CrO|- -----> Hg2CrO41
■>

black
red
The precipitate dissolves readily in excess reagent
forming [Bil* ]" — orange coloured NaOH turns the precipitate into black mercury(l)
oxide :
Bil3+r [Bil,]-
tetraiodobismuthate(III) ions Hg2CrOj+2OH- ■> Hg2OJ' + Cr04“+H20
black
When diluted with H2O, the above reaction is
reversible forming Bil3 (black). Heating the precipitate (c) Ag+ : Ag+ salts form red precipitate of Ag2CrO4
with H2O, it turns orange due to the formation of (silver(I) chromate)
bismuthyl iodide. 2Ag+ +CrOj- Ag2CrO4X
Bil3 + H2O BiOU +2HI red
orange
The precipitate is soluble in dilute HNO3 and in
(h) Arsenate(V) AsO3" : Arsenate(V) ions oxidise I- to NH3(ag) solution.
I2 in acidic medium which appears as brown coloured
2Ag2CrO4 X + 2H+ 2Ag+ + Cr2O7- + H2O
solution due to formation of I3 orange
AsO43- + 2H+ + 21“ ■> AsO3 +12 + H2O
Ag2CrOj+4NH3 -----> 2[Ag(NH3)2]++CrO^’
Iz+KI KI3
brown The acidified solution turns to oragne because of the
formation ofCr2O^_.
(i) Antimony (Sb3+): KI form yellows coloured
solution with Sb3+ due to formation of [Sbl6]3- HC1 converts the precipitate into AgCl (white)
Sb3++6I“ ----- > [Sblgl3- Ag2CrO4-l+2C1" -----> 2AgCl X + CrOj’
yellow
(d) Ba2+ : K2CrO4 gives yellow precipitate ofBaCrO<
with Ba2 + salts in presence ofCH3COOH
3. K2CrO4
Ba2+ +CrOT -----> BaCrOj
yellow
DEEP Focus
The metallic chromates are usually coloured solids, yielding (e) HgS : H2S reduces acidified chromate solution to
yellow solution when soluble in water. In the presence of dilute green chromium(III) ion accompanied by the separation of
mineral acids, chromates are converted into dichromates which sulphur.
yield orange-red aqueous solution. In alkali solution,
dichromates change to chromate. 2CrOih + 3H2S + 10H+ > 2Cr3+ + 3S i + 8H2O
green
2CrO?"+2H+ Cr2O3"+H9O
yellow (f) SO^ : SC^’/H* or SO2 reduces CrOj' to Cr3*
Cr2O2- +2OH- 2CrOj- + H2O (green)
orange-red yellow 2CrO4_ + 3SO2 + 4H * 2Cr3+ + 3SO4"+2H20
green
The reaction may also be expressed as :
2CrO*"+2H+ 2HCrO; Cr2O?“ + H2O (f) Fe2+ : Fe2+ in presence of mineral acids reduces
chromates to Cr3+ (green)
(a) Pb2+ : K2CrO4 gives yellow precipitate of PbCrO4 CrOj- + 3Fe2+ +8H+ * Cr3+ + 3Fe3++4H20
on reaction of Pb2+ salts after acidification with green
CH3COOH (g) 1“ (Iodides): Iodide (KI) reduces CrC^" to Cr3<
Pb2+ + CrO^“ PbCrO4 X (green), and itself oxidised to I2
yellow
2CrOj“ +61- + 16H+ -----> 2Cr3+ + 3I2 + 8H2O
HNO3 or NaOH dissolves the precipitate
12 goes into solution as I3 (deep brown or bluish brown)
2PbCrO4X+2H+ 2Pb2++Cr2O27-+H2O
2PbCrO41 + 4OH- [Pb(OH)4f‘+Cr(^"
Chapter 19.Salt Analysis | 859

(h) CgHgOH: Ethanol (C^HgOH) in presence of (f) Ag+ : Silver salts (AgNO3) give white precipitate
mineral acid reduces chromates (or dichromates) slowly in of Ag4 [Fe(CN)61 with K, (Fe(CN),'6J
cold but rapidly if the solution is heated 4Ag++[Fe(CN)6r - Ag4[Fe(CN)6]l
2CrOl" + SCaHgOH + 10H+ ---- > 2Cr3++ 3CH3CHO ? white
green
Precipitate dissolves in KCN and Na2S2O3 but
+ 8H2O
remains insoluble in NH3(ag)
Smell of acetaldehyde (CH3CHO) is observed on
Ag4[Fe(CN)6]X+8CN" 4[AgfCN)2r +[Fe(CN)6J|-
heating the mixture. soluble
(4) K4 [Fe(CN)6 ] Potassium hexacyanoferrate(II) Ag4[Fe(CN)6]l +8S2O23----- » 4[Ag(S,O3)2 J:
soluble
(Potassium ferrocyanide)
+ [Fe(CN)6]‘-
(a) Cu2+ : Cu2+ salts on reaction with CH3COOH and
5. KSCN (Potassium thiocyanate solution)
K4[Fe(CN)6] give reddish brown precipitate Of Note : Instead NH4SCN can also be used
Cu2[Fe(CN)6]
(a) Hg2* (Mercuric ion) When NHjSCN and cobalt
Cu2+ +[Fe(CN)6]4- ■> Cu2[Fe(CN)6]i
reddish brown acetate is added to Hg2* solution, a blue colour precipitate
is produced
(b) Cd2+ : Cd2+ salts give bluish white precipitate of
Hg2++Co2+ + 4SCN‘ Co[Hg(SCN)4 ]l
Cd2[Fe(CN)6] under on reaction with K4[Fe(CN)6] and blue
CH3COOH cobalt tetrathiocyanato
mercurate(II)
Cd2+ +[Fe(CN)6]4" Cd2[Fe(CN)6]l
bluish white White precipitate of Hg(SCN)2 readily soluble in
excess of NHjSCN is obtained
(c) Zn2+ : (Similar to Cd2+)
Hg2++2SCN“ ---- > Hg(SCN)2
(d) Fe3+ : Fe3+ salts forms blue coloured precipitate Hg(SCN)2+2SCN" ---- » [Hg(SCN)4f~
ofFe4[Fe(CN)6]3 called Prussian blue
(b) Cu2* : NHjSCN on reaction with Cu2+ salts gives
4FeCl3 +3K4[Fe(CN)6] ---- > Fe4[Fe(CN)6]3 +12KC1 black precipitate of Cu(SCN)2
Prussian blue Cu2+ +2SCN- Cu(SCN)2i
black
A large excess of the reagent dissolves Prussian blue
partly or entirely when an intense blue colour is obtained. The precipitate decomposes slowly to form white
precipitate of CuSCN along with thiocyanogen gas
Fe4[Fe(CN)6]3+K4[Fe(CN)6] ---- > 4KFe[Fe(CN)6]
soluble 2Cu(SCN)2X -----> 2CuSCNi + (SCN)2T
black white thiocyanogen
(e) Fe^ : When K3[Fe(CN)6] is added to Fe2+ salts,
first K3[Fe(CN)6] oxidises Fe2+ to Fe3+ and itself reduced Cu(SCN)2 can be converted to CuSCN immediately by
suitable reducing agent like SOj'/H*
toK4[Fe(CN)6]
[Fem(CN)6]3- +Fe2+ ---- > Fe3++[FeII(CN)6]<- SO3"+2H+ ---- > SOz +H2O
2Cu(SCN)2 +SO2 +2H2O ---- > 2CuSCNX + 2SCN"
Fe3+
3+ and [Fe(CN)6]4'* combine to form a blue black white
precipitate of Turnbull’s blue
+ SOt+4H+
4Fe3+ +3[Fe(CN)6]4" Fe4[Fe(CN)6]3l
Turball’s blue (c) Fe3+ : NF^SCN reacts with Fe2+ salts in slightly
acidic medium giving red colour solution of Fe(SCN)3
DEEP Focus Fe3++3SCN" ---- > Fe(SCN)3
red
We can observe that the composition of this precipitate is
identical to that of Prussian blue. Earlier it was suggested that Sometime (Fe(SCN)f + is also formed. Fe(SCN)3 can be
its composition was Fe3[Fe(CN)6]2 hence the different name. extracted by ether or amyl alcohol. Fluorides, oxalates and
The identical composition and structure of Turnbull’s blue and Hg(II) ions bleach the red colour of Fe(SCN)3.
Prussian blue has recently been proved by Mossbauer
Fe(SCN)3 +6F- -----> [FeF6]3- +3SCN“
spectroscopy. The precipitate is decomposed by NaOH or red more stable
.KOH forming Fe(OH)3 — brown coloured precipitate. (colourless)
860 | Essential Inorganic Chemistry

2Fe(SCN)3+3Hg2+ —> 2Fe3++3Hg(SCN)2 (g) Zn2+ : Zn2+ salts are precipitated as Zn3(PO4)2
Fe(SCN)3 +3(COO)2- - * [Fe{(COO)2}3]3-+3SCN- on the addition ofNa2HPO4.
oxalate
3Zn2+ +2HPOt * Zn3(PO4)2 1 +2H+
(d) Co2+ : On adding a few crystals of NHjSCN to a white
neutral or acid solution of cobalt(II), a blue colour appears
However, in presence of NH3(aq), Zn(NH4)PO4 is
owing to the formation of [Co(SCN)4 j2-
precipitated
Co2++4SCN" ----- > [Co(SCN)4f-
tetrathiocyanato-cobaltdl) ion Zn2++NH3(og) + HPOt Zn(NH4)PO41
(blue) white
Both the precipitates are soluble in dilute acids.
6. Na2HP04 Excess ofNH3(aq) dissolves the precipitates
(a) Pb2+ : Na2HPO4 gives white precipitate with Pb2+
Zn3(PO4)2 +12NH3 ----- > 3[Zn(NH3)4f+ +2PO?'
salts soluble
3Pb2+ +2HPO^- Pb3(PO4)2l+2H+
Zn(NH4)PO4 +3NH3 [Zn(NH3)4f* + HPOJ'
white
soluble
The reaction is reversible; strong acids (HNO3)
(h) Mg24: Mg2+ salts are precipitated as
dissolve the precipitate. The precipitate is also soluble in
NaOH. Mg(NH4)PO4 -6H2O whenNa2HPO4 is added in presence
(b) Hg^+ (Mercurous): Na2HPO4 gives white of basic buffer (N^Cl + NHjOH).
precipitate with Hg2+ Mg^+NHjj+HPOj" Mg(NH,)P04 J-
Hg^+HPO?- * Hg2HPOj white
white The precipitate is sparingly soluble in water; soluble
(c) Ag+ : Na2HPO4 in neutral solution gives yellow in acetic acid and in mineral acids. The precipitate
separates slowly from dilute solutions because of its
precipitate of silver phsophate with Ag+
tendency to form supersaturated solutions; this may be
3Ag+ +HPOt ----- > Ag3PO4X+H+ overcome by cooling and by rubbing the test-tube with a
yellow glass rod.
(d) Fe3+ : NaH2PO4 gives yellowish-white In neutral solution, a white flocculant precipitate of
precipitate with Fe3+ salts MgHPO4 is produced
Fe3+ 4-HPOS’ - -> FePO4«L + H+ Mg2+ +HPO?" MgHPO4i
yellowish white

The reaction is reversible due to formation of strong


7. NlnO;
acid (H+) which dissolves FePO4. Hence, FePO4 is All permanganates are soluble in water forming
precipitated in presence of acidic buffer of CH3COOH and purple (reddish-violet) solutions
CH3COONa. (a) C2O^“ (Oxalate): C^O^- decolourises KMnO4 in
(e) Al3+ : White gelatinous precipitate of A1PO4 is acidic medium.
obtained when Al3+ salts react withNa2HPO4. 5C2(^-+2MnO4-+16H+ * 10CO2 +2Mn2++8H20
A13++HPO|- A1POJ+H*
The reaction is reversible; strong acids dissolve the DEEP Focus
precipitate of A1PO4. Precipitates are also soluble in NaOH The reaction is slow at room temperature but is rapid at
60°C. Manganese(II), Mn2+ ions, catalyse the reaction; thus
but insoluble in CH3COOH.
reaction is autocatalytic; once Mn2+ ions are formed, it becomes
(f) Mn2* : Mn2+ salts are precipitated as manganese faster.
ammonium phosphate, MnfNH^PO,! • 7H2O, (pink) on the (b) SOf- : SOI" /H+ or SO2 also decolourises MnO<
addition ofNa2HPO4 in presence ofNH3(aq)
SO23"+2H+ -----> SO2+H2O
Mn2+ +NH3 + HPO^“ ----- > Mn(NH4)PO4X
pink 2MnO4- +5SO2 +2H2O -----> 2Mn2+ +5S^'+4H+
Some dithionate (S2O6-) may also be formed in this
IfNH3(o^) is absent, Mn3(PO4)2 is precipitated
reaction, the quantity being dependent upon the
3Mn2++2HPO?’ ----- > Mn3(PO4)2 +2H+ experimental conditions :
Both precipitates are soluble in acids. 2MnO4 + 6SO2 + 2H2O * 2Mn2+ +S2O26" +4S(^-
+ 4Hf
Chapter 19 . Salt Analysis | 861

(c) Fe2+ : Fe2+ reduces MnO4 to Mn2+: and solution 3Mn(^“ + 3H2O 2MnO; + MnO(OH)2 + 40H“
becomes yellow due to formation of Fe3+ green purple +4
+6 +7 f
MnO4 + 5Fe2+ +8H+ ---- > 5Fe2++Mn2++ 4H2O t
The yellow colour disappears if phosphoric acid or TheMnO^* disproportionatestoMn04 (oxidation) and
potassium fluoride is added due to formation of colourless MnO2 (reduction).
complexes with Fe3+. 8. Thioacetamide
(d) I- : KI reduces MnO4 /H+ and solution turn dark (Analysis of cations without H-jS)
H2S in the laboratory is produced by reaction of H2SO4
brown due to formation of I3
with FeS
2MnO4 + 10F + 16H+ ---- > 2Mn2+ + 8H2O + 5I2
FeS+H2SO4 ---- > FeSO4+H2S
5I2 +51“ 5I3 This causes lot of pollution and hence formation ofH2S
dark brown by the above reaction should be discouraged.
Thioacetamide serves as a source of H2S. On hydrolysis in
(e) NO2 : NO2 also decolourises MnO4 /H+
hot acidic solution, it formsH2S (in situ) and CH^OONHj.
2MnO4 +5NO2 +6H+ ---- > 2Mn2++5NO3+3H2O S 0
(f) Upon warming a concentrated solution of KMnO4
II AA il
CH3CNH2 +2H2O CHjCONHf +H2S
with concentrated NaOH solution, a green solution of thioacetamide
K2MnO4 (potassium manganate) is produced and O2 is Thus, simply addition of thioacetamide in hot solution
evolved. When K2MnO4 solution is diluted or acidified is the instant source of H2S in the test-tube itself.
with dil. H2SO4, the purple colour of KMnO4 is restored Pb2++H2S ---- > PbSi+2H+
and MnO2 (manganese dioxide) is precipitated (as black
MnO(OH)2). Cu2++H2S ---- > CuSi+2H+
4MnO4+4OH“ ---- > 4MnOj“+Q,?+2H2O
black
K.

Total Practice Set


(Read, Plan & Solved)
j., ■JS

Problem 1. (A) is yellow coloured solid partially Solution (A) + tap water ---- > white turbidity soluble
soluble in aqueous NH3. (A) is soluble in Na2S2O3 (hypo) in aq NH3. Tap water has Cl" and turbidity is soluble in aq
solution forming a complex (B) which on heating is NH3.
converted into (C) (black). (C) is converted into white ppt => turbidity is of AgCl and thus (A) has Ag+
(D) on reaction with HC1 and HNO3. (D) is soluble in (A) also gives ring test of N03
aqueous NH3 forming (E). Identify (A), (B), (C), (D) and (E) => (A) has N03 and thus (A) is AgNO3
and explain reactions.
Explanation: 2AgNO3 A 2Ag +2NO2 +02
Solution (A) is AgBr (present in photographic plate) (B) (C)

AgBr+2Na2S2O3 ---- > Na3[Ag(S2O3)2] +NaBr N02 is paramagnetic due to one unpaired electron as
hypo soluble shown and thus forms dimer by using unpaired electron.
(B) 2NO2 N2O4
A dimer (VD = 46)
HCI+HNO3 ” o ,0
[Ag(NH3)2Cl] + NH3 AgCU «----------- ~Ag2S
(D) (C)
• N.
(E)
0
Problem 2.. Identify (A) and (B) in the following and Problem 4. (A), an important laboratory reagent,
explain reactions given VD of (A) is 17. turns red litmus blue, imparts golden yellow colour in
A decolourises acidified KMnO4. (A) also gives brown ppt flame and is a good precipitating agent. (A) reacts with Zn
(B) with alkaline KMnO4. (A) also liberates from or Al forming H2 gas. (A) gives white ppt with ZnCl2 or
acidified KI solution. (A) also removes black stains from A1C13 but ppt dissolves in excess of (A). What is (A) and
old oil paintings. explain reactions?
Solution (A) turns blue litmus red => (A) is basic in
Solution Molecular weight of (A) = 2 x VD nature.
= 34 g mol-1. Also (A) removes stains from old oil (A) imparts golden yellow colour in flame => (A) has Na”.
paintings. (A) gives H2 gas with Zn or Al => (A) is NaOBL
=> (A) is H2O2 (mol. wt. = 34)
Explanation:
2MnO4"+ 5H2O2 + 6H+ ---- > 2Mn2+ + 81^0 + 5O2
2NaOH + Zn ->Na2ZnO2 + 10
2MnO4 + + 2MnO2 X + 3O2 + 2 OH" + 2H.2O 2NaOH + 21^0 + 2A1 + 2NaA102 +3H0
(B) (brown)
ZnCl2 + 2NaOH + Zn(OH)21 + 2NaCl
4H2O2 + PbS PbSO4 + 4H2O white ppt
black (of oil paintings)
Zn(OH)2 + 2NaOH---- > Na2[Zn(OH)4]
Problem 3. (A) + tap water + white turbidity or Na2ZnO2
soluble in aq NH3 sodium zincate
AlCfy + 3NaOH---- > Al(OH)3i + 3NaCl
(A) —residue (B) + (C) (oxides of N) + 02 white ppt
aqueous (A) gives brown ring on adding FeSO4 and A1(OH)3 + NaOH + Na[Al(OH)4]
conc.HgSQj. (C) is paramagnetic and forms dimer of VD or NaA102
46. Identify (A), (B) and (C) and explain reactions. sodium meta-aluminate
Chapter 19 . Salt Analysis | 863

Problem 5. Identify (A) and (B) based on following Problem 7. A colourless mixture of two salts (A) and
reactions : (B) (excess) is soluble in H^O. (A) turns blue litmus red and
(i) U) Na°H>(B)(g) — —white fumes (B) turns red litmus blue. (A) gives white ppt with (B),
which dissolves in excess of (B) forming (C). (A) when
(ii) After (B) is expelled completely, resultant alkaline placed in atmosphere gives fumes and can form dimer. (A)
solution again gives gas (B) on heating with zinc. gives white ppt with NH4C1 and NH4OH soluble in (B).
Identify (A), (B) and (C) and explain reactions.
(iii) (A) —N20 + H2O
Solution (A) gives white ppt with NH4C1 and NH40H
Solution By (iii) (A) gives N20, J^O on heating, hence => (A) isAl3+
(A) isNH4NO3 => ppt is of A1(OH)3
A N20 + 2H2O A1(OH)3 is soluble in (B) => (B) is NaOH
NH4NO3 => (C) is NaA102 (sodium meta-aluminate)
(A) forms dimer indicating (A) is electron-deficient that
(i) NH4NO3 4-NaOH A+NaN03 +NH3 -t-HaO
(B) is the possible case of A1C13.
(A) is thus AlCLj (acidic due to hydrolysis, turns blue
NH3 +HC1---- > NH4C1 +H20 litmus red)
(B) white fumes (B) is NaOH (alkaline, turns red litmus blue)
(ii) 2NaOH + Zn Na2ZnO2 + 2H AICI3 +3NaOH---- > A1(OH)3 +3NaCl
(A) (B) white ppt
A NaOH +NH3 + 2^0
NaN03 + 8H
(B) A1(OH)3 + NaOH -> NaA102 + ^0
(excess) soluble (C)
(NH3 gas is obtained due to reduction of N03)
A1C13+3H2O- * A1(OH)3 + 3HC1 ? (fumes)
NH44C1 >
Problem 6. Identify (A), (B), (O and (D) based on AICI3 + 3NH4OH -------- A1(OH)3 X + 3NH4C1
following observations:
2A1C13---- > A12C16
(i) (A) —glassy transparent bead (B) on platinum
wire CK /Cl Cl
(B) + CuSO4-----> coloured bead (O >A1 Al
ignite^ green flame Cl Cl Cl
(ii) (A) + cone. H2SO4 + CHgCHjjOH
(D)
(iii) Aqueous solution of (A) is alkaline. Problem 8. Identify (A) based on following facts:
(a) (A) reduces HgCl2 solution to white ppt changing to
Solution (i) (A) forms glassy transparent bead which grey.
is characteristic property of borax. (b) (A) turns FeCl3 yellow coloured solution to green.
=> (A) is borax (Na^O.? • 10H2O) (c) (A) gives white ppt with NaOH soluble in excess of
NaOH.
Na2B4O7 • lOHaO NaBO2 + BA + lO^O
(d) (A) gives yellow dirty ppt on passingHgS gas, soluble
(B) glassy bead in yellow ammonium sulphide (YAS).
A (e) (A) gives chromyl chloride test.
B2O3+CuSO4 —> Cu(BO2)2 + SO3T
coloured bead (C)
Solution (A) gives yellow dirty ppt with H^S soluble in
(ii)Na2B4O7 +^804 + 5^0 > Na2SO4 + 4H3BO3 YAS
=> (A) has Sn2+
HjBOg + aqjigOH ((^115)3603 +31^0
green flame (A) gives chromyl chloride test (of Cl“)
(on ignition) => (A) has Cl“ ion and thus (A) is SnCl^
(D)
(iii)Na2B4O7 +5^0---- > 2H3BO3 +2Na[B(OH)4] Explanation:
weak acid (a) SnCl2 + 2HgCl, ■»Hg2Cl2X +SnCl4
white
Na[B(OH)4 ] reacts with acid (HC1) hence aqueous solution
SnCl2 + Hg2Cl2 2Hg 1 + SnCl4
of (A) is alkaline. grey
864 | Essential Inorganic Chemistry

(b) 2FeCl3 + SnCl2 2FeCl2 + SnCl4 Problem 11. (A) is a colourless salt with divalent
yellow green cation. (A) gives yellow ppt with K^CrO4 as well as with
(0 SnCl2 + 2NaOH Sn(OH)2 i + 2NaCl AgNO3. (A) gives apple green colour in flame. (A) is
white precipitated by H2S neither in acidic nor ammoniacal
medium, but addition of (NH4)2CO3 in NH40H gave white
Sn(0H)2 + 2Na0H Na2SnO2 + 2H2O ppt (B). 0.297 g of (A) gave 0.197 g of (B). Identify (A) and
soluble
(sodium stannite) explain reactions.
(d) SnCl2 + ---- > 2HC1 +SnS J-
yellow Solution (A) gives yellow ppt with KI as well as with
SnS + (NH4)2S2 (NH4)2SnS3 AgNO3.
YAS soluble => (A) can havePb2+ orBa2+ (both divalent cations) and
Br". (A) is not precipitated by HgS in acidic and
Problem 9. You have a mixture of CO2, SO2 and O2 ammoniacal medium.
=> (A) does not have Pb2+ (precipitated as PbS in acidic
gas. You have been provided following reagents for testing medium by H2S gas)
(a) lime water => (A) contains Ba2+ (apple green flame)
(b) acidified potassium dichromate solution => (A) is BaBr2
(c) pyrogallol
Arrange these reagents in order such that only one gas BaBrj + 2AgNO3 2AgBr 1 + Ba(NO3)2
yellow
is tested at one time.
BaBr2 + KgCrC^ BaCrO4 I + 2KBr
Solution The correct order is (b), (a), (c) yellow

CO2, SO2, O2 [ KgCi^O^—turns green due to S02 BaBrj + (NH4)2CO3 ► BaCO3 + 2NH4Br
|cO2, 02 (A) (B)

Ca(OH)2 —turns milky due to CO2 BaBi^ +(NH4)2CO3 BaCO3 +2NH4Br


? 197 g
I 02 0.297 0.197 g
pyrogallol —turns black due to O2 M = 197
0.297 " 0.197
Note: • SO2 andCO2 both turn lime water milky henceCa(OH)2
can’t be used before K2Cr2O7.
M(BaBig) = 297 g mol-1
• O2 is soluble in pyrogallol. Thus, A is BaBr2
(Ba = 137, Br = 80).
Problem 10. Identify gas (X) based on following facts:
(a) (X) occupies 0.35 L per g at NTP Problem 12. A colourless salt (A), soluble in water,
(b) (X) turns acidified K^Ci^C^ solution green. gives a mixture of three gases (B), (C) and (B) along with,
water vapours. (B) is blue, (C) is red and gas (B) is neutral
(c) (X) decolourises acidified KMnO4 solution.
towards litmus paper. Gas (B) is also obtained when (A) is
(d) (X) gives white turbidity when H2S gas is passed heated with NaOH and gives brown ppt with B^Hgl^.
into Solution thus obtained gives white ppt (B) with CaCl2
its aqueous solution. solution in presence of CH3COOH. (B) decolorises
MnO4 / H+. Gas (C) turns lime water milky while gas (P)
Solution (a) (X) occupies 0.35 L per g at NTP bums with blue flame and is fatal when inhaled.
=> molecular weight of (X) is 64 gmol-1
Identify (A) to (B) and explain reactions.
(b) (X) turns K2Cr2O7 /H+ green, (X) is a reducing agent
Solution Gas (B) gives brown ppt with K-jHg^
and green colour is due to formation ofCr3+.
Ci2O72’+ 14H+ + 6e"---- > 2Cr3+ + 7^,0 => gas (B) is NH3
=> (A)hasNH4
(c) (X) also reduces MnO4/H+
Gas (C) turns lime water milky
MnO4- + 8H+ + 5e”---- » Mn:2+ + 41^0 => gas (C) can be SO2 or CO2
=> probably (X) is SO2.
Gas (B) is also obtained along with (O. Gas (B) bums
SO2 + 21^0 ---- > SOj- +4H+ +2e~
with blue flame and is fatal when inhaled
(d) SO2 in aqueous solution gives white turbidity due to
=> gas (B) is CO
formation of colloidal sulphur.
=> gas (C) is CO2
SO2 + 2H2S---- > 21^0 + 3S => (A) has CjOj" ion
(colloidal)
Chapter 19: Salt Analysis | 865

It is confirmed by the fact that CaCl2 gives white ppt (b)K2CrO4: Cu2+ + CrOj- CuCrO4 X
CaCA (E) which decolourises MnO4 / H+ chocolate
Pb2+ + Cry- PbCrOj
=>(A)is(NH4)2C2O4 yellow
Explanation:
(c)AgNO3: Agl X
(NH4)2CA^-» 2NH3+CO2+CO +H2O yellow
(A) (B) (C) (D) Ag++Cl~ AgClX
white
(B) is blue towards litmus (basic)
(C) is red towards litmus (acidic) (d) Cl^ water + CHC13 :
(B) is neutral
2r + Cl2 12 + 2C1"
(NH4)2CA + 2NaOH -^Na2G2O4 +2NH3 + 2^0 (violet in
(B) CHC13 layer)
Na2CA + CaCl2 CaQA X + 2NaCl 2Br“ + Cl2 > Br, + 2C1"
white ppt (orange in
(E) CHC13 layer)
/Hg\ (e) BaC^: BaCl2 +SO23- ---- > BaSO3X
3NaOH + NH3 + 2K2HgI4 ---- > 0< ^NHjX
\Hg/ white ppt soluble in cone. HC1
brown ppt BaCl2 + 8^- ---- > BaSO4 X
(Iodide of Millon's base) white ppt insoluble in cone. HC1
+ 4KI + 3NaI + 2H2O (f)NH4SCN: Fe3+ + SCN- ---- > (Fe(SCN)f+
red colour
2MnO4"+ 16H+ + 5CA2"---- > 10 C02 + 2Mn2+ Cu2+ +2SCN" ---- > Cu(SCN)2X
violet colourless black
+ 8H2O (g)NH4SCN: Co'►2+ + 4SCN" ■> [Co(SCN)4f-
blue colour
Problem 13. Match the following using three columns
Cu2+ +2SCN- Cu(SCN)2X
with (X) containing radicals, (Y) the reagents used and (Z)
black
the compound formed when (X) and (Y) react.
(X) (Y) (Z) Problem 15. What single reagent solution (including
(I) Fe2+ (a) NH4SCN (A) Cherry red ppt HA could be used to affect the separation of the following
(II) Fe3+ (B) Turnbull’s blue pairs of solids ?
(b) DMG
(III) Ni2+ (a) NaOH andFe(OH)3 (b)Ni(OH)2 andFe(OH)'3:
(c) K4[Fe(CN)6] (C) Prussian blue
(c) CiA and Fe(OH)3 (d) MnS and CoS
(IV) Co2+ (d) K3[Fe(CN)6] (D) Blue
(e) AgCl and Agl
Solution (X) (Y) (Z)
Solution (a) H2O can dissolve NaOH, Fe(OH)3 remains
(I) (d) (B)
insoluble in water.
(ID (c) (O (b) NH4OH dissolves Ni(OH)2, Fe(OH)3 is insoluble
(III) (b) (A)
(IV) (a) (D) Ni(OH)2 + 4NH4OH----->[Ni(NH3)4f+
soluble
Problem 14. Name one common reagent that can (c) Excess of NaOH can dissolve CiA but FefOH)3
precipitate or react and differentiate following pairs remains insoluble
(a) Ag+ and Ba2+ (b)Cu2+ and Pb2+ Cr,O3 + SHjO---- > 2Cr(OH)3
(c)F and Ci­ (d)I“ and Br“
Cr(OH)3 +NaOH--- > Na[Cr(OH)4]
te) SO?" and SO^ (f)Fe3+ and Cu2+ soluble
(g) Co2+and Cu2+ (d) Dil. HC1 dissolves MnS while CoS remains
insoluble.
Solution (a) K2CrO4:
MnS + 2HC1-----> MnCl2 + l^S T
2Ag+ + Cr^- Ag2CA X (e) Aqueous NH3 would dissolve AgCl while Agl
red
remains insoluble
Ba2++Cry-—> BaCA'l'
yellow AgCl+2NH3(ag) -----> [Ag(NH3)2]Cl
soluble
866 | Essential Inorganic Chemistry

Problem 16. (a) If CO2(g) under pressure is passed 3H2SO4 + 6FeSO4 + 2HNO3 3Fe2(SO4)3
into Na2CrO4(aQ), Na-jCi^O/ag) is formed. What is the + 2N0 + 41^0
[unction of the CO2(g) ? [Fe(H2O)6]SO4 +NO [Fe(H2O)5NO]SO4 +H20
(b) When Zn is added to acidic solution of Na2Cr2O7, the brown coloured ring (C)
colour of the solution changes from orange to green
then to blue, and, over a period of time, back to NH4NO3 N2O +211,0
(A) (D)
(D) (F)
green. Explain.
(c) In acidic solution silver (II) oxide first dissolves to NaO + HgO > H2N2O2
produce Ag2* (aq) and then reduces to Ag+ by H20 (F)
OH OH
which is oxidised to 02. Write reactions. N N

Solution (a) C02 makes the solution acidic that can


I I
-N N-
convert Na2CrO4 to Na2Ci2O7 HO OH
trans cis
C02 + H20 4 HC03+H+
F (hyponitrous acid)
2Cr(^- + 2H+ -> Cr2O^_ + H20
(b) Zn reduces Cr2O^“ to Cr3+ (green) and then to Problem 18. Identify (A) to (G) in the following scheme
Cr2+(blue) over a long time, Cr2* is oxidised to Cr3+ and name the process.
by atmospheric 02 and thus blue colour changes to CaCO3 (A) +(B) gas
green.
Ci^O^" + 14H+ + 3Zn + 2Cr3+ + 7H2O + 3Zn2+ (A) + HjO----- > (O
orange green (C) + (B)----- >CaCO3+H2O
Cr3+ +e" -» Cr2*
green blue (D) + (C) (E) gas

C?+ -> Cr3+ + e~ (E) + H20 + (B) (F)


green
NaCl + (F) —> (G) + (D)
(0 AgO + 2H+ -» Ag2++H2O
A
4Ag2+ + 21^0 4 4Ag+ + 4H+ + 02
(G) —> Na2CO3 + HaO+CB)

Solution Scheme represents the Solvay process of


Problem 17. Colourless salt (A), on heating with manufacture ofNaaCOg.
NaOH, gave gas (B) that can also be obtained when Mg^
A
reacts with H^O. When reaction of (A) with NaOH was CaCO3 —> CaO +CO2(g)
complete, solution obtained on reaction with FeSO4 and (A) (B)
cone. H2SO4 gave a brown coloured ring (0) between two
CaO + H>0----- > Ca(0H)2
layers. (A) on heating strongly forms (D) and (E). (E) is a (A) (C)
neutral oxide of a dibasic acid (F) that exists as cis and
—> CaCO3 +1^0
Ca(OH)2 + C02 -----
trans isomer. Identify (A) to (F) and explain reactions.
A
2NH4C1 +Ca(0H)'22 —> 2NH3?+ CaCl2 +2^0
Solution (A) NaQH’ (B) <■ HaO MggN2 (£)) (C) (F)
=> (A) has NH4 ion and (B) is NH3 NH3 + HLjO +C02 ----- > NH4HCO3
(F) (B) (F)
Mg3N2 + 6H2O----- > 3Mg(OH)2 + 2NH3
NaCl +NH4HCO3 ■» NaHC03 +NH4C1
NH4+ + NaOH----- > Na+ + NH3 + HjO (F) (G) (D)
Resultant solution after NH3 has escaped completely
2NaHCO3 Na2CO3 + HaO +C02
contains Na+ and anion of (A) which also gives ring test (B)
(G)
ofNO3.
Thus, (A) is NH4NO3. Problem 19. How might you separate the following
Explanation: pairs of ions by addition of a single reagent? Include
NH4NO3 + NaOH NH3 ? + NaN03 +H^O formulae for the major products of the reactions.
(A) (B) (a) Fe3+ andNa+
(b) Cr3+ andFe3+
NaN03 + H2SO4 ----- > NaHS04 + HN03 (c) Fe3+ andCu2+
Chapter 19 . Salt Analysis | 867

Solution (a)Fe3+ andNa+: Problem 21. The following solid substances are in
Add excess of aq KOH. Fe3+ is precipitated as Fe(OH)3. separate but unlabelled test tubes :
Na+(aq) remains in solution. A12(SO4)318H2O, BaCl2-2H2O, KOH
Describe how could you identify the compounds by
Fe3+(aq)+3OH"(aq) ■» Fe(OH)3 (s) X chemical tests using only these substances and water?
brown ppt
(b) Cr3+ and Fe3+ : Solution All the above substances are unlabelled.
Label the test-tubes I, II and III.
Add excess of aq NaOH. Fe3+ is precipitated as Fe(OH)3
Mix I and II in aqueous solution. If there is formation of
while Cr(OH)3 first formed dissolves in excess of
white ppt, then these are A12(SO4)318H2O and
aq NaOH. BaCl2 -211,0
Fe3+(aq)+30H“ (aq) Fe(0H),(s)^U ppt does : +S02'
Bai2+ BaSO4 X
not dissolve white ppt
Cr3+(aq)+3OH (aq) Cr(OH)3(s) III is certainly KOH.
soluble in excess • Add III into I, if there is white ppt and ppt dissolves in
of NaOH (a^) excess of III then I is A12(SO4)3-18H2O
[Cr(OH)4 ]“(aq)
soluble A12(SO4)3 + 6KOH---- > 2A1(OH)3 X + SKjSO.,
(deep green) white ppt
(c)Fe3+ andCu2+: A1(OH)3 + KOH---- > KA1(OH)4
Add excess of aq NH3(NH4OH). Fe3+ is precipitated as soluble
Fe(OH)3 while Cu2+ forms deep blue coloured soluble • Add III into II and there is no ppt formation then II is
complex [Cu(NH3)4 f+. certainly BaCl2 • 21^0 thus
lisAl^SOPj-lSELjO, IIisBaC^^BLjO and Ulis
Fe3+(aq)+ 3NH4OH(aq)--- >Fe(OH)3(s)+ 3NH4+
KOH.
Cu2+ + 4NH4OH--- > [Cu(NH3)4f+ + 4^0
Problem 22. Baking powders contain sodium (or
Problem 20. Write a balanced net ionic equation for potassium) hydrogen carbonate and an acidic substance.
each of the following reactions : When water is added to a baking powder, carbon dioxide is
(a) A CrO|- solution turns from yellow to orange upon released. One kind of baking powder contains NaHCO3
addition of acid. and sodium aluminium sulphate, NaAl (SO4)2. Write the
(b) Fe3+(aq) reacts with KSCN(aq) to give deep red net ionic equation for the reaction that occurs in water
solution. solution.
(c) Copper metal reacts with nitric acid to give NO gas
and a blue solution. Solution NaHCO3(aq) Na*(aq) + HCO3(aq)
(d) A deep green solution of Cr(OH)3 in excess base
NaAl(SO4)2 (aq) Na+(aq) +Al3+(aq) + 2S0j"(aq)
turns yellow on addition of hydrogen peroxide.
Al3+ solution is acidic due to hydrolysis
Solution (a) 2CrOj (aq) + 2H3O+(aq)
Al3+(aq) + 3H2O(Z) A1(OH)3 + 3H+(aq)
Yellow
C^O?" (aq) + SHjOfZ) H+ (aq) decomposes HCO3 (aq) forming C02 (g)
orange H+ (aq) + HCO3 (aq)---- > HjOtZ) + C02 (g)
(b) (FeO^O^toHSCN-to)—> or AKH^O)?*(aq)+ HCO3(aq)---- > [AKHaO^OHrf\aq)
[Feffl,O)6(SCN)f ♦(ag)+ + HjOfZ) + CO2(g)
(c) 3Cu(s) + 2NOj( + H,O(Z))+ 8H*(a?)---- >
Problem 23. A light coloured crystalline solid A has
3Cu2‘(a«) + 2NO(g)+41^0(0
27.55% H>0. A gives following reactions :
(d) Cr(OH)3(s)+ OH-(ag)—> Cr(OH);(o<?) I: A BaCl2 solution) white ppt (B) insoluble in cone. HN03
deep green
K3[Fe(CN)6l solution , a ppt (C)
II :A
2Cr(OH)4 (aq) + 3^0-2 + 20H“ (aq)---- >
III : A Ws/NaOH solution, ppt
2CrO^“(aq) + SELjOG)
yellow Identify A
868 | Essential Inorganic Chemistry

Solution Solution The suspected arsenic-containing material is


Observation treated with zinc in an acidic solution:
Inference
Zn(s) + 2H+(ag)----- > Zn2+(ag) + 2H(g)
A gives white ppt insoluble in A contains SO4’ as anion The freshly prepared (nascent) H(g) reduces the arsenic
cone. HNO3
compounds to arsine, AsH3.
A gives dark-blue ppt with A contains Fe2 + as cation
K3[Fe(CN)6] As4O6 + 24H2(g) * 4AsH3(g) + GHjO
(arsenic sample)
A also gives brown ppt with A also contains NH4 as cation.
K2Hgl4/NaOH (Nessler's Finally, the AsH3 is decomposed by heating it in a glass
reagent) tube; the As deposits as a thin film (a mirror).
2AsH3(g)----- > 2As(s) + 3H2(g)
Thus, A is a double salt. mirror
Even trace quantities of arsenic can be detected
FeSO4 (NH4)2SO4 xH20
by this method.
H2O is 27.55%
Problem 25. Identification of cations can also be done
Molar mass of A is = (284 + 18x) using a dry test called sodium carbonate-bead test which
18x is similar to borax-bead test. Identify A, B, C and D in the
% of H2O = x 100 = 27.55
284 +18x following:

x=6 KNOo fuse and heat in Cl2


Na2CO3 -------- ------------------------ > A
oxidising flame with MnO
Thus, A is FeSO4 • (NH4)2SO4 ■ 6H2O
repeat with Cr2O3 1H2SO4
1D
Reactions :

I: SO?" + Ba2+ *BaSO4J- Solution Na2CO3 + O2 (from KN03) + MnO —>

II : [Fe (CN)6]3- first oxidises Fe2+ to Fe3+ and itself Na2MnO4 +C0.2
reduced to [Fe (CN)6 (A green)
Fe2+ +[Fe(CN)6]3- ----- > Fe3+ + [Fe[CN)6 ]*" 2Na2MnO4 + Cl2----- > 2NaMnO4 + 2NaCl
A B (purple)
Products thus formed combined to form a blue coloured 2Ci2O3 + 4Na2CO3 + 3O2 * 4Na2CrO4 + 4CO2
ppt called Turnbull's blue C (yellow)
4Fe3+ + 3[Fe(CN)6 ----- > Fe4[Fe(CN)6]3 2Na2CrO4 + H2SO4----- > Na2C^O7 + Na2SO4 + H20
Turnbull’s blue C D (orange)
(However, if [Fe(CN)6]3" is in excess, ppt dissolves
Problem 26. When metallic copper is heated with
partially or completely forming a blue coloured solution. concentrated sulphuric acid, in addition to copper (II)
Fe3* + [Fe (CN)6]*~ ----- > Fem[Fen(CN)6 T sulphate and sulphur dioxide, some copper (II) sulphide is
also formed. Explain.
final product of redox reaction Turnbull s blue
Solution When Cu reacts with cone. H2SO4 there is
DEEP Focus formation of CuS04 (by oxidation of Cu to Cu2+) and S02
(by reduction ofSC^- toSO2)
Note : Composition of this Turnbull’s blue is identical to that
of Prussian blue and has been confirmed by spectroscopy. Cu — ■» Cu2+ + 2e"
SO?" + 2e" + 4H+ — > SO2 + 21^0
III: NH4+ + NaOH > NH3 + 0,0 + Na+ There is also side reaction :
/Hgx SO?" + 8H+ + 8e’ ----- > S2" + 4^0
Cu2++S2‘ ----- > CuS
3NaOH + NH3 + 2K2HgI4 > 0 •W
Nessler's reagent xHg/ This reaction is favourable due to low solubility of CuS
Iodide of Millon's base and high [H+ ].
(brown ppt)
Problem 27. Silver nitrate gives a white precipitate
+ 3NaI + 4KI+ 2^0
(A) with aqueous sodium thiosulphate (B); the precipitate
dissolves in excess of thiosulphate. If the precipitate is
Problem 24. A sensitive test for arsenic that has been
heated with water, it turns black (C), and the supemant
popularised through mystery novels is the Marsh test. liquid then gives a white precipitate (D) with acidified
Write the chemical equations. barium nitrate solution. Explain.
Chapter 19: Salt Analysis | 869

Solution 2AgNO3+Na2S2O3 Ag2S2O3 X FeSO4 + K3[Fe(CN)6] KFeII[FeIII(CN)6] + K2SO4


(B) white ppt (A) Turnbull’s blue (F)
+ 2NaNO3 (A): CuS + FeS (CuFeS2) (B): CuSO4
White ppt dissolves in excess of B forming a complex. (C): FeSO4 (D): HjS
Ag2S2O3 + 3Na2S2O3 ---- > 2Na3[Ag(S2O3)2 ] (E): (F): KFen[Feni(CN)6]
soluble (G): PbS (H): BaSO4
When A is heated with water, there is (I) : CuS
disproportionation ofS20|“ to SO^- andS2-
^&2^2^3 + ^0 ---- > Ag2S -I + H,SO4 Problem 29. (A)(black) + dil.H2SO4 ■>(B)(a9) + (O
(c) black ppt soluble (gas)

H,S04 remains in supemant (upper) layer and gives (O+(CH3COO)2Pb ---- > black
white ppt (D) with Ba(NO3)2 solution (B) + NaOH ---- > green ppt(D) changing to brown (B)
H2SO4 +Ba(NO33'2
)2 ---- > BaSO4X+ 2HNO3 by atmospheric 02
D
(B)+ K3[Fe(CN)6] ---- > blue(F)
Problem 28. Identify (A),(B),(C) ... (H) in the (B) + HC1 deep yellow coloured solution (G).
following and explain reactions. (F) + KCNS ---- > red coloured solution (H).
(A) + black+dil.H2SO4 ---- > B(aq) + C(aq) + D(g)
What are (A) to (H) and explain reactions?
(B) + KI ---- > brown coloured solution
Solution (C) + (CH3COO)2Pb black
hypo
=> (O is H^S and (A) has S2~
white ppt (B)
(B) + K3[Fe(CN)6 ] ---- > blue
(O + Mn04 + H+ ---- > colourless solution
=> (B) has Fe2+ ion and thus (A) is FeS.
(O + K3[Fe(CN)6 ] ---- > blue(F)
Explanation:
(D) + (CH3COO)2Pb ---- > black (G)
FeS + H,S04 -> FeSO4 + H,S
(B) or (C) + BaCl2 ---- > white ppt CH) (A) (B) (C)
(B) + (D) ---- > black ppt (Z) (CH3COO)2Pb+H2S 4 PbS(black)X + 2CH3COOH
Solution CD) + (CH3COO)2Pb —> black (G) FeSO4 + 2NaOH 4 Fe(OH)2X +Na2SO4
green ppt (B)
=> (D) is HgS gas and (G) in PbS
=>(A) also has S2" 4Fe(OH)2 + 02 2Fe2O3 • H2O + 2^0
air brown (E)
(A) + dil.H2SO4 ---- > (B),(C)
(B) and(C) both are sulphates as confirmed by white ppt Fe2O3+6HCl ■> 2FeCl3 + 3^0
deep yellow (G)
(H) with BaCl2
(B) + KI ---- > brown coloured solution changing to FeSO4 + K3[Fe(CN)6] ---- » KFe[Fe(CN)]6 + K2SO4
white ppt (B) (B) Turnbull’s blue (F)

Thus, (B) is CuSO4. FeCl3+KCNS ---- > [Fe(CNS)]Cl2 + KC1


(G) red (H)
2CuSO4 +4KI CuJjj +2K2SO4 4-^
(E)
L> +KI KI3 Problem 30. Black coloured (insoluble in H>0) solid
Due to KI3, solution appears reddish brown. On adding (A) does not dissolve in dil. HNO3. Aqua regia can dissolve
hypo, La disappears and white ppt of Cu^ appears. (A) forming (B). (B) gives yellow ppt. (C) with NaOH. (B)
also gives orange ppt(D) with KI;(D) dissolves in excess of
(IpL> +2Na2S2O3 ---- > 2NaI + Na2S4O6 KI forming (B). (B) gives brown ppt with NH4 salt in
hypo
presence of NaOH. (A) is precipitated if PLS gas is passed
(O decolourises Mn04/H+. (O also gives blue colour into solution of (B) in dil. HC1. Identify (A) to (B) and
with K3[Fe(CN)6 ] thus (C) is FeSO4. explain reactions.
870 | Essential Inorganic Chemistry

Solution (A) is insoluble in dil. HN03. CHgCHjOH+O -> CHgCHO + HjO


=>(A) is of group II (copper subgroup) and is HgS [only] CH3CHO + 3C12 * CC13CHO + 3HC1
HgS (black) is insoluble in dil. HN03] 2CC13CHO+Ca(OH)2 > (HCOO)2Ca+2CHC13
(A) + 3HC1 +HNO3 ----- > HgCl2
aqua -—
--------- :----'
regia (B) Problem 32. Aqueous solution of (A) is deep yellow
but (A) (solid) is brown. (A) is aqueous solution gives white
HgCl2 + 2NaOH * Hg(H +2NaCl+H2O ppt with AgNO3 soluble in NH3. Yellow solution of (A)
yellow ppt (C)
changes to green solution (B) on passing f^S. (A) is
HgCl2 +2KI ----- > HgL,X +2KC1 precipitated as brown ppt (C) on adding NH40H in
(D) (orange)
presence of NH4C1. Precipitate (C) is insoluble in NaOH
HgL, +2KI * K^Hgl, even in presence of oxidising agents but (O dissolves in
(E) (soluble) HC1 forming (A) again. Aqueous (A) gives blue colouration
(Z>) with K4[Fe(CN)6]. Aqueous (B) also gives blue
NH4 +NaOH NH3+H2O+Na+
colouration (E) with K3[Fe(CN)6 ]. Identify (A) to (E) and
/Hgx explain differece between (P) and (E).
3NaOH+NH3+2K2HgI4 * O •NHgli
Solution A gives white ppt with AgNO3 soluble inNH3.
xHg/
brown ppt =>(A) hasCF
(iodide of Millon’s base)
cr + Ag+ AgCU
+ 4KI + 3NaI + 2H2O white ppt

HgCl2 +H2S ----- > HgSi+2HCl AgCl + 2NH3(aq) ----- > [Ag(NH3)2]Cl
(B) black soluble
(A) is precipitated as brown ppt. (C) by NI^OH in
Problem 31. Identify (A) to (O in the following : presence ofNH4Cl
A(aq) + KI ----- > => (A) has Fe3+ (of III group) which is confirmed by
A(aq) + CO2 + CH3COOH ----- > milky (B) formation of blue colouration with K4 [Fe(CN)6 ]
=>(A) isFeClg
A(agXpaste) + CHgCl^OH —product (C)
(an anaesthetic) Explanation:

Solution CI^CHjjOH + A(oq) (paste) - 2FeCl3 + H2S •> 2FeCl2 +S+2HC1


+ product (O (A) (yellow) (B) (green)
(C) is an anaesthetic from CHgCH^OH
FeCl3 + 3NH4OH -> Fe(OH)3^+3NH4Cl
=>(C) isCHClg
(C) (brown)
=>(A) contains Cl
(A) with CO2 gives milkyness Fe(OH)3+3HCl ----- > FeCl3+3H2O
(C) (A)
=> (A) contains Ca(OH)2 in aqueous solution.
(A) is CaOC^ (bleaching powder) FeCl3+K4[Fe(CN)6] ----- > KFein[Fen(CN)6] + 3KCl
(A) (D) (Prussian blue)
Explanation:
FeCl2 + K3[Fe(CN)6] * KFeII[Fem(CN)6] +2KC1
CaOCl2 +H2O ■> Ca(OH)2 + C12 (available) (B) (E) (Turnbull’s blue)
(A)
Cl2 +2KI ----- > 2KC1+I,
DEEP Focus
Ca(OH)2 + Cl2 + CO2 ----- > CaCO3 + H^O + Cl2
milky (B) X-ray studies have established that Prussian blue and
Turnbull’s blue are identical. In (P), Fe3+ partially
A
CH3CH2OH + Cl2(CaOCl2) > CHC13 oxidises [Fen(CN)6]4" forming some [Feffl(CN)8]3’ and
(C) in (E), Fe2+ partially reduces [Fera(CN)e]3' forming
Formation of C is explained by following reactions : some [Fen(CN)e]4". Thus, both CD) and (E) have Fe2+,
CaOCl2 +1^0 -> Ca(OH)2 +C12 Fe3+, [Fen(CN)e]4- and [Fem(CN)e]3-.
Cl2 +H2O * 2HC1+O
Chapter 19: Salt Analysis | 871

Problem 33. (A) is scarlet (reddish-orange) inorganic Fe2O3+6HCl ----- > 3FeCl3 -hSI^O
salt insoluble in HjO. (A) on reaction with HNO3 gives C0nC- yellow (D)
blackish-brown residue (B) and colourless solution of (O. FeCl3+NH4CNS ----- > [Fe(CNS)]Cl2+NH4C1
(0 gives yellow ppt with KI solution as well as with (D) red
I^CrC^. (C) also gives black ppt (D) with H2S in HC1
FemCl3+K4[Fen(CN)6] ----- > KFein[Fen(CN)6]
solution, ppt(D) dissolves in dil. HNO3.(B) on heating with
(D) (Prussian blue)
cone. HNO3 and Mn(NO3)2 forms pink coloured solution
(E). Identify (A), (B), (C), (D) and (E). FeCla+SHjO ----- > Fe(OH)3+3HCl
Solution (O is colourless soluble solution. 2HC1 + Zn ----- > ZnCl2 + 2H (nascent hydrogen)
(C) give black ppt with HgS/HCl FeCl3+H ----- > FeCl2 +HC1
=> (O is of group II green (E)
(C) gives yellow ppt with KI as well as with K^CrC^
i
=>(C) hasPb2+ ion DEEP Focus
Thus, (C) is Pb(NO3)2 obtained from (A), (B) on heating [Nascent hydrogen is more reactive than molecular H2 hence
withMn2+ and HN03 gives purple colour which is due to H2 gas passed into FeCl3 solution has no effect. FeCl3 solution
oxidaton ofMn2+ of HMnO4 (purple) is acidic due to hydrolysis, hence Zn added into it gives
nascent hydrogen (in situ) that causes reduction o/"FeCl3 into
=> (B) is oxidising agent
FeCl2]
=> (B) is PbO2 (blackish-brown)
=> (A) is double oxide of lead. 2Na2CrO4 + H2SO4 -> Na2SO4 + Na2Ci^O7 +H2O
(A) is Pb3O4 (red lead) (2PbO • PbO2) (B) orange (F)

Pb3O4 +4HNO3 ----- > PbO2 +2Pb(NO3)2 +21^0 Na2Ci2O7 +2NaOH ----- > 2Na2CrO4 4-HjO
(A) (B) (C) (F) (B)
Pb(NO3)2 +2KI ----- » PblJ +2KNO3 Na2C^O7 +2NH4C1 ----- > (NH^Cr^ +2NaCl
(C) yellow (F)
A
Pb(NO3)2 +K2CrO4 PbCrOj +2KNO3 (NH4)2Ci2O7 ■> N2 + +4H2O
yellow (H) green (G)

Pb(NO3)2 + H^S ----- > PbSl +2HNO3 Note : (Cr2O3 can be reduced to Cr by Al.)
(D) black
Problem 35. Colourless salt, (A) (insoluble in water)
5PbO2 +2Mn2+ +4H+ ----- > 2MnO4~ +5Pb2+ +2^0
GE) pink + dil.H2SO4 ----- > (B) + C(g)
+ NaOH NaOH
(B) ----------- ■>> (D) (white ppt) — —(E) (soluble)
Problem 34. Chromite ore (A) (FeC^O4) is fused with (excess)
NaOH in presence of H2O2 when yellow coloured solution NH.C1 + 4NH.0H (C)
4
(B) and residue (C) are obtained. (C) is separated by
filtration and dissolved in cone. HC1 forming a yellow no ppt a (A) (A) (white ppt)
coloured solution (D). (D) gives red colour with NH4CNS
(C) + (F) (yellow coloured solution)----- > light green (G)
and blue colour with K4 [Fe(CN)6 ].(D) changes to green (E)
when Zn is added into it, however, H2 gas if passed into (D) K4[Fe(CN)6]
blue(H)
has no effect. (B) changes to orange (F) on reaction with dil.
HgSC^ and again (F) changes to (B) on adding NaOH. (F) AgNO2
---------- > white ppt (I) (soluble in NH3)
on reaction with NH4C1 and subsequent heating gives G
(green) and H (gas). Identify (B) to (H) and explain +02, A ,+vC
—-—> HgO + (J) -------- > colloidal white turbidity
reactions.
Cr2C)2-/H+
Solution A(FeCi2O4) is an ore of chromium. -------------- > green

2FeCr>04 +8NaOH + 7H2O 4Na2CrO4 +Fe2O3 Ca(OH)2


milky
(A) (B)
+ 11H2O Identify (A) to (J) and explain reactions.
872 | Essential Inorganic Chemistry

Solution Gas (O is vital compound in this scheme. Problem 36. A solution may contain any of the
_ O 2
following ions : Fe3+, Ni2+, Cr3+, Zn2+, Mn2+. Based on the
Gas (C) * HjO + fJ) following experiment and results therein, indicate which
A
of the ions would be present? Indicate any wrong
Ca(0H)2 Cr2O2-/H+ information, if any.
Gas (J)
milky <----------- ------------ > green (a) The original solution is treated with (NH4)2S
(a substitute of H2S) in a buffered basic solution. A
=>(J) is S02
dark precipitate is obtained.
=> (C) is (giving H20 and S02 by combustion in 02). (b) The ppt for (a) dissolves in aqua-regia
(B) gives white ppt only if (0) is passed into its (c) The filtrate after separating ppt in (a) is treated
ammoniacal solution containing with NaOH and H2O2. A dark ppt is separated.
NH.CKNI^OH + NI^Cl) Filtrate is colourless.
(d) The ppt from (c) dissolves in HC1 (aq) giving a
=>(B) has Zn2+ and (B) is ZnSO4 coloured solution.
=> (A) has Zn2+ ion (e) The solution from (d) is treated with aqueous NH3.
=> (A) is ZnS A dark ppt forms.
(f) The ppt from (e) is soluble in HC1 (aq) and solution
(F) gives blue colour with K4[Fe(CN6)] develops an intense red colour when treated with
=> (F) has Fe3+ ion SCN-(aq).
(F) gives white ppt with AgNO3 Solution
=> (F) has Cl- ion. Thus, F is FeClg. (a) Precipitate can be of NiS
Explanation: (b) NiS dissolves in aqua regia
(c) Precipitate can be of Fe(OH)3
ZnS +H2SO4 ZnSO4 +H2S (d) Fe(OH)3 is soluble in HC1 (aq) giving (yellow)
(B) (C) coloured solution of FeCl3.
(e) This step is incorrect since filtrate of step (c) is
ZnSO4 +2NaOH ---- > Zn(OH)2 +Na2SO4
(B) (D) already colourless indicating absence ofCr3+.
(f) This confirms Fe3+ by forming intense red colour
Zn(OH)2 +2NaOH * Na2ZnO2 +2H2O Fe3++SCN" ---- > [Fe(SCN)f+
(D) soluble (E)
Thus, Fe3+ — confirmed
Na2ZnO2 +H2S ZnS+2NaOH Ni2+ — probable
(E) (O (A)
Mn2+
Zn2+,,Mn 2+ , Cr3+ — absent
H2S+2FeCl3 -> 2FeCl2 +2HC1+S
(C) (F) (G) green
yellow Problem 37. On treatment with cold water, an
element (A) reacted quietly, liberating a colourless,
FeCl3 + K4[Fe(CN)6] KFeni[Fen(CN)6]+ 3KC1 odourless gas (B), and a solution (C). Lithium reacted with
(F) (H), Prussian blue (B) yielding a solid product (D) which effervesced with
water to give a strongly basic solution (E) and gas (F).
FeCl3 + 3AgNO3 ---- > Fe(NO3)3 + 3AgCU When C02 was bubbled through sdlution (C) and initial
white ppt (I)
white ppt (G) was formed, but this re-dissolved, forming
AgCl+2NH3 ---- > [Ag(NH3)2]Cl solution (H) when more C02 was added, ppt (G) effervesced
(Z) • soluble when moistened with cone. HC1, and gave a deep brick red
colouration to a Bunsen burner flame. When (G) was
2H2S+3O2 2H2O + 2SO2 heated with carbon at 1000°C a caustic white compound
(J) (I) was formed, which when heated withN2 at 1000°C gave
SO2 +2H2S ---- > 21^0 + 3SX a solid (J) of some commerical importance. Name the
(J) (C) colloidal compounds (A) to (J) and explain reactions.
white turbidity
Solution When C02 is passed into (0), white ppt (G)
3SO2 + Ci2O27" +2H+ 2Cr3++3SO2"+ILO was formed which re-dissolved in excess ofCO2
green
=>(C) isCa(OH)2
S02 + Ca(OH)2 ---- > CaSO3 + HgO =$(A) is Ca
milky
Chapter 19: Salt Analysis | 873

Explanation: HC1, H2S


A ----------- > black ppt insoluble in NaOH.
Ca + 21LO ■> Ca(OH)2 + Hg
(A) (C) (B) Thus, A (colourless salt) can contain Hg2* or Pb2+ or
Bi3+ as cation.
2Li+H2 2LiH
(B) (D) Since, A gives yellow ppt with I^CrO,,, hence A contains
Pb2+ as cation. Thus, A is PbCl2.
LiH+ELO ----- > LiOH + Hj,
Reactions:
(£) (F)
PbCl2 + 2NaOH > Pb(OH)2 + 2NaCl
Ca(OH)2 +CO2 CaCOg+HjjO (A) white ppt
(C) (G)
Pb(OH)2 +2NaOH Na2[Pb(OH)4]
CaCO3 4-HgO+ CO2 ----- » Ca(HCO3)2 soluble (B)
(G) (H)
CaCO3+2HCl ----- > CaCl2+H2O + CO2 Na2[Pb(OH)4J + H2O2 PbO2 +2H2O + 2NaOH
(B) black ppt
Brick red colour in flame is due to CaCl2 (C)

CaCO3 + 4C -1000^ CaCj +3CO


PbCl2 +H2SO4 PbSO4>l +2HC1
(Z) • white ppt
CaCg +N2 —> CaCN2 + C
(Z) J (nitrolim) PbSO4 +2CH3COONH4 ----- > (CH3COO)2Pb+(NH4)2SO4
soluble (D)

DEEP Focus PbCl2 +K2CrO4 ----- > PbCrOj +2KC1


CaCN2 is of industrial importance. It is used to make urea, yellow ppt (F)
thiourea etc.
PbCl2 +H2S ----- > PbSX +2HC1
CaCN2 +H2SO4. —> CaSO4 + NH2CN black ppt (G)
NHZ9CN + HZ90 ---- > NHZ 9C0NH
z
2 (urea)
NHZ9CN + Hz9S ---- > NHz9CSNHz2 (thiourea) Problem 39. Identify (A) to (F) based on the following
reactions:
Problem 38. Identify colourless salt (A) based on the Brownish black oxide (A) + dil. HC1 ----- > green
following reactions: coloured solution (B) + gas (O
(B) + aq NH3 + HgS gas----- > black ppt (D)
NaOH
Aqueous solution of (A) ------- > white ppt soluble in (B) + aq NH3----- > green ppt (B) soluble in excess of
excess of NaOH (B) aqNH3
(O + aq KI ----- > reddish brown solution (F)
„ H2SO4 ,H2°2
black ppt (O Solution Ammoniacal solution ofB (green) gives black
white ppt soluble
ppt withHjS hence (B) hasNi2+ ion and exists asNiCl2 due
inCH3COONH4 (D) to dissolution of (A) in dil HC1.
AgN°% white ppt soluble is aqueous (A) also contains nickel ion but in +3 oxidation state
Aqueous solution of A which is reduced +2 by dil HC1
NH3(B) Thus, A is NigOg
K2CrO4 Reactions:
■> yellow ppt (F)
Ni2O3 + 6HC1 2NiCl2 + 3^0 +C12
HC1, H2S A B C
■> black ppt(G) insoluble in NaOH

-AgNOjlfr white ppt soluble in aq NH3 NiCl2 +H2S NiS-l +2HC1


Solution A D
Thus, A contains Cl as anion. NiCl2 + 2NH4OH Ni(OH)2 + 2NH4C1
E
Cr + Ag+ —> AgCll soluble in excess NH3
white ppt C12 + 2KI 2KC1 + Ig
(C)
AgCl +2NH3 [Ag(NH3)2]Cl ^+KI KI3
soluble (E) (F)
874 | Essential Inorganic Chemistry

Problem 40. Identify (A) to (J) based on following observations :

green coloured solution of treat with NaOH


yellow coloured solution (8)
a compound A and H2O2
pass
AgNO3 CO2
gas
white ppt soluble in aq NH3

orange coloured solution NH4CI orange coloured solution of


<-
of compound (D) compound (C)

C in solid state
(D) in solid state
A KCI, cone. H2SO4

ZZF reddish brown gas (E)

gas (G) h2o green residue (H) NaOH

Li, A B
H2O
solid I gas (J) CH3COOH + (CH3COO)2Pb
HCI
yellow ppt (F)
white fumes

Solution
Observations Inference
A gives white ppt. soluble in aq. NH3, with AgNO3 A contains Cl
This is chromyl-chloride test thus C is Na2Cr2O7.
C + cone. ------
H2SO4 + KCI Ax NaOH^ B

ch3cooh Thus, 8 is &O4 (Na2CrO4).


8 + (CH3COO)2Pb yellow ppt

Other compounds are thus :


(A): CrCl3 (B) : Na2CrO4 (C): Na2Ci2O.'7 (D): (NH4)2Ci2O7 (E): CrO2Cl2
(F): PbCrO4 (G): N2 (H): Ci&03 (I): Li3N (J): NH3
Reactions (unbalanced/poartial):
CrCl-j+NaOH+HA Na2CrO4 +NaCl
(A) (B)
CO2 +H2O ---- > HC03 +H+
2CrO*’ +2H+ ---- > CrjO?- +H2O
(C)
Na2C^O7+2NH4C1 ---- > (NH^CzA +2NaCl
CD)
A
(NH4)2Ci2O7 ■> N2 + C^Og + 4H2O
(D) (G) (B)
4KC1 +Na2Ct2O7 +3H2SO4 - 2CrO2Cl2 + 2K2SO4 + SHgO + Na2SO4
cone. (£)
chromyl chloride
CrO2Cl2 +2NaOH ---- > Na2CrO4 +2HC1
(E) (B)
Na2CrO4 +(CH3COO)2Pb > PbCrO4 +2CH3COONa
(F)
Chapter 19: Salt Analysis | 875

6Li+N2 ---- > 2LiaN KMnO4 can oxidise Fe2+, C,05~ in acidic medium and
(G) (/) used as reagent (Baeyer) in basic medium for testing
LiaN+SHaO ---- > 3Li0H+NH3 unsaturation of alkenes and alkynes.
(«7) Thus, (A) is Mn02.
Explanation:
froblem 41. Identify (A), (B) and (O in the following Mn02 +2KOH + HJO2 ---- > Kz,Mn04 +2H2O
theme: (A) black (B) (green)
KOH, H2O2
A (black) * (B) (green) 2K2M11O4 + Cl2 2KMnO4 +2KC1
A (C) (pink)
C12 A
(B) * (O (pink-violet) Mn02 + 4HC1 MnCl2 +C12 +2H2O
(A) cone.
(A) + conc.HCl ---- > Cl2(g)
2KMnO4 +16HC1 ---- > 2KC1 + 2MnCl2 + 81^0 + 5C12
(O + conc.HCl ---- > Cl2(^) (C) conc-

(0 is good laboratory reagent in acidic, basic and acidic


Mn04 +8H+ +5e" 4 Mn2+ + 4H,0
neutral medium. neutral*
Mn04 + e~
(0 can be decolourised by Fe2+, C,C^- in acidic medium
basic
and byC,H4 (ethene) in basic medium (in which brown ppt Mn04+2H,O + 3e_ DaBIC > Mn02 +40H~
occurs) MnO; + 8H+ + 5Fe2+ ---- > Mn2+ +5Fe3+ + 411,0
(O+KOH ---- > (B) 2Mn04 +16H+ + 50,05“ ---- > 2Mn2+ +10CO2 +811,0
Solution Reactions indicate that (O is KMnO4 and (A) 2KMnO4+2K0H ---- > 2K2MnO4 +H,0 + 0
is thus Mn02. (C) (B)

(Also refer KMn04 in chapter 16)


----------- ----------- —....

Chapter Proficiency
Exercises
Exercise 1
(Stage 1: Learning)
Short Answer Type Questions 7. (A) + KBr yellow ppt (B)
1. BaCl2 gives white precipitate with an aqueous (A) + cone. H2SO4 brown vapours intensified
solution of a salt. Identify acid radical. with Cu-tumings.
2. PO4~ and AsO4~ can be identified by a reagent. Name (B) dissolves in hypo forming a soluble complex (C).
the reagent. Write reaction. What are (A), (B) and (C) and explain their reactions?
3. A mixture contains, CO^“, SO^~ and SO^“. Give 8. (A) + dil. H2SO4 gas (B). Gas (B) turns
scheme to identify them. KgCijO.? /H+ solution green.
4. SO^“ and SO^' both give white ppt with BaCl2 Aqueous solution of (A) + BaCl2---- > white ppt (0.
solution. How is SO^~ detected in presence of SO^~ ? Filtrate after removing (O + Br2 water—> white ppt
A CHoOH, A
(C) . Identify (A), (B) and (C).
5. Na2B4O7 • IOH2O + cone. H2SO4 * (A) ----- --------- > 9. In the top space write the formula for each substance
(B). formed when the following ion pairs are mixed. Use
Identify (A) and (B). the bottom space to describe any visible change, i.e.,
6. SO^- also gives white ppt with lead acetate but ppt white precipitate formed, solution turns red, etc. If no
visible change occurs, leave the space blank.
dissolves in ammonium acetate solution. Explain.

Cations
Reagent added Pb2+ Cu2+ Fe3 + Co2+ Zn2+ Mg2+ Ba2+ K+
PbSO4
(NH4)2so4
white ppt

(NH4)2S

Nal (Petroleum ether)

NH4SCN

NaOH (Limited amount)

NaOH (Excess amount)

NH3(aq) (Limited amount)

NH3 (aq) (Excess amount)


Chapter 19: Salt Analysis | 877

10. If Cu2+ and Cd2+ both are present, it is difficult to A


22. A(orange) D(g) + residue (C) (green) + HgO
analyse. Outline a scheme to analyse in a mixture.
NaOH
11. HgS is soluble in aqua regia forming HgCl2. What
A
happens if Cu turnings are added to HgCl2? HC1
B(g) 4 white fumes
12. Sometimes it happens that when HgS gas is passed
Al HgO
into solution in dil. HC1, yellowish white turbidity D(g) 4 (E) -=—» gas(B)
appears. What do you conclude? What precautions A
are taken to check this turbidity? cone. HC1 „ NaOH, H2O2
residue (C) ---------- > (F) ” A~ ■> (G)
13. II B (arsenic group) sulphides are soluble in YAS. If
(yellow)
cone. HC1 is added to this soluble portion, coloured
ppt are formed. Write reactions. CH3COOH
■> (H) (yellow ppt)
(CH3COO)2Pb
14. Identify (A), (B), (C), (P) and (F). (A) (black) + dil. HC1
—^-4 (B) (Z) + (C)(g). Gas (C) turns lead acetate paper Identify (A) to (H) and explain reactions.
black. (B) gives orange ppt (P) soluble in excess of KI 23. Solution containing Mg2* and NH4 ions is called
forming (B). magnesia mixture and can be used to identify PO3“
15. Test tube (A) contains aqueous zinc acetate solution ions. What can be other anion (acidic radical) which
while test tube (B) contains aqueous zinc chloride can be identified by this magnesia mixture?
solution. What happens if HgS gas is passed into each 24. You have following salts on your table in a lab.
solution? KI, HgLj, NaOH, NH4OH, SnCl2.
16. (A) (colourless solution) gives white ppt (B) with (a) Select salts to prepare Nassler’s reagent.
NaOH solution but ppt dissolves in excess of NaOH (b) Above reagent is used to identify which ion?
forming (C). (C) does not give ppt with HgS but on (c) Write reaction
boiling with NH4C1, white ppt (B) appears. (A) also
gives yellow ppt with AgNO3. Identify (A), (B) and (C). Analytical Questions
17. Colourless solution of (A) gives white ppt (B) with 1. A monomeric complex of cobalt gave the following
AgNO3 solution, soluble in aq NH3. (A) also gives results on analysis:
white ppt (C) with NaOH soluble in excess of it Co3* NH. o CF SO2"I
forming (P). (P) gives white ppt (F) with HgS.
% 21.24 24.77 12.81 34.65
Identify (A), (B),(C),(P) and (F).
The compound is diamagnetic and contains no other
18. A reagent can be used to identify Ba2 + andPb2+ both.
groups or elements, except water which might be
Name the reagent is. present. What is probable formula of the compound?
19. Without going into systematic mixture analysis (Co = 59)
specify the test that can identify Ba2+/Ca2+/Sr2+
2. Identify (A) to (H) giving reactions taking place :
20. Teacher demonstrated that (CgO^~) oxalate ion dil. H,SO4
... ................
Mineral (A) —U (B) + (O + (P)(gas)
decolourised MnO4/H+. He also demonstrated that
Ca2+ can be precipitated as CaCgO4 (white) and these (P) + (CH3COO)2Pb -A> black ppt
precipitate also decolourised MnO4/H+. While
performing salt analysis quickly, a student (P)
(B) + (O E (black ppt) separated from (C) by
performed this experiment without precipitation of
CgQ^" but salt given to him confirmed MnO4 /H+ test. filtration
He reported C2O^- but teacher awarded him zero. A
(F) + HNO3 (F) blue coloured solution
Can you explain positive MnO4 /H+ test? dil.
21. Identify (A), (B), (C), (P) and (F). (F) + K4[Fe(CN)6] 4 (C) chocolate coloured ppt
(C) + K3[Fe(CN)6] 4 (H)blue
(A) + NaOH -±> Btg) + C(aq)
(slightly alkaline) 3. An aqueous solution of salt (A) gives a white
HC1 |zn crystalline precipitate (B) with NaCl solution. The
A ,, filtrate gives a black precipitate (C) when H2S is
HgO + D(g) white fumes B(g) passed through it. Compound (B) dissolves in hot
water and the solution gives yellow precipitate (P) on
| Al, A
4 E
H.0 f
treatment with potassium iodide and on cooling. The
compound (A) does not give any gas with dilute HC1
878 | Essential Inorganic Chemistry

but liberates a reddish brown gas on heating. Identify 8. Identify (A) to (G) in the following scheme:
the compounds (A) to CD) giving the involved
Colourless (A) > (B) gas + (C) aq
equations.
4. A white amorphous powder (A) when heated gives a (alkaline due to excess NaOH added)
colourless gas (B), which turns lime water milky and (B) + Na — * (D) + H2
the residue (C) which is yellow when hot but white C(ag) + Zn (B)
when cold. The residue (C) dissolves in dilute HC1
and the resulting solution gives a white precipitate (A) (SJ + HaO
on addition of potassium ferrocyanide solution. (A) (E) + (D) * (F)
dissolves in dilute HC1 with the evolution of a gas
UO + HsSQ, * (G)
which is identical in all respects with (B). The
solution of (A) as obtained above gives a white (G) contains 97.67% N, rest hydrogen.
precipitate (D) on addition of excess ofNH4OH and on 9. 0.98 g of an acid (A) of phosphorus in 100 mL solution
passing H^S. Another portion of this solution gives is 0.1 M and neutralises 300 mL of 0.1 N NaOH.
initially a white precipitate (E) on addition of NaOH
which dissolves in excess of it. Identify (A) to (B) and (A) (B) (O
explain reactions. If 1.11 of (C) is obtained from given (A), identify (A),
5. Compound (A) is a light green crystalline solid. It (B) and (C).
gives the following tests : 10. An acid (A) can oxidise I" to . One mole of (A) can
(i) It dissolves in dilute sulpuric acid. No gas is neutralise one mole of Ca(OH)2 in aq solution. One
produced. mole of (A) in aqueous solution, on hydrolysis, gives
(ii) A drop of KMnO4 is added to the above solution. two moles of another acid (B) and one mole of liquid
The pink colour disappears. (C) . 0.98 g of acid (B) gives 2.33 g of white ppt (D)on
(iii) Compound (A) is heated strongly. Gases (B) and reaction with excess of BaCl2. Liquid (C) has
(C) with pungent smell came out. A brown oxidising as well as reducing property. (C) can
residue (D) is left behind. decolourise KMnO4/H+, blackened oil paintings.
(iv) The gas mixture ((B) and (C)] is passed into a Identify (A) to (D) and explain reactions.
dichromate solution. The solution turns green.
(v) The green solution from step (iv) gives a white 11. A certain metal (A) is boiled in dilute nitric acid to
precipitate (B) with a solution of barium nitrate. give a salt (B) and an oxide of nitrogen (0). An
(vi) Residue (D) from (v) is heated on charcoal in aqueous solution of (B) with brine gives a precipitate
reducing flame. It gives a magnetic substance. CD) which is soluble in ammonium hydroxide. On
Identify the compound (A) to (B). adding aqueous solution of (B) to hypo solution, a
white precipitate (E) is obtained. (E) on standing
6. When 20.02 g of white solid (X) was heated, 4.4 g of turns to a black compound (F). Identify (A) to (F).
acid gas (A) that turned lime water milky was driven
off together with 1.8 g of a gas (B) which condenses to 12. An aqueous solution of a gas (X) gives the following
a colourless liquid. The solid (Y) that remained is reactions :
dissolved in water to give an alkaline solution, with (i) It turns an acidified ILjC^O? solution green.
which excess barium chloride solution gave a white (ii) On boiling it with H2O2, cooling it and then
precipitate (Z). The precipitate effervesced with acid adding an aqueous solution of BaCl2, a
giving off carbon dioxide. Identify (A), (B) and (Y) and precipitate insoluble in dilute hydrochloric acid
write down the equation for the thermal is obtained.
decomposition of (X). (iii) On passing HgS in the solution a white turbidity
is obtained.
7. A hydrated metallic salt (A), light green in colour,
gives a white anhydrous residue (B) after being Identify (X) and give equations for the reactions at
heated gradually. (B) is soluble in water and its steps (i), (ii) and (iii).
aqueous solution reacts with NO to give a dark brown 13. (a) A compound (X) imparts a golden yellow flame
compound (C). (B) on strong heating gives a brown and shows the following reactions :
residue CD) and a mixture of two gases (B) and (F). (i) Zinc powder when boiled with a concentrated
The gaseous mixture, when passed through acidified aqueous solution of (X) dissolves and
permanganate, discharged the pink colour and when hydrogen is evolved.
passed through acidified BaCl2 solution, gives a (ii) When an aqueous solution of (X) is added to
white precipitate. Identify (A), (B), (C), (D), (E) and an aqueous solution of stannous chloride, a
(F).
Chapter 19: Salt Analysis | 879

white precipitate is obtained first which (ii) When CO2 is passed through an aqueous
dissolves in excess of solution of (X). suspension of (X), the turbidity transforms to a
Identify (X) and write equations for reactions precipitate.
at steps (i) and (ii). (iii) When a paste of (X) in water is heated with ethyl
(b) Write equations for the action of strong heat on alcohol, a product of anaesthetic use is obtained.
borax and green vitriol. Identify (X) and write down chemical equations for
14. A metal chloride (X) shows the following reactions : reactions at steps (i), (ii) and (iii).
(i) When H2S is passed in an acidified aqueous 18. An aqueous solution of an inorganic compound (X)
solution of (X), a black precipitate is obtained. shows the following reactions :
(ii) The precipitate obtained at step (i) is not soluble (i) It decolourises an acidified KMnO4 solution
in yellow ammonium sulphide. accompanied by the evolution of oxygen.
(iii) When a solution of stannous chloride is added to (ii) It liberates iodine from an acidified potassium
an aqueous solution of (X), a white precipitate is iodide solution.
obtained which turns grey on addition of more of (iii) It gives a brown precipitate with alkaline
stannous chloride. KMnO4 solution with the evolution of oxygen.
(iv) When an aqueous solution of KI is added to an (iv) It removes black stains from old oil paintings.
aqueous solution of (X), a red precipitate is Identify (X) and give chemical equations for the
obtained which dissolves on addition of excess of reactions at steps (i) to (iv).
KI. 19. Two solid laboratory reagents (A) and (B) give the
Identify (X) and write down the equations for following reactions :
the reactions at steps (i), (iii) and (iv). Compound (A):
15. (a) An aqueous solution of a gas (X) shows the (i) On strongly heating it gives two oxides of
following reactions : sulphur.
(i) It turns red litmus blue. (ii) On adding aqueous NaOH solution to its
(ii) When added in excess to a copper sulphate aqueous solution, a dirty green precipitate is
solution, a deep blue colour is obtained. obtained which starts turning brown on
(iii) On addition to a ferric chloride solution, a exposure to air.
brown precipitate, soluble in dilute nitric acid, Compound (B):
(iii) It imparts green colour to the flame.
is obtained. Identify (X) and give equations for
(iv) Its solution does not give a precipitate on
the reactions at steps (ii) and (iii). passing H^S.
(b) A certain compound (X) is used in laboratory for (v) When it is heated with solid KoC^O? and
analysis. Its aqueous solution gives the following concentrated H2SO4, a red gas is evolved. The
reactions : gas when passed in an aq NaOH solution turns
(i) On addition to a copper sulphate solution, a it yellow.
brown precipitate is obtained which turns
white on addition of excess of Na2S2O3 Identify (A) and (B) and give chemical equations for
reactions at step (i), (ii)and (v).
solution.
(ii) On addition to Ag+ ion solution, a yellow 20. Identify (A) to (D) in following steps and give
curdy precipitate is obtained which is chemical equations :
insoluble in ammonium hydroxide. (i) A white amorphous powder (A) on strongly
heating gives a colourless non-combustible gas
Identify (X) and give equations for the reactions at (B) and solid (C).
step (i) and (ii). (ii) The gas (B) turns lime water milky and
16. A certain salt (X) gives the following tests : turbidity disappears with the passage of excess
(i) Its aqueous solution is alkaline to litmus. of gas.
(ii) On strongly heating it swells to give a glassy (iii) The solution (C) in dilute HC1 gives a white
material. precipitate with an aqueous solution of
(iii) When concentrated sulphuric acid is added to a K4[Fe(CN)6].
hot concentrated solution of (X), white crystals (iv) The solution of (A) gives a white precipitate (D)
of a weak acid separate out. on passing HoS in presence of excess ofNH4OH.
Identify (X) and write down the chemical equations 21. (i) A black mineral (A) on heating in presence of air
for reactions at steps (i), (ii) and (iii). gives a gas (B).
17. A certain compound (X) shows the following (ii) The mineral (A) on reaction with dilute H,SO4
reactions : gives a gas (C) and solution of a compound (D).
(i) When KI is added to an aqueous suspension of (iii) On passing gas (C) into an aqueous solution of
(X) containing acetic acid, iodine is liberated. (B), a white turbidity is obtained.
880 | Essential Inorganic Chemistry

(iv) The aqueous solution of compound (D) on reaction (iv) On adding a solution of (A) into the solution of
with potassium ferricyanide gives a blue cupric chloride, a white precipitate is first
. ’■ compound (E). Identify (A) to (E) and give formed which dissolves on adding excess of (A)
chemical equations for reactions at steps (i) to (iv). forming a compound (E).
22. An inorganic Lewis acid (X) shows the following Identify (A) to (E) and give chemical equations for the
reactions : reactions at steps (i) to (iv).
~(i) It fumes in moist air. 27. (i) A black coloured compound (B) is formed on
(ii) The intensity of fumes increases when a rod passing hydrogen sulphide through the solution
dipped in NHjOH is brought near it. of a compound (A) in NH4OH.
(iii) An acidic solution of (X) on addition of NH4C1 (ii) (B) on treatment with hydrochloric acid and
and NH4OH gives a precipitate which dissolves potassium chlorate gives (A).
in NaOH solution. (iii) (A) on treatment with potassium cyanide gives a
(iv) An acidic solution of (X) does not give precipitate buff coloured precipitate which dissolves in
with HgS. Identify (X) and give chemical excess of this reagent forming a compound (0.
equations for reactions steps (i) to (iii). (iv) The compound (C) is changed into a compound
(D) when its aqueous solution is boiled.
23. The certain inorganic compound (X) shows the (v) The solution of (A) is treated with excess of
following reactions : sodium bicarbonate and then with bromine
(i) On passing H2S through an acidified solution of water. On cooling and shaking for sometime, a
(X), a brown precipitate is obtained. green colour of compound (E) is formed. No
(ii) The precipitate obtained at step (i) dissolves in change is observed on heating.
excess of yellow ammonium sulphide. Identify (A) to (E) and give chemical equations for the
(iii) On adding an aqueous solution of NaOH to a reactions at steps (i) to (v).
solution of (X), first a white precipitate is 28. (i) An aqueous solution of a white coloured
obtained which dissolves in excess of NaOH. compound (A) on reaction with HC1 gives a
(iv) The aqueous solution of (X) reduces ferric white precipitate of compound (B).
chloride. Idenitfy the cation of (X) and give
(ii) (B) becomes soluble in chlorine water with the
chemical equations for reactions at steps (i), (ii)
formation of (C).
and (iv).
24. (i) An inorganic iodide (A) on heating with a (iii) (C) reacts with KI to give a precipitate which
becomes soluble in excess of if forming a
solution of KOH gifes a gas (B) and the solution
compound (D). The compound (D) is used for
of a compound (C).
detecting ammonium salts.
(ii) The gas (B) oq ignition in air gives a compound
CD) and water. , (iv) (B) and (C) both, on treatment with SnCl2 give a
grey precipitate of (E).
(iii) Copper sulphate is finally reduced to the metal
on passing (B) thorugh its solution. (v) When cone. HgSC^ is added slowly into a
(iv) A precipitate of compound (E) is formed on mixture of cold solutions of (A) and FeSO4, a
brown ring of compound (F) is formed. Identify
reaction of (C) with CuSO4 solution. Identify (A)
(A) to (F) and give chemical equations for the
to (E) and explain reactions (i) to (iv).
reactions at steps (i) to (v).
25. A black mineral (A) is soluble in NaCN solution 29. (i) A blue coloured compound (A) on heating gives
forming (B). On adding Zn into (B), metal (C) is two products, (B) and (C).
formed. Metal (C) on boiling with HNO3 forms (D) (ii) A metal (D) is deposited on passing hydrogen
which gives white ppt (E) with NaCI-soluble in NH3 through heated (B).
solution. (A) can be obtained when CD) reacts with (iii) The solution of (B) in HC1 on treatment with
hypo. Identify (A) to (E). K4[Fe(CN)6] gives a chocolate brown coloured
precipitate of compound (E).
26. (i) An inorganic compound (A) is formed on passing (iv) (C) turns lime water milky which disappears on
a gas (B) through a concentrated liquor continuous passage of (C) forming a
containing sodium sulphide and sodium compound (F). •
sulphite.
(ii) On adding (A) into a dilute solution of silver Identify (A) to (F) and give chemical equations for the
nitrate, a white precipitate appears which quickly reactions at steps (i) to (iv).
changes into a black coloured compound (C). 30. (i) The yellow coloured precipitate of compound (A)
(iii) On adding two or three drops of ferric chloride is formed on passing H2S through a neutral
into the excess of solution of (A), a violet solution of a salt (B).
coloured compound (D) is formed. This colour (ii) (A) is soluble in hot dilute HNO3, but insoluble
disappears quickly. in yellow ammonium sulphide.
Chapter 19: Salt Analysis | 881

(iii) The solution of (B) on treatment with small using chemical tests that involve .only these
quantity of NH3 gives white precipitate which compounds plus water? '
becomes soluble in excess of it forming a 34. You are given three unlabeled test tubes. One test
compound (C). tube contains a solution of Na2SO4, the second
(iv) The solution of (B) gives white precipitate with contains a solution of NaHSO4 and the third contains
small concentration of KCN which becomes a solution of NaHSO3. Describe how you could
soluble in excess of this reagent forming a identify the solutions.
compound (D).
(v) The solution of (D) on treatment with I^S gives 35. Identify each of the following substances from the
(A). description.
(vi) The solution of (B) in dilute HC1 on treatment (a) A yellow solid that burns with a blue flame,
with a solution of BaCl2 gives white precipitate giving off a gas with a choking odour.
of compound (E) which is insoluble in cone. (b) A white solid that reacts with water to give
HNO3. phosphoric acid.
(c) A colourless gas that poisons by attaching
Idenitfy (A) to (E) and give chemical equations for the strongly to iron atoms in the haemoglobin of red
reactions at steps (i) and (iii) to (vi). blood cells.
31. (i) An aqueous solution of a compound (A) is acidic (d) A hard, lustrous grey solid used as the base for
towards litmus and (A) is sublimed at about solid state semiconductor devices.
300°C. 36. Addition of HC1 (aq) to a solution containing several
(ii) (A) on treatment with an excess of NH4SCN different cations produces a white precipitate. The
gives a red coloured compound (B) and on filtrate is removed and treated with l^SCaq) in
treatment with a solution ofK4[Fe(CN)6 ] gives a 0.3 M HC1. No precipitate forms. Which of the
blue coloured compound (C). following conclusions is/are valid? Explain
(iii) (A) on heating with excess of in (a) Ag+ and/or Hg2+ probably present
presence of concentrated ^864 evolves deep red (b) Mg2* probably not present
vapours of (D).
(iv) On passing the vapours of (D) into a solution of (c) Pb2+ probably not present
NaOH and then adding the solution of acetic (d) Fe2+ probably not present
acid and lead acetate, a yellow precipitate of 37. Suppose you did a cation group I analysis and treated
compound (E) is obtained. the chloride precipitate with NH3(a<7), without first
Identify (A) to (E) and give chemical equations for the treating it with hot water. What might you observe
reactions at steps (ii) to (iv). and what vaild conclusions could you reach about
cations present, cations absent, and cations in doubt?
32. (a) Pyrolusite on heating with KOH in the presence
of air gives a dark green compound (A). The 38. Write net ionic equations for the following qualitative
solution of (A) on treatment with H2SO4 gives a analysis procedures:
purple coloured compound (B), which gives (a) Precipitation of PbCl2(s) from a solution
following reactions : containing Pb2+(ag)
(i) KI on reaction with alkaline solution of (B) (b) Dissolving of Zn(OH)2(s) in NaOH(a^).
changes into a compound (C). (c) Dissolving of Fe(OH)3(s) in HCl(aq).
(ii) The colour of the compound (B) disappears on (d) Precipitation of antimony sulphide from a
treatment with the acidic solution ofFeSO4. solution containing H^S and the complex ion
(iii) With cone. 1^864 compound (B) gives (D) SbCl;.
which can decompose to yield (E) and oxygen. 39. Identify A to E based on following observations and
Identify (A) to (E) and write balanced write balanced reactions therein :
chemical equations for the formation of (A) neutral FeCL solution ,, ,
and (B) and for the steps (i) to (iii). (A) ------------------------- > blood red colour (B)
(b) Complete and balance the following chemical I I dil.HCl.H2S gas
equations. ‘-------- ——------ > yellow ppt (C), (C dissolves in
I hot dil. HNO3 but insoluble in YAS)
(i) Au + HC1+HNO3 ---- > .......+ ....+^0
aq NHo
(ii) C+ HNO3 (cone.) ---- > CO2 + .. ...+H2O ---------> white ppt soluble in excess of aq NH3
(iii) Sn + KOH (hot) + HjO —> .... +..... forming (D)
(iv) Cu(OH)2 +NH4NO3+NH4OH —> ... + H2O az? KCN
---------- > white ppt soluble in excess of aq KCN
33. Unlabeled test tube contains solid A1C13 SHjO in one,
forming (E)
Ba(OH)2 -8^0 in another, and MgSO4 TI^O in the
H2S gas
other. How could you find what is in each test tube
yellow ppt (C)
882 | Essential Inorganic Chemistry

40. Identify A to G based on following reactions : and conducts electricity. When exposed to air,
+ gas B a slowly forms a white powder giving a basic solution­
solution of the compound (A) ■> (C) water. What can you conclude about the elemerz
(A) + (D) from these observations?
cone. H2SO4 45. Deduce what you can about the nature of tl
cone, solution of (C) - * (E) (weak acid) following reactions :
(D) + cobalt oxide (F) (coloured bead) (a) One mole of NHjOH reacts with two moles
Ti(III) in the presence of excess of alkali, and tE
(El + CjHzjOH green edged flame (G) Ti(III) is converted into Ti(IV).
41. Colourless substance (b) Silver phosphite, Ag2HPO3, is wanned wit:
dilH2SO4 water, and dilute the silver is precipitated as tla
(A)------------ 4 gas (B) + colourless solution (C) metal.
(CHgCOOh Pb ... .... (c) When one mole of hypophosphorous acid, H^PCU
gas (B)------ ------- ----- > black ppt (D)
NaOH solution , NaOH (excess) is warmed with excess of iodine in acidic solution
(C)-------------------- > white ppt (E) ---------- - -------- > one mole of iodine is reduced; on making th_
(F) (soluble) solution alkaline, another mole of iodine l
gasB consumed.
(i) NH4C1, A, NH4OH 46. The action of concentrated sulphuric acid on ure;
> (A) results in the production of a white crystalline solii
(ii) gas (B)
X of formula H^NOgS, a monobasic acid. Or
What are (A), (B), (C), (D), (E) and (F) and explain treatment with sodium nitrite and diluti
reactions. hydrochloric acid at 0°C, it liberates one mole of N.
42. Element M is a shiny and highly reactive metal and per mol of X, and on addition of aqueous bariuc
element Xis a highly reactive non-metal. They react chloride, the resulting solution yields one mole o-
to form a compound MX, a colourless brittle white BaSO4 per mol of X taken. Deduce the structure ofX
solid. MX conducts electricity in aqueous solution
47. Identify, A, B, C, D, E and F in the following:
and in molten state. When chlorine gas is bubbled
(A)
through an aqueous solution, containing MX, a ------ >) soluble (C
(A) + AgNO3 ----- > white ppt (B) -------
excess
reddish-brown liquid appears and Cl- ions are
formed. H2O,
A
2MX + Cl2----- >2MC1 + X2
black ppt (D) + (E) (soluble)
X2 is decolourised by unsaturated hydrocarbon and
aq solution of MX containing 1.19 gMXin 100 gHgO (E) BaC12> white ppt (F)
freezes at -3.72°C. Molar depression constant of HgO
48. In strongly alkaline solution containing excess o
is 1.86° mol-1 kg. Deduce the identity of the elements
barium ions, a solution containing 0.01587 g of I wa
MandX.
treated with 0.1 molar MnO4 until a pink coloui
43. An element X of group 2 reacts with Hg gas at 200°C presisted in the solution : 10.0 mL of the MnO,
to form a compound Y. When Y is heated to a higher solution were required. Under these conditions MnO,
temperature, it decomposes to the element X and is converted into the sparingly solubleBaMnO4.Wha
hydrogen gas in the ratio of 559 mL Hg (measured at is the product of the oxidation of iodide?
STP) for 1.00 g X reacted. X also combines with 49. In neutral solution, one mole periodate ion, I04
chlorine to form a compound Z, which contains
reacts with excess of iodide to produce one mole o
63.89 per cent by mass of chlorine. Deduce the idenity
iodine; on acidification of the resulting solution, i
of the element X, compound Y and Z.
further three moles of iodine are liberated.
44. A student is given samples of three elements X, Y,
Derive equations of the reactions which occur unde
and Z, which could be an alkali metal, a member of
these conditions.
group IVA, and a member of group VA. He makes the
following observations: 50. To prevent the formation of oxides, peroxides an
Element X has a metallic lusture and conducts superoxides, alkali metals are sometimes stored i
electricity. It reacts slowly with HC1 to produce Hg an inert atmosphere. Which of the following gase
gas. Element Y is a light-yellow solid that does not should not be used for lithium? Why? Ne, Ar,N2, Ki
conduct electricity. Element Z has a metallic lusture
Chapter 19: Salt Analysis | 883

ii 51. The following are two reaction-schemes involving (ii) Nitrogen is obtained in the reaction of aqueous
is magnesium. Identify A to J based on the observations ammonia with potassium permanganate.
Jia given : (iii) Elemental phosphorus reacts with cone. HNO:3
to give phosphoric acid.
Colourless (iv) Sulphur is precipitated in the reaction of
dil. H2SO4 02, A
solutions •4-------------------------- Mg A hydrogen sulphide with sodium bisulphite
G dil. HC1 solution.
3S NaOH (v) Carbon dioxide is passed through a suspension
B
of limestone in water.
White ppt H Na2CO3 57. Give reasons in one or two sentences for the
Ji
is following:
HNO3
White ppt C (i) The hydroxides of aluminium and iron are
Solution I A
insoluble in water. However, NaOH is used to
2< Gas D separate one from the other.
1, evaporate (ii) Ammonium chloride is acidic in liquid ammonia
£ A Ca(OH)2
Solid I E solvent.
J
White ppt F 58. A light bluish green crystalline compound responds
a Gas J to the following tests :
11 (i) Its aqueous solution gives a brown precipitate or
52. When aqueous KCN is added to a solution of colouration with alkaline [Hgl^ ] solution.
copper(II) sulphate, a white precipitate, soluble in an (ii) Its aqueous solution gives a blue colour with
excess of KCN is formed. No precipitate is formed K3[Fe(CN)6 ] solution.
! when H2S is passed into it. Explain.
1 (iii) Its solution in hydrochloric acid gives a white
53. The reaction precipitate with BaCl2 solution.
4AfO2($)+2CO2(g) —» 2M2CO3(s) + 3O2(g) Identify the ions present and suggest the formula of
is to be used to regenerate oxygen in space-craft. the compound.
Discuss the choice of M among the alkali metals. 59. Complete and balance the following:
54. Write balanced chemical equations for the following (i) NH3+NaOCl---- > ............. + .+
reactions :
(i) A mixture of potassium dichromate and sodium (ii) AgBr + Na2S2O3---- > ............ +
chloride is heated with concentrated H2SO4. (iii) (NH4)2S2O8 -f-HjO + MnSOj -
(ii) Aqueous solution of sodium nitrate is heated with
+............ + ..............
zinc dust and caustic soda solution.
(iii) Sodium iodate is added to a solution of sodium 60. The acidic, aqueous solution of ferrous ion forms a
bisulphite. brown complex in the presence of NO3 by the
(iv) Iron reacts with cold dilute nitric acid. following two steps. Complete and balance the
(v) Potassium permanganate is added to a hot equations:
solution of manganous sulphate. [FeCHjOJsf+NOJ+H* —> (a) + [FelHjOlJ3’
55. The gas liberated on heating a mixture of two salts + H.0
with NaOH, gives a reddish brown precipitate with [FefHjOJef ’ + (6) —> (c) + H,0
an alkaline solution ofKoHgIpThe aqueous solution
of the mixture on treatment with BaCl2 gives a white 61. An orange solid (A) on heating gave a green residue
precipitate which is sparingly soluble in cone. HC1. (B), a colourless gas (C) and water vapours. The dry
On heating the mixture with K2Cr2O7 an cone. H2SO4, gas (C) on passing over heated Mg gave a white solid
red vapours (A) are produced. The aqueous solution (D) . CD) on reaction with water gave a gas (E) which
of the mixture gives a deep blue colouration (B) with formed dense white fumes with HC1. Identify (A) to
potassium ferricyanide solution. Identify the radicals (E) and give the reactions involved.
in the given mixture and write the balanced 62. (A) is a binary compound of a univalent metal. 1.422 g
equations for the formation of (A) and (B). of (A) reacts completely with 0.321 g of sulphur in an
56. Write balanced chemical equations for the following : evacuated and sealed tube to give 1.743 g of a white
(i) Sodium nitrite is produced by absorbing the crystalline solid (B), that forms a hydrated double
oxides of nitrogen in aqueous solution of salt (C) with A12(SO4)3. Identify (A), (B) and (C).
washing soda.
884 | Essential Inorganic Chemistry

63. When gas (A) is passed through dry KOH at low 68. During the qualitative analysis of a mixture
temperature, a deep red coloured compound (B) and a containing Cu2+ and Zn2+ ions, H2S gas is passed
gas (C) are obtained. The gas A, on reaction with through an acidified solution containing these ions in
but-2-ene, followed by treatment with Zn/H2O yields order to testCu2+ alone. Explain briefly.
acetaldehyde. Identify (A), (B) and (C).
69. A white solid is either Na2O or Na2O2. A piece of red
64. A scarlet compound (A) is treated with cone. HNO3 to litmus paper turns white when it is dipped into a
give a chocolate brown precipitate (B). The freshly made aqueous solution of the white solid.
precipitate is filtered and the filtrate is neutralised (i) Identify the substance and explain with
with NaOH. Addition of KI to the resulting solution balanced equation.
gives a yellow precipitate (C). The precipitate (B) on (ii) Explain what would happen to the red litmus if
wanning with cone. HNO3 in the presence of the white solid were the other compound.
Mn(NO3)2 produces a pink-coloured solution due to 70. An aqueous solution containing one mole of Hg^ and
the formation of (D). Identify (A), (B), (C) and (D).
two moles of Nal is orange in colour. On addition of
Write the reactions in sequence.
excess Nal the solution becomes colourless. The
65. (i) A white precipitate (B) is formed when a
mineral (A) is boiled with Na2CO3 solution. orange colour reappears on subsequent addition of
(ii) The precipitate is filtered an the filtrate NaOCl. Explain with equations.
contains two compounds (C) and (D). The 71. (a) In the following equation,
compound (C) is removed by crystallisation and (A) + 2(B) + HaO---- > (C) + 2(D)
when CO2 is passed through the mother liquor [(A) = HNO2, (B) = H2SO3, (C) = NH2OH].
left, (D) changes to (C).
Identify (D). Draw the structures of (A), (B), (0
(iii) The compound (C) on strong heating gives two and (D).
compound (D) and (E).
(b) The Haber process can be represented by the
(iv) E an heating with CoO forms blue colour bead
following scheme :
(F). Identify (A) to (F).
C3CO3
66. A colourless inorganic salt (A) decomposes completely
at about 250°C to give only two products (B) and (C) CaO co2
leaving no residue. The oxide (C) is a liquid at room
temperature and neutral to moist litmus paper while
the gas (B) is a neutral oxide. White phosphorus
burns in excess of (B) to produce a strong while h2o H3-H2O
dehydrating agent CD). Write balanced equations for
the reactions involved in the above process.
NaCI
67. (a) Element (A) burns in nitrogen to given an ionic
compound (B). Compound (B) reacts with water
to give (C) and (D). A solution of (C) becomes c + h2o NH3 + H2O + E
‘milky1 on bubbling carbon dioxide. Identify (A),
(B), (C) and (D).
(b) Aluminium sulphide gives a foul odour when it
becomes damp. Write a balanced chemical
equation for the reaction. Identify (A), (B), (C), (D) and (E).

Exercise 2
(Stage 2: High Skill Problem Solving)
Only One Option Correct 2. Fe2+ does not give blue colour with K4[Fe(CN)6] but
1. Yellow coloured solution of FeCl3 changes to light on its reaction with (A), blue colour appears. (X) can
green when be
(a) SnCl2 is added (b) Zn is added (a) MnO4/H+ (b) H2SO4
(c) H2S gas is passed (d) all true (c) NH3 (d) HC1
Chapter 19: Salt Analysis | 885

; 3. Fe(0H)3 and Cr(OH)3 ppt are separated by 13. H2S would separate the following at pH < 7
(a) Zn2+, Co2+ (b) Cu2+, Cd2+
(a)Ag. NH3 (b) HC1
(c) Cu2+,Cr3+ (d) Cu2+,As3+
(c) NaOH/HA (d) H^
14. Solution of (X) in dil. HC1 + HjO---- > white
4. Turnbull’s blue and Prussian's blue respectively are
H S/HC1
I. Fen[Fen(CN)6f- IL FenI[Fein(CN)6] turbidity. (X) —------- > black ppt (Y). (Y) is soluble in
III. Fen[FeIU(CN)6r IV. Feni[FeII(CN)6T (a) H2SO4 (b) YAS
(c) HNO3 (d) HC1
(a) I, II (b) I, III
15. I^C^O? + cone. HjSQj + H^ + ether---- > blue
(c) III, IV (d) IV, III
perchromic anhydride (in ethereal layer). Blue colour
5. Which of the following are soluble in excess of NaOH is due to
(X): As2S3; (Y): CuS; (Z): A1C13? (a) CrO3 (b) H>CrO4
(a) X, Y,Z (b) Y,Z (c) (d) CrO5
(c) X,Z (d) X,Y 16. There is foul smell in presence of moisture with
(a) A1C13 (b) A12(SO4)3
6. A mixture on heating gave a gas used as an (c) FeS (d) FeSO4
anaesthetic, 1.1 g of gas occupies 0.56 L at NTP.
17. AgNO3 gives white ppt with hypo changing to black
Mixture contains
after sometime. Black ppt is of
(a) NaNO3 + NH4C1 (b) NaNO2 + NH4C1 (a) Ag2S2O3 (b) Ag2SO4
(c) CaCO3 + MgCO3 (d) NH4C1 + Na2SO4 (c) Ag2S4O6 (d) Ag2S
7. Aqueous solution contains Zn(CH3COO)2, 18. SO2 and CO2 both turn lime water (A) milky, SO2 also
Cd(CH3COO)2 and Cu(CH3COO)2. On passing I^S turns I^Ci^Ot/H* (B) green while O2 is soluble in
gas, there is a precipitation of..... as sulphide pyrogallol (C) turning it black. These gases are to be
(a) Zn2+, Cd2+ (b) Cu2+, Cd2+ detected in order by using these reagents. The order is
(c) Zn2+, Cu2+ (d) Zn2+, Cu2+, Cd2+ (a) (A), (B), (C) (b) (B), (C), (A)
(c) (B), (A), (C) (d) (A), (O, (B)
8. Ferric alum gives red colour with NH4SCN due to
19. Aluminium sulphate (X) is slightly insoluble in
formation of water. It is converted into soluble sodium sulphate by
(a) A1(SCN)3 (b) [Fe(SCN)3r using Na2CO3 in the preparation of sodium carbonate
(c) Fe(SCN)3 (d) [Fe(SCN)f+ extract. Moles of (Y), required for complete
2+ conversion of 1 mole of (X) into soluble sulphate, is
A Cu , A (a) 1 (b) 2
9. Colourless salt (X) (X) ■> coloured
(0 3 (d)4
bead (Z). (X) can be
(a) borax (b) microcosmic salt 20. CoCl2 gives blue colour with NH4SCN in ethereal
(c) both (a) and (b) (d) None of these layer due to formation of
NaOH (y) (a) (NH4)2[Co(SCN)4] (b) (NH4)4[Co(SCN)6]
10. KC1 + cone H2SO4 + K^CrA (X)
(c) (NH4)3[Co(SCN)6] (d) (NH4)[Co(SCN)4]
(X) is reddish brown coloured gas soluble in NaOH
NHo/NdOH
forming (Y). (X) and (Y) are 21. HgCl2 + excess of KI---- > (A) ------ (B).
(a) Ci^OCl2, Na2CrO3 (b) Ci2O2Cl2) Na2CrO3 (A) and (B) respectively are
(c) CrO2Cl, Na2CrO4 (d)CrO2Cl2, Na2CrO4 Hg
11. Aqueous solution of BaBi^ gives yellow ppt with (a) KjHg^ (Nessler’s reagent), 0 NHjI
xHg/
(a) K2CrO4 (b) AgNO3 (X)
(c) both (a) and (b) (d) None of these (Iodide of Millon's base)
pH =x (Y)
12. Ci^O^ e -—
<- .... - >*- CrO^-. This change is based on (b) (Y), (X)
orange pH PH = yJ green (c) both (X)
change in pH. Probable values of x and y can be ' (d) both (Y)
(a) 8, 6 22. NH4SCN can be used to test one or more out of Fe3+,
(b) 8,10 Co2+,Cu2+
(c) 4,6 (a) Fe3+ only (b) Co2*, Cu2+
(d) change is independent of pH (c) Fe3+, Cu2+ (d) all of these
886 | Essential Inorganic Chemistry

23. K4[Fe(CN)6 ] can be used to detect one or more out of 30. To increase the molar solubility of CaCO3(s) in
Fe2+,Fe3+, Zn2+, Cu2+, Cd2+ saturated aqueous solution add
(a) Fe2+, Fe3+ (b) Fe3+, Zn2+, Cu.22++ (a) more water (b) Na2CO3
(c) all but Fe3+ (d) allbutFe2+ (c) NaOH (d)NaHSO4
24. Aqueous solution containing one mole borax reacts 31. Cu(OH)2 is highly soluble in all of the followin
with two moles of acids. This is because of except one. The exception is
(a) formation of 2 moles of B(OH)3 only (alHgO (b)NH3(u<7)
(b) formation of 2 moles of[B(OH)4]_ only (c)HCl(aq) (d)HNO3(ag)
(c) formation of 1 mole each of B(OH)3 and [B(OH)4 ]" 32. To increase significantly the concentration of fre=
(d) formation of 2 mol each of[B(OH)4r andB(OH) 3’ Zn2+ ion is a solution of the complex ion [Zn(NH3)4
of which only [B(OH)4 ]" reacts with acid.
Zn2+(aq)+ 4NH3(aq) (Zn(NH3)4f+(a4?)
25. Ag2S is soluble in NaCN due to formation of
add to the solution some
(a) Na[Ag(CN)2] (b) Ag(CN)2 (al^O (b)HCl(aq)
(c) Na2[Ag(CN)3] (d) Na2[Ag(CN)2] (c)NH3(aq) (d)NH4Cl(ag)
26. Match the substances given in B based on reactions 33. The best way to ensure complete precipitation from
given in A and select correct answe

You might also like